Вы находитесь на странице: 1из 953

SPECTRUM’S

Handbook for
GENERAL STUDIES
PAPER II
(UPSC Civil Services Preliminary Examination)

@UPSC_THOUGHTS
2020
Editor
Kalpana Rajaram

Assistant Editor
R. Vidya

With Contributions from


C. Rashmi, B.Tech.
Usha Govind, M.A.
R. Vidya, M.A.
Priyadarshi Kar, Ph.D.
S. Manikantan, Ph.D.
Ajit Kumar, M.A.
Kalpana Rajaram, M.A.

SPECTRUM BOOKS (P) LTD.


A1 291, First Floor, Janakpuri,
New Delhi 110 058
Editor’s Note

A few words about the name of this examination first—it is not CSAT. The civil services
examination, conducted by the Union Public Service Commission (UPSC), is a three-phase selection
process, of which the Preliminary Examination is the first phase. And the UPSC has called the
first stage just that—Civil Services Preliminary (CSP for short) examination. Since 2011, there
is no ‘optional’ paper in the Preliminary Examinations. Instead, there are two compulsory papers—
General Studies Paper I and General Studies Paper II. There have been changes in the syllabus;
in 2015 it was decided that marks for English Language Comprehension Skills would not be counted.
Also, the candidate was required to get 33 per cent to pass in GS II; however, the marks were
not to be counted for appearing in the main examination. In the 2015 exam, English Language
Comprehension passages were altogether omitted. Indeed, as their marks were not to be counted,
it seemed pointless to include the passages. Then the syllabus for 2016 left out English Language
Comprehension Skills.
The coverage of the syllabus in this book is concise and includes only what was felt to be absolutely
relevant. So a detailed discussion of theory in subjects like Interpersonal Skills and Decision-Making
has been eschewed, and more emphasis has been placed on the application aspect in most areas
of study. The questions have been devised with an emphasis on quality rather than mere quantity.
The Introduction explains the scope of the syllabus and the approach of this book more fully. Here,
@UPSC_THOUGHTS

I would like to acknowledge the help provided by many experts in the various areas of study,
some of whom are in the academic field and some in the civil services (and, therefore, having
a first-hand knowledge of what kind of attitude and abilities are required in the service), besides
the writers of the numerous reference books, without which this endeavour would not have been
possible.
The first edition of this book came out in April 2011, just two months before the examination
which was to be held for the first time according to the new pattern and syllabus. The syllabus,
all said and done, is somewhat ‘economical’ in its description of the areas of study. However, we
are happy that what we covered in our first edition proved to be quite near the mark—even in
the new areas of ‘Interpersonal Skills’ and ‘Decision-Making’. Now, with a few examinations based
on the new pattern behind us, we have a clearer picture of what the scope of the paper is likely
to be, though there is no clearcut pattern followed by the UPSC. It is to be noted that the emphasis
may vary: one year there may be more questions on numeracy/data interpretation and, in another,
more items on logical reasoning. In almost every year, there have been several comprehension
passages. From 2014 onwards, no question on Interpersonal Skills and Decision-Making have been
set, but till 2019 these subjects had not been removed from the syllabus. However, it would not
be advisable to analyse the papers and decide to be selective in study. Attention needs to be given
to all the subjects and different types of questions practised, so that a candidate is confident attempting
any kind of paper. As of now, we are confident that this book will go a long way in helping
many a candidate cross the important first step towards a career in the civil service.
We welcome suggestions from our readers to further improve this book.

Kalpana Rajaram

(iii)
Contents
Editor’s Note (iii) Worked Examples 1.7
Introduction (ix) Practice Exercises 1.13
Sample Tests ST.1—ST.180 Answers with Explanatory Notes 1.51
1. General Studies Paper II (2011) ST.2 2. INTERPERSONAL SKILLS 2.1—2.64
Practice Answer Sheet ST.15 INCLUDING COMMUNICATION
Answers and Explanatory Notes ST.16 SKILLS
2. General Studies Paper II (2012) ST.24
Practice Answer Sheet ST.39 Orientation 2.3
Answers and Explanatory Notes ST.40 Interpersonal Skills 2.3
3. General Studies Paper II (2013) ST.48 Trying to Define Interpersonal Skills 2.3
Practice Answer Sheet ST.63 Dimensions of Interpersonal Skills 2.3
Answers and Explanatory Notes ST.64 and How to Improve them
4. General Studies Paper II (2014) ST.72 1. Self-awareness 2.3
Practice Answer Sheet ST.83 2. Emotional Intelligence 2.4
Answers and Explanatory Notes ST.84 3. Self-management 2.4
5. General Studies Paper II (2015) ST.89 4. Persuasive Ability 2.4
@UPSC_THOUGHTS

Practice Answer Sheet ST.99 5. Leadership Styles 2.5


6. Teamwork Skills 2.6
Answers and Explanatory Notes ST.100
7. Interpersonal Communication 2.6
6. General Studies Paper II (2016) ST.105
8. Interpersonal Skills and Negotiation,
Practice Answer Sheet ST.114
Decision-Making, Problem-Solving
Answers and Explanatory Notes ST.115 and Conflict Resolution 2.9
7. General Studies Paper II (2017) ST.121 9. Stress management 2.10
Practice Answer Sheet ST.131 Can One Overdo Interpersonal Skills? 2.11
Answers and Explanatory Notes ST.132 Developing Interpersonal Skills: 2.11
8. General Studies Paper II (2018) ST.137 A Few Tips
Practice Answer Sheet ST.151
Communication Skills 2.11
Answers and Explanatory Notes ST.152
Importance of Communication 2.12
9. General Studies Paper II (2019) ST.157
Process of Communication 2.12
Practice Answer Sheet ST.168
Purposes of Communication 2.13
Answers and Explanatory Notes ST.180
Channels of Communication 2.13
1. COMPREHENSION 1.1—1.64 Modes of Communication 2.14
Barriers to Effective Communication 2.14
Orientation 1.3 Elements Necessary for Effective 2.16
Reading the Passage 1.3 Communication
Make a Consistent Effort 1.3 Importance of Listening in Communication 2.17
Barriers to Listening 2.18
Test Basics 1.3
Ways to Improve Listening Skills 2.19
Test Strategies 1.4
Tips to Improve Communication Skills 2.19
How to read... 1.4
To Resolve Conflicts and Create a
Answering Questions 1.5 more Positive Outcome 2.20
General Guidelines 1.5 Verbal Communication Skills 2.21
Tackling Comprehension Questions 1.5 Interpersonal Skills for Civil Servants 2.21
Pitfalls to avoid 1.6
To sum up 1.7 Question Types 2.24

(iv)
Contents

Practice Exercises 2.27 Bases for Decision-Making 4.7


Group Decision-Making 4.8
Answers with Explanatory Notes 2.55
Techniques of Decision-Making and 4.9
Problem Solving
3. LOGICAL REASONING AND 3.1—3.88
Errors in Decision-Making 4.11
ANALYTICAL ABILITY
Ethical Decision-Making 4.11
Orientation 3.3 Ethics and Decision-Making in 4.12
Basic Principles 3.3 Public Administration
Terms Explained 3.3 Characteristics of Effective Decision-Makers 4.16
Logical Deduction 3.4 Tips for Effective Decision-Making 4.16
I. Categorical Propositions 3.4 Problem-Solving and Decision-Making 4.17
II. Hypothetical Syllogisms 3.10 Compared
III. Disjunctive Syllogisms 3.11 Models of Problem Solving 4.17
IV. Relational Syllogisms 3.11 Problem-Solving Process 4.17
Inductive Logic: Analogy and 3.11 Causes of Poor Problem-Solving 4.18
Probable Inference Question Types 4.18
Order and Context of Premises, 3.13 Some Questions Analysed 4.21
Assumptions and Unstated Conclusions
Practice Exercises 4.25
Analytical Ability and Analytical Reasoning 3.14
Analytical Reasoning Problems 3.14 Answers with Explanatory Notes 4.52
@UPSC_THOUGHTS

Best Approach to Analytical Reasoning 3.15


Problems
5. GENERAL MENTAL ABILITY 5.1—5.56
Worked Examples 3.15
Orientation 5.3
I. Statement/Conclusion 3.15
II. Logical Compatibility or Logically 3.17 Scope of this Section 5.3
Related Statements Series 5.3
III. Statement-Assumptions 3.18 Alphabet Series 5.3
IV. Cause and Effect 3.20 Number Series 5.5
V. Passage-Based Reasoning 3.20 Letter-Number Mixed Series 5.6
VI. Arrangement, Selection and 3.23 Figure Series 5.6
Allocation Problems
Analogy 5.7
Practice Exercises 3.29 Verbal Analogy 5.7
Number Analogy 5.7
Answers with Explanatory Notes 3.66
Figure Analogy 5.8
4. DECISION-MAKING AND 4.1—4.64 Classification/Odd-man-out 5.8
PROBLEM-SOLVING Verbal Classification 5.8
Numerical Classification 5.9
Orientation 4.3
Figure Classification 5.9
Basic Ideas 4.3
Matrix and Other Patterns 5.10
What is Decision-Making? 4.3
Matrix 5.10
Characteristics of Decision-Making 4.3
Other Patterns 5.10
Kinds of Decisions 4.3
Analogy-based Patterns 5.10
Factors Influencing Decision-Making 4.4
Decision-Making Processes 4.4 Coding / Decoding 5.11
Models of Decision-Making 4.6 Letter Coding 5.11

(v)
Contents

Coding with Numbers 5.11 HCF and LCM 6.15


Substituted Words 5.12 Highest Common Factor (HCF) 6.15
Sentence Coding 5.12 Least Common Multiple (LCM) 6.16
Using Mathematical Symbols 5.13
Indices, Roots and Surds 6.17
Passwords 5.13 Indices 6.17
Coded Statements and Conclusions 5.14 Finding Square Root 6.18
Direction Problems 5.15 Finding Cube Root 6.19
Problems on Linear Direction 5.15 Surds 6.19
Problems on Angular Direction 5.15 Simplification 6.20
Problems of Distance and Directions 5.16 Using Vbodmas 6.20
Ranking 5.17 Fast-track formulae to Remember and
their Application 6.21
Blood Relations 5.17
Percentage 6.22
Figure Partition 5.17
Basics 6.22
Building Designs 5.18 Facts and Fast-Track Formulae to Remember 6.22
Dices 5.18 Worked Examples 6.24

Images 5.19 Average 6.27


Mirror Images 5.19 Basics 6.27
Fast-Track Formulae to Remember 6.27
@UPSC_THOUGHTS

Water Images 5.19


Worked Examples 6.28
Embedded Figures 5.20
Ratio and Proportion 6.30
Completing Patterns 5.20 Basics of Ratio 6.30
Figure Matrix 5.20 Basics of Proportion 6.30
Fast-Track Methods and Formulae 6.31
Paper Cutting 5.21 Worked Examples 6.32
Paper Folding 5.21 Unitary Method 6.35
Practice Exercises 5.23 Basics 6.35
Variation and Proportion 6.35
Answers with Explanatory Notes 5.42
Points to Remember 6.35
Worked Examples 6.35
6. BASIC NUMERACY AND 6.1—6.256
Time and Work, Work and Wages 6.37
DATA INTERPRETATION
Basics 6.37
Orientation 6.3 Points to Remember 6.37
Fast-Track Methods 6.37
Numbers and Fundamental Operations 6.3
Worked Examples 6.37
Numbers, Numerals and Digits 6.3
Number Classification 6.3 Pipes and Cisterns 6.40
Fundamental Operations: Addition, 6.6 Basics 6.40
Subtraction, Multiplication and Division Fast-Track Methods 6.40
Test of a Prime Number 6.10 Worked Examples 6.40

Fractions and Decimals 6.12 Time and Distance 6.41


Vulgar Fractions 6.12 Basics 6.41
Decimals 6.12 Fast-Track Methods 6.42

(vi)
Contents

Worked Examples 6.42 Clocks 6.74


Basics 6.74
Boats and Streams 6.44
Points to Remember 6.74
Basics 6.44
Worked Examples 6.74
Fast-Track Methods 6.44
Worked Examples 6.45 Algebraic Formulae and Equations 6.75
Basics 6.75
Races and Games 6.45
Linear Equations 6.75
Basics 6.45
Quadratic Equations 6.76
Points to remember 6.46 Points to Note 6.77
Fast-Track Formulae 6.46 Some Facts about Algebraic Expressions
Worked Examples 6.46 and Equations 6.77
Profit and Loss 6.47 Problems on Ages 6.77
Basics 6.47
Fast-Track Methods 6.48 Permutation and Combination 6.79
Basics 6.79
Worked Examples 6.49
Formulae to Remember 6.79
Alligation and Mixtures 6.51 Worked Examples 6.80
Basics 6.51
Probability 6.81
Fast-Track Methods 6.52
Basics 6.81
Worked Examples 6.52
Measuring Probability 6.82
Interest 6.54 Probability Theorems 6.83
@UPSC_THOUGHTS

Basics 6.54 Worked Examples 6.83


Formulae for Simple Interest 6.54 Progressions 6.85
Formulae for Compound Interest 6.54 Basics 6.85
Fast-Track Methods 6.55 Arithmetic Progression 6.85
Worked Examples 6.56 Geometric Progression 6.85
Discount 6.61 Harmonic Progression 6.86
True Discount 6.62 Worked Examples 6.86
Formulae to Remember for True Discount 6.62 Geometry 6.86
Banker’s Discount 6.63 Lines and Angles 6.86
Formulae to Remember for Banker’s 6.63 Polygon 6.87
Discount Triangle 6.87
Worked Examples 6.63 Quadrilaterals 6.88
Circles 6.89
Partnership 6.66
Points to Remember 6.89
Basics 6.66
Worked Examples 6.91
Formulae to Remember 6.66
Worked Examples 6.67 Coordinate Geometry 6.92
Basics and Some Formulae 6.92
Stocks, Shares and Debentures 6.69
Basics 6.69 Mensuration: Area and Volume 6.92
Formulae for Problems on Stocks 6.70 Perimeter, Area, Volume 6.92
Formulae for Problems on Shares 6.70 Tables of Metric Units of Measurement 6.93
Two-Dimensional Figures 6.94
Worked Examples 6.71
1. Triangle 6.94
Calendars 6.73 2. Quadrilateral 6.94
Basics 6.73 3. Parallelogram 6.94
Worked Examples 6.74 4. Rectangle 6.95

(vii)
Contents

5. Rhombus 6.95 Worked Examples 6.106


6. Square 6.95
Data Sufficiency 6.107
7. Trapezium 6.95
Types and Strategies 6.107
8. Regular Polygon 6.95
Worked Examples 6.107
9. Circle 6.96
Fast-Track Formulae 6.96 Data Interpretation 6.111
Three-Dimensional Figures 6.97 Statistical Methods 6.111
1. Rectangular Solid 6.97 Interpretation 6.130
2. Cube 6.97 Interpreting From Tables 6.130
3. Prism 6.97 Interpreting Graphs and Diagrams 6.133
4. Cylinder 6.97 Practice Exercises 6.139
5. Cone 6.98
6. Pyramid 6.98 Answers with Explanatory Notes 6.182
7. Sphere 6.99
Worked Examples 6.99
7. PRACTICE SESSION 7.1—7.160
Heights and Distances 6.101 I. Practice Exercise Sets 7.1
(Basic Trigonometry)
Practice Set 1 7.1
Trigonometry and Heights and Distances 6.101
Answers with Explanatory Notes 7.23
Measuring Angles 6.101
Trigonometrical Ratios 6.102 Practice Set 2 7.34
@UPSC_THOUGHTS

Sign of Trigonometrical Ratios 6.102 Answers with Explanatory Notes 7.56


Trigonometric Ratios of Complementary
Practice Set 3 7.65
Angles 6.102
Answers with Explanatory Notes 7.88
Trigonometrical Identities 6.102
Angles and Trigonometrical Ratios 6.103 II. Practice Test Papers 7.107
Worked Examples 6.104 Practice Test One 7.107
Logarithms 6.105 Answers with Explanatory Notes 7.118
Order of Magnitude and Logarithm 6.105 Practice Test Two 7.123
Logarithm 6.105 Answers with Explanatory Notes 7.135
Properties of Logarithms 6.105
Practice Test Three 7.141
Characteristic and Mantissa of Logarithm 6.105
Applications of Logarithm 6.106 Answers with Explanatory Notes 7.154

(viii)
Introduction
From 2011, the pattern and syllabus for the preliminary examination are different. There is
no longer an ‘optional’ paper; instead, there are two compulsory papers—General Studies
Paper I and General Studies Paper II. Both papers are of the objective multiple choice question
type. While Paper I covers most subjects that were covered in the compulsory General Studies
paper of earlier years (before 2011), the second paper covers several new subjects. The objective,
apparently, is to create a ‘level playing field’ for candidates through a common test. No longer
will there be that sneaking feeling that some of the optional subjects are more ‘scoring’ than
others; nor will there be speculation about how subjects such as mathematics and geography
or history could be given the same kind of marking, or what kind of normalisation is done
to make the scores more ‘fair’. General Studies Paper I and Paper II each now carries 200
marks, and each is for the duration of 2 hours. The General Studies Paper I has 100 questions
and the General Studies Paper II has 80 questions. The General Studies Paper II has been
made a qualifying test since 2015, with candidates requiring 33 per cent to pass it, though
its marks will not be counted for appearing in the main examination.

THE SYLLABUS FOR PAPER II


@UPSC_THOUGHTS

200 marks – Duration : Two hours


● Comprehension
● Interpersonal skills including communication skills
● Logical reasoning and analytical ability
● Decision making and problem solving
● General mental ability
● Basic numeracy (numbers and their relations, orders of magnitude, etc.—Class X
level), Data interpretation (charts, graphs, tables, data sufficiency, etc.—Class X
level)

SAMPLE QUESTIONS FOR PAPER II PUT OUT BY THE UPSC IN 2011


Read the following passage and answer the given questions. Your answers should be drawn from
the content of the given passage only.
The economy of contemporary India is a great paradox. It is a strange combination of outstanding
achievements as well as grave failures. Since independence, India has achieved remarkable
progress in overcoming its economic backwardness. From being a very poor country in the 1950s
and a ‘basket case’ in the mid 1960s, it has emerged as the fourth largest economy in the world
(in terms of purchasing power parity). Our economy has become one of the fastest growing
economies in the world. Now the country is one of the leading players in the world knowledge
economy with vast intellectual capital and booming software and information technology services.
While our country has joined the league of the world’s top five fastest growing economies, we
are in the bottom 20 among all countries in terms of the Human Development Index. While
the country is celebrating its growth rate and technological wonders, it is witnessing social
contradictions and the paradox and ironies of development. Thus, there are ‘two Indias’ in
contemporary India.

(ix)
Introduction

1. Why is the Indian economy considered ‘a great paradox’?


(a) It is a leading player in information technology services with low levels of literacy.
(b) There is poverty amidst plenty in agricultural produce.
(c) It is one of the largest economies with low human development.
(d) It has scientific achievements with social contradictions.

2. Why is India being referred to as a leading player in the world knowledge


economy?
(a) India’s knowledge base in science and technology is one of the world’s best.
(b) India has huge reserves of human intellectual capital and information technology
services.
(c) India is among the world’s five fastest growing economics and technology reserves.
(d) India has a huge reservoir of human capital and scientific knowledge export
potential.
3. What does the author imply by the phrase ‘two Indias’?
(a) There is the India that has vast intellectual capital and the other that is largely
illiterate.
(b) There is the India of burgeoning growth and the India of widespread want and
misery.
(c) There is the India of progressive mindsets and the others who are socially conservative.
@UPSC_THOUGHTS

(d) There is an India of outstanding achievements combined with gigantic failures.

4. Consider the following statement and also the conclusions. Answer the question that
follows:
Statement : Education is in the Concurrent List. The State government cannot bring
reforms in education without the consent of the Central Government.
Conclusion I : For bringing about quick reforms in education, it should be in the State
List.
Conclusion II : States are not willing to bring about quick reforms in education.
Which one of the following is correct?
(a) Conclusion–I only follows from the statement
(b) Conclusion–II only follows from the statement
(c) Both conclusions I and II follow from the statement
(d) Neither conclusion I nor conclusion II follows from the statement

5. Five persons P, Q, R, S, T are sitting in a row. Q is between P and T. To find who among
them is in the middle, which of the information given in the following statements is/
are sufficient?
1. P is left of Q and right of S.
2. R is at the right end.
Select the correct answer using the code given below:
(a) 1 only
(b) 2 only
(c) Either 1 or 2
(d) Both 1 and 2

(x)
Introduction

6. Which one among , , , is the smallest one?


(a)
(b)
(c)
(d)

7. The following pie charts show that a man spends 10% on clothes, 20% on rent, 30%
on food and rest on miscellaneous items in the month of January and spends 15% on
clothes, 25% on rent, 35% on food and rest on miscellaneous items in the month of
February.

@UPSC_THOUGHTS

Consider the following statements:


1. The money spent on food over rent in the month of January is same as the money
spent on food over rent in the month of February.
2. The money spent on rent over clothes is same as money spent on food over rent
in the month of January.
Which of the statements given above is/are correct?
(a) 1 only
(b) 2 only
(c) Both 1 and 2
(d) Neither 1 nor 2

HOW THIS BOOK CAN HELP YOU

The subjects have been mentioned by the UPSC syllabus, but the scope of each subject or what
exactly it entails has not been elaborated. The sample questions are basic and do not, in any
case, seem to cover every subject prescribed. The recent examination papers which are
discussed later in this book give a further idea of the kind of questions that may be expected.
Let’s take the subjects one by one.

Comprehension
The section on Comprehension – which will be in English as well as in Hindi—will obviously
consist of passages followed by questions. The length of the passages would differ. All the
questions are not likely to be direct; some will call upon you to think deeply before you choose

(xi)
Introduction

your answer response. Many of the passages call for critical reasoning. Besides your native
intelligence and ability to understand a language in all its nuances, you need practice: practice
at reading fast even while grasping the meaning, and then answering the questions correctly
without wasting time.
Though the 2011 paper—the first based on the new syllabus—contained several short to
medium length passages, the 2012 paper had longer passages. The papers since then have
had a mixed bag of long and short passages. The subjects range from philosophical, political
and economic to the sociological and environmental. There is no certainty about the length
or type of passage. So, we have given passages of varying lengths for practice as well as
the techniques for tackling passages of different lengths and different types. The questions
are not all straightforward but devised to test your analytical ability and understanding of
the language. Moreover, each answer has been explained to give you a full understanding
of how to choose the correct answer. It does not matter which subject the passage relates
to; what matters is your ability to understand various aspects of the passage—and learn not
to go beyond its extent unless asked to do so specifically.

Interpersonal Skills including Communication Skills


This is a new area, and has not as such been a subject of objective tests in competitive
@UPSC_THOUGHTS

examinations. Interpersonal skills involves attitudinal traits that determine how you behave
with other people and, as many experts have pointed out, are best tested in interviews or
group discussions, may be even in written tests—what is known as ‘subjective’ format—rather
than in objective type tests. How does one, for example, gauge the tone and gestures of a
speaker—so much part of interpersonal communication—in an objective test? Moreover, these
aspects are not amenable to set evaluations and easy judgement. But since the subject is in
the syllabus, we have briefly covered some important theoretical aspects, including interpersonal
communication skills.
Communication skills, again, could mean theoretical as well as applied aspects. You should
improve your vocabulary and understanding of words and expressions – in whichever
language you choose for the examination. We have given the basic features of communication
skills very briefly.
The practice exercises that we have designed emphasise more on the practical or applied
aspects than on the theoretical, as the test is not likely to call for ‘expert’ knowledge in this
field. The questions in the paper are more likely to test your ‘attitude’ in certain situations
or circumstances—how you deal with colleagues, subordinates, and higher authorities in
various situations. On the whole, question-types in this subject are limited, and we have
avoided repetitive themes. The explanatory notes give a clear idea of the attitudinal aspects
required in a civil services aspirant. We have also included among the questions a variety
of items that test linguistic communication skills, such as clarity in written communication,
and the ability to choose the correct word/phrase in the given context and express a thought
briefly and effectively. Communication skills, after all, include linguistic ability.

(xii)
Introduction

Logical Reasoning and Analytical Ability


Many competitive examinations have items on logical and analytical reasoning. These
questions test your ability to think clearly and understand and analyse a given statement –
for that indicates your ability to approach a situation in real life rationally and analytically
enough to be able to find a solution to complex problems. While formal logic may not be
required for answering questions of the type set in the test, a basic knowledge of terms and
rules of deductive and inductive logic is helpful in understanding and answering the questions
with accuracy and reasonable speed.
We have given an elementary review of logic along with a number of examples of questions
that call for the application of logic. It is necessary to know how statements can be deduced
from others; how assumptions and inferences can (or cannot) be made from given situations
or statements; how to read implicit meanings; how to determine whether certain events are
causes or the effects of other events, and so on. If you get the basics right you will be able
tackle any question relating to logical reasoning whatever form it takes. We have also given
questions that go under the name of ‘analytical reasoning’ in competitive examinations. These
are mainly based on arrangement of elements and selection of elements from given data. At
times, these data are presented in long passages that require steady reading and correct
@UPSC_THOUGHTS

understanding. These questions require a technique of approach which you need to master.
The questions we have included in the practice exercise range over all types, including several
tough ones. The explanatory notes that go with almost every answer will help you to
understand how to reach the answer to the particular kind of problem.

Decision-making and Problem-solving


This is another section which is normally not a part of competitive examinations comprising
objective type questions. But, of course, the ability to make decisions and solve problems is
expected of a civil service aspirant. Though the UPSC syllabus does not spell out what is
meant to be tested in this section, it may be understood that certain elements go into any
and every kind of decision-making. Broadly, decisions may have to be taken immediately on
some emerging situation or they may have to be taken over time on some well-defined problem.
Approaches will differ from case to case. So we have given the elements that go into decision-
making and problem-solving. The basics are the same – whether for the corporate or for the
wider civil service sector. However, we have not gone deeply into the theories.
The emphasis in the civil service set-up would be more administration oriented, with
politico-socio-economic factors thrown in, rather than corporate-oriented. So, cost-benefit
analysis and decisions on marketing a product are less likely to be the subject of questions.
There will be questions of the situational judgement or reaction type. These questions present
a situation and ask you what you would do in the circumstances, how you would respond
to a situation. Your answer depends on the kind of person you are – somewhat like in
interpersonal skills. Your response depends on your attitude to life in general and to the many

(xiii)
Introduction

issues around you, on your character and, to an extent, the environment and the way in which
you have been brought up. A civil servant – in fact, any person of character – needs to have
or needs to develop certain qualities – integrity, honesty, tolerance, impartiality, courage in
the face of adversity, presence of mind, alertness to evolving situations and the requisite
flexibility. It is obvious that some of these can be learnt, while others have to be present in
the first place to be developed further. It will also be obvious that, in this section of the test
as in the section on interpersonal skills, you cannot ‘learn’ all possible responses or ‘practise’
all possible questions. What you can do is get familiar with the kind of questions that are
likely to come and develop your own character to be responsive to the needs of this diverse
society, and also develop a deep respect for the rule of law and the principle of equality before
the law – two essential elements of constitutional government.
We have given a wide variety of questions encompassing a variety of ‘situations’, besides
questions that test how well and truly you are able to follow a set of given rules. Again, we
have given explanatory notes to elucidate why a certain answer would be considered ‘more
correct’ than another. This ought to help you to understand how to tackle a problem.
Incidentally, these problems are not of the mathematical type, and there could be more than
one ‘correct’ answer though the degree of correctness would vary. Significantly, in the
@UPSC_THOUGHTS

examination, questions on Interpersonal Skills and Decision-Making have not been subject to
negative marking—implying that the ‘correctness’ of answers is not so fixed and, after all,
one should not be penalised for a partially correct answer.

General Mental Ability


The name would be familiar-sounding to those who remember the syllabus of the General
Studies paper in the Prelims till 2010. The General Mental Ability (GMA) section comprised
logical and analytical reasoning as well as numerical aptitude and visual/spatial aptitude
questions. Now, as logical reasoning and analytical ability as well as numeracy are independent
subjects, we have kept for this section several questions that usually go under the name of
‘intelligence’ tests. We have given the principles required to solve them as well as an adequate
number of questions of each type for practice. Needless to say, explanatory notes add to
information while elucidating the problem.

Basic Numeracy (including Data Interpretation)


The UPSC syllabus says that the level of this section will be that of X standard. Now, basic
numeracy can be restricted to arithmetic or it can include a few more areas of mathematics.
To be on the safe side, we have gone a little beyond arithmetic and included elementary algebra,
geometry, trigonometry and logarithm to refresh your memory. The section also covers data
interpretation along with the statistical methods required to solve questions of this type.
We have discussed the basic principles and formulae, and given some short cut methods
of reaching the solution quickly wherever possible. The questions range over a wide variety,
and focus on the ‘tricky’ types rather than just on the conventional kind. Explanatory notes

(xiv)
Introduction

are appended to practically every answer so that you know how to solve each problem and
how and where to use the short cuts.
Just remember, you are not allowed to use calculators for objective tests.

Practice Session
This is the last section in the book. As the name suggests, there is a large collection of select
multiple choice questions covering the syllabus. This section is intended to enable you to assess
yourself and locate weak points so that you can pay special attention to those areas. This
is followed by three Practice Tests. This section ought to help you with all the necessary practice
not only to answer questions accurately but also to do so within the time limit.

GENERAL DO’S AND DON’TS IN THE APPROACH TO


MULTIPLE CHOICE QUESTIONS
There are a few things to be kept in mind while answering multiple choice questions.
1. In a multiple choice question paper, the answer is there before you – all you have
to do is find it. You can do so in many cases by eliminating some of the answer choices
rather than by using all the knowledge at hand to arrive at the answer. In some items,
a couple of answer choices are obviously incorrect. Cross them out if you locate them,
@UPSC_THOUGHTS

and concentrate on the narrowed down options. This is especially helpful in those
questions where you are somewhat unsure of the answer. In our worked examples
and the explanatory notes accompanying the answers to various questions, we have
pointed out those types of questions where such elimination is possible and where
it isn’t.
2. As time is of the essence, don’t get stuck on a question you are unable to understand
or whose answer you are unable to find; make a mark against the question and move
on to the next question and the next . . . Answer all the questions you are sure of
and then come back to the unanswered ones. It happens at times that another question
actually provides a hint about the answer to a question that has puzzled you. You
could differentiate in the marks you put against those questions you hope to solve
and those which you feel are just beyond you, so that when you come back to solve
the questions a second time round, you can simply skip those that you never intended
to attempt. And don’t feel let down if there are such stinkers – there are bound to
be at least some of those, but they will be just a very few, and most likely will puzzle
every other candidate as well.
3. Every time you skip a question, be sure to skip that number in the answer sheet as
well, otherwise you may mark the correct answer in the wrong place and lose precious
marks. Talking of answer sheets, remember to use a soft black pencil that will not
scratch or perhaps tear your answer sheet. Keep a good eraser ready to rub out
completely any wrong answer that you want to change. You don’t need reminding
that the computer checking system will be confused and mark you wrong if it gets
two answers for a question.

(xv)
Introduction

4. Last but not least, read the directions to the questions carefully, understand them
and follow them meticulously; don’t assume something that is not said in the
directions. Read the questions carefully, understand what it wants. Then read the
options equally carefully and mark your choice after due thought. Be especially careful
in those questions which are phrased ‘negatively’ – asking you what is ‘not’ true or
what should ‘not’ be done. The options almost always have one among them that
gives an answer to the positive question, and thus could trap you into giving a wrong
answer in a hurry even though you know the right one.

Above all, remain calm and cool. It is just a test, after all. And everyone is in the same
boat anyway.

@UPSC_THOUGHTS

(xvi)
Introduction

Now, go on to the SAMPLE TESTS—the General


Studies Paper II of the last few years set by the
UPSC—to get an idea of the type of questions that
are likely in the new pattern of tests.
✫ There are 80 questions for 200 marks in each
paper, all questions carrying equal marks; so
each question is of 2.5 marks.
@UPSC_THOUGHTS

✫ All questions except those on Interpersonal Skills


and Decision-Making are subject to negative
marking for wrong answers.
Try the Questions and see how you fare.
Use the practice answer sheets to mark your answers.
Check your answers with the key provided, and
understand the answers with the help of the extensive
explanatory notes provided.

ST . 1
Sample Test : General Studies Paper II (2011)

SAMPLE TEST 1 : GENERAL STUDIES PAPER II (2011)


Directions for the following 4 (four) items: 2. Consider the following assumptions:
Read the following passage and answer the items that 1. A country under foreign domination cannot
follow. Your answers to these items should be based on indulge in spiritual pursuit.
the passage only. 2. Poverty is an impediment in the spiritual
pursuit.
Passage
3. Subject peoples may become other-worldly.
A country under foreign domination seeks escape
from the present in dreams of a vanished age, and finds With reference to the passage, which of the above
consolation in visions of past greatness. That is a assumptions is/are valid?
foolish and dangerous pastime in which many of us (a) 1 and 2
indulge. An equally questionable practice for us in (b) 2 only
India is to imagine that we are still spiritually great (c) 2 and 3
though we have come down in the world in other (d) 3 only
respects. Spiritual or any other greatness cannot be
3. The passage thematically centres on
founded on lack of freedom and opportunity, or on
starvation and misery. Many western writers have (a) the state of mind of oppressed people
@UPSC_THOUGHTS

encouraged the notion that Indians are other-worldly. (b) starvation and misery
I suppose the poor and unfortunate in every country (c) the growth of civilization
become to some extent other-worldly, unless they become (d) body, mind and spirit of people in general
revolutionaries, for this world is evidently not meant
4. According to the passage, the torture of the mind
for them. So also subject peoples.
and spirit is caused
As a man grows to maturity he is not entirely
(a) by the impact of foreign domination
engrossed in, or satisfied with, the external objective
world. He seeks also some inner meaning, some (b) by the desire to escape from foreign
psychological and physical satisfactions. So also with domination and find consolation in visions
peoples and civilizations as they mature and grow of past greatness
adult. Every civilization and every people exhibit these (c) due to lack of equilibrium between an
parallel streams of an external life and an internal life. external life and an internal life
Where they meet or keep close to each other, there is (d) due to one’s inability to be either
an equilibrium and stability. When they diverge conflict revolutionary or other-worldly
arises and the crises that torture the mind and spirit.
Directions for the following 3 (three) items:
1. The passage mentions that “this world is evidently
Read the passage given below, study the graph that
not meant for them”. It refers to people who
follows and answer the three items given below the figure.
1. seek freedom from foreign domination
2. live in starvation and misery During a party, a person was exposed to
3. become revolutionaries contaminated water. A few days later, he developed
Which of the statements given above is/are fever and loose motions. He suffered for some days
correct? before going to a doctor for treatment. On starting the
(a) 1 and 2 treatment, he soon became better and recovered
(b) 2 only completely a few days later. The following graph shows
(c) 2 and 3 different phases of the person’s disease condition as
(d) 3 only regions A, B, C, D and E of the curve.

ST . 2
Sample Test : General Studies Paper II (2011)

Number of Bacteria 10. Consider the figure given below and answer the
item that follows:

0
0
Time In the figure shown above, OP1 and OP2 are two
5. Which region/regions of the curve correspond/ plane mirrors kept perpendicular to each other.
corresponds to incubation phase of the infection? S is the direction of a beam of light falling on the
(a) A only mirror OP1. The direction of the reflected beam of
(b) B only light from the mirror OP2 will be
(a) Perpendicular to the direction S.
(c) B and C
(b) At 45° to the direction S.
(d) No part of the curve indicates the incubation
(c) Opposite and parallel to the direction S.
phase
(d) At 60° to the direction S.
6. Which region of the curve indicates that the
11. Consider the following figure and answer the
person began showing the symptoms of infection? item that follows:
(a) A (b) B
(c) C (d) D
@UPSC_THOUGHTS

7. Which region of the curve indicates that the


treatment yielded effective relief?
(a) C
(b) D
(c) E
(d) The curve does not indicate the treatment
8. There are four routes to travel from city A to city
B and six routes from city B to city C. How many What is the minimum number of different colours
routes are possible to travel from the city A to required to paint the figure given above such that
city C? no two adjacent regions have the same colour?
(a) 24 (b) 12 (a) 3 (b) 4
(c) 10 (d) 8 (c) 5 (d) 6
12. Consider the following figure and answer the
9. A contract on construction job specifies a penalty
item that follows:
for delay in completion of the work beyond a
certain date as follows : ` 200 for the first day, 15
` 250 for the second day, ` 300 for the third day
etc., the penalty for each succeeding day being
` 50 more than that of the preceding day. How
much penalty should the contractor pay if he 48
delays the work by 10 days?
(a) ` 4950
(b) ` 4250
(c) ` 3600
(d) ` 650 A square is divided into four rectangles as shown

ST . 3
Sample Test : General Studies Paper II (2011)

above. The lengths of the sides of rectangles are 2. Need of the hour is to have an enabling
natural numbers. The areas of two rectangles are Government.
indicated in the figure. What is the length of each 3. The Government should engage in
side of the square? maximum interference in market processes.
(a) 10 4. There is a need to change the size of the
(b) 11 Government.
(c) 15 Which of the statements given above are correct?
(d) Cannot be determined as the given data are (a) 1 and 2 only
insufficient (b) 2 and 3 only
(c) 1 and 4 only
Directions for the following 8 (eight) items:
(d) 1, 2, 3 and 4
Read each of the following two passages and answer the
14. According to the passage, the strategy of inclusive
items that follow. Your answers to these items should be
growth can be effected by focussing on
based on the passages only.
(a) Meeting all the needs of every citizen in the
Passage-1 country.
For achieving inclusive growth there is a critical (b) Increasing the regulations over the
need to rethink the role of the State. The early debate manufacturing sector.
among economists about the size of the Government (c) Controlling the distribution of manufactured
can be misleading. The need of the hour is to have an goods.
enabling Government. India is too large and complex (d) Delivery of the basic services to the deprived
a nation for the State to be able to deliver all that is sections of the society.
@UPSC_THOUGHTS

needed. Asking the Government to produce all the 15. What constitutes an enabling Government?
essential goods, create all the necessary jobs, and keep 1. A large bureaucracy.
a curb on the prices of all goods is to lead to a large 2. Implementation of welfare programmes
cumbersome bureaucracy and widespread corruption. through representatives.
The aim must be to stay with the objective of 3. Creating an ethos that helps individual
inclusive growth that was laid down by the founding enterprise.
fathers of the nation and also to take a more modern 4. Providing resources to those who are
view of what the State can realistically deliver. underprivileged.
This is what leads to the idea of an enabling State, 5. Offering direct help to the poor regarding
that is, a Government that does not try to directly basic services.
deliver to the citizens everything that they need. Instead,
Select the correct answer from the codes given
it (1) creates an enabling ethos for the market so that
below:
individual enterprise can flourish and citizens can, for
(a) 1, 2 and 3 only
the most part, provide for the needs of one another, and
(b) 4 and 5 only
(2) steps in to help those who do not manage to do well
(c) 3, 4 and 5 only
for themselves, for there will always be individuals, no
(d) 1, 2, 3, 4 and 5
matter what the system, “who need support and help.
Hence we need a Government that, when it comes to 16. Why is the State unable to deliver “all that is
the market, sets effective, incentive-compatible rules and needed”?
remains on the sidelines with minimal interference, 1. It does not have sufficient bureaucracy.
and, at the same time, plays an important role in 2. It does not promote inclusive growth.
directly helping the poor by ensuring that they get basic Select the correct answer from the codes given
education and health services and receive adequate below:
nutrition and food. (a) 1 only
13. According to the passage: (b) 2 only
1. The objective of inclusive growth was laid (c) Both 1 and 2
down by the founding fathers of the nation. (d) Neither 1 nor 2

ST . 4
Sample Test : General Studies Paper II (2011)

17. What is the essential message being conveyed by 2. Instigation by external forces.
the author of the passage? 3. Quest for social equality and individual
(a) The objectives of inclusive growth laid freedom.
down by the founding fathers of the nation 4. Urge for granting privileges and self-respect
should be remembered. to disparaged sections of the society.
(b) The Government needs to make available
Select the correct answer using the codes given
more schools and health services.
below:
(c) The Government needs to establish markets
(a) 1 and 3 only
and industries to meet the needs of the poor
(b) 2 and 4 only
strata of the society.
(c) 3 and 4 only
(d) There is a need to rethink the role of the
(d) 1, 2, 3 and 4
State in achieving inclusive growth.
20. With reference to the passage, consider the
Passage-2 following statements:
The concept of ‘creative society’ refers to a phase 1. To be a creative society, it is essential to
of development of a society in which a large number have a variety of social movements.
of potential contradictions become articulate and active. 2. To be a creative society, it is imperative to
This is most evident when oppressed social groups get have potential contradictions and conflicts.
politically mobilised and demand their rights. The
Which of the statements given above is/are
upsurge of the peasants and tribals, the movements for
correct?
regional autonomy and self-determination, the
(a) 1 only
environmental movements, and the women’s movements
@UPSC_THOUGHTS

(b) 2 only
in the developing countries are signs of emergence of
creative society in contemporary times. The forms of (c) Both 1 and 2
social movements and their intensity may vary from (d) Neither 1 nor 2
country to country and place to place within a country. 21. Consider the following three statements:
But the very presence of movements for social 1. Only students can participate in the race.
transformation in various spheres of a society indicates 2. Some participants in the race are girls.
the emergence of a creative society in a country. 3. All girl participants in the race are invited
18. What does the author imply by “creative society”? for coaching.
1. A society where diverse art forms and Which one of the following conclusions can be
literary writings seek incentive. drawn from the above statements?
2. A society where social inequalities are (a) All participants in the race are invited for
accepted as the norm. coaching.
3. A society where a large number of (b) All students are invited for coaching.
contradictions are recognised.
(c) All participants in the race are students.
4. A society where the exploited and the
(d) None of the statements (a), (b) and (c) given
oppressed groups grow conscious of their
above is correct.
human rights and upliftment.
Directions for the following 2 (two) items:
Select the correct answer using the codes given
below: Each of the following two items consists of four statements.
(a) 1, 2 and 3 Of these four statements, two cannot both be true, but both
(b) 4 only can be false. Study the statements carefully and identify
(c) 3 and 4 the two that satisfy the above condition. Select the correct
(d) 2 and 4 answer using the codes given below each set of statements:

19. What according to the passage are the 22. Examine the following statements:
manifestations of social movements? 1. All animals are carnivorous.
1. Aggressiveness and being incendiary. 2. Some animals are not carnivorous.

ST . 5
Sample Test : General Studies Paper II (2011)

3. Animals are not carnivorous. Directions for the following 2 (two) items:
4. Some animals are carnivorous. The following pie charts show the break-up of
Codes: disease categories recorded in the patients from two
(a) 1 and 3 towns, Town A and Town B. Pie charts plot the disease
(b) 1 and 2 categories as percentage of the total number of patients.
(c) 2 and 3 Based on these, answer the two items that follow the
(d) 3 and 4 charts.
23. Examine the following statements:
Distribution of diseases in Town – A
1. All trains are run by diesel engine.
2. Some trains are run by diesel engine.
3. No train is run by diesel engine.
4. Some trains are not run by diesel engine. Others Diabetes
Codes:
(a) 1 and 2
(b) 2 and 3
(c) 1 and 3 High BP
(d) 1 and 4
Obesity
24. Consider the four age pyramids given below
namely A, B, C and D representing four different
countries. Heart Disease
@UPSC_THOUGHTS
Age

Distribution of diseases in Town – B


Per cent in age class

Which one of them indicates the declining


population? Others Diabetes
(a) A (b) B
(c) C (d) D
25. The following figure has four curves namely A,
B, C and D. Study the figure and answer the item High BP
that follows. Obesity
Population (billions)

Heart Disease

26. Which of the two towns has a higher number of


persons with Diabetes?
Year
(a) Town A
Which curve indicates the exponential growth? (b) Town B
(a) A (b) B (c) Same in Town A and Town B
(c) C (d) D (d) No inference can be drawn

ST . 6
Sample Test : General Studies Paper II (2011)

27. What can we say about persons with more than relevant. This is so because childhood does not last. If
one disease from these graphs? a legal battle fought on behalf of a child is eventually
(a) There are likely to be persons with more won, it may be of little use to the boy or girl because
than one disease in Town A. the opportunity missed at school during childhood
(b) There are likely to be persons with more cannot serve the same purpose later in life. This may
than one disease in Town B. be painfully true for girls because our society permits
(c) There are likely to be persons with more them only a short childhood, if at all. The Right to
than one disease in both Towns A and B. Education (RTE) has become law at a point in India’s
(d) No inference can be drawn. history when the ghastly practice of female infanticide
has resurfaced in the form of foeticide. This is
28. Consider the following Velocity – Time graph. It “symptomatic of a deeper turmoil” in society which is
shows two trains starting simultaneously on compounding the traditional obstacles to the education
parallel tracks. of girls. Tenacious prejudice against the intellectual
potential of girls runs across our cultural diversity and
the system of education has not been able to address
it.
Velocity

29. With reference to the passage, consider the


B
ain following statements:
Tr 1. When children are denied education, adult
in A
Tra society does not act on behalf of them.
2. Right to Education as a law cannot be
enforced in the country.
@UPSC_THOUGHTS

Time
Which of the statements given above is/are
With reference to the above graph, which one of correct?
the following statements is not correct? (a) 1 only
(a) Train B has an initial acceleration greater (b) 2 only
than that of Train A. (c) Both 1 and 2
(b) Train B is faster than Train A at all times. (d) Neither 1 nor 2
(c) Both trains have the same velocity at
30. According to the passage, what could be the
time t0.
traditional obstacles to the education of girls?
(d) Both trains travel the same distance in
1. Inability of parents to fight a legal battle
time t0 units.
when the Right to Education is denied to
Directions for the following 5 (five) items: their children.
Read the following passage and answer the items that 2. The traditional way of thinking about girls’
follow. Your answers to these items should be based on role in society.
the passage only. 3. The prejudice against the intellectual
potential of girls.
Passage
4. Improper system of education.
Now India’s children have a right to receive at least
eight years of education, the gnawing question is Select the correct answer from the codes given
whether it will remain on paper or become a reality. below:
(a) 1 and 2 only
One hardly needs a reminder that this right is different
(b) 2, 3 and 4 only
from the others enshrined in the Constitution, that the
(c) 1, 3 and 4 only
beneficiary—a six year old child cannot demand it, nor
(d) 1, 2, 3 and 4
can she or he fight a legal battle when the right is
denied or violated. In all cases, it is the adult society 31. On the basis of the passage, consider the following
which must act on behalf of the child. In another statements:
peculiarity, where a child’s right to education is denied, 1. Right to Education is a legal right and not
no compensation offered later can be adequate or a fundamental right.

ST . 7
Sample Test : General Studies Paper II (2011)

2. For realising the goal of universal education, 36. Whose wife is the teacher?
the education system in the country must (a) C (b) D
be made identical to that of developed (c) A (d) B
countries.
Read the following passage and answer the 3 (three)
Which of the statements given above is/are items that follow:
correct? In a survey regarding a proposed measure to be
(a) 1 only introduced, 2878 persons took part of which 1652 were
(b) 2 only males. 1226 persons voted against the proposal of
(c) Both 1 and 2 which 796 were males. 1425 persons voted for the
(d) Neither 1 nor 2 proposal. 196 females were undecided.
32. Which one of the following statements conveys 37. How many females voted for the proposal?
the key message of the passage? (a) 430 (b) 600
(a) India has declared that education is (c) 624 (d) 640
compulsory for its children.
38. How many males were undecided?
(b) Adult society is not keen on implementing
(a) 31 (b) 227
the Right to Education. (c) 426 (d) 581
(c) The Right to Education, particularly of a
girl child, needs to be safeguarded. 39. How many females were not in favour of the
(d) The system of education should address proposal?
the issue of Right to Education. (a) 430 (b) 496
(c) 586 (d) 1226
@UPSC_THOUGHTS

33. Which one of the following statements conveys


the inference of the passage? 40. In a queue, Mr. X is fourteenth from the front and
(a) The society has a tenacious prejudice Mr. Y is seventeenth from the end, while Mr. Z
is exactly in between Mr. X and Mr. Y. If Mr. X
against the intellectual potential of girls.
is ahead of Mr. Y and there are 48 persons in
(b) Adults cannot be relied upon to fight on
the queue, how many persons are there between
behalf of children for their Right to
Mr. X and Mr. Z?
Education.
(a) 6 (b) 7
(c) The legal fight to get education for children
(c) 8 (d) 9
is often protracted and prohibitive.
(d) There is no sufficient substitute for Directions for the following 6 (six) items:
education received in childhood. Read each of the following two passages and answer the
items that follow. Your answers to these items should be
Read the following passage and answer the 3 (three) based on the passages only.
items that follow:
Passage-1
A, B, C, D and E are members of the same family. Ecosystems provide people with a variety of goods
There are two fathers, two sons, two wives, three males and services; food, clean water, clean air, flood control,
and two females. The teacher was the wife of a lawyer soil stabilization, pollination, climate regulation,
who was the son of a doctor. E is not a male, neither spiritual fulfilment and aesthetic enjoyment, to name
also a wife of a professional. C is the youngest person just a few. Most of these benefits either are irreplaceable
in the family and D is the eldest. B is a male. or the technology necessary to replace them is
34. How is D related to E? prohibitively expensive. For example, potable fresh
(a) Husband (b) Son water can be provided by desalinating sea-water, but
only at great cost.
(c) Father (d) Wife
The rapidly expanding human population has
35. Who are the females in the group? greatly modified the Earth’s ecosystems to meet their
(a) C and E (b) C and D increased requirements of some of the goods and services,
(c) E and A (d) D and E particularly food, fresh water, timber, fibre and fuel.

ST . 8
Sample Test : General Studies Paper II (2011)

These modifications have contributed substantially to Which of the statements given above is/are
human well being and economic development. The correct?
benefits have not been equally distributed. Some people (a) 1 only
have actually been harmed by these changes. Moreover, (b) 2 only
short-term increases in some ecosystem goods and (c) Both 1 and 2
services have come at the cost of the long-term (d) Neither 1 nor 2
degradation of others. For example, efforts to increase
Passage-2
the production of food and fibre have decreased the
ability of some ecosystems to provide clean water, A moral act must be our own act; must spring from
regulate flooding and support biodiversity. our own will. If we act mechanically, there is no moral
content in our act. Such action would be moral, if we
41. With reference to the passage, consider the
think it proper to act like a machine and do so. For,
following statements.
in doing so, we use our discrimination. We should bear
Expanding human population has an adverse in mind the distinction between acting mechanically
effect on: and acting intentionally. It may be moral of a king to
1. Spiritual fulfilment pardon a culprit. But the messenger carrying out the
2. Aesthetic enjoyment order of pardon plays only a mechanical part in the
3. Potable fresh water king’s moral act. But if the messenger were to carry out
4. Production of food and fibre the king’s order considering it to be his duty, his action
5. Biodiversity would be a moral one. How can a man understand
Which of the statements given above are correct? morality who does not use his own intelligence and
(a) 1, 2 and 3 only
@UPSC_THOUGHTS

power of thought, but lets himself be swept along like


(b) 2, 4 and 5 only a log of wood by a current? Sometimes a man defies
(c) 3 and 5 only convention and acts on his own with a view to absolute
(d) 1, 2, 3, 4 and 5 good.
42. The passage mentions that “some people have 44. Which of the following statements best describe/
actually been harmed by these changes”. What describes the thought of the writer?
does it imply?
1. A moral act calls for using our discretion.
1. The rapid expansion of population has
2. Man should react to a situation
adversely affected some people.
immediately.
2. Sufficient efforts have not been made to
3. Man must do his duty.
increase the production of food and fibre.
4. Man should be able to defy convention in
3. In the short term some people may be
order to be moral.
harmed, but in the long term everyone will
benefit from modifications in the Earth’s Select the correct answer from the codes given
ecosystems. below:
Which of the statements given above is/are (a) 1 only
correct? (b) 1 and 3
(a) 1 only (c) 2 and 3
(b) 2 (d) 1 and 4
(c) 1 and 3 45. Which of the following statements is the nearest
(d) None of the statements given above definition of moral action, according to the writer?
43. With reference to the passage, consider the (a) It is a mechanical action based on official
following statements: orders from superiors.
1. It is imperative to modify the Earth’s (b) It is an action based on our sense of
ecosystems for the well being of mankind. discretion.
2. Technology can never replace all the goods (c) It is a clever action based on the clarity of
and services provided by ecosystems. purpose.

ST . 9
Sample Test : General Studies Paper II (2011)

(d) It is a religious action based on under- 48. Consider the following figures:
standing.
46. The passage contains a statement “lets himself be 2 6 3 ?
swept along like a log of wood by a current.”
Among the following statements, which is/are 80 24 120 36
nearest in meaning to this?
1. A person does not use his own reason. What is the missing number?
2. He is susceptible to influence/pressure. (a) 7 (b) 8
3. He cannot withstand difficulties/ (c) 9 (d) 10
challenges. 49. Study the following figure:
4. He is like a log of wood.
A person goes from A to B always moving to the
Select the correct answer using the codes given right or downwards along the lines. How many
below: different routes can he adopt?
(a) 1 only A
(b) 1 and 2
(c) 2 and 3
(d) 1 and 4
47. Consider the following distance–time graph. The B
graph shows three athletes A, B and C running
Select the correct answer from the codes given
side by side for a 30 km race.
below:
@UPSC_THOUGHTS

(a) 4 (b) 5
(c) 6 (d) 7
50. Consider the following figure and answer the
item that follows:
Time in min.

Distance in km
What is the total number of triangles in the above
With reference to the above graph, consider the grid?
following statements: (a) 27 (b) 26
1. The race was won by A. (c) 23 (d) 22
2. B was ahead of A up to 25 km mark.
51. A person has only ` 1 and ` 2 coins with her.
3. C ran very slowly from the beginning.
If the total number of coins that she has is 50 and
Which of the statements given above is/are the amount of money with her is ` 75, then the
correct? number of ` 1 and ` 2 coins are, respectively
(a) 1 only (a) 15 and 35
(b) 1 and 2 only (b) 35 and 15
(c) 2 and 3 only (c) 30 and 20
(d) 1, 2 and 3 (d) 25 and 25

ST . 10
Sample Test : General Studies Paper II (2011)

52. Three persons start walking together and their A study at the University of Washington
steps measure 40 cm, 42 cm and 45 cm demonstrated the influence of Pisaster on species richness
respectively. What is the minimum distance each by removing sea stars from selected parts of the intertidal
should walk so that each can cover the same zone repeatedly over a period of five years. Two major
distance in complete steps? changes occurred in the areas from which sea stars
(a) 25 m 20 cm were removed. First, the lower edge of the mussel bed
(b) 50 m 40 cm extended farther down into the intertidal zone, showing
(c) 75 m 60 cm that sea stars are able to eliminate mussels completely
(d) 100 m 80 cm where they are covered with water most of the time.
Second, and more dramatically, 28 species of animals
53. If a bus travels 160 km in 4 hours, and a train
and algae disappeared from the sea star removal zone.
travels 320 km in 5 hours at uniform speeds, then
Eventually only Mytilus, the dominant competitor,
what is the ratio of the distances travelled by
occupied the entire substratum. Through its effect on
them in one hour?
competitive relationships, predation by Pisaster largely
(a) 8 : 5 (b) 5 : 8
determines which species live in these rocky intertidal
(c) 4 : 5 (d) 1 : 2
ecosystems.
54. There are 100 students in a particular class. 60%
56. What is the crux of the passage?
students play cricket, 30% students play football
(a) Sea star has a preferred prey.
and 10% students play both the games. What is
(b) A preferred prey determines the survival of
the number of students who play neither cricket
a keystone species.
nor football?
(c) Keystone species ensures species diversity.
(a) 25 (b) 20
@UPSC_THOUGHTS

(d) Sea star is the only keystone species on the


(c) 18 (d) 15 Pacific coast of North America.
55. A village having a population of 4000 requires 57. With reference to the passage, consider the
150 litres of water per head per day. It has a tank following statements:
measuring 20 m × 15 m × 6 m. The water of this 1. Mussels are generally the dominant species
tank will last for in intertidal ecosystems.
(a) 2 days (b) 3 days 2. The survival of sea stars is generally
(c) 4 days (d) 5 days determined by the abundance of mussels.
Directions for the following 4 (four) items: Which of the statements given above is/are
correct?
Read the following passage and answer the items that
(a) 1 only
follow. Your answers to these items should be based on
(b) 2 only
the passage only.
(c) Both 1 and 2
Passage (d) Neither 1 nor 2
A species that exerts an influence out of proportion 58. Which of the following is/are implied by the
to its abundance in an ecosystem is called a keystone passage?
species. The keystone species may influence both the 1. Mussels are always hard competitors for
species richness of communities and the flow of energy sea stars.
and materials through ecosystems. The sea star Pisaster 2. Sea stars of the Pacific coast have reached
ochraceus, which lives in rocky intertidal ecosystems on the climax of their evolution.
the Pacific coast of North America, is also an example 3. Sea stars constitute an important
of a keystone species. Its preferred prey is the mussel component in the energy flow in intertidal
Mytilus californianus. In the absence of sea stars, these ecosystem.
mussels crowd out other competitors in a broad belt of Which of the statements given above is/are
the intertidal zone. By consuming mussels, sea star correct?
creates bare spaces that are taken over by a variety of (a) 1 and 2 (b) 2 only
other species. (c) 1 and 3 (d) 3 only

ST . 11
Sample Test : General Studies Paper II (2011)

59. Consider the following assumptions: (a) C and D live on the same street.
1. The food chains/food web in an ecosystem (b) C’s house faces south.
are influenced by keystone species. (c) The houses of C and D are less than 20 km
2. The presence of keystone species is a specific apart.
characteristic of aquatic ecosystems. (d) None of the above
3. If the keystone species is completely
Directions for the following 9 (nine) items:
removed from an ecosystem, it will lead to
the collapse of the ecosystem. The following nine items are based on three passages
in English to test the comprehension of English language
With reference to the passage, which of the above and therefore these items do not have Hindi version.
assumptions is/are valid? Read each passage and answer the items that follow.
(a) 1 only
(b) 2 and 3 only Passage-I
(c) 1 and 3 only He walked several miles that day but could not get
(d) 1, 2 and 3 anything to eat or drink except some dry bread and
60. Consider the following argument: some water, which he got from cottagers and farmers.
As night fell, he slept under a haystack lying in a
“In order to be a teacher one must graduate from meadow. He felt frightened at first, for the wind blew
college. All poets are poor. Some Mathematicians awfully over the empty fields. He felt cold and hungry,
are poets. No college graduate is poor.”
and was feeling more lonely than he had ever felt
Which one of the following is not a valid before. He, however, soon fell asleep, being much tired
conclusion regarding the above argument? with his long walk. When he got up next day, he was
@UPSC_THOUGHTS

(a) Some Mathematicians are not teachers. reeling terribly hungry so he purchased a loaf of bread
(b) Some teachers are not Mathematicians. with a few coins that he had.
(c) Teachers are not poor.
64. When the night fell, he slept
(d) Poets are not teachers.
(a) in the open field
61. A student on her first 3 tests received an average (b) under a pile of dry grass
score of N points. If she exceeds her previous (c) in a farmer’s cottage
average score by 20 points on her fourth test, then (d) under a tree
what is the average score for the first 4 tests?
(a) N + 20 (b) N + 10 65. He soon fell asleep because
(c) N + 4 (d) N + 5 (a) he was exhausted
(b) he was all alone
62. In a group of persons, 70 per cent of the persons
(c) he had not slept for days
are male and 30 per cent of the persons are
(d) he was very frightened
married. If two-sevenths of the males are married,
what fraction of the females is single? 66. With reference to the passage, consider the
2 1 following statements:
(a) (b) 1. He was walking through the countryside.
7 3
2. The cottagers and farmers gave him enough
3 2
(c) (d) food so that he could sleep at night without
7 3
feeling hungry.
63. The houses of A and B face each other on a road
going north-south, A’s being on the western side. Which of the statements given above is/are
A comes out of his house, turns left, travels 5 km, correct?
turns right, travels 5 km to the front of D’s house. (a) 1 only
B does exactly the same and reaches the front of (b) 2 only
C’s house. In this context, which one of the (c) Both 1 and 2
following statements is correct? (d) Neither 1 nor 2

ST . 12
Sample Test : General Studies Paper II (2011)

Passage-II 70. Female polar bears give birth during


(a) spring
I opened the bag and packed the boots in; and then,
(b) summer
just as I was going to close it, a horrible idea occurred
(c) autumn
to me. Had I packed my toothbrush? I don’t know how
(d) winter
it is, but I never do know whether I’ve packed my
toothbrush. 71. Mother bear
My toothbrush is a thing that haunts me when I’m (a) takes sides over cubs
travelling, and makes my life a misery. I dream that I (b) lets the cubs fend for themselves
haven’t packed it, and wake up in a cold perspiration, (c) feeds only their favourites
(d) sees that all cubs get an equal share
and get out of bed and hunt for it. And, in the morning,
I pack it before I have used it, and it is always the last 72. With reference to the passage, the following
thing I turn out of the bag; and then I repack and forget assumptions have been made:
it, and have to rush upstairs for it at the last moment 1. Polar bears fast as long as eight months
and carry it to the railway station, wrapped up in my due to non-availability of prey.
pocket-handkerchief. 2. Polar bears always give birth to triplets.

67. When he was going to close the bag, the idea that Which of the assumptions given above is/are
occurred to him was valid?
(a) unpleasant (a) 1 only
(b) sad (b) 2 only
(c) Both 1 and 2
(c) fantastic
@UPSC_THOUGHTS

(d) Neither 1 nor 2


(d) amusing
Directions for the following 8 (eight) items:
68. What makes his life miserable whenever he
undertakes travelling? Given below are eight items. Each item describes a
(a) Going to railway station situation and is followed by four possible responses.
Indicate the response you find most appropriate. Choose
(b) Forgetting the toothbrush
only one response for each item. The responses will be
(c) Packing his bag
evaluated based on the level of appropriateness for the
(d) Bad dreams
given situation.
69. His toothbrush is finally Please attempt all the items. There is no penalty for
(a) in his bag wrong answers for these eight items.
(b) in his bed
(c) in his handkerchief 73. You have been asked to give an explanation for
(d) lost not attending an important official meeting. Your
immediate boss who has not informed you about
Passage-III the meeting is now putting pressure on you not
In spring, polar bear mothers emerge from dens to place an allegation against him/her. You
with three month old cubs. The mother bear has fasted would
for as long as eight months but that does not stop the (a) send a written reply explaining the fact.
young from demanding full access to her remaining (b) seek an appointment with the top boss to
explain the situation.
reserves. If there are triplets, the most persistent stands
(c) admit your fault to save the situation.
to gain an extra meal and it may have the meal at the
(d) put the responsibility on the coordinator of
expense of others. The smallest of the litter forfeits many
the meeting for not informing.
meals to stronger siblings. Females are protective of
their cubs but tend to ignore family rivalry over food. 74. A local thug (bad element) has started illegal
In 21 years of photographing polar bears, I’ve only once construction on your vacant plot. He has refused
seen the smallest of triplets survive till autumn. your request to vacate and threatened you of dire

ST . 13
Sample Test : General Studies Paper II (2011)

consequences in case you do not sell the property 78. You are the officer-in-charge of a village
at a cheap price to him. You would administering distribution of vaccine in an
(a) sell the property at a cheap price to him. isolated epidemic hit village, and you are left
(b) go to the police for necessary action. with only one vaccine. There is a requirement of
(c) ask for help from your neighbours. that vaccine from the Gram Pradhan and also a
(d) negotiate with the goon to get a higher poor villager. You are being pressurised by the
price. Gram Pradhan to issue the vaccine to him. You
would
75. You have to accomplish a very important task for
(a) initiate the procedure to expedite the next
your Headquarters within the next two days.
supply without issuing the vaccine to either.
Suddenly you meet with an accident. Your office
(b) arrange vaccine for the poor villager from
insists that you complete the task. You would
the distributor of another area.
(a) ask for an extension of deadline.
(c) ask both to approach a doctor and get an
(b) inform Headquarters of your inability to
input about the urgency.
finish on time.
(d) arrange vaccine for the Gram Pradhan from
(c) suggest alternate person to Headquarters
the distributor of another area.
who may do the needful.
(d) stay away till you recover. 79. You have taken up a project to create night-
shelters for homeless people during the winter
76. You are an officer-in-charge for providing basic
season. Within a week of establishing the shelters,
medical facilities to the survivors of an earthquake
you have received complaints from the residents
affected area. Despite your best possible effort,
of the area about the increase in theft cases with
people put allegations against you for making
@UPSC_THOUGHTS

a demand to remove the shelters. You would


money out of the funds given for relief. You
(a) ask them to lodge a written complaint in
would
the police station.
(a) let an enquiry be set up to look into the
(b) assure residents of an enquiry into the
matter.
matter.
(b) ask your senior to appoint some other
(c) ask residents to consider the humanitarian
person in your place.
effort made.
(c) not pay attention to allegations.
(d) continue with the project and ignore their
(d) stop undertaking any initiative till the
complaint.
matter is resolved.
80. You, as an administrative authority, have been
77. You have been made responsible to hire boats at
approached, by the daughter-in-law of an
a short notice to be used for an area under flood.
influential person regarding harassment by her
On seeing the price mentioned by the boat owners
in-laws on account of insufficient dowry. Her
you found that the lowest price was
parents are not able to approach you because of
approximately three times more than the approved
social pressures. You would
rate of the Government. You would
(a) call the in-laws for an explanation.
(a) reject the proposal and call for a fresh price.
(b) counsel the lady to adjust, given such a
(b) accept the lowest price.
circumstance.
(c) refer the matter to the Government and
(c) take action after her parents approach you.
wait.
(d) ask her to lodge a complaint with the
(d) threaten the boat owners about a possible
police.
cancellation of the licence.

ST . 14
Sample Test : General Studies Paper II (2011)

PRACTICE ANSWER SHEET

Directions: Use HB Pencil. Erase completely to change. Example

1. 17. 33. 49. 65.

2. 18. 34. 50. 66.

3. 19. 35. 51. 67.

4. 20. 36. 52. 68.

5. 21. 37. 53. 69.

6. 22. 38. 54. 70.


@UPSC_THOUGHTS

7. 23. 39. 55. 71.

8. 24. 40. 56. 72.

9. 25. 41. 57. 73.

10. 26. 42. 58. 74.

11. 27. 43. 59. 75.

12. 28. 44. 60. 76.

13. 29. 45. 61. 77.

14. 30. 46. 62. 78.

15. 31. 47. 63. 79.

16. 32. 48. 64. 80.

ST . 15
Sample Test : General Studies Paper II (2011)

ANSWERS AND EXPLANATORY NOTES

1. (b) Read the sentence, ‘I suppose the poor and [If you don’t remember the formula or don’t
unfortunate…’ to which ‘them’ refers. The ‘poor know it, you can still solve the problem by
and unfortunate’ may be considered as people adding up the penalty amounts for ten days.
living in ‘starvation and misery’. Day 1 ` 200
[Elimination Method (EM): Locate the part of
Day 2 ` 250
the passage from which the quoted words
come. It is clear that only statement 2 is Day 3 ` 300
applicable; 1 is not mentioned, and 3 is Day 4 ` 350
mentioned as an alternative to being ‘other Day 5 ` 400
worldly’. So reject any answer response that Day 6 ` 450
has 1 or 3 in it; you are left with (b).] Day 7 ` 500
2. (c) Assumption 1 is not supported by the passage Day 8 ` 550
which says that spiritual or any other ‘greatness’ Day 9 ` 600
cannot be achieved in a condition of lack of
Day 10 ` 650
freedom and opportunity—a lack that can be
brought about by foreign domination. However, Total ` 4250
one cannot, therefore, assume that a country
under foreign domination cannot ‘indulge’ in The total penalty for ten days is ` 4250]
spiritual pursuit. So assumption 1 is to be
rejected as incorrect. Refer to the sentence, 10. (c) Remember the laws of reflection.
@UPSC_THOUGHTS

‘Spiritual or ... misery’. It is clear that assumption


2 is correct. Assumption 3 is supported by
the sentence, ‘I suppose the poor … for them.’ S
3. (a) The passage mentions ‘starvation and misery’
and ‘civilizations’ but they do not form the
theme of the passage. The second paragraph
mentions the search of humans for inner
meaning, but this is not to say that the theme 11. (a)
of the passage is ‘body, mind and spirit of R Y
B
people in general’.
R Y R
4. (c) Refer to the last two sentences, “Where they Y B Y = Yellow
meet … the mind and the spirit.” Y Y R B = Blue
5. (a) Y
R Y R = Red
6. (b) R

7. (c) Y R
8. (a) 4 × 6 = 24
[However, if all circles had sections, more
9. (b) It is a case of arithmetic progression (AP).
number of colours would be needed under the
If Sn is the sum of n terms, with the first term
same conditions.]
being a and the difference between terms
being d, and l being the last term, 12. (b) Now the rectangle of area 15 may have sides
n of 15 and 1, or 5 and 3. The rectangle with
Sn = [2a + (n – 1) d ] area 48 may have sides of 48 and 1, 24 and
2
2, 12 and 4, 16 and 3, or 6 and 8. However,
10
= [2 × 200 + (10 – 1) 50] we need sides for the two rectangles to add
2
= 5 (400 + 9 × 50) up to the sides of the square of which they
= 5 (400 + 450) are parts; as the sides of a square are equal,
= 5 × 850 = ` 4250 the sides of the two given rectangles must

ST . 16
Sample Test : General Studies Paper II (2011)

add up to give the same sum. This happens 16. (d) Neither statement is either mentioned or implied
only if we take the sides to be in the passage.
5 × 3 = 15 17. (d) Response (a) is mentioned in the passage, but
and 6 × 8 = 48 it is not the ‘essential message’ sought to be
Now 5 + 6 = 11 conveyed. Responses (b) and (c) are not
and 3 + 8 = 11 stated as such in the passage, nor are they
the main message.
5 6 18. (c) Statement 1 is not mentioned or implied in the
3 15 3 passage. Statement 2 is not correct as the
passage clearly defines ‘creative society’ as
a phase of development in society in which
the oppressed begin to demand their rights,
8 48 8
so it cannot be a society in which social
inequalities are accepted as a norm. It is the
movements for social transformation—not the
status quo of inequalities—that indicates the
5 6 ‘creative society’. Statements 3 and 4 are
So, the length of each side of the square expressed in the passage.
is 11. [EM: Once you realise statements 1 and 2
13. (a) Only Statements 1 and 2 are correct. Statement are wrong, you may reject answer responses
1 is clear from paragraph 2. Statement 2 is (a) and (d). As you note statement 3 to be
implied in paragraph 3. Statement 3 contradicts correct, you are left with only (c) as the correct
what is said in the passage: ‘Hence we need answer response.]
@UPSC_THOUGHTS

… minimal interference …’ (paragraph 3). The 19. (c) Statements 1 and 2 find no mention, directly
passage says that discussion about the size or by implication, in the passage. Statement
of the government is misleading; it nowhere 3 is supported by the passage: “Oppressed
mentions the need to change the size of the social groups … rights”. Statement 4 is implied
government. So statement 4 is wrong. in the movements seeking social
14. (d) See last section of paragraph 3: ‘basic education transformation. So statements 3 and 4 are
and health services’, ‘adequate food and correct.
nutrition’ are ‘basic services’ that the [EM: As soon as you note that statements
government needs to deliver to the poor. 1 and 2 are wrong, you may safely reject any
Response (a) contradicts what the passage response that includes either—such as (a), (b)
says in paragraph 1. Response (b) is not and (d). You are left with (c) which is the
correct as the passage does not mention correct answer.]
increasing regulations, but talks of ‘effective,
incentive-compatible rules’. An ‘enabling ethos’ 20. (a) The last sentence of the passage supports
is not compatible with increased regulations. statement 1. Statement 2 seems at first sight
Response (c) is contradicted by paragraph 1 to be substantiated by the first sentence of
and paragraph 3. the passage; however, it is not just the
potential contradictions and conflicts in a
15. (c) Statement 1 is contradicted by the passage society but their becoming articulate and
in paragraph 1. A ‘cumbersome’ bureaucracy
active that leads to a creative society. So
is the result of a government that controls all
statement 2 is not acceptable. Answer response
manufacture and delivery of goods. Statement
(a) is correct.
2 is not implied or mentioned in the passage.
Statements 3, 4 and 5 are what paragraph 3 21. (c) This is a question involving logical reasoning.
says. Statement 1 means no one, but students can
participate in the race or that all participants
[EM: As soon as you realise statements 1 and
in the race must be (or are) students. Straight-
2 are wrong, you may reject answer responses
(a) and (d). As soon as you check out away answer response (c) is the correct one.
statement 3 to be correct, you are left with 22. (a) Statements 1 and 3 contradict each other, so
answer response (c) being the correct one.] both cannot be true at the same time.

ST . 17
Sample Test : General Studies Paper II (2011)

However, both can be false, if ‘some animals’ system of education has not been able to
are found to be ‘not carnivorous’ and ‘some address it”.
animals’ are found to be ‘carnivorous’. [EM: As soon as you noted that statement
Statements 1 and 2 are contradictory and 1 is incorrect, you can reject (a), (c) and (d)
cannot both be true, but both statements and choose (b) as the correct answer response.]
cannot be false.
31. (d) While the passage conveys to us that the
If ‘some animals’ are found to be ‘carnivorous’, Right to Education is a law that can be fought
statement 2 is not rendered false. So (b) is for, the fact that it mentions it as a right
not a correct answer response. Statements 2 enshrined in the Constitution indicates that it
and 3 can be both true at the same time, so is more than a mere legal right, and that it
do not meet the required condition. Answer is a fundamental right. Statement 1 is not
response (c) is not correct. Statements 3 and correct. Statement 2 is neither stated nor
4 cannot both be true, but statement 4 need implied in the passage. So neither statement
not be false, even if ‘some animals’ are ‘not is correct.
carnivorous’. So (d) is also not correct. 32. (c) While (a) is indicated, it is not the ‘message’
23. (c) The same kind of reasoning as in Q. 22 is of the passage. Response (b) is nowhere
to be applied here. Statements 1 and 3 cannot stated in the passage. Nor is (d) indicated.
both be true. However, they can both be false 33. (d) While (a) and (c) are mentioned, neither is a
if ‘some trains’ are found not to be run by general inference; they go to show how
diesel engine or ‘some trains’ are found to be opportunity is missed in childhood. Statement
run by diesel engine. (b) is not mentioned in the passage.
24. (c) For items 34 to 36, you need to analyse the data
@UPSC_THOUGHTS

25. (c) Exponential growth is geometric growth where and make a rough diagram of the relationships. We
are given that D is the oldest and that there are three
each increment is twice the size of the last.
males of whom two are fathers. B is a male. E is
Curve C is nearest to this expression. So
a female who is not the wife of a professional so the
answer response (c) is correct.
teacher cannot be E. Nor can the teacher be D as
26. (d) As we have not been given the total number she is married to the lawyer who is the son of a doctor,
of persons in each town, we cannot say which and thus cannot be the oldest member. So we may
town has a higher number of persons with conclude D is a male, as there are only two females
diabetes. in the group. E could be the doctor married to D. As
27. (b) The segments in the pie chart for Town A add there are two fathers (given) and three males, C the
up to 100. The segments in the pie chart for youngest must be a male—the son of the lawyer and
the teacher. As B is a male, the lawyer must be B.
Town B, however, add up to 121. So there
So the teacher must be A.
are likely to be persons with more than one
disease in Town B. [Though it is somewhat
E m D
odd for a pie chart with percentage segments
to add up to more than 100.] (female, doctor) (oldest, male)
28. (d)
29. (d) Statement 1 is not valid in the light of the
passage which merely says adult society must
A m B
act on the behalf of the child. The reference
to a legal battle fought on behalf of the child
(teacher, female) (lawyer, male)
indicates that the Right to Education as a law
can be enforced. So neither statement is
correct. C
30. (b) Statement 1 is not indicated in the passage; (youngest, male)
furthermore, the statement refers to ‘children’
34. (a)
and not ‘girls’. Statements 2 and 3 are stated
in the last two sentences of the passage, and 35. (c)
statement 4 is implied in the words “the 36. (d)

ST . 18
Sample Test : General Studies Paper II (2011)

For items 37 to 39, draw a chart and plug in the data, rapidly expanding population that has modified
deriving the missing figures. the ecosystems has adversely affected some
Total males females people. So statement 1 is implied; answer
response (d) may be rejected. Answer response
Persons 2878 1652 (2878 – 1652)
who took part = 1226 (b) may also be rejected as it does not include
1. Now check out statement 3. It is not
Voted 1226 796 (1226 – 796)
against = 430 substantiated by the passage which points out
that short-term benefits are at the cost of long-
Voted for 1425 (1425 – 600) (1226 – 430 – 196)
= 825 = 600 term degradation. So the correct answer
response is (a). However, for your own
Undecided (2878 – 1226 – (227 – 196) 196
1425) = 227 = 31
satisfaction check out statement 2; it is not
supported by the passage.
[All figures in bold are derived.]
43. (d) Statement 1 is not supported by the passage;
37. (b)
the passage says modifications of the Earth’s
38. (a)
ecosystems have been made to provide certain
39. (a)
40. (c) X is 14th from the front and there are 48 benefits—so for the well-being of mankind—
persons in the queue. If Y is 17th from the but it does not suggest that it is imperative
end, this position is 48 – 17 = 31st from front. to modify the ecosystem. So answer responses
So the number of persons between X and Y (a) and (c) may be rejected. Answer response
will be 31 – 14 = 17. Z is in mid-position (b) is not correct. Statement 2 is too categorical
between X and Y. to be accepted—we do not know, and the
@UPSC_THOUGHTS

8 8 passage does not say so—if technology can


never replace the goods produced by eco-
X Y system.
Z
There will be 8 persons between X and Z. 44. (d) Statement 1 is supported by the passage. An
act would be moral if it has been the outcome
41. (c) The passage says ecosystems provide people of conscious decision after considering its
with spiritual fulfilment, but does not mention suitability or rightness. Both the words
spiritual fulfilment in the context of adverse ‘discrimination’ and ‘discretion’ convey this
effects of expanding human population. Nor is meaning. There is nothing in the passage to
aesthetic enjoyment mentioned in this context. support statement 2. In fact, if a moral act
So (1) and (2) are not correct. The last involves using one’s discrimination, thought
sentence of the passage gives us the answer: becomes necessary, and an immediate reaction
the adverse effects being on clean water, to a situation is impulsive rather than reasoned.
control of floods and support to biodiversity. Statement 3 sounds correct, but the writer
So only 3 and 5 are correct. Production of considers duty to be a moral act only if the
food and fibre (4) is not an adverse effect. performer thinks about it. There is no indication
The correct answer response is (c). that the writer thinks man must do his duty.
[EM: As soon as you note (1) and (2) are not So statement 3 is not correct. Statement 4
correct, you can eliminate answer responses is implied in the last sentences of the
(a), (b) and (d).] passage. The writer seems to think that man
should have the ability to defy convention in
42. (a) This is a tricky item. According to the
the interests of morality. So statements 1 and
passage, the expanding human population has
4 are correct, which is answer response (d).
caused changes in the ecosystems and it is
these changes that have harmed some people. 45. (b) This is clear from the passage. Response (a)
On the face of it, therefore, statement 1 seems contradicts what is said in the passage.
wrong. However, the question is “What does Responses (c) and (d) are not mentioned in
it imply?” By implication, or indirectly, the the passage.

ST . 19
Sample Test : General Studies Paper II (2011)

46. (b) Both statements 1 and 2 explain the assertion. 50. (c) Label the triangles and count carefully without
Being swept along implies not using one’s own missing any and without counting any one
more than once.
reason. Being swept by a current implies the
A
unreasoned susceptibility to influence or
pressure: just as a log of wood is susceptible
B C
to the influence of a current. There is nothing
in the passage to support statement 3. As for F
D
statement 4, the comparison with a log of E

wood is made only in the context of being G J


H I
swept along by a current and not in every way.
So statement 4 is not acceptable.
K L M N O
47. (b) The question requires an understanding of The triangles are
reading graphs. Winning the race means 1 (AKO)
reaching the end point in the shortest time.
+ 12 (ABC, BDE, BEC, CEF, DGH, DEH,
The end point is on the X-axis—30 km. From EHI, EFI, FIJ, HLM, HIM, IMN)
the reading on the Y-axis, the time taken by + 7 (ADF, BGI, CHJ, DKM, FMO, DFM, ELN)
A to reach the 30 km point is least compared
+ 3 (AGH, BKN, CLO)
to B and C. So statement 1 is correct.
Total 23
Statement 2 is correct as the time taken by 51. (d) Let the number of ` 1 coins be x.
B to reach 25 km is less than that taken by Then the number of ` 2 coins = 50 – x
A to reach the same distance. If we look at
@UPSC_THOUGHTS

From the given data, we can say


the curve for C, it is noticeable that C reached
x + 2 (50 – x) = ` 75
20 km much earlier than either A or B, so
C ran very fast in the beginning. Statement Þ x + 100 – 2x = 75
3 is therefore not correct. Þ x = 25
Number of ` 1 coins = 25
48. (c) Establish the relationship between the numbers.
Number of ` 2 coins = 50 – 25 = 25
2 × 40 = 80
52. (a) We must find the L.C.M. of 40, 42 and 45.
6 × 4 = 24
Factorise each number
Similarly,
3 × 40 = 120 40 ® 2 × 2 × 2 × 5
? × 4 = 36 42 ® 2 × 3 × 7
Clearly, 9 replaces the (?) mark. 45 ® 3 × 3 × 5
L.C.M. = 2 × 2 × 2 × 3 × 3 × 5 × 7
49. (c) A 1 2 = 2520
The required minimum distance is 2520 cm
or 25 m and 20 cm
5 3 53. (b) The speed is uniform for the bus and train
4
Distance travelled by bus in 1 hour
B 160
6 7 = = 40 km
4
The routes under the given conditions are: Distance travelled by train in 1 hour
1. A 1 2 3 B 320
= = 64 km
2. A 1 4 3 B 5
3. A 1 7 B Ratio of distances travelled by bus and train is
4. A 5 6 7 B 40 : 64
5. A 5 4 7 B 40 5
6. A 5 4 3 B or = or 5 : 8
64 8

ST . 20
Sample Test : General Studies Paper II (2011)

54. (b) 60. (b) If you represent the statements


Cricket Football
diagrammatically, it will become clear. The first
sentence means: All teachers are graduates
60 10 30
from college. It is given that no graduate is
poor, so the classes of graduates and poor
U = 100 are exclusive of each other. So we have
Students who play only football G = graduates
1
= 30 – 10 = 20
M Pr T = teachers
Students who play only cricket G
= 60 – 10 = 50 T Pt
M
Pt = poets
Students who play both = 10
Number of students who play neither cricket 3
M Pr = poor people
nor football 2
= 100 – (50 + 20 + 10) M = mathematicians
= 100 – 80
We can see that teachers are not poor (as
= 20
all teachers are graduates and no graduate is
55. (b) Water required by 4000 persons per day
= 4000 × 150 = 600,000 litres poor). So answer response (c) is a valid
Volume of water conclusion. As all poets are poor, they are
= 600 m3 outside the class of graduates and teachers.
Volume of the tank So poets are not teachers; answer response
= 20 × 15 × 6 = 1800 m3 (d) is a valid conclusion. The given statements
do not specifically say how mathematicians
@UPSC_THOUGHTS

The number of days the tank of water will last


are related to teachers. But under the given
1800 m3
= = 3 days conditions—as the diagram shows, some
600 m3
mathematicians are not teachers, however that
56. (c) Answer response (a) is true but it is not the
class is distributed. So answer response (a)
crux of the passage. Answer response (b) and
is a valid conclusion. So, obviously answer
(d) are not supported by the passage.
response (b) is not a valid conclusion. Check
57. (d) Nothing in the passage indicates that mussels it out: from circles 1 and 2 showing the
are ‘generally’ the dominant species in intertidal position of mathematicians, some teachers are
ecosystems; we are told they are the dominant not mathematicians but circle 3 shows it is
competitors in the absence of the sea stars possible for some teachers to be
that feed on them. So statement 1 is not mathematicians. So it is not valid to conclude
correct. Statement 2 is not supported by the categorically that some teachers are not
passage which merely says mussels are the mathematicians.
preferred prey of sea stars. So statement 2
is incorrect. Answer response (d) is correct. 61. (d) Total score in the first three tests = 3N
Score in the fourth test = N + 20
58. (d) Statements 1 and 2 are not supported by the
Average score for the first four tests
passage. Statement 3 is supported by the
3N + N + 20
second sentence of the passage. =
4
59. (a) Statement 1 is supported by the entire passage. 4N 20
= + = N + 5
Statement 2 is not indicated in the passage. 4 4
The example of a keystone species just
[Alternative Method: The extra marks received
happens to belong to an aquatic ecosystem.
in the fourth test is 20. All you have to do
The removal of a keystone species is shown
is find how these marks are to be averaged
to drastically change an ecosystem, but the
for four tests and add it to the average of the
passage does not mention the collapse of an 20
ecosystem. So statement 3 is not correct. first three tests. So, = 5; and new average
4
Answer response (a) is correct. is N + 5.]

ST . 21
Sample Test : General Studies Paper II (2011)

62. (d) Let the number of persons in the group So statement 1 is correct. Statement 2
be 100 contradicts what is said in the passage: ‘He
Males number = 70 felt cold and hungry….’
Females number = 30
Married number = 30 67. (a) A ‘horrible idea…’ which clearly implies
2 something ‘unpleasant’ rather than ‘sad’ or
Married males = × 70 = 20
7 ‘fantastic’. It certainly does not imply ‘amusing’.
So, married females = 30 – 20 = 10
68. (b) Refer to the sentence, ‘My toothbrush …
Single females = 30 – 10 = 20
misery.’ The entire passage is about the
20 2
The required fraction = = writer’s fears about forgetting his toothbrush
30 3
[We have assumed that no male in the group when setting out on his travels.
is married to any female from the same group.] 69. (c) Refer to ‘wrapped … handkerchief’ at the end
63. (c) Sketch a diagram based on the given data. of the passage.
D 70. (d) As the polar bear mother emerges in spring
with cubs that are three months old, the birth
5 km must have taken place in winter.
B
71. (b) Answer responses (a), (b) and (c) are not
N S correct, as we are told that the female bears
5 km 5 km
tend to ignore family rivalry over food. So (b)
A
5 km seems to be the best response, as ‘fend for
@UPSC_THOUGHTS

themselves’ means ‘look after themselves


without any help from anyone’.
C
It is not possible that C and D live on the 72. (d) The passage does not indicate why the polar
same street. Answer response (a) is not bears fast, though it is likely to be due to
correct. C’s house may face north, south or hibernation over the winter months rather than
east; we are just told that B travels to reach lack of prey. So statement 1 is not valid. The
the front of C’s house. So answer response phrase ‘If there are triplets …’ indicates that
(b) is not certain. As for the distance between there is no certainty about polar bears always
the houses of C and D, it is the sum of the giving birth to triplets. So statement 2 is also
hypotenuses of two isosceles right-angled not valid.
triangles. We know that in such a triangle the
hypotenuse 73. (d) The question tests your interpersonal skills.
c = a 2 where a is the side. Clearly, your boss does not want to accept
responsibility for his lapse. However, it would
Here c = 5 2 not be tactful on your part to blame your boss
The sum of the hypotenuses = 2c = 2 ( 5 2 ) in a written explanation which is quite a serious
which is less than 20. matter, so response (a) is not appropriate.
So the distance between the houses of C and Response (b) seems an over-reaction, and is
D is less than 20 km. Answer response (c) for much more serious situations. Response
is correct. (c) would not only be a falsity, it would also
64. (b) He slept under a haystack which is a ‘pile of set a wrong precedent and indicate that you
dry grass’. can be bullied into accepting blame
undeservedly to be in the good books of your
65. (a) Refer to the sentence, “He, however, soon fell
boss. Response (d) is the best of the given
asleep, being much tired …”. ‘Exhausted’
responses. It may seem unethical at first
means ‘very tired’.
sight, but it is the responsibility of the
66. (a) If the place has farmers, cottagers, meadow coordinator to inform you (whether or not your
and haystacks, clearly it is the countryside. boss has informed you), especially if the

ST . 22
Sample Test : General Studies Paper II (2011)

meeting is important enough for your absence them. Response (d) is again defeatist and
to be noted. shows a lack of commitment to work at hand.

74. (b) It is the correct action to take the help of the 77. (b) The area is under flood and needs quick
law in the circumstances. Answer response (a) attention. In the circumstances, answer
shows you to be easy prey to threats and responses (a) and (c) would cause unaffordable
shows a defeatist attitude. Asking neighbours delay. Answer response (d) shows you as a
to help is unrealistic and of little use. So bully willing to abuse your power and position.
Besides, when the market is free, you can
response (c) is not acceptable. Response (d)
bargain, but not threaten the boat owners over
would show you are unscrupulous and willing
prices.
to adopt illegal means yourself besides
encouraging the goons. 78. (c) This is a fair and practical decision. Both (b)
and (d) are unfair to one or the other person.
75. (b) Informing headquarters of your inability to Response (a) implies withholding the vaccine
finish on time—response (b)—is fine even perhaps from one who is in dire need of it,
though it seems somewhat passive. Response and indicates that you want to play safe at
(c) shows enterprise but it might be felt by all costs.
headquarters as overstepping your brief. The
79. (b) It would be best to first enquire into the matter
work needs to be accomplished within the next
to see if the complaints are justified before
two days, so there is no point in asking for
responding with (a). Response (c) is somewhat
an extension of deadline [response (a)].
futile, as you cannot expect people to overlook
Response (d) shows irresponsibility and is the security because of a humanitarian cause.
@UPSC_THOUGHTS

worst response. Response (d) shows high handedness and a


76. (a) If you have worked with integrity and put in callousness towards the residents.
your best efforts, you should welcome an 80. (d) You could choose response (a) but it depends
inquiry so that your name can be cleared and on whether you have the authority to call for
the people who are putting allegations against an explanation in the matter. The correct
you are silenced, even as the public at large procedure is to lodge a complaint with the
becomes aware of the facts. Answer response police. Response (b) is not correct; besides
(b) is defeatist and escapist, showing an showing the age-old bias in family relationships,
unwillingness to face challenges. If just one it implies connivance in the illegal practice of
or two persons are putting the allegations, you harassment for dowry, and ignoring a call for
could choose response (c), but even so one help may lead to great harm. As for (c) it
should try to clear up things rather than ignore appears to be a delaying tactic.

ST . 23
Sample Test 2 : General Studies Paper II (2012)

SAMPLE TEST 2: GENERAL STUDIES PAPER II (2012)


Directions for the following 11 (eleven) items: numerous species of plants, invertebrates and vertebrates
found in continental Europe but absent from the British
Read the following three passages and answer the items
Isles (many because they have so far failed to recolonise
that follow each passage. Your answers to these items should
after the last glaciations). Their introduction would be
be based on the passages only.
likely to augment British biodiversity. The significant
detrimental effect noted above arises where aggressive
Passage-1 species provide a novel challenge to endemic biotas ill-
Invasions of exotic species into new geographic equipped to deal with them.
areas sometimes occur naturally and without human
agency. However, human actions have increased this 1. With reference to the passage, which of the
trickle to a flood. Human-caused introductions may following statements is correct?
occur either accidentally as a consequence of human (a) Introduction of exotic species into new
transport, or intentionally but illegally to serve some geographical areas always leads to reduced
private purpose or legitimately to procure some hoped- biodiversity.
for public benefit by bringing a pest under control, (b) Exotic species introduced by man into new
producing new agricultural products or providing novel areas have always greatly altered the native
recreational opportunities. Many introduced species are ecosystems.
@UPSC_THOUGHTS

assimilated into communities without much obvious (c) Man is the only reason to convert a hugely
effect. However, some have been responsible for dramatic diverse range of local community
changes to native species and natural communities. For compositions into more homogeneous ones.
example, the accidental introduction of the brown tree (d) None of the statements (a), (b) and (c) is
snake Boiga irregularis into Guam, an island in the correct in this context.
Pacific, has through nest predation reduced 10 endemic 2. Why does man introduce exotic species into new
forest bird species to the point of extinction. geographical areas?
One of the major reasons for the world’s great 1. To breed exotic species with local-varieties.
biodiversity is the occurrence of centers of endemism 2. To increase agricultural productivity.
so that similar habitats in different parts of the world 3. For beautification and landscaping.
are occupied by different groups of species that happen Which of the above statements is/are correct?
to have evolved there. If every species naturally had (a) 1 only (b) 2 and 3 only
access to everywhere on the globe, we might expect a (c) 1 and 3 only (d) 1, 2 and 3
relatively small number of successful species to become
3. How is homogenization prevented under natural
dominant in each biome. The extent to which this
conditions?
homogenization can happen naturally is restricted by
(a) Evolution of groups of species specific to
the limited powers of dispersal of most species in the
local habitats.
face of the physical barriers that exist to dispersal. By
(b) Presence of oceans and mountain ranges.
virtue of the transport opportunities offered by humans,
(c) Strong adaptation of groups of species to
these barriers have been breached by an ever-increasing
local physical and climatic conditions.
number of exotic specie. The effects of introductions
(d) All the statements (a), (b) and (c) given
have been to convert a hugely diverse range of local
above are correct in this context.
community compositions into something much more
homogeneous. 4. How have the human beings influenced the
It would be wrong, however, to conclude that biodiversity?
introducing species to a region will inevitably cause a 1. By smuggling live organisms.
decline in species richness there. For example, there are 2. By building highways.

ST . 24
Sample Test 2 : General Studies Paper II (2012)

3. By making ecosystems sensitive so that must depend on what they do to the lives and freedom
new species are not allowed. of the people involved, which must be central to the
4. By ensuring that new species do not have idea of development.
major impact on local species. If development is understood in a broader way,
Which of the statements given above are correct? with a focus on human lives, then it becomes
(a) 1 and 2 (b) 2 and 3 immediately clear that the relation between development
(c) 1 and 3 (d) 2 and 4 and democracy has to be seen partly in terms of their
constitutive connection, rather than only through their
5. What can be the impact of invasion of exotic
external links. Even though the question has often been
species on an ecosystem?
asked whether political freedom is “conducive to
1. Erosion of endemic species.
development”, we must not miss the crucial recognition
2. Change in the species composition of the
that political liberties and democratic rights are among
community of the ecosystem.
the “constituent components” of development. Their
Select the correct answer using the codes given relevance for development does not have to be
below: established indirectly through their contribution to the
(a) 1 only growth of GNP.
(b) 2 only
6. According to the passage, why is a serious
(c) Both 1 and 2
tension perceived between democracy and
(d) Neither 1 nor 2
development by the detractors of democracy?
(a) Democracy and development are distinct
Passage-2 and separate goals.
@UPSC_THOUGHTS

Most champions of democracy have been rather (b) Economic growth can be promoted
reticent in suggesting that democracy would itself successfully without pursuing a democratic
promote development and enhancement of social system of governance.
welfare—they have tended to see them as good but (c) Non-democratic regimes deliver economic
distinctly separate and largely independent goals. The growth faster and far more successfully
detractors of democracy, on the other hand, seemed to than democratic ones.
have been quite willing to express their diagnosis of (d) All the statements (a), (b) and (c) given
what they see as serious tensions between democracy above are correct in this context.
and development. The theorists of the practical split—
7. According to the passage, what should be the
“Make up your mind : do you want democracy, or
ultimate assessment/aim/view of development ?
instead, do you want development ?”—often came, at
(a) Rise in the per capita income and industrial
least to start with, from East Asian countries, and their
growth rates.
voice grew in influence as several of these countries
(b) Improvement in the Human Development
were immensely successful—through the 1970s and
Index and GNP.
1980s and even later—in promoting economic growth
(c) Rise in the savings and consumption trends.
without pursuing democracy.
(d) Extent of real freedom that citizens enjoy.
To deal with these issues we have to pay particular
attention to both the content of what can be called 8. What does a “constitutive” connection between
development and to the interpretation of democracy (in democracy and development imply?
particular to the respective roles of voting and of public (a) The relation between them has to be seen
reasoning). The assessment of development cannot be through external links.
divorced from the lives that people can lead and the (b) Political and civil rights only can lead to
real freedom that they enjoy. Development can scarcely economic development.
be seen merely in terms of enhancement of inanimate (c) Political liberties and democratic rights are
objects of convenience, such as a rise in the GNP (or essential elements of development.
in personal incomes), or industrialization—important (d) None of the statements (a), (b) and (c) given
as they may be as means to the real ends. Their value above is correct in this context.

ST . 25
Sample Test 2 : General Studies Paper II (2012)

Passage-3 3. Affiliates in a particular market/sector lose


The need for Competition Law becomes more evident their independence as their parent
when foreign direct investment (FDI) is liberalised. The companies overseas merge.
impact of FDI is not always pro-competitive. Very often 4. Foreign companies lower the cost of their
FDI takes the form of a foreign corporation acquiring products as compared to that of products
a domestic enterprise or establishing a joint venture of domestic companies.
with one. By making such an acquisition the foreign Which of the statements given above are correct?
investor may substantially lessen competition and gain (a) 1 and 2 only
a dominant position in the relevant market, thus (b) 2 and 3 only
charging higher prices. Another scenario is where the (c) 1, 2 and 3 only
affiliates of two separate multinational companies (d) 1, 2, 3 and 4
(MNCs) have been established in competition with one
11. What is the inference from this passage?
another in a particular developing economy, following
(a) Foreign investors and multinational
the liberalisation of FDI. Subsequently, the parent
companies always dominate the domestic
companies overseas merge. With the affiliates no longer
market.
remaining independent, competition in the host country
(b) It is not in the best interests of the domestic
may be virtually eliminated and the prices of the
economy to allow mergers of companies.
products may be artificially inflated. Most of these
(c) With competition law, it is easy to ensure
adverse consequences of mergers and acquisitions by
a level playing field between domestic and
MNCs can be avoided if an effective competition law
foreign firms.
is in place. Also, an economy that has implemented an
(d) For countries with open economy, Foreign
@UPSC_THOUGHTS

effective competition law is in a better position to attract


Direct Investment is essential for growth.
FDI than one that has not. This is not just because most
MNCs are expected to be accustomed to the operation 12. Examine the following statements:
of such a law in their home countries and know how 1. I watch TV only if I am bored.
to deal with such concerns but also that MNCs expect 2. I am never bored when I have my brother’s
competition authorities to ensure a level playing field company.
between domestic and foreign firms. 3. Whenever I go to the theatre I take my
brother along.
9. With reference to the passage, consider the
following statements: Which one of the following conclusions is valid
1. It is desirable that the impact of Foreign in the context of the above statements?
Direct Investment should be pro-competitive. (a) If I am bored, I watch TV.
2. The entry of foreign investors invariably (b) If I am bored, I seek my brother’s company.
leads to the inflated prices in domestic (c) If I am not with my brother, then I watch
markets. TV.
(d) If I am not bored, I do not watch TV
Which of the statements given above is/are
correct? 13. Only six roads A, B, C, P, Q and R connect a
(a) 1 only military camp to the rest of the country. Only one
(b) 2 only out of A, P and R is open at any one time. If B
(c) Both 1 and 2 is closed, so is Q. Only one of A and B is open
during storms. P is closed during floods. In this
(d) Neither 1 nor 2
context, which one of the following statements is
10. According to the passage, how does a foreign correct?
investor dominate the relevant domestic market? (a) Under normal conditions only three roads
1. Multinational companies get accustomed are open.
to domestic laws. (b) During storms at least one road is open.
2. Foreign companies establish joint ventures (c) During floods only three roads are open.
with domestic companies. (d) During calamities all roads are closed.

ST . 26
Sample Test 2 : General Studies Paper II (2012)

14. Examine the following statements: 17. Consider the following statement:
1. None but students are members of the club
The Third World War, if it ever starts, will end
2. Some members of the club are married very quickly with the possible end of civilization.
persons. It is only the misuse of nuclear power which will
3. All married persons are invited for dance. trigger it.
Which one of the following conclusions can be Based on the above statement, which one of the
drawn from the above statements? following inferences is correct?
(a) All students are invited for dance. (a) Nuclear power will be used in the Third
(b) All married students of the club are invited World War.
for dance. (b) There will be no civilization left after the
(c) All members of the club are married Third World War.
persons. (c) The growth of nuclear power will destroy
(d) None of the above conclusions can be civilization in the long run.
drawn. (d) The Third World War will not take place.
15. Four political parties W, X, Y and Z decided to 18. The elements of the problem figures given below
set up a joint candidate for the coming are changing with a certain rule as we observe
parliamentary elections. The formula agreed by them from left to right:
them was the acceptance of a candidate by most
of the parties. Four aspiring candidates, A, B, C
and D approached the parties for their tickets.
@UPSC_THOUGHTS

A was acceptable to W but not to Z. According to this rule, which of the following
B was acceptable to Y but not to X. would be the next figure if the changes were
C was acceptable to W and Y. continued with the same rule?
D was acceptable to W and X.
When candidate B was preferred by W and Z, (a)
candidate C was preferred by X and Z, and
candidate A was acceptable to X but not to Y;
who got the ticket? (b)
(a) A (b) B
(c) C (d) D
16. Consider the following statements: (c)
1. All X-brand cars parked here are white.
2. Some of them have radial tyres.
3. All X-brand cars manufactured after 1986 (d)
have radial tyres.
4. All cars are not X-brand. 19. Consider the following information regarding the
performance of a class of 1000 students in four
Which one of the following conclusions can be
different tests:
drawn from the above statements?
(a) Only white cars are parked here. Tests I II III IV
(b) Some white X-brand cars with radial tyres Average marks 60 60 70 80
are parked here.
(c) Cars other than X-brand cannot have radial Range of marks 30 45 20 0
tyres. to to to to
(d) Most of the X-brand cars are manufactured 90 75 100 100
before 1986. If a student scores 74 marks in each of the four

ST . 27
Sample Test 2 : General Studies Paper II (2012)

tests, in which one of the following tests is her


K H B
performance the best comparatively?
(a) Test I B K H
(b) Test II A M P
(c) Test III
(d) Test IV
What is the letter opposite to A?
20. Six squares are coloured, front and back, red (R), (a) H (b) P
blue (B), yellow (Y), green (G), white (W) and (c) B (d) M
orange (O) and are hinged together as shown in
the figure given below. If they are folded to form 23.
a cube, what would be the face opposite the white
face?

R B
?
G Y O

(a) R Which one of the figures shown below occupies


@UPSC_THOUGHTS

(b) G the blank space (?) in the matrix given above?


(c) B
(d) O
(a)

21. P Q
10 1 (b)
11

9 7 2
8 6 (c)
4 5 3
R S
(d)
In the above figure, circle P represents
hardworking people, circle Q represents intelligent 24. Consider the following statements:
people, circle R represents truthful people, and 1. All artists are whimsical.
circle S represents honest people. Which region 2. Some artists are drug addicts.
represents the people who are intelligent, honest 3. Frustrated people are prone to become drug
and truthful but not hardworking? addicts.
(a) 6
From the above three statements it may be
(b) 7
concluded that
(c) 8
(a) Artists are frustrated.
(d) 11
(b) Some drug addicts are whimsical.
22. Three views of a cube following a particular (c) All frustrated people are drug addicts.
motion are given below: (d) Whimsical people are generally frustrated.

ST . 28
Sample Test 2 : General Studies Paper II (2012)

25. Examine the following statements: order) and pants of green, yellow and orange
1. Either A and B are of the same age or A colours (not necessarily in that order). No person
is older than B. wore shirt and pants of the same colour. Further,
2. Either C and D are of the same age or D it is given that
is older than C. 1. A did not wear shirt of black colour.
3. B is older than C. 2. B did not wear shirt of blue colour.
Which one of the following conclusions can be 3. C did not wear shirt of orange colour.
drawn from the above statements? 4. A did not wear pants of green colour.
(a) A is older than B 5. B wore pants of orange colour.
(b) B and D are of the same age What were the colours of the pants and shirt
(c) D is older than C worn by C, respectively?
(d) A is older than C (a) Orange and black
26. Examine the following statements: (b) Green and blue
l. Only those who have a pair of binoculars (c) Yellow and blue
can become the members of the (d) Yellow and black
birdwatcher’s club.
29. Ten new TV shows started in January—5 sitcoms,
2. Some members of the birdwatcher’s club
3 drama and 2 news-magazines. By April only
have cameras.
seven of the new shows were still on, five of them
3. Those members who have cameras can take
being sitcoms.
part in photo-contests.
@UPSC_THOUGHTS

Based on the above information, four conclusions,


Which of the following conclusions can be drawn
as given below, have been made. Which one of
from the above statements?
these logically follows from the information given
(a) All those who have a pair of binoculars are
above?
members of the birdwatcher’s club.
(b) All members of the birdwatcher’s club have (a) Only one news magazine show is still on.
a pair of binoculars. (b) Only one of the drama shows is still on.
(c) All those who take part in photo-contests (c) At least one discontinued show was a
are members of the birdwatcher’s club. drama.
(d) No conclusion can be drawn. (d) Viewers prefer sitcoms over drama.

27. During the last summer vacation, Ankit went to 30. Read the passage given below and the two
a summer camp where he took part in hiking, statements that follow (given on the basis of the
swimming and boating. This summer, he is passage):
looking forward to a music camp where he hopes Four men are waiting at Delhi airport for a Mumbai
to sing, dance and learn to play the guitar. flight. Two are doctors and the other two are
businessmen. Two speak Gujarati and two speak Tamil.
Based on the above information, four conclusions,
No two of the same profession speak the same language.
as given below, have been made. Which one of
Two are Muslims and two are Christians. No two of
these logically follows from the information given
the same religion are of the same profession, nor do they
above?
(a) Ankit’s parents want him to play the guitar. speak the same language. The Tamil-speaking doctor
(b) Ankit prefers music to outdoor activities. is a Christian.
(c) Ankit goes to some type of camp every 1. The Christian-businessman speaks
summer. Gujarati.
(d) Ankit likes to sing and dance. 2. The Gujarati-speaking doctor is a Muslim.

28. Three persons A, B and C wore shirts of black, Which of the above statements is/are correct
blue and orange colours (not necessarily in that conclusion/conclusions?

ST . 29
Sample Test 2 : General Studies Paper II (2012)

(a) 1 only Who scored the highest?


(b) 2 only (a) Rama
(c) Both 1 and 2 (b) Padma
(d) Neither 1 nor 2 (c) Rani
31. Consider the following statement: (d) Ratna
“Though quite expensive, television is not a Directions for the following 6 (six) items:
luxury item, as one can learn many things through
Read the following two passages and answer the
television.”
items that follow each passage. Your answers to these
Which one of the following is a valid inference items should be based on the passages only.
from the above statement?
(a) All expensive things are regarded as luxury. Passage-1
(b) All essential things for learning are not
The poor, especially in market economies, need the
luxury.
strength that collectivities offer for creating more
(c) Television is essential for learning.
economic, social and political space for themselves, for
(d) Television is not a luxury item.
enhancing their socio-economic well-being and voice,
32. Mr Kumar drives to work at an average speed of and as a protection against free market individualism.
48 km per hour. The time taken to cover the first It has been argued that a group approach to farming,
60% of the distance is 10 rninutes more than the especially in the form of bottom up agricultural
time taken to cover the remaining distance. How production collectivities, offers substantial scope for
far is his office? poverty alleviation and empowering the poor as well
@UPSC_THOUGHTS

(a) 30 km as enhancing agricultural productivity. To realise this


(b) 40 km potential, however, the groups would need to be
(c) 45 km voluntary in nature, small in size, participative in
(d) 48 km decision making and equitable in work sharing and
33. Gita is prettier than Sita but not as pretty as Rita. benefit distribution. There are many notable examples
Then, of such collectivities to be found in varied contexts,
(a) Sita is not as pretty as Gita. such as in the transition economies. All of them bear
(b) Sita is prettier than Rita witness to the possibility of successful cooperation
(c) Rita is not as pretty as Gita under given conditions. And although the gender impact
(d) Gita is prettier than Rita. of the family cooperatives in the transition economies
is uncertain, the Indian examples of women-only groups
34. Given that,
farming offer considerable potential for benefiting
1. A is the brother of B
women.
2. C is the father of A.
3. D is the brother of E. 36. Agricultural collectivities such as group based
4. E is the daughter of B. farming can provide the rural poor
Then, the uncle of D is 1. empowerment.
(a) A 2. increased agricultural productivity.
(b) B 3. safeguard against exploitative markets.
(c) C 4. surplus production of agricultural
(d) E commodities.
35. Examine the following statements: Select the correct answer using the codes given
1. Rama scored more than Rani. below :
2. Rani scored less than Ratna. (a) 1, 2, 3 and 4
3. Ratna scored more than Rama. (b) 1, 2 and 3 only
4. Padma scored more than Rama but less (c) 2 and 4 only
than Ratna. (d) 1, 3 and 4 only

ST . 30
Sample Test 2 : General Studies Paper II (2012)

37. What does the author imply by “gender impact”? 39. According to the passage, what does “deepening
(a) Women are doubtful participants in of democracy” mean in the Western context?
cooperatives. (a) Consolidation of group and class identities.
(b) Family cooperatives may not include (b) Democratization translated as greater
women. involvement of people.
(c) Women benefiting from group farming. (c) Democratization as greater involvement of
(d) Women’s role in transition economies is ‘atomised’ individuals in the public sphere.
highly restrictive. (d) None of the statements (a), (b) and (c) given
above is correct in this context.
38. Consider the following assumptions:
1. It is imperative for transition economies to 40. Greater democratization in India has not
have agricultural collectivities. necessarily led to
2. Agricultural productivity can be increased (a) the dilution of caste and communal
by group approach to farming. identities in the public sphere.
(b) irrelevance of community identity as a
With reference to the above passage, which of governing force in Indian politics.
these assumptions is/are valid? (c) marginalization of elite groups in society.
(a) 1 only (d) relative unimportance of hereditary
(b) 2 only identities over class identities.
(c) Both 1 and 2
(d) Neither 1 nor 2 41. What is the “silent revolution” that has occurred
in the Indian democratic process?
@UPSC_THOUGHTS

Passage-2 (a) Irrelevance of caste and class hierarchies in


In a typical Western liberal context, deepening of political processes.
democracy invariably leads to consolidation of ‘liberal (b) Loosening of social strictures in voting
values’. In the Indian context, democratization is behaviour and patterns.
translated into greater involvement of people not as (c) Social change through transfer of power
‘individuals’ which is a staple to liberal discourse, but from upper caste elites to subaltern groups.
(d) All the statements (a), (b) and (c) given
as communities or groups. Individuals are getting
above are correct in this context.
involved in the public sphere not as ‘atomised’
individuals but as members of primordial communities Directions for the following 5 (five) items:
drawn on religious or caste identity. Community-identity
Examine the information given in the following
seems to be the governing force. It is not therefore
paragraph and answer the items that follow:
surprising that the so-called peripheral groups continue
to maintain their identities with reference to the social Guest lectures on five subjects viz., Economics,
groups (caste, religion or sect) to which they belong History, Statistics, English and Mathematics have to be
while getting involved in the political processes despite arranged in a week from Monday to Friday. Only one
the fact that their political goals remain more or less lecture can be arranged on each day. Economics cannot
identical. By helping to articulate the political voice of be scheduled on Tuesday. Guest faculty for History is
the marginalized, democracy in India has led to ‘a available only on Tuesday. Mathematics lecture has to
loosening of social strictures’ and empowered the be scheduled immediately after the day of Economics
peripherals to be confident of their ability, to improve lecture. English lecture has to be scheduled immediately
the socio-economic conditions in which they are placed. before the day of Economics lecture.
This is a significant political process that had led to 42. Which lecture is scheduled on Monday?
a silent revolution through a meaningful transfer of (a) History
power from the upper caste elites to various subaltern (b) Economics
groups within the democratic framework of public (c) Mathematics
governance. (d) Statistics

ST . 31
Sample Test 2 : General Studies Paper II (2012)

43. Which lecture is scheduled between Statistics 49. Examine the following statements:
and English? 1. None but the rich can afford air-travel.
(a) Economics 2. Some of those who travel by air become
(b) History sick.
(c) Mathematics 3. Some of those who become sick require
(d) No lecture treatment.
44. Which lecture is the last one in the week? Which one of the following conclusions can be
(a) History
drawn from the above statements?
(b) English
(a) All the rich persons travel by air.
(c) Mathematics
(b) Those who travel by air become sick.
(d) Economics
(c) All the rich persons become sick.
45. Which lecture is scheduled on Wednesday? (d) All those who travel by air are rich.
(a) Statistics
(b) Economics 50. In five flats, one above the other, live five
(c) English professionals. The professor has to go up to meet
(d) History his IAS officer friend. The doctor is equally
friendly to all, and has to go up as frequently as
46. Which lecture is scheduled before the Mathematics
go down. The engineer has to go up to meet his
lecture?
MLA friend above whose flat lives the professor’s
(a) Economics
(b) History friend.
(c) Statistics
@UPSC_THOUGHTS

From the ground floor to the top floor, in what


(d) English order do the five professionals live?
47. Two glasses of equal volume are respectively half (a) Engineer, Professor, Doctor, IAS officer,
and three-fourths filled with milk. They are then MLA
filled to the brim by adding water. Their contents (b) Professor, Engineer, Doctor, IAS officer, MLA
are then poured into another vessel. What will (c) IAS officer, Engineer, Doctor, Professor, MLA
be the ratio of milk to water in this vessel? (d) Professor, Engineer, Doctor, MLA, IAS officer
(a) 1 : 3
(b) 2 : 3 Directions for the following 15 (fifteen) items:
(c) 3 : 2 Read the following three passages and answer the
(d) 5 : 3 items that follow each passage. Your answers to these
48. Consider the following statements: items should be based on the passages only.
1. All machines consume energy.
2. Electricity provides energy. Passage-1
3. Electrically operated machines are cheap to Education without a doubt, has an important
maintain. functional, instrumental and utilitarian dimension. This
4. Electrically operated machines do not cause is revealed when one asks questions such as ‘what is
pollution. the purpose of education?’ The answers, too often, are
Which one of the following inferences can be ‘to acquire qualifications for employment/upward
drawn from the above statements? mobility’ ‘wider/higher (in terms of income)
(a) All machines are run by electric energy. opportunities’, and ‘to meet the needs for trained
(b) There is no form of energy other than human power in diverse fields for national
electricity. development’. But in its deepest sense education is not
(c) Most machines are operated on electric instrumentalist. That is to say, it is not to be justified
energy. outside of itself because it leads to the acquisition of
(d) Electrically operated machines are preferable formal skills or of certain desired psychological-social
to use. attributes. It must be respected in itself. Education is

ST . 32
Sample Test 2 : General Studies Paper II (2012)

thus not a commodity to be acquired or possessed and a few individuals may be unusually resistant (perhaps
then used, but a process of inestimable importance to because they possess an enzyme that can detoxify the
individuals and society, although it can and does have pesticide). If the pesticide is applied repeatedly, each
enormous use value. Education then, is a process of successive generation of the pest will contain a larger
expansion and conversion, not in the sense of converting proportion of resistant individuals. Pests typically have
or turning students into doctors or engineers, but the a high intrinsic rate of reproduction, and so a few
widening and turning out of the mind—-the creation, individuals in one generation may give rise to hundreds
sustenance and development of self-critical awareness or thousands in the next, and resistance spreads very
and independence of thought. It is an inner process of rapidly in a population.
moral-intellectual development. This problem was often ignored in the past, even
though the first case of DDT (dichlorodiphenyltri-
51. What do you understand by the ‘instrumentalist’
chloroethane) resistance was reported as early as 1946.
view of education?
There is exponential increase in the numbers of
(a) Education is functional and utilitarian in
invertebrates that have evolved resistance and in the
its purposes.
number of pesticides against which resistance has
(b) Education is meant to fulfil human needs.
evolved. Resistance has been recorded in every family
(c) The purpose of education is to train the
of arthropod pests (including dipterans such as
human intellect.
mosquitoes and house flies, as well as beetles, moths,
(d) Education is meant to achieve moral
wasps, fleas, lice and mites) as well as in weeds and
development.
plant pathogens. Take the Alabama leafworm, a moth
52. According to the passage, education must be
@UPSC_THOUGHTS

pest of cotton, as an example. It has developed resistance


respected in itself because in one or more regions of the world to aldrin, DDT,
(a) it helps to acquire qualifications for dieldrin, endrin, lindane and toxaphene.
employment. If chemical pesticides brought nothing but
(b) it helps in upward mobility and acquiring problems—if their use was intrinsically and acutely
social status. unsustainable—then they would already have fallen
(c) it is an inner process of moral and out of widespread use. This has not happened. Instead,
intellectual development. their rate of production has increased rapidly. The ratio
(d) All the (a), (b) and (c) given above are of cost to benefit for the individual agricultural producer
correct in this context. has remained in favour of pesticide use. In the USA,
53. Education is a process in which insecticides have been estimated to benefit the
(a) students are converted into trained agricultural products to the tune of around $5 for every
professionals. $1 spent.
(b) opportunities for higher income are Moreover, in many poorer countries, the prospect
generated. of imminent mass starvation, or of an epidemic disease,
(c) individuals develop self-critical awareness are so frightening that the social and health costs of
and independence of thought. using pesticides have to be ignored. In general the use
(d) qualifications for upward mobility are of pesticides is justified by objective measures such as
acquired. ‘lives saved’, ‘economic efficiency of food production’
and ‘total food produced’. In these very fundamental
Passage-2 senses, their use may be described as sustainable. In
Chemical pesticides lose their role in sustainable practice, sustainability depends on continually
agriculture if the pests evolve resistance. The evolution developing new pesticides that keep at least one step
of pesticide resistance is simply natural selection in ahead of the pests—pesticides that are less persistent,
action. It is almost certain to occur when vast numbers biodegradable and more accurately targeted at the
of a genetically variable population are killed. One or pests.

ST . 33
Sample Test 2 : General Studies Paper II (2012)

54. “The evolution of pesticide resistance is natural Which of the statements given above is/are
selection in action.” What does it actually correct?
imply? (a) 1 only
(a) It is very natural for many organisms to (b) 1 and 2 only
have pesticide resistance. (c) 3 only
(b) Pesticide resistance among organisms is a (d) 1, 2 and 3
universal phenomenon.
58. Why is the use of chemical pesticides generally
(c) Some individuals in any given population
justified by giving the examples of poor and
show resistance after the application of
developing countries?
pesticides.
1. Developed countries can afford to do away
(d) None of the statements (a), (b) and (c) given
with use of pesticides by adapting to organic
above is correct.
farming, but it is imperative for poor and
55. With reference to the passage, consider the developing countries to use chemical
following statements: pesticides.
1. Use of chemical pesticides has become 2. In poor and developing countries, the
imperative in all the poor countries of the pesticide addresses the problem of epidemic
world. diseases of crops and eases the food
2. Chemical pesticides should not have any problem.
role in sustainable agriculture. 3. The social and health costs of pesticide use
3. One pest can develop resistance to many are generally ignored in poor and
@UPSC_THOUGHTS

pesticides. developing countries.


Which of the statements given above is/are Which of the statements given above is/are
correct? correct?
(a) 1 and 2 only (a) 1 only
(b) 3 only (b) 1 and 2 only
(c) 1 and 3 only (c) 2 only
(d) 1, 2 and 3 (d) 1, 2 and 3
56. Though the problems associated with the use of 59. What does the passage imply?
chemical pesticides is known for a long time, (a) Alternative options to chemical pesticides
their widespread use has not waned. Why? should be promoted.
(a) Alternatives to chemical pesticides do not (b) Too much use of chemicals is not good for
exist at all. the ecosystem.
(b) New pesticides are not invented at all. (c) There is no scope for the improvement of
(c) Pesticides are biodegradable. pesticides and making their use sustainable.
(d) None of the statements (a), (b) and (c) given (d) Both the statements (a) and (b) above are
above is correct. correct.
57. How do pesticides act as agents for the selection
of resistant individuals in any pest population? Passage-3
1. It is possible that in a pest population the Today’s developing economies use much less energy
individuals will behave differently due to per capita than developed countries such as the United
their genetic makeup. States did at similar incomes, showing the potential for
2. Pests do possess the ability to detoxify the lower-carbon growth. Adaptation and mitigation need
pesticides. to be integrated into a climate-smart development
3. Evolution of pesticide resistance is equally strategy that increases resilience, reduces the threat of
distributed in pest population. further global warming, and improves development

ST . 34
Sample Test 2 : General Studies Paper II (2012)

outcomes. Adaptation and mitigation measures can Select the correct answer using the codes given
advance development, and prosperity can raise incomes below:
and foster better institutions. A healthier population (a) 1 only
living in better-built houses and with access to bank (b) 2, 3 and 4 only
loans and social security is better equipped to deal with (c) 1 and 4 only
a changing climate and its consequences. Advancing (d) 1, 2, 3 and 4
robust, resilient development policies that promote
adaptation is needed today because changes in the 61. What does low-carbon growth imply in the
climate, already begun, will increase even in the short present context?
term. 1. More emphasis on the use of renewable
The spread of economic prosperity has always been sources of energy.
intertwined with adaptation to changing ecological 2. Less emphasis on manufacturing sector
conditions. But as growth has altered the environment and more emphasis on agriculture sector.
and as environmental change has accelerated, sustaining 3. Switching over from monoculture practices
growth and adaptability demands greater capacity to to mixed farming.
understand our environment, generate new adaptive 4. Less demand for goods and services.
technologies and practices, and diffuse them widely. As
economic historians have explained, much of Select the correct answer using the codes given
humankind’s creative potential has been directed at below:
adapting to the changing world. But adaptation cannot (a) 1 only
cope with all the impacts related to climate change, (b) 2, 3 and 4 only
@UPSC_THOUGHTS

especially as larger changes unfold in the long term. (c) 1 and 4 only
Countries cannot grow out of harm’s way fast (d) None of the above implies low-carbon
enough to match the changing climate. And some growth
growth strategies, whether driven by the government or
the market, can also add to vulnerability—particularly 62. Which of the following conditions is/are
if they overexploit natural resources. Under the Soviet necessary for sustainable economic growth?
development plan, irrigated cotton cultivation expanded 1. Spreading of economic prosperity more.
in water-stressed Central Asia and led to the near 2. Popularising/spreading of adaptive
disappearance of the Aral Sea, threatening the technologies widely.
livelihoods of fishermen, herders and farmers. And 3. Investing on research in adaptation and
clearing mangroves, the natural coastal buffers against mitigation technologies.
storm surges—to make way for intensive farming or
housing development, increases the physical Select the correct answer using the codes given
vulnerability of coastal settlements, whether in Guinea below:
or in Louisiana. (a) 1 only
(b) 2 and 3 only
60. Which of the following conditions of growth can
(c) 1 and 3 only
add to vulnerability?
(c) 1, 2 and 3
1. When the growth occurs due to excessive
exploitation of mineral resources and 63. Which of the following inferences can be made
forests. from the passage?
2. When the growth brings about a change in 1. Rainfed crops should not be cultivated in
humankind’s creative potential. irrigated areas.
3. When the growth is envisaged only for 2. Farming under water-deficient areas should
providing houses and social security to the not be a part of development strategy.
people.
4. When the growth occurs due to emphasis Select the correct answer using the codes given
on farming only. below:

ST . 35
Sample Test 2 : General Studies Paper II (2012)

(a) 1 only little tufts of grass and odd bits of stone as old friends.
(b) 2 only I was not alone in my cell, for several colonies of wasp
(c) Both 1 and 2 and hornets lived there, and many lizards found a
(d) Neither 1 nor 2 home behind the rafters, emerging in the evenings in
search of prey.
64. Consider the following assumptions:
1. Sustainable economic growth demands the 66. Which of the following explains best the sentence
use of creative potential of man. in the passage “I was almost a part of it”?
2. Intensive agriculture can lead to ecological (a) I was not alone in the cell.
backlash. (b) I was familiar with every bit of the cell.
3. Spread of economic prosperity can (c) I greeted little tufts of grass like old friends.
adversely affect the ecology and (d) I felt quite at home in the cell.
environment. 67. The passage attempts to describe
With reference to the passage, which of the above (a) the general conditions of the country’s jails.
assumptions is/are valid? (b) the prisoner’s capacity to notice the minute
(a) 1 only details of his surroundings.
(b) 2 and 3 only (c) the prisoner’s conscious efforts to overcome
(c) 1 and 3 only the loneliness.
(d) 1, 2 and 3 (d) the prisoner’s ability to live happily with
other creatures.
65. Which one of the following statements constitutes
68. The author of the passage seems to suggest that
@UPSC_THOUGHTS

the central theme of this passage?


(a) it is possible to adjust oneself to uncongenial
(a) Countries with greater economic prosperity
surroundings.
are better equipped to deal with the
(b) the conditions in Indian prisons are not
consequences of climate change.
bad.
(b) Adaptation and mitigation should be
(c) it is not difficult to spend one’s time in a
integrated with development strategies.
prison.
(c) Rapid economic growth should not be
(d) there is a need to improve the conditions
pursued by both developed and developing
in our jails.
economies.
(d) Some countries resort to overexploitation of
natural resources for the sake of rapid Passage-2
development. We started pitching the highest camp that has ever
been made. Everything took five times as long as it
Directions for the following 8 (eight) items:
would have taken in a place where there was enough
The following eight items (questions 66 to 73) are air to breathe; but at last we got the tent up, and when
based on three passages in English to test the we crawled in it was not too bad. There was only a
comprehension of English language and therefore these light wind, and inside it was not too cold for us to take
items do not have Hindi version. Read each passage off our gloves. At night most climbers take off their
and answer the items that follow. boots; but I prefer to keep them on. Hillary, on the other
hand, took his off and laid them next to his sleeping
Passage-1 bag.
For fourteen and a half months I lived in my little 69. What does the expression “pitching the
cell or room in the Dehradun jail, and I began to feel highest camp” imply?
as if I was almost a part of it. I was familiar with every (a) They reached the summit of the highest
bit of it. I knew every mark and dent on the whitewashed mountain in the world.
walls and on the uneven floor and the ceiling with its (b) Those who climbed that far earlier did not
moth-eaten rafters. In the little yard outside I greeted pitch any camp.

ST . 36
Sample Test 2 : General Studies Paper II (2012)

(c) So far nobody has ever climbed that high. response for each item. The responses will be evaluated based
(d) They were too many climbers and needed on the level of appropriateness for the given situation.
to pitch a big camp. Please attempt all the items. There is no penalty for
70. They took a long time to finish the work because wrong answers for these seven items.
(a) they were very tired. 74. You have differences of opinion regarding the
(b) there was not enough air to breathe. final report prepared by your subordinate that is
(c) it was very cold. to be submitted urgently. The subordinate is
(d) it was very dark. justifying the information given in the report. You
would...
71. When they crawled into the tent
(a) Convince the subordinate that he is wrong.
(a) they took off their gloves because it was not
(b) Tell him to reconsider the results.
very cold.
(c) Revise the report on your own.
(b) they could not take off their gloves because
(d) Tell him not to justify the mistake.
it was very cold.
(c) they took off their gloves though it was very 75. You are competing with your batch-mate for a
cold. prestigious award to be decided based on an oral
(d) they did not take off their gloves though it presentation. Ten minutes are allowed for each
presentation. You have been asked by the
was not cold.
committee to finish on time. Your friend, however,
is allowed more than the stipulated time period.
Passage-3
You would...
A local man, staying on the top floor of an old
@UPSC_THOUGHTS

(a) Lodge a complaint to the chairperson


wooden house, was awakened at midnight by a fire. against the discrimination.
Losing his way in a smoke-filled passage, he missed (b) Not listen to any justification from the
the stairway and went into another room. He picked committee.
up a bundle to protect his face from the fire and (c) Ask for withdrawal of your name.
immediately fell through the floor below where he (d) Protest and leave the place.
managed to escape through a clear doorway. The 76. You are handling a time-bound project. During
“bundle” proved to be the baby of the Mayor’s wife. the project review meeting, you find that the
The “hero” was congratulated by all. project is likely to get delayed due to lack of
72. The man went into another room because cooperation of the team members. You would...
(a) he did not know where exactly the stairway (a) Warn the team members for their non-
cooperation.
was.
(b) Look into reasons for non-cooperation.
(b) the passage was full of smoke.
(c) Ask for the replacement of team members.
(c) he was extremely nervous.
(d) Ask for extension of time citing reasons.
(d) he stumbled on a bundle.
77. You are the chairperson of a state sports
73. The man was called a hero because he committee. You have received a complaint and
(a) expressed his willingness to risk his life to later it was found that an athlete in the junior
save others. age category who has won a medal has crossed
(b) managed to escape from the fire. the age criteria by 5 days. You would...
(C) showed great courage in fighting the fire. (a) Ask the screening committee for a
(d) saved a life. clarification.
(b) Ask the athlete to return the medal.
Directions for the following 7 (seven) items:
(c) Ask the athlete to get an affidavit from the
Given below are seven items. Each item describes a court declaring his/her age.
situation and is followed by four possible responses. Indicate (d) Ask the members of the sports committee
the response you find most appropriate. Choose only one for their views.

ST . 37
Sample Test 2 : General Studies Paper II (2012)

78. You are handling a priority project and have been fixed sum for the installation of taps and
meeting all the deadlines and are therefore the usage of water be free.
planning your leave during project. Your (c) Recommend that a fixed monthly charge be
immediate boss does not grant your leave citing levied only on the non-BPL families and for
the urgency of the project. You would... BPL families water should be free.
(a) Proceed on leave without waiting for the (d) Recommend that the users pay a charge
sanction. based on the consumption of water with
(b) Pretend to be sick and take leave. differentiated charges for non-BPL and BPL
(c) Approach higher authority to reconsider families.
the leave application.
80. As a citizen you have some work with a
(d) Tell the boss that it is not justified.
government department. The official calls you
79. You are involved in setting up a water supply again and again; and without directly asking
project in a remote area. Full recovery of cost is you, sends out feelers for a bribe. You want to
impossible in any case. The income levels in the get your work done. You would...
area are low and 25% of the population is below (a) Give a bribe.
poverty line (BPL). When a decision has to be (b) Behave as if you have not understood the
taken on pricing you would... feelers and persist with your application.
(a) Recommend that the supply of water be (c) Go to the higher officer for help verbally
free of charge in all respects. complaining about feelers.
(b) Recommend that the users pay a one time (d) Send in a formal complaint.
@UPSC_THOUGHTS

ST . 38
Sample Test 2 : General Studies Paper II (2012)

PRACTICE ANSWER SHEET

Directions: Use HB Pencil. Erase completely to change. Example

1. 17. 33. 49. 65.

2. 18. 34. 50. 66.

3. 19. 35. 51. 67.

4. 20. 36. 52. 68.

5. 21. 37. 53. 69.

6. 22. 38. 54. 70.


@UPSC_THOUGHTS

7. 23. 39. 55. 71.

8. 24. 40. 56. 72.

9. 25. 41. 57. 73.

10. 26. 42. 58. 74.

11. 27. 43. 59. 75.

12. 28. 44. 60. 76.

13. 29. 45. 61. 77.

14. 30. 46. 62. 78.

15. 31. 47. 63. 79.

16. 32. 48. 64. 80.

ST . 39
Sample Test 2 : General Studies Paper II (2012)

ANSWERS AND EXPLANATORY NOTES

1. (d) None of the statements (a), (b) or (c) is 6. (c) At first sight, answer responses (a), (b) and
supported by the passage. Responses (a) and (c) all seem acceptable and one may be
(b) are negated by the sentences, “Many tempted to mark (d) as correct. However, if
introduced … without much obvious effect. you have read the passage carefully and
However, some have been … communities.” understood the question, you will see that
(Paragraph 1) and “It would be wrong, however, response (a) is what the champions of
to conclude … species richness there … Their democracy think and it does not express
introduction … augment British biodiversity.” “serious tension” or conflict between democracy
(Paragraph 3). Response (c) is negated by the and development. Statement (b) is exemplified
very first sentence of the passage. (It is by some of the East Asian countries and this
advisable to be careful of statements with is actually stated in the passage at the end
words like ‘always’, ‘never’, ‘only’: most likely of Paragraph 1. However, it is the conclusion
they are not correct in the context of passages.) from this illustration that answers the question—
”Make up your mind … development” expressing
2. (b) Check each statement. Statement 1 is not
the ‘practical split’ between democracy and
supported by the passage. Statement 2 may
development—that the two are not compatible.
not be directly mentioned in the passage but
This is best expressed by answer response
is implied in the words, “by bringing a pest
(c).
under control, producing new agricultural
7. (d) Refer to the sentences, “The assessment of
@UPSC_THOUGHTS

products” (Paragraph 1). So it is correct.


Statement 3 may also be considered as development … enjoy” (Paragraph 2) and
implied in the words, “providing novel “Even though the question … development”
recreational opportunities” which could include (Paragraph 3).
beautification and landscaping. So response [EM: There is no reference in the passage to
(b) is correct. human development index, rise in savings and
consumption trends; so answer responses (b)
[Elimination Method (EM): If you check the
and (c) may be rejected immediately; sentence,
responses after checking and rejecting
“Development can scarcely … real ends”
statement 1, you will see that responses (a),
(Paragraph 2) negates answer response (a); so
(c) and (d) may be rejected immediately as
you are left with answer response (d) which
they include 1.]
alone can be correct.]
3. (b) Refer to “The extent to which … physical
8. (c) Answer response (a) is contradicted by the
barriers” (Paragraph 2). Oceans and mountain
passage: “… it becomes immediately clear …
ranges are such physical barriers.
external links” (Paragraph 3). Answer response
4. (a) A look at the statements will tell you that (3) (b) is not mentioned. The word ‘constitutive’
and (4) are in no way supported by the implies ‘constituent’, and we are told in the
passage. So the correct answer response can passage that political liberties and democratic
be only (a). Statement 1 is implied in paragraph rights are the “constituent components” of
1: human beings may introduce exotic species development; so the implication is clear.
(live organisms) “illegally”—i.e. by smuggling.
9. (a) The passage favours a Competition Law in the
Statement 2 is indicated in the sentence, “By
context of the impact of FDI entry not always
virtue of the transport opportunities …” in
being pro-competitive. So Statement 1 is
Paragraph 2. implied. The use of the word ‘invariably’ makes
5. (c) Refer to the sentences, “However, some have Statement 2 unacceptable; the passage merely
been responsible for dramatic changes …” says FDI entry may lead to inflated prices in
(Paragraph 1) and “The effects of introductions certain circumstances. So only statement 1 is
… homogeneous” (Paragraph 2). correct.

ST . 40
Sample Test 2 : General Studies Paper II (2012)

10. (b) Statement 1 may be a fact as stated in the are invited to the dance, conclusion in (b)
passage but it is not a factor leading to the follows logically.
dominance of the foreign investor in the 15. (c) Make a chart.
domestic market. Statement 2 is supported by
the passage: “Very often FDI takes the form Candidates Parties
… higher prices.” Statement 3 is supported by W X Y Z
the passage: “Another scenario … inflated.”
A ✓ ✓ X ✓
Statement 4 is not correct as the passage
nowhere mentions lower cost of products in B ✓ X ✓ ✓
the context of foreign companies. So only
statements 2 and 3 are correct. C ✓ ✓ ✓ ✓
[EM: As soon as you realise Statement 1 is D ✓ ✓
wrong, a look at the answer responses will help
you to eliminate (a), (c) and (d) as all of them [The encircled symbols are entered at the
include 1.] second step.]
11. (c) This may be inferred from the last sentence It is clear candidate C is acceptable to all
of the passage. The passage does not say parties and so got the ticket.
that foreign investors and MNCs ‘always’ 16. (b) From statements 1 and 4, we can see that
dominate the domestic market; so (a) is not some cars are X-brand and some of those X-
correct. The passage talks of the “adverse brand cars (all those parked here) are white.
consequences of mergers”; it does not say There is no basis for selecting answer response
such mergers are not in the best interests of (a) as we know nothing about the colour of
@UPSC_THOUGHTS

the domestic economy. So (b) is not correct. all cars parked here.
Answer response (d) is nowhere mentioned in
Taking statements 2 and 3 together, we can
the passage.
conclude that some of the white X-brand cars
12. (d) parked here (those manufactured after 1986)
13. (b) It is clear from the statement: “Only one of have radial tyres. So answer response (b) is
A and B is open during storms.” So at least correct.
one road is open during storms. Check out answer responses (c) and (d). The
14. (b) Statement 1 can be restated as: All members statements do not tell us whether cars other
of the club are students. Now if we represent than X-brand have or do not have radial tyres.
the statements diagrammatically, we have— Nor can we conclude anything about when all
or most X-brand cars were manufactured. So
(c) and (d) may be rejected. We are left with
S
(b) as the best possible choice.

D 17. (a) This inference follows from the sentence, “It


CM M
is only … it.” It is only if nuclear power is
used that the possibility of its misuse is there.
Answer response (b) is not correct as the
statement indicates “possible end”, and not a
CM = club members categorical end of civilisation after the Third
S = students World War. Answer response (c) is not
M = married persons supported by the statement. Answer response
D = people invited for the dance (d) is not supported by the statement.
Clearly only answer response (b) can be 18. (d) Take the movement of the triangle. It gets
concluded. inverted and moves to the next corner in an
A more direct method: As only students are anti-clockwise direction. The next figure in
members of the club and only married persons continuation will have the triangle with apex

ST . 41
Sample Test 2 : General Studies Paper II (2012)

at top. This is only in answer response (d). 25. (d) Symbolically,


It is also in the correct corner. So (d) must
A ³ B
be correct.
D ³ C
19. (b) In Test II, anyone scoring 74 marks will be B > C
near the very top which is 75 marks.
∴ A ³ B > C
20. (c)
It is clear that answer responses (a) and (c)
21. (a) The required region is the overlap of Q, S and are invalid, as A may be the same age as
R, excluding P. This region is 6. B and D may be the same age as C. Even
22. (a) When A is seen, H is not; when H is seen, if D is older than C, there is no way to
A is not. So A and H are on opposite sides. establish that B and D are of the same age.
23. (d) Consider the top and bottom rows which are B could be older than D. So answer response
complete. The black segment moves to the (b) is incorrect. Answer response (d) is correct:
opposite quarter. So in the middle row, the even if A and B are of the same age, B is
missing figure will have the black segment in older than C, so A must be older than C.
the lower right quarter—as in (d).
26. (b) Statement 1 means that all members of the
24. (b) Represent the statements diagrammatically. birdwatcher’s club have a pair of binoculars.
(Statement 3 would be: some frustrated people This is what answer response (b) says. Now
are prone to be drug addicts.) check the other responses.

F
From statement 1 we know that all members
of the birdwatcher’s club have binoculars.
W
@UPSC_THOUGHTS

From that we cannot conclude answer response


A (a), for we are not told about all persons who
D
have binoculars. (‘All S is P’ is not ‘All P is
F S’.) From statements 2 and 3 we may
conclude that some of the members have a
camera and these members can participate in
F photo-contests. We have not been told that
only members of the birdwatcher’s club can
A = artists
participate in photo-contests. So answer
W = whimsical persons
response (c) is incorrect as there may be
D = drug addicts
contestants who are not members of the club.
F = frustrated people
It can be seen that answer response (a) cannot As answer response (b) is valid, answer
be a conclusion as frustrated people need not response (d) does not apply.
include artists at all. Answer response (b) is 27. (d) We are not told about Ankit’s parents’ wishes,
a valid conclusion: if some artists are drug so answer response (a) is invalid. We do not
addicts, some drug addicts are artists; as all know about Ankit ‘preferring’ anything to
artists are whimsical it follows that the drug something else, so response (b) is also invalid.
addicts who are artists are whimsical. From the information given it may be concluded
Check responses (c) and (d). that Ankit has gone to one summer vacation
Answer response (c) is not valid as statement camp and is likely to go on another. So (c)
3 cannot be interpreted as: all frustrated people seems valid. However, we are told of just two
are drug addicts. Answer response (d) is not summers; it may not be correct to conclude
valid as no firm relationship can be drawn from that Ankit goes camping every summer.
between whimsical persons and frustration Now the words ‘looking forward to’ in the
from the given statements. All one can say information gives the impression that Ankit
is that some frustrated people may (or may
‘likes to sing and dance’. So (d) seems an
not) be whimsical.
acceptable conclusion. It is the best among
So answer response (b) is correct. the given responses.

ST . 42
Sample Test 2 : General Studies Paper II (2012)

28. (b) Making a chart will help. Fill in the information businessman and a Christian. So statement
by steps. 1 is also correct. Answer response (c) is
correct.
A B C
You may also convert the information into a
Shirts chart and reach the conclusion.
Black X ✓ X Person Profession Language Religion
Blue X X ✓ I Doctor Tamil Christian

Orange ✓ X X II Doctor Gujarati Muslim


III Businessman Tamil Muslim
Pants
IV Businessman Gujarati Christian
Green X X ✓
Yellow X 31. (a) This is the inference. It is because expensive
things are regarded as luxury that the given
Orange ✓
statement says “Though quite expensive ….”
Answer responses (a) and (c) cannot be
As B wears orange pants, B cannot wear
inferred from the given statement which only
orange shirt. As neither B nor C wears orange
says that television is not a luxury as one
shirt, A must wear orange shirt. As B cannot
wear either blue or orange shirt, B must wear can learn things through a television; it does
black shirt. Under the given conditions, C can not make any connection between all essential
wear only blue shirt. Now, looking at the learning things and luxury or say that learning
@UPSC_THOUGHTS

answer responses, you can reject responses is only possible through television. Answer
(a) and (d). For pants, as B wears orange, response (d) could lead to some confusion,
B cannot be wearing green or yellow pants. but note that it is a stated fact in the given
We are told A does not wear green pants, so sentence and cannot be an ‘inference’.
only C can wear green pants. Answer response 32. (b) Remaining Distance (100 – 60)%
(b) is correct.
= 40%
29. (c) Out of the original ten, seven shows are on— Let time taken during the 40% distance
out of which 5 are sitcoms. The remaining 2
= x minutes
have to be found out. These could only be
Time taken during the 60% distance
combinations of (i) 2 drama, (ii) 2 news
magazines or (iii) 1 drama and 1 news = (x + 10) minutes
magazine. In case of (i) and (ii), answer Under the given conditions,
response (a) cannot be correct. In these
cases, answer response (b) is also not correct. 60% of D 40% of D
=
In all cases, at least one drama show is ( x + 10) x
discontinued. So answer response (c) is correct. 60 40
´ D ´ D
We are not told why all sitcoms have continued; Þ 100 = 100
it could be because viewers like sitcoms, but (x + 10) x
we have no clue as to the preference regarding Þ 6x = 4 (x + 10)
drama or news magazines. Conclusion in
Þ 6x = 4x + 40
answer response (d) is not a certainty, so it
does not logically follow from the given Þ 6x – 4x = 40
information. Þ 2x = 40
30. (c) From the last sentence of the passage, it is Þ x = 20 minutes
clear, under the given conditions, that the other ∴ x + 10 minutes
doctor speaks Gujarati and is a Muslim. So
= 20 + 10 minutes
Statement 2 is correct. In that case, the other
Gujarati-speaking person must be a = 30 minutes

ST . 43
Sample Test 2 : General Studies Paper II (2012)

Total distance (D) = D1 + D2 democratisation that the relevance of such


identities has grown in Indian politics. While
30 20
= 48 ´ + 48 ´ the passage talks of “a meaningful transfer of
60 60
power from upper caste elites to various
= 24 + 16 subaltern groups”, it does not indicate
= 40 km marginalisation of elite groups. So (c) is not
33. (a) Symbolically, acceptable. Answer response (d) is not
mentioned in the passage.
Rita > Gita > Sita
41. (c) Answer response (a) is contradicted by the
(> prettier than) passage. “Loosening of social strictures” is
34. (a) Draw a diagram mentioned in the context of giving expression
C to political voice of the marginalised and
father of improving their socio-economic conditions not
in voting patterns; so (b) is not a correct
A brother of B answer response. In the circumstances, answer
response (d) is not applicable.
For answering questions 42 to 46, represent
E D (brother)
(daughter) the given Information in a chart.
D is B’s son and A must be D’s uncle. Day Lecture
35. (d) Rama > Rani Monday Statistics
Ratna > Rani Tuesday History
@UPSC_THOUGHTS

Ratna > Rama Wednesday English


Ratna > Padma > Rama Thursday Economics
It is clear that Ratna scored the highest. Friday Mathematics
36. (b) A reading of the passage—the first two 42. (d)
sentences—clearly indicates that agricultural
43. (b)
collectivities such as group approach to farming
helps in empowerment, poverty alleviation and 44. (c)
enhancing agricultural productivity. Such 45. (c)
collectivities also offers “protection against 46. (a) This question has clearly been answered in
free market individualism”, i.e., exploitative the passage: “Mathematics lecture …
markets. So 1, 2 and 3 are valid. As for 4, Economics lecture.”
surplus production of agricultural commodities
47. (d) Let capacity of each glass = x
is not mentioned in the passage. So (b) is
the correct answer choice. Glass Milk Water
x x
37. (c) 1
2 2
38. (b) The passage does not say anything about 3x x
2
agricultural collectivities being imperative. It 4 4
merely speaks of the benefits of such collective x 3x
approach. So assumption 1 is not correct. ∴ Quantity of milk = +
2 4
39. (c) It is clear from the words, “In the Indian
2x + 3x 5x
context, … liberal discourse”. = =
4 4
40. (a) Answer response (b) also appears to be x x
∴ Quantity of water = +
correct but it actually follows from (a). It is 2 4
because caste and communal identities have 2x + x 3x
= =
not been diluted as a consequence of 4 4

ST . 44
Sample Test 2 : General Studies Paper II (2012)

5x 3x 51. (b) Answer response (a) repeats words from the


∴ Required proportion is : passage that are used along with the word
4 4
‘instrumentalist’, so it cannot be the meaning
=5:3 of the word. The meaning becomes clear from
Shorter Method the sentences, “But in its deepest sense …
psychological-social attributes.” The
Milk Water
‘instrumentalist’ view considers education as
æ1 3 ö æ1 1 ö a means to (which is what ‘instrumental’
çè + ÷ : çè + ÷
2 4ø 2 4ø stands for) acquiring some skills or fulfilling
æ2 + 3 ö æ2 + 1ö some human needs. Answer responses (c) and
çè ÷ : çè ÷ (d) are what education is when it is not
4 ø 4 ø
instrumentalist.
5 3
: 52. (c) Refer to “Education then … development.”
4 4
= 5 : 3 53. (c)

48. (d) This can be inferred from statements 3 54. (d) Answer response (c) is just a part of the
and 4. explanation and is not the whole answer. So
(d) is the best option.
49. (d) Statement 1 may be translated as ‘Only the
rich can afford air-travel’ or ‘All those who can 55. (b) The use of the word ‘all’ makes statement 1
travel by air are rich’. This is what answer unacceptable. The passage refers to “many
response (d) says, so it is the likely conclusion. poorer countries” in this context. Statement 2
However, check the other answer responses. is contradicted by the passage: “In these very
@UPSC_THOUGHTS

Answer response (a) is invalid – a case of fundamental senses … sustainable” (Paragraph


converting ‘All S is P’ into ‘All P is S’. 4). The last sentences of Paragraph 2 support
Statement 2 says ‘some’ of those who travel statement 3. So (b) is the correct answer
by air become sick; so to convert a particular response.
to a universal statement as in (b) is wrong. EM: As soon as statement 1 is found to be
Answer response (c) is way off mark as there unacceptable, you may reject answer responses
is nothing said in the statements about all rich (a), (c) and (d) and choose (b).
persons. So only (d) is correct. 56. (d) The passage does not refer to alternative
50. (d) Sketch a diagram. As the doctor is ‘equally pesticides, so (a) is unacceptable. Neither (b)
friendly to all’ and apparently has to move up nor (c) has been stated in the passage.
or down equally to meet friends, he is likely 57. (a) Statement 1 is implied in “One or a few
to be at the centre with two people above and individuals may be unusually resistant ….”
two below his flat. The professor’s friend (the Statement 2 is too generalised to be correct,
IAS officer) lives above the MLA’s flat. As the as the passage says only ‘One or a few
engineer and professor both have to move up individuals’ may possess an enzyme that can
to meet their friends, the likely arrangement detoxify the pesticide. Statement 3 is not
of the flats is supported by the passage. So only (a) is
(i) IAS officer or (ii) IAS officer correct.

MLA MLA EM: As soon as statement 1 is found correct


but statement 2 is found to be unacceptable,
Doctor Doctor answer response (a) becomes the only choice.
Engineer Professor 58. (c) Nothing in the passage supports statement 1.
There is no mention of organic farming.
Professor Engineer
Statement 2 is supported by the passage –
As only (i) is among the answer responses see the last paragraph. Statement 3 is not
it is (d) that is to be chosen. quite correct: the passage says social and

ST . 45
Sample Test 2 : General Studies Paper II (2012)

health costs of pesticide use “have to be the present context” and not “in the context
ignored” in the context of problems – which of the passage”. Statements 2 and 3 would
is not the same as these costs being “generally not in themselves lead to low-carbon growth
ignored”. Answer response (c) is correct. unless other mitigation efforts are included for
EM: As soon as statement 1 is found to be agriculture also contributes to climate change.
unacceptable, answer responses (a), (b) and 62. (d) Statement 3 is implied.
(d) may be rejected and (c) chosen. 63. (b) The question asks for ‘inferences’. Considering
59. (d) Answer response (b) is certainly implied in the what the passage says in Paragraph 1 on
passage. But the last paragraph after adaptation and mitigation it would be correct
considering the fact that chemical pesticides to infer that a development strategy should not
cannot be rejected altogether suggests the include farming under water-deficient areas.
need to develop new pesticides that not only However, statement 1 cannot be inferred.
meet the challenge of pests that develop
64. (c) Refer to Paragraph 2. It is only human
resistance but are also ecologically less harmful.
creativity that can adapt to and manage
So (a) is also implied. The last sentence of
environmental challenges to maintain
the passage contradicts answer response (c).
sustainable growth. And spread of economic
So (d) is the correct response.
prosperity can affect ecology and environment
60. (a) Refer to the third paragraph. A look at the if adaptation lags behind. Statement 2 is not
statements tells you that statement 1 is correct as there is nothing in the passage to
correct. Statement 2 is not correct: the suggest that intensive agriculture by itself can
passage says human creative potential helps lead to ecological backlash.
@UPSC_THOUGHTS

in adapting to a changing world. It is not a


65. (b)
condition of growth that adds to vulnerability.
It may prove unable to cope with changes, 66. (d) Answer responses (a), (b) and (c) simply
but that is not what the statement says. repeat words from the passage and illustrate
Statements 3 and 4 may appear correct, but how the writer “was almost a part of it”; they
housing projects and intensive farming are do not explain the sentence.
mentioned as examples of the overexploitation 67. (c) The prisoner’s capacity to notice the minute
of natural resources. Incidentally, the words details is a part of his efforts to overcome
‘social security’ in statement 3 indicates there the loneliness.
is something wrong in the statement. Statement
68. (a) The tone of the passage does not suggest (c).
1 points at the basic vulnerability. So answer
A conscious effort has to be made to adjust.
response (a) is correct. [If you are still
confused, have a look at the answer responses: 69. (b) There is nothing in the passage to suggest
(b) and (d) may be rejected as they include any of the other answer responses.
statement 2 which is certainly wrong. Now 70. (b) Refer to “Everything took five times as long
consider statement 4. The passage does not … breathe”.
mention ‘farming’ in general but speaks of
71. (a) Refer to “it was not too cold for us to take
growth based on a particular kind of farming
off our gloves”, meaning it was not very cold.
that overexploits natural resources. So
statement 4 is not correct.] 72. (b) This is the best answer response. ‘Missing the
staircase’ is not the same as ‘not knowing
61. (c) or (d) If one goes by general knowledge, low-
where the staircase was’ – answer response
carbon growth involving adaptation and mitigation
(a). It was because the passage was “smoke-
“integrated into a climate smart development
filled” that he lost his way and went into
…” (Paragraph 1), statements 1 and 4 would
another room.
be correct. So (c) would be the correct answer
response. However, if one goes literally by 73. (d) Answer responses (a) and (c) are not justified
what the passage spells out, (d) would be the in the context of the passage, as the man
answer. The question, significantly, says “in did not do anything with conscious effort.

ST . 46
Sample Test 2 : General Studies Paper II (2012)

Merely escaping from the fire—answer response (d) may just lead to avoidable discussion and
(b)—would not be considered heroic. He saved delay.
a life, though unknowingly; he was therefore [The question, however, does not make it clear
called a hero.
as to what exactly the ‘complaint’ was and
74. (b) or (c) As a senior, your views will ultimately whether an investigation was made into it, or
matter and the report needs to reflect them whether the age issue just came to light by
as you will be responsible for it. If your accident!]
subordinate has a different opinion he needs
78. (c) Answer response (a) would be an act of
to reconsider it and revise the report accordingly
indiscipline. Response (b) shows irresponsibility
if you tell him to do so. As for urgency, your
and an inability to tackle a difficulty as it
subordinate will have to show the same
should be. Response (d) would get no desirable
urgency as you would need to if you were to
result as the boss has already justified not
revise the report yourself (option c). Option (c)
granting your leave because of the urgency
is also not wrong as such, especially
of the work. So the best option among those
considering the urgency and the difference of
views on the matter. Option (a) may take given is (c).
time—which you do not have. Response (d) 79. (d) Freebies are always welcomed but seldom
is the least appropriate as it shows arrogance respected. Of course, the conditions stated in
and poor interpersonal skills. the situation may lead one to think ‘free’
75. (a) The step to be taken is straightforward. supply of water, at least to the BPL families,
Responses (c) and (d) show a petty attitude is justified. However, there is always more
which, furthermore, would achieve nothing wastage when something is given free.
@UPSC_THOUGHTS

useful. Response (b) shows you to be Furthermore, you get a sense of self-respect
unreasonable. when you pay for something you use. So
responses (a) and (b) may be rejected. Having
76. (b) Response (a) would in all likelihood lead to
a fixed monthly charge in itself is not fair as
even more non-cooperation. Response (c) is
usage may vary from family to family. Also,
an extreme step and would also expose you
as the general income level is low, there may
as inept at handling team-mates. Response (d)
may get you more time but there is no be resentment if BPL families get free water.
certainty that your team members will begin So (c) is not acceptable. Option (d) is the best,
to cooperate. The problem is lack of cooperation, as it is fair even while allowing some concession
and this can only be tackled if the reasons to BPL families.
are known. 80. (c) Response (a) is unacceptable. Besides
77. (a) If the athlete has been allowed to participate encouraging corruption it shows a readiness
in the first place though over-age, the screening to run when asked to walk. Response (b) may
committee either has a good reason for it or show your unwillingness to pay bribes but it
must have been negligent in its work. So the is doubtful if you will get your work done; your
first step is to ask for a clarification from the application may just be kept pending. Sending
screening committee as to what criteria besides in a formal complaint—option (d)—would be
age, if any, governed the selection. Asking the necessary if a firm demand of a bribe has been
medal winner to return the medal (b) would made and after other avenues have been
depend on what the screening committee exhausted. It will also stall your application as
says, though at first sight it seems the correct a formal inquiry is set in motion. Option (c)
thing to do – uphold rules. Response (c) is is best as it will work as a check on the official
ridiculous, as the age has been established expecting a bribe and at the same time move
as per the situation in the question. Response your application.

ST . 47
Sample Test 3 : General Studies Paper II (2013)

SAMPLE TEST 3: GENERAL STUDIES PAPER II (2013)


Directions for the following 8 (eight) items: 1. Which of the following is closest to the view
of democracy as mentioned in the above
Read the following four passages and answer the
passage?
items that follow each passage. Your answers to
(a) The subject of democracy is a muddle
these items should be based on the passages only.
due to a desire to portray it as a Western
Passage-1 concept, ‘alien’ to non-Western countries.
(b) The language or imposition of democracy
The subject of democracy has become severely
is inappropriate. There is, however, a
muddled because of the way the rhetoric surrounding
need to consider this concept in the
it has been used in recent years. There is increasingly,
backdrop of culture of ‘own ways’ of
an oddly confused dichotomy between those who
non-Western society.
want to ‘impose’ democracy on countries in the non-
(c) While democracy is not essentially a
Western world (in these countries’ ‘own interest’, of
Western idea belonging exclusively to the
course) and those who are opposed to such
West, the institutional structure of current
‘imposition’ (because of the respect for the countries’ democratic practices has been their
‘own ways’). But the entire language of ‘imposition’, contribution.
used by both sides, is extraordinarily inappropriate (d) None of the statements (a), (b) and (c)
@UPSC_THOUGHTS

since it makes the implicit assumption that democracy given above is correct.
belongs exclusively to the West, taking it to be a
quintessentially ‘Western’ idea which has originated 2. With reference to the passage, the following
and flourished only in the West. assumptions have been made:
But the thesis and the pessimism it generates 1. Many of the non-Western countries are
about the possibility of democratic practice in the unable to have democracy because they
world would be extremely hard to justify. There were take democracy to be a specialised cultural
several experiments in local democracy in ancient product of the West.
2. Western countries are always trying to
India. Indeed, in understanding the roots of
impose democracy on non-Western
democracy in the world, we have to take an interest
countries.
in the history of people participation and public
reasoning in different parts of the world. We have Which of the above is/are valid assumption/
to look beyond thinking of democracy only in terms assumptions?
of European and American evolution. We would fail (a) 1 only
to understand the pervasive demands for participatory (b) 2 only
living, on which Aristotle spoke with far-reaching (c) Both 1 and 2
insight, if we take democracy to be a kind of a (d) Neither 1 nor 2
specialised cultural product of the West.
It cannot, of course, be doubted that the
Passage-2
institutional structure of the contemporary practice
of democracy is largely the product of European and Corporate governance is based on principles such
American experience over the last few centuries. This as conducting the business with all integrity and
is extremely important to recognise since these fairness, being transparent with regard to all
developments in institutional formats were immensely transactions, making all the necessary disclosures
innovative and ultimately effective. There can be little and decisions, complying with all the laws of the
doubt that there is a major ‘Western’ achievement land, accountability and responsibility towards the
here. stakeholders and commitment to conducting business

ST . 48
Sample Test 3 : General Studies Paper II (2013)

in an ethical manner. Another point which is criterion for foreign institutional investors
highlighted on corporate governance is the need for when they decide to buy a company.
those in control to be able to distinguish between
Select the correct answer using the codes given
personal and corporate funds while managing a
below:
company.
(a) 1 only
Fundamentally, there is a level of confidence that (b) 2 and 3 only
is associated with a company that is known to have (c) 1 and 3 only
good corporate governance. The presence of an (d) 1, 2 and 3
active group of independent directors on the board
contributes a great deal towards ensuring confidence Passage-3
in the market. Corporate governance is known to be Malnutrition most commonly occurs between the
one of the criteria that foreign institutional investors ages of six months and two years. This happens
are increasingly depending on when deciding on despite the child’s food requirements being less than
which companies to invest in. It is also known to that of an older child. Malnutrition is often attributed
have a positive influence on the share price of the to poverty, but it has been found that even in
company. Having a clean image on the corporate households where adults eat adequate quantities of
governance front could also make it easier for food, more than 50 per cent of children-under-five
companies to source capital at more reasonable costs. do not consume enough food. The child’s dependence
Unfortunately, corporate governance often becomes on someone else to feed him/her is primarily
the centre of discussion only after the exposure of responsible for the malnutrition. Very often the
a large scam. mother is working and the responsibility of feeding
@UPSC_THOUGHTS

3. According to the passage, which of the following the young child is left to an older sibling. It is
should be the practice/practices in good therefore crucial to increase awareness regarding the
corporate governance? child’s food needs and how to satisfy them.
1. Companies should always comply with 5. According to the passage, malnutrition in
labour and tax laws of the land.
children can be reduced
2. Every company in the country should
(a) if the children have regular intake of
have a government representative as one
food.
of the independent directors on the board
(b) after they cross the age of five.
to ensure transparency.
(c) if the food needs of younger children are
3. The manager of a company should never
known.
invest his personal funds in the company.
(d) if the responsibility of feeding younger
Select the correct answer using the codes given children is given to adults.
below:
6. According to the author, poverty is not the
(a) 1 only
main cause of malnutrition, but the fact that
(b) 2 and 3 only
1. taking care of younger ones is not a
(c) 1 and 3 only
(d) 1, 2 and 3 priority for working mothers.
2. awareness of nutritional needs is not
4. According to the passage, which of the following propagated by the Public Health
is/are the major benefit/benefits of good authorities.
corporate governance?
1. Good corporate governance leads to Select the correct answer using the codes given
increase in share price of the company. below:
2. A company with good corporate (a) 1 only
governance always increases its business (b) 2 only
turnover rapidly. (c) Both 1 and 2
3. Good corporate governance is the main (d) Neither 1 nor 2

ST . 49
Sample Test 3 : General Studies Paper II (2013)

Passage-4 (a) One can identify a single policy that can


reduce risk without any side-effect.
A number of empirical studies find that farmers
(b) No single risk-specific policy is sufficient
are risk-averse, though only moderately in many
to reduce agricultural risk.
cases. There is also evidence to show that farmers’
(c) Policies which affect risk indirectly can
risk aversion results in cropping patterns and input
eliminate it.
use designed to reduce risk rather than to maximise
(d) Government’s policy intervention can
income. Farmers adopt a number of strategies to
mitigate agricultural risk completely.
manage and cope with agricultural risks. These
include practices like crop and field diversification, 9. Consider the following statements:
non-farm employment, storage of stocks and strategic (i) A primary group is relatively smaller in
migration of family members. There are also size.
institutions ranging from share tenancy to kinship, (ii) Intimacy is an essential characteristic of
extended family and informal credit agencies. One a primary group.
major obstacle to risk sharing by farmers is that the (iii) A family may be an example of a primary
same type of risks can affect a large number of group.
farmers in the region. Empirical studies show that
In the light of the above statements, which one
the traditional methods are not adequate. Hence
of the following is true?
there is a need for policy interventions, especially
(a) All families are primary groups.
measures that cut across geographical regions.
(b) All primary groups are families.
Policies may aim at tackling agricultural risks
(c) A group of smaller size is always a
directly or indirectly. Examples of risk-specific policies
primary group.
@UPSC_THOUGHTS

are crop insurance, price stabilisation and the


(d) Members of a primary group know each
development of varieties resistant to pests and diseases.
other intimately.
Policies which affect risk indirectly are irrigation,
subsidised credit and access to information. No single 10. Four friends, A, B, C and D distribute some
risk-specific policy is sufficient to reduce risk and is money among themselves in such a manner
without side-effects, whereas policies not specific to that A gets one less than B, C gets 5 more than
risk influence the general situation and affect risks D, D gets 3 more than B. Who gets the smallest
only indirectly. Crop insurance, as a policy measure amount?
to tackle agricultural risk directly, deserves careful (a) A
consideration in the Indian context and in many (b) B
other developing countries because the majority of (c) C
farmers depend on rain-fed agriculture and in many (d) D
areas yield variability is the predominant cause of
Directions for the following 4 (four) items:
their income instability.
Read the following statements and answer the four
7. The need for policy intervention to mitigate
items that follow:
risks in agriculture is because
(a) farmers are extremely risk-averse. Five cities P, Q, R, S and T are connected by
(b) farmers do not know how to mitigate different modes of transport as follows:
risks. P and Q are connected by boat as well as rail.
(c) the methods adopted by farmers and S and R are connected by bus and boat.
existing risk sharing institutions are not Q and T are connected by air only.
adequate. P and R are connected by boat only.
(d) majority of farmers depend on rain-fed T and R are connected by rail and bus.
agriculture.
11. Which mode of transport would help one to
8. Which of the following observations emerges reach R starting from Q, but without changing
from the above passage? the mode of transport?

ST . 50
Sample Test 3 : General Studies Paper II (2013)

(a) Boat (b) A, B, C and Z


(b) Rail (c) A, B, C and X
(c) Bus (d) A, W, Y and Z
(d) Air
17. If all the three males are selected, then how
12. If a person visits each of the places starting many combinations of four-member teams are
from P and gets back to P, which of the possible?
following places must he visit twice? (a) 1
(a) Q (b) 2
(b) R (c) 3
(c) S (d) 4
(d) T 18. The music director of a film wants to select four
13. Which one of the following pairs of cities is persons to work on different aspects of the
connected by any of the routes directly without composition of a piece of music. Seven persons
going to any other city? are available for this work; they are Rohit,
(a) P and T Tanya, Shobha, Kaushal, Kunal, Mukesh and
(b) T and S Jaswant.
(c) Q and R Rohit and Tanya will not work together. Kunal
(d) None of these and Shobha will not work together. Mukesh
and Kunal want to work together.
14. Between which two cities among the pairs of
cities given below are there maximum travel Which of the following is the most acceptable
@UPSC_THOUGHTS

options available? group of people that can be selected by the


(a) Q and S music director?
(b) P and R (a) Rohit, Shobha, Kunal and Kaushal
(c) P and T (b) Tanya, Kaushal, Shobha and Rohit
(d) Q and R (c) Tanya, Mukesh, Kunal and Jaswant
(d) Shobha, Tanya, Rohit and Mukesh
Directions for the following 3 (three) items:
19. Five people A, B, C, D and E are seated about
Read the following passage and answer the three
a round table. Every chair is spaced equidistant
items that follow: from adjacent chairs.
A tennis coach is trying to put together a team (i) C is seated next to A.
of four players for the forthcoming tournament. For (ii) A is seated two seats from D.
this 7 players are available: males A, B and C; and (iii) B is not seated next to A.
females W, X, Y and Z. All players have equal
capability and at least 2 males will be there in the Which of the following must be true?
team. For a team of four, all players must be able I. D is seated next to B.
to play with each other. But, B cannot play with W, II. E is seated next to A.
C cannot play with Z and W cannot play with Y. Select the correct answer from the codes given
15. If Y is selected and B is rejected, the team will below:
(a) I only
consist of which one of the following groups?
(b) II only
(a) A, C, W and Y
(c) Both I and II
(b) A, C, X and Y
(d) Neither I nor II
(c) A, C, Y and Z
(d) A, W, Y and Z Directions for the following 3 (three) items:
16. If B is selected and Y is rejected, the team will Examine carefully the following statements and
consist of which one of the following groups? answer the three items that follow:
(a) A, B, C and W Out of four friends A, B, C and D,

ST . 51
Sample Test 3 : General Studies Paper II (2013)

A and B play football and cricket, Sovereign risk concerns, particularly in the Euro area,
B and C play cricket and hockey, affected financial markets for the greater part of the
A and D play basketball and football, year, with the contagion of Greece’s sovereign debt
C and D play hockey and basketball. problem spreading to India and other economies by
way of higher-than-normal levels of volatility.
20. Who does not play hockey?
The funding constraints in international financial
(a) D
markets could impact both the availability and cost
(b) C
of foreign funding for banks and corporates. Since
(c) B
the Indian financial system is bank dominated,
(d) A
banks’ ability to withstand stress is critical to overall
21. Who plays football, basketball and hockey? financial stability. Indian banks, however, remain
(a) D robust, notwithstanding a decline in capital to risk-
(b) C weighted assets ratio and a rise in non-performing
(c) B asset levels in the recent past. Capital adequacy levels
(d) A remain above the regulatory requirements. The
financial market infrastructure continues to function
22. Which game do B, C and D play?
without any major disruption. With further
(a) Basketball
globalisation, consolidation, deregulation, and
(b) Hockey
diversification of the financial system, the banking
(c) Cricket
business may become more complex and riskier.
(d) Football
Issues like risk and liquidity management and
23. Geeta is older than her cousin Meena. Meena’s enhancing skill therefore assume greater significance.
@UPSC_THOUGHTS

brother Bipin is older than Geeta. When Meena


24. According to the passage, the financial markets
and Bipin visit Geeta, they like to play chess.
in the emerging market economies including
Meena wins the game more often than Geeta.
India had the adverse impact in recent years
Based on the above information, four due to
conclusions, as given below, have been made. 1. weak global economic prospects.
Which one of these logically follows from the 2. uncertainties in the international financial
information given above? markets.
(a) While playing chess with Geeta and 3. sovereign risk concerns in the Euro area.
Meena, Bipin often loses. 4. bad monsoons and the resultant crop
(b) Geeta is the oldest among the three. loss.
(c) Geeta hates to lose the game.
Select the correct answer using the codes given
(d) Meena is the youngest of the three. below:
Directions for the following 4 (four) items: (a) 1 and 2 only
(b) 1, 2 and 3
Read the following passage and answer the four
(c) 2 and 3 only
items that follow. Your answers to these items
(d) 2, 3 and 4
should be based on the passage only.
25. The Indian financial markets are affected by
Passage
global changes mainly due to the
Financial markets in India have acquired greater (a) increased inflow of remittances from
depth and liquidity over the years. Steady reforms abroad.
since 1991 have led to growing linkages and (b) enormous increase in the foreign exchange
integration of the Indian economy and its financial reserves.
system with the global economy. Weak global (c) growing global linkages and integration
economic prospects and continuing uncertainties in of the Indian financial markets.
the international financial markets therefore, have (d) contagion of Greece’s sovereign debt
had their impact on the emerging market economies. problem.

ST . 52
Sample Test 3 : General Studies Paper II (2013)

26. According to the passage, in the Indian financial (b) D belongs to R


system, banks’ ability to withstand stress is (c) A belongs to Q
critical to ensure overall financial stability (d) B belongs to S
because Indian financial system is
30. Seven men, A, B, C, D, E, F and G are standing
(a) controlled by the Government of India.
in a queue in that order. Each one is wearing
(b) less integrated with banks.
a cap of a different colour like violet, indigo,
(c) controlled by the Reserve Bank of India.
blue, green, yellow, orange and red. D is able
(d) dominated by banks.
to see in front of him green and blue, but not
27. Risk and liquidity management will assume violet. E can see violet and yellow, but not red.
more importance in the Indian banking system G can see caps of all colours other than orange.
in future due to If E is wearing an indigo coloured cap, then
1. further globalisation. the colour of the cap worn by F is
2. more consolidation and deregulation of (a) Blue
the financial system. (b) Violet
3. further diversification of the financial (c) Red
system. (d) Orange
4. more financial inclusion in the economy.
31. There are some balls of red, green and yellow
Select the correct answer using the codes given colour lying on a table. There are as many red
below: balls as there are yellow balls. There are twice
(a) 1, 2 and 3 as many yellow balls as there are green ones.
(b) 2, 3 and 4 The number of red balls
@UPSC_THOUGHTS

(c) 1 and 2 only (a) is equal to the sum of yellow and green
(d) 3 and 4 only balls.
28. There are five hobby clubs in a college viz, (b) is double the number of green balls.
photography, yachting, chess, electronics and (c) is equal to yellow balls minus green balls.
gardening. The gardening group meets every (d) cannot be ascertained.
second day, the electronics group meets every Directions for the following 2 (two) items:
third day, the chess group meets every fourth
day, the yachting group meets every fifth day Read the following passage and answer the two
and the photography group meets every sixth items that follow. Your answers to these items
day. How many times do all the five groups should be based on the passage only.
meet on the same day within 180 days? Passage
(a) 3
(b) 5 Crude mineral oil comes out of the earth as a
(c) 10 thick brown or black liquid with a strong smell. It
(d) 18 is a complex mixture of many different substances,
each with its own individual qualities. Most of them
29. A, B, C, D and E belong to five different cities
are combinations of hydrogen and carbon in varying
P, Q, R, S and T (not necessarily in that order).
proportions. Such hydrocarbons are also found in
Each one of them comes from a different city.
other forms such as bitumen, asphalt and natural
Further it is given that:
gas. Mineral oil originates from the carcasses of tiny
1. B and C do not belong to Q.
animals and from plants that live in the sea. Over
2. B and E do not belong to P and R.
millions of years, these dead creatures form large
3. A and C do not belong to R, Sand T.
deposits under the sea-bed and ocean currents cover
4. D and E do not belong to Q and T.
them with a blanket of sand and silt. As this mineral
Which one of the following statements is not hardens, it becomes sedimentary rock and effectively
correct? shuts out the oxygen, so preventing the complete
(a) C belongs to P decomposition of the marine deposits underneath.

ST . 53
Sample Test 3 : General Studies Paper II (2013)

The layers of sedimentary rock become thicker and (a) 24


heavier. Their pressure produces heat, which (b) 33
transforms the tiny carcasses into crude oil in a (c) 42
process that is still going on today. (d) 66
32. Mineral oil deposits under the sea do not get Directions for the following 7 (seven) items:
completely decomposed because they
Read the following two passages and answer the
(a) are constantly washed by the ocean
items that follow each passage. Your answers to
currents.
these items should be based on the passages only.
(b) become rock and it prevents oxygen from
entering them. Passage -1
(c) contain a mixture ‘of hydrogen and The law in many parts of the world increasingly
carbon. restricts the discharge of agricultural slurry into
(d) are carcasses of organisms lying in saline watercourses. The simplest and often the most
conditions. economically sound practice returns the material to
33. Sedimentary rock leads to the formation of oil the land as semisolid manure or as sprayed slurry.
deposits because This dilutes its concentration in the environment to
(a) there are no saline conditions below it. what might have occurred in a more primitive and
(b) it allows some dissolved oxygen to enter sustainable type of agriculture and converts pollutant
the dead organic matter below it. into fertilizer. Soil microorganisms decompose the
(c) weight of overlying sediment layers causes organic components of sewage and slurry and most
of the mineral nutrients become available to be
@UPSC_THOUGHTS

the production of heat.


(d) it contains the substances that catalyse absorbed again by the vegetation.
the chemical reactions required to change The excess input of nutrients, both nitrogen and
dead organisms into oil. phosphorus-based, from agricultural runoff (and
human sewage) has caused many ‘healthy’
34. In a class of 45 students, a boy is ranked 20th. oligotrophic lakes (low nutrient concentrations, low
When two boys joined, his rank was dropped plant productivity with abundant water weeds, and
by one. What is his new rank from the end? clear water) to change to eutrophic condition where
(a) 25th high nutrient inputs lead to high phytoplankton
(b) 26th productivity (sometimes dominated by bloom-forming
(c) 27th toxic species). This makes the water turbid, eliminates
(d) 28th large plants and, in the worst situations, leads to
35. A thief running at 8 km/hr is chased by a anoxia and fish kills; so called cultural eutrophication.
policeman whose speed is 10 km/hr. If the thief Thus, important ecosystem services are lost, including
is 100 m ahead of the policeman, then the time the provisioning service of wild-caught fish and the
required for the policeman to catch the thief cultural services associated with recreation.
will be The process of cultural eutrophication of lakes
(a) 2 min has been understood for some time. But only recently
(b) 3 min did scientists notice huge ‘dead zones’ in the oceans
(c) 4 min near river outlets, particularly those draining large
(d) 6 min catchment areas such as the Mississippi in North
America and the Yangtze in China. The nutrient-
36. A train travels at a certain average speed for enriched water flows through streams, rivers and
a distance of 63 km and then travels a distance lakes, and eventually to the estuary and ocean where
of 72 km at an average speed of 6 km/hr more the ecological impact may be huge, killing virtually
than its original speed. If it takes 3 hours to all invertebrates and fish in areas up to 70,000 km2
complete the total journey, what is the original in extent. More than 150 sea areas worldwide are
speed of the train in km/hr? now regularly starved of oxygen as a result of

ST . 54
Sample Test 3 : General Studies Paper II (2013)

decomposition of algal blooms, fuelled particularly by 40. What is/are the characteristics of a water body
nitrogen from agricultural runoff of fertilisers and with cultural eutrophication?
sewage from large cities. Oceanic dead zones are 1. Loss of ecosystem services
typically associated with industrialised nations and 2. Loss of flora and fauna
usually lie off countries that subsidise their agriculture, 3. Loss of mineral nutrients
encouraging farmers to increase productivity and use
more fertiliser. Select the correct answer using the codes given
below:
37. According to the passage, why should the (a) 1 only
discharge of agricultural slurry into (b) 1 and 2 only
watercourses be restricted? (c) 2 and 3 only
1. Losing nutrients in this way is not a good (d) 1, 2 and 3
practice economically.
2. Watercourses do not contain the 41. What is the central theme of this passage?
microorganisms that can decompose (a) Appropriate legislation is essential to
organic components of agricultural slurry. protect the environment.
3. The discharge may lead to the (b) Modern agriculture is responsible for the
eutrophication of water bodies. destruction of environment.
Select the correct answer using the codes given (c) Improper waste disposal from agriculture
below: can destroy the aquatic ecosystems.
(a) 1 only (d) Use of chemical fertilisers is undesirable
(b) 2 and 3 only in agriculture.
(c) 1 and 3 only Passage-2
@UPSC_THOUGHTS

(d) 1, 2 and 3
The miseries of the world cannot be cured by
38. The passage refers to the conversion of “pollutant physical help only. Until man’s nature changes, his
to fertiliser”. What is pollutant and what is physical needs will always arise, and miseries will
fertiliser in this context? always be felt, and no amount of physical help will
(a) Decomposed organic component of slurry remove them completely. The only solution of the
is pollutant and microorganisms in soil problem is to make mankind pure. Ignorance is the
constitute fertiliser. mother of evil and of all the misery we see. Let men
(b) Discharged agricultural slurry is pollutant
have light, let them be pure and spiritually strong
and decomposed slurry in soil is fertiliser.
and educated; then alone will misery cease in the
(c) Sprayed slurry is pollutant and
world. We may convert every house in the country
watercourse is fertiliser.
into a charitable asylum, we may fill the land with
(d) None of the above expressions is correct
hospitals, but human misery will continue until
in this context.
man’s character changes.
39. According to the passage, what are the effects
of indiscriminate use of fertilisers? 42. According to the passage, which of the following
1. Addition of pollutants to the soil and statements is most likely to be true as the reason
water. for man’s miseries?
2. Destruction of decomposer micro- (a) The poor economic and social conditions
organisms in soil. prevailing in society.
3. Nutrient enrichment of water bodies. (b) The refusal on the part of man to change
4. Creation of algal blooms. his character.
(c) The absence of physical and material
Select the correct answer from the codes given
help from his society.
below:
(d) Ever increasing physical needs due to
(a) 1, 2 and 3 only
(b) 1, 3 and 4 only changing social structure.
(c) 2 and 4 only 43. With reference to the passage, the following
(d) 1, 2, 3 and 4 assumptions have been made:

ST . 55
Sample Test 3 : General Studies Paper II (2013)

1. The author gives primary importance to 46. Problem figures:


physical and material help in eradicating
human misery.
2. Charitable homes, hospitals, etc. can
remove human misery to a great extent.
Which of the assumptions is/are valid? 1 2 3 4
(a) 1 only Answer figures:
(b) 2 only
(c) Both 1 and 2
(d) Neither 1 nor 2
44. Consider the following figures 1, 2, 3 and 4:
(a) (b) (c) (d)
47. Consider the following diagrams:
x men, working at constant speed, do a certain
job in y days. Which one of these diagrams
1 2 3 4 shows the relation between x and y?
In the figures from 1 to 4 above, two symbols y y
are shown to change their position in a regular
direction. Following the same sequence, which one
of the following will appear at the fifth stage?
Days

Days
@UPSC_THOUGHTS

0 x 0 x
Men Men
(a) (b) (c) (d) Diagram I Diagram II
y y
Directions for the following 2 (two) items:
In each item, there are two sets of figures: first four
figures are named Problem figures and the next four
Days

Days

figures are named Answer figures, indicated as (a), (b),


(c) and (d). The problem figures follow a particular
sequence. In accordance with the same, which one of the
four answer figures should appear as the fifth figure? 0 x 0 x
Men Men
45. Problem figures: diagram III diagram IV

(a) diagram I
(b) diagram II
(c) diagram III
(d) diagram IV
1 2 3 4
Answer figures: 48. Consider the following matrix:
3 370 7
2 224 6
1 730 X
What is the number at ‘X’ in the above matrix?
(a) (b) (c) (d) (a) 5

ST . 56
Sample Test 3 : General Studies Paper II (2013)

(b) 8 (a) 2 and 3


(c) 9 (b) 6 and 1
(d) 11 (c) 1 and 4
(d) 3 and 1
49. Four cars are hired at the rate of ` 6 per km
plus the cost of diesel at ` 40 a litre. In this Directions for the following 5 (five) items:
context, consider the details given in the
Study the two figures given below and answer the
following table:
five items that follow:
Car Mileage Hours Total
(km/l) Payment (`) Male Female
32
A 8 20 2120
28
B 10 25 1950
24
C 9 24 2064 20
D 11 22 1812 16
Which car maintained the maximum average 12
speed? 8
(a) Car A 4
(b) Car B
0
(c) Car C PHYSICS MATHEMATICS CHEMISTRY BOTANY PSYCHOLOGY ECONOMICS

(d) Car D Figure 1: Number of Professors in selected


@UPSC_THOUGHTS

50. Examine the following three figures in which disciplines in a University by sex
the numbers follow a specific pattern:

84 81 88 35 – 44
14 12 18 9 ? 11 20%
40% 25 –34
The missing number (?) in the third figure
10%
above is
(a) 7 60 –65
(b) 16 30%
(c) 21 45 –59
(d) 28
51. A cube has six numbers marked 1, 2, 3, 4, 5 Figure 2: Age of Physics Professors
and 6 on its faces. Three views of the cube are
shown below: 52. How many Physics professors belong to the age
group 35–44?
1 3 3 (a) 18
4 2 6 (b) 16
6 1 5 (c) 14
(d) 12
What possible numbers can exist on the two
faces marked A and B , respectively on the 53. Which one of the following diciplines has the
cube? highest ratio of males to females?
B (a) Physics
(b) Mathematics
5 (c) Chemistry
A
(d) Economics

ST . 57
Sample Test 3 : General Studies Paper II (2013)

54. What percentage of all Psychology professors Directions for the following 2 (two) items:
are females?
Read the following passage and answer the two
(a) 40%
items that follow. Your answers to these items
(b) 50%
should be based on the passage only.
(c) 60%
(d) 70%
Passage
55. If the number of female Physics professors in
the age group 25 – 34 equals 25% of all the Ecological research over the last quarter of the
Physics professors in that age group, then what century has established the deleterious effects of
is the number of male Physics professors in the habitat fragmentation due to mining, highways and
age group 25 – 34? such other intrusions on forests. When a large block
(a) 9 of forests gets fragmented into smaller bits, the edges
(b) 6 of all these bits come into contact with human
(c) 3 activities resulting in the degradation of entire forests.
(d) 2 Continuity of forested landscapes and corridors gets
disrupted affecting several extinction-prone species of
56. If the Psychology professors in the University
wildlife. Habitat fragmentation is therefore considered
constitute 2% of all the professors in the
as the most serious threat to biodiversity conservation.
university, then what is the number of professors
Ad hoc grants of forest lands to mining companies
in the university?
coupled with rampant, illegal mining is aggravating
(a) 400
this threat.
(b) 500
@UPSC_THOUGHTS

(c) 600 58. What is the central focus of this passage?


(d) 700 (a) Illegal mining in forests
(b) Extinction of wildlife
57. Consider the following figures:
(c) Conservation of nature
(d) Disruption of habitat
59. What is the purpose of maintaining the
continuity of forested landscapes and corridors?
(1) (2) (3) (4) 1. Preservation of biodiversity.
2. Management of mineral resources.
3. Grant of forest lands for human activities.

? Select the correct answer using the codes given


below:
(5) (6) (7) (a) 1 only
(b) 1 and 2
Which one of the following figures would (c) 2 and 3
logically come in the 7th position indicated (d) 1, 2 and 3
above by a question mark? 60. In a rare coin collection, there is one gold coin
for every three non-gold coins. 10 more gold
coins are added to the collection and the ratio
of gold coins to non-gold coins is then 1 : 2.
(a) (b)
Based on the information; the total number of
coins in the collection now becomes
(a) 90
(b) 80
(c) (d) (c) 60
(d) 50

ST . 58
Sample Test 3 : General Studies Paper II (2013)

61. A gardener has 1000 plants. He wants to plant (a) Two hours
them in such a way that the number of rows (b) Two and a half hours
and the number of columns remains the same. (c) Five and a half hours
What is the minimum number of plants that (d) Four hours
he needs more for this purpose?
Directions for the following 8 (eight) items:
(a) 14
(b) 24 The following eight items (items 67 to 74) are
(c) 32 based on three passages in English to test the
(d) 34 comprehension of English language and therefore
these items do not have Hindi version. Read
62. A sum of ` 700 has to be used to give seven
each passage and answer the items that follow.
cash prizes to the students of a school for their
overall academic performance. If each prize is
` 20 less than its preceding prize, what is the English Passage-1
least value of the prize?
Seven-year-old Jim came home from the park
(a) ` 30
without his new bicycle. “An old man and a little
(b) ` 40
boy borrowed it,” he explained. “They are going to
(c) ` 60
bring it back at four o’clock.” His parents were upset
(d) ` 80
that he had given his expensive new bicycle, but
63. Out of 120 applications for a post, 70 are male were secretly proud of his kindness and faith. Came
and 80 have a driver’s license. What is the ratio four o’clock, no bicycle. The parents were anxious.
between the minimum to maximum number of But at 4:30, the door bell rang, and there stood a
@UPSC_THOUGHTS

males having driver’s licence? happy man and a boy, with the bicycle and a box
(a) 1 to 2 of chocolates. Jim suddenly disappeared into his
(b) 2 to 3 bedroom, and then came running out. “All right,”
(c) 3 to 7 he said, after examining the bicycle. ‘’You can have
(d) 5 to 7 your watch back!”
64. In a garrison, there was food for 1000 soldiers 67. When Jim came home without his bicycle, his
for one month. After 10 days, 1000 more parents
soldiers joined the garrison. How long would (a) were angry with him.
the soldiers be able to carry on with the (b) were worried.
remaining food? (c) did not feel concerned.
(a) 25 days (d) were eager to meet the old man and the
(b) 20 days little boy.
(c) 15 days
(d) 10 days 68. Jim returned the watch to the old man and the
little boy because
65. The tank-full petrol in Arun’s motor-cycle lasts
(a) they gave him chocolates.
for 10 days. If he starts using 25% more
(b) his father was proud of him.
everyday, how many days will the tank-full
(c) he was satisfied with the condition of his
petrol last?
bicycle.
(a) 5
(d) they were late only by 30 minutes.
(b) 6
(c) 7
(d) 8 English Passage-2
66. A person can walk a certain distance and drive It was already late when we set out for the next
back in six hours. He can also walk both ways town, which according to the map was about fifteen
in 10 hours. How much time will he take to kilometres away on the other side of the hills. There
drive both ways? we felt that we would find a bed for the night.

ST . 59
Sample Test 3 : General Studies Paper II (2013)

Darkness fell soon after we left the village, but luckily anywhere. Individual liberty would have become
we met no one as we drove swiftly along the narrow social anarchy.
winding road that led to the hills. As we climbed
72. It was pointed out to the lady that she should
higher, it became colder and rain began to fall,
walk on the pavement because she was
making it difficult at times to see the road. I asked
(a) a pedestrian.
John, my companion, to drive more slowly. After we
(b) carrying a basket.
had travelled for about twenty kilometres, there was
(c) stout.
still no sign of the town which was marked on the
(d) an old lady.
map. We were beginning to get worried. Then
without warning, the car stopped and we found we 73. The lady refused to move from the middle of
had run out of petrol. the street because
(a) she was not afraid of being killed.
69. The author asked John to drive more slowly
(b) she felt that she was entitled to do
because
whatever she liked.
(a) the road led to the hills.
(c) she did not like walking on the pavement.
(b) John was an inexperienced driver.
(d) she was confused.
(c) the road was not clearly visible.
(d) they were in wilderness. 74. The old lady failed to realise that
(a) she was not really free.
70. The travellers set out for the town although it
(b) her liberty was not unlimited.
was getting dark because
(c) she was an old person.
(a) they were in a hurry.
(d) roads are made for motor vehicles only.
@UPSC_THOUGHTS

(b) the next town was a short distance away


and was a hill resort. Directions for the following 6 (six) items:
(c) they were in wilderness. Given below are six items. Each item describes a
(d) the next town was a short distance away situation and is followed by four possible responses.
and promised a good rest for the night. Indicate the response you find most appropriate.
71. The travellers were worried after twenty Choose only one response for each item. The
kilometres because responses will be evaluated based on the level of
(a) it was a lonely countryside. appropriateness for the given situation.
(b) they probably feared of having lost their Please attempt all the items. There is no
way. penalty for wrong answers for these six items.
(c) the rain began to fall
(d) it was getting colder as they drove. 75. You are the head of your office. There are
certain houses reserved for the allotment to the
English Passage-3 office staff and you have been given the
A stout old lady was walking with her basket discretion to do so. A set of rules for the
down the middle of a street in Petrograd to the great allotment of the houses has been laid down by
confusion of the traffic and no small peril to herself. you and has been made public. Your personal
It was pointed out to her that the pavement was secretary, who is very close to you, comes to
the place for foot-passengers, but she replied, “I’m you and pleads that as his father is seriously
going to walk where I like. We’ve got liberty now.” ill, he should be given priority in allotment of
It did not occur to the dear lady that if liberty entitled a house. The office secretariat that examined
the foot-passenger to walk down the middle of the the request as per the rules turns down the
road it also entitled the taxi-driver to drive on the request and recommends the procedure to be
pavement, and that the end of such liberty would followed according to the rules. You do not
be universal chaos. Everything would be getting in want to annoy your personal secretary. In such
everybody else’s way and nobody would get circumstances, what would you do?

ST . 60
Sample Test 3 : General Studies Paper II (2013)

(a) Call him over to your room and personally (b) Tell him that the matter is actually to be
explain why the allotment cannot be dealt with by a junior officer and that
done. he should give the application to him.
(b) Allot the house to him to win his loyalty. (c) Call one of your senior subordinate officers
(c) Agree with the office note to show that and ask him to solve the villager’s problem.
you are not biased and that you do not (d) Quickly take the application from him,
indulge in favouritism. ask him a few relevant questions
(d) Keep the file with you and not pass any regarding his problem and then proceed
orders. to the meeting.
76. While travelling in a Delhi-registered commercial 78. There is a shortage of sugar in your District
taxi from Delhi to an adjacent city (another where you are the District Magistrate. The
state), your taxi driver informs you that as he Government has ordered that only a maximum
has no permit for running the taxi in that city, amount of 30 kg sugar is to be released for
he will stop at its Transport Office and pay the wedding celebrations. A son of your close
prescribed fee of ` forty for a day. While paying friend is getting married and your friend
the fee at the counter you find that the requests you to release at least 50 kg sugar for
transport clerk is taking an extra fifty rupees his son’s wedding. He expresses annoyance
for which no receipt is being given. You are when you tell him about the government’s
in a hurry for your meeting. In such restrictions on this matter. He feels that since
circumstances, what would you do? you are the District Magistrate you can release
(a) Go up to the counter and ask the clerk any amount. You do not want to spoil your
@UPSC_THOUGHTS

to give back the money which he has friendship with him. In such circumstances,
illegally taken. how would you deal with the situation?
(b) Do not interfere at all as this is a matter (a) Release the extra amount of sugar which
between the taxi driver and the tax your friend has requested for
authorities. (b) Refuse your friend the extra amount and
(c) Take note of the incident and subsequently strictly follow the rules.
report the matter to the concerned (c) Show your friend the copy of the
authorities. government instructions and then
(d) Treat it as a normal affair and simply persuade him to accept the lower amount
forget about it. as prescribed in the rules.
(d) Advise him to directly apply to the
77. A person lives in a far-off village which is
allotting authority and inform him that
almost two hours by bus. The villager’s
you will not interfere in this matter.
neighbour is a very powerful landlord who is
trying to occupy the poor villager’s land by 79. You are incharge of implementing the Family
force. You are the District Magistrate and busy Planning programme in an area where there
in a meeting called by a local minister. The is a strong opposition to the present policy. You
villager has come all the way, by bus and on want to convince the residents of the need for
foot, to see you and give an application seeking keeping small families. What would be the best
protection from the powerful landlord. The way of communicating this message?
villager keeps on waiting outside the meeting (a) By logically explaining to the residents
hall for an hour. You come out of the meeting the need for family planning to improve
and are rushing to another meeting. The villager the health and living standards.
follows you to submit his application. What (b) By encouraging late marriages and proper
would you do? spacing of children.
(a) Tell him to wait for another two hours (c) By offering incentives for adopting family
till you come back from your next meeting. planning devices.

ST . 61
Sample Test 3 : General Studies Paper II (2013)

(d) By asking people who have been sterilised informs you that his son will suffer from
or are using contraceptives to directly depression if he fails in this examination. In
talk to the residents. such circumstances, what would you do?
80. You are a teacher in a university and are (a) In view of the help he had given you,
setting a question paper on a particular subject. extend your help to him.
One of your colleagues, whose son is preparing (b) Regret that you cannot be of any help
for the examination on that subject, comes to to him.
you and informs you that it is his son’s last (c) Explain to your colleague that this would
chance to pass that examination and asks be violating the trust of the University
whether you could help him by indicating authorities and you are not in a position
what questions are going to be in the to help him.
examination. In the past, your colleague has (d) Report the conduct of your colleague to
helped you in another matter. Your colleague the higher authorities.

@UPSC_THOUGHTS

ST . 62
Sample Test 3 : General Studies Paper II (2013)

PRACTICE ANSWER SHEET

Directions: Use HB Pencil. Erase completely to change. Example

1. 17. 33. 49. 65.

2. 18. 34. 50. 66.

3. 19. 35. 51. 67.

4. 20. 36. 52. 68.

5. 21. 37. 53. 69.

6. 22. 38. 54. 70.


@UPSC_THOUGHTS

7. 23. 39. 55. 71.

8. 24. 40. 56. 72.

9. 25. 41. 57. 73.

10. 26. 42. 58. 74.

11. 27. 43. 59. 75.

12. 28. 44. 60. 76.

13. 29. 45. 61. 77.

14. 30. 46. 62. 78.

15. 31. 47. 63. 79.

16. 32. 48. 64. 80.

ST . 63
Sample Test 3 : General Studies Paper II (2013)

ANSWERS AND EXPLANATORY NOTES

1. (c) The last paragraph of the passage supports C gets x + 3 + 5 (or x + 8)


this. ∴ A gets the smallest amount.
2. (d) [Alternatively: D gets 3 more than B, and C
3. (a) There is no mention of ‘government gets 5 more than D; so C obviously gets more
representative’. In fact the passage speaks of than B. A is the only one to get less than B,
‘independent directors’. So nothing supports 2. so obviously A gets the least.]
As for 3, to be able to distinguish between For items 11 to 14, a sketch would help.
personal and corporate funds does not mean
P boat
that a manager cannot invest his own funds
rail Q
in the company.
4. (a) There is no mention in the passage of increase
boat air
in business turnover because of corporate
governance. (The use of ‘always’ in the
statement should make you suspect the rail
correctness of the statement.) So 2 is s
bu R bus T
unacceptable. The passage says that corporate t
boa
governance is ‘one’ of the criteria, not the S
‘main’ criterion for foreign investors deciding on 11. (a) Q to P to R by boat.
investing in a company; the passage does not
@UPSC_THOUGHTS

12. (b) P → Q → T → R → S → R → P. However,


mention anything about the investors buying
one goes to S from P will have to visit R twice.
companies. So 3 is not correct.
13. (d)
5. (c) The last sentence of the passage suggests
this. 14. (b)
6. (d) The passage says nothing about working 15. (b) As W cannot play with Y, response (a) may
mothers not considering the care of younger be rejected. In response (b) there are two
children as a ‘priority’. Statement 1 is wrong. males (A and C) and none of the given players
While lack of awareness is a major factor in face any restrictions, so it is the correct one.
child malnutrition, the passage does not tell us Check out (c) and (d). In (c) there are C and
about the public health authorities not Z who cannot be on the same team; in (d)
propagating such awareness. One may deduce/ again there are W and Y who cannot play
assume this but the conclusion goes beyond together and there is only one male on the
what the passage says. So 2 is not acceptable. team.
7. (c) The penultimate sentence of the first paragraph 16. (c) As B cannot play with W, (a) is not possible.
suggests this. As C cannot play with Z, (b) is not possible.
No restrictions are there on the players in the
8. (b)
team in (c), and it fulfils given conditions. As
9. (d) Notice the word ‘may’ in statement (iii), so it one condition is for B to be selected and Y
negates the ‘all’ in responses (a) and (b). None rejected, (d) is obviously not possible.
of the statements makes a categorical statement
17. (b) Under the given conditions, A, B and C are
about the size of all primary groups, so (c) is
on the team, and W and Z cannot be on the
not correct. Response (d) follows from statement
team. So the possible teams are A, B, C, X
(ii).
and A, B, C, Y.
10. (a) Assuming B gets Rs x,
18. (c) As Shobha and Kunal cannot work together,
A gets x – 1; response (a) is to be rejected. As Tanya and
D gets x + 3 Rohit do not want to work together, response

ST . 64
Sample Test 3 : General Studies Paper II (2013)

(b) cannot be accepted. Response (c) gives an 25. (c) Refer to the first paragraph.
acceptable group. As Rohit and Tanya do not 26. (d) Refer to the sentence, “Since the Indian …
want to work together, (d) is not acceptable. stability” in the second paragraph.
19. (c) The sketch makes it clear: 27. (a) Refer to the last two sentences of the second
A paragraph. There is no mention of ‘financial
inclusion’ in the passage, so statement 4 is not
acceptable.
E 28. (a) Find the L.C.M. of the days: 2, 3, 4, 5 and
C
6. The L.C.M. is 60. So the groups will meet
on the same day every 60 days. In 180 days
they will meet on the same day 180 ÷ 60 =
3 times.

B 29. (d) Remember the condition that each person


D
(D) (B) comes from a different city.
Make a chart from the given data.
As B cannot be next to A and D is two seats
away from A, B and D can occupy two Persons Cities
positions. Either way they are placed next to P Q R S T
each other and E has to be next to A. So both
A ✔ ✕ ✕ ✕
statements I and II are true.
B ✕ ✕ ✕
For items 20 to 22, make a chart from the
@UPSC_THOUGHTS

given information. C ✔ ✕ ✕ ✕ ✕
Friends Football Cricket Hockey Basketball D ✕ ✔ ✕
A ✔ ✔ ✔ E ✕ ✕ ✕ ✔ ✕
B ✔ ✔ ✔ Clearly C can come from P only and E can
C ✔ ✔ ✔ come from S only.
D ✔ ✔ ✔ At this stage you know B cannot be from S,
so (d) is not correct. [A can only come from
A more rough and ready version would be: Q, as from the options of P and Q, P is already
A – Fb, Cr, Bb reserved for C; so responses (a) and (c) are
B – Fb, Cr, Ho
correct. As C and E belong to P and S,
C – Cr, Ho, Bb
D – Bb, Fb, Ho respectively, D can only come from R; so
response (c) is also correct.]
Now you can answer the questions easily.
30. (c) Sketch a diagram.
20. (d)
21. (a) A Green/Blue (D can see green and
B blue, not violet)
22. (b)
C
23. (d) Nothing in the information given tells us whether
anyone loses often or hates to lose; so D Violet (E can see violet and yellow
responses (a) and (c) may be rejected. We are but D cannot see violet)
told Bipin is older than Geeta, so (b) cannot E Indigo (given)
be correct. As we are told that Bipin is older
F Red
than Geeta and Geeta is older than Meena,
it follows that Meena is the youngest of the G Orange (As G cannot see orange, he
three—response (d). must be wearing an orange cap.)
24. (b) Statements 1, 2 and 3 are supported by the [Alternatively, from what D and E can see and
first paragraph of the passage. There is no what E is wearing, we can eliminate green,
mention of bad monsoons in the passage. blue, violet and yellow, and indigo. If G can

ST . 65
Sample Test 3 : General Studies Paper II (2013)

see all colours except orange, and he is last Either x = 42 or x = –3


in line, he must be wearing the orange cap. As a negative value for x is not possible here,
Only (d) from the options can be correct x = 42
for F.] The original speed of the train is 42 km per
31. (b) As the red balls and yellow balls are equal in hour.
number, if the yellow balls are twice the 37. (c) Statement 1 is implied in the first paragraph.
number of green ones, the red balls will also As for statement 2, the passage says excess
be twice (or double) the number of green balls. nutrients damage water ecosystems. But there
32. (b) is no information about water courses not
containing microorganisms that can decompose
33. (c)
organic components. It could be that water
34. (c) The new rank of the boy is 21st, and the new bodies do not have ‘enough’ of such
total of students is 47. There are 47 – 21 = 26 microorganisms. So we cannot say anything
boys after the boy concerned. His position or categorically about statement 2. Statement 3
rank from the end is therefore 26 + 1 = 27th. is supported by the second paragraph.
distance 38. (b)
35. (b) Time =
relative speed 39. (b) Statement 2 is not supported by the passage.
10 − 8 Statements 3 and 4 are indirectly the results
Here relative speed = × 1000
60 of indiscriminate use of fertilisers.
(km converted to m, and hour converted to
40. (b) Cultural eutrophication involves loss of large
minutes) plants and loss of fish.
@UPSC_THOUGHTS

100 41. (c)


Time =
10 − 8
× 1000 42. (b)
60
43. (d) The first and last sentences of the passages
100 × 60
= clearly negate the given statements.
2 × 1000
44. (b)
= 3 minutes
45. (c) The triangle moves 120° in a clockwise direction
36. (c) Let the original speed of the train be x km/h. at every step. The arrow gets inverted every
Then as per the conditions given, time in the same place. So the next position
Distance1 Distance2 will be (c).
Total time = +
x x +6 46. (b) The last problem figure is a vertical inversion
Distance of the first problem figure. The next figure will
Time = be a vertical inversion of the second problem
Speed
figure—which is (b).
63 72
3 = + 47. (d) Under the given conditions, the product of x
x x +6
and y (the work) has to be constant; this is
63(x + 6) + 72x
3 = only seen in diagram IV.
x (x + 6)
48. (c) The sum of the first digit and last digit in each
63 (x + 6) + 72x
x (x + 6) = row of the matrix is the same as the sum of
3 the digits of the number in the middle. Thus
x (x + 6) = 21 (x + 6)3 + 24x 3 + 7 = 10; 3 + 7 + 0 = 10
x 2 + 6x = 21x + 126 + 24x 2 + 6 = 8; 2 + 2 + 4 = 8
x 2 + 6x – 45x – 126 = 0 Now 1 + X must be the same as 7 + 3 + 0
x 2 – 39x – 126 = 0 or 10. X can only be 9.
(x – 42) (x + 3) = 0 49. (a) For each car work out the cost for each km
(42 × 3 = 126; we can factorise –39x into (which is the sum of the fixed rate per km and
–42x and 3x) the cost of diesel ÷ mileage). Then find the

ST . 66
Sample Test 3 : General Studies Paper II (2013)

total distance by dividing total payment by unit Clearly the average speed of Car A was
cost for km. Dividing the distance by the total maximum.
hours gives the average speed. [You need not calculate the decimal places for
40 average speed, as you can estimate with the
For Car A: Unit cost/km = 6 +
8 whole numbers itself as to which is the
= Rs 11 maximum.]

2120 50. (b) The pattern is:


Total distance =
11 14 14
Figure 1: = 7; 7 × 12 = 84; × 12 = 84
= 192.72 km 2 2
192.72 18
Average speed = Figure 2: × 9 = 81
20 2
= 9.6 km/hr ?
∴ In Figure 3: × 11 = 88
40 2
For Car B: Unit cost/km = 6 +
10 88 × 2
?= = 16
= Rs 10 11
1950 51. (a) From the three views of the cube shown, we
Total distance = can see that 1 has 4, 6, 2 and 3 on its
10
neighbouring faces. So 5 must be on the face
= 195 km
opposite to 1. Neither A nor B can therefore
195 be 1. Responses (b), (c) and (d) have 1 on
@UPSC_THOUGHTS

Average speed =
25 them and may therefore be rejected. Response
= 7.8 km/h (a) is correct.
40 52. (b) Total number of Physics professors
For Car C: Unit cost/km = 6 +
9
= 32 + 8 = 40
94 Physics professors in the age group 35–44
= Rs
9
= 40% of 40
94
Total distance = 2064 ÷ 40
9 = × 40 = 16
100
9
= 2064 × 53. (a) The ratio for physics is 32 : 8 = 4 : 1; it is
94 28 : 8, 16 : 22 and 24 : 8 for Mathematics,
= 197.6 km Chemistry and Economics, respectively. So the
197.6 ratio is highest in Physics. It is obvious from
Average speed =
24 the graph that in Mathematics, Chemistry and
= 8.2 km/h Economics, the ratio of males to females is
lower than it is for Physics. (In Mathematics
40
For Car D: Unit cost/km = 6 + and Economics, the number of males is lower
11 than in Physics though number of females is
106 the same in the two cases. In Chemistry, the
= Rs
11 number of males is lower than the number of
females.)
11
Total distance = 1812 ×
106 54. (c) There are 6 female and 4 male professors in
= 188 km Psychology. The total is 10.
188 The percentage of females of total Psychology
Average speed =
22 6
professors = × 100 = 60%
= 8.5 km/h. 10

ST . 67
Sample Test 3 : General Studies Paper II (2013)

55. (a) Total Physics professors = 40 2x + 20 = 3x


30% of 40 are in the age group 25–34 x = 20
30% of 40 = 12
Female Physics professors in age group 25–34 Total number of coins in the collection
= 25% of 12 = 3 = (x + 10) + 3x
Male Physics professors in the age group 25–34 = (20 + 10) + (3 × 20)
= 12 – 3 = 9
= 30 + 60 = 90.]
[If you had been in a hurry and marked (c)
61. (b) If the number of rows and the number of
you would have been wrong as that gives the
columns are to be the same, the total number
number of female, not male Physics professors
of plants must be a square of a natural
in the required age group.]
number. Here the number must be greater
56. (b) Let x be the total number of professors in the than 1000.
university.
Now 302 = 900;
Total Psychology professors are 10.
312 = 961;
As per given data,
322 = 1024.
2% of x = 10
100 So the minimum number of plants required is
x = 10 × 1024. The gardener therefore needs 1024 –
2
1000 1000 = 24 more plants
= = 500
2 [Alternatively, you can try out each response.
The total number of professors is 500.
(a) 1000 + 14 = 1014 whose square root is not
57. (d) The next figure will have a circle in the middle,
@UPSC_THOUGHTS

a whole number.
so (c) may be rejected. The arrows will be
pointed in the same direction, so (a) may be (b) 1000 + 24 = 1024 whose square root is 32;
rejected. However, the arrows are to be in the so (b) is the correct answer.]
clockwise direction as opposed to figure 5. So 62. (b) This is an arithmetic progression where the
(d) is the obvious choice. common difference is given, so we apply the
58. (d) formula

59. (a) n
Sn = [2a + (n – 1) d ]
60. (a) If the ratio is 1 : 2, the total would be divisible 2
by 3. Responses (b) and (d) may be rejected. where Sn is the sum of the terms = 700;
Taking (a), 1 : 2 ratio of 90 gives 30 : 60 of
n is number of terms = 7;
gold coins to non-gold coins.
d is the difference = 20; and
Now see if this works with the conditions of
the initial ratio. The gold coins then were 10 a is the first term (to be found out)
less or (30 – 10 =) 20. The ratio will be 20 : 60 So we have
which is 1 : 3. Thus response (a) must be
7
correct. 700 = [2a + (7 – 1) 20]
2
Checking out for (c), we get 1 : 2 as 20 : 40.
7
If we reduce gold coins by 10, we have 10 : 40 = [2a + 120)
2
which does not give the initial ratio of 1 : 3
700
[Conventional method: Let the number of gold = 2a + 120
3.5
coins be x. The non-gold coins, as per given 200 – 120 = 2a
ratio, will be 3x.
80
After 10 gold coins are added, we have a = = 40
2
x + 10 1
The least value of the prize is Rs 40.
=
3x 2

ST . 68
Sample Test 3 : General Studies Paper II (2013)

[Alternative method: Let the least value be Rs x. Now he uses 25% more or 1.25 l every day,
The prizes are: x, x + 20, x + 40 . . . x + 120 10
so the petrol lasts = 8 days.
1.25
As per given conditions,
x + (x + 20) + (x + 40) + (x + 60) + (x + 80) 10
66. (a) Time taken to walk one way = = 5 hours
+ (x + 100) + (x + 120) 2

= 700 If the combined time for walking and driving


both ways is 6 hours, the driving time for one
7x + 420 = 700
way is 6 – 5 = 1 hour
7x = 700 – 420
The person will therefore take 2 hours to drive
= 280 both ways.
280
x = 67. (b)
7
= 40.] 68. (c) Obviously, the watch was kept as a surety for
the return of the bicycle in good condition.
63. (c) Total applicants = 120
69. (c)
Males are 70
70. (d) Though (b) also says the town was a short
∴ Females are 120 – 70 = 50
distance the passage does not mention a hill
Maximum number of males with driving licence resort; in fact the town is shown on the map
can be 70. on the “other side of the hill”. There is no
Minimum number of males with driving licence suggestion of ‘wilderness’.
@UPSC_THOUGHTS

can be 80 – 50 (i.e., if all females have driving 71. (b) The other responses are true about the situation
licence) = 30 in which the travellers were but these were not
The ratio of minimum to maximum number of cause of their worry.
males with driving licence is 30 : 70 or 3 : 7.
72. (a)
[Conventional method: Using the formula,
73. (b)
n (A ∪ B) = n (A) + n (B) – n (A ∩ B), we have
74. (b)
120 = 70 + 80 – n (A ∩ B)
75. (a) First, as head of your office, you have been
n (A ∩ B) = 70 + 80 – 120 given discretion to allot certain houses to the
= 30 (minimum) office staff. If that were just the case, response
(b) would be fine, even though it could be
The maximum has to be 70
called misuse of discretionary powers. However,
∴ The ratio is 30 : 70 or 3 : 7.]
you have framed certain rules for such allotment
64. (d) After 10 days, there are 20 days left in the and these rules have been made public. In the
month. The food would have lasted for 20 days circumstances, to allot the house just to win
for 1000 soldiers. If the soldiers are doubled loyalty would be unfair and seen as favouritism.
to 2000, the days for which the food will last Would you be able to do the same for others
get halved. So the remaining food would last in the same or similar circumstances? As a
for 20 ÷ 2 = 10 days. responsible officer running a department, you
[Conventional method: Let the number of days have to act and be seen to act without bias.
be x. The office secretariat has examined the request
Then 1000 × (30 – 10) = (1000 + 1000) × x “as per the rules” and turned it down. By opting
for response (a) you are in effect agreeing with
1000 × 20
= x = 10.] the office note and showing no favouritism
2000
(option c) but just being more humane, showing
65. (d) Suppose Arun uses 1 litre every day, then the consideration for a person who works with you,
petrol lasts for 10 days. is close to you. Option (c) is too brusque in

ST . 69
Sample Test 3 : General Studies Paper II (2013)

the circumstances. Response (d) is a delaying schedule. But if it is just to fob him off for the
tactic considered typical of bureaucrats for time being; it is a cynical attitude.
which they are criticised by the common people. [Our ranking of the responses in order of
[Our ranking of the responses in order of acceptability: (c) (d) (b) (a)]
acceptability: (a) (c) (b) (d)] 78. (c) You are a public official with responsibilities
76. (c) Response (a) is impulsive, though many of us towards an entire district and you cannot let
would want to do it. It may work if you have personal friendships influence your decisions
the support of the taxi drivers and some others and make you flout rules. Option (a) is not
around. But most likely it will achieve nothing acceptable. Option (b) is what you have to do,
useful; the clerk would deny taking extra but there are ways of doing things without
money; and the taxi driver, who would have being harsh and inconsiderate of another’s
to travel the same way and have to face the feelings. Option (c) is best in the circumstances:
clerk again, may not support you. You would you are not breaking rules to favour a friend,
end up creating a scene and wasting time that but you are humane and making an effort to
you cannot afford. Options (b) and (d) are what take your friend along with your decision—
most people would do. Option (b) is correct in which is better in the long run. Option (d) is
so far as it is upto the taxi driver to take up also possible; it relieves you of an unpleasant
the matter with the tax authorities as he has situation and may actually work in your friend’s
been wronged. But it is doubtful if the taxi driver favour. However, in the face of the shortage
will do anything. However, as a member of the in your district, it is more necessary to get your
friend to understand the situation and cooperate.
@UPSC_THOUGHTS

public you have witnessed a corrupt act—bribe


taking and giving, that too for a legal transaction. [Our ranking of the responses in order of
If you are conscious of your duty as a citizen acceptability: (c) (b)/(d) (a)]
and want reduction in corruption, you must 79. (a) Option (b) is a long-term aspect of a policy on
report the matter to the appropriate authorities— family planning and is not a way available to
option (c). Option (d) shows a cynical acceptance you for convincing people who appear to be
of corruption that can harm our society. opposed to a policy on keeping a small family.
[Our ranking of the responses in order of In any case you can encourage people on late
acceptability: (c) (a) (b) (d)] marriages only after convincing them of its
benefit. Option (c) would be useful if people
77. (c) Considering that the villager has already spent
are merely indifferent to or lacking the will to
so much time waiting to see you, it would be
adopt family planning; here people need to be
inconsiderate, indeed callous, of you to ask him
convinced about its necessity itself, so it is
to wait another two hours. Option (a) shows
doubtful if such incentives would work. Moreover,
you to be arrogant and unconcerned about the
you want to convince the people about the
people you are meant to serve. Option (b) may
need to have small families, not merely appeal
be acceptable if it were true but, considering
to their greed for incentives. Option (d) may
the case, it seems to be an effort to ‘pass the
be a part of your effort to make people realise
buck’ or get rid of your responsibility. Option
the importance of family planning, but cannot
(c) is best: you are getting things to move even
be sufficient. Response (a) is the best of the
as you are unable to take up the matter
given options: to convince people, you have to
yourself, having to attend another meeting.
explain the rationale behind a policy and make
Option (d) is rather meaningless, unless you
them understand how it can and will benefit
intend to act on the information—but that is not
them.
clear in the option. However, it indicates your
interest in the villager’s problem and readiness [Our ranking of options in their order of
to give some time to him amidst your busy acceptability: (a) (d) (c) (b)]

ST . 70
Sample Test 3 : General Studies Paper II (2013)

80. (c) Favours need to be returned, but you should a practice with your colleague, you should look
not, and cannot be expected to do something upon him with consideration. He may be acting
illegal and immoral to return a favour. Option in desperation, in which state he cannot think
(a) is not to be considered. Option (b) merely properly. He may see reason if you explain to
states your regret on your inability to ‘help’ him the immorality of what he is asking you
without reference to the nature of help sought. to do, and he may regret his own action. So
Would you like to help in such a manner? (c) appears the best option. Of course, if he
Option (d) is not incorrect: you are taking a insists on his demand, (d) has to be adopted.
stand of moral high ground, and seeing to it
[Our ranking of options in their order of
that illegal/unprincipled actions are not
condoned. However, unless this happens to be acceptability: (c) (d) (b) (a)]

@UPSC_THOUGHTS

ST . 71
Sample Test 4 : General Studies Paper II (2014)

SAMPLE TEST 4: GENERAL STUDIES PAPER II (2014)


Directions for the following 5 (five) items: Hence the criterion being suggested here is a statistical
summing up of the idea of inclusive growth, which,
Read the following two passages and answer the items in turn, leads to two corollaries: to wish that India
that follow each passage. Your answers to these items must strive to achieve high growth and that we must
should be based on the passages only. work to ensure that the weakest segments benefit
from the growth.
Passage – 1
1. The author’s central focus is on
In recent times, India has grown fast not only (a) applauding India’s economic growth not
compared to its own past but also in comparison with only against its own past performance,
other nations. But there cannot be any room for but against other nations.
complacency because it is possible for the Indian (b) emphasising the need for economic growth
economy to develop even faster and also to spread which is the sole determinant of a
the benefits of this growth more widely than has been country’s prosperity.
done thus far. Before going into details of the kinds (c) emphasising inclusive growth where gains
of micro-structural changes that we need to of growth are shared widely by the
conceptualise and then proceed to implement, it is population.
worthwhile elaborating on the idea of inclusive (d) emphasising high growth.
growth that constitutes the defining concept behind
this Government’s various economic policies and 2. The author supports policies which will help
decisions. A nation interested in inclusive growth (a) develop economic growth.
views the same growth differently depending on (b) better distribution of incomes irrespective
whether the gains of the growth are heaped primarily of rate of growth.
@UPSC_THOUGHTS

on a small segment or shared widely by the population. (c) develop economic growth and redistribute
The latter is cause for celebration but not the former. economic gains to those getting left behind.
In other words, growth must not be treated as an end (d) put an emphasis on the development of
in itself but as an instrument, for spreading prosperity the poorest segments of society.
to all. India’s own past experience and the experience 3. Consider the following statements:
of other nations suggest that growth is necessary for
According to the author, India’s economy has
eradicating poverty but it is not a sufficient condition.
grown but there is no room for complacency
In other words, policies for promoting growth need
as
to be complemented with policies to ensure that more
1. growth eradicates poverty.
and more people join in the growth process and,
2. growth has resulted in prosperity for all.
further, that there are mechanisms in place to
redistribute some of the gains to those who are unable Which of the statements given above is/are
to partake in the market process and, hence, get left correct?
behind. (a) 1 only (b) 2 only
A simple way of giving this idea of inclusive (c) Both 1 and 2 (d) Neither 1 nor 2
growth a sharper form is to measure a nation’s
progress in terms of the progress of its poorest Passage – 2
segment, for instance, the bottom 20 per cent of the It is easy for the government to control State-
population. One could measure the per capita income owned companies through nods and winks. So what
of the bottom quintile of the population and also really needs to be done as a first step is to put petrol
calculate the growth rate of income; and evaluate our pricing on a transparent formula—if the price of
economic success in terms of these measures that crude is x and the exchange rate y, then every month
pertain to the poorest segment. This approach is or fortnight, the government announces a maximum
attractive because it does not ignore growth like some price of petrol, which anybody can work out from
of the older heterodox criteria did. It simply looks at the x and the y. The rule has to be worked out to
the growth of income of the poorest sections of the make sure that the oil-marketing companies can, in
population. It also ensures that those who are outside general, cover their costs. This will mean that if one
of the bottom quintile do not get ignored. If that were company can innovate and cut costs, it will make
done, then those people would in all likelihood drop greater profits. Hence, firms will be more prone to
down into the bottom quintile and so would innovate and be efficient under this system. Once the
automatically become a direct target of our policies. rule is announced, there should be no interference by

ST . 72
Sample Test 4 : General Studies Paper II (2014)

the government. If this is done for a while, private 9. Consider that:


companies will re-enter this market. And once a 1. A is taller than B.
sufficient number of them are in the fray, we can 2. C is taller than A.
remove the rule-based pricing and leave it truly to 3. D is taller than C.
the market (subject to, of course, the usual regulations 4. E is the tallest of all.
of anti-trust and other competition laws). If they are made to sit in the order of their
4. Consider the following statements: height, who will occupy the mid position?
(a) A (b) B
According to the passage, an oil company can
(c) C (d) D
make greater profits if a transparent formula
for petrol pricing is announced every fortnight 10. Consider the following statements:
or month, by There are six villages: A, B, C, D, E and F.
1. promoting its sales. F is 1 km to the west of D.
2. undertaking innovation. B is 1 km to the east of E.
3. cutting costs. A is 2 km to the north of E.
4. selling its equity shares at higher prices. C is 1 km to the east of A.
D is 1 km to the south of A.
Which of the statements given above is/are
correct? Which three villages are in a line?
(a) 1 only (b) 2 and 3 (a) A, C, B (b) A, D, E
(c) 3 and 4 (d) 1, 2 and 4 (c) C, B, F (d) E, B, D
5. Consider the following statements: 11. Four children are sitting in a row. A is occupying
the seat next to B but not next to C. If C is not
According to the passage, private oil companies sitting next to D, who is/are occupying seat/
re-enter the oil producing market if seats adjacent to D?
@UPSC_THOUGHTS

1. a transparent rule-based petrol pricing (a) B (b) A


exists. (c) B and A (d) Impossible to tell
2. there is no government interference in
the oil-producing market. 12. Assume that
3. subsidies are given by the government. 1. the hour and minute hands of a clock
4. regulations of anti-trust are removed. move without jerking.
2. the clock shows a time between 8 o’clock
Which of the statements given above are correct? and 9 o’clock.
(a) 1 and 2 (b) 2 and 3 3. the two hands of the clock are one above
(c) 3 and 4 (d) 2 and 4 the other.
6. Five persons fire bullets at a target at an After how many minutes (nearest integer) will
interval of 6, 7, 8, 9 and 12 seconds respectively. the two hands be again lying one above the
The number of times they would fire the bullets other?
together at the target in an hour is (a) 60 (b) 62
(a) 6 (b) 7 (c) 65 (d) 67
(c) 8 (d) 9
Directions for the following 6 (six) items:
7. A group of 630 children is seated in rows for
a group photo session. Each row contains three Read the following two passages and answer the items
less children than the row in front of it. Which that follow each passage. Your answers to these items
one of the following number of rows is not should be based on the passages only.
possible?
(a) 3 (b) 4 Passage – 1
(c) 5 (d) 6 Climate change poses potentially devastating
8. There are seven persons up on a ladder, A, B, effects on India’s agriculture. While the overall
C, D, E, F and G (not in that order). A is farther parameters of climate change are increasingly
up than E but is lower than C. B is in the accepted—a 1 °C average temperature increase over
middle. G is between A and B. E is between B the next 30 years, sea level rise of less than 10 cm
and F. If F is between E and D, the person on in the same period, and regional monsoon variations
the bottom step of the ladder will be and corresponding droughts—the impacts in India
are likely to be quite site and crop specific. Some
(a) B (b) F
crops may respond favourably to the changing
(c) D (d) E

ST . 73
Sample Test 4 : General Studies Paper II (2014)

conditions, others may not. This emphasises the need harnessing renewable energy on a large scale as a
to promote agricultural research and create maximum national imperative. This country is extremely well
flexibility in the system to permit adaptations. endowed with solar, wind and biomass sources of
The key ingredient for “drought proofing” is the energy. Where we have lagged, unfortunately, is in
managed recharge of aquifers. To ensure continued our ability to develop and to create technological
yields of important staple crops (e.g. wheat), it may solutions for harnessing these resources.
also be necessary to shift the locations where these One particular trajectory for carrying out stringent
crops are grown, in response to temperature changes mitigation of greenhouse gas emissions assessed by
as well as to water availability. The latter will be a the Intergovernmental Panel on Climate Change
key factor in making long-term investment decisions. (IPCC) clearly shows the need for ensuring that
For example, water runoff from the Himalayas global emissions of greenhouse gases peak no later
is predicted to increase over the next 30 years as than 2015 and reduce rapidly thereafter. The cost
glaciers melt, but then decline substantially thereafter. associated with such a trajectory is truly modest and
It will be critical to provide incentives to plan for would amount, in the estimation of IPCC, to not more
these large-scale shifts in agro-ecological conditions. than 3 per cent of the global GDP in 2030. In other
India needs to make long term investment in words, the level of prosperity that the world would
research and development in agriculture. India is have reached without mitigation would at worst be
likely to experience changed weather patterns in postponed by a few months or a year at the most.
future. This is clearly not a very high price to pay for
protecting hundreds of millions of people from the
13. Consider the following statements: worst risks associated with climate change. Any such
Climate change may force the shifting of effort, however, would require lifestyles to change
locations of the existing crops due to appropriately also. Mitigation of greenhouse gas
1. melting of glaciers. emissions is not a mere technological fix, and clearly
requires changes in lifestyles and transformation of
@UPSC_THOUGHTS

2. water availability and temperature


suitability at other locations. a country’s economic structure, whereby effective
3. poor productivity of crops. reduction in emissions is brought about, such as
4. wider adaptability of crop plants. through the consumption of much lower quantities
of animal protein. The Food and Agriculture
Which of the statements given above are correct? Organisation (FAO) has determined that the emissions
(a) 1, 2 and 3 (b) 2 and 3 only from the livestock sector amount to 18 per cent of
(c) 1 and 4 only (d) 1, 2, 3 and 4 the total. The reduction of emissions from this source
14. According to the passage, why is it important is entirely in the hands of human beings, who have
to promote agricultural research in India? never questioned the impacts that their dietary habits
(a) To predict variations in monsoon patterns of consuming more and more animal protein are
and to manage water resources bringing about. Mitigation overall has huge co-benefits,
(b) To make long-term investment decisions such as lower air pollution and health benefits, higher
for economic growth energy security and greater employment.
(c) To facilitate wider adaptability of crops 15. According to the passage, which of the following
(d) To predict drought conditions and to would help in the mitigation of greenhouse
recharge aquifers gases?
1. Reducing the consumption of meat
Passage – 2 2. Rapid economic liberalisation
It is essential that we mitigate the emissions of 3. Reducing consumerism
greenhouse gases and thus avoid some of the worst 4. Modern management practices of
impacts of climate change that would take place in livestock
coming years and decades. Mitigation would require
Select the correct answer using the code given
a major shift in the way we produce and consume
below:
energy. A shift away from overwhelming dependence
(a) 1, 2 and 3 (b) 2, 3 and 4
on fossil fuels is now long overdue, but unfortunately,
(c) 1 and 3 only (d) 2 and 4 only
technological development has been slow and
inadequate largely because government policies have 16. Why do we continue to depend on the fossil
not promoted investments in research and fuels heavily?
development, myopically as a result of relatively low 1. Inadequate technological development
prices of oil. It is now, therefore, imperative for a 2. Inadequate funds for research and
country like India to treat the opportunity of development

ST . 74
Sample Test 4 : General Studies Paper II (2014)

3. Inadequate availability of alternative (a) B and C (b) E and F


sources of energy (c) C and E (d) C and F
Select the correct answer using the code given 22. A straight line segment is 36 cm long. Points
below: are to be marked on the line from both the end
(a) 1 only (b) 2 and 3 only points. From each end, the first point is at a
(c) 1 and 3 only (d) 1, 2 and 3 distance of 1 cm from the end, the second point
is at a distance of 2 cm from the first point and
17. According to the passage, how does the
the third point is at a distance of 3 cm from
mitigation of greenhouse gases help us?
the second point and so on. If the points on the
1. Reduces expenditure on public health
ends are not counted and the common points
2. Reduces dependence on livestock
are counted as one, what is the number of
3. Reduces energy requirements
points?
4. Reduces rate of global climate change
(a) 10 (b) 12
Select the correct answer using the code given (c) 14 (d) 16
below:
23. If Sohan, while selling two goats at the same
(a) 1, 2 and 3 (b) 1, 3 and 4
price, makes a profit of 10% on one goat and
(c) 2, 3 and 4 (d) 1 and 4 only
suffers a loss of 10% on the other,
18. What is the essential message of the passage? (a) he makes no profit and no loss.
(a) We continue to depend on fossil fuels (b) he makes a profit of 1%.
heavily (c) he suffers a loss of 1%.
(b) Mitigation of the greenhouse gases is (d) he suffers a loss of 2%.
imperative
(c) We must invest in research and 24. Out of a total of 120 musicians in a club, 5%
development can play all the three instruments, guitar, violin
@UPSC_THOUGHTS

and flute. It so happens that the number of


(d) People must change their lifestyle
musicians who can play any two and only two
19. There are 60 students admitted to a nursery of the above instruments is 30. The number of
class. Some students can speak only English musicians who can play the guitar alone is 40.
and some can speak only Hindi. 10 students can What is the total number of those who can play
speak both English and Hindi. If the number violin alone or flute alone?
of students who can speak English is 21, then (a) 45 (b) 44
how many students can speak Hindi, how (c) 38 (d) 30
many can speak only Hindi and how many can
speak only English? 25. Six identical cards are placed on a table. Each
(a) 21, 11 and 29 respectively card has number ‘1’ marked on one side and
number ‘2’ marked on its other side. All the six
(b) 28, 18 and 22 respectively
cards are placed in such a manner that the
(c) 37, 27 and 13 respectively
number ‘1’ is on the upper side. In one try,
(d) 39, 29 and 11 respectively
exactly four (neither more nor less) cards are
20. A gardener increased the area of his rectangular turned upside down. In how many least number
garden by increasing its length by 40 per cent of tries can the cards be turned upside down
and decreasing its width by 20 per cent. The such that all the six cards show number ‘2’ on
area of the new garden the upper side?
(a) has increased by 20 per cent. (a) 3 (b) 5
(b) has increased by 12 per cent. (c) 7 (d) This cannot be achieved
(c) has increased by 8 per cent.
(d) is exactly the same as the old area. Directions for the following 8 (eight) items:
21. Six books are labelled A, B, C, D, E and F and Read the following two passages and answer the items
are placed side by side. Books B, C, E and F have that follow each passage. Your answers to these items
green covers while others have yellow covers. should be based on the passages only.
Books A, B and D are new while the rest are
Passage – 1
old volumes. Books A, B and C are law reports
while the rest are medical extracts. Which two The Himalayan ecosystem is highly vulnerable to
books are old medical extracts and have green damage, both due to geological reasons and on
covers? account of the stress caused by increased pressure of

ST . 75
Sample Test 4 : General Studies Paper II (2014)

population, exploitation of natural resources and diversity, makes the Himalayan region a
other related challenges. These aspects may be biodiversity hotspot.
exacerbated due to the impact of climate change. It (d) The Himalayan biogeographic region
is possible that climate change may adversely impact should be enabled to adapt to climate
the Himalayan ecosystem through increased change smoothly.
temperature, altered precipitation patterns, episodes 28. What is the most important message conveyed
of drought and biotic influences. This would impact by the passage?
not only the very sustenance of the indigenous (a) Endemism is a characteristic feature of
communities in uplands but also the life of Himalayan region.
downstream dwellers across the country and beyond. (b) Conservation efforts should emphasise
Therefore, there is an urgent need for giving special on biogeographic ranges rather than on
attention to sustain the Himalayan ecosystem. This some species or habitats.
would require conscious efforts for conserving all the (c) Climate change has adverse impact on
representative systems. the Himalayan ecosystem.
Further, it needs to be emphasised that the (d) Without Himalayan ecosystem, the life of
endemics with restricted distribution, and most often the communities of uplands and down-
with specialised habitat requirements, are among the streams will have no sustenance.
most vulnerable elements. In this respect the
Himalayan biodiversity hotspot, with rich endemic 29. With reference to the passage, the following
diversity, is vulnerable to climate change. The threats assumptions have been made:
include possible loss of genetic resources and species, 1. To maintain natural ecosystems,
habitats and concomitantly a decrease in ecosystem exploitation of natural resources should
services. Therefore, conservation of endemic elements be completely avoided.
in representative ecosystems/habitats assumes a great 2. Not only anthropogenic but also natural
significance while drawing conservation plans for the reasons can adversely affect ecosystems.
@UPSC_THOUGHTS

region. 3. Loss of endemic diversity leads to the


Towards achieving the above, we will have to extinction of ecosystems.
shift toward contemporary conservation approaches, Which of the above assumptions is/are correct?
which include a paradigm of landscape level (a) 1 and 2 (b) 2 only
interconnectivity between protected area systems. (c) 2 and 3 (d) 3 only
The concept advocates a shift from the species-
habitat focus to an inclusive focus on expanding the
Passage – 2
biogeographic range so that natural adjustments to It is often forgotten that globalisation not only is
climate change can proceed without being restrictive. about policies on international economic relationships
and transactions, but also has equally to do with
26. Consider the following statements:
domestic policies of a nation. Policy changes
According to the passage, the adverse impact necessitated by meeting the internationally set
of climate change on an ecosystem can be a conditions (by WTO etc.) of free trade and investment
1. permanent disappearance of some of its flows obviously affect domestic producers and
flora and fauna. investors. But the basic philosophy underlying
2. permanent disappearance of ecosystem globalization emphasises absolute freedom to markets
itself. to determine prices and production and distribution
patterns, and views government interventions as
Which of the statements given above is/are
processes that create distortions and bring in
correct?
inefficiency. Thus, public enterprises have to be
(a) 1 only (b) 2 only
privatised through disinvestments and sales; sectors
(c) Both 1 and 2 (d) Neither 1 nor 2
and activities hitherto reserved for the public sector
27. Which one of the following statements best have to be opened to the private sector. This logic
implies the need to shift toward contemporary extends to the social services like education and
conservation approach? health. Any restrictions on the adjustments in
(a) Exploitation of natural resources causes workforce by way of retrenchment of workers should
a stress on the Himalayan ecosystem. also be removed and exit should be made easier by
(b) Climate change alters precipitation removing any restrictions on closures. Employment
patterns, causes episodes of drought and and wages should be governed by free play of market
biotic interference. forces, as any measure to regulate them can discourage
(c) The rich biodiversity, including endemic investment and also create inefficiency in production.

ST . 76
Sample Test 4 : General Studies Paper II (2014)

Above all, in line with the overall philosophy of Y


reduction in the role of the State, fiscal reforms
should be undertaken to have generally low levels of 6

Average profit in thousands (`)


taxation and government expenditure should be kept
5
to the minimum to abide by the principle of fiscal
B
prudence. All these are policy actions on the domestic 4
front and are not directly related to the core items
of the globalisation agenda, namely free international 3 A
flow of goods and finance.
30. According to the passage, under globalisation, 2
government interventions are viewed as 1
processes leading to
(a) distortions and inefficiency in the 0 X
economy. 1995 1996 1997 1998 1999 2000
(b) optimum use of resources. Year
(c) more profitability to industries.
(d) free play of market forces with regard to 34. In which year is the average profit of A and
industries. B same?
(a) 1995 (b) 1996
31. According to the passage, the basic philosophy (c) 1997 (d) 1998
of globalisation is to
(a) give absolute freedom to producers to 35. What is the difference between the average
determine prices and production. profit of B and A in the year 1998?
(b) give freedom to producers to evolve (a) – ` 100 (b) – ` 1,000
(c) + ` 600 (d) – ` 300
@UPSC_THOUGHTS

distribution patterns.
(c) give absolute freedom to markets to 36. How much more average profit did A make in
determine prices, production and the year 2000 than in the year 1999?
employment. (a) ` 200 (b) ` 1,000
(d) give freedom to producers to import and (c) ` 1,500 (d) ` 2,000
export.
37. What is the trend of the average profit of B
32. According to the passage, which of the following from the year 1997 to the year 2000?
is/are necessary for ensuring globalisation? (a) Non-increasing (b) Non-decreasing
1. Privatisation of public enterprises (c) Steady (d) Fluctuating
2. Expansionary policy of public expenditure
38. The following table shows the marks obtained
3. Free play of market forces to determine
by two students in different subjects:
wages and employment
4. Privatisation of social services like Student Maximum Student Maximum
education and health A Marks B Marks

Select the correct answer using the code given English 60 100 80 150
below: Psychology 70 100 70 100
(a) 1 only (b) 2 and 3 only History 50 100 60 100
(c) 1, 3 and 4 (d) 2, 3 and 4 Sanskrit 30 50 15 25
33. According to the passage, in the process of The difference in the mean aggregate percentage
globalisation, the State should have marks of the students is:
(a) expanding role. (a) 2.5% (b) 13.75%
(b) reducing role. (c) 1.25% (d) Zero
(c) statutory role. 39. Examine the following figure:
(d) none of the above roles.
Directions for the following 4 (four) items:
The following graph shows the average profit of two
fruit-sellers A and B in thousands (`) per year from the
year 1995 to 2000. Consider the graph and answer the Which one of the following figures has the
4 (four) items that follow: above figure embedded in it?

ST . 77
Sample Test 4 : General Studies Paper II (2014)

43. Consider the following matrix with one empty


block in the lower extreme corner:

(a) (b) (c) (d)


40. Consider the following matrix:

Which of the following figures could fit in the


empty block and thus complete the matrix?

Which one of the following figures fits into the


blank part of the above matrix?
(a) (b) (c) (d)
44. With reference to the figure given below, the
number of different routes from S to T without
retracing from U and/or V, is
@UPSC_THOUGHTS

(a) (b) (c) (d)


41. The following table gives population and total S T
income of a city for four years:
U V
Year 1992 1993 1994 1995
Population 20 21 22 23 (a) 3 (b) 6
in lakhs (c) 9 (d) 18
Income 1010 1111 1225 1345 45. Consider the following figures:
in crores (`)

Which one of the following statements is correct ?


in respect of the above data?
(a) Population increased by 5% or more every Change in positions of beads in the four figures
year. above follows a sequence. Following the same
(b) Income increased by 10% or more every sequence, which of the figures below should
year. appear as the fifth figure above?
(c) Per capita income was always above
` 5,000.
(d) Per capita income was highest in 1994.
42. Consider the table given below in which the (a) (b) (c) (d)
numbers bear certain relationship among 46. A bell rings every 18 minutes. A second bell
themselves along the rows: rings every 24 minutes. A third bell rings every
29 13 18 32 minutes. If all the three bells ring at the same
33 X 19 time at 8 o’clock in the morning, at what other
time will they all ring together?
30 27 3 (a) 12 : 40 hrs (b) 12 : 48 hrs
Which one of the following numbers is the (c) 12 : 56 hrs (d) 13 : 04 hrs
missing number indicated above by X? 47. “Price is not the same thing as value. Suppose
(a) 19 (b) 15 that on a day the price of everything viz., coal,
(c) 14 (d) 8 bread, postage stamps, a day’s labour, the rent

ST . 78
Sample Test 4 : General Studies Paper II (2014)

of houses, etc. were to double. Prices then Directions for the following 7 (seven) items:
would certainly rise, but values of all things
except one would not.” Read the following two passages and answer the items
that follow each passage. Your answers to these items
The writer wants to say that if prices of all should be based on the passages only.
things were doubled
(a) the values of all things would remain Passage – 1
constant. Many nations now place their faith in capitalism
(b) the values of the things sold would be and governments choose it as the strategy to create
doubled. wealth for their people. The spectacular economic
(c) the values of the things bought would be growth seen in Brazil, China and India after the
halved. liberalisation of their economies is proof of its
(d) the value of money only would be halved. enormous potential and success. However, the global
48. A and B decide to travel from place X to place banking crisis and the economic recession have left
Y by bus. A has ` 10 with him and he finds many bewildered. The debates tend to focus on free
that it is 80% of the bus fare for two persons. market operations and forces, their efficiency and
B finds that he has ` 3 with him and hands their ability for self correction. Issues of justice,
it over to A. In this context, which one of the integrity and honesty are rarely elaborated to highlight
following statements is correct? the failure of the global banking system. The apologists
(a) Now the money A has is just enough to of the system continue to justify the success of
buy two tickets. capitalism and argue that the recent crisis was a blip.
(b) A still needs ` 2 for buying the tickets. Their arguments betray an ideological bias with
(c) After buying the two tickets A will be left the assumptions that an unregulated market is fair
with 50 paise. and competent, and that the exercise of private greed
(d) The money A now has is still not sufficient will be in the larger public interest.
@UPSC_THOUGHTS

to buy two tickets. Few recognise the bidirectional relationship


49. As per agreement with a bank, a businessman between capitalism and greed; that each reinforces
had to refund a loan in some equal instalments the other. Surely, a more honest conceptualisation of
without interest. After paying 18 instalments the conflicts of interest among the rich and powerful
he found that 60 per cent of his loan was players who have benefited from the system, their
refunded. How many instalments were there in biases and ideology is needed; the focus on the wealth
the agreement? creation should also highlight the resultant gross
(a) 22 (b) 24 inequity.
(c) 30 (d) 33 52. The apologists of the “Free Market System”,
50. A worker reaches his factory 3 minutes late if according to the passage, believe in
his speed from his house to the factory is 5 km/ (a) market without control by government
hr. If he walks at a speed of 6 km/hr, then he authorities.
reaches the factory 7 minutes early. The distance (b) market without protection by the
of the factory from his house is government.
(a) 3 km (b) 4 km (c) ability of market to self correct.
(c) 5 km (d) 6 km (d) market for free goods and services.
51. “Liberty, therefore, is never real unless the 53. With reference to “ideological bias”, the passage
Government can be called to account when it implies that
invades rights.” (a) free market is fair but not competent.
Which one of the following is the best (b) free market is not fair but competent.
justification of the above statement? (c) free market is fair and competent.
(a) In the realisation that the government (d) free market is neither fair nor biased.
can be brought to book in a court of law 54. “The exercise of private greed will be in the
(b) In identifying a man as a political unit larger public interest” from the passage
in a way which distinguishes him from 1. refers to the false ideology of capitalism.
other citizens 2. underlies the righteous claims of the free
(c) In a decentralised society wherein the market.
basic needs of men can find satisfaction 3. shows the benevolent face of capitalism.
(d) In the understanding that liberty and 4. ignores resultant gross inequity.
restraints are complementary

ST . 79
Sample Test 4 : General Studies Paper II (2014)

Which of the statements given above is/are reason/reasons for saying that the time has
correct? come to review the role of public sector?
(a) 1 only (b) 2 and 3 1. Now public sector has lost its relevance
(c) 1 and 4 (d) 4 only in the industrialisation process.
2. Public sector does not perform
Passage – 2 satisfactorily.
Net profits are only 2.2% of their total assets for 3. Entrepreneurship in private sector is
central public sector undertakings, lower than for the expanding.
private corporate sector. While the public sector or 4. Effective competition policies are available
the State-led entrepreneurship played an important now.
role in triggering India’s industrialisation, our
Which of the statements given above is/are
evolving development needs, comparatively less-than-
correct in the given context?
satisfactory performance of the public sector
(a) 1 and 3 only (b) 2 only
enterprises, the maturing of our private sector, a
(c) 2, 3 and 4 only (d) 1, 2, 3 and 4
much larger social base now available for expanding
entrepreneurship and the growing institutional 56. According to the passage, rural roads should be
capabilities to enforce competition policies would in the domain of public sector only. Why?
suggest that the time has come to review the role of (a) Rural development work is the domain of
public sector. government only.
What should the portfolio composition of the (b) Private sector cannot have monetary gains
government be? It should not remain static all times. in this.
The airline industry works well as a purely private (c) Government takes money from tax payers
affair. At the opposite end, rural roads, whose sparse and hence it is the responsibility of
traffic makes tolling unviable, have to be on the government only.
@UPSC_THOUGHTS

balance-sheet of the State. If the government did not (d) Private sector need not have any social
own rural roads, they would not exist. Similarly, responsibility.
public health capital in our towns and cities will need 57. The portfolio composition of the government
to come from the public sector. Equally, preservation refers to
and improvement of forest cover will have to be a
(a) Public sector assets quality.
new priority for the public sector assets.
(b) Investment in liquid assets.
Take the example of steel. With near-zero tariffs,
(c) Mix of government investment in different
India is a globally competitive market for the metal.
industrial sectors.
Indian firms export steel into the global market,
(d) Buying Return on Investment yielding
which demonstrates there is no gap in technology.
capital assets.
Indian companies are buying up global steel
companies, which shows there is no gap in capital 58. The author prefers government as the umpire
availability. Under these conditions, private ownership and private sector as players because
works best. (a) Government prescribes norms for a fair
Private ownership is clearly desirable in regulated play by the private sector.
industries, ranging from finance to infrastructure, (b) Government is the ultimate in policy
where a government agency performs the function formulation.
of regulation and multiple competing firms are located (c) Government has no control over private
in the private sector. Here, the simple and clean sector players.
solution—government as the umpire and the private (d) None of the above statements is correct
sector as the players—is what works best. In many in this context.
of these industries, we have a legacy of government 59. A question paper must have a question on one
ownership, where productivity tends to be lower, fear of the eight poets: A, B, C, D, E, F, G or H. The
of bankruptcy is absent, and the risk of asking for first four belong to the medieval period while
money from the tax payer is ever present. There is the rest are considered modern poets. Generally,
also the conflict of interest between government as modern poets figure in the question paper in
an owner and as the regulator. The formulation and alternate years. Generally those who like H like
implementation of competition policy will be more G also; and those who like F like E also. The
vigorous and fair if government companies are out paper-setter does not like to ask about F as he
of action. has written a book on F, but he likes F. Last
55. According to the passage, what is/are the year, the paper contained a question on A. On

ST . 80
Sample Test 4 : General Studies Paper II (2014)

the basis of the information given, this year’s In the context of political development, the
paper is most likely to contain a question on assumption in the above passage is that
(a) C (b) E (a) political leadership is not an effective
(c) F (d) H instrument.
(b) military fills in political vacuum.
60. In a group of six women there are four dancers,
(c) military intervention is inevitable for
four vocal musicians, one actress and three
development.
violinists. Girija and Vanaja are among the
(d) None of the above
violinists while Jalaja and Shailaja do not know
how to play on the violin. Shailaja and Tanuja 65. Four persons, Alok, Bhupesh, Chander and
are among the dancers. Jalaja, Vanaja, Shailaja Dinesh have a total of ` 100 among themselves.
and Tanuja are all vocal musicians and two of Alok and Bhupesh between them have as much
them are also violinists. If Pooja is an actress, money as Chander and Dinesh between them,
who among the following is certainly a dancer but Alok has more money than Bhupesh; and
and a violinist? Chander has only half the money that Dinesh
(a) Jalaja (b) Pooja has. Alok has in fact ` 5 more than Dinesh has.
(c) Shailaja (d) Tanuja Who has the maximum amount of money?
(a) Alok (b) Bhupesh
61. The letters L, M, N, O, P, Q, R, S and T in their
(c) Chander (d) Dinesh
order are substituted by nine integers 1 to 9 but
not in that order. 4 is assigned to P. The 66. Examine the following statements:
difference between P and T is 5. The difference 1. George attends Music classes on Monday.
between N and T is 3. What is the integer 2. He attends Mathematics classes on
assigned to N? Wednesday.
(a) 7 (b) 5 3. His Literature classes are not on Friday.
4. He attends History classes on the day
@UPSC_THOUGHTS

(c) 4 (d) 6
following the day of his Mathematics
62. The number of deaths among the army personnel classes.
is 8 in 1000, but among the civilian population 5. On Tuesday, he attends his Sports classes.
it is 20 per 1000. Which one of the following
inferences can be drawn from this statement? If he attends just one subject in a day and his
(a) It is better to join the army. Sunday is free, then he is also free on
(b) The relationship is fortuitous. (a) Monday (b) Thursday
(c) Quality of Life Index is very high within (c) Saturday (d) Friday
the armed forces. 67. In a row ‘A’ is in the 11th position from the
(d) The groups cannot be compared due to left and ‘B’ is in the 10th position from the
their heterogeneity. right. If ‘A’ and ‘B’ interchange, then ‘A’
63. Given the statement: “Buses are the cause of becomes 18 from the left. How many persons
more accidents than cars, and trucks cause are there in the row other than ‘A’ and ‘B’?
fewer accidents than buses”, which of the (a) 27 (b) 26
following conclusions can we draw? (c) 25 (d) 24
(a) There are more buses on the road than 68. Location of B is north of A and location of C
trucks. is east of A. The distances AB and AC are 5
(b) Car drivers are more careful than bus km and 12 km respectively. The shortest
drivers. distance (in km) between the locations B and
(c) Truck drivers are more skilled than either C is
car or bus drivers. (a) 60 (b) 13
(d) None of the above (c) 17 (d) 7
64. “If political leadership fails to emerge, there is 69. Two cars start towards each other, from two
likelihood of military taking over power in places A and B which are at a distance of 160
developing countries. Radical student groups or km. They start at the same time 08 : 10 AM.
labour may try to raise revolution but they are If the speeds of the cars are 50 km and 30 km
not likely to compete with the military. Military per hour respectively, they will meet each other
intervention, rule, and withdrawal from politics at
is closely related to a society’s level of political (a) 10 : 10 AM (b) 10 : 30 AM
development.” (c) 11 : 10 AM (d) 11 : 20 AM

ST . 81
Sample Test 4 : General Studies Paper II (2014)

Directions for the following 6 (six) items: (c) she lacked self-confidence.
(d) she did not like school plays.
The following six items are based on two passages in
English to test the comprehension of English language 73. Cynthia’s classmates were chatting because
and therefore these items do not have Hindi version. Read (a) it was their turn to act next.
each passage and answer the items that follow. (b) they were bored of the performances.
(c) Cynthia did not act well.
Passage – 1 (d) the teacher had no control over them.
In front of us was walking a bare-headed old man 74. Cynthia’s knees were quaking because
in tattered clothes. He was driving his beasts. They (a) she felt nervous and shy.
were all laden with heavy loads of clay from the hills (b) the teacher scolded her.
and looked tired. The man carried a long whip which (c) she was very thin and weak.
perhaps he himself had made. As he walked down (d) she was afraid of her classmates.
the road he stopped now and then to eat the wild
berries that grew on bushes along the uneven road. 75. The transformation that occurred during the
When he threw away the seeds, the bold birds would audition refers to
fly to peck at them. Sometimes a stray dog watched (a) the nervousness of Cynthia.
the procession philosophically and then began to (b) the eruption of the entire room in
thunderous applause.
bark. When this happened, my two little sons would
(c) the surprise on the faces of her classmates.
stand still holding my hands firmly. A dog can
(d) the stunning performance of Cynthia.
sometimes be dangerous indeed.
76. If the 3rd day of a month is Monday, which
70. The author’s children held his hands firmly
one of the following will be the fifth day from
because
21st of this month?
(a) they were scared of the barking dogs.
(a) Monday (b) Tuesday
@UPSC_THOUGHTS

(b) they wanted him to pluck berries.


(c) Wednesday (d) Friday
(c) they saw the whip in the old man’s hand.
(d) the road was uneven. 77. For a charity show, the total tickets sold were
420. Half of these tickets were sold at the rate
71. The expression “a stray dog watched the of ` 5 each, one-third at the rate of ` 3 each
procession philosophically” means that and the rest for ` 2 each. What was the total
(a) the dog was restless and ferocious. amount received?
(b) the dog stood aloof, looking at the (a) ` 900 (b) ` 1,540
procession with seriousness. (c) ` 1,610 (d) ` 2,000
(c) the dog looked at the procession with big,
wondering eyes. Directions for the following 3 (three) items:
(d) the dog stood there with his eyes closed.
Read the passage given below and answer the items that
Passage – 2 follow.
Cynthia was a shy girl. She believed that she was A, B, C, D, E, F are members of a family. They
plain and untalented. One day her teacher ordered are engineer, stenographer, doctor, draughtsman,
the entire class to show up for audition for the school lawyer and judge (not in order). A, the engineer, is
married to the lady stenographer. The judge is married
play. Cynthia nearly died of fright when she was told
to the lawyer. F, the draughtsman, is the son of B and
that she would have to stand on stage in front of the
brother of E. C, the lawyer, is the daughter-in-law of
entire class and deliver dialogues. The mere thought
D. E is the unmarried doctor. D is the grandmother
of it made her feel sick. But a remarkable
of F. There are two married couples in the family.
transformation occurred during the audition. A thin,
shy girl, her knees quaking, her stomach churning in 78. What is the profession of B?
terror, began to stun everyone with her excellent (a) Judge (b) Lawyer
performance. Her bored classmates suddenly stopped (c) Draughtsman (d) Cannot be determined
their noisy chat to stare at her slender figure on the 79. Which of the following is/are a couple/couples?
stage. At the end of her audition, the entire room (a) AD only (b) BC only
erupted in thunderous applause. (c) Both AD and BC (d) Both AC and BD
72. Cynthia was afraid to stand on stage because 80. What is the profession of D?
(a) she felt her classmates may laugh at her. (a) Judge (b) Stenographer
(b) her stomach was churning. (c) Doctor (d) Cannot be determined

ST . 82
Sample Test 4 : General Studies Paper II (2014)

PRACTICE ANSWER SHEET

Directions: Use black ball pen. Example

1. 17. 33. 49. 65.

2. 18. 34. 50. 66.

3. 19. 35. 51. 67.

4. 20. 36. 52. 68.

5. 21. 37. 53. 69.

6. 22. 38. 54. 70.


@UPSC_THOUGHTS

7. 23. 39. 55. 71.

8. 24. 40. 56. 72.

9. 25. 41. 57. 73.

10. 26. 42. 58. 74.

11. 27. 43. 59. 75.

12. 28. 44. 60. 76.

13. 29. 45. 61. 77.

14. 30. 46. 62. 78.

15. 31. 47. 63. 79.

16. 32. 48. 64. 80.

ST . 83
Sample Test 4 : General Studies Paper II (2014)

ANSWERS AND EXPLANATORY NOTES


1. (c) The entire passage is focused on inclusive get 648 for 4 rows, and 12 to 648 to get 660
growth. for 5 rows, and 15 to 660 to get 675 for 6 rows.
2. (d) This is clear from the sentences in Paragraph Now, with 675 ÷ 6, you at once know that it is
2: “If that were done . . . policies.” not fully divisible, so 6 rows are not possible.
3. (d) Calculations need to be done mentally to save
4. (b) Refer to the sentence, “This will mean . . . time.]
profits.” 8. (c) You have seven persons in this order:
5. (a) C
6. (b) The L.C.M. of the time intervals needs to be A
found.
L.C.M. of 6, 7, 8, 9 and 12 is 504. G
The five persons will fire together every 504
seconds. B
There are 60 × 60 = 3600 seconds in an hour. E
The five persons will fire together
3600 F
= 7 times in an hour.
504 D
7. (d) Let the number of children in the first row be
9. (c) We are given D > C > A > B and E is tallest
x; the number of children in the next few rows of all. The heights in descending order: E D C
will be x – 3, x – 6, x – 9, and so on.
A B. So the middle position is occupied by C.
Take the options one-by-one.
10. (b) Place F and D. N
(a) Number of rows is 3, so we have
@UPSC_THOUGHTS

x + (x – 3) + (x – 6) = 630 Now look for a


W E
3x – 9 = 630 village whose
630 + 9 639 direction is given S A
x = = = 213 1 km
relative to one of F D
3 3 these. The last
This is a possibility. 1 km
statement says D is
[What one has to see is if the value 1 km to south of A. D
F
639 Place A. 1 km
of x as in is a whole number. 1 km
3 A C
Now look for
If it is, the number of rows given is possible.]
statements that 1 km
(b) Number of rows is 4, so we have relate to any of these
x + x – 3 + x – 6 + x – 9 = 630 villages whose F 1 km
D A 1 km
630 + 3 + 6 + 9 648 positions are fixed. C
x = = = 162 Take the statement
4 4 before the last one:
This is possible. 1 km
(c) Number of rows is 5, so we have C is 1 km to the east
1 km
of A. Place C. F D 2 km
x + x – 3 + x – 6 + x – 9 + x – 12 = 630
Now the third
630 + 3 + 6 + 9 + 12 660 1 km
x = = = 132 statement: A is 2 km
5 5 to north of E. Clearly
This is possible. E is 1 km south of D. E
At this stage itself you may choose (d) as Place E. A 1 km
answer; with (a) (b) and (c) being possible, C
At this point, you 1 km
only (d) will not be possible.
have your answer: A,
However, just to check. 1 km
D, E are three D
(d) With 6 rows, we have F
villages in a line. So
x + x – 3 + x – 6 + x – 9 + x – 12 + x – 15 = 630 option (b) is correct. 1 km
630 + 3 + 6 + 9 + 12 + 15 675 Check for B if you
x = = feel the need to.
B
6 6 E 1 km
which is not likely to be a whole number. [Note: You should be able to arrive at the
[Note: Calculations need not be repeated. Once answer when you read ‘A is 2 km to the north
you have 639 for 3 rows, just add 9 to 639 to of E’ and ‘D is 1 km to the south of A’.]

ST . 84
Sample Test 4 : General Studies Paper II (2014)

11. (b) Given that A and B sit next to each other but 50 = 21 – 10 + x
that A does not sit next to C and C does not 50 – 21 + 10 = x
sit next to D, only two rows are possible with x = 29].
the four children: 20. (b) 40% = 0.4.; 20% = 0.2.
D A B C or C B A D If length L increases by 40%, the new length
(A will be either to the left of B or to the right is L + 0.4 L which is 1.4 L
of B.) Either way, A is adjacent to D. If breadth B decreases by 20%, the new breadth
12. (c) The hands will coincide next between 9 o’clock is B – 0.2 B = 0.8 B
and 10 o’clock which will happen after 65 Original area = LB
minutes. (The minute hand would move one full New area = 1.4 L × 0.8 B = 1.12 LB
revolution, i.e., 60 minutes, to reach its original Area change is 1.12 LB – LB = 0.12
place between 8 o’clock and 9 o’clock and then which is a 12 per cent increase.
5 minutes more to reach the required position [If the length and breadth of a rectangle are
between 9 o’clock and 10 o’clock.) increased by x% and y% respectively, the area
13. (b) Statements 2 and 3 are clear from: “To ensure xy
continued yields. . . water availability.” of the rectangle will change by x + y + %
100
(Paragraph 2) Statement 1 relates to water
Now, in the given problem, breadth is decreased,
availability in a particular way and is just an
so it is (–y)%. Substituting in the formula we
example of how water availability may change.
have
Statement 4 would not lead to the need to shift
location of crops. 40 × (−20)
40 + ( −20) + %
14. (c) Refer to the last sentence of Paragraph 1. 100
15. (c) Meat is a form of animal protein. Reducing 80
consumerism is implied in ‘changes in lifestyles’. = 40 − 20 − %
100
Refer to Paragraph 2. = (20 – 8)% = 12%
@UPSC_THOUGHTS

[Now check the options. As (a) and (c) have 1, As the sign is positive, there is an increase of
one of them must be correct. Check Statement 12% in the area. (This formula is given in the
2. The passage does not support it. So (a) chapter on Mensuration—Fast-Track Formulae—
cannot be correct; (c) alone can be correct.] in the section, Basic Numeracy and Data
16. (a) Refer to Paragraph 1. Interpretation.)]
17. (d) One of the co-benefits of mitigation is health
benefit, implying reduced expenditure on public 21. (b) Make a chart of the information:
health; so Statement 1 is correct. While reduced A B C D E F
dependence on livestock will lead to mitigation, y g g y g g
nothing in the passage suggests mitigation will
new new old new old old
reduce dependence on livestock. Statement 2
is not correct. [So answer responses (a) and (c) law law law medical medical medical
cannot be correct.] The passage does not say Clearly E and F are the required items.
anywhere that mitigation of greenhouse gases [However, even without making a chart, and just
will reduce energy requirements. So Statement by checking the options, you can get the correct
3 is not correct. Statement 4 is self-evident. answer.
18. (b) As A, B and C are law books, D, E and F are
19. (d) Those who speak only English is 21-10 (as 10 medical extracts. The question relates to medical
can also speak Hindi) which is 11. Now, among extracts, so the answer must have two of the
the answer responses, only (a) and (d) have 11. items D, E and F. Answer responses (a), (c) and
The question places English at the end, so (d) (d) may be eliminated as they mention B and
is obviously correct. C—both law books.]
[Check: English only = 11 22. (b) A rough diagram with the numbers plugged in
Hindi only = Total – English speakers from both ends as per the given intervals will
= 50 – 21 = 29 give you the answer fast.
Hindi = Hindi only speakers + Speakers of 1 2 3 4 5 6 7 8
both Hindi and English
A 0 1 3 6 10 15 21 28 36 B
= 29 + 10 = 39
50 8 15 21 26 30 33 35
8 7 6 5 4 3 2 1
21 10 x You see that 15 and 21 are common and must
be counted once each. So leaving out the end
points (0 and 36) you have a total of 12 points.

ST . 85
Sample Test 4 : General Studies Paper II (2014)

23. (c) If two articles have the same selling price, and 25. (a) Four cards need to be turned every time.
gain per cent on one is equal to the loss per The lowest number of turns among the answer
cent on the other, there is always a loss on the responses is 3. Let’s see if we can get all six
whole and the loss per cent is given by cards showing 2 on the top side in 3 moves.
Gain % or Loss %
2 After 1st move : 1 1 2 2 2 2
After 2nd move : 2 1 1 1 1 2
10 After 3rd move : 2 2 2 2 2 2
Here gain % = loss % = 10 So the required condition can be achieved in
2
10 3 moves. In the second move, turn one 1 and
∴ The loss per cent = % = 1% three 2’s to get four 1’s top side. These four 1’s
10
[This formula is given in the chapter on Profit can then be turned to get all 2’s top side.
and Loss (Fast-Track) in the section, Basic 26. (c) Both statements are implied in the sentence,
Numeracy and Data Interpretation.] “The threats include . . . services.” (Paragraph 2)
Alternative method: If selling price of each 27. (c) This appears to be the best answer, as it is
goat is x, because of the rich biodiversity that the region
needs conservation. Response (d) actually
100 10
Cost price of one goat is x = x explains contemporary conservation approaches
110 11 rather than implying the ‘need’ for them.
100 10 28. (b) The other responses are to be found in the
Cost price of the other goat is x = x
90 9 passage but all of them lead to the focus of the
Selling price of both goats = 2x passage expressed in (b).
10 10 200 29. (b)
Total cost price = x + x = x 30. (a) Refer to the sentence, “But the basic philosophy
11 9 99
As selling price is lower than cost price, clearly . . . inefficiency.”
a loss is indicated. 31. (c)
@UPSC_THOUGHTS

32. (c) Refer to the sentence, “This logic . . . health.”


200x
Loss = – 2x 33. (b)
99 34. (b) The readings for A and B are at the same point
200 − 198 2 in 1996, so the average profit of both is the
= x = x
99 99 same that year (Rs 4,000).
2x 200x 35. (c) From the graph this seems to be the correct
Percentage of loss = × 100 response. The point for A in 1998 is a little lower
99 99
than the half point between 3,000 and 4,000
2x 99 indicating a figure less than Rs 3,500. B’s
= × × 100
99 200 x average profit for the same year is Rs 4,000.
= 1%. So the distance between the points cannot be
24. (b) 5% of 120 = 6; so 6 musicians play all three as great as Rs 1,000 or as low as Rs 100 or
instruments. Number of musicians who play any Rs 300.
two (only) instruments = 30 36. (d) It is clear from the graph.
Those who play guitar only = 40 37. (b)
The total number of musicians who play the 38. (d) Mean aggregate percentage marks
violin or the flute alone is: Total marks obtained
= × 100
Total – (guitar only + any two + all three) Total maximum marks
= 120 – (40 + 30 + 6) 210
= 120 – 76 = 44 For Student A : × 100 = 60%
350
Alternatively: Use circles to stand for each
225
instrument and draw a diagram: For Student B : × 100 = 60%
375
The difference is zero.
a x
39. (c)
Guitar 40 Violin 40. (b) Taking the first column and the last column, we
6 see that while the left hand (LH) arrowhead
b c remains in the same position, the right hand
(RH) arrowhead moves in a clockwise direction
y 90° at a time. So in the middle column, the next
Flute
position of the RH arrowhead will be vertical.
a + b + c = 30 (given) As the LH arrow will also remain vertical, (b)
x + y = Total – (40 + 30 + 6) is the correct answer response.
= 120 – 76 = 44.

ST . 86
Sample Test 4 : General Studies Paper II (2014)

[If we take the rows, the pattern is, RH arrow anticlockwise movements alternate), with the
is reversed at every step while the LH arrow is shaded portion at the top. Of (b) and (c), only
reversed in the last step. So in the last row, the (b) fits, so (b) is the correct answer. (You may
middle place will have both arrows in vertical choose any two circles and trace the pattern.)
positions.] 46. (b) Find the L.C.M. of the times to get the time when
41. (c) It is seen that there is an increase of one lakh they will ring again together.
in population every year. As the base population L.C.M. of 18, 24 and 32 = 288 minutes or
increases every year, there will not be a 4 hours 48 minutes.
percentage increase but a percentage decrease. So the bells will ring together again at 8 + 4 : 48
The increase cannot be 5% every year. or at 12 : 48 hours.
[Percentage increase in 1994 over 1993, for 47. (d) Price change implies that value of money
changes. So if prices double, value of money
1
example = × 100 ≠ 5% but <5%] is halved. As the author clearly says “values of
21 all things except one”, option (a) is not correct.
So (a) is not correct.
48. (c) ` 10 is 80 per cent of the bus fare for two. The
In case of (b), the increase was 10% for 1993 fare is
1111 − 1010 100
× 100 = 10% ; increase was more
1010 × 10 = ` 12.50
80
than 10% for 1994 over 1993 With B giving ` 3 to A, A has ` 13 with him.
1225 − 1111 After paying ` 12.50 for the fare he will have
× 100 = 10.26% ; but income in
1111 50 paise left.
1995 over 1994 decreased 49. (c) If 60 instalments have been paid, full number
1345 − 1225 of instalments is 100.
× 100 = 9.79% . If 18 have been paid, full number of instalments
1225 100
So option (b) is not correct. = 60 × 18 = 30.
@UPSC_THOUGHTS

Income 50. (c) Let the distance be d km. We know


Per capita income =
Population d
Per capita income for the given years:
Speed = time
101000 111100 122500 134500 If the worker reaches late by 3 minutes and early
, , ,
20 21 22 23 by 7 minutes at two different speeds,
(Converting crores into lakhs) we have
It is seen that all are above Rs 5,000. d d 10
So (c) is correct. − = (3 + 7 =) 10 minutes or hour
5 6 60
There is no need to check further. However, if
6d − 5d 1
you check you will find per capita income is =
highest in 1995, so (d) is not correct. 30 6
30
42. (d) The sum of the numbers in each row is 60. d = = 5 km.
So X = 60 – (33 + 19) = 8; 6
43. (a) Take either the rows or columns and establish 51. (a) 52. (a)
the pattern. Row-wise, moving left to right, a 53. (b) The free market supporters believe that the free
circle is deleted and a triangle added at every market is fair and competent, but the implication
step. So in the last box, there will be no circle in the passage is that it is competent—it creates
and two triangles—as in (a). [The same pattern wealth and economic growth—but it is not fair
is to be seen columnwise, top to bottom.] as there is ‘gross inequity’. See the last
44. (d) Number the vortices and trace the routes. paragraph.
45. (b) Take one circle at a time, say top left (TL). The 54. (c)
shaded half first reverses then moves 90° 55. (c) Statements 2, 3 and 4 are endorsed in Paragraph
clockwise, then reverses from figure to figure by 1.
turn. Next it must move 90°, but is it clockwise? 56. (b) Refer to Paragraph 2, sentence “At the opposite
No figure in the answer responses shows TL end, rural roads, whose sparse traffic makes
circle with shaded portion moved 90° clockwise tolling unviable . . .”. This implies lack of
(which would show shaded portion on right half monetary gains to the private sector.
of the circle). So, it must move 90° anticlockwise. While (c) may be true, it is not mentioned in the
Options (b) and (c) fit the picture. passage.
Now take top right (TR) circle. The shaded 57. (c) 58. (a)
portion is reversed and then turned 90° 59. (b) As a question on A, a medieval period poet,
anticlockwise, then again reversed. Now it would figured in last year’s paper, this year a question
turn 90° clockwise (as the clockwise/ on medieval poets will not be there, so option

ST . 87
Sample Test 4 : General Studies Paper II (2014)

(a) can be ruled out. As the paper-setter would 68. (b) B


not like to set a question on F, (c) is ruled out.
But as he likes F, he also likes E, so he is most
5 km
likely to set a question on E. Option (b) is the
best answer. (The question is a little vague, and
does not give enough information as to why a A 12 km C
question on H should not be set!) The shortest distance in this case will be the
60. (d) A rough chart will help. measure of the hypotenuse of the right triangle
v = violinist, m = musician, d = dancer, formed, BAC.
a = actress BC2 = (AB)2 + (AC)2,
Girija v BC = (AB)2 + (AC)2
Vanaja v, m
Jalaja not v, m = 52 + 122
Shailaja not v, d, m = 25 + 144
Tanuja d, m
Pooja a = 169 = 13 km.
Only Tanuja can certainly be a dancer and a 69. (a) The factor to be found is the time.
violinist. Distance
[Even without a chart you can get the answer Time =
Speed (here, sum of the two rates)
fast. As you are asked about who is dancer and 160 160
violinist, you may just focus on Shailaja and
Time = = = 2 hours
50 + 30 80
Tanuja. Given that Shailaja does not know how The two cars will meet at
to play the violin, only Tanuja can be both 8 : 10 + 2 = 10.10 A.M.
dancer and violinist.] 70. (a) 71. (b)
@UPSC_THOUGHTS

61. (d) As P is 4, T must be 9 (9 – 4 being 5) 72. (c) The implication is lack of self-confidence. Refer
If the difference between N and T is 3, N has to the first two lines of the passage.
to be 9 – 3 = 6. 73. (b) The sentence, “Her bored classmates . . . .”
62. (d) 63. (d) 64. (b) makes this clear.
65. (a) As Alok has more money than Bhupesh, Dinesh 74. (a) 75. (d)
has more money than Chander, and Alok has 76. (c) If the 3rd is a Monday, every 7th day from the
3rd will be a Monday. So, 10th, 17th and 24th
` 5 more than Dinesh, clearly Alok has the
will be a Monday. The fifth day from the 21st
maximum.
is the 26th which will be Wednesday (two days
66. (d) As per statement 4, George attends one class
after 24th—a Monday).
in a day. As his History classes are on the day
77. (c) Half of 420 = 210
following the day of his Math classes, History
One-third of 420 = 140
classes are on Thursday. With Monday,
Rest = 420 – 350 = 70
Tuesday, Wednesday, Thursday accounted for Amount received = 210 × 5 + 140 × 3 + 70 × 2
and being told his literature classes are not on = 1050 + 420 + 140 = Rs 1610.
Friday (Statement 3), Friday must be the only For Qs 78 to 80: Draw a family-tree chart. Start with
other day (besides Sunday) that he is free. D as she is the grandmother and the others must be
A chart may simplify things:
related in some way to her.
Monday : Music
D (grandmother)
Tuesday : Sports
Wednesday : Mathematics
Thursday : History
(judge) B m. E (lawyer and
Friday : Not Literature d-in-law of D)
Saturday : Must be Literature
Sunday : Free
Friday is free also. F E
(draughtsman, (brother of F,
67. (c)
grandson of D and unmarried doctor)
A (11th) B (10th) son of B)
A (18th) It is now clear that A, the engineer, must be
There must be 9 persons from the right before married to D who is the lady stenographer.
B, so total number in the row is 18 + 9 = 27. Now the question can be answered.
Leaving out A and B, the number is 25. 78. (a) 79. (c) 80. (b)

ST . 88
Sample Test 5 : General Studies Paper II (2015)

SAMPLE TEST 5: GENERAL STUDIES PAPER II (2015)


Directions for the following 8 (eight) items: (a) Building of weapons systems by us has
Read the following six passages and answer the items that instigated our neighbours to wage wars
follow. Your answers to these items should be based on the against us.
passages only. (b) The greater spending on weapon-building
by us would lessen the possibility of armed
Passage – 1 conflict with our neighbours.
Human history abounds in claims and theories (c) It is necessary to have state of the art
confining the right of governing to a few select citizens. weapons systems for national security.
Exclusion of the many is justified on the ground that (d) Many people in India believe that we are
human beings may be rightfully segregated for the good wasting our resources on weapon-building.
of society and viability of the political process.
Passage – 4
1. Which one of the following statements is least
essential as a part of the argument in the above India accounts for nearly a fifth of the world’s child
passage? deaths. In terms of numbers, it is the highest in the
(a) Man seeks control over external things world—nearly 16 lakhs every year. Of these, more than
affecting him. half die in the first month of life. Officials believe that
(b) In society, there are ‘super’ and ‘sub’ human the reason for this is the absence of steps to propagate
beings. basic health practices relating to breast feeding and
(c) Exceptions to universal citizen participation immunisation. Also the large reproductive population
are conducive to systemic efficacy. of 2-6 crore remains bereft of care during the critical
(d) Governing implies recognition of disparities phases of pregnancy and post-delivery. Added to this
in individual capacities. is the prevalence of child marriages, anaemia among
young women and lack of focus on adolescent sanitation,
@UPSC_THOUGHTS

Passage – 2 all of which impact child death rates.


By 2050, the Earth’s population will likely have 4. Which is the critical inference that can be made
swelled from seven to nine billion people. To fill all from the above passage?
those stomachs—while accounting for shifting (a) A lot of Indians are illiterate and hence do
consumption patterns, climate change, and a finite not recognise the value of basic health
amount of arable land and potable water—some experts practices.
say food production will have to double. How can we (b) India has a very huge population and the
make the numbers add up? Experts say higher yielding government alone cannot manage public
crop varieties and more efficient farming methods will health services.
be crucial. So will waste reduction. Experts urge cities (c) Universalisation and integration of maternal
to reclaim nutrients and water from waste streams and health and child health services can
preserve farmland. Poor countries, they say, can improve effectively address the problem.
crop storage and packaging and rich nations could cut (d) The nutrition of women in child bearing age
back on resource-intensive foods like meat. does not affect child mortality rate.
2. Which one of the following statements best sums Passage – 5
up the above passage?
(a) The population of the world is growing Foods travel more than the people who eat them.
very fast. Grocery stores and supermarkets are loaded with
(b) Food security is a perennial problem only preserved and processed foods. This, however, often
in developing countries. leads to environmental threats, such as pollution
(c) The world does not have enough resources generated by long distance food transportation and
to meet the impending food scarcity. wastage of food during processing and transportation,
(d) Food security is increasingly a collective destruction of rain forests, reduced nutritional content,
challenge. increased demand for preservation and packaging. Food
insecurity also increases as the produce comes from
Passage – 3 regions that are not feeding their own population
Many people in India feel that if we cut our defence properly.
expenditure on weapon-building, we can create a climate 5. With reference to the above passage, which of the
of peace with our neighbours, subsequently reducing following statements is/are true?
the conflict or creating a no-war situation. People who 1. Consuming regionally grown food and not
proclaim such ideas are either the victims of war or the depending on long travelled food is a part
propagators of false argument. of eco-friendly behaviour.
3. With reference to the above passage, which of the 2. Food processing industry puts a burden on
following is the most valid assumption? our natural resources.

ST . 89
Sample Test 5 : General Studies Paper II (2015)

Select the correct answer using the code given 10. An automobile owner reduced his monthly petrol
below: consumption when the prices went up. The price-
(a) 1 only (b) 2 only consumption relationship is as follows:
(c) Both 1 and 2 (d) Neither 1 nor 2
Price (in ` per litre) 40 50 60 75
Passage – 6 Monthly consumption (in litres) 60 48 40 32
I must say that, beyond occasionally exposing me If the price goes up to ` 80 per litre, his expected
to laughter, my constitutional shyness has been of no consumption (in litres) will be
disadvantage whatever. In fact I can see that, on the (a) 30 (b) 28
contrary, it has been all to my advantage. My hesitancy (c) 26 (d) 24
in speech, which was once an annoyance, is now a 11. Consider the figures given below:
pleasure. Its greatest benefit has been that it has taught
me the economy of words. I have naturally formed the
habit of restraining my thoughts. And I can now give
myself the certificate that a thoughtless word hardly ever
escapes my tongue or pen. I do not recollect ever having
had to regret anything in my speech or writing. I have
thus been spared many a mishap and waste of time. To fit the question mark, the correct answer is
Experience has taught me that silence is part of the
spiritual discipline of a votary of truth. Proneness to
exaggerate, to suppress or modify the truth, wittingly (a) (b)
or unwittingly, is a natural weakness of man, and silence
is necessary in order to surmount it. A man of few words
will rarely be thoughtless in his speech; he will measure (c) (d)
every word. We find so many people impatient to talk.
There is no chairman of a meeting who is not pestered
12. Consider the following matrix:
with notes for permission to speak. And whenever the
@UPSC_THOUGHTS

permission is given the speaker generally exceeds the 3 8 10 2 ? 1


time-limit, asks for more time, and keeps on talking
without permission. All this talking can hardly be said 6 56 90 2 20 0
to be of any benefit to the world. It is so much waste
of time. My shyness has been in reality my shield and What is the missing number at ‘?’ in the matrix?
buckler. It has allowed me to grow. It has helped me (a) 5 (b) 0
in my discernment of truth. (c) 7 (d) 3
6. The author says that a thoughtless word hardly 13. What is the missing number ‘X’ of the series
ever escapes his tongue or pen. Which one of the 7, X, 21, 31, 43 ?
following is not a valid reason for this? (a) 11 (b) 12
(a) He has no intention to waste his time. (c) 13 (d) 14
(b) He believes in the economy of words.
14. Four cardboard pieces of specific shapes are
(c) He believes in restraining his thoughts.
(d) He has hesitancy in his speech. shown in the following figure:
7. The most appropriate reason for the author to be
spared many a mishap is that
(a) he hardly utters or writes a thoughtless
word.
(b) he is a man of immense patience.
(c) he believes that he is a spiritual person.
(d) he is a votary of truth.
8. For the author, silence is necessary in order to
surmount
(a) constitutional shyness. Which one of the following figures given can be
(b) hesitancy in speech. formed by joining these pieces together?
(c) suppression of thoughts.
(d) tendency to overstate.
9. Twelve people form a club. By picking lots, one (a) (b)
of them will host a dinner for all once in a month.
The number of dinners a particular member has
to host in one year is
(a) One (b) Zero (c) (d)
(c) Three (d) Cannot be predicted

ST . 90
Sample Test 5 : General Studies Paper II (2015)

15. In a test, a candidate attempted only 8 questions (b) no athlete who does not eat a well-balanced
and secured 50% marks in each of the questions. diet is a good athlete.
If he obtained a total of 40% in the test and all (c) every athlete who eats a well-balanced diet
questions in the test carried equal marks, how is a good athlete.
many questions were there in the test? (d) all athletes who want to win are good
(a) 8 (b) 10 athletes.
(c) 15 (d) 16 Directions for the following 8 (eight) items:
16. A father is nine times as old as his son and the Read the following seven passages and answer the items that
mother is eight times as old as the son. The sum follow. Your answers to these items should be based on the
of the father’s and the mother’s age is 51 years. passages only.
What is the age of the son? Passage – 1
(a) 7 years (b) 5 years
The richer States have a responsibility to cut down
(c) 4 years (d) 3 years
carbon emissions and promote clean energy investments.
17. Four persons A, B, C and D consisting of two These are the States that got electricity, grew faster and
married couples are in a group. Both the women now have high per capita income, making them capable
are shorter than their respective husbands. A is of sharing India’s burden of becoming eco-friendly.
the tallest among the four. C is taller than B. D Delhi, for example, can help by generating its own clean
is B’s brother. In this context, which one of the electricity using solar rooftop panels or even help poor
following statements is not correct? States finance their clean energy projects. It is no secret
(a) All four have family ties that State Electricity Boards, which control 95 per cent
(b) B is the shortest among the four of the distribution network, are neck-deep in losses.
(c) C is taller than D These losses further discourage State utilities from
(d) A is B’s husband adopting renewable energy as it is more expensive than
fossil fuels.
18. Consider the following statements:
1. A man had a wife, two sons and two 21. Which among the following is the most logical
@UPSC_THOUGHTS

daughters in his family. and rational assumption that can be made from
2. The daughters were invited to a feast and the above passage?
the male members of the family went out (a) The richer States must lead in the production
to take part in a picnic. and adoption of renewable energy.
(b) The poor States always have to depend on
3. The man’s father did not return from his
rich States for electricity.
work.
(c) The State Electricity Boards can improve
Which of the following statements is true? their finances by undertaking clean energy
(a) Only the man’s wife was left at home. projects.
(b) It is likely that the man’s wife was left at (d) The high economic disparity between the
home. rich and poor States is the major cause of
(c) None was left at home. high carbon emissions in India.
(d) More than one person was left at home.
Passage – 2
19. Geeta : Naresh has become a better boxer since
Set against a rural backdrop, ‘Stench of kerosene’ is
he started meditation. the story of a couple, Guleri and Manak, who have been
Radha : Impossible. A boxer’s most important happily married for several years but do not have a
asset is his aggressiveness. child. Manak’s mother is desperate to have a grandchild
Radha’s statement reflects her belief that to carry on the family name. Hence, she gets Manak
(a) meditation tends to make a person less remarried in Guleri’s absence. Manak, who acts as a
aggressive. reluctant but passive spectator, is meanwhile, informed
(b) meditation has little or no effect on the by a friend that Guleri, on hearing about her husband’s
person who practises it. second marriage, poured kerosene on her clothes and
(c) Naresh was a poor boxer earlier because he set fire to them. Manak is heartbroken and begins to live
was not aggressive enough. as if he were a dead man. When his second wife delivers
(d) Naresh would not have taken to meditation a son, Manak stares at the child for a long time and
as he was a boxer. blurts out, “Take him away! He stinks of kerosene.”
20. All good athletes want to win and all athletes who 22. This is a sensitive issue-based story which tries
want to win eat a well-balanced diet; therefore all to sensitise the readers about
athletes who do not eat a well-balanced diet are (a) Male chauvinism and infidelity.
bad athletes. (b) Love and betrayal.
The best conclusion from this statement is that (c) Lack of legal safeguards for women.
(a) no bad athlete wants to win. (d) Influence of patriarchal mindset.

ST . 91
Sample Test 5 : General Studies Paper II (2015)

Passage – 3 Passage – 5
The ultimate aim of government is not to rule or Flamingos in large flocks in the wild are social and
control by fear, nor to demand obedience, but conversely, extremely loyal. They perform group mating dances.
to free every man from fear, that he may live in all Parents are very fond of their chicks, gathering them into
possible security. In other words, to strengthen his crèches for protection while both males and females fly
natural right to exist and work without injury to himself off to search for food.
or others. The object of government is not to change men 25. Which among the following is the most logical
from rational beings into beasts or puppets. It should corollary to the above passage?
enable them to develop their minds and bodies in (a) Mass nesting in all species of birds is
security, and to employ their reason unshackled. essential to ensure complete survival of
23. Which among the following is the most logical their offspring.
and rational inference that can be made from the (b) Only birds have the capacity to develop
social behaviour and thus can do mass
above passage?
nesting to raise their chicks in safety.
(a) The true aim of government is to secure the
(c) Social behaviour in some species of birds
citizens their social and political freedom. increases the odds of survival in an unsafe
(b) The primary concern of government is to world.
provide absolute social security to all its (d) All species of birds set up crèches for their
citizens. chicks to teach them social behaviour and
(c) The best government is the one that allows loyalty.
the citizens to enjoy absolute liberty in all
matters of life. Passage – 6
(d) The best government is the one that provides Vast numbers of Indian citizens without bank
absolute physical security to the people of accounts live in rural areas, are financially and
the country. functionally illiterate, and have little experience with
technology. A research study was conducted in a
Passage – 4 particular area in which electronic wage payments in
@UPSC_THOUGHTS

Our municipal corporations are understaffed. The Mahatma Gandhi National Rural Employment Guarantee
issue of skills and competencies of the staff poses an Scheme (MGNREGS) are meant to go directly to the
even greater challenge. Urban services delivery and poor. It was observed that recipients often assume that
infrastructure are complex to plan and execute. They the village leader needs to mediate the process, as was
require a high degree of specialisation and the case under the previous paper-based system. Among
professionalism. The current framework within which households under this research study area who claimed
municipal employees, including senior management, to have at least one bank account, over a third reported
still receiving MGNREGS wages in cash directly from
are recruited does not adequately factor in the technical
a village leader.
and managerial competencies required. Cadre and
recruitment rules only specify the bare minimum in 26. What is the most logical, rational and crucial
academic qualifications. There is no mention of message that is implied in the above passage?
managerial or technical competencies, or of relevant (a) MGNREGS should be extended only to
work experience. This is the case with most municipal those who have a bank account.
corporations. They also suffer from weak organisation, (b) The paper-based system of payments is
more efficient than electronic payment in
design and structure.
the present scenario.
24. Which among the following is the most logical (c) The goal of electronic wage payments was
and rational assumption that can be made from not to eliminate mediation by village leaders.
the above passage? (d) It is essential to provide financial literacy
(a) The task of providing urban services is a to the rural poor.
complex issue which requires the Passage – 7
organisational expansion of municipal
bodies all over the country. Individuals, groups and leaders who promote human
(b) Our cities can provide better quality of life development operate under strong institutional,
if our local government bodies have structural and political constraints that affect policy
options. But experience suggests broad principles for
adequate staff with required skills and
shaping an appropriate agenda for human development.
competencies. One important finding from several decades of human
(c) Lack of skilled staff is due to the absence development experience is that focusing exclusively on
of institutions which offer the requisite economic growth is problematic. While we have good
skills in city management. knowledge about how to advance health and education,
(d) Our country is not taking advantage of the the causes of growth are much less certain and growth
demographic dividend to manage the is often elusive. Further, an unbalanced emphasis on
problems associated with rapid urbanisation. growth is often associated with negative environmental

ST . 92
Sample Test 5 : General Studies Paper II (2015)

consequences and adverse distributional effects. The (a) peas are as tasty as lady’s finger.
experience of China, with its impressive growth record, (b) peas are as tasty as cauliflower and lady’s
reflects these broader concerns and underlines the finger.
importance of balanced approaches that emphasise (c) cabbage is the least tasty of the four
investments in the non-income aspects of human vegetables.
development. (d) cauliflower is tastier than cabbage.
27. With reference to the above passage, consider the 33. Shahid and Rohit start from the same point in
following statements: opposite directions. After each 1 km, Shahid
1. In developing countries, a strong always turns left and Rohit always turns right.
institutional framework is the only Which of the following statements is correct?
requirement for human development and (a) After both have travelled 2 km, the distance
policy options. between them is 4 km.
2. Human development and economic growth (b) They meet after each has travelled 3 km.
are not always positively inter-related. (c) They meet for the first time after each has
3. Focusing only on human development travelled 4 km.
should be the goal of economic growth. (d) They go on without ever meeting again.
Which of the above statements is/are correct? 34. In a 500 metres race, B starts 45 metres ahead of
A, but A wins the race while B is still 35 metres
(a) 1 only (b) 2 and 3 only
behind. What is the ratio of the speeds of A to
(c) 2 only (d) 1, 2 and 3
B assuming that both start at the same time?
28. With reference to the above passage, the following (a) 25 : 21 (b) 25 : 20
assumptions have been made: (c) 5 : 3 (d) 5 : 7
1. Higher economic growth is essential to 35. Two equal glasses of same type are respectively
ensure reduction in economic disparity. 1/3 and 1/4 full of milk. They are then filled up
2. Environmental degradation is sometimes a with water and the contents are mixed in a pot.
consequence of economic growth. What is the ratio of milk and water in the pot?
Which of the above is/are valid assumption/ (a) 7 : 17 (b) 1 : 3
@UPSC_THOUGHTS

assumptions? (c) 9 : 21 (d) 11 : 23


(a) 1 only (b) 2 only 36. Out of 130 students appearing in an examination,
(c) Both 1 and 2 (d) Neither 1 nor 2 62 failed in English, 52 failed in Mathematics,
29. If A runs less fast than B, and B runs as fast but whereas 24 failed in both English and Mathematics.
not faster than C, then, as compared to A, C runs The number of students who passed finally is
(a) slower than A (a) 40 (b) 50
(b) faster than A (c) 55 (d) 60
(c) with same speed as A 37. In a group of persons travelling in a bus, 6
(d) Given data is not sufficient to determine persons can speak Tamil, 15 can speak Hindi and
30. Each of A, B, C and D has ` 100. A pays ` 20 6 can speak Gujarati. In that group none can speak
to B, who pays ` 10 to C, who gets ` 30 from D. any other language. If 2 persons in the group can
speak two languages only and one person can
In this context, which one of the following
speak all the three languages, then how many
statements is not correct?
persons are there in the group?
(a) C is the richest. (a) 21 (b) 22
(b) D is the poorest. (c) 23 (d) 24
(c) C has more than what A and D have
together. 38. In a parking area, the total number of wheels of
(d) B is richer than D. all the cars (four-wheelers) and scooters/
motorbikes (two-wheelers) is 100 more than twice
31. In a town, 45% population read magazine A, 55% the number of parked vehicles. The number of
read magazine B, 40% read magazine C, 30% read cars parked is
magazines A and B, 15% read magazines B and (a) 35 (b) 45
C, 25% read magazines A and C; and 10% read (c) 50 (d) 55
all the three magazines. What percentage do not
read any magazine? 39. The mangroves can shed tons of leaves per acre
every year; fungi and bacteria break down this
(a) 10% (b) 15%
leaf litter and consume it, they then are consumed
(c) 20% (d) 25%
by tiny worms and crustaceans, which in turn feed
32. Examine the following statements: small fish, which feed larger fish and birds and
1. Lady’s finger is tastier than cabbage. crocodiles.
2. Cauliflower is tastier than lady’s finger. Which among the following is the most logical
3. Cabbage is not tastier than peas. inference of the above statement?
The conclusion that can be drawn from these (a) Coastal areas cannot have food chains
statements is that without mangroves.

ST . 93
Sample Test 5 : General Studies Paper II (2015)

(b) Mangroves are an essential component of United States Federal Reserve’s imminent reversal of its
all marine ecosystems. “Quantitative Easing” policy.
(c) Mangroves have a crucial role in some of 42. Which among the following is the most rational
the coastal food chains. and critical inference that can be made from the
(d) The composition of marine flora and fauna above passage?
is largely determined by mangroves. (a) Foreign portfolio investments are not good
40. “By liberty I mean the eager maintenance of that for emerging economies.
atmosphere in which men have the opportunity (b) Advanced economies undermine the global
to be their best selves.” financial stability.
(c) India should desist from accepting foreign
Which one of the following expresses the view portfolio investments in the future.
implied in the above statement? (d) Emerging economies are at a risk of shock
(a) Liberty is the absence of restraint on human from advanced economies.
action.
(b) Liberty is what law permits people to Passage – 3
perform. Open defecation is disastrous when practised in
(c) Liberty is the ability to do what one desires. very densely populated areas, where it is impossible to
(d) Liberty is the maintenance of conditions for keep away human faeces from crops, wells, food and
the growth of human personality. children’s hands. Groundwater is also contaminated by
open defecation. Many ingested germs and worms
Directions for the following 7 (seven) items: spread diseases. They prevent the body from absorbing
Read the following six passages and answer the items that calories and nutrients. Nearly one-half of India’s children
follow. Your answers to these items should be based on the remain malnourished. Lakhs of them die from preventable
passages only. conditions. Diarrhoea leaves Indians’ bodies smaller on
average than those of people in some poorer countries
Passage – 1 where people eat fewer calories. Underweight mothers
Climate change is already making many people produce stunted babies prone to sickness who may fail
hungry all over the world, by disrupting crop yields and to develop their full cognitive potential. The germs
@UPSC_THOUGHTS

pushing up prices. And it is not just food but nutrients released into environment harm rich and poor alike,
that are becoming scarcer as the climate changes. It is even those who use latrines.
the poorest communities that will suffer the worst effects 43. Which among the following is the most critical
of climate change, including increased hunger and inference that can be made from the above passage?
malnutrition as crop production and livelihoods are (a) The Central and State governments in India
threatened. On the other hand, poverty is a driver of do not have enough resources to afford a
climate change, as desperate communities resort to latrine for each household.
unsustainable use of resources to meet current needs. (b) Open defecation is the most important
public health problem of India.
41. Which among the following is the most logical (c) Open defecation reduces the human capital
corollary to the above passage? of India’s workforce.
(a) Government should allocate more funds to (d) Open defecation is a public health problem
poverty alleviation programmes and in all developing countries.
increase food subsidies to the poor
communities. Passage – 4
(b) Poverty and climate impacts reinforce each We generally talk about democracy but when it
other and therefore we have to re-imagine comes to any particular thing, we prefer a belonging to
our food systems. our caste or community or religion. So long as we have
(c) All the countries of the world must unite this kind of temptation, our democracy will remain a
in fighting poverty and malnutrition and phoney kind of democracy. We must be in a position
treat poverty as a global problem. to respect a man as a man and to extend opportunities
(d) We must stop unsustainable agricultural for development to those who deserve them and not to
practices immediately and control food those who happen to belong to our community or race.
prices. This fact of favouritism has been responsible for much
discontent and ill-will in our country.
Passage – 2 44. Which one of the following statements best sums
The Global Financial Stability Report finds that the up the above passage?
share of portfolio investments from advanced economies (a) Our country has a lot of diversity with its
in the total debt and equity investments in emerging many castes, communities and religions.
economies has doubled in the past decade to 12 percent. (b) True democracy could be established by
The phenomenon has implications for Indian policy providing equal opportunities to all.
makers as foreign portfolio investments in the debt and (c) So far none of us have actually understood
equity markets have been on the rise. The phenomenon the meaning of democracy.
is also flagged as a threat that could compromise global (d) It will never be possible for us to establish
financial stability in a chain reaction, in the event of truly democratic governance in our country.

ST . 94
Sample Test 5 : General Studies Paper II (2015)

Passage – 5 only if the Government has wider


The existence/establishment of formal financial discretionary power.
institutions that offer safe, reliable and alternative (d) None of the above statements is a logical
assumption that can be made from this
financial instruments is fundamental in mobilising
passage.
savings. To save, individuals need access to safe and
reliable financial institutions, such as banks, and to 48. A selection is to be made for one post of Principal
appropriate financial instruments and reasonable financial and two posts of Vice-Principal. Amongst the six
incentives. Such access is not always available to all candidates called for the interview, only two are
people in developing countries like India and more so, eligible for the post of Principal while they all are
in rural areas. Savings help poor households manage eligible for the post of Vice-Principal. The number
volatility in cash flow, smoothen consumption, and of possible combinations of selectees is
build working capital. Poor households without access (a) 4 (b) 12
to a formal savings mechanism encourage immediate (c) 18 (d) None of the above
spending temptations. 49. A student has to opt for 2 subjects out of 5 subjects
for a course, namely, Commerce, Economics,
45. With reference to the above passage, consider the Statistics, Mathematics I and Mathematics II.
following statements: Mathematics II can be offered only if Mathematics
1. Indian financial institutions do not offer any I is also opted. The number of different
financial instruments to rural households to combinations of two subjects which can be opted
mobilise their savings. is
2. Poor households tend to spend their (a) 5 (b) 6
earnings/savings due to lack of access to (c) 7 (d) 8
appropriate financial instruments.
50. A person ordered 5 pairs of black socks and some
Which of the statements given above is/are correct? pairs of brown socks. The price of a black pair
(a) 1 only (b) 2 only was thrice that of a brown pair. While preparing
(c) Both 1 and 2 (d) Neither 1 nor 2 the bill, the bill clerk interchanged the number of
46. What is the crucial message conveyed in the black and brown pairs by mistake which increased
the bill by 100 per cent. What was the number
@UPSC_THOUGHTS

passage?
of pairs of brown socks in the original order?
(a) Establish more banks
(a) 10 (b) 15
(b) Increase the Gross Domestic Product (GDP) (c) 20 (d) 25
growth rate
(c) Increase the interest rate of bank deposits 51. The number of persons who read magazine X only
(d) Promote financial inclusion is thrice the number of persons who read magazine
Y. The number of persons who read magazine Y
Passage – 6 only is thrice the number of persons who read
Governments may have to take steps which would magazine X. Then, which of the following
otherwise be an infringement on the Fundamental Rights conclusions can be drawn?
of individuals, such as acquiring a person’s land against 1. The number of persons who read both the
his will, or refusing permission for putting up a building, magazines is twice the number of persons
who read only magazine X.
but the larger public interest for which these are done 2. The total number of persons who read
must be authorised by the people (Parliament). either one magazine or both the magazines
Discretionary powers to the administration can be done is twice the number of persons who read
away with. It is becoming more and more difficult to both the magazines.
keep this power within limits as the government has
many number of tasks to perform. Where discretion has Select the correct answer using the code given
to be used, there must be rules and safeguards to below:
prevent misuse of that power. Systems have to be (a) 1 only (b) 2 only
devised which minimise, if not prevent, the abuse of (c) Both 1 and 2 (d) Neither 1 nor 2
discretionary power. Government work must be 52. The graph below depicts the earnings of A and
conducted within a framework of recognised rules and B over the period 2000 to 2010:
principles, and decisions should be similar and
predictable.
47. Which among the following is the most logical
assumption that can be made from the above
passage?
Earnings

A
(a) Government should always be given wide
discretionary power in all matters of
administration. B
(b) The supremacy of rules and safeguards
should prevail as opposed to the influence
of exclusive discretion of authority.
(c) Parliamentary democracy is possible 2000 2007 2010
Years

ST . 95
Sample Test 5 : General Studies Paper II (2015)

From the graph, which one of the following can 50


be concluded?
(a) On the average A earned more than B 40
during this period.

Price in Rupees
(b) On the average B earned more than A 30
during this period.
(c) The earnings of A and B were equal during
this period. 20
(d) The earnings of A were less as compared
to B during this period. 10
53. Two pipes A and B can independently fill a tank 0
completely in 20 and 30 minutes respectively. If 1980 1990 2000 2010
both the pipes are opened simultaneously, how
much time will they take to fill the tank Years
completely? The price of the commodity in the year 1990
(a) 10 minutes (b) 12 minutes (a) must have been ` 10
(c) 15 minutes (d) 25 minutes (b) must have been ` 12
54. Each of the six different faces of a cube has been (c) must have been anywhere between ` 10 and
coated with a different colour i.e., V, I, B, G, Y ` 20
and O. Following information is given: (d) is higher than that in the year 1991
1. Colours Y, O and B are on adjacent faces.
2. Colours I, G and Y are on adjacent faces. 59. The proportion of expenditure on various items
3. Colours B, G and Y are on adjacent faces. by two families A and B is represented in the
4. Colours O, V and B are on adjacent faces. following Bar Charts:
Which is the colour of the face opposite to the 10% Food
face coloured with O? 50%
(a) B (b) V Food
@UPSC_THOUGHTS

(c) G (d) I 60%


Other items
55. Consider the following statements followed by
two conclusions:
30%
Statements : Some men are great.
Other items
Some men are wise. 30%
Conclusion I : Men are either great or wise. 20% Education
Conclusion II : Some men are neither great nor Education
wise. Family A Family B
Which one of the following is correct? Total expenditure: Total expenditure :
(a) Only conclusion I is valid ` 20,000 per month ` 1,00,000 per month
(b) Only conclusion II is valid From these charts, we can conclude that
(c) Both the conclusions are valid
(a) Family A spent more money on food than
(d) Neither of the conclusions is valid
Family B.
56. Consider the following statements: (b) Family B spent more money on food than
1. Some claim to have seen UFOs (Unidentified Family A.
Flying Objects). (c) Family A and Family B spent the same
2. Life on other heavenly bodies is considered amount on food.
to be a possibility. (d) The expenditure on food by Family A and
3. Voyage to space is now an established fact. Family B cannot be compared.
From the above statements, it may be concluded 60. Usha runs faster than Kamala, Priti runs slower
that than Swati, Swati runs slower than Kamala. Who
(a) UFOs are heavenly bodies is the slowest runner?
(b) UFOs are sent from other heavenly bodies (a) Kamala (b) Priti
(c) Some living species in other heavenly bodies (c) Swati (d) Usha
are more intelligent than man
(d) Nothing definite can be said about the Directions for the following 7 (seven) items:
UFOs Read the following four passages and answer the items that
57. If ABC × DEED = ABCABC; where A, B, C, D and follow. Your answers to these items should be based on the
E are different digits, what are the values of D passages only.
and E?
(a) D = 2, E = 0 (b) D = 0, E = l Passage – 1
(c) D = 1, E = 0 (d) D = 1, E = 2 India has suffered from persistent high inflation.
58. Year-wise variation of the price of a certain Increase in administered prices, demand and supply
commodity is shown in the following graph: imbalances, imported inflation aggravated by rupee

ST . 96
Sample Test 5 : General Studies Paper II (2015)

depreciation, and speculation—have combined to keep (c) Old values, ideas and traditions persist
high inflation going. If there is an element common to despite the dynamic nature of human
all of them, it is that many of them are the outcomes society.
of economic reforms. India’s vulnerability to the effects (d) Constitutional guarantee of freedom of
of changes in international prices has increased with speech is not in the interest of society.
trade liberalisation. The effort to reduce subsidies has
resulted in a continuous increase in the prices of Passage – 4
commodities that are administered. Climate change is a complex policy issue with major
61. What is the most logical, rational and crucial implications in terms of finance. All actions to address
message that is implied in the above passage? climate change ultimately involve costs. Funding is vital
(a) Under the present circumstances, India for countries like India to design and implement
should completely avoid all trade adaptation and mitigation plans and projects. Lack of
liberalisation policies and all subsidies. funding is a large impediment to implementing
(b) Due to its peculiar socio-economic situation, adaptation plans. The scale and magnitude of the
India is not yet ready for trade liberalisation financial support required by developing countries to
process. enhance their domestic mitigation and adaptation actions
(c) There is no solution in sight for the problems are a matter of intense debate in the multilateral
of continuing poverty and inflation in India negotiations under the United Nations Framework
in the near future. Convention on Climate Change (UNFCCC). The
(d) Economic reforms can often create a high Convention squarely puts the responsibility for provision
inflation economy. of financial support on the developed countries, taking
into account their contribution to the stock of greenhouse
Passage – 2 gases (GHGs) in the atmosphere. Given the magnitude
No Right is absolute, exclusive or inviolable. The of the task and the funds required, domestic finances
Right of personal property, similarly, has to be perceived are likely to fall short of the current and projected needs
in the larger context of its assumed legitimacy. The Right of the developing countries. Global funding through the
of personal property should unite the principle of multilateral mechanism of the Convention will enhance
liberty with that of equality, and both with the principle their domestic capacity to finance the mitigation efforts.
@UPSC_THOUGHTS

of cooperation. 64. According to the passage, which of the following


62. In the light of the argument in the above passage, is/are a matter of intense debate in the multilateral
which one of the following statements is the most negotiations under UNFCCC regarding the role of
convincing explanation? developing countries in climate change?
(a) The Right of personal property is a Natural 1. The scale and size of required financial
Right duly supported by statutes and support.
scriptures. 2. The crop loss due to climate change in the
(b) Personal property is a theft and an developing countries.
instrument of exploitation. The Right of 3. To enhance the mitigation and adaptation
personal property is therefore violative of actions in the developing countries.
economic justice.
(c) The Right of personal property is violative Select the correct answer using the code given
of distributive justice and negates the below:
principle of cooperation. (a) 1 only (b) 2 and 3 only
(d) The comprehensive idea of economic justice (c) 1 and 3 only (d) 1, 2 and 3
demands that the Right of each person to 65. In this passage, the Convention puts the
acquisition of property has to be reconciled responsibility for the provision of financial support
with that of others. on the developed countries because of
1. their higher level of per capita incomes.
Passage – 3 2. their large quantum of GDP.
The conflict between man and State is as old as State 3. their large contribution to the stock of
history. Although attempts have been made for centuries GHGs in the atmosphere.
to bring about a proper adjustment between the Select the correct answer using the code given
competing claims of State and the individual, the below:
solution seems to be still far off. This is primarily (a) 1 only (b) 1 and 2 only
because of the dynamic nature of human society where (c) 3 only (d) 1, 2 and 3
old values and ideas constantly yield place to new ones.
It is obvious that if individuals are allowed to have 66. With regards to developing countries, it can be
absolute freedom of speech and action, the result would inferred from the passage that climate change is
be chaos, ruin and anarchy. likely to have implications on their
1. domestic finances.
63. The author’s viewpoint can be best summed up in 2. capacity for multilateral trade.
which of the following statements?
(a) The conflict between the claims of State and Select the correct answer using the code given
individual remains unresolved. below:
(b) Anarchy and chaos are the obvious results (a) 1 only (b) 2 only
of democratic traditions. (c) Both 1 and 2 (d) Neither 1 nor 2

ST . 97
Sample Test 5 : General Studies Paper II (2015)

67. Which one of the following is essentially discussed (a) 39 (b) 30


in the passage? (c) 21 (d) 20
(a) Conflict between developed and developing 75. Two men, Anil and David, and two women,
countries regarding support for mitigation Shabnam and Rekha are in a sales group. Only
(b) Occurrence of climate change due to two speak Tamil. The other two speak Marathi.
excessive exploitation of natural resources
Only one man and one woman can drive a car.
by the developed countries
Shabnam speaks Marathi. Anil speaks Tamil. Both
(c) Lack of political will on the part of all the
countries to implement adaptation plans Rekha and David can drive.
(d) Governance problems of developing Which of the following statements is true?
countries as a result of climate change (a) Both the Tamil speakers can drive a car.
68. Between 6 PM and 7 PM the minute hand of a (b) Both the Marathi speakers can drive a car.
clock will be ahead of the hour hand by 3 minutes (c) Both of those who can drive a car speak
at Marathi.
(a) 6 : 15 PM (b) 6 : 18 PM (d) One of those who can drive a car speaks
(c) 6 : 36 PM (d) 6 : 48 PM Tamil.
69. There are 5 tasks and 5 persons. Task-1 cannot be 76. In a plane, line X is perpendicular to line Y and
assigned to either person-1 or person-2. Task-2 parallel to line Z; line U is perpendicular to both
must be assigned to either person-3 or person-4. lines V and W; line X is perpendicular to line V.
Every person is to be assigned one task. In how Which one of the following statements is correct?
many ways can the assignment be done? (a) Z, U and W are parallel.
(a) 6 (b) 12 (b) X, V and Y are parallel.
(c) 24 (d) 144 (c) Z, V and U are all perpendicular to W.
70. The monthly incomes of Peter and Paul are in the (d) Y, V and W are parallel.
ratio of 4 : 3. Their expenses are in the ratio of 77. A cow costs more than 4 goats but less than 5
3 : 2. If each saves ` 6,000 at the end of the month, goats. If a goat costs between ` 600 and
their monthly incomes respectively are (in `) ` 800, which of the following is a most valid
(a) 24,000 and 18,000
@UPSC_THOUGHTS

conclusion?
(b) 28,000 and 21,000 (a) A cow costs more than ` 2,500.
(c) 32,000 and 24,000 (b) A cow costs less than ` 3,600.
(d) 34,000 and 26,000 (c) A cow costs between ` 2,600 and ` 3,800.
71. Two cities A and B are 360 km apart. A car goes (d) A cow costs between ` 2,400 and ` 4,000.
from A to B with a speed of 40 km/hr and returns 78. A society consists of only two types of people—
to A with a speed of 60 km/hr. What is the fighters and cowards. Two cowards are always
average speed of the car? friends. A fighter and a coward are always enemies.
(a) 45 km/hr (b) 48 km/hr
Fighters are indifferent to one another. If A and
(c) 50 km/hr (d) 55 km/hr
B are enemies, C and D are friends, E and F are
Directions for the following 2 (two) items: indifferent to each other, A and E are not enemies,
Read the following passage and answer the 2 (two) items that while B and F are enemies.
follow: Which of the following statements is correct?
(a) B, C and F are cowards.
A, B, C, D, E and F are cousins. No two cousins (b) A, E and F are fighters.
are of the same age, but all have birthdays on the same (c) B and E are in the same category.
day of the same month. The youngest is 17 years old (d) A and F are in different categories.
and the oldest E is 22 years old. F is somewhere between 79. In a box of marbles, there are three less white
B and D in age. A is older than B. C is older than D. marbles than the red ones and five more white
A is one year older than C.
marbles than the green ones. If there are a total
72. Which one of the following is possible? of 10 white marbles, how many marbles are there
(a) D is 20 years old in the box?
(b) F is 18 years old (a) 26 (b) 28
(c) F is 19 years old (c) 32 (d) 36
(d) F is 20 years old
80. Candidates in a competitive examination consisted
73. What is the number of logically possible orders of 60% men and 40% women. 70% men and 75%
of all six cousins in terms of increasing age? women cleared the qualifying test and entered the
(a) 1 (b) 2 final test where 80% men and 70% women were
(c) 3 (d) 4 successful.
74. In a society it is customary for friends of the same Which of the following statements is correct?
sex to hug and for friends of opposite sex to shake (a) Success rate is higher for women.
hands when they meet. A group of friends met (b) Overall success rate is below 50 per cent.
in a party and there were 24 handshakes. (c) More men cleared the examination than
Which one among the following numbers indicates women.
the possible number of hugs? (d) Both (a) and (b) above are correct.

ST . 98
Sample Test 5 : General Studies Paper II (2015)

PRACTICE ANSWER SHEET

Directions: Use black ball pen. Example

1. 17. 33. 49. 65.

2. 18. 34. 50. 66.

3. 19. 35. 51. 67.

4. 20. 36. 52. 68.

5. 21. 37. 53. 69.

6. 22. 38. 54. 70.


@UPSC_THOUGHTS

7. 23. 39. 55. 71.

8. 24. 40. 56. 72.

9. 25. 41. 57. 73.

10. 26. 42. 58. 74.

11. 27. 43. 59. 75.

12. 28. 44. 60. 76.

13. 29. 45. 61. 77.

14. 30. 46. 62. 78.

15. 31. 47. 63. 79.

16. 32. 48. 64. 80.

ST . 99
Sample Test 5 : General Studies Paper II (2015)

ANSWERS AND EXPLANATORY NOTES


1. (a) The other three answer responses are supported 7 X 21 31 43
by what the passage says.
2. (d) 6 8 10 12
3. (b) Response (b) is the assumption going by the 2 2 2
last line of the passage. Nothing in the passage Try out 8 (which is 10 less 2) to be the difference
supports (c) or (d). As for (a), the author does between 21 and X: 21 – 8 = 13; X = 13. Now
not agree with this. 13 less 6 is 7 which is the first number.
[Alternatively, work with each option in the place
4. (c) of X. None will give you uniform progression or
5. (c) The sentence, ‘This ..... packaging’ implies both some pattern in the difference between the
the given statements. numbers except 13.]
6. (a) The other three responses are clearly implied 14. (d) The given figure is composed of shapes each
or stated in the passage. of which is right angled. Only (d) is composed
7. (a) of such shapes.
8. (d) Refer to sentence, ‘Proneness ..... surmount it’. 15. (b) Let each question be of 10 marks. Total marks
9. (d) The host is decided by the picking of lots. As will be 100.
there is no condition on how many times a If a candidate obtains 50 per cent marks for each
person can be chosen, we cannot came to a of the 8 questions, he gets 5 marks for each.
conclusion on how many times a member has So he gets a total of 8 × 5 = 40 marks out of
to host a dinner in a year. 100, i.e., 40 per cent (which is also given)
[Don’t make the mistake of marking (a) in a 100
∴ Number of questions is = 10
@UPSC_THOUGHTS

hurry, assuming that one member each will be 10


the host every month.] [Alternatively: If the student had scored 100 per
cent in the questions attempted, he would have
10. (a) The automobile owner clearly wants to maintain got 80 per cent marks. In that case if 8 questions
the same expenditure on petrol as is evident are equal to 80 marks, each question is for 10
from the price-consumption relationship given: marks and the total number of questions would
40 × 60 = Rs 2400 be 10.]
50 × 48 = Rs 2400
16. (d) Let the son’s age be x, then by the given
60 × 40 = Rs 2400
conditions,
75 × 32 = Rs 2400
9x + 8x = 51
Now if the price of petrol goes up to Rs 80 per
17x = 51
litre, and expenditure is maintained at Rs 2400,
51
2400 x = = 3
the consumption will be = 30 litres. 17
80 The son would be 3 years of age.
11. (a) Take the figure of the small triangle and see
how it moves. It moves clockwise from corner 17. (c) You have to choose the response that is not
to corner getting inverted at each move. So the correct. As A is the tallest and both women are
next position of the triangle will be top left with shorter than their respective husbands, A must
apex at top. Now, both answer responses (a) be a man. D is the other man, being the ‘brother’
and (b) fit that condition. So take the dark dot of B. Now B and C are the women. As D is
next; it moves anticlockwise and its next position B’s brother, A must be B’s husband, and D is
will be at the bottom. Answer response (a) is C’s husband. We are told that both women are
correct. shorter than their respective husbands, so C
12. (a) At first sight, there seems to be no uniform must be shorter than D. Answer response (c)
relationship or pattern of progression among the is therefore not correct.
figures. However, taking each column, we note [As A is married to B who is D’s sister, and D
that the lower number is a multiple of the is married to C, all four are related either by
number at the top in each case. Among the blood or through marriage. So answer response
given answer responses, only (a) fits: 20 is a (a) is correct. C is the taller woman of C and
multiple of 5. B, and since C is shorter than A as well as D,
B must be the shortest among the four. Answer
13. (c) Work out how the differences between the response (b) is correct. Answer response (d) is
numbers progress. obviously correct.]

ST . 100
Sample Test 5 : General Studies Paper II (2015)

18. (b) We cannot be categorical about only the wife Percentage of people reading magazines
being left at home, so (a) is not acceptable. Nor = 20 + 20 + 5 + 10 + 15 + 10 = 80
can (c) or (d) be concluded from the given Percentage of people not reading any magazine
statements.
= 100 – 80 = 20.
19. (a) Radha considers it ‘impossible’ for Naresh to
have become a better boxer after meditation, 32. (d) From statements 1 and 2 we can deduce that
because she thinks aggressiveness is important cauliflower > (tastier) lady’s finger > cabbage
in a boxer. So meditation and aggressiveness From statement 3 we deduce that peas are
do not go together. tastier or as tasty as cabbage.
20. (b) Answer response (b) is just a negative form of (Note that ‘cabbage is not tastier than peas’
the conclusion in the question item, ‘therefore could imply cabbage is as tasty as peas or less
all athletes ..... bad athletes’. Here a universal tasty than peas.)
affirmative proposition has been turned into a So we have cauliflower > lady’s finger > cabbage
universal negative proposition which is obversion ≤ peas
and valid. The only definite conclusion is that cauliflower
We do not know about bad athletes wanting to
is tastier than cabbage.
or not wanting to win; so (a) is not correct.
If ‘All S is P’ we cannot conclude ‘All P is S’, [Cabbage and peas could be equally ‘least
so (c) is not correct. The same is true for (d). tasty’ of the lot, so option (c) cannot be correct.]

21. (a) The first sentence suggests that (a) is the correct 33. (b) The diagram explains:
answer response.
Second right for Second left for
22. (d) Refer to the sentence, ‘Manak’s mother is Rohit Shahid
desperate to ..... family name.’ It clearly indicates
1 km 1 km
@UPSC_THOUGHTS

the patriarchal mindset out of which the rest of


the story develops. 1 km 1 km
23. (b)
1 km 1 km
24. (b) This is the assumption.
First right turn Starting First left turn
25. (c) for Rohit Point for Shahid
26. (d)
After Shahid travels 3 km and Rohit travels 3
27. (c) Statement I is not to be found in the passage. km they would meet.
As for statement III, the passage talks of the
importance of balanced growth and not on any 34. (a) The distance covered by A to win the race is
exclusive sector. 500 m. In the same time, B would have covered
28. (c) 500 – (45 + 35) = 420 m.
29. (b) A < B; B = C. Clearly C runs faster than A. The ratio of their speeds is
30. (c) A : 100 -> 20 to B; Balance 80 500 : 420 or 25 : 21
B : 100 + 20 (from A) – 10 (to C); balance 110 (Time being the same, ratio of distances covered
C : 100 + 10 (from B) + 30 (from D); balance 140 gives the ratio of speeds.)
D : 100 – 30 (to C); balance 70 1
Now A + D = 80 + 70 = 150 35. (a) In one case, the quantity of milk is ; the
2 3
C has 140. quantity of water is
So answer response (c) is the incorrect one. 3 1
N the other case, the quantity of milk is ; the
31. (c) 3 4
100 quantity of water is .
A = 45% B = 55% 4
Mixed in the pot,
– 20 20 1 1 7
Milk = + =
10 3 4 12
15 5 2 3 17
Water = + =
10 3 4 12
7 17
C = 40% The ratio of milk to water is : or 7 : 17.
12 12

ST . 101
Sample Test 5 : General Studies Paper II (2015)

36. (a) Failed Failed 49. (c) If Math II is offered, Math I must also be opted
English Math for; so the number of ways is 1. Now there are
62 52 four subjects. The possible combinations of 2
subjects is 4C2 = 6.
Total combinations = 6 + 1 = 7.
38 24 28 50. (d) Let the number of brown socks be x and the
price of a pair of brown rocks be y. If the correct
bill had been made for the purchase of 5 black
Number of failed students = 38 + 24 + 28 = 90 socks and x brown socks, it would have
Number of students who passed = 130 – 90 = 40 amounted to
[Alternatively, the number of students who passed 5 × 3y + xy
is or 15y + xy
130 – (62 + 52 – 24) However, as the quantities were exchanged by
= 130 – 90 = 40.] the billing clerk, the bill would be 5y + x × 3y
As this is an increase of 100 per cent of the
37. (c) If each person speaks only one language the actual bill,
total number of persons in the group would be 5y + 3xy = 2 (15y + xy)
6 + 15 + 6 = 27. 5 + 3x = 30 + 2x
However, since 2 people speak two languages 3x – 2x = 30 – 5
each one has been counted twice, and one x = 25
person who can speak three languages has The number of brown socks pairs is 25.
been counted thrice to get the figure 27, thus
there is a total of 1 + 1 + 2 or 4 extra counts. 51. (d)
The actual number of persons in the group 52. (a) From the graph it is obvious that the average
would, therefore, be 27 – 4 = 23. earnings of A is more than the average earnings
@UPSC_THOUGHTS

38. (c) Let the number of cars be x and the number of B.


of two-wheelers be y 53. (b) With both pipes opened, the time taken to fill
By the given conditions, the tank completely is
4x + 2y = 100 + 2 (x + y)
4x + 2y – 2x – 2y = 100 20 × 30 600
= = 12 minutes.
2x = 100 20 + 30 50
100 54. (c)
x = = 50 G
2
The number of cars parked is 50. V
39. (c) B
40. (d)
I Y
41. (b) Just tackling poverty would not be enough; O
climate change and its effects have also to be
tackled. Therefore our food systems have to be
re-imagined so that even the poor can have 55. (d) From the given statements, we cannot conclude
sufficient to eat and adopt more sustainable use that wisdom and greatness are mutually
of resources. exclusive. What if all wise men are also great?
42. (d)
great
43. (b)
wise
44. (b)
45. (b)
46. (d)
men
47. (b)
48. (d) The selection of principal can be made in two
ways. some wise men
After the principal has been selected, 5 who are also great
candidates are left from whom 2 vice principals So conclusion I is not valid.
may be selected in 5C2 or 10 ways.
From a particular affirmative statement ‘Some S
Total possible combinations = 2 × 10 = 20.
is P’ we cannot conclude a negative ‘Some S

ST . 102
Sample Test 5 : General Studies Paper II (2015)

is not P’. We do not know this for certainty. So the 30-minute and 35-minute positions; at 48-
conclusion II is not valid. minute position, the minute hand would be
much more than 3 minutes ahead of the hour
56. (d)
hand. So option (d) is not correct.
57. (c) To get consecutively repeated three digits in the
69. (c)
answer, of six digits, a three digit number must
be multiplied by 1001. So DEED must be 1001. 70. (a) Let the monthly income of Peter and Paul be
Even if you don’t know this rule, you could try 4x and 3x respectively.
out the options with any three-digit number. Expenses of Peter = 4x – 6000
Take ABC to be 123. The required answer must Expenses of Paul = 3x – 6000
be 123123. By the given conditions
(a) DEED = 2002; 123 × 2002 you get a 4x − 6000 3
=
product whose last digit is 6 which is not 3x − 6000 2
the same as the last digit of 123. So it is 8x – 12000 = 9x – 18000
an incorrect option. x = 18000 – 12000 = Rs 6000
(b) DEED = 0110; 123 × 0110 last digit will Peter’s income is 4 × 6000 = Rs 24000
be 0. Option is wrong. Paul’s income is 3 × 6000 = Rs 18000
(c) DEED = 1001; 123 × 1001 123123
Option is correct.
Distance
71. (b) Time =
Speed
58. (c) Here total time taken to travel from A to B and
59. (c) Since all the options relate to what is spent on B to A
food, we may calculate for that item alone. 360 360
= + = 9 + 6 = 15 hours
Family A spends on food 50% of Rs 20,000 40 60
@UPSC_THOUGHTS

= Rs 10,000 Total distance is 720 km


Family B spends on food 10% of Rs 1,00,000 720
Average speed = = 48 km/hour.
= Rs 10,000 15
Both families spend the same amount on food.
For Questions 72 and 73, we need to work out the order
60. (b) It is evident that Priti is the slowest of the cousins in age.
Priti < Swati < Kamala < Usha We are given that E is the oldest at 22 years.
61. (d) If the youngest is 17 years (given), there is a
difference of one year between one cousin and
62. (d) Option (c) is not correct as the passage does the next. We are told A is one year older than
not speak of negative aspects of the Right to C and also older than B. So we can conclude
Property. C is older than B. In that case C cannot be the
63. (a) youngest. Now A, E and C cannot be the
64. (a) Statement (2) is not mentioned at all. As for youngest. Either B or D must be the youngest.
statement (3) it is mentioned only in the context (F cannot be the youngest as F is between B
of financial support required. and D.) There are thus two possibilities.
I : With D at 17 years
65. (c)
D (17) < F (18) < B (19) < C (20) < A (21) < E (22)
66. (a) There is no mention of multilateral trade in the II : With B at 17 years
passage. B (17) < F (18) < D (19) < C (20) < A (21) < E (22)
67. (a)
72. (b) F is 18 years old in either case.
68. (c) Just visualise the face of a clock. At 6 : 15 PM,
the hour hand would be a little ahead of 6; the 73. (b) As shown, there are two logical orders possible.
minute hand at 15 minutes would not be ahead 74. (c) If there are 24 handshakes, the break-up of
of the hour hand. The same would apply in case opposite sexes in the group would be the factors
of 6 : 18 PM. So options (a) and (b) are incorrect. of 24. So the possible sets of men and women
In case of 6 : 36 PM, the hour hand would be would be
between 6 and 7 at the 33-minute position and 24, 1 or 12, 2 or 8, 3 or 6, 4
the minute hand at 36 minutes—3 minutes If x and y denote the opposite sexes, the number
ahead. So option (c) is correct. If option (d) is of hugs would be
to be checked, at 6 : 48 PM, the hour hand x
C2 + yC2
would still be between 6 and 7, i.e., between

ST . 103
Sample Test 5 : General Studies Paper II (2015)

So, for each set: The most valid conclusion is that the cow costs
24
C2 = 276 (1 woman/man has no one to hug) between Rs 2,400 and Rs 4,000.
12
C2 + 2C2 = 67
8 78. (b) If A and B are enemies, one must be a fighter
C2 + 3C2 = 31
6 and the other a coward.
C2 + 4C2 = 21
C and D are friends, so both are cowards.
Only 21 is among the given options. So (c) is
E and F are indifferent to each other, so both
the correct answer.
are fighters.
75. (d) Make a chart. Fill in the known details. If A is not an enemy of E, A cannot be a coward;
A must be a fighter.
Anil David Shabnam Rekha
Now B and F are enemies and F, we know, is
Man man woman woman a fighter, so B is a coward.
Tamil Marathi Checking the options, only (b) is correct.
Can drive can drive
79. (b) Number of white marbles = 10
See if any of the options fit in with the available Number of red marbles = 10 + 3 = 13
details. Number of green marbles = 10 – 5 = 5
Option (a) is obviously incorrect as Anil can’t Total number of marbles = 10 + 13 + 5 = 28
drive.
Option (b) is obviously incorrect as Shabnam 80. (c) Let there be 100x candidates.
can’t drive. So there are 60x men and 40x women.
Option (c) is again incorrect as, by the given Men who cleared the qualifying test
conditions, only one of the two capable of = 70% of 60x = 42x
driving—David and Rekha—can speak Marathi Women who cleared the qualifying test
(as Shabnam already speaks Marathi). = 75% of 40x = 30x
Option (d) is correct, as either Rekha or David In the final test
@UPSC_THOUGHTS

must speak Tamil under the given conditions


Men who were successful
and both can drive.
= 80% of 42x = 33.6x
76. (d) Make a diagram. It will be clear that option (d) Women who were successful
is correct. = 70% of 30x = 21x
Now check out each option.
Y V W 21x
(a) is incorrect as is 0.525 and
40x
33.6x
U is 0.56
60x
Z (b) is incorrect as both men and women have a
21
success rate of more than 50 per cent; and
X 40
33.6
are both more than 50 per cent.
60
77. (d) The minimum price for 4 goats is
4 × 600 = Rs 2,400 (c) is correct. (This is clear from the figures of
The maximum price for 5 goats is 33.6x and 21x.)
5 × 800 = Rs 4,000 (d) is obviously incorrect.

ST . 104
Sample Test 6 : General Studies Paper II (2016)

SAMPLE TEST 6: GENERAL STUDIES PAPER II (2016)


Directions for the following 6 (six) items: Read the 2. Lack of sufficient professional training to the
following two passages and answer the items that follow each officers of All India Services
passage. Your answers to these items should be based on the 3. Lack of proper service benefits in civil services
passages only. 4. Lack of Constitutional provisions to define
the respective roles of professional civil
Passage-1 services vis-a-vis political executive in this
context
Accountability, or the lack of it, in governance
generally, and civil services, in particular, is a major Select the correct answer using the code given
factor underlying the deficiencies in governance and below:
public administration. Designing an effective framework (a) 1 only (b) 2 and 3 only
for accountability has been a key element of the reform (c) 1 and 4 only (d) 2, 3 and 4
agenda. A fundamental issue is whether civil services 2. With reference to the passage, the following
should be accountable to the political executive of the assumptions have been made:
day or to society at large. In other words, how should 1. Political executive is an obstacle to the
internal and external accountability be reconciled? Internal
accountability of the civil services to the society
accountability is sought to be achieved by internal
performance monitoring, official supervision by bodies 2. In the present framework of Indian polity, the
like the Central Vigilance Commission and Comptroller political executive is no longer accountable to the
and Auditor General, and judicial review of executive society
decisions. Articles 311 and 312 of the Indian Constitution Which of these assumptions is/are valid?
provide job security and safeguards to the civil services, (a) 1 only (b) 2 only
especially the All India Services. The framers of the (c) Both 1 and 2 (d) Neither 1 nor 2
@UPSC_THOUGHTS

Constitution had envisaged that provision of these 3. Which one of the following is the essential message
safeguards would result in a civil service that is not implied by this passage?
totally subservient to the political executive but will (a) Civil services are not accountable to the
have the strength to function in larger public interest. society they are serving
The need to balance internal and external accountability (b) Educated and enlightened persons are not
is thus built into the Constitution. The issue is where taking up political leadership
to draw the line. Over the years, the emphasis seems (c) The framers of the Constitution did not
to have tilted in favour of greater internal accountability envisage the problems being encountered by
of the civil services to the political leaders of the day the civil services
who in turn are expected to be externally accountable (d) There is a need and scope for reforms to
to the society at large through the election process. This improve the accountability of civil services
system for seeking accountability to society has not
worked out, and has led to several adverse consequences 4. According to the passage, which one of the
for governance. following is not a means of enhancing internal
Some special measures can be considered for accountability of civil services?
improving accountability in civil services. Provisions of (a) Better job security and safeguards
articles 311 and 312 should be reviewed and laws and (b) Supervision by Central Vigilance Commission
regulations framed to ensure external accountability of (c) Judicial review of executive decisions
civil services. The proposed Civil Services Bill seeks to (d) Seeking accountability through enhanced
address some of these requirements. The respective participation by people in decision making
roles of professional civil services and the political process
executive should be defined so that professional Passage-2
managerial functions and management of civil services
are depoliticised. For this purpose, effective statutory In general, religious traditions stress our duty to
civil service boards should be created at the centre and god, or to some universal ethical principle. Our duties
in the states. Decentralisation and devolution of authority to one another derive from these. The religious concept
to bring government and decision making closer to the of rights is primarily derived from our relationship to
people also helps to enhance accountability. this divinity or principle and the implication it has on
1. According to the passage, which of the following our other relationships. This correspondence between
factor/factors led to the adverse consequences for rights and duties is critical to any further understanding
governance/public administration? of justice. But, for justice to be practiced; rights and
1. Inability of civil services to strike a balance duties cannot remain formal abstractions. They must be
between internal and external accountabilities grounded in a community (common unity) bound

ST . 105
Sample Test 6 : General Studies Paper II (2016)

together by a sense of common union (communion). (a) 3 km east, then 2 km south


Even as a personal virtue, this solidarity is essential to (b) 3 km east, then 1 km north
the practice and understanding of justice. (c) 1 km north, then 2 km west
5. With reference to the passage, the following (d) 3 km south, then 1 km north
assumptions have been made: 10. Consider the following statement:
1. Human relationships are derived from their “We shall go either for a picnic or for trekking”.
religious traditions Which of the following, if true, would falsify this
2. Human beings can be duty bound only if claim?
they believe in god (a) We go for a picnic but not for trekking
3. Religious traditions are essential to practice (b) Activities such as picnic and trekking are
and understand justice encouraged by the health authorities
Which of these assumption(s) is/are valid? (c) We go for trekking and not for picnic
(a) 1 only (b) 2 and 3 only (d) We do not go either for picnic or for trekking
(c) 1 and 3 only (d) 1, 2 and 3 11. There were 50 faculty members comprising 30
6. Which one of the following is the crux of this males and the rest females. No male faculty
passage? member knew music, but many of the female
(a) Our duties to one another derive from our faculty members did. The Head of the institution
religious traditions invited six faculty members to a tea party by draw
(b) Having relationship to the divine principle is of lots. At the party it was discovered that no
a great virtue member knew music. The conclusion is that:
(c) Balance between and duties is crucial to the (a) the party comprised male faculty members
delivery of justice in a society only
(d) Religious concept of rights is primarily (b) the party comprised only those female faculty
derived from our relationship to god members who could not give renderings in
music
7. A ate grapes and pineapple; B ate grapes and (c) the party comprised both male and female
oranges; C ate oranges, pineapple and apple; D
@UPSC_THOUGHTS

faculty members
ate grapes, apple and pineapple. After taking (d) nothing can be said about the gender
fruits, B and C fell sick. In the light of the above composition of the party
facts, it can be said that the cause of sickness was:
(a) Apple (b) Pineapple 12. Five people A, B, C, D and E are seated about
(c) Grapes (d) Oranges a round table. Every chair is spaced equidistant
from adjacent chairs.
8. Consider the following statements. (i) C is seated next to A
1. The rate of population growth is increasing (ii) A is seated two seats from D
in the country (iii) B is not seated next to A
2. The death rate is declining faster in the
country compared to birth rate On the basis of above information, which of the
3. The birth rate is declining faster in the country following must be true?
compared to death rate 1. D is seated next to B
4. Rural-urban migration is taking place 2. E is seated next to A
regularly in the country 3. D and C are separated by two seats
Which one of the following conclusions may be Select the correct answer using the code given
true in the light of the above facts? below:
(a) The rate of population growth is increasing (a) 1 only
due to rural-urban migration (b) 1 and 2 only
(b) The rate of population growth is increasing (c) 3 only
due to decline in death rate only (d) Neither 1 nor 2 nor 3
(c) The rate of population growth is increasing 13. There are five hobby clubs in a college—
due to increase in birth rate only photography, yachting, chess, electronics and
(d) The rate of population growth is increasing gardening. The gardening group meets every
due to faster decline in death rate than in second day, the electronics group meets every
birth rate third day, the chess group meets every fourth day,
the yachting group meets every fifth day and the
9. A person X was driving in a place where all roads
photography group meets every sixth day. How
ran either north-south or east-west, forming a grid.
many times do all the five groups meet on the
Roads are at a distance of 1 km from each other
same day within 180 days?
in a parallel. He started at the intersection of two
(a) 5 (b) 18
roads, drove 3 km north, 3 km west and 4 km
(c) 10 (d) 3
south. Which further route could bring him back
to his starting point, if the same route is not 14. There are some nectar-filled flowers on a tree and
repeated? some bees are hovering on it. If one bee lands on

ST . 106
Sample Test 6 : General Studies Paper II (2016)

each flower, one bee will be left out. If two bees primary energy in 2050. However the maximum
land on each flower, one flower will be left out. sustainable technical potential of biomass resources
The number of flowers and bees respectively are: (both residues and energy crops) without disruption of
(a) 2 and 4 (b) 3 and 2 food and forest resources ranges from 80 – 170 exajoules
(c) 3 and 4 (d) 4 and 3 a year by 2050, and only part of this is realistically and
economically feasible. In addition, some climate models
Directions for the following 5 (five) items: Consider the rely on biomass-based carbon capture and storage, an
following information and answer the five items that unproven technology, to achieve negative emissions and
follow: to buy some time during the first half of the century.
There are five persons in a group—P, Q, R, S and Some liquid biofuels such as corn-based ethanol,
T. The group has one doctor, one lawyer and one artist. mainly for transport, may aggravate rather than
P and S are unmarried students. T is a man married ameliorate carbon emissions on a lifecycle basis. Second
to one of the group members. Q is the brother of P and generation biofuels, based on ligno-cellulosic feedstocks
is neither doctor nor artist. R is not doctor. – such as straw, bagasse, grass and wood – hold the
15. Who is the doctor? promise of sustainable production that is high-yielding
(a) T (b) P and emit low levels of greenhouse gases, but these are
(c) Q (d) R still in the R&D stage.
16. Who is the artist? 21. What is/are the present constraint/constraints in
(a) P (b) Q using biomass as fuel for power generation?
(c) R (d) T 1. Lack of sustainable supply of biomass
2. Biomass production competes with food
17. Who is the spouse of R? production
(a) P (b) T 3. Bio-energy may not always be low carbon on
(c) Q (d) S a life-cycle basis
18. Who is the lawyer? Select the correct answer using the code given
(a) P (b) Q below:
(c) R (d) S
@UPSC_THOUGHTS

(a) 1 and 2 only (b) 3 only


19. Who of the following is definitely a man? (c) 2 and 3 only (d) 1, 2 and 3
(a) P (b) S 22. Which of the following can lead to food security
(c) Q (d) None of the above problem?
20. There is an order of 19000 quantity of a particular 1. Using agricultural and forest residues as
product from a customer. The firm produces 1000 feedstock for power generation
quantity of that product per out of which 5 per 2. Using biomass for carbon capture and storage
cent are unfit for sale. In how many days will the 3. Promoting the cultivation of energy crops
order be completed? Select the correct answer using the code given
(a) 18 (b) 19 below:
(c) 20 (d) 22 (a) 1 and 2 only (b) 3 only
Directions for the following 5 (five) items: Read the (c) 2 and 3 only (d) 1, 2 and 3
following two passages and answer the items that follow
each passage. Your answers to these items should be 23. In the context of using biomass, which of the
based on the passages only. following is/are the characteristic/characteristics
of the sustainable production of biofuel?
Passage-1 1. Biomass as a fuel for power generation could
meet all the primary energy requirements of
Biomass as fuel for power, heat, and transport has the the world by 2050
highest mitigation potential of all renewable sources. It 2. Biomass as a fuel for power generation does
comes from agriculture and forest residues as well as not necessarily disrupt food and forest
from energy crops. The biggest challenge in using resources
biomass residues is a long-term reliable supply delivered 3. Biomass as a fuel for power generation could
to the power plant at reasonable costs; the key problems help in achieving negative emissions, given
are logistical constraints and the costs of fuel collection. certain nascent technologies
Energy crops, if not managed properly, compete with
Select the correct answer using the code given
food production and may have undesirable impacts on
below:
food prices. Biomass production is also sensitive to the
(a) 1 and 2 only (b) 3 only
physical impacts of a changing climate.
(c) 2 and 3 only (d) 1, 2 and 3
Projections of the future role of biomass are probably
overestimated, given the limits to the sustainable biomass 24. With reference to the passage, following
supply, unless breakthrough technologies substantially assumptions have been made:
increase productivity. Climate-energy models project 1. Some climate-energy models suggest that the
that biomass use could increase nearly four-fold to use of biomass as a fuel for power generation
around 150 – 200 exajoules, almost a quarter of world helps in mitigating greenhouse gas emissions

ST . 107
Sample Test 6 : General Studies Paper II (2016)

2. It is not possible to use biomass as a fuel for 28. 30g of sugar was mixed in 180 ml water in a vessel
power generation without disrupting food A, 40 g of sugar was mixed in 280 ml of water
and forest resources in vessel B and 20 g of sugar was mixed in 100
Which of these assumptions is/are valid? ml of water in vessel C. The solution in vessel
(a) 1 only (b) 2 only B is
(c) Both 1 and 2 (d) Neither 1 nor 2 (a) sweeter than that in C
(b) sweeter than that in A
Passage-2 (c) as sweet as that in C
We are witnessing a dangerous dwindling of biodiversity (d) less sweet than that in C
in our food supply. The green revolution is a mixed 29. In aid of charity, every student in a class contributes
blessing. Over time farmers have come to rely heavily as many rupees as the number of students in that
on broadly adapted, high yield crops to the exclusion class. With the additional contribution of Rs 2 by
of varieties adapted to the local conditions. one student only, the total collection is Rs 443.
Monocropping vast fields with the same genetically Then how many students are there in the class?
uniform seeds helps boost yield and meet immediate (a) 12 (b) 21
hunger needs. Yet high-yield varieties are also genetically (c) 43 (d) 45
weaker crops that require expensive chemical fertilisers
and toxic pesticides. In our focus on increasing the 30. Anita’s mathematics test had 70 problems carrying
amount of food we produce today, we have accidentally equal marks i.e., 10 arithmetic, 30 algebra and 30
put ourselves at risk for food shortages in future. geometry. Although she answered 70 per cent of
the arithmetic, 40 per cent of the algebra and 60
25. Which among the following is the most logical
and critical inference that can be made from the per cent of the geometry problems correctly, she
above passage? did not pass the test because she got less than
(a) In our agricultural practices, we have become 60 per cent marks. The number of more questions
heavily dependent on expensive chemical she would have to answer correctly to earn a 60
fertilisers and toxic pesticides only due to per cent passing marks is:
(a) 1 (b) 5
@UPSC_THOUGHTS

green revolution
(b) Monocropping vast fields with high-yield (c) 7 (d) 9
varieties is possible due to green revolution 31. In a class, there are 18 very tall boys. If these
(c) Monocropping with high-yield varieties is constitute three-fourths of the boys and the total
the only way to ensure food security to number of boys is two-thirds of the total number
millions of students in the class, what is the number of girls
(d) Green revolution can pose a threat to in the class?
biodiversity in food supply and food security (a) 6 (b) 12
in the long run (c) 18 (d) 21
26. A class starts at 11:00 am and lasts till 2:27 pm. 32. Consider the following statements:
Four periods of equal duration are held during 1. Either A and B are of the same age or A is
this interval. After every period, a rest of 5 older than B
minutes is given to the students. The exact duration
2. Either C and D are of the same age or D is
of each period is:
older than C
(a) 48 minutes (b) 50 minutes
3. B is older than C
(c) 51 minutes (d) 53 minutes
Which of the following conclusions can be drawn
27. Four friends A, B, C and D need to cross a bridge.
A maximum of two persons can cross it at a time. from the above statements?
It is night and they just have one lamp. Persons (a) A is older than B
that cross the bridge must carry the lamp to find (b) B and D are of the same age
the way. A pair must walk together at the speed (c) D is older than C
of slower person. After crossing the bridge, the (d) A is older than C
person having faster speed in the pair will return 33. The monthly average salary paid to all the
with the lamp each time to accompany another employees of a company was Rs 5000. The monthly
person in the group. Finally, the lamp has to be average salary paid to male and female employees
returned at the original place and the person who was Rs 5200 and Rs 4200 respectively. Then the
returns the lamp has to cross the bridge again percentage of males employed in the company is
without lamp. To cross the bridge, the time taken (a) 75 per cent (b) 80 per cent
by them is as follows : A: 1 minute, B: 2 minutes, (c) 85 per cent (d) 90 per cent
C: 7 minutes and D: 10 minutes. What is the total
minimum time required by all the friends to cross Direction for the following 3 (three) items: Consider the
the bridge? given formation and answer the three items that follow.
(a) 23 minutes (b) 22 minutes Six boxes A, B, C, D, E and F have been painted
(c) 21 minutes (d) 20 minutes with six different colours viz., violet, indigo, blue, green,

ST . 108
Sample Test 6 : General Studies Paper II (2016)

yellow and orange and arranged from left to right (not follows each passage. Your answers to these items
necessarily either kept or painted with the colours in should be based on the passages only.
the same order). Each box contains a ball of any one
of the following six games: cricket, hockey, tennis, golf,
Passage-1
football and volleyball (not necessarily in the same By killing transparency and competition, crony capitalism
order). The golf ball is in violet box and is not in the is harmful to free enterprise, opportunity and economic
box D. The box A which contains tennis ball is orange growth. Crony capitalism, where rich and the influential
in colour and is at the extreme right. The hockey ball are alleged to have received land and natural resources
is neither in box D nor in box E. The box C having cricket and various licences in return for payoffs to venal
ball is painted green. The hockey ball is neither in the politicians, is now a major issue to be tackled. One of
box painted blue nor in the box painted yellow. The the greatest dangers to growth of developing economies
box C is fifth from right and next to box B. The box like India is the middle-income trap where crony
B contains volleyball. The box containing the hockey ball capitalism creates oligarchies that slow down the growth.
is between the boxes containing golf ball and volleyball. 41. Which among the following is the most logical
34. Which one of the following boxes contains the golf corollary to the above passage?
ball? (a) Launching more welfare schemes and
allocating more finances for the current
(a) F (b) E
schemes are urgently needed
(c) D (d) None of the above
(b) Efforts should be made to push up economic
35. Which of the following statements is/are correct? growth by other means and provide licences
(a) D is painted yellow to the poor
(b) F is painted indigo (c) Greater transparency in the functioning of the
(c) B is painted blue government and promoting the financial
(d) All of the above inclusion are needed at present
(d) We should concentrate more on developing
36. The football is in the box of which colour?
manufacturing sector than service sector
(a) Yellow
@UPSC_THOUGHTS

(b) Indigo Passage-2


(c) Cannot be determined as data are inadequate Climate adaptation may be rendered ineffective if policies
(d) Blue are not designed in the context of other development
37. Two numbers X and Y are respectively 20 per cent concerns. For instance, a comprehensive strategy that
and 28 per cent less than a third number Z. By seeks to improve food security in the context of climate
what percentage is the number Y less than the change may include a set of coordinated measures
number X? related to agricultural extension, crop diversification,
(a) 12 per cent (b) 10 per cent integrated water and pest management and agricultural
(c) 9 per cent (d) 8 per cent information series. Some of these measures may have
to do with climate changes and others with economic
38. A daily train is to be introduced between station development.
A and station B starting from each at 6 AM and
the journey is to be completed in 42 hours. What 42. What is the most logical and rational inference
is the number of trains needed in order to that can be made from the above passage?
(a) It is difficult to pursue climate adaptation in
maintain the Shuttle Service?
the developing countries
(a) 2 (b) 3
(b) Improving food security is a far more complex
(c) 4 (d) 7 issue than climate adaptation
39. A piece of tin is in the form of a rectangle having (c) Every developmental activity is directly or
length 12 cm and width 8 cm. This is used to indirectly linked to climate adaptation
construct a closed cube. The side of the cube is: (d) Climate adaptation should be examined in
(a) 2 cm (b) 3 cm tandem with other economic development
(c) 4 cm (d) 6 cm options
40. In a question paper there are five questions to be Passage-3
attempted and answer to each question has two Understanding of the role of biodiversity in the
choices – True (T) or False (F). It is given that no hydrological cycle enables better policymaking. The
two candidates have given the answers to the five term biodiversity refers to the variety of plants, animals,
questions in an identical sequence. For this to microorganisms, and the ecosystems in which they
happen the maximum number of candidates is: occur. Water and biodiversity are interdependent. In
(a) 10 (b) 18 reality, the hydrological cycle decides how biodiversity
(c) 26 (d) 32 functions. In turn, vegetation and soil drive the movement
of water. Every glass of water we drink has, at least in
Directions for the following 8 (eight) items: Read the part, passed through fish, trees, bacteria, soil and other
following eight passages and answer the item that organisms. Passing through these ecosystems, it is

ST . 109
Sample Test 6 : General Studies Paper II (2016)

cleansed and made fit for consumption. The supply of one man is capable, without the aid of society, of
water is a critical service that the environment provides. supplying his own wants; and those wants, acting upon
43. Which among the following is the most critical every individual, impel the whole of them into society.
inference that can be made from the above passage? 46. Which among the following is the most logical
(a) Biodiversity sustains the ability of nature to and rational inference that can be made from the
recycle water above passage?
(b) We cannot get potable water without the (a) Nature has created a great diversity in human
existence of living organisms society
(c) Plants, animals and microorganisms (b) Any given human society is always short of
continuously interact among themselves its wants
(d) Living organisms could not have come into (c) Social life is a specific characteristic of man
existence without hydrological cycle (d) Diverse natural wants forced man towards
social system
Passage-4
Passage-7
In the last decade, the banking sector has been
restructured with a high degree of automation and The nature of the legal imperatives in any given state
products that mainly serve middle-class and upper corresponds to the effective demands that state
middle-class society. Today there is a need for a new encounters, and that these, in their turn, depend, in a
agenda for the banking and non-banking financial services general way, upon the manner in which economic power
that does not exclude the common man is distributed in the society which the state controls.
44. Which one of the following is the message that 47. The statement refers to:
is essentially implied in the above passage? (a) the antithesis of Politics and Economics
(a) Need for more automation and more products (b) the interrelationship of Politics and Economics
of bank (c) the predominance of Economics over Politics
(b) Need for a radical restructuring of our entire (d) the predominance of Politics over Economics
public finance system Passage-8
@UPSC_THOUGHTS

(c) Need to integrate banking and nonbanking


About 15 percent of global greenhouse gas emissions
institutions
come from agricultural practices. This includes nitrous
(d) Need to promote financial inclusion oxide fertilisers; methane from livestock, rice production,
Passage-5 and manure storage; and carbon dioxide (CO2) from
burning biomass, but this excludes CO2 emissions from
Safe and sustainable sanitation in slums has soil management practices, savannah burning and
immeasurable benefits to women and girls in terms of deforestation. Forestry, land use, and land-use change
their health, safety, privacy and dignity. However, account for another 17 per cent of greenhouse gas
women do not feature in most of the schemes and emissions each year, three quarters of which come from
policies on urban sanitation. The fact that even now the tropical deforestation. The remainder is largely from
manual scavenging exists, ones to show that not enough draining and burning tropical peatland. About the same
has been done to promote pour-flush toilets and amount of carbon is stored in the world’s peatlands as
discontinue the use of dry latrines. A more sustained is stored in the Amazon rainforest.
and rigorous campaign needs to be launched towards
the right to sanitation on a very large scale. This should 48. Which among the following is the most logical
primarily focus on the abolition of manual scavenging. and rational inference that can be made from the
above passage?
45. With reference to the above passage, consider the (a) Organic farming should immediately replace
following statements: mechanised and chemical dependent
1. Urban sanitation problems can be fully solved agricultural practices all over the world
by the abolition of manual scavenging only (b) It is imperative for us to modify our land use
2. There is a need to promote greater awareness practices in order to mitigate climate change.
on safe sanitation practices in urban areas (c) There are no technological solutions to the
Which of the statements given above is/are correct? problem of greenhouse gas emissions
(a) 1 only (d) Tropical areas are the chief sites of carbon
(b) 2 only sequestration
(c) Both I and 2 49. A person climbs a hill in a straight path from point
(d) Neither 1 nor 2 ‘O’ on the ground in the direction of north-east
and reaches a point ‘A’ after travelling a distance
Passage-6 of 5 km. Then, from the point ‘A’ he moves to
To understand the nature and quantity of Government point ‘B’ in the direction of north-west. Let the
proper for man, it is necessary to attend to his character. distance AB be 12 km. Now, how far is the person
As nature created him for social life, she fitted him for away from the starting point ‘O’?
the station she intended. In all cases she made his (a) 7 km (b) 13 km
natural wants greater than his individual powers. No (c) 17 km (d) 11 km

ST . 110
Sample Test 6 : General Studies Paper II (2016)

50. An agricultural field is in the form of a rectangle broken at C which is at a height of 12 meters,
having length X1 meters and breadth X2 meters (X1 broken part is partially attached to the vertical
and X2 are variable). If X1 + X2 = 40 meters, then portion of the trunk at C. If the end of the broken
the area of the agricultural field will not exceed part B touches the ground at D which is at a
which one of the following values? distance of 5 meters from A, then the original
(a) 400 sq m (b) 300 sq m height of the trunk is:
(c) 200 sq m (d) 80 sq m (a) 20 m (b) 25 m
51. The sum of the ages of 5 members comprising a (c) 30 m (d) 35 m
family, 3 years ago was 80 years. The average age 58. A person walks 12 km due north, then 15 km due
of the family today is the same as it was 3 years east, after that 19 km due west and then 15 km
ago, because of an addition of a baby during the due south. How far is he from the starting point?
intervening period. How old is the baby? (a) 5 km (b) 9 km
(a) 6 months (c) 37 km (d) 61 km
(b) 1 year 59. A cube has all its faces painted with different
(c) 2 years colours. It is cut into smaller cubes of equal sizes
(d) 2 years and 6 months such that the side of the small cube is one-fourth
52. The total emoluments of two persons are the the big cube. The number of small cubes with only
same, but one gets allowances to the extent of 65 one of the sides painted is:
per cent of his basic pay and the other gets (a) 32 (b) 24
allowances to the extent of 80 per cent of his basic (c) 16 (d) 8
pay. The ratio of the basic pay of the former to 60. Ram and Shyam work on a job together for four
the basic pay of the latter is: days and complete 60 per cent of it. Ram takes
(a) 16 : 13 (b) 5 : 4 leave then and Shyam works for eight more days
(c) 7 : 5 (d) 12 : 11 to complete the job. How long would Ram take
53. A person is standing on the first step from the to complete the entire job alone?
bottom of a ladder. If he has to climb 4 more steps (a) 6 days (b) 8 days
@UPSC_THOUGHTS

to reach exactly the middle step, how many steps (c) 10 days (d) 11 days
doe he ladder have? 61. A military code writes SYSTEM as SYSMET and
(a) 8 (b) 9 NEARER as AENRER. Using the same code,
(c) 10 (d) 11 FRACTION can be written as:
Direction for the following 3 (three) items: Consider the (a) CARFTION (b) FRACNOIT
given information and answer the three items that (c) NOITCARF (d) CARFNOIT
follow. 62. If R and S are different integers both divisible by
When three friends A, B and C met, it was found 5, then which of the following is not necessarily
that each of them wore an outer garment of a different true?
colour. In random order, the garments are: jacket, (a) R – S is divisible by 5
sweater and tie; and the colours are: blue, white and (b) R + S is divisible by 10
black. Their surnames in random order are: Ribeiro, (c) R × S is divisible by 25
Kumar and Singh. Further, we know that: (d) R2 + S2 is divisible by 5
1. Neither B nor Ribeiro wore a white sweater
2. C wore a tie 63. How many numbers are there between 100 and
3. Singh’s garment was not white 300 which either begin with or end with 2?
4. Kumar does not wear a jacket (a) 110 (b) 111
5. Ribeiro does not like to wear the black colour (c) 112 (d) None of the above
6. Each of the friends wore only one outer Directions for the following 8 (eight) items: Read the
garment of only one colour following five passages and answer the items that follow
54. What is C’s surname? each passage. Your answers to these items should be
(a) Riberio (b) Kumar based on the passages only.
(c) Singh (d) Cannot be determined
Passage-1
55. What is the colour of the tie?
(a) Black (b) Blue As we look to 2050, when we will need to feed two
(c) White (d) Cannot be determined billion more people, the question of which diet is best
has taken on new urgency. The foods we choose to eat
56. Who wore the sweater? in the coming decades will have dramatic ramifications
(a) A (b) B for the planet. Simply put, a diet that revolves around
(c) C (d) Cannot be determined meat and dairy, a way of eating that is on the rise
57. AB is a vertical trunk of a huge tree with A being throughout the developing world, will take a greater toll
the point where the base of the trunk touches the on the world’s resources than one that revolves around
ground. Due to a cyclone, the trunk has been unrefined grains, nuts, fruits and vegetables.

ST . 111
Sample Test 6 : General Studies Paper II (2016)

64. What is the critical message conveyed by the Passage-4


above passage?
(a) Our increasing demand for foods sourced An increase in human-made carbon dioxide in the
from animals puts a greater burden on our atmosphere could initiate a chain reaction between
natural resources plants and microorganisms that would unsettle one of
(b) Diets based on grains, nuts, fruits and the largest carbon reservoirs on the planet—soil. In a
vegetables are best suited for health in study, it was found that the soil, which contains twice
developing countries the amount of carbon present in a plants and Earth’s
(c) Human beings change their food habits from atmosphere combined, could become increasingly volatile
time to time irrespective of the health concerns as people add more carbon dioxide to the atmosphere.
(d) From a global perspective, we still do not This is largely because of increased plant growth.
know which type of diet is best for us Although a greenhouse gas and a pollutant, carbon
dioxide also supports plant growth. As trees and other
Passage-2 vegetation flourish in a carbon dioxide-rich future, their
roots could stimulate microbial activity in soil that may
All humans digest mother’s milk as infants, but until in turn accelerate the decomposition of soil carbon and
cattle began being domesticated 10,000 years ago, children its release into the atmosphere as carbon dioxide.
once weaned no longer needed to digest milk. As a
result, they stopped making the enzyme lactase, which 67. Which among the following is the most logical
breaks down the sugar lactose into simple sugars. After corollary to the above passage?
humans began herding cattle, it became tremendously (a) Carbon dioxide is essential for the survival
advantageous to digest milk, and lactose tolerance of microorganisms and plants
evolved independently among cattle herders in Europe, (b) Humans are solely responsible for the release
the middle East and Africa. Groups not dependant on of carbon dioxide into the atmosphere
cattle, such as the Chinese and Thai, remain lactose (c) Microorganisms and soil carbon are mainly
intolerant. responsible for the increased plant growth
(d) Increasing green cover could trigger the
65. Which among the following is the most logical release of carbon trapped in soil
@UPSC_THOUGHTS

assumption that can be made from the above


passage? Passage-5
(a) About 10,000 years ago, the domestication of
animals took place in some parts of the world Historically, the biggest Challenge to world agriculture
(b) A permanent change in the food habits of a has been to achieve a balance between demand for and
community can bring about a genetic change supply of food. At the level of individual countries, the
in its members demand-supply balance can be a critical policy issue for
(c) Lactose tolerant people only are capable of a closed economy, especially if it is a populous economy
getting simple sugars in their bodies and its domestic agriculture is not growing sufficiently
(d) People who are not lactose tolerant cannot enough to ensure food supplies, on an enduring basis;
digest any dairy product it is not so much and not always, of a constraint for an
open, and growing economy, which has adequate
Passage-3 exchange surpluses to buy food abroad. For the world
as a whole, supply-demand balance is always an
“The conceptual difficulties in National Income inescapable prerequisite for warding off hunger and
comparisons between underdeveloped and industrialised starvation. However, global availability of adequate
countries are particularly serious because a part of the supply does not necessarily mean that food would
national output in various underdeveloped countries is automatically move from countries of surplus to countries
produced without passing through the commercial of deficit if the latter lack in purchasing power. The
channels.” uneven distribution of hunger, starvation, under or
66. In the above statement, the author implies that: malnourishment, etc., at the world-level, thus owes itself
(a) the entire national output produced and to the presence of empty-pocket hungry mouths,
consumed in industrialised countries passes overwhelmingly confined to the underdeveloped
through commercial channels economies. Inasmuch as ‘a two-square meal’ is of
(b) the existence of a non-commercialised sector elemental significance to basic human existence, the
in different underdeveloped countries renders issue of worldwide supply of food has been gaining
the national income comparisons over significance, in recent times, both because the quantum
countries difficult and the composition of demand has been undergoing
(c) no part of national output should be produced big changes, and because, in recent years, the capabilities
and consumed without passing through of individual countries to generate uninterrupted chain
commercial channels of food supplies have come under strain. Food
(d) a part of the national output being produced production, marketing and prices, especially price-
and consumed without passing through affordability by the poor in the developing world, have
commercial channels is a sign of become global issues that need global thinking and
underdevelopment global solutions.

ST . 112
Sample Test 6 : General Studies Paper II (2016)

68. According to the above passage, which of the (a) 1/3 (b) 1/2
following are the fundamental solutions for the (c) 1/4 (d) 1/6
world food security problem?
74. In a class of 60 students, where the number of girls
1. Setting up more agro-based industries
is twice that of boys, Kamal, a boy, ranked
2. Improving the price affordability by the poor
3. Regulating the conditions of marketing seventeenth from the top. If there are 9 girls ahead
4. Providing food subsidy to one and all of Kamal, the number of boys in rank after him
is:
Select the correct answer using the code given (a) 13 (b) 12
below: (c) 7 (d) 3
(a) 1 and 2 only (b) 2 and 3 only
(c) 1, 3 and 4 only (d) 1, 2, 3 and 4 75. A and B walk around a circular park. They start
at 8 a.m. from the same point in the opposite
69. According to the above passage, the biggest directions. A and B walk at a speed of 2 rounds
challenge to world agriculture is:
per hour and 3 rounds per hour respectively. How
(a) to find sufficient land for agriculture and to
many times shall they cross each other after 8.00
expand food processing industries
(b) to eradicate hunger in underdeveloped a.m. and before 9.30. a.m.?
countries (a) 7 (b) 6
(c) to achieve a balance between the production (c) 5 (d) 8
of food and non-food items 76. W can do 25 per cent of a work in 30 days, X
(d) to achieve a balance between demand for and can do 1/4 of the work in 10 days, Y can do 40
supply of food per cent of the work in 40 days and Z can do 1/
70. According to the above passage, which of the 3 of the work in 13 days. Who will complete the
following helps/help in reducing hunger and work first?
starvation in the developing economies? (a) W (b) X
1. Balancing demand and supply of food (c) Y (d) Z
2. Increasing imports of food 77. The average monthly income of a person in a
3. Increasing purchasing power of the poor
@UPSC_THOUGHTS

certain family of 5 is Rs 10,000. What will be the


4. Changing the food consumption patterns and average monthly income of a person in the same
practices family if the income of one person increased by
Select the correct answer using the code given Rs 1,20,000 per year?
below: (a) Rs 12,000 (b) Rs 16,000
(a) 1 only (b) 2, 3 and 4 only (c) Rs 20,000 (d) Rs 34,000
(c) 1 and 3 only (d) 1, 2, 3 and 4
78. In a race, a competitor has to collect 6 apples
71. The issue of worldwide supply of food has gained which are kept in a straight line on a track and
importance mainly because of: a bucket is placed at the beginning of the track
1. overgrowth of the population worldwide which is a starting point. The condition is that the
2. sharp decline in the area of food production competitor can pick only one apple at a time, run
3. limitation in the capabilities for sustained back with it and drop it in the bucket. If he has
supply of food to drop all the apples in the bucket, how much
Select the correct answer using the code given total distance he has to run if the bucket is 5
below: meters from the first apple and all other apples
(a) 1 and 2 only (b) 3 only are placed 3 meters apart?
(c) 2 and 3 only (d) 1, 2 and 3 (a) 40 m (b) 50 m
72. Four-digit numbers are to be formed using the (c) 75 m (d) 150 m
digits 1, 2, 3 and 4; and none of these four digits 79. A round archery target of diameter 1 m is marked
are repeated in any manner. Further, with four scoring regions from the centre outwards
1. 2 and 3 are not to immediately follow each as red, blue, yellow and white. The radius of the
other red band is 0.20 m. The width of all the remaining
2. 1 is not to be immediately followed by 3 bands is equal. If archers throw arrows towards
3. 4 is not to appear at the last place the target, what is the probability that the arrows
4. 1 is not to appear at the first place fall in the red region of the archery target?
How many different numbers can be formed? (a) 0.40 (b) 0.20
(a) 6 (b) 8 (c) 0.16 (d) 0.04
(c) 9 (d) None of the above 80. A person allows a 10 per cent discount for cash
73. A cylindrical overhead tank of radius 2 m and payment from the marked price of a toy and still
height 7 m is to be filled from an underground he makes a 10 per cent gain. What is the cost price
tank of size 5.5m x 4m x 6m. How much portion of the toy which is marked Rs 770?
of the underground tank is still filled with water (a) Rs 610 (b) Rs 620
after filling the overhead tank completely? (c) Rs 630 (d) Rs 640

ST . 113
Sample Test 6 : General Studies Paper II (2016)

PRACTICE ANSWER SHEET

Directions: Use black ball pen. Example

1. 17. 33. 49. 65.

2. 18. 34. 50. 66.

3. 19. 35. 51. 67.

4. 20. 36. 52. 68.

5. 21. 37. 53. 69.

6. 22. 38. 54. 70.


@UPSC_THOUGHTS

7. 23. 39. 55. 71.

8. 24. 40. 56. 72.

9. 25. 41. 57. 73.

10. 26. 42. 58. 74.

11. 27. 43. 59. 75.

12. 28. 44. 60. 76.

13. 29. 45. 61. 77.

14. 30. 46. 62. 78.

15. 31. 47. 63. 79.

16. 32. 48. 64. 80.

ST . 114
Sample Test 6 : General Studies Paper II (2016)

ANSWERS AND EXPLANATORY NOTES


1. (c) The sentences, “The issue is ...... for governance”, 10. (d) The statement implies that one of the activities
support statement 1. Statement 2 finds no is chosen definitely. So choosing to do neither
mention in the passage. Statement 3 is negated falsifies the claim. Response (b) is irrelevant.
by the passage. The passage says constitutional
provisions as they exist have proved insufficient, 11. (d) The six faculty members could have been all
male or all female, so response (a) and (c) are
hence the civil services bill has been proposed.
not acceptable. As we do not know how many
The passage says the roles need to be defined.
of the females knew music, we cannot accept
So statement 4 is correct.
response (c).
2. (d) Statement 1 is not implied; the passage assumes
12. (b)
that accountability as envisaged by the
A A
Constitution has not worked out satisfactorily.
Statement 2 is not correct, as it is clearly stated
that “the political leaders of the day ...... are E C C E
expected to be externally accountable to the
or
society at large .......”
3. (d)
D B B D
4. (d) The question refers to ‘internal accountability’;
(d) refers to external accountability. [Incidentally, A cannot be 2 seats from D in both
5. (a) Refer to the sentences, “Our duties to one directions.]
another ...... our other relationships.” Statements 13. (d) We need to find the LCM of the days on which
@UPSC_THOUGHTS

2 and 3 are not supported by the passage. the clubs meet.


6. (c) Refer to the sentences, “This correspondence LCM of 2, 3, 4, 5, 6
...... justice.” This is central to the passage. = 60
In 180 days the five groups meet on the same
7. (d) Since B and C fell sick, we have to see which day every 60th day.
was the only fruit common to both of them. 180
Clearly, the oranges are to blame as only B and They meet = 3 times.
60
C ate them and A and D did not have them.
14. (c)
8. (d) Rural-urban migration has nothing to do with
overall rate of population growth. If the birth rate For Question items 15 to 19, a chart will help. See
declines faster than the death rate, the population below.
would decrease. The given facts do not support P — unmarried student
Q — brother of P (so male); not doctor or artist;
response (c).
so must be lawyer (and cannot be married
9. (b) See the diagram. Clearly (b) is the correct to T, a man)
option: 3 km east and 1 km north. R — not a doctor; must be artist
S — unmarried student
1 km N T — a man; must be doctor; and can be married
W E only to R the artist (so R is female)
1 km
S 15. (a)
1 km 16. (c)
17. (b)
1 km
18. (b)
1 km
19. (c)
1 km X starts 20. (c) Normally 19000 items would take 19 days,
given 1000 pieces are produced per day. As
end point there are defective items to the order of 5 per
1 km
cent or 50 items per day, clearly responses (a)
1 km 1 km 1 km 1 km 1 km 1 km 1 km and (b) are not correct. Take option (c).

ST . 115
Sample Test 6 : General Studies Paper II (2016)

In 20 days 20 × 1000 or 20000 items will be 30. (b) We may assume each problem carries 1 mark.
produced. If she had to get 60 per cent to pass, she would
In 20 days, defective items will be have to get
50 × 20 or 1000 items. 60
× 70 = 42 marks
So we have 20000 – 1000 = 19000 items. 100
Option (c) is correct. Anita has got
Alternatively, use the formula 70 40 60
= × 10 + × 30 + × 30
19000 100 100 100
= 20 days.
(1000 − 50) or 37 questions correct
21. (d) All three statements are supported by the She should have answered 42 – 35 = 5 more
passage. questions correctly.
22. (b) 31. (b) If x is the total number of students in the class,
under the given conditions
23. (b)
3 2
24. (b) Statement 1 has been made in the passage but 18 = of x
4 3
it is not an assumption of the passage. 1
18 = x
25. (d) 2
x = 36
26. (a) Total time from 11 a.m. to 2.27 p.m. is 3 hours Now, if total number of boys is two-thirds of the
27 minutes class; the girls would be one-third of the class
1
If there are four periods, there are 3 intervals; of 36 = 12.
if each is for 5 minutes, total time for intervals 3
is 5 × 3 = 15 minutes 32. (d) A ≥ B > C
@UPSC_THOUGHTS

So, total time for classes will be D ≥ C.


3 hours 27 minutes – 15 minutes 33. (b) Let the total employees number 100. Let the
= 3 hours, 12 minutes number of males be x, and number of females
= 180 minutes + 12 minutes 100 – x
= 192 minutes Under the given conditions,
192 5200x + 4200 (100 – x) = 5000 × 100
Each period will be for = 48 minutes. 5200x + 420000 – 4200x = 500000
4
27. (a) A (being the fastest) will be the person 5200x – 4200x = 500000 – 420000
accompanying each of the others and returning 1000x = 80000
every time. 80000
x = = 80
A and B → 2 + 1 (back) → 3 minutes 1000
A and C → 7 + 1 (back) → 8 minutes The males in the company form 80%.
A and D → 10 + 1 (back) → 11 minutes
A crosses alone without lamp → 1 minute For Question items 34 to 36, a chart will help see
below.
Total 23 minutes. D C B F E A
Football Cricket Volleyball Hockey Golf Tennis
28. (d) Given the ratio of sugar to water in the vessels,
Yellow/Blue Green Yellow/Blue Indigo Violet Orange
the solution in vessel C must be the sweetest.
So clearly (d) is the correct response. 34. (b)
30 35. (b) It cannot be ascertained which box is painted
[The ratios are = 1 : 6 → A
180 yellow and which, blue.
40
= 1 : 7 → B 36. (c) Refer to explanation to item 35.
280
20 37. (b) Let Z be 100
= 1 : 5 → C]. Then X = 100 – 20 = 80
100 and Y = 100 – 28 = 72
29. (b) The number of students in the class must be So Y is 8 less than X
8
443 − 2 = 441 = 21. Percentage wise the difference is × 100 = 10%
80

ST . 116
Sample Test 6 : General Studies Paper II (2016)

38. (c) The trains do not reach their destinations in 24 OB 2 = OA 2 + AB 2


hours (i.e., a day). If the service has to be daily, OB = +
an additional train from each station must start
on the second day. = 52 + 122
39. (c) If the side of the cube is a, its surface area is = 25 + 144
6a2. = 169
The area of the given rectangle
= 12 × 8 = 96 cm2 OB = 13 km.
Surface area of the cube = Area of the rectangle 50. (a) The maximum possible length and breadth of a
6a2 = 96 cm2 rectangle in which x1 + x2 = 40 metres can be
96 19 + 21.
a2 = cm2 = 16 cm2 Area would be 19 × 21 = 399 sq. m. while is
6
less than 400 sq. m.
a = 16 = 4 cm.
[Another way to look at the problem is to note
40. (d) Each of the five questions has two options; so that a square among all rectangles has the
each question can be answered with two choices. largest area, and in the case of a square
Under the given conditions, the candidates will x1 = x2.
number 2 × 2 × 2 × 2 × 2 = 32. As x1 + x2 = 40 m, x1 = x2 = 20 m, and the area
41. (c) will be 20 × 20 = 400 sq. m. So, a rectangle
whose length and breadth add up to 40 m
42. (d)
cannot have an area exceeding 400 sq. m.]
43. (a) Response (b) is not derivable from the passage
which merely states that water is made fit for 51. (b) If 3 years ago the sum of the ages of 5 members
consumption by passing through various of a family was 80 years, the average was
ecosystems. Response (c) may be true but it is 80
= 16 years
@UPSC_THOUGHTS

not the answer to the question. Response (d) 5


is not supported by the passage. Three years later a total of 3 × 5 or 15 years
have been added so that the sum of the 5
44. (d) Response (b) is not correct, as there is no
members is now 80 + 15 = 95 years. The
mention of radical restructuring of public finance
95
system. average in that case would be = 17 years.
5
45. (b) Refer to the sentence, “A more sustained . . . However, it is given that the average remains
large scale.” While the passage calls for a focus the same, i.e., 16 years, with the addition of
on abolishing manual scavenging, it does not another member to the family.
say abolition of manual scavenging is the solution So the total age would be 16 × 6 = 96 years.
to urban sanitation problems. Clearly the difference is 96 – 95 = 1 year
46. (d) The baby is 1 year old.
47. (b) Nothing in the passage supports antithesis of 52. (d) Let the basic pay of one person be x and that
politics and economics or the domination of one of the other be y. As their emoluments are the
over the other. same and given their allowances, we have
48. (b) Response (a) is too narrow. Response (c) is x + 0.65x = y + 0.80y
nowhere indicated by the passage. x(1 + 0.65) = y(1 + 0.80)
49. (b) x 1.80 12
= =
y 1.65 11
B
The basic pays of the two are in the ratio 12 : 11.
N
53. (b) From the first step the person takes four more
12 km W to reach the middle rung which clearly is the
E
fifth. There must be four more steps to the top.
So the ladder has 9 steps.
S
For Question items 54 to 56 a chart would help.
A It is to be presumed that the sweater is white, otherwise
5 km nothing certain can be determined from the statements.
O Also B cannot be assumed to be Ribeiro.
The path taken to reach B creates a 90° angle Just jot down in a rough chart the given details and what
at A. So using the Pythagoras Theorem, may be derived from them.

ST . 117
Sample Test 6 : General Studies Paper II (2016)

A B C 58. (a) A diagram will help. Just mark the directions and
White✓ White ✕ Tie✓ distances correctly.
Sweater✓ Sweater ✕ Blue ?
↓ Jacket✓ Black ? 19 km
Kumar✓ Blue ? Singh
Black ? or ? 4 km 15 km
Singh or Ribeiro
Ribeiro N
Kumar Singh Ribeiro 12 km W E
Jacket ✕ White ✕ White ✕ 15 km 12 km
[So not B; as Sweater ✕ Sweater ✕
Ribeiro and Black Black ✕ S
Singh do not Tie Blue ✓
wear white, C Tie 4 km
Kumar is left Jacket C Starting point
with white B Jacket 3 km
sweater.] B
Now the questions can be answered.
We have a right angled triangle. Derived from
54. (d) It is not Kumar, but it could be Ribeiro or Singh. the Pythagorean theorem there is the formula
55. (d) It cannot be white, but it could be either black that if the sides enclosing the right angles are
or blue. 4 and 3, the longest side (the hypotenuse)
would be 5. So the person is 5 km from the
56. (a) starting point.
57. (b) Make a diagram: [Alternatively, calculate as per the Pythagoras
Theorem:
@UPSC_THOUGHTS

B The distance from the starting point to the end


point = 4 2 + 32
= 16 + 9
C = 25
= 5 km.]
59. (b) If the side of the small cube is one-fourth the
Original height of the
side of the big cube, there will be a total of 64
trunk AB which is smaller cubes.
12m
? now = AD + CD

D 5m A

We need to find the length of CD.


CAD is a right angled triangle, so
CD 2 = AC 2 + AD 2
CD = AC 2 + AD 2
= 122 + 52
= 144 + 25
= 169
CD = 13 m
The original height of the trunk There are sixteen smaller cubes to each face.
AB = AC + CD (There are six faces but only three are shown
= 12 + 13 in the diagram; the other three can be visualised.)
= 25 m. The cubes on the sides would have at least two

ST . 118
Sample Test 6 : General Studies Paper II (2016)

sides painted. The four cubes in the middle of 63. (d) Between 100 and 199, there are 10 numbers
each side (shaded) will have only one side ending in 2:
painted. There are four such cubes on each of 102, 112, 122, 132, 142, 152, 162, 172, 182
the six faces of the larger cube. So in total there and 192
will be From 200 to 300, there are 100 numbers beginning
6 × 4 = 24 with 2:
Cubes with only one side painted. 200, 201, 202, …. 299
(It is irrelevant to the question that all the faces There are also 10 numbers ending in 2:
are painted in different colours.) 202, 212 …. 292
60. (c) If Ram and Shyam complete the work alone in So numbers beginning or ending with 2 total
x and y days respectively. 10 + 100 + 10 = 120.
1
Ram’s one day’s work = 64. (a) Refer to the sentence, “Simply put .............….
x 1
Shyam’s one day’s work = resources .............….”
y
So in 4 days as per given conditions, 65. (b)
4 4 6 3 66. (b)
+ = =
x y 100 5
67. (d) This is the corollary.
2
The rest of the work is 68. (b) This is the best answer; statements 1 and 4 find
5
Shyam takes 8 days to complete the work no mention in the passage.
8 2 69. (b) At present, according to the passage, this is the
=
y 5 challenge.
8×5
@UPSC_THOUGHTS

70. (c)
y = = 20
2 71. (b)
Putting this value of y in the equation
72. (d)
4 4 3
+ =
x y 5 73. (a) The volume of the overhead cylindrical tank
We have = πr2h
4 4 3 22
+ = = × (2)2 × 7 = 88 m3
x 20 5 7
4 12 − 4 8 The volume of the underground tank
= = = 5.5 × 4 × 6 = 132 m3
x 20 20 After filling the overhead tank there will be
20 132 – 88 = 44 m3 of water left in the underground
x = × 4 = 10
8 tank. 44 1
Ram can finish the work alone in 10 days. That indicates or portion of the
132 3
61. (d) The word is broken into two halves, and the underground tank still filled with water.
letters of each half reversed.
74. (b) If the number of boys is x, the number of girls
SYS TEM
is 2x.
SYS MET
x + 2x = 60 (given)
NEA RER 60
AEN RER x = = 20
3
FRAC TION Number of boys is 20; number of girls is 40.
CARF NOIT As Kamal is seventeenth from top, the number
of students behind Kamal
62. (b) Try out the answer responses with integers
divisible by 5. = 60 – 17 = 43
In the case of divisibility by 10, the number There are 9 girls ahead of Kamal.
needs to end with a zero. Now 5 + 10 = 15 There must be 40 – 9 or 31 girls behind Kamal
which is divisible by 5 but not by 10. So (b) is So, the number of boys behind Kamal
not necessarily true. = 43 – 31 = 12.

ST . 119
Sample Test 6 : General Studies Paper II (2016)

75. (a) As A and B are walking in opposite directions, Additional monthly increment
their relative speeds will be 2 + 3 = 5 rounds 1,20,000
per hour. In one hour they will cross each other = = 10,000
12
5 times, or once every 12 minutes. 10,000
In half an hour they will cross each other twice. Average increase = = 2,000
5
1
So, between 8 a.m. and 9.30 a.m., which is 1 So new average = 10,000 + 2,000 = Rs 12,000
2
hours, they will cross each other [If, on the other hand, each person in the family
5 + 2 = 7 times. got an increment of Rs 1,20,000 a year, the new
average would be Rs 20,000.]
1 25 1
76. (d) The capacity of W = × = 78. (d) The total distance that the competitor covers
30 100 120
part of the work in a day. while collecting the apples and putting them in
the bucket will be
1 1 1
The capacity of X = × = 2 [5 + (5 + 3) + (5 + 3 + 3) + (5 + 3 + 3 + 3)
10 4 40
+ (5 + 3 + 3 + 3 + 3) + (5 + 3 + 3 + 3 + 3 + 3)]
part of the work in a day.
= 2 (5 + 8 + 11 + 14 + 17 + 20)
1 40 1 = 2 × 75 = 150 m.
The capacity of Y = × =
40 100 100
79. (c) As we are concerned only with the red band
part of the work in a day.
at the centre, we may treat the rest of the archery
1 1 1 target as a single ring.
The capacity of Z = × =
13 3 39 d2
Area of a circle = πr 2 or π
part of the work in a day. 4
1
Comparing the fractions Here the area of the target = π or 0.25 π.
@UPSC_THOUGHTS

4
1 1 1 1 Area of the red band at the centre
, , and
120 40 100 39 = π (0.20)2
we may conclude Z’s capacity is highest, so, = 0.04 π.
Z will complete the work first. The arrows thrown by the archers may all fall
77. (a) Average monthly income of a person in a family within the red band or all fall outside the red
of 5 = Rs 10,000. band. The probability of arrows falling in the red
The total monthly income of the family region
= 10,000 × 5 = Rs 50,000 0.04 π
One person gets an increment of Rs 1,20,000 = = 0.16.
per year, which is a monthly increment of 0.25 π
1,20,000 80. (c) The marked price (MP) of the toy = Rs 770
= Rs 10,000
12 Discount = 10% of the MP = 0.1 × 770 = Rs 77
After increment, the total monthly income of the Price after discount = 770 – 77 = Rs 693
family
If x be the cost price, as per given data,
= Rs 50,000 + Rs 10,000
= Rs 60,000 10
693 – x = x
The average monthly income now will be 100
60,000 693 = 0.1x + x
= = Rs 12,000
5 693 = 1.1x
Shortcut: Find the average of the monthly
693
increase in increment and add to the original x = = Rs 630.
average. 1.1

ST . 120
Sample Test 7 : General Studies Paper II (2017)

SAMPLE TEST 7: GENERAL STUDIES PAPER II (2017)


Directions for the following 8 (eight) items: Read the undermine the whole Arctic food web, atop which stand
following eight passages and answer the items that polar bears.
follow the passages. Your answers to these items should 3. Which among the following is the most crucial
be based on the passages only. message conveyed by the above passage?
(a) Climate change has caused Arctic summer to
Passage-1 be short but temperature to be high.
What climate change will undeniably do is cause or (b) Polar bears can be shifted to South Pole to
amplify events that hasten the reduction of resources. ensure their survival.
Competition over these diminishing resources would (c) Without the presence of polar bears, the food
ensue in the form of political or even violent conflict. chains in Arctic region will disappear.
Resource-based conflicts have rarely been overt and are (d) Climate change poses a threat to the survival
thus difficult to isolate. Instead they take on veneers that of polar bears.
appear more politically palatable. Conflicts over resources
like water are often cloaked in the guise of identity or Passage-4
ideology. Why do people prefer open defecation and not want
toilets or, if they have them, only use them sometimes?
1. What does the above passage imply? Recent research has shown two critical elements : ideas
(a) Resource-based conflicts are always politically of purity and pollution, and not wanting pits or septic
motivated. tanks to fill because they have to be emptied. These are
(b) There are no political solutions to resolve the issues that nobody wants to talk about, but if we want
environmental and resource-based conflicts. to eradicate the practice of open defecation, they have
(c) Environmental issues contribute to resource to be confronted and dealt properly.
@UPSC_THOUGHTS

stresses and political conflict.


(d) Political conflict based on identity or ideology 4. Which among the following is the most crucial
cannot be resolved. message conveyed by the above passage?
(a) The ideas of purity and pollution are so deep-
Passage-2 rooted that they cannot be removed from the
minds of the people.
The man who is perpetually hesitating which of the two (b) People have to perceive toilet use and pit-
things he will do first, will do neither. The man who emptying as clean and not polluting.
resolves, but suffers his resolution to be changed by the (c) People cannot change their old habits.
first counter-suggestion of a friend—who fluctuates from (d) People have neither civic sense nor sense of
opinion to opinion and veers from plan to plan—can privacy.
never accomplish anything. He will at best be stationary
and probably retrograde in all. It is only the man who Passage-5
first consults wisely, then resolves firmly and then In the last two decades, the world’s gross domestic
executes his purpose with inflexible perseverance, product (GDP) has increased by 50 per cent, whereas
undismayed by those petty difficulties which daunt a inclusive wealth has increased by a mere 6 per cent. In
weaker spirit—that can advance to eminence in any line. recent decades, GDP-driven economic performance has
2. The keynote that seems to be emerging from the only harmed inclusive wealth like human capital; and
passage is that natural capital like forests, land and water. While the
(a) we should first consult wisely and then resolve world’s human capital which stands at 57 per cent of total
inclusive wealth grew by only 8 per cent, the natural
firmly
capital which is 23 per cent of total inclusive wealth
(b) we should reject suggestions of friends and
declined by 30 per cent worldwide in the last two
remain unchanged
decades.
(c) we should always remain broad-minded
(d) we should be resolute and achievement- 5. Which of the following is the most crucial inference
oriented from the above passage?
(a) More emphasis should be laid on the
Passage-3 development of natural capital.
(b) The growth driven by GDP only is neither
During the summer in the Arctic Ocean, sea ice has been desirable nor sustainable.
melting earlier and faster, and the winter freeze has been (c) The economic performance of the countries of
coming later. In the last three decades, the extent of the world is not satisfactory.
summer ice has declined by about 30 per cent. The (d) The world needs more human capital under
lengthening period of summer melt threatens to the present circumstances.

ST . 121
Sample Test 7 : General Studies Paper II (2017)

Passage-6 Directions for the following 2 (two) items: Consider the


given information and answer the two items that follow.
By 2020, when the global economy is expected to run
short of 56 million young people, India, with its youth No supporters of ‘party X’, who knew Z and supported
surplus of 47 million, could fill the gap. It is in this his campaign strategy, agreed for the alliance with
context that labour reforms are often cited as the way ‘party Y’; but some of them had friends in ‘party Y’.
to unlock double-digit growth in India. In 2014, India’s 9. With reference to the above information, which one
labour force was estimated to be about 40 per cent of among the following statements must be true?
the population, but 93 per cent of this force was in (a) Some supporters of ‘party Y’ did not agree for
unorganised sector. Over the last decade, the compound the alliance with the ‘party X’.
annual growth rate (CAGR) of employment has slowed (b) There is at least one supporter of ‘party Y’
to 0.5 per cent, with about 14 million jobs created during who knew some supporters of ‘party X’ as a
last year when the labour force increased by about 15 friend.
million. (c) No supporters of ‘party X’ supported Z’s
6. Which of the following is the most rational inference campaign strategy.
from the above passage? (d) No supporters of ‘party X’ knew Z.
(a) India must control its population growth so 10. With reference to the above information,
as to reduce its unemployment rate. consider the following statements :
(b) Labour reforms are required in India to make 1. Some supporters of ‘party X’ knew Z.
optimum use of its vast labour force 2. Some supporters of ‘party X’, who opposed
productively. Z’s campaign strategy, knew Z.
(c) India is poised to achieve the double-digit 3. No supporters of ‘party X’ supported Z’s
growth very soon. campaign strategy.
(d) India is capable of supplying the skilled young
Which of the statements given above is/are not
people to other countries.
correct?
Passage-7 (a) 1 only
@UPSC_THOUGHTS

(b) 2 and 3 only


The very first lesson that should be taught to us when (c) 3 only
we are old enough to understand it, is that complete (d) 1, 2 and 3
freedom from the obligation to work is unnatural, and
11. If second and fourth Saturdays and all the Sundays
ought to be illegal, as we can escape our share of the
are taken as only holidays for an office, what
burden of work only by throwing it on someone else’s
would be the minimum number of possible working
shoulders. Nature ordains that the human race shall
days of any month of any year?
perish of famine if it stops working. We cannot escape
(a) 23 (b) 22
from this tyranny. The question we have to settle is how
(c) 21 (d) 20
much leisure we can afford to allow ourselves.
12. If there is a policy that l/3rd of a population of
7. The main idea of the passage is that
a community has migrated every year from one
(a) it is essential for human beings to work
place to some other place, what is the leftover
(b) there should be a balance between work and
population of that community after the sixth year,
leisure
if there is no further growth in the population
(c) working is a tyranny which we have to face
during this period?
(d) human’s understanding of the nature of work
(a) 16/243rd part of the population
is essential
(b) 32/243rd part of the population
Passage-8 (c) 32/729th part of the population
(d) 64/729th part of the population
There is no harm in cultivating habits so long as they
are not injurious. Indeed, most of us are little more than 13. Four tests—Physics, Chemistry, Mathematics and
bundle of habits. Take away our habits and the residuum Biology are to be conducted on four consecutive
would hardly be worth bothering about. We could not days, not necessarily in the same order. The
get on without them. They simplify the mechanism of Physics test is held before the test which is
life. They enable us to do a multitude of things conducted after Biology. Chemistry is conducted
automatically, which, if we had to give fresh and original exactly after two tests are held. Which is the last
thought to them each time, would make existence an test held?
impossible confusion. (a) Physics (b) Biology
(c) Mathematics (d) Chemistry
8. The author suggests that habits
(a) tend to make our lives difficult 14. The sum of income of A and B is more than that
(b) add precision to our lives of C and D taken together. The sum of income of
(c) make it easier for us to live A and C is the same as that of B and D taken
(d) tend to mechanise our lives together. Moreover, A earns half as much as the

ST . 122
Sample Test 7 : General Studies Paper II (2017)

sum of the income of B and D. Whose income is 20. Gopal bought a cell phone and sold it to Ram at
the highest? 10% profit. Then Ram wanted to sell it back to
(a) A (b) B Gopal at 10% loss. What will be Gopal’s position
(c) C (d) D if he agreed?
15. Consider the following: (a) Neither loss nor gain
Statement: (b) Loss 1%
(c) Gain 1%
Good voice is a natural gift but one has to keep (d) Gain 0.5%
practising to improve and excel well in the field
of music. Directions for the following 7 (seven) items: Read the
following seven passages and answer the items that
Conclusions:
follow the passages. Your answers to these items should
I. Natural gifts need nurturing and care.
be based on the passages only.
II. Even though one’s voice is not good, one can
keep practising. Passage-1
Which one of the following is correct in respect
We have hard work ahead. There is no resting for any
of the above statement and conclusions?
of us till we redeem our pledge in full, till we make all
(a) Only conclusion I follows from the statement.
the people of India what destiny intends them to be. We
(b) Only conclusion II follows from the statement.
are citizens of a great country, on the verge of bold
(c) Either conclusion I or conclusion II follows
advance, and we have to live up to that high standard.
from the statement.
All of us, to whatever religion we may belong, are
(d) Neither conclusion I nor conclusion II follows
equally the children of India with equal rights, privileges
from the statement.
and obligations. We cannot encourage communalism or
16. There are three pillars X, Y and Z of different narrow-mindedness, for no nation can be great whose
heights. Three spiders A, B and C start to climb people are narrow in thought or action.
on these pillars simultaneously. In one chance, A
21. The challenge the author of the above passage
climbs on X by 6 cm but slips down 1 cm. B climbs
@UPSC_THOUGHTS

throws to the public is to achieve


on Y by 7 cm but slips down 3 cm. C climbs on
(a) a high standard of living, progress and
Z by 6.5 cm but slips down 2 cm. If each of them
privileges
requires 40 chances to reach the top of the pillars,
(b) equal privileges, fulfilment of destiny and
what is the height of the shortest pillar?
political tolerance
(a) 161 cm (b) 163 cm
(c) spirit of adventure and economic parity
(c) 182 cm (d) 210 cm
(d) hard work, brotherhood and national unity
17. “Rights are certain advantageous conditions of
social well-being indispensable to the true Passage-2
development of the citizen.”
“The individual, according to Rousseau, puts his person
In the light of this statement, which one of the and all his power in common under the supreme direction
following is the correct understanding of rights? of the General Will and in our corporate capacity we
(a) Rights aim at individual good only. receive each member as an indivisible part of the whole.”
(b) Rights aim at social good only.
22. In the light of the above passage, the nature of
(c) Rights aim at both individual and social good.
General Will is best described as
(d) Rights aim at individual good devoid of social
(a) the sum total of the private wills of the
well-being.
individuals
18. 15 students failed in a class of 52. After removing (b) what is articulated by the elected
the names of failed students, a merit order list has representatives of the individuals
been prepared in which the position of Ramesh is (c) the collective good as distinct from private
22nd from the top. What is his position from the wills of the individuals
bottom? (d) the material interests of the community
(a) 18th (b) 17th
(c) 16th (d) 15th Passage-3
19. Consider the following: In a democratic State, where a high degree of political
A + B means A is the son of B. maturity of the people obtains, the conflict between the
A – B means A is the wife of B. will of the sovereign law-making body and the organised
will of the people seldom occurs.
What does the expression P + R – Q mean?
(a) Q is the son of P. 23. What does the above passage imply?
(b) Q is the wife of P. (a) In a democracy, force is the main phenomenon
(c) Q is the father of P. in the actual exercise of sovereignty.
(d) None of the above (b) In a mature democracy, force to a great extent

ST . 123
Sample Test 7 : General Studies Paper II (2017)

is the main phenomenon in the actual exercise Which of these assumptions is/are valid?
of sovereignty. (a) 1 only
(c) In a mature democracy, use of force is (b) 2 only
irrelevant in the actual exercise of sovereignty. (c) Both 1 and 2
(d) In a mature democracy, force is narrowed (d) Neither 1 nor 2
down to a marginal phenomenon in the actual
exercise of sovereignty. Passage-7
There is more than a modicum of truth in the assertion
Passage-4 that “a working knowledge of ancient history is necessary
A successful democracy depends upon widespread interest to the intelligent interpretation of current events”. But the
and participation in politics, in which voting is an sage who uttered these words of wisdom might well have
essential part. To deliberately refrain from taking such added something on the benefits of studying particularly
an interest, and from voting, is a kind of implied anarchy; the famous battles of history for the lessons they contain
it is to refuse one’s political responsibility while enjoying for those of us who lead or aspire to leadership. Such
the benefits of a free political society. a study will reveal certain qualities and attributes which
24. This passage relates to enabled the winners to win—and certain deficiencies
(a) duty to vote which caused the losers to lose. And the student will see
(b) right to vote that the same pattern recurs consistently, again and again,
(c) freedom to vote throughout the centuries.
(d) right to participate in politics 27. With reference to the above passage, the following
assumptions have been made:
Passage-5 1. A study of the famous battles of history would
In a free country, the man who reaches the position of help us understand the modern warfare.
leader is usually one of outstanding character and ability. 2. Studying the history is essential for anyone
Moreover, it is usually possible to foresee that he will who aspires to be a leader.
reach such a position, since early in life one can see his Which of these assumptions is/are valid?
@UPSC_THOUGHTS

qualities of character. But this is not always true in the (a) 1 only
case of a dictator; often he reaches his position of power (b) 2 only
through chance, very often through the unhappy state (c) Both 1 and 2
of his country. (d) Neither 1 nor 2
25. The passage seems to suggest that 28. Suppose the average weight of 9 persons is 50 kg.
(a) a leader foresees his future position The average weight of the first 5 persons is 45 kg,
(b) a leader is chosen only by a free country whereas the average weight of the last 5 persons
(c) a leader must see that his country is free from is 55 kg. Then the weight of the 5th person will
despair be
(d) despair in a country sometimes leads to (a) 45 kg (b) 47.5 kg
dictatorship (c) 50 kg (d) 52.5 kg
29. In a group of six women, there are four tennis
Passage-6
players, four postgraduates in Sociology, one
The greatest blessing that technological progress has in postgraduate in Commerce and three bank
store for mankind is not, of course, an accumulation of employees. Vimala and Kamla are the bank
material possessions. The amount of these that can be employees while Amala and Komala are
effectively enjoyed by one individual in one lifetime is unemployed. Komala and Nirmala are among the
not great. But there is not the same narrow limit to the tennis players. Amala, Kamla, Komala and Nirmala
possibilities of the enjoyment of leisure. The gift of leisure are postgraduates in Sociology of whom two are
may be abused by people who have had no experience bank employees. If Shyamala is a postgraduate in
of making use of it. Yet the creative use of leisure by Commerce, who among the following is both a
a minority in societies has been the mainspring of all tennis player and a bank employee?
human progress beyond the primitive level. (a) Amala (b) Komala
(c) Nirmala (d) Shyamala
26. With reference to the above passage, the following
assumptions have been made : 30. P = (40% of A) + (65% of B) and Q = (50% of A)
1. People always see the leisure time as a gift + (50% of B), where A is greater than B.
and use it for acquiring more material In this context, which of the following statements
possessions. is correct?
2. Use of leisure by some people to produce new (a) P is greater than Q.
and original things has been the chief source (b) Q is greater than P.
of human progress. (c) P is equal to Q.

ST . 124
Sample Test 7 : General Studies Paper II (2017)

(d) None of the above can be concluded with (a) 25% (b) 30%
certainty. (c) 35% (d) 40%
31. A watch loses 2 minutes in every 24 hours while 38. There are 4 horizontal and 4 vertical lines, parallel
another watch gains 2 minutes in every 24 hours. and equidistant to one another on a board. What
At a particular instant, the two watches showed is the maximum number of rectangles and squares
an identical time. Which of the following statements that can be formed?
is correct if 24-hour dock is followed? (a) 16 (b) 24
(a) The two watches show the identical time (c) 36 (d) 42
again on completion of 30 days. 39. A freight train left Delhi for Mumbai at an average
(b) The two watches show the identical time speed of 40 km/hr. Two hours later, an express
again on completion of 90 days. train left Delhi for Mumbai, following the freight
(c) The two watches show the identical time train on a parallel track at an average speed of 60
again on completion of 120 days. km/hr. How far from Delhi would the express
(d) None of the above statements is correct. train meet the freight train?
32. In a city, 12% of households earn less than ` 30,000 (a) 480 km (b) 260 km
per year, 6% households earn more than ` 2,00,000 (c) 240 km (d) 120 km
per year, 22% households earn more than ` 1,00,000 40. In a test, Randhir obtained more marks than the
per year and 990 house-holds earn between total marks obtained by Kunal and Debu. The total
` 30,000 and ` 1,00,000 per year. How many marks obtained by Kunal and Shankar are more
households earn between ` 1,00,000 and ` 2,00,000 than those of Randhir. Sonal obtained more marks
per year? than Shankar. Neha obtained more marks than
(a) 250 (b) 240 Randhir. Who amongst them obtained highest
(c) 230 (d) 225 marks?
33. A clock strikes once at 1 o’clock, twice at 2 o’clock (a) Randhir (b) Neha
and thrice at 3 o’clock, and so on. If it takes 12 (c) Sonal (d) Data are inadequate
seconds to strike at 5 o’clock, what is the time taken
@UPSC_THOUGHTS

by it to strike at 10 o’clock? Directions for the following 8 (eight) items: Read the
(a) 20 seconds (b) 24 seconds following seven passages and answer the items that
(c) 28 seconds (d) 30 seconds follow the passages. Your answers to these items should
be based on the passages only.
34. Consider the given statement and the two
conclusions that follow: Passage-1
Statement: Disruption of traditional institutions, identifications and
Morning walk is good for health. loyalties is likely to lead to ambivalent situations. It is
Conclusions: possible that some people may renew their identification
1. All healthy people go for morning walk. with traditional groups whereas others align themselves
2. Morning walk is essential for maintaining with new groups and symbols emergent from processes
good health. of political development. In addition, political development
What is/are the valid conclusion/ conclusions? tends to foster group awareness of a variety of class, tribe,
(a) 1 only (b) 2 only region, clan, language, religion, occupation and others.
(c) Both 1 and 2 (d) Neither 1 nor 2 41. Which one of the following is the best explanation
35. There are thirteen 2-digit consecutive odd numbers. of the above passage?
If 39 is the mean of the first five such numbers, (a) Political development is not a unilinear process
then what is the mean of all the thirteen numbers? for it involves both growth and decay.
(a) 47 (b) 49 (b) Traditional societies succeed in resisting
(c) 51 (d) 45 positive aspects of political development.
36. Six boys A, B, C, D, E and F play a game of cards. (c) It is impossible for traditional societies to
Each has a pack of 10 cards. F borrows 2 cards break away from lingering loyalties.
from A and gives away 5 to C who in turn gives (d) Sustenance of traditional loyalties is conducive
3 to B while B gives 6 to D who passes on 1 to to political development.
E. Then the number of cards possessed by D and
E is equal to the number of cards possessed by
Passage-2
(a) A, B and C (b) B, C and F There has been a significant trend worldwide towards
(c) A, B and F (d) A, C and F regionalism in government, resulting in a widespread
37. There is a milk sample with 50% water in it. If transfer of powers downwards towards regions and
l/3rd of this milk is added to equal amount of pure communities since 1990s. This process, which involves
milk, then water in the new mixture will fall down the creation of new political entities and bodies at a sub-
to national level and an increase in their content and

ST . 125
Sample Test 7 : General Studies Paper II (2017)

powers, is known as devolution. Devolution has been till now have been growing rapidly, will stop growing
characterised as being made up of three factors—political beyond a point—a point that is well short of incomes
legitimacy, decentralisation of authority and in the developed West. The IMF identifies a number of
decentralisation of resources. Political legitimacy here causes of middle-income trap—none of which is
means a mass demand from below for the decentralisation surprising—from infrastructure to weak institutions, to
process, which is able to create a political force for it to less than favourable macroeconomic conditions. But the
take place. In many cases, decentralisation is initiated by broad, overall cause, says IMF, is a collapse in the growth
the upper tier of government without sufficient political of productivity.
mobilisation for it at the grassroots level, and in such 44. Which among the following is the most logical,
cases the decentralisation process often does not fulfil its rational and critical inference that can be made
objectives. from the above passage?
42. Which among the following is the most logical, (a) Once a country reaches middle-income stage,
rational and critical inference that can be made it runs the risk of falling productivity which
from the above passage? leads to stagnant incomes.
(a) Emergence of powerful mass leaders is essential (b) Falling into middle-income trap is a general
to create sub-national political entities and characteristic of fast growing economies.
thus ensure successful devolution and (c) There is no hope at all for emerging Asian
decentralisation. economies to sustain the growth momentum.
(b) The upper tier of government should impose (d) As regards growth of productivity, the
devolution and decentralisation on the regional performance of Asian economies is not
communities by law or otherwise. satisfactory.
(c) Devolution, to be successful, requires a
democracy in which there is free expression Passage-5
of the will of the people at lower level and An innovative India will be inclusive as well as
their active participation at the grassroots technologically advanced, improving the lives of all
level. Indians. Innovation and R&D can mitigate increases in
@UPSC_THOUGHTS

(d) For devolution to take place, a strong feeling social inequality and relieve the pressures created by
of regionalism in the masses is essential. rapid urbanisation. The growing divergence in
productivity between agriculture and knowledge-intensive
Passage-3 manufacturing and services threatens to increase income
We live in digital times. The digital is not just something inequality. By encouraging India’s R&D labs and
we use strategically and specifically to do a few tasks. universities to focus on the needs of poor people and by
Our very perception of who we are, how we connect to improving the ability of informal firms to absorb
the world around us, and the ways in which we define knowledge, an innovation and research agenda can
our domains of life, labour and language are hugely counter this effect. Inclusive innovation can lower the
structured by the digital technologies. The digital is costs of goods and services and create income-earning
everywhere and, like air, invisible. We live within digital opportunities for the poor people.
systems, we live with intimate gadgets, we interact 45. Which among the following is the most logical and
through digital media, and the very presence and rational assumption that can be made from the
imagination of the digital has dramatically restructured above passage?
our lives. The digital, far from being a tool, is a condition (a) Innovation and R&D is the only way to reduce
and context that defines the shapes and boundaries of rural to urban migration.
our understanding of the self, the society, and the (b) Every rapidly growing country needs to
structure of governance. minimise the divergence between productivity
43. Which among the following is the most logical and in agriculture and other sectors.
essential message conveyed by the above passage? (c) Inclusive innovation and R&D can help create
(a) All problems of governance can be solved by an egalitarian society.
using digital technologies. (d) Rapid urbanisation takes place only when a
(b) Speaking of digital technologies is speaking of country’s economic growth is rapid.
our life and living.
(c) Our creativity and imagination cannot be Passage-6
expressed without digital media. Climate change is likely to expose a large number of
(d) Use of digital systems is imperative for the people to increasing environmental risks forcing them to
existence of mankind in future. migrate. The international community is yet to recognise
this new category of migrants. There is no consensus on
Passage-4 the definition and status of climate refugees owing to the
The IMF has pointed out that the fast growing economies distinct meaning the term refugees carry under
of Asia face the risk of falling into ‘middle-income trap’. international laws. There are still gaps in understanding
It means that average incomes in these countries, which how climate change will work as the root cause of

ST . 126
Sample Test 7 : General Studies Paper II (2017)

migration. Even if there is recognition of climate refugees, 3. They are essential to maintain the biodiversity
who is going to provide protection? More emphasis has of any ecosystem.
been given to international migration due to climate Select the correct answer using the code given
change. But there is a need to recognise the migration below.
of such people within the countries also so that their (a) 1 only (b) 1 and 2 only
problems can be addressed properly. (c) 1 and 3 only (d) 1, 2 and 3
46. Which of the following is the most rational inference 49. Certain 3-digit numbers have the following
from the above passage? characteristics :
(a) The world will not be able to cope with large- 1. All the three digits are different.
scale migration of climate refugees. 2. The number is divisible by 7.
(b) We must find the ways and means to stop
3. The number on reversing the digits is also
further climate change.
divisible by 7.
(c) Climate change will be the most important
reason for the migration of people in the How many such 3-digit numbers are there?
future. (a) 2 (b) 4
(d) Relation between climate change and migration (c) 6 (d) 8
is not yet properly understood. 50. Examine the following statements :
1. All colours are pleasant.
Passage-7 2. Some colours are pleasant.
Many farmers use synthetic pesticides to kill infesting 3. No colour is pleasant.
insects. The consumption of pesticides in some of the 4. Some colours are not pleasant.
developed countries is touching 3000 grams/hectare.
Unfortunately, there are reports that these compounds Given that statement 4 is true, what can be
possess inherent toxicities that endanger the health of the definitely concluded?
farm operators, consumers and the environment. Synthetic (a) 1 and 2 are true (b) 3 is true.
pesticides are generally persistent in environment. Entering (c) 2 is false. (d) 1 is false.
@UPSC_THOUGHTS

in food chain they destroy the microbial diversity and 51. How many numbers are there between 99 and 1000
cause ecological imbalance. Their indiscriminate use has such that the digit 8 occupies the units place?
resulted in development of resistance among insects to (a) 64 (b) 80
insecticides, upsetting of balance in nature and resurgence (c) 90 (d) 104
of treated populations. Natural pest control using the
52. If for a sample data
botanical pesticides is safer to the user and the environment
Mean < Median < Mode then the distribution is
because they break down into harmless compounds
(a) symmetric
within hours or days in the presence of sunlight. Plants
with pesticidal properties have been in nature for millions (b) skewed to the right
of years without any ill or adverse effects on the (c) neither symmetric nor skewed
ecosystem. They are easily decomposed by many microbes (d) skewed to the left
common in most soils. They help in the maintenance of 53. The age of Mr. X last year was the square of a
biological diversity of predators and the reduction of number and it would be the cube of a number next
environmental contamination and human health hazards. year. What is the least number of years he must
Botanical pesticides formulated from plants are wait for his age to become the cube of a number
biodegradable and their use in crop protection is a again?
practical sustainable alternative. (a) 42 (b) 38
47. On the basis of the above passage, the following (c) 25 (d) 16
assumptions have been made : 54. P works thrice as fast as Q, whereas P and Q
1. Synthetic pesticides should never be used in together can work four times as fast as R. If P, Q
modern agriculture. and R together work on a job, in what ratio should
2. One of the aims of sustainable agriculture is they share the earnings?
to ensure minimal ecological imbalance. (a) 3 : 1 : 1 (b) 3 : 2 : 4
3. Botanical pesticides are more effective as (c) 4 : 3 : 4 (d) 3 : 1 : 4
compared to synthetic pesticides.
55. Consider the following relationships among
Which of the assumptions given above is/are members of a family of six persons A, B, C, D, E
correct? and F:
(a) 1 and 2 only (b) 2 only 1. The number of males equals that of females.
(c) 1 and 3 only (d) 1, 2 and 3 2. A and E are sons of F.
48. Which of the following statements is /are correct 3. D is the mother of two, one boy and one girl.
regarding biopesticides? 4. B is the son of A.
1. They are not hazardous to human health. 5. There is only one married couple in the family
2. They are persistent in environment. at present.

ST . 127
Sample Test 7 : General Studies Paper II (2017)

Which one of the following inferences can be number or rating. This index is ideally kept constantly
drawn from the above? updated and available in different places. The AQI is
(a) A, B and C are all females. most useful when lots of pollution data are being
(b) A is the husband of D. gathered and when pollution levels are normally, but not
(c) E and F are children of D. always, low. In such cases, if pollution levels spike for
(d) D is the daughter of F. a few days, the public can quickly take preventive action
56. A bag contains 20 balls. 8 balls are green, 7 are (like staying indoors) in response to an air quality
white and 5 are red. What is the minimum number warning. Unfortunately, that is not urban India. Pollution
of balls that must be picked up from the bag levels in many large Indian cities are so high that they
blindfolded (without replacing any of it) to be remain well above any health or regulatory standard for
assured of picking at least one ball of each colour? large part of the year. If our index stays in the ‘Red/
(a) 17 (b) 16 Dangerous’ region day after day, there is not much any
(c) 13 (d) 11 one can do, other than getting used to ignoring it.
57. If 2 boys and 2 girls are to be arranged in a row 61. Which among the following is the most logical and
so that the girls are not next to each other, how rational inference that can be made from the above
many possible arrangements are there? passage?
(a) 3 (b) 6 (a) Our governments are not responsible enough
(c) 12 (d) 24 to keep our cities pollution free.
58. The outer surface of a 4 cm × 4 cm × 4 cm cube (b) There is absolutely no need for air quality
is painted completely in red. It is sliced parallel indices in our country.
to the faces to yield sixty four 1 cm × 1 cm × 1 (c) Air quality index is not helpful to the residents
cm small cubes. How many small cubes do not of many of our large cities.
have painted faces? (d) In every city, public awareness about pollution
(a) 8 (b) 16 problems should increase.
@UPSC_THOUGHTS

(c) 24 (d) 36
Passage-2
59. Consider the following :
Productive jobs are vital for growth and a good job is
A, B, C, D, E, F, G and H are standing in a row the best form of inclusion. More than half of our
facing North. population depends on agriculture, but the experience of
B is not neighbour of G other countries suggests that the number of people
F is to the immediate right of G and neighbour dependent on agriculture will have to shrink if per capita
of E. incomes in agriculture are to go up substantially. While
G is not at the extreme end. industry is creating jobs, too many such jobs are low-
A is sixth to the left of E. productivity non-contractual jobs in the unorganised
H is sixth to the right of C. sector, offering low incomes, little protection, and no
Which one of the following is correct in respect benefits. Service jobs are relatively of high productivity,
of the above? but employment growth in services has been slow in
(a) C is to the immediate left of A. recent years.
(b) D is immediate neighbour of B and F.
62. Which among the following is the most logical and
(c) G is to the immediate right of D.
rational inference that can be made from the above
(d) A and E are at the extreme ends.
passage?
60. In a certain code, ‘256’ means ‘red colour chalk’, (a) We must create conditions for the faster growth
‘589’ means ‘green colour flower’ and ‘254’ means of highly productive service jobs to ensure
‘white colour chalk’. The digit in the code that employment growth and inclusion.
indicates ‘white’ is (b) We must shift the farm workers to the highly
(a) 2 (b) 4 productive manufacturing and service sectors
(c) 5 (d) 8 to ensure the economic growth and inclusion.
Directions for the following 7 (seven) items: Read the (c) We must create conditions for the faster growth
following seven passages and answer the items that of productive jobs outside of agriculture even
follow the passages. Your answers to these items should while improving the productivity of
be based on the passages only. agriculture.
(d) We must emphasise the cultivation of high-
Passage-1 yielding hybrid varieties and genetically
An air quality index (AQI) is a way to combine modified crops to increase the per capita
measurements of multiple air pollutants into a single income in agriculture.

ST . 128
Sample Test 7 : General Studies Paper II (2017)

Passage-3 (d) India has already gained a great share in


A landscape-scale approach to land use can encourage global markets in sectors showing a rising
greater biodiversity outside protected areas. During trend in demand.
hurricane ‘Mitch’ in 1998, farms using ecoagricultural
practices suffered 58 per cent, 70 per cent and 99 per cent
Passage-5
less damage in Honduras, Nicaragua and Guatemala, Over the last decade, Indian agriculture has become more
respectively, than farms using conventional techniques. robust with record production of food grains and oilseeds.
In Costa Rica, vegetative windbreaks and fencerows Increased procurement, consequently, has added huge
boosted farmers’ income from pasture and coffee while stocks of food grains in the granaries. India is one of the
also increasing bird diversity. Bee pollination is more world’s top producers of rice, wheat, milk, fruits and
effective when agricultural fields are closer to natural or vegetables. India is still home to a quarter of all
seminatural habitat, a finding that matters because 87 per undernourished people in the world. On an average,
cent of the world’s 107 leading crops depend on animal almost half of the total expenditure of nearly half of the
pollinators. In Costa Rica, Nicaragua and Colombia households is on food.
silvopastoral systems that integrate trees with pastureland
65. Which among the following is the most logical
are improving the sustainability of cattle production, and
corollary to the above passage?
diversifying and increasing farmers’ income.
(a) Increasing the efficiency of farm-to-fork value
63. Which among the following is the most logical and chain is necessary to reduce the poverty and
rational inference that can be made from the above malnutrition.
passage? (b) Increasing the agricultural productivity will
(a) Agricultural practices that enhance biodiversity automatically eliminate the poverty and
can often increase farm output and reduce the malnutrition in India.
vulnerability to disasters. (c) India’s agricultural productivity is already
(b) All the countries of the world should be great and it is not necessary to increase it
encouraged to replace ecoagriculture with further.
@UPSC_THOUGHTS

conventional agriculture. (d) Allocation of more funds for social welfare


(c) Ecoagriculture should be permitted in and poverty alleviation programmes will
protected areas without destroying the ultimately eliminate the poverty and
biodiversity there. malnutrition in India.
(d) The yield of food crops will be very high if
ecoagricultural practices are adopted to Passage-6
cultivate them. The States are like pearls and the Centre is the thread
which turns them into a necklace; if the thread snaps,
Passage-4 the pearls are scattered.
The medium term challenge for Indian manufacturing is
66. Which one of the following views corroborates the
to move from lower to higher tech sectors, from lower
above statement?
to higher value-added sectors, and from lower to higher
(a) A strong Centre and strong States make the
productivity sectors. Medium tech industries are primarily
federation strong.
capital intensive and resource processing; and high tech
(b) A strong Centre is a binding force for national
industries are mainly capital and technology intensive.
integrity.
In order to push the share of manufacturing in overall
(c) A strong Centre is a hindrance to State
GDP to the projected 25 per cent, Indian manufacturing
autonomy.
needs to capture the global market in sectors showing
(d) State autonomy is a prerequisite for a
a rising trend in demand. These sectors are largely high
federation.
technology and capital intensive.
64. Which among the following is the most logical and Passage-7
rational inference that can be made from the above Really I think that the poorest he that is in England has
passage? a life to live, as the greatest he, and therefore truly, I
(a) India’s GDP displays high value-added and think it is clear that every man that is to live under a
high productivity levels in medium tech and government ought first by his own consent to put himself
resource processing industries. under the government, and I do think that the poorest
(b) Promotion of capital and technology intensive man in England is not at all bound in a strict sense to
manufacturing is not possible in India. that government that he has not had a voice to put
(c) India should push up the public investments himself under.
and encourage the private investments in
research and development, technology 67. The above statement argues for
upgradation and skill development. (a) distribution of wealth equally to all

ST . 129
Sample Test 7 : General Studies Paper II (2017)

(b) rule according to the consent of the governed (a) A (b) B


(c) rule of the poor (c) D (d) E
(d) expropriation of the rich 73. A 2-digit number is reversed. The larger of the two
68. The average rainfall in a city for the first four days numbers is divided by the smaller one. What is
was recorded to be 0.40 inch. The rainfall on the the largest possible remainder?
last two days was in the ratio of 4 : 3. The average (a) 9 (b) 27
of six days was 0.50 inch. What was the rainfall (c) 36 (d) 45
on the fifth day? 74. The monthly incomes of X and Y are in the ratio
(a) 0.60 inch (b) 0.70 inch of 4 : 3 and their monthly expenses are in the ratio
(c) 0.80 inch (d) 0.90 inch of 3 : 2. However, each saves ` 6,000 per month.
What is their total monthly income?
Directions for the following 3 (three) items: Consider the (a) ` 28,000 (b) ` 42,000
given information and answer the three items that follow. (c) ` 56,000 (d) ` 84,000
A, B, C, D, E, F and G are Lecturers from different cities— 75. Two walls and a ceiling of a room meet at right
Hyderabad, Delhi, Shillong, Kanpur, Chennai, Mumbai angles at a point P. A fly is in the air 1 m from
and Srinagar (not necessarily in the same order) who one wall, 8 m from the other wall and 9 m from
participated in a conference. Each one of them is specialised the point P. How many meters is the fly from the
in a different subject, viz., Economics, Commerce, History, ceiling?
Sociology, Geography, Mathematics and Statistics (not (a) 4 (b) 6
necessarily in the same order). Further (c) 12 (d) 15
1. Lecturer from Kanpur is specialised in
Geography Directions for the following 3 (three) items: Consider the
2. Lecturer D is from Shillong given information and answer the three items that follow.
3. Lecturer C from Delhi is specialised in Eight railway stations A, B, C, D, E, F, G and H are
Sociology connected either by two-way passages or one-way
@UPSC_THOUGHTS

4. Lecturer B is specialised in neither History nor passages. One-way passages are from C to A, E to G,
Mathematics B to F, D to H, G to C, E to C and H to G. Two-way
5. Lecturer A who is specialised in Economics passages are between A and E, G and B, F and D, and
does not belong to Hyderabad E and D.
6. Lecturer F who is specialised in Commerce
76. While travelling from C to H, which one of the
belongs to Srinagar
following stations must be passed through?
7. Lecturer G who is specialised in Statistics
(a) G (b) E
belongs to Chennai
(c) D (d) F
69. Who is specialised in Geography? 77. In how many different ways can a train travel from
(a) B F to A without passing through any station more
(b) D than once?
(c) E (a) 1 (b) 2
(d) Cannot be determined as data are inadequate (c) 3 (d) 4
70. To which city does the Lecturer specialised in 78. If the route between G and C is closed, which one
Economics belong? of the following stations need not be passed
(a) Hyderabad through while travelling from H to C?
(b) Mumbai (a) E (b) D
(c) Neither Hyderabad nor Mumbai (c) A (d) B
(d) Cannot be determined as data are inadequate
79. There are certain 2-digit numbers. The difference
71. Who of the following belongs to Hyderabad? between the number and the one obtained on
(a) B reversing it is always 27. How many such maximum
(b) E 2-digit numbers are there?
(c) Neither B nor E (a) 3 (b) 4
(d) Cannot be determined as data are inadequate (c) 5 (d) None of the above
72. In a school, there are five teachers A, B, C, D and 80. What is the total number of digits printed, if a book
E. A and B teach Hindi and English. C and B teach containing 150 pages is to be numbered from 1 to
English and Geography. D and A teach Mathematics 150?
and Hindi. E and B teach History and French. Who (a) 262 (b) 342
teaches maximum number of subjects? (c) 360 (d) 450

ST . 130
Sample Test 7 : General Studies Paper II (2017)

PRACTICE ANSWER SHEET

Directions: Use black ball pen. Example

1. 17. 33. 49. 65.

2. 18. 34. 50. 66.

3. 19. 35. 51. 67.

4. 20. 36. 52. 68.

5. 21. 37. 53. 69.

6. 22. 38. 54. 70.


@UPSC_THOUGHTS

7. 23. 39. 55. 71.

8. 24. 40. 56. 72.

9. 25. 41. 57. 73.

10. 26. 42. 58. 74.

11. 27. 43. 59. 75.

12. 28. 44. 60. 76.

13. 29. 45. 61. 77.

14. 30. 46. 62. 78.

15. 31. 47. 63. 79.

16. 32. 48. 64. 80.

ST . 131
Sample Test 7 : General Studies Paper II (2017)

ANSWERS AND EXPLANATORY NOTES


1. (c)
2. (d) Response (a) appears to be the correct one at 13. (c) Chemistry is clearly the third test. Biology
first sight. But the question asks you to identify cannot be the last test as there is a test after
the keynote emerging from the passage. Biology, and Physics is not the last, as it is
Response (a) simply takes a few words from before Biology. So Mathematics must be the
the passage. If you read the full sentence, ‘It last.
is only the man ... any line’. Response (d) is 14. (b) A + B > C + D
more apt as the emerging idea of the passage. A + C = B + D
3. (d) The passage contradicts Response (a). B +D
A = 2A = B + D
Responses (b) and (c) are not indicated in the 2
passage. ∴ A + C = 2A C = A
Given A + B > C + D
4. (b) Response (a) seems correct at first sight, but
the passage does not say the ideas of purity A + B > A + D
and pollution cannot be removed. ∴ B > D.

5. (b) This is the ‘crucial inference’. 15. (d) While conclusion I may be correct as a statement,
it does not follow from the given statement. One
6. (b) cannot generalise on the basis of a single
example. Conclusion II clearly does not follow
7. (a)
from the given statement.
8. (c)
16. (b) In the case of Pillar X, the height gained at one
@UPSC_THOUGHTS

9. (b) Response (b) follows from the statement: ‘but move = 6 – 1 = 5 cm


some ... in party Y’. After 40 chances the height gained
= 40 × 5 + 1 or 39 × 5 + 6 or 201 cm.
10. (b) Statement 1 is correct. From the given
(At the last chance, the spider would have
information, it is not possible to conclude
climbed the full 6 cm to reach the top.)
statement 2 as there may or may not be such
Now in the case of Pillar Y, the height would
supporters. Statement 3 contradicts the given
be 39 × 4 + 7 = 163 cm.
information. So only 2 and 3 are not correct.
(It is shorter than X)
11. (b) Since February is the month with fewest days, In case of Pillar Z, the height would be
let’s take it as our measure. In a February of 39 × 4.5 + 6.5
28 days, there will be 4 Sundays and 2 This would clearly be more than the height of
Saturdays off, so working days will amount to Pillar Y. So the height of the shortest pillar is
28 – 4 – 2 = 22. 163 cm.
If we have 5 Sundays in a February of 29 days,
17. (c)
the working days will amount to
29 – 5 – 2 = 22 days. 18. (c) Number of students who passed
If in a month of 30 days there are 5 Sundays, = 52 – 15 = 37
the number of working days will be Ramesh’s position from top is 22nd.
30 – 5 – 2 = 23. There are 37 – 22 or 15 positions below him.
So 22 is the minimum number of working days
in any month. top R bottom
1 22
12. (d) Since of the population leaves, at every 15
3 16
2
stage there will be of the population left. At From the bottom Ramesh is at the 16th position.
3
the end of the sixth year, the population left will
be 19. (c) P + R P is the son of R.
2 2 2 2 2 2 64 R – Q R is the wife of Q.
× × × × × =
3 3 3 3 3 3 729 Clearly P is the son of Q, or Q is the father
of the original population. of P.

[26 is 64; as only Response (d) has 64 in the 20. (c) Let Gopal’s cost price be Rs 100.
numerator, that is your correct response.] With 10% profit, selling price of cellphone by

ST . 132
Sample Test 7 : General Studies Paper II (2017)

Goplal = Rs 110 = cost price for Ram = 40 + 32.5 = 72.5


Selling price for Ram at 10% loss Q = 50 + 25 = 75
90 So Q > P.
= 110 × = Rs 99.
100 31. (d) If the two watches have a time gap of 4 minutes
The cost price for Gopal will be Rs 99. in 1 day, the gap will increase to 1 hour (or
Difference between Gopal’s original cost price 60 minutes) in
and new cost price 1
× 60 = 15 days
= 100 – 99 = Rs 1 gain 4
1 The watches being 24-hour watches, they will
Profit per cent = × 1 = 1 per cent.
100 show identical times in
21. (d) 15 × 24 = 360 days.

22. (c) 32. (b) If 22 per cent of households earn more than
Rs 1,00,000 and 6 per cent earn more than Rs
23. (d) 2,00,000, percentage of households earning
24. (a) between Rs 1,00,000 and Rs 2,00,000 is
22 – 6 = 16.
25. (d) Percentage of households earning less than
Rs 30,000 is 12.
26. (d) Statement 1 is not supported by the passage.
So percentage of households earning between
Statement 2 is a mere paraphrase of the last
Rs 30,000 and Rs 1,00,000 is
line of the passage. So this is what the author
100 – (22 + 12) = 66.
is saying. It cannot be the assumption.
Now, it is given that 990 households earn
27. (b) between Rs 30,000 and Rs 1,00,000
Or 66 per cent is 990 households
@UPSC_THOUGHTS

28. (c) The weight of the 5th person is


(45 × 5 + 55 × 5) – (50 × 9) 990
∴ 16 per cent is × 16 = 240.
= 225 + 275 – 450 66
= 500 – 450 33. (b)
= 50 kg.
34. (d) From the given statement it cannot be concluded
29. (c) We are given that Amala and Komala are that all healthy people go for a morning walk.
unemployed. So they cannot be the bank It is possible that some people are healthy
employees. Now Komala and Nirmala are tennis without going for a morning walk.
players. We are also told that among Amala, Again, the second conclusion is not valid as the
Kamla, Komala and Nirmala, two are bank statement does not indicate morning walks are
employees. Vimala and Kamla are said to be ‘essential’.
bank employees. Amala and Komala being
unemployed, Nirmala is the third bank employee. 35. (a) If 39 is the mean of the first five consecutive
So clearly Nirmala is the tennis player and a odd numbers, there must be two odd numbers
bank employee. before it, namely 37 and 35. So the series
begins from 35. Since the series has 13
30. (d) The answer depends on how great the difference consecutive odd numbers, we have
between A and B is. 35, 37, 39, 41, 43, 45, 47, 49, 51,
Plug in some numbers and calculate. 53, 55, 57, 59
Say, A = 11 and B = 10, under the given N +1
conditions, The mean for odd N numbers = item
2
40 65 So in the 13 number series, the mean is
P = × 11 + × 10
100 100 13 + 1
item, i.e., the 7th item
= 4.4 + 6.5 = 10.9 2
Q = 5.5 + 5 = 10.5 The 7th item is 47.
So, P > Q. 36. (b) The cards possessed by the boys:
Now take A = 100 and B = 50 A 10 – 2 = 8
Under the given conditions, B 10 + 3 – 6 = 7
40 65 C 10 + 5 – 3 = 12
P = × 100 + × 50
100 100 D 10 + 6 – 1 = 15

ST . 133
Sample Test 7 : General Studies Paper II (2017)

E 10 + 1 = 11 40. (d)
F 10 + 2 – 5 = 7
41. (a)
∴ D + E = 15 + 11 = 26
B + C + F = 7 + 12 + 7 = 26 42. (c)
D + E = B + C + F.
1 43. (b)
37. (a) Water in the first sample = 50% = .
2 44. (d) This is the inference. Response (a) seems
Water in the second sample = 0
correct but the passage refers to the economies
Let the water in the mixture be x.
of Asia and is not referring to the world in
By alligation,
general.
1 0
2 45. (c)
46. (d)
x
47. (b)

1 48. (a)
0–x x–
2 49. (b) The numbers are 168 – 861; 259 – 952.
[Note the numbers that satisfy conditions 1 and
0−x Quantity of mixture
= 2, checking for condition 3 as you go on. You
1 Quantity of pure milk
x− may stop when you reach 259 whose reverse
2 – 952 – would most likely be the last such
1 3-digit number. Use the formula for divisibility
–x = x – by 7.]
2
1 50. (d) Given that statement 4 is true, Response (a)
–2x = –
@UPSC_THOUGHTS

2 is not correct. Responses (b) and (c) cannot be


1 definitely concluded. Response (d) can be
2x =
2 definitely concluded, as it is given as true that
1 some colours are not pleasant.
x =
4
or 25%. 51. (c) Between 99 and 1000, there will be 10 numbers
with 8 in the units place. There are 9 sets of
38. (c) The lines would form a figure as follows. hundreds between 99 and 1000, and in each
there will be 10 numbers with 8 in the units
place. So, totally there are 9 × 10 or 90 such
numbers.
52. (d)
53. (b) Most likely, the man’s present age is 26 years:
it fulfils the given conditions.
There will be 14 squares and 22 rectangles, a
Last year he was 52 or 25 years and next year
total of 36.
he will be 33 or 27 years.
39. (c) Let the two trains meet after t hours. The next time his age will be a cube of a number
∴ Distance covered by the express train in t hours is 64 or 43.
= distance covered by the freight train in The years he has to wait to become 64 is
t + 2 hours 64 – 26 = 38.
t × 60 = (t + 2) × 40
[as distance = time × speed] 54. (a) If Q finished the work in x days,
60t = 40t + 80 x
P can do it in days.
80 3
t = = 4 hours (P + Q)’s one day work
20
Distance covered by the express train is 4 hours 1 3 4
= + =
= 60 × 4 = 240 km x x x
[Distance covered by the freight train R’s one day work
= (4 + 2) × 40 or 6 × 40 = 240 km] 1 4 1
The two trains will meet 240 km from Delhi. = × =
4 x x

ST . 134
Sample Test 7 : General Studies Paper II (2017)

Ratio of efficiency 66. (b)


3 1 1
P : Q : R = : : 67. (b)
x x x
3 : 1 : 1 68. (c) In the last two days, the rainfall was
The ratio in which the three share the earnings (6 × 0.5) – (4 × 0.4)
will also be 3 : 1 : 1. 3.0 – 1.6 = 1.4 inches
As the ratio of rainfall on the fifth and sixth day
55. (b) It is clear from the diagram that A and D must was 4 : 3, rainfall on the fifth day
be married and A is the husband of D.
4
= × 1.4 = 0.80 inches
7
male male female female male female
Note: Make a chart to answer Questions 69, 70 and
71.
A B C D E F
Lecture City Subject
B son son daughter A E
of A (B) (C) sons A [Not Hyderabad] Economics
Mumbai [Not History or Math]
[You can get the answer just by studying the B Kanpur Geography
responses. Response (a) can be rejected as it
is given A is ‘son’ of F. Response (c) is not C Delhi Sociology
correct as it is given that E is child of F. D Shillong History or Math
Response (d) cannot be derived from the given E Hyderabad Math or History
data. But since D has two children—a boy and
a girl—and A has B as his son, and since A F Srinagar Commerce
is male and B is female, Response (b) would G Chennai Statistics
@UPSC_THOUGHTS

be the best choice.]


We can see that B can only be specialised in
56. (b) If you pick all 8 green balls and all 7 white balls,
Geography. So put in Geography against B. As
you would have picked 15 balls and exhausted
the Geography specialist is from Kanpur, B must
two colours. The next ball you pick has to be
be from Kanpur. Fill in Kanpur. That leaves us
a red one—the third colour. This is the only
with Mumbai and Hyderabad. As A is not from
assured way to pick balls of all colours.
Hyderabad, he is from Mumbai. Put Mumbai
57. (c) against A; so Hyderabad must be E’s city.
[In the above chart, the items in bold have been
58. (a)
filled in at the second stage.]
59. (c) The arrangement would be The questions can be answered now.
A C B D G F E H ↑ N 69. (a)
G is to the immediate right of D.
70. (b)
60. (b) (a) 256 red colour chalk
(b) 589 green colour flower 71. (b)
(c) 254 white colour chalk 72. (b) A chart will help.
We can see that ‘5’ is common to all the codes
A Hindi, English, Mathematics
and ‘colour’ is common in all the meanings; so
B Hindi, English, Geography, French, History
5 means colour. C English, Geography
From (a) and (c) we see 2 is common and so D Hindi, Mathematics
is chalk; 2 means chalk. E History, French
So in 254, 4 must be white.
Clearly, B teaches the maximum number of
61. (c) subjects.
62. (c) 73. (d) The number should be such that, when reversed,
63. (a) the smaller number is very slightly above half
the larger number. This will ensure a dividend
64. (c) of 1 and a large remainder. This is most likely
in the 90s. Try some numbers out. 91? No, 19
65. (a)

ST . 135
Sample Test 7 : General Studies Paper II (2017)

is too small. 92 and 93 are also similar. Take Draw a diagram to answer Questions 76 to 78.
94, the reverse is 49. 94 ÷ 49 gives a dividend
of 1 and a remainder of 45. Response (d) is
correct. [A remainder of 95 is unrealistic with
A E B D
a two-digit number.]
74. (b) Let the monthly incomes of X and Y be 4x and
3x respectively, and their monthly expenses 3y
and 2y respectively.
Under the given conditions, C G F H
4x – 3y = 3x – 2y
4x – 3x = 3y – 2y
x = y = 6,000 76. (b)
Total income of X and Y is 7x
Or 7 × 6,000 = 42,000 77. (d) FDHGCA; FDEA; FDECA; FDEGCA.

75. (a) Let the fly be x metres from the ceiling. 78. (c) The route will be HGBFDEC.
As per the conditions given, the distance between 79. (d) 14-41, 25-52, 36-63, 47-74, 58-85, 69-96 all fulfil
the fly and the ceiling will be the conditions. Clearly more than 5 such two-
82 + 12 + x 2 = 92 digit numbers are there.
64 + 1 + x 2 = 81
80. (b) One-digit 1 to 9 = 9
x 2 = 81 – 64 – 1
Two-digits 10 to 99 = 2 × 90 = 180
x 2 = 16
x = 4 m. Three-digits 100 to 150 = 3 × 51 = 153
Total = 9 + 180 + 153 = 342.
@UPSC_THOUGHTS

ST . 136
Sample Test 8 : General Studies Paper II (2018)

SAMPLE TEST 8: GENERAL STUDIES PAPER II (2018)


1. Consider the following three-dimensional figure: What is the ratio between the distances covered
by vehicles A and B in the time interval OL?
(a) 1 : 2 (b) 2 : 3
(c) 3 : 4 (d) 1 : 1
6. A train 200 metres long is moving at the rate of
40 kmph. In how many seconds will it cross a man
standing near the railway line?
(a) 12 (b) 15
(c) 16 (d) 18
Directions for the following 4 (four) items:
Read the following four passages and answer the items
that follow. Your answers to these items should be
How many triangles does the above figure have? based on the passages only.
(a) 18 (b) 20
(c) 22 (d) 24 Passage—1
2. Consider the following sum: Global population was around 1.6 billion in 1990—
• + 1• + 2• + •3 + •1 = 21• today it is around 7.2 billion and growing. Recent
In the above sum, • stands for
(a) 4 (b) 5 estimates on population growth predict a global
population of 9.6 billion in 2050 and 10.9 billion in 2100.
@UPSC_THOUGHTS

(c) 6 (d) 8
Unlike Europe and North America, where only three to
3. Consider the following pattern of numbers:
8 10 15 13 four per cent of population is engaged in agriculture,
6 5 7 4 around 47 per cent of India’s population is dependent
4 6 8 8 upon agriculture. Even if India continues to do well in
6 11 16 ? the service sector and the manufacturing sector picks up,
What is the number at ? in the above pattern? it is expected that around 2030 when India overtakes
(a) 17 (b) 19 China as the world’s most populous country, nearly 42
(c) 21 (d) 23 per cent of India’s population will still be predominantly
4. How many diagonals can be drawn by joining the dependent on agriculture.
vertices of an octagon?
(a) 20 (b) 24 7. Which of the following is the most logical and
(c) 28 (d) 64 rational inference that can be made from the above
5. The figure drawn below gives the velocity graphs passage?
of two vehicles A and B. The straight line OKP (a) Prosperity of agriculture sector is of critical
represents the velocity of vehicle A at any instant, importance to India.
whereas the horizontal straight line CKD represents (b) Indian economy greatly depends on its
the velocity of vehicle B at any instant. In the agriculture.
figure, D is the point where perpendicular from
(c) India should take strict measures to control
P meets the horizontal line CKD such that
its rapid population growth.
1
PD = LD: (d) India’s farming communities should switch
2 over to other occupations to improve their
P economic conditions.

Vehicle B
Passage—2
C K D
Velocity

Many pathogens that cause foodborne illnesses are


unknown. Food contamination can occur at any stage
le A
ehi c from farm to plate. Since most cases of food poisoning
V go unreported, the true extent of global foodborne
illnesses is unknown. Improvements in international
O Time L monitoring have led to greater public awareness, yet the

ST . 137
Sample Test 8 : General Studies Paper II (2018)

rapid globalisation of food production increases 10. The author wants India to rid herself of certain
consumers’ vulnerability by making food harder to past bonds because
regulate and trace. “We have the world on our plates”, (a) he is not able to see the relevance of the past
says an official of WHO. (b) there is not much to be proud of
(c) he is not interested in the history of India
8. Which of the following is the most logical corollary
(d) they obstruct her physical and spiritual growth
to the above passage?
(a) With more options for food come more risks. Directions for the following 3 (three) items:
(b) Food processing is the source of all foodborne The following three items are based on the graph given
illnesses. below which shows imports of three different types of
(c) We should depend on locally produced food steel over a period of six months of a year. Study the
only. graph and answer the three items that follow.
(d) Globalisation of food production should be 44
curtailed. 42

Thousands of tons imported


40
Passage—3 38
36
I am a scientist, privileged to be somebody who tries 34
to understand nature using the tools of science. But it 32
30
is also clear that there are some really important 28
questions that science cannot really answer, such as: 26
24
Why is there something instead of nothing? Why are we 22
here? In those domains, I have found that faith provides 20
January February March April May June
a better path to answers. I find it oddly anachronistic
@UPSC_THOUGHTS

that in today’s culture there seems to be a widespread Coil ($ 320) Sheet ($ 256) Scrap ($ 175)
presumption that scientific and spiritual views are
incompatible. The figures in the brackets indicate the average cost per
ton over six months period.
9. Which of the following is the most logical and
11. By how much (measured in thousands of tons) did
rational inference that can be made from the above
the import of sheet steel exceed the import of coil
passage?
steel in the first three months of the year?
(a) It is the faith and not science that can finally (a) 11 (b) 15
solve all the problems of mankind. (c) 19 (d) 23
(b) Science and faith can be mutually
12. What was the approximate total value (in $) of
complementary if their proper domains are
sheet steel imported over the six months period?
understood. (a) 45,555 (b) 50,555
(c) There are some very fundamental questions (c) 55,550 (d) 65,750
which cannot be answered by either science
13. What was the approximate ratio of sheet steel and
or faith.
scrap steel imports in the first three months of the
(d) In today’s culture, scientific views are given
year?
more importance than spiritual views. (a) 1 : 1 (b) 1.2 : 1
Passage—4 (c) 1.4 : 1 (d) 1.6 : 1

Though I have discarded much of past tradition and Directions for the following 3 (three) items :
custom, and am anxious that India should rid herself Rotated positions of a single solid are shown below. The
of all shackles that bind and contain her and divide her various faces of the solid are marked with different
people, and suppress vast numbers of them, and prevent symbols like dots, cross and line. Answer the three items
the free development of the body and the spirit; though that follow the given figures.
I seek all this, yet I do not wish to cut myself off from
that past completely. I am proud of that great inheritance
that has been and is, ours and I am conscious that I too,
like all of us, am a link in that unbroken chain which
goes back to the dawn of history in the immemorial past
of India. (I) (II) (III) (IV)

ST . 138
Sample Test 8 : General Studies Paper II (2018)

14. What is the symbol on the face opposite to that stress-free; and a massive programme for curricular
containing a single dot? reform should be initiated to provide for a child-friendly
(a) Four dots learning system, that is more relevant and empowering.
(b) Three dots Teacher accountability systems and processes must
(c) Two dots ensure that children are learning, and that their right to
(d) Cross learn in a child-friendly environment is not violated.
14. What is the symbol on the face opposite to that Testing and assessment systems must be reexamined
containing two dots? and redesigned to ensure that these do not force children
(a) Single dot to struggle between school and tuition centres, and
(b) Three dots bypass childhood.
(c) Four dots 17. According to the passage, which of the following
(d) Line is/are of paramount importance under the Right
15. What is the symbol on the face opposite to that to Education?
containing the cross? 1. Sending of children to school by all parents
(a) Single dot 2. Provision of adequate physical infrastructure
(b) Two dots in schools
(c) Line 3. Curricular reforms for developing child-
(d) Four dots friendly learning system
Directions for the following 4 (four) items: Select the correct answer using the code given
Read the following passage and answer the four items below.
that follow. Your answers to these items should be (a) 1 only
based on the passage only. (b) 1 and 2 only
@UPSC_THOUGHTS

Passage (c) 3 only


(d) None of the above
It is no longer enough for us to talk about providing
for universal access to education. Making available 18. With reference to the above passage, the following
schooling facilities is an essential prerequisite, but is assumptions have been made:
insufficient to ensure that all children attend school and 1. The Right to Education guarantees teachers’
participate in the learning process. The school may be accountability for the learning process of
there, but children may not attend or they may drop children.
out after a few months. Through school and social 2. The Right to Education guarantees 100%
mapping, we must address the entire gamut of social, enrolment of children in the schools.
economic, cultural and indeed linguistic and pedagogic 3. The Right to Education intends to take full
issues, factors that prevent children from weaker sections advantage of demographic dividend.
and disadvantaged groups, as also girls, from regularly Which of the above assumptions is/are valid?
attending and complementing elementary education.
(a) 1 only
The focus must be on the poorest and most vulnerable
(b) 2 and 3 only
since these groups are the most disempowered and at
(c) 3 only
the greatest risk of violation or denial of their right to
education. (d) 1, 2 and 3
The right to education goes beyond free and 19. According to the passage, which one of the
compulsory education to include quality education for following is critical in bringing quality in
all. Quality is an integral part of the right to education. education?
If the education process lacks quality, children are being (a) Ensuring regular attendance of children as
denied their right. The Right of Children to Free and well as teachers in school
Compulsory Education Act lays down that the curriculum (b) Giving pecuniary benefits to teachers to
should provide for learning through activities, motivate them
exploration and discovery. This places an obligation on
(c) Understanding the socio-cultural background
us to change our perception of children as passive
of children
receivers of knowledge, and to move beyond the
(d) Inculcating learning through activities and
convention of using textbooks as the basis of
discovery
examinations. The teaching-learning process must become

ST . 139
Sample Test 8 : General Studies Paper II (2018)

20. What is the essential message in this passage? Directions for the following 7 (seven) items:
(a) The Right to Education now is a Fundamental Read the following four passages and answer the items
Right. that follow. Your answers to these items should be
(b) The Right to Education enables the children based on the passages only.
of poor and weaker sections of the society to
attend schools. Passage—1
(c) The Right to Free and Compulsory Education ‘Desertification’ is a term used to explain a process of
should include quality education for all. decline in the biological productivity of an ecosystem,
(d) The Government as well as parents should leading to total loss of productivity. While this
ensure that all children attend schools. phenomenon is often linked to the arid, semi-arid and
21. If LSJXVC is the code for MUMBAI, the code for sub-humid ecosystems, even in the humid tropics, the
DELHI is impact could be most dramatic. Impoverishment of
(a) CCIDD (b) CDKGH human-impacted terrestrial ecosystems may exhibit itself
(c) CCJFG (d) CCIFE in a variety of ways: accelerated erosion as in the
mountain regions of the country, salinisation of land as
22. If RAMON is written as 12345 and DINESH as
in the semi-arid and arid ‘green revolution’ areas of the
675849, then HAMAM will be written as
(a) 92233 (b) 92323 country, e.g., Haryana and western Uttar Pradesh, and
(c) 93322 (d) 93232 site quality decline—a common phenomenon due to
general decline in tree cover and monotonous
23. If X is between –3 and –1, and Y is between monoculture of rice/wheat across the Indian plains. A
–1 and 1, then X2 – Y2 is in between which of the major consequence of deforestation is that it relates to
following?
adverse alterations in the hydrology and related soil and
@UPSC_THOUGHTS

(a) –9 and 1 (b) –9 and –1


nutrient losses. The consequences of deforestation
(c) 0 and 8 (d) 0 and 9
invariably arise out of site degradation through erosive
24. X and Y are natural numbers other than 1, and losses. Tropical Asia, Africa and South America have the
Y is greater than X. Which of the following highest levels of erosion. The already high rates for the
represents the largest number? tropics are increasing at an alarming rate (e.g., through
(a) XY (b) X/Y the major river systems—Ganga and Brahmaputra, in the
(c) Y/X (d) (X + Y)/XY Indian context), due to deforestation and ill-suited land
Directions for the following 2 (two) items: management practices subsequent to forest clearing. In
the mountain context, the declining moisture retention
Read the following information and answer the two
of the mountain soils, drying up of the underground
items that follow.
springs and smaller rivers in the Himalayan region
The plan of an office block for six officers A, B, C, D, could be attributed to drastic changes in the forest cover.
E and F is as follows : Both B and C occupy offices to
An indirect consequence is drastic alteration in the
the right of the corridor (as one enters the office block)
upland-lowland interaction, mediated through water.
and A occupies on the left of the corridor. E and F
The current concern the tea planter of Assam has is
occupy offices on opposite sides of the corridor but their
about the damage to tea plantations due to frequent
offices do not face each other. The offices of C and D
inundation along the flood-plains of Brahmaputra, and
face each other. E does not have a corner office. F’s office
the damage to tea plantation and the consequent loss
is further down the corridor than A’s, but on the same
in tea productivity is due to rising level of the river
side.
bottom because of siltation and the changing course of
25. If E sits in his office and faces the corridor, whose the river system. The ultimate consequences of site
office is to his left? desertification are soil degradation, alteration in available
(a) A (b) B water and its quality, and the consequent decline in
(c) C (d) D food, fodder and fuel-wood yields essential for the
26. Who is/are F’s immediate neighbour/neighbours? economic well-being of rural communities.
(a) A only (b) A and D 27. According to the passage, which of the following
(c) C only (d) B and C are the consequences of decline in forest cover?

ST . 140
Sample Test 8 : General Studies Paper II (2018)

1. Loss of topsoil are needed to conserve a wider pool of genetic resources


2. Loss of smaller rivers of existing crops, breeds, and their wild relatives.
3. Adverse effect on agricultural production Relatively intact ecosystems, such as forested catchments,
4. Declining of groundwater mangroves, wetlands, can buffer the impacts of climate
change. Under a changing climate, these ecosystems are
Select the correct answer using the code given
themselves at risk, and management approaches will
below.
need to be more proactive and adaptive. Connections
(a) 1, 2 and 3 only
between natural areas, such as migration corridors, may
(b) 2, 3 and 4 only
be needed to facilitate species movements to keep up
(c) 1 and 4 only
with the change in climate.
(d) 1, 2, 3 and 4
30. With reference to the above passage, which of the
28. Which of the following is/are the correct inference/
following would assist us in coping with the
inferences that can be made from the passage?
climate change?
1. Deforestation can cause changes in the course
1. Conservation of natural water sources
of rivers. 2. Conservation of wider gene pool
2. Salinisation of land takes place due to human 3. Existing crop management practices
activities only. 4. Migration corridors
3. Intense monoculture practice in plains is a
Select the correct answer using the code given
major reason for desertification in Tropical
below.
Asia, Africa and South America.
(a) 1, 2 and 3 only
Select the correct answer using the code given (b) 1, 2 and 4 only
below. (c) 3 and 4 only
@UPSC_THOUGHTS

(a) 1 only (d) 1, 2, 3 and 4


(b) 1 and 2 only
31. With reference to the above passage, the following
(c) 2 and 3 only
assumptions have been made:
(d) None of the above is a correct inference
1. Diversification of livelihoods acts as a coping
29. With reference to ‘desertification’, as described in strategy for climate change.
the passage, the following assumptions have been 2. Adoption of monocropping practice leads to
made: the extinction of plant varieties and their wild
1. Desertification is a phenomenon in tropical relatives.
areas only. Which of the above assumptions is/are valid?
2. Deforestation invariably leads to floods and (a) 1 only
desertification. (b) 2 only
Which of the above assumptions is/are valid? (c) Both 1 and 2
(a) 1 only (d) Neither 1 nor 2
(b) 2 only Passage—3
(c) Both 1 and 2
Today, the top environmental challenge is a combination
(d) Neither 1 nor 2
of people and their aspirations. If the aspirations are
Passage—2 more like the frugal ones we had after the Second World
A diversity of natural assets will be needed to cope with War, a lot more is possible than if we view the planet
climate change and ensure productive agriculture, as a giant shopping mall. We need to get beyond the
forestry, and fisheries. For example, crop varieties are fascination with glitter and understand that the planet
needed that perform well under drought, heat, and works as a biological system.
enhanced CO2. But the private-sector and farmer-led 32. Which of the following is the most crucial and
process of choosing crops favours homogeneity adapted logical inference that can be made from the above
to past or current conditions, not varieties capable of passage?
producing consistently high yields in warmer, wetter, (a) The Earth can meet only the basic needs of
or drier conditions. Accelerated breeding programmes humans for food, clothing and shelter.

ST . 141
Sample Test 8 : General Studies Paper II (2018)

(b) The only way to meet environmental challenge (c) Pencil = Cap
is to limit human population. (d) Pencil > Cap
(c) Reducing our consumerism is very much in 38. A bookseller sold ‘a’ number of Geography
our own interest. textbooks at the rate of ` x per book, ‘a + 2’
(d) Knowledge of biological systems can only number of History textbooks at the rate of
help us save this planet.
` (x + 2) per book and ‘a – 2’ number of
Passage—4 Mathematics textbooks at the rate of `(x – 2) per
book. What is his total sale in ` ?
Some people believe that leadership is a quality which
(a) 3x + 3a (b) 3ax + 8
you have at birth or not at all. This theory is false, for
the art of leadership can be acquired and can indeed (c) 9ax (d) x 3a3
be taught. This discovery is made in time of war and 39. A bag contains 15 red balls and 20 black balls.
the results achieved can surprise even the instructors. Each ball is numbered either 1 or 2 or 3. 20% of
Faced with the alternatives of going left or right, every the red balls are numbered 1 and 40% of them
soldier soon grasps that a prompt decision either way are numbered 3. Similarly, among the black balls,
is better than an endless discussion. A firm choice of 45% are numbered 2 and 30% are numbered
direction has an even chance of being right while to do 3. A boy picks a ball at random. He wins if the
nothing will be almost certainly wrong. ball is red and numbered 3 or if it is black and
33. The author of the passage holds the view that numbered 1 or 2. What are the chances of his
(a) leadership can be taught through war winning?
experience only 1 4
(a) (b)
(b) leadership can be acquired as well as taught 2 7
(c) the results of training show that more people
@UPSC_THOUGHTS

5 12
acquire leadership than are expected (c) (d)
9 13
(d) despite rigorous instruction, very few leaders
40. Two persons, A and B are running on a circular
are produced
track. At the start, B is ahead of A and their
34. A number consists of three digits of which the positions make an angle of 30° at the centre of
middle one is zero and their sum is 4. If the the circle. When A reaches the point diametrically
number formed by interchanging the first and last opposite to his starting point, he meets B. What
digits is greater than the number itself by 198, then is the ratio of speeds of A and B, if they are
the difference between the first and last digits is running with uniform speeds?
(a) 1 (b) 2 (a) 6 : 5 (b) 4 : 3
(c) 3 (d) 4 (c) 6 : 1 (d) 4 : 2
35. A solid cube of 3 cm side, painted on all its faces, 41. A student has to get 40% marks to pass in an
is cut up into small cubes of 1 cm side. How many examination. Suppose he gets 30 marks and fails
of the small cubes will have exactly two painted by 30 marks, then what are the maximum marks
faces?
in the examination?
(a) 12 (b) 8
(a) 100 (b) 120
(c) 6 (d) 4
(c) 150 (d) 300
36. While writing all the numbers from 700 to 1000,
42. 19 boys turn out for playing hockey. Of these, 11
how many numbers occur in which the digit at
are wearing hockey shirts and 14 are wearing
hundred’s place is greater than the digit at ten’s
hockey pants. There are no boys without shirts
place, and the digit at ten’s place is greater than
and/or pants. What is the number of boys wearing
the digit at unit’s place?
full uniform?
(a) 61 (b) 64
(a) 3 (b) 5
(c) 85 (d) 91
(c) 6 (d) 8
37. If Pen < Pencil, Pencil < Book and Book > Cap,
Directions for the following 6 (six) items:
then which one of the following is always true?
(a) Pen > Cap Read the information given below and answer the six
(b) Pen < Book items that follow.

ST . 142
Sample Test 8 : General Studies Paper II (2018)

A, B, C and D are students. They are studying in to design and implement adaptation and mitigation
four different cities, viz., P, Q, R and S (not necessarily plans and projects. The problem is more severe for
in that order). They are studying in Science college, Arts developing countries like India, which would be one of
college, Commerce college and Engineering college (not the hardest hit by climate change, given its need to
necessarily in that order), which are situated in four finance development. Most countries do indeed treat
different States, viz., Gujarat, Rajasthan, Assam and climate change as real threat and are striving to address
Kerala (not necessarily in that order). Further, it is given it in a more comprehensive and integrated manner with
that— the limited resources at their disposal.
(i) D is studying in Assam
49. With reference to the above passage, the following
(ii) Arts college is located in city S which is in
assumptions have been made:
Rajasthan
(iii) A is studying in Commerce college 1. Climate change is not a challenge for
(iv) B is studying in city Q developed countries.
(v) Science college is located in Kerala 2. Climate change is a complex policy issue and
also a development issue for many countries.
43. A is studying in
3. Ways and means of finance must be found
(a) Rajasthan (b) Gujarat
to enable developing countries to enhance
(c) city Q (d) Kerala
their adaptive capacity.
44. Science college is located in
Which of the above assumptions is/are valid?
(a) city Q (b) city S
(a) 1 and 2 only
(c) city R (d) city P
(b) 3 only
45. C is studying in (c) 2 and 3 only
(a) Science college
@UPSC_THOUGHTS

(d) 1, 2 and 3
(b) Rajasthan
(c) Gujarat Passage—2
(d) city Q Cooking with biomass and coal in India is now
46. Which one of the following statements is correct? recognised to cause major health problems, with women
(a) D is not studying in city S. and children in poor populations facing the greatest risk.
(b) A is studying in Science college. There are more than 10 lakh premature deaths each year
(c) A is studying in Kerala. from household air pollution due to polluting cooking
(d) Engineering college is located in Gujarat. fuels with another 1.5 lakh due to their contribution to
47. Which one of the following statements is correct general outdoor air pollution in the country. Although
regarding Engineering college? the fraction of the Indian population using clean cooking
(a) C is studying there. fuels, such as LPG, natural gas and electricity, is slowly
(b) B is studying there. rising, the number using polluting solid fuels as their
(c) It is located in Gujarat. primary cooking fuel has remained static for nearly 30
(d) D is studying there. years at about 70 crore.
48. Which one of the following statements is correct? 50. Which of the following is the most crucial and
(a) Engineering college is located in Assam. logical inference that can be made from the above
(b) City Q is situated in Assam. passage?
(c) C is studying in Kerala. (a) Rural people are giving up the use of
(d) B is studying in Gujarat. polluting solid fuels due to their increasing
Directions for the following 8 (eight) items: awareness of health hazards.
(b) Subsidising the use of clean cooking fuels
Read the following eight passages and answer the items
will solve the problem of India’s indoor air
that follow. Your answers to these items should be
pollution.
based on the passages only.
(c) India should increase its import of natural
Passage—1 gas and produce more electricity.
All actions to address climate change ultimately involve (d) Access to cooking gas can reduce premature
costs. Funding is vital in order for countries like India deaths in poor households.

ST . 143
Sample Test 8 : General Studies Paper II (2018)

Passage—3 years, the subsidy to producers will exceed 10 per cent


of the value of agricultural production.
Scientific knowledge has its dangers, but so has every
great thing. Over and beyond the dangers with which 53. What is the crucial message conveyed by the
it threatens the present, it opens up as nothing else can, above passage?
the vision of a possible happy world; a world without (a) India should revise its PDS.
poverty, without war, with little illness. Science, whatever (b) India should not be a member of WTO.
unpleasant consequences it may have by the way, is in (c) For India, food security collides with trade.
its very nature a liberator. (d) India provides food security to its poor.

51. Which one of the following is the most important Passage—6


implication of the passage? India’s educational system is modelled on the mass
(a) A happy world is a dream of science. education system that developed in the 19th century in
(b) Science only can build a happy world, but Europe and later spread around the world. The goal of
it is also the only major threat. the system is to condition children as ‘good’ citizens and
(c) A happy world is not possible without productive workers. This suited the industrial age that
science. needed the constant supply of a compliant workforce
(d) A happy world is not at all possible with or with a narrow set of capabilities. Our educational
without science. institutes resemble factories with bells, uniforms and
Passage—4 batch-processing of learners, designed to get learners to
conform. But, from an economic point of view, the
The Arctic’s vast reserves of fossil fuel, fish and minerals
environment today is very different. It is a complex,
are now accessible for a longer period in a year. But
volatile and globally interconnected world.
unlike Antarctica, which is protected from exploitation
54. With reference to the above passage, the following
@UPSC_THOUGHTS

by the Antarctic Treaty framed during the Cold War and


is not subject to territorial claims by any country, there assumptions have been made:
is no legal regime protecting the Arctic from 1. India continues to be a developing country
industrialisation, especially at a time when the world essentially due to its faulty education system.
craves for more and more resources. The distinct 2. Today’s learners need to acquire new-age
possibility of ice-free summer has prompted countries skill-sets.
with Arctic coastline to scramble for great chunks of the 3. A good number of Indians go to some
melting ocean. developed countries for education because
the educational systems there are a perfect
52. Which one of the following is the most important
reflection of the societies in which they
implication of the passage?
function.
(a) India can have territorial claims in the Arctic
territory and free access to its resources. Which of the above assumptions is/are valid?
(b) Melting of summer ice in the Arctic leads to (a) 1 and 3 only
changes in the geopolitics. (b) 2 only
(c) The Arctic region will solve the world’s (c) 2 and 3 only
future problem of resource crunch. (d) 1, 2 and 3
(d) The Arctic region has more resources than
Passage—7
Antarctica.
The practice of dieting has become an epidemic; everyone
Passage—5 is looking out for a way to attain that perfect body. We
Being a member of the WTO, India is bound by the are all different with respect to our ethnicity, genetics,
agreements that have been signed and ratified by its family history, gender, age, physical and mental and
members, including itself. According to Article 6 of the spiritual health status, lifestyles and preferences. Thereby
Agriculture Agreement, providing minimum support we also differ in what foods we tolerate or are sensitive
prices for agricultural products is considered distorting to. So we really cannot reduce so many complexities into
and is subject to limits. The subsidy arising from one diet or diet book. This explains the failure of diets
‘minimal supports’ cannot exceed 10 per cent of the across the world in curbing obesity. Unless the reasons
value of agricultural production for developing countries. for weight gain are well understood and addressed and
PDS in India entails minimum support prices and public unless habits are changed permanently, no diet is likely
stockholding of food grains. It is possible that, in some to succeed.

ST . 144
Sample Test 8 : General Studies Paper II (2018)

55. What is the most logical and rational inference between adjacent squares. What is the area of each
that can be made from the above passage? square?
(a) Obesity has become an epidemic all over the 5
(a)
sq cm
world. 7
(b) A lot of people are obsessed with attaining 7
(b) sq cm
a perfect body. 5
(c) Obesity is essentially an incurable disease. (c) 1 sq cm
(d) There is no perfect diet or one solution for 25
obesity. (d) sq cm
12
Passage—8 61. Consider’ the following graph:
Monoculture carries great risks. A single disease or pest 100%
can wipe out swathes of the world’s food production,
an alarming prospect given that its growing and wealthier Expected

% of work
75% progress
population will eat 70% more by 2050. The risks are
magnified by the changing climate. As the planet warms 50%
and monsoon rains intensify, farmlands in Asia will Actual
flood. North America will suffer more intense droughts, progress
25%
and crop diseases will spread to new latitudes.
56. Which of the following is the most logical, rational 0%
and crucial message given by the passage? 1st April May June July Aug
(a) Preserving crop genetic diversity is an
Which one of the following statements is not correct with
insurance against the effects of climate change. reference to the graph given above?
@UPSC_THOUGHTS

(b) Despite great risks, monoculture is the only (a) On 1st June, the actual progress of work was
way to ensure food security in the world. less than expected.
(c) More and more genetically modified crops (b) The actual rate of progress of work was the
only can save the world from impending greatest during the month of August.
shortages of food. (c) The work was actually completed before the
(d) Asia and North America will be worst expected time.
sufferers from climate change and the (d) During the period from 1st April to 1st
consequent shortage of food. September, at no time was the actual progress
57. A shopkeeper sells an article at ` 40 and gets X% more than the expected progress.
profit. However, when he sells it at ` 20, he faces 62. For a sports meet, a winners’ stand comprising
same percentage of loss. What is the original cost three wooden blocks is in the following form:
of the article?
(a) ` l0 (b) ` 20
(c) ` 30 (d) ` 40
58. There are 24 equally spaced points lying on the
circumference of a circle. What is the maximum
number of equilateral triangles that can be drawn
by taking sets of three points as the vertices? There are six different colours available to choose
(a) 4 (b) 6 from and each of the three wooden blocks is to
(c) 8 (d) 12 be painted such that no two of them has the same
59. Consider the sequence given below: colour. In how many different ways can the
4/12/95, 1/1/96, 29/1/96, 26/2/96..... winners’ stand be painted?
What is the next term of the series? (a) 120 (b) 81
(a) 24/3/96 (b) 25/3/96 (c) 66 (d) 36
(c) 26/3/96 (d) 27/3/96 Directions for the following 2 (two) items:
60. Twelve equal squares are placed to fit in a Consider the following graph in which the birth rate and
rectangle of diagonal 5 cm. There are three rows death rate of a country are given, and answer the two
containing four squares each. No gaps are left items that follow.

ST . 145
Sample Test 8 : General Studies Paper II (2018)

30 E
40
Birth and death rates per thousand

30
Birth rate
20
Hourly
earnings 20
Death rate
10
10

2013 14 15 16 17 Y
Years
0 Fig. B
1970 1980 1990 2000 2010 From the figures, it is observed that the
Years
63. Looking at the graph, it can be inferred that from (a) values of E are different
1990 to 2010 (b) ranges (i.e., the difference between the
(a) population growth rate has increased maximum and the minimum) of E are different
(b) population growth rate has decreased (c) slopes of the graphs are same
(c) growth rate of population has remained stable (d) rates of increase of E are different
(d) population growth rate shows no trend
66. Consider the figures given below:
64. With reference to the above graph, consider the
following statements considering 1970 as base
@UPSC_THOUGHTS

year:
1. Population has stabilised after 35 years.
2. Population growth rate has stabilised after 35
years.
3. Death rate has fallen by 10% in the first 10
years.
4. Birthrate has stabilised after 35 years. ?
Which of the above are the most logical and
rational statements that can be made from the
above graph? To fit the question mark, the correct answer is
(a) 1 and 2 only
(b) 1, 2 and 3 (a) (b)
(c) 3 and 4
(d) 2 and 4 (c) (d)
65. Average hourly earnings per year (E) of the
workers in a firm are represented in figures A and 67. Consider the following figures A and B:
B as follows:
8
Cost of production

E 7
(` in lakhs)

6
40
5
Hourly 35
4
earnings 30
25
20 1000 2000 3000
2013 14 15 16 17 Y No. of pieces manufactured
Years Fig. A
Fig. A

ST . 146
Sample Test 8 : General Studies Paper II (2018)

500
Selling price per

450 (a) (b)


pieces (`)

400
350
300
(c) (c)
1000 2000 3000
No. of pieces sold
Fig. B 71. Consider the following graphs. The curves in the
The manufacturing cost and projected sales for a graphs indicate different age groups in the
product are shown in the above figures A and B populations of two countries A and B over a
respectively. What is the minimum number of period of few decades:
pieces that should be manufactured to avoid a Country A
1.2
loss?

Population (in million s)


(a) 2000 (b) 2500 1.0
(c) 3000 (d) 3500 0.8
68. A lift has the capacity of 18 adults or 30 children. 0.6
How many children can board the lift with 12 0.4
adults? 0.2
@UPSC_THOUGHTS

(a) 6 (b) 10
0
(c) 12 (d) 15
1950 1970 1990 2010 2030 2050
69. A person bought a refrigerator worth ` 22,800 with Years
12.5% interest compounded yearly. At the end of <15 15–64 64+
first year, he paid ` 8,650 and at the end of second
year ` 9,125. How much will he have to pay at Country B
the end of third year to clear the debt? 1.2
Population (in millions )

(a) ` 9,990 (b) ` 10,000 1.0


(c) ` 10,590 (d) ` 11,250 0.8
70. Consider the following figures: 0.6
0.4
0.2
0
1950 1970 1990 2010 2030 2050
Years
(I) (II) (III)
<15 15–64 64+

With reference to the above graphs, which of the


following are the most logical and rational
inferences that can be made?
1. Over the last two and a half decades, the
(IV) (V) (VI)
dependency ratio for country B has decreased.
In the figures (I) to (VI) above, some parts are 2. By the end of next two and a half decades,
shown to change their positions in regular the dependency ratio of country A will be
directions. Following the same sequence, which of much less than that of country B.
the figures given below will appear at (VII) stage? 3. In the next two decades, the workforce relative

ST . 147
Sample Test 8 : General Studies Paper II (2018)

to its total population will increase in country Directions for the following 2 (two) items:
B as compared to country A. The following table gives the GDP growth rate and Tele-
Select the correct answer using the code given density data of different States of a country in a
below. particular year. Study the table and answer the two
(a) 1 and 2 only items that follow.
(b) 2 and 3 only
(c) 1 and 3 only States Per capita GDP growth Tele-density
(d) 1, 2 and 3
income ($) rate (%)
72. Lakshmi, her brother, her daughter and her son
are badminton players. A game of doubles is State 1 704 9.52 70.27
about to begin: State 2 419 5.31 35.88
(i) Lakshmi’s brother is directly across the net
State 3 254 10.83 50.07
from her daughter.
(ii) Her son is diagonally across the net from the State 4 545 9.78 5.94
worst player’s sibling. State 5 891 10.8 76.12
(iii) The best player and the worst player are on
State 6 1077 11.69 77.5
the same side of the net.
State 7 900 8.88 104.86
Who is the best player?
(a) Her brother State 8 395 5.92 6
(b) Her daughter State 9 720 7.76 82.25
(c) Her son
State 10 893 9.55 96.7
(d) Lakshmi
@UPSC_THOUGHTS

State 11 363 4.7 57.7


73. The graph given below indicates the changes in
key policy rates made by the Central Bank several State 12 966 7.85 63.8
times in a year: State 13 495 9.37 52.3
State 14 864 5.46 97.9
Key Policy rate in % 7.50
(+.25) State 15 497 7.48 62.3
7.50 7.25
State 16 777 7.03 93.8
7.00 Repo Rate State 17 335 5.8 49.9
6.50 6.25 State 18 599 7.49 47.84
6.00 CRR
6.00
6.00 6.00 74. With reference to the above table, which of the
5.50 (0)
Reverse Repo Rate following is/are the most logical and rational
5.50
5.00 inference/inferences that can be made?
1. Higher per capita income is generally
4.50 associated with higher Tele-density.
4.00 2. Higher GDP growth rate always ensures
4.00 higher per capita income.
3.50 3. Higher GDP growth rate does not necessarily
3.00 ensure higher Tele-density.
Jul 2 Jul 27 Sep 16 Nov 2 Dec 16 Jan 25 Mar 17 May 3 Jun 16
Select the correct answer using the code given
2010 2011
below.
Which one of the following can be the most likely (a) 1 only (b) 2 and 3
reason for the Central Bank for such an action? (c) 1 and 3 (d) 3 only
(a) Encouraging foreign investment
75. With reference to the above table, the following
(b) Increasing the liquidity assumptions have been made:
(c) Encouraging both public and private savings 1. Nowadays, prosperity of an already high
(d) Anti-inflationary stance performing State cannot be sustained without

ST . 148
Sample Test 8 : General Studies Paper II (2018)

making further large investments in its 3. If x is negative, y must be positive for any
telecom infrastructure. value of x and y.
2. Nowadays, a very high Tele-density is the
Select the correct answer using the code given
most essential condition for promoting the
below.
business and economic growth in a State.
(a) 1 only
Which of the above assumptions is/are valid?
(b) 2 only
(a) 1 only
(c) Both 1 and 2
(b) 2 only
(d) Neither 1 nor 2 nor 3
(c) Both 1 and 2
(d) Neither 1 nor 2 Directions for the following 3 (three) items:
76. The following graph indicates the composition of Read the following two passages and answer the items
our tax revenue for a period of two decades: that follow. Your answers to these items should be
45 based on the passages only.
Per cent of gross tax revenue

40
35 Passage—1
30
The quest for cheap and plentiful meat has resulted in
25
20 factory farms where more and more animals are squeezed
15 into smaller lots in cruel and shocking conditions. Such
10 practices have resulted in many of the world’s health
5
0
pandemics such as the avian flu. Worldwide, livestock
are increasingly raised in cruel, cramped conditions,
1990–91

1995–96

2003–04

2004–05

2005–06

2006–07

2007–08

2008–09

2009–10

2010–11
(BE)
(Prov)

where animals spend their short lives under artificial


@UPSC_THOUGHTS

light, pumped full of antibiotics and growth hormones,


Years until the day they are slaughtered. Meat production is
Excise Customs Corporate Personal Service water-intensive. 15000 litres of water is needed for every
tax income tax tax kilogram of meat compared with 3400 litres for rice, 3300
With reference to the above graph, which of the litres for eggs and 255 litres for a kilogram of potatoes.
following is/are the most logical and rational 78. What is the most rational and crucial message
inference/inferences that can be made? given by the passage?
1. During the given period, the revenue from (a) Mass production of meat through industrial
Direct Taxes as percentage of gross tax revenue farming is cheap and is suitable for providing
has increased while that of Indirect Taxes protein nutrition to poor countries.
decreased.
(b) Meat-producing industry violates the laws
2. The trend in the revenue from Excise Duty
against cruelty to animals.
demonstrates that the growth of
(c) Mass production of meat through industrial
manufacturing sector has been negative during
farming is undesirable and should be stopped
the given period.
immediately.
Select the correct answer using the code given (d) Environmental cost of meat production is
below. unsustainable when it is produced through
(a) 1 only industrial farming.
(b) 2 only
(c) Both 1 and 2 Passage—2
(d) Neither 1 nor 2 A male tiger was removed from Pench Tiger Reserve
77. If x – y = 8, then which of the following must be and was relocated in Panna National Park. Later, this
true? tiger trekked towards his home 250 miles away. The trek
1. Both x and y must be positive for any value of this solitary tiger highlights a crisis. Many wildlife
of x and y. reserves exist as islands of fragile habitat in a vast sea
2. If x is positive, y must be negative for any of humanity, yet tigers can range over a hundred miles,
value of x and y. seeking prey, mates and territory. Nearly a third of

ST . 149
Sample Test 8 : General Studies Paper II (2018)

India’s tigers live outside tiger reserves, a situation that 80. With reference to the above passage, the following
is dangerous for both human and animal. Prey and assumptions have been made:
tigers can only disperse if there are recognised corridors 1. The strategy of conservation of wildlife by
of land between protected areas to allow unmolested relocating them from one protected area to
passage. another is not often successful.
79. Which of the following is the most rational and 2. India does not have suitable legislation to
crucial message given by the passage? save the tigers, and its conservation efforts
(a) The conflict between man and wildlife cannot have failed which forced the tigers to live
be resolved, no matter what efforts we make. outside protected areas.
(b) Safe wildlife corridors between protected areas
is an essential aspect of conservation efforts. Which of the above assumptions is/are valid?
(c) India needs to declare more protected areas (a) 1 only
and set up more tiger reserves. (b) 2 only
(d) India’s National Parks and Tiger Reserves (c) Both 1 and 2
need to be professionally managed. (d) Neither 1 nor 2

@UPSC_THOUGHTS

ST . 150
Sample Test 8 : General Studies Paper II (2018)

PRACTICE ANSWER SHEET

Directions: Use black ball pen. Example

1. 17. 33. 49. 65.

2. 18. 34. 50. 66.

3. 19. 35. 51. 67.

4. 20. 36. 52. 68.

5. 21. 37. 53. 69.

6. 22. 38. 54. 70.


@UPSC_THOUGHTS

7. 23. 39. 55. 71.

8. 24. 40. 56. 72.

9. 25. 41. 57. 73.

10. 26. 42. 58. 74.

11. 27. 43. 59. 75.

12. 28. 44. 60. 76.

13. 29. 45. 61. 77.

14. 30. 46. 62. 78.

15. 31. 47. 63. 79.

16. 32. 48. 64. 80.

ST . 151
Sample Test 8 : General Studies Paper II (2018)

ANSWERS AND EXPLANATORY NOTES


1. (b) There are two sets of triangles to be seen. vertices, you may count 8 × 5 = 40 diagonals,
Those in the front, in solid lines, number 2 (at but then you are counting each diagonal twice
bottom) + 5 (in the middle) +3 (at top). At the (AD is the same as DA, AE is the same as EA,
back are the triangles in dotted lines: 40
and so on). So there are or 20 diagonals.]
3 (at bottom) + 5 (in the middle) + 2 (at top). 2
Total is 20. 5. (c) From the figure, it is clear that the velocity (or
2. (d) The unit digit on the RHS needs to be the same speed) at which vehicle B is travelling is constant.
as the digit to replace the • on the LHS. Try Let us assume this velocity to be 10 kmph.
out the options, counting only the units digits on Vehicle A has moved from 0 to 15 kmph in the
the LHS. same time
(a) 4 + 4 + 4 + 3 + 1 → 1 6 4 1
(Given that PD = LD and LD = 10, PD = 5
(b) 5 + 5 + 5 + 3 + 1 → 1 9 5 2
and LP = 15)
(c) 6 + 6 + 6 + 3 + 1 → 2 2 6 15
Average speed of A = = 7.5 kmph
(d) 8 + 8 + 8 + 3 + 1 → 2 8 8 2
Ratio of speeds of A and B = 7.5 : 10
Check: 8 + 18 + 28 + 83 + 81 = 218 or 3 : 4.
3. (a) The pattern for all the columns is: Now, time being the same, ratios of speed and
Top number – middle number + bottom number distance will be the same, i.e. 3 : 4
= resultant number distance
8 – 6 + 4 = 6 (Velocity or speed = ).
time
@UPSC_THOUGHTS

10 – 5 + 6 = 11
15 – 7 + 8 = 16 6. (d) The distance covered = the length of the moving
So, body = 200 m
13 – 4 + 8 = 17
length of the moving body
4. (a) The number of diagonals of a polygon with n Now, speed =
number of sides time (t )
n (n − 3) We need to convert all figures into the same
= units – metre and seconds.
2
In this case, n = 8
40 × 1000
8(8 − 3) 40 So, 40 kmph = m/s
∴ Number of diagonals = = 20 60 × 60
2 2 40 × 1000 200
[Refer to the chapter on Mensuration (under ∴ =
60 × 60 t
Regular Polygons) in the section, Basic
Numeracy).] 200 × 60 × 60
[If you don’t remember the formula, just draw an t = = 18 seconds
40 × 1000
octagon
A [Refer to the chapter on Time and Distance in
B
the section, Basic Numeracy.]
7. (a) The answer could be (b) also. However, the
H C line, “Even if India continues to do well in …”,
does imply that ‘doing well’ is of importance; so
(a) is a better choice. Option (b) is merely stating
a fact, not an inference.
G D 8. (a) Option (a) follows from the passage.
9. (b)
F E 10. (d)
Starting from one vertex, you will see five 11. (c) Read the difference between sheet steel and
vertices are non-adjacent, making possible five coil steel on the graph for January, February
diagonals from one vertex. So from each vertex, and March and add.
five diagonals can be drawn. As there are 8 In January the difference is 10 (40 – 30)

ST . 152
Sample Test 8 : General Studies Paper II (2018)

In February the difference is 6 (37 – 31) [Since E cannot be in a corner office, the office
In March the difference is 3 (36 – 33) has to be in the middle. As E and F do not face
The total difference is 19 (in thousand tons). each other and F is further than A on the same
side, E is on the right side, E must be faced
12. (c) Total import of sheet metal in the given period
by A.]
(in thousand tons)
Now, you can answer the question items.
= 40 + 37 + 36 + 36 + 34 + 34 = 217
Value = 217 × 256 = 55,552 25. (c)
which is approximately 55,550.
26. (a)
13. (b) The ratio is 113 : 98 which is closest to
27. (d)
1.2 : 1.
28. (a) The passage does not imply that salinisation of
14. (b) From figures I and II it is clear three dots are
land takes place only due to human activities.
opposite one dot.
29. (d)
15. (c) This is clear from figures II and III.
30. (b)
16. (c) Figures I and IV make this clear.
31. (b)
17. (c)
32. (c) Refer to the sentence, “… a lot more … shopping
18. (d) The question asks for the ‘assumptions’ made.
mall”.
19. (d)
33. (b)
20. (c)
34. (b) Let x and y be the first and last digits such that
21. (a) The code works on a pattern taking the letters x + 0 + y = 4.
of the alphabet. We have to get a number such that
@UPSC_THOUGHTS

M U M B A I x 0y – y 0x = 198
Try out some numbers as x and y to meet these
–1 ↓ –2 ↓ –3 ↓ –4 ↓ –5 ↓ –6 ↓
conditions.
L S J X V C The number could be 2
2 + 0 + 2 = 4
So, D E L H I But interchanging the digits would give you the
–1 ↓ –2 ↓ –3 ↓ –4 ↓ –5 ↓ same number, 202, which, following the next
condition, will give you 0; that is not an option
C C I D D
Try with 3 and 1, i.e. 301
22. (b)
3 + 0 + 1 = 4
23. (c) Assume X and Y to be integers. Interchanging the first and last digits
As X lies between –3 and –1, You get 103
X2 must be between (–3)2 and (–1)2 301 – 103 = 198
As Y lies between –1 and 1, The difference between the first and last digits
Y2 must be between (–1)2 and (1)2 is 3 – 1 = 2.
The maximum value for X2 – Y2 [Taking the algebraic way
= 9 – 1 = 8 Let the number be x0y,
The minimum value for X2 – Y2 = 1 – 1 = 0 100x + 0 + y
So, X2 – Y2 would lie between 0 and 8. As the question asks for the difference between
the first and last digits, let us establish an
24. (a) Take X to be 2 and Y to be 3 equation with the given data
XY = 2 × 3 = 6 which is greater than the value Interchanged, the number will be
of the other options. 100y + 0 + x
For 25-26 Draw a chart and place the offices Given,
Left
(100x + 0 + y) – (100y + 0 + x) = 198
Right
100x + y – 100y – x = 198
F B
99x – 99y = 198
99(x – y) = 198
A E 198
x – y = = 2
99
The difference between the first and last digits
D C
is 2.]

ST . 153
Sample Test 8 : General Studies Paper II (2018)

35. (a) In each layer, there will be 4 cubes painted on 41. (c) If the boy gets 30 marks and fails by 30 marks,
two faces. So 4 × 3 = 12. the passing mark
= 30 + 30 = 60
36. (c) Numbers between 700 to 799 in which digits
By the given information
at hundred’s place is greater than the digit at
40 per cent of maximum marks = 60
the ten’s place and the latter is greater than the
The maximum marks in the exams are
units digit are:
765, 764, 763, 762, 761, 760 100
× 60 = 150
754, 753, 752, 751, 750 40
743, 742, 741, 740 42. (c) The number of boys wearing full uniform is
732, 731, 730 11 + 14 – 19 = 6
721, 720 For 43-48: Draw a chart and fill in the information.
710 At first instance, the chart will look like:
Total 21
Now from 800 to 899, there will be 7 more such Student Subject City State
numbers
21 + 7 = 28 A Commerce
From 900 to 999, there will be yet another 8 B Q
more such numbers C
28 + 8 = 36
D Assam
So the total such numbers from 700 to 1000 will
be Now, from (ii) we see that Arts college is in City
21 + 28 + 36 = 85. S in Rajasthan state. So Arts, S and Rajasthan—
all in one row. This is possible only for C.
37. (b) The given data does not establish a conclusive
Science college can only be for B. Gujarat can
relation between pen and cap or pencil and cap.
only go in the first column against A, and D has
@UPSC_THOUGHTS

All we know is book > cap or cap < book


to be doing engineering. Now the chart looks
We could have cap in different positions:
like:
cap ≤ pen ≤ cap ≤ pencil ≤ cap < book
Options (a), (c) and (d) may or may not be true Student Subject City State
However, pen < pencil < book is established.
A Commerce Gujarat
38. (b) Total sales would be
ax + (a + 2) (x + 2) + (a – 2) (x – 2) B Science Q Kerala
ax + ax + 2x + 2a + 4 + ax – 2x – 2a + 4 C Arts S Rajasthan
3ax + 8.
D Engineering Assam
39. (b) The total number of balls: 15 + 20 = 35
The boy has to pick red balls no. 3 or black Two city slots are not definite. The Questions
balls no. 1 or no. 2 can, however, be easily answered now.
Percentage of red balls numbered 3 = 40 43. (b)
Percentage of black balls numbered 3 = 30
Percentage of black balls numbered 44. (a)
1 = 100 – (45 + 30) = 25
45. (b)
Chances of the boy winning
46. (a)
40 15 70 20 4
= × + × =
100 35 100 35 7 47. (d)
40. (a) As A reaches diametrically opposite his starting 48. (a)
point when he meets B, he has covered 180°
of the circular track. 49. (c) Statement 1 contradicts the passage.
B was 30° ahead of A at the start so now he 50. (d) There is no reference to the other options in the
has covered passage.
180 – 30 = 150°
The required ratio of speeds of A and B (as they 51. (c)
are running at uniform speeds)
52. (b)
= 180° : 150°
6 : 5. 53. (a) Can (c) be correct? It does not seem correct to

ST . 154
Sample Test 8 : General Studies Paper II (2018)

say food security collides with ‘trade’ just 62. (a) The first block can be painted in 6 ways, the
because of India’s being bound by agreements second in 5 ways and the last in 4 ways.
drawn by the WTO.
The different ways for all of the stands to be
54. (b) painted = 6 × 5 × 4 = 120.
55. (d) 63. (a) The fall in birth rate is slower than the fall in
death rate, so there is an increase in the growth
56. (a) Option (b) contradicts the passage; there is no rate of population.
reference to genetically modified crops in the
passage, so option (c) is not correct; the passage 64. (d) As birth rate continues to be higher than the
merely mentions the consequences for Asia and death rate, population could not have stabilised;
so statement 1 is incorrect. Statement 2 is
North America, but does not say Asia and North
clearly correct from the graph. Statement 3 is
America will be ‘worst sufferers’.
wrong as visible from the graph. Statement 4
57. (c) As percentage of profit is the same as percentage is correct as is evident from the graph: birth rate
of loss, actual profit = actual loss. line is almost horizontal.
Let cost price be C 65. (c) The two graphs represent the same data.
As per data given,
66. (a) Note that each row/column has one of each type
40 – C = C – 20
of ‘leg’ and ‘head’. In the required figure, the
2C = 60
head will be a square and the legs will be
60 angled towards left.
C = = 30.
2
[Just looking at the numbers 40 and 20, you can 67. (a) Have a look at the answer responses. Take
get the answer as the same profit or loss means response (a). The cost of production of 2000
half the difference between cost price and items is ` 7 lakh. The selling price of 2000 items
@UPSC_THOUGHTS

selling price is ` 350 × 2000 or ` 7 lakh. So, with 2000 items,


20 the cost and selling price are the same, entailing
So, 40 – 20 = 20; = 10
2 no loss. The other options are obviously
So, CP in case of gain is 40 – 10 = 30 and irrelevant.
CP in case of loss is 20 + 10 = 30.]
68. (b) The ratio of number of adults to number of
58. (c) The number of equilateral triangles will be children in such a lift would be inverse. 12
24 2
= 8 adults is of 18 which is the full capacity of
3 3
59. (b) The series represents dates from 4th December 1 1
the lift. With of the lift capacity unfilled,
1995 with a gap of 27 days in between. You 3 3
1
would be wrong to choose (a) as February in of the children can board it; of 30 is 10.
1996 would have 29 days (as 1996 would be 3
a leap year). 69. (d) An interest of 12.5 per cent on ` 22,800
= ` 2,850
60. (c) If the side of each square is a the sides of the
given rectangle are 3a and 4a 12.5
Given, diagonal = 5cm. × 22,800 = 2,850
100
For a rectangle, diagonal = side2 + side2 Amount payable at the end of the first year is
` 22,800 + ` 2,850 = ` 25,650.
∴ 5 = (3a )2 + (4a )2 After paying ` 8,650, amount left = ` 25,650 –
5 = 9a 2 + 16a 2 = 25a 2 = 5a ` 8,650 = ` 17,000. Interest being 12.5 per cent,
5 amount payable at the end of the second year
= a = 1 = ` 17,000 + ` 2,125 = amount = ` 19,125.
5 After paying ` 9,125, amount left
Area of each square will be 12 = 1
19,125 – 9,125 = ` 10,000.
61. (d) Option (a) is clearly correct from the graph as At the end of the third year, amount to repaid
is option (b). It was expected that 100% of the will be ` 10,000 + interest
work would be done by September 1, but ` 10,000 + ` 1,250 = ` 11,250.
actually was done by mid-August. So option (c) [Marking (b) would be hasty and wrong as the
is also correct. The period between August and question asks for the amount at the end of the
September as shown on the graph contradicts third year.]
the statement in (d).

ST . 155
Sample Test 8 : General Studies Paper II (2018)

70. (b) The square moves clockwise one segment at


Brother Son
a time while the triangle moves anti-clockwise
one segment at a time. net
71. (c) The dependency ratio is the ratio of the persons Daughter Lakshmi
of less than 15 years and persons more than
64 year to the persons in the age group of [Son is the worst player.]
15–64 years. Take the reference to be the In either case, Lakshmi’s brother is the best
current time (2018?). player.
From graph for country B, we see that the
15–64 group line has increased steeply 73. (d) Liquidity is decreased, and thus inflation is
compared to the lines for <15 and 64+ groups. sought to be controlled.
Clearly the dependency ratio has decreased for 74. (d) The figures clearly disprove statements 1
country B. and 2.
So statement 1 is correct.
We may take statement 2 to refer to 2050. 75. (d)
We see that the line for 15–64 years age group 76. (a) The graph represents percentage and not
for country A has declined whereas the combined absolute values. So statement 2 cannot be
numbers of <15 and 64+ have increased. In the inferred.
graph for country B, the line for 15–64 years age
group has risen even as the <15 has declined 77. (d) You can just plug in some numbers and check
and 64+ is less than 64+ in country A. So out the statements.
statement 2 is wrong. As the line for 15–64 Let x be – 4 and y be –12
years age group is at a higher position for x – y = – 4 – (–12) = 8
country B than for country A, statement 3 is So statement 1 is wrong.
correct. (The 15–64 years age group is Incidentally, this proves statement 3 is also
@UPSC_THOUGHTS

considered to be the working population.) wrong.


If x = 14 and y = 6
[Note the divisions indicating the years]
x – y = 14 – 6 = 8
72. (a) From the given information we have two possible Both x and y can be positive.
placements on the court: Statement 2 is also wrong.

Brother Lakshmi 78. (d)

net 79. (b)


Daughter Son 80. (d) Statement 1 is what has been said in the
passage; it is not an assumption.
[Lakshmi is the worst player]

ST . 156
Sample Test 9 : General Studies Paper II (2019)

SAMPLE TEST 9: GENERAL STUDIES PAPER II (2019)


Directions for the following 8 (eight) items: we should aim at is producing men who possess both
Read the following seven passages and answer the items culture and expert knowledge. Their expert knowledge
that follow each passage. Your answers to these items will give them a firm ground to start from and their
should be based on the passages only. culture will lead them as deep as philosophy and as
high as art. Together it will impart meaning to human
Passage—1 existence.
Political theorists, no doubt, have to take history of 3. On the basis of the above passage, the following
assumptions have been made:
injustice, for example, untouchability, seriously. The
1. A society without well educated people cannot
concept of historical injustice takes note of a variety of
be transformed into a modern society.
historical wrongs that continue into the present in some 2. Without acquiring culture, a person’s education
form or the other and tend to resist repair. Two reasons is not complete.
might account for resistance to repair. One, not only are
the roots of injustice buried deep in history, injustice itself Which of the above assumptions is/are valid?
(a) 1 only
constitutes economic structures of exploitation, ideologies
(b) 2 only
of discrimination and modes of representation. Two, the (c) Both 1 and 2
category of historical injustice generally extends across (d) Neither 1 nor 2
a number of wrongs, such as economic deprivation, social
discrimination and lack of recognition. This category is Passage—3
complex, not only because of the overlap between a
number of wrongs, but because one or the other wrong, Soil, in which nearly all our food grows, is a living
generally discrimination, tends to acquire partial autonomy resource that takes years to form. Yet, it can vanish in
@UPSC_THOUGHTS

minutes. Each year, 75 billion tonnes of fertile soil is lost


from others. This is borne out by the history of repair
to erosion. That is alarming—and not just for food
in India.
producers. Soil can trap huge quantities of carbon dioxide
1. What is the main idea that we can infer from the in the form of organic carbon and prevent it from
passage? escaping into the atmosphere.
(a) Untouchability in India has not been taken
4. On the basis of the above passage, the following
seriously by political theorists.
assumptions have been made:
(b) Historical injustice is inevitable in any society
1. Large-scale soil erosion is a major reason for
and is always beyond repair.
widespread food insecurity in the world.
(c) Social discrimination and deprivation have 2. Soil erosion is mainly anthropogenic.
their roots in bad economies. 3. Sustainable management of soils helps in
(d) It is difficult, if not impossible, to repair every combating climate change.
manifestation of historical injustice.
Which of the above assumptions is/are valid?
2. On the basis of the above passage, the following (a) 1 and 2 only
assumptions have been made: (b) 3 only
1. Removal of economic discrimination leads to (c) 2 and 3 only
removal of social discrimination. (d) 1, 2 and 3
2. Democratic polity is the best way to repair
historical wrongs. Passage—4
Which of the above assumptions is/are valid? Inequality is visible, even statistically measurable in
(a) 1 only many instances, but the economic power that drives it
(b) 2 only is invisible and not measurable. Like the force of gravity,
(c) Both 1 and 2 power is the organising principle of inequality, be it of
(d) Neither 1 nor 2 income, or wealth, gender, race, religion and region. Its
effects are seen in a pervasive manner in all spheres, but
Passage—2 the ways in which economic power pulls and tilts visible
Education plays a great transformatory role in life, economic variables remain invisibly obscure.
particularly so in this rapidly changing and globalising 5. On the basis of the above passage, the following
world. Universities are the custodians of the intellectual assumptions have been made:
capital and promoters of culture and specialised 1. Economic power is the only reason for the
knowledge. Culture is an activity of thought, and existence of inequality in a society.
receptiveness to beauty and human feelings. A merely 2. Inequality of different kinds, income, wealth,
well-informed man is only a bore on God’s earth. What etc., reinforces power.

ST . 157
Sample Test 9 : General Studies Paper II (2019)

3. Economic power can be analysed more through Passage—7


its effects than by direct empirical methods.
Around 56 million years ago, the Atlantic Ocean had not
Which of the above assumptions is/are valid?
fully opened and animals, perhaps including our primate
(a) 1 and 2 only
ancestors, could walk from Asia to North America
(b) 3 only
through Europe and across Greenland. Earth was warmer
(c) 1 and 3 only
(d) 1, 2 and 3 than it is today, but as the Palaeocene epoch gave way
to Eocene, it was about to get much warmer still—rapidly
Passage—5 and radically. The cause was a massive geologically
sudden release of carbon. During this period called
Climate change may actually benefit some plants by Palaeocene-Eocene Thermal Maximum or PETM, the
lengthening growing seasons and increasing carbon carbon injected into the atmosphere was roughly the
dioxide. Yet other effects of a warmer world, such as amount that would be injected today if humans burned
more pests, droughts, and flooding, will be less benign.
all the Earth’s reserves of coal, oil and natural gas. The
How will the world adapt? Researchers project that by
PETM lasted for about 1,50,000 years, until the excess
2050, suitable croplands for four commodities—maize,
carbon was reabsorbed. It brought on drought, floods,
potatoes, rice and wheat—will shift, in some cases
pushing farmers to plant new crops. Some farmlands may insect plagues and a few extinctions. Life on Earth
benefit from warming, but others won’t. Climate alone survived—indeed, it prospered—but it was drastically
does not dictate yields; political shifts, global demand, different.
and agricultural practices will influence how farms fare 8. Based on the above passage, the following
in the future. assumptions have been made:
6. Which one of the following is the most logical and 1. Global warming has a bearing on the planet’s
rational inference that can be made from the above biological evolution.
passage? 2. Separation of land masses causes the release
(a) Farmers who modernise their methods and of huge quantities of carbon into the
diversify their fields will be in an advantageous atmosphere.
@UPSC_THOUGHTS

position in future. 3. Increased warming of Earth’s atmosphere can


(b) Climate change will adversely affect the crop change the composition of its flora and fauna.
diversity. 4. The present man-made global warming will
(c) Shifting major crops to new croplands will finally lead to conditions similar to those
lead to a great increase in the total area under which happened 56 million years ago.
cultivation and thus an increase in overall Which of the assumptions given above are valid?
agricultural production. (a) 1 and 2
(d) Climate change is the most important factor (b) 3 and 4
affecting the agricultural economy in the future. (c) 1 and 3
(d) 2 and 4
Passage—6 9. The number of times the digit 5 will appear while
A bat’s wings may look like sheets of skin. But underneath, writing the integers from 1 to 1000 is
a bat has the same five fingers as an orangutan or a (a) 269
human, as well as a wrist connected to the same cluster (b) 271
of wrist bones connected to the same long bones of the (c) 300
arm. What can be more curious than that the hand of (d) 302
a man, formed for grasping, that of a mole for digging, 10. A solid cube is painted yellow, blue and black such
the leg of the horse, the-paddle of the porpoise, and the that opposite faces are of same colour. The cube
wing of the bat, should all be constructed on the some is then cut into 36 cubes of two different sizes such
pattern? that 32 cubes are small and the other four cubes
7. Which one of the following is the most logical, are big. None of the faces of the bigger cubes is
scientific and rational inference that can be made painted blue. How many cubes have only one face
from the above passage? painted?
(a) Different species having similar structure of (a) 4
hands is an example of biodiversity. (b) 6
(b) Limbs being used by different species for (c) 8
different kinds of work is an example of (d) 10
biodiversity.
(c) Man and the aforementioned animals having 11. A and B are two heavy steel blocks. If B is placed
similar structure of limbs is an example of on the top of A, the weight increases by 60%. How
coincidence in evolution. much weight will reduce with respect to the total
(d) Man and the aforementioned animals have a weight of A and B, if B is removed from the top
shared evolutionary history. of A?

ST . 158
Sample Test 9 : General Studies Paper II (2019)

(a) 60% 75% of the amount required to buy the handset.


(b) 45.5% Therefore, he borrows ` 2000 from a friend. Then
(c) 40% (a) Raju still does not have enough amount to buy
(d) 37.5% the handset.
12. Mr ‘X’ has three children. The birthday of the first (b) Raju has exactly the same amount as required
child falls on the 5th Monday of April, that of the to buy the handset.
second one falls on the 5th Thursday of November. (c) Raju has enough amount to buy the handset
On which day is the birthday of his third child, and he will have ` 500 with him after buying
which falls on 20th December? the handset.
(a) Monday (d) Raju has enough amount to buy the handset
(b) Thursday and he will have ` 1000 with him after buying
(c) Saturday the handset.
(d) Sunday 18. In 2002, Meenu’s age was one-third of the age of
13. Consider the following Statements and Conclusions: Meera, whereas in 2010, Meenu’s age was half the
Statements: age of Meera. What is Meenu’s year of birth?
1. Some rats are cats. (a) 1992
2. Some cats are dogs. (b) 1994
3. No dog is a cow. (c) 1996
Conclusions: (d) 1998
I. No cow is a cat. 19. Rakesh and Rajesh together bought 10 balls and
II. No dog is a rat. 10 rackets. Rakesh spent ` 1300 and Rajesh spent
III. Some cats are rats. ` 1500. If each racket costs three times a ball does,
Which of the above conclusions is/are drawn from then what is the price of a racket?
the statements? (a) ` 70
(a) I, II and III (b) ` 90
(b) Only I and II (c) ` 210
@UPSC_THOUGHTS

(c) Only III (d) ` 240


(d) Only II and III 20. In a conference, out of a total 100 participants, 70
14. The number of parallelograms that can be formed are Indians. If 60 of the total participants are
from a set of four parallel lines intersecting another vegetarian, then which of the following statements
set of four parallel lines, is is/are correct?
1. At least 30 Indian participants are vegetarian.
(a) 18
2. At least 10 Indian participants are non-
(b) 24
vegetarian.
(c) 32
(d) 36 Select the correct answer using the codes given
below:
15. In a school, every student is assigned a unique
(a) 1 only
identification number. A student is a football (b) 2 only
player if and only if the identification number is (c) Both 1 and 2
divisible by 4, whereas a student is a cricketer if (d) Neither 1 nor 2
and only if the identification number is divisible
by 6. If every number from 1 to 100 is assigned Directions for the following 8 (eight) items:
to a student, then how many of them play cricket Read the following six passages and answer the items
as well as football? that follow each passage. Your answers to these items
(a) 4 should be based on the passages only.
(b) 8
(c) 10 Passage—1
(d) 12 Low-end IoT (Internet of Things) devices are cheap
16. When a runner was crossing the 12 km mark, she commodity items: addressing security would add to the
was informed that she had completed only 80% of cost. This class of items is proliferating with new
the race. How many kilometres was the runner applications; many home appliances, thermostats, security
supposed to run in this event? and monitoring devices and personal convenience devices
(a) 14 are part of the IoT. So are fitness trackers, certain medical
(b) 15 implants and computer-like devices in automobiles. The
(c) 16 IoT is expected to expand exponentially—but new security
(d) 16.5 challenges are daunting.
17. Raju has ` 9000 with him and he wants to buy 21. Which one of the following statements is the most
a mobile handset; but he finds that he has only logical and rational inference that can be made

ST . 159
Sample Test 9 : General Studies Paper II (2019)

from the above passage? 24. What is the most logical and rational corollary to
(a) Development of enabling technologies in India the above passage?
can be a big boost to its manufacturing sector. (a) Supporting small farmers is an important part
(b) India is not yet fully ready to adopt IoT in of any agenda regarding environmentally
view of the imminent security challenges. sustainable development.
(c) Life becomes more comfortable with the (b) Poor countries have little role to play in the
development of cheap low-end IoT devices. mitigation of global warming.
(d) As we go digital, we must recognise the huge (c) Due to a large number of farmer households,
threat to Internet security from some IoT India will not have food security problem in
devices. the foreseeable future.
(d) Only small-holder farmers in India can ensure
Passage—2 food security.
With the digital phenomenon restructuring most social 25. The above passage implies that
sectors, it is little surprise that global trade negotiations 1. There is a potential problem of food insecurity
are now eyeing the digital area in an attempt to pre- in India.
emptively colonise it. Big Data is freely collected or 2. India will have to strengthen its disaster
mined from developing countries, and converted into management capabilities.
digital intelligence in developed countries. This intelligence Which of the above assumptions is/are valid?
begins to control different sectors and extract monopoly (a) 1 only
rents. A large foreign company providing cab service, for (b) 2 only
instance, is not a work of cars and drivers, it is digital (c) Both 1 and 2
intelligence about commuting, public transport, roads, (d) Neither 1 nor 2
traffic, city events, personal behavioural characteristics of
commuters and driver, and so on. Passage—4
22. Which one of the following is the most logical and A changing climate, and the eventual efforts of
rational corollary to the above passage? governments (however reluctant) to deal with it, could
@UPSC_THOUGHTS

(a) Globalisation is not in the interests of India have a big impact on investors’ returns. Companies that
as it undermines its socio-economic structures. produce or use large amounts of fossil fuels will face
(b) India should be careful to protect its digital higher taxes and regulatory burdens. Some energy
sovereignty in global trade talks. producers may find it impossible to exploit their known
(c) India should charge monopoly rents from reserves, and be left with ‘stranded assets’—deposits of
multinational companies in exchange for Big oil and coal that have to be left in the ground. Other
Data. industries could be affected by the economic damage
(d) The loss of Big Data from India is proportional caused by more extreme weather—storms, floods, heat
to the degree/value of its foreign trade. waves and droughts.
23. Which of the following is most definitively implied 26. On the basis of the above passage, the following
by the above passage? assumptions have been made:
(a) Big Data is the key resource in the digital 1. Governments and companies need to be
space. adequately prepared to face the climate change.
(b) Big economies create Big Data. 2. Extreme weather events will reduce the
(c) Access to Big Data is the prerogative of economic growth of governments and
developed countries. companies in future.
(d) Access to and possession of Big Data is a 3. Ignoring climate change is a huge risk for
characteristic of developed countries. investors.
Passage—3 Which of the above assumptions is/are valid?
(a) 1 and 2 only
The rural poor across the world, including India, have (b) 3 only
contributed little to human-induced climate change, yet (c) 1 and 3 only
they are on the frontline in coping with its effects. (d) 1, 2 and 3
Farmers can no longer rely on historical averages for
rainfall and temperature, and the more frequent and Passage—5
extreme weather events, such as droughts and floods, can
spell disaster. And there are new threats, such as sea level Access to schooling for those coming of school age is close
rise and the impact of melting glaciers on water supply. to universal, but access to quality exhibits a sharp
How significant are small farms? As many as two billion gradient with socio-economic status. Quotas for the
people worldwide depend on them for their food and weaker sections in private schools is a provision introduced
livelihood. Small-holder farmers in India produce 41 per by the Right of Children to Free and Compulsory
cent of the country’s food grains, and other food items Education Act, 2009. The quotas have imposed a debate
that contribute to local and national food security. on issues of social integration and equity in education

ST . 160
Sample Test 9 : General Studies Paper II (2019)

that private actors had escaped by and large. The idea To ensure that any two consecutive floors have
of egalitarian education system with equality of different colours
opportunity as its primary goal appears to be outside the (a) Only statement 2 is sufficient
space that private school principals inhabit. Therefore, the (b) Only statement 3 is sufficient
imposition of the quotas has led to resistance, sometimes (c) Statement 1 is not sufficient, but statement 1
justified. along with statement 2 is sufficient
27. With reference to the above passage, the following (d) Statement 3 is not sufficient, but statement 3
assumptions have been made: along with statement 2 is sufficient
1. Making equality of opportunity a reality is the 30. P, Q and R are three towns. The distance between
fundamental goal of the Indian education P and Q is 60 km, whereas the distance between
system. P and R is 80 km. Q is in the West of P and R
2. The present Indian school system is unable to is in the South of P. What is the distance between
provide egalitarian education. Q and R?
3. Abolition of private schools and establishment (a) 140 km
of more government schools is the only way (b) 130 km
to ensure egalitarian education. (c) 110 km
Which of the above assumptions is/are valid? (d) 100 km
(a) 1 and 2 only 31. All members of a club went to Mumbai and stayed
(b) 2 only in a hotel. On the first day, 80% went for shopping
(c) 2 and 3 only and 50% went for sightseeing, whereas 10% took
(d) 3 only rest in the hotel. Which of the following
conclusion(s) can be drawn from the above data?
Passage—6 1. 40% members went for shopping as well as
sightseeing.
A majority of the TB infected in India are poor and lack 2. 20% members went for only shopping.
sufficient nutrition, suitable housing and have little
Select the correct answer using the code given
@UPSC_THOUGHTS

understanding of prevention. TB then devastates families,


makes the poor poorer, particularly affects women and below:
children, and leads to ostracisation and loss of (a) 1 only
(b) 2 only
employment. The truth is that even if TB does not kill
(c) Both 1 and 2
them, hunger and poverty will. Another truth is that (d) Neither 1 nor 2
deep-seated stigma, lack of counselling, expensive
treatment and lack of adequate support from providers 32. In a school, 60% students play cricket. A student
and family, coupled with torturous side-effects demotivate who does not play cricket, plays football. Every
football player has got a two-wheeler. Which of the
patients to continue treatment—with disastrous health
following conclusions cannot be drawn from the
consequences. above data?
28. Which one of the following is the most logical, 1. 60% of the students do not have two-wheelers.
rational and crucial message conveyed by the 2. No cricketer has a two-wheeler.
above passage? 3. Cricket players do not play football.
(a) TB is not a curable disease in Indian Select the correct answer using the code given
circumstances. below:
(b) Curing TB requires more than diagnosis and (a) 1 and 2 only
medical treatment. (b) 2 and 3 only
(c) Government’s surveillance mechanism is (c) 1 and 3 only
deficient; and poor people have no access to (d) 1, 2 and 3
treatment.
33. The ratio of a two-digit natural number to a
(d) India will be free from diseases like TB only
number formed by reversing its digits is 4 : 7. The
when its poverty alleviation programmes are
number of such pairs is
effectively and successfully implemented. (a) 5
29. A five-storeyed building with floors from I to V (b) 4
is painted using four different colours and only one (c) 3
colour is used to paint a floor. (d) 2
Consider the following statements: 34. In an examination, A has scored 20 marks more
1. The middle three floors are painted in different than B. If B has scored 5% less marks than A, how
colours. much has B scored?
2. The second (II) and the fourth (IV) floors are (a) 360
painted in different colours. (b) 380
3. The first (I) and the fifth (V) floors are painted (c) 400
red. (d) 420

ST . 161
Sample Test 9 : General Studies Paper II (2019)

35. Seeta and Geeta go for a swim after a gap of every `Intellectual Property Rights’ regime that seeks to create
2 days and every 3 days respectively. If on 1st private monopolies for such technologies. If GM
January both of them went for a swim together, technology is largely corporate driven, it seeks to maximise
when will they go together next? profits and that too in the short run. That is why
(a) 7th January corporations make major investments for herbicide-tolerant
(b) 8th January and pest-resistant crops. Such properties have only a
(c) 12th January short window, as soon enough, pests and weeds will
(d) 13th January evolve to overcome such resistance. This suits the
36. X, Y and Z are three contestants in a race of 1000 corporations. The National Farmers Commission pointed
m. Assume that all run with different uniform out that priority must be given in genetic modification
speeds. X gives Y a start of 40 m and X gives Z to the incorporation of genes that can help impart
a start of 64 m. If Y and Z were to compete in resistance to drought, salinity and other stresses.
a race of 1000 m, how many metres start will Y
give to Z? 41. Which one of the following is the most logical,
(a) 20 rational and crucial message conveyed by the
(b) 25 above passage?
(c) 30 (a) Public research institutions should take the
(d) 35 lead in GM technology and prioritise the
technology agenda.
37. If x is greater than or equal to 25 and y is less than (b) Developing countries should raise this issue
or equal to 40, then which one of the following in WTO and ensure the abolition of Intellectual
is always correct?
Property Rights.
(a) x is greater than y
(c) Private corporations should not be allowed to
(b) (y – x) is greater than 15
do agribusiness in India, particularly the seed
(c) (y – x) is less than or equal to 15
business.
(d) (x + y) is greater than or equal to 65
(d) Present Indian circumstances do not favour
38. Ena was born 4 years after her parents’ marriage. the cultivation of genetically modified crops.
@UPSC_THOUGHTS

Her mother is three years younger than her father


and 24 years older than Ena, who is 13 years old. 42. On the basis of the above passage, the following
At what age did Ena’s father get married? assumptions have been made:
(a) 22 years 1. The issue of effects of natural calamities on
(b) 23 years agriculture is not given due consideration by
(c) 24 years GM technology companies.
(d) 25 years 2. In the long run, GM technology will not be
able to solve agricultural problems arising due
39. Rakesh had money to buy 8 mobile handsets of to global warming.
a specific company. But the retailer offered very
good discount on that particular handset. Rakesh Which of the above assumptions is/are valid?
could buy 10 mobile handsets with the amount he (a) 1 only
had. What was the discount the retailer offered? (b) 2 only
(a) 15% (c) Both 1 and 2
(b) 20% (d) Neither 1 nor 2
(c) 25%
(d) 30% Passage—2
40. The average marks of 100 students are given to be Most invasive species are neither terribly successful nor
40. It was found later that marks of one student very harmful. Britain’s invasive plants are not widespread,
were 53 which were misread as 83. The corrected not spreading especially quickly, and often less of a
mean marks are nuisance than vigorous natives such as bracken. The
(a) 39 arrival of new species almost always increases biological
(b) 39.7 diversity in a region; in many cases, a flood of newcomers
(c) 40 drives no native species to extinction. One reason is that
(d) 40.3 invaders tend to colonise disturbed habitats like polluted
Directions for the following 7 (seven) items: lakes and post-industrial wasteland, where little else
lives. They are nature’s opportunists.
Read the following six passages and answer the items
that follow each passage. Your answers to these items 43. Which one of the following is the most logical and
should be based on the passages only. rational inference that can be made from the above
passage?
Passage—1 (a) Invasive species should be used to rehabilitate
desert areas and wastelands of a country.
What stands in the way of the widespread and careful (b) Laws against the introduction of foreign plants
adoption of ‘Genetic Modification (GM)’ technology is an are unnecessary.

ST . 162
Sample Test 9 : General Studies Paper II (2019)

(c) Sometimes, the campaigns against foreign substantial employment. But this is not so. When our
plants are pointless. economy was growing at 3 per cent per year, employment
(d) Foreign plants should be used to increase the in the organised sector was growing at 2 per cent per
biodiversity of a country. year. As the economy began to grow at 7–8 per cent per
year, the rate of growth of employment in the organised
Passage—3 sector actually declined to 1 per cent per year.
Diarrhoeal deaths among Indian children are mostly due 46. The above passage seems to imply that
to food and water contamination. Use of contaminated 1. most of modern economic growth is based on
groundwater and unsafe chemicals in agriculture, poor technological progress.
hygiene in storage and handling of food items to food 2. much of modern Indian economy does not
cooked and distributed in unhygienic surroundings; there nurture sufficient symbiotic relationship with
are myriad factors that need regulation and monitoring. labour-intensive, natural resource-based
People need to have awareness of adulteration and ways livelihoods.
of complaining to the relevant authorities. Surveillance 3. service sector in India is not very labour-
of food-borne diseases involves a number of government intensive.
agencies and entails good training of inspection staff. 4. literate rural population is not willing to enter
Considering the proportion of the urban population that organised sector.
depends on street food for its daily meals, investing in Which of the statements given above are correct?
training and education of street vendors is of great (a) 1 and 2 only
significance. (b) 3 and 4 only
44. On the basis of the above passage, the following (c) 1, 2 and 3 only
assumptions have been made: (d) 1, 2, 3 and 4
1. Food safety is a complex issue that calls for
a multipronged solution. Passage—6
2. Great investments need to be made in India has banking correspondents, who help bring people
developing the manpower for surveillance
@UPSC_THOUGHTS

in the hinterland into the banking fold. For them to


and training.
succeed, banks cannot crimp on costs. They also cannot
3. India needs to make sufficient legislation for
afford to ignore investing in financial education and
governing food processing industry.
literacy. Banking correspondents are way too small to be
Which of the above assumptions is/are valid? viewed as a systemic risk. Yet India’s banking regulator
(a) 1 and 2 only has restricted them to serving only one bank, perhaps
(b) 3 only to prevent arbitrage. Efforts at banking outreach may
(c) 1 and 3 only succeed only if there are better incentives at work for
(d) 1, 2 and 3 such last-mile workers and also those providers who
Passage—4 ensure not just basic bank accounts but also products
such as accident and life insurance and micro pension
The interests of working and poor people have historically schemes.
been neglected in the planning of our cities. Our cities
are increasingly intolerant,’, unsafe and unlivable places 47. Which one of the following is the most logical,
for large numbers of citizens and yet we continue to plan rational and crucial inference that can be derived
via the old ways—the static Development Plan—that from the above passage?
draws exclusively from technical expertise, distanced (a) Efforts to bring people in India’s hinterland
from people’s live experiences and needs, and actively into the banking system are not successful.
excluding large number of people, places, activities and (b) For meaningful financial inclusion, India’s
practices that are an integral part of the city. banking system needs more number of banking
correspondents and other such last-mile
45. The passage seems to argue
(a) against the monopoly of builders and the workers.
interests of elite groups. (c) Meaningful financial inclusion in India requires
(b) against the need for global and smart cities. that banking correspondents have diverse skills
(c) in favour of planning cities mainly for working (d) Better banking outreach would be impossible
class and poor people. unless each banking correspondent is allowed
(d) in favour of participation of peoples’ groups to serve a number of banks.
in city planning. 48. What is X in the sequence 132, 129, 124, 117, 106,
93, X?
Passage—5 (a) 74
A vast majority of Indians are poor, with barely 10 per (b) 75
cent employed in the organised sector. We are being (c) 76
convinced that vigorous economic growth is generating (d) 77

ST . 163
Sample Test 9 : General Studies Paper II (2019)

49. A wall clock moves 10 minutes fast in every 24 56. Suppose you have sufficient amount of rupee
hours. The clock was set right to show the correct currency in three denominations: ` 1, ` 10 and
time at 8:00 a.m. on Monday. When the clock ` 50. In how many different ways can you pay a
shows the time 6:00 p.m. on Wednesday, what is bill of ` 107 ?
the correct time? (a) 16
(a) 5:36 p.m. (b) 17
(b) 5:30 p.m. (c) 18
(c) 5:24 p.m. (d) 19
(d) 5:18 p.m. 57. ‘A’ started from his house and walked 20 m
50. If the numerator and denominator of a proper towards East, where his friend ‘B’ joined him. They
fraction are increased by the same positive quantity together walked 10 m in the same direction. Then
which is greater than zero, the resulting fraction ‘A’ turned left while ‘B’ turned right and travelled
is 2 m and 8 m respectively. Again ‘B’ turned left
(a) always less than the original fraction to travel 4 m followed by 5 m to his right to reach
(b) always greater than the original fraction his office. ‘A’ turned right and travelled 12 m to
(c) always equal to the original fraction reach his office. What is the shortest distance
(d) such that nothing can be claimed definitely between the two offices?
(a) 15 m
51. What is X in the sequence
(b) 17 m
4, 196, 16, 144, 36, 100, 64, X? (c) 19 m
(a) 48 (d) 20 m
(b) 64 58. Consider two statements S1 and S2 followed by a
(c) 125 question:
(d) 256 S1: p and q both are prime numbers.
52. In a group of 15 people; 7 can read French, 8 can S2: p + q is an odd integer.
read English while 3 of them can read neither of Question: Is pq an odd integer?
@UPSC_THOUGHTS

these two languages. The number of people who Which one of the following is correct?
can read exactly one language is (a) S1 alone is sufficient to answer the question
(a) 10 (b) S2 alone is sufficient to answer the question
(b) 9 (c) Both S1 and S2 taken together are not sufficient
(c) 5 to answer the question
(d) 4 (d) Both S1 and S2 are necessary to answer the
53. A printer numbers the pages of a book starting question
with 1 and uses 3089 digits in all. How many pages 59. Which year has the same calendar as that of 2009?
does the book have? (a) 2018
(a) 1040 (b) 2017
(b) 1048 (c) 2016
(c) 1049 (d) 2015
(d) 1050 60. Number 136 is added to 5B7 and the sum obtained
54. Consider the following sequence that follows some is 7A3, where A and B are integers. It is given that
arrangement: 7A3 is exactly divisible by 3. The only possible
c_accaa_aa_bc_b value of B is
(a) 2
The letters that appear in the gaps are (b) 5
(a) abba (c) 7
(b) cbbb (d) 8
(c) bbbb
(d) cccc Directions for the following 7 (seven) items:
55. A family has two children along with their parents. Read the following five passages and answer the items
The average of the weights of the children and that follow each passage. Your answers to these items
their mother is 50 kg. The average of the weights should be based on the passages only.
of the children and their father is 52 kg. If the Passage—1
weight of the father is 60 kg, then what is the
weight of the mother? India’s economic footprint, given its population, still
(a) 48 kg remains small compared to the US, the European Union
(b) 50 kg or China. It has much to learn from other economies, yet
(c) 52 kg must implement solutions that fit its unique circumstances.
(d) 54 kg India especially needs an effective long-term regulatory

ST . 164
Sample Test 9 : General Studies Paper II (2019)

system based on collaboration rather than the current top- 63. Which one of the following is the most logical,
down approach. Regulations seek desirable outcomes yet rational and crucial inference that can be derived
are repeatedly used as political tools to push one agenda from the above passage?
or another. Often, regulations fail to consider impacts on (a) If malnourished condition in children is caused
jobs and economic growth — or less restrictive alternatives. by gut bacteria, it cannot be treated.
Regulations may be used to protect local markets at the (b) The guts of malnourished babies should be
expense of more widely shared prosperity in the future. inoculated with mature microbiomes.
Additionally, regulations inevitably result in numerous (c) Babies of malnourished mothers should be fed
unintended consequences. In today’s hyper competitive with dairy milk fortified with sialylated
global economy, regulations need to be viewed as oligosaccharides instead of mother’s milk.
‘weapons’ that seek cost-justified social and environmental (d) Research on benign effects of gut bacteria on
benefits while improving the economic well-being of nutrition has policy implications.
most citizens. 64. On the basis of the above passage, the following
61. Which one of the following is the most logical, assumptions have been made:
rational and crucial inference that can be derived 1. Processed probiotic foods are a solution to
from the above passage? treat the children suffering from malnutrition
(a) A better regulatory system will help India due to immature gut bacteria composition.
achieve the size of economy appropriate to its 2. The babies of malnourished mothers generally
population. tend to be malnourished.
(b) In a competitive global economy, India must Which of the above assumptions is/are valid?
use regulations strategically. (a) 1 only
(c) Regulations in India do not favour its (b) 2 only
integration with today’s hyper competitive (c) Both 1 and 2
global economy. (d) Neither 1 nor 2
(d) Job creation and economic growth should be
dominant considerations in developing India’s Passage—3
@UPSC_THOUGHTS

regulatory system.
Temperatures have risen nearly five times as rapidly on
62. On the basis of the above passage, the following the Western Antarctic Peninsula than the global average
assumptions have been made: over the past five decades. Researchers have now found
In today’s global economy, that melting glaciers are causing a loss of species diversity
1. regulations are not effectively used to protect among benthos in the coastal waters off the Antarctic
local markets. Peninsula, impacting an entire seafloor ecosystem. They
2. social and environmental concerns are believe increased levels of suspended sediment in water
generally ignored by the governments across to be the cause of the dwindling biodiversity in the
the world while implementing the regulations. coastal region.
Which of the above assumptions is/are valid? 65. On the basis of the above passage, the following
(a) 1 only assumptions have been made:
(b) 2 only 1. Regions of glaciers warm faster than other
(c) Both 1 and 2 regions due to global warming.
(d) Neither 1 nor 2 2. Global warming can lead to seafloor
sedimentation in some areas.
Passage—2 3. Melting glaciers can reduce marine biodiversity
In a study, scientists compared the microbiomes of poorly in some areas.
nourished and well nourished infants and young children. Which of the above assumptions is/are valid?
Gut microbes were isolated from faecal samples of (a) 1 and 2 only
malnourished and healthy children. The microbiome was (b) 3 only
‘immature’ and less diverse in malnourished children (c) 2 and 3 only
compared to the better developed ‘mature’ microbiome (d) 1, 2 and 3
found in healthy children of the same age. According to
some studies, the chemical composition of mother’s milk Passage—4
has shown the presence of a modified sugar (sialylated A research team examined a long-term owl roost. Owls
oligosaccharides). This is not utilised by the baby for its prey on small mammals and the excreted remains of
own nutrition. However, the bacteria constituting the those meals that accumulated over the time, provide us
infant’s microbiome thrive on this sugar which serves as an insight into the composition and structure of small
their food. Malnourished mothers have low levels of this mammals over the past millennia. The research suggested
sugar in their milk. Consequently, the microbiomes of that when the Earth went through a period of rapid
their infants fail to mature. That, in turn, leads to warming about 13,000 years ago, the small mammal
malnourished babies. community was stable and resilient. But, from the last

ST . 165
Sample Test 9 : General Studies Paper II (2019)

quarter of the nineteenth century, human-made changes capitalised, then how is the first month of the
to the environment had caused an enormous drop in second half of the year written?
biomass and energy flow. This dramatic decline in energy (a) JuLY
flow means modern ecosystems are not adapting as easily (b) jULy
as they did in the past. (c) jUly
66. On the basis of the above passage, the following (d) jUlY
assumptions have been made: 69. Sunita cuts a sheet of paper into three pieces.
1. Global warming is a frequently occurring Length of first piece is equal to the average of the
natural phenomenon. three single digit odd prime numbers. Length of
2. The impending global warming will not the second piece is equal to that of the first plus
adversely affect small mammals. one-third the length of the third. The third piece
3. Humans are responsible for the loss of the is as long as the other two pieces together. The
Earth’s natural resilience. length of the original sheet of paper is
Which of the above assumptions is/are valid? (a) 13 units
(a) 1 and 2 only (b) 15 units
(b) 3 only (c) 16 units
(c) 2 and 3 only (d) 30 units
(d) 1, 2 and 3 70. In the sequence 1, 5, 7, 3, 5, 7, 4, 3, 5, 7, how many
such 5s are there which are not immediately
Passage—5 preceded by 3 but are immediately followed by 7?
Food varieties extinction is happening all over the world— (a) 1
and it is happening fast. For example, of the 7,000 apple (b) 2
(c) 3
varieties that were grown during the nineteenth century,
(d) None
fewer than a hundred remain. In the Philippines,
thousands of varieties of rice once thrived; now only up 71. A joint family consists of seven members A, B, C,
D, E, F and G with three females. G is a widow
@UPSC_THOUGHTS

to a hundred are grown there. In China, 90 per cent of


the wheat varieties cultivated just a century ago have and sister-in-law of D’s father F. B and D are
disappeared. Farmers in the past painstakingly bred and siblings and A is daughter of B. C is cousin of B.
developed crops well suited to the peculiarities of their Who is E?
local climate and environment. In the recent past, our 1. Wife of F
2. Grandmother of A
heavy dependence on a few high-yielding varieties and
3. Aunt of C
technology-driven production and distribution of food is
causing the dwindling of diversity in food crops. If some Select the correct answer using the code given
mutating crop disease or future climate change decimates below:
the few crop plants we have come to depend on to feed (a) 1 and 2 only
our growing population, we might desperately need (b) 2 and 3 only
some of those varieties we have let go extinct. (c) 1 and 3 only
(d) 1, 2 and 3
67. On the basis of the above passage, the following
assumptions have been made: 72. Each face of a cube can be painted in black or white
1. Humans have been the main reason for the colours. In how many different ways can the cube
large-scale extinction of plant species. be painted?
2. Consumption of food mainly from locally (a) 9
cultivated crops ensures crop diversity. (b) 10
3. The present style of production and distribution (c) 11
of food will finally lead to the problem of food (d) 12
scarcity in the near future. 73. How many triplets (x, y, z) satisfy the equation
4. Our food security may depend on our ability x + y + z = 6, where x, y and z are natural numbers?
to preserve the locally cultivated varieties of (a) 4
crops. (b) 5
Which of the above assumptions are valid? (c) 9
(a) 1 and 3 (d) 10
(b) 2 and 4 74. If $ means ‘divided by’; @ means ‘multiplied by’;
(c) 2 and 3 # means ‘minus’, then the value of 10#5@1$5 is
(d) 1 and 4 (a) 0
68. If every alternative letter of the English alphabet (b) 1
from B onwards (including B) is written in lower (c) 2
case (small letters) and the remaining letters are (d) 9

ST . 166
Sample Test 9 : General Studies Paper II (2019)

75. An 8-digit number 4252746B leaves remainder 0 (c) Fourth


when divided by 3. How many values of B are (d) Second or third
possible? 78. If E is ranked third, then which one of the
(a) 2 following is correct?
(b) 3 (a) E gets more marks than C
(c) 4 (b) C gets more marks than E
(d) 6 (c) A is ranked fourth
Directions for the following 3 (three) items: (d) D is ranked fifth
Read the following information and answer the three Directions for the following 2 (two) items:
items that follow: Read the following statements SI and S2 and answer the
Six students A, B, C, D, E and F appeared in several two items that follow:
tests. Either C or F scores the highest. Whenever C scores S1: Twice the weight of Sohan is less than the
the highest, then E scores the least. Whenever F scores weight of Mohan or that of Rohan.
the highest, B scores the least. S2: Twice the weight of Rohan is greater than the
In all the tests they got different marks; D scores weight of Mohan or that of Sohan.
higher than A, but they are close competitors; A scores
higher than B; C scores higher than A. 79. Which one of the following statements is correct?
(a) Weight of Mohan is greatest
76. If F stands second in the ranking, then the position (b) Weight of Sohan is greatest
of B is (c) Weight of Rohan is greatest
(a) Third (d) ‘Whose weight is greatest’ cannot be
(b) Fourth determined
(c) Fifth
(d) Sixth 80. Which one of the following statements is correct?
(a) Weight of Mohan is least
77. If B scores the least, the rank of C will be (b) Weight of Sohan is least
(a) Second (c) Weight of Rohan is least
@UPSC_THOUGHTS

(b) Third (d) ‘Whose weight is least’ cannot be determined

ST . 167
Sample Test 9 : General Studies Paper II (2019)

PRACTICE ANSWER SHEET

Directions: Use black ball pen. Example

1. 17. 33. 49. 65.

2. 18. 34. 50. 66.

3. 19. 35. 51. 67.

4. 20. 36. 52. 68.

5. 21. 37. 53. 69.

6. 22. 38. 54. 70.


@UPSC_THOUGHTS

7. 23. 39. 55. 71.

8. 24. 40. 56. 72.

9. 25. 41. 57. 73.

10. 26. 42. 58. 74.

11. 27. 43. 59. 75.

12. 28. 44. 60. 76.

13. 29. 45. 61. 77.

14. 30. 46. 62. 78.

15. 31. 47. 63. 79.

16. 32. 48. 64. 80.

ST . 168
Sample Test 9 : General Studies Paper II (2019)

ANSWERS AND EXPLANATORY NOTES


1. (d) The passage does not say untouchability has power is an important, perhaps the most
not been taken seriously by political theorists; important, factor responsible for the existence
nor does it say that historical injustice is of inequality, but that does not justify saying it
inevitable, or anything about ‘bad economics’ is the ‘only reason’ for such inequality. So,
being the root of social discrimination response; statement 1 is not acceptable. Statement 2 is
so, answer responses (a), (b), and (c) are not not justified in the light of the passage. It is
correct. The passage in its entirety presents the power that drives inequality. Statement 3 is an
idea that the manifestations of historical injustice assumption that is supported by the passage,
continue to this day and pose complex problems, from the first and last sentences.
and resist repair, hence making repair ‘difficult’,
6. (a) Nothing in the passage supports what is stated
though the ‘history of repair’ suggests that some
in answer responses (b) and (c). The last
repair has been done; so, repair is not
sentence of the passage contradicts what is
‘impossible’. Answer response (d) is correct.
stated in answer response (d). Answer response
2. (d) The sentence, “This category is complex ...” (a) is the most logical inference to be drawn
points out the overlap of several wrongs; just from the passage.
the removal of economic discrimination cannot
7. (d) The passage is not on biodiversity. So, answer
lead to the removal of social discrimination, as
responses (a) and (b) are not correct. If one
they exist side by side along with other wrongs.
goes on a scientific basis, there is no coincidence
As for the second statement on democratic
in evolution; everything has a purpose. So,
polity being the best way to repair historical
answer response (c) is not acceptable. Answer
@UPSC_THOUGHTS

wrongs, there is no indication in the passage


response (d) is the most logical inference. All
of any particular kind of polity in relation to the
the species mentioned have similar structures,
best way out of the problem. So, neither
which points to common beginnings, but with
statement is valid.
changes suited to each one’s needs as
3. (b) The passage states that education has a role demanded by evolution.
in transforming the life of a person. It also
8. (c) The last sentence of the passage supports
states, “What we should aim at is producing
statements 1 and 3. So, answer response (c)
men who possess both culture and expert
is correct. Nothing in the passage justifies
knowledge. . . Together they will impart meaning
statement 2. Humans would have to burn the
to human existence.” So, statement 2 is certainly
entire reserves of coal, natural gas, and oil to
an assumption based on the passage. Statement
achieve the conditions of the period called
1 is rather far-fetched as an assumption, as it
PETM, and that is not anywhere equal to the
makes a leap from individual life adapting to
present man-made global warming.
global change to society’s transformation into
modernity. So, (b) is our choice for answer 9. (c) Between 1 and 1000, 5 will occur in the units,
response. It is possible that some may choose tens, and hundreds places.
(c) on the basis that individuals make up society. In the units place, between 1 and 100, 5 occurs
in 5, 15, 25, 35, 45, 55, 65, 75, 85, and 95,
4. (b) The passage does not mention causes for soil
i.e., 10 times. Between 1 and 1000, 5 in the
erosion, so assuming that soil erosion is mainly
units place will occur in 10 × 10 = 100 times.
anthropogenic is not justified (even if we are
In the tens place, 5 occurs 10 times between
accustomed to hearing this nowadays). So,
1 and 100: 50, 51, ... 59. So, between 1 and
statement 2 is not acceptable. Answer responses
1000, 5 occurs in the tens place in 10 × 10 =
with 2 in them cannot be correct; (a), (c) and
100 times.
(d) may be rejected, leaving only (b) as correct.
In the hundreds place, 5 occurs 100 times
The passage does not justify making an
between 1 and 1000: 500 to 599.
assumption on food security. So, statement 1
So, between 1 and 1000, 5 occurs 100 + 100
is not acceptable.
+ 100 = 300 times.
5. (b) From the passage it is clear that economic [Any digit between 2 and 9 will occur 300 times

ST . 169
Sample Test 9 : General Studies Paper II (2019)

between 1 and 1000. The number 1 would occur If the 1st is on Wednesday, the fifth Thursday
301 times, as it is also in the thousands place will be the 30th, in which case 20th December
in 1000.] will be a Wednesday. (There is no Wednesday
among the answer responses, however.)
10. (c) Under the given conditions, the four big cubes
will have to form the middle layer of the solid Now, if the 1st is on Thursday, the fifth Thursday
cube, otherwise one face of each big cube will will be the 29th, in which case 20th December
be painted blue. The 32 small cubes will be 16 will be a Thursday—which is answer response
to one side and 16 to the other. The solid cube (b).
will look like this: So, the birthday of Mr X’s third child falls on
a Thursday.
13. (c) Represent the statements diagrammatically
Rats = R; Cats = C; Dog = D; Cows = Co
Bl

D
ue

R C
Co
Black

Co Co
Co

Each of the 4 shaded small cubes will have only Take the conclusions one by one.
@UPSC_THOUGHTS

one of their faces painted. There will be another I. We just know that dogs and cows are
set of 4 such cubes on the opposite side. So, mutually exclusive. There are many
there are 8 cubes in total with only one face possibilities for how cows stand in relationship
painted. with cats (or rats), so we cannot categorically
say ‘No cow is a rat’, for a cow may or may
11. (d) Answer response (a) may seem the correct one
not be a rat. Conclusion I cannot be drawn.
but think it over.
If the weight increases by 60% on adding B, II. We are given that some cats are dogs. But
it is the whole weight of A + B that has to be we do not know about all dogs: some may
considered when B is removed. or may not be rats. Conclusion II cannot be
Let weight of A be 100, then weight of B is 60. drawn.
Total weight A + B = 160. III. If some cats are rats, it follows that some
60 rats are cats. So, conclusion III is acceptable.
B as a proportion of A + B = .
160 Answer response (c) is correct.
60 14. (d)
Removing B will cause a reduction of in 1 2
160 3 4
the total weight. As a percentage, this is 1
A B C D

60 75 E F G H
× 100 = = 37.5. 2
160 2
I J K L
So 37.5% of the total weight will be reduced 3
on removing B. M N O P
4
12. (b) Let’s see if we can get the solution without
taking April into consideration, as we are asked Easy to note 1 big parallelogram ADPM and the
only about the day for the December date. In nine small parallelograms, together amounting
any month of 30 days, two days will occur 5 to 10.
times. In November (a month of 30 days), if Now count systematically:
there are 5 Thursdays, the first of the month
ADHE, EHLI, ILPM, ABNM, BCON, CDPO = 6
will be on either a Wednesday or a Thursday.

ST . 170
Sample Test 9 : General Studies Paper II (2019)

ABJI, BCKJ, CDLK, EFNM, FGON, GHPO = 6 20. (c)


ACGE, EGKI, IKOM, DBFH, HFJL, LJNP = 6
100
ACOM, BDPN, ADLI, EHPM = 4
ACKI, BDLJ, EGOM, FHPN = 4 70 30 60
Total = 10 + 6 + 6 + 6 + 4 + 4 = 36
[The shorter method: 4C2 × 4C2 Indians Vegetarians

4×3 4×3
× = 6 × 6 = 36.] Total participants = 100
2 2
Indians + Vegetarians = 70 + 60 = 130
15. (b) The identification numbers assigned to those Common to both = 130 – 100 = 30
who play football as well as cricket must be Non-vegetarians will be 100 – 60 = 40
divisible by the LCM of 4 and 6.
The LCM of 4 and 6 = 12. 100
The numbers between 1 and 100 that are
divisible by 12 are: 70 10 40
12, 24, 36, 48, 60, 72, 84, 96 which is a total
Indians Non-vegetarians
of 8.
16. (b) If 80% of the race measured 12 km, 100% Total participants = 100
would measure Indians + Non-vegetarians = 70 + 40 = 110
12 Common to both = 110 – 100 = 10
= × 100 = 15 km.
80
@UPSC_THOUGHTS

So, at least 30 Indian participants are vegetarian


17. (a) 75% of the cost of the mobile is ` 9000 and at least 10 Indian participants are non-
The full cost of the mobile will be vegetarian.
9000 21. (d) Answer responses (a) and (c) may be true in
× 100 = ` 12000.
75 themselves, but the passage makes no reference
With the borrowed amount of ` 2000, Raju has to these aspects of IoT. Answer response (b)
9000 + 2000 = ` 11000 has no justification with reference to the passage.
This is short of the amount he requires. The passage is certainly concerned with security
issues of IoT, and this is reinforced by the last
18. (b) If Meenu’s age in 2002 was x years, Meera’s
line. Answer response (d) is the only logical
age was 3x.
inference to be drawn from the passage.
In 2010, Meenu’s age is x + 8 and Meera’s age
is 3x + 8. 22. (b) The passage is about protection of digital data
It is given that in 2010 in the context of global trade. There is nothing
in it to suggest that globalisation is against
2 (x + 8) = 3x + 8
India’s interest, as answer response (a) states.
2x + 16 = 3x + 8 Answer response (b) follows logically from the
x = 8 passage which points out that an attempt is
Meenu was 8 years old in 2002. being made by developed countries to control
So, she was born 8 years earlier in and colonise the digital data of developing
countries. Obviously, in the circumstances, India,
2002 – 8 = 1994.
as a developing country, should protect digital
19. (c) If the price of 1 ball is x, the price of 1 racket sovereignty in global trade negotiations. There
is 3x. is nothing in the passage that can lead to the
As per the given conditions, conclusions in answer responses (c) and (d).
10x + 30x = 1,300 + 1,500 23. (a) Only answer response (a) is implied by the
40x = 2,800 passage that illustrates the use and importance
x = ` 70 of Big Data. The other answer responses do not
follow from the passage. Nowhere is it said that
The price of 1 racket = 3 × 70 = ` 210.

ST . 171
Sample Test 9 : General Studies Paper II (2019)

Big Data is created by big economies; it is 28. (b) There is nothing in the passage to conclude
collected and mined from developing countries. what is said in answer responses (a) and (c).
The passage is critical of the situation in which Answer response (b) is the logical and rational
developed countries access Big Data to try and message to be drawn from the passage. There
establish a monopoly over digital intelligence. are other factors involved in the cure of the
disease besides diagnosis and medicines.
24. (a) The question asks what the ‘corollary’ to or
Poverty, lack of awareness, social attitudes, and
possible result of what is stated in the passage.
the severe side effects of medicines that
Only answer response (a) is a logical and
demotivate patients from continuing treatment
rational corollary to the passage. Refer to the
are factors that have to be tackled besides
lines, “How significant . . . national food security.”
medical treatment. Answer response (d) may be
Obviously, supporting this section of the farming
partially correct; if poverty is tackled through the
community is required if the effects of climate
effective implementation of poverty alleviation
change are to be countered and sustainable
programmes, disease control would be easier,
development along with food security is to be
but the statement ignores the other ‘truth’
achieved. Answer response (b) is contradictory
expressed in the passage. Answer response (b)
to what the passage says. Neither (c) nor (d)
would include poverty eradication when it says
are supported by the passage.
curing TB requires more than diagnosis and
25. (c) The fact that small farms produce 41 per cent medical treatment.
of the country’s food grains and food products
29. (b) If the first and fifth floors are of one colour, each
implies that the food security of India depends
of the other three floors will have a different
on these small farmers to a significant extent,
@UPSC_THOUGHTS

colour.
and if they are adversely affected, food security
is likely to pose a problem. So, statement 1 is 30. (d) Draw a diagram based on the directions and
a correct implication. Climate change is expected the distances of the towns. You will get a right
to bring about natural disasters which would angled triangle. The distance between Q and
adversely affect the farmers, so it follows that R is the hypotenuse. Use the Pythagoras
India will need to strengthen its disaster Theorem to solve.
management capabilities. So, statement 2 is 60
Q P
also implied.
26. (c) Statements 1 and 3 are clearly assumptions in
? 80
the passage. Would statement 2 also be an
assumption? The passage points out the
possibility of governments and companies facing
R
economic problems: note the use of words like
‘could have’, ‘could be’, and ‘may’. Moreover, We know that PQ2 + PR2 = QR2
only some kind of companies are likely to be
PQ 2 + PR 2 = QR 2
affected. One cannot, therefore, categorically
assume that extreme weather events ‘will’ reduce (60)2 + (80)2 = QR
economic growth of governments and companies
3600 + 6400 = QR
in general. So, statement 3 is not acceptable.
10000 = 100 = QR
27. (a) Statement 1 is an assumption as the passage The distance between Q and R is 100 km.
elaborates on the goal not being achieved.
Statement 2 is an assumption in that it admits 31. (a) The question seems a little confusing. However,
that the goal has not been achieved. The let us assume the total number of members of
the club to be 100.
passage does not support statement 3 as a valid
assumption. So, 80 members went shopping, 50 went
sightseeing while 10 rested, i.e., neither went

ST . 172
Sample Test 9 : General Studies Paper II (2019)

shopping nor went sightseeing. means a football player does not play cricket.
Diagrammatically: From that we cannot conclude that cricket
players do not play football; a cricket player may
90 or may not play football. So, conclusion 3 cannot
100
be drawn from the given data.
80 40 50 10
33. (b) All counting numbers are natural numbers—
resting 1, 2, 3, …
shopping sightseeing
Let the two-digit number be 10x + y.
As 10 were resting, it is out of 90 members that After reversing the digit, we have 10y + x
80 went shopping and 50 went sightseeing. Given,
The members common to the two groups 10x + y 4
= 80 + 50 – 90 =
10y + x 7
= 130 – 90 = 40
70x + 7y = 40y + 4x
So, 40 members out of 100 (in total) or 40%
70x – 4x = 40y – 7y
went shopping as well as sightseeing. Statement
1 is correct. 66x = 33y
Now, those who went only for shopping number x 33 1
80 – 40 = 40. Statement 2 is not correct. = =
y 66 2
[If 80% and 50% are to be of 90 (that is after
That means if x is 1, y is 2. We need to find
leaving out the 10 who choose to rest at the
pairs of numbers to suit the conditions. The
hotel), the answers are going to be very different.
numbers would be:
@UPSC_THOUGHTS

In that case, the numbers would be


(12, 21), (24, 42), (36, 63), and (48, 84)
80
Shoppers = × 90 = 72 Beyond that we are not able to get two-digit
100
numbers and their reverse form in the ratio of
50 4 : 7. So, there are 4 such pairs.
Sightseers = × 90 = 45
100 [You could just take two-digit numbers and their
Common figure = 72 + 45 – 90 = 27
reverse forms and see if they are in the ratio
27 of 4 : 7.
Percentage = × 100 = 30%
90 Obviously, 10 and 11 do not fit the needs.
Statement 1 is not correct in this case. 12 ? The reverse is 21.
Those who went only for shopping would be
12 4
72 – 27 = 45 = 3
21 7
45
Percentage = × 100 = 50% So, 12, 21 is a suitable pair of numbers.
90
Statement 2 is also wrong in this case. Multiples of 12 will clearly give you further such
pairs:
The answer response would be (d).]
24 4
32. (d) None of the conclusions follows from the given 24, 42 → =
42 7
statements. Just because every football player
36 4
has a two-wheeler but does not play cricket, we 36, 63 → =
cannot jump to any conclusion about the cricket 63 7
players (60%) and two-wheelers. So statement 48 4
1 cannot be a conclusion. Statement 2 is almost 48, 84 → =
84 7
the same as statement 1: though we are told
that all football players have two-wheelers, we The next multiple of 12 would be 60; its reverse
do not know if cricket players (who form 60% is 06 and the pair is not in the ratio of 4 : 7.
of the students) do or do not have a two- So, there are 4 pairs of such two-digit numbers.
wheeler. As per the information given, a player You may try out two-digit numbers beyond 12,
who does not play cricket plays football—which but you will find none conforms to the given

ST . 173
Sample Test 9 : General Studies Paper II (2019)

conditions. You need a number that is divisible maximum and minimum values. Answer response
by 4 and its reverse divisible by 7 at the same (c) is correct.
time. Check (d) anyway. Take x as 30 and y as 25;
34. (b) From the given data, if B scores x marks, A x + y = 30 + 25 which is neither greater nor
scores x + 20 marks; also B’s marks are 5% equal to 65. So, (d) is not always correct.
less than those of A. 38. (b) Ena’s mother is 24 years older than Ena who
In other words, 5% of (x + 20) = 20 marks. is now 13 years old; so Ena’s mother is now
Try out the answer responses: 24 + 13 = 37 years old. As Ena was born 4
(a) → If B scores 360, A scores 360 + 20 = 380. years after her parents’ marriage, her parents
have been married for 4 + 13 = 17 years. Ena’s
5 mother must have been 37 – 17 = 20 years
5% of 380 = × 380 = 19 ≠ 20 (×)
100 old at the time of the marriage. As she is 3 years
(b) → If B scores 380, A scores 380 + 20 = 400 younger than Ena’s father, Ena’s father must
have been 20 + 3 = 23 years old at the time
5 of his marriage.
5% of 400 = × 400 = 20 ( )
100
[Alternatively, Ena’s mother is 24 years older
So, (b) is the answer. than Ena. Ena was born 4 years after her
If A’s marks are more than 400, 5% of those parents’ marriage. So, her mother must have
marks will be above 20, so would not meet the been 24 – 4 = 20 years old at her marriage.
conditions. Ena’s father, being 3 years older than her
35. (d) Seeta’s dates (with a gap of 2 days): mother, must have been 20 + 3 = 23 years old
at the time of marriage.]
1, 4, 7, 10, 13
@UPSC_THOUGHTS

Geeta’s dates (with a gap of 3 days): 39. (b) Let’s suppose the price of each mobile before
discount was ` 100.
1, 5, 9, 13
Rakesh could have bought 8 mobiles with the
So, the two will go together again on 13th amount in hand, i.e., he had 8 × 100 = ` 800.
January. After discount, he was able to buy 10 mobiles
36. (b) The distance covered by X is 1000 m; the for ` 800. So, the discounted price for a mobile
distance covered by Y is 1000 – 40 = 960 m; 800
was = ` 80.
and the distance covered by Z is 10
1000 – 64 = 936 m. 100 − 80
Discount percentage = × 100 = 20%.
100
For a distance of 960 m,
Y gives Z a start of 960 – 936 = 24 m. 40. (b) The total marks of 100 students before correction
= 40 × 100 = 4000.
For a distance of 1000 m, Y will give Z a start
24 By mistake, 83 – 53 = 30 extra marks have been
of × 1000 = 25 m. included in this total.
960
The corrected total marks = 4000 – 30 = 3970.
37. (c) The question asks about a value that is always
The corrected mean
correct. Just plug in some numbers for x and
y, and check each answer response. 3970
= = 39.7.
Suppose x is 30, y can be 39, x < y; so, 100
response (a) need not always be correct.
41. (a) This is the only logical and crucial message to
Taking the same values for x and y, y – x =
be drawn from the passage. The passage does
39 – 30 = 9, which is not greater than 15.
not say anything about abolishing intellectual
So, response (b) is not always correct. property rights or taking the matter to the WTO.
The minimum value of x is 25, and the maximum Nor does it lead one to call for a ban on private
value of y is 40; y – x = 40 – 25 = 15. If the enterprise in agribusiness in India, or to conclude
value of x is greater than 25 or the value of that the circumstances in India are against GM
y is lower than 40, y – x will always be less crops. The passage seems to endorse research
than 15, which is the difference between their in the field of genetic modification to concentrate

ST . 174
Sample Test 9 : General Studies Paper II (2019)

on areas relevant to India; as a corollary one beneficial) relationship between the economy
could conclude that public sector institutions and labour-intensive and natural resource-based
would be more suitable than private corporates livelihoods, the latter would be contributing more
for undertaking this sort of research. to the growth of the former. So, statements 1
and 2 are implied in the passage. There is no
42. (a) Statement 1 is correct as the passage points
comment in the passage on the service sector;
out that private corporates are interested in
moreover, the statement by itself is not quite
profit and, therefore, concentrate on genetic
correct, as many areas of the service sector in
modifications that would be short-lived. They do
India are labour-intensive. Statement 4 seems
not pay attention to issues such as resistance unfounded.
to the stresses of natural calamities like drought
and flood. Statement 2 cannot be an assumption 47. (b) The other answer responses are not logical
as the passage does not talk of global warming, inferences to be drawn from the passage.
though it does talk of drought and flood and
48. (c) Trace the difference between the numbers in
other stresses. However, it does not seem to
sequence:
assume that GM technology will not be able to
cope with problems caused by these stresses,
but indicates that research in this direction was
needed. The numbers are reduced by consequent prime
numbers. After 13, the next prime number is 17;
43. (a) This seems to be the most acceptable ‘inference’.
minus 17 from 93 to get X.
However, answer responses (c) and (d) also
could be inferred. 49. (a) By 8:00 a.m. of Wednesday, the clock would
have gained 20 minutes. By 8:00 p.m. of
44. (a) Statement 1 is an assumption supported by the
Wednesday (another 12 hours), it would have
passage on the whole. The fact that there is
@UPSC_THOUGHTS

gained half of 10 or 5 more minutes, i.e., a total


a need for surveillance machinery involving
of 25 minutes from Monday 8:00 a.m.
several government agencies and training staff
for efficient inspection speaks of the assumption At 6:00 p.m. on Wednesday, therefore, the clock
would have gained less than 25 minutes.
that great investments have to be made as
pointed out in statement 2. Statement 3 may The correct time would be less than 25 minutes
be correct in itself, but it does not seem to be behind 6:00 p.m.
an assumption in the given passage. Only answer response (a) seems likely as it
shows a difference of 24 minutes (6:00 – 24
45. (d) The passage clearly speaks for the need to plan minutes → 5:36). Answer response (b) shows
for cities, taking into consideration the interests a difference of 30 minutes. Answer response (c)
of people who live in them. So, (d) is the correct shows a difference of 36 minutes. Answer
answer response. Answer responses (a) and (b) response (d) shows a difference of 42 minutes.
are irrelevant in the context of the passage.
[Alternatively, you may work out how much the
There is nothing in the passage to suggest that
clock gains in 1 hour:
cities have become the monopoly of builders
and the interests of the elites. According to the In 24 hours, the clock gains 10 minutes. In 1
passage, technical experts, without much idea hour, it gains 10/24 minutes. From Monday 8:00
of what the people actually need or want, have a.m. to Wednesday 8:00 a.m., it is clearly 48
been drawing up plans for cities. Pointing out hours, so the clock would have gained 20
that the interests of working and poor people minutes by then. From 8:00 a.m. to 6:00 p.m.,
have been neglected in the planning of cities it is 10 hours. In that period, the clock would
does not mean arguing for cities to be planned have gained
mainly for these people; answer response (c) 10 100
× 10 =
is not acceptable. 24 24
100 4 1
46. (a) Though the passage does not mention =4 =4
technological progress as being the main impetus 24 24 6
1
for modern economic growth, it is implied; if Converting to seconds.
6
people (workers) are not behind the growth, 1
technology is the alternative. Statement 2 is × 60 = 10 seconds.
6
fairly obvious: if there were a symbiotic (mutually = 4 minutes and 10 seconds.

ST . 175
Sample Test 9 : General Studies Paper II (2019)

Remember 60 seconds to a minute, and 60


12
minutes to an hour. French
Time gained by 6:00 p.m. will be 20 + 4:10 = 7 English
24 minutes and 10 seconds. 8
So, the correct time is 4 3 5
6:00 p.m. – 24 minutes (approximately)
= 5:36 p.m.]
50. (b) A proper fraction is always less than 1, with the Both French and English
numerator less than the denominator.
Answer responses (a), (c), and (d) are not Those who can read only French are:
correct. You can try out with a few fractions. 7 – 3 = 4.
1 1+ 1 2 Those who can read only English are:
→ =
2 2 +1 3 8 – 3 = 5.
The total number of people who can read only
2 2 +1 3 one language is: 4 + 5 = 9.
→ or
3 3 +1 4 Answer response (b) is correct.

1 1+ 1 2 53. (c) Pages 1 – 9 → digits used 9


→ = Pages 10 – 99 → digits used 2 × 90 = 180
4 4 +1 5
Pages 100 – 999 → digits used 3 × 900 = 2700
1 1+ 1 2 1 Total digits = 9 + 180 + 2700 = 2889
→ = =
5 5 +1 6 3 Number of digits left = 3089 – 2889 = 200.
@UPSC_THOUGHTS

In all the above cases, the resultant fraction is 200


Number of pages with 4 digits = = 50.
greater than the original fraction. 4
Total number of pages = 999 + 50 = 1049.
51. (b) The numbers in the sequence progress in a Answer response (c) is correct.
highly uneven way. There does not seem to be
a clear link between the numbers taken 54. (b) Plug in the answer responses and see if you
sequentially. The sequence is most likely to be get a meaningful sequence. Only answer
composed of two alternating sequences. response (b) works:
Take alternate numbers from 4 onwards: ccacc / aabaa / bbcbb
55. (d) Weight of the 2 children and the mother
4, 196, 16, 144, 36, 100, 64, X
= 3 × 50 = 150 kg.
As the difference between the numbers is large, Weight of the 2 children and the father
it is likely that they are squares of numbers. = 3 × 52 = 156 kg.
Now, X is in the sequence 196, 144, 100 … Weight of the children
Try out if the numbers are squares. = 156 – 60 = 96 kg.
196 = 142 Weight of the mother
144 = 122 = 150 – 96 = 54 kg.
100 = 102
56. (c) The different ways the bill can be paid using
It is clear that this sequence is composed of
all the rupee denominations are as follows:
squares of numbers decreasing by 2 at every
step. 50 + 50 + (7 × 1);
So, X = (10 – 2)2 or 82 = 64. 50 + (57 × 1);
Answer response (b) is correct. 50 + (5 × 10) + (7 × 1);
If you check the other sequence (4, 16, 36, 64), 50 + (4 × 10) + (17 × 1);
you will find it composed of squares of numbers 50 + (3 × 10) + (27 × 1);
increasing by Z at every step. 50 + (2 × 10) + (37 × 1);
50 + 10 + (47 × 1)
52. (b) If 3 out of the 15 people cannot read either
→ total of 7 ways.
language, we are left with 12 people who are
Now, leaving out the 50-rupee notes but using
able to read either or both French and English.

ST . 176
Sample Test 9 : General Studies Paper II (2019)

the 10s, the bill can be paid in the following So, S2 alone is sufficient to answer the given
ways: question.
(10 × 10) + (1 × 7); 10 × 9 + (17 × 1); and
59. (d) The dates fall on the same day every 6 years.
so on till we use (10 × 1) + (97 × 1) → a
(If there were no leap year, the repetition would
total of 10 ways.
take place every 7 years.) So, 6 years after
Now using just the 1-rupee notes, the bill can
2009, namely, 2015 would have the same
be paid in only 1 way => 107 × 1
calendar (2012 having been a leap year).
So, the bill can be paid in 7 + 10 + 1 = 18
different ways. 60. (d) The test of divisibility by 3 says that a number
is divisible by 3 if the sum of the individual digits
57. (b) Make a diagram of the routes, being careful of
is divisible by 3. If 7A3 is to be exactly divisible
the directions and the distances. We may begin
by 3, then 7 + A + 3 must be divisible by 3.
with A and B moving together.
If A is 1, 7A3 is 713, the sum of the digits being
A 12 m 83, which is not divisible by 3.
A’s office
y Let us take A to be 2; 7 + 2 + 3 = 12 which
A 2m z
10 m is divisible by 3.
N
A and B 4m Now, according to the question,
W E B 723 = 5B7 + 136
8m 723 – 136 = 5B7
8m ?
S 587 = 5B7
∴ B = 8.

B
61. (b) Answer response (a) also seems correct, as the
4m 5m
passage considers the necessity of an effective
@UPSC_THOUGHTS

B’s office x regulatory system in the context of improving


B
India’s economic footprint, which does mean
XYZ in the diagram forms a right-angled triangle ‘size’ as well as ‘impression’, and points out
with the distance between the two offices XZ what is lacking in the regulatory system as it
being the hypotenuse. exists in India at present. So, a better regulatory
XY = 5 + 8 + 2 = 15 system would help India achieve a size of
YZ = 12 – 4 = 8 economy appropriate to the population. But the
These are entries in a Pythagorean triple: 8, 15, ‘crucial’ message seems to be the strategic use
17. So, the hypotenuse is 17. of the regulations (seen as ‘weapons’) if India
The distance between the two offices is 17 m. is to make an impact economically in a holistic
[You may solve it if you do not remember manner in the competitive global economy. So,
(b) seems the better answer response. Answer
Pythagorean triples.
response (c) is not acceptable as the passage
XZ 2 = XY 2 + YZ 2 does not speak about India’s integration with
= global economy. Answer response (d) is not
152 + 82
correct. To say that the regulations fail to
= 289 consider the impact on jobs and economic
XZ = 17]. growth is not to say that these aspects have
been dominant in developing a regulatory system.
58. (b) Take S1: p and q being prime numbers, pq may
At most, these aspects must be kept in mind
or may not be odd; e.g., 2 and 3 are prime
while drawing up the regulations.
numbers, 2 × 3 = 6 (even); 3 and 5 are prime
numbers, where 3 × 5 = 15 (odd). 62. (d) The passage says: “Regulations may be used
Take S2: if p is 3 and q is 4, p + q = 7 (odd), to protect local markets at the expense of more
and pq = 3 × 4 = 12 (even); if p is 5 and q widely shared prosperity.” Nowhere in the
is 2, p + q = 7 (odd), and pq = 2 × 5 = 10 passage is it said that social and environmental
(even); if p is 7 and q is 4, p + q = 11 (odd), concerns are generally ignored by governments
and pq = 4 × 7 = 28 (even). So, the answer while implementing regulations. So, neither
to the given question can always be ‘No’. statements are valid assumptions.

ST . 177
Sample Test 9 : General Studies Paper II (2019)

63. (c) There is nothing in the passage to lead us to consumption ensuring crop diversity. Assumption
infer anything about the treatment of 1 is implied in the passage, which speaks of
malnourished children; so, answer response (a) technology-driven production and distribution of
is not acceptable. The passage has nothing food and high dependence on high-yielding
suggesting the inoculation of malnourished seeds, which have led to reduction of food crops
babies. Answer response (b) may be rejected. —which are plant species. But it is the human
Answer response (c) is the best. This is a logical mind that is behind the technology and the
and crucial inference to be made. As the low tendency to depend on the limited diversity. So,
levels of the particular essential sugar in the milk assumptions 1 and 4 are correct.
of malnourished mothers lead to malnourished
68. (d) Just write the alphabet according to the given
babies, it would help to feed the babies with
conditions, starting from b and ending with y.
fortified dairy milk to make them healthy. Answer
bCdEfGhljKlMnOpQrStUvWxY
response (d) seems irrelevant.
Answer response (d) is correct.
64. (b) Fortified dairy milk is not processed probiotic [You may just go backwards. You know ‘b’ is
food, so, there is nothing in the passage that lower case, so ‘A’ must be capitalised. Taking
supports assumption 1. Assumption 2 is ‘z’ as previous to A, we know ‘z’ is small, so
supported by the passage. ‘Y’ must be capitalised. Only answer responses
(a) and (d) conform to this. Now just check for
65. (b) Assumption 1 generalises from the particular
‘J’ which comes to be lower case; (d) is the
instance of Western Antarctic Peninsula, which
correct answer.]
is not justified. The passage talks of suspended
sediment in water causing loss of biodiversity 69. (d) Let the length of first piece be x, that of the
@UPSC_THOUGHTS

in coastal regions, not seafloor sedimentation, second piece be y, and that of the third piece
so, assumption 2 is not acceptable. Assumption be z.
3 is correct, as it is supported by the passage. As per the given conditions,
3+5+7
66. (b) The passage is not about the frequent or x = = 5
3
infrequent occurrence of global warming, so z
y = 5+
assumption 1 cannot be accepted on the basis 3
of the passage. Assumption 2 contradicts what z
z = 5+5 +
is said in the passage. Assumption 3 seems to 3
be correct as the passage elaborates on the 15 + 15 + z
z =
difference in adaptability of ecosystems to global 3
warming of an earlier era and the global 3z – z = 30
warming of recent times, pointing out that 30
z = = 15
resilience has been lost by ecosystems affected 2
by human-induced changes to the environment. z
Now, y = 5 +
3
67. (d) This seems to be the most acceptable answer 15
response. Reading through the assumptions, y = 5 + = 10
3
assumption 4 is certainly supported by the The length of the original piece of cloth
passage: “If some mutating . . . let go extinct.” = x + y + z
Assumption 3 has a categorical tone in the ‘will = 5 + 10 + 15
finally’ that is not justified in the context of the = 30 m.
passage, which only suggests a possibility and
70. (a) 1, 5, 7 is the only such 5. The other 5s are
not a certainty; moreover, there is no reference
to the ‘near future’. So, assumption 3 is not preceded by 3.
acceptable. Assumption 2 is not based on the 71. (d) Make a diagrammatic representation of the
passage. The sentence, “Farmers in the past given data.
. . . environment” speaks of production and not Use all the known facts.

ST . 178
Sample Test 9 : General Studies Paper II (2019)

f → female; m → male Total number of triplets is 6 + 3 + 1 = 10


[If you do not know or remember the formula,
m
you can still solve the problem.
(i) x y z
G Sister-in-law of F 1 2 3
Wife
1 3 2
f 2 1 3 = 6 ways
2 3 1
3 1 2
Siblings 3 2 1
Cousin
D B C (ii) x y z
1 1 4
Daughter 1 4 1 = 3 ways
4 1 1
A f (iii) x y z
2 2 2 → 1 way
Only E is left. E must be the wife of F to Total 6 + 3 + 1 = 10].
complete the joint family. Now, statement 1 is 74. (d) 10 # 5 @ 1 $ 5
correct. It follows that E is the grandmother of 10 – 5 × 1 ÷ 5
A; so, statement 2 is correct. If C is the cousin Using the BODMAS rule, we get
of B, E must be C’s aunt. Statement 3 is also 10 – 1 = 9.
correct. All three statements are correct.
75. (c) According to the rule of divisibility by 3, if the
@UPSC_THOUGHTS

72. (b) Visualise the cube in its different facets. sum of the digits of a number is divisible by
3, then the number is also divisible by 3. So,
Black face White face ways
we may assume:
6 0 1
4 + 2 + 5 + 2 + 7 + 4 + 6 = 30 + B.
5 1 1
4 2 2* B has to be 0, or a multiple of 3.
3 3 2** The possible values for B are: 0, 3, 6, 9.
2 4 2*** Therefore, 4 values of B are possible.
1 5 1
0 6 1 For items 76 to 78, there are two possibilities
for the highest and lowest scores.
* White faces opposite each other or adjacent (i) F B
to each other I rank VI rank
** White faces may be adjacent to one another, (ii) C E
or alternate with the black faces I rank VI rank
*** Same as for 2 white faces Now, take the questions keeping in mind
There are 1 + 1 + 2 + 2 + 2 + 1 + 1 = 10 D > A > B; and C > A
different ways to paint the cube. As we are told A and D are close competitors,
it is likely that no one is between them.
73. (d) If x, y, z are natural numbers whose sum is 6,
there are the following possibilities: 76. (c) This is case (ii). We know
(i) x = 1, y = 2, z = 3 C > F _ _ _ E
1 + 2 + 3 = 6 The only possibility is:
3 ! = 6 C > F > D > A > B > E
(ii) x = 1, y = 1, z = 4 B’s position is fifth.
1 + 1 + 4 = 6
77. (d) This will be a case of (i). Now, the possibilities
3!
= 3 of the ranks are:
2! 1. F > C > D > A > E > B
(iii) x = 2, y = 2, z = 2
2. F > E > C > D > A > B
2 + 2 + 2 = 6
3. F > C > E > D > A > B
3!
= 1 So, the rank of C will be either second or third.
3!

ST . 179
Sample Test 9 : General Studies Paper II (2019)

78. (b) If E is ranked third, it is a case of (i) 79. (d) From the given data we cannot determine
The ranking will be as follows: whose weight is the greatest. From S1 we can
F > C > E > D > A > B gather that Sohan weighs less than Mohan or
C gets more marks than E. Rohan (as 2X Sohan’s weight is less than the
For Items 79 and 80, analyse the two given
weight of Mohan or Rohan). From S2 we cannot
statements to reach conclusions.
infer that Rohan singly weighs either more or
If S is Sohan, M is Mohan, and R is Rohan,
less than Mohan.
2 S < M or R → M or R > S or S < M
or R 80. (b) From S1 we can infer that Sohan weighs less
2 R > M or S. than Mohan or Rohan.

@UPSC_THOUGHTS

ST . 180
1
COMPREHENSION
Comprehension means the ability to understand something. Civil servants
are expected to go through a lot of information (just think of all those
files), perhaps at times very quickly, before having to make a note for
the senior officer’s attention, writing out a reply to a question in
Parliament, making a decision or recommending a course of action. As
such, they need to develop the ability to read quickly and assimilate and
correctly understand the essentials of a matter. Comprehension tests are
probably intended to assess the ability of a potential civil
@UPSC_THOUGHTS

servant in this area.

Comprehension tests are usually a means of assessing one’s understanding


of a language, and are thus almost always a part of a language test or
what is called ‘verbal ability’ test. As per the UPSC syllabus,
‘Comprehension’ will be in Hindi as well as in English. The questions
could demand analytical skill, as they would not only ask about the
literal content of the passage but would ask about the general theme,
the central idea, statements that support or oppose, what the writer
intends, and so on.

In order to do well in the test, you need to develop some skills, and
you need to practise answering questions. The following pages are
designed to provide you with both the strategies and the practice.
We have included passages of varying lengths—ranging from some
100 words to 450 words—so that you can tackle any type of
passage and get the required practice.
Comprehension

@UPSC_THOUGHTS

1.2
Comprehension

Comprehension
ORIENTATION
This approach will stand you in good stead in
READING THE PASSAGE
your future career besides preparing you effectively
for the task in hand—tackling the comprehension
Make a Consistent Effort
section in the examination.
A comprehension test is always a test of language—
even if language-specific questions (such as finding Test Basics
synonyms and antonyms) are not asked. It is only by The test is of the multiple choice type. There will be
knowing a language well that you can understand a passage followed by some questions. The passage
what is being said in a piece of writing. The use of may relate to any subject. Each question will have
words, the style of writing and the meaning—obvious a set of answer responses out of which you have to
and implied—conveyed by the words in a piece of choose the correct (or the most nearly correct) one
writing have to be understood. Knowledge of a wide to answer the question. You will have to mark your
vocabulary will help you detect subtle differences in choice on the answer sheet as indicated in the test
meaning of sentences that could be the key to the booklet.
meaning of an entire passage. So the command of a There may be more than one passage, and the
language is necessary. length of a passage may vary from the short (about
@UPSC_THOUGHTS

Your work in this direction should have begun 100 words) to the long (about 400 or more words).
long before this moment. Even now, however, it is In the 2011 paper—the first one after the change in
not too late to improve your basic knowledge of the syllabus—several passages (7 to be exact) were
language. Set yourself a target and read standard given. These passages were not all given in sequence,
newspaper and magazine articles every day. Make but were dispersed over the test paper. They ranged
your reading varied: the subjects should range over in length from about 150 words to 300 words. Over
politics, economics and business, science, culture and the years, the number has varied; so has the length.
topics of social interest—in fact, everything under the There is obviously no set pattern, except that several
sun. passages are to be expected. We have not stuck to
As you read, try to understand not just what the any specific length in our selection of passages or a
writer is writing about, but also how the matter has fixed number of questions. The emphasis is on helping
been organised. Try to locate the main points of the you to tackle all kinds of passages and types of
article and summarise the views to yourself. Do all questions, so that you may confidently face the test.
this, not because the comprehension passage may be Whatever the length of the passage, there are two
set from one of the articles you read, but because you points you have to keep in mind while preparing for
learn to read and understand several things, get the test.
familiar with varied subjects, and develop the ability 1. Time management is important: you don’t
to analyse a writer’s approach to a subject, the style have an indefinite amount of time for reading
of writing, the way a piece of writing is organised, the passage and answering the questions.
whether the writer is biased or neutral or aggressive; 2. You need to choose the correct answer response
do it, in other words, to get the practice that will be quickly: your aim is to answer correctly as
of use in the test. many questions as possible—preferably all—
Develop your vocabulary by consulting a good in order to get a high score. You are not being
dictionary whenever you come across a new word, assessed for exhaustive reading!
or when you find a word used in a way you are not So, is reading to be avoided? That’s, of course,
familiar with. The word could have different meanings a silly question; answering the questions demands
in different contexts. By the way, this need to look that you read and understand the given passage.
up words is a lifelong affair. What matters is how you read it.

1.3
Comprehension

Test Strategies A passage will usually be structurally organised.


Should I read the passage or the questions first? This The main ideas of the paragraphs must in some way
question bothers most students, especially in the face be related to the main idea—or theme—of the passage
of consistent advice from teachers that the passage must as a whole. So, if you understand the main ideas
get a ‘preview’ first. But why not the questions first— of the paragraphs and the connections between
after all, you get an idea of what is required when you them, you will be able to comprehend the passage
go to the passage, and you will be able to find the as a whole. You will understand the purpose of the
answer quickly. There is really no hard and fast rule; supporting details. Recognising the structure of the
passage helps you to easily locate any detail asked
do what suits you after trying out both ways. Just
for in a question.
remember that whatever you choose to do should save
Some common structures of passages are:
you time and help you to choose the correct answer
(i) cause-and-effect development showing how
response. Teachers advise reading first because,
an event led to another;
according to them, reading the questions first could be
(ii) comparison or contrast between two things;
distracting and could thus make you waste time. (iii) argumentative discussion of an issue, giving
The ‘questions first’ approach could be useful if the opposing viewpoints;
given passage is short, i.e., in the range of 75-100 words. (iv) description of a single topic seen from the
Looking up the questions first can help you locate the angle of different theories;
answer faster when you read the passage, precisely (v) an idea illustrated with many examples;
because it is short and, therefore, easy to keep in mind. (vi) an idea or theory supported by many
But this approach is not found to work in the case of arguments;
longer passages: you may spend too much time looking (vii) a sequence of events in chronological order;
@UPSC_THOUGHTS

for specific answers, being forced to go back to reading (viii) definition and explantation of a special term
the entire passage more than once. In the case of longer It is possible that a single passage may fit into more
passages, you just have to do some methodical reading than one structure; a cause-and-effect sequence may
first. also be a comparison-contrast structure. It is not
recognition of the pattern, as such, that matters. You
How to read...
may choose any of the patterns (if more than one
● Skim the passage, or glance through the passage. are validly applicable) as the pattern is only a
One way is to glance through the passage, allowing vehicle for following the writer’s thought.
your eyes to move down the page, skimming the
● Look out for words that indicate a change or just
contents and gathering whatever information you
continuation. Some such transitional words are
are able to. The other is to actually read some of the
given below.
sentences from the passage; the first few sentences
cause-effect: because, led to, consequently, therefore,
of the passage; the first and last sentences of each since
paragraph; the last few sentences of the passage. comparison: similarly, like, more, less
● Scan the passage—read it with purpose—to find the contrasting information to come: however, but,
main ideas of each paragraph. How to locate the although, nevertheless, yet, conversely
main idea? Try to identify the subject—who or what definition: is called, is defined, is known as, is, is
the writer is writing about—in each paragraph. referred to as
Identify the topic of the paragraph, i.e., the aspect attention please: main, central, major, important,
of the subject being discussed. Try to express in a primary
sequence indicators: first, second, after, now, later,
concise sentence what the writer is attempting to
next
say on the topic. That would be the main idea.
clarification: for instance, for example, such as
Main ideas are usually of a broad and general not quite sure: may be, apparently, perhaps, alleged,
nature, while the supporting details tend to be was reported, seems.
specific and more limited. The main idea often These words could guide you along the writer’s
(though not always) occurs at the very beginning thought process and help you understand the
or at the end of a paragraph. passage better.

1.4
Comprehension

● As you read, watch out for the author’s style. A


ANSWERING QUESTIONS
technical piece of writing would generally not carry
words of an emotional overtone, for instance. But General Guidelines
in other kinds of writing the author’s intention or Let’s recall that this is a test of multiple choice
viewpoint comes out clearly through the choice of questions. So remember the very important—and
words, especially the verbs. There is clearly a often ignored—guidelines that we listed in our
difference between ‘sneaking’ into a room and Introduction. These can help you take the test
‘sauntering’ into a room; the former conveys an idea successfully.
of slyness or deception, while the latter conveys a In brief, don’t spend too much time over one
sense of casual, lazy, purposeless walk. A careful question; move on to the next one if you get stuck. After
study of the words and word pattern would tell you answering the questions you know, come back to the
if the writer’s tone is angry, sarcastic, humorous or questions which you found difficult first time round.
just straight-forward and factual. Questions are, at Now see if any of the other questions asked can help
times, directed at the ‘tone’ of the passage. you answer the ‘tough’ ones. Sometimes questions are
● Make marginal notes and mark the text of the answered in later questions asking for different things.
passage. This is another advice consistently given However, try this only after you have finished the test
by teachers and equally consistently ignored by and have some time remaining to try the questions with
most students who feel that it is a waste of time. which you had you have had problems.
In fact, this effort will save time in the long run.
Tackling Comprehension Questions
One usually takes a test under pressure; in the When you come to the questions that follow the passage,
circumstances, it is easy to forget things, lose your
@UPSC_THOUGHTS

teachers advise that you look at the stem or head of the


focus, and then feel stressed and out of control. The question first, but avoid looking at the answer responses.
more notes you make on your test booklet – or Frame your own answer to the question and then look
‘rough paper’, the less likely you are to get lost in at the answer responses. You are bound to find a
a mental drift, or be unable to locate the information response which tallies with the answer you framed:
you “know” you “saw somewhere in the passage”. that is the correct answer choice. Go through the other
If you make the right markings, whatever a question responses just to be sure.
asks, you will know where in the passage you are If none of the responses tallies with what you came
likely to find the answer to particular questions. So up with, read the responses carefully, see which ones
underline words that strike you as important, make can be eliminated, and then choose from those left what
marginal marks (say, a star or two small vertical appears to be the most correct one.
lines) against those parts that contain significant Work as quickly as you can on the easy questions
details, facts, or bits of information as you go without being careless. On more difficult questions
through the passage. This will maximise your where you are not exactly sure about the answer, using
‘comprehension’ of the passage and minimise the the elimination method can increase your chances of
time you take to find the correct answer. choosing the correct answer:
● Create a passage ‘map’ as you read—summarise— (i) Strike out (or put a cross against) answers that
very briefly—the main idea of each paragraph, you know are incorrect. At least one such
stylistic features and transitions. Once you know response is bound to be there.
the main idea, you will know that details supporting (ii) Read the remaining answers. These answers
the idea would be found around those words. Use could appear very similar. Probably, they will
abbreviations if you like (but do remember those all be correct to some degree. However, one
special abbreviations). answer will be more appropriate, more right
Once you practise in this mode, you learn to read than the others. The correct answer is likely
efficiently, understanding the most important features to be more precise and complete than the other
of the passage. For the other details, you can always likely responses.
come back to the passage—you know where to look. Questions can be broadly categorised as general

1.5
Comprehension

and specific. analytical ability to reach the correct answer.


Examples of general questions relating to the
passage as a whole are:
Pitfalls to avoid...
What is the main purpose/theme of the passage? A common mistake made by students is to bring their
How would you describe the structure of the passage? own knowledge to bear on their answers. This happens
What is the tone of the author? when the subject of the passage is familiar to the
student.
The very first reading of the passage should have
If you know a great deal about the subject of the
provided you with the answers to these questions.
passage, be doubly careful in answering the questions:
Ideally, there would be no reason to go back to the
you are required to base your answer or choice of
passage to answer these questions. But there is nothing
answer entirely on the passage. So, if the passage says,
to stop you from having another glance at the passage
for some reason, that only black cats exist, you had
if you feel uncertain.
better forget—for the space of answering the questions—
Such general questions have general answers, so
that you have seen white cats, spotted cats, and striped
you may ignore the responses that focus on one part
cats. In the multiple-choice type, especially, you may be
of the passage or are too specific in some other way.
tempted to pick an answer choice just because you
Specific questions relate to specific details, facts
know it to be factually true, or because you agree with
and information that would be scattered all through the
what it says. In either case, you would be committing
passage. The answers to these have to be found. And
a mistake if the substance of those answer choices is
this is when those jottings, underlining and marginal
not based on the information in the passage. The correct
notes—if you made them while reading the passage—
answer choice must reflect the opinions and facts
will come in useful. You will know where exactly in
@UPSC_THOUGHTS

expressed in the passage, irrespective of your own


the passage a particular bit of information is to be
beliefs and knowledge.
found. Go there, and read around the particular line,
Question-setters often resort to tricky answer choices
and you are bound to find the answer. Check the
in the multiple-choice type. Some of these sound
responses and choose the correct one.
plausible, and are often placed among the first few
If your jottings do not help, or you have not made
choices, say, as (a) or (b). Restrain yourself from marking
the notes required, don’t worry. The question is bound
it off immediately as the correct one; if you go on to
to have some ‘lead’ word or phrase which will certainly
read the other choices, you may find a better answer.
be there in the passage. Run your eye down the passage
In these questions, unlike in mathematics questions,
—don’t read—and locate that word or phrase: you have
there may be degrees of right and wrong, and you are
located the part that will almost certainly provide you
required to choose the best of the answer choices given.
with the answer. Now read carefully and check the
Yet another pitfall to avoid is marking as correct
responses for the correct one. Use the elimination
an answer choice just because it seems familiar; it
method if you need to.
probably is, because the idea occurs in some part of the
There may be questions that ask you to draw an
passage, but it may not be the answer to the question
inference. Such questions may belong to the ‘general’
or the ‘specific’ category. Examples: asked. It is always better to refer back to the passage
What can you infer about the author’s attitude in the to get the correct answer.
passage? Beware of the ‘Negative’ in the question. Common
What does the author imply by.... (reference to something examples are:
specific in the passage)? All the following are mentioned in the passage
Why does the author say.... (something from the passage)? except .....
Which of the following is not correct according to the
Inference questions may ask you to judge the passage?
author’s opinion, or even ask you to guess at the
author’s further conclusions. These may appear to be Among the answer responses there is bound to be
difficult questions, but they are not likely to be too one or more representing what is correct according to
‘subtle’ or revolutionary. You need to draw on your the passage. In your eagerness to answer, you may

1.6
Comprehension

choose one of these responses because you know it tips on how to use the elimination method to arrive
is what the passage says, whereas you had to choose at the correct answer and save time.
the response that is not given in the passage or is
Example 1
not correct in the context of the passage.
The industrial age has made us worshippers of
There is a question type which has become quite
wealth. In the social hierarchy, anyone who has the
a favourite with test setters. In this type, you are given
luck or ability to make money occupies a high place.
two or three statements (labelled 1, 2, 3 ... A, B, C ...
Before the industrial revolution, we had different
or I, II, III ...) within the question itself, and you have
standards of social valuation. Saints, men of learning,
to check if one or two or all of them are correct or if
poets and philosphers were at the top of the social
all of them are wrong – all according to the given
ladder. The days when human values like honesty,
passage. It is a bit tricky, but practice can gain you
sympathy, understanding and justice were considered
mastery over this type. What you need to check is
the most desirable qualities are over. The consumer’s
whether each statement within the question is correct.
appetite grows by what it feeds upon. Money-making
And be careful to check all the statements before you
has become one of the most popular industries of the
choose the answer response.
modern world.
To sum up....
Q 1. What is said in the passage about poets and
Comprehension tests are designed to assess your ability philosophers before the industrial revolution?
to analyse a written passage from several perspectives. (a) They belonged to the richest class of society
You are required to understand what is stated as well (b) They lived in poverty
as what is implied and the assumptions behind (c) They were highest in the social hierarchy
statements or arguments in the passage. (d) They had high values
@UPSC_THOUGHTS

As you read through the passage, pay attention to


Ans. You are asked something factual about “poets
the function of words in their context, how they relate
and philosophers”. Look through the passage to locate
to the larger segment of the passage; grasp the thread
the words: you will find them in the fourth sentence.
of ideas running through the passage- the relationships
among the various ideas; and the author’s relationship Form your own answer. Now look at the answer
to the subject of the passage and to the reader, i.e., the responses. If your answer is correct, it would tally with
author’s reasoning and tone. (c). There is nothing in the passage about how ‘rich’
Questions will relate to (i) the main idea; (ii) facts the poets and philosophers were, or if they lived in
and details of information explicitly stated in the poverty; (a) and (b) are not correct. As for (d), ‘high’ is
passage; (iii) implied ideas and information; (iv) the too vague, and we are not told about the values of the
author’s reasoning method or technique of convincing poets and philosophers as such in the passage.
the reader; (v) inferences you can draw from the passage Q 2. Which of the following statements is/are correct
—what assumptions might underlie certain arguments, in the context of the passage?
and the possible ways or areas in which the author’s I. People with money are held in high esteem
reasoning or ideas could be applied. in the industrial age.
Use the elimination method carefully in arriving at II. There is no honesty or sympathy for people
an answer if you are unable to come up with the correct in the industrial age.
answer at first go. (a) I only
(b) II only
WORKED EXAMPLES (c) Both I and II
The solved examples in these pages will help you to (d) Neither I nor II
understand how to put the strategies to work. Further, Ans. You have two statements within the question
the practice exercises will help you get the necessary whose correctness you have to first check before choosing
practice; the answers will help you check your your answer response. The very first sentence supports
proficiency even as the detailed explanatory notes will statement I. So I is correct. Put a tick mark against it.
help you understand why a particular answer response
But you have also to check if II is correct. The key words
is correct and the others are wrong while offering more
in the statement are ‘honesty and sympathy’. You find

1.7
Comprehension

the words in the fifth sentence. The sentence, however, on the main idea. Option (b) is correct insofar as the
does not say that these values are absent, only that they passage does list sources of food of the fungus fact, but
are no longer the most desirable qualities. Read it is again not the ‘purpose of the author’. Option (c)
around the sentence as well. Nothing to support is also something that can be found in the passage,
statement II is available. Statement II is not correct. but it is just an example of a situation.
Put a cross against it. Now look at the answer Q 2. Which of the following statements is/are correct
responses. Only (a) is valid. according to the passage?
Example 2 I. There is no plant that is safe from fungus
Observe the dilemma of the fungus: it is a plant, but action.
it possesses no chlorophyll. While all other plants put II. Fungus creates all the rot and decay in the
the sun’s energy to work for them, combining the world.
nutrients of ground and air into the body structure, the III. Some fungi are used to create important
chlorophyll-less fungus must look elsewhere for an medicines.
energy supply. It finds it in those other plants which, (a) I only
having received their energy free from the sun, relinquish (b) I and II
it at some point in their cycle either to other animals (c) II and III
or to fungi. (d) None of them
In this search for energy the fungus has become the Ans. The question demands that you check the
earth’s major source of rot and decay. Wherever you see correctness of three statements before you look at the
mold forming on a piece of bread, or a pile of leaves answer responses. Statement I is contradicted towards
turning to compost, or a blown-down tree becoming the end of the passage where it is specifically stated
pulp on the ground, you are watching a fungus eating.
@UPSC_THOUGHTS

that certain plants that contain a resin toxic to fungi


Without fungus action the earth would be piled high would last indefinitely—indicating that these plants
with dead plant life of past centuries. In fact, certain are safe from fungus action. Cross out statement I.
plants which contain resins that are toxic to fungi will Statement II is a little tricky. The passage does associate
last indefinitely; specimens of the redwood, for instance, fungus with the source of rot and decay, but there is
can still be found resting on the forest floor centuries a qualifying word —“major”; so we cannot correctly
after having been blown down. say that “all” the decay is caused by fungus. Cross out
Q 1. What is the author’s purpose in the passage? statement II. Statement III says something that most of
(a) Explaining the dilemma faced by the fungus us know to be correct—remember penicillin? But wait,
which has no chlorophyll does the passage say anything about fungus being used
(b) Describing where fungi get their food from for medicines? No. Recall that you have to base your
(c) Explaining the long life of some redwoods answer on the given passage. Statement III is not valid
(d) Pointing out the utility of fungi here. Cross it out. So, what do we have – three crossed
out statements; in other words, none of the statements
Ans. The question asks the purpose of the author is correct. Choose answer response (d).
in the passage. Though the passage starts by saying
that the fungus has a dilemma, this is not the main idea Q 3. Which of the following best describes the fungus
of the passage; recall that there are passages in which as depicted in the passage?
the idea is developed over a number of lines, and may (a) Toxic
not be actually stated clearly. You have to read further. (b) A plant without chlorophyll
The author goes on to describe the fungus in action as (c) Parasitic
it searches for energy to live, and points out how the (d) Strange
fungus is useful: “... the fungus has become the earth’s Ans. When you think up your answer for Q3, you
major source of rot and decay”; “Without fungus action might have come up at first instance with ‘non-green
the earth would be piled high with dead plant life...” plant’—a plant without chlorophyll. Now when you
Clearly the author is writing about the usefulness of look at the responses, you would straightaway pick
fungus. Look at the answer responses; (d) is correct. option (b). But wait, you should always check the other
Option (a) is implied, but does not explain the options before you put that mark on the answer sheet.
‘purpose’ of the author; it is just a fact stated that leads Option (a) may be true in the light of your own

1.8
Comprehension

knowledge of fungi, but the passage does not say so. viable financing methods,
Cross it out. When you come to option (c), however, you therefore, is an urgent requirement
are likely to be struck with its aptness. ‘Parasitic’ for many countries, including
exactly describes the non-chlorophyll plant depending India. The World Health
on other plants for its food. The correct answer response, Organisation makes a timely
therefore, is (c). You may cross out option (d) as too non- intervention by calling for reforms
specific; there is nothing in the passage to support it. in the way nations finance
Example 3 healthcare in its World Health Report
—Health systems financing: the path to
This is a long passage and you should read it before you
universal coverage.
look at the questions. If you note the points and make the
correct markings, you will have no difficulty in answering Any policy that aims at
the questions. reducing personal financial burden
(It is not necessary that the markings and notings done related to healthcare should focus
by us are to be followed; you may follow your own thoughts on bringing down the direct who
and ideas on what to mark.) payments by the individual. This pays?
means a change in who pays for
The world’s growing riches healthcare, now borne over-
health whelmingly by individuals in
seem to make little difference to over
100 million people globally as they
costs countries that have weak
slide into poverty every year because poverty government-paid healthcare
of healthcare costs. One of the systems. The question of ‘who-else-
@UPSC_THOUGHTS

unsolved conundrums in many should-pay’ gives itself two choices


countries is the inability to provide globally: integration of provision and
for universal healthcare coverage, payment, which calls for a lead role
despite economic growth and by governments; and an
development. While the financial institutionally separate agency, say,
consequences of illness are severe an insurer or a government body
for many in poorer countries that do that pays for healthcare on behalf of
not have appropriate systems in individuals. In India, there has been
place, those in richer nations are by an increase in the share of private
no means immune from this malady. how ?
insurance to meet private health
Researchers at Harvard have made
expenditure, up from 1.1 per cent in
the point that illness or medical
1995 to 2.2 per cent in 2008. But this
bills were behind 62 per cent of
is no substitute for the state’s role
personal bankruptcies in the United
in providing basic, affordable
States in 2007. In India, high
healthcare. The report’s suggested
spending on health is a major
domestic options for innovative
reason for people sliding into
poverty. Inadequate state delivery financing—for instance, diaspora
systems mean India’s private bonds, and a minimal tax on foreign
expenditure on health accounts for exchange transactions in currency
72 per cent of the total health
need markets (0.005 per cent in the case
expenditure. Moreover, with poor for of India)—are timely as they could
re-financing options, a staggering financing help governments fund better state-
89.5 per cent of this private health provided healthcare. Revenues
healthcare
expenditure is met out of pocket, raised from such measures should
from the immediately available funds be used for putting in place strong
of individuals. Coming up with and affordable delivery systems,

1.9
Comprehension

particularly in the two important by government (or state) in healthcare provision and
areas of primary and preventive financing, not for ‘total’ public healthcare system. Cross
healthcare. out statement I. Statement II is correct: the passage does
suggest the setting up of “an institutionally separate
Q 1. According to the passage, the cost of healthcare
agency, say, an insurer or a government body that
I. is exorbitant in the private sector
pays for healthcare on behalf of individuals”.
II. affects people in developing countries rather
Statement III too has to be verified before you mark
than those in the richer countries
your answer response. While the passage says that
III. has led millions into poverty all over the there has been an increase in the share of private
world insurance, there is no indication that it favours an
(a) I only increase. Cross out statement III. So only statement
(b) II only II is correct. Looking at the answer responses, you
(c) III only find (b) is correct.
(d) II and III
Q 4. How, according to the passage, can public
Ans. The question has three components, each of
healthcare costs be met?
which has to be verified. From personal experience, you
I. Creation of a state fund
may feel statement I is correct, but the passage does not
II. A small cess on income
say it. Cross it out. As for statement II, the passage III. Government bonds
clearly says “those in richer nations are by no means (a) I and II
immune from this malady”; so this statement contradicts (b) II and III
what is said in the passage and may be crossed out. (c) I, II and III
Statement III is what the passage says at the very
@UPSC_THOUGHTS

(d) None of them


beginning; so III is correct. Mark answer response (c).
Ans. This question also has statements to be verified.
Q 2. In India, healthcare costs are mostly met by A government body or insurer that pays for individual
(a) the state healthcare cost is not a ‘state fund’, so statement I may
(b) private insurance be crossed out. A specific kind of tax is mentioned to
(c) personal funds finance healthcare costs, but it is not a cess on income.
(d) public healthcare system Cross out statement II. Statement III mentions state
Ans. The question is fairly straightforward. The bonds, which is not mentioned in the passage. Cross
only real choice is between options (b) and (c). While out III. None of the statements is correct. So your answer
private insurance is mentioned in the passage and response should be (d).
stated to be growing, there is nothing to indicate that Q 5. The passage is mainly concerned with
most of the costs is met by it. The passage clearly states (a) poverty resulting from poor healthcare system
that more than 89 per cent of private expenditure on (b) the necessity of public financing of healthcare
health is “met out of pocket”, i.e., from personal funds. (c) affordable health insurance
The correct answer response is (c). (d) expansion of the public health system
Q 3. The passage is in favour of Ans. The question asks for the main concern or idea
I. total public healthcare system in the passage. If you have read the passage with
II. a government institution that pays for attention, you would have come up with the answer—
individual’s healthcare financing individual costs of healthcare. When you
III. increase in the private insurance system read the options, you find that (b) tallies with your
(a) I only answer. However, check the other options: the passage
(b) II only certainly mentions that more people have become poor
(c) III only because of healthcare costs, not poor healthcare system;
(d) II and III furthermore, the comment is a starting point to develop
the main concern—how to make healthcare affordable.
Ans. In this question, once again, three statements
Option (c) mentions affordable ‘insurance’—not
need to be verified. The passage calls for a leading role
mentioned in the passage. While the expansion of

1 . 10
Comprehension

public healthcare system mentioned in option (d) of quality schools and colleges to churn out skilled
may be desirable, the passage only talks of greater professionals to cater to the needs of emerging
state role in providing ‘affordable healthcare’ and businesses. This in turn will have a positive trickle-
‘strong ... delivery systems’, and not about ‘expansion’ down effect and galvanise the rural economy of the
of the system. So (b) is the correct answer. respective states.
Example 4 In planning these new urban hubs, errors of the
You need to read the passage and make your notes and past that have given rise to chaotic and dysfunctional
markings as this is a long passage. cities must not be repeated. Our metros may have
reached a point of saturation. While they should by no
In a welcome development, small cities and towns means be ignored, pay attention to Tier II and III cities
appear to be doing more to power India’s growth story as well to continue India’s growth story and make it
than big metros. Confirming this are the latest income more inclusive.
tax statistics, which indicate that Tier II and Tier III
cities like Patna, Lucknow, Meerut and Kanpur have Q 1. Which of the following indicators has/have been
far outstripped Delhi, Mumbai, Chennai and Kolkata used to highlight growth?
in terms of growth in personal and corporate tax I. Per capita income
collections. In fact, Patna has seen as much as 95 per II. Migration
cent growth in personal income tax figures over the III. Corporate tax
2009-10 period compared to a measly 4 per cent for IV. Tax collections
Delhi and 6 per cent for Mumbai. Such a shift towards (a) I and II
growth driven by regional centres can help mitigate the (b) III only
@UPSC_THOUGHTS

problems ensuing from unequal development and, (c) IV only


therefore, needs to be encouraged. (d) None of them
The current growth and development model centred Ans. The statements need to be verified; there is no
on big metros is unsustainable. Having experienced mention of I in the passage. While ‘migration’ is
years of economic migration, these large cities are mentioned, it is not in the context of growth. So
literally bursting at the seams. They are left with statement II is not acceptable. ‘Corporate tax’ sounds
creaking infrastructure—compounded by shoddy urban correct but is incomplete; corporate tax collections is
planning—and poor civic amenities, all of which is only one indicator. So statement III is only partially
reflected in the fast depreciating quality of life. Yet correct. Statement IV gives the complete answer; refer
people continue to be drawn to metros due to the allure to ‘in terms of growth in personal and corporation tax
of better career prospects. The only way to reverse this collections’ in paragraph 1. So the correct answer
trend is to have multiple growth poles spread across response is (c).
the length and breadth of the country. It is encouraging
Q 2. Which of the following is definitely true as per
that many of the small cities showing robust economic
the passage?
growth are located in the backward regions. They could
(a) The tax collection from Patna were the
serve as magnets for intra-state migration and take the
highest for 2009-10
burden off traditional metropolitan hubs.
(b) Growth in corporate tax collections is
As emerging markets within the Indian economy,
always followed by growth in personal tax
these small urban centres can become hotspots for new
collections
investment opportunities. Many outsourcing companies
(c) The growth in personal tax figures for
are already setting up operations in Tier II and Tier III
Chennai and Kolkata was not more than
cities to minimise their running costs. Conducive
4 per cent for 2009-10
conditions need to be created to encourage India Inc as
(d) None is true
well as foreign investors to increasingly invest in small
cities and townships. Crucial to this is creating sound Ans. Read paragraph 1. While (a), (b) and (c) are
infrastructure. There needs to be a significant number not mentioned, Patna has seen 95 per cent growth in

1 . 11
Comprehension

‘personal income tax figures’ (not corporate tax would be better


collection) in 2009-10. So (d) is the correct answer Ans. In the third paragraph you are told that
response. outsourcing companies have moved to Tier II/III cities
Q 3. Which of the following may not be an objective to ‘minimise their running costs’. Answer response (a)
of developing Tier II/III cities? seems correct. While (b) is suggested as something to
I. To have more inclusive growth be encouraged through the creation of suitable
II. To have growth spread over the country conditions, and not as something that is happening,
III. To improve the quality of schools and responses (c) and (d) are not supported by the passage.
So (a) is the correct answer.
colleges to produce skilled professionals
IV. To take off pressure from the over-burdened Q 5. What attracts people to the metros?
infrastructure of metros (a) Better living conditions
(a) I and II (b) Improved source of income
(b) I, II and III (c) Better education facilities
(c) III and IV (d) Seamless economic and social activities
(d) III only
Ans. ‘Better career prospects’ in the fourth sentence
Ans. The question has a ‘negative’ in it: you have of paragraph 2 implies improved source of income. The
to find which statement is not correct according to the other options may be true in your own experience of
passage. So you need to eliminate the statements that metros but they are not mentioned in the passage. So
are supported by the passage. The passage is about the (b) is the correct answer.
development of Tier II/III cities. The last sentence of Q 6. Heavy migration of people seeking a living into
@UPSC_THOUGHTS

paragraph 1, ‘Such a shift … unequal development …’ large cities has led to


implies inclusive growth, and so supports statement I. I. the quality of life being lowered
Statement I may hence be rejected. Statement II is also II. infrastructure that is in bad condition
supported by the passage, ‘The only way to … the III. poor urban planning
country’ (paragraph 2), so it may be rejected. Statement IV. higher economic growth
III finds a mention in paragraph 3. However, this is (a) I only
mentioned as a ‘need’, not as an objective, so this is (b) I and II
not supported by the passage as an ‘objective’. Have a (c) III and IV
look at the answer responses; I and II are in (a) and (d) I, II and III
(b) so those are wrong. Keep your options open as (c)
Ans. Check the statements. You need to understand
has III and IV; move on to statement IV—it is clearly
the meaning of words and what they convey. Statement
an objective for developing Tier I/III cities. So (d) is your
I is correct as the term ‘fast depreciating quality of life’
answer. suggests; ‘depreciating’ means lowering of value.
Q 4. Which of the following would be the advantage Statement II is correct as seen in the term ‘creaking’
of setting up operations in Tier II and Tier III which means ‘no longer working well or in good
cities? condition’. Statement III at first sight seems to be correct,
(a) The operating costs would be lower but if you read the second paragraph carefully, poor
(b) It will attract better investment urban planning (‘shoddy’ = low standard = poor)
(c) It will attract tax exemptions from the worsens (‘compounded’ = making something worse)
government the ‘creaking infrastructure’; so poor urban planning is
(d) The manpower available for employment not a consequence of the heavy migration. Statement III
is not acceptable. Looking at the answer responses, you
may at once reject (c) and (d) and choose (b). However,
if you want to check statement IV, the first paragraph
by implication contradicts it. So statement IV is not
correct. Answer response (b) is correct.

1 . 12
Comprehension

PRACTICE EXERCISES

Directions In this section you have a number of Which of the statements given above is/are
passages. After each passage, some questions are given correct?
based on the passage. Read each passage and answer the (a) I only
questions based on it. (b) I and II
(c) II and III
(d) I, II and III
Passage 1
Passage 2
There are many ways of communicating without using
speech. Signals, signs, symbols and gestures may be Trade exists for many reasons. No doubt it started from
found in every culture. The basic function of a signal a desire to have something different. Men also realised
is to impinge upon the evironment in such a way that that different men could make different products. Trade
it attracts attention. While less adaptable to the encouraged specialisation, which led to improvement
codification of words, signs contain greater meaning in quality. Trade started from person to person but grew
in and of themselves. A stop sign conveys meaning to involve different towns and different lands. Some
quickly and conveniently. Symbols are more difficult to found work in transporting the goods or selling them.
describe than either signals or signs because of their Merchants grew rich as the demand for products
intricate relationship with the receiver’s cultural increased. Craftsmen were able to sell more products
perceptions. In some cultures, applauding in a theatre at home and abroad. People, in general, got a great
variety of things to choose. The knowledge of new
provides performers with an auditory symbol of
@UPSC_THOUGHTS

products led to an interest in the lands which produced


approval. Gestures such as waving and handshaking
them. The more adventurous ventured out to see other
also communicate certain cultural messages.
lands. Those who stayed at home asked many questions
1. Which of the following is correct about signals, of the travellers. As people learned about the products
signs and symbols? and the conditions in other countries, they compared
(a) They are part of all primitive cultures them with their own. This often led to a desire for better
(b) They are difficult to understand as they are conditions or a hope for a better life. Thus trade was
closely related to cultural perceptions mainly an economic force but it affected many aspects
(c) They are non-verbal forms of communication of a society.
(d) They are all easy to interpret 5. Consider the following statements in the light of
2. Symbols are difficult to decipher because the passage.
(a) they use difficult codes I. Trade makes countries prosperous.
(b) of cultural differences of the receiver and the II. Trade enables people to enjoy more choice in
things.
sender
III. Trade leads to a desire for a better standard
(c) they are less likely to be adapted to common
of life.
speech
(d) they are not related to effective communication Which of the statements given above is/are
correct?
3. Applauding in a theatre is an example of a (a) I and II
(a) sign of ending the show (b) II and III
(b) symbol of appreciation (c) I, II and III
(c) gesture of encouragement for the performers (d) None of them
(d) signal that the curtain has risen
6. Which of the following best describes the effect of
4. Consider the following statements in the light of trade?
the passage? (a) It brings prosperity to traders
I. Signs can be understood by themselves. (b) It spreads knowledge about unknown lands
II. Gestures have cultural meaning. (c) It provides employment to craftsmen
III. Signals are means to draw attention. (d) It affects various aspects of social life

1 . 13
Comprehension

7. Which of the following is true of trade according 10. Malnutrition in children can be reduced by creating
to the passage? an awareness of
(a) Trade began as an interaction between (a) nutritive values of food
countries (b) regular intake of food
(b) Trade encouraged travel (c) balanced diet
(c) It was because of trade that some countries (d) the food needs of younger children
became economically powerful 11. The author reasons that poverty is not the main
(d) An improvement in the quality of goods led cause of malnutrition because
to specialisation and more trade I. adults in poor families eat good quantities of
food.
Passage 3 II. children are malnourished whatever the
economic status of the family.
Malnutrition most commonly occurs between the ages
Which of the statements given above is/are
of six months and two years. This happens despite the
correct?
child’s food requirements being less than that of an
(a) I only
older child. Malnutrition is often attributed to poverty,
(b) II only
but it has been found that even in households where
(c) Both I and II
adults eat adequate quantities of food, more than 50 per (d) Neither I nor II
cent of children under five do not consume enough
food. The child’s dependence on someone else to feed Passage 4
@UPSC_THOUGHTS

him/her is primarily responsible for the malnutrition.


Modern economics does not differentiate between
Very often the mother is working and the responsibility
renewable and non-renewable materials, as its method
of feeding the young child is left to an older sibling.
is to measure everything by means of a money price.
It is therefore crucial to increase awareness regarding
Thus, taking various alternative fuels like coal, oil,
the child’s food needs and how to satisfy them.
wood or water-power, the only difference between them
8. Consider the following statements about recognised by modern economics is relative cost per
malnutrition. equivalent unit. The cheapest is automatically the one
I. Malnutrition occurs as a small child requires to be preferred, as to do otherwise would be irrational
more food than an older child. and ‘uneconomic’. From a Buddhist point of view, of
II. Malnutrition occurs mainly because of poverty. course, this will not do; the essential difference between
III. Malnutrition occurs because of lack of nutritive non-renewable fuels like coal and oil on the one hand
feeding habits. and renewable fuels like wood and water-power on the
other cannot be simply overlooked. Non-renewable
Which of the statements given above is/are correct goods must be used only if they are indispensable, and
in the light of the passage? then only with the greatest care and the highest concern
(a) I only for conservation. To use them carelessly or extravagantly
(b) II only is an act of violence, and while complete non-violence
(c) I, II and III may not be possible on this earth, it is none the less
(d) None of them the duty of man to aim at the ideal of non-violence in
all he does.
9. According to the author, the major cause of
malnutrition is 12. In this passage the author is trying to
(a) ignorance of a child’s food needs (a) differentiate between renewable and non-
(b) dependence of a child on others renewable materials
(c) non-availability of food in most families (b) show that the modern economist is concerned
(d) working mothers only with costs

1 . 14
Comprehension

(c) underline the need for conserving natural to receive higher wages. Despite their greater education,
resources white women’s actual median income is only 2 to 5 per
(d) differentiate between two economic cent higher than that of black women.
philosophies The above data is consistent with the hypothesis
that the intensity of discrimination against women
13. The Buddhist economist’s attitude implies that
differs little between the whites and blacks. Therefore,
fuels like coal and oil must be used only if
racial discrimination adds little to the effects of existing
(a) there is a plentiful supply
sex discrimination.
(b) wood and water-power can be dispensed
These findings suggest that a black woman does
with
not necessarily suffer relatively more discrimination in
(c) the relative cost of each is lower than that of
the labour market than does a white woman. Rather,
wood and water for women, the effects of sexual discrimination are so
(d) there is no alternative fuel available pervasive that the effects of racial discrimination are
14. To use non-renewable goods carelessly is not negligible. Of course, this is not to say that the more
favoured by Buddhists because it would generalised racial discrimination of which black women,
(a) go against their economic tenets like black men, are victims does not disadvantage black
(b) constitute an act of violence women in their search for work. After all, one important
(c) be an act against natural law productivity factor is the level of education, and the
(d) be sinful difference between white and black women on this
scale is largely the result of racial discrimination.
15. Consider the following statements in the context
@UPSC_THOUGHTS

of the passage. 16. The primary purpose of the passage is to


I. Buddhists do not use non-renewable (a) give a historical account of racial and sexual
resources. abuse in the labour market
II. Buddhists do not believe in modern economic (b) show that racial discrimination against black
women is less important than sexual
development.
discrimination
III. Buddhists are aware that violence may not be
(c) show how education levels influence the
completely eradicated from this earth.
earnings of black workers
Which of the statements given above is/are not (d) show what effect sexual discrimination has
true? on women, both black and white
(a) I only
(b) I and II 17. Which of the following best describes the tone of
(c) III only the passage?
(d) II and III (a) Somewhat biased
(b) Tentative and inconclusive
Passage 5 (c) Objective and critical
(d) Overbearing
The existence of both racial and sexual discrimination
in employment is well documented, and policy-makers 18. If the hypothesis mentioned by the author is
and responsible employers are sensitive to the plight correct, a higher white-black women income ratio
of the black female employee on the theory that she is would lead to
doubly the victim of discrimination. (a) a lower female-male income ratio
To obtain a true measure of the effect of racial (b) an increase in the level of education of women
discrimination in employment it is necessary to adjust (c) a general lessening of discrimination against
the gross black-white income ratio for their productivity women
factors. White women in urban areas have a higher (d) a general aggravation of discrimination
educational level than black women and can be expected against women

1 . 15
Comprehension

19. Which of the following can be said to be the cause humans, like chimpanzees, are more gregarious and
of difference in the educational level of black resourceful than orangutans, the latter provide
women and white women? anthropologists with useful information about the
(a) Sexual discrimination behaviour of prehominid primates and how apelike
(b) Racial discrimination behaviour influenced our ancestors’ search for food and
(c) Rural-urban divide family beneath the forest’s canopy.
(d) Low income of blacks 21. The primary purpose of this passage is to
20. Consider the following statements in the light of (a) describe some behavioural and evolutionary
the passage. characteristics of orangutans
I. Uneducated black women suffer more (b) analyse the reasons why early primates left
discrimination than black men. their forest dwellings
II. Women suffer more discrimination than men. (c) illustrate the dangers posed to orangutans by
poachers
Which of the statements given above is/are
(d) show how behaviour of the orangutans differs
correct?
(a) I only from that of other primates
(b) II only 22. The passage indicates that it is difficult to return
(c) Both I and II orangutans to the wild because of
(d) Neither I nor II I. the threat posed by newcomers to other
orangutans’ territory.
@UPSC_THOUGHTS

Passage 6
II. the conflict between males over available
Anthropologists who study orangutans, distant cousins females.
of the human race, find in the animals’ behaviour hints III. the scarcity of available food in the orangutan’s
of how our earliest ancestors may have lived. It has long environment.
been accepted that primates originally dwelt in the tree- Which of the statements given above is/are
tops and only migrated to the ground as forests began correct?
to dwindle. While to a certain extent all primates except (a) I only
humans spend at least some time dwelling in trees, the (b) I and II only
orangutan hardly ever ventures to the forest floor. Adult (c) I and III only
orangutans can grow as heavy as 330 pounds and live
(d) II and III only
four decades, requiring copious amounts of fruit simply
to stay alive. Thus, they become very jealous of the 23. Which of the following can be inferred about
territory where they find their food. Compounding this differences between the behaviour of orangutans
territoriality are the breeding habits of orangutans, and that of other ape species?
since females can only breed every few years and, like (a) While orangutans spend much of their time
humans, give birth not to litters but single offspring. in the treetops, other apes live exclusively on
Consequently, orangutans are solitary, territorial the ground
animals who have difficulty foraging in any part of the (b) Orangutans and other types of apes are all
forest where they were not raised. Orangutans taken sociable species, but orangutans are more
from poachers by customs agents undergo incredible likely to bond for life
hardship on their return to the wild. Incorrectly relocating (c) Apes such as chimpanzees rely less upon
a male orangutan is especially problematic, often ending their size than the average orangutan does
in the animal’s death at the hands of a rival who sees (d) Orangutans spend less time in the company
not only his territory but also the females of his loosely of other members of their species than do
knit community under threat from an outsider. While some other apes

1 . 16
Comprehension

24. According to the author, anthropologists study despite being an ancient civilisation that traces itself to
the behaviour of orangutans in order to the very dawn of human habitation, India is among the
(a) prevent orangutans from becoming the target youngest countries in the world.. More than half the
of poaching country is under 25 years of age and more than a third
(b) assist customs agents in the relocation of is under 15 years of age.
orangutans Brought up in the shadow of the rise of India’s
(c) analyse the causes and consequences of service industry boom, this group feels it can be at least
contemporary human behaviour as good if not better than anyone else in the world. This
(d) better understand the factors that influenced confidence has them demonstrating a great propensity
human evolution to consume, throwing away ageing ideas of asceticism
25. Which of the following are factors that the author and thrift. The economic activity created by this
indicates as contributing to the orangutan’s combination of a growing labour pool and rising
territoriality? consumption demand is enough to propel India to
(a) The lack of available food and anti-social double-digit economic growth for decades. This
nature of orangutans opportunity also represents the greatest threat to India’s
(b) The orangutan’s need for large quantities of future. If the youth of India are not properly educated
food and the infrequency with which it mates and if there are not enough jobs created, India will have
(c) The threat posed by poachers and the forever lost its opportunity.
orangutan’s inability to protect itself from India’s Information Technology and Business
them Process Outsourcing industries are engines of job
@UPSC_THOUGHTS

(d) The difficulties that orangutans face when creation, but they still account for only 0.2 per cent of
compelled to socialise with other ape species India’s employment. The country has no choice but to
such as chimpanzees dramatically industrialise and inflate its economy.
According to a recent survey, more than half of India’s
26. It can be inferred from the passage that one
unemployed within the next decade could be its educated
development responsible for the evolution of
youth.
distinct ape species was the
(a) early primates’ inability to survive in the 27. Consider the following statements:
forest I. India’s rich cultural heritage prevents India
(b) shrinking of the available primitive forest from surging ahead to become an active partner
(c) growth of human and chimpanzee in the global economy.
communities II. By and large, India’s youth still believe in a
(d) encroachment of other species into the thrifty lifestyle.
primitive forest Which of the statements given above is/are
correct?
Passage 7 (a) I only
(b) II only
Today, India looks to be on course to join the league
(c) Both I and II
of developed nations. It is beginning to establish a
(d) Neither I nor II
reputation not just as the technology nerve-centre and
back-office to the world, but also as its production 28. What is the approximate number of people in
centre. India’s secularism and democracy serve as a role India who are in the age group 15-25 years?
model to other developing countries. There is great (a) 500 million
pride in an Indian that easily integrates with a global (b) 350 million
economy, yet maintains a unique cultural identity. (c) 210 million
But what is breathtaking is India’s youth. For, (d) 180 million

1 . 17
Comprehension

29. In the recent past, which sector has witnessed a effectiveness and success depend on their ability to
phenomenal growth? organise their ideas and present them effectively.
(a) Heavy industry Delivering your message in person provides immediate
(b) Service industry feedback that helps you clarify points and answer
(c) Petrochemical industry questions.
(d) Textile industry Oral presentations are often more persuasive. As far
as possible, one should never read a presentation or
30. Consider the following statements:
memorise it. Then the presentation will lose flexibility
I. Rising consumption demand will retard and communication will suffer. The spoken word wields
economic growth. great power. Face-to-face interaction demands thinking
II. India’s youth are its greatest opportunity as and speaking. Anecdotes, quotations and humorous
well as threat. touches often make a presentation interesting. One may
Which of the statements given above is/are consult one’s notes frequently when one is making
correct? one’s presentation. This may create a feeling among
(a) I only listeners that the speaker has taken pains to prepare
(b) II only for the occasion. A positive response will be generated
(c) Both I and II and the speaker will be heard with respect. Speaker’s
(d) Neither I nor II enthusiasm and confidence can influence people to
accept or reject an idea in a way that a written
31. Consider the following statements:
document cannot. A presentation should be persuasive
I. Information Technology sector provides a and should change the audience’s attitude. The topic
@UPSC_THOUGHTS

relatively large proportion of jobs in India. of the presentation must be interesting to the audience.
II. In the coming decade, only uneducated youth The topic should be of interest to the speaker also
will remain deprived of employment otherwise he will go through the motions of making a
opportunity. presentation. No perfunctory approach should ever be
Which of the statements given above is/are resorted to while making a presentation. It is very
correct? important that the speaker is perceived by the audience
(a) I only as credible and qualified to speak about the topic. The
(b) II only speaker must adapt to the intellectual level of the
(c) Both I and II audience.
(d) Neither I nor II 32. Consider the following statements:
1. An effective presentation about the product of
Passage 8 a company can help in increasing sales
volumes.
The art of effective presentation is the fruit of persistent
2. Impromptu presentation can leave a more
efforts and practice. Your personality is reflected in
forceful impact on the audience.
your presentation. Adequate planning and preparation
Which of the statements given above is/are
are essential for a successful presentation. A thorough
correct?
preparation is the best antidote for nervousness. If a
(a) Only 1
person is not successful in presenting his views and
(b) Only 2
ideas then it will become the greatest obstacle in his
(c) Both 1 and 2
career and life. People form a perception about how
(d) Neither 1 nor 2
competent you are by seeing how you present yourself
when you stand and speak. 33. Consider the following statements:
A successful presentation can help a person in 1. One of the drawbacks of presentations is that
winning orders for the company he works for. Most they fail to provide a feedback from the
people who work in organisations find that their audience.

1 . 18
Comprehension

2. While making a presentation one should, at 37. Consider the following statements:
times, refer to one’s written material. 1. Persuasive skill-set is a prerequisite to an
Which of the statements given above is/are effective presentation.
correct? 2. At the end of a presentation, offering small
(a) Only 1 gifts to the audience by the speaker is a good
(b) Only 2 strategy.
(c) Both 1 and 2 Which of the statements given above is/are
(d) Neither 1 nor 2
correct?
34. Consider the following statements: (a) Only 1
1. Presentations are not meant to change the (b) Only 2
point of view of the audience about the subject (c) Both 1 and 2
of the presentation. (d) Neither 1 nor 2
2. Recounting of quotations in the presentation
should be avoided as it makes the presentation Passage 9
appear superficial.
A major consequence of fast-paced motorisation and
Which of the statements given above is/are
expansion of roads and highways in India is the
correct?
mounting rate of fatalities and injuries from traffic
(a) Only 1
(b) Only 2 accidents. More than 110,000 people are killed on the
(c) Both 1 and 2 roads each year, with the death toll rising by eight per
@UPSC_THOUGHTS

(d) Neither 1 nor 2 cent annually; the estimate for serious injuries is 1.6
million. India’s roads are now rated the worst in the
35. Consider the following statements:
world. Viewed against this background, the road safety
1. A speaker must memorise his talk so as to
initiative launched by the central government and the
introduce more flexibility.
World Bank to cover 3,000 km of high-risk national and
2. A written document is more efficacious than
State highways in Assam, Gujarat, and Karnataka is an
an oral presentation as it leaves a lasting
incremental step to improve the situation. Under the
impression on the reader.
plan, affordable improvements based on the latest
Which of the statements given above is/are
technologies will be put in place to reduce crashes and
correct?
fatalities. The project will draw upon the experience of
(a) Only 1
the International Road Assessment Programme
(b) Only 2
supported by the World Bank in several countries. The
(c) Both 1 and 2
investments can improve the safety record of some
(d) Neither 1 nor 2
roads. What is important, however, is for the government
36. Consider the following statements: to demonstrate the political will to move beyond limited
1. Innate stage fright of a speaker can be schemes in a few States. The continuing carnage
countered by meticulous preparation of his demands a policy of zero tolerance to crashes covering
presentation. the entire network of 65,000-plus km of national
2. Confidence of a speaker is generally taken by highways and the quarter million km of urban roads.
the audience as a sign of arrogance. Almost three years ago, the Sundar Committee
Which of the statements given above is/are recommended a national road safety policy but precious
correct? little has been done by way of implementation.
(a) Only 1 There is no justification for delayed action on road
(b) Only 2 safety when the national economic loss on account of
(c) Both 1 and 2 death and disability from accidents is officially reckoned
(d) Neither 1 nor 2
to be of the order of ` 75,000 crore a year. Research on

1 . 19
Comprehension

the challenge facing India points to specific areas that disability from accidents has reached ` 75,000
need urgent action. Pedestrians, bicyclists, and non- annually
motorised vehicle users constitute 60 per cent of those (c) Roads and vehicles should be scientifically
killed on urban roads; and motorcyclists and small car designed so that the consequences of accidents
users make up 25 per cent. Unsurprisingly, there is a are not serious
disproportionate involvement of trucks and buses in (d) None of the above
fatal crashes, highlighting a key problem. These data
41. Consider the following statements.
point to the need for segregation of vulnerable road
1. Urban roads in India are unsafe mainly
users and appropriate traffic calming measures to
because heavy vehicles form a very high
reduce risk. Equally, scientific design of roads and
proportion of the traffic.
vehicles can reduce conflicting interactions among road
2. Segregating road users will effectively reduce
users and mitigate the consequences of accidents. There
risk on roads.
should be a sincere attempt to analyse such data
Which of the statements given above is/are
emerging from studies conducted by injury prevention
correct?
researchers in the country. The Sundar Committee has
(a) 1 only
rightly pointed out that the State transport departments,
(b) 2 only
which now play the relatively minor role of licensing
(c) Both 1 and 2
and vehicle registration, should be made legally
(d) Neither 1 nor 2
responsible for coordination of multi-sectoral safety.
The time to act is now. 42. The tone of the passage is
@UPSC_THOUGHTS

(a) analytical but critical of the government


38. The number of vehicles have grown in India. This
(b) analytical but supportive of the government
has led to
(c) somewhat biased in favour of vulnerable road
(a) larger and greater number of roads and
users
highways
(d) aggressive
(b) more traffic accidents
(c) increasing deaths and injuries from traffic
Passage 10
accidents
(d) bad roads Italian architects have developed a new ‘transparent
39. The road safety initiative has been launched cement’ that allows daylight to flood into a room
1. on a limited scale. making the walls look like giant windows.
2. with the help of the World Bank. Created by researchers at Italcementi Group, the
3. on the recommendations of the Sundar material, called i.light, has dozens of tiny holes in it
Committee. which let light through without compromising the
structural integrity of a building. Up close, the 2-3 mm
Which of the statements given above is/are
gaps make a startling pattern and from certain angles
correct?
or at a distance appear exactly the same as normal
(a) 1 only
concrete. But on a sunny day, the effect is akin to little
(b) 2 only
more than a light mesh on the wall filtering the light
(c) 1 and 2
coming in.
(d) 2 and 3
Italcementi researchers, who created the cement by
40. Which of the following statements is correct bonding special resins in a new mix, have so far used
according to the passage? it for only one building—the Italian pavilion at last
(a) A policy of zero tolerance to accidents on all year’s Expo in Shanghai. However, it has already been
kinds of roads in India is required suggested it could save electricity that would otherwise
(b) National economic loss in terms of death and be required for daytime lighting.

1 . 20
Comprehension

Italcementi used i.light for around 40 per cent of II. The holes in the material allow light in but
the 18-metre high Expo pavilion, or 3,774 transparent don’t let the heat in the room escape.
panels and semi-transparent panels made from 189 Which of the statements given above is/are
tonnes of the product. In each transparent panel there correct?
are approximately 50 holes, leading to about 20 per cent (a) I only
transparency. The semi-transparent panels were around (b) II only
10 per cent see-through and were created by modulating (c) Both I and II
the insertion of the resins. (d) Neither I nor II
Previous attempts at a similar feat had been tried
using fibre optic cables through concrete, but Italcementi Passage 11
is claiming that its version is better. It is claimed that
Biologists have long known that some types of
the transparent cement made from plastic resins is
electromagnetic radiation such as x-rays and gamma
much cheaper than the one made from optical fibres
rays can be dangerous to human beings. Operating at
and costs less. Moreover, the ability to ‘capture’ light a frequency of 1018 through 1022 MHz, these rays, which
is greater, since the resins contain a wider visual angle
are well above the visible light spectrum, were first
than optical fibres. This characteristic in fact increases
detected in the early years of the twentieth century.
the transparency properties of the material and the
However, until now, no one has ever suggested that
luminous effects given to buildings. microwave radiation might also be harmful. In
According to Borgarello, director of Italcementi, his
preliminary laboratory results, Cleary and Milham have
company took up the challenge to build the pavilion
@UPSC_THOUGHTS

found elevated growth rates in cancer cells exposed to


because it wanted to find a “creative, efficient solution”.
low doses of microwaves. Cleary exposed cancer cells
43. Italcementi is to levels of radiation that are commonly found in
(a) a new kind of cement microwave ovens and found that the abnormal cells
(b) transparent cement grew 30 per cent faster than did unexposed cells.
(c) a research group Milham’s study focused on ham radio operators who
(d) a company are commonly exposed to levels of radiation slightly
higher than those emitted by cellular telephones. He
44. Which of the following is i.light?
discovered elevated levels of myeloid leukemia.
(a) A new kind of glass
The methodology of Cleary and Milham has been
(b) A new kind of cement
questioned by other scientists in the field. However, no
(c) A mixture of chemicals created under heavy
one seriously disputes that their preliminary findings
pressure
must be taken seriously or that new studies should be
(d) A plastic resin
set up to try to duplicate their results. Although
45. It would be correct to say about the new material government guidelines for how much electromagnetic
that it energy can be allowed to enter the work and home
(a) has been used extensively since its invention environment have been made more stringent since they
(b) could be environmentally beneficial were first implemented in 1982, the recent studies pose
(c) makes use of optical fibres to capture light troubling questions about the safety of microwaves.
(d) allows 50 per cent transparency
47. The main concern of the passage is
46. Consider the following statements regarding the (a) the impact of electromagnetic waves
new material. (b) comparison of the work of scientists on
I. The resins used in the material are able to microwaves
capture light because of their wide visual (c) the dangers posed by X-rays and gamma rays
angle. (d) none of the above

1 . 21
Comprehension

48. The findings of Cleary and Milham of any society. Such unfolding requires understanding
(a) have been rejected by other scientists and imagination, integrity and compassion, cooperation
(b) led to further studies on the safety of among people and harmony between the human species
microwaves that have confirmed their and the rest of nature. Acquisitiveness and the pursuit
hypothesis of power have made the modern man an aggressor
(c) showed a connection between cancer and against everything that is non-human, an exploiter and
microwaves oppressor of those who are poor, meek and unorganised,
(d) have been accepted by scientists and a pathological type which hates and distrusts the
government guidelines have been modified world and suffers from both acute loneliness and false
accordingly pride.
49. Consider the following statements. The need for a new renaissance is deeply felt by
I. The studies were not scientifically valid. those sensitive and conscientious men and women who
II. The studies indicated that microwaves were not only perceive the dimensions of the crisis of our age
more harmful than X-rays. but also realise that only through conscious and
III. When the passage was written, the final cooperative human effort may this crisis be met and
results were not in. probably even overcome.
Which of the above can be inferred from the given 51. The author appears to be advocating which of the
passage about the studies of Cleary and Milhan? following approaches to be adopted by the society?
(a) I only (a) Capitalistic
(b) II only (b) Communist
@UPSC_THOUGHTS

(c) III only (c) Humanistic


(d) I and III
(d) None of the above
50. According to the passage which of the following
52. Which of the following best describes the behaviour
could be expected to be found in a study of ham
of modern man?
radio operators?
(a) Imaginative and sympathetic
(a) Unusual cells growing 30 per cent faster than
(b) Cruel and greedy
normal
(c) Conscientious and cooperative
(b) Radiation exposure level similar to that found
(d) Perceptive and creative
in microwave ovens
(c) Radiation levels identical to those emitted by 53. According to the passage, why has modern man
cell phones turned out as an enemy of everything that is non-
(d) Higher levels of a certain type of leukemia human?
(a) He hates and distrusts other human beings
Passage 12 (b) Non-humans have refused to cooperate with
Power and possession have been central pursuits of human beings
modern civilisation for a long time. They have blocked (c) He has been dominated by the drives of
out or distorted other features of the western renaissance acquisitiveness and power
(revival) which promised so much for humanity. What (d) He consciously practises spirit of cooperation
people have been and are still being taught to prize are 54. Which of the following statements is not true in
money, success, control over the lives of others, and the context of the given passage?
acquisition of more and more objects. Modern social, (a) Power and possession go hand in hand
political and economic systems, whether capitalist, (b) There is need for a new renaissance
fascist or communist, reject in their working the basic (c) Poor and weak people are oppressed by the
principle that the free and creative unfolding of every modern man
man, woman and child is the true measure of the worth (d) The modern man is not individualist

1 . 22
Comprehension

55. The author hopes that the present crisis can be eosinophils, basophils, monocytes, and lymphocytes.
solved by The neutrophils and monocytes are search-and-destroy
(a) devoted individual efforts cells. The monocytes follow chemical trails to inflamed
(b) different political systems tissues where they develop into macrophages that can
(c) purposeful and collective human efforts engulf invaders and debris. Two classes of lymphocytes,
(d) spiritually developed individuals B cells and T cells, make highly specific defence
responses.
Passage 13 Some stem cells in bone marrow give rise to giant
Blood, a connective tissue, is a sticky fluid that has cells called megakaryocytes. These shed fragments of
multiple functions. It transports oxygen, nutrients, and cytoplasm enclosed in a bit of plasma membrane. The
other solutes to cells; carries away metabolic wastes membrane-bound fragments are platelets, which initiate
and secretions; and helps stabilise internal pH. Plasma, blood clotting. Each platelet only lasts five to nine days,
red blood cells, white blood cells, and platelets are its but hundreds of thousands are always circulating in
components. blood.
Plasma, which is mostly water, functions as a 56. The passage is primarily concerned with
transport medium for blood cells and platelets. It also (a) blood
serves as a solvent for ions and molecules, including (b) blood circulation
hundreds of different kinds of plasma proteins. Some (c) the components of blood
of the plasma proteins transport lipids and fat-soluble (d) blood functions
vitamins through the body. Others have roles in blood
57. According to the passage, plasma is
@UPSC_THOUGHTS

clotting or in defense against pathogens. Collectively,


the concentration of plasma proteins affects the blood’s (a) made up of ions and molecules
fluid volume, for it influences the movement of water (b) a biconcave disc
between blood and interstitial fluid. (c) interstitial fluid
Erythrocytes, or red blood cells, are biconcave discs. (d) mostly water
They transport the oxygen used in aerobic respiration 58. The red colour is given to blood by
and carry away some carbon dioxide wastes. When (a) oxygen
oxygen diffuses into blood, it binds with hemoglobin, (b) iron
the iron-containing pigment that gives red blood cells (c) enzymes and other proteins
their colour. (d) none of the above
Mature red blood cells no longer have their nucleus,
59. The use of words such as “defence’, ‘patrol’,
nor do they require it. They have enough hemoglobin,
‘armies’ and ‘battle’ implies
enzymes, and other proteins to function for about 120
(a) white blood cells can become uncontrollable
days. At any time, phagocytes are engulfing the oldest
(b) phagocytes attack old red blood cells
red blood cells or the ones already dead, but ongoing
(c) white blood cells serve as protection against
replacements keep the cell count fairly stable.
invaders
Leukocytes, or white blood cells, arise from stem
(d) white blood cells are violent
cells in bone marrow. They function in daily
housekeeping and defence. Many patrol tissues, where 60. Which of the following is/are correct?
they target or engulf damaged or dead cells and anything 1. Stem cells give rise to leukocytes and
chemically recognised as foreign to the body. Many megakaryocytes.
others are massed together in the lymph nodes and 2. There is no nucleus in mature red blood cells.
spleen. There they divide to produce armies of cells that (a) 1 only
battle specific viruses, bacteria, and other invaders. (b) 2 only
White blood cells differ in size, nuclear shape, and (c) Both 1 and 2
staining traits. There are five categories: neutrophils, (d) Neither 1 nor 2

1 . 23
Comprehension

Passage 14 (c) they wanted to take parts of the ship and sell
them
For months the old tanker, African Queen, lay turned (d) it was an interesting exercise
over on her side, stuck fast in the sands off the coast
of Maryland. She had run aground so badly that her 62. How did the two men propose to float the ship
owners had decided to leave her to her fate. It was again?
considered impossible to refloat her and the ship began (a) By sending divers to examine the damage
to rust and sink deeper and deeper into the sands. Men (b) By closing the large hole in her side
frequently came out in small boats and removed any (c) By joining a large number of steel sheets
parts that could be sold—until two men decided to together
attempt the impossible: to float the African Queen once (d) By pumping air into the tanks
more. Both men were engineers and had no experience 63. What was the danger which the divers faced?
of ships so that few people thought they could succeed. (a) The rough seas
The men began by studying the exact state of the (b) The cold and dark situation under water
African Queen and came to the conclusion that she (c) Having to contend with sharks
would float again if air were pumped into the tanks (d) The cutting edges of the steel sheets
which were now full of sea-water. A diver was sent
down to examine the underside of the ship. In the cold, 64. The two men felt proud because
dark water he found an enormous hole in her side (a) they could float the ship in three months
which had been torn when the ship ran aground. It was (b) they had succeeded when everyone thought
plain that nothing could be done until the hole was they would fail
@UPSC_THOUGHTS

repaired. As no single sheet of steel would cover it, the (c) the African Queen was coming into the harbour
men were obliged to order a great number of sheets (d) the African Queen began to stir in the water
which had to be joined together. For several weeks
Passage 15
divers worked continually to close the hole. At times,
the sea was so rough that it was difficult to go down; Energy is neither created nor destroyed; it is only
and on more than one occasion, they had to contend recycled and recast in different forms. Even the human
with sharks. body is a form of energy. Call it prana or jivatma or
At last the hole was covered and the men began simply vayu, energy sustains the gross body and
to pump the sea-water out of the ship’s tanks. It seemed expresses itself through sensory perceptions and the
as if they were bound to succeed, for when the tanks basic physical elements.
were full of air, the African Queen began to stir in the The three fundamental gunas—satvik, rajasik and
water. The men could not understand why she still tamasik—which characterise the nature of the human
would not float until they discovered that her rudder being, very often in a combination, are further subdivided
was embedded in mud. Huge cranes were brought to into many basic qualities commonly known as human
haul the sunken rudder out and the ship was again values. Love, affection, integrity and truth, for instance,
afloat. By this time, the men were almost exhausted.
are values that are as important to life as breathing or
They had worked ceaselessly for three months to save
eating. These values are positive but in the course of
the African Queen and had succeeded when everyone
practice and because they emanate from a mind that
thought they would fail. Now they stood on the bridge
is susceptible to negative thoughts, they get corrupted.
of the ship, tired but proud, as tugs brought the African
They generate negativity and manifest in the form of
Queen into the harbour.
lying, cheating or causing others harm. The very fact
61. Men frequently went out to the African Queen that human civilisation has survived over centuries
because shows that despite all the negativity, the force of
(a) it was a rare sight to see a sunken ship positive energy within all of us continues to expand
(b) they attempted to float the ship once again and enrich.

1 . 24
Comprehension

According to the theory of karma, every action (c) The chance that the reaction to every action
generates a corresponding reaction, good or bad as the can either be good or bad
case may be. The process might take place in this life (d) The outcome of good deeds is good and adds
or the next, but take place, it will. The theory prompted to the common good
sages to exhort humanity to be good and do good. This
68. What did Swami Vivekananda believe?
way, the result of a good deed will invariably be good
1. Perceiving goodness in human nature is
which will add to the collective good of the human
important.
species. This is what Sri Aurobindo called the Goodness
2. Doing good deeds benefits the individual.
Quotient. Swami Vivekananda firmly believed that
human nature is basically good. It is only to be perceived 3. Individual good multiplies into social good.
as such and being aware, one must follow it through (a) Only 1
for the good of oneself so that it multiplies for social (b) Only 2
good. Goodness will help improve the quality of life of (c) 1 and 3
not just the individual but an entire society. There is (d) 1, 2 and 3
no such thing as ‘negative’ energy, all energy being 69. Which of the following characterises human
positive and constantly recycled. Negativity is therefore nature?
a dysfunction of thought or outlook. This has to be (a) Actions such as breathing and eating
resisted from within by reinforcing the positive qualities (b) Good deeds
inherent in human nature. (c) Satvik, rajasik, vayu
The instant gratification of the senses might be the
(d) Three basic gunas comprising basic human
driving force in a society driven by consumerism but
values
@UPSC_THOUGHTS

when you realise that all the material success and


prosperity you have acquired over a lifetime means 70. Which of the following is true in the context of
nothing once your body lies inert, life takes on a new the passage?
meaning. 1. Energy helps to sustain our physical body.
2. Negativity is manifested in the form of lying
65. The fact that human civilisation has survived
and cheating.
indicates that
3. Negativity can ultimately end human
(a) positive energy within us increases despite
negativity civilisation
(b) too much positive energy is not harmful (a) 1 only
(c) negative energy usually overpowers positive (b) 1 and 2
energy (c) 1, 2 and 3
(d) positive values do not get corrupted if they are (d) None of them
practised 71. Which of the following is true about negativity?
66. Which of the following is suggested by the theory (a) It existed before positive energy
of karma? (b) Negativity comes from a deviation of our
(a) A person is rewarded or punished only in his thoughts
lifetime (c) Negativity cannot be fought
(b) Human civilisations will continue to survive (d) Negative energy is stronger than positive
over the years energy
(c) Every action has some reaction
(d) Energy is recycled Passage 16

67. What is the Goodness Quotient? The function of capital markets is to facilitate an
(a) Human nature though basically bad can be exchange of funds among all participants, and yet, in
trained practice we find that certain participants are not on par
(b) Goodness need not improve the quality of life with others. Members of society have varying degrees

1 . 25
Comprehension

of market strength in terms of information they bring (b) the allocation of financial resources takes
to a transaction, as well as of purchasing power and place among separate individual participants,
credit worthiness, as defined by lenders. each of whom has access to the market
For example, within minority communities, capital (c) since transaction costs for stocks, bonds and
markets do not properly fulfil their functions; they do other financial instruments are not equally
not provide access to the aggregate flow of funds in the apportioned among all minority group
United States. The financial system does not generate members, the financial market is subject to
the credit or investment-vehicles needed for underwriting criticism
economic development in minority areas. The problem (d) the existence of certain factors adversely
underlying this dysfunction is found in a rationing affecting members of minority groups shows
mechanism affecting both the available alternatives for that financial markets do not function as
investment and the amount of financial resources. This conventional theory says they function
creates a distributive mechanism penalising members
73. Which of the following can be inferred about
of minority groups because of their socio-economic
minority communities on the basis of the passage?
differences from others. The existing system expresses
(a) They are not granted governmental subsidies
definite socially biased investment preferences that
to assist in underwriting the cost of economic
result from the previous allocation of income and that
development
influence the allocation of resources for the present and
(b) They do not receive the share of the amount
future. The system tends to increase the inequality of
of funds available for investment that would
income distribution. And, in the United States economy,
be expected according to traditional financial
@UPSC_THOUGHTS

a greater inequality of income distribution leads to a


market analysis
greater concentration of capital in certain types of
(c) They provide a significant portion of the
investments.
funds that become available for investment in
Most traditional financial market analysis studies
the financial market
ignore financial markets’ deficiencies in allocation
(d) They provide the same access to alternative
because of analysts’ inherent preferences for the simple
sources of credit to finance businesses as do
model of perfect competition. Conventional financial
majority communities
analysis pays limited attention to issues of market
structure and dynamics, relative costs of information, 74. The passage states that traditional studies of the
and problems of income distribution. Market financial market overlook imbalances in the
participants act as entirely independent and allocation of financial resources because
homogeneous individuals with perfect foresight about (a) those performing the studies choose an
capital market behaviour. Also it is assumed that each oversimplified description of the influences
individual in the community at large has the same on competition
access to the market and the same opportunity to (b) the analysts who study the market are
transact and to express the preference appropriate to unwilling to accept criticism of their methods
his or her individual interest. Moreover, it is assumed as biased
that transaction costs for various types of financial (c) such imbalances do not appear in the statistics
instruments (stocks, bonds, etc.) are equally known and usually followed to measure the market’s
equally divided among all community members. behaviour
(d) an optimum allocation of resources is the
72. The main point made by the passage is
final result of competition among participants
that
(a) investments in minority communities can be 75. According to the passage, analysts have
made by the use of various alternative financial conventionally tended to view those who
instruments, such as stocks and bonds participate in financial markets as

1 . 26
Comprehension

(a) varying in market power with respect to one Some of the questions that scholars ask seem to the
another world to be scarcely worth asking let alone answering.
(b) having equal opportunities to engage in They ask questions too minute and specialised for you
transactions and me to understand without years of explanation. If
(c) influencing the allocation of funds through the world inquires of one of them why he wants to
prior ownership of certain kinds of assets know the answer to a particular question he may say,
(d) basing judgements about future events mainly especially if he is a scientist, that the answer will in
on chance some obscure way make possible a new machine or
76. A difference in which of the following would be weapon or gadget. He talks that way because he knows
an example of inequality in transaction costs as that the world understands and respects utility. But to
alluded to in the last lines of the passage? you who are now part of the university, he will say that
(a) Fees charged to large and small investors for he wants to know the answer simply because he does
purchasing stocks not know it. The way a mountain climber wants to
(b) Exchange rates in dollars for currencies of climb a mountain simply because it is there. Similarly
different countries a historian when asked by outsiders why he studies
(c) Maximum amount of loans extended by a history may come out with the argument that he has
bank to business in different areas learnt to repeat on such occasions, something about
(d) Fees paid to different attorneys for preparing knowledge of the past making it possible to understand
legal suits for damages the present and mould the future. But if you really want
77. According to the passage, a questionable to know why a historian studies the past, the answer
@UPSC_THOUGHTS

assumption of the conventional theory about the is much simpler: something happened, and he would
operation of financial markets is that like to know what.
(a) market structure and market dynamics depend All this does not mean that the answers which
on income distribution scholars find to their questions have no consequences.
(b) those who engage in financial market They may have enormous consequences but these seldom
transactions are perfectly well informed about form the reason for asking the question or pursuing the
the market answers. It is true that scholars can be put to work
(c) inequalities in income distribution are answering questions for the sake of the consequences
increased by the functioning of the financial as thousands are working now, for example, in search
market of a cure for cancer. But this is not the primary function
(d) credit worthiness as determined by lenders is of the scholar, for the consequences are usually
a factor determining market access subordinate to the satisfaction of curiosity.

Passage 17 78. Common people consider some of the questions


asked by scholars as unimportant
The world dismisses curiosity by calling it idle, or mere (a) since they are not worth asking or answering
idle curiosity—even though curious persons are seldom
(b) because the question is related to new
idle. Parents do their best to extinguish curiosity in their
machines and gadgets
children because it makes life difficult when one is
(c) because the common man doesn’t understand
faced everyday with a string of unanswerable questions
questions without years of explanations
about what makes fire hot or why grass grows. Children
(d) because scholars ask very minute, specialised
whose curiosity survives parental discipline are invited
questions beyond the comprehension of the
to join our university. Within the university, they go on
common man
asking their questions and trying to find the answers.
In the eyes of a scholar, that is mainly what a university 79. In the statement ‘. . . that is mainly what a
is for. university is for’, ‘that’ refers to:

1 . 27
Comprehension

(a) parents’ refusal to answer questions competitive strength in the market. Higher prices usually
(b) children’s curiosity that survives parental mean fewer customers, reduced profit means less capital
strictures investment, and low-cost materials mean poor product
(c) questions not worth answering quality. But, increasing labour productivity by enhancing
(d) opportunity to curious minds to find out skills and motivation creates an almost unlimited
answers to their questions resource. The development of economic resources,
80. According to the passage the general public human as well as non-human, is the product of human
respects effort, and the quality of human effort in large part
(a) new inventions depends on human motivation.
(b) any useful invention Enthusing employees with workaholic spirit through
(c) any invention that makes life easier for them traditional authority and financial incentives has become
(d) a scientist who invents gadgets and machines increasingly difficult as employees become economically
for them secure and their dependency on any one particular
organisation decreases. According to expectancy
81. The writer compares the scientist to
theorists, the motivation to work increases when an
(a) a historian and a mountain climber
employee feels his performance is an instrument for
(b) a historian
obtaining desired rewards. Nevertheless, in many
(c) a mountain climber
organisations today employees are entitled to
(d) a scholar
organisational rewards just by being employed. Unions,
82. The primary function of a scholar is different from governmental regulations, and the nature of the job
@UPSC_THOUGHTS

the search for a cure for cancer because itself in some cases prevent management from relating
(a) the answers to the scholar’s question have no financial rewards to performance. People may be
consequence unlike the results of the research attracted to join and remain in organisations to receive
involving a cure for cancer organisational rewards, but being motivated to join an
(b) the answer sought by the scholar is selfish organisation is not the same as being motivated to exert
unlike the consequences of cancer research effort in an organisation. The challenge to management
which are for the common weal is to find and administer alternative forms of incentives
(c) the scholar seeks satisfaction of his mental which will induce employees to improve work
curiosity, while research involving a cure for performance. Such alternative forms of reinforcement
cancer demands a constant, systematic and
will require increased understanding of motivational
planned pursuit by several scholars
theories and programmes.
(d) several scholars work for a cancer cure while
a single scholar works with a selfish motive 83. Which of the following statements is true in the
context of the passage?
Passage 18 1. Development of economic resources is
There is absolutely no point in complaining that, over primarily the product of market conditions.
the years, there has been pressure for increased 2. Increasing the selling price of products is one
productivity and higher earnings for workers in of the best ways to motivate employees.
industry. There are several ways of increasing 3. All employees should be entitled to
employees’ earnings. Employee earnings can be organisational rewards just by being
increased by raising the selling price of the firm’s employed.
products and services, reducing profits or costs of raw (a) 1 and 2
materials, or augmenting labour productivity. However, (a) 2 only
increasing employee earnings by means other than (a) 3 only
increased labour productivity jeopardises the firm’s (a) None of them

1 . 28
Comprehension

84. Organisations can derive maximum advantages 3. Employees can be easily motivated with
by traditional authority today.
(a) providing financial incentives to employees (a) 1 only
regardless of performance (b) 3 only
(b) enhancing labour productivity by increasing (c) 1 and 3
skills and motivation (d) 2 and 3
(c) encouraging employees to expend greater
89. Which of the following factors determine the
physical energy
quality of human efforts?
(d) inducing employees to improve work
performance and control their demands (a) Desire and willingness of an individual to
excel in whatever he undertakes
85. According to the passage, all of the following (b) Economic resources available with the
contribute to an increase in employee earnings, organisation
except (c) The individual’s innovativeness
(a) reducing profits in favour of employees
(d) Authoritarian leadership and job security
(b) providing incentives and fringe benefits to
employees 90. In the context of the passage, a company’s
(c) enhancing labour productivity competitive strength in the market is affected
(d) increased capital investment mainly because of
1. a slump in the international market.
86. Employees feel motivated to work when they
2. poor inter-departmental coordination.
1. experience good working conditions in the
@UPSC_THOUGHTS

3. labour productivity.
organisation.
(a) Only 1
2. decide to produce goods and services as a
(b) Only 2
result of team work.
(c) Only 3
3. think of performance as a tool for obtaining
rewards. (d) None of them
(a) 1 and 2 Passage 19
(b) 2 and 3
(c) 3 only In order to better understand conservation in China, it
(d) 1, 2 and 3 is essential that one has a grasp of what the term
‘Chinese conservatism’ means. Chinese conservatism is
87. Which of the following factors, according to the
passage, adversely affects the organisation’s markedly different from the conservatism of the modern
competitive strength? West. The political term ‘conservative’ came about
(a) Making rewards contingent on performance during the French Revolution and inspired men who
(b) Anti-productivity and anti-management were determined to preserve Christian and aristocratic
activities of labour unions elements in European society. Chinese conservatism
(c) Motivating employees with traditional began around the time of the Taiping rebellion and had
authority as its primary objectives the preservation of both
(d) Increasing employee earnings regardless of Confucian society and non-feudal strains of pre-Opium
their productivity War Chinese society, while Western conservatism
88. Which of the statements is/are not true in the believes in sacredness of private property and distrust
context of the passage? of cosmopolitan order. Thus, the only common area of
1. Human effort is the cause of the development agreement between European and Chinese conservatism
of economic resources. is the intent to conserve.
2. Management is free to relate financial rewards During the Tung-chin Restoration the great aim
to performance. was the revival of Confucian values and institutions.

1 . 29
Comprehension

But these aims had to be modified so that they might 92. The major similarity between Chinese and Western
endure. Restoration statesmen had no desire to create conservatism is
a new society—they wanted to restore a society that (a) that Chinese conservatism developed during
they believed had been based on truth. The statesmen the Taiping revolution
of the Restoration stretched the traditional ideology to (b) that both attempted to preserve tradition
its limits in an effort to make the Confucian system (c) that Chinese conservatism is primarily land-
work under new conditions. They were true oriented
conservatives in a great tradition, living in an age when (d) the cosmopolitan nature of Western
revolutionary change was unavoidable. The aim of the conservatism
Restoration was to restore to their original vitality, the 93. A primary objective in the development of
best of the ancient institutions. During the Restoration, Restoration thought was
the two immediate problems were the suppression of (a) to create a new society based on modern
rebellion and the stabilisation of foreign relations. In values
addition, the people were striving for a restoration of (b) the knowledge that Chinese conservatism is
the system of government by superior civil officials. superior to Western conservatism
The men in the hierarchy of the Restoration rose (c) to restore traditional Chinese society under
to prominence through proven ability in both civil and new conditions
military affairs. They emphasised human and social (d) to restore the system of government by superior
training—that is, indoctrination, morality and the art civil officials
of leadership through the cultivation of character. The 94. The Western conservatives intended to preserve
@UPSC_THOUGHTS

great majority of the officials rose through the all the following except
examination system. (a) Christianity
During the chaos of this period, the examination (b) private property
system had lost much of its effectiveness. This is (c) cosmopolitanism
important and must be noted because the examination (d) aristocratic elements
system was the traditional avenue for selecting officials.
95. The traditional method for selecting officials was
The senior officials of Restoration realised that their
(a) by the civil government
policies would be ineffective unless the quality of the
(b) through a subjective testing system
junior official was improved, so it was their duty to
(c) sponsorship by a high government official
weed out the officials who had attained office in
(d) the examination system
irregular ways and to promote the examination system
as the only way to high position. But these men of the 96. During restoration, ancient institutions were
Restoration had enough foresight to determine that it (a) responsible for bringing out a revolutionary
was impossible to select officials automatically on the change in China
basis of the objective type test alone. As a result, the (b) no longer accepted as a viable alternative to
system of recommendation was ushered in, whereby a Western technology
high official sponsored the career of a promising young (c) considered as a primary reason for the decline
man. This acted as an important supplement to the of traditional China
examination system. (d) to be the cornerstone of a changing but
traditional society
91. The most significant Chinese philosopher mentioned
in the passage is 97. The most appropriate title for this passage will be
(a) Tung-chin (a) The Chinese Examination System
(b) Ching (b) Impact of Western Conservatism
(c) Taiping (c) Chinese Conservatism
(d) none of these (d) Revival of Confucian Values and Institutions

1 . 30
Comprehension

Passage 20 The final argument concerns the relationship


between delayed development and the state. Countries
In many underdeveloped countries, the state plays an wishing to industrialise today have more competitors,
important and increasingly varied role in economic and these competitors occupy a more differentiated
development. There are four general arguments, all of industrial terrain than previously. This means that the
them related, for state participation in economic available niches in the international system are more
development. First, the entrance requirements in terms limited. For today’s industrialisers, therefore, the process
of financial capital and capital equipment are very large of industrialisation cannot be a haphazard affair, nor
in certain industries, and the size of these obstacles will can the pace, content, and direction be left solely to
serve as barriers to entry on the part of private investors. market forces. Part of the reason for a strong state
One can imagine that these obstacles are imposing in presence, then, relates specifically to the competitive
industries such as steel production, automobiles, international environment in which modern countries
electronics, and parts of the textile industry. In addition, and firms must operate.
there are what Myint calls “technical indivisibilities in
98. Which of the following best states the main point
social overhead capital.” Public utilities, transport, and
of the passage?
communications facilities must be in place before
(a) State participation should not have a role in
industrial development can occur, and they do not lend the industrialisation of any nation
themselves to small-scale improvements. (b) Underdeveloped countries face a crisis of
A related argument centres on the demand side of over-population, and lack of effective demand
the economy. This economy is seen as fragmented, that requires outside assistance to overcome
@UPSC_THOUGHTS

disconnected, and incapable of using inputs from other (c) Without state intervention, many less
parts of the economy. Consequently, economic activity developed countries would be unable to carry
in one part of the economy does not generate the out the interconnected tasks necessary to
dynamism in other sectors that is expected in more achieve industrialisation
cohesive economies. Industrialisation necessarily (d) State economic planning can make non-
involves many different sectors; economic enterprises industrialised countries develop their
will thrive best in an environment in which they draw resources rapidly
on inputs from related economic sectors and, in turn,
release their own goods for industrial utilisation within 99. According to the passage, all of the following
their own economies. argue in favour of state intervention except
A third argument concerns the low-level equilibrium (a) the state must mediate between demand and
trap in which less developed countries find themselves. supply sides of the economy
At subsistence levels, societies consume exactly what (b) the pace and process of industrialisation are
too important to be left to market trends alone
they produce. There is no remaining surplus for
(c) public amenities are required to facilitate a
reinvestment. As per capita income rises, however, the
favourable environment for industrialisation
additional income will not be used for savings and
(d) the security of workers should not be affected
investment. Instead, it will have the effect of increasing
by the variability of industrial trends
the population that will eat up the surplus and force
the society to its former subsistence position. Fortunately, 100. Which of the following, according to the passage,
after a certain point, the rate of population growth will are appropriate roles for the state in economic
decrease; economic growth will intersect with and development?
eventually outstrip population growth. The private I. Development of infrastructure facilities to
sector, however, will not be able to provide the one-shot service industry.
large dose of capital to push economic growth beyond II. Provision of capital inputs sufficient for
those levels where population increases eat up the growth to surpass increases in per capita
incremental advances. consumption.

1 . 31
Comprehension

III. safeguarding the domination of local markets derives from science and manifests itself in physical
with a single source of capital. form). The scope to apply technology to both farm and
(a) I and II non-farm activities in rural areas is huge, as are the
(b) II and III potential benefits. In fact, crop yields are far lower than
(c) I and III what they are in demonstration farms, where science
(d) I, II and III and technology are more fully applied. Technologies
101. According to the passage, “low-level equilibrium that reduce power consumption of pumps are vital;
trap” results from unfortunately, their use is minimal, since agricultural
(a) lack of state intervention power is free or largely subsidised. Similarly, there is
(b) the inability of market forces to overcome the little incentive to optimise—through technology or
effects of population growth otherwise—water use, especially in irrigated areas (a
(c) the tendency of societies to increase their third of total arable land), given employment and
populations incomes, and at present deployment of technology is
(d) subsistence-level economies marginal. Cold storage and cold-chains for
transportation to market is of great importance for many
102. Which of the following, if true, would weaken the
agricultural products—particularly, fruits and
author’s contention that state participation is
vegetables—but are non-existent. These are clearly
important in the launch of large-scale industries?
technologies with an immediate return on investment,
I. Associations of private sector investors can
raise large amounts of capital by pooling their and benefits for all: the farmer, the end-consumer, the
resources. technology provider. However, regulatory and structural
@UPSC_THOUGHTS

II. A series of small-scale improvements can help barriers are holding back investments.
to build up transportation and communi- Power is a key requirement in rural areas, for
cations facilities. agricultural as well as domestic use. Technology can
(a) I only provide reliable power, at comparatively low cost in a
(b) II only decentralised manner. However, this needs to be
(c) Both I and II upgraded and scaled in a big way, with emphasis on
(d) Neither I nor II renewable and non-polluting technologies. Reliable and
low cost means of transporting goods and people is an
Passage 21 essential need for rural areas. The bullock-cart and the
Rural India faces serious shortages—power, water, tractor-trailer are at present the vehicles of choice.
health facilities, roads, etc.; these are known and Surely, technology can provide a better, cheaper and
recognised. However, the role of technology in solving more efficient solution? Information related to commodity
these and other problems is barely acknowledged and prices, agricultural practices, weather, etc., is crucial for
the actual availability of technology in rural areas is the farmer. Technology can provide these through
marginal. The backbone of the rural economy is mobile phones, which is a proven technology; however,
agriculture which also provides sustenance to over half the challenge to ensure connectivity remains. Thus
the country’s population. The ‘Green Revolution’ of the there is a pressing need for technology as currently
1970s was, in fact, powered by the scientific work in economic growth—though skewed and iniquitous—
various agricultural research institutions. While some has created an economically attractive market in rural
fault the Green Revolution for excessive exploitation of India.
water and land resources through overuse of fertilisers, 103. According to the author, which of the following
it did bring about a wheat surplus and prosperity in is/are the problem/s facing India’s rural
certain pockets of the country. population?
In rural India today, there is a dire inadequacy of 1. Unavailability of healthcare facilities.
both science (i.e. knowledge) and technology (which 2. The technological advancements which have

1 . 32
Comprehension

been borrowed from abroad have not been 108. What has hampered investment in post-harvest
suitably adapted to the Indian scenario. technologies?
3. Lack of awareness about the importance of (a) Cost of implementing such technology is
utilising technology in the agricultural sector. higher than the returns
(a) Only 1 (b) No tangible benefits to technology suppliers
(b) Only 3 (c) Obstacles from statutory authorities
(c) Both 2 and 3
(d) Rapid economic growth has drawn investors
(d) None of these
away from agriculture to more commercially
104. Which of the following is not an impact of the viable sectors
Green Revolution?
109. What, according to the passage, is the role of
(a) Over-utilisation of water resources
mobile technology in the rural economy?
(b) Application of scientific research only in
demonstration farms 1. It will not play a large role since the technology
(c) Wealth creation restricted to certain areas is largely untested.
(d) Supply of wheat surpassed demand 2. It provides opportunities for farmers to
manipulate commodity prices.
105. Why is there no motivation to reduce power
3. It will largely be beneficial since such
consumption?
technology is cheap.
(a) Freely available renewable sources of energy
(a) Only 1
(b) Government will have to subsidise the cost of
(b) Both 2 and 3
technology required to reduce power
@UPSC_THOUGHTS

consumption (c) Only 3


(c) The cost of implementing power saving (d) None of these
technology is exorbitant for the customer. 110. Which of the following is currently not a threat
(d) None of these to the rural economy, according to the passage?
106. The author’s main objective in writing the passage 1. Inadequate rural infrastructure such as roads.
is to 2. Excessive utilisation of technology.
(a) criticise farmers for not utilising experimental, 3. Fluctuating power supply.
low cost post-harvesting technology (a) Only 1
(b) exhort the government to subsidise the cost of (b) Only 2
utilising technology (c) Both 1 and 2
(c) promote a second Green Revolution (d) Both 2 and 3
(d) advocate broadening the scope of research
and use of technology in agriculture 111. Which of the following is/are true in the context
of the passage?
107. Which of the following is not true in the context
1. About 33 per cent of arable land in India is
of the passage?
irrigated.
1. In recent times the benefits of science and
2. There is hardly any motivation to utilise
technology have not been felt in agriculture.
technology to optimise water usage among
2. The current means of rural transportation are
ideal i.e. low cost and non-polluting. farmers.
3. Agriculture provides livelihood to over 50 per 3. Climatic information can easily be made
cent of the Indian population. available to farmers.
(a) Both 1 and 2 (a) Only 1
(b) Only 2 (b) Both 1 and 2
(c) Only 3 (c) Both 2 and 3
(d) Both 1 and 3 (d) 1, 2 and 3

1 . 33
Comprehension

Passage 22 112. Which of the following is true in the context of


the passage?
Modern biotechnology, especially the creation of (a) Genetically modified crops have been
genetically modified (GM) crops, is often presented as universally recognised as a solution to
a magic solution or universal panacea for the problems poverty and environmental degradation
of poverty, inadequate nutrition and even environmental (b) The only way to improve the deficit in food
degradation across the world. Conversely, there are requirement and food production in the world
people who present the picture of tech generated is by adapting genetically modified crops
monsters and major human health hazards being created (c) Genetically modified crops produce more yield
by science. Many of the technological changes currently as compared to yield from the traditional
in the process of being utilised in agriculture can have methods
unforeseen consequences, and their safety and future (d) Taking advantage of absence of regulatory
viability are far from secure. standards, scientists have been dumping new
The reality, as always, is far more complex than products in the markets without appropriate
either of these two extremes. Even today the total food approval
production in the world is adequate to feed the hungry
113. The author of the given passage seems to be
of the world; the problem is rather one of unequal
definitely
distribution, which deprives a large part of the
(a) suggesting the use of traditional methods of
population of even their minimal nutritional
agriculture as against biotechnology by
requirements. Similarly, farmers, especially in developing
@UPSC_THOUGHTS

developing countries owing to their poor


countries, face many problems such as lack of
regulatory standards
infrastructure, poor or unstable market access, volatile
(b) in favour of utilising biotechnology as a tool
input and output prices etc. that biotechnology does not
for alleviation of poverty from the world
address, much less solve.
(c) urging the policy makers to improve
It is true that transgenic plants can offer a range
infrastructural facilities so that farmers can
of benefits which are above and beyond those which
maximise the benefits of genetically modified
emerged from more traditional innovations in cultivation.
crops
It is suggested that such new technology offers more
(d) unconvinced of the long-term effects and
effective pest resistance of seeds and crops through
rationale for immediate requirement of
genetic control mechanisms, which also reduces the
genetically modified products
need for pesticide use and leads to improved yield. A
basic question, of course, is whether the new GM 114. Why, according to the author, is genetic
technology is safe, and whether this is absolutely modification of crops not an answer to the
crucial since the effects may only be known much later. problem of hunger in the world?
The jury is still very much out on this matter, and it 1. People being highly doubtful of the long term
does not appear that the controversy would be resolved effects of genetically modified crops, do not
quickly. buy the products grown by such methods.
The trouble is that most governments in developing 2. The problem of hunger in the world is not due
countries have relatively low food and beverage to inadequate production of food but due to
regulatory standards, and public systems for monitoring unequal distribution of it.
and surveillance of such items are poor or non-existent. 3. Many developing countries have banned
This leaves them open to entry and even dumping of genetically modified products as developed
a range of agricultural products of the new technology, countries have been using these countries as
which may not pass regulatory standards in the more dumping grounds for new genetically
developed countries. modified products.

1 . 34
Comprehension

(a) Only 1 IV. are checked to verify the suitability of


(b) Only 2 applicants to some jobs
(c) Both 2 and 3 (a) I and II
(d) Both 1 and 3 (b) I, II and III
(c) I, II and IV
115. The author definitely questions all the following
(d) I, II, III and IV
except
(a) the morality of countries that dump GM food 117. The main reason why fingerprinting is used for
on developing countries identification is that
(b) the safety of GM technology (a) every individual has a unique set of
(c) the contention that food is in short supply fingerprints
(d) the absolute necessity of using GM technology (b) every set of fingerprints falls into a pattern
(c) records of fingerprints can be maintained
Passage 23 (d) fingerprints can be picked up even after a
crime
Fingerprints are the marks made by the ridges on the
118. Who first used finger impressions to authenticate
ends of the fingers and thumbs. These ridges form a
documents?
pattern that stays the same throughout a person’s life.
(a) Sir Francis Galton
No two persons have ever had the same fingerprints.
(b) Sir E.R. Henry
So fingerprints are a foolproof way of identifying a
(c) The Chinese
person.
@UPSC_THOUGHTS

(d) The British


A fingerprint record is made in an interesting
manner. A small piece of metal is coated with a thin 119. When a person’s fingerprint record is made, the
film of ink. Next, a person’s finger and thumb tips are impression is taken of
pressed against the inked surface. Then the fingertips (a) both thumbs
are pressed on a white card. The prints are recorded (b) the right thumb
in exact detail. (c) the tips of fingers and the thumb
Fingerprinting is often used to solve crimes. (d) the entire palm
Fingerprints are picked up at the scene of a crime. These
Passage 24
are compared with those of a suspect. Millions of
fingerprints are kept on files by police departments. The ease with which democratic governments have
Fingerprinting is also used in finding missing given way to authoritarian regimes in one Asian country
persons and identifying unknown dead. It is used to after another has made many persons ask in despair
screen people who apply for certain jobs. whether the parliamentary system based on the Western
It is thought that the Chinese used thumbprints to model is suited to underdeveloped countries. People
sign documents a long time before Christ. The system who do not know how to read and write, they argue,
used today was invented by Sir Francis Galton in the can hardly know how to vote. Popular elections often
1880s. In 1901, Sir E.R. Henry found a simple way of bring incompetent men to the top, they contend, and
grouping fingerprints. His system is used by many law- the division of party spoils breeds corruption. What is
enforcement organisations. worse, the system of perpetual party warfare obstructs
the business of government.
116. According to the passage, it would be correct to They point to the dismal results of the last ten years.
say fingerprints The pace of social and economic change has been far
I. are marks made by raised lines on the fingers too slow and the governments in most of the
II. remain unchanged all through one’s life underdeveloped countries have failed to come to grip
III. are reliable means of identifying a person with the problems which face the people. What they say

1 . 35
Comprehension

is no doubt true to some extent but it is pertinent to II. are not able to deal with the problems of
remember that every alternative to democracy, while it people
in no way guarantees greater integrity or efficiency in III. rule by trial and error
the administration, lacks even the saving merit of IV. have no experience in good governance
regimes which, based on the suffrage of the people, (a) I only
leave it to the people to find out, by trial and error, who (b) II only
is their best friend. The people can peacefully get rid (c) II and III
of a democratic government which has failed to keep (d) I, II, III and IV
its promise, they can overthrow a dictatorial regime 124. The writer
only through a violent revolution. Those who feel sore I. criticises democratic governments in
over the ills from which democratic regimes suffer underdeveloped countries in Asia for their
should therefore be wary of suggesting a cure which incompetency
is likely to undermine the democratic structure of the II. favours democratic regimes in spite of their
state. The people can at least raise their voice of protest shortcomings
against the injustices of a democratic government; they III. admits that critics of democratic regimes are
can only suffer in silence the tyranny of a regime which correct to some extent
is responsible to no one but itself. (a) I and III
(b) II and III
120. Democratic governments have given way to
(c) I, II and III
authoritarian regimes in several Asian countries
(d) does not subscribe to any of the given
because
statements
@UPSC_THOUGHTS

(a) Asians are underdeveloped


(b) Asians like powerful leaders 125. Why is a democratic regime to be preferred over
(c) Asians cannot read and write and can hardly a dictatorial one?
know how to vote I. It does not require a violent revolution to
(d) Asians have not been able to adapt themselves change a democratic government.
to the Western parliamentary system II. Under a democratic regime, people can voice
their displeasure over the injustices they suffer.
121. Popular elections in underdeveloped countries
III. A dictatorial government shows greater
(a) give power to men who lack ability inefficiency than a democratic one.
(b) result in the fragmentation of political parties (a) I only
(c) lead to misgovernance (b) II only
(d) do not give true representation to the people’s (c) I and II
choice (d) I, II and III
122. ‘They point to the dismal results of the last ten
years.’ The word ‘They’ represents Passage 25
(a) the incompetent leaders
When flowers bloom in the lush bamboo plantations
(b) critics of all democracies
in the hills of the northeast, the tribesmen are thrown
(c) people who feel that the Western parliamentary
into a state of panic. The rare phenomenon of the
system is unsuitable for underdeveloped
flowering of a dwarf-sized bamboo species triggers a
countries
boom in the rat population. They devour the crops in
(d) people who feel that underdeveloped countries
nearby farmlands. The result is famine.
are better off with autocratic rule
The mauve-coloured flowers sprouting in the hill-
123. According to one view presented in the passage, slopes in the sprawling Seppa valley in east Kameng
governments in underdeveloped countries district, Arunachal Pradesh, are a palpable threat to the
I. are not interested in the socio-economic State Government. The worries are justified, if the
development of the citizens catastrophe caused by the flowering of a bamboo

1 . 36
Comprehension

species named Mau in Mizoram in 1959 is anything World countries where the natural tendency has been
to go by. Rodents had multiplied in millions during the to fix prices, have licences and block imports. This
flowering of bamboos and ravaged the crops in the cannot be changed by doctrinaire arguments like
foothills. ‘everything should be left to the market’ which have
The flowering of the bamboo species in Arunachal been recommended and at times imposed on Third
Pradesh was first noticed in September this year. The World countries in the name of the fundamental theorem
depredations by the rats in the paddy, maize and millet
of economics. In order to use the market to the advantage
fields from Bhalukpong on the Assam border to Seppa
of the society, it is important to be aware of its limitations.
valley in the north were reported soon. As the agriculture
For those who believe that individual rationality is
department rushed its field-staff to the affected areas to
invariably efficient, a drive through Delhi’s Chandni
fight the rampaging rodents with rat-traps and zinc
Chowk area can be instructive. Overtaking from the
phosphate, fresh alarms were sounded with flowering
wrong side, grazing past other vehicles, stepping on the
reported from the Tezu circle.
accelerator at the sight of a zebra crossing to frighten
According to the local MLA much of the crops in
Kameng had been destroyed by October. The State away pedestrians, drivers here can be accused of
Veterinary Minister also confirmed the reports of a near- anything but the lack of individual rationality and
famine condition in that inaccessible district. The rats, enterprise. And the outcome of this naked display of
feeding on the flower seedlings, continue to multiply. self-interest is chaos. At least in this situation there is
Till today, 33,000 rats, of the 26 varieties in the State, much to be gained by supplementing the ‘invisible
have been trapped with the help of indigenous devices. hand’ of a multitude acting rationally with the visible
hand of the traffic warden.
126. What happens when the bamboos flower in the
@UPSC_THOUGHTS

luxuriant plantations of the northeast? 128. The general tendency in Third World countries
I. The tribesmen are overjoyed. has been to
II. There is a huge increase in the rat population. (a) increase prices of non-essential goods
(a) I only (b) control industries
(b) II only (c) encourage imports
(c) Both I and II (d) allow individual enterprise enough licence
(d) Neither I nor II
129. According to the writer of the passage,
127. Which of the following statements is/are correct I. everything should be left to the market.
with reference to what is given in the passage? II. individuals can rationally work out everything
I. The flowering of a species of bamboo on the to the advantage of society.
hill slopes is linked with the possibility of III. regulating the market is unwarranted.
crop damage in the foothills. IV. the market is highly controlled in socialist
II. The agricultural department imports rat traps economies.
to deal with the rodent menace. (a) I and II
III. The bamboo flower seedlings are a source of (b) I, II and III
food for rodents. (c) III only
(a) I only (d) None of the statements is correct
(b) I and II
(c) I and III 130. It may be said of the writer that he or she
(d) I, II and III (a) is a staunch supporter of the free market
(b) favours a controlled economy as in socialistic
countries
Passage 26
(c) is moderate, favouring some regulation of the
The market mechanism has an important role to play free market
not only in capitalist economies but also in socialist (d) does not like the chaotic conditions of traffic
ones. The truth of this is especially felt in many Third in Delhi

1 . 37
Comprehension

Passage 27 (b) the human species does not have many


biologically fixed action patterns
The specific meaning of social change depends, first of (c) human beings have a great capacity to learn
all, on the social entity considered. Changes in a small and create
group may be important on the level of that group itself, (d) the human species is capable of no change
but negligible on the level of the larger society. Similarly, that is not genetically determined
the observation of social change depends on the time
span taken; most short-term changes are negligible if
Passage 28
a social development is studied in the long run. Even
if one abstracts from small-scale and short-term changes, Today, with a Nobel Prize to its credit, Grameen is one
social change is a general characteristic of human of the largest microfinance organisations in the world.
societies: customs and norms change, inventions are It started out lending small sums to poor entrepreneurs
made and applied, environmental changes lead to new
in Bangladesh to help them grow from a subsistence
adaptations, conflicts result in redistribution of power.
living to a livelihood. The great discovery its founders
This universal human potential for social change
made was that even with few assets, these entrepreneurs
has a biological basis. It is rooted in the flexibility and
repaid on time. Grameen and microfinance have since
adaptability of the human species—the near absence of
biologically fixed action patterns on the one hand and become financial staples of the developing world.
the enormous capacity for learning, symbolising, and Grameen’s approach, unlike other microfinancers, uses
creating on the other hand. The human biological the group-lending model. Costs are kept down by
constitution makes changes possible that are not having borrowers vet one another, tying together their
@UPSC_THOUGHTS

biologically (genetically) determined. Social change, in financial fates and eliminating expensive loan officers
other words, is only possible by virtue of biological entirely. The ultimate promise of Grameen is to use
characteristics of the human species, but the nature of business lending as a way for people to lift themselves
the actual changes cannot be reduced to these species’ out of poverty.
traits. Recently Grameen has taken on a different
131. Which one of the following statements is correct challenge—by setting up operations in the US. Money
regarding social change? may be tight in the waning recession, but it is still a
(a) Social change has to be studied in the long nation of 1,00,000 bank branches. Globally, the working
term to understand it microfinance equation consists of borrowing funds
(b) All human societies are characterised by social cheaply and keeping loan defaults and overhead
change expenses sufficiently low. Microlenders, including
(c) Environmental changes drastically affect social Grameen, do this by charging colossal interest rates—
mores as high as 60 per cent or 70 per cent—which is
(d) All the above are true of social change necessary to compensate for the risk and attract bank
132. Social change has biological connotations because funding. But loans at rates much above the standard
I. it is genetically determined. 15 per cent would most likely be attacked as usurious
II. it is based in human capability for adaptation. in America.
(a) I only So, the question is whether there is a role for a Third
(b) II only World lender in the world’s largest economy. Grameen
(c) Both I and II America believes that in a few years it will be successful
(d) Neither I nor II and turn a profit, thanks to 9 million US households
133. According to the passage it is not correct to say untouched by mainstream banks and 21 million using
that the likes of payday loans and pawn shops for financing.
(a) inventions and their application is an aspect But enticing the unbanked won’t be easy. After all,
of social change profit has long eluded US microfinanciers and if it is

1 . 38
Comprehension

not lucrative, it is not microlending, but charity. When 136. Which of the following can be inferred from the
Grameen first went to the US, in the late 1980s, it passage?
tripped up. Under Grameen’s tutelage, banks started (a) Microfinance has been successful only in
micro loans to entrepreneurs with a shocking 30 per Asian countries
cent loss. But Grameen America says that this time (b) America has the largest number of banks in
the world
results will be different because Grameen employees
(c) There is scope for microfinance institutions to
themselves will be making the loans, not training an
be profitable in developed countries
American bank to do it. More often than not, the
(d) There are no informal sources of credit in
borrowers in the US, Grameen finds, already have jobs
developed countries
(as factory workers, for example) or side businesses—
selling toys, cleaning houses etc. The loans from 137. According to the author, what has enhanced the
likelihood of success for Grameen America at
Grameen, by and large, provide a steadier source of
present?
funding, but they don’t create businesses out of nothing.
(a) Its success in Bangladesh and other
But money isn’t everything. More importantly for many
developing countries
entrepreneurs, group members are tremendous sources
(b) Absence of other microfinance institutions for
of support to one another. So even if studies are yet to competition
determine if Grameen is a clear-cut pathway out of (c) The fact that America is currently in the midst
poverty, it still achieves something useful. of a recession
134. What has adversely affected the success of (d) None of these
@UPSC_THOUGHTS

microfinance institutions in the US? 138. Which of the following can be said about
(a) The focus of these institutions is on making Grameen?
a profit at any cost instead of being charitable 1. Its success in developing countries will ensure
to the needy its success in developed countries.
(b) American banks engaged in microlending 2. It ensures that the poor in developing countries
were the most severely hit during the recession enjoy a subsistence standard of living.
3. It has demonstrated that the poor are far more
(c) A widespread perception among bankers that
likely to repay loans than the affluent.
these institutions are better suited to
(a) Only 2
developing countries
(b) Only 1 and 2
(d) Their failure to attract those outside the formal (c) Only 2 and 3
banking system as customers (d) None of them
135. Why has Grameen made a second attempt to 139. What is the central theme of the passage?
launch itself in the US? (a) A comprehensive evaluation of the current
(a) The willingness of US banks to provide the status of the American economy
necessary staff and funds to facilitate the (b) A discussion about the prospects of Grameen
spread of microfinance and microfinance in the US
(b) The rates of interest on loans in the US are (c) The role of banks in facilitating microlending
exorbitant, making it easier to recover capital efforts in developed nations
(c) The realisation that a large percentage of the (d) Microfinance efforts are useful in developing
countries but are futile in developed ones
American population not reached by
mainstream banks can be tapped 140. Why was Grameen America’s initial US initiative
(d) Recognition of the fact that disbursing credit a flop?
in developing countries during the recession 1. Lack of proper training to Grameen America
is too risky personnel.

1 . 39
Comprehension

2. It ended up giving loans at half their customary trypotophan level and the concentrations of trypotophan
rates of interest. and of serotonin in the brain increased after the meal.
(a) Only 1 Surprisingly, however, when we added a large
(b) Only 2 amount of protein to the meal, brain trypotophan and
(c) Both 1 and 2 serotonin levels fell. Since protein contains trypotophan,
(d) Neither 1 nor 2 why does it depress brain trypotophan levels? The
answer lies in the mechanism that provides blood
141. Which of the following is a benefit of the Grameen
trypotophan to the brain cells. This same mechanism
system of microfinance?
also provides the brain cells with other amino acids
(a) If a single member is unable to repay a loan,
found in protein, such as tryrosine and leucine. The
other group members will repay it
consumption of protein increases blood concentration
(b) It utilises the vast bank network already
of the other amino acids much more proportionately
existing in a country
than it does that of trypotophan.
(c) Group members can sanction loans and verify
The more protein is in the meal, the lower is the
if borrowers have sufficient collateral
ratio of the resulting blood trypotophan concentration
(d) Backing that borrowers receive from other
to the concentration of competing amino acids, and the
group members
more slowly is trypotophan provided to the brain. Thus
the more protein in the meal, the less serotonin
Passage 29
subsequently produced and released.
It was once believed that the brain was independent
142. According to the passage, one reason that the
@UPSC_THOUGHTS

of metabolic processes occurring elsewhere in the body.


authors gave rats carbohydrates was to
In recent studies, however, we have discovered that the (a) compare the effects of carbohydrates with the
production and release in brain neurons of the effects of proteins
neurotransmitter serotonin (neurotransmitters are (b) depress the rats’ trypotophan levels
compounds the neurons use to transmit signals to other (c) cause the rats to produce insulin
cells) depend directly on the food that the body (d) demonstrate that insulin is the most important
processes. substance secreted by the body
Our first studies sought to determine whether the
increase in serotonin observed in rats given a large 143. The authors’ discussion of the “mechanism that
provides blood trypotophan to the brain cells” is
injection of the amino acid trypotophan might also
meant to
occur after rats ate meals that change trypotophan
(a) provide supporting evidence for a
levels in the blood. We found that, immediately after
controversial scientific theory
the rats began to eat, parallel elevations occurred in
(b) refute the conclusions of a previously
blood trypotophan, brain trypotophan, and brain
controversial scientific theory
serotonin levels. These findings suggested that the
(c) help explain why a particular research finding
production and release of serotonin in brain neurons
was obtained
were normally coupled with blood trypotophan
(d) stimulate further research studies
increases. In later studies we found that injecting
insulin into a rat’s blood stream also caused parallel 144. Which of the following titles best summarises the
elevations in blood and brain trypotophan levels and content of the passage?
in serotonin levels. We then decided to see whether the (a) Amino Acids and Neurotransmitters: The
secretion of the animal’s own insulin similarly affected connection between serotonin levels and
serotonin production. We gave the rats a carbohydrate- tyrosine
containing meal that we knew would elicit insulin (b) Neurotransmitters: Their crucial function in
secretion. As we had hypothesised, the blood Cellular Communication

1 . 40
Comprehension

(c) The Effects of Food Intake on the Production (c) The rats’ brain trypotophan levels would
and Release of Serotonin: Some Recent decrease
Findings (d) The rats would produce more insulin
(d) The Blood Supply and the Brain: A Reciprocal
149. Which of the following can be inferred from the
Dependence
passage to be least likely as a potential source of
145. According to the passage, the speed with which aid to a patient who was not adequately producing
trypotophan is provided to the brain cells of a rat and releasing serotonin?
varies with the (a) Meals that would elicit insulin secretion
(a) concentration of trypotophan in the brain (b) Meals consisting almost exclusively of protein
before a meal (c) Meals consisting almost exclusively of
(b) concentration of serotonin in the brain before carbohydrates
a meal (d) None of the above
(c) concentration of leucine in the blood rather
than the concentration of tyrosine in the blood Passage 30
after a meal
An important feature of the labour market in recent
(d) amount of protein present in the meal
years has been the increasing participation of women,
146. According to the passage, an injection of insulin particularly married women. Many analysts suggest,
was most similar in its effect on rats to an however, that women comprise a secondary labour
injection of market where rates of pay and promotion prospects are
@UPSC_THOUGHTS

(a) blood inferior to those available to men. The principal reason


(b) trypotophan is that women have, or are assumed to have, domestic
(c) tyrosine responsibilities which compete with paid employment.
(d) leucine Such domestic responsibilities are strongly influenced
by social values which require women to give priority
147. According to the passage, when the authors began
to home and family over paid employment.
their first studies, they were aware that
The difficulties which women face in the labour
(a) trypotophan levels in the blood were difficult
market and in their ability to reach senior positions in
to monitor with accuracy
organisations are accentuated with the arrival of
(b) there were many neurotransmitters whose
children. In order to become full-time employees, women
production was dependent on metabolic
with children must overcome the problems of finding
processes elsewhere in the body
good, affordable childcare and the psychological barriers
(c) they would eventually need to design
of workplace marginality. Some women balance domestic
experiments that involve feeding rats high
and workplace commitments by working part-time.
concentrations of protein
However, part-time work is a precarious form of
(d) serotonin levels increased after rats were
employment. Women part-timers are often the first laid
injected with a large amount of trypotophan
off in a difficult economy. These workers are often
148. It can be inferred from the passage that the referred to as the ‘reserve army’ of female labour.
authors initially held which of the following One researcher has found that approximately 80
hypotheses about what would happen when they per cent of women in their twenties who have children
fed large amounts of protein to rats? remain at home. Such women who later return to work
(a) The rats’ tyrosine levels would increase less represent another sector of the workforce facing
quickly than would their leucine levels difficulties. When the typical houseworker returns to
(b) The rats’ brain serotonin levels would not the labour market she is unsure of herself in her new
decrease environment. This doubt is accentuated by her recent

1 . 41
Comprehension

immersion in housework, a very private form of work. 2. The threat of being laid off first in the event
Without recent employment experience, these women of the economy slowing down.
confront a restricted range of opportunities and will 3. Obsolete skills and fewer opportunities.
almost certainly be offered low-status jobs with poor (a) 1 only
prospects. (b) Both 2 and 3
Even women professionals who interrupt their (c) Both 1 and 2
careers to have children experience difficulties. Their (d) 3 only
technical skills may become rusty or obsolete, important 153. According to the passage, men generally receive
networks of business contacts are broken, and their higher salaries and have a better chance of being
delayed return to work may mean that they are likely promoted because women
to come up for promotion well after the age that would (a) are not equally well-qualified professionally
be otherwise normal. Consequently, women, even those (b) tend to take up part-time work
of high ability, may find themselves blocked in the (c) suffer discrimination in the male-dominated
lower echelons of an organisation, overlooked, or even corporate environment
‘invisible’ to senior management. (d) have responsibilities outside the workplace
150. The main concern of the passage is to that demand considerable attention
(a) advocate changes in employers’ practices
towards women and children Passage 31
(b) describe the psychological consequences for Slums are not caused by the poor but by governments
@UPSC_THOUGHTS

women of working outside home denying people the right to own and exchange property.
(c) examine some of the reasons for women rarely When people own their own property they have
reaching the higher positions of paid labour incentives to invest time, money and energy to improve
(d) take issue with those who believe women it because they know that they will be able to benefit
should not work outside the home from any such improvements i.e. the ability to obtain
151. Which of the following statements about women mortgages, etc. In short, property rights beget capital,
workers is supported by information provided in which begets innovation, which begets wealth. Sadly
the passage? the poor typically don’t have secure title to their land
(a) Providing childcare facilities for working as there are bureaucratic restrictions on transferring
women is the responsibility of employers title or there is no clear system for titling. Without legal
(b) The decision to work outside the home is deeds they live in constant fear of being evicted by
often a source of considerable anxiety for landlords or municipal officials. Illiteracy is a major
reason why poor people often choose not to seek the
women with children
protection of local courts since in so many countries
(c) Conditions for working mothers are better
laws established under colonial rule have never been
today than they were twenty-five years ago
translated into local languages. When entrepreneurs do
(d) Women with professional qualifications reach
set out to legally register business they are discouraged
high positions and are easily able to integrate
by red tape and costly fees. The proprietors of such
career and children
businesses cannot get loans, enforce contracts or expand
152. Which of the following is/are not difficulties a personal network of familiar customers and partners.
faced by women who return to work after As a result the poor have no choice but to accept
remaining at home for some time to look after their insecurity and instability as a way of life.
children? In India, severe restrictions on free transfer of
1. Social values that dictate that women should property in most rural areas inhibit investment and
put home and family before career. encourage urban flight. Planning policies, however,

1 . 42
Comprehension

discourage building homes for these migrants as (b) praise government initiatives for migrant slum
numerous homes are destroyed if they do not comply dwellers
with planning rules, essentially forcing people to live (c) convince governments to empower the poor
in slums and perversely blaming it on population (d) enlist the aid of developed countries to tackle
growth. U.N. Habitat, the UN agency for housing the the issue of slums
poor, has implemented more plans to stabilise the 157. Which of the following is true in the context of
unplanned aspects of urban growth but grandiose the passage?
plans like UN schemes and government housing projects (a) Additional UN projects will exacerbate the
simply ignore or worsen the underlying problems. It is plight of slum dwellers
when governments grant people legal means to control (b) Although the government allocates land for
their assets that they empower them to invest and plan them the poor choose not to invest in building
ahead. In Buenos Aires, economists studied the houses
experience of two Argentine communities. One had (c) With the spread of slums populations are
received legal title to its land in the 1980s and surpassed drifting back to rural areas
the other group, which had not, in a range of social (d) The UN has declared the right to housing as
indicators including quality of house construction and a fundamental right
education levels. As the growth of illegal settlements
amply demonstrates, the poor are not helpless; all they 158. What is the objective of the Commission on Legal
need is governments to grant them fundamental human Empowerment of the Poor?
rights of freedom and responsibility. (a) Coerce international leaders to implement
@UPSC_THOUGHTS

housing projects
154. What did the Argentine study indicate? (b) Selecting experts to recommend ideas to do
1. Argentina’s economy is booming and the away with poverty
percentage of poor has fallen.
(c) Establish practical ways for governments to
2. When the government gives people the legal empower the underprivileged
means to control their assets they plan for the
(d) None of these
future.
3. The government succeeded in widening the 159. Which of the following difficulties do unregistered
gap between the rich and the poor. businesses face?
(a) Only 1 1. Banks do not give loans in the absence of
(b) Only 2 security.
(c) Both 2 and 3 2. They are unable to earn the loyalty of any
(d) None of the above customer.
3. They cannot enforce contracts.
155. According to the author, which of the following
(a) Only 1
factors is responsible for the creation of a slum?
(b) Both 1 and 3
(a) Migration of landless labourers to cities
(c) Both 1 and 2
(b) Municipal authorities building low-cost
(d) 1, 2 and 3
housing for the poor.
(c) Unchecked population growth
(d) Government failure to secure property rights
Passage 32
for citizens Manufacturers are currently competing with each other
156. The author’s main objective in writing the passage to produce a form of green packaging. Packaging is an
is to important part of marketing these days, but much of it
(a) exhort the UN to play a greater role in is a threat to the environment. There are two reasons
rehabilitating slum dwellers for this. The production of such packaging uses up a

1 . 43
Comprehension

great deal of energy and the cartons, wrappers, etc are (c) Packaging that produces little waste
often difficult to dispose of when they become waste (d) Packaging that reduces pollutants released in
material. air
People in most countries have become aware of the
161. Which of the following kind of pollution is not
damage which modern living is doing to the
explained in the passage?
environment, and many of them are concerning
(a) Pollution caused by industries
themselves with the conservation of the environment for
(b) Nuclear pollution
future generations. Thus, both politicians and scientists
(c) Pollution caused by plastics
are now looking at the issues of energy-saving and
(d) Pollution caused by vehicles
waste disposal with a view to making them more
environmentally-friendly. 162. Which of the following statements is incorrect in
As far as packaging is concerned, it is vital that it the context of the passage?
is either recyclable or biodegradable. For example, (a) Biodegradable wastes like vegetable peelings
instead of throwing out newspapers and glass bottles are recyclable
with their household rubbish, people in several countries (b) Packaging is a threat to the environment
are being encouraged to put these in special containers (c) Burying plastic would cause land pollution
to allow the material to be recycled. Some household (d) Developed countries produce a large amount
waste, such as vegetable peelings, is naturally of waste material
biodegradable and so decomposes gradually until it 163. Which of the statements given below is/are correct
disappears. according to the passage?
@UPSC_THOUGHTS

Man-made goods are not so easily disposed of. I. When standards of living become high, people
Goods and packaging made of plastic create waste become aware of environmental problems and
material that is particularly difficult to get rid of. This reduce waste material.
means that huge landfill sites have to be dug out so II. Political leaders have not yet understood the
as to bury the plastic waste underground, possibly importance of safe waste disposal.
causing problems for future generations. (a) Only I
Just as much of a problem is industrial waste, since (b) Both I and II
the effluent from factories often contains chemicals (c) Only II
which can lead to the pollution of water supplies. (d) Neither I nor II
Waste from factories has to be monitored carefully in
order to avoid this. 164. What is the meaning of the word ‘green’ as used
Technological advances using nuclear power have in the passage?
added to the waste problem. The disposal of nuclear (a) Ecological
waste causes particular concern because it is radioactive (b) Healthy
and so possibly dangerous to life. (c) Concerned with the protection of the
The high standard of living, which the people of environment
many countries now enjoy, has resulted in a huge (d) Biodegradable
increase in waste material. This could have a terrible
effect on the ecology of the planet. There is no doubt Passage 33
that urgent action must be taken to save our environment
Educational qualifications are considered to be very
from possible disaster.
important in the modern world. They are essential for
160. How would you define green packaging? people who want to find reasonably well-paid
(a) Biodegradable packaging employment in the professions. For this reason, most
(b) Packaging concerned with protection and parents try to get their children to work hard at school
conservation of the environment and achieve academic success by doing well in exams.

1 . 44
Comprehension

Many parental aspirations also include their children (c) Parents are sometimes reluctant to have their
going to university and graduating with a good children opting for their professions
degree. (d) Parents often cause stress in their children
Not all children, however, are capable of achieving 166. What do you think is the main theme of the
academic success. This does not matter as long as passage?
parents are willing to accept this, but it is quite common (a) Parental-children conflict in families
for parents to think that all their children have to do (b) Whether children should follow their parents’
is to study hard and they will pass their exams. All wishes and professions
too often, they just succeed in causing too much stress (c) Parental aspirations
in their offspring, with the result that the children either (d) The many abilities of children
get ill or fail exams that they might otherwise have 167. What is the perspective adopted by the writer in
passed. the passage? That of a
There are some children who are quite bright, but (a) philosopher
who are simply not interested in formal learning. Some (b) critic
might be of an artistic bent and wish to become an artist (c) cynic
or designer while some might have a talent for acting. (d) teacher
Others may show an aptitude for working with their 168. The writer of the passage
hands, or want to start their own businesses and I. has a very high opinion of children who excel
become entrepreneurs. in academics.
@UPSC_THOUGHTS

It is perfectly possible for children to achieve such II. speaks against children taking up their
ambitions. However, their parents may well have other parents’ professions.
ideas, which can lead to family conflict. Strangely III. wants children to get educated so that they
enough, many parents are often reluctant to allow can get well-paid jobs.
children to follow in their footsteps. For example, actors (a) I and II
may not wish their children to have a career in the (b) II and III
theatre, because of the uncertainty of the profession. (c) I, II and III
(d) None of them
Business people may feel that their children will have
more status in an academic profession than in the 169. What leads to the generation gap according to the
world of commerce. writer of the passage?
The opposite situation also arises. Parents who (a) Lack of communication
have worked hard to establish a business may want (b) The young and their parents thinking
their children to become part of it, only to find that their differently on a matter
(c) Different perspectives on education
sons and daughters prefer to look for completely different
(d) Conflict between the academic aspirations of
occupations.
parents and the vocational pursuits of the
Each generation has different ideas, making
children
communication between the two extremely difficult.
Thus, has arisen the aptly named generation gap. 170. According to the passage
(a) actors hardly ever want their children to
165. Which of the following cannot be said with become actors because the profession is risky
reference to the passage? (b) all parents want their children to follow their
(a) The parents’ wishes for their children may own inclinations if they are good at something
contradict those of their children (c) formal education is preferred by parents as it
(b) Parents want their children to achieve more enables people to get into professional jobs
status than what they have (d) all the above are true

1 . 45
Comprehension

Passage 34 Thus passed, as simply as a child, the man whom


the French people were to vote at a plebiscite as the
In spite of all the honours that we heaped upon him, greatest man that France had ever produced. Napoleon,
Pasteur, as has been said, remained simple at heart. who has always been considered the idol of France,
Perhaps the imagery of his boyhood days, when he was placed fifth.
drew the familiar scenes of his birthplace, and the No greater tribute could have been paid to Louis
longing to be a great artist, never wholly left him. In Pasteur, the tanner’s son, the scientist, a man of peace,
truth he did become a great artist, though after his the patient worker for humanity.
sixteenth year he abandoned the brush for ever. Like
every artist of worth, he put his whole soul and energy 171. Why did not even accolades and honours change
into his work, and it was this very energy that in the the simple man that Pasteur was?
end wore him out. For to him, each sufferer was (a) He believed in work and put his whole soul
something more than just a case that was to be cured. into his work
He looked upon the fight against hydrophobia as a (b) Because he remained a simple person at heart
battle, and he was absorbed in his determination to whom the sight of suffering moved to an
win. The sight of injured children, particularly, moved indescribable extent
him to an indescribable extent. He suffered with his (c) He revered the faith of his fathers
patients, and yet he would not deny himself a share (d) He was a man of peace
in that suffering. His greatest grief was when sheer 172. How did Pasteur view those who suffered from
physical exhaustion made him give up his active work. diseases?
@UPSC_THOUGHTS

He retired to the estate at Villeneuve Etang, where he (a) Pasteur believed in work and viewed those
had his kennels for the study of rabies, and there he
who suffered as the goal of his work
passed his last summer, as his great biographer, Vallery
(b) For Pasteur the grief of the suffering was the
Radot, has said, ‘practicing the Gospel virtues’.
work of nature
‘He revered the faith of his fathers,’ says the same
(c) Pasteur viewed the suffering person as
writer, ‘and wished without ostentation or mystery to
something more than just a patient who had
receive its aid during his last period.’
come for a cure
The attitude of this man to the science he had done
(d) Pasteur viewed the suffering as his bread and
so much to perfect can be best summed up in a sentence
butter
that he is reputed once to have uttered, concerning the
materialism of many of his contemporaries in similar 173. How did Pasteur engage himself in the estate?
branches of learning to his own: ‘The more I contemplate I. He conducted a study of rabies.
the mysteries of Nature, the more my faith becomes like II. He practised the Gospel virtues.
that of a peasant.’ III. He actively engaged himself in attending to
But even then in retirement he loved to see his the sick.
former pupils, and it was then he would reiterate his Choose the correct option from those given below:
life principles: ‘Work,’ he would say, ‘never cease to (a) Only I
work.’ So well had he kept this precept that he began (b) Only I and II
rapidly to sink from exhaustion. (c) Only I and III
Finally on September 27, 1895, when someone leant (d) All of the above
over his bed to offer him a cup of milk, he said sadly:
‘I cannot,’ and with a look of perfect resignation and 174. What advice did he always give to his pupils?
peace, seemed to fall asleep. He never again opened his (a) Always attend to the sick and needy
eyes to the cares and sufferings of a world, which he (b) Believe in the Gospel values
had done so much to relieve and to conquer. He was (c) Never stop working
within three months of his seventy-third birthday. (d) None of the above

1 . 46
Comprehension

175. Which of the following is/are mentioned/implied discussion about changes as you experience them. It is
in connection with Pasteur in the passage? an unconditional experience where you receive as
I. A man of peace. much as you give. You can explain yourself to a friend
II. A worker for humanity. openly without the fear of hurting a family member.
III. A spiritualist. How do friendships grow? The answer is simple. By
IV. A person of religious values. revealing yourself; being attentive; remembering what
Choose from the options given below: is most important to your friend and asking them about
(a) Only I and II it; putting yourself in their position; showing empathy;
(b) Only II and III seeing the world through the eyes of your friend, you
(c) Only III and IV will understand the value of friendship. All this means
learning to accept a person from a completely different
(d) Only I, II and IV
family to your own or perhaps someone from a
completely different cultural background. This is the
Passage 35
way we learn tolerance. In turn, we gain tolerance and
The role friends play in our lives has become acceptance for our own differences.
significantly greater than at any other time in our Friendships are made by being considerate which
history. Today many of us live and work great distances means all the communication skills come into play:
from where we were born or grew up and are separated active listening skills, questioning skills, negotiation
from our original families. The pain we feel when we skills, reflecting content skills, reflecting emotion skills
are away from our families can be significant. and editing yourself.
@UPSC_THOUGHTS

The happiness of the individual relies on Friendships offer a great opportunity to learn about
friendships which form a necessary human connection. yourself because a friend can reflect back to you ‘how
It is perfectly normal to need and want friends and you come across in the world’. They also allow you to
depression is more prevalent among those who lack practise skills in dealing with ‘personal boundaries’ by
friends. They lack the intimacy and richness friends can looking after yourself as well as your friend. They help
bring into our lives. Frequently, friends reflect similar you develop resilience in relation to the wider social
values to us. Yet these values are often different from world beyond your family.
the ones we grew up with; they are the values we
176. Why do friends play a more significant role today
created for ourselves in our adult lives.
than ever before?
Communication skills are fundamental in all
(a) Our happiness depends on our friendships
friendships. The more friends and acquaintances one
(b) People go to work at places distant from their
has, the greater are one’s communication skills. Some
families
call these, people skills.
(c) It is friendship that forms the foundation of
Like watering a plant, we grow our friendships
values that help us to grow
(and all our relationships) by nurturing them.
(d) Friends play a positive role in our lives
Friendships need the same attention as other
relationships if they are to continue. These relationships 177. How is friendship different from other
can be delightfully non-judgemental, supportive, relationships?
understanding, and fun. (a) Friendship offers a great opportunity to help
Sometimes a friendship can bring out the positive one learn about oneself
side that you never show in any other relationship. This (b) We are more sensitive and show greater
may be because the pressure of playing a ‘role’ (daughter, empathy in the company of friends
partner or child) is removed. With a friend you are to (c) Friendship helps us to develop our
be yourself and free to change. Of course you are free communication skills
to do this in all other relationships as well but in (d) In friendship one gets the opportunity to be
friendships you get to have lots of rehearsals and oneself and not be constrained

1 . 47
Comprehension

178. What are the essential human values that help composed of inactivated virus of the three types, and
friendship to grow? Select the correct option from the ‘live’ (Sabin) vaccine, composed of genetically
the answer responses given: attenuated viruses of the three types. These vaccines,
I. Communication skills which were introduced in the 1950s, have lowered the
II. Tolerance incidence in developed countries of paralysis resulting
III. Empathy from poliomyelitis. The disease still occurs in developing
IV. Kindness countries and recurs in some developed countries where
(a) Only II vaccination programmes have not been enforced. Rare
(b) Only I and III cases of poliomyelitis occur, particularly in the United
(c) Only II and III States, from the Sabin vaccine strain of type-3 poliovirus,
(d) Only I, II and III which is genetically unstable and occasionally reverts
to the virulent form.
179. Which communication skills help in building
Vaccination can prevent diseases caused by strictly
friendship?
human viruses that exist in only one antigenic and
Select the correct option from the answer responses
stable type. Measles has been prevented in developed
given:
countries with routine vaccination. Measles, however,
I. Listening skills
may still be the major cause of death in children in
II. Questioning skills
developing countries. Vaccination for mumps and
III. Answering skills
chicken pox promises to be successful because the
IV. Editing yourself
causative viruses of these diseases show little tendency
(a) Only I, II and IV
to vary antigenically and are confined to humans. On
@UPSC_THOUGHTS

(b) Only I, III and IV


the other hand, development of vaccines for the common
(c) Only I, II and III
cold caused by rhinoviruses, similar to polioviruses,
(d) All of them
will be a formidable, if not impossible, task because
180. From the passage it is clear that friendship is there are at least 100 antigenic types of the rhinovirus.
(a) a practical necessity Also daunting is the task of developing a vaccine
(b) a human need against HIV. The major antigenic component of this
(c) necessary to understand cultural differences virus is a surface membrane-inserted glycoprotein
(d) unconditional (gpl20) which has a startling rate of mutation. The
extreme antigenic diversity that results from the
181. According to the passage, friendship
mutability of the gene that codes for this protein would
(a) grows only among people of the same cultural
prevent HIV from being identified and attacked by
backgrounds
circulating antibodies or killer T lymphocytes.
(b) expects you to sacrifice a lot of your personal
time 182. Which of the following statements is/are not
(c) often occurs between people who share values correct with reference to the passage?
(d) is all about communication skills I. Measles is caused by a virus that infects only
humans.
Passage 36 II. All vaccines developed against poliomyelitis
have been equally effective.
Vaccines are most successful when directed against III. Vaccines can only be produced for diseases
those viruses that do not mutate and that infect only caused by viruses that exist in one antigenic
humans. In addition to smallpox, a successful vaccine type.
programme has been carried out against poliomyelitis. (a) I and II
Polioviruses exist in only three antigenic types, each of (b) II only
which has not changed significantly for decades. The (c) II and III
vaccines available are the ‘killed’ (Salk) vaccine, (d) I, II and III

1 . 48
Comprehension

183. The Salk vaccine must recognise that violence is an unfortunate breach
(a) is made of viruses rendered less harmful than of community, and devise other ways of establishing
the original satisfactory relationships. Bernard Shaw remarks
(b) is composed of milder types of live viruses somewhere that, in a really civilised society, flogging
(c) is made of genetically unstable viruses that would be impossible, because no man could be
sometimes return to their dangerous form persuaded to flog another. But as it is, any decent
(d) has all three types of viruses in an inactivated warder will do it for a rupee, probably not because he
form likes it or thinks it desirable on penal grounds, but
because it is expected of him. It is obedience to social
184. Which of the following is/are regarded, as per the
expectation. The pity and the sordidness of war lie in
passage, to be the difficulties in producing a
this: that without any evil in us we engage in it, not
vaccine against HIV?
because we are in any way cruel, but because we mean
I. The element in the HIV virus that causes the
to be kind. We engage in wars to save democracy, to
infected body to produce antibodies can very
win freedom for the world, to guard our women and
quickly change genetically.
children, to protect our hearths and homes. At least we
II. The HIV virus does not infect humans only.
believe so.
(a) I only
(b) II only 185. Why is the soldier likened to the hunter by the
(c) Both I and II writer of the passage?
(d) Neither I nor II (a) Both use weapons of destruction
(b) Both have outlived their utility
@UPSC_THOUGHTS

Passage 37 (c) Hunting and soldiering were once necessary


but have now become merely fashionable
Before the domestication of animals the hunter (d) Both love their professions
discharged a social duty by providing food. Today the
hunter is not needed for that purpose; yet hunting is 186. The writer does not contend that
(a) the human tendency is to perpetuate a
fashionable, because hunting for sport has taken the
tradition, no longer useful, on the basis of
place of hunting for livelihood. Even so, when we were
surrounded by raiding barbarians the soldier helped to imagined justifications
(b) war is a manifestation of the love of power
make life more tolerable; but is war essential today?
and success
Man is the only animal who kills for reasons which
are more or less metaphysical, for an obsolete claim to (c) war is necessary in rare and exceptional
circumstances
territory, for a childish passion for a mistress, for
(d) human beings must find other ways than
prestige, for drawing the frontiers at one line and not
another. When an institution is no longer necessary, we violence to settle the problems in their
relationships
invent fictitious reasons for satisfying our acquired
tastes which long habit has produced. War was the 187. What is the core of the writer’s argument against
sport of kings and the game of the upper classes, in human indulgence in war?
which the prizes were wealth and honour. War has (a) Human beings still seek security from
become an end in itself, an exciting game, a vested barbarians through war
interest of financiers. Those who engage in war are not (b) War is resorted to a mechanical way in
bad men who believe themselves to be doing wrong, but obedience to what is expected and not because
good men who are convinced that they are doing right. of inherent cruelty
So long as power and success are worshipped, the (c) Human beings are incapable of forsaking the
military tradition, in its modern form of mechanical pursuit of power and success
inhumanity, will flourish. We must alter our values, we (d) None of the above

1 . 49
Comprehension

Passage 38 188. From the passage it is clear that the writer


I. is against reservation for the socio-
What is the situation today with respect to Ambedkar’s economically deprived classes.
mission of annihilation of caste and, in view of that, II. sees no improvement in the condition of the
the relevance of his manifesto of social emancipation?
scheduled caste population.
First, the state’s reservation policy for the Scheduled
III. feels that literacy and higher education alone
Castes and Scheduled Tribes has made a positive
can lead to improvement in the field of work
impact on their socio-economic condition. Though the
among the scheduled castes.
gap between them and the rest of society persists, and
(a) I and II
they lag behind the others with respect to many
(b) II only
indicators of development, the overall situation has
(c) III only
improved. A small strata in all areas of national
(d) None of them is correct
importance—education, professions, governance,
politics, art, literature and so on—has emerged in these 189. Social emancipation among the scheduled castes,
communities. This upward occupational mobility has according to the writer, has led to
been accompanied by some social prestige, which was (a) entirely positive developments among the
unthinkable earlier. The spread of literacy and higher scheduled castes
and professional education, the pace of urbanisation, (b) the complete destruction of caste
the development of means of communication and consciousness as hoped for by Ambedkar
transport, and so on have been instrumental in loosening (c) the development of caste-based alignments to
@UPSC_THOUGHTS

somewhat the rigidity and hangover of the caste system, achieve political power
particularly in the urban areas. (d) the breakdown of caste differences
Second, and contrary to this positive development,
caste has come to be used blatantly and indiscriminately 190. Which one of the following is correct from a
for political ends. This has sharpened caste and sub- reading of the passage?
caste identities and resulted in caste alliances of different (a) Urbanisation and education have contributed
types in different regions for the sole purpose of wielding to a change in the attitude to the caste system
political power. (b) The marginal castes have entered the centre
Thus, while the dominant castes, either in numerical of the power struggle
strength or in terms of economic clout, struggle to retain (c) The castes which have been powerful so far
their monopoly over power, the marginal castes, have now lost all their importance
operating on the periphery of the power structure, have (d) The dominance of certain castes in the political
aroused their caste consciousness for political and the economic spheres has been completely
mobilisation. destroyed

1 . 50
Comprehension

ANSWERS WITH EXPLANATORY NOTES


PASSAGE 1 7. (b) The passage clearly says that trade began
1. (c) The question refers to signals, signs and from person to person, so option (a) is not
symbols. The very first sentence of the correct. Option (b) is supported by the lines:
passage tells you all three are forms of ‘The knowledge … other lands.’ Option (c) is
‘communicating without using speech’. Option neither mentioned directly nor implied in the
(c) says ‘non-verbal’, and is correct. However, passage, though you may know it to be
you have to read the passage to see if factually correct. Option (d) inverts a statement
anything else has been said about the three made by the passage which says that
collectively. The passage says that they are specialisation led to the improvement in the
found in ‘every culture’. Option (a), however, quality of goods.
mentions ‘primitive’—which is not what is said
PASSAGE 3
in the passage. So (a) is wrong. Option (b)
describes symbols only, so cannot be the 8. (d) Verify each statement. Statement I is
correct choice. The passage does not say that contradicted by the passage: ‘This happens
all are easy to interpret; so (d) may be crossed despite the child’s food requirements being
out. less than that of an older child.’ Statement
II is contradicted by the passage which says
2. (b) The passage implies this clearly in the lines
that, though malnutrition is ‘attributed’ to
about symbols. The other options are not
poverty, it also occurs in households where
@UPSC_THOUGHTS

found in the passage in relation to symbols.


enough food is eaten by adults. Statement III
3. (b) Refer to the sentence ‘In some cultures …
is not to be found in the passage. So all three
auditory symbol of approval’.
are wrong.
4. (d) You need to check the correctness of three
9. (b) The answer is directly stated in the sentence:
statements. All three statements are supported
‘The child’s dependence … malnutrition.’
by the passage.
10. (d) The last sentence of the passage supports this
PASSAGE 2 option, even though the other options may be
5. (b) Three statements are to be checked. So the factually correct.
passage needs to be read well. Statement I 11. (d) You need to verify the two statements. You
may be factually correct, but the passage does will find neither statement is supported by the
not say so; the passage only mentions that passage. So (d) is the correct answer response.
merchants grew rich from trade. Cross out I.
Statement II is supported by the sentence: PASSAGE 4
‘People … choose’. Statement III is implied: 12. (d) This is a ‘purpose of the passage’ question.
Trade brings knowledge about new products Reading the passage shows that the difference
and curiosity about life in other countries, this between two perspectives on renewable and
in turn leading to a desire for better conditions. non-renewable materials has been given. Option
So two statements—II and III—are correct. (d) is correct. Not much has been said to
Answer response (b) is correct. differentiate between the two types of materials;
6. (d) The key word in the question stem is ‘best’— cross out (a). While the modern economist’s
not just ‘describes’ but ‘best describes’. While concern with costs is stated, it is not the
options (a), (b) and (c) are all true of the purpose of the author. Cross out (b). The
effects of trade as described in the passage, Buddhist concern for conservation is mentioned,
the ‘best’ is (d), as it sums up the effects but the passage is not about conservation of
of trade. natural resources. Option (c) is not correct.

1 . 51
Comprehension

13. (d) The answer is found in the sentence: ‘Non- 18. (c) Nothing in the passage supports options (a)
renewable goods must be used only if they or (d). An increase in the level of education
are indispensable …’; i.e., when an alternative of women—option (b)—would probably lead to
is not to be found. The other options are not higher income; it is not an effect of higher
supported by the passage. white-black women income ratio.
14. (b) While option (a) may be indirectly true, option 19. (b) The answer is to be directly found in the last
(b) is more specific, and is directly mentioned line of the passage.
in the passage: ‘To use … act of violence 20. (b) Check both statements. There is nothing in
…’ The other options are not mentioned or the passage saying black women suffer more
implied in the passage. than black men; in fact the passage links both
15. (b) The three statements have to be verified. black men and black women as victims of
Statements I and II find no support in the racial discrimination. Cross out I. Statement
passage. So (b) is your answer. Statement III II is correct, as the passage makes the point
is stated in the sentence: ‘… while complete that women suffer discrimination—obviously
non-violence may not be possible on this compared to men. Answer response (b) is
earth…’ correct.

PASSAGE 5 PASSAGE 6
16. (b) The passage provides data to show that racial 21. (a) When you read the passage, it strikes you that
discrimination ‘adds little to the effects of sex it is about orangutans and how their beheaviour
discrimination’ and that a black woman does tells us something about human ancestors.
@UPSC_THOUGHTS

not suffer more than a white woman in the Option (a) tallies with this reading (The very
job market. It observes that compared to the first sentence and the last sentence of the
widely existent effects of sexual discrimination, passage support this conclusion.) While the
the effects of racial discrimination are reason for primates leaving their forest dwellings
‘negligible’. This is the main idea of the is given in the second sentence of the
passage. There is no historical account of paragraph, the passage does not ‘analyse’ the
abuse in the labour market, so option (a) can reasons for this move. Option (b), therefore,
be crossed out. Though a mention is made is not correct. Options (c) and (d) are not
of how education levels differentiate black supported by the passage.
women and white women, it is in the context 22. (b) You need to check the correctness of all three
of observing the effect of racial discrimination; statements before choosing your answer
the main concern of the passage is not what response. Statements I and II can be drawn
is stated in option (c). Option (d) is not correct from the second paragraph: ‘Incorrectly
as the passage does not give the ‘effects’ of relocating … from an outsider.’ The newcomer
sexual discrimination, merely giving data to is seen as a threat to the territory of the earlier
prove that such discrimination exists. inhabitants, and to the females in the
17. (c) As you read the passage you note the author community. Read along with the last sentence
considers an issue objectively, providing of the first paragraph: the suggestion of
supportive evidence of the idea presented. It ‘guarding’ over the females and a conflict
is also critical of the assumption that black about the available females may be understood.
women suffer more discrimination on the job So I and II are correct. Nothing in the passage
market because of being black, and points out supports statement III. So answer response
that sexual discrimination is more intense than (b) is correct.
the racial kind. So (c) is the correct choice. 23. (d) In this kind of question which is rather general
There is no bias in the tone, nor is the tone and not specific, it helps to have a look at
tentative or overbearing. the options first. You may then use the

1 . 52
Comprehension

elimination method. You may cross out (a); ‘The economic activity… growth for decades’
the passage clearly says that all primates in the third paragraph. Cross it out. The last
except humans spend some time dwelling in sentences of the paragraph confirm statement
trees. (You should know an ape is a primate.) II. Choose answer response (b) as correct.
Option (b) may be crossed out as it is 31. (d) Check out the given statements. The first
contradicted by the first sentence of the sentence of the last paragraph contradicts
second paragraph. Chimpanzees are compared statement I. The last sentence contradicts
to orangutans in the matter of sociability; the statement II. So neither statement is supported
passage, however, makes no comparison of by the passage. Choose (d) as your answer.
the effects of size. Cross out option (c). Option
(d) is supported by the fact that orangutans PASSAGE 8
are ‘solitary’ animals.
32. (a) Check the statements for their correctness.
24. (d) The answer is provided by the first and last The first sentence of the second paragraph
sentences of the passage. Options (a), (b) and supports statement 1. As for statement 2,
(c) are not supported by the passage. ‘impromptu’ means ‘spontaneous’, ‘produced or
25. (b) The orangutans develop their territoriality in done without care or planning’; nothing in the
association with availability of food. In addition, passage supports the statement even as many
they are possessive of the females because sentences call for good preparation: ‘The art
they breed only every few years. So (b) of effective… practice’, ‘A thorough
corresponds to this idea. The other options are preparation…’, ‘demands thinking’, ‘One may
not supported by the passage. consult… notes’, ‘No perfunctory approach
@UPSC_THOUGHTS

should even be resorted to…’ We should be


26. (b) The sentence ‘It has long … dwindle.’
careful not to confuse ‘oral’ presentations with
(paragraph 1) supports this option. The other
‘impromptu’ presentations. Only statement 1 is
options are not supported by the passage.
correct. So your answer choice should be (a).
PASSAGE 7 33. (b) Check both statements for their correctness.
27. (d) Check if the given statement is correct. The last sentence of paragraph 2 contradicts
Statement I is contradicted by what is said statement 1. Statement 2 is supported by the
at the end of paragraph one. Statement II is sentence. ‘One may consult… presentation’ in
contradicted by what is said in paragraph paragraph three. (There may be some confusion
three—‘This confidence… thrift.’ So, neither here as the passage says ‘may’ and the
statement is correct in the context of the statement uses the more inflexible ‘should’.
passage. However, we may risk marking (b) as the
correct answer response.)
28. (d) You need to remember India’s total population
figures (approximately 1000 million) and do 34. (d) Statement 1 contradicts what the passage
some calculations, if you want to answer this says in paragraph three: ‘A presentation should
question. Paragraph 2 tells you half the be persuasive…’ Statement 2 is contradicted
population is under 25, and a third is under by ‘Anecdotes, quotations… interesting’ in the
15 years of age. So the population in the 15- same paragraph. So neither statement is
25 age group would be ( 12 of 1000 million) – correct.
( 13 of 1000 million) which is approximately 180 35. (d) Both statements are contradicted by the
million. opening lines of the third paragraph.
29. (b) This is straightforward. The third paragraph 36. (a) Statement 1 is supported by the sentence, ‘A
begins with a reference to the ‘service industry thorough… nervousness’ in paragraph one. As
boom’. for statement 2, there is nothing in the
30. (b) Check if the given statements are correct. passage suggesting that the confidence of the
Statement I is contradicted by the sentence speaker is seen as arrogance by the audience.

1 . 53
Comprehension

On the other hand, ‘Speaker’s… cannot’ 41. (b) Statement 1 is contradicted by the research
suggests something different. So response (a) findings reported in the passage. Trucks and
is correct. buses—heavy vehicles—form a small proportion
37. (a) You have to draw an inference here. It is of the traffic; their involvement in accidents
practice and effort and use of various features— is disproportionately large. Statement 2 is
quotations, anecdotes, interesting topic—that supported by the passage as a measure to
make a presentation persuasive enough to reduce risk. So (b) is your answer.
change the audience’s attitude. So, it would 42. (a) The passage analyses the state of road safety
be correct to say that ‘persuasive skill-set’ is in India and various aspects of road usage
a prerequisite to an effective presentation. So and measures to mitigate risk. It is critical of
statement 1 can be considered correct. There government apathy.
is nothing in the passage to support statement
2. Answer response (a) is correct. PASSAGE 10
43. (d) The answer is obvious. It is also supported
PASSAGE 9 by the last paragraph or the passage.
38. (c) The first few lines of the passage give you 44. (b) The passage clearly calls it ‘cement’.
the answer. While (b) may also seem correct,
45. (b) The options have to be read alongwith the
it is something one has to assume—more
question for this item. It is stated in the
fatalities and injuries could indicate more
passage (paragraph 3) that it could save
accidents; however, the passage mentions the
electricity in daytime; so it can be inferred that
mounting fatalities and injuries from traffic
@UPSC_THOUGHTS

it could be environmentally beneficial. Option


accidents, not the increase in the accidents.
(a) is contradicted in paragraph 3: the material
Option (a) speaks of ‘larger’ roads—not
has so far been used only for one building.
mentioned in the passage, and, any way, it
Option (c) is incorrect as the passage clearly
is not an answer to the question. Option (d)
says the use of resins is better than use of
is not an answer either.
optical fibres (paragraph 5). Check out paragraph
39. (c) The statements have to be checked for their 4 and you see that option (d) is wrong. So
correctness. Statement 1 is correct—‘the road (b) is your answer.
safety initiative … to cover 300 km … in
46. (a) The statements need to be checked for
Assam, Gujarat and Karnataka …’ The same
correctness. Statement I is corroborated by
sentence supports statement 2. So 1 and 2
paragraph 5. There is nothing in the passage
are correct. Towards the end of the first
about not letting heat escape from a room.
paragraph, the author comments that nothing
So only statement I is correct.
has been done to implement the Sundar
Committee recommendations; so statement 3 PASSAGE 11
is not correct. The correct answer choice is
47. (d) If you read the passage, the main topic will
(c).
strike you: the dangers posed by microwaves
40. (c) The options have to be read alongwith the to health. Check the options. While
question in this item. The passage calls for electromagnetic radiation, X-rays and gamma
zero tolerance on highways and urban roads, rays are mentioned in the passage (paragraph
whereas ‘all kinds of roads’ would include rural 1), this is just an introductory thought to
roads as well. So (a) is wrong. As for (b) the develop the main idea. Options (a) and (c) may
loss is to the tune of ` 75,000 crore annually; be eliminated. Though two scientists are
so (b) is not correct. Option (c) is directly mentioned more than once, their work is never
supported by the passage, so it is correct. compared. So (b) too is not the correct option.
Option (d) naturally cannot be correct after (c) You are left with (d)—none of the other options
has been considered. is correct.

1 . 54
Comprehension

48. (c) The options need to be read first in this kind 53. (c) The last sentence of paragraph 1,
of question-item, as the question stem could ‘Acquisitiveness … pride’ supports the option.
lead to anything. Option (a) may be eliminated, Option (a) is what man has become—also
as in paragraph 3, you read that the because of acquisitiveness and pursuit of
‘methodology’ of the two scientists has been power. Option (c) gives the reason asked for
‘questioned’. The same sentence helps you to in the question. The other two options are not
eliminate options (b) and (d). The passage supported by the passage.
says nothing so categorical about anything 54. (d) The other options are all true in the context
being confirmed—as in option (b). Option (c) of the passage.
is correct, not just because it is the only one
55. (c) The last sentence of the passage supports this
left but because the passage supports it in
option.
paragraph 2.
49. (c) Check each statement. There is nothing in the PASSAGE 13
passage to conclude that the studies were not 56. (c) The passage is certainly about blood, but that
scientifically valid; it is just stated in paragraph is a very general term. The first paragraph
3 that the methodology was ‘questioned’. suggests a detailed treatment; the topics
Statement I is wrong. Take statement II. mentioned are blood, its characteristics,
Nothing in the passage supports it, so cross functions and components. Option (a) is too
it out. You are left with statement III. If you general. The passage does not talk about
now look at the options, you find you can blood circulation; option (b) thus goes beyond
eliminate not only (a) and (b) but also (d)— the scope of the passage. Option (d) may
@UPSC_THOUGHTS

as it contains I which is wrong. So option (c) seem correct but the passage does not just
must be correct. The passage supports it in discuss functions; it also describes the
paragraph 3 with the words ‘preliminary findings’. components and the functions of those individual
50. (d) The reference to ham radio operators in components. So (c) is the best answer
paragraph 2 is in the context of Milham’s response.
study. The finding was the presence of higher 57. (d) The answer is pretty obvious. (Refer to the
levels of myeloid leukemia. A look at the first sentence of paragraph 2.) According to
options leads you to (d). Check the other the passage, plasma is a solvent for ions and
options quickly: (a) and (b) refer to Cleary’s molecules, but it is not stated to be composed
study which had nothing to do with ham radio of ions and molecules; eliminate option (a).
operators; option (c) could confuse you, but Option (b) does not refer to plasma, and option
careful reading shows that the passage (c) is also wrong.
mentions levels of radiation slightly higher 58. (b) Paragraph 3 gives the relevant information. It
than, and not identical to, those emitted by is a little tricky: you might assume that oxygen
cell phones. is responsible for the red colour. However,
oxygen is said to bind with haemoglobin. The
PASSAGE 12 specific mention of ‘iron-containing’ pigment
51. (c) The passage speaks for ‘the free and creative make (b) a better choice. Option (c) is not
unfolding’ of human beings as the ‘true measure correct.
of the worth of any society’. The ‘need for a 59. (c) The words given in the question are immediately
new renaissance’ also implies the humanistic associated with protection and are for the white
plea by the author. The other options are blood cells. Option (a) is not implied in the
obviously wrong. context of the passage. As for option (b),
52. (b) The very first sentence suggests the answer. phagocytes are mentioned in the context of
The other options refer to those features that red blood cells, and the given words are used
the author wants humans to develop. for leukocytes or white blood cells. So option

1 . 55
Comprehension

(b) is not acceptable. White blood cells may supports this. The second sentence of the
be considered ‘violent’ from the invader’s point same paragraph mentions that the ‘process’
of view, but they act only when provoked. may also take place in the next life, so option
Option (d) may be rejected. (a) is not valid. Option (b) is not stated by
60. (c) Statement 1 is supported by paragraphs 5 and the passage which says ‘human civilisation
has survived over centuries’; even if we take
7 respectively. Statement 2 is supported by
the statement as implied by the passage, it
paragraph 4. So both statements are correct.
is not in the context of the theory of karma.
PASSAGE 14 So option (b) is not acceptable. While option
(d) is stated by the passage, it is not in the
61. (c) The passage clearly says so in the first
context of karma.
paragraph. It is only two men who decide to
try and get the ship to float, so option (b) does 67. (d) This is supported by the sentence, ‘This way,
not give the reason asked for in the question the result … species’ in the third paragraph
stem. Options (a) and (d) are irrelevant. of the passage.

62. (d) Options (a) and (b) are what the men did after 68. (d) All the given statements are supported by the
they decided that the ship would float if air third paragraph of the passage.
was pumped into the tanks. Option (c) is what 69. (d) The beginning of the second paragraph supports
they did to plug the hole in the side of the this.
ship. Option (d) is correct. 70. (b) The passage only supports statements 1 and
63. (c) Option (a) seems correct, but the last sentence 2.
@UPSC_THOUGHTS

of paragraph 2 tells you that rough seas made 71. (b) The third paragraph has the sentence,
it ‘difficult’ for the divers, not ‘dangerous’. ‘Negativity is therefore a dysfunction of thought
Having to contend with sharks can certainly or outlook’. This supports option (b). The other
be considered a ‘danger’. So (c) is the better options are not supported by the passage.
option, while option (b) is mentioned as a fact
in the passage, and not associated with PASSAGE 16
danger. Option (d) is not mentioned in the 72. (d) When you read the passage, it might strike
passage. you that it is pointing out the inadequacies of
64. (b) The feeling of pride is associated with the the financial markets with regard to minorities.
accomplishment of the task when everyone Option (a) is contradicted by what is said in
thought they would fail. The time taken was the second paragraph: the problem affects the
three months, certainty, but the passage does available alternatives. Option (b) is contradicted
not give an impression that the time period in the third paragraph: the given statement is
is significant except to indicate how long it said to be ‘assumed’, not existing as a fact.
took to set the ship afloat. As for option (c), the passage is itself critical
in that it points out the lacunae in the financial
PASSAGE 15 markets; it does not say that the financial
65. (a) The last sentence of the second paragraph market is subject to criticism—that too only
supports this. The passage also says that because of ‘transaction costs’.
there is no ‘negative energy’, so option (c) is 73. (b) Option (a) mentions ‘subsidies’—nowhere
not valid. The passage says that positive mentioned in the passage. Options (c) and (d)
values get corrupted in the course of practice contradict what is said in the passage.
because of negativity, so option (d) is not 74. (a) The passage calls the system ‘biased’, but
valid. While option (b) may be understood, it says nothing about market analysts being
is not given as the reason for the survival of ‘unwilling’ to accept criticism of their methods.
human civilisation. It is more like the market analysts being
66. (c) The first sentence of the third paragraph unaware of bias. So option (b) is not correct.

1 . 56
Comprehension

The fact that the passage says that 82. (c) The passage does not say that the answers
‘conventional financial analysis pays limited to a scholar’s question have no consequence;
attention’ to many issues makes it likely that only that consequence does not determine the
imbalances do appear in the statistics but they search for answers. So option (a) is not
are ignored. So option (c) is not correct. As acceptable. The word ‘selfish’ gives a negative
for option (d), the passage does not support quality to the scholar which the tone of the
it. passage does not support. Furthermore, the
75. (b) Options (a) and (c) are stated by the passage, passage says that the consequences of the
but not as the analysts’ views. Option (d) is scholar’s search for answers may be
not supported by the passage. ‘enormous’. Option (b) draws conclusions that
the passage does not imply. Option (d) is
76. (a) Options (b) and (d) would affect all equally.
wrong for similar reasons as (b).
Option (c) cannot be called a transaction cost.
77. (b) Paragraph 3 lists the ‘assumptions’ of the PASSAGE 18
analysts. Option (b) is implied in the last
83. (d) Each statement has to be checked. Statement
sentences of the passage. Options (a) and (c)
1 is contradicted by the last sentence of the
are not ‘questionable’ assumptions. As for
first paragraph. Cross it out. Statement 2 is
option (d), ‘credit worthiness’ is mentioned as
negated by the sentence, ‘However, increasing
a variant market strength but it is again not
…’ in the first paragraph. Statement 3 sounds
a ‘questionable’ assumption of the conventional
almost correct, but read it carefully; the term
theory about financial markets.
‘should be’ is different from what the passage
@UPSC_THOUGHTS

PASSAGE 17 conveys. The sentence ‘Nevertheless …


employed’ in the passage uses the word ‘are’
78. (d) The passage says ‘seem’ to the world to be
to indicate what exists, not what ‘should be’.
worth asking or answering, so option (a) is not
Cross out statement 3. So (d) is the correct
acceptable. Option (b) is way off the mark.
answer response.
Of options (c) and (d), the latter is more
precise and correct in the context of the 84. (b) The idea is supported by the sentence, ‘But
passage. increasing … resource’ towards the end of the
first paragraph. The other options are not
79. (d) Options (a) and (c) are easily rejected as
supported by the passage.
frivolous. Option (b) are the type of people
invited to join the university. Option (d) provides 85. (d) Here you need to find the option that is not
the aim of the university which is what ‘that’ supported by the passage. Options (a) and (c)
refers to. are mentioned in the first paragraph. Option
(b) will obviously increase an employee’s
80. (b) Option (a) is too general. The sentence, ‘He
earnings, and is mentioned in the second
talks that way … respects utility’ in the second
paragraph. Option (d) finds no mention as a
paragraph supports option (b). Option (c) may
way to increase employee earnings. Capital
be acceptable, from our knowledge of humans,
investment is mentioned as being reduced if
but is outside the scope of the passage.
Option (d) is not what the passage says. profits are reduced to increase employee
earnings.
81. (c) The writer compares the scientist to the
mountain climber. The word ‘similarly’ may 86. (c) Check each statement. While you may feel
mislead you into choosing option (a), but the statements 1 and 2 are correct in themselves,
scientist is not being compared to the historian. they are outside the scope of the passage;
It is the comparison of scientist to mountain they are not mentioned anywhere in the given
climber that is being paralleled in the historian passage. Cross them out. Statement 3 is
and his motive for studying the past. The other supported by a sentence in the second
options are irrelevant. paragraph: ‘According to … desired rewards.’

1 . 57
Comprehension

87. (d) The other options are not supported by the mentioned as what ‘the people were striving
passage. Option (a) would not have an ‘adverse’ for’—in fact something of a problem—and not
effect. Nothing in the passage says anything as an objective in the development of
mentioned in (b). Option (c) is contradicted by Restoration thought, so the option is not
the beginning of the second paragraph. correct.
88. (d) Here you have to check which statement is 94. (c) The first paragraph gives the answer.
not supported by the passage. The last 95. (d) This is clearly stated in the fourth paragraph.
sentence of the first paragraph supports
96. (d) The other options are not supported by the
statement 1. So cross it out. Be careful not
passage.
to mark (a)—because it is correct. In the
second paragraph, we are told that many 97. (c)
elements ‘prevent’ management from relating
financial rewards to performance. So to say PASSAGE 20
management is free to do so is wrong: 98. (c) The passage as a whole gives reasons for the
statement 2 is not true. The first sentence necessity of state intervention in the
of the second paragraph contradicts statement industrialisation of many less developed
3; so statement 3 is also not true. Option (d) countries. Option (a) is not supported by the
is thus the correct answer. passage. Option (b) introduces the idea of
89. (a) The other options are not supported by the ‘outside assistance’—not found in the passage.
passage. Option (d) goes beyond what the passage
says.
90. (c) Statements 1 and 2 do not get a mention in
@UPSC_THOUGHTS

the passage. Statement 3 is implied in the 99. (d) Here you need to look for a reason not
sentence, ‘However, increasing employee mentioned in the passage. Option (a) is
earnings … in the market’—in the first paragraph. mentioned in the second paragraph, option (b)
The idea is reinforced later by the sentence, in the fourth, and option (c) in the first. While
‘But, increasing … unlimited resource.’ option (d) may be factually correct according
to your knowledge, it is not mentioned in the
PASSAGE 19 passage.
91. (d) There are references in the passage to 100.(a) Check each statement. Statement I is supported
‘Confucian’ or that associated with the Chinese in the first paragraph. Statement II is supported
philosopher, Confucius. Tung-chin and Taiping in the third paragraph. There is nothing in the
do not appear to be philosophers from the passage to support statement III. So statement
passage. Ching is not mentioned. III is not correct. Answer response (a) is
correct.
92. (b) The last sentence of the first paragraph gives
the point of agreement. The rest of the 101.(b) The phrase quoted in the question is explained
paragraph clarifies that both wanted to preserve in the third paragraph; it explains how economic
their own tradition. Option (a) is true as a fact, growth in subsistence-level economies tends
but it does not show the similarity between to lead to population growth which negates the
the two conservatisms. Options (c) and (d) are effects of economic growth. Option (b) is the
contradicted by the passage. most precise and correct answer.
93. (c) Option (a) is contradicted by the sentence, 102.(c) You need to see if the statements contradict
‘Restoration statesmen … on truth’ (paragraph the author’s arguments. Statement I contradicts
2). There is nothing in the passage that speaks the author’s argument in the first and third
of the superiority of one conservatism over paragraphs that only the state can raise the
another; so option (b) is not correct. Option amount of capital necessary to launch large-
(c) is supported by paragraph 2; so it is the scale industries. Statement II contradicts the
correct answer response. Option (d) is author’s contention in the first paragraph that

1 . 58
Comprehension

transportation and communications facilities statement 1 may be crossed out. Now look
‘do not lend themselves to small-scale at the responses; you may eliminate any
improvements’. So both statements are valid; response that has 1 in it. Cross out (a) and
answer response (c) is correct. (d). Now take statement 2; it is not supported
by the passage which says technology could
PASSAGE 21 surely ‘provide a better, cheaper and more
103.(d) Statement I is not quite correct: the passage efficient solution’ to the current vehicles of
talks of ‘serious shortages’, not ‘unavailability’. choice—in the third paragraph. The terms ‘low
Cross it out. Statement 2 finds no mention cost and non-polluting’ are not used in the
in the passage. Cross it out. Statement 3 context of the ‘current’ or ‘present’ vehicles
appears correct at first sight. However, if you of choice. Statement 2 is, therefore, not true
have read the passage, you would have noted in the context of the passage. With response
that the passage mentions lack of (b) that says ‘only 2’, it is most likely to be
‘acknowledgement’, not ‘awareness’. There is the correct response. For your own satisfaction,
enough in the passage to show that awareness you may check out statement 3; it is clearly
about importance of technology in agricultural confirmed in the first paragraph. So cross it
sector is not lacking, but other factors prevent out.
the development of technologies. The fact that 108.(c) Options (a) and (b) are contradicted in the
the Green Revolution was ‘powered by scientific passage in the second paragraph. The last
research’ shows the awareness of the sentence of this paragraph supports option (c).
importance of technology in the agricultural Option (d) is outside the scope of the passage.
@UPSC_THOUGHTS

sector as does the fact that there are 109.(d) Check each statement. Statement I is not
demonstration farms where science and supported by the passage—see the third
technology are applied. So statement 3 is not paragraph: the technology of mobile phones is
quite correct. Answer response (d) is the best ‘proven’, so not ‘largely untested’. Nothing in
choice. the passage mentions ‘opportunities … to
104.(b) You have to look for an option that is not manipulate commodity prices’. So statement
supported by the passage. Options (a), (c) and 2 also is incorrect. Now statement 3 is largely
(d) are mentioned in association with the Green true, but only if you take into consideration
Revolution in the first paragraph. Option (b) is knowledge outside the passage. The passage
not correct because the Green Revolution itself does not mention the fact. So statement
brought about a wheat surplus—something that 3 also has to be rejected. Option (d) is correct.
would not have happened if the application of 110.(d) Here again you have to check if any statement
scientific research had been confined to is not supported by the passage. Statement
demonstration farms. 1 is supported by the passage, so cross it
105.(d) The answer to the given question is to be out. Statement 2 is contradicted by the
found in the second paragraph in the sentence, passage which points out that there is an
‘Technologies that reduce power … subsidised.’ ‘inadequacy’ of technology and that more
Options (a), (b) and (c) are either contradicted technological solutions are required for the
by or not mentioned in the passage. rural economy. Now look at the options. Option
(a) is not correct. As option (c) has 1 in it,
106.(d) The other options are not supported by the
it too cannot be correct. Option (b) says ‘only
passage.
(2)’; this could be the correct choice. However
107.(a) You have to check which of the statements option (d) has 2 and 3; so you need to check
is or are not supported by the passage. statement 3 as well. The passage does not
Statement 1 is supported by the second mention ‘fluctuating power supply’. So it too
paragraph. The Green Revolution was way qualifies as ‘not a threat’. You need to choose
back in the 1970s—not in recent times. So (d) as the correct answer response. Remember,

1 . 59
Comprehension

you should not go beyond what the passage Statement IV is supported by paragraph 4. To
states. screen a person is to get information to check
111.(d) Check each statement. All the statements are his/her suitability for something. Fingerprints
supported by the passage—1 and 2 in the may be checked with police records, for
second paragraph and 3 in the third paragraph. instance, to see if a candidate has a criminal
So all the statements are true. record. So all statements are correct.
117.(a) Refer to the first paragraph: ‘So the … person’.
PASSAGE 22
118.(c) Refer to the last paragraph, first sentence. A
112.(c) The question requires a look at the options— signature authenticates a document, and the
which need to be carefully understood. Option thumb print does the work of the signature.
(a) is not correct. The first sentence of the
119.(c) Refer to the second paragraph.
first paragraph uses the words ‘is often
presented as a magic solution or universal PASSAGE 24
panacea’—which is not the same as an
120.(d) The best answer is (d). The passage mentions
endorsement of genetically modified crops
underdeveloped countries, not underdeveloped
being ‘universally recognised as a solution to
people, so answer response (a) is not
poverty…’ Option (b) is contradicted in the
acceptable. Nothing in the passage supports
second paragraph—which questions the
response (b). The passage does say that
existence of a deficit and lists other problems
according to some illiterate people would not
not addressed by biotechnology. Option (c) is
know how to vote, but this is the reason given
close to what is said in the third paragraph.
@UPSC_THOUGHTS

for the unsuitability of the Western parliamentary


Because of the effective pest resistance they
system to the underdeveloped countries.
have, these crops reduce pesticide use and
Response (d) gives the basic reason for
give improved yield. Even here, however, the
democratic governments in Asian countries
writer uses the words, ‘it is suggested’, rather
giving way to authoritarian rule.
than a categorical assertion. Let us consider
option (d); it is clearly not correct, as there 121.(a) Responses (b) and (d) are not supported by
is no mention in the passage of ‘scientists’ the passage. It is because of incompetent men
dumping anything in the market. The best elected to power through popular elections that
answer response, therefore, is (c). misgovernance occurs, so (c) is not as good
a response as (a).
113.(d) The answer is clear from the third paragraph.
122.(c) The passage does not refer to critics of
114.(b) Check each statement. Statement 2 is
democracy in all countries, but only to critics
supported by the second paragraph. Statements
of the democratic government as practised in
1 and 3 are outside the scope of the passage.
underdeveloped countries – a model that does
So the correct answer response is (b).
not suit them.
115.(a) The author states a fact; he does not question
123. (b) Response (a) is not correct because, according
the ‘morality’ of the dumping countries but
to the passage, it is the ‘pace’ of socio-
makes a point about the low regulatory standard
economic development that is slow; the
in the developing countries. The other options
development is not mentioned to be absent.
are supported by the passage.
124.(b) Refer to the second paragraph: ‘What they say
PASSAGE 23 is no doubt . . . best friend’. As for response
(a) the writer presents the view of critics, and
116.(d) Statement I refers to the ridges on one’s
is not himself (or herself) a critic of democracy.
fingertips, so it is correct. Statement II is
correct; refer to ‘These ridges … person’s life’. 125.(c) A democratic government, according to the
Statement III is supported by the last sentence passage, can be overthrown peacefully whereas
of paragraph 1 (Foolproof = error free). a dictatorial one can only be overthrown by

1 . 60
Comprehension

a violent revolution. This is an advantage of is not acceptable; the passage merely says
democratic regimes. So statement I is correct. that new adaptations take place because of
Another advantage is that people can raise environmental changes, and nothing about how
their voice against the injustices of a democratic drastic these changes are or are not.
government whereas under a dictatorship people 132.(b)
have to suffer silently. Statement II is correct.
133.(d) This is contradicted by the sentence, ‘The
The passage says that there is no guarantee
human biological . . . determined.’
that an alternative form to the democratic one
is more efficient; this is not a categorical
PASSAGE 28
assertion that a dictatorship is more inefficient
that a democratic regime. So statement III is 134.(d) This is the best option. Refer to the third
not correct. paragraph, ‘But enticing … easy.’ Option (a)
uses the words ‘making a profit at any cost’
PASSAGE 25 which is not what the passage says. The other
options are not supported by the passage at
126.(b) Statement I is not correct. The tribesmen are
all.
‘thrown into a state of panic’, meaning they
are worried and fearful. There is a ‘boom’ 135.(c) Refer to the third paragraph, ‘9 million … shops
(sudden increase) in the rat population. So for financing’. Tapping this section remains
statement II is correct. Grameen’s reason for launching itself in the
US a second time. Options (a), (b) and (d)
127.(c)
are not supported by the passage.
@UPSC_THOUGHTS

PASSAGE 26 136.(c) The passage leads to the inference that


128.(b) This is the correct response. Having licences microfinance can be profitable in developed
implies control of industries. countries. See the sentence, ‘Grameen America
believes … turn a profit …’ in the third
129.(d) The writer clearly indicates that such fixed
paragraph, and the reasons offered later in the
beliefs such as everything must be left to the
paragraph. The passage says that microfinance
market does not work. The passage says that
has become a financial staple in the ‘developing
the market’s limitations need to be recognised,
world’ which cannot lead to the inference that
and disagrees with the notion that individual
it has been successful only in Asian countries.
rationality is always efficient or works to the
So option (a) is not correct. The passage says
advantage of society. The illustration given by
the US has 1,00,000 bank branches—which
the writer of the chaotic conditions of traffic
cannot lead to the inference that America has
if left entirely to individual enterprise and the
the largest number of banks; so option (b) is
need for the traffic warden implies that
not acceptable. Option (d) is contradicted in
statement III is not supported by the passage.
the third paragraph in the reference to ‘payday
So I, II and III are not correct. Response IV
is not mentioned in the passage. So none of loans and pawn shops’—which are informal
the statements is correct. sources of credit.

130.(c) It is clear that the writer neither favours 137.(d) None of the given options answers the question.
complete freedom to the market nor the strict The answer is to be found in the last paragraph
control of the market. Answer response (d) is in the sentences, ‘But Grameen America says
not indicated in the passage. The writer merely … out of nothing.’
uses the chaotic conditions of traffic as an 138.(d) The statements need to be checked for their
illustration. correctness. Nothing in the passage suggests
what statement 1 says, so cross it out.
PASSAGE 27 Statement 2 is a bit tricky, but read the
131.(b) Answer response (a) is not correct, as social statement and the passage, especially the first
change may be short term also. Response (c) paragraph, carefully and you can see that it

1 . 61
Comprehension

is not correct. Grameen, says the passage, had already been observed—the increase in
helped the poor entrepreneurs in Bangladesh serotonin in rats injected with trypotophan.
to ‘grow from a subsistence living to a 148.(b) The word ‘surprisingly’ at the beginning of the
livelihood’ and later in the same paragraph, third paragraph holds the clue. The authors’
‘The ultimate promise of Grameen … out of hypothesis was most probably the same in the
poverty’. It does not ensure subsistence case of a protein meal as in a carbohydrate
standard; it aims at lifting the poor above meal—that the trypotophan (which increases
subsistence standard. So statement 2 is not serotonin) level would increase. That is why
correct; cross it out. The passage says that they asked the question, ‘Since protein …
the poor, to whom Grameen lent, repaid on levels?’
time; no comparison is made between the poor
149.(c) The question is a bit long, but it is clear from
and the affluent. So statement 3 is also
the passage that more protein in a meal leads
incorrect; cross it out. The correct answer
to a depression in the level of serotonin
response is (d).
production and release. So, naturally, in the
139.(b) The passage is about Grameen and its given case, a meal high in proteins alone is
microfinance attempt in the US. It hardly to be avoided.
attempts a comprehensive evaluation of the
status of the American economy, nor is that PASSAGE 30
the concern of the passage. So option (a) is 150.(c) This is the only answer response that covers
not correct. Options (c) and (d) are out of the the topic, scope and point of view of the
scope of the passage. passage. Option (a) enlarges the scope to
@UPSC_THOUGHTS

140.(d) There is nothing to suggest ‘lack of proper employers’ ‘practices’ in general, whereas the
training’. Statement 1 is not correct. Nor is passage is concerned only with pay and
statement 2 supported by the passage. So promotion. Moreover, the author does not
neither statement is correct. The real reason advocate anything. Option (c) refers to the
is to be found in the third paragraph, ‘Under second and third paragraphs but does not
Grameen’s tutelage … bank to do it.’ qualify as the ‘main concern’ of the passage.
Option (d) is outside the topic of the passage.
141.(d) See the last paragraph, ‘More importantly …
Moreover, the author does not ‘take issue’ with
one another.’ others’ views.
PASSAGE 29 151.(b) The third paragraph suggests the anxiety
women experience anxiety: ‘When the typical
142.(c) This is clear from the sentence, ‘We gave the
… she is unsure of herself …’ Option (a) is
rats … secretion’ in the second paragraph.
not supported by the passage which merely
143.(c) There is no reference to ‘controversial’ scientific states in the second paragraph that women
theories in the passage, merely statements of face the problem of ‘finding good, affordable
how a research finding was made. So options childcare…’ Option (c) is not discussed in the
(a) and (b) may be rejected. Nothing in the paragraph at all. Option (d) is contradicted by
passage refers to ‘further research’. So option the passage in the fourth paragraph.
(d) is not correct.
152.(c) Check each statement for not being supported
144.(c) The other options fail to give the essence of by the passage. Statement 1 is mentioned in
the passage. the first paragraph but not in the context of
145.(d) Refer to the third and fourth paragraphs. women who return to the labour market after
a gap. So it qualifies as a correct response
146.(b) The second paragraph gives the answer.
in this question. Statement 2 refers only to
147.(d) The first sentence of the second paragraph part-time workers as per the passage. So it
confirms what option (d) says. The author’s qualifies in the context of this question.
first studies sought to move ahead of what Statement 3 is mentioned in the third and

1 . 62
Comprehension

fourth paragraphs. So the statement is not 158.(d) The answer is to be found in the second
correct for this question. Answer response (c) paragraph.
is correct.
159.(d) Statement 2 may appear wrong, but the words
153.(d) The first paragraph supports this option: it in the first paragraph, ‘personal network of …’
attributes the situation of lower rates of pay implies customer loyalty.
and promotion for women in comparison with
men to the fact that women have domestic PASSAGE 32
responsibilities. Option (a) is not the reason
160.(b) Green packaging is environment-friendly; it
for men receiving higher salaries in general;
ensures that waste is either biodegradable or
it is not said that women lack qualifications,
but that, after some time away from work, their recyclable.
technical skills may become ‘rusty or obsolete’. 161.(d)
Option (b) does not apply to women in general.
162.(a) Biodegradable wastes like vegetable peelings
Option (c) goes beyond the scope of the
can be decomposed easily. ‘Recyclable’ means
passage: the writer does not discuss general
materials like paper and glass bottles can be
discrimination against women at the workplace.
put through a process that allows them to be
PASSAGE 31 used again. (b) is stated in para 1; (c) is stated
154.(b) There is nothing in the passage about in para 4; the last para implies what is stated
Argentina’s economy as such. So statement in (d).
1 is out. The Argentine study is illustrative of 163.(d)
@UPSC_THOUGHTS

what the passage says (in the second


164.(c) Green as used in the passage means ‘concerned
paragraph) just before referring to the study—
with the conservation of the environment’. Note
that when government gives legal means to
control their assets, then people plan ahead. that ‘green’ does not mean ‘healthy’.
While the study does not specifically refer to
PASSAGE 33
planning for the future in the Argentine study,
the social indicators referred to—house 165.(b) Read paras 2 and 3 for (a) and (c). Para 2
construction and education levels—indicate states (d). (b) is not stated or implied in the
building assets or planning for the future. So passage.
statement 2 is correct. Statement 3 is not
166.(c) Though (a) and (d) are discussed and it is
supported by the passage. So answer choice
mentioned as to what parents want their
(b) is correct.
children to do vis-à-vis their professions, none
155.(d) The passage clearly states that when of these is the main theme of the passage.
government grants people the legal means to
control their assets, the people are interested 167.(b)
in building assets and wealth. Only then will 168.(d)
the growth of slums stop. So (d) is correct.
169.(b) Response (c) is part of (b). It is because of
See the second sentence of the first paragraph.
the difference in ideas that communication is
The passage does not blame migration, so
difficult; there is not a ‘lack’ of communication.
reject (a). There is no mention of municipal
authorities building low-cost housing; reject (b). 170.(c)
Blaming population growth, the passage implies,
is not correct; reject (c). PASSAGE 34
156.(c) None of the other options is supported by the 171.(b) See the opening lines of the passage.
passage. 172.(c) Though (a) is implied in the passage that was
157.(a) The other options are either contradicted by not the way in which Pasteur essentially
or outside the scope of the passage. viewed the suffering.

1 . 63
Comprehension

173.(b) Read the end of the para 1. sentences of paragraph 2. So you may
174.(c) See para 4. eliminate answer responses (a) and (d) as both
have I in them. Statement II is not supported
175.(d) Though throughout it is indicated that he was
by the passage; refer to ‘Rare cases … form.’
not a materialistic person, we cannot call him
So your answer can be (b) or (c). Statement
a ‘spiritualist’—a believer in spiritualism—with
III is contradicted by the fact that poliomyelitis
reference to the passage.
caused by three antigenic virus types does
PASSAGE 35 have a vaccine for its control. So both II and
III are not supported by the passage. The
176.(b) It is stated in the very first paragraph.
correct answer response is (c).
177.(d) See paragraph 5. Though (a), (b) and (c) are
183.(d) Responses (a) and (c) refer to the Sabin
mentioned in the passage, these cannot be
vaccine, while (b) is not supported by the
said to be unique to friendship as against other
passage.
relationships in the context of the passage.
184.(a) Refer to the last two sentences of the second
178.(c) In friendship, all communication skills come
paragraph. Response (b) may be correct taking
into play, but these cannot be termed a ‘human
into account the fact that vaccines can be
value’.
developed against viruses that infect humans
179.(a) See paragraph 6. only; however, in the passage, HIV is
180.(b) Option (c) helps friendship to grow; there is considered in the context of the mutative
nothing in the passage to suggest (d). capability of the virus. So (b) is not mentioned
in the passage.
@UPSC_THOUGHTS

181.(c) Option (a) is contradicted by what is said in


paragraph 5. Option (b) is nowhere suggested. PASSAGE 37
As for (d), communication skills are important
185.(b) Response (c) is not correct because only
in all friendships but friendship is not ‘all about
hunting is mentioned as having become
communication skills’.
fashionable, and (d) finds no mention in the
PASSAGE 36 passage.
182.(c) The question has a negative – not – in it, so 186. (c) 187. (b)
you need to point out the statement/s that is/
PASSAGE 38
are not supported by the passage. Statement
I is supported by implication in the first two 188. (d) 189. (c) 190. (a)

1 . 64
2
INTERPERSONAL
SKILLS
INCLUDING
COMMUNICATION
SKILLS
@UPSC_THOUGHTS

Interpersonal skills are variously called social skills, people skills, soft skills
or even simply communication skills. It is, however, broadly understood to be
the skills used by a person to properly interact with others. In a place of
work, the term generally refers to an employee’s ability to get along with
others while getting the work done. A very important part of these skills is
communication skills, as most of us interact with others through some form of
communication: words, gestures, facial expression, all communicate what we
think, feel and expect from others. Though interpersonal skills are, to a large
extent, related to character traits of an individual, they can be learnt and
improved upon. Persons with good interpersonal skills consider and respond
appropriately to the needs, feelings and capabilities of different people in
different situations. They are tactful, sensitive, and compassionate and treat
others with respect. Obviously, these aspects need to be absorbed and put to
work by civil servants who have to deal not just with office colleagues but
with members of a vast public from a land of stupendous diversity.
The following pages consider the meaning, various aspects and importance
of interpersonal skills without going too deeply into theories. While
communication skills have been broadly and simply discussed, questions on
verbal linguistic skills have been included as well—specially the skills of
understanding shades of meanings, being precise, and communicating clearly.
Interpersonal Skills including Communication Skills

@UPSC_THOUGHTS

2.2
Interpersonal Skills including Communication Skills

Interpersonal Skills including


Communication Skills
ORIENTATION

INTERPERSONAL SKILLS Dimensions of Interpersonal Skills and


How to Improve them
Trying to Define Interpersonal Skills Within an organisation, people with good interpersonal
skills are likely to be more productive than those with
Interpersonal skills are usually considered to be mental
poor interpersonal skills, because the former are
and communicative processes or set of rules applied in
considered to be more likely to project a positive attitude
the course of social interaction to create certain effects
and look for solutions to problems. A person with good
or to reach certain results. They are abilities that help
interpersonal skills is usually one who considers and
a person to interact effectively with other people.
responds appropriately to the needs, feelings, and
Interpersonal skills are essential if one wants to succeed
capabilities of different people in different situations.
in a modern organisational set-up. They are essential
Such a person is tactful, sensitive, compassionate, and
in interactions with the superior, peers and subordinates.
treats others with respect. So one has to work towards
@UPSC_THOUGHTS

They are equally important in interacting with people


acquiring these traits if one does not possess them
outside the organisation.
naturally.
Interpersonal skills are often interchangeably called
people skills, soft skills or even communication skills. 1. Self-awareness
Interpersonal skill is a set of several skills of which no This is the first step towards improving one’s
definitive list can be made. The choice of, and emphasis interpersonal skills. You may have a certain opinion
on, different skills vary according to the topic and local about yourself, but do others around you perceive you
conditions. Many skills are used simultaneously in in the same light? Do you know how they perceive you?
practice. You need to know as much as you can about yourself.
Interpersonal skills, however, include more than The more you know yourself, the better you will
communicating skills. They also involve habits, attitudes, understand how you are perceived by others and why
behaviour, manners, even appearance and deportment. others respond to you the way they do. Where did you
While interpersonal skills begin early in life and are come from? How does that play into your present? How
influenced by family, friends and one’s experience of does your past affect the decision you make today?
the world around, some are even inherited: our genes Understanding these things will give you a greater
to a large extent affect our appearance and some discernment and direction. However, self-awareness is
personality traits. How far is it, then, correct to use the something that may be learnt on one’s own; it is not
term ‘skills’ in this context? Though certain basic traits something that can be taught.
cannot be altogether overturned, they can be worked Consider what you are good at and develop a
upon to show some changes if those changes are realistic view of those skills. Which are the skills you
desirable for your position in an organisation or even should utilise? Which ones do you need to continue
outside it. And any ability that can be acquired by to hone and improve? Focus on developing those
training is, after all, a skill. Indeed, as we grow up, it strengths. Get your bearings straight and build your
is our responsibility—no longer our parents’ or teachers’ leadership around your natural abilities. However, it is
responsibility—to initiate the changes in interpersonal necessary to know and accept your weaknesses also.
skills that might be required. And in this process, we Recognising your weakness will help you determine
have to start with ourselves before judging others. how to compensate for them.

2.3
Interpersonal Skills including Communication Skills

Self-awareness can help you to work towards problems; and readily smoothing out differences and
developing abilities like adjustment, extroversion, mending relationships.
agreeableness, conscientiousness, openness to Emotional learning comes more through practice—
experience, recognition of motives, and discovery of repetition of ‘proper’ responses—than through conscious
feeling. noticing and memorising effort. To develop emotional
skills, you should ‘watch’ your own emotional reactions,
2. Emotional Intelligence noting how you feel in various situations and what
Emotional intelligence or emotional quotient (EQ) is arouses various feelings in you. Notice how others react
another dimension of interpersonal skills. It is based on to situations and, if possible, talk to others about their
the premise that in order to be successful we need to emotional responses to check the accuracy of your
be able to manage our own emotions and be aware of perceptions. Watch and try to imitate those who do
and sensitive to those of others. have highly developed emotional intelligence in the
Emotional intelligence includes both personal and ways they display, control or use their emotions in
social competence. In the personal sense, an emotionally various situations.
intelligent person is self-aware, able to assess his or her While emotional intelligence certainly helps to make
own strengths and act with self-confidence, and can possible warmer and positive friendships with
control his or her emotions and keep in check destructive colleagues at work, it has benefits for work success as
emotions. The social aspect of emotional intelligence well. It has a positive impact on cohesion of work
involves an ability to perceive and interpret with groups, results in a better compatibility between
accuracy another person’s emotions leading to an appraisals by self and superior and actual performance,
@UPSC_THOUGHTS

ability to develop healthy relationship. It is the ability and can prevent emotional, ethical and job conflicts
to distinguish between emotions, such as between from interfering with the satisfaction at work.
frustration and irritability, or between happiness and
excitement. 3. Self-management
The principle of emotional intelligence provides a Self-management skills tell an employer whether or not
new way to understand and assess people’s behaviours, your personality fits what the organisation requires.
management styles, attitudes, interpersonal skills, and Self-management refers to methods, skills, and strategies
potential. There are five domains of emotional by which individuals can effectively direct their own
intelligence: activities towards the achievement of objectives. It
(i) knowing your own emotions, and recognising includes goal-setting, decision-making, focusing,
and understanding other people’s emotional planning, scheduling, task tracking, self-evaluation,
expressions; self-intervention and self-development.
(ii) managing or self-regulating your own
emotions; 4. Persuasive Ability
(iii) selecting socially appropriate responses to Persuasiveness is an important interpersonal skill,
situations and others’ emotions especially if you are in a position of leadership.
(iv) managing the emotions of others; and Persuasion means to convince others to your point of
(v) using emotional knowledge to solve problems. view to induce them to do something they otherwise
The behavioural traits of EQ include the abilities may not do. It simply denotes change in which you
of showing your feelings honestly and in a calm must engage other people to be an active participant,
manner; dealing with others with compassion and where they must listen to your suggestions in order for
sensitivity; assessing the potential responses of others that change to take effect. Setting an example is one of
and acting accordingly; being open to others’ ideas; the most effective persuasion techniques to influence
keeping a balance between personal and the professional others. To effect external change you should act in the
life; firmly confronting people who are prone to create way that you would like others to act. For instance, if

2.4
Interpersonal Skills including Communication Skills

you would like people to work hard, then you yourself Skill of managing change is another dimension of
should work hard. interpersonal skills. Change management is a structured
The behavioural aspects of persuading skill include approach to shifting or transitioning individuals, teams
having patience, establishing credibility, using positive and organisations from a current state to a desired
tactful tone, harvesting the subjects’ self-interest, using future state. It is an organisational process aimed at
logical presentation of ideas and arguments, using empowering employees to accept and embrace change
emotional appeals judiciously and enlisting the help of in their current business environment. Many
others when persuading. One should be careful not to organisations have failed to survive because there was
present too many ideas at a time in seeking change. One no one to lead the change needed for them. Change
idea at a time is good persuasive tactic. management may include creative actions apart from
deep social understanding about leadership styles and
5. Leadership Styles
group dynamics.
Using different leadership styles to suit different persons Successful change management is more likely to
being managed and different situations at hand is an occur if the following are included:
asset. Transactional leadership involves an exchange ● Effective communication that informs various
relationship between leaders and followers. stakeholders of the reasons for the change (why?),
Transformational leadership is based more on a leader’s the benefits of successful implementation (what
shifting the values, beliefs and needs of his or her is in it for us, and for you?) as well as the details
followers. Effective transactional leaders identify of the change.
themselves as agents of change; they are courageous ● Effective education, training or skill upgrading
@UPSC_THOUGHTS

and they believe in people; they are value driven; they schemes for the organisation.
are life-long learners; they have the ability to deal with ● Provision to counter resistance from the
complexity, ambiguity and uncertainty; they are, in fact, employees and align them to overall strategic
visionaries.
direction of the organisation.
Rensis Likert categorised four styles of leadership—
● Availability of personal counselling to alleviate
exploitative authoritative, benevolent authoritative,
any change-related fear.
consultative, and participative/democratic. In
● Creation of a sense of urgency to loosen up or
exploitative authoritative leadership decisions are taken
‘unfreeze’ employees.
at the top and communicated downwards for
● Developing a compelling vision and strategy
implementation. Threats, rewards and punishments are
and empowerment of the employees to act on
used to secure performance. In benevolent authoritative
them.
style some power of control and decision-making is
delegated to the middle and lower levels. Economic As a leading administrator or manager, you need
rewards for good performance and threats or to develop goal setting skills. Goal setting is a powerful
punishments for non-performance and disobedience process for thinking about future and for motivating
are used as motivational tools. In consultative style oneself and/or others to turn the future vision into
there is adequate delegation of control and decision- reality. The process helps to choose the way one wants
making processes to lower levels. Rewards and to go. Goal setting gives long-term vision and short-term
occasional punishments are used for motivation. In motivation. It helps to organise time and resources.
participative group style there is complete trust in SMART goal setting includes the features of being
subordinates. It is the optimal approach to lead all specific, measurable, attainable, relevant, and time-
people in our dynamic and educated society. bound. Behavioural characteristics of effective goal
An effective leader should establish trust with setting include explaining the purpose and benefits of
followers, assess the readiness level of followers, clarify being focused to reach desired outcomes; knowing and
objectives, clearly structure the task, and motivate the showing the goal of the organisation; specifying deadline
followers. for each goal and prioritising the goals; building feedback

2.5
Interpersonal Skills including Communication Skills

mechanisms to monitor the progress; and giving rewards The most effective way to develop people in an
on attainment of goals. organisation is to ensure to give regular feedback to the
Coaching and counselling skills improve the team members on their work. The top priority for a team
employee and workplace performance. It is a truism manager is delegation. Successful delegation starts with
that the best managers (and leading administrators) are matching people and tasks, so the roles and goals of
excellent teachers. Coaching helps increasing an the team are fully understood. Another key duty of a
employee’s ability. Counselling helps an employee to leader is to motivate team members.
solve his/her personal emotive problems including the The behavioural characteristics needed for team
attitudinal ones that hinder work performance. Coaching skill are: to assess strength and weaknesses of the team;
provides an individual with feedback, insight and to establish a common goal for the team; to develop
guidance on achieving their full potential in their specific individual goals; to develop simple techniques
business or personal life. for creating cohesive work groups by recognising and
Persons with coaching and counselling skills would praising each other; to identify and practise new
discover sources of a problem through asking questions; methods of dealing with others that promotes team
develop an action plan for improvement of employees; spirit, cooperation and open communication; to practise
use proven methods to help staff become more self- active listening skills; to learn to receive feedback from
sufficient; learn how to delegate and which task to others without becoming defensive or angry; to provide
delegate first; educate and not assist; accept mistakes the members the required training and resources; to
and use them as learning opportunities; and recognise
encourage responsibility for both individual and team
and reward even trivial improvements.
performance; and to gain skills that allow positive
@UPSC_THOUGHTS

The power of coaching is to guide the behaviour


actions to be taken on group performance problems
through reinforcement. For effective coaching the actions
without causing conflicts and tension.
must be honest, consistent and specific.
Teamwork creates a greater output than what its
Politicking may sound ‘bad’, but is important to an
individuals on their own can create.
extent in real life situations. Politicking means actions
taken to influence how the advantages and 7. Interpersonal Communication
disadvantages are distributed within an organisation. Communication, an important aspect of interpersonal
A person with politicking skill frames arguments in skills, occurs not only when you interact with someone,
terms of organisational goals, gains control of
but also when you treat the other person as a unique
organisational resources, develops coalitions and makes
human being. So interpersonal communication can
himself or herself appear indispensable.
occur even between total strangers interacting
In an organisation, or in other interpersonal
respectfully, and even though the topic of conversation
relations, the text-book approach/prescription does not
might be something dull or task-oriented.
always work. There are times when you have to protect
In a workplace, each interpersonal relationship
your interests against others’ manoeuvres directed
exists within a complex network of inter-related
against you.
relationships. In the circumstances, the ability to
6. Team Work Skills communicate interpersonally—that is, with people as
Ability to work in and with teams This is also an people—becomes a basic skill for virtually any other
important interpersonal skill. A team is composed of organisational activity.
individuals who have different outlooks and abilities, Skilful interpersonal communication involves basic
and are at different stages of their careers. For some, conversational skills including listening and speaking
the task allocated may be challenging, and they need abilities; a basic understanding of how personalities
support. Others may be ‘old hands’ and may be looking and cultures affect communication; an understanding
for opportunities to stretch their skills. A leader/manager of your own personality, culture, and preferences; and
has responsibility to develop all the people. knowledge of techniques of conflict prevention.

2.6
Interpersonal Skills including Communication Skills

In fact, interpersonal communication is more about


Principles of Effective Interpersonal content and character of the exchange than about the
Communication mechanics of how it happens or with whom. Effective
Interpersonal communication may be improved by communication requires considerable skill in both
following the principles given below— sending and receiving information. Effective
● Treat the other person with respect.
communication skill is needed to provide all relevant
● Have positive thinking and mindset.
information, use multiple channels of communication,
● Do not interrupt one another.

● Do not criticise others or yourself.


be complete and specific, send congruent verbal and
● Do not volunteer others—i.e., do not say ‘so and non-verbal messages, use languages to be understood
so will do it’ without that person’s prior consent. by the receiver, maintain credibility of the message sent,
● Have patience. communicate in a warm and friendly way, obtain
● Listen to learn and learn to listen.
feedback to know that your communication has been
● Do not indulge in gossiping and be sensitive to
understood as you meant it to be understood.
others’ feelings.
A successful member of an organisation will display
● Use sense of humour at appropriate time and

situation—specially to defuse conflict. a variety of interpersonal skills when he/she uses


● Do not complain. communication to maintain relationships, share
● Look for solutions. knowledge, or exchange information about values,
● Be cheerful and smile at proper occasion. motivations and emotions.
● Praise and compliment people when they deserve
[Some aspects of interpersonal communication are
it.
discussed under Communication Skills.]
@UPSC_THOUGHTS

● Don’t immediately disagree with or boast about a


compliment given to you; when someone Today’s workplace, specially if it is of an
compliments you, simply say thanks with a smile administrative nature, would not be homogeneous; it is
and move on. made up of people from a range of socio-cultural and
● When you are unhappy, try your best to maintain economic backgrounds. Interaction thus cuts across
a positive outlook; you will end up feeling better cultures and classes.
and so will the people around you, as the mood Cross-cultural communication or inter-cultural
is contagious.
communication is the field of study which covers how
● Treat the members of your organisation and

colleagues as friends and not as strangers or people from different cultural backgrounds communicate
subordinates. in similar and different ways among themselves.
● Do not speak too often or for too long. Communicating across cultures integrates a number of
● Challenge the behaviour and not the person. topics including self-awareness, listening, feedback and
● Respect confidentiality.
knowledge. For effective cross-cultural communication
it is essential that people understand the potential
Interpersonal communication includes message-
problems of cross-cultural communication, and make a
sending and message-reception between two or more
conscious effort to overcome these problems. For effective
individuals. It includes all aspects of communication—
cross-cultural communication the behavioural traits
listening, persuading, asserting, non-verbal
needed are—
communication, etc. Individuals communicate on
● Understanding cultural diversity
different interpersonal levels depending on who they
● Developing awareness of individual cultures
are engaged in communication with. Interpersonal
● Showing tolerance
communication may be conducted using both direct
● Using simple language
and indirect mediums of communication such as face-
● Avoiding the use of stereotypes to explain the
to-face interaction and computer-mediated communi-
behaviour of different cultures
cation. Successful communication assumes that both
● Getting help from mediator/language interpreter
the sender and receiver of the message will interpret
and understand the message being sent. One has to develop empathy—the ability to

2.7
Interpersonal Skills including Communication Skills

understand another person’s situation, perceptions and The key behavioural dimensions of skill of valuing
feelings from their point of view—and the ability to diversity are to have self-respect and respect for others;
communicate that understanding back to the other to recruit broadly and fairly in an organisation; to be
person. Empathy is not sympathy; the latter implies trustworthy, self-controlled and self-confident; to
actually being affected by the other’s perceptions, reinforce productive differences; and to provide
opinions and feelings. Sympathy has its place but it can orientation and training to value diversity.
get in the way of effective consultation or coaching, for Communication involves listening skill. Listening
instance. is making sense out of what we hear. Hearing means
People who have travelled or worked in merely picking up sound vibrations. In listening we pay
multicultural environments generally (though not attention, differentiate sound stimuli and interpret the
always) develop the skill of empathy. Empathy is of sender’s intended meaning. The listening skill may be
special value in interaction with members of the learned and improved through practice. Listening
organisation or the public who have come to you with requires the temporary suspension of all unrelated
their grievances or complaints. It helps you understand thoughts. Listening helps in building relationships,
a problem from others’ point of view and thus helps solving problems, ensuring understanding, resolving
you reach a solution that benefits great numbers. conflicts and improving accuracy. At work, effective
Empathy is a critical skill for you to have as a coach listening means fewer errors and less wastage of time.
and counsellor. It is what helps in becoming effective One needs to develop the ability to be attentive and
leaders and supervisors. It has special use in conflict maintain eye contact; keep an open mind; not to
resolution. interrupt; wait for the speaker to pause in order to ask
@UPSC_THOUGHTS

Employee participation in workplace is valuable. If clarifying questions; not to overtalk; try to walk in the
such participation is hindered by divisive feelings or speaker’s shoes and feel what the speaker is feeling;
because small groups of individuals feel isolated because and give regular feedback to the speaker.
of the ‘different’ tag attached to them, work suffers. An Effective questioning shows that you have been
‘inclusive workplace’ implies that the people at the listening and have the ability to understand the caller’s
helm of affairs and most others have made an effort at real needs. There may be two kinds of questions—open-
bridging cultural differences. ended questions and close-ended questions. An open-
Even as one avoids stereotyping, one should respect ended question allows the person to create any one of
differences, and find out what kind of behaviour is a number of possible answers that work for the person.
acceptable or not acceptable to people from another A close-ended question means the person can only
culture rather than foist one’s own ways on them. One answer in ‘yes’ or ‘no’.
needs to learn not to evaluate or judge hastily. It is Feedback is a communication that gives a person
necessary to make sure that you have understood the information about some aspect of his or her
words and behaviour correctly. Look at things from the behaviour. In an organisation it is used to denote
other’s point of view. performance feedback. It may be positive or negative.
Valuing diversity is important. A ‘diverse By giving and receiving feedback the performance of
organisation’ is one which values differences. It team members may be kept high and well-integfrated.
recognises that people with different background, skills, Negative feedback can be unnerving and fear-provoking;
attitudes and experiences bring fresh ideas and as a result, it is often avoided, delayed or substantially
perceptions. It encourages and harnesses these distorted.
differences to make their services relevant and Giving feedback effectively is a skill and like all
approachable. It draws upon the widest possible range skills it may be developed through practice. While
of views and experiences, so it can listen to and meet giving feedback one should try to make it a positive
the changing needs of its users, staff, volunteers, partners process and experience. The purpose for giving feedback
and supporters. should be to improve the performance. It should be

2.8
Interpersonal Skills including Communication Skills

given in time and closer to the event. Prepare your At the same time, you need to develop the power
comment before giving feedback and be specific. Criticism of visualising the various effects of your decision on
should be done in private. And positive aspects should others. Interpersonal skills are important here as you
be spoken of. Specific suggestions should also follow have to gather information, get inputs from various
a feedback. Debates must be avoided, and one should people to decide better, understand how certain actions
listen actively, ask for examples, not become obsessive would affect different groups, and use your persuasive
over certain issues, and keep things as impersonal and skills to convince people that your action is for the best
job related as possible. and necessary.
Ethical action is a process fundamental to a
8. Interpersonal Skills and Negotiation,
professional practice of counselling. Ethical codes of
Decision-Making, Problem-Solving and
conduct generated by professional organisations rest on
Conflict Resolution
the general moral principles that guide an administrator
Interpersonal skills play a role in negotiations, decision-
or a manager’s behaviour within professional
making, and problem-solving and conflict resolution.
relationships.
Each of these areas calls for special interpersonal skills,
The behavioural characteristics included in ethical
though many of these skills overlap. Basically,
decision-making are to understand the organisation’s
interpersonal communication skills are involved here.
policy on ethics and determine all affected parties; to
[More on these topics will be found in the section on
interpret one’s action relative to the potential impact on
Communication Skills.]
others; to devise a moral course of action to assess
Negotiating is a contract sport. For the negotiation
potential impact on others; to select the more moral
@UPSC_THOUGHTS

to be effective you must be able to persuade others to


outcome from the choices generated; to think creatively
listen to your arguments, consider the arguments, and
about alternatives; and to have an intellectual attitude
decide that they want to help you is some way to
to deal with the complexities of human interactions in
achieve your goals.
a deliberate and systematic manner.
Negotiation is, after all, an aspect of communication
Problem solving is a mental process considered as
process. Clarity, objectivity, a positive and goal-oriented
attitude help in any negotiation situation. It is also the most complex of all intellectual functions. It has
important to develop the skill of addressing the problem been defined as higher order cognitive process that
rather than personalities—as is true in any argument. requires the modulation and control of the more routine
If you are to be successful at negotiating, you need to or fundamental skills. There are seven steps in problem
be clear of what you want, but you should also be able solving—
to see it from the other’s point of view. (i) Define the problem
A very important aspect of success in negotiations (ii) Look at potential causes of problem
is power: greater the power of a party, the greater its (iii) Identify alternatives for approaches to solve
chance of success. the problem
Decision-making skill is used to solve problems by (iv) Select an approach to resolve the problem
selecting one course of action from several possible (v) Plan the implementation of the best alternative
alternatives. Decision-making is also a key element of (vi) Monitor implementation of the plan
time management skills. Almost all decisions involve (vii) Verify if the problem has been resolved or not
some conflict. The difficult part of decision-making is The interpersonal skills involved in problem solving
to pick one solution where the positive outcome can include the ability to draw new ideas from others to
outweigh possible losses. bear upon the problem resolution. A good problem
The behavioural characteristics involved in solver would have or develop a systematic approach
decision-making include self-confidence; analytical towards solving the problem. He or she must be able
thinking; critical thinking; researching; and conflict to inspire confidence in and establish trust with the
management. involved people, encourage creativity from participants,

2.9
Interpersonal Skills including Communication Skills

spend time analysing the problem from all angles, and 9. Stress management
in case the original plan fails, come up with a backup Stress is our physical, mental, and emotional response
plan. to the various demands, changes and events. Stress
A good executive anticipates crisis and is prepared management is a collection of skills, tools and techniques
to deal with it. The crisis may be because of bad that help you to reduce, manage, and even counteract
employee relations or public relations, poor products/ the negative side-effects of stress. The potential cost of
services or outdated technology. Whatever be the case, stress to an organisation are in high turnover,
the scenario is anticipated and the plan of action absenteeism, low morale and reduced productivity.
thought out ahead of time. A good problem solver Stress management methods may be seen in two
should maintain an open and honest communication perspectives—(i) methods from the manager’s view;
with the people involved and not resort to personal and (ii) methods from the employee’s view.
attacks or finger-pointing. From the manager’s view, sources of stress are:
In any workplace or even outside where there is (i) Imperfect match between staff and jobs: the
human interaction, conflict is possible. Conflict is a distress may arise because the employee’s
situation of competition in which the parties are aware competence is not adoptive to the post’s
of the incompatibility of each with the wishes of the demand.
other. It becomes a problem when people are unable to (ii) Inappropriate performance appraisal system
manage and resolve it effectively. If it is not dealt with could bring about employees distress.
constructively, it can be a powerful destructive force
Some methods to reduce stress would be:
between people and within organisations. If it is
@UPSC_THOUGHTS

(i) An incentive mechanism matching up with


managed effectively, it can be turned into a constructive
the stress level is recommended.
force. The common sources of conflict are values,
(ii) Create an organisation culture in which the
attitudes, needs, expectations, perceptions, resources,
employee is felt to be concerned.
and personalities. The steps involved in conflict
(iii) Implement special human resource training
resolution are:
for the employees.
(i) Explore the reasons for disagreement
(ii) Put forward alternative solutions Methods from employees’ point of view are—
(iii) Get the people involved to agree on the most (i) To adjust the conception towards stress at
appropriate solution workplace: the employees should learn how
(iv) Implement the best solution to adjust their emotion in unchangeable
(v) Evaluate the solution working condition and environment.
(vi) Practise the conflict resolution process (ii) To enhance time management: to apply time
We are all capable of resolving conflict to a greater management principles which can help
or lesser degree. However, we must be able to identify employees to cope with stress.
the styles of conflict resolution with which we are (iii) To try variable techniques to alleviate the
comfortable. Communication is an important tool in stress: the physiological techniques like
conflict resolution. Persuasion, listening and feedback breathing, meditation, exercising, massaging,
are important aspects of conflict resolution. You must etc., and the psychological techniques like
be aware of which conflicts you can and want to imagining one is sitting in a comfortable
handle. You need to evaluate the sources of the conflict; place, learning to relax and enjoy, setting
keep people and problems separate; pay objective appropriate goals, etc., may be applied to
attention to the interests of the conflicting parties being alleviate stress.
presented; set out the facts that have an impact on the The behavioural characteristics involved in stress
decision; explore options together; and select the best management are positive thinking, effort to identify
option. causes of stress, changing the stressor or environment,

2 . 10
Interpersonal Skills including Communication Skills

change in the attitude and perceptions that affect ● If the other person has a point of view different
responses; and change in the direct response to the from yours, find out more about it. Your own
stressors. point of view may get broadened from the
exchange of ideas.
Can One Overdo Interpersonal Skills? ● Share information, confide your thoughts and
There is a flip side to almost everything. While the feelings, and invite a response. Make sure that
cultivation of interpersonal skills is no doubt desirable, the other person knows he or she has a value.
there can be too much of a good thing as well. ● Even if you are uncomfortable around a group
Overdoing the interpersonal skills can result in— of people, put in an effort to be at your best, and
● wasting time in building rapport in meetings try to learn something about the others—their
and building up networks; interests, their families. Initiate the first move, ask
● being misinterpreted as easy-going or easy to questions about their opinions and preferences.
influence and not being taken as substantive by ● Be careful about body language; strive to appear
some; relaxed, interested and calm. Fidgeting or
● having too strong a desire to be liked, thus shuffling papers around, looking frequently at
leading to avoidance of necessary negative or your watch—these can put off the other person.
unpleasant transactions; ● Even under stress, exercise your interpersonal
● not being able to assume credible leadership skills. If you are being criticised or verbally
when required to do so. attacked, retain your cool. Rather than shouting
In a position of responsibility in a workplace, or back, ask clarifying questions. Just allow the
@UPSC_THOUGHTS

as an administrator, you cannot always avoid other person to blow off steam till he or she runs
unpleasantness. You cannot be liked always and by out of it. And rephrase the criticism as an attack
one and all. Sometimes you will have to take unpopular on the problem. Who knows, the criticism may
decisions. How you manage depends on your be valid, and you could learn from it.
interpersonal skills, but overdoing these skills may ● Neither be pushy not be a pushover. Be assertive,
hinder you in doing your job well. conscious of your own rights just as you are
conscious of others’ rights and maintain the
Developing Interpersonal Skills: A Few Tips right balance in interactions with others.
● Accept others, alongwith their style and usual ● Use objective data when you judge others. It is
way of doing things. Understand that every better to study people than to judge them, anyway.
person is unique and works in distinct ways. ● Examine and evaluate your own performance in
Use your knowledge of what makes them work various roles—professional, contributor to
best to interact effectively with them. problem solving, managing change.
● Choose your approach from the other person ● Try to learn from experience, feedback and other
rather than impose your way on him or her. people. Models of strength as well as models of
● Work on being open and approachable. Be weaknesses can teach. However, learn from the
interested in the other person and gather characteristics, not the whole person.
information about the people you interact with,
and direct your actions towards putting them at COMMUNICATION SKILLS
ease.
● Listen to others: interpersonal communication is The word ‘communication’ is derived from the Latin
at least a two-way process. Ask clarifying word communis which means ‘common’. Hence,
questions without interrupting and judging communication implies sharing of ideas in common.
hastily. Getting your message across depends a Communication is a process by which information is
lot on getting the others’ message right first. channelled and imparted by a sender to a receiver

2 . 11
Interpersonal Skills including Communication Skills

through some medium. The receiver decodes the message An organisation is a group and communi-
and provides the sender with a feedback. Thus, all cation is the foundation of all group activities.
communications require a sender, a message and a Communication permits upward, downward
receiver. Communication is an integral part of and horizontal interaction between members
management. Most of the time of a successful manager of all levels of authority.
or administrator is spent in communicating with his/ (v) Communication helps to improve
her peers, subordinates, superiors, clients or customers, relationships. Communication helps to cement
members of the public, etc. Communication in an the superior-subordinate relationship. It binds
organisation may be internal, external and interpersonal. individuals to a common purpose. It enables
Internal communication deals with the relationship superiors to understand the problems,
between the organisation and its employees. The external difficulties and opinions of the employees.
communication deals with the relations of the agency (vi) Communication improves morale and
with the public and is called ‘public relations’. The motivation. Communication keeps the people
interpersonal communication is related to the in an organisation informed and, thus, helps
relationship among the employees of an agency. in improving their morale and motivation.
Good communication induces people to give
Importance of Communication their best to the organisation. Communication
Communication is a means, not an end. It makes brings about meeting of minds which in turn
also improves morale and motivation.
possible the management process and services; it acts
(vii) Communication is an effective device for
as the lubricant for smooth operation. Communication
@UPSC_THOUGHTS

helps managerial planning to be performed effectively, achieving participation by the workers.


Management can consult the workers and
managerial organising to be carried out effectively,
receive their grievances, complaints and
managerial actuating to be followed effectively, and
managerial controlling to be applied effectively. suggestions.
Communication is important because of the following
factors: Process of Communication
(i) It is the basis of action. Through In the process of communication a sender develops a
communication the employees associate the message which he or she encodes and transmits through
objectives of the company with their own. a channel to a recipient who decodes the message
Communication keeps the people working in which then provides meaning to the receiver who, in
accordance with the policies of the turn, provides feedback to the sender.
organisation.
Noise Noise Noise Noise Noise
(ii) Communication facilitates planning. Through
Source Encoding Channel Decoding Receiver
communication the executive interact and
provide vital inputs to plans. Through it the
Noise
executive can strike rapport with subordinates,
Feedback
seek their opinions and source and provide
realistic information on which sound plans The source is the initiator or sender of the
could be prepared. communication, who wants to transmit his ideas,
(iii) Communication facilitates decision-making. thoughts, needs, intentions or any other piece of
Through communication the right type of information to another person. Encoding is a process in
information reaches a manager and enables which the ideas to be conveyed are translated into a
him or her to consider the pros and cons of code or set of symbols or some other format of expression.
the matter before arriving at a decision. The message is the actual physical product from the
(iv) Communication makes coordination possible. source-encoding. It represents the meaning which the

2 . 12
Interpersonal Skills including Communication Skills

source wants to convey. The channel is the medium ● To express the interest of the management in
through which the message transmits. It is the connection their employees.
link between the sender (the source) and the receiver. ● To reduce or prevent labour turnover.
Decoding is the process which translates the message ● To use for gate-keeping by building linkages
into a form that can be understood by the receiver. The between the organisation and outside world.
receiver is the person to whom the message is directed ● To influence people in the organisation to create
(conveyed). Feedback is the response from the receiver a healthy climate, proper attitudes and
which enables the sender to determine whether the cooperative relations.
message was received and understood as originally ● To help in problem solving in the organisation.
intended.
Channels of Communication
At every stage there is noise. Noise includes those
factors in each of the components of communication Channels or networks of communication are of two
that reduce the accuracy or fidelity of the message. It types, viz., formal and informal. A formal channel of
may be sloppy handwriting, heavy accent, soft speech, communication is deliberately established by
unintelligible language or other factors that disturb, management for the transmission of official information.
confuse or otherwise interfere with the communication. An informal channel of communication, on the other
hand, is an unofficial channel and is the result of the
Purposes of Communication operation of social forces at the workplace. It is
The purposes of communication in an organisation are also known as ‘grapevine’ and supplements formal
as follows: communications.
@UPSC_THOUGHTS

● To increase the acceptance of organisational Major characteristics of formal communication are


rules by subordinates. as follows:
● To gain greater commitment to organisational ● It is intentionally created for flow of

objectives by motivating, controlling and communication between various positions in the


evaluating the performance of employees. organisation.
● To keep employees informed of the progress of ● Commands, instructions and orders travel

the organisation. downward; reports regarding performance,


● To provide data necessary for decision-making. grievances and suggestions travel upward; and
● To solicit information from the employees which coordination travels horizontally.
may be of help to the management. ● It is orderly, systematic and supports the

● To clarify task responsibilities, identify authority authority of superiors.


positions and provide accountability for ● It works very slowly because it has to follow the

performance. path laid down by the management.


● To indoctrinate employees with positive work ● It is largely written and is always on record.

attitude. ● It is oriented towards team events.

● To instil in each employee personal pride in ● It is controlled and motivated by the management.

being a member of the organisation. ● It is rigid as deviations are not allowed.

● To provide feedback to various elements in the ● There are few chances of distortion of

organisation. information.
● To make each employee interested in his ● It is authentic.

respective job and in the work of the organisation ● The status or position of the party to

as a whole. communication is very important.


● Management information system (MIS) is an The major characteristics of informal communi-
effective control device which helps in monitoring cation are as follows:
and controlling various activities in the ● It works very rapidly as it has not to follow any

organisation. particular path.

2 . 13
Interpersonal Skills including Communication Skills

● It is spontaneous and largely oral. by some medium other than speech or writing. Non-
● It is mostly off the record. verbal signals include proximity, posture, physical
● It is people oriented. appearance, gesture and facial expressions, and
● It is employee controlled and employee serving. direction of gaze.
● It is flexible. The distances (proximities) people choose to sit or
● It has very high chances of distortion of stand from each other can communicate quite a bit
information. about their relationship.
● Status or position of the party to communication Body language includes: (i) how people carry their
is not relevant. body; (ii) how the body is postured; (iii) how much and
where people touch each other; (iv) the extent to which
Modes of Communication people maintain eye contact; and (v) gestures.
People communicate through a number of different Paralinguistics includes the tone of voice, pacing,
modes or channels—verbal, non-verbal, and symbolic. and extralinguistic aspects of speech. How something
Verbal Communication In verbal communication is said, the way in which silence is used, the use of
words are used as mode of expression. Verbal ‘filled pauses’, and the pitch and quality of voice have
communication may be oral or written. Oral a range of meanings that are conveyed to others.
communication can take place either face-to-face or Symbolic Communication Symbolic communication
through mechanical devices. The most natural way of conveys a lot of information to other people. The places
transmitting messages is face-to-face communication. where we live, the clothes we wear, and the cars we
This method is useful in securing cooperation and drive all portray certain things about us to other people.
@UPSC_THOUGHTS

resolving conflict. It is also very useful in keeping the Similarly, the type of decoration a person has in an
message confidential. The mechanical devices used in office, and the way in which chairs are arranged and
modern organisations are telephone, intercom, electronic the manner in which physical space is used can
or radio paging, dictating machines, internet, internal communicate a great deal about a manager.
TV broadcast, etc. Both types of oral communications
are used in giving instructions to co-workers, Barriers to Effective Communication
interviewing a job candidate, lecturing to the workers, The goal of effective communication is understanding.
and negotiating with a supplier. Verbal-written However, considering the complex ways in which
communication is generally formal and, therefore, taken verbal, non-verbal and symbolic means are used to send
seriously. Records are maintained in it; therefore, it is messages, such understanding is not always achieved.
verifiable. Written communication may be in the form There are a number of physical, interpersonal and
of letters, circulars, memos, bulletins, manuals, intrapersonal barriers to effective communication. Some
handbooks, notes, orders, instructions, regulations, etc. of the barriers are as follows:
It is useful when the assignment is important and it (i) Semantic barriers: Semantic barriers refer to
is necessary to fix responsibility. However, as compared language difficulties that occur due to the
to oral communication, it is very slow. differences in the individual interpretations of
Drawbacks Drawbacks of verbal communication words and symbols used in the process of
are: (i) words can mean quite different things to different communication. The use of jargon can also
people; (ii) many organisations have developed a create problems.
specialised language that often facilitates (ii) Lack of skill Managers and supervisors who
communication between insiders but largely excludes lack writing, oral, reading and listening skills
others from understanding; and (iii) business talk may often fail to express themselves clearly. For
be interpreted at face value. example, young workers usually lack writing
Non-verbal Communication Non-verbal skill and, therefore, cannot write explicit
communication refers to the transmission of messages messages.

2 . 14
Interpersonal Skills including Communication Skills

(iii) Ideological barriers The members of the and wherever possible to improve the quality,
organisation do not share the same ideological accuracy and understanding of the message
perspectives and orientation. Differences in being sent.
background, education and expectation result (x) Dogmatism Sometimes the attitudes, opinions
in different social and political views. These and beliefs possessed by a person prevents
are probably the greatest handicaps to effective him from accepting accurate additional
communication and probably the most difficult information as it conflicts with the current
to overcome. situation. This is dogmatism, and it affects
(iv) Status differences Differences in status tend effective communication.
to inhibit upward communication. These (xi) Specialisation Specialisation often leads to
differences in status are sometimes created by fragmentation of work that causes people to
the organisations through titles, offices and be more loyal to their particular departments
amount of support resources distributed. In than to their organisations as a whole. Factors
general, the juniors are physically and like tunnel vision, parts-mentality, sectional
psychologically discouraged from approach- interest and departmental loyalty prevent
ing the superiors. employees from looking at organisational
(v) Filtering Filtering refers to the sender’s problems in a broad perspective.
purposeful and deliberate manipulation of (xii) Halo effect The halo effect is the result of two-
information to be passed on to the receiver. valued thinking. It leads to a situation where
The longer the chair of communication, the things are seen only as dichotomies—good or
@UPSC_THOUGHTS

greater the chance of distortion and vice- bad, right or wrong, black or white, etc.
versa. The purpose of filtering is to present a However, most situations are not dichotomous
more favourable/unfavourable impression. and, therefore, such thinking may oversimplify
(vi) Source credibility When the message sender most real situations.
has high credibility, the message is perceived (xiii) Confidentiality Sensitive matters of the
as truthful and significant. But when the organisation, e.g., personnel files, techno-
sender is negatively evaluated, the message is logical advances, policies, etc., must be kept
likely to be ignored. confidential. But extending the line of
(vii) Assumptions about self and others A person’s confidentiality beyond a certain reasonable
self-concept is probably the most important limit hinders communication. Confidentiality
factor affecting his/her communication. becomes a barrier when it is used as a means
Persons with low self-esteem are unwilling to of attracting or acquiring status rather than
communicate. Similarly, one’s assumption for operational effectiveness.
about the receiving member affects (xiv) Kind of information Information which fits
communication. One composes different our self-concept tends to be received and
messages for different people. accepted much more readily than the data
(viii) Time constraints Time constraints can create which contradicts what we already know.
communication distortion. Managers work (xv) Physical location The physical location and
under time pressure. They must make decisions proximity between the sender and receiver
without delay and meet deadlines. also influence effective communication.
(ix) Noise Noise is quite common but an often Research suggests that the probability of two
overlooked barrier to communication. Noise persons communicating with each other
means any kind of disturbance in the decreases by the square of the distance
communication system, i.e., unintelligible between them. Other physical barriers such as
language, heavy accent, sloppy handwriting, corners, indirect paths and stairs also hinder
etc. You must try to reduce noise whenever communication efforts.

2 . 15
Interpersonal Skills including Communication Skills

(xvi) Negativity Use of negative language is always (ii) Integrity of purpose The relationship between
destructive. Where the balance is over- what is said and what is meant is integrity
whelmingly positive, people tend to react of purpose. This means using clear, concise
quickly and positively to the negatives. Where and unambiguous terms, so that there is no
the balance is overwhelmingly negative, the doubt about the impact on the receiver. The
positive gets lost. message should be honest and straightforward.
(xvii) Interest People listen attentively only to those (iii) Suitable language and media The language
communications which relate to their interest and media used for the communication
and needs. should be suitable to the receiver.
(xviii) Information overload Information overload (iv) Timeliness Communication should be timely,
refers to a situation where we have more neither too late nor too early.
information than we can possibly sort out (v) Feedback It refers to the transmission to the
and use. To reduce the overload we delegate sender of the reaction of the receiver to the
others to attend to the information, put off information. Its purpose is to reinforce or
information until the overload situation is correct the action implied in any act of
over, forget information, or avoid it altogether. communication. It also provides opportunity
(xix) Defensiveness Individuals who feel for suggestions and criticism.
threatened by communication may use defence (vi) Flexibility A good communication system
mechanisms. For example, when communi- should be flexible enough to adjust itself to
cation appears evaluative or judgemenal, the changing requirements.
@UPSC_THOUGHTS

instead of attempting to discuss and (vii) Visibility People tend to respond much more
understand the message, the individuals positively if they know who is sending the
involved usually act defensively to protect message.
their self-concept by attacking the other person, (viii) Consistency Communication should be
resorting to sarcasm and ridicule, questioning consistent with the expectations of the receiver.
the motives and competence of the other (ix) Adequacy Information in the communication
person, or by attempting to avoid the situation should be sufficient, neither over-burdening
altogether. nor too little.
(xx) Other barriers In addition to the above (x) Uniformity Communication should be uniform
barriers, communication is affected by factors and not discriminatory for all those who are
like degree of motivation of the sender or the supposed to behave similarly.
receiver; influence of reference group; (xi) Acceptability Communication should
invisibility of sender; long, drawn out and stimulate acceptance and positive response in
complex rules and regulations; out-lived the receiver.
policies and procedures of the organisation; (xii) Multiplicity of channels Multiple channels
hierarchical barriers; cultural barriers; and may be used to stimulate a number of receiver’s
feedback barriers. senses (visual, auditory, etc.).
For improving communication the American
Elements necessary for Effective Management Association has developed a set of ten
Communication suggestions which are as follows:
Elements or characteristics which are necessary to make ● Clarify communication before attempting to
communication effective are as follows: communicate.
(i) Clarity of purpose It means addressing the ● Examine the purpose of communication.
questions like what the message is, and why ● Understand the physical and human
it is being sent. environment when communicating.

2 . 16
Interpersonal Skills including Communication Skills

● In planning communication, consult others to (iii) Critical listening Critical listening is the ability
obtain their support, as well as the facts. whereby a person can judge and evaluate the
● Consider the content and the overtones of the other person. It demands high levels of real-
message. time thoughtful efforts.
● Communicate something that helps, or is valued (iv) Biased listening When a listener hears what
by the receiver. he/she wants to hear, it is called biased
● Follow up makes communication effective. listening. The interpretation of communication
● Clear mention of whether messages are short- is based upon the stereotypes and biases that
run and or long-run importance. the listener has.
● Actions must be congruent with communication. (v) Evaluative listening Evaluative listening is
● Be a good listener. also called critical, judgemental or interpretive
listening. In this process the listener makes
Importance of Listening in Communication
judgment about what the other person is
Listening is the act of hearing attentively. Good listening
saying. It is particularly pertinent when the
requires one to carefully consider what one is hearing.
other person is trying to persuade the listener
There are three basic types of listening—(i) Competitive
perhaps to change his behaviour and even
listening, (ii) Passive listening, and (iii) Active listening.
beliefs.
Competitive listening is also called combative
(vi) Appreciative listening The main purpose of
listening. It occurs when a person is more interested in
projecting his/her part of view than in understanding appreciative listening is to appreciate and
thus enjoy the way the message is given, e.g.,
@UPSC_THOUGHTS

what the speaker is trying to communicate. The person


keeps on waiting for the opening or is busy in listening to music.
formulating his/her rebuttal reply to emerge as winner. (vii) Informational listening The listener’s goal is
Passive listening is also known as attentive listening. to understand the message as completely as
It occurs when a person is genuinely interested in possible for which he/she may ask questions
listening and knowing the other person’s point of view. or request clarification to get the full message.
The listener becomes attentive and keeps on listening (viii) Relationship listening The listener seeks to
passively. The listener assumes that he/she has listened better the relationship between people through
and understood the message well, and does not bother relationship listening. It is also known as
to verify the same. therapeutic listening.
Active listening, also called reflective listening, is (ix) Sympathetic listening In sympathetic listening
the most useful listening skill. It occurs when a person the listener’s role is often not to respond at
is genuinely interested in understanding and knowing all. The speaker who seeks sympathetic
what the other person is thinking, feeling, wanting or listening might have suffered a tragedy.
what the message means. The listener verifies his (x) Marginal listening In marginal listening, the
understanding before responding with his new massage. speaker is listened to in bits and pieces rather
The types of listening based on the purpose of than in whole. It may be classified as negative
listening are as follows: kind of listening since the important part of
(i) Discriminative listening The differences the message may be ignored. It is also referred
between different types of sounds are identified to as selective listening.
under this process. (xi) Projective listening In projective listening the
(ii) Comprehensive listening Comprehensive listener absorbs the information in accordance
listening, also known as content listening, is with his/her own view or perspective which
nothing but comprehending what the other dominates the perspective of the speaker. This
person is intended to communicate. is also a negative kind of listening.

2 . 17
Interpersonal Skills including Communication Skills

(xii) Empathetic listening Opposite to projective overheated room, a big crowd, excessive sound,
listening is empathetic/sensitive listening. In etc.
this only the speaker’s view is taken predomi- (iii) Attitudinal barriers The attitude of the listener
nantly while that of the listener is either may sometimes act as a barrier to communi-
completely ignored or given less importance. cation. Among common attitudinal barriers
It is a negative kind of listening. are egocentrism, pride and prejudice.
(xiii) Literal listening In literal listening only the (iv) Cultural barriers Cultural barriers like
content of the message is taken and the different accents and cultural values may
relationship between the facts in the content interfere with the listening process, when two
is ignored. This results in loss of the meaning
people from two different cultures
of the message.
communicate.
(xiv) False listening False listening occurs when a
(v) Gender barriers Researches have revealed
person is actually not listening, but is behaving
that men and women listen very differently
or pretending that he/she is listening. The
for different purposes. Women are more likely
listener may nod, smile and grunt at the
to listen for the emotion behind a speaker’s
appropriate time without actually listening to
the message. It is, in fact, not listening at all. words, whereas men listen more for the facts
(xv) Full listening In full listening the listener pays and the content.
close and careful attention to the message (vi) Lack of training People are not good listeners
received and tries to understand the full by birth. The art of listening has to be developed
@UPSC_THOUGHTS

content of the message. Full listening requires through practice and training. In India, lack
skills like concentration, understanding and of training in listening skills is a barrier to
summary. listening.
(xvi) Deep listening Deep listening is more intense (vii) Bad listening habits Most of the people have
than full listening. In this not only is the poor listening habits. Some people have the
message understood but the person behind habit of faking attention and listening to
the words is also perceived. It is, in fact, nothing at all. Others may tend to listen to
listening between the lines of what is said. everything spoken without discrimination and,
The sender’s emotion, body language, values, as a result, miss out on the main points.
biases, beliefs, etc., are also understood. (viii) Knowing the answer This barrier acts when
the listener indicates that he already knows
Barriers to Listening
what the speaker is going to say. Sometimes
There are a number of obstacles that stand in the way
the listener tries to cut short the speaker
of effective listening. Some of them are as follows.
and tries to complete the sentence for the
(i) Physiological barriers The impairment of
speaker.
physical organs participating in communi-
(ix) Trying to be helpful Sometimes, the listener
cation acts as barrier to communication i.e.,
hearing disability, problem in processing tries to be helpful to the speaker by encour-
information, etc. Rapid thought is also a aging the speaker on some point and this acts
physiological barrier in which the attention of as a barrier to good listening.
the listener is deviated because of his more (x) Reaching to red flag words Sometimes, some
rapid information processing ability in words spoken by the speaker may lead to a
comparison to the pace of the information reaction in the listener which the speaker did
passed by the sender. not mean. If this happens, the listener is
(ii) Physical barriers Physical barriers refers to seldom able to fully listen to what the speaker
the distortions in the environment i.e., an is saying.

2 . 18
Interpersonal Skills including Communication Skills

(xi) Believing in language Some words may carry ● Control your fear.
different meanings in the minds of the ● Think before you talk.
speakers and listeners and this may act as a ● Learn to listen.
barrier to listening. ● Believe in your message.
(xii) Desire to talk People love to talk most of the ● Repeat major points.
time, to be the centre of attention. If a listener (ii) Regarding Communication techniques
insists on monopolising the conversation, he/ ● Reduce jargon in your language.
she is not going to hear very much. ● Use acronyms only after first defining them.
(xiii) Close mindedness Our mind will not be open ● Use humour to reduce boredom.
to new ideas if we think that we have all the ● Level objections, if any.
answers and that the things we know are the ● Ask for feedback.
right answers. ● Cite anecdotes and stories.
● Eliminate audible pauses.
Ways to Improve Listening Skills ● Make eye contact.

Listening skills may be improved by following the ● Increase your vocabulary.

suggestions given below: ● Enunciate clearly.


● Practise pronouncing words properly.
● Provide verbal or non-verbal clues that you are
● Exercise tongue, jaws and lips.
actively involved in listening.
● Use gestures, but in a limited way.
● Concentrate on listening. It requires willingness
● Use pauses.
@UPSC_THOUGHTS

and practice.
● Record your voice to know how it sounds.
● Refrain from formulating an immediate response.
● Watch your tone.
● Try to prepare for an interaction beforehand.
● Vary your volume.
● Be prepared to accept revisions.

● Be sure the environment is conducive to listening. (iii) Regarding listening


● While listening, lean forward and sit upstraight ● Prepare yourself to listen.

in your chair. ● Concentrate on the message.


● Focus on the speaker.
● Make eye contact with the speaker.
● Screen out distractions.
● Listen between the lines instead of hearing only

scattered information. (iv) Regarding speaking and listening


● Pen down your question, if you have any, and ● Accept different accents.
ask it at the appropriate time. ● Use mind mapping.
● Stay focused and have patience. ● Ask questions.
● Try to remove distractions.
(v) Regarding speaking and writing key points
● Keep your emotions in check when you disagree
● Write a purposeful statement.
with the speaker.
● Outline the messages.
● Keep your mind open and react to ideas instead
● Use a simple method.
of reacting to the speaker in person. ● Be sparing with absolutes and generalities.
● Express your emotion.
Tips to Improve Communication Skills ● Explain abstract words.
The following tips are crucial to improving your ● Illustrate with personal examples.
communication skills: ● Ask for what you want.
(i) Regarding speaking ● Cite source of statistical data.

● Know what you want to say. ● Get to the point.

● Find out what you listener wants. ● Use gender-neutral language.

2 . 19
Interpersonal Skills including Communication Skills

● Use active verbs. To Resolve Conflicts and Create a more


● Use short sentences. Positive Outcome
● Quote authorities. Some points on effective communication to be kept in
● Watch semantics. mind are as follows.
● Support statements with details. (i) Stay focused Try not to bring up past hurts or
● Be concise. other topics into the current issue. Stay focused
● Create a verbal picture. on the present, your feelings, understanding
(vi) Regarding reading one another and finding a solution.
● Keep up with current events.
(ii) Listen carefully We often think we are listening,
but are really thinking about what to say next
● Read something inspirational.
when the other person stops talking. So try
● Use internet.
really to listen to what the other person is
(vii) Regarding non-verbal communication saying. Don’t interrupt; don’t get defensive.
● Check your posture. (iii) Try to understand the viewpoint of others.
● Give non-verbal cues. (iv) Respond to criticism with empathy While criticism
● Smile at appropriate times. is hard to hear, and often exaggerated or
● Dress appropriately. coloured by other person’s emotions, it is
● Touch people lightly for emphasis or important to listen to the other person’s pain
guidance. and respond with empathy to their feelings.
● Shake hands properly. (v) Own your responsibility Effective communi-
@UPSC_THOUGHTS

cation involves admitting when you are wrong.


(viii) Regarding attitude
Sharing responsibility diffuses the tense
● Be flexible.
situation, sets a good example and shows
● Be agreeable.
maturity.
● Intend to improve.
(vi) Use ‘I’ messages Instead of saying, ‘You really
● Commit to being truthful.
messed up here’, say ‘I feel frustrated when
● Be realistic.
this happens’. It is less accusatory, sparks
● Recognise the impact of stress on
less defensiveness, and helps the other person
communication.
understand your point of view rather than
● Respect the point of view of others.
feeling attacked.
● Take responsibility.
(vii) Look for compromise Effective communication
● Be receptive to new ideas.
involves finding a solution that both sides
● Eliminate negative feelings.
can be happy with.
● Don’t take yourself too seriously.
(viii) Take a break If you feel yourself or your partner
● Handle disagreement with tact.
in communication is starting to get too angry
● Change abrasive behaviour.
to be constructive, it is better to take a
● Avoid words that hurt.
break from the discussion until you both cool
● Recognise condescending manners.
off.
● Recognise manipulative behaviour.
(ix) Don’t give up After taking a break, always
● Be well-mannered
come back to communication with a
(ix) Miscellaneous constructive attitude, mutual respect and a
● Present a good personal image. willingness to find a solution to the conflict.
● Give importance to timing. (x) Ask for help if you need it Help of a therapist
● Consider experience level of listeners. or counsellor or just your superior officer may
● Organise productive meetings. be taken to resolve conflict.

2 . 20
Interpersonal Skills including Communication Skills

Verbal Communication Skills it could happen if a sentence is grammatically incorrect


Verbal communication—the use of words—is an or there is some usage error. Such mistakes are ‘noises’
important part of interpersonal communication. Letters, because the reader is either distracted by the mistakes
memos, notes—all these form a channel of written and misses the message, or he/she simply cannot
interpersonal communication. Whether we say what we understand what is meant.
mean or mean what we say—two different things, by Verbal communication skills are vastly improved
the way—we have to make an effort to say or write what by (i) using appropriate and direct language—which
we want the other person to understand. It might be means avoiding the use of jargon and complex terms
of help to us if we can read between the lines and where simpler words will do, and (ii) following rules
understand what others mean even when they hide it of grammar and usage; being precise and brief (without
behind well chosen words. If we speak or write a leaving out important details) instead of verbose—after
language with some ‘communication skill’, we will find all, one does not want to bore the receiver to distraction.
that others mostly do understand our message.
It is necessary to keep out jargon unless the INTERPERSONAL SKILLS FOR
communication is between people who are familiar CIVIL SERVANTS
with it—the ‘insiders’ of a group, for instance. Even if A civil service aspirant must have good interpersonal
jargon is kept out, linguists have shown that ‘common’ skills as well as the ability to communicate with clarity
words are interpreted quite differently by individuals and precision. For the most part, these skills are the
because so often meaning exists in people’s minds and same for everyone, but the canvas of the civil servant
not in the words themselves. There are many words that is wider and much more diverse (and complex) than
@UPSC_THOUGHTS

have more than one meaning. that with which a corporate manager works. The civil
The very word ‘communication’, for instance, can servant is required to know a great deal of the
mean different things. It can mean (i) ‘pass on’ Constitution and legal niceties as well as socio-economic
information (or energy such as heat, illness, etc.) to dimensions of a huge country and these ideas are
somebody or something; (ii) exchange or share news expected to influence him/her in interpersonal skills;
with someone; and (iii) the state of being connected; you a worker in a corporate house, be it the largest ever,
even can have rooms with ‘communicating’ doors. needs to know much less and works in a much more
These variations of meaning demand that we know limited area.
them well before using the word—or such words—and Cognitive Ability is one of the requisites of
use them correctly. There are some standard practices interpersonal skills. By cognitive abilities we mean the
to be followed in the choice of words and the way in brain-based skills we require to carry out anything.
which we put these words together if we want the These skills are not so much to do with any actual
receiver to understand our message as we intended it knowledge as to do with the mechanisms of how we
to be understood, and if we, in our turn, want to learn, remember, solve problems, and pay attention. A
understand the message sent to us. task can be broken down into the different cognitive
Our language skills should be able to take our full skills or functions needed to complete that task
message to our receiver without any loss in quality. The successfully. Even answering the door involves at least:
‘noise’ that can distract our receiver’s attention from perception (hearing the door bell or knock), decision
our message in the present context could be precisely taking (answering or not), motor skill (unbolting or
because the word used allows two meanings or more unlocking the door), language skills (talking and
to get into the receiver’s head. ‘Call me a taxi’, for understanding language with reference to whoever is
instance, could mean two things, and the person at the door) and social skills (interpreting tone of voice
addressed may misinterpret the message and start and interacting properly with another human being).
calling the sender ‘Taxi’. Or the noise could happen if Cognitive skills involve attention or the ability to
we allow an odd spelling mistake to distort a word. Or sustain concentration on a particular object, action, or

2 . 21
Interpersonal Skills including Communication Skills

thought and the ability to manage competing demands Problem-solving and decision-making ability must
in our environment. Memory also has a role, both short- be developed, if it is not already there, to define the
term and long-term. Language skills, allowing us to problem in the right way so that you can then generate
translate sounds into words and generate verbal output, solutions and pick the right one. (You can see the
are another aspect of cognitive ability. A civil service common factor between interpersonal skills and
aspirant would do well to learn at least three languages decision-making.) In interpersonal skills, your decisions
well, including English and Hindi. India being a relate to people, your responses to their actions and
country with a variety of languages and dialects, if you words, and your behaviour with them in different
circumstances and under different emotional conditions.
want to be people-friendly, you must know the language
One must develop the ability to make decisions even
of the region to which you are posted. Similarly, if you
on incomplete information and on emotions (ours and
are seeking a foreign service, it would do well to know
others’).
a couple of foreign languages. A civil servant must also
Memory, or the capacity to hold and manipulate
have executive skills or the abilities that enable goal-
information, is of importance in interpersonal relations,
oriented behaviour, such as the ability to plan and
especially if you are in a responsible position. You must
execute a goal The goals of a civil servant would be be up to date with policies and programmes and their
different and wider than those of a corporate manager, goals and activities, even as you must keep in mind the
for they relate to the public at large and their welfare. idiosyncrasies of colleagues, superiors and juniors in
Executive skills include the following. office as well as the cultural milieu of the region in
You must have flexibility, or the capacity for quickly which you work.
switching to the appropriate mental mode as demanded Emotional self-regulation is very important to be
@UPSC_THOUGHTS

by the situation—whether you are talking to your able to identify and manage one’s own emotions for
superior, your colleagues or your juniors, friends or good performance. Even if you are angry with the
children and what you are talking about and under shoddy work of a colleague or a junior, it is necessary
what conditions. Civil servants have to interact with a to convey your thoughts in an acceptable manner—a
variety of people—politicians, illiterate villagers, manner that will make the receiver of the information
conservative religious groups, corporate heads, understand the problem with his or her work and
professionals in many fields, and so on. With none of accept the need to rectify the matter. So too if a superior
these people can you be high-handed or rude, even if picks out faults in you or your work, look at it from
you disagree with some of their ideas and disapprove a balanced point of view—that these faults if rectified
of their behaviour. You have to be accommodating, would make you a better worker. Don’t indulge in the
blame game. Even if you feel the criticism is unjustified,
though always keeping in mind the broad dictates of
it is better to be calm rather than answer back in anger
the nation’s Constitution and legal principles and
or show your displeasure. Your willingness to listen
rules.
will calm down the other person and probably make
You must develop insight into other people’s inner
him or her look at the situation afresh and to your
world, their plans, their likes and dislikes. And this is
advantage. If a member of the public is annoyed about
a difficult thing to do, given the diversity of India’s
some lack of service on the part of your department, you
people. have to treat the person with respect and offer your
You must develop the ability of anticipation to help, even if you think he or she is in the wrong. If a
make predictions based on recognition of patterns of person has been the victim of a disaster, you must treat
behaviour or occurrences. As a civil servant, your him or her sympathetically, irrespective of his/ her
canvas is much larger and perhaps even more crucial status in society, wealth, caste, creed or gender. At the
than those of a corporate manager. You may need to same time, you cannot allow yourself to become
deal with disasters – natural and man-made, or riots emotionally upset and over-involved in the personal
and demonstrations and protests. It always helps if you grief of a person struck by some tragedy, even if you
are alert enough to read the signs and take preventive/ have been designated as an officer responsible for
quick action. offering relief to people affected by the disaster.

2 . 22
Interpersonal Skills including Communication Skills

Sequencing or the ability to break down complex you reach senior posts, you need to speak compellingly
actions into manageable units and prioritise them in the as well when briefing or presenting information. You
right order is an important skill. This is as important have to learn to organise your thoughts and present
in interpersonal relations as in decision-making. If you them in a concise manner so that your message gets
have an ill-tempered boss who is for some reason across. You need to strike the right note with external
antagonistic towards you as well as a scheming stakeholders in meetings/by phone, etc. These days,
colleague – and this can and does happen as not all you must be information technology savvy as well.
people develop good interpersonal skills – you cannot Analytical Skills/Ability to Deal with Complex
just weep over your ‘misfortune’. You have to consider Information and Data You must be able to consider an
what steps you can take and in what order, and that issue in a detached manner, pick out important items
too without compromising your own ethical principles, in the problem you’re dealing with, and find solutions/
so that your career and mental well-being do not suffer ways forward. Moreover, you should be able to relate
while you work in the office. your issues to those that others within the office are
You must know the skill of inhibition. While you dealing with and to the bigger picture for the
are dealing with other persons, you must be able to pay organisation as a whole including overall aims and
attention to them and the situation at hand, and objectives.
withstand distraction and internal urges. As a civil Initiative and Multi-tasking You need to show
servant you may be placed in a crowded scene where initiative and not wait to be told what to do all the time.
it becomes necessary to focus on the person requiring You should be a person who has ideas, is quick to
your attention rather than on other things. learn, makes suggestions for improvements, and get on
A civil service aspirant should be ready and able with the work without constant supervision and advice.
@UPSC_THOUGHTS

to work in a team. So it is necessary to ask yourself: However, you should also be careful when showing
are you a friendly and approachable colleague and will initiative; there are situations when you have to work
you get on with the team you are in and the wider according to rules and procedures laid down. You need
culture, including working with other teams without to be able to think on your feet and respond to changing
any problem, and freely sharing information? You must situations, even as you need to be a good multi-tasker,
be ready to accept new bits of work thrown at you or able to juggle a number of things at once and be able
to work for a different boss from time to time/work in to work out which is most important or urgent. At the
project management style in different teams with people same time, you must know when to ask for help and
from around the office. from whom.
Management Skills/Organisational Skills As a Representation Skills If you are required to do so,
civil servant, you must be able to manage other people you must be able to represent the organisation in public
without taking up all their work and doing it for them fora or at meetings with members of the public or other
as you try to guide them; on the other hand, you cannot organisations. You need to develop the skill of
just assign others tasks and leave them without guidance. demonstrating an understanding of issues around office
You have to delegate but also guide and monitor. You and an ability to relate them coherently to others. You
have to coach and counsel and help others develop. need to be confident and have the courage of conviction
You should be able to get people to perform well in a when briefing senior people.
no-blame culture. Deadlines do matter: and you have Regulatory Skills You must develop the ability of
to see to it that others as well as you yourself perform being comfortable with both collaborating with and
the assigned work within specific time limits. You must confronting people, who may sometimes be external
be able to prioritise when there is a lot to do. Above stakeholders, and know when to choose the correct one
all, you must readily share credit with others who have of these approaches. There may be times when you have
worked with you and not selfishly claim all credit. to confront your own colleagues, such as when public
Communication Skills As a civil servant you must grievances against them are voiced. You may have to
know how to write well and speak coherently so that defend a policy or programme against criticism as also
people understand what needs to be done and how. As seek seniors’ authority, or political help, to bring about

2 . 23
Interpersonal Skills including Communication Skills

changes in programmes when there are glitches observed to anger with reasonableness. The language questions
in their implementation. are specifically designed to test (i) understanding of
As a civil servant in a developing country, you meaning, especially of words that have more than one
must (or must learn to) feel comfortable with economic meaning, words that sound the same but have different
concepts and working with economists. You must also spelling and meaning, and words that are often confused
feel comfortable with working within a legal framework with one another and hence misused; (ii) ability to spot
which controls the limits of what you can do. There is redundancies and improve a verbose piece of writing;
a need to be able to anticipate trouble before it happens. and (iii) the ability to arrange a set of sentences into
You must have a questioning mind that doesn’t just a logical order to convey sense. In other words, the
accept things you’re told at face value, and must learn questions focus on the skills of clarity, precision and
to negotiate with difficult people. Besides, you must the ability to be concise.
learn to respect confidentiality.
Q.1. Interpersonal skills are
If you get drawn into the policing department, you
(a) how people present a project at meetings and
must know how to interact with witnesses, victims and
conferences
the public on a daily basis and must possess strong
(b) how people relate to others within an
interpersonal skills including social perceptiveness and
organisation
listening skills. Critical thinking and problem-solving
(c) how people relate to one another
skills are important in analysing a situation and
(d) nothing but communication skills
determining a course of action. Life-saving skills such
as CPR and first aid are important fields of knowledge Solution Interpersonal skills are more to do with
that anyone, but especially civil servants, must learn. face-to-face interaction; it is the sum total of one’s ability
@UPSC_THOUGHTS

Since police work can be stressful and dangerous, to interact effectively with other people. The best answer
officers must possess courage, stamina and stress is (c), for these skills extend beyond interactions
management skills. You need to develop the ability to within an organisation to interaction with people
work under stressful conditions, integrity, ethics, self- outside. Communication skills is a part of interpersonal
control, ability to maintain confidentiality, teamwork, skills. Presentations at meetings are quite another skill,
ability to follow rules and respect for others. though one’s psychological disposition guides one
there as well.
QUESTION TYPES
Q.2. How would you cope with a difficult co-worker?
Interpersonal Skills including Communication Skills is You would
a new subject in the Civil Services Preliminary (a) be extra considerate and helpful in the face
examination introduced in 2011, and it is a somewhat of rudeness
uncharted area for the examinee. The questions in this (b) avoid him/her and make fun of him/her as
section could range from theoretical aspects of humour may work
interpersonal skills and their application, and the (c) be kind, but also realistic in not expecting the
basics of communication skills to specific questions on person to improve or to respond to your
linguistic skills. We have included various types of efforts to influence his/her behaviour
questions in the practice exercises. Questions on (d) create conditions that make it difficult for
interpersonal skills would test you for cognitive ability, him/her to continue and forcing him/her to
agreeableness, conscientiousness and emotional resign the job
stability. They will test your ability to understand Solution Option (c) is the best choice. You would
people and relate to them sensitively but without losing probably improve things even as you do not delude
a sense of your own rights; your ability to act ethically yourself with high expectations. And you prove your
in your interactions with others—be it friend, colleague, own interpersonal skills—which you don’t with options
junior or senior officer, or a member of the public; and (b) and (d). Option (a) makes you a pushover rather
your ability to remain calm under stress and respond than an effective person.

2 . 24
Interpersonal Skills including Communication Skills

Q.3. A member of the public comes to you with a the bickering of your colleagues. Option (b) is a waste
grievance just as you are leaving for an important of time. Option (c) offers no certainty that things will
meeting after a hectic morning of work. You change; besides it seems like offering a sweet to a child
would to stop him/her crying.
(a) immediately get back to your table and attend
Q.5. You are in charge of a department that has the
to her
responsibility of implementing an important socio-
(b) ask her to come the next day
economic programme in the district. Recently you
(c) tell her to go and meet a colleague in the next
have been receiving several complaints from the
room
members of the public that your officers are not
(d) request a colleague to attend to her explaining
able to explain the features of the programme. As
your inability to do so
a first step you would
Solution Option (a) shows you to be sensitive and (a) ask the complainants to come to you
responsive, but remember you have that important personally so that you may explain the features
meeting to attend. Option (b) is the typical behaviour of the programme properly to them
we have come to associate with insensitive bureaucracy (b) issue memos to the officers that they explain
and needs to be strictly avoided. Option (c) shows an their irresponsible conduct
inconsiderate attitude towards a colleague and the (c) call a meeting of the officers and try to find
drawback of ‘volunteering another’. Of course he or she out why exactly the public feels they are
is there for work but, in the circumstances, it is more unable to explain the programme
polite and considerate to get the same result with a (d) ignore the complaints as some people will
@UPSC_THOUGHTS

direct request to the colleague. Option (d) is best. always be dissatisfied


Q.4. You are a member of a team assigned an important Solution Option (c) is best as a first step, as you
project that has to be completed in a fortnight’s have to understand why a communication gap has
time. Anil and Ranil, the other two colleagues on occurred—is it the officers’ inability or inefficiency or
the team, are hard working and efficient, but their is it due to language problems or some other issue.
bickering with one another often holds up work. Option (a) means you involve yourself more than
You have tried hard to make them give up this necessary—you have officers to do the job, after all.
behaviour and cooperate in the work, but they do Option (b) is acting unfairly—you don’t know that the
not bother to change. You would officers have been irresponsible. Ignoring public
(a) accept them as they are and carry on as best complaints (option d) is the worst option: as a public
as you can, taking over their work also when official, your duty is to attend to the public’s grievances
they argue and problems.
(b) try again to resolve the conflict between them
Q.6. Which of the following is not a part of the
(c) offer to take them out to dinner at an expensive
message in communication?
hotel if they stop bickering and attend to their
(a) symbols
work
(b) gestures
(d) take up the issue with a superior officer
(c) posture
asking him/her to replace either Anil or
(d) active listening
Ranil, or talk them into working without
quarrelling Solution Option (d) is not an aspect of the message,
though it is a part of communication.
Solution Option (d) is best, as you have already
tried your best to get them to stop quarrelling and the Q.7. Which one of the following is/are barriers to
project requires their cooperative effort. At times, you communication?
have no choice but to bring a superior officer into the I. Walls hung with beautiful paintings.
picture. Option (a) is impractical as it will overburden II. Assumption that the other person is aware of
you and the atmosphere will continue to be vitiated by all the necessary background information.

2 . 25
Interpersonal Skills including Communication Skills

III. Cultural bias cowboy type. (b) is only a slight improvement: the
(a) II only caterpillar eats shoots, but then does it eat leaves or
(b) II and III decide to leave the place? (c) is not much better than
(c) III only (a). (d) is correct: the sentence needs no comma if it is
(d) I, II and III to convey the meaning of the caterpillar’s normal eating
behaviour.
Solution: All three are barriers. You may wonder
why walls hung with beautiful paintings should be a Q.10. Which of the given options would you choose to
barrier to communication, but decorations tend to express the meaning of the following sentence in
distract attention; you may tend to admire the paintings a concise and improved style?
rather than listen attentively to what is being said. The
Sentence: You are hereby requested through this
assumption that the other person knows all the facts
letter to submit immediately and with no delay
may not be true, in which case your message may not
all the documents that this office requires or
be clear enough. It is best to make sure that the other
you may have to countenance avoidable delay
person is as well informed as you are. Cultural bias also
in your documents being processed.
interferes with clear communication as your perception
(a) You are hereby requested to immediately
becomes less objective. So (d) is the best option.
supply all the required documents so as to
Q.8. You are to visit nearby villages to assess the process them speedily.
results of a certain social welfare programme (b) You are requested to submit the documents
initiated some time ago by the government. You that need processing urgently.
have to interact with several villagers in the (c) You are requested to immediately submit all
@UPSC_THOUGHTS

process, but are not very familiar with the language the required documents so that this office may
of the region. You would process them without delay.
(a) not use jargon when speaking to the villagers (d) If you want no delay, you should submit all
(b) rely on non-verbal communication skills alone your documents immediately.
(c) make sure you have a good interpreter with
you Solution ‘Hereby’ and ‘in this letter’ are almost
(d) speak slowly using the local language repetitive, and both can be left out. If you are writing
wherever you can and using plenty of signs all this in a letter, it is obvious enough. You have to
and gestures get the message across that the receiver has to submit
the required documents so that they may be processed
Solution Option (c) is the best choice. Jargon must quickly. (a) is confusing as to who will process the
in any case be avoided in such meetings so (a) is not
document, and there is a difference in meaning between
enough in the circumstances in which understanding
‘supply’ and ‘submit’ (which is what the original
your language is the main issue. Options (b) or (c) can
sentence says). (b) is not fully informative. (c) is the best
be a choice only if you cannot get hold of an interpreter.
option. (d) is too informal and also fails to mention who
Q.9. Study the following options and choose the one will process the documents besides asking for ‘all your
that shows appropriate punctuation. documents’, which is not what the original sentence
(a) The caterpillar eats, shoots, and leaves. mentions.
(b) The caterpillar eats shoots, and leaves.
(c) The caterpillar eats, shoots and leaves. This is just a sample of question types that may be
(d) The caterpillar eats shoots and leaves. set in the test. In the following pages, there are many
Solution Options (a), (b) and (c) give an idea of how more questions for practice. Wherever required, we have
punctuation can distort meaning, as the words ‘shoots’ provided detailed explanations. It must be kept in mind
and ‘leaves’ can be interpreted in different ways that questions on this subject can only be indicative;
depending on usage as noun or verb. (a) gives the besides, sometimes more than one answer response
impression of the caterpillar being a reckless ‘bad’ may be acceptable.

2 . 26
Interpersonal Skills including Communication Skills

PRACTICE EXERCISES
1. Which of the following statements are correct? 5. A person with good persuading skills
I. Interpersonal skills are skills that a person I. has patience.
uses to interact effectively with other people. II. uses a tactful tone.
II. Interpersonal skills are essential to get success III. appeals to the emotions.
in modern organisational set-up. IV. uses logical arguments.
III. Good interpersonal skills have nothing to do Which of the above statements are correct?
with emotion. (a) I, II and III only
IV. Interpersonal skills involve effective
(b) II, III and IV only
communication skills.
(c) I, II and IV only
(a) I and II only
(d) All the above
(b) I, II and III only
(c) I, II and IV only 6. Which of the following is not a characteristic of
(d) All the above an effective leader?
2. The personality characteristics of a self-aware (a) The leader uses threats, rewards and
person are punishments to secure performance
I. discovery of what one is passionate about. (b) The leader assesses the follower’s readiness
II. openness to experience. level
III. adjustment. (c) The leader clarifies objectives
IV. conscientiousness. (d) The leader clearly structures the task
@UPSC_THOUGHTS

(a) I and II only


7. Interpersonal communication includes
(b) I, II and III only
I. listening.
(c) II, III and IV only
(d) All the above II. persuasion.
III. eye-to-eye contact
3. Which of the following are domains of emotional IV. personal space.
quotient?
(a) I, II and III only
I. Managing your own emotions.
(b) II, III and IV only
II. Managing the emotions of others.
(c) I, III and IV only
III. Motivating yourself.
(d) All the above
IV. Motivating others.
(a) I, II and III only 8. The main purpose of ‘feedback’ in an organisation
(b) II, III and IV only is
(c) I, III and IV only (a) to improve performance
(d) All the above (b) to initiate debates
4. Which of the following behavioural traits are (c) to criticise the speaker
shown by a person with high emotional quotient? (d) to delay communication
I. He/she deals with others with compassion.
9. Consider the following statements:
II. He/she is close to the ideas of others.
I. Feedback may be positive or negative.
III. The person has no balance between personal
II. Listening skill may be learned and improved
and professional life.
IV. The person assesses the upcoming reactions through practice.
of others and acts accordingly. III. Questioning skill shows the ability to
(a) I and II only understand the caller’s real needs.
(b) II and III only IV. Use of stereotypes to explain the behaviour of
(c) III and IV only different cultures helps in cross-cultural
(d) I and IV only communication.

2 . 27
Interpersonal Skills including Communication Skills

Which of the above statements are correct? 14. Consider the following statements.
(a) I, II and III only I. The characteristic needed for ethical decision-
(b) II, III and IV only making involves interpreting one’s action
(c) I, III and IV only relative to the potential impact on others.
(d) All the above II. For making an ethical decision, the leader
needs to have an intellectual attitude to deal
10. Which of the following traits are important for with complexities of human interaction in a
effective cross-cultural skill? systematic manner.
I. Understanding cultural diversity. Which of the above statements is/are correct?
II. Showing tolerance to other cultures. (a) I only
III. Use of simple language. (b) II only
IV. Taking help from the mediator. (c) Both I and II
(a) I, II and III only (d) None of the above
(b) II, III and IV only
15. Consider the following statements.
(c) I, III and IV only I. A good problem solver should maintain open
(d) All the above and honest communication with the people
11. Effective goal-setting skill involved and not resort to personal attacks or
I. helps to choose the way one wants to go. finger-pointing.
II. helps to accept your weakness. II. A good problem solver discourages creativity
from participants.
III. gives long-term vision and short-term
Which of the above statements is/are correct?
@UPSC_THOUGHTS

motivation.
(a) I only
IV. helps to organise time and resources.
(b) II only
Which of the above statements are correct?
(c) Both I and II
(a) I, II and III only
(d) None of the above
(b) II, III and IV only
(c) I, III and IV only 16. The common sources of conflict in an organisation
(d) All the above are—
I. values.
12. The top priority for a team manager is delegation. II. attitudes.
Successful delegation starts with III. expectations.
(a) matching people and tasks IV. personalities.
(b) establishing a common goal for the team Select the correct answer from the codes given
members below.
(c) developing simple techniques for creating (a) I, II and III only
cohesive workgroups (b) II, III and IV only
(d) None (c) I, III and IV only
(d) All the above
13. Good decision-making involves
I. identifying the purpose of the decision. 17. A good conflict resolution skill includes
II. identifying principles to judge the alternatives. I. keeping people and problems separate.
II. evaluation of sources of conflict.
III. evaluating each choice in terms of
consequences. Select the correct answer from the codes given
IV. not taking any action on the decisions taken. below.
(a) I, II and III only (a) I only
(b) II, III and IV only (b) II only
(c) I, III and IV only (c) Both I and II
(d) All the above (d) None of the above

2 . 28
Interpersonal Skills including Communication Skills

18. Stress in an organisation leads to 22. All types of communication requires


I. high production. I. a sender.
II. high turnover II. words.
III. absenteeism. III. a message.
IV. low morale. IV. a receiver.
Select the correct code from those given below.
Select the answer from the codes given below.
(a) I, II and III only
(a) I, II and III only (b) II, III and IV only
(b) II, III and IV only (c) I, III and IV only
(c) I, III and IV only (d) All the above
(d) All the above
23. Consider the following statements.
19. Which of the following is/are likely to be I. Communication is a process by which
understood as a desirable interpersonal skill across information is channelled and imparted by a
cultures? sender to a receiver through some medium.
I. Greeting people with a smile. II. Communication helps managerial planning
II. Maintaining direct eye contact while speaking to be performed effectively.
to anyone. III. Communication is a means, not an end.
III. Offering a handshake as soon as you meet a IV. Management can be hindered by communi-
person. cation.
(a) I only Select the correct code from those given below.
@UPSC_THOUGHTS

(b) I and II (a) I, II and III only


(c) I and III (b) II, III and IV only
(d) I, II and III (c) I, III and IV only
(d) All the above
20. In a team whose members come from diverse
cultures and social backgrounds, it is best for the 24. The functions of communication include—
leader to I. facilitating decision-making.
II. making coordination possible.
(a) emphasise the diversities
III. improving morale and motivation.
(b) look for and emphasise the commonalities
IV. engulfing superior-subordinate relationship.
among members
Select the correct code from those given below.
(c) identify himself/herself with the minority and
(a) I, II and III only
support them
(b) II, III and IV only
(d) play by the ear and be ready to resolve
(c) I, III and IV only
misunderstandings
(d) All the above
21. Being assertive means
25. The processes involved in a good communication
I. to be clear and specific in stating what you include
want. I. encoding.
II. directing your message to the person for II. decoding.
whom it is meant. III. giving feedback.
III. encourage the other person/persons to give IV. reducing noise.
you specific feedback on what you say. Select the correct code.
(a) I only (a) I, II and III only
(b) I and II (b) II, III and IV only
(c) I and III (c) I, III and IV only
(d) I, II and III (d) All the above

2 . 29
Interpersonal Skills including Communication Skills

26. The purpose of communication in an organisation III. the vehicle we use.


includes IV. how the body is postured.
I. providing data necessary for decision-making.
Select the correct code
II. indoctrinating employees with positive work
(a) I, II and III only
attitude.
(b) II, III and IV only
III. increasing the acceptance of organisational
(c) I, III and IV only
rules.
(d) All the above
IV. providing feedback to various elements in the
organisation. 31. Which of the following is not a barrier of
Select the correct code. communication?
(a) I, II and III only (a) Noise
(b) II, III and IV only (b) Dogmatism
(c) I, III and IV only (c) Feedback
(d) All the above (d) Halo effect
27. Which of the following is not a characteristic of 32. Barriers of communication include
formal communication? I. filtering information.
(a) It is intentionally created II. information overload.
(b) It is flexible III. defensiveness.
(c) It is largely written and is always on record IV. integrity of purpose.
(d) It is controlled and motivated by the
@UPSC_THOUGHTS

Choose the correct code from those given below.


management
(a) I, II and III only
28. Which of the following is not a drawback of (b) II, III and IV only
verbal communication? (c) I, III and IV only
(a) Records are maintained in it, therefore, it is (d) All the above
verifiable
(b) Words can mean quite different things to 33. Noise in communication interferes with
different people (a) sender
(c) Many organisations have developed a (b) the message
specialised language that often facilitates (c) receiver
communication between insiders but largely (d) all the above
excludes outsiders 34. Which of the following statements are true about
(d) Business talk may be interpreted at face value an effective communication system?
29. Non-verbal communication signals include I. Communication should be neither too late nor
I. Internet. too early.
II. proximity. II. Communication should stimulate acceptance
III. facial expression. and positive response in the receiver.
IV. direction of gaze. III. Multiple channels may be used to stimulate
(a) I, II and III only a number of senses of the receiver.
(b) II, III and IV only IV. A good communication should be rigid.
(c) I, III and IV only Select the correct code:
(d) All the above (a) I, II and III only
30. Symbolic communication includes (b) II, III and IV only
I. the places we live. (c) I, III and IV only
II. the clothes we wear. (d) All the above

2 . 30
Interpersonal Skills including Communication Skills

35. The three basic types of listening are III. taking responsibility.
I. competitive listening. IV. eliminating negative feelings.
II. committed listening. Choose the correct code:
III. passive listening. (a) I, II and III only
IV. active listening. (b) II, III and IV only
(c) I, III and IV only
Select the correct code:
(d) All the above
(a) I, II and III only
(b) II, III and IV only 40. Which of the following is not the way to improve
(c) I, III and IV only listening skill?
(d) None of the above (a) Refrain from formulating an immediate
response
36. Which of the following statements is/are correct?
(b) Refrain from preparing for an interaction
I. In ‘marginal listening’, the listener absorbs beforehand
the information in accordance with his/her (c) Provide verbal or non-verbal cues to show
own view or perspective. active involvement
II. Biased listening occurs when a person is (d) Listen between the lines instead of simply
actually not listening, but is behaving or hearing what is being said
pretending that he/she is listening.
Select the correct code: 41. Addressing a person with a gender-specific title
(a) I only is to be followed by one
(a) only in the case of superiors
@UPSC_THOUGHTS

(b) II only
(b) always in the case of superiors
(c) Both I and II
(c) in case of subordinates if they are new and
(d) None of them
the relationship is formal
37. Which of the following is/are barriers to effective (d) always in an organisation
listening?
42. Which one of the following can be considered
(a) Bad listening habits
most reliable?
(b) Knowing the answer
I. Body language.
(c) Trying to be helpful
II. Tone of voice.
(d) All the above
III. Verbal-written communication.
38. Match List I and List II. (a) I only
List I List II (b) II and III
A. Language barrier (i) Gestures (c) I and II
B. Body language (ii) Inadequate (d) III only
vocabulary 43. You and a colleague are casually discussing some
C. Paralinguistics (iii) Personal space current event when there is a difference of opinion
D. Spatial cues (iv) Tone of voice and an ugly argument develops. You would
A B C D (a) stick to your view and insist on arguing out
(a) (ii) (i) (iii) (iv) the issue
(b) (i) (ii) (iv) (iii) (b) accept your colleague’s point of view to end
(c) (ii) (i) (iv) (iii) the argument
(d) (i) (ii) (iii) (iv) (c) calm down and say, ‘let’s agree to disagree’,
39. For effective communication skills, one should and part with your colleague amicably
have attitude which shows traits of (d) walk off as you know you cannot agree with
I. flexibility. your colleague’s view, and staying there could
II. respect for the view of others. lead to a worse situation

2 . 31
Interpersonal Skills including Communication Skills

44. Your colleague who has become a good friend in work and making careless mistakes. You would
invites you for dinner at his house. You reach his as a first step
house on time but find it locked. You wait for a (a) call her into your office and reprimand her as
few minutes and then you mistakes can cause problems and reflect badly
(a) feel hurt at such forgetfullness or casual on your department
behaviour and decide to remain aloof from (b) tell her to take a few days off and assign her
this person from then on work to another person
(b) ring up your wife and tell her to cook (c) talk to her and try and find out the reasons
something for you, even though it is quite late, for her sudden lack of interest
as you are hungry and would be reaching (d) overlook it and check her work on your own
home soon so that mistakes are corrected as you feel such
(c) would ring up your friend and inform him of periods of fall in work happen once in a while
your arrival at his house and ask him if there and will pass off in due course of time
was a mistake about the date or some problem
48. You need to clarify some matter in a document
(d) wait for some more time, sure that your friend
urgently with a colleague as you have to report
will soon come back
on the matter to your superior. You find your
45. Your colleague comes to you and confides that colleague engaged in some important
she is being sexually harassed by another colleague documentation. You would
who happens to be your friend. You would (a) ask for the clarification you require because
(a) ask her to overlook it as such things always
@UPSC_THOUGHTS

it is urgent
happen (b) withdraw and come sometime later
(b) put up a defence for your friend and say the (c) interrupt with a polite “Excuse me, can you
woman has probably misunderstood the spare time to discuss this urgent matter?”
situation (d) try and assess which work is more urgent and
(c) tell her to complain to the authorities act accordingly
concerned
49. You work in a department having to deal with
(d) assure her that you will take it up with your
the public. You have an officer under you who
friend and if the problem continues she should
supervises the staff dealing with the public. Once
complain to the authorities concerned
as you walk through the outer office you notice
46. You have suffered a bereavement and in spite of a member of the staff speaking rudely to an
your own effort to come to terms with it and your outsider who has come with a grievance. You
friends’ efforts to cheer you up, you continue to would
be depressed. You are, as a consequence, unable (a) intervene and reprimand the member of the
to do your work properly. You should staff so that the outsider gets the idea that
(a) pull yourself up; after all one has to move on rudeness will not be tolerated and discipline
(b) take leave from work or resign the job as you is important in the department
are not able to work efficiently (b) go into your office and call up the officer in
(c) seek professional counselling to help you get charge and tell him/her to deal with it
over the depression (c) wait for complaints from the member of the
(d) continue to do what you can at work and public, which is bound to happen
devote yourself to religious pursuits to calm (d) intervene to defuse the situation, reassure the
your mind member of the public, and later call the
47. You find that one of the employees over whom member of the staff and counsel him/her on
you supervise is suddenly showing less interest better behaviour

2 . 32
Interpersonal Skills including Communication Skills

50. You are hard working and ethical in your conduct 53. One of your subordinates often stays back late in
and your department head has always spoken office saying he has to complete his assigned
firmly of the need to behave ethically. Yet, when work. You think he is a diligent worker. One day
the time for recommendation for a new, important you come back to office late to collect something
post comes, your department head puts forward you have forgotten and find this person playing
the name of a colleague whose ethical conduct is cards and drinking with some men unknown to
questionable. Your first step would be to you. You would
(a) go to a higher authority and complain about (a) ignore it as it is after office hours
the unfairness of your department head (b) ask the subordinate and his friends to leave
(b) ignore it and continue to work in your usual immediately and never do such things again
way (c) ask all the other men to leave and ask your
(c) seek out the head of your department and subordinate to give a written explanation of
discuss your grievance informally his conduct
(d) place a formal complaint with the grievance (d) report you subordinate’s conduct to the top
redressal authorities boss
51. You have been in a responsible position in a 54. A young person you know has made a water filter
department for a couple of years and in recent that can be of great use in rural areas, but he has
months you have been receiving feedback from the little capital to invest to make the product
public with which your department deals that marketable. You would
there is dissatisfaction with the service. Your first (a) extend a personal loan to him
@UPSC_THOUGHTS

step would be to (b) ask him to advertise in two national


(a) circulate a note expressing your strong newspapers asking for investment
disapproval of falling services and place a (c) help him get a loan from a bank
deadline for improvement
(d) tell him to approach the sarpanch of every
(b) devise an on-job training programme so as to
village and get orders and advance payment
improve the skills of those who work in your
from villagers to make more of the machines
department
(c) tell your second-in-command to keep a sharp 55. You are new to a department and you have
lookout for those who are at fault and bring several older people who are now your
a list of their names so that you can take subordinates. You should
suitable action against them I. defer to them as they are older than you as
(d) issue a general note to all the members of the one should be respectful to elders.
staff sharing your concern with the fall in II. treat them as individuals, just like anyone
services and expressing the conviction that else, and not as a ‘class’.
things will improve III. have value for their experience and try to
understand what it is based on.
52. You have some urgent personal work for which
IV. make it clear to them that, though younger,
you require leave. Your superior is not ready to
you are their superior now and they should
grant leave. You should
follow what you say.
(a) put in a leave application and go on leave
(a) III and IV
(b) go on your personal work after asking some
(b) II and III
close colleagues to cover for you
(c) II, III and IV
(c) meet and discuss your problem with your
superior and try to convince him/her of the (d) I and II
urgent need of leave 56. You have just joined a new department and find
(d) take it up with a higher authority and put in that most members of the staff come late habitually.
a leave application with him/her Your immediate action would be to

2 . 33
Interpersonal Skills including Communication Skills

(a) put a red mark in the attendance register 59. A colleague who shares the room with you has
against names of latecomers and take them to the habit of taking too loudly on phone on
task personal matters during office hours. He does this
(b) write a letter to each member of staff telling despite several hints from you that it disturbs you.
them that you are aware of the latecomers and You would
that things cannot continue like that under (a) complain about him to a superior official and
you ask for a change of room
(c) call a meeting with the employees and (b) make it clear to him that he is not only being
reprimand them inconsiderate but also unethical by wasting
(d) tell the establishment in-charge to look into official time on personal matters and that you
the matter and to tell latecomers to come in would take it up with the boss if he did not
time stop
(c) start singing loudly so that he too is disturbed
57. You are immersed in urgent work that has to be
and realises that he is doing something
completed in two days’ time, and you know your
annoying and wrong
office is not going to extend the deadline. Your
(d) adjust to the situation as he has not taken any
colleague who is new to the department
of your hints so far and you are not interested
approaches you for advice on how to plan his
in confronting or causing trouble for your
work which too has to be completed fast. You
colleague.
would
(a) tell him that you have no time and point out 60. You are in charge of a department that has a lot
@UPSC_THOUGHTS

another colleague whom he could approach of public dealing. A new official, who is well
for help qualified and efficient, is assigned to the
(b) attend to him as he is new to the department department. He has a bad stammer which becomes
even though your own work could be delayed worse when he speaks with more than one person,
in the process especially if they are strangers. What would you
(c) make it a condition that he helps you finish do?
your task if you help him with advice, as give (a) Assign him more tasks that require speaking
and take are part of life to people
(d) plan out your work in such a way that you (b) Ask the head office to transfer him as he is
can finish it in time and help your colleague unsuitable for your department
as much as time allows though you may have (c) Ask him to take leave and go to speech
to put in some extra hours at work in the therapy sessions
process (d) Assign him documentation or other work that
does not require him to speak with the public
58. A member of the public calls you up and wants
to know about a certain scheme that your 61. A colleague does not do his work properly and
department has introduced. You know all the gets out of doing his assignments often by bullying
details but another officer has been designated to the more amicable colleagues. All of you have
deal with public queries. You would been putting in more labour to do his share and
(a) tell the caller you know nothing about the get the work done in time. He becomes abusive
scheme and taunts and threatens to get his way and
(b) give the caller all the information you have simply ignores any attempt to talk things over.
on the scheme You would
(c) put the caller on to the person designated to (a) take up the matter with the senior officer and
deal with queries ask him/her to intervene as the morale of the
(d) tell the caller the scheme is not worth his/her others and the office environment is
attention deteriorating

2 . 34
Interpersonal Skills including Communication Skills

(b) ignore him and continue to work as before as (a) admit your error, thank the person for having
some colleagues are bound to be incorrigible pointed it out, and continue with your talk
(c) organise other colleagues into a group to gang (b) tell the junior officer to keep silent while you
up on him and under threat of a good are talking and bring his doubts to you later
thrashing make him accept his responsibility on
(d) socially boycott him till he rectifies his ways (c) ignore the interruption and continue with
your talk, and later on issue a memo on the
62. You have taken leave from office to complete some correct data
urgent clearing up work at home. As you are (d) tell the junior officer to explain the programme
working hard, your elderly neighbour drops in and leave the room
and seems inclined to stay and gossip. You
65. You have asked your colleagues to an official
would
discussion over lunch at a restaurant. They are
(a) go on with your work nodding every now and
late by more than 20 minutes. How would you
then as though you were listening to him
react?
(b) feel irritated but stop your work, offer the
(a) You would sulk at the insult of having been
neighbour some tea and entertain him till he
kept waiting, tell them that you are not
chooses to leave
pleased, and will be curt with them all through
(c) spend a few minutes with him and then
the meeting
politely explain you have a lot to do and ask (b) You would smile and make excuses for them
him to excuse you but remind them every now and then in a
(d) readily enjoy the gossip session leaving the
@UPSC_THOUGHTS

sarcastic manner of their late coming


work for another day (c) You would ignore their coming late, and go
63. A member of the public approaches you saying on to lunch and discussion of the matter at
that she has come to you, a senior officer, after hand
failing to get redressal for her grievance for a long (d) You would mention the time and ask if they
time from the staff in your department. What had been delayed by some sudden problem
would you do? 66. You, in your official capacity, have helped an
(a) Tell her you are sorry but the department has unemployed youth to get a bank loan and set up
been very busy and it may take some time to a departmental store, and it is doing good business.
address her problem and request One day, you meet the youth just outside his stop.
(b) You would listen to her complaint, then ask He insists you come in and, as you need to buy
one of the officers in your staff to attend to some things, you agree. After you have chosen a
her immediately and report to you on the few things, the youth declares he will not accept
action taken any payment as he is eternally grateful for your
(c) You would listen to her patiently and ask her help. You would
to write down her complaint again and submit (a) protest but agree in the end as he is just
it to you and you would promise to act on showing his appreciation for what you have
it yourself done for him
(b) accept as a gift symbolising his gratitude but
(d) You would advise patience and tell her it
secretly decide you will reciprocate it soon
takes time to attend to so many grievances
(c) leave all the goods you have chosen on the
brought in by so many people
counter and tell him that you are not the sort
64. You are explaining a programme to your team- to accept bribes and leave the shop
mates and inadvertently quote wrong data. A (d) insist that you will pay for the goods, and
junior officer points out the mistake in front of the assure him that you cannot accept free goods
others. You would for having done your duty

2 . 35
Interpersonal Skills including Communication Skills

67. You as a senior administrative officer have received 70. Your department organises a ‘Health Mela’ every
several complaints about one of your junior officers year. For this year the date has been fixed and
that he does nothing without taking bribes and announced. You are in charge of overseeing all
that he is close to certain bad elements whom he arrangements but just when a week is left two
uses to threaten anyone who opposes him. You team members fall ill and cannot work. There is
would still a lot to be done. You would
(a) ignore such complaints as they are mostly (a) request headquarters to postpone the event
motivated by frustration or jealousy and make the necessary announcement
(b) ask his colleagues about his character and (b) work doubly hard, taking over all the work
behaviour assigned to the two sick persons yourself so
(c) order an inquiry into the officer’s conduct, that everything is ready in time
suspending him till the completion of the (c) take stock of the pending work, see what can
inquiry be cut down, and share pending work with
(d) summarily dismiss the officer as you cannot
the rest of the workers
tolerate corruption and bullying tactics
(d) request headquarters to assign extra helpers
68. You are the team leader in a project that has to so that work can be finished in time
be completed within a week. Information reaches
71. You have to reach office on time as there is an
you that the mother of one of the team members
important meeting to attend. However, though
has died. What would you do?
you are always punctual, that day your vehicle
(a) Keep the information from the person
breaks down and you know you will be late. You
concerned till the project is completed, as it
@UPSC_THOUGHTS

would
is a matter of just a few days
(b) Inform the person concerned, offer your (a) call up the coordinator of the meeting and
condolences, and allow him to go on explain the situation and ask him/her to
compassionate leave at once. inform the senior officer and your colleagues
(c) Ask another team member to pass on the (b) call up a colleague and ask him/her to make
information and tell the person concerned to excuses for you
put in an application for leave (c) not bother to inform anyone but just reach the
(d) Inform the person concerned, offering your venue and slip into a chair at the back
condolences, and allow him immediate leave (d) forget about the meeting and go home and
after getting the details of progress with his report sick
share of the project work so that you can 72. One of your colleagues, Rajan, is inefficient and
continue with what is left.
shirks work. He is not liked by the others. One
69. You are very busy at work when you notice Anil, day you enter the room and hear him unjustly
a junior member of your staff, hovering at the criticising you and your work methods to your
door. You are informed by your assistant that Anil other colleagues. You would
has a problem that he wants to discuss with you. (a) give Rajan a piece of your mind
You would (b) start talking loudly about the pot calling the
(a) immediately leave your work and call Anil in kettle black
and look into his problem (c) tell him firmly not to disturb everyone and
(b) tell your assistant to fix up an appointment otherwise ignore his comments
two days later for Anil to meet you (d) justify yourself to your other colleagues
(c) call Anil in and tell him he should consult pointing out how inefficient Rajan himself is
some counsellor and not waste your time
(d) complete what work is absolutely urgent and 73. You have received an e-mail from your
then call Anil in, listen to his problem and headquarters telling you to send them ten sacks
assure him of your help to resolve it of wheat. You are surprised at the message and

2 . 36
Interpersonal Skills including Communication Skills

feel there must be some mistake as your department (c) give a firm lecture to the entire group on the
has nothing to do with foodgrains or agriculture necessity of the project and how their
or even disaster relief. You would behaviour will not be tolerated
(a) do as you are told as e-mails are not likely (d) meet with teammates one-to-one as well as in
to get distorted small groups and find out what is troubling
(b) send an e-mail seeking confirmation of the them, and address the problems
order from headquarters
(c) call a meeting of your trusted officers to come 77. A junior officer comes to you in a disturbed state
to a decision on it of mind and tells you she is being harassed for
(d) ignore the message as it is obviously a mistake dowry by her in-laws. You know her in-laws and
they have always seemed good-natured and
74. You are at a meeting to discuss possible solutions modern minded, so you are surprised. You would
to a specific problem. The majority of officers are
(a) tell the woman you cannot believe her story
in favour of a certain action. However, you do not
and she had better try to live amicably with
agree. You would
her in-laws
(a) go silently with the majority decision
(b) request her not to bring her personal problems
(b) quietly leave the meeting as you simply cannot
to you
agree with the proposed action
(c) assure her that you will have a talk with her
(c) strongly express your disagreement with the
in-laws and see how the problem can be
majority view and give your reasons for this
(d) put forward your views and offer an resolved
(d) tell her to go to the police
@UPSC_THOUGHTS

alternative plan of action without belittling


the majority view 78. You as team leader are outlining a project to your
75. You have recently taken charge of a department. colleagues when one of them yawns loudly at
On checking the cash book you find shocking which there is some laughter. You would
discrepancies, and the cash in hand is short of (a) reprimand the person who yawned for his
what is projected in the cash book. You would lack of manners and tell him to apologise
(a) call a meeting of all senior officers, state the (b) join in the laughter and make a joke of it and
position and ask them what is to be done return to the matter at hand
(b) demand an explanation from the accounts (c) feel resentful and say that you are leaving as
officer some people seem so bored
(c) report the matter to headquarters and await (d) tell senior officers about the incident and
instructions recommend that the person be removed from
(d) wait for some time before you take action as that team
somebody who needed the cash urgently might
79. You and a colleague of yours are trying to complete
have taken it and may put it back very soon
some urgent work when your colleague receives
76. A project is underway and has to be completed a call from his wife saying their child has met
within a few weeks and there is some stress of with an accident and asks him to reach the
work. You, as the leader of the team in charge of hospital immediately. What would you do?
the project, come to hear that some team members (a) Put aside the work and immediately
are grumbling. You would
accompany your friend to hospital
(a) ignore it; there’s always a little amount of
(b) Tell your friend to leave and you will inform
such grumbling
the senior officer
(b) find out the instigator of the unhappiness and
(c) Tell your friend to leave after informing the
have a serious talk with him and threaten him
senior officer and ask him not to worry about
with an adverse confidential report if he
the work as you will take care of it
persists

2 . 37
Interpersonal Skills including Communication Skills

(d) Tell your friend that his wife will manage and (b) Meet the four officers and delegate the work
the hospital will take care of the child and to them and set a deadline
that he had better put in some more time at (c) Call a meeting with the officers, acquaint
the work which is very urgent them with the project details, and ask for their
inputs on how it can be efficiently completed
80. One of your colleagues suggests to you some
(d) You know you will have to do most of the
procedures that can make office work and
work, so you start working and inform your
communication much easier and more efficient
teammates every now and then
than at present, but she hesitates to tell the senior
officers. You encourage her to present her ideas 83. Meena and Sheena are two junior officers assigned
at the next meeting, but she reports sick on the to you for documenting some research findings.
day of the meeting. You put forward the ideas Each one has her own work to do. While Meena
yourself and they are greatly appreciated. You is always focused on her work and methodically
would goes about it, Sheena spends a part of her time
(a) gracefully accept the appreciation listening to music on her i-pod, doodling or
(b) take the opportunity to let everyone know that reading a novel. The work produced by both at
you get many good ideas the end of the day, is equally good. You, however,
(c) say it was all the grace of god and elders become aware that Meena resents this. You would
(d) acknowledge that the ideas were, in fact, your (a) tell Sheena to focus more on her work and
colleague’s follow Meena’s method
@UPSC_THOUGHTS

81. As a senior officer assigned charge of a new (b) call both of them to your room and explain
department you become aware of some employees that a cooperative spirit is good for the team
who seem to deliberately obstruct the smooth to produce good work
functioning of the office and prevent others from (c) tell Meena that people have different work
doing their work. You would styles and her work is greatly appreciated
(a) call a meeting of all the employees and give and she should continue in her own way
them a lecture on cooperative functioning (d) tell Meena not to work so hard and to relax
(b) call the employees creating trouble to your like Sheena does every now and then to
room, try to find out why they are behaving improve her mood
as they are, and advise them to do their work 84. Jayant is of breezy temperament but efficient. Able
properly to finish his own work fast he has a tendency to
(c) call the employees creating trouble to your go to other officers and chat with them every now
room and shout at them and threaten them and then. The others enjoy this but it prevents
with a transfer to a remote place them from doing their work in time. As a senior
(d) wait for a while to see if things improve before officer you would
taking action (a) reprimand Jayant
82. You have been briefed by your senior officer on (b) call Jayant to your room, express appreciation
a new project and made the team leader and of his abilities and then suggest that if he
assigned a team of four officers to carry out the finishes has own work he could listen to
project. How would you involve the team music or read something or take up a fresh
members? assignment
(a) Call a meeting with the four officers and hand (c) tell the other officers that they should not chat
out the papers documenting the details of the but be as efficient as Jayant
project and tell them to read the papers and (d) ignore the situation, as the other officers after
get on with the work a time will tell Jayant not to disturb them

2 . 38
Interpersonal Skills including Communication Skills

85. You want to change some procedures in the office (b) allow him to apply without the necessary age
which you know will reduce wastage, and for proof document as the project idea is too
which you have the discretionary powers. You brilliant to ignore
would (c) Accept the application provisionally giving
(a) issue a circular detailing the changes and see the candidate time to submit his certificate of
that every officer follows directions age proof
(b) call a meeting of the officers concerned and (d) Tell the applicant that you will use your
explain the changes and your reasons for discretionary powers to process the
effecting them and how they will benefit application without the age proof document
everyone if he says your son is a co-author of the project
(c) call each officer to meet you and explain the
88. One of your colleagues is very good at dealing
changes so that you can know how he/she
with the members of the public when they come
reacts to the information
with their problems to your department. However,
(d) introduce the new procedures on an
she does not have a good grasp over grammar
experimental basis to see how they work
when she writes her reports. This is causing some
before you seek the endorsement of your
friction with the senior officer. What would you
senior officer for them
do?
86. You have been placed as head of a new (a) console her, saying that some senior officers
department. Over time you have come to rely on are always complaining, and tell her to
delegate the report writing to a junior officer
@UPSC_THOUGHTS

a junior officer, X, who is intelligent, hard working


and witty. One day he comes to you and tells you with good language skills
that another officer Y has a nexus with a local (b) suggest that she brush up her language skills
criminal and is making money on the side. You and offer her guidance in this direction
would (c) go to the senior officer and explain that your
(a) take an immediate decision to suspend Y and colleague is good at public dealing and one
institute an official inquiry against him cannot be expected to do well in every field
(b) think X is taking advantage of your friendship (d) offer to write her reports for her as your
and badmouthing his colleague and become language skills are good and you can write
aloof from X fast
(c) tell X not to indulge in telling tales 89. Your senior officer is giving you an outline of the
(d) find out from other sources how far the projects that have to be accomplished in the next
information is true or if there are some personal few weeks. You are not clear about some aspects.
reasons behind X’s leveling charges against You would
a colleague (a) interrupt him and ask for clarifications as
87. You are interviewing candidates for grant of loans later on you may forget what you had to ask
for projects beneficial for rural development. The (b) keep quiet while he is speaking and later take
eligible candidates have to be under 30 years of your doubts to him when he is alone
age. A local youth has a brilliant project idea and (c) keep quiet and later ask some colleagues to
presents you with all the required documents but explain the matter to you
has forgotten the vital document of age proof. You (d) wait till he finishes speaking and then ask
would him about your doubts
(a) suspect that he has something to hide and 90. You have worked hard at a project assigned to
reject his application even though his idea is your department, and everyone in the department
good has shown appreciation for your contribution.

2 . 39
Interpersonal Skills including Communication Skills

When the project is complete, your senior officer all circumstances, but request him to keep it
is given credit for the project’s success by to himself till the policy is announced
headquarters and she quietly accepts it without (c) firmly refuse to tell your friend anything as
mentioning your valuable contribution. You would the matter is confidential
(a) feel hurt and take leave from work for some (d) tell him details that are not confidential but
time to recover ask him to wait for the policy to be announced
(b) take the matter to someone you know at for all the details
headquarters so that people in the right place
93. You have assigned a junior officer the charge of
do not remain ignorant your contribution
relief work in an earthquake affected district. You
(c) go to a friend in the media and ask her to
receive complaints from some people that he is
publicise your effort in the project
siphoning off some of the relief supplies and
(d) take the appreciation expressed for your senior
making money out of it. The officer in your
officer as appreciation for your team and, by
opinion is a man of integrity and is unlikely to
implication, for you as a member of the team
commit such acts. You would
91. Your senior officer has assigned you some work (a) ignore the complaints as being made by some
to be completed in a month’s time. Ten days later, envious people
however, he tells you that the work needs to be (b) replace the officer after explaining the matter
completed earlier and must be handed over in to him and telling him that you personally
another week’s time. You know that it is almost have a good opinion of him but you have to
@UPSC_THOUGHTS

impossible to meet the new deadline. You would be seen taking action on the complaints
(a) start working overtime to get the work (c) make your own inquiries subtly before taking
completed in time as the senior officer has to further action
be obeyed (d) report the complaints to your senior and ask
(b) ask some of your colleagues to help you out for instructions
with the work
94. A new official has been posted as your senior in
(c) acquaint the headquarters with the situation
the department. You find that he sits over files
and request for some more time as it would
and postpones decisions on most matters. This
not be possible to finish the work so soon
delay annoys the members of the public who are
(d) request the senior officer to assign some
affected, and they complain to you about it. You
assistants to help you complete the work in
have tried to get the officer concerned to speed up
time
his decisions, but to no avail. You would
92. You and a close friend of yours are having coffee (a) assure the people that you would do something
at a restaurant when you realise that he is asking about it but do nothing as you cannot argue
certain probing questions about a certain policy with your senior
that your office has been working on and which (b) leak the matter to a friend in the local media
is soon going to be announced. You also realise who would publicise the matter of delay and
that the policy to be announced is likely to have thus make the officer concerned to rectify his
some effect on the company your friend works in. ways
You would (c) take the matter up with a higher officer who
(a) be vague and change the topic and resolve not can deal with your senior
to continue friendship with such a person (d) suggest to the affected persons that they take
(b) give him all the information as he is your their complaints to a higher officer in the
friend and it is your duty to help a friend in department

2 . 40
Interpersonal Skills including Communication Skills

95. You are posted in a city and you do not know 98. You assign a new piece of work to a junior who
the regional language. The staff in most shops objects saying that she does not know how to go
dealing with the necessities of life – groceries and about it. You would
vegetables, for example – know only the regional (a) insist that she does it, as it is a matter of office
language. What would you do? discipline
(a) Point the goods out to a sales assistant and (b) you feel that she is merely making an excuse,
write out the quantity you require so you tell her she has to do the assigned
(b) Do not shop till you get an official interpreter work
(c) Explain your difficulty to a neighbour and (c) explain the nature of the task and give her an
arrange for her to buy you the needed goods assistant to help her
for which you would pay the requisite amount (d) ask her to discuss with you where she lacks
and ask her to teach you essential terms in the knowledge or expertise to tackle the task
the local language
99. You are assigned an office room to be shared
(d) Request a colleague who belongs to the region
with another colleague who already occupies it.
to help you in your shopping till you learn
You want to be near the window, but your table
the language a little
is away from the window whereas your
96. You have been appointed team leader of a project. colleague’s table is next to it. You would
Your team mates, who are your juniors and (a) make slight changes in the position of the
@UPSC_THOUGHTS

somewhat new to the work at hand, point out furniture so that both of you can share the
some problems. You would space near the window for your tables
(a) provide them with clear answers (b) sit in the space assigned to you, resenting
(b) tell them to discuss the problems among your colleague for hogging the space near the
themselves and come up with solutions and window
then come to you for confirmation (c) explain your wanting to sit near the window
(c) get them together and put insightful questions to the senior officer and get his permission
that will help them to think and come up with before making requisite changes in the position
solutions themselves of tables
(d) tell them to leave all the problems to you and (d) place your thoughts on the subject before your
get on with what they can colleague and convince him that suitable
changes could be made without much
97. Your senior officer has delegated to you some
disturbance to his space
official task that not only calls for extra effort but
is also not really your job. You would 100. You are assigning an important and urgent task
(a) seek her out and explain your feelings in a to a junior officer. He says he could simply not
polite manner and suggest a substitute who meet the deadline. You would
would be able to do the work well (a) tell him that, if he does not undertake the task,
(b) complain about it to colleagues and ask them he will get adverse comments on his
to help out confidential report, and assign the task to
(c) accept it as a challenge and do it to the best another person
of your ability (b) tell him to get the help of his colleagues if he
(d) go to a higher officer with your grievance if feels he cannot handle it
your immediate senior is not ready to change (c) immediately extend the deadline and hope he
her mind is satisfied

2 . 41
Interpersonal Skills including Communication Skills

(d) reconsider and, if necessary, break up the task (b) listen respectfully and offer no comment of
into smaller segments and spread out the your own
responsibility among more staff members (c) conceal what you really think
(d) have a frank exchange of views
101. You have just been given an important task by
your senior officer. You have begun working on 104. You are on your way to an important meeting in
it, when you get an urgent message that your your car, but get stuck in a traffic jam. There is
child has met with an accident and is at a certain little you can do to reach on time, as you are too
hospital. You rush to your senior officer’s room far from your destination to walk. You would
to seek his permission for leave, but you find that (a) ring your office and call for the staff car
he has left for a meeting and will not be coming (b) get out and walk fast to try to reach the place
back to office for a while. You ring him on his of the meeting
mobile but you get a switched off message. You (c) call up the person/s you were to meet, explain
would the situation and tell them to wait till you are
(a) leave a message on the senior officer’s table able to reach
and delegate the urgent work to junior officers (d) call up the person/s you were to meet and
and leave for the hospital explain the situation and decide something
(b) leave immediately, as it is an emergency, and mutually convenient
apply for leave the next day and explain to
105. A colleague who has always picked quarrels with
your boss
@UPSC_THOUGHTS

(c) tell some responsible and sympathetic you has a got a better job and is leaving. The office
colleague and family members to attend to arranges a farewell party for him. You would
matters at the hospital and keep you informed (a) contribute towards the party but would make
of the developments as you are not able to an excuse of illness and not attend as you feel
come without leave it to be hypocritical to attend the party when
(d) seek a higher officer and explain the situation you could not get along with the man
to him/her and ask for leave as it is an (b) avoid contributing to the party
emergency (c) you would contribute to the party and attend
the party just as you would in the case of any
102. You have been assigned the duty of inspecting an
other colleague leaving office
over-bridge under construction. When you reach
(d) you would not attend the party but would
the site, you are shocked to see that welding is
express your delight at the prospect of an
being carried out by workers without welders’
glasses and gloves. You would office without his presence
(a) scold the workers and blacklist the contractor 106. You have invited your boss for dinner at 8.30 p.m.
(b) ask the workers to wear their safety gear and You also want to invite a friend, who is a good
work conversationalist, to the dinner. Your friend,
(c) stop the welding work and ask the supervisor however, is in the habit of turning up late at any
why the workers are without safety gear such appointment by some half-an-hour. You
(d) ignore the matter as the contractor knows best would
and some workers are more comfortable (a) regretfully give up the idea of inviting your
without dark glasses and gloves
friend
103. When you are in conversation with a senior (b) tell your friend again and again before the
officer you meet at a social gathering, you day in question to be on time, as it is your
(a) always agree with what she says boss who is coming

2 . 42
Interpersonal Skills including Communication Skills

(c) threaten your friend with a complete boycott (c) take a deep breath and examine the camera
if he is late on this occasion for damage, telling your friend to be more
(d) tell him to come by 7.30 p.m. careful
(d) pretend to take it lightly and tell your friend
107. You are in a group of officers given the task of
not to mind as cameras can be bought again
studying a certain social policy and suggesting
but not friendship, and decide in your mind
some changes. A majority of your team mates
to avoid the person in future
agree upon suggestions A, B and C. You, however,
are convinced that suggestion C is retrograde and 110. You are walking home in a hurry when you notice
should be modified. You would an old man looking vague and walking about
(a) keep quiet as the majority view would anyway aimlessly. You accost him, but he does not seem
prevail, and just write a dissenting note to remember anything. You would
(b) loudly express your disagreement with the (a) go about your business as you are in a hurry
majority view and shout down the opposition (b) collect some other passersby to help you find
to your view as you are convinced it is correct out what the old man means
(c) try your best to convince the others about your (c) guide the old man to nearest police station
point of view, putting forward cogent reasons and explain the situation to the officer on
for the way you think duty
(d) take the old man home with you and see who
(d) keep quiet at the meeting but later on write
reports him missing
a note to senior officers putting forward your
@UPSC_THOUGHTS

ideas on the subject 111. The daughter of a friend comes to you accompanied
by a young man who is an office colleague of hers.
108. You are a person of honesty and integrity.
She says the two want to get married but her
Suddenly one of your colleagues tells you that
parents are objecting because the man belongs to
someone has made a complaint to the senior
a different caste. She asks you for your help. You
officer that you take bribes and undue favours to
would
do your work. You would
(a) tell her to do as she wants, and that you will
(a) feel panicky that such a complaint has been
stand by her as you are against caste
made and that the mud may stick however
discrimination
innocent you may be
(b) tell her to convince her parents that her
(b) ignore your colleague’s words as a pack of
decision is good and that you will help her
lies
in talking to her parents
(c) go immediately to your senior officer and
(c) tell her you are sorry but you would not like
refute the allegations to intervene in her personal matters
(d) wait for word from your senior officer and be (d) tell her that her parents are right and she
ready with what you need to say should listen to them
109. You are hosting a party and your friend drops
Directions: In each of the questions (112 to 121) given
your camera while he is taking a picture. You
below, a word has been used in its different meanings
would
to convey meaningful sentences that are grammatically
(a) feel too angry for words and leave the room
correct. However, one of the sentences is incorrect. Select
so as not to lose your temper and make a
the sentence that is incorrect.
scene
(b) lose your temper and shout at your friend for 112. Round
being so careless, and later apologise to him (a) The constable was on his daily round when
for being angry he saw the accident.

2 . 43
Interpersonal Skills including Communication Skills

(b) The round table was placed in the centre of 119. Strain
the room. (a) She could not stand the strain and had a
(c) He turned and looked round when I shouted. breakdown.
(d) The earth rounds the sun. (b) The writing was strained with humour.
(c) They strained the rope and it broke.
113. Head (d) There is a strain of optimism in his speech.
(a) The officer’s error of judgement cost him his
head. 120. Odd
(a) The odd is that it will rain today.
(b) Matters came to a head when a formal
(b) He has an odd taste in clothing.
complaint was filed.
(c) The numeral 7 is an odd number.
(c) He hurt his head portion and fell unconscious.
(d) She is engaged in doing odd jobs.
(d) He has been chosen to head the committee.
121. Grave
114. Keep
(a) She was engaged in grave thoughts of an
(a) It was a keeping that suited them both. uncertain future.
(b) Even without a regular income, he wished to (b) His image was graven on her mind.
enjoy the comforts of life for keeps. (c) He left behind a grave of unfulfilled ambitions.
(c) What is keeping you from accepting the role? (d) The voice grave informed us of the tragedy.
(d) The documents are safe in his keeping.
Directions: In each of the following questions (122 to
115. Note 129) there are two sentences that attempt to use the
@UPSC_THOUGHTS

(a) He was lucky to strike the right notes at the same word in two different ways. Identify the incorrect
beginning of his job. sentence(s) and indicate the answer option accordingly.
(b) He is a scholar of note.
(c) There is a note of optimism in his report. 122. A. Let us converse with him regarding the matter.
B. What you say is correct but the converse is
(d) He made a note of the development.
also true.
116. Kind (a) A only
(a) They differ in degree rather than in kind. (b) B only
(b) They will prefer the payment in kinds. (c) Both A and B
(c) He is kind to animals. (d) Neither A nor B
(d) It is kind of dark in here. 123. A. He is an invalid who spends his time making
117. Mean beautiful music.
(a) Ram is a person of means. B. Your argument is invalid as you have not
(b) They mean that they are moving out in a studied the issue deeply.
(a) Only A
week’s time.
(b) Only B
(c) That was a mean remark he uttered.
(c) Both A and B
(d) He has a mean position in the class.
(d) Neither A nor B
118. Straight
124. A. The police want to apprehend the accused
(a) A straight path leads to the river. without delay.
(b) The police asked the accused to tell his B. He apprehend his position in front of the jury.
account of the happening straight. (a) Only A
(c) He was willing to do it straights off. (b) Only B
(d) They thought they would have some straight (c) Both A and B
talk to sort out the matter. (d) Neither A nor B

2 . 44
Interpersonal Skills including Communication Skills

125. A. The breach of contract led to tensions between 130. He saw no need to say it again and again.
the two groups. (a) reinforce it
B. The people want a breach of the island from (b) stress it
the mainland. (c) reiterate it
(a) Only A (d) state it often
(b) Only B
131. The officer has been made to bear the blame for
(c) Both A and B
others in the corruption scam.
(d) Neither A nor B
(a) a truant
126. A. The expensive car was only for effects. (b) a scapegoat
B. He wants to bring the plan into effect by the (c) foolish
next year. (d) culpable
(a) Only A
132. They have been kind to an extreme degree.
(b) Only B
(a) infinitely kind
(c) Both A and B
(b) extremely kind
(d) Neither A nor B
(c) over kind
127. A. He had a rough time in the fields. (d) too much kindly
B. He is engaged in rough manual labour.
133. In addition to what is stated, you may note that the
(a) Only A
leave period may not extend to more than a week
@UPSC_THOUGHTS

(b) Only B
(c) Both A and B at a stretch.
(d) Neither A nor B (a) Adding to it
(b) Stating further
128. A. It was a singular success for the company.
(c) Adding to what is stated
B. The girl’s singular behaviour attracted the
(d) Moreover
attention of the passers-by.
(a) Only A 134. There is a need to implant values by repeated
(b) Only B statement or admonition.
(c) Both A and B (a) introduce repeated values
(d) Neither A nor B (b) inculcate values
(c) infuse values
129. A. The imminent lawyer hails from the state of
(d) force values into the mind
Uttar Pradesh.
B. He has slightly protruding eyes and an 135. We warn you that he is a person who only deludes
imminent nose. or deceives people.
(a) Only A (a) humbug
(b) Only B (b) counterfeit
(c) Both A and B (c) cheat
(d) Neither A nor B (d) misanthrope

Directions: In each of the following questions (130 to 136. You must not break or go against the rules.
136) replace the group of words or phrase that is (a) threaten
italicised with word(s) from among the options given (b) oppose
below the question in order to express the sentence (c) contravene
succinctly and correctly. (d) bend

2 . 45
Interpersonal Skills including Communication Skills

Directions: In the following questions (137 and 138) C. The minister’s action was ruled to be ultra
match the groups of words or phrases given in one vires and he won accolades for his bold effort.
column with their one-word meanings given in another (a) Only A
column. (b) Only B
(c) Only C
137. A. Bring a countercharge (i) Contravene
(d) None of them
against an accuser
B. Go against the rules (ii) Circumscribe 141. A. The assembly had a hurried meeting and
C. Enclose within bounds (iii) Prevaricate adjourned sine die.
D. Deliberately misstate (iv) Recriminate B. The media cannot report on the case because
(a) A-i, B-ii, C-iii, D-iv it still sub poena.
(b) A-ii, B-i, C-iii, D-iv C. The session was suspended as there was no
(c) A-iv, B-i, C-ii, D-iii quorate.
(d) A-iii, B-iv, C-i, D-ii (a) Only A
(b) Only C
138. A. Formal acceptance (i) Injunction
(c) Both A and B
into a group
(d) Both B and C
B. Court order compelling (ii) Deputation
one to stop doing 142. A. They signed the contract with the provost that
something the terms can be discussed again later.
C. Act of appointing a (iii) Testimonial B. Fixing of the rates is not per se a matter for
@UPSC_THOUGHTS

person to act for the consideration of the housing committee.


another C. He is a man who can bear misfortunes with
D. Written declaration (iv) Initiation sangfroid.
certifying a person’s (a) Only B
character or conduct (b) Only C
(a) A-iv, B-i, C-ii, D-iii (c) Both A and B
(b) A-iv, B-ii, C-i, D-iii (d) Both B and C
(c) A-i, B-ii, C-iii, D-iv
143. A. The commission is a fait accomplish and
(d) A-i, B-iii, C-iv, D-ii
nobody can ignore it.
Directions: In each of the following questions (139 to B. It was the minister’s resignation that was the
143), select the sentence(s) that are correct and choose coup de main that led the collapse of the
the answer options accordingly. already – tottering government.
C. The agreement is a sine qua non of all
139. A. An ad hoc committee has been constituted to
look into the matter. employment contracts.
B. He stands to be condemned ipso facto. (a) Only B
C. We are the alma mater of the college. (b) Only C
(a) Only A (c) Both A and B
(b) Both A and B (d) Both B and C
(c) Both B and C
Directions: In each of the following questions (144 and
(d) A, B and C
145) a passage has been given that may contain a few
140. A. The student tried to translate the ideas of his redundant expressions. Read the paragraph carefully and
professor ad infinitum. find out whether any redundant expressions have been
B. There is prima facie evidence that the intruder used. These are expressions that are superfluous and need
committed the crime. to be avoided if we want to express ourselves to the point

2 . 46
Interpersonal Skills including Communication Skills

and correctly. Once you have spotted the redundant (a) Only A
expressions, choose the correct option from those given (b) Only B
below. (c) Both A and B
(d) Neither A nor B
144. It is still a matter of debate whether or not persons
who become reinfected can spread the virus to 147. A. “Can we have the true facts of the case?” the
other susceptible persons. However, there is no judge retorted.
dearth of various different arguments and theories B. The company’s expenditure would be to the
on the issue. tune of rupees sixty lakhs.

How many redundant expressions are used in the Which of the above sentences has/have redundant
above paragraph? expressions?
(a) One (a) Only A
(b) Two (b) Only B
(c) Three (c) Both A and B
(d) Four (d) Neither A nor B
145. More and more people are opting for online 148. A. He thought to himself that the best way out
shopping, something that is seen as preferable to of the situation was to get legal help.
actually sweating it out in the sun. It is argued B. He, along with his friend, was left stranded
that usually, people normally enjoy shopping not on the highway.
@UPSC_THOUGHTS

so much for what they buy but for the ‘experience’


of shopping—lazily sauntering from shop to shop, Which of the sentences has/have redundant
seeing varieties of things, voicing their opinions expressions?
and finally settling for what they see as the best. (a) Only A
Unless and until this experience is enjoyed, (b) Only B
shopping is not quite shopping. (c) Both A and B
(d) Neither A nor B
How many redundant expressions can you find
in the paragraph? 149. A. The boys have successfully developed a gadget
(a) Two that can locate hidden land mines.
(b) Three B. The company’s assurance that it would not
(c) Four retrench its employees provided a brief respite
(d) Five to them.
Directions: In each of the following questions (146 to Which of these sentences has/have redundant
153) there is a set of sentences that may or may not expressions?
contain redundant expressions. Locate the redundant (a) A only
expressions—words/phrases—and select the correct option (b) B only
from those given below the sentences. (c) Both A and B
146. A. The scientist’s untimely death proved a huge (d) Neither A nor B
setback. 150. A. If no other claimant comes forward, the right
B. He is undoubtedly the best musician in the to the money will revert back to him.
circuit. B. The reason why the police are finding it
Which of the above sentences has/have redundant difficult to apprehend the criminal is that he
expressions? or she has left absolutely no trace behind.

2 . 47
Interpersonal Skills including Communication Skills

Which of the above sentences has/have redundant Directions: Each of the following questions (154 to
expressions? 162) has a sentence. The sentence may be rewritten in
(a) Only A a terse and correct manner, making the sentence more
(b) Only B direct, exact and effective. In each question, see whether
(c) Both A and B the sentence can be rewritten in an improved style. If
(d) Neither A nor B it needs to be rewritten, select one of the options as
151. A. The people in the colony are keen to put an the correct form. If it is terse, simple and correct as
end to the monkey menace. it is, choose option (d) wherever the option is given.
B. It is possible that she may decide to go after Remember, the meaning of the sentence should not
all. change in the course of rewriting it and there should
C. The Irish talk of their past history with great be no grammatical error in the rewritten sentence.
pride.
154. There is a lot of access to the site where men and
Which of the above sentences has/have redundant materials are concerned.
expressions? (a) Men and materials can reach the site readily.
(a) Only A (b) There is adequate access for men and materials
(b) Only A and B to the site.
(c) Only A and C
(c) The site is largely open to men and materials.
(d) A, B and C
(d) Where men and materials are concerned, there
152. A. The understanding between the parties was a is a lot of access to the site.
@UPSC_THOUGHTS

verbal understanding that was never written


155. The publishing house, which is a leading publisher
down.
of books in the city, brought out a hundred
B. Inside each folder is one or more sheets of
publications last year.
information.
(a) The publishing house, a leading publisher of
C. The search for new breakthroughs has spurred
books in the city, has published a hundred
extra spending in recent years.
publications this year.
The redundant expressions are in: (b) The publishing house, a leading publisher of
(a) Only A and B books in the city, brought out a hundred
(b) Only A and C books last year.
(c) Only B and C
(c) The publishing house brought out a hundred
(d) A, B and C
publishings last year.
153. A. The company is throwing a party which may (d) No improvement required.
well be one of the most expensive ones to be
156. The company has developed a new gas lighter.
held this year.
(a) The company has evolved a new gas lighter.
B. The joke was so funny that she laughed and
(b) The company has created a new gas lighter.
literally choked on her food.
(c) The company has designed a new gas lighter.
C. This book isn’t very good, but neither are any
(d) No improvement needed.
of the others available in the shop.
Which of the above sentences has/have redundant 157. The police decided to activate a search for the
expressions? miscreants by utilising various means including
(a) Only A and B the media.
(b) Only A and C (a) The police wanted to start searching for the
(c) A, B and C miscreants by employing various means
(d) None of them including the media.

2 . 48
Interpersonal Skills including Communication Skills

(b) The police searched for the miscreants by 162. I speak with all sincerity that I would have done
using means like the media. it in gladness.
(c) The media and other means were used by the (a) I speak sincerely that glad is the manner in
police to search the miscreants. which I would have done it.
(d) The police decided to start a search for the (b) I say with all sincerity that I would have done
miscreants by using various means including it gladly.
the media. (c) I say sincerely that gladly I would have done
158. We are endeavouring to construct a society that it.
is more inclusive. (d) No improvement needed.
(a) We are wanting to build a society that has Directions: In the following questions (163 to 170),
inclusion. each passage consists of six sentences. The first sentence
(b) We are endeavouring to build a society that
(S1) and the sixth sentence (S6) are given in the
is more inclusive.
beginning. The four sentences in middle in each have
(c) We are trying to form an all encompassing
been removed and jumbled up. These are labelled P, Q,
society.
R and S. You are required to find out the correct logical
(d) We are going to make a society in which no
order of the four sentences.
one is left out.
159. Overtime went up as a result of the strikes held 163. S1 : Indian media still has not matured as far as
recently. election coverage goes.
@UPSC_THOUGHTS

(a) Overtime increased as a result of the recent S6 : There is nothing wrong in the demand for
strikes. unbiased, non-casteist and non-communal
(b) As a result of the recent strikes, overtime went reporting.
up. P : Today, it is common for public relations
(c) The recent strikes shot up the overtime. firms to cultivate journalists and plant
(d) No improvement needed. stories.
Q : At the same time, nobody has addressed the
160. The machine will cost rupees six lakhs but save
menace of corruption in the media itself.
an annual Rs. 50,000 on administration.
R : In such a scenario, the reader feels
(a) The machine will save Rs. 50,000 annually on
shortchanged as unbiased election coverage
administration and cost only rupees six lakhs.
is not available.
(b) The machine will cost rupees six lakhs
annually but save Rs. 50,000. S : Press conferences are used openly for giving
(c) Though costing rupees six lakhs for the gifts and bribes to journalists.
administration, the machine will save Which one of the following is the correct sequence?
Rs. 50,000 on an annual basis. (a) PQSR (b) QPRS
(d) No improvement needed. (c) PQRS (d) QPSR

161. I determined my mind to say the truth even 164. S1 : Over decades, we have made things a lot
though I had a good cause for not doing so. worse.
(a) I determined my mind to speak truly even S6 : In the end, it can destroy the entire village.
though my excuse was good for not doing so. P : It has proved quite disastrous.
(b) I determined my mind to speak the truth Q : The unregulated spread of borewells was an
though I could have avoided it. early form of water privatisation.
(c) I determined to speak the truth even though R : Many poor farmers have seen their dug
I had a reasonable excuse for not doing so. wells sucked dry as neighbours collar all the
(d) No improvement needed. groundwater.

2 . 49
Interpersonal Skills including Communication Skills

S : The richer you are, the more wells you can P : As Ramananda came down the steps before
sink, the deeper you can go. daybreak for his usual bath, he trod on the
Which one of the following is the correct sequence? sleeping man.
(a) QSPR (b) PQRS Q : Kabir at once jumped up and threw himself
(c) QPRS (d) PSQR at the feet of the preacher.
R : “Ram, Ram”, he exclaimed in astonishment.
165. S1 : Sachin has scored centuries against all
S : One dark night, Kabir went to the ‘ghat’ and
oppositions, in all countries.
lay down on the river steps.
S6 : But largely, he has been the dictator, giving
Which one of the following is the correct sequence?
nightmares to quality bowlers of the world.
(a) PQSR (b) PRQS
P : There have been those rare occasions when
(c) RQPS (d) SPRQ
he looked entangled at the crease.
Q : In fact, some of his most memorable essays 168. S1 : Smoke oozed up between the planks.
came in adverse conditions. S6 : Most people bore the shock bravely.
R : Gavaskar called him “the closest thing to P : Passengers were told to be ready to quit the
batting perfection this game has ever seen”. ship.
S : Sachin has established his stamp over all Q : The rising gale fanned the smouldering fire.
types of attack. R : Everyone now knew there was a fire on
board.
Which one of the following is the correct sequence?
S : Flames broke out here and there.
(a) PQSR (b) RQSP
Which one of the following is the correct sequence?
@UPSC_THOUGHTS

(c) PSQR (d) RSQP


(a) QPSR (b) QSRP
166. S1 : Diabetes is a silent killer and because it does (c) RSPQ (d) SRQP
not seem as potent as cancer or AIDS, it is
169. S1 : Most of the perishable foods are shipped by
very often overlooked without too much
refrigerator ships.
heed by patients.
S6 : Ripe bananas are poor travellers and even
S6 : Though diabetes can never be totally cured
one ripe banana at the start of the trip can
it can definitely be controlled from causing
spoil a whole shipload of fruit.
serious consequences.
P : They are placed in the refrigerated hold of
P : Worse, as many as a third of them don’t the ship.
even know it. Q : Some foods, such as bananas, are shipped
Q : Untreated diabetes can lead to heart and before they get ripe.
kidney failure, amputations and even death. R : As the green bananas are loaded, a man
R : Millions of people suffer from diabetes. watches closely the signs of yellow on them.
S : The early symptoms of diabetes are often S : The cool temperatures keep the bananas
confused with other less grave conditions. from getting ripe during the trip.
Which one of the following is the correct sequence? Which one of the following is the correct sequence?
(a) SQRP (b) RPSQ (a) PQRS (b) PSQR
(c) SPRQ (d) RQSP (c) QPSR (d) SRPQ
167. S1 : Kabir knew that Ramananda got up very 170. S1 : Evolution is not progress.
early in the morning and went down on the S6 : For, like progress, evolution does, over the
steps of the ‘ghat’ to bathe in the waters of long run, imply betterment.
the sacred Ganges. P : And yet, for all their differences, it is not
S6 : He said, “You have given me the mantra, wholly wrong to identify evolution with
‘Ram, Ram.’ I have become your disciple.” progress.

2 . 50
Interpersonal Skills including Communication Skills

Q : As a noted scientist had said, “The tapeworm S : We hardly realise what pranks the birth rate
in its inglorious lot in man’s intestine is an plays with our theories and our arguments.
outcome of evolution as well as the lark at T : Voltaire preferred monarchy to democracy
heaven’s gate.” on the ground that in a monarchy it was
R : Three hundred million years after the first only necessary to educate one man; in a
land creatures crawled out of the sea, the democracy you must educate millions, and
one-celled amoeba is man himself. the grave digger gets them all before you can
S : The physical facts of evolution betray such educate ten per cent of them.
advance. (a) PQSTR (b) SRTQP
Which one of the following is the correct sequence? (c) TSPQR (d) TRSQP
(a) QPSR (b) RPSQ
173. P : The state replaces spontaneous society and
(c) SPQR (d) SRQP
the corporation replaces the small dealer.
Directions: Each of these questions (171 to 175) Q : The aggregation of people in cities breaks
consists of four or five sentences followed by four down neighbourhood morality as a source
sequential arrangements. Select the best sequential of spontaneous order.
arrangement as your answer. R : Every egoistic impulse is free in the protecting
171. P : How much more depends on the current anonymity of the crowded.
uses to which the rupee can be put. S : The developing complexity of life has bound
Q : Conversely, at a 5 per cent interest rate, a us into a highly integrated whole, and has
@UPSC_THOUGHTS

rupee received a year from now is worth taken from us that independence of parts
only Re. 1.05, or 95.2 paise today; at a rate which once was possible when each family
of 10 per cent, 90.9 paise. was economically a self-sufficient
R : The theory of discounting the future is sovereignty.
simple: a rupee received today is worth T : Where natural order is still powerful, as in
more than a rupee received tomorrow. simple rural communities, little law is
S : If it can earn 5 per cent interest, a rupee necessary; where natural order is weak, as
today will be worth Re. 1.05 after a year; if in sprawling cities, legislation grows.
10 per cent, Re. 1.10. (a) STRQP (b) TSRQP
T : The determination of a future rupee’s present (c) QRTSP (d) TQPSR
value is, according to the accepted theory, 174. P : So money is beautiful rather than useful to
the appropriate way to compare future the miser.
benefits with present costs. Q : Obviously beauty, as distinguished from
(a) RQPST (b) RTPQS use, is bound up with a certain keenness of
(c) RPTQS (d) RPSQT satisfaction that reflects the intensity of
172. P : Nearly all of each generation are brought up desire.
in homes where the income is too small to R : Ugliness lowers our vitality, and disturbs
provide for the luxury of knowledge. our digestion and our nerves; it may produce
Q : The minority acquire education, and have nausea, or make poets call for a revolution.
small families; the majority have no time for S : Hence the beauty of light and rhythm and
education and have large families. a gentle touch.
R : Hence the perennial futility of political T : Anything takes on beauty, if it stimulates
liberalism: the propaganda of intelligence and invigorates the organism.
cannot keep pace with the propagation of (a) TPSRQ (b) RSTQP
the ignorant. (c) SRTQP (d) QRSTP

2 . 51
Interpersonal Skills including Communication Skills

175. P : The beautiful, then, is in its lowest stages the 177. Which of the following should be the sixth (last)
sensory aspect of that which satisfies a sentence after rearrangement?
strong desire. (a) F (b) E
Q : Anything that meets a fundamental need of (c) D (d) C
our natures has in it certain aesthetic
178. Which of the following should be the second
possibilities.
sentence after rearrangement?
R : At bottom, it differs from the useful only in
(a) F (b) E
the intensity of our need.
(c) D (d) C
S : To the author who has struggled for years
to find the way into print, his first published 179. Which of the following should be the third
page will seem to him a thing of compelling sentence after rearrangement?
beauty, but to a farmer or an artisan who (a) F (b) E
has healthier ambitions than to write books, (c) D (d) C
the same page may only be of use to wipe
180. Which of the following should be the first sentence
his razor on.
after rearrangement?
T : A plateful of food is beautiful to a starving
(a) F (b) E
man as a pretty girl to a young Romeo; let
(c) D (d) C
the young Romeo be starved, and his
aesthetic sense will be dulled even to the 181. Some new procedures have been evolved for your
loveliest nymph; he will consider her only department which all officers in the department
@UPSC_THOUGHTS

as something good to eat. need to know of. Which of the following would
(a) PQRTS (b) QTSPR be an effective way of ensuring everyone concerned
(c) PRSQT (d) TSRQP becomes aware of the procedural changes?
(a) Calling a meeting and presenting the changes
Directions: Rearrange the following sentences to form
and inviting comments
a meaningful paragraph and then answer the questions
(176 to 180) below them. (b) Advertising in the local newspaper
(c) Using a bulletin board to publicise the
A. For other wishes, it enlists the help of those who
procedures
make or own what the child desires.
(d) Bringing out a handbook of the procedures
B. It grants some wishes with its own funds
and giving one to each officer concerned
depending upon the availability of funds.
C. I was deeply moved when I learnt about the 182. You have introduced a new scheme to benefit the
activity of “Make-a-Wish Foundation”. people in nearby villages, and want to further
D. From parents, friends, or hospital attendants, the improve the scheme to meet the people’s needs.
foundation learns about the child's wish for How would you seek this feedback?
anything from a special toy to a visit to I. Ask for e-mails to be sent to a special e-mail
Disneyland. id created for the purpose.
E. I am sure you also now must have been moved II. Set up suggestion boxes at the reception of
by the noble act of the foundation. your office and convey to the villagers that
F. It grants the wishes of children who are terminally they may drop their suggestions in these
ill. boxes.
176. Which of the following should be the fourth III. Organise meetings every day in the villages
sentence after rearrangement? to get people’s ideas directly.
(a) F (b) E IV. Tell the sarpanchs of the villages to collect the
(c) D (d) B people’s suggestions and let you know.

2 . 52
Interpersonal Skills including Communication Skills

(a) I only manual is to be released next day at a function.


(b) II only You would
(c) I, II and IV (a) request a postponement of the function and
(d) II, III and IV release of the manual as the mistake needs to
be rectified
183. The district where you work as an administrative
(b) allow the release of the manual with a slip
officer is facing water shortage. You want people
attached indicating the error on the particular
to reduce the use of water and use it with care
page, and rectify the mistake for the next print
where necessary. What would you do as a first
run
step?
(c) take the printer to task and threaten to blacklist
(a) Whenever you see water being wasted, you
him
will talk severely to the persons who are
(d) keep quiet about it as the error is not serious
wasting water and tell them to be careful
and no one would probably notice it
(b) Distribute pamphlets and use local radio and
television to spread the message 187. You are conducting a workshop on cultural bias
(c) Put up a warning in prominent places that among people. The participants are all from
those caught misusing/wasting water will different places and do not know each other. How
have their water supply cut off will you make it easy for the people to participate
(d) Restrict supply of water to 2 hours a day to constructively and lose their inhibition to
the households contribute to the discussion?
@UPSC_THOUGHTS

(a) Crack a Sardarji joke to make people laugh


184. As coordinator of a meeting it is your main
(b) Smile at all of the participants
responsibility to
(c) Get the participants to introduce themselves
(a) inform everyone who is supposed to attend
briefly and tell about some weakness they
about the venue and time of the meeting
have
(b) choose the participants of the right kind
(d) Get the participants to suggest incidents of
(c) make sure that all participants attend the
cultural bias
meeting
(d) follow up absentees and ask for an 188. You have gone to a nearby village to speak about
explanation for their absence a welfare scheme recently introduced by the
government. You have given the assembled
185. As a public official probing the alleged criminal
villagers an outline of the scheme and then asked
activities of a senior officer, you would
for their comments and doubts, if any, regarding
(a) keep the media updated every few days on the
the scheme. You receive blank looks and a general
progress of the investigation
silence broken by some murmurs. You would
(b) call a press conference to reveal anything
(a) assume that everyone likes the scheme and no
sensational that you find
one has any comment to make
(c) keep aloof from the media even if they pester
(b) think the villagers are afraid of speaking out
you for details
against government schemes for some reason
(d) give only such information to the media, if
and encourage them to express their ideas
asked, that would not affect your investigations
(c) wonder if the audience has understood your
if publicised
language, and would ask a local officer or
186. The manual you have got printed has to be official to interpret what you have said in the
distributed to the public. You discover a minor local dialect
grammatical error which, however, does not have (d) think the villagers are uninterested, and take
any impact on the meaning of the sentence. The leave as you have done your duty

2 . 53
Interpersonal Skills including Communication Skills

189. A colleague is talking to you about a programme (d) tell your colleague that you are interested
she is organising for the department for the in what she is saying and could both of
welfare of the drought affected. You are getting you talk some time later as you have to
late for a meeting. You would attend an urgent meeting
(a) look pointedly at your watch hoping your
190. You need to communicate an important change
colleague gets the hint that you are getting
in the policy of your organisation to your junior
late officers. What mode/s of communication would
(b) look surreptitiously at your watch to see you choose?
how much time you can spare but smile (a) Written note
brightly and nod as you do not want to (b) Oral communication
hurt your colleague’s feelings (c) oral communication followed by a written
(c) tell your colleague frankly that you are in note
a hurry to attend a meeting so you cannot (d) oral communication followed by a notice in
stop to listen to her the local media

@UPSC_THOUGHTS

2 . 54
Interpersonal Skills including Communication Skills

ANSWERS AND EXPLANATORY NOTES

1. (c) Understanding one’s own and others’ emotions popularity, especially in the urban corporate
are crucial to good interpersonal skills. scenario, even today there are cultures in
2. (d) Self-awareness leads to discovery of passion, which a bow or folded hands with palms
recognition of motives, adjustment, together (namaste) is more common. Indeed,
agreeableness, conscientiousness, and in India, even among relatively modern
openness to experience. educated groups, people adopt the namaste,
especially where women are greeted.
3. (a)
20. (b) This will help build a cohesive team which
4. (d) Apart from having the traits given in I and
in turn works best. Being aware of and
IV, the person with high EQ is open to the
respecting cultural differences is not the
ideas of others and has a balance between
same as emphasising them or for that matter
personal and professional life.
identifying with them and supporting them. So
5. (d) options (a) and (c) are not appropriate.
6. (a) An effective leader establishes trust with the Resolution of misunderstandings is a team
followers and motivates them. leader’s function any way, irrespective of
7. (d) Interpersonal communication includes all whether team members come from diverse
aspects of communication. socio-cultural groups or not. Moreover,
misunderstandings do not occur only because
8. (a) of cultural differences.
@UPSC_THOUGHTS

9. (a) For effective cross-cultural communication, 21. (d) Do not confuse assertiveness with being
the use of stereotypes to explain the behaviour aggressive.
of different cultures is avoided.
22. (c) 23. (a)
10. (d)
24. (a) Communication cements the superior-
11. (c) Accepting one’s weakness is a self-awareness
subordinate relationship.
skill, not an effective goal-setting skill.
25. (d) 26. (d)
12. (a)
27. (b) Formal communication is rigid as deviations
13. (a) Good decision-making also involves putting
are not allowed.
the decision into action.
28. (a) 29. (b)
14. (c)
30. (a) How the body is postured is body language
15. (a) A good problem solver encourages creativity
which is a kind of non-verbal communication.
from participants.
31. (c) Feedback is the response from the receiver
16. (d) 17. (c)
which enables the sender to determine whether
18. (b) Stress in an organisation may lead to reduced the message was received and understood
production. as originally intended.
19. (a) Most interpersonal skills counsellors will 32. (a) Integrity of purpose facilitates communication.
consider maintaining eye contact as a
33. (d)
universal sign of paying attention, and therefore
necessary, but in some cultures, avoiding eye 34. (a) A good communication system should be
contact or looking at the ground while talking flexible enough to adjust itself to the changing
to elders or one of higher social status is requirements.
considered not just normal but a sign of 35. (c)
respect. Maintaining direct eye contact may
be construed as confrontational or disrespectful 36. (d) Statement I is projective listening and
(on the part of the doer) in these cultures. statement II is false listening.
As for III, though the handshake is gaining 37. (d) 38. (c)

2 . 55
Interpersonal Skills including Communication Skills

39. (d) casually without finding out more. Response


40. (b) Try to prepare for an interaction beforehand. (c) is not incorrect; however, in the
circumstances, you may pause before going
41. (c) The other options are not correct because of
this advice as (i) it is your friend who is being
the use of ‘only’ or ‘always’. If the superior
accused and you will want to give him a
gives permission or specifically demands it,
chance to explain things, and (ii) the woman
one may dispense with the gender-specific
has probably approached you because you
title (Sir, Madam or Mr/Mrs/Miss …). With
are a friend of the accused colleague and,
subordinates and colleagues, once a familiarity
may be, would be able to resolve the matter
has been established, or in informal situations,
without making it a formal issue. If response
first names may be used.
(d) had not been there, (c) would be best.
42. (d) Both I and II may be misinterpreted. Specially
46. (c) Response (a) merely states something which
I needs great caution in interpreting as it
you are unable to do. Response (b) is not
depends on cultural norms, personal style and
going to help, as you are likely to sink further
physical setting. Tone of voice too can
into depression. Response (d) would be unfair
mislead, as nasty things can be said in a
to the job you have if you are not able to
sweet tone. Even III can be—as words can
give your full attention to work. One should
be taken to mean different things. However,
be ready to admit to weakness and
of the three, verbal-written is the most
psychological problems and seek professional
reliable—one can always check it.
help to overcome them. Such a step would
43. (c) If views are divergent, it is best to agree to be beneficial to you as well as those who
disagree and not prolong the argument. have to interact with you. So response (d)
@UPSC_THOUGHTS

Response (a) will only lead to more is best.


unpleasantness. Response (b) implies you
47. (c) It is best to try and find out what is bothering
are not firm in your convictions or that you
the person, so that some suitable remedial
will resort to dishonesty to escape an
steps could be taken.
unpleasant situation. Response (d) would be
the last resort—that is, if your colleague 48. (c) This gives the colleague time to adjust to
continues to be vociferous even after you your request, though it is understood that an
have suggested that the argument be left off. ‘urgent’ request will generally get priority. It
gives the receiver of the request the sense
44. (c) You have to give your friend the benefit of
of judging independently the importance of the
the doubt—that he would not have committed
request and agreeing to further interaction.
such a discourtesy without a good reason.
So it is best to find out what happened. 49. (d) Reprimanding a staff member in public is poor
Response (a) shows an over-reaction; your interpersonal skill. Nor can you have a hands-
initial reaction should be of bewilderment off approach here, as you have observed a
rather than ‘hurt’. Instead of feeling concern situation in which a staff member’s behaviour
that your friend may be in some difficulty, is questionable, and a member of the public
you are self-centred in your reaction. Response requires attention.
(b) is a little inconsiderate to your wife; you 50. (c) Your first step should be to bring it to the
could easily eat at some hotel on the way ears of your department head that you are
home or make yourself a snack when you surprised at the decision and discuss the
reach home. Response (d) is a waste of time matter with the head. There may be some
and will achieve nothing constructive without specific reason for the decision; in any case,
further information, as you have already your department head will get to know how
waited for some time. you feel. Response (a) may be resorted to
45. (d) Response (a) shows you to be insensitive to if such oversight continues to happen even
gender issues. Response (b) may be natural, on later occasions. Response (b) could also
as the person being accused is your friend, be acceptable if you are not too bothered
but it is not correct to dismiss such a charge about the head’s decision and also because

2 . 56
Interpersonal Skills including Communication Skills

the decision of a superior is seldom questioned 60. (d) There is always likely to be work that does
or reversed. Response (d) must come after not require direct public dealing. Response (a)
you have been overlooked a few times and may be suitable in a training session where
your informal efforts have failed to get you this would be a means to create confidence
a satisfactory explanation. in a stammerer but in dealing with the public
on behalf of a department this move may be
51. (d) This ought to be the first step. Option (a)
counter productive leading to the members of
besides being authoritative as a first step
the public becoming impatient. Response (b)
may even place you at a disadvantage—what
shows callousness on your part towards
if the deadline passes and things don’t
disability as well as your lack of
improve? Do you have some back up action
resourcefulness as a leader. Response (c) is
in mind? Option (b) could come later, but not
not appropriate; you could counsel him to
as a first step. Option (c) is a little underhand,
take speech therapy sessions in his spare
and may put your second-in-command in an
time but not ask him to go on leave.
uncomfortable position with colleagues. It
would be different if you asked him or her 61. (a) There are times when it becomes necessary
to talk to and counsel those against whom to involve the senior officer. Response (b) is
defeatist and lets an errant person get his
there are specific complaints, if any.
way and enjoy himself at others’ expense.
52. (c) Anyone who is part of a team must put in
53. (c) Office premises cannot be used in such a his/her best effort and it is unfair that others
way so (a) is not correct. Response (b) is do his/her share always when there is no
not effective. Response (c) is better than need for it. Response (c) would make you
liable of taking the law into your own hands
@UPSC_THOUGHTS

response (d).
and being violent. Option (d) is futile; it seems
54. (c)
unlikely that such an action will lead to the
55. (b) In a work set up, age does not or ought not person concerned changing his ways.
influence respect. Respect as individuals is
62. (c) This is the best response and shows you can
due to one and all. So I is not correct. IV
handle an awkward situation frankly and
shows poor interpersonal skill. Anyway having
effectively. Response (a) shows you consider
been around longer than you, they must be the other person as a non-entity even as you
aware of hierarchy and would know the rules. adopt an escapist route out of the situation.
56. (d) Option (a) is rather pushy, and option (b) Response (b) shows you to be ineffective at
somewhat impractical if the staff is large. handling an awkward situation while response
Option (c) is not required for this kind of lapse (d) indicates you lack a commitment to work
specially as a first step. that needs to be done.
57. (d) 63. (b) The other responses are delaying tactics.
58. (c) Even though you know the details, making 64. (a) Errors can be made by anyone, and to accept
the person contact the designated official is a mistake does not adversely affect your
the correct procedure. Responses (a) and (d) reputation. Also, it is best for all to know the
show a serious lack of commitment to your correct figures. Response (d) shows you are
work and department. Response (b) is unable to tolerate even valid criticism.
assuming duties that have not been assigned 65. (d) It is impolite of your colleagues to be late
to you. for a meeting, so you cannot ignore it.
59. (b) Since hints have not worked, this is the next Response (a) and (b) shows immature and
step. Of course, if he still persists, option rude behaviour on your own part. Response
(a) would have to be taken up. Option (c) (d) is best—polite as well as assertive.
could easily lead to an unpleasant situation 66. (d) While (a) and (b) are not ethical behaviour,
besides disturbing other people as well. (c) is unnecessarily rude for the youth’s
Option (d) shows you are ready to give up intention may be genuinely to show his
easily even when you are in the right. appreciation rather than to offer a bribe.

2 . 57
Interpersonal Skills including Communication Skills

67. (c) Responses (a) and (b) are ineffective. As for resourcefulness. Calling a meeting on the
response (d), everyone deserves a fair inquiry matter is a waste of time and will be futile.
before action is taken, and you probably will 74. (d) If you disagree with some view, you must
not have the power to dismiss any officer express yourself and try to convince/persuade
summarily. others to that point of view without belittling
68. (d) As the project is also important, the team others’ views. Responses (a) and (b) show
member’s inputs have to be taken before he you are too submissive and unwilling to
proceeds on leave. So response (b) is accept challenge, while (c) shows avoidable
incomplete. While response (a) is aggressiveness.
unscrupulous, response (c) shows you shirking 75. (b) The accounts officer needs to explain the
your responsibility as team leader besides discrepancies. Response (a) is not relevant
showing insensitivity to a person who has and response (c) is unnecessary waste to
suffered bereavement. time. Response (d) is questionable conduct
69. (d) The efficiency of workers depends on their on your part. Money that belongs to the
morale and you as a senior officer must be officer cannot be taken by anyone for personal
ready to help out juniors. While response (a) use without procedural rules being followed.
does not show a balanced approach, response 76. (d) Leading a team calls for cooperation and
(b) is not urgent enough to show concern. understanding from all members, and the
Response (c) is unwarranted as you do not leader must be positive in his/her attitude in
know the nature of the problem. considering the problems he/she has.
70. (c) Your first step should be to assess how much Responses (b) and (c) show high-handedness
@UPSC_THOUGHTS

of the work can be left out so that the event and aggressiveness while (a) is not advisable—
can be good even as workload becomes less. such grumbling would adversely affect the
Then the pending work should be fairly efficiency of the team.
shared. Response (a) is impractical just a 77. (c) Here, as you know something about the in-
week before the event, specially as the laws which does not accord with what the
circumstances are not so serious. While (b) junior officer says, response (c) seems better
can be your response, it is unnecessarily than response (d). Response (a) is biased.
burdening (and stressing out) yourself. While it is true that you as an officer should
Response (d) would require time—which you not interfere in personal matters of your staff,
do not have. here an officer has come to you of her own
71. (a) Clearly you must call up and inform the proper accord, and has spoken of a matter that is
authority which is the coordinator of the legally wrong, if true. So (b) is not correct.
meeting. Response (b) may be resorted to 78. (b) Annoying situations should be managed
if you are unable to get through to the tactfully, and not by losing one’s cool and
coordinator. Response (c) shows you do not over-reacting.
like to face up to things while response (d) 79. (c) 80. (d)
shows you to be unethical and irresponsible.
81. (b) This ought to be the first step.
72. (c) This particular colleague is inefficient and is
82. (c) You have to work as a team and your
not liked by the others. So there is no need
colleagues’ inputs will help make the delegation
to justify yourself or demean yourself by
of work easier and contribute towards
responding to his comments.
efficiency.
73. (b) If you are in doubt about an order you have
83. (c) So long as the assigned work is done well
to confirm it from the source so that you can
in time, one cannot complain. Response (b)
be sure of the message. You cannot ignore
is not relevant as you are told that each one
it as you cannot be sure it is not what
has her own work.
headquarters wants. Just to go ahead and
carry out the order shows a lack of 84. (b)

2 . 58
Interpersonal Skills including Communication Skills

85. (b) Response (a) is too abrupt when changes are discussed the matter with him first; this is
sought to be introduced. Response (c) is a not correct.
waste of time; even in a general meetings 92. (d) Of course, confidential information should not
reactions can be gauged and doubts, if any, be revealed to anyone, but you can be tactful
settled. As you have discretionary powers to as well. However, even as you learn to be
effect the changes, there is no need to ask on guard with people, there is no reason why
for the endorsement of a senior officer, so you should stop being friends with anyone in
response (d) is not correct. the circumstances.
86. (d) You cannot accept accusations on their face 93. (c) You should not ignore complaints, however
value even if they come from someone you unreasonable they seem. It is your duty to
consider reliable, especially if the charges are attend to them and reach a fair conclusion
against another colleague about whom you about them. Replacing the officer would send
know nothing negative. Not can you reject the wrong signals to outsiders and cast
serious allegations out of hand. Response (d) doubts on the officer’s reputation, and that
is the best. would be unfair to the officer if he is not guilty
87. (c) Response (d) is unethical and response (a) of the offence. Response (d) shows that you
is impulsive. You can’t do as response (b) are not able to show initiative and shoulder
says as age limit is a part of the scheme. responsibility.
88. (b) If you care for your colleague’s well being, 94. (d) This is the best option. You should not
this is the best response. Response (a) is promise something and not do it. It is not
hardly going to help. It means your colleague your business to take the matter to a higher
@UPSC_THOUGHTS

does nothing to improve herself but merely officer, unless you are responsible for the
loads extra work on junior officers which could files that are getting delayed by your senior
be resented by them. Response (c) is not officer; even so, such a step should be a
going to work besides being confrontational ‘last resort’. Leaking the matter to the media
with a senior officer. There is, after all, no may appear to be a good idea, but you as
reason why your colleague should not have a public official must deal with the media with
good language skills as well. Response (d) caution, and must go to it if other avenues
means burdening yourself with extra work and do not help.
this could well spoil the relationship with your 95. (d) Responses (a) and (c) could be temporary
colleague in the longer term, besides making measures, while (b) is impractical.
her beholden to you all the time.
96. (c) This is the best response, though (a) is also
89. (d) Interrupting a person while they are speaking not wrong. Response (c) involves the team
could make them lose their line of thought members in evolving solutions.
for which they would not be happy with you.
97. (c) You should make your official tenure a
The situation does not call for a later meeting
learning experience.
when the officer is alone. There is nothing
wrong in asking colleagues, but it is always 98. (d) You cannot assume that she is making
better to get it right with the person who is excuses; she might really be having difficulties.
first presenting the project. So the best option is to consider the particular
problems she has instead of explaining the
90. (d) Such things happen at work, and one cannot
task all over to her. An assistant may be
be over-sensitive to them.
assigned only if there is a need for doing so.
91. (d) You need to assess the work and ask for To tell her get with the job shows you to be
help if there is a need for it. Asking your insensitive and impatient.
colleagues would not be the right thing to do,
as they would have their own work to 99. (d) Courtesy demands that you talk it over with
complete. Going to headquarters means you your colleague before making changes.
go over your senior officer without having 100. (d)

2 . 59
Interpersonal Skills including Communication Skills

101. (d) Response (a) would be acceptable if no other ‘kinds’ in plural is incorrect; it should be ‘kind’
way is possible. Response (c) could happen (n.)—‘in kind’ meaning ‘in goods or services,
in rare cases of emergency work from which rather than money’.
you simply cannot take leave. 117. (d) In (a), ‘means’ (n.) means ‘considerable
102. (c) Response (d) shows you to be insensitive to financial resources’. The word is always used
safety concerns of workers and evading in plural in this sense. In (b) ‘mean’ is a verb
responsibility. meaning ‘to intend to express’ or ‘indicate’.
103. (d) 104. (d) 105. (c) In (c), ‘means’ (adj.) means ‘uncharitable’ or
‘malicious’. ‘Means’ as an adjective has the
106. (d) This is practical and tactful. Responses (b) meaning of ‘occupying a middle position or
and (c) may or may not work. intermediate place’ or ‘average’. But it has
107. (c) 108. (d) to be used with that meaning only when we
109. (c) Though none of the other responses is talk in terms of value, etc. It cannot be used
‘wrong’, (a) and (b) show you as a better as in (d).
personality that (d). 118. (c) ‘Straight’ (adj.) means ‘without a bend’ in (a)
110. (c) This would be the best option. and ‘direct’ or ‘candid’ in (d). In (b), ‘straight’
is used as an adverb meaning ‘without
111. (b)
embellishment’. In (c), the correct verbal
112. (d) ‘Round’ has been used as a noun in (a) to phrase is ‘straight off’ meaning ‘immediately’.
mean ‘revolving motion’, as an adjective in
119. (b) ‘Strain’ (n.) means ‘great effort in pursuit of
(b) to mean circular; and as an adverb in (c)—
a goal’ in (a) and ‘streak’ or ‘trace’ in (d).
meaning, rotation or with change to opposite
@UPSC_THOUGHTS

In (c), ‘strain’ (v.) means ‘over stretch’.


position. ‘Round’ cannot be used as a verb
‘Strain’ cannot be used as a verb as given
as given in (d) as verbal phrase round up,
in (b). ‘Marked’, can be used here.
can mean ‘make number a round figure’ (He
rounded up the amount) or gather (He rounded 120. (a) ‘Odd’ (adj.) means ‘peculiar’ in (b); and ‘not
up the cattle). Round on means ‘make regular’ or ‘occasional’ in (d). ‘Odd’, in the
unexpected retort’ or ‘inform against’. mathematical sense, means leaving a
remainder of 1 when divided by 2; an ‘odd
113. (c) ‘Head’ has been used as a noun in (a) to
number’ is such a number. In (a), the correct
mean ‘life’ and in (b) to mean a ‘crisis’ or
word is ‘odds’ (n.) meaning ‘the probability
‘climax’. In (d), ‘head’ is a verb with meaning
that something is so or is more likely to occur
‘lead’. ‘Head’ cannot be used as an adjective
than something else’.
(c).
121. (d) Grave (adj.) means ‘serious’, or ‘solemn’ in
114. (a) ‘For keeps’ is a colloquial expression that
(a); ‘graven’ (v.) means ‘to impress deeply’
means ‘permanently’. In (c), ‘keeping’ has the
in (b); and ‘grave’ (n.) means ‘a receptacle
meaning of ‘preventing’. In (d) ‘keeping’ (n.)
of what is dead, lost or past’ in (c). ‘Grave’
means ‘custody’ or ‘change’. We cannot use
cannot be used as an adverb; the correct
‘keeping’ to mean ‘agreement’ as in (a).
adverb is ‘gravely’.
115. (a) ‘Note’ is used as a noun meaning ‘eminence’
122. (d) ‘Converse’ (v.) means ‘talk informally’ in A.
in (b) and a noun meaning ‘an underlying
‘Converse’ (n.) has the meaning of ‘the
expression of a quality, emotion, etc.’ in (c).
opposite’ or ‘contrary’ in B.
‘Note’ is a verb in (d), meaning ‘observation’.
In (a), the correct word is ‘note’ in singular. 123. (d) ‘Invalid’ (n.) means ‘an infirm or sickly person’
‘Note’ here would mean ‘a distinctive quality, in A; in B, ‘invalid’ is used as an adjective
tone’ or ‘mood’. meaning ‘deficient in substance or cogency’.

116. (b) ‘Kind’ (n.) means ‘nature’ or ‘character’ in (a). 124. (b) In A, ‘apprehend’ means ‘to take into custody’.
It is used as an adjective meaning ‘considerate’ ‘Apprehend’ cannot be used as a verb
or ‘humane’ in (c). In (d), ‘kind of’, an informal meaning ‘defended’, as it is done in B.
expression, means ‘to some extent’. In (b), However, ‘apprehend’ (v.) can mean ‘anticipate’

2 . 60
Interpersonal Skills including Communication Skills

( He apprehends violence against his however, means ‘a school, college or university


community) or ‘understand’ (It is necessary at which one has studied and usually from
to apprehend in order to forgive). which one has graduated’. The word is
125. (b) In A, ‘breach’ means ‘violation’ or ‘break’. In incorrectly used in C; the right word is
B, the word should be ‘breakage’— ‘alumni’, meaning ‘former students’.
‘separation’—and not ‘breach’. 140. (b) In A, the words in bold should ‘ad verbum’,
126. (a) In A, the correct word is ‘effect’ (n. singular)— that is, ‘word for word’, and not ‘ad infinitum’
not ‘effects’ (n. plural)—meaning ‘the making meaning ‘without limit’. B is correct, ‘prima
of a desired impression’. In B, ‘effect’ means facie’ meaning ‘on the face of it’. But in C,
‘operation’. it should be ‘intra vires’—‘within the permitted
powers’—and not ‘ultra vires’—‘beyond
127. (d) In A, ‘rough’ (adj.) means ‘unpleasant’ or powers’—in keeping with the meaning (‘won
‘difficult’. In B, ‘rough’ (adj.) means ‘requiring accolades’) conveyed by the sentence.
exertion or strength’.
141. (a) A is correct, as ‘sine die’ means ‘without
128. (d) ‘Singular’ (n.) means ‘extraordinary’ in A and fixing a day for future action or meeting’. B
‘odd’ or ‘strange’ in B. is incorrect. ‘Sub poena’ means ‘court order
129. (c) ‘Imminent’ means ‘near’ or ‘impending’. It has requiring someone to appear before a
been incorrectly used in both the sentences. parliamentary committee or a court of law’.
The right word is ‘eminent’ (adj.) meaning The right word to be used is ‘sub judice’—
‘reputed’ in A and ‘jutting’ or ‘prominent’ ‘under the law’ or ‘being considered by a
in B. court’. C is incorrect: the word should be
‘quorum’ (n.)—‘number of people who have to
@UPSC_THOUGHTS

130. (c) ‘Reiterate’ means ‘say or do again and again’.


‘State it often’ does not convey the same be present to make a meeting valid’—and not
meaning as the italicised part of the sentence. ‘quorate’ which is an adjective meaning
‘Reinforce’ means ‘strengthen with some ‘having a quorum’ as in The resolution could
added support or material’. ‘Stress’ (v.) means not be passed through as the shareholders’
‘to emphasise’. meeting was not quorate.

131. (b) A ‘scapegoat’ is ‘a person or group made to 142. (a) In A, the correct word is ‘proviso’ meaning
bear the blame for others or to suffer in their ‘condition in a contract’ and not ‘provost’. ‘Per
place’. A ‘truant’ is ‘a person who shirks or se’ means ‘on its own’ or ‘by itself’; ‘sangfroid’
neglects his or her duty’. ‘Foolish’ is a very means ‘coolness of mind’.
general word; it cannot be used as an exact 143. (b) ‘Fait accompli’, and not ‘fait accomplish’,
equivalent of the italicised phrase. ‘Culpable’ means ‘an established fact or thing’. In B,
means ‘deserving blame or censure’. the bold phrase should be ‘coup de grace’
132. (b) 133. (d) meaning ‘a finishing or decisive stroke’ and
not ‘coup de main’—‘a surprise attack or
134. (b) ‘Inculcate’ means ‘implant by repeated
sudden development’. C is correct as ‘sine
statement or admonition’. ‘Infuse’ means ‘to
qua non’ means ‘condition without which
introduce or instil’.
something cannot work’.
135. (a) A ‘counterfeit means ‘a sham’, ‘a person who
144. (b) ‘Whether or not’ is a redundant expression
is not genuine’ or ‘false’. ‘Misanthrope’ means
as far as the given sentence is concerned.
‘a hater of humankind’. ‘Cheat’ is much more
‘Or not’ adds nothing to the expression as
limited; it means to deceive for one’s own
‘whether’ is equivalent to ‘if’ here. For, we
gain, and is closer to ‘fraud’. A ‘humbug’ may
can also say ‘a matter of debate if persons’.
deceive for the fun of it.
However, note that ‘or not’ is necessary when
136. (c) 137. (c) 138. (a) what is being stressed is an alternative, as
139. (b) ‘Ad hoc’ means ‘temporary’ or ‘for the particular in ‘I am keen to come whether or not it is
purpose’. ‘Ipso facto’ means ‘by the very necessary’.
nature of the deed’ here. ‘Alma mater’, The other redundant expression here is ‘various

2 . 61
Interpersonal Skills including Communication Skills

different’: it should be either ‘various’ or 152. (b) A ‘verbal’ understanding means one that is
‘different’, as they have almost the same ‘spoken’ or ‘uttered’, that is ‘not written down’.
meaning here. ‘Breakthroughs’ mean ‘new developments’; so
‘new’ is superfluous in C.
145. (b) Using ‘usually’ and ‘normally’ in the same
sentence amounts to telling twice that a thing 153. (d) ‘Literally’ means ‘actually’, not ‘figuratively’.
is customary. Either ‘usually’ or ‘normally’ has So ‘literally’ has been used correctly here. But
to go. In the sentence, ‘It is argued … the it is incorrect to use the word when we mean
best’, it is enough to say ‘and finally settle to convey a metaphorical meaning, as in We
for the best’; ‘what they see as’ is redundant stared so hard our eyes were literally glued
here as the paragraph is consistently referring to the dark.
to the experience of the shoppers only. 154. (a) The use of ‘adequate access’ (b) and ‘site
‘Unless and until’ is a redundant phrase that is largely open’ (c) are awkward construction
is frequently used. One of them will do. Note that are better to be avoided. (d) re-writes the
that ‘more and more’ is an accepted expression given sentence in a similar circuitous manner.
that denotes a tendency or trend on the rise. 155. (b) When we say ‘brought out a hundred books’,
‘Actually’ in the first sentence is necessary; we are being more specific about the nature
‘actually sweating it out’ means ‘experiencing of the publications. (a) changes the meaning
a thing in reality’. The ‘quite’ in the last of the given sentence with ‘this year’.
sentence is also needed; it means ‘in the ‘Publishings’ does not mean ‘publications’; it
fullest sense’ or ‘entirely’. is an incorrect word.
146. (a) ‘Untimely death’ is a common but inane 156. (c) It is better to use ‘designed’ here, rather than
@UPSC_THOUGHTS

expression. Note that death is generally never ‘developed’, ‘evolved’ or ‘created’.


termed ‘timely’. ‘Undoubtedly’ means ‘without 157. (d) ‘Start’ is a simpler word for ‘activate’; ‘using’
doubt’. for ‘utilising’. (b) changes the meaning of the
147. (c) ‘True facts’ is redundant, for a fact is not given sentence. In (c) ‘search the miscreants’,
a fact unless it is true. ‘To the tune of’ is that is, ‘actually search them’ gives a meaning
different from ‘search for’ or ‘look for’ the
a hackneyed circumlocution; we could express
miscreants.
the sentence succinctly as ‘The company’s
expenditure would be rupees sixty lakhs.’ 158. (d) This option simplifies what is conveyed in the
given sentence. The phrase ‘waiting to build’
148. (a) ‘Thought to himself’ is redundant; one can
in (a) is awkward. The ‘all-encompassing’ in
only think to oneself. Similarly with the
(c) does not give the same meaning as
expression, ‘I pictured in my mind’: ‘pictured’
inclusive.
means ‘visualised’ (in one’s mind, of course).
159. (b) The given sentence has an awkward
149. (a) ‘Successfully developed’ is redundant; a thing
construction. ‘Shot up the overtime’ (c) does
cannot be developed ‘unsuccessfully’. not convey the meaning correctly.
150. (c) ‘Revert back’ is a redundant expression; it 160. (a) The meaning of the sentence is distorted in
should simply be ‘revert’. ‘Why’ is redundant (b)—it will not cost rupees six lakhs annually.
in B: ‘The reason the police …’ is correct. (c) is a long sentence that does not present
151. (d) ‘Put an end to’ is commonly used but the the given sentence in a simple and correct
expression has redundancy. Simply ‘end’ or style.
‘stop’ will do. In ‘B’ only one of the two— 161. (c) ‘I determined my mind’ is not a correct
‘possible’ or ‘may’—may be used. Both these expression; ‘I determined’ itself means ‘made
words convey the same meaning. Likewise up my mind’. ‘Speak the truth’ is preferable
in C, either their ‘past’ or their ‘history’, not to ‘say the truth’. ‘Good cause’ has to be
both together. History has to do with the past. replaced by a phrase like ‘reasonable excuse’
So are ‘past experiences’, ‘past achieve- because that is what is meant. We can,
ments’, ‘past records’ and ‘past precedents’. however, say We are fighting for a good

2 . 62
Interpersonal Skills including Communication Skills

cause where ‘good cause’ means ‘a just’ or P can follow only S. Q then follows P (Kabir
‘right movement deserving support’. would have ‘jumped up’, only when he was
162. (b) The sentence in (a) expresses the meaning ‘trod on’). The SPRQ order is presented by
in a round about manner. In (c), the adverb option (d).
‘gladly’ is incorrectly placed. It may be noted at the beginning itself that
163. (a) The initial choice is between PQ and QP. As SP as the first two options is presented by
P explains how media is gullible enough to option (d) only.
be manipulated by public relations firms, it 168. (b) If it is difficult to make out which one of the
indicates media immaturity—the substance of given sentences would come first, find out
S1. Q goes on to talk of another aspect— the order of a couple of sentences that
media corruption. So PQ seems to be the logically make sense. R, for instance, can
better order to follow S1. After Q, S is the come only after Q. Only (a) and (b) offer this
correct sentence to follow as it gives an order and so the final order of all the
example of media corruption. R follows S and sentences must begin with Q. Immediately
smoothly leads to S6. So (a) or PQSR after Q, what? Obviously S (only after the
appears to be the correct answer response. flames break out, can the passengers be
164. (a) The next sentence must be P or Q, going asked to quit the ship). QSRP is the right
by the responses. The sentence to follow S1 order.
cannot be P, as it does not clarify what has 169. (a) The right order of the sentences is difficult
proved disastrous or how things have been to be made out at first sight as more than
made worse. So, Q is the sentence to follow one order appears to be correct. It is
@UPSC_THOUGHTS

S1, and it makes sense. So it is either (a) especially difficult to choose the first sentence
or (c). Of the two, (a) seems to be the better that would follow S1. In such a case, see
choice, as the transition from R to S6 is whether any two sentences make logical
smoother than that from S to S6. sense. It is obvious that S must follow R
165. (d) Either P or R is the sentence to follow S1. (it is only after the bananas are checked for
It could be P. Taking (a), the sequence their greenness and loaded that one can point
appears awkward after PQ. And the transition out what factor keeps them green during the
from R to S6 makes no sense. Now (c) also trip. Only in (a) does S follow R. But check
has R as the sentence before S6. We may through the P and Q order also.
reject (a) and (c) responses. In (b), taking 170. (d) At first sight, Q, R or S may follow S1.
Q and S in that order, even if Q is acceptable However, note that ‘such advance’ (S) refers
after R, S seems awkward coming after Q; to the ‘progress’ mentioned in S1. P is more
moreover, the transition from S to P is abrupt. likely to refer to Q than R, as a contrast is
The order in (d) is more acceptable; the being emphasised and not a similarity.
transition from P to S6 makes sense. However, only P can precede S6 as they
166. (b) Either S or R could be the sentence after together explain why evolution can be equated
S1. In such a situation, take the responses with progress to some extent.
one by one. However, the sequence in (a) 171. (d) R is the beginning (all the options show it).
SQRP seems unacceptable, specially as P After R, what? The ‘is worth more’ in R is
does not lead smoothly to S6. Taking (b) explained further by P. So RP—(c) or (d). S
RPSQ, the sequence appears correct as the further explains P (how much more a rupee
paragraph is coherent. As for (c) SPRQ, R is worth today is explained by a scenario
coming after P is awkward. The sequence in where it can earn 5 per cent or 10 per cent
(d) is obviously too haphazard to be correct. interest and the implication of this over time).
167. (d) To find out which sentence will follow S1, read Q easily follows S (d) but Q cannot follow
all the sentences carefully. S can follow S1 T with the same ease (c). So RPSQT is the
but P cannot, as it refers to ‘the sleeping correct order. T actually provides a fitting
man’—that is, ‘Kabir’ as evident from S. So conclusion to the paragraph.

2 . 63
Interpersonal Skills including Communication Skills

172. (d) T would come first as it presents the problem It would be useful to write down the order of the
with democracy. Hence the problem with statements as you think they should appear in a para.
political liberalism (R). The ‘propagation of the The five questions can then be answered by just
ignorant’ in R connects to the misleading looking at the order you’ve written down.
aspect of ‘birth rate’ in S, which in turn relates 176. (d) 177. (b) 178. (a) 179. (c) 180. (d)
to the ‘small and large families’ and ‘education’
181. (d) This would be most efficacious, ensuring that
of Q. P explains Q further. The order of the
only those who require it have the information
sentences is TRSQP.
and can refer to it whenever required.
173. (a) Which statement would be the best beginning?
182. (c) Organising meetings everyday would be
This question has to be answered viewing the
impractical; after all you would have other
natural progression of ideas in a paragraph.
work to do also.
S talks of ‘the developing complexity of life’
and the loss of the family as a ‘self-sufficient 183. (b) This would be the first step; you have to
sovereignty’ that makes legislation necessary make people aware of the situation and trust
(T). R explains need for legislation further as them to act sensibly. Response (a) would be
does Q with reference to the situation in a haphazard move and would not ensure that
cities. Note that the ‘weak natural order’ in the message reaches all concerned. Response
(c) is unfriendly and adversarial. Response (d)
T connects to the ‘free egoistic impulse’ in
is a step to be taken if water shortage and
crowded places/cities (R). So R must follow
wastage continue.
T. The ‘breakdown’ of ‘spontaneous order’ (Q)
leads to ‘the state’ replacing the ‘spontaneous 184. (a)
society’ (P). 185. (d) It is not advisable to keep aloof from the
@UPSC_THOUGHTS

174. (b) In such questions where the beginning is not media, but it is equally not advisable to give
provided, find the statements that cannot, in out crucial information which, if publicised,
an obvious sense, begin the para. P (So) Q may harm investigations.
(obviously) and S (Hence) cannot begin the 186. (b)
para. If T is to begin the para, P can follow 187. (d) Response (c) may appear to be correct, but
it but S cannot follow P. to ask for ‘weaknesses’ may prove awkward
175. (b) in a congregation of people unknown to one
For 176-180: Understand what the paragraph is about another.
first of all. It aims to acquaint us with the objectives 188. (c) If an entire assembly of villages look blank,
and functioning of the Make-a-Wish Foundation. So, it must be because people have not understood
C is the first sentence introducing the theme. What you. While (a) is too facile, there is little
does it do? F makes the general statement. So it is reason to conclude that villagers are afraid
the second sentence of the para. D, B and A explain as in (b) unless there are exceptional
its functioning further. A has to come after B as the circumstances. Response (d) shows a lack
former talks about the ‘other wishes’ that do not come of commitment to work.
under B (‘some wishes’). E is the summing-up statement. 189. (d) 190. (c)

2 . 64
3
LOGICAL
REASONING
AND
ANALYTICAL
@UPSC_THOUGHTS

ABILITY
Logic, reasoning and analytical ability are so closely related to one
another that it is difficult to define or describe one without reference
to the others. When one uses the term ‘logical’, one implies ‘clear or
valid reasoning’. And ‘reasoning’ is ‘the act or process of drawing
conclusions from facts’—which is also what logic is concerned with.
‘Analytical ability’ is what helps you break down complex problems
into its constituent parts and examine how they are related to one
another. It also helps you to assess the truth or falsity of arguments
on the basis of the meanings of words. In every field of activity and
every stage of life, one is required to reason and analyse and put to
use the ability to think logically. These abilities are specially to be
developed by persons aspiring to the civil services, as important
decisions concerning public welfare and socio-economic development
call for an analytical mind and the ability to reason ‘correctly’. Of
course, in a ‘Test of Reasoning’, there are specific formats of
questions by which your reasoning and analytical abilities are
sought to be assessed—and the following pages will familiarise
you with them.
Logical Reasoning and Analytical Ability

@UPSC_THOUGHTS

3.2
Logical Reasoning and Analytical Ability

Logical Reasoning
and Analytical Ability
ORIENTATION
of philosophy, understanding some terms associated
BASIC PRINCIPLES
with the discipline can help you understand the
Reasoning is the process of drawing conclusions from questions better and reach correct answers faster.
given facts or evidence. Logic is concerned with An argument in logic is any set of propositions. A
differentiating between ‘correct’ and ‘incorrect’ proposition is the content of a sentence that affirms or
reasoning. denies something and is capable of being true or false.
In a sense, any reasoning test involves logic at all A proposition—or statement as some people prefer to
levels. Patterns have to be perceived and identified, call it—is not exactly the same as a sentence. Two
whether in thought/argument, use of words or numbers, sentences may present the same proposition; for instance,
or in symbols before arriving at valid conclusions or ‘Seema wrote the letter’ and ‘The letter was written by
answers. Similarly, application of rules, principles, and Seema’ are two different sentences but assert the same
method to solve a given problem involves logical proposition (or make the same statement). Also, in
@UPSC_THOUGHTS

thinking and analysis, or what is known as analytical different contexts, the same sentence can be used to
ability. At every stage and in every field of life, you need make different statements: to have said in 1953 ‘India’s
to be able to distinguish between correct and incorrect prime minister at present is a man who took part in
reasoning. More specifically, of course, formal logic is the freedom struggle’ would have been a true statement,
concerned with drawing or inferring conclusions from but the same sentence said in 2010 would have been
given ‘premises’ and judging the validity of statements making a false statement.
and conclusions. The knowledge of language—words One proposition in an argument is claimed to
and their meanings, and forms of sentences—is essential follow from other propositions that offer support for the
for logical reasoning and analytical ability. truth of that one. The argument, in the logician’s sense,
Formal logic—deductive and inductive—calls for has a structure: the propositions become premises (also
deriving conclusions from premises, and assessing the premisses) and conclusions. The conclusion is the
correctness/validity of assumptions, causes and proposition that is affirmed in the argument on the
consequences; critical reasoning which calls for
basis of the support (or evidence) provided by the
comprehension not only of the contents of a passage
premises. The validity, correctness or soundness of an
but also of the tone and implied meanings; and, finally,
argument depends on how well the premises provide
interpretation and analysis of data.
evidence for the conclusion to follow.
Many arguments in real life situations do not
An argument may be deductive or inductive,
present their ideas in a ‘logical’ manner; the conclusions
depending on how the conclusion follows or is inferred
may be anywhere but at the end, or may not even be
from the premises.
specifically stated but only implied. Here it is one’s
An argument may be considered deductive if it is
analytical ability that helps one to analyse the argument
impossible for the conclusion to be false when all the
and ‘discover’ the conclusion and one’s logical ability
premises are true. In a deductive argument, the premises
that helps one to judge whether the conclusion is
necessitate the conclusion. An example of a deductive
‘correct’ or ‘incorrect’.
argument is:
Terms Explained 1. All men are mortal.
Even though answering the questions in this test may 2. A is a man.
not require an in-depth study of logic as a discipline 3. Therefore, A is mortal.

3.3
Logical Reasoning and Analytical Ability

If both premises are true, then the conclusion I. Categorical Propositions


follows automatically. Categorical propositions have four different standard
An argument is inductive if it is improbable that the forms, which can be studied in terms of how the classes
conclusion is false when all the premises are true. The mentioned are related to one another. A class is the
premises do not necessitate but just make probable the collection of all objects sharing specified characteristic.
conclusion. Take the proposition:
Whether the conclusion in an argument has been
All cats are animals.
arrived at through deductive reasoning or through
inductive reasoning can often be discerned from the There are two classes here—cats and animals. The
wording of the statement or sentence. Words such as first class—cats—is included or contained in the second
‘usually’, ‘sometimes’ and ‘generally’ are signals of class of ‘animals’. The reference to cats is all-
induction. enveloping—all cats without exception. So the subject
It is in deductive arguments that we talk of ‘validity’ (S) term is distributed. As for the predicative (P) term
and ‘invalidity’. In inductive arguments, we may at ‘animals’, it does not refer to all animals in this
most consider the ‘soundness’ of the argument. The proposition. It is only referring to those animals which
soundness of the argument depends on the strength of are cats. We cannot say ‘All animals are cats’, as there
the evidence relative to the conclusion. An inductive are many other animals besides cats. So the predicative
argument is evaluated by degrees—weaker or stronger, term ‘animals’ is not distributed here.
convincing or not-so-convincing—depending on the If a term refers to all the things it stands for, it is
premises provided. used in its complete denotation, and is a distributed
Consider: term.
@UPSC_THOUGHTS

This is a universal affirmative proposition and is


It is likely that Jonu, a drug addict, comes
symbolically represented as
from an unhappy family, as ninety per cent
of drug addicts come from unhappy families. All S is P.
This is known as an A proposition.
The conclusion—which is, ‘It is likely that Jonu
comes from an unhappy family’—has a higher degree Universal negative propositions are in the form
of truth in this argument than it would have if ‘thirty- No S is P.
five per cent’ replaced ‘ninety per cent’ in the argument. No cat is a dog.

Logical Deduction Here all cats (subject) are being referred to but in
a negative perspective, i.e., all cats are outside the
As mentioned earlier, a deductive argument is valid
category of all dogs. This is an E proposition.
when its premises, if true, provide conclusive grounds
for the truth of its conclusion. If the premises are true, Particular affirmative propositions affirm something
the conclusion is true. The relation between premises about ‘some’ of a class. They are in the form
and conclusion, however, follow certain forms if the Some S is P.
argument is to be valid. Every deductive argument is Some cats are black.
either valid or invalid. The word ‘some’ is customarily taken to mean ‘at
A syllogism is a logical argument containing three least one’. The proposition is known as an I proposition.
statements—two premises and a conclusion drawn It is affirmed here that some members of the class of
from those premises. There are syllogisms corresponding cats are members of the class of black. The proposition
to each type of logical proposition. neither affirms nor denies that all cats are black. The
Traditionally, the study of deduction focused on class of black does not refer to all black things but only
arguments with propositions of a special kind known to cats that are black, so neither term is distributed. The
as categorical propositions. Now, however, hypothetical, statement does not literally say that some black things
disjunctive and relational propositions are also are not cats, though it may be taken to suggest it. The
recognised as types of logical propositions. literal and minimal interpretation of the statement is

3.4
Logical Reasoning and Analytical Ability

that the class of cats and the class of black things have logically equivalent to, and therefore to translate into,
some member or members in common. the following standard-form propositions; ‘Some
The fourth type of categorical proposition is the children are not believers in Santa Claus’, ‘Some
particular negative: elephants are white things’, ‘No elephants are pink
Some S is not P. things’, and ‘No round objects are square objects’.
Some cats are not black. Categorical propositions involving the words ‘only’
or ‘none but’ are often called ‘exclusive propositions’
Here the subject does not refer to all and the
statement is negative. The predicate, however, refers to because in general they assert that the predicate applies
all black things. This is an O proposition. exclusively to the subject named. Examples of such
The proposition says that at least one member of usages are ‘Only citizens can vote’ and ‘None but the
the class S is excluded from the whole of the class P. brave deserve the fair’. The first translates into the
The subject and predicate terms always designate standard-form categorical proposition ‘All those who
classes, but those terms may be more complicated can vote are citizens’, and the second into the standard-
expressions than the simple ‘cats’ and ‘black’ and form categorical proposition ‘All those who deserve the
‘animals’. fair are those who are brave’.
Language and the use of words are important in Some examples of exceptive propositions are ‘All
identifying propositions. except employees are eligible’, ‘All but employees are
Words such as all could confuse one into thinking eligible’, and ‘Employees alone are not eligible’.
a proposition to be A. For example, ‘All cats are not Translating exceptive proposition into standard form is
black’ implies that ‘some cats are not black’. As the somewhat complicated because propositions of this
@UPSC_THOUGHTS

proposition is negative, it is an ‘O’ proposition. kind (like singular propositions) make two assertions
Whenever a noun or name is given it is a universal
rather than one. Thus the three illustrative propositions
proposition. If it is affirmative, it is A and if negative,
about eligibility translate identically into ‘All non-
it is E. Thus, ‘Man is mortal’ is A and ‘Man is not
employees are eligible persons, and no employees are
immortal’ is E (‘No man is mortal’).
eligible persons’.
The sentence, ‘All persons except one are present’
The grammatical particles ‘a’ and ‘an’ may also
is an I proposition (‘some persons are present’). But if
a particular name is an exception, the statement would serve to indicate quantity, but whether they are being
be A. Thus ‘All except Seema are present’ is an A used to mean ‘all’ or ‘some’ depends largely on the
proposition as ‘all except Seema’ is taken as subject.] context. Thus ‘A bat is a mammal’ and ‘An elephant
Where there is no quantifier, what the sentence is is a pachyderm’ are reasonably interpreted as meaning
intended to express may be doubtful. We may be able ‘All bats are mammals’ and ‘All elephants are
to determine its meaning only by examining the context pachyderms.’ But ‘A bat flew in at the window’ and
in which it occurs, and that examination will usually ‘An elephant escaped’ quite clearly do not refer to all
clear up our doubts. It is very probable that ‘Dogs are bats or all elephants; they are properly reduced to ‘Some
carnivorous’ refers to all dogs and is to be translated bats are creatures that flew in the window’ and ‘Some
as ‘All dogs are carnivores’. On the other hand, it is elephants are creatures that escaped’.
highly probable that only some children are referred to Although affirmative statements beginning with
in ‘Children are present’; thus it would translate to ‘every’ and ‘any’ are translated into ‘All S is P,’
‘Some children are beings who are present’. negative statements beginning with ‘not every’ and ‘not
Now take any’ are quite different. Their translations are much less
Not all children believe in Santa Claus. obvious and require great care. Thus, for example, ‘Not
There are white elephants. every S is P’ means that some S is not P, whereas ‘Not
There are no pink elephants. any S is P’ means that no S is P.
Nothing is both round and square. In this case, while the subject is not distributed, the
On reflection, these propositions will be seen to be predicate is distributed.

3.5
Logical Reasoning and Analytical Ability

The following table will help: Though the definite conclusion is that some
members are shared by cats and black things,
Proposition Subjective Predicative
it is possible to infer that ‘some black things
term term
are not cats’ or Some P is non-S. The part of
A distributed undistributed circle S that does not overlap circle P may be
E distributed distributed taken to represent those S that are not P. The
I undistributed undistributed part of the circle labelled P that does not
overlap the circle labelled S may be taken to
O undistributed distributed
represent all P that are not S. So,
We can use Venn diagrams in a simplified form to Some black things are not cats.
illustrate the four propositions A, E, I and O so as to Some cats are not black.
understand them better. (iv) ‘O’ proposition: Some cats are not black.
(i) ‘A’ proposition: All cats are animals. (All S (Some S is not P.)
is P)
P S P Black things
animals
Some cats
S x
cats that are
S not black

We only know that ‘some cats’ are not black;


@UPSC_THOUGHTS

One circle representing the class of ‘cats’ is we do not know about the other cats—we
enclosed within another circle representing cannot assume that some of these other cats
the class of ‘animals’. are black. They could be brown, white or
It is implied that orange; they would still not be black. Of
‘Some animals are cats’. (Some P is S.) course, they may be black. We cannot say any
of these things definitely from the given
(ii) ‘E’ proposition: No cat is a dog. (No S is P.)
statement.
As you may have understood from the above
S P matter, from a given proposition, another proposition
cats dogs can be inferred by interchanging or transforming the
subject and predicate. This is called conversion. (This
is already evident from the preceding diagrams.)
The two classes—cats and dogs—exclude each
(i) A can be converted to I.
other. All cats are animals. A
Implied is the statement: ‘No dog is a cat’. (No Some animals are cats. I (valid)
P is S.)
(ii) E can be converted to E.
(iii) ‘I’ proposition: Some cats are black. (Some S No cat is a dog. E
is P.) No dog is a cat. E (valid)
(iii) I can be converted to I.
black Some cats are black. I
cats things
S P
Some black things are cats. I (valid)
(iv) O cannot be converted.
black cats Some cats are not black. O
SP (a) Some black things are not cats. (invalid)
In this case, we may also infer that ‘Some (b) Some not black things are cats. (invalid)
black things are cats. (Some P is S.) (c) Some non-cats are not black. (invalid)

3.6
Logical Reasoning and Analytical Ability

The converted propositions (a), (b) and (c) are that contains the major term, which is by definition the
invalid as they may or may not be true. (As it is a predicate term of the conclusion. Likewise, the minor
particular proposition, it cannot be changed into a premise is not defined in terms of its position, but as
universal one.) the premise that contains the minor term, which is
We may also validly infer the following by what defined as the subject term of the conclusion.
is called obversion.
Obversion is another kind of valid immediate Rules for Standard-Form Syllogisms There are
inference. some rules that categorical syllogisms have to follow
(i) All cats are animals. A if they are to be valid.
No cat is a non-animal. E (valid) 1. A valid standard-form categorical syllogism
must contain exactly three terms, each of which
(ii) No cat is a dog. E
is used in the same sense throughout the
All cats are non-dogs. A (valid)
argument.
(iii) Some metals are conductors. I
The fallacy of equivocation occurs when words
Some metals are not non-conductors. O (valid)
or phrases that have more than one meaning are
(iv) Some cats are not black. O used. The ambiguity may occur in both premises
Some cats are non-black. I (valid) or in one premise and the conclusion. In the
These inferences—which are immediate following example, the structure of the argument
inferences—are also clear from the Venn diagrams is valid but an equivocation occurs.
used to illustrate the four categorical propositions. Happiness is the end of life. (p is q )
Coming to mediate inference, which is what a The end of life is death. (q is r )
@UPSC_THOUGHTS

syllogism involves, we must know about major, minor So happiness is death. (p is r )


and middle terms.
The conclusion of a categorical syllogism is a The fallacy is that the expression “end of life”
categorical proposition that contains two of the has different meanings in premises. What has
syllogism’s three terms. The conclusion is always used been asserted with one sense of the expression
to identify the terms of the syllogism. is then wrongly regarded as having been proved
The term that occurs as the predicate of the with respect to the other expression.
conclusion is called the major term of the syllogism. 2. In a valid standard-form categorical syllogism,
The term that occurs as the subject term of the the middle term must be distributed in at least
conclusion is called the minor term of the syllogism. one premise.
The third term of the syllogism, which does not Take
occur in the conclusion, appearing instead in both
All cats are animals.
premises, is called the middle term.
All dogs are animals.
The major and minor terms must each occur in a
Therefore all dogs are cats.
different one of the premises. The premise containing
the major term is called the major premise, and the The middle term, ‘animals’, is not distributed in
premise containing the minor term is called the minor either premise, so the syllogism commits the
premise. fallacy of the undistributed middle.
Take the syllogism
For the two terms of the conclusion really to be
All mammals are animals. (major premise)
connected through the third, at least one of them
All whales are mammals. (minor premise)
must be related to the whole of the class
Therefore, all whales are animals.
designated by the third or middle term.
The term ‘whales’ is the minor term; the term Otherwise, each may be connected with a
‘animals’ is the major term. The term ‘mammals’ which different part of that class, and the two not
is not there in the conclusion is the middle term. necessarily connected with each other at all.
It should be emphasised that the major premise is This is obviously what occurs in the given
not defined in terms of its position, but as the premise example. Cats are included in part of the class

3.7
Logical Reasoning and Analytical Ability

of animals, and dogs are included in part of the Any syllogism that breaks this rule is said to
class of animals. commit the fallacy of exclusive premises.
3. In a valid standard-form categorical syllogism, 5. If either premise of a valid standard-form
if either term is distributed in the conclusion, categorical syllogism is negative, the conclusion
then it must be distributed in the premises. must be negative.
When a syllogism contains its major term No poets are managers.
undistributed in the major premise but Some artists are poets.
distributed in the conclusion, the argument is Therefore some artists are managers.
said to commit the fallacy of illicit process of
It will be seen immediately that the exclusion
the major term or, more briefly, the illicit major.
of poets and managers, asserted by the first
An example of this fallacy is
premise, does not justify any valid inference
All dogs are mammals. regarding the inclusion of artists and managers.
No cats are dogs. Any syllogism that breaks this rule may be said
Therefore no cats are mammals. to commit the fallacy of drawing an affirmative
The conclusion makes an assertion about all conclusion from a negative premise.
mammals, saying that all of them are excluded 6. No valid standard-form categorical syllogism
from the class of cats. But the premises make with a particular conclusion can have two
no assertion about all mammals, so the universal premises.
conclusion illicitly goes beyond what the
An example of a syllogism that breaks this rule
premises assert. Since ‘mammals’ is the major
is
@UPSC_THOUGHTS

term, the fallacy here is an illicit major.


All household pets are domestic
When a syllogism contains its minor term
animals.
undistributed in its minor premise but
No unicorns are domestic animals.
distributed in its conclusion, the argument
Therefore some unicorns are not
commits the fallacy of illicit process of the
household pets.
minor term (more briefly called the illicit minor).
An example of this fallacy is The given syllogism is invalid because its
All reactionaries are subversive
conclusion asserts that there are unicorns,
elements. whereas its premises do not assert the existence
All reactionaries are critics of the of unicorns (or of anything at all).
present administration. Venn diagrams provide the simplest and most
Therefore all critics of the present direct method of testing the validity of categorical
administration are subversive elements. syllogisms.
4. No standard-form categorical syllogism having Take the following example.
two negative premises is valid. All men are mortal.
X is a man
Here is an example of a syllogism that breaks
X is mortal.
this rule:
No astrologers are scientists. Step One The first premise states that all men
Some scientists are not magicians. belong to a class which is called mortality. Therefore,
Therefore some magicians are not we need one circle to represent the term ‘mortal’ and
astrologers. another circle for ‘men’.

The premises state the relation of the middle Mortals


term, ‘scientists,’ to the subject and predicate
terms of this syllogism, but from these premises Men
no valid inference can be drawn about the
relation between magicians and astrologers.

3.8
Logical Reasoning and Analytical Ability

Step Two The second premise states that X is a Now take


man. We draw a third circle for X. X’s circle is within All whales are mammals.
the circle of men. All whales are animals.
Therefore all animals are mammals.
mortals
The second premise can be represented in more
men
than one way.
X
A
M M

W A
M W W
A
Step Three Is the argument valid? Yes, it is, because
we see that the conclusion—X is mortal—follows from
the premises. X is in the ‘mortal’ space, so X is also (i) In which case (ii) In which case (iii) In which case
mortal. only some only some the conclusion
animals are animals are would be
Venn diagrams can also be used to show if mammals, and mammals, and correct.
arguments are invalid. For example, take the argument: the conclusion the conclusion
‘Because all rupees are money and all yen are is wrong. is wrong.
money, all rupees must be yen.’
But there can only be one valid conclusion, and
By placing rupees, money, and yen in circles and
here no conclusion is definite. So the violation of Rule
arranging them appropriately, we arrive at at least two
3 is illustrated.
possibilities that invalidate the conclusion.
If we take Rule 4, we can see how the following
@UPSC_THOUGHTS

syllogism breaks the rule.


M M
M = money No astrologers are scientists.
Y R = rupees Some scientists are not magicians.
R R Y
Y = yen Therefore some magicians are not
astrologers.

We can see that the conclusion—all rupees are Using Venn diagrams, we have at least two
yen—is not valid, since rupees and yen can be separate possibilities which invalidate the conclusion.
or rupees may be yen, and yen may be rupees, but we some scientists who
cannot be sure of this. All we can conclude is that both are not magicians
rupees and yen are money.
This illustrates how the violation of Rule 2 leads
to an invalid (or uncertain) conclusion. A x
Take the syllogism
All dogs are mammals.
astrologers magicians
No cats are dogs.
Therefore no cats are mammals. (i) In which ‘No magician is an astrologer’.

It would be illustrated as: some scientists who


magicians are not magicians
M
C mammals = M
D C dogs =D
cats =C A x

Cats may or may not be in the class of mammals.


So no definite conclusion follows from the premises. astrologers
The given conclusion is thus invalid. (ii) In which some magicians are astrologers.

3.9
Logical Reasoning and Analytical Ability

As for Rule 5, the given syllogism can be illustrated. The fallacy of affirming the consequent should be
avoided. It is invalid if the categorical premise affirms
No poets are managers.
the consequent.
Some artists are poets.
The fallacy of denying the antecedent should be
Therefore some artists are managers.
avoided. It is invalid if the categorical premise denies
poets =P the antecedent.
A P A M managers = M Both the following arguments are invalid.
artists =A 1. If it rains, I take an umbrella.
2. I take an umbrella. × (affirming the consequent)
It is clear that some artists may or may not be Conclusion: Therefore it rains. ×
managers. The conclusion is thus invalid. Therefore it does not rain. ×
3. It does not rain. × (denying the antecedent)
Let’s take the syllogism (see under Rule 6) that Conclusion: Therefore I take an umbrella. ×
breaks Rule 6. Therefore I do not take an umbrella. ×
By illustration we have [It is possible that taking the umbrella or not
DA
taking it is not solely dependent on rain. Rain
domestic animals = DA
is just one condition.]
HP U household pets = HP
unicorns =U In a pure hypothetical syllogism, all propositions
are hypothetical or conditional. For example,
We can reach the valid conclusion that ‘No unicorns If Arun catches the plane, he will attend
are household pets’. We cannot say ‘Some unicorns are the conference.
not household pets’ as we do not know that unicorns If Arun attends the conference, the new
@UPSC_THOUGHTS

exist at all from the premises. When we say ‘Some are’ project will be launched.
or ‘Some are not’ we affirm the existence of at least one Therefore, if Arun catches the plane, the
of the class. new project will be launched.

Validity rules for pure hypothetical syllogisms are:


II. Hypothetical Syllogisms 1. Both premises have a common factor; in the
In mixed hypothetical syllogisms, the first premise above example, ‘he will attend the conference’
consists of a hypothetical proposition (with an ‘if’ is the common factor.
clause). In the second premise, which is a categorical 2. The common factor should be the antecedent in
proposition, the antecedent (the ‘if’ clause) may be one premise and the consequent in another.
affirmed, in which case the conclusion, which is also 3. The conclusion should not have the common
categorical, affirms its consequent. For example: factor, but the other antecedent of one premise
and the other consequent of another premise.
antecedent consequent Symbolically, the following are invalid hypothetical
syllogisms:
1. If it rains, I take an umbrella. (i) If p, then q.
2. It rains. (antecedent affirmed) If q, then r.
Conclusion: Therefore I take an umbrella. Therefore, if r, then p.
(consequent affirmed) (ii) If q, then r.
If p, then q.
It is also valid for the minor premise to deny the
Therefore, if r, then p.
consequent of the major premise, in which case the
conclusion denies the antecedent of the major premise. (iii) If p, then q.
For example: If r, then q.
1. If it rains, I take an umbrella. Therefore, if p, then r.
2. I do not take an umbrella. (consequent denied) (iv) If p, then q.
Conclusion: Therefore it does not rain. If p, then r.
(antecedent denied) Therefore, if q, then r.

3 . 10
Logical Reasoning and Analytical Ability

III. Disjunctive Syllogisms Inductive Logic: Analogy and


The disjunctive (or ‘alternative’) proposition is of the Probable Inference
form ‘Either ... or’, i.e., it has two component propositions, Inductive arguments may be based on examples,
which are its disjuncts. generalisations, analogy, causal connection or other
An example: grounds for belief.
Argument by Example One may infer conclusions
‘Either he is a student or he is an official.’
from specific cases or examples. The number of cases
The two components are ‘He is a student’ and ‘He or examples used may vary from one to several. Take
is an official’. The disjunction does not categorically the example:
affirm the truth of either one of its disjuncts, but
The World Bank gives billions of dollars
says that at least one of them is true; may be, both are
in foreign aid to Catoxia. The people of
true.
Catoxia resent foreign aid. The World Bank
If one premise is a disjunction, and the other is a should discontinue direct foreign aid to
denial or contradictory of one of the two disjuncts, then developing countries.
we can validly conclude that the other disjunct of the
In the above argument, only one example—that of
premise is true. The following is thus a valid argument:
Catoxia—has been used as a premise for reaching the
Either he is a thief or he is a policeman. conclusion that foreign aid should be discontinued to
He is not a thief. developing countries. If it could be shown that Catoxia
Therefore, he is a policeman. is not a developing country, then one premise is false,
The truth of one disjunct of a disjunction does and the argument and conclusion are invalid. If both
@UPSC_THOUGHTS

not imply the falsehood of the other disjunct. Thus a premises are true, can the conclusion be valid? Just one
valid argument is there only where the categorical example has been given. One might argue that most
developing countries welcome foreign aid and, therefore,
premise contradicts one disjunct and the conclusion
that the single example given is irrelevant or atypical.
affirms the other disjunct of the disjunctive premise.
This would weaken the given argument.
IV. Relational Syllogisms Argument by Analogy Reasoning by analogy
In this kind of argument, the propositions show some consists of making a comparison between two similar
relation to one another. Some examples: cases, and inferring that what is true in one case is true
(i) A is more educated than B. in the other.
B is more educated than C. Most of our everyday inferences are by analogy.
Therefore A is more educated than C. (valid) Analogy is at the root of most of our ordinary reasoning
(ii) A is as popular as B. from past experience to what the future will hold.
B is as popular as C. Symbolically, we may express an analogical
argument as
Therefore A is as popular as C. (valid)
A, B, C, D are entities having characteristics
However, there are asymmetrical relational
x1, x2, x3, x4
arguments where there can be some difficulty.
A, B, C also have the characteristic x5
A is a friend of B. Therefore D also probably has the
B is a friend of C. characteristic x5.
We cannot infer that A is a friend of C, as A may Suppose there are the books A, B and C, all written
not even know C! Such relationships cannot be by an author x1. All of them are detective novels, so have
transferred. But if ‘friend’ were replaced by ‘brother’, the common factor x2. They all have a racy style—
we could validly conclude ‘A is brother of C’. So the another common factor x3. They are all interesting to
relation between the subjective and predicative terms read—x4. You come across book D which has the
has to be carefully studied before working out the characteristics x1, x2, and x3. You would conclude that
inference or judging the validity of the inference. D will also have the characteristic of x4—i.e., be

3 . 11
Logical Reasoning and Analytical Ability

interesting to read. The conclusion, as in all induction, A Venn diagram of the above argument is:
is likely or probable, not absolutely certain. The
Benefits
likelihood of the conclusion would be reduced if, for
instance, you establish an analogy on the basis of the
Tariffs on
cover, the number of pages and binding of the book. textiles Tariffs on
It is not just the amount of similarity that makes an X
induction by analogy reliable; it is the relevance of the
Tariff on
similarities. sugar
Another criterion by which analogical arguments
may be judged is the modesty of their conclusions relative
to their premises. If A has a new car that goes 25 km
The analogy here assumes that because two
to a litre of petrol, from this B can infer with some
industries benefit from tariffs, all others will also benefit.
probability that her new car—which is of the same
However, this argument may or may not be correct.
make and model as A’s car—will also give a good
mileage. If B concludes that her car will make 20 km Causality We have already mentioned that relevance
per litre of petrol, her conclusion is very probable; if she is an important criterion in judging analogical
says her car will go 23 km per litre, her conclusion is arguments. The factor of relevance is to be explained
bolder but not so strong; if she concludes her car will in terms of causality: relevant analogies are those that
give an exact mileage of 25 km per litre, she is being deal with causality related attributes or circumstances.
bolder but has a much weaker argument. Analogical arguments may be probably whether they
The number of disanalogies or points of difference go from cause to effect or from effect to cause. They are
between the instances mentioned in the premises and even probable when the attribute in the premise and
@UPSC_THOUGHTS

the instance with which the conclusion is concerned that in the conclusion are all effects of the same cause.
also affects analogical arguments. If, for instance, in the If some symptoms of a disease are present in a person,
argument concerning the cars of A and B, it is added a doctor can predict other symptoms—not because one
that A drives at a steady speed of about 30 km per hour symptom causes the other, but because they are jointly
most of the time while B hardly ever goes at less than caused by the same infection.
60 km per hour, the argument is considerably weakened A common causal fallacy is to treat an insignificant
and the probability of the conclusion greatly reduced. relationship as a causal factor and assuming that a
Greater the dissimilarities among the cases of the sequential relationship implies a causal relationship. If
premises, the stronger the argument. This is not to be two events occur in sequence it is not evidence of a
confused with ‘disanalogies’. Take an example. If five causal relationship.
different people who have bought A’s model of car The following is an example of a causal fallacy.
drive at different speeds and with different degrees of Romi develops a rash whenever exposed to pollen.
care, and yet get an average of 25 km per litre of petrol, On his way home from a hike, he breaks out in a rash.
B’s conclusion that her car will give a good mileage gets Upon applying some ointment, he exclaims. ‘I must
stronger. have taken in pollen.’
A faulty analogy assumes that things that are Romi’s argument may be expanded as:
similar in one respect must be similar in other respects. 1. Rashes are caused by pollen.
Suppose that an economist argues that a ‘tariff on sugar 2. I have a rash.
will help our sugar industry, a tariff on textiles will Therefore, I must have been affected by pollen.
help the textile industry, a tariff on every imported
product will benefit the economy.’ Romi may be correct. However, other phenomena
The above analogy may be stated as: may have caused his rash: unless these can be ruled
1. Tariffs on sugar benefit the sugar industry. out, Romi’s argument is fallacious.
2. Tariffs on textiles benefit the textiles industry. One type of fallacy of causality is a fallacy termed
Therefore, post hoc ergo hoc (after this, on account of this). This
3. A tariff on every imported product benefits the is the proposition that because events follow one
economy. another, one causes the other.

3 . 12
Logical Reasoning and Analytical Ability

Consider the following scenario. In one of a Words and phrases that indicate conclusions
company’s five sales districts, the advertising budget include
was increased 20 per cent, while in the other four so, therefore, hence (it follows that), thus
districts, advertising expenditure was unchanged. Sales
(one way infer), accordingly, in
increased in the first district by nearly 20 per cent while
consequence, as a result
sales remained unchanged in the other four districts.
Did the increased advertising cause the sales increase? All arguments need not, however, have such
This argument is in the form: indicators. In some cases, the context helps us to
1. Event a followed event b. identify the premises and conclusions. You have to
2. So b is the cause of a. locate which statement is the conclusion and which
statements offer supporting evidence. Take the passage:
The inference is weak, as a may have been the effect
of a third factor c. For example, in the sales district with The destruction of forests amounts to self-
the increase, a major competitor may have withdrawn destruction. Their role in controlling pollution,
from the market. absorbing carbon dioxide and slowing global
warming is crucial to the life of the earth.
Order and Context of Premises, Assumptions
and Unstated Conclusions There is no indicator of premise or conclusion, but
you can locate both. Forests are crucial to life on earth
Most arguments do not come in the form of syllogisms
because.… So the destruction of forests amounts of self-
or in ordered premises and conclusions. Also, in real
destruction.
life, deductive and inductive reasoning are not clearly
A conclusion may even be unstated as such, but
separated. One has to do some of both. There are no
understood from the context of the argument. Take:
fixed position for the premises and conclusion in an
@UPSC_THOUGHTS

argument. Take the following statements: Jeevan is interested only in making money,
but you cannot be a good person and be
(i) Arun was talking during the lesson, so
interested only in making money.
he didn’t understand the teacher’s
instructions. You can understand the conclusion here—‘Jeevan
(ii) Arun did not understand the teacher’s is not a good person’—even though it is not stated.
instructions because he was talking It is necessary to classify and connect things and
during the lesson. events in order to analyse the arguments. To aid this
analysis, think of events in terms of time sequence or
The position of the words do not alter the conclusion:
causal (cause-effect) relationships.
in either case, the conclusion is the same, i.e., ‘Arun did
In some cases, an argument is presented in which
not understand the teacher’s instructions’.
one premise is not stated, because the person presenting
Just any proposition cannot be a premise or a
the argument supposes that it is common knowledge
conclusion. A proposition is a premise only where it
or will, for other reasons, be supplied or assumed by
occurs as an assumption in an argument and it is a
conclusion only where it occurs in an argument in the reader or listener. Take the example:
which it is claimed to follow from propositions assumed All transgenic animals are manmade and
in that argument. Arguments need not be confined to as such patentable.
just two premises and a conclusion. There may be many
Here the missing premise is understood—
premises and more than one conclusion. Indeed, one
conclusion may become a premise for a further All manmade things are patentable things.
conclusion. So we can complete the syllogism:
In some arguments, one can identify the premises
All transgenic animals are manmade.
and conclusions from the use of certain words which
Therefore all transgenic animals are
may be called ‘indicators’. Some such indicators for
patentable things.
premises are:
It is a valid syllogism.
because, since, as (may be inferred from),
for, as shown by, follows from (i.e., may Questions are often asked on which assumption/s
be derived from) is/are made in reaching a certain given conclusion.

3 . 13
Logical Reasoning and Analytical Ability

Or a question is asked on which assumption, if true, Analytical Reasoning Problems


can strengthen or support the conclusion in a given The following are, broadly, the types of analytical
passage. Assumptions are the hidden or unstated reasoning problems.
premises. They are valid if the given conclusion can be
1. Ordering or arrangement In this type of question,
drawn from them. At times assumptions appear
you are asked to place the ‘players’ in a certain
correct, but are not so. It is your analytical ability that sequence or arrange them according to the given
helps you analyse or break down the argument conditions. The arrangement may be linear, circular or
and see whether given assumptions are correct in the vertical or in order of finishing in a race or test or in
context. ascending/descending order of height, weight, etc. The
Your ability to analyse an arguments and identify information will not be complete, but enough for you
premises (stated or otherwise) will be tested by questions to derive some more information, if necessary, and
that ask which statement, if true, can strengthen or answer the questions.
weaken a given argument. An unstated conclusion In ordering problems, you may be asked to
may have to be identified and understood to answer ● determine the location within the sequence of a
a question on logical inference. Questions may be particular player;
asked on the truth or falseness of inferences. Or on ● determine which players may or must be next
cause and effect of events. The rules of logic and to/across from each other in the sequence;
fallacies need to be kept in mind while answering all ● determine which players cannot be next to/

such questions. across from each other in the sequence;


● identify all players that must come earlier or
Analytical Ability and Analytical Reasoning later in the sequence than a given player;
@UPSC_THOUGHTS

It has already been pointed out that logical reasoning ● identify the number of positions separating two
and analytical ability are not exactly two separate players within the sequence.
skills. They are closely related. Passages, even single 2. Selection These problems require you to select a
statements at times, have to be analysed before you smaller group of ‘players’ from a larger group according
arrive at the solution asked for, to say whether the given to certain conditions. These questions will provide the
inference is correct, and what could make an argument list of ‘players’ and certain facts/characteristics unique
stronger or weaker. When ‘analytical reasoning’ is to each player and the conditions according to which
considered in the context of competitive examinations, the groups have to be chosen.
specific question formats come to mind. Often known Analytical reasoning selection problems may require
as ‘games’ with ‘players’, these questions test your you to answer questions such as:
ability to work on the basis of set conditions; you have ● identify the players that must be selected based
to interpret those conditions and apply the rules to a on the criteria/facts provided;
specific question or , more likely, a number of questions. ● identify the players that are eligible for selection

Generally, the information provided is partial—you based on the criteria/facts provided;


have to use your reasoning ability to fill in the gaps. ● identify the players that are ineligible for selection

This is what tests your analytical power. based on the criteria/facts provided;
The question does not necessarily consist of a ● assuming some specific player or players are

single cohesive statement, but presents a set of conditions selected, identify the players that must be
usually in two parts. The first section introduces the selected;
● assuming some specific player or players are
problem, presenting the ‘players’; the players could be
people, animals, things—anything. The second section selected, identify the players that cannot be
selected;
outlines the conditions or rules that apply to the
● given certain facts, identify the number of players
‘players’. Then comes the actual question which you
eligible for selection.
are to answer. Of course, the information on the players
and the conditions that govern them may be put 3. Allocation or Matching Allocation problems are
together in a single passage. similar to selection problems but differ in one vital

3 . 14
Logical Reasoning and Analytical Ability

aspect: allocation problems take care of accommodating ● It is only after you have understood and visualised
everyone, whereas selection problems ask you to choose the information given and arranged it
a few. Allocation problems may ask you to match the diagrammatically (in most cases) that you can pick
players with certain characteristics. your answer response; in other words, simple
In analytical reasoning (games) allocation problems, elimination method does not usually work in these
you may be asked to answer questions such as: problems.
● identify which players must be included in a Go through the sample of worked examples, and
certain group/category; practise the questions given in the next part. They cover
● identify which players could be included in a almost every type of question that go under the name
certain group/category; of logical reasoning and analytical ability. As we have
● identify which players are ineligible for inclusion provided explanations for practically every question,
in a certain group/category; you can not only check your answer but also understand
● determine the number of players that are to be
how each one is worked out.
placed in a certain group/category;
● determine which players have to be, cannot be,
WORKED EXAMPLES
or could be paired with other specific players.
Within the above types, there may be a variety of I. Statement/Conclusion
problems. Some may combine the types. There may be A common type of question in logical reasoning is the
routes to choose, touching or not touching certain Statement/Conclusion type. Two or three statements
locations; there may be buildings to allot to different are given followed by two or three (sometimes four)
people who have their own conditions of co-existing conclusions and the answer responses. You have to
@UPSC_THOUGHTS

with one another; there may be time schedules to match pick the answer response that has the correct numbers
with plays and people or with trains and planes; there of the conclusion/s that should follow from the given
may be recruitment problems based on certain rules, or statements.
allotment of duties as per conditions. The easiest way to solve these problems is to reduce
the statements to Venn diagrams, from which you can
Best Approach to Analytical deduce whether any or more than one of the given
Reasoning Problems conclusions follows. It helps to remember the rules of
● First and foremost, you need to read all the logical deduction and the fallacies.
conditions carefully, and absorb them so that you
can apply them to the question proper.
Directions (Qs. 1 and 2): In each of the questions
below are given two statements followed by four
● Confine yourself to the information provided and
conclusions. You have to take the given statements to
what can be derived from the information given.
be true even if they seem to be at variance with
● As you read the problem, work on it mentally and
commonly known facts. Read all the conclusions and
jot down the facts—the qualities/characteristics/
then decide which of the given conclusions logically
details associated with each ‘player’ as well as the
follows from the given statements disregarding commonly
relationships among them, time sequences if any,
known facts.
and so on. Use the usual symbols— > for greater,
taller, coming before another in a race; < for the Q.1. Statements: Some houses are offices. Some offices
opposite; × for not having a characteristic, e.g., ‘not are schools.
R’ for not red; for son for daughter (or any Conclusions: I. Some schools are houses.
symbol you like—just remember what it is). II. Some offices are houses.
● You need to work the information into a diagram III. No house is school.
or table/chart—visualising the problem makes it IV. Some schools are offices.
easier to answer the questions. (a) Only I follows
● See to it that you incorporate all the given (b) Only I and II follow
information into your diagram/chart. Once you do (c) Only IV follows
that, you can fill in the gaps with a bit of thinking. (d) Only II and IV follow

3 . 15
Logical Reasoning and Analytical Ability

Q.2. Statements: Some taxis have horns. Some taxis placing the circle for S, as you have not been told that
have lights. ‘No house is a school’ in the statements.
Conclusions: I. Every taxi has either horn or
light.
S
II. Some taxis have neither horn nor
light.
III. Some taxis have horns as well as O H
lights.
IV. No taxi has horn as well as light.
(a) Only I and II follow
In this case, there could be some houses that are
(b) Only II and III follow
schools, and some schools that are houses (Conclusion
(c) Only II and IV follow
I). However, in this case, too, conclusions II and IV are
(d) Either III or IV follows
valid.
Solutions: 1. As you cannot reach a universal So the only definite conclusions are II and IV
conclusion from two particular statements, you may Conclusion I may or may not follow. In the case of I
cross out III. If some houses are offices, some offices being valid, III would become invalid.
would also be houses. So conclusion II is correct. As Answer response (d) is correct.
some offices are schools, some schools would be offices.
So conclusion IV is correct. The most likely correct 2. At first sight, as the statements are both particular,
answer response is (d). no universal conclusion should follow, and conclusion
The picture is clearer and it is easier to verify the IV may be rejected. However, among the answer
@UPSC_THOUGHTS

validity of the conclusions, if you draw a Venn diagram responses there is an either/or option. So you cannot
for the statement. reject any conclusion out of hand.
Take a circle to represent House = H Let’s draw a Venn diagram for the statements.
Office = O There are three entities—taxis (T), horns (H) and
School = S lights (L)
As some houses are offices, you have overlapping Take a circle to represent each and make a diagram
circles. for the statements.
We can represent them in more than one way.

O H T taxis with
taxis with H horn
(i) lights L

offices that are offices/ taxis with neither


houses that are offices horn nor light

As some offices are schools, add another circle taxis with


L T
overlapping that of offices. taxis that neither horn
(ii) have both nor light
houses that are offices/ light and
horn H
offices that are houses

T
S O H [as ‘some’ could mean
50 per cent or half the
(iii) H L total number]

offices that are schools/


schools that are offices
Now, let’s check the conclusions with reference to
If you think about it, there may be another way of each diagram.

3 . 16
Logical Reasoning and Analytical Ability

From (i) we can say ‘Some taxis have neither horn C. All poor people are lazy people.
nor light’ (conclusion II). D. Some who are lazy are not poor people.
From (ii) we can say ‘Some taxis have horns as well E. Some poor people are Indians.
as lights’ (conclusion III). F. All lazy people are poor people.
However, the two conclusions are mutually (a) ABD
exclusive, and cannot be valid at the same time, so (b) CEA
answer response (b) may be rejected—II and III cannot (c) FBA
follow together. (d) CAB
From (iii) we can say ‘Every taxi has either horn Solution: Take each answer response and see if the
or light’. So conclusion I may follow. However, it cannot set of statements form a logically related group.
follow alongwith conclusion II which says ‘Some taxis Take (a):
have neither horn nor light’. So answer response (a) A. Some Indians are lazy people.
may be rejected—I and II cannot follow. B. Some Indians are poor people.
Conclusions II and IV cannot follow together. D. Some who are lazy are not poor people.
From diagrams (i), (ii) and (iii) we can see that The rule says no negative conclusion follows two
conclusions II and IV are not the ‘only’ ones to follow. affirmative premises. The argument is not valid. The
It is possible as seen from (iii) that conclusion II may answer becomes clearer with a Venn diagram.
be invalid when conclusion IV is valid, i.e., one cannot
say from this representation of the statement that ‘Some I = Indian
taxis have neither horn nor light’. So answer response L P = Poor people
P I
(c) is out. L = Lazy people
@UPSC_THOUGHTS

Now diagram (ii) validates conclusion III. Diagrams


(i) and (iii) validate conclusion IV. But obviously, the
two cannot occur at the same time; either III or IV can So, you may say, it is possible that ‘Some who are
lazy are not poor people’.
occur.
But the same statements may be sketched as:
So the correct answer response is (d).

II. Logical Compatibility or Logically I


Related Statements P

Questions may be set on logical order or compatibility.


In such questions a series of numbered statements is L
given. Each answer response puts some of these in a
set of certain order. You have to choose the answer In this case, at least some poor and lazy people
have members in common. So (D) cannot be concluded
response that gives a ‘logical order’ or the set of
definitely from A and B. And we are concerned only
statements that make a ‘logical group’. You need to
with definite conclusions, not with ‘may be’s’ and
remember the rules of logical validity in syllogisms as
possibilities. Validity follows only if the premises give
well as the fallacies to avoid to answer these questions.
rise to a definite conclusion.
However, even Venn diagrams can be helpful in
Answer response (a) is out.
eliminating the incorrect options and reaching the
correct answer. Take (b):
C. All poor people are lazy people.
Directions: The following question contains six E. Some poor people are Indians.
statements followed by four answer responses each giving A. Some Indians are lazy people.
a combination of three. Choose the answer response that As poor people are totally contained within lazy
gives the set in which the statements are logically related. people, whoever is poor will also be lazy. So if some
A. Some Indians are lazy people. poor people are Indians, those Indians who are poor
B. Some Indians are poor people. will also be lazy.

3 . 17
Logical Reasoning and Analytical Ability

followed by two (sometimes three) other statements


L L which are the assumptions—or the premises that could
I
lead to the given conclusion. You are asked to decide
P I P
if either, neither or both of the assumptions is/are
implicit. In other words, you have to see if the conclusion
can follow any of the assumptions. You have to use
In either case, some Indians are lazy people. some of your analytical ability here, as it is at times easy
So CEA is valid; answer response (b) is correct. to confuse assumption with inferences. You need to
Check (c) and (d) as well to make sure. judge whether a given assumption can be a premise
(c) gives: that validly leads to the given conclusion.
F. All lazy people are poor. Sometimes, you have to consider an assumption as
B. Some Indians are poor people. just that—as something you assume is possible or
A. Some Indians are lazy people. likely—before doing some action. In, for instance, you
post a certain notice, your assumption is that people
will read it and follow what it says.
I
P 1. Consider the following set of statement and
conclusion:
L Statement: All those who are born in independent
India are Indian citizens.
Conclusion: Sarita is an Indian citizen.
@UPSC_THOUGHTS

Which of the following would be the assumption


Conclusion (A) does not follow from (F) and (B).
necessary to reach the given conclusion?
(Even if there is one possibility of the conclusion not
(a) Sarita is born to parents of Indian origin
following from the premises, no definite conclusion
(b) Sarita was born after August 15, 1947
follows, and the statements are not logically related.)
(c) Sarita was born in independent India
So (c) is out.
(d) Sarita’s parents are Indian citizens
Take (d):
E. Some poor people are Indians. Solution: Remember that you have to establish a
F. All lazy people are poor people. valid connection between the statement and the
A. Some Indians are lazy people. conclusion. Nothing is said about parents of Indian
origin or parents being Indian citizens in the statement
or conclusion, so (a) and (d) are not suitable assumptions.
P
The link is to be found in (c):
I
L All those born in independent India are Indian
citizens.
(c) Sarita was born in independent India.
(A) need not follow from E and F. Answer response (Therefore) Sarita is an Indian citizen.
(d) is out. Option (b) appears correct—after all, you know
The correct answer is (b). India got independence of August 15, 1947. But the
statement says ‘born in independent India’, not after
III. Statement-Assumptions
Indian independence—two rather different propositions.
An assumption could be an unstated premise in a By this assumption, Sarita could have been born outside
logical argument. However, the assumption should be India, too. So (b) is not correct.
something that can truly be taken for granted in reaching Answer response (c) is correct.
the given or valid conclusion.
In this type of question, a statement—which is Directions (Qs 2 to 6): In each question below
taken as the conclusion of an argument—is given, is given a statement followed by two assumptions

3 . 18
Logical Reasoning and Analytical Ability

numbered I and II. An assumption is something (c) Only II and III are implicit
supposed or taken for granted. You have to consider the (d) None of them is implicit
statement and the following assumptions and decide
6. Statement: In spite of heavy rains, the traffic has
which of the assumptions is implicit in the statement.
not been disrupted this year.
2. Statement: Detergents should be used to clean Assumptions:
clothes. I. Traffic has been light this year.
Assumptions: II. The roads have been repaired and drains
I. Detergents form lather. cleaned before the rainy season.
II. Detergents help to dislodge grease and dirt. III. Heavy rains tend to disrupt traffic movement.
(a) Only I is implicit (a) Only I
(b) Only II is implicit (b) Only II and III
(c) Both I and II are implicit (c) Only III
(d) Neither I nor II is implicit (d) Only I and III
3. Statement: ‘Private property; trespassers will be Solutions: 2. The statement mentions detergents
prosecuted.’—A notice on a plot of land. and their use in cleaning. Whether they produce lather
Assumptions: or if it is the lather that cleans is not known; it has not
I. Passersby will read the notice and not been mentioned. So I is not a valid assumption. Clearly
trespass. if detergents are used to clean clothes, one assumes that
II. People are scared of prosecution. they will have a way of doing so—what assumption
(a) Only I is implicit II suggests. So only II is implicit. Answer response (b)
(b) Only II is implicit is correct.
@UPSC_THOUGHTS

(c) Neither I nor II is implicit


3. One puts up a notice or issues a circular with
(d) Both I and II are implicit
the assumption that it will be read and followed by
4. Statement: The regulatory authority has set up a people. So assumption I is valid. One issues a threat
review committee to find the reasons for the with the assumption that it will cause people to desist
fluctuating stock prices. from a certain action, and the threat is effective only
Assumptions: if those it is meant to affect are afraid of it—and so will
I. Investors may regain confidence in the stock desist from a certain action. So assumption II is also
market. valid. Both assumptions being valid, the correct answer
II. The review committee has the expertise to find response is (d).
out the causes for volatility in the stock
4. Assumption I clearly states an aim of setting up
market.
a review committee; the assumption is valid. The
(a) Only I is implicit
committee is likely to do what it is intended to do only
(b) Only II is implicit
if it has the required expertise. So assumption II is also
(c) Neither I nor II is implicit
valid. Answer response (d) is correct.
(d) Both I and II are implicit
5. If there is a boom in a certain business, you can
5. Statement: There is a big boom in the drug
assume there is a demand for that product, so an
business and a number of jhuggi-jhopari dwellers
increase in drug addiction could lead to the increased
in Delhi can be seen peddling small pouches of
drug business. And as Delhi is mentioned as a scene
smack and brown sugar.
for such drug peddling, one can assume I. Assumption
Assumptions:
II is not warranted: You can peddle drugs without
I. Drug addiction is on the increase in Delhi.
using them. Assumption III is nowhere indicated. So
II. Jhuggi-jhopari dwellers are drug addicts.
only I is implicit. Answer response (a) is correct.
III. It is possible for jhuggi-jhopari people to
peddle drugs without being caught. 6. Assumption I is not warranted. In any case it is
(a) Only I is implicit an inference rather than an assumption. II is also an
(b) Only I and II are implicit inference: if the heavy rains have not affected the traffic

3 . 19
Logical Reasoning and Analytical Ability

it is possible that road repair and drain cleaning have 3. I. Rural and semi-urban areas around Delhi
been done. Only III is valid, as the statement indicates have been suffering heavy load shedding for
by use of ‘in spite of’. So answer response (c) is correct. some time now.
II. If steps are not taken soon, urban areas of
IV. Cause and Effect Delhi will face load shedding.
Many events have a cause and many actions or events
Solutions: 1. The two statements appear causally
have an effect. Logical reasoning helps in connecting
connected. II is the effect of I. So (a) is the correct answer
cause and effect, identifying and differentiating the
response.
cause and effect. Even as some events have a clear
cause-effect relationship, there are others which have 2. Clearly I and II are not cause or effect of one
no such causal truth; they merely ‘seem’ connected another. They don’t seem to have a common cause.
causally, but are in truth independent of one another. They have independent causes. So (c) is the correct
There may also be events that have a common cause. answer response.
Analytical ability and logical reasoning come into 3. It is clear that both I and II are related to power
play when you consider statements to see if a causal shortage. Both have a common cause. So (d) is the
relationship exists between them. answer response.
Questions usually come in the form of two
statements. You are expected to analyse them and V. Passage-Based Reasoning
answer if one is the causative agent and the other, the This is not to be confused with ‘Comprehension’,
effect of that causative agent. There are also options though in both cases, a passage is given with questions
stating that the two events have different causes and following the passage. Here the passage has to be read
are unrelated; or that they are not causally related, but even more carefully—no skimming will do; for shades
@UPSC_THOUGHTS

are both effects of the same cause. of meaning are important. Your analytical ability and
logical reasoning skill are both required here—you
Directions: Each question below has two statements must be able to isolate and identify various components
numbered I and II. There may be a cause and effect of a given argument. You need to determine the precise
relationship between them, or the two may both be the function of every sentence in the passage—usually
effects of the same cause. Or they may be the effect of called the ‘stimulus’. Recall the important facts of
independent causes. Read both statements and decide logical reasoning—premise and conclusion indicators,
which of the following answer responses correctly depicts stated and unstated premises and conclusions, what
the relationship between the two statements, and mark strengthens or weakens an argument. Above all, unless
accordingly. you are specifically asked your opinion, don’t give it;
(a) If statement I is the cause and statement II is keep within the scope of the argument presented.
the effect ● Inferences or Main Point: You may be asked
(b) If statement II is the cause and statement I is what inference can be made from the passage.
the effect The process of inferring is a matter of
(c) If both statements I and II are effects of considering one or more statements as
independent causes evidence and then drawing a conclusion from
(d) If both statements I and II are effects of some them. An inference can but need not be the
common cause author’s main point alone; it can be a less
central point. To be valid, it must be true if the
1. I. The government reduced the tax on petroleum
statements in the passage are true; it could be
products last week. a small extension of the argument rather than
II. The prices of petroleum products dropped a necessary part of it.
this week.
● True/False: Besides being asked: ‘What is the
2. I. Many elderly people are harassed by inference?’ or ‘Which inference is true?’, you
youngsters in locality X. may be asked to decide on the degree of truth
II. Children living in locality X play till late in or falsity of statements with regard to the
the evening. passage.

3 . 20
Logical Reasoning and Analytical Ability

● Assumptions: You may be asked about the adequately supported by the proposition that any
assumptions made in the passage. This is a drug that treats the disease is more effective than
little more demanding than the statement no treatment at all. What must be taken into
assumption questions (see III in this part), but account is that drug ABC is very expensive and
the principles are the same. You must not has notable side effects.
confuse ‘assumptions’ and ‘inferences’. Of Which one of the following most accurately
course, assumptions are not stated in the expresses the main point of the doctor’s argument?
passage. But if one of the answer responses (a) Drug ABC is more effective than no treatment
is something not stated in the passage, it does at all
not necessarily qualify as an assumption; it (b) Drug ABC is more expensive than other forms
could be an unstated conclusion or inference. of treatment for the disease
Of course, if an answer response can be found (c) Drugs should not be used to treat the disease
directly in the passage, it cannot be an unless they are either effective or inexpensive
assumption. Assumptions support the (d) The possible effectiveness of drug ABC in
conclusion of the passage. An assumption treating disease XYZ is not sufficient
bridges the gap between an argument’s justification for using it
evidence and conclusion, so first locate the
Solution: Here you are required to find the main
conclusion. If the argument suggests a cause
point of the argument by the doctor. Option (a) may
for some event/thing, you should ask yourself
state a truth, but it is not the main point made by the
if any other cause might be there. If the
doctor. The doctor’s argument suggests that other factors
argument makes use of statistics, you might
beyond mere effectiveness—for instance, cost and side
see if it is representative. This will also help
@UPSC_THOUGHTS

effects—have to be considered before deciding to use the


you determine whether the assumptions are
drug. If you glance at the options, (d) appears to be the
valid. If you find a new idea or word in the
correct one: the drug’s effectiveness is not enough to
conclusion, as compared to the ideas in the justify its use. Option (b) goes beyond the passage
evidence, that idea/word should give you a which makes no comparison of the cost of this drug
clue about the hidden premise or assumption. and other treatments of the disease. Option (c) distorts
● Strengthening and Weakening Arguments: the words of the doctor who makes no either/or
Questions relating to strengthening or statement between effectiveness and low cost. Answer
weakening the argument of the passage are response (c) is correct.
often based on the assumptions. You need to Q. 2. In recent years, attacks by Rottweilers on small
choose answer responses to confirm the children have risen greatly. Last year saw 40 such
assumptions of the passage to strengthen the attacks in Delhi alone, an increase of almost 20
argument; preferably, the ‘strengthening’ per cent over the previous year’s total. Clearly,
answer response will have some new then, it is unsafe to keep dogs as pets if one has
information which will support the argument. small children in the house.
However, there could be similarities in logic.
If you are asked to find a response that The argument above depends upon which of the
following assumptions?
weakens the argument, you have to look for
(a) Other animals such as cats are better pets
something that makes the conclusion less
than dogs if there are small children in the
tenable—say conditions that would contradict
house
or call into doubt the conclusion.
(b) Rottweilers are dangerous and could have
been trained to prevent attacks
Directions: In each of the items below (Qs. 1 to 5)
(c) No reasonable justification for the attacks by
read the given passage and the questions below it and
Rottweilers has been discovered
select the best answer response.
(d) The behaviour of Rottweilers towards children
Q. 1. A doctor says: The continued use of drug ABC is representative of the behaviour of dogs in
to treat patients with disease XYZ cannot be general

3 . 21
Logical Reasoning and Analytical Ability

Solution: The passage talks about attacks by judges are more lenient, but that would actually
Rottweilers, but the conclusion is a generalisation about strengthen the argument. So (d) is not the right choice.
dogs. That may be a faulty argument, but you have not Option (a) is best.
been asked to find fault, but to find the assumption. It
Q.4. Around 1950, there were about 300 farms in XY
is only if the behaviour of the Rottweiler is representative
district. By the 1970s the number had dropped to
of the behaviour of dogs in general that the conclusion
200, and by 1998 there were only 100 farms in
of the argument can be reached. So (d) is the answer.
operation. Therefore, the amount of land in the
Option (a) is outside the scope of the passage. One
district that is devoted to farming has dropped by
does not know whether training would or would not
about 65 per cent.
have helped; anyway option (b) does not bridge the gap
between the conclusion and the rest of the passage. The Point out the flaw in the above argument, if any.
same holds true for (c); whether the attacks were (a) From absolute numbers it moves on to
justified is also beside the point, as the conclusion does percentages
not change. (b) It does not say whether the land diverted from
farming is being used productively
Q.3. The rate of violent crime in XYZ state has risen
(c) It ignores the possibility that the average size
35 per cent compared to last year. The fault lies
of farms has changed
entirely in our court system: our judges in recent
(d) There is no flaw
times have handed out such lenient sentences that
criminals can now do almost anything without Solution: Consider the answer responses. Moving
fear of a long time in prison. from absolute numbers to percentages is no flaw, so (a)
is out. The conclusion is not about productive use of
Which of the following, if true, would weaken the
@UPSC_THOUGHTS

land, only about land devoted to farming; so option (b)


argument above?
is irrelevant. Option (c) points out the flaw. A reduction
(a) The vacancies in the police force of XYZ state
in the number of farms does not automatically mean
have increased in the last year and have not
less land for farming; it could mean many farms have
been filled due to budget constraints
been brought together. Option (c) is correct. Option (d)
(b) A 25 per cent increase in white collar crime
becomes irrelevant.
is also seen in XYZ state
(c) Human rights groups oppose the death Q.5. Scientists today accept that the increased severity
penalty of hurricanes in the last ten years has been a
(d) Almost 15 per cent of the judges have been result of warmer water in the Caribbean, which
appointed last year adds to the intensity of the storms as they pass
over it by a mechanism not yet completely
Solution: You are required to weaken the argument.
understood. Thus, these severe hurricanes are yet
The crux of the argument is that the rise in crime is
more evidence of global warming.
entirely due to the court system. The argument would
be weakened if we can find another cause for the rise Which of the following, if true, would strengthen
in violent crime. the above argument?
Answer response (a) gives you an alternative cause: (a) The arctic ice caps have been losing an average
as the police force is depleted, crime control suffers. of 1 metre in circumference for the past three
However, check the other answer responses as well. years
Option (b) strengthens the argument: if there is also an (b) There is a lack of accurate statistics on the
increase in white collar crime, the court’s leniency may warming of the earth for the time before 100
be responsible for the overall increase in crime. So (b) years or so
is out. If human rights activists oppose death penalty, (c) There is no undersea volcano whose eruption
they are not against long prison terms. Further the could have fuelled a new undersea current
‘opposition’ to death penalty does not quite translate which could have funneled warmer water
into increase in crime ‘compared to last year’. So (c) is into the Caribbean Sea
not relevant. It could be that the newly appointed (d) A group of scientists believe they will soon

3 . 22
Logical Reasoning and Analytical Ability

prove that convection is the mechanism by mentions wounds being a factor in turning tigers into
which a storm picks up energy from warm maneaters, and wounds are not directly related to old
water age. Nothing in the passage contradicts the statement
Solution: You have to strengthen the argument either, so (c) is not correct. So one may choose (d)—data
which connects the warming of the waters of the is inadequate for reaching the inference.
Caribbean to global warming. Consider the answer (ii) This seems to be true at first glance, but you
responses. Option (a) strengthens the case for global should read the statement carefully. The word ‘only’
warming in general rather than support the particular makes the statement untrue, for the passage clearly
argument under consideration. It cannot be the correct mentions other factors than wounds that lead a tiger
choice. Option (b) actually weakens the argument, so to kill humans. So the inference is definitely false.
it is out. Option (c) seems to be the right one, as it Answer response (c) is correct.
removes a possible alternate cause for warmer water in
(iii) The passage says that worn down claws
the Caribbean Sea. (If to weaken an argument, you have
contribute to a tiger’s inability to catch animals, thus
to find an alternate cause, to strengthen an argument,
leading them to kill human beings. It implies that a tiger
alternate causes have to be removed.) Option (d) does
not help to make the argument stronger or weaker. So needs sharp claws to catch the animals in the forest.
the correct answer is (c). So you may go with answer response (a).

Q.6. A tiger, when killing its prey, which it does either (iv) This is a little tricky. The statement is not to
by stalking or lying in wait for it, depends for the be found in the passage. However, it is clear from the
success of its attack on its speed and, to a lesser passage that when disabilities prevent tigers from
catching animals—their natural prey—tigers turn to
@UPSC_THOUGHTS

extent, on the condition of its teeth and claws.


When, therefore, a tiger suffers from one or more killing humans. It is implied that humans are easier to
painful wounds or when its teeth go missing or catch and kill than are other animals. The statement is,
are defective and its claws get worn down, and therefore, probably true. Answer response (b) is correct.
it is unable to catch the animals it has been [The last type of passage-inference item requires
accustomed to eating, it is driven by necessity to you to read the passage and statements very carefully.
killing human beings. You should take care not to supply information from
your own knowledge in choosing the answer response.
Now consider the following statements and choose You need to stick to what the passage says.]
the answer response in the context of the passage.
(a) if the inference is definitely true VI. Arrangement, Selection and
(b) if the inference is probably true Allocation Problems
(c) if the inference is definitely false
(d) if the data is inadequate to reach the given Directions (Qs. 1 to 3): Study the following information
inference to answer the given questions—
(i) Old age propels tigers towards becoming Five boys A, B, C, D and E are sitting in a circle
maneaters. with three girls F, G and H. They are all facing the
centre. A is the only boy seated between two girls. B
(ii) A tiger kills humans only when it has been
is seated second to the left of E. F is seated second to
incapacitated by wounds.
the right of H who is a neighbour of A. C is to the
(iii) Sharp claws are needed by tigers to catch and immediate left of B. One of B’s neighbours is a girl. A
kill animals in the forest. is third to the left of B.
(iv) Human beings are easy prey of tigers.
Q.1. Who is G’s right hand side neighbour?
Solutions: (i) The passage does not mention (a) E
anything about old age. You may infer that missing (b) B
teeth and worn down claws are to be associated with (c) C
old age, but it is not necessarily so. Also, the passage (d) F

3 . 23
Logical Reasoning and Analytical Ability

Q.2. What is A’s position with respect to E? reference to whom the question is put. Here you have
(a) Third to the left to determine A’s position from E’s place—which is
(b) Immediate right third to E’s right. It could also be fifth to the left—but
(c) Third to the right that is not asked.
(d) Fourth to the right
3. (b) Here again, direction is an important
Q.3. How many people are sitting between C and H component of the question. Clockwise means towards
when counted in a clockwise direction from C? left. You have F and A between C and H; i.e. two people.
(a) 1
(b) 2 Directions (Qs. 4 to 6): Study the following
(c) 6 information carefully and answer the questions, which
(d) 4 follow:
Solution: In such questions a sketch helps. Draw Five plays A, B, C, D and E were organised in a
a circle and cut off as many places on the circumference week from Monday to Saturday with one play each day
as there are ‘players’—here 8 persons of which 5 are
and no play was organised on one of these days. Play
boys and 3, girls. Start by placing any one at any place.
D was organised before Thursday but after Monday.
Just remember the direction ‘right’ and ‘left’ is with
Play E was organised on Saturday. Play C was not
regard to the ‘players’. Mark the direction so that you
organised on the first day. Play B was organised on the
don’t make a careless mistake. You get the information
day next to that on which play C was organised. Play
H is neighbour of A who is the only boy sitting between
two girls. A is third to left of B. C is to the immediate A was organised on Tuesday.
left of B and B is second to the left of E. Sketch all that
@UPSC_THOUGHTS

Q.4. On which day was play B held?


in. (a) Monday
A (b) Friday
Right (c) Saturday
(d) None of these days
C
Left Q.5. On which day was there no play?
B E
(a) Monday
Now you know F, G and H are girls. H is a (b) Thursday
neighbour of A, but if F—also a girl—is second to the (c) Saturday
right of H, H cannot be to the right of A, as C occupies (d) Data inadequate
that place. So H must be A’s neighbour on the left. As
Q.6. Which of the following play and day is correctly
A has girls on both sides, A’s right neighbour must be
matched?
F—second to right of H. As one of B’s neighbours is
I. Friday – D
a girl, place G there. D is clearly between E and H.
Complete the diagram: II. Wednesday – C
II. Thursday – B
A (a) I and II
F H
Right
(b) II and III
C D (c) Only I
Left (d) I, II and III
B E
G Solution: It is best to spend some time arranging
the available information in a table/chart, for then the
So now all the places are clear. And the questions
question can be answered accurately and fast. [No
may be answered easily.
Sunday. And it is easier if you have the days in order—
1. (a) B is on G’s left, and E is on her right. then you can just place the play against each day.] Now
2. (c) Be careful of the direction and the person with fill in information you definitely have.

3 . 24
Logical Reasoning and Analytical Ability

Day Play will remain unchanged as compared to the original


Monday seating positions?
Tuesday A (a) One
Wednesday (b) Two
Thursday (c) None
Friday (d) Cannot be determined
Saturday E Solutions: This is a straight line with the ‘players’
As play D was organised after Monday but before facing north. Draw the line and mark off and number
Thursday, it can only be on Wednesday. Now, as C was the 6 positions. Plug in what is definitely known first.
not on the first day, and B followed on the day after N
C, the only days for C and B are Thursday and Friday, ?P/Q/T P/R/T?
respectively. Complete the table: 6 5 4 3 2 1

Day Play × Q Q ×
S ×
Monday × No play × S
× L L ×
Tuesday A
As Q sits fourth to the left of T, but not at the end,
Wednesday D
Q can only be in place 5 and T is in 1. As only one
Thursday C person sits between R and T, R occupies place no. 3.
Friday B
As L can’t sit next to Q, L can only be in place no. 2.
Saturday E
The rest is easy.
4. (b) N
@UPSC_THOUGHTS

5. (a) 6 5 4 3 2 1
6. (d) P Q S R L T

Directions (Qs. 7 to 9): Study the following 7. (b) S and R


information carefully and answer the given questions. 8. (c)
9. (b) For this you have to rearrange the ‘players’
L, P, Q, R, S and T are sitting in a straight line alphabetically and compare with the earlier positions.
facing North. It is seen that the positions of T and R are unchanged.
Q sits fourth to the left of T and Q does not sit at
an extreme end of the line. L P Q R S T
R is not an immediate neighbour of Q.
Directions (Qs. 10 and 11): Read the following and
Only one person sits between R and T.
answer the questions:
Neither S nor L sits at the extreme end of the line.
L is not an immediate neighbour of Q. A man plants only five different kinds of
vegetables—beans, corn, peas, potato, cabbage. Every
Q.7. How many persons sit between Q and L?
year the man plants exactly three kinds of vegetables
(a) One according to the following restrictions:
(b) Two
(c) Three If corn is planted, beans is also planted that year.
(d) Cannot be determined If cabbage is planted one year, it is not planted next
year.
Q.8. The left end position is occupied by In any year the man plants no more than one of
(a) T the vegetables he planted in the previous year.
(b) L
Q.10. Which of the following is a possible sequence of
(c) P
combinations for the man to plant in two
(d) Either T or R
successive years?
Q.9. If all the persons are made to sit in alphabetical (a) Beans, corn, peas; beans, corn, potato
order from left to right, the positions of how many (b) Beans, peas, potato; beans, corn, cabbage

3 . 25
Logical Reasoning and Analytical Ability

(c) Corn, peas, potato; beans, cabbage, peas If A washes on day 4, D washes on day 5.
(d) Cabbage, peas, potato; beans, corn, cabbage If B washes on day 2, G washes on day 5.
If H washes on day 6, D washes on day 4.
Q.11. If the man plants beans, corn and cabbage in the
first year, which of the following can be a Q.12. Which of the following is an acceptable order in
combination for the third year? which the campers can wash dishes from the first
(a) Beans, corn, and cabbage to the last day?
(b) Beans, corn and peas (a) D, B, A, G, C, H
(c) Beans, peas and potato (b) B, A, H, C, G, D
(d) None of the above (c) H, G, B, C, D, A
(d) C, B, A, D, G, H
Solutions: This problem needs no sketch; you can
reach your answer by taking up the responses one by Q.13. If A washes on day 1, who washes on day 2?
one and seeing if it follows the given conditions. (a) B
(b) C
10. Take response (a). You would reject it (c) D
immediately as it violates the last condition: two (d) G
vegetables cannot be planted again in successive years,
and this option shows beans and corn appearing in Q.14. If B washes on day 2, which of the following is
both. a complete and accurate list of the days that could
Option (b) seems to follow the conditions. Only be the day on which H washes?
(a) 4
beans is repeated next year, and other vegetables are
(b) 1, 4
all new.
(c) 4, 6
@UPSC_THOUGHTS

Option (c) is wrong as corn cannot be planted


(d) 1, 4, 6
without beans.
Option (d) is not correct as cabbage appears in two Solution: This is another type of problem where
successive years. there is no need to sketch or mark a table. Looking up
So option (b) is correct. the answer responses and comparing each with the
conditions will get you the answers.
11. Just work out what can be planted in the
second year. Corn cannot, since if it were, beans too 12. Take (a). So the conditions do not mention D
would have to be planted, and then the third restriction on the first day. Move on to B on the second day. The
would be broken. Now, as per the second restriction, sixth condition says if B washes on day 2, G is on day
cabbage cannot be planted in successive years; only the 5. But here it is C. So (a) is out.
other three vegetables can be grown in the second year, Take (b). The very first condition is violated here;
namely, beans, peas and potato. Beans can be planted B cannot wash on the first day. So (b) is out.
without corn, as the first restriction only implies that Take (c). We have B on day 3, violating the very
corn cannot be planted without beans. So, the vegetables first condition. So option (d) alone can be correct. Check
of the third year can only be cabbage, beans and corn— it out. Start with B on day 2—G on day 5 is correct.
option (a). The last condition is also fulfilled: H on day 6, D is
on day 4. The fifth condition is also fulfilled: if A
Directions (Qs. 12 to 14): Six campers—A, B, C, washes on day 4, D is on day 5. The only day left for
D, G and H—are arranging a dish washing schedule C is the first, and there is no restriction on that.
for the six days of their camping trip so that each of So option (d) is correct.
them wash dishes on only one day.
Directions (Qs. 15 to 17): Read the following
B washes either on day 2 or on day 6.
information and answer the questions.
If A washes on day 1, C washes on day 4; C does
not wash on day 4 unless A washes on day 1. Five friends, P, Q, R, S and T travelled to five
If A washes on day 1, H washes on day 5; H does different cities—Chennai, Kolkata, Delhi, Bengaluru
not wash on day 5 unless A washes on day 1. and Hyderabad—by the different modes of transport of
If G does not wash on day 3, A washes on day 3. bus, train, plane, car and boat from Mumbai.

3 . 26
Logical Reasoning and Analytical Ability

The person who travelled to Delhi did not travel C, D and E are parallel to one another, and roads G,
by boat. H, I, J, K, L and M are parallel to one another.
R went to Bengaluru by car and Q went to Kolkata (i) A is 1 km east of B.
by plane. (ii) B is ½ km west of C.
S travelled by boat whereas T travelled by train. (iii) D is 1 km of west of E.
Mumbai is not connected by bus to Delhi or Chennai. (iv) G is ½ km south of H.
(v) I is 1 km north of J.
Q.15. Which of the combinations of place and mode of
(vi) K is ½ km north of L.
transport is not correct? (vii) K is 1 km south of M.
(a) Bengaluru – car
(b) Hyderabad – train Q.18. Which of the following is necessarily true?
(c) Chennai – boat (a) E and B intersect
(d) Cannot be determined (b) D is 2 km west of B
(c) D is at least 2 km west of A
Q.16. The person travelling to Delhi went by (d) M is 1.5 km north of L
(a) bus
(b) train Q.19. If E is between B and C, which of the following
(c) car cannot be true?
(a) C is less than 1.5 km from D
(d) None of the above
(b) Distance from E to B added to distance
Q.17. Who travelled to Delhi? between E and C is ½ km
(a) P (c) D is 2 km west of A
(b) S (d) E is less than 1 km from A
@UPSC_THOUGHTS

(c) T
Solutions: There seems to be a lot of information
(d) Cannot be determined
and distances, etc. Let’s go step by step. It may be
Solutions: Sketch a chart and fill in what is available necessary to draw different sketches instead of a
at first reading: composite one. In these kind of questions, we should
P → not waste time trying to get a single diagram; nor
Q → Kolkata by plane should we assume that it is impossible to answer the
R → Bengaluru by car questions. Let’s take the data.
S → by boat The first two statements we can conclude A, B and
T → by train C run in the north-south direction. So we have:
Now only P is left with bus as the only mode of B C A
travel. As Delhi and Chennai are not reachable by bus, N
P must have gone to Hyderabad (the only place left). ½ km
W E
S travelling by boat can’t go to Delhi, so T must have
gone to Delhi, and S to Chennai. You have all the
S
necessary information to answer the questions.
P → Hyderabad by bus 1 km
(i) and (ii)
Q → Kolkata by plane
From (iii) and (iv) we have figures
R → Bengaluru by car
S → Chennai by boat D E
T → Delhi by train H
15. (b) 16. (b) 17. (c) 1 km ½ km
G
Directions (Qs. 18 and 19): Study the given
information and answer the questions.
All the roads of a city are straight and either (iii) (iv)
perpendicular or parallel to one another. Roads A, B, From (v), (vi) and (vii) we have figures

3 . 27
Logical Reasoning and Analytical Ability

M Option (a) may or may not be true; if E is closer


to C than half way between B and C, the distance
between C and D could be less than 1.5 km. But if E
I 1 km
is closer to B than to C, the distance between C and
D could be more than 1.5 km. So option (a) is uncertain.
1 km K Option (b) is true: as distance between B and C is
½ km ½ km, and E is between B and C; wherever E is
J L positioned, EB + EC will be ½ km. So (b) is to be rejected.
Option (c) says D is 2 km west of A. D is west of
A, but the distance of 2 km can never be true unless
(v) (vi) and (vii)
E coincides with B. So (c) definitely cannot be true.
Now let’s take the questions. It would be best to Let’s check (d). As AB is 1 km and E lies between
look at the options and use the elimination method. A and B, E will always be less than 1 km from A. So
18. Option (a) may be rejected immediately as being (d) is true.
parallel, E and B cannot intersect. It is not possible with The correct answer response is (c).
the given data to say whether options (b) and (c) are Q.20. Golu is taller than Sallu but not taller than
true or not. Option (d) must be correct. And checking Rollo. Rollo and Tullu are of the same height. Golu is
it with the diagram for (vi) and (vii) we find it is true. shorter than Alu. Which one amongist these is the
So option (d) is correct. shortest?
(a) Alu
19. More data is provided for this question, namely,
(b) Rollo and Tullu
that E is between B and C. So combine sketch for (iii)
(c) Golu
@UPSC_THOUGHTS

with sketch for (i) and (vi):


(d) Sallu
D B E C A Solution: Take the first letter of each person to
represent the height, and use symbols for ‘taller’, etc.
So we have
½ km G > S
R G (Rollo could be equal in height to Golu)
R = T
1 km
1 km
A > G
From the above we can conclude that Golu is either
We have used dotted lines for E and D, as their shorter than or equal in height to each of the others
exact positions are not known. Now take the options except Sallu. This indicates that Sallu is the shortest
for Q.19 one by one. You are asked to find which one amongst the given persons.
cannot be true, i.e., definitely cannot be true. Conclusion (d) is correct.

3 . 28
Logical Reasoning and Analytical Ability

PRACTICE EXERCISES
Directions: In the following questions, two or three (b) If only conclusion II follows
statements are given. Choose the answer response that (c) If both conclusions follow
gives the correct conclusion logically following from the (d) If either conclusion follows
given statements. You have to take the given statements
6. Statements: 1. All hands are machines.
as true even if they seem at variance with commonly
known facts. 2. All machines are wheels.
Conclusions: I. All wheels are hands.
1. Statements: 1. Some clocks are radios.
II. All hands are wheels.
2. No radio is a laptop.
(a) No clock is a laptop. 7. Statements: 1. Some stones are shells.
(b) Some clocks are laptops. 2. All shells are pearls.
(c) No laptop is a radio. Conclusions: I. Some stones are pearls.
(d) All the above. II. All pearls are shells.
2. Statements: 1. Some pins are forks.
8. Statements: 1. Some eyes are ears.
2. All forks are keys.
2. Some ears are hands.
(a) Some pins are keys.
(b) Some keys are pins. Conclusions: I. No hand is eyes.
(c) Some keys are forks. II. Some eyes are hands.
(d) All the above.
@UPSC_THOUGHTS

9. Statements: 1. Some caps are shirts.


3. Statements: 1. Some rats are dogs. 2. All shirts are papers.
2. Some dogs are horses. Conclusions: I. All shirts which are caps are
3. Some horses are camels.
papers.
(a) Some horses are rats.
II. Some shirts are caps.
(b) Some camels are horses.
(c) Some horses are not rats. 10. Statements: 1. Some lions are tigers.
(d) No logical conclusion can be drawn. 2. All lions are cats.
4. Statements: 1. All banks are beams. Conclusions: I. Some cats are lions
2. All bridges are beams. II. Some tigers are cats.
3. All bridges are cows.
(a) Some bridges are banks. Directions: In each of the questions below there are
(b) All cows are beams. three statements followed by two conclusions. Decide
(c) Some banks are bridges. which of the conclusions logically and always follow/
(d) None of the above. follows from the three statements. You have to take the
5. Statements: 1. All roofs are cameras. given statements as true even if they seem at variance
2. Some cameras are photographs. with commonly known facts. Give answer
3. Some photographs are stores. (a) If only I follows
(a) Some stores are cameras. (b) If only II follows
(b) Some stores are roofs. (c) If neither I nor II follows
(c) Some cameras are roofs. (d) If both I and II follow
(d) Some stores are not cameras.
11. Statements: All districts are cities.
Directions: In the following questions, two statements All states are cities.
are followed by two conclusions. Read them and mark Some cities are countries.
your answers as follows: Conclusions: I. Some states are districts.
(a) If only conclusion I follows II. Some countries are states.

3 . 29
Logical Reasoning and Analytical Ability

12. Statements: All keys are locks. Directions: In each of the following questions, three
No lock is a door. or four statements are given followed by three or four
All doors windows. conclusions. Decide which of the conclusions logically
Conclusions: I. No key is a door. follows or follow from the given statements and choose
II. Some windows are locks. your answer from the given answer responses. You have
to take the given statements as true even if they seem
13. Statements: All books are pages.
at variance with commonly known facts.
All libraries are books.
All words are pages. 21. Statements: All books are teachers.
Conclusions: I. All words are books. All teachers are chairs.
II. All libraries are pages. All schools are chairs.
14. Statements: Some clouds are ashes. Conclusions: I. Some chairs are schools.
Some ashes are particles. II. Some teachers are books.
All particles are elements. III. Some chairs are books.
(a) Only I follows
Conclusions: I. No particle is a cloud.
(b) Only I and III follow
II. Some elements are ashes.
(c) I, II and III follow
15. Statements: All ships are aeroplanes. (d) None follows
All trucks are ships.
All cars are trucks. 22. Statements: All snakes are trees.
All roads are mountains.
Conclusions: I. Some ships are not cars. Some trees are roads.
II. All cars are aeroplanes.
@UPSC_THOUGHTS

Conclusions: I. Some mountains are snakes.


16. Statements: No table is wood. II. Some roads are snakes.
Some wood are chairs. III. Some mountains are trees.
All chairs are rocks. (a) Only I follows
Conclusions: I. No rock is table (b) Only II follows
II. Some rock is wood. (c) Only III follows
(d) None follows
17. Statements: All typhoons are hurricanes.
All winds are typhoons. 23. Statements: Some fruits are dogs.
All clouds are winds. Some dogs are cats.
Conclusions: I. All typhoons are clouds. All cats are goats.
II. All clouds are hurricanes. Conclusions: I. No goat is a fruit.
II. Some goats are fruits.
18. Statements: Some floppies are mouses.
III. No goat is a dog.
All mouses are speakers.
All speakers are monitors. (a) Only I follows
Conclusions: I. Some monitors are floppies. (b) Either I or II follows
II. All mouses are monitors. (c) I, II and III follow
(d) None of them follows
19. Statements: All rooms are buses.
No bus is a train. 24. Statements: Some pens are swords.
Some trains are cars. All swords are dogs.
Some dogs are foxes.
Conclusions: I. No room is a train
II. Some cars are buses. Conclusions: I. Some pens are foxes.
II. All dogs are sowrds.
20. Statements: All islands are countries. III. All foxes are sowrds.
All stars are islands. (a) Only I follows
All planets are countries. (b) Only II follows
Conclusions: I. Some stars are not countries. (c) Only III follows
II. No planet is a star. (d) None of them follows

3 . 30
Logical Reasoning and Analytical Ability

25. Statements: Some apples are pumpkins. Conclusions: I. Some machines are boxes.
All pumpkins are rotten. II. Some taps are calculators.
Some potatoes are rotten. III. Some boxes are calculators.
Conclusions: I. Some apples are rotten. IV. No machine is calculator.
II. Some potatoes are apples. (a) Only I and II follow
III. Some pumpkins are potatoes. (b) Only I and IV follow
IV. Some pumpkins are apples. (c) Only II and III follow
(a) Only I follows (d) I, II and III follow
(b) Only I and II follow 30. Statements: All fruits are leaves.
(c) Only I and IV follow Some leaves are trees.
(d) Only IV follows No tree is house.
26. Statements: All pens are pencils. Conclusions: I. Some houses are fruits.
Some pens are erasers. II. Some trees are fruits.
Some erasers are clips. III. No house is fruit.
Conclusions: I. Some clips are pens. IV. Some trees are leaves.
II. No clip is a pen. (a) I and IV follow
III. Some erasers are pencils. (b) Only IV follows
IV. No eraser is a pencil. (c) Either I or III and IV follow
(a) Only I and II follow (d) I, III and IV follow
(b) Only III follows 31. Statements: Some trains are cars.
(c) IV and either I or II follow All cars are buses.
@UPSC_THOUGHTS

(d) III and either I or II follow All buses are shoes.


27. Statements: Some doors are windows. Some shoes are dresses.
All windows are blue. Conlcusions: I. Some dresses are cars.
Some blue are brown. II. Some shoes are trains.
Conclusions: I. Some windows are brown. III. Some buses are trains.
II. All doors are blue. IV. Some dresses are trains.
III. Some doors are blue. (a) Only II follows
IV. No. window is brown. (b) Only II and III follow
(a) Only III follows (c) Only I, III and IV follow
(b) III and either I or IV follow (d) Either II or IV follow
(c) I and IV follow 32. Statements: Some pens are keys.
(d) II and IV follow Some keys are doors.
28. Statements: All teachers are doctors All doors are trees.
All doctors are engineers. All trees are monkeys.
All engineers are typists. Conclusions: I. Some monkeys are pens.
Conclusions: I. Some typists are teachers. II. Some trees are pens.
II. All doctors are typists. III. Some monkeys are doors.
III. Some engineers are teachers. IV. Some trees are keys.
IV. All doctors are teachers. (a) Only I and II follows
(a) Only I follows (b) Only II and III follows
(b) Only I and III follow (c) Only III and IV follows
(c) Only IV follows (d) I, II, III and IV follows
(d) I, II and III follow 33. Statements: All pigs are cows.
29. Statements: All calculators are boxes. Some cows are bats.
All boxes are taps. Some bats are lamps.
Some taps are machines. All lamps are rats.

3 . 31
Logical Reasoning and Analytical Ability

Conclusions: I. Some rats are bats. Some buds are flowers.


II. Some lamps are cows. No hammer is a flower.
III. Some bats are pigs. Conclusions: I. All hammers are tools.
IV. Some rats are cows. II. No tool is a flower.
(a) Only I and II follow III. Some hammers are buds.
(b) Only I, II and III follow IV. Some flowers are tools.
(c) Only III and IV follow (a) Only I follows
(d) None of the above (b) Only III follows
34. Statements: All pencils are clips. (c) III and either II or IV follow
Some clips are trains. (d) II, III and IV follow
Some trains are wheels. 38. Statements: Some chairs are handles.
Some wheels are buses. All handles are pots.
Conclusions I. Some buses are trains. All pots are mats.
II. Some wheels are clips. Some mats are buses.
III. Some wheels are pencils. Conclusions: I. Some buses are handles.
IV. Some buses are clips. II. Some mats are chairs.
(a) Only I follows III. No bus is handle.
(b) Only II follows IV. Some mats are handles.
(c) Both I and IV follow (a) Only I and II follow
(d) None follows (b) Only II and IV follow
@UPSC_THOUGHTS

35. Statements: All sickles are houses. (c) Only I and III follow
No house is a room. (d) None follows
Some rooms are doors. 39. Statements: Some benches are walls.
All doors are windows. All walls are houses.
Conclusions: I. Some windows are sickles. Some houses are jungles.
II. Some windows are rooms. All jungles are roads.
III. No window is a sickle. Conclusions: I. Some roads are benches.
IV. Some rooms are sickles. II. Some jungles are walls.
(a) Only I follows III. Some houses are benches.
(b) Either I or III follows IV. Some roads are houses.
(c) II and either I or III follow (a) Only I follows
(d) Only II follows (b) Only III follows
36. Statements: All spiders are dogs. (c) Either III or IV follows
No dog is a cow. (d) Only III and IV follow
All cows are pigs. 40. Statements: Some sticks are lamps.
Some pigs are mynas. Some flowers are lamps.
Conclusions: I. No myna is a cow. Some lamps are dresses.
II. No spider is a cow. All dresses are shirts.
III. No pig is a dog. Conclusions: I. Some shirts are sticks.
IV. No myna is a dog. II. Some shirts are flowers.
(a) Only II and III follow III. Some flowers are sticks.
(b) Only II follows IV. Some dresses are sticks.
(c) Only III and IV follow (a) Only I follows
(d) None follows (b) Only IV follows
37. Statements: Some tools are hammers. (c) II and III follow
All tools are buds. (d) None follows

3 . 32
Logical Reasoning and Analytical Ability

Directions: Each of questions 41 to 50 contains six 3. Some athletes play cricket.


statements followed by four sets of combinations. Choose 4. Some cricket players are vegetarians.
the set in which the statements are logically related. 5. No cricket player is vegetarian.
41. 1. All voters are residents. 6. Some athletes are vegetarians.
2. Some voters are citizens. (a) 3 4 1
3. All citizens are residents. (b) 3 4 6
4. No citizen is a resident. (c) 1 2 5
5. All voters are citizens. (d) 2 5 6
6. Some voters are residents. 46. 1. All secretaries are stenographers.
(a) 1 3 2 2. Some girls are secretaries.
(b) 1 5 4 3. Some stenographers are boys.
(c) 3 5 1 4. All girls are stenographers.
(d) 3 5 4 5. Some boys are stenographers.
6. Some secretaries are girls.
42. 1. No wealthy persons are vagrants. (a) 1 6 5
2. All lawyers are wealthy persons. (b) 5 6 1
3. Some lawyers are vagrants. (c) 1 4 2
4. Some wealthy persons are not vagrants. (d) 4 2 3
5. No lawyers are vagrants.
6. All lawyers are vagrants. 47. 1. No one who is strict is employed.
(a) 1 2 3 2. All those who are employed are strict.
(b) 2 5 4 3. No one who is strict is a friend.
@UPSC_THOUGHTS

(c) 1 2 6 4. All friends are strict.


(d) 1 2 5 5. Some friends are employed.
6. Some friends are not strict.
43. 1. Some intellectuals are shy and retiring people. (a) 1 3 5
2. Some shy and retiring people are successful (b) 1 5 6
politicians. (c) 1 2 3
3. No intellectuals are successful politicians. (d) 2 4 5
4. All shy and retiring people are intellectuals.
5. Some successful politicians are intellectuals. 48. 1. Some politicians are honest people.
6. No shy and retiring people are successful 2. Some politicians work hard.
3. All honest people work hard.
politicians.
4. All people who work hard are honest.
(a) 3 6 1
5. Some who work hard are politicians.
(b) 3 4 6
6. Some who are honest are politicians.
(c) 6 1 5
(a) 5 6 4
(d) 3 2 4
(b) 3 2 1
44. 1. All teachers are women. (c) 1 2 3
2. All students are women. (d) 3 6 2
3. All teachers are students.
49. 1. Some cooks are lazy people.
4. All students are teachers.
2. Some cooks are poor people.
5. Some women are teachers.
3. All poor people are lazy people.
6. Some women are students.
4. Some who are lazy are not poor.
(a) 6 2 5
5. Some poor people are cooks.
(b) 1 2 3
6. All lazy people are poor people.
(c) 2 3 1
(a) 3 5 1
(d) 5 6 3
(b) 1 2 4
45. 1. No athletes are vegetarians. (c) 3 1 2
2. All cricket players are athletes. (d) 6 2 1

3 . 33
Logical Reasoning and Analytical Ability

50. 1. All participants in this programme are boys. of the former. Failing to file a tax
2. All participants in this programme are girls. return illustrates the latter. In some
3. No boy is a girl. countries, no distinction is made
4. Some girls are participants in this programme. between the two kinds of crime.
5. No girl is a participant in this programme. Conclusions:
6. Some boys are girls. I. Some countries have no legislature.
(a) 1 3 6 II. In some countries a distinction between the
(b) 5 4 1 two kinds of crime is made.
(c) 1 2 3 (a) Only I follows
(d) 1 3 5 (b) Only II follows
(c) Both I and II follow
Directions: Each of questions 51 to 65 has one or two (d) Neither I nor II follows
statements or a passage followed by some conclusions.
54. Statements: Electronic governance helps to
Choose the correct answer response about which conclusion
involve stakeholders such as
follows in the light of the statements.
interested citizens and social groups
51. Statements: There are several charges of in devising ways of meeting public
corruption against officer X. Anyone challenges. E-governance promotes
facing corruption charges should be democracy. Not all developing
suspended, and an inquiry launched countries have adopted e-gover-
into the charges. nance.
Conclusions: Conclusions:
@UPSC_THOUGHTS

I. Officer X should be questioned about the I. Some developing countries are not democratic.
charges and then suspended. II. Some developing countries do not have e-
II. Officer X should be suspended. governance systems.
(a) Only I follows III. Without e-governance democracy ceases to
(b) Only II follows exist.
(c) Both I and II follow (a) Only I and III follow
(d) Neither I nor II follows (b) Only II follows
52. Statements: There were more than two lakh (c) I, II and III follow
immigrants in country X in 2001. (d) None of them follows
Many of these immigrants were 55. Statements: It takes more than six months to start
employed in professional occupa- a small business in India because of
tions. For instance, many were legal formalities to be completed. A
engineers or doctors. country in which legal formalities
Conclusions: take more than two months is
I. Some of the engineers working in the country backward.
were immigrants. Conclusions:
II. Some of those employed in professional I. India is backward.
occupations were not immigrants. II. A country in which legal formalities are
(a) Only I follows completed within two months is forward.
(b) Only II follows (a) Only I follows
(c) Either I or II follows (b) Only II follows
(d) Neither I nor II follows (c) Both I and II follow
53. Statements: Crimes are often characterised as (d) Neither I nor II follows
either inherently evil or criminal 56. Statements: Regions showing high economic
because they are declared offences growth have also shown good
by a legislature. Murder is an example human development indicators.

3 . 34
Logical Reasoning and Analytical Ability

Conclusions: (a) Only I follows


I. All regions with good human development (b) Only II follows
indicators have high economic growth. (c) Both I and II follow
II. There is no link between economic growth (d) Neither I nor II follows
and human development indicators.
60. Statements: Bindas Hoteliers Co. was unable to
(a) Only I follows
(b) Only II follows pay interest on its debts. Bankrupt
(c) Both I and II follow companies are unable to pay interest
(d) Neither I nor II follows on their debts.
Conclusions:
57. Statements: All countries with good governance I. Bindas Hoteliers Co. was bankrupt.
show good economic growth but II. Bindas Hoteliers Co. was criminal.
only some democratic countries show
(a) Only I follows
good economic growth.
(b) Only II follows
Conclusions:
(c) Both I and II follow
I. Some democratic states do not have good
(d) Neither I nor II follows
governance.
II. Democratic countries with good governance 61. Statement: In deserts, camels are indispensable
show good economic growth. for people to travel from one place
(a) Only I follows to another.
(b) Only II follows Conclusions:
(c) Both I and II follow I. Camels are the cheapest mode of transport.
@UPSC_THOUGHTS

(d) Neither I nor II follows II. Camels are easily available in deserts.
58. Statements: A study of indigenous inhabitants of (a) Only I follows
country X found that two-thirds of (b) Only II follows
the children developed considerable (c) Both I and II follow
levels of eye trouble after starting (d) Neither I nor II follows
school, while their illiterate parents
62. Statement: Smoking is one of those human
and grandparents, who had no
weaknesses which tends to test the
opportunity for formal schooling,
will power of the smoker to the edge.
showed no signs of eye trouble.
Conclusions:
Conclusions:
I. Human beings have many weaknesses.
I. Illiterate people have no eye trouble.
II. It is difficult for smokers to give up smoking
II. One-third of the children are illiterate.
III. Only literate people develop eye trouble. even if they want to do so.
(a) Only I and II follow (a) Only I follows
(b) Only II and III follow (b) Only II follows
(c) I, II and III follow (c) Neither I nor II follows
(d) None of them follows (d) Both I and II follow

59. Statements: Being a good football player is 63. Statement: The old order changes yielding place
contingent upon being able to run to the new.
fast, jump high, and lift weights. Conclusions:
Suraj can run fast, jump high, and I. Old ideas should be discarded.
lift weights, but Mayank cannot do II. Change is always there.
all this. (a) Only I follows
Conclusions: (b) Only II follows
I. Suraj is a good football player. (c) Both I and II follow
II. Mayank is not a good football player. (d) Neither I nor II follows

3 . 35
Logical Reasoning and Analytical Ability

64. Statements: Television programmes telecast Assumptions:


specially for women are full of 1. Gambling is normally reprehensible.
recipes and household hints. 2. Those who conduct the lotteries are evading
Conclusions: taxes.
I. Programme producers think women are 68. Statement: The government decided to levy a
interested most in cooking and housekeeping. charge of Rs 50 on every vehicle
II. Most women like learning recipes and using the super highway between
household hints and are not interested in two major cities of the state.
other things. Assumptions:
(a) Only I follows 1. The cost of constructing the highway can be
(b) Only II follows recovered over time from this charge.
(c) Neither I nor II follows 2. Use of the highway by cyclists and slow-
(d) Both I and II follow moving vehicles needs to be stopped.
65. Statements: Those who ignore the facts are likely 69. Statement: A ban has been imposed by the state
to be mistaken. No one who is truly administration of country X on the
objective is likely to be mistaken. gathering of more than twenty people
Conclusions: at any place during the visit of a
I. No one who ignores the facts is truly objective. certain foreign dignitary.
II. Some objective people are mistaken. Assumptions:
(a) Only I follows 1. Huge rallies against the foreign dignitary
were expected as many exiles from his country
@UPSC_THOUGHTS

(b) Only II follows


were residing in country X.
(c) Neither I nor II follows
2. A red alert had been issued on the security
(d) Both I and II follow
of the visiting dignitary.
Directions: In each of questions 66 to 90, is a statement 70. Statement: Please send an official letter rather
followed by two assumptions. Consider the statement and than a semi-official letter on this
assumptions and decide which of the assumptions is/ subject.
are implicit in the given statement. Assumptions:
Choose option: 1. The format and emphasis of the different
(a) if only assumption I is implicit types of letters are different.
(b) if only assumption II is implicit 2. One type of letter may be more effective than
(c) if both assumptions are implicit the other on certain subjects.
(d) if neither assumption is implicit 71. Statement: The government has decided to
66. Statement: Even though the number of factories auction the construction of highways
to private sector organizations in
producing sugar in this country has
several blocks across the country on
increased, sugar import has
a build-operate-transfer basis.
continued.
Assumptions:
Assumptions:
1. There are many private sector organizations
1. Demand for sugar has increased more than
in the country capable of constructing
what is produced even after the increase in highways within reasonable time.
number of factories producing sugar. 2. Private sector organizations would be willing
2. Sugar prices may increase in the future, and to undertake the project as the proposal of
import of the commodity would keep the build-operate-transfer would make it
prices in check. financially attractive.
67. Statement: Government has banned gambling 72. Statement: The government has urged the
through lotteries. citizens to use the electronic media

3 . 36
Logical Reasoning and Analytical Ability

for their daily transactions whenever Assumptions:


possible, as the use of paper requires 1. The army can be used for such purposes.
cutting down large number of trees, 2. The army can succeed in this goal where
thus causing damage to the other authorities have failed.
ecosystem.
78. Statement: The cotton crop continues to be poor
Assumptions:
even after the introduction of
1. Citizens are environmentally aware enough
improved variety of seeds.
to adapt to the directive.
Assumptions:
2. Most people have access to electronic media
1. The new variety of seeds was expected to
to conduct daily transactions.
boost production of cotton.
73. Statement: Government has decided to have 2. The cotton crop is important for the economy.
special coaching classes in all state
79. Statement:A. Jonathan has taken over as the
capitals for civil service aspirants
new director of XYZ institution and
from the weaker sections of society.
Assumptions: now the administration of the
1. All civil service aspirants require coaching. institution will improve.
2. Aspirants from weaker sections of society do Assumptions:
not have the money to study in private 1. XYZ institution had no director before A.
coaching institutions. Jonathan was appointed.
2. A new director is required for administration
74. Statement: The United Nations is hardly of XYZ institution to improve.
@UPSC_THOUGHTS

approached by member nations for


settlement of bilateral disputes these 80. Statement:The support price for various crops
days. should be announced before their
Assumptions: sowing seasons.
1. There is an erosion in the authority of the Assumptions:
United Nations. 1. Farmers are influenced by the price of their
2. One of roles of United Nations has been to products before they choose what to sow.
settle international disputes. 2. Hoarding is to be checked.

75. Statement: Revenue from direct tax collections 81. Statement: The oil producing countries decided
has increased greatly in the last five to reduce the production of crude in
years. order to increase profit.
Assumptions: Assumptions:
1. Tax laws have been made stringent. 1. The price of crude would increase due to less
2. More people have come within the income tax production.
bracket. 2. Countries buying crude from the producers
would continue to do so.
76. Statement: The laws against terrorism have been
made very stringent after the recent 82. Statement: Give adequate job training to the
attacks on government institutions. new employees before assigning
Assumptions: them full-fledged work.
1. Stringent laws can be effective in countering Assumptions:
terrorism. 1. Training leads to efficiency in performance.
2. Government has failed in its attempts to 2. New employees lack the specific skills required
control terrorism. for the work.
77. Statement: The army should be called in to 83. Statement: The government has released a large
restore peace and normalcy in the quantity of foodgrains from its buffer
riot-infested area. stock in the pre-harvest kharif season.

3 . 37
Logical Reasoning and Analytical Ability

Assumptions: would be fined Rs 1,000 and their


1. There could be a shortage of foodgrains in the driving licences would be
market in this season. impounded.
2. Kharif production will be enough to replenish Assumptions:
the buffer stock. 1. Such a threat would be effective in getting
drivers to comply with the warning.
84. Statement: The driver of a huge truck put the
2. Drivers would continue to use their cell phone
emergency brakes when he saw a
while driving and be forced to pay huge fines
cyclist suddenly veer into the truck’s
and lose their licences.
way.
Assumptions: 89. Statement: The government has decided to run
1. The cyclist would see the danger and move all commercial vehicles on bio-fuel
out of the truck’s way. in order to save the depleting fossil
2. The truck would be able to stop before it hit fuel resources.
the cyclist. Assumptions:
85. Statement: The chairman of the company issued 1. Enough bio-fuel can be produced in the
a circular urging the employees to country.
refrain from long personal calls 2. Technical changes in vehicles required to
during working hours. switch over from fossil fuels to bio-fuels are
Assumptions: easy to accomplish.
1. Most employees would respond positively if 90. Statement: The local music society decided to
@UPSC_THOUGHTS

the chairman issued such an appeal. organise a concert to raise money for
2. Making long personal calls in working hours the construction of the club building.
adversely affects productivity. Assumptions:
86. Statement: The police cordoned off the entire 1. The response to the concert would be large
locality for an entire day and stopped enough to raise a substantial amount of
all vehicular traffic when a highly money.
placed government official was to 2. People are all interested in music.
visit, in view of the security threat
Directions: In each of questions 91 to 95, a statement
perception, and advised all residents
is followed by three assumptions. Consider the statement
of the area to limit their movement
and assumptions and decide which of the assumptions
outside their dwellings.
Assumptions: is/are implicit in the statement.
1. Security would be easier to handle with no 91. Statement: Considering the tickets sold in the
vehicles and few pedestrians on the streets. last seven days, the circus authorities
2. People living in the area may move out of decided to continue the show for
their houses in defiance of police action. another fortnight which includes two
weekends.
87. Statement: The doctor warned the patient
Assumptions:
against the consumption of alcohol
I. People coming at weekends would be more.
if his ailment was to be cured.
II. The daily average number of visitors to the
Assumptions:
circus would be maintained.
1. It was the doctor’s duty.
III. Other places may not show enough
2. The patient would heed the doctor’s advice.
interest.
88. Statement: The traffic police has put up huge (a) Only I is implicit
hoardings along roads, warning (b) I and II are implicit
drivers to refrain from using cell (c) II and III are implicit
phones while driving, else they (d) I, II and III are implicit

3 . 38
Logical Reasoning and Analytical Ability

92. Statement: The telephone company informed 95. Statement: “If he proves a single charge against
the subscribers through a notice that me, I vow that I will quit politics
those who did not pay their bills by forever.”—A politician in a speech.
the due date would be charged Assumptions:
penalty for every day of default. I. The charges levelled against the politician are
Assumptions: false.
I. Majority of people would pay their bills by II. The charges levelled against the politician are
due date to avoid penalty. beyond proof even if true.
II. Money collected as penalty would compensate III. The politician is fed up with politics.
for the losses incurred due to delayed payment (a) Only I is implicit
of bills. (b) Only II is implicit
III. People generally heed notices. (c) I or III is implicit
(a) I and II are implicit (d) I, II and III are implicit
(b) II and III are implicit
(c) I, II and III are implicit Directions: For questions 96 to 125, passages are given
(d) None of them is implicit followed by inferences. Read each passage and examine
the inferences in the context of the passage concerned
93. Statement: “Why should we not protest? When
and decide upon their degree of truth or falsity. Choose
we ask for drinking water they are
your answer from the given answer responses. The
giving us Pepsi and Coca-Cola.”—A
politician in an interview. inferences range from
Assumptions: ● ‘probably true’ though not ‘definitely true’ in
@UPSC_THOUGHTS

I. Only drinking water is good for people’s the light of the facts given.
health. ● ‘definitely true’, i.e., it properly follows from
II. Providing people drinking water is more the statement of facts given.
important than providing Pepsi and Coca- ● ‘data are inadequate’, i.e., from the facts given
Cola. you cannot say whether the inference is likely
III. Pepsi and Coca-Cola do not contain healthy to be true or false.
drinking water. ● ‘probably false’, though not ‘definitely false’
(a) Only I is implicit in the light of the facts given.
(b) Only II is implicit ● ‘definitely false’, i.e., it cannot possibly be
(c) I and III are implicit drawn from the facts given or it contradicts
(d) Either II or III is implicit the given facts.

94. Statement: “To err is human. Every man makes Passage: Coal and power are the mainstays of
mistakes. But to learn from those India’s energy sector. Coal is the country’s backbone of
mistakes is what makes great men. energy supply and is likely to continue playing a
And this is why there are few of significant role in meeting energy demands. It accounts
great men around us.”—A for over 50 per cent of the commercial energy require-
philosopher in his speech. ments in the country. Despite reported figures of a
Assumptions: relatively large coal resource compared with other fossil
I. Mistakes do not make significant issues.
fuels, the country has been facing shortages from time
II. There are very few persons who learn from
to time. Projections for coal by various study groups
mistakes.
show an increasing gap between the demand and
III. If a man learns from mistakes, he is a great
supply of domestic coal, which may result in increased
man.
(a) Only I is implicit imports. Moreover, the country’s resources are often
(b) Only II is implicit overstated as assessment process does not consider the
(c) Only III is implicit quality, technical feasibility and economic viability of
(d) I and III are implicit coal mining and exploration.

3 . 39
Logical Reasoning and Analytical Ability

96. The coal mines of the developed countries generate (a) Definitely true
more profits than those in India. (b) Definitely false
(a) Probably true (c) Probably true
(b) Probably false (d) Probably false
(c) Definitely true
102. Investors need to critically evaluate the risk of
(d) Cannot say from the given data
each investment option.
97. India’s coal mines and mining activities are (a) Definitely true
commercially viable. (b) Definitely false
(a) Definitely true (c) Probably true
(b) Probably true (d) Probably false
(c) Probably false
103. Present day investors need to use their judgement
(d) Cannot say from the given data
more critically before investing.
98. India’s dependence on coal for generating energy (a) Definitely true
will continue in the time to come. (b) Probably true
(a) Definitely true (c) Probably false
(b) Probably true (d) Cannot say on the given data
(c) Probably false
104. Multiple investment options of similar type makes
(d) Cannot say from the given data
for difficult investment decisions.
99. India’s coal reserves are much more than its (a) Definitely true
requirement. (b) Probably true
@UPSC_THOUGHTS

(a) Probably true (c) Definitely false


(b) Probably false (d) Cannot say on the given data
(c) Definitely false
105. In the past, investors were generally guided by the
(d) Cannot say from the given data
fund managers.
100. Assessment process of coal reserves is not capable (a) Definitely false
of identifying coal reserves accurately. (b) Probably false
(a) Definitely true (c) Probably true
(b) Probably true (d) Cannot say on the given data
(c) Probably false
(d) Cannot say on the given data Passage: Privatisation is no panacea when it comes
Passage: Investors today have more investment to education. Nor can high-cost intervention at the
options than were available just a few years ago. Choice tertiary stage produce quality talent. The backbone of
in any decision making is good in so far it provides quality education is primary schooling. And improving
variety, differentiation and benchmarking. It could also, that is not just a question of funding. The government
however, at times lead to clutter and ‘noise’ if the has taken some steps to improve the situation by
options are mostly similar and undifferentiated. To increasing the percentage of allocation in the budget.
make sense of this choice conundrum, it is imperative But it has done precious little to increase the efficacy
for an investor to define objectives—both returns and of public spending in education. For that is a political
digestible risk and then identify the possible options. and administrative task. Teachers who do not turn up
The investor also needs to select the mix and regularly to teach at rural schools (absenteeism is about 30 per
monitor that objectives and investment outcomes remain cent according to one estimate) and teachers who are
aligned. Sounds simple, but can present the most not equipped or motivated to teach but continue in
confounding situations which multiplies with the service are part of India’s socio-political reality.
quantum of wealth.
106. Motivating the primary school teachers and
101. Investment of higher amount is relatively simpler improving attendance of these teachers are major
than smaller amounts. challenges in India.

3 . 40
Logical Reasoning and Analytical Ability

(a) Definitely true 111. India has failed to take measures to recharge
(b) Probably true groundwater adequately in the northern part.
(c) Probably false (a) Definitely true
(d) Data inadequate (b) Probably true
(c) Probably false
107. The Indian government has not done enough to
(d) Cannot say on the given data
improve the quality of secondary education.
(a) Definitely true 112. Wheat cultivation in India requires comparatively
(b) Definitely true more water than paddy.
(c) Probably false (a) Definitely true
(d) Data inadequate (b) Definitely false
(c) Probably true
108. Government has been unsuccessful in achieving (d) Probably false
the desired result in education.
(a) Definitely true 113. Water level in other parts of India was stable
(b) Probably true during the last decade.
(c) Probably false (a) Definitely false
(d) Cannot say on the given data (b) Probably true
(c) Probably false
109. Quality of education is ensured in all the privately (d) Cannot say from the given data
run educational institutions in India.
114. India has now put in place a system to reduce
(a) Definitely true
over-dependence on groundwater.
(b) Probably true
@UPSC_THOUGHTS

(a) Probably false


(c) Definitely false
(b) Definitely false
(d) Probably false
(c) Probably true
110. Allocating more funds in the budget for primary (d) Cannot say from the given data
education will substantially improve the quality
115. Adequate monsoon helps in drawing less ground
of education.
water for cultivation and thus preserve balance.
(a) Probably true
(a) Definitely true
(b) Definitely false
(b) Definitely false
(c) Probably false
(c) Probably true
(d) Cannot say on the given data
(d) Cannot say from the given data

Passage A recent report that satellite data shows Passage: The first time I saw The Wizard of Oz, the
groundwater levels in northern India depleting by as story bewitched me. The second time I saw The Wizard
much as a foot per year, over the past decade, is a matter of Oz, the special effects amazed me. The third time I
of concern. The clear writing on the wall is that India saw The Wizard of Oz, the photography dazzled me.
faces a turbulent water future and veritable crisis Have you ever seen a movie twice, three times? You
without a proactive policy and sustainable practices. notice subtleties and hear sounds you completely missed
Besides, the poor monsoon this season and the resultant the first time around.
drought situation pan-India calls for sustained policy It’s the same on the phone. Because your business
focus on the water economy. A whole series of glaring conversations are more consequential than movies, you
anomalies need to be addressed. For one, there’s far too should listen to them two, may be three times. Often
much reliance on groundwater. For another, our water we have no clear idea of what really happened in our
infrastructure for storage and supply is sorely phone conversation until we hear it again. You’ll find
inadequate. Worse, policy distortions in artificially shadings more significant than the colour of Toto’s
underpricing key agri-inputs like power have perversely collar—and more scarecrows than you imagined who
incentivised cultivation of water-intensive crops like ‘haven’t got a brain!’
paddy in traditionally wheat-growing areas. How do you listen to your important business

3 . 41
Logical Reasoning and Analytical Ability

conversations again? Simply legally and ethically tape bureaucracy, it can have a significant dampening effect
record them. I call the technique of recording and on business.
analysing your business conversations for subtleties
121. No other developing country except India claims
Instant Replay.
that they have highly trained technical manpower.
116. The movie ‘The Wizard of Oz’ will help improve (a) Definitely true
business conversation. (b) Probably true
(a) Probably true (c) Definitely false
(b) Definitely true (d) Data is inadequate
(c) Probably false
122. Foreign companies are more equipped than
(d) Definitely false domestic companies to provide quality service in
117. For most, if they watch a movie more than once, good time.
different aspects in different order would impress (a) Definitely true
in a better way. (b) Probably true
(a) Probably true (c) Definitely false
(b) Definitely true (d) Probably false
(c) Probably false 123. Official formalities are less cumbersome in almost
(d) Cannot say from the given data all the countries except India.
118. Tips are given for understanding business (a) Probably true
conversation. (b) Probably false
(a) Definitely true (c) Definitely true
@UPSC_THOUGHTS

(b) Probably true (d) Data is inadequate


(c) Probably false 124. Indian service industry was more comfortable
(d) Cannot say from the given data before economic liberalisation.
119. The advice is being given to the sales team. (a) Definitely true
(a) Definitely false (b) Probably true
(b) Definitely true (c) Probably false
(c) Probably false (d) Definitely false
(d) Cannot say from the given data 125. India at present is to some extent on par with
developed countries in terms of technological
120. The author watches most movies more than twice.
development.
(a) Definitely true
(a) Definitely true
(b) Definitely false
(b) Probably true
(c) Probably true
(c) Probably false
(d) Cannot say from the given data
(d) Definitely false
Passage: Economic liberalisation and globalisation Directions: In the questions below (126 to 135), two
have put pressures on Indian industry, particularly on statements (A) and (B) are given. These statements may
the service sector, to offer quality products and services be either independent causes or effects of independent
at low costs and with high speed. Organisations have causes or of a common cause. One of these statements
to compete with unequal partners from abroad. It is well may be the effect of the other statement. Read both the
recognised that developing countries like India are statements and decide which of the following answer
already behind other countries technologically, in many choices correctly depicts the relationship between these
areas, although some of them, particularly India, boast two statements.
of huge scientific and technical manpower. In addition Mark answer (a) if statement (B) is the cause
to this, if an entrepreneur or industrialist has to spend and statement (A) is its effect.
a lot of his time, money and energy in dealing with Mark answer (b) if both the statements (A) and
unpredictable services and in negotiating with the local (B) are independent causes.

3 . 42
Logical Reasoning and Analytical Ability

Mark answer (c) if both the statements (A) and 134. (A) : Increase in rainfall and rising flood
(B) are effects of independent situations are regular phenomena for past
causes. few years.
Mark answer (d) if both the statements (A) and (B) : People avoid going out in heavy rains.
(B) are effects of some common
135. (A) : The health department has advised people
cause.
to drink boiled and filtered water and
126. (A) : Many schools have banned the sale of fast maintain hygiene during the monsoon.
food in their premises. (B) : The health department has instructed the
(B) : Obesity in youngsters has been linked to civic hospitals to equip themselves with
their poor eating habits. adequate stock of medicines during the
127. (A) : The share prices are touching an all-time monsoon.
low.
Directions: In each question below (136 to 140) are
(B) : Most of the organizations have been
grounding or terminating employees and given two statements (A) and (B). These statements may
undergoing cost-cutting exercises wherever be either independent causes or may be effects of
possible. independent causes or a common cause. One of these
statements may be the effect of the other statement. Read
128. (A) : The glaciers at the poles of the earth are
both the statements and decide which of the following
melting at a fast rate.
(B) : In recent times there has been a substantial answer choices correctly depicts the relationships between
increase in the incidents of earthquakes these two statements.
and volcanic eruptions. Mark answer—
@UPSC_THOUGHTS

129. (A) : Though mobile phones find a good number (a) if statement (A) is the cause and statement (B)
of users in rural India, computers and is its effect.
internet still remain a distant dream. (b) if statement (B) is the cause and statement (A)
(B) : In the recent past there has been a large- is its effect.
scale migration from the rural parts of (c) if both the statements (A) and (B) are
India to the urban sectors. independent causes.
130. (A) : The government has decided to take strict (d) if both the statements (A) and (B) are effects
action against the builder responsible for of independent causes.
construction of the bridge which collapsed. 136. (A) : Huge tidal waves wrecked the vast coastline
(B) : The government has decided to build a early in the morning killing thousands of
vehicular bridge over the river connecting
people.
the suburban area to the city.
(B) : Large number of people gathered along the
131. (A) : Many people prefer to shop at the big retail coastline to enjoy the spectacular view of
chains than the local grocery stores. sunrise.
(B) : Retail chains offer a discount of more than
25 per cent as compared to the local grocery 137. (A) : The government has suspended several
shops. police officers in the city.
(B) : Five persons carrying huge quantities of
132. (A) : Indian citizens are willing to incur the cost
illicit liquor were apprehended by police.
of using environment-friendly technology.
(B) : Many countries are taking steps to cut their 138. (A) : The authorities have made speed breakers
carbon emissions. near the intersection of two roads in a quiet
133. (A) : The government has amended tax laws to locality.
boost exports. (B) : There have been many accidents at the
(B) : The export sector has been passing through intersection involving vehicles moving at
difficult times due to heavy tax burdens. high speed.

3 . 43
Logical Reasoning and Analytical Ability

139. (A) : More than two billion pounds of chemicals (c) The communication system was severely
are spewed in the air everyday by industries affected and continues to be out of gear
and vehicles of some countries. (d) Government ordered that all the offices and
(B) : International Pollution Control Organi- schools should be kept open
sation is expected to penalise the countries
144. Statement: The prices of petroleum products have
which cross the maximum emission limits.
increased by about twenty per cent in the past two
140. (A) : The production of pulses has dropped for months.
the third consecutive year.
Which of the following can be a probable cause
(B) : India has decided to import pulses this
of the above effect?
year.
(a) The prices of foodgrains and vegetables have
141. Statement: At least 20 school children were shot up by more than thirty per cent
seriously injured while going for a school picnic (b) The truck owners’ association has decided to
during the weekend. increase their rent by about 20 per cent with
Which of the following can be a probable cause immediate effect
of the above effect? (c) The prices of crude oil in the international
(a) The picnic spot was always considered market have increased considerably during
dangerous as it was very close to a deep lake. the past few weeks
(b) The bus in which the children were travelling (d) People have decided to demonstrate against
met with an accident while taking a turn on the government’s apathy towards rise in
the main highway prices of essential commodities
@UPSC_THOUGHTS

(c) The driver of the bus in which the children 145. Statement: The government has recently increased
were travelling was inexperienced its taxes on petrol and diesel by about 10 per cent.
(d) The school authority banned all school picnics
for the next six months with immediate effect Which of the following can be a possible effect of
the above cause?
142. Statement: Government has recently decided to (a) The petroleum companies will reduce the
hike the procurement price of paddy for the rabi prices of petrol and diesel by about 10 per
crops.
cent
Which of the following will be a possible effect (b) The petroleum companies will increase the
of the above cause? prices of petrol and diesel by about 10 per
(a) The farmers may be encouraged to cultivate cent
paddy for the rabi season (c) The petroleum companies will increase the
(b) The farmers may switch over to other cash prices of petrol and diesel by about 5 per cent
crops in their paddy fields (d) The petrol pumps will stop selling petrol and
(c) There was a drop in production of paddy diesel till the taxes are rolled back by the
during kharif season government
(d) Government may not increase the procurement
price of paddy during the next kharif season Passage (for Q. 146): A recent review reported that
143. Statement: A severe cyclonic storm swept most India’s fashion industry has been affected severely due
parts of the district. to a slump in demand for luxury apparel, leather goods,
besides gems and jewellery in domestic as well
Which of the following cannot be a possible effect international markets.
of the above cause?
(a) Heavy rainfall was reported in most part of 146. Which of the following contradicts the views
the district expressed in the above statement?
(b) Many people were rendered homeless as their (a) India’s fashion industry had been growing
houses were blown away till last year

3 . 44
Logical Reasoning and Analytical Ability

(b) Government has helped India’s fashion (a) Civic mindedness is closely connected to voter
industry to grow in the past participation
(c) Domestic market of fashion goods reflects the (b) Being civic minded requires participation in
sentiments of international market voting
(d) Share of fashion goods manufactured by India (c) Good citizenship is behind a thriving society
is negligible in the international market (d) Mr Balan does his duty as a good citizen
Passage (for Qs. 147 and 148): Rage killings are on Passage (for Qs. 150 and 151): Giving computer
the rise in the city. Police stations have registered 16 education in primary schools is a waste of money. Some
per cent more cases as compared to the corresponding private schools fleece parents in the name of computer
period last year. In most of these cases minor altercations education. Government should better use its resources
have led to the crime. While cautioning the public, the in appointing more teachers and creating facilities.
commissioner of police in a press conference said that Children in primary schools are too young to learn how
the number of rage killings in the city is fast approaching to use computers effectively. They need to learn the
the figure for deaths due to road accidents which is on basics like numbers, arithmetic, reading and writing.
decline. A leading psychologist has attributed this to
the growing complexity of city life and resulting 150. Which of the following, if true, would strengthen
intolerance due to exponential rise in population, large the speaker’s argument?
disparity between social strata and other modern-day (a) Cost of ownership of computers is very high
socio-economic factors. (b) A recent report suggests that computers can
hamper holistic growth of children
147. Which of the following, if true, would make the
(c) Research on the effect of internet on children
psychologist’s argument stronger?
@UPSC_THOUGHTS

(a) Residents dislike influx of outsiders into the (d) Examples of high school students who use
city computers to visit adult web-sites
(b) There is a large and widening difference in 151. Which of the following, if true, would weaken the
income levels speaker’s argument?
(c) Increase in city population has put immense (a) Computers can be used to enhance learning
pressure on basic infrastructure like water, in arithmetic and language
electricity and open space (b) A study on preference of general public
(d) The number of deaths due to road accidents towards computers in primary schools
is declining
(c) Report on computer illiteracy among teachers
148. Which of the following was never intended by the (d) Students in USA have designed a new
Police Commissioner? microchip
(a) The number of deaths due to rage killing can Passage (for Qs 152 and 153): English should be
cross the number of deaths through road the only language used in the Parliament. There is no
accidents reason for the country to spend money printing
(b) The road-safety measures taken by the police documents in several different languages just to cater
have shown positive results
to people who cannot speak English. The government
(c) The exponential increase in city population is
has better ways to spend taxpayers’ money, almost all
responsible for rise in rage killings
of whom are comfortable with English. Legislators who
(d) Public should be more cautious during social
come to the capital should learn to speak English
interactions
152. Which of the following, if true, would make the
Passage (for Q. 149): Voting displays good
speaker’s argument stronger?
citizenship. Good citizenship is required for any society
(a) There is currently a law that says the
to thrive. Mr. Balan votes in every election, so he must
government must provide legislators with
be civic minded.
documents in their language of choice
149. Which of the following best states the assumption (b) Legislators who do not speak English are
made in the passage? more close to the common man

3 . 45
Logical Reasoning and Analytical Ability

(c) India has the largest English speaking calls the employees are found to be more
population in the world motivated to carry out their duties than those
(d) Individual states have different official working in other organisations
languages (d) Many organisations who provide cell phones
to their employees for making official calls
153. Which of the following, if true, would make the
advise them to refrain from making personal
speaker’s argument weaker?
calls during office hours
(a) The government currently translates official
documents into more than twenty languages Passage (for Q. 156): A researcher studying drug
(b) English is the most difficult language in the addicts found that, on an average, they tend to mani-
world to learn pulate other people a great deal more than non-addicts
(c) Most legislators who do not know English do. The researcher concluded that people who frequently
learn English within two months of their manipulate other people are likely to become addicts.
election to the Parliament.
156. Which of the following, if true, most seriously
(d) Making English the official language is a
weakens the researcher’s conclusion?
politically unpopular idea
(a) People who are likely to become addicts exhibit
Passage (for Q. 154): Many patients suffering from unusual behaviour patterns other than frequent
malaria were administered anti-malarial drug for a manipulation of other people
week. Some of them did not respond to the traditional (b) Some non-addicts manipulate other people
drug and their condition deteriorated after four days. more than some addicts do
(c)) After becoming addicts to drugs, drug addicts
154. Which of the following would weaken the findings
learn to manipulate other people as a way of
mentioned in the above statement?
@UPSC_THOUGHTS

obtaining drugs
(a) Those patients who responded to the
(d) The addicts that the researcher studied were
traditional drugs and recovered were needed
often unsuccessful in obtaining what they
to be given additional doses as they reported
wanted when they manipulated other people
relapse of symptoms
(b) The mosquitoes carrying malaria are found to Passage (for Q. 157): Within 20 years it will
be resistant to traditional malarial drugs probably be possible to identify the genetic susceptibility
(c) Majority of the patients suffering from malaria an individual may have toward any particular disease.
responded well to the traditional malarial Eventually, effective strategies will be discovered to
drugs and recovered from the illness counteract each such susceptibility. Once these effective
(d) Many drug companies have stopped strategies are found, the people who follow them will
manufacturing traditional malarial drugs never get sick.
Passage (for Q. 155): Many people are of the 157. The argument above is based on which of the
opinion that the use of cell phones in offices for following assumptions?
personal use should totally be banned. It has been (a) People will follow medical advice when they
found in a research study that there was significant are convinced that it is effective
drop in output of employees in the organisation where (b) All human sicknesses are in part the result
the use of cell phones was liberally allowed vis-a-vis of an individual’s genetic susceptibilities
those organisations where the use of cell phones was (c) The DNA, the basic genetic material, of all
banned for making personal calls. human beings is the same
(d) In future, genetics will be the only medical
155. Which of the following contradicts the findings
specialty of any importance
stated in the above statement?
(a) People spend more time on talking while Passage (for Q. 158): The number of people
using cell phone for personal calls diagnosed as having a certain intestinal disease has
(b) The use of cell phones has become common dropped significantly in small towns over the last 5
in all the organisations years in a western state of India, as compared to earlier
(c) In the organisation where employees were years. Health officials attribute this decrease entirely to
allowed to use cell phones for making personal improved sanitary conditions at water treatment plants,

3 . 46
Logical Reasoning and Analytical Ability

which made available cleaner water during the last 5 160. If all of the statements above are true, then which
years and thus reduced the incidence of the disease. of the following must also be true?
(a) There are more specialised producers in
158. Which of the following, if true, would most
planned economies than market economies
seriously weaken the health officials’ explanation
(b) A manager’s task is easier in a market
for the lower incidence of the disease?
economy than in a planned economy
(a) Because of medical advances over the 5 years,
(c) Division of labour functions more effectively
far fewer people who contract the intenstinal
in market economies than in planned
disease will develop severe cases of the disease
economies
(b) Bottled water has not been consumed in
(d) The need for coordination in market economies
significantly different quantities by people
is greater than in planned economies
diagnosed as having the intestinal disease as
compared to people who did not contract the Passage (for Q. 161): Appearing before the
disease government-appointed committee going into allegations
(c) Many new and advanced water treatment that cigarette manufacturers in country X had increased
plants have been built in the last 5 years in nicotine levels in cigarettes in order to get consumers
the small towns previously known susceptible addicted to their products, representatives of the tobacco
to water borne disease industry of country X argued that cigarette smoking is
(d) Because of new diagnostic technique many not addictive. The main reason they gave in support
people who until recently would have been of their claim was that cigarette smoking was not
diagnosed as having the intestinal disease are regulated by the National Drug Act.
now correctly diagnosed as suffering from
intestinal ulcer of a bacterial infection not 161. For the argument of the representatives of the
@UPSC_THOUGHTS

known before tobacco industry to be logically correct, which of


the following must be assumed?
Passage (for Q. 159): The overall operating costs (a) Some substances that are not addictive are not
borne by many small farmers are reduced when the regulated by the National Drug Act
farmers eliminate expensive commercial chemical (b) Law does not regulate some addictive
fertilizers and pesticides in favour of crop rotation and substances
twice-yearly use manure as fertilizer. Therefore, large (c) No scientific proof is available on cigarette
farmers should adopt the same measures. They will smoking being addictive
then realise even greater total savings than do the small (d) Those substances that are not regulated by the
farmers. National Drug Act are not addictive
159. The argument above assumes that Passage (for Q. 162): There is clear evidence that
(a) large farmers generally look to small farmers the mandated use of seat belts has resulted in fewer
for innovative ways of increasing crop yields fatalities over the past five years. Compared to the five-
or reducing operating costs year period prior to the passage of laws requiring the
(b) the smaller the farm, the more control the use of seat belts, fatalities have decreased by 25 per cent.
farmer has over operating costs
(c) a sufficient amount of manure will be available 162. Which one of the following, if true, most
for fields of large farmers substantially strengthens the argument above?
(d) None of these (a) Car accidents have decreased sharply over
the past five years
Passage (for Q. 160): The greater the division of (b) The number of serious car accidents has
labour in an economy, the greater the need for remained steady over the past five years
coordination. This is because increased division of (c) The use of air bags in cars has increased by
labour entails a larger number of specialised producers, 25 per cent over the past five years
which results in a greater burden on managers and, (d) The number of teenage drivers has increased
potentially in a greater number of disruptions of supply by 25 per cent over the past five years
and production.
There is always more division of labour in market Passage (for Q. 163): Three powerful anti-oxidants—
economies than in planned economies. lycopene, glutathione, and glutamine—neutralise the

3 . 47
Logical Reasoning and Analytical Ability

free radicals that are produced in the body as a result pressure as it increases the level of potassium in the
of routine bodily processes. An excess of these free blood stream.
radicals in your system causes rapid aging because
166. Which of the following contradicts the findings
they increase the rate of cellular damage. Aging is
reported in the above statement?
simply the result of this damage. It is thus necessary
(a) Increased level of potassium in blood stream
to supplement your diet with these anti-oxidants on a
enhances quality of health
daily basis in order to slow down aging.
(b) Many people who consume ripe mangoes
163. Which of the following, if true, most seriously regularly were found to be suffering from
undermines the author’s contention? hypertension
(a) Exercise associated with normal daily activities (c) Consumption of ripe mango helps in storing
can effectively neutralise and dissipate the anti-oxidants in the body.
free radicals that are produced as a result of (d) Ripe mango is a good source of many vitamins
day-to-day bodily processes
Passage (for Q. 167): The government has decided
(b) The cost of antioxidants is too high for the
to instruct the banks to open new branches in such a
budget of most consumers
way that there is one branch of any of the banks in
(c) Only obese people who do not exercise on a
every village of population 1,000 and above or a cluster
daily basis are likely to have an excess of
of villages with population less than 1,000 to provide
these free radicals in their systems
banking services to all the citizens.
(d) Smoking cigarettes is one of the main causes
of cellular damage in humans and should be 167. Which of the following will weaken the step taken
avoided by the government?
(a) The private sector banks in India have stepped
@UPSC_THOUGHTS

Passage (for Q. 164): Chitra has never paid a


up their branch expansion activities in rural
penalty for breaking traffic rules in all the 15 years that
India
she has driven her car. She must be a good driver.
(b) Many government-owned banks have surplus
164. Which of the following can be said about the manpower in its urban branches
above reasoning? (c) All the banks, including those in private
(a) It cannot be substantiated sector, will follow the government directive
(b) The argument is build upon hidden (d) Large number of branches of many
assumptions government-owned banks in the rural areas
(c) It is an argument by analogy are making huge losses every year due to lack
(d) It uses circular reasoning of adequate business activities
Passage (for Q. 165): Cars are safer than trains. Passage (for Q. 168): Exit polls, conducted by an
Fifty per cent of train accidents result in death, while independent organisation, AMS, among voters at five
only one per cent of car accidents result in death. polling locations during a recent election, suggested
that the incumbent MLA—of Party X—was going to
165. Which of the following, if true, would most
lose the election by a huge margin. But by the time the
seriously weaken the argument above?
final results were tabulated, the incumbent had won by
(a) The number of car accidents is several hundred
a narrow margin.
thousand times higher than the number of
train accidents 168. Which of the following, if true, would explain the
(b) Engine drivers never work under the influence apparent contradiction?
of alcohol, while car drivers often do (a) The voters chosen by AMS at random
(c) Train accidents are usually the fault of those happened to belong to Party X
manning the signals, not that engine drivers (b) The incumbent MLA promised to bring better
(d) Trains carry more passengers than cars do drinking water facilities if elected
(c) The exit poll locations were in areas with
Passage (for Q. 166): Based on the findings of a majority of voters opposed to Party X
recent study, it has been reported that regular (d) The exit polls were conducted in the afternoon
consumption of ripe mango helps in controlling blood when most voters had already voted

3 . 48
Logical Reasoning and Analytical Ability

Passage (for Q. 169): A newly discovered disease to performance, especially growth in terms of sales and
is thought to be caused by a certain bacterium. However, profits. Of course, big organisations are more complex
recently released data note that the bacterium thrives than the small, but all CEOs require significant amount
in the presence of a certain virus, implying that it is of energy and time in managing organisations. There
actually the virus that causes the new disease. is no proof that CEOs of big organisations are more
stressed than CEOs of small organisations.
169. Which of the following pieces of evidence would
most support the implication of the data? 171. Which of the following can be used to refute the
(a) The virus has been shown to aid the growth above argument?
of the bacteria (a) All CEOs should be paid equally
(b) The disease has been observed to follow (b) If small companies are managed well, they
infection by the bacterium in the absence of would soon become big, when the CEOs
the virus would also get higher salaries
(c) In cases where the disease does not develop, (c) Managing a large organisation is much more
infection by the bacterium is usually followed challenging than managing a small one
by infection by the virus (d) CEOs of larger organisations are usually better
(d) The virus alone has been observed in many qualified than those of smaller organisations
cases of the disease
Passage (for Q. 172): United Nations members
Passage (for Q. 170): Historically, periods of rising contribute funds, proportionate to their population, for
wages have generally followed famines, because when facilitating the smooth functioning of the UN. By 20XX,
a labour force is reduced, workers are more valuable India, being the most populous nation on the planet,
in accordance with the law of supply and demand. The would contribute the maximum amount to the UN.
@UPSC_THOUGHTS

Irish potato famine of the 1840s is an exception; it Therefore, the official language of United Nations should
resulted in the death or emigration of half of Ireland’s be changed to Hindi.
population, without a significant rise in the average
172. Which of the following is true?
wages in Ireland in the following decade.
(a) The conclusion contradicts the speaker’s
170. Which one of the following, if true, would least argument
contribute to an explanation of the exception to (b) The conclusion goes beyond the speaker’s
the generalisation? argument
(a) Eviction policies of the landowners in Ireland (c) The conclusion sums up the speaker’s
were designed to force emigration of the elderly argument
and infirm, who could not work, and to retain (d) The conclusion strengthens the speaker’s
a high percentage of able-bodied workers argument
(b) Advances in technology increased the
Passage (for Q. 173 and 174): I. The Chintapuri-
efficiency of industry and agriculture, and so
Khushganj corner needs a speed-breaker. The children
allowed maintenance of economic output with
of Gulmohan school cross this intersection on their way
less demand for labour.
to the school, and many a times do not check out for
(c) The birth rate increased during the decade
traffic. I get to read regular reports of cars and other
following the famine, and this compensated
vehicles hitting children. I know that speed-breakers are
for much of the loss of population that was
irritating for drivers, and I know that children cannot
due to the famine.
be protected from every danger, but this is one of the
(d) England, which had political control of
worst intersections in town. There needs to be a speed-
Ireland, legislated artificially low wages to
breaker so that vehicles have to slow down and the
provide English-owned industry and
children be made safer.
agriculture in Ireland with cheap labour
II. According to a recent research conducted by the
Passage (for Q. 171): It was recently reported that district road planning department, ten per cent students
CEOs of large organisations are paid more than CEOs come with parents in cars, twenty percent students use
of small organisations. It does not seem fair that just auto-rickshaws, twenty per cent students use taxis,
because a CEO is heading a big organisation he or she forty per cent students use the school buses and ten per
should be paid more. A CEOs’ salary should be related cent students live in the hostel inside the school.

3 . 49
Logical Reasoning and Analytical Ability

173. Which of the following arguments is used in Passage (for Q. 176): As man casts off worn-out
passage I? garments and puts on others which are new, similarly
(a) Analogy—comparing the intersection to the embodied soul, casting off worn-out bodies, enters
something dangerous into others which are new.
(b) Emotive—referring to the safety of children to
176. Which one of the following best describes the
get people interested
argument?
(c) Personalisation—telling the story of one
(a) Inductive conclusion
child’s near accident at the intersection
(b) Cause-and-effect
(d) Attack—pointing out people who are against
(c) Argument by analogy
speed-breakers as being uncaring about
(d) None of the above
children
Passage (for Q. 177): In this era of global capital
174. Taking passage II in the context of passage I,
flows, so much money is now flowing throughout the
which of the following is true of passage II?
world that no single country can fight the problem of
(a) It uses analogy to extend the speaker’s
inflation effectively by tightening its monetary policy.
argument in I
(b) It is similar to the argument of the speaker 177. If the above is true, which of the following could
in I be most logically concluded?
(c) It uses statistics to prove the argument of the (a) Changes in cash reserve ratio by Reserve
speaker in I Bank of India will control the rate of inflation
(d) It uses statistical data to counter the speaker’s in India
argument in I (b) Finance ministers have sufficient control over
their respective economies
@UPSC_THOUGHTS

Passage (for Q. 175): The Yoga system is divided


(c) Finance ministers have insufficient control
into two principal parts—Hatha and Raja Yoga. Hatha
over their respective economies
Yoga deals principally with the physiological part of
(d) Inflation does not matter as long as incomes
man with a view to establishing his health and training
increase
his will. The processes prescribed to arrive at this end
are so difficult that only a few resolute souls go through Passage (for Q. 178): “There is nothing so stupid
all the stages of its practice. Many have failed and some as an educated man, if you can get him off the thing
have died in the attempt. It is therefore strongly he was educated in.”
denounced by all the philosophers. The most illustrious
178. Which one of the following, if true, weakens the
Shankaracharya has remarked in his treatise called
author’s contention?
Aparokshanubhuti that “the system of Hatha Yoga was
(a) It is the cost of general education that confines
intended for those whose worldly desires are not
a person to one field of education
pacified or uprooted.”
(b) Education in the true sense implies exposure
175. Which of the following, if true, substantially to wide range of important subjects
strengthens the idea presented in the passage? (c) Stupidity is relative, like intelligence
(a) The percentage of people in a given ashram (d) Education causes people to develop their
practising Raja Yoga is more than the faculties
percentage of people practising Hatha Yoga
Passage (Q. 179): Ram, an economist, and Ramesh,
(b) The number of people in a given ashram
an astrologer, had a debate. Ram said “Astrology does
practising Raja Yoga is more that the number
of people practising Hatha Yoga not work. It just cannot predict.”
(c) Yoga schools teaching Raja Yoga are more in “It can predict better than your subject” rebutted
Ramesh.
number than Yoga schools teaching Hatha
Yoga 179. Which of the following would best resolve the
(d) The percentage of students who have debate?
successfully learnt Raja Yoga is more than the (a) Compare past performance of astrologers and
percentage of students who have successfully economists in terms of number of predictions
learnt Hatha Yoga which have come true

3 . 50
Logical Reasoning and Analytical Ability

(b) Conduct a survey among economists asking 183. Who is sitting between A and D?
their opinion regarding the ability of economic (a) B
theory to predict economic phenomena (b) B and E
(c) Conduct an experiment where both astrologers (c) G and B
and economists would be asked to predict the (d) None of these
future and then compare the number of
predictions that come true 184. In which of the following pairs is the first person
(d) Conduct an experiment where both astrologers sitting to the immediate right of the second person?
and economists would be asked to predict the (a) BE
future and then compare the percentage of (b) DG
predictions that come true (c) CF
(d) In none of these cases
Passage (for Q. 180): In an experiment conducted
at a laboratory, 160 white mice were injected with 185. How many are sitting between H and E?
Serum E. Another 160 other white mice were injected (a) Two
with a harmless sugar solution. In two weeks’ time 39 (b) Three
per cent of the white mice, who were injected with (c) Four
Serum E, contracted the highly contagious and often (d) Depends on whether it is to the right or left
fatal disease, jungle fever. Hence, it can be concluded
of H
that jungle fever is caused by some elements similar to
the elements in Serum E. Information for Qs. 186 to 190: P, Q, R, S, T, V, W
180. The above discussion would be seriously weakened and Z are sitting around a circle facing the centre. S
in case it is shown that is second to the right of V who is third to the right of
@UPSC_THOUGHTS

(a) people contracting jungle fever are usually the T. Q is second to the left of T and fourth to the right
victims of the bite of the South American of Z. W is third to the right of P who is not an immediate
Lesser Hooded Viper neighbour of T.
(b) one among the 160 white mice had already
186. Who is to the immediate left of W?
contracted jungle fever prior to the laboratory
(a) R
experiment
(c) the natural habitats of white mice does not (b) T
contain any of the elements found in Serum (c) S
E (d) None of these
(d) the scientists administered the injections being 187. Who is opposite R?
ignorant of the contents of the solutions used (a) P
Information for Qs. 181 to 185: A, B, C, D, E, F, (b) Q
G and H are sitting around a circular table facing the (c) S
centre. D is second to the left of H, who is third to the (d) None of these
left of A. B is fourth to the right of C, who is the
188. Who is fourth to the left of P?
immediate neighbour of H. G is not a neighbour of B
or C. F is not a neighbour of B. (a) T
(b) R
181. Who is third to the left of B? (c) W
(a) A (d) None of these
(b) H
(c) F 189. In which of the following pairs is the first person
(d) None of these sitting immediately to the right of the second
person?
182. What is F’s position with respect to G?
(a) QW
(a) Second towards left
(b) Second towards right (b) TW
(c) Third towards left (c) ZV
(d) Third towards right (d) SQ

3 . 51
Logical Reasoning and Analytical Ability

190. Three of the following four are alike in a certain 196. Who is to the immediate left of E?
way based on their positions. Which group does (a) H
not show the same order? (b) F
(a) ZRV (c) G
(b) WQT (d) I
(c) PSQ 197. Which of the following gives the names of persons
(d) RTZ sitting in alternate positions?
I. I E D
Information for Qs. 191 to 195: A, B, C, D, E, F,
II. A B H
G and H are sitting around a circle facing the centre.
III. D A I
D is fourth to the right of H and second to the left of (a) I and II
B. F is fourth to the right of B. C is fourth to the right (b) II and III
of E who is not an immediate neighbour of B or D. A (c) Only I
is not an immediate neighbour of D. (d) I, II and III
191. Who is third to the right of A? 198. With respect to E, C is
(a) C I. Fifth to the right
(b) D II. Fifth to the left
(c) F III. Fourth to the right
(d) G IV. Fourth to the left
(a) I only
192. Who is to the immediate left of D? (b) I and II
(a) C (c) III and IV
(b) F (d) II and III
@UPSC_THOUGHTS

(c) H
(d) G Information for Qs. 199 to 201: A, B, C, D, E, F,
G and H are sitting around a circular table, facing the
193. Who is fourth to the left of G? centre. A sits third to the left of C and second to the
(a) E right of E. B sits second to the right of D, who is not
(b) F an immediate neighbour of E. H sits second to the left
(c) A of F. G is not an immediate neighbour of D.
(d) H
199. Which of the following pairs has only one person
194. In which of the following combinations is the sitting between them, if the counting is done in
third person sitting in between the first and the clockwise direction?
second persons? (a) H, D
(a) ABC (b) F, G
(b) GCD (c) H, G
(d) None of these
(c) AHE
(d) CBA 200. What is the position of G with respect to A’s
position?
195. What is B’s position with respect to G?
(a) Immediately to the right
(a) Third to the right (b) Second to the left
(b) Third to the left (c) Third to the right
(c) Fifth to the right (d) Third to the left
(d) Fourth to the left
201. Starting from A’s position, if all the eight are
Information for Qs. 196 to 198: A, B, C, D, E, F, arranged in alphabetical order in clockwise
G, H and I are sitting around a circle facing at the direction, the seating position of how many
centre. D is third to the left of H who is second to the members (excluding A) would remain unchanged?
left of B. A is fourth to the left of E who is second to (a) Two
the right of D. C is third to the right of H. I is not an (b) Four
immediate neighbour of D. G is not an immediate (c) Only one
neighbour of E. (d) All would change

3 . 52
Logical Reasoning and Analytical Ability

Information for Qs 202 to 205: A, B, C, D, E, F, G 207. What is the position of L with respect to S?
and H are sitting around a circular table facing the (a) Third to the left
centre. (b) Third to the right
1. B sits third to the right of F. (c) Immediately to the right
2. A sits second to the right of D who is not an (d) Fourth to the left
immediate neighbour of B and F. 208. Four of the following five are alike in a certain
3. C and E are immediate neighbours of each way based on their seating positions in the above
other. arrangement and so form a group. Which is the
4. H is not an immediate neighbour of A. one that does not belong to that group?
5. No one sits between C and F. (a) PR
202. Four of the following five are similar in a certain (b) ST
(c) LQ
way based on their positions in the seating
(d) WT
arrangement and so form a group. Which of the
following does not belong to that group? 209. How many persons sit between S and W?
(a) BC (a) 2
(b) BD (b) 3
(c) DA (c) 4
(d) HG (d) None

203. What is the position of D with respect to E in the 210. If all the persons are made to sit in alphabetical
above arrangement? order from left to right, the positions of how many
will remain unchanged as compared to the original
(a) Third to the right
seating positions?
@UPSC_THOUGHTS

(b) Fourth to the left


(a) One
(c) Second to the right (b) Two
(d) Fourth to the right (c) Three
204. In which of the following pairs is the second (d) None
person sitting on the immediate right of the first Information for Qs. 211 to 215: Eight friends Q, R,
person? S, T, V, W, Y and Z are sitting around a circular table,
(a) HB facing the centre. There are three males and five females
(b) GD in the group of friends. No two males are immediate
(c) FA neighbours of each other.
(d) None of these 1. V sits second to the right of his wife.
205. Who amongst the following sits between G 2. S sits third to the right of V.
3. W sits second to the right of her husband Z.
and F?
Z is not an immediate neighbour of V’s wife.
(a) A
4. T is a male and Y is not an immediate
(b) B
neighbour of V.
(c) H
5. R sits second to the right of Q.
(d) E
211. Which of the following statements regarding S is
Information for Qs 206 to 210: L, P, Q, R, S, T and definitely correct?
W are sitting in a straight line facing north. P sits fourth (a) S is one of the male members of the group
to the left of L and L sits second to the left of Q. R, (b) Both the immediate neighbours of S are females
who is not an immediate neighbour of L, sits to the (c) S sits third to the left of T
immediate right of S. T sits second to the left of W. (d) W is an immediate neighbour of S
206. Which of the following pairs sit at the extreme 212. Who amongst the following has a male sitting
corners of the line? to the immediate left and right?
(a) S, T (a) Y
(b) W, Q (b) R
(c) P, Q (c) Q
(d) S, L (d) S

3 . 53
Logical Reasoning and Analytical Ability

213. Which of the following is/are true regarding T? (a) Only I and II
I. T is an immediate neighbour of Z’s wife. (b) Only II, III and IV
II. No male is an immediate neighbour of T. (c) Only I, II and III
III. Q sits second to right of T. (d) None is correct
IV. The one who sits third to the left of T is a
218. In which of the following pairs is the representative
male.
from the first country sitting on the immediate left
(a) Only I
of the representative from the second country?
(b) I and II
(a) India-Thailand
(c) I, II and III
(b) Austria-US
(d) I, II, III and IV
(c) Thailand-Holland
214. Which of the following pairs represents the (d) Spain-Germany
immediate neighbours of T?
(a) RQ 219. The representative from which of the following
(b) WZ countries sits exactly in the middle of the
(c) YV representatives from Thailand and France?
(d) WY (a) Holland
(b) Austria
215. How many people sit between V and S when (c) Germany
counted in anti-clockwise direction? (d) None of these
(a) One
(b) Two 220. Which of the following pairs represents the
(c) Three immediate neighbours of the representative from
(d) Four Holland?
@UPSC_THOUGHTS

(a) CG
Information for Qs 216 to 220: In an international (b) BE
conference, representatives A, B, C, D, E, F, G and H (c) AH
from eight different countries, viz., Thailand, France, (d) HB
Holland, Austria, US, Spain, India and Germany (not
necessarily in the same order), sit around a circular Information for Qs. 221 to 223: P, Q, R, S, T, W
table facing the centre. A, who represents Germany, sits and Z are seven students studying in three different
third to the left of E. The one who is from India sits institutes—A, B and C. There are three girls among the
on the immediate right of A. D, who is from Holland, seven students who study in each of the three institutes.
sits second to the right of B. B is not an immediate Two of the seven students study BCA, two study
neighbour of E. C, who is from Spain, sits exactly in Medicine and one each studies Aviation Technology,
the middle of people representing US and India. G, the Journalism and MBA. R studies in the same college as
representative from France, sits second to the left of H, P who studies MBA in college B. No girl studies
who is from Thailand. Journalism or MBA. T studies BCA in college A and
his brother W studies Aviation Technology in college
216. How many people sit between A and the
C. S studies Journalism in the same college as Q.
representative from Austria when counted in
Neither R nor Z studies BCA. The girl who studies BCA
clockwise direction?
does not study in college C.
(a) One
(b) Two 221. Which of the following pairs of students study
(c) Three medicine?
(d) Four (a) QZ
217. Which of the following is correct regarding (b) WZ
representative F? (c) PZ
I. F is the representative of Austria. (d) None of these
II. F sits second to the left of the representative 222. In which of the colleges do three of them study?
from Thailand. (a) A
III. The representative from Germany is not an (b) B
immediate neighbour of F. (c) A and B
IV. E sits third to the right of F. (d) C

3 . 54
Logical Reasoning and Analytical Ability

223. Which of the following three represents girls? 226. Which of the following groups represents the
(a) SQR students of college Y?
(b) QRZ (a) C, E, G
(c) SQZ (b) A, C, D
(d) None of these (c) A, B, C
(d) D, B, C
Information for Qs. 224 and 225: Seven persons H,
227. Who is in medicine?
I, J, K, L, M and N are working in different cities
(a) G
Ahmedabad, Bengaluru, Chennai, Hyderabad, Kolkata,
(b) E
Delhi and Mumbai, not necessarily in the same order.
(c) D
Each one has a different mother tongue—Tamil, (d) A
Kannada, Telugu, Hindi, Marathi, Punjabi and Bangla,
not necessarily in the same order. 228. Which of the following combinations of person,
J works in Bengaluru and his mother tongue is not college and profession is definitely correct?
Tamil or Marathi. K’s mother tongue is Punjabi and he (a) E-X-Fashion Designing
works in Ahmedabad. L and M do not work in Chennai (b) F-X-Engineering
(c) A-Y-Businessman
and none of them has Marathi mother tongue. I works
(d) D-Z-Teaching
in Hyderabad and his mother tongue is Telugu. The one
who works in Delhi has Bangla mother tongue. N Information for Qs. 229 and 230: Seven friends H,
works in Mumbai and his mother tongue is Hindi. L I, J, K, V, W and X study different disciplines, viz., Arts,
does not work in Kolkata. Commerce, Science, Engineering, Architecture,
Management and Pharmacy, not necessarily in the
224. What is J’s mother tongue? same order. Each of them belongs to a different state,
@UPSC_THOUGHTS

(a) Telugu viz., Andhra Pradesh, Uttar Pradesh, Maharashtra,


(b) Hindi Karnataka, Kerala, Madhya Pradesh and Punjab, but
(c) Bangla not necessarily in the same order.
(d) Kannada
J studies Engineering and does not belong to either
225. Which of the following is/are correct? Uttar Pradesh or Punjab. The one who belongs to
I. Marathi—N—Bengaluru Madhya Pradesh does not study Architecture or
II. Tamil—M—Kolkata Pharmacy. H belongs to Maharashtra. V belongs to
III. Bangla—L—Delhi Kerala and studies Science. The one who belongs to
(a) Only I and II Andhra Pradesh studies Commerce. K studies
(b) Only II and III Management and X studies Arts. I belongs to Karnataka
(c) All I, II and III and does not study Architecture. The one who studies
(d) None of them Arts does not belong to Punjab.
229. Which of the following person and subject
Information for Qs. 226 to 228: Seven friends A, combination is correct?
B, C, D, E, F and G studied in colleges X, Y and Z and (a) J—Architecture
are currently in different professions, namely Medicine, (b) W—Commerce
Fashion Designing, Engineering, Business, Acting, (c) I—Management
Teaching and Architecture (not necessarily in the same (d) V—Arts
order). At least two and not more than three friends had 230.Which of the following state-subject combination
studied in the same college. is/are correct?
C is an architect and studied in college Y. E is not I. Madhya Pradesh : Pharmacy
a businessman. Only G amongst the seven friends II. Uttar Pradesh : Arts
studied in college X along with E. F is an engineer and III. Punjab : Management
did not study in college Y. B is an actor and did not IV. Kerala : Commerce
study in the same college as F. A did not study in (a) Only I and II
college Z. Those who studied in college X are neither (b) Only II and III
fashion designers nor teachers. None of those who (c) Only III and IV
studied in college Y is a teacher. (d) Only I and IV

3 . 55
Logical Reasoning and Analytical Ability

Information for Qs. 231 and 232: A, B, C, D and and are posted at different places viz., Chennai,
E are sons of P, Q, R, S and T but not in the same order. Kozhikode, Kolkata, Ranchi, Patna, Bhopal, Nagpur
Match the right mother and son on the basis of the and Hyderabad not necessarily in the same order. At
information given below— least two and not more than three executives work in
Q is not B’s or C’s mother. A’s or E’s mother is not any of the three departments.
T. C is not R’s son. E is not Q’s or S’s son and his G works in Engineering department at Chennai. H
mother’s name does not start with the letter ‘R’. A’s is posted in Ranchi but not in Systems department. No
mother is not Q or R. one from Marketing department is posted in Hyderabad.
The only other person in same department as that of
231. Who is the son of S?
G is posted in Kolkata. D is posted in Hyderabad and
(a) D
F in Kozhikode. V is not posted in Kolkata and works
(b) C
in the same department as that of D. B and T both work
(c) A
in Marketing department. The one who works in
(d) B
Marketing is not posted in Bhopal. T is not posted in
232. Which of the following does not give the correct Nagpur.
son-mother combination?
235. T is placed at
(a) B—R
(a) Kolkata
(b) C—T
(b) Patna
(c) E—P
(c) Nagpur
(d) D—S
(d) None of these
Information for Qs. 233 and 234: P, Q, R, S, T, V
236. Who is posted at Kolkata?
and W are seven students of a school. Each of them
@UPSC_THOUGHTS

(a) K
studies in a different standard—from Standard IV to
(b) K or T
Standard X—not necessarily in the same order. Each
(c) T
of them has a favourite subject from English, Science,
(d) V
History, Geography, Mathematics, Hindi and Sanskrit,
not necessarily in the same order. 237. Which department has only two executives?
Q studies in VII Standard and does not like either (a) Systems
Mathematics or Geography. R likes English and does (b) Marketing
not study either in V or in IX. T studies in VIII Standard (c) Engineering
and likes Hindi. The one who likes Science studies in (d) Marketing or Systems
X Standard. S studies in IV Standard. W likes Sanskrit.
238. Which of the following group of persons work in
P does not study in X Standard. The one who likes marketing department?
Geography studies in V Standard. (a) KBT
233. Which one of the following is a wrong (b) BTF
combination of student and standard? (c) BHD
(a) P – V (d) BHT
(b) Q – VII
239. Which of the following combinations of
(c) R – VIII
department, person and place is correct?
(d) S – IV
(a) Marketing - B - Bhopal
234. Which of the following combinations of student- (b) Engineering - G - Kolkata
standard-subject is correct? (c) Systems - T - Patna
(a) T - VIII - Mathematics (d) None of these
(b) W - VII - Sanskrit
Information for Qs. 240 to 243: There is a group
(c) Q - VII - Geography
of six persons A, B, C, D, E and F in a family. They
(d) V - X - Science
are psychologist, manager, lawyer, jeweller, doctor and
Information for Qs 235 to 239: Eight executives B, engineer. The doctor is the grandfather of F, who is a
G, H, K, D, F, T and V are working in three departments psychologist. The manager D is married to A. C, the
Engineering, Systems and Marketing of the organisation jeweller, is married to the lawyer. B is the mother of F

3 . 56
Logical Reasoning and Analytical Ability

and E. There are two married couples in the family. The 247. What is/was Ajay’s grandfather’s occupation?
psychologist is a female while the engineer is a male. (a) Teacher
(b) Lawyer
240. Which of the following is correctly matched?
(c) Doctor
(a) A – female – doctor
(d) Cannot be determined
(b) D – male – manager
(c) B – female – lawyer Information for Qs. 248 to 251: If ranks of five
(d) C – female – jeweller candidates P, Q, R, S and T are arranged in ascending
241. Which of the following are the two married order of their marks in Numerical Ability, T is the fourth
couples in the family? and S is the first. When they are arranged in the
(a) A-D and E-F ascending order of marks in General Awareness, P
(b) A-D and B-C takes the place of T and T takes the place of Q. R’s
(c) A-D and B-E position remains the same in both the arrangements.
(d) B-C and A-E Q’s marks are lowest in one test and highest in the other
test. P has more marks than R in Numerical Ability.
242. How many males are there in the family?
(a) Two 248. Who has secured the highest marks in General
(b) Three Awareness?
(c) Four (a) Q
(d) Cannot be determined (b) T
(c) S
243. D is E’s (d) R
(a) grandfather
249. Who has secured the highest marks in Numerical
@UPSC_THOUGHTS

(b) mother
(c) father Ability?
(d) grandmother (a) S
(b) R
Information for Qs. 244 to 247: (c) P
(i) In a family of 6 persons, there are two couples. (d) Q
(ii) The lawyer is the head of the family and has
only two sons—Mukesh and Rakesh—both 250. Which of the following groups of candidates has
teachers. improved in rank in General Awareness as
(iii) Mrs. Reena and her mother-in-law are both compared to that in Numerical Ability?
lawyers. (a) SPT
(iv) Mukesh’s wife is a doctor and they have a (b) PSR
son, Ajay. (c) QST
(d) SPQ
244.Which of the following is definitely a couple?
(a) Lawyer-Teacher 251. Whose marks in General Awareness are more
(b) Doctor-Lawyer than R’s marks in General Awareness?
(c)) Teacher-Teacher (a) Only P’s
(d) None of these (b) P’s, Q’s and S’s
(c) P’s, S’s and T’s
245. What is the profession of Rakesh’s wife?
(d) P’s, Q’s and T’s
(a) Teacher
(b) Doctor Information for Qs. 252 to 255:
(c) Lawyer (i) A, B, C, D, E, F, G and H are eight students,
(d) Cannot be determined each having a different height.
246. How many male members are there in the family? (ii) D is shorter than A but taller than G.
(a) Two (iii) E is taller than H but shorter than C.
(b) Three (iv) B is shorter than D but taller than F.
(c) Four (v) C is shorter than G.
(d) Cannot be determined (vi) G is not as tall as F.

3 . 57
Logical Reasoning and Analytical Ability

252. Which of the following is/are true? 256. The ideal candidate for the office farthest from
I. G is shorter than B Mr. Balu would be
II. D is taller than E (a) Ms. Heera
III. B is taller than H (b) Mr. Mohan
(a) Only I and II (c) Mr. Tarun
(b) Only II and III (d) Mr. Dina
(c) I, II and III
(d) None of them 257. The three employees who are smokers should get
offices
253. If another student J, who is taller than E but (a) 1, 2 and 3
shorter than G, is added to the group, which of (b) 2, 3 and 4
the following will be definitely true? (c) 2, 3 and 6
(a) C and J are of the same height
(d) 1, 2 and 4
(b) J is shorter than D
(c) J is shorter than H 258. Mr. Mohan would get office
(d) J is taller than C (a) 2
(b) 3
254. Which of the following will definitely be the third
(c) 4
from top when the eight students are arranged in
(d) 6
descending order of height?
(a) B 259. In the event of what occurrence, within a period
(b) F of one month since the assignment of the offices,
(c) G would a request for a change in office be put forth
@UPSC_THOUGHTS

(d) B or G by one or more employees?


255. How many of the eight are definitely shorter than (a) Mr. Dina quitting smoking
F? (b) The installation of a noisy teletype machine
(a) Three by Ms. Heera in her office
(b) Four (c) Mr. Balu suffering from laryngitis
(c) Five (d) Mr. Tarun taking over the duties formerly
(d) None of these taken care of by Ms. Reena
Information for Qs 256 to 259: An employee has Information for Qs 260 to 264: Nine individuals—
been assigned the task of allotting offices to six of the Z, Y, X, W, V, U, T, S and R—are the only candidates,
staff members. The offices are numbered 1 to 6. The who can serve on three committees—A, B and C, and
offices are arranged in a row and they are separated each candidate should serve on exactly one of the
from each other by six-foot high dividers. Hence voices, committees.
sounds and cigarette smoke flow easily from one office Committee A should consist of exactly one member
to another. more than committee B.
Ms. Reena needs to use the telephone quite often It is possible that there are no members of committee
throughout the day. Mr. Mohan and Mr. Balu need C.
adjacent offices as they need to consult each other often Among Z, Y and X none can serve on committee A.
while working. Ms. Heera, is a senior employee and has
Among W, V and U none can serve on committee B.
to be allotted the office number 5, having the biggest
Among T, S and R none can serve on committee C.
window.
Mr. Dina requires silence in the offices next to his. 260. In case T and Z are the individuals serving on
Mr. Tarun, Mr. Mohan and Mr. Dina are all smokers. committee B, how many of the nine individuals
Ms. Heera finds tobacco smoke allergic and would serve on committee C?
consecutively the offices next to hers to be occupied by (a) 3
non-smokers. (b) 4
Unless specifically stated all the employees maintain (c) 5
an atmosphere of silence during office hours. (d) None

3 . 58
Logical Reasoning and Analytical Ability

261. Of the nine individuals, the largest number that Information for Qs 267 to 271: A flat wilderness
can serve together on committee C is area has four widely separated shelters— M, N, O and
(a) 9 P—that are connected by exactly four straight paths—
(b) 8 D, E, F, and G—that equal to each other in length and
(c) 6 connect the shelters in the following ways:
(d) 5 • D connects M and O only
262. In case R is the only individual serving on • E connects N and O only
committee B, which among the following should • F connects M and N only
serve on committee A? • G connects N and P only
(a) V and U The shelters are at the ends of the paths.
(b) V and T 267. If a person walks the full length of each path
(c) U and S exactly once, which of the following lists all those
(d) T and S shelters and only those shelters at which the
263. In case T, S and X are the only individuals serving person must be exactly twice?
on committee B, the membership of committee C (a) N
should be: (b) N and O
(a) Z and Y (c) N, O, P
(b) Z and W (d) M and N
(c) Y and V
268. If, by taking shortcuts that stray from the paths,
(d) X and V
a person could travel from O to P over a shorter
264. Among the following combinations which would distance than the shortest between O and P by
@UPSC_THOUGHTS

constitute the membership of committee C? path alone, which of the following must be true?
(a) Y and T (a) The route composed of E and G is not a
(b) X and U straight line
(c) Y, X and W (b) The route composed of F and G is not a
(d) Z, X, U and R straight line
Information for Qs 265 and 266: The only speakers (c) E meets G at a right angle
at a conference were four school principals and the (d) The shortest sequence of paths between M
secretaries of three of those principals. The principals and P is the shortest distance between M
were L, M, N and O; the secretaries were A, D and G. and P
Each person in turn delivered a report to the assembly 269. Which of the following is the order in which a
as follows: person, starting at M, using only paths and using
Each of the secretaries delivered their report exactly no path more than once, must reach the other
after his or her principal. The first principal to speak
shelters?
was M, and N spoke after him.
(a) P, N, O
265. Among the following which is not an appropriate (b) O, P, N
order of delivered reports? (c) N, O, P
(a) M, A, N, G, O, L, D (d) O, N, P
(b) M, D, N, G, L, O, A
270. If a person is at P and wants to reach M by a
(c) M, N, A, L, D, O, G
(d) M, N, G, D, O, L, A sequence of paths no longer than necessary, there
are how many path sequence(s) of minimal length
266. In case L speaks after A, and A is the third of the from which to choose?
secretaries to speak, then among the following (a) One
statements which cannot be true? (b) Two
(a) O spoke immediately after G (c) Three
(b) The order of the first four speakers could be (d) Four
M, G, N, D
(c) A was the fourth speaker after M. 271. If a straight-line distance between M and P is the
(d) The principals spoke in the order M, N, O, L same as the straight-line distance between O and

3 . 59
Logical Reasoning and Analytical Ability

P, which of the following can result if new (c) Bannerji - Doctor, Chatterji - Accountant,
straight paths are added between M and P and Mukherji - Dentist, and Pestonji - Lawyer
between O and P? (d) Bannerji - Lawyer, Chatterji - Dentist, Mukherji
(a) The shortest distance by path between M and - Doctor, and Pestonji - Accountant
O is less than the shortest distance between
Information for Qs. 275 to 277: In a game, exactly
O and P
six inverted cups stand side by side in a straight line,
(b) The shortest distance by path between any
and each has exactly one ball hidden under it. The cups
shelter and any other shelter is the same
are numbered consecutively 1 through 6. Each of the
(c) A person must travel fewer paths to travel the
shortest distance between M and P than to balls is painted a single solid colour. The colours of the
travel the shortest distance between M and N balls are green, magenta, orange, purple, red, and
(d) The number of paths required for the shortest yellow. The balls have been hidden under the cups in
possible travel by path between any shelter a manner that conforms to the following conditions:
and any other shelter is one The purple ball must be hidden under a lower-
numbered cup than the orange ball.
Information for Qs. 272 and 273: In a city there are The red ball must be hidden under a cup
three bus routes 1, 2 and 3 between A and F. Route- immediately adjacent to the cup under which the
1 has intermediate stops at B and D. Route-2 has stops magenta ball is hidden.
at C and D. The shortest route-3 with a length of 10 The green ball must be hidden under cup 5.
km, stops at C only, which is exactly at the middle of
this route. The longest route has 3 km more length than 275. Which of the following could be the colours of the
the shortest one. The distances between C and D, B and balls under the cups, in order from 1 through 6?
@UPSC_THOUGHTS

D, and F and D are 4, 3 and 2 kilometres respectively. (a) Green, yellow, magenta, red, purple, orange
(b) Magenta, red, purple, yellow, green, orange
272. What is the distance between A and B?
(C) Orange, yellow, red, magenta, green, purple
(a) 5 km
(d) Red, purple, magenta, yellow, green, orange
(b) 6 km
(c) 7 km 276. A ball of which of the following colours could be
(d) 8 km under cup 6?
(a) Magenta
273. What is the length of route-2?
(b) Purple
(a) 11 km
(c) Red
(b) 12 km
(d) Yellow
(c) 13 km
(d) Insufficient information 277. Which of the following must be true?
(a) The green ball is under a lower-numbered cup
Information for Q. 274: The surnames of four
than the yellow ball
professionals are: Bannerji, Chatterji, Mukherji and
(b) The orange ball is under a lower-numbered
Pestonji. Their professions are accountant, lawyer,
dentist and doctor (not necessarily in this order). The cup than the green ball
accountant and lawyer work in their offices, while the (c) The purple ball is under a lower-numbered
dentist and doctor work in their nursing homes. The cup than the green ball
accountant looks after Mukherji’s and Chatterji’s (d) The purple ball is under a lower-numbered
account. Chatterji does not know Bannerji, although his cup than the red ball
nursing home is on the same street as Bannerji’s office. Information for Q. 278 to 280: A museum curator
Chatterji is not a doctor. must group nine paintings—F, G, H, J, K, L, M, N, and
274. What are the occupations of the four people? O—in twelve spaces numbered consecutively from 1-12.
(a) Bannerji - Doctor, Chatterji - Dentist, Mukherji The paintings must be in three groups, each group
- Accountant, and Pestonji - Lawyer representing a different century. The groups must be
(b) Bannerji - Lawyer, Chatterji - Dentist, Mukherji separated from each other by at least one unused wall
- Accountant, and Pestonji - Doctor space. Three of the paintings are from the eighteenth-

3 . 60
Logical Reasoning and Analytical Ability

century, two from the nineteenth-century, and four from 282. If K and T are the first two display items to be
the twentieth century. selected, how many acceptable groups of items are
Unused wall spaces cannot occur within groups. there that would complete the display?
G and J are paintings from different centuries. (a) 1 (b) 2
J, K, and L are all paintings from the same century. (c) 3 (d) 4
Space number 5 is always empty.
F and M are eighteenth-century paintings. Information for Qs 283 to 285: A pastry chef who
N is a nineteenth-century painting. is visiting a culinary school wishes to schedule three
classes on pastry making—one at 10 a.m., one at 2 p.m.,
278. If space 4 is to remain empty, which of the
and one at 6 p.m. Eight student chefs—Q, R, S, T, W,
following is true?
X, Y, and Z—who have registered to attend class will
(a) Space number 10 must be empty
each be assigned to one of the three classes. Each class
(b) The groups of paintings must be hung in
chronological order by century will contain either two or three student chefs. The
(c) An eighteenth-century painting must be hung assignment of student chefs to each class must conform
in space 3 to the following restrictions:
(d) A nineteenth-century painting must be hung Q must be assigned to a class to which only one
in space 1 other student is assigned.
R must be assigned to the same class as Y.
279. If the paintings are hung in reverse chronological
S must not be assigned to the same class as X.
order by century, the unused wall spaces could
T must be assigned to either the 10 a.m. class or
be
the 6 p.m. class.
(a) 1, 6 and 10
X must be assigned to a class that meets earlier
@UPSC_THOUGHTS

(b) 4, 7 and 8
(c) 5, 8 and 12 in the day than the class to which W is
(d) 5, 9 and 10 assigned.

280. If J hangs in space 11, which of the following is 283. Which of the following is a possible assignment
a possible arrangement for spaces 8 and 9? of students to the classes?
(a) F in 8 and M in 9 10 a.m. 2 p.m. 6 p.m.
(b) K in 8 and G in 9 (a) Q, X R, T, Z S, W, Y
(c) N in 8 and G in 9 (b) Q, W R, X, Y S, T, Z
(d) G unused and H in 9 (c) R, X, Y S, W, Z Q, T
(d) S, T, X R, Y, Z Q, W
Information for Qs. 281 and 282: The owner of a
computer store is planning a window display of five 284. If S and Q are assigned to the 6 p.m. class, which
products. Three are to be hardware items selected from of the following must be the group of students
K, L, M, N, and O, and two are to be software manuals assigned to the 10 a.m. class?
selected from R, S, T, and U. The display items are to (a) R, X, Y
be selected according to the following conditions: (b) R, Y, Z
If K is displayed, U must be displayed. (c) T, W, Z
M cannot be displayed unless both L and R are (d) T, X, Z
also displayed.
If N is displayed, O must be displayed, and if O 285. If Q and Z are assigned to the 10 a.m. class, which
is displayed, N must be displayed. of the following must be the group of students
S is displayed, neither T nor U can be displayed. assigned to the 6 p.m. class?
(a) R, W, Y
281. Which of the following could be a display as per
(b) R, T, Y
conditions?
(c) S, T, X
(a) N, O, R, S, T
(d) S, T, W
(b) K, M, N, O, R
(c) M, N, O, T, V Directions: (Qs. 286 to 295): Each of these questions
(d) None of the above given below consists of a question and two statements

3 . 61
Logical Reasoning and Analytical Ability

numbered I and II given below it. You have to decide whether I. Of the one thousand novels in the library, fifty
the data provided in the statements is sufficient to answer per cent novels are in English and Hindi.
the given question. Read both the statements and II. The number of Hindi novels is double the
Give answer (a) if statement I alone is sufficient to number of English novels.
answer the question, while statement II alone is
292. Mohan is taller than Pradeep, and Ram is taller
not sufficient to answer the question. than Deepak. Who is the tallest?
Give answer (b) if statement II alone is sufficient I. Pradeep is taller than Ram.
to answer the question, while statement I is not II. Mohan is taller than Ram.
sufficient to answer the question.
Give answer (c) if either statement I alone or 293. Among E, F, G, H and I, who is in the middle
statement II alone is sufficient to answer the while standing in a row?
question. I. F, who is 4th from the right end, is immediately
Give answer (d) if both the statements I and II to the left of G.
together are necessary to answer the question. II. H, who is not the neighbour of E or F, is to
the immediate left of I who is at the right end.
286. How far is Akola from Nagpur?
294. Five friends P, Q, R, S and T are standing in a
I. Akola is 70 km away from Raipur and these
row facing north. Who is standing at the extreme
two cities are equidistant from Nagpur.
right end?
II. Bilaspur is 35 km away from Akola and is
I. Only P is between S and T. R is to the
exactly midway between Akola and Nagpur.
immediate right of T.
287. Four seminars A, B, C and D were organised one II. R is between T and Q.
@UPSC_THOUGHTS

on each day on four consecutive days but not


295. What day is the fourteenth of a particular month?
necessarily in that order. On which day was
I. The last day of the month is a Wednesday.
seminar C organised?
II. The third Saturday of the month was
I. The first seminar was held on 23rd, a
seventeenth.
Thursday and was followed by seminar D.
II. Seminar A was not organised on 25th and Directions: (Qs. 296 to 300): Consider the following
there was a gap of one day between seminars diagrams.
A and B. (a) Indicates that one thing (group)
is within the other one, but the
288. Who is father of Kamal? third one is altogether different.
I. Kamal and Navin are brothers.
II. Navin’s mother is wife of Ashok’s brother. (b) Indicates that two things are
overlapping while the third one
289. Who scored the highest among A, B, C, D and E? is altogether different.
I. B scored more than D, but not as much as C. (c) Indicates that one thing is within
II. E scored more than C but not more than A. the other while the third one
290. Among A, B, C, D and E who is in the middle includes the other two.
while standing in a row? (d) Indicates that there are three
I. C, who is third to the left of D, is to the things separate from one another.
immediate right of A and 2nd to the left of In the following questions, study the relationship
E. of each set of words and mark (a), (b), (c) or (d) as fits
II. C is 2nd to the left of E, who is not at any the relationship.
of the ends and who is third to the right of 296. Animals, Amphibians, Frogs
A. D is at one of the ends. 297. Red, Flower, Grass
291. A library contains only English, Hindi and Telugu 298. Europe, Russia, Angola
novels. How many Telugu novels are there in the 299. Doctors, Dogs, Vessels
library? 300. Sun, Planet, Earth

3 . 62
Logical Reasoning and Analytical Ability

Directions: (Qs. 301 to 305): From the options (a), (b), 310. Brinjals, Cauliflowers, Vegetables
(c) and (d) choose the diagram that best illustrates the set (a) A (b) B
of words in each question. (c) D (d) E
(a) One class is contained in the Directions: Choose from the following diagrams the one
other, but the third is separate. that best represents the relationship between the different
groups given in each of the questions 311 to 315.
(b) Two classes are contained in the
third.

(c) Neither class is contained in the


other completely but the two (1) (2) (3)
have common members and are
both contained in another class.

(d) Two classes are inter-related (4) (5)


and the third one is separate. 311. Artist, Painter, Brush
301. Protons, Electrons, Atoms (a) 1 (b) 2
(c) 5 (d) None of them
302. Science, Biology, Chemistry
303. Hydrogen, Atmosphere, Nitrogen 312. Cows, Animals, Carnivores
304. Machine, Lathe, Mathematics (a) 1 (b) 2
305. Novelists, Poets, Books (c) 3 (d) 5
@UPSC_THOUGHTS

313. Furniture, Wood, Steel


Directions: In each of the questions 306 to 310, out of
the five response figures given below, you are to indicate (a) 2 (b) 3
which figure will best represent the relationship amongst the (c) 4 (d) 5
three classes. 314. Singer, Writer, Actor
(a) 1 (b) 2
(c) 3 (d) 4
315. Vertebrates, Tiger, Camel
(a) 2 (b) 3
(c) 4 (d) None of them
316. In the given diagram, circle represents strong men,
A B C D E
square represents tall men, triangle represents
army officers. Which region represents army officers
306. Teachers, Graduates, Athletes who are tall but not strong?
(a) A (b) C
(c) D (d) E
5
307. Police, Criminals, Lawyers 1 2 4
(a) A (b) B 3
(c) C (d) E 6

308. Women, Civil Servants, Married Persons


(a) B (b) C (a) 1 (b) 2
(c) D (d) E (c) 3 (d) 4
309. Women, Married Persons, Working Wives 317. In the following diagram, parallelogram represents
(a) A (b) B women, triangle represents inspectors of police
(c) C (d) D and circle represents post-graduates. Which

3 . 63
Logical Reasoning and Analytical Ability

numbered area represents women post-graduate


inspectors of police? 3
5
13 4 2 1

5 8 6
3
6
4 (a) Muslims and Hindus
(b) Hindus and Christians
(a) 5 (b) 13
(c) 4 (d) 3 (c) Christians and Muslims
(d) All three
318. The triangle, square and circle shown below
represent honest, learned, rich people respectively. Diagram (Qs. 321 and 322): In the diagram given,
the triangle represents educated people, the rectangle
Which one of the areas marked A, B, C, D, E, F
represents scientists, the circle represents poets, the
and G represents the honest, and learned who are
square represents politicians. Study the figure and
not rich? answer the questions.
Educated people
A
Scientists
B C D Poets

F E Politicians
@UPSC_THOUGHTS

G
321. Which statement is not true?
(a) All politicians are educated
(a) B (b) A (b) Some politicians are poets and some are
(c) D (d) C scientists
319. The diagram below represents the students who (c) Some poets are educated but are not politicians
study Physics, Chemistry and Mathematics. Study (d) All scientists are educated
the diagram and identify the region which 322. Which of the statements can be true?
represents the students who study Physics and (a) All poets are scientists
Mathematics but not Chemistry.
(b) Some educated politicians are scientists and
poets
Physics Chemistry (c) Some scientists are politicians but not educated
(d) All poets are educated
P V Q
x y
U Directions: Questions 323 and 324 are based on the
T R following diagram.

S H
F
z Mathematics

G E
(a) T (b) P + T + S
(c) U (d) P + T
320. In the following diagram, triangle represents the The triangle stands for Hindi-speaking people,
Christians, square represents the Hindus and the circle for French-speaking, the square for
circle the Muslims. Each part is numbered. Which English-speaking and the rectangle for German-
communities occupy the area indicated by 5? speaking people.

3 . 64
Logical Reasoning and Analytical Ability

323. Which one of the following statements is true with (b) Some of the educated shopkeepers are road
reference to the given diagram? tax payers even though they discharge duties
I. All French-speaking people speak Hindi. of a policeman
II. All Hindi-speaking people speak French. (c) Some of the educated policemen who pay
III. No German speaks French. road tax are sharing profits with uneducated
IV. There are some English-speaking people who shopkeepers
speak Hindi and French. (d) No shopkeeper is a policeman and no
(a) Only I and III policeman is a shopkeeper
(b) Only II and IV Directions:: Questions 327 to 330 are based on the
(c) Only I, III and IV following diagram. The numbers represent the number of
(d) Only III and IV students taking up the different disciplines for study.
324. In the diagram, which one of the following
Chemistry
statements is true? 16
(a) There are some people who speak all four
14 13 12
languages
(b) Some German-speaking people can speak Botany 9 18 20 18 9 Zoology
either Hindi or English 15 13 16
(c) All English-speaking people can speak at
least one of the other languages Physics 19
(d) All Hindi-speaking people speak French but
not German
@UPSC_THOUGHTS

327. How many students took both Chemistry and


Directions: (for Qs. 325 and 326): The following Zoology?
figure represents a set of persons—the triangle represents (a) 60
educated persons, the rectangle represents policemen, the (b) 65
bigger ellipse represents road tax payers and the smaller (c) 66
ellipse represents shopkeepers. (d) 63
328. What is the number of students who have taken
all four subjects?
(a) 82
(b) 20
(c) 48
(d) Cannot be estimated
325. Looking at the given figure, which of the following
can be said? 329. Which of the following is true from the diagram?
(a) Some persons who are neither shopkeepers I. Those who study chemistry plus zoology and
nor policemen are educated but do not pay no other subject number 12.
road tax II. Those who study chemistry only number 16.
(b) Some persons who are neither shopkeepers III. Those who study all four subjects number 20.
nor policemen, pay road tax, though (a) I only
uneducated (b) II only
(c) Some persons who are either shopkeepers or (c) II and III
policemen pay road tax and are also educated (d) I, II and III
(d) All the above statements are correct 330. The largest number of students are found to study.
326. On the basis of the figure, it can be concluded (a) Chemistry
(a) None of the educated shopkeepers is a (b) Zoology
policeman though an uneducated policeman (c) Physics
is a shopkeeper (d) Cannot be determined

3 . 65
Logical Reasoning and Analytical Ability

ANSWERS WITH EXPLANATORY NOTES


1. (c) The conclusion follows clearly from statement Beams
2. Option (a) is not certain. The statement
Banks Bridges
does not say whether clocks are or are not
laptops. A universal proposition cannot be
drawn from a particular proposition. The We are told all bridges are beams, and all
diagrams show how some clocks may or may bridges are cows. Once again we cannot draw
not be laptops. a definite conclusion about cows and bridges.
All cows may be beams, or some cows (that
C = clocks are bridges) may be beams. See diagram.
C R L
R = radios
Beams
L = laptops Cows
Bridges
(i) In this case, no clock is a loptop.

L
C R (i) All cows are beams
Beams
(ii) In this case, some clocks are laptops. In
Bridges cows
both cases, no radio is a laptop, and no
laptop is a radio.
@UPSC_THOUGHTS

2. (d) If some pins are forks and all forks are keys, (ii) Some cows are beams.
some pins are keys. So (a) is correct. If some If one answer response had been ‘either
pins are keys, some keys are pins. So option some cows are beams’ or ‘all cows are
(b) is correct. From statement 2, it follows beams’, that answer response would have
that some keys are forks. So (c) is correct. been correct. But in the absence of such
As (a) (b) and (c) are correct, answer response an option, answer response (d) is correct.
(d) is best. 5. (c) The other conclusions are not valid; they may
or may not follow. Two possibilities are shown
P = pins in the diagrams.
P F K F = forks
P S P
K = keys
R C R
(i) C (ii)
3. (b) Statement 3 leads to this conclusion. Option S
(a) may or may not follow.
(i) R = rats R = roofs P = photographs
D = dogs C = cameras S = stores
R D H C H = horses 6. (b) All hands are machines, but all machines are
C = camels wheels. So all hands are wheels. Conclusion
II follows. By the given statements, ‘some
H C wheels are machines’ (conversion of statement
(ii) 2) and ‘some machines are hands’ (conversion
R D
of statement 1). Only some of those wheels
(c) is not valid. which are machines can be hands. So
conclusion I is not valid. See diagram:
4. (d) If we take the first two statements, we cannot
W
arrive at option (a) as a conclusion, as banks M
and bridges can both be beams and yet be H H = hands
exclusive of one another. Option (c) is invalid M = machines
on the same grounds. See diagram. W = wheels

3 . 66
Logical Reasoning and Analytical Ability

7. (a) Those stores which are shells are pearls. We statement we get ‘some cities are states’ by
know all shells are pearls, from which we can conversion. The third statement says ‘some
say ‘some pearls are shells’, and not ‘all cities are districts’. These statements could
pearls are shells’ So conclusion II is not valid. lead to many conclusions of which the given
See diagram. conclusions are not always valid. Diagramati-
cally some possibilities are:
S = stones
S Sh P
Sh = shells C = cities, D = districts
P = pearls S = states, Co = countries
8. (d) From statement 1, you get ‘some ears are
C Co
eyes’. One possibility is that some of the ears S C
that are eyes are also hands. In that case (i) Co D (ii) D S
conclusion II follows. The other possibility is
that the ears that are eyes and the ears that C C
are hands are not common; in which case S
D S D
conclusion I follows. Now either I or II follows; (iii) (iv)
both cannot be possible at the same time. Co Co
So (d) is the correct answer response.
So some states may or may not be districts
A diagram can show this.
and some countries may or may not be
states. Neither conclusion follows definitely.
eyes ears hands
(i) 12. (a) As all keys are locks and no lock is a door,
no key will be a door. So conclusion I is valid.
@UPSC_THOUGHTS

No hand is an eye. But conclusion II may or may not be valid.


We do not know if windows are locks or keys.
hands See diagram for possibilities.
eyes
L W
(ii) K = keys (i) K D
ears L = locks
L W
Some eyes are hands. D = door (ii)
K D
9. (c) From statement 1, it follows that ‘some shirts W = windows
are caps’ (conclusion II). From the two 13. (b) From the first two statements, you can
statements together, it follows that the shirts deduce conclusion II. All libraries are books,
which are caps are papers (conclusion I). So but all books are pages. So all libraries are
both I and II follow, and (c) is the correct pages. If you take the first and second
answer response. statements, you can deduce ‘Some pages are
Diagramatically we can show it: books’ and ‘Some books are libraries’,
respectively; from these two particular
C = caps
statements, a universal statement cannot be
C S P S = shirts validly drawn. So conclusion I is not valid.
P = papers See the diagram. The possibilities for ‘words’
x = all shirts that are caps. are many, out of which conclusion I could be
just one. However, Conclusion II definitely
10. (c) Conclusion I follows from statement 2. follows. P W
Conclusion II follows from statement 1. B = books W
B
P = pages L
C C = cats W
L T L = libraries
L = lions W
T = tigers W = words W

11. (c) From the first statement we get ‘some cities 14. (b) From the second and third statements you
are districts’ by conversion. From the second can deduce that some ashes are elements.

3 . 67
Logical Reasoning and Analytical Ability

By conversion you get ‘some elements are


ashes’. So conclusion II is valid. From the R
(iii) T C W
first two statements which are particular
affirmative, a negative universal conclusion
In (i) no rock is a table, but in (ii) and (iii)
cannot be drawn. So conclusion I is invalid.
some table is rock and some rock is table.
We do not know—we have not been told—
So conclusion I is uncertain, and only (b) is
if some clouds are particles or elements.
the correct answer response.
Diagrammatically we can visualise the
possibilities. 17. (b) See the diagram:
C = clouds A = ashes H C = clouds
P = particles E = elements W
T
W = winds
E C
E P T = typhoons
(i) C A P (ii)
C H = hurricanes
A
Only some typhoons are clouds. So conclusion
Conclusion I is not valid in the case of (ii)
I is invalid. All clouds are hurricanes. So
being true.
conclusion II is valid.
15. (b) Going from the third to the first statement,
18. (d) See the diagram:
conclusion II follows.
F = floppies
C = cars Mo = mouses
T = trucks F Mo S Mn
C T A S = speakers
S S = ships Mn = monitors
@UPSC_THOUGHTS

A = aeroplanes Both conclusions are valid.


As for conclusion I, we cannot deduce a 19. (a) See the diagram:
negative conclusion from positive statements. (i) R = rooms
We know ‘Some ships are cars’, from the B C B = buses
second and third statements, but we know R T
nothing about whether there are ships which C T = train
are not cars from the given statements. So (ii) C = cars
conclusion I is not valid. As all rooms are buses and no bus is a train,
16. (b) From the second statement you can derive, it follows that no room is a train. Conclusion
‘Some chairs are wood’. As the third statement I is valid. There are at least two possibilities
says that ‘all chairs are rocks’, you can derive for cars; either ‘some cars are buses—(i);
that some rock that is chair is wood. So hence ‘some buses are cars’. Or ‘no bus is
conclusion II is valid. Conclusion I is not the a car’ (ii). So conclusion II is uncertain.
only one to be drawn because there is a
Answer response (a) is correct.
possibility of some rock or chair being table,
as the first statement just tells us that no 20. (c) See the diagram:
table is wood. See the possibilities from the C
(i) I = islands
diagram: P
T = table C = chair I S C = countries
P P (ii) S = stars
W = wood R = Rock
P = planets
R (iii)
C From the first and second statements, it is
(i) clear that some countries are stars. So
T W
conclusion I is not valid. There are three
R
possibilities for planets. From (iii) it is clear
that planets may be stars. So conclusion II
T C W is not valid. Answer response (c) is correct.
(ii)
21. (c) Conclusion I follows from statement 3: ‘All

3 . 68
Logical Reasoning and Analytical Ability

schools are chairs’ (an A proposition) converted There are two possibilities (i) and (ii) for
to ‘Some chairs are schools’ (an I proposition). statement 3, but in both cases conclusion III
Conclusion II follows from statement 1: ‘All is valid, whereas conclusions I and II are
books are teachers (an A proposition) converted uncertain. So answer response (c) is correct.
to ‘Some teachers are books’ (an I proposition). 23. (b) Conclusion I talks of goats and fruits. So read
Conclusion III follows from statements I and the statements to arrive at the relationship
2. Books are totally included in teachers and between these two terms. Converting statement
teachers are totally included in chairs, So by 1, you get: ‘Some dogs are fruits’. Statement
statements 1 and 2, we can conclude: ‘All
2 says ‘Some dogs are cats’. Statement 3
books are chairs’ which leads to ‘Some chairs
says ‘All cats are goats’. So you may
are books’.
conclude that at least some dogs (those which
A diagrammatic representation will help you
are cats) are goats, and that means, by
answer the question easily.
conversion, ‘some goats are dogs’. Conclusion
B = books T = teachers III may thus be rejected. Now from statement
C = chairs S = schools
I you have derived that some dogs are fruits.
C
Now, the goats which are dogs could also be
T S fruits or may be outside the term fruits, though
B S S
they cannot be both at the same time. So
S either conclusion I or conclusion II follows.
S
If you look at the options, you will see that
only (b) can be correct. Option (a) is not
[You can see that there are various possibilities correct because I is not the only conclusion
@UPSC_THOUGHTS

for schools vis-a-vis books and teachers, but that can follow. Option (c) is not correct as
from statements 3, conclusion I always I and II cannot follow at the same time and
follows—‘Some chairs are schools.] III is wrong. Option (d) is not relevant here,
22. (c) As ‘some trees are roads’ (statement 3) and though it would be correct if the question
‘all roads are mountains’ (statement 2), we asked: which conclusion would ‘definitely’
may conclude ‘some trees are mountains’. follow from the given statements.
Conclusion III is therefore correct.
A diagrammatic representation makes it all
Statement I says ‘all snakes are trees’ which
we can convert to get ‘some trees are easier to understand.
possibility
snakes’. Taken with ‘some mountains are F = fruits G (i)
snakes’? Not with certainty: Some mountains C
may or may not be snakes; it depends on D = dogs
whether those ‘some trees that are snakes’ F D
are also ‘some trees that are mountains’. So C = cats C
conclusion I does not follow. By converting G (ii)
statement 1 we get: ‘Some trees are snakes’. G = goats
possibility
We are given in statement 3 that ‘some treeas
are roads’. Again we cannot conclude that While there is only one possible way to
some roads are snakes from those premises. represent statement 1, statements 2 and 3
So conclusion II also does not follow. in combination have two possibilities. According
Only answer response (c) is correct. to (i), ‘No goat is a fruit’—conclusion I.
A diagrammatic representation of the According to (ii), ‘Some goats are fruits’—
statements makes it easier to understand. conclusion II. So either conclusion I or
conclusion II is possible though both cannot
S = snakes M (i)
R be possible at the same time. It is also clear
T = trees T that conclusion III is invalid as in all cases
S ‘some goats are dogs’. So the correct answer
R = roads R response is (b).
M = mountains M (ii) 24. (d) From the given statement 2, you would easily

3 . 69
Logical Reasoning and Analytical Ability

reject conclusion II. If all swords are dogs, Conclusion II, it is seen, is not definite.
you cannot conclude that all dogs are swords; Conclusion III, too, it is seen, is not definite.
you can only say ‘some dogs are swords’. So answer response (c) is correct.
Option (b) is eliminated. Now as some pens
26. (d) All pens are pencils, so some pencils are
are swords, and all swords are dogs, you can
pens (Proposition A converts to I). Some pens
say some pens are dogs and, by conversion,
are erasers, so some pencils will be erasers
some dogs are pens. But taken with another
(as all pens are pencils). Convert and you get:
particular affirmative (I) statement,—some
some erasers are pencils (I → I). So conclusion
dogs are foxes—no definite conclusion follows.
III is correct. But don’t be in a hurry to mark
Some pens which are dogs may or may not
be among those dogs which are foxes. So (b); you need to check the other option with
conclusion I is uncertain. Option (a) is III in it.
eliminated. By statement 2 ‘all swords are With conclusion III being correct, conclusion
dogs’, so, by conversion, ‘some dogs are IV cannot be correct. So you may cross out
swords’—an (I) proposition. Statement 3 is answer response (c) which has IV in it.
also an (I) proposition. Taken together they Conclusion I says ‘Some clips are pens’. Look
lead to no definite conclusion. So option (c) up statement 3, from which you can conclude
is also elininated. None of the conclusions ‘Some clips are erasers’. From statement 2,
being solely correct, option (d) is the best you can conclude ‘Some erasers are pens’.
choice. Diagrammatically, you can represent Now, there is more than one possibility. Some
the statements. or all clips that are erasers may be erasers
F that are pens. In that case conclusion I
P = pens
@UPSC_THOUGHTS

follows. Another possibility is that as only


S = swords D
P S erasers are pens, some may not be pens, and
D = dogs F
F some of these erasers are clips. In that case,
F = foxes
no clip is an eraser—conclusion II. You can
It is clear that there are more than one see that either I or II can follow, though not
possibility for statement 3. So conclusion I
both together.
may follow in a certain case, but is not the
A diagram will make it easy to understand.
only conclusion to follow. However, conclusion (iii)
II and III definitely do not follow. E P = pens
C
25. (c) From statement I, it follows that ‘Some Pl Pl = pencils
pumpkins are apples.’ (I proposition converts P
C
to another I proposition). So conclusion IV is E
(ii) E = erasers
correct. A look at the answer responses tells (i)
you the choice lies between (c) and (d) as C = clips
C E
IV is mentioned only in these options. Now
(c) mentions I as well. So check if conclusion By possibilities (i) and (ii), no clip is a pen—
I follows from the statements. From statement conclusion II.
2 you know that all pumpkins are included By possibility (iii), some clips are pens—
in ‘rotten’ things and from statement 1, you conclusion I.
know some apples are pumkins, so some So either I or II can follow, but not both at
apples (which are pumpkins) are also rotten. the same time
Conclusion I is valid. Your answer choice is 27. (b) As from statements 1 and 2 you know some
(c). Option (a) is not correct, as IV also doors are windows and all windows are blue,
follows. A diagram makes it clear:
you can conclude some doors are blue.
A = apples Conclusion III is valid. Since answer responses
Po R
R = rotten (a) and (b) have III in them, one of them must
A
Pu Po Pu = pumpkins be correct. You have to check out conclusions
Po Po = potatoes I and IV. Draw a diagram.

3 . 70
Logical Reasoning and Analytical Ability

D = doors (i) T T
L L
Br (ii)
Br H
W = windows (i) F (ii) H F
D W B
B = blue

Br (iii) L T
Br = brown
(iii) H F
Possibility (i) and possibility (iii) show how
some windows can be brown (conclusion I).
Possibility (ii) shows how no window is brown Conclusion IV is valid from (i), (ii) and (iii).
(conclusion (IV). You can see that the windows From (i) and (ii) conclusion III is possible.
can be blue or not-blue; they cannot be both From (iii) conclusion I is possible. However,
at the same time. So it is either conclusion the two conclusions cannot work together. So
I or conclusion IV. Now you have conclusion answer response (d) is not correct. Only (c)
III being valid, and conclusion I or IV being is correct.
possible. Answer response (b) is best.
31. (b) The diagram makes it clear:
Incidentally conclusion II is not valid as it
(ii)
clashes with valid conclusion III. D T = trains
(i) D (iii)
28. (d)
D C = cars
Ty T = teachers
E B S
C B = buses
D D = doctors T
@UPSC_THOUGHTS

T S = shoes
E = engineers
D D = dresses
(iv)
Ty = typists
While it is clear that some shoes are trains
Conclusion I is valid and some buses are trains, dresses may or
Conclusion II is valid may not be cars or trains, as there are various
Conclusion III is valid possibilities. So conclusion II and III are
Conclusion IV is invalid. (An A proposition can correct, while conclusions I and IV are
be converted to I, not A.) uncertain. The best answer response is (b).
29. (c) Look at the diagram: 32. (c) The diagram makes it clear.

T M C = calculators M P = pens
B T
D (i)
C B = boxes K = keys
M
T = taps K D = doors
P
M M = machines D
T = trees
(ii)
As we are not told that machines are not
M M = monkeys
boxes or calculators, conclusion IV is not
valid. With the possibilities for machines, we It is clear that some monkeys are doors and
cannot categorically say I can follow. some trees are keys in both possibilities of
Conclusions II and III follow from the first and doors being keys. So conclusions III and IV
second statements. are valid. But because there is more than one
30. (c) From the second statement, conclusion IV possibility for ‘some keys are doors’, conclusion
certainly follows. But as the answer responses I and II are uncertain.
have an either/or option, check them out. These 33. (d) Some bats are lamps, but as all lamps are
diagrams will help to work out possibilities. rats, some bats are rats, and hence some

3 . 71
Logical Reasoning and Analytical Ability

rats are bats, conclusion I follows. As for the


other conclusions, they are not certain. The H R
W
diagram makes it clear. S D
R
P = pigs
L
B C = cows (ii) Some windows are sickles – Conclusion I.

C B = bats Conclusion II follows and either conclusion I


P B
L or conclusion III may follow. So answer
L B R L = lamps response (c) is correct.
R
R = rats (Whenever the options have an ‘either/or’
feature, you must be very careful to check
Some lamps may or may not be cows. Some the conclusions.)
bats may or may not be pigs. Some rats may
or may not be cows. There are various 36. (b) Conclusion II certainly follows from the first
possibilities. So conclusions II, III and IV are two statements: all spiders are dogs, but as
uncertain. Only conclusion I is valid, but as no dog is a cow, no spider is a cow. The
none of the answer responses gives this, (d) other conclusions may or may not follow, as
is the correct option. we are told nothing about whether pigs and
mynas are or are nor spiders and dogs. Or
34. (d) There are any number of possibilities for the whether some cows are or are not mynas.
three particular propositions. The given See the diagram:
conclusions are uncertain; they may or may
M
not follow. Some of the possibilities are shown
@UPSC_THOUGHTS

D P
in the diagram.
S C
B (i)
P = pencils
W

T C = clips No myna is a cow – Conclusion I


C No pig is a dog – Conclusion III
B T = trains
P No myna is a dog – Conclusion IV
T
W T W = wheels M
W D
B B = buses S C
(ii)
P
35. (c) Conclusion II certainly follows from the third
and fourth statements: as some rooms are
doors, and all doors are windows, it follows Some pigs are dogs – negates conclusion III
that some rooms are windows and some Some mynas are dogs – negates conclusion
windows are rooms. Conclusion I and III are IV
mutually exclusive, so they cannot be valid D P M
at the same time. But one or the other can
S C
be valid, as the statements say nothing about (iii)
windows not being houses or sickles, and as
only some windows are rooms. See the
Some pigs are dogs—negates Conclusion III
diagram for possibilities.
Some mynas are cows—negates Conclusion I
S = sickles
W 37. (c) Some tools are hammers, so some (not all)
H
D
H = houses
hammers are tools; Conclusion I is not valid.
S R R = rooms
Option (a) is eliminated. From the first and
D = doors second statements, you may conclude that
W = windows some hammers are buds: as all tools are buds
(i) No window is a sickle – Conclusion III. and some tools are hammers, those hammers

3 . 72
Logical Reasoning and Analytical Ability

that are tools will be buds. Conclusion III is possibility in which none of the conclusions
valid. However, there is an either/or option as follows. As there is no either/or option, only
well. So check it out. A diagram will help. option (d) is correct.
(i) F T = tools S = sticks
F
L = lamps
H
H = hammers L
B T S F = flowers
B = buds D = dresses
D
F Sh
(ii) F = flowers Sh = shirts
In possibility (i), no tool is a flower—conclusion Note: In solving questions 41 to 50 you need to be
II is valid. In possibility (ii), some flowers are clear about which propositions can be premises and
tools—conclusion IV is valid. Either II or IV what kind of conclusion follows from those premises.
is possible but II and IV are not both possible. Remember the rules of syllogisms and the fallacies
Correct answer response is (c). to avoid. While choosing the answer response, you
38. (b) A diagram helps in understanding the should pay attention to the order in the combinations,
statements and possible conclusions. for only a certain order will be logical.

C = chairs 41. (c) In the set in (a), the middle term ‘residents’
B B is not distributed in either premise. So the
B H = handles order is not logical. Diagrammatically you may
H P M P = pots see the possible invalidity of the conclusion,
C ‘some voters are citizens’.
@UPSC_THOUGHTS

M = mats
B residents
B = buses citizens

As we are not told anything about buses being voters


or not being pots, handles or chairs, there are
many possibilities, and no certain conclusion. It is clear that from the premises 1 and 3
So conclusions I and III are uncertain. But it cannot be logically concluded that some
it is clear that ‘some mats are chairs’ voters are citizens. It is possible they are—
(conclusion II) and ‘some mats are handles’ if within the class of residents, voters and
(conclusion IV) are certain.
citizens overlap, but it is not certain.
39. (d) Draw a diagram:
Option (b) has a negative conclusion to follow
R B = benches affirmative premises, so it cannot be valid.
H J W = walls Option (c) is logical.
W H = houses 3. All citizens are residents.
B
J = jungles 5. All voters are citizens.
R = roads ∴ 1. All voters are residents.
With ‘some’ statements taken together, there Option (d) does not give a logical conclusion.
are always more than one possibility. The 42. (d) Option (a) is invalid: one cannot conclude that
given diagram gives one possibility from which some lawyers are vagrants when all lawyers
you can see that conclusions I and II need are wealthy persons and all wealthy persons
not follow. From the first and second
are excluded from the class of vagrants.
statements, conclusion III follows. The third
and fourth statements lead to conclusions IV. Option (b) is invalid: when one premise is
So only answer response (d) is correct. negative, the conclusion must be negative.

40. (d) With so many particular statements, any of Option (c) is obviously invalid: the conclusion
the given conclusions would appear to be contradicts the premises taken together.
difficult to reach. The diagram shows one Besides, it violates the rule that the conclusion

3 . 73
Logical Reasoning and Analytical Ability

must be negative if one of the premises is term. The diagram shows how the conclusion
negative. is not certain (or valid).
43. (b) Of the given options, only 3-4-6 is logical: people who
3. No intellectuals are successful politicians. work hard some politicians
4. All shy and retiring people are honest who work hard
intellectuals. people

6. No shy and retiring people are successful


politicians. Option (c) has two particular premises from
Another logical combination would be 2 1 5 which a universal premise cannot be drawn.
but this is not among the answer responses. Option (d) draws a valid particular conclusion
from a universal premise and a particular
44. (c) This is the only option that has a combination—
premise. (3 should be rephrased as ‘All honest
AAA—that makes logical sense. Option (a)
people are people who work hard’.)
may lead to various conclusions but the set
is not a syllogism. Option (b) has an people who
undistributed middle term (This shows that the work hard
honest politicians
order matters: 1-2-3 is invalid, but 2-3-1 is all honest
people
valid). Option (d) has two particular premises
from which a universal conclusion cannot be
drawn. Conclusion: At least some politicians are of
the class of people who work hard.
45. (c) Option (a) is invalid as it draws a universal
negative conclusion from two particular 49. (a) This is a valid conclusion from the universal
@UPSC_THOUGHTS

affirmative premises. Option (b) is invalid as and particular premises.


it has all particular premises which leads to lazy people
an uncertain conclusion. Besides, no term is cooks
poor people
distributed. Option (d) is invalid as, with a
negative premise, only a negative conclusion
can be drawn. Option (b) is invalid as a negative conclusion
cannot follow from affirmative premises. The
46. (a) The set is All—a valid syllogism. Option (b)
conclusion is uncertain. Option (c) is invalid.
with the same propositions has an invalid
If we diagram propositions 3 and 1 we have
order, as the premises are both particular and
a universal conclusion does not follow. Option
lazy people cooks who
(c) has two universal premises from which a
are lazy
particular premise does not follow. Option (d) poor people
is invalid because the predicate in the
conclusion is not there in the major premise.
We cannot conclude from this that some
47. (b) From a universal negative and particular cooks are poor. Option (d) is invalid on the
affirmative, a valid particular negative same grounds—undistributed middle term which
conclusion has been drawn. Option (a) may leads to conclusions that are uncertain.
be rejected as it has an affirmative conclusion
50. (d) This conclusion is valid. See diagram for the
from two negative premises; indeed the two
premises.
premises cannot both be negative in the first
boys
place for a valid conclusion. Option (c) is not girls
all participants
in syllogistic form. Option (d) is invalid as the
middle term ‘strict’ is not distributed.
48. (d) Option (a) has two particular premises from Conclusion: No girl is a participant.
which a valid universal conclusion cannot be Option (a) is invalid as an affirmative conclusion
drawn. Option (b) has an undistributed middle cannot follow from a negative premise. In

3 . 74
Logical Reasoning and Analytical Ability

option (b), the two premises contradict each to definite conclusions. Clear cause-effect
another. Option (c) is invalid as the premises relationship has not been established.
are contradictory.
59. (b) The form of the first statement is: ‘All good
51. (b) You may consider I to be a more ‘fair’ option, football players are those people who can run
but it is not a conclusion based on the given fast, jump high and lift weights’. This means
statements. the class of good footballers is within the
52. (a) You have to assume ‘many’ to mean ‘some’. class of those who run fast; jump high, and
Conclusion I follows. You cannot conclude II lift weights. See diagram for the interpretation:
from the given statements, which are all
affirmative: no negative conclusion can be S = Suraj
people who S
drawn from them. can run fast, etc.
53. (d) The first 3 sentences make general statements S
about two kinds of crime. The next statement all good
footballers
tells us that some countries do not make a M M = Mayank
distinction between the two kinds. We cannot
conclude that this is because they have no
It is seen that with the abilities that Suraj has
legislature. Nor can we conclude from this
he may or may not be a good footballer. So
statement that some states do distinguish
between the two kinds of crimes. conclusion I does not necessarily follow.
Mayank is outside the class of both good
54. (b) ‘Not all’ means ‘some’. So conclusion II
footballers and those who are able to run fast,
follows. The other conclusions are not certain,
etc. So conclusion II is valid.
as the link between e-governance and
@UPSC_THOUGHTS

democracy has not been firmly established. 60. (d) If you read the statements carefully, and then
conclusion I, you will see that there is an error
55. (a) Conclusion I follows from the given premises.
of the undistributed middle. The conclusion is
Conclusion II does not, as what constitutes
not certain and therefore invalid.
‘forward’ has not been stated.
BHC = Bindas
56. (d) The statement is a universal affirmative one
Hoteliers Co.
by which conclusion I cannot follow. We may UPI
conclude some (not all ) regions showing good BC = Bankrupt
BC Companies
human development indicators show high BHC
economic growth. As we cannot categorically UPI = Companies
say whether the two features are or are nor unable to pay
interest on
linked—there is no mention of a cause-effect
debts
relationship in the statement, but we cannot
therefore reject it—conclusion II is invalid. Unless we are told that ‘all companies that
are unable to pay interest on their debt are
57. (b) Since ‘all’ countries with good governance
bankrupt’, conclusion I would not be valid.
show good economic growth, democratic
Conclusion II is clearly invalid, as the
countries with good governance fall within that
statements make no mention of criminals or
class. So conclusion II is correct. However,
criminality.
conclusion I is invalid, as we are only given
an affirmative statement on some democratic 61. (d) The clue word in the statement is
countries showing good economic growth from ‘indispensable’—which means the camel is
which we cannot draw a negative conclusion practically the only mode of travel suitable for
on democracies and good governance. Even the deserts. The given conclusions indicate
if we leave aside formal logic, there is nothing why camels are to be ‘preferred’ in deserts.
in the statements that attributes good economic 62. (d) Both conclusions can be drawn from the
growth exclusively to good governance. statement.
58. (d) The term ‘two-thirds’ is to be taken as ‘some’. 63. (b) The statement does not say what should
The statements are too indeterminate to lead happen but only what happens. So conclusion

3 . 75
Logical Reasoning and Analytical Ability

I is not valid. Conclusion II is implied: from weaker sections lack facilities to attend
everything gets old and gives way to fresh private coaching classes which are
and new things. concentrated in only some cities, and also
64. (a) Nothing in the statement indicates that women may suffer a disadvantage on the front of
are not interested in other things. So conclusion education or experience in examination
II is invalid. compared to other aspirants.

65. (a) Diagramatically, the statements may be 74. (b) It is only if the United Nations is assumed
represented as follows. to have this role that one can state that
member nations are not approaching the
persons likely institution for settlement of disputes.
to be mistaken
truly Assumption (2) is implicit. Assumption (1)
objective those seems correct at first glance, but it is an
persons who
ignore inference from the statement rather than an
facts assumption behind it.
75. (d) Both ‘assumptions’ are, in fact, ‘inferences’
or possible reasons for the better revenues.
The only assumption here could be that it is
So only conclusion I follows. possible for revenues from direct tax collection
66. (a) Import of sugar may be a policy action to to increase.
check prices but the reference to ‘future’ in 76. (a) 1 is an implicit assumption. The second is
(2) does not warrant continued import of sugar. irrelevant as an assumption, though it could
Only (1) is correct.
@UPSC_THOUGHTS

be an inference.
67. (d) If (1) is implicit, all gambling would have to 77. (c) Both assumptions are implicit.
be banned, not just ‘lotteries’. Banning lotteries
78. (a) The second assumption is too far-fetched in
is no way to control tax evasion by those who
the context of the statement.
conduct lotteries; so (2) is not implicit.
79. (b) The first assumption is not actually supported
68. (a) Assumption (2) need not lead to a charge on by the statement. We are not told whether
vehicles; it could simply read to a restricting the post was created for Jonathan or whether
order on use of the road by such vehicles, Jonathan replaced another director.
or even a ban. There is no guarantee that
80. (a) The second assumption is not relevant.
the charge of Rs 50 will ‘stop’ these vehicles
from using the road, even though it may 81. (c) If the profit has to increase despite reduced
reduce the use. Assumption (1) is implicit: the production, both assumptions are implicit.
charge would help to recover cost of 82. (c) Both assumptions are implicit.
construction.
83. (c) Both assumptions could be behind the stated
69. (c) Both assumptions could lead to the ban order, action.
though (1) is stronger than (2).
84. (b) The truck driver could not make assumptions
70. (c) Both assumptions are possible. for the cyclist’s behaviour as he has no
71. (c) Both assumptions are implicit. control over the latter. So assumption (1)
cannot be implicit.
72. (c) Yet another assumption would be that most
people are capable of using this media for 85. (c) Both assumptions are implicit.
their daily transactions. 86. (a) Assumption (2) seems out of context.
73. (d) The first assumption is not likely as all 87. (b) A doctor’s duty is not an ‘assumption’,
aspirants do not require or take coaching. As whereas the expectation of the patient’s
for the second assumption, the statement heeding the advice could be an assumption
does not mention subsidised or free classes. behind the doctor’s words.
Likely assumptions could be that aspirants 88. (a) The second assumption is rather cynical and

3 . 76
Logical Reasoning and Analytical Ability

can hardly be what motivated the warning and many words, the sentence, ‘To make sense
threat. of … possible options’ indicates it.
89. (b) Only assumption 2 is implicit. If biofuel is not 103. (a) This is indicated in the sentence, ‘The investor
produced in the country, it can be imported; also needs to select … aligned.’
so assumption 1 need not be required. 104. (a) The first few lines of the passage confirm the
90. (a) The second assumption is too general to be truth of the inference.
considered an implicit one. 105. (d) There is no reference to this in the passage,
91. (b) Including two weekends implies the assumption so we cannot say whether the inference is
that more people come at weekends. Also, true or false.
the decision to continue with a show would 106. (b) The last sentence indicates it, though does
be backed by the assumption that the number not actually say so.
of visitors would be maintained if not increased.
107. (d) There is no mention of government effort in
Assumption III is irrelevant, as it is not likely
the field of secondary education. So data is
to lead to the decision of continuing with the
not adequate to say whether the inference is
show.
true or false.
92. (c) All the assumptions are implicit.
108. (a) The passage confirms the truth of the inference.
93. (b) There is no reference to health in the
109. (c) The very first sentence contradicts the
statements; so assumptions I and III are
inference.
irrelevant.
110. (b) Read the sentences, ‘And improving that …
94. (b) Assumption I is not implicit. III is more of
spending in education.’ Clearly, allocating
@UPSC_THOUGHTS

an ‘inference’ than an assumption.


more funds is not going to improve the quality
95. (c) Either I or II could be an assumption implicit of education.
in the statement which reflects confidence 111. (a) The passage confirms the inference.
that the charges cannot be proved. III is not
indicated. 112. (b) The inference contradicts what is said in the
last sentence of the passage.
96. (d) The passage has nothing on coal mines of
113. (d) The passage does not have information on
developed countries.
the subject of ‘last decade’, though it mentions
97. (a) Unless the mines and activities are poor monsoon ‘this season’ resulting in drought
commercially viable, coal could not account situation ‘pan-India’.
for 50 per cent of the commercial energy
114. (b) The statement contradicts what is said in the
requirements of the country.
passage about the need to address anomalies
98. (a) This is stated at the very beginning of the such as over-reliance on ground water and
passage. lack of storage infrastructure. Earlier, the
99. (c) We are told that assessments of the resources passage implies a lack of ‘proactive policy
are often overstated and that projections for and sustainable practices’.
coal by study groups show demand may 115. (c) The mention of poor monsoon in the context
exceed supply, and that the country has faced of depleting ground water levels suggests this,
shortages. so it is probably true.
100. (b) We cannot categorically say that the 116. (d) The movie is cited as an analogy to illustrate
assessment process is not capable of how repetition of something helps us to
accurately identifying coal reserves, but it is understand it better. The movie does not help
likely from what is said at the end of the directly in improving business conversation.
passage. 117. (a) The last sentence of the first paragraph
101. (b) It is clear from the last sentence of the suggests this.
passage. 118. (a) That is what the passage leads up to, so it
102. (c) Though the passage does not say it in so is definitely true.

3 . 77
Logical Reasoning and Analytical Ability

119. (d) Nothing in the passage suggests this, but we 135. (d) Both are effects and have a common cause:
cannot, therefore, conclude it to be false as the health department fears that diseases may
it does not contradict anything in the passage. break out during the monsoon.
120. (d) We only know the author watched The Wizard 136. (d) The two are not linked, but both are results
of Oz thrice. The passage does not indicate of independent causes. (A) could be caused
if watching a movie more than twice is a habit by earthquake or climate disturbances; (B)
could be because the sky was clear and the
with the author. From the tone of surprise in
weather, fine.
‘Have you ever seen… ?’ One could even
choose ‘probably false’ if that option were 137. (d) The two are effects of independent causes.
there. 138. (b) The action in (A) is caused by (B).
121. (c) The passage clearly mentions ‘some of them’. 139. (a) It is the heavy emission that has led to (B).
122. (b) It is indicated in the first two sentences of 140. (a) The shortage in production is the cause for
the passages, though not stated as such. the decision to import pulses.
123. (d) The passage mentions ‘negotiating with the 141. (b) The statement mentions ‘while going’, so (b)
local bureaucracy’ in a general way; but no is the likely cause, rather the (a). Simply
because the driver was inexperienced (c) is
information is available on less cumbersome
not necessarily a cause of the children being
affairs in ‘almost all countries except India’.
injured. Incidentally option (d) could be an
124. (b) It is indicated in the first few sentences, if effect of the statement.
not clearly stated. 142. (a)
125. (d) The passage states that countries like India 143. (d) Option (d) is not a likely effect. In fact the
@UPSC_THOUGHTS

are behind other countries technologically so government is likely to have asked all schools
the inference is false. and offices to be closed for some time.
126. (a) The cause and effect are clear. 144. (c) Options (a), (b) and (d) could be effects of
the statement.
127. (d) At first the two seem unrelated, but, in fact,
both are the effects of a common economic 145. (c) The percentage tax increase will not be fully
cause, namely, recession. reflected in the price increase, so (c) will
probably be the effect rather than (b). Options
128. (c) Both are effects and have independent causes. (a) and (d) are unlikely.
129. (c) You could find independent causes for (A) and 146. (d) If India’s share is negligible, what happens in
(B); it will be difficult to link them as cause the international market would not greatly
and effect or to consider them as causes. affect the Indian industry.
130. (c) Statement (B) can hardly be the cause of (A). 147. (c) You may think of (a) but the matter at hand
Both are effects of independent causes. is rage killings which need not involve conflict
between residents and outsiders, but between
131. (a) It is because of the discount offered that
residents themselves—refer to ‘minor
many people show a preference for big retail altercations’. Option (c) would reinforce the
chains. psychologist’s views. Option (b) is already
132. (d) Both, in all likelihood, are effects of a common implicit in the psychologist’s reference to
cause, such as the need to prevent environment ‘disparity between social strata’. Option (d) is
degradation. irrelevant here.

133. (a) The tax burden has led to difficulty for 148. (c) This statement is by the psychologist. Option
(a) is implied in the police commissioner’s
exports, so the government has amended the
caution about rage killings ‘fast approaching’
laws to improve exports.
the number of deaths due to road accidents;
134. (c) The cause of (A) would be climate change. so the number of the former may soon
The cause of (B) may be to avoid getting wet. overtake the number of the latter. Option (b)
The effects (A) and (B) have unrelated is implied in the decline in the number of
causes. deaths due to road accidents. Option (d) is

3 . 78
Logical Reasoning and Analytical Ability

implicit in the mention of ‘minor altercations’ correct. Option (b) makes a wrong inference;
leading to rage killings—in other words, the the argument favours the large farmers’
public should be more cautions in social adoption of the measures of the small farmers
interactions. and not the size of the farms. Option (c) is
149. (b) Option (a) is not incorrect, but (b) is better gives a possible assumption by the author of
as it is more specific. Option (c) is vague. the passage: only if enough manure is available
Option (d) is an inference rather than an can large farmers adopt this measure. Another
assumption. possible assumption could be: the methods
that work for small farms will also work for
150. (b) Option (a) is not relevant, as it is the schools big farms.
who make use of the computers, and they
already charge the parents ‘in the name of 160. (d) Options (a) and (b) contradict the given
computer education’. Option (b) is correct as statements. Option (c) goes beyond the
this reinforces the speaker’s argument that statements. Option (d) logically flows from the
computer education in primary schools is a statements.
waste of money. Option (c) is vague, and 161. (d) The claim is that cigarette smoking is not
option (d) talks of ‘high school students’, not addictive, and the supporting evidence given
primary school children which is the speaker’s is that the Drug Act does not regulate
concern. smoking. It follows that the claim would be
correct only if the Drug Act regulated all
151. (a) This statement would take on the speaker on
addictive substances, and substances not so
the very fact that children in primary schools
regulated would not be addictive.
should learn the basics. Option (b) is not
relevant as primary school education needs 162. (b) It is only when the number of accidents
@UPSC_THOUGHTS

cannot depend on general public perception. remains steady that a valid comparison of
Options (c) and (d) are of no relevance either. fatalities can be made. Option (a) is too
generalised. Options (c) and (d) are not
152. (a) This is the only statement that would
relevant to the issue at hand.
strengthen the speaker’s argument.
163. (a) If (a) is true, there is no need for the
153. (b) The other options do not weaken the argument.
supplements that the author calls for. Option
154. (c) (b) does nothing to counter the author’s
155. (c) Option (a) is absurd. Option (b) is no argument. contention. Option (c) merely implies that only
Option (c) contradicts the findings as it implies obese people may require the supplements.
that motivated employees doing their duties Option (d) goes off at a tangent and is
would be more productive. Option (d) has irrelevant here.
nothing in the way of contradiction of what 164. (b) Just because Chitra has not paid a penalty
the passage says. is no reason to conclude that she is a good
156. (b) Option (a) merely adds another dimension to driver, unless the contention is that all those
the research. Options (c) and (d) go beyond who have not paid a penalty are good drivers.
the matter at hand. After all, she may not have been caught
breaking a traffic rule—which has nothing to
157. (b) It is only if this statement is assumed that
do with being a good driver. The other options
it can be concluded that strategies to counteract
are not relevant.
each susceptibility will prevent disease. Option
(a) is not relevant. Options (c) and (d) go 165. (a) If the number of car accidents were so large,
beyond the passage. the one per cent would result in many more
deaths than the 50 per cent in train accidents.
158. (d) Option (a) speaks of fewer serious cases, Arguments depending on incomplete statistics
which is not what is contended in the can be faulty. Option (b) does nothing to
passage. Options (b) and (c) actually support weaken the argument that cars are safer than
the health officials’ claim. trains. Option (c) is talking about ‘causes’,
159. (c) The passage asks the large farmer to adopt which is not relevant here. Option (d) talks
the measures of the small farmers, so it is of passengers, whereas the argument is about
not an assumption, so option (a) is not number of accidents.

3 . 79
Logical Reasoning and Analytical Ability

166. (b) Options (a), (b) and (c) go beyond what is 174. (d) No use of analogy is made, so option (a) may
stated in the passage which only links ripe be rejected. The passages are not similar, so
mango consumption with blood pressure. option (b) is not valid. It certainly uses
167. (d) Obviously in the circumstances, the statistical data, but not to prove, but to
disprove the speaker’s argument in I; so reject
government will have to rethink the decision,
(c) and select (d). If you read II, you will
as there does not seem to be such a demand
gather from the figures, that there is little or
for banks in villages.
no requirement for the children to ‘cross’ the
168. (c) Clearly, the sampling of the voters was not intersection.
representative of all voters, so the skewed
175. (d) The passage gives the idea that Hatha Yoga
result of the exit polls. Option (a) would lead
is much more difficult than Raja Yoga. Only
one to expect the exit polls to have predicted
option (d) strengthens the argument: as the
victory for the MLA. Option (b) is outside the
percentage of students who have mastered
scope of the argument. Option (d) does not
Raja Yoga is more than the percentage of
quite explain why the exit poll results would students who have mastered Hatha Yoga,
be skewed; after all even few voters could Raja Yoga by implication is not as difficult
have been a representative sample. as Hatha Yoga. The other options refer to
169. (d) The argument ends with a line that suggests students ‘practising’ the forms of yoga or
the correct response. The cause of the schools ‘teaching’ the forms of yoga.
disease was thought to be one thing, but now 176. (c) There is a comparison drawn, so the argument
it is believed to be something else—and that uses analogy. There is no general inference
cause, according to recent evidence is a from the specific, so option (a) is wrong. Nor
virus. So if the virus alone is observed in is there a cause: effect argument, so (b) is
@UPSC_THOUGHTS

many cases of the disease, the argument is wrong.


supported. Option (a) accords a secondary 177. (c) Option (a) specifies India as able to control
role to the virus. Options (b) and (c) are inflation, whereas the argument clearly says
outside the scope of the argument. ‘no single country …’, so it is incorrect. Option
170. (c) The other options would explain the situation. (b) contradicts the argument, so cannot be a
The increase in birth rate in a decade would logical conclusion. Option (d) goes beyond
not produce able-bodied labour within that what is stated in the argument and hence is
decade, so (c) is the correct answer response. invalid. Option (c) is the logical conclusion.
171. (c) Option (a) endorses and does not refute the 178. (b) Option (a) is wrong as it seems to be making
given argument. Option (b) holds out some excuses for the author’s very contention.
‘carrot’ type of future reward which is anyway Option (c) brings in intelligence, whereas the
outside the scope of the argument. Option (d) passage talks of education, so it goes beyond
also makes a statement which is outside the the argument. Option (d) goes way beyond the
scope of the argument. Option (c) gives a argument. Option (b) undermines the argument
as it says a person educated in the true sense
reason why CEOs of big organisations may
would be conversant with many subjects, so
get higher salaries than those of smaller
would not be stupid: there would be few
organisations.
subjects you could get him or her ‘off’.
172. (b) There is nothing within the passage to suggest
179. (d) In such cases of comparisons, unless the
that amount of contribution and the official
same numbers are compared, percentages
language are linked in any way. So it is not
alone will give an accurate picture. So any
logical—because it goes beyond the argument.
option that compares numbers may be rejected,
173. (b) The intersection is not compared to anything, such as (a) and (c). Option (b) is one-sided
so option (a) is invalid. There is no story of in its approach and may be rejected.
one child’s near accident in the passage, so 180. (b) This would mean that the infection would have
option (c) is not correct. Option (d) is off-track. spread from that mouse, and the results of
Option (b) is correct, as it appeals to the the experiment would be corrupted. Option (a)
emotions of the readers. and (c) are irrelevant, and option (d) is absurd.

3 . 80
Logical Reasoning and Analytical Ability

For Qs. 181 to 185, Draw a diagram based on the Next, as Q is second to the left of T and
information: there are eight persons, so draw a circle fourth to the right of Z, we have
and mark off eight places. With the first bit of S
information you get a figure: Q

H V
Right
T Z
D
Next, plug in the positions of W, P and R,
Left
A taking the clue that P is not an immediate
neighbour of T. (So P can only be between
[Just get the ‘right’ and ‘left’ positions clear—
it is ‘right’ or ‘left’ with respect to the persons S and V.) With W in place, the only vacant
in the question, and not with respect at you.] place is filled by R.
C, we are told, is the immediate neighbour S
Q P
of H. If C is on the left of H, we cannot place Right
B at fourth place to the right of C, as the
W V
position is already taken by A. So C is
immediately on the right of H, and we now Left
T Z
have the figure:
R
H
C
Now the questions can be easily answered.
186. (d) The correct person would be Q.
D
187. (c) 188. (a) 189. (b) 190. (c)
@UPSC_THOUGHTS

A B For Qs. 191 to 195, draw a diagram.


We are next told G is not a neighbour of B D
or C. So the only position G can occupy is C G
Right
that between H and D. Also, F is not a
neighbour of B, so F must be immediately B F
to the right of C. Once these are in place, Left
the position of E is clear. The figure with all A E
of them seated will be: H
H 191. (c) 192. (d) 193. (c) 194. (b) 195. (a)
C G
For Qs. 196 to 198, the following diagram will help.
F D Note that there are 9 persons.

I H
A B
E Right
E B
Now questions 181 to 185 can be easily F
answered. C Left
181. (c) 182. (d) 183. (b) 184. (a) D
A G
185. (b) H and E are opposite one another and there
are three places between them whichever side 196. (b) 197. (a) 198. (d)
you take. For Qs. 199 to 201, the following diagram will help.
For Qs. 186 to 190, draw a diagram.
A
At first instance, you have F H Right
S
D E
Left (clockwise)
V C G
B
T 199. (c) 200. (d)

3 . 81
Logical Reasoning and Analytical Ability

201. (d) Draw the diagram and compare with the earlier W (Z’s wife)
one: S

A
B V’s wife Z (male)
H

G C
V (male)
F D
As T is a male, he can only be between W
E
and V’s wife. And if Y is not an immediate
For Qs. 202 to 205, the following diagram will help.
neighbour of V, and Y can only be a female
A
F G (as all three males have been named), Y must
be V’s wife. The position of R and Q now
C D become clear, as R sits second to the right
of Q:
E H
B W (Z’s wife)
202. (a) 203. (a) 204. (d) 205. (a) T (male) S (female)

For Qs. 206 to 210, draw a diagram. You are given Y Z (male)
seven persons in a straight line facing north. As P (V’s wife)
sits fourth to the left of L and L is second to the left Q (female) R (female)
of Q, the only position for P is extreme left, and L V (male)
is second from right end. Right extreme is Q.
N
@UPSC_THOUGHTS

211. (d) 212. (b) 213. (d) 214. (d) 215. (b)
P L Q
Now it is easy to fill in the other positions. For Qs. 216 to 220, the diagram will be:
A (Germany)
N
F (India) D (Holland)
P S R T L W Q
C (Spain) H (Thailand)
206. (c) 207. (b) 208. (d) 209. (b)
E (USA) B (Austria)
210. (d) Compare the new seating positions with the
earlier one: G (France)

216. (b) 217. (d) 218. (c) 219. (b) 220. (c)
L P Q R S T W

For Qs. 211 to 215, draw a diagram. At the first For Qs. 221 to 223, you need to tabulate the
instance we have: information
S
Student Gender College Subject

P boy B MBA
(V’s wife)
Q girl A BCA
V
We deduce from the given statement that Z R girl B Medicine
is a male. If Z is not an immediate neighbour S boy A Journalism
of V’s wife, and his wife W sits second to
the right from him, Z cannot sit between S T boy A BCA
and V’s wife. He cannot sit between V and W boy C Av. Tech.
V’s wife because no two males can be
immediate neighbours. So Z must be Z girl C Medicine
immediately to the left of S (who must thus
221. (d) 222. (a) 223. (b)
be a female). We have:

3 . 82
Logical Reasoning and Analytical Ability

For Qs. 224 and 225, tabulate the data. For Qs. 229 and 230, make a table.

Persons City Mother Tongue Person Subject State

H Chennai Marathi H Architecture Maharashtra

I Hyderabad Telugu I Pharmacy Karnataka × Arch.

J Bangalore
× Tamil
Kannada × Marathi J Engineering Madhya Pradesh × UP
× Punjab
K Ahmedabad Punjabi K Management Punjab

× Chennai/Kolkata V Science Kerala


L Delhi Bangla × Marathi
M Kolkata Tamil × Chennai W Commerce Andhra Pradesh
× Marathi
N Mumbai Hindi X Arts Uttar Pradesh × Punjab

With the available data in the first place you


(The later additions to the table are encircled.
can place all the data that is not encircled.
×-marks indicate ‘not’-features.) The person from Andhra Pradesh studies
From the initial plug-in of data, it is clear that Commerce; this can only be plugged in
Marathi can be the mother tongue of H. Now against W which is completely vacant. As I
does not study architecture, only H is left to
J can only have Kannada as mother tongue.
do so. Plug in pharmacy—the only subject
And as L and M don’t work in Chennai, and left—against I. J can only belong to Madhya
@UPSC_THOUGHTS

L does not work in Kolkata, either H or M Pradesh. So K is from Punjab and X is from
is working in Kolkata. But you are given that Uttar Pradesh. The table will now have all the
the one who is at Delhi has Bangla as mother encircled data.
tongue. Only H, L and M have to be 229. (b) 230. (b)
considered. As L and M are not for Chennai, For Qs. 231 and 232, make a rough sketch and plug
H must be in Chennai. As L is not for Kolkata, in data:
Sons Mothers
M must be in Kolkata. Plug in Delhi and × Q, R A P
Bangla for L, and the other two get happily
settled. B Q × B, C
224. (d) 225. (b) ×R C
R
For Qs. 226 to 228, tabulate the information. D

S
Friend College Profession × Q, S, R E T × A, E

A Y fashion The pairs are A-S, B-R, C-T, D-Q, E-P.


231. (c) 232. (d)
B Y actor
For 233 and 234 tabulate the data:
C Y architect Student Standard Subject
P V Geography
D Z teacher
Q VII History

E X medicine R VI English
S IV Mathematics
F Z engineer T VIII Hindi
V X Science
G X business
W IX Sanskrit
226. (c) 227. (b) 228. (d) 233. (c) 234. (d)

3 . 83
Logical Reasoning and Analytical Ability

For Qs. 235 to 239, tabulate the data. For Qs. 252 to 255, Work out as follows:
From (ii) we have G < D < A
Person Work Place From (iii) we have H < E < C
B Marketing Nagpur
From (iv) we have F < B < D
From (v) we have C < G
G Engineering Chennai From (vi) we have G < F
Now from (vi), (v) and (iv)
H Marketing Ranchi We have C < G < F < B < D
K Engineering Kolkata Plug in what we know from (v), (iii) and (ii)
and we have:
D Systems Hyderabad
H < E < C < G < F < B < D < A
F Systems Kozhikode 252. (c)
T Marketing Patna 253. (b) We cannot say (a) or (d) as J may or may
not be shorter than C. Option (c) is not true.
V Systems Bhopal
254. (a) 255. (b)
235. (b) 236. (a) 237. (c) 238. (d) 239. (d)
For Qs. 256 to 259, draw the office plan and
For Qs. 240 to 243, tabulate the information: occupants:
Persons Gender Profession D T M B H R
A male doctor—(grandfather of F) 1 2 3 4 5 6
B female lawyer—(mother of F and E) 256. (d) 257. (a) 258. (b)
@UPSC_THOUGHTS

C male jeweller—(married to lawyer) 259. (d) Mr. Tarun is next to Mr. Dina who requires
silence in the office next to his. If Mr. Tarun
D female manager—(married to A) takes over Ms. Reena’s duties there would
be the disturbance from telephone use.
E male engineer

F female psychologist } —siblings


260. (b) If TZ serve on committee B, committee A will
have exactly one individual more, so it will
have 3 members. So committee C will have
9 – (2 + 3) = 4 members.
240. (c) 241. (b) 242. (b) 243. (d) 261. (c) The least number to serve on committee B
For Qs. 244 to 247, make a sketch of the data: is 1 and, consequently, there will be 2
members on committee A. So there can be
Lawyer (head of family)
= Reena’s mother-in-law a maximum of 6 members in committee C.
262. (d) In the case of any other option, T and S will
be left without a committee.
Son Son 263. (a) If T, S and X serve on committee B,
Wife Mukesh Rakesh Mrs Reena
(lawyer)
committee A has to have 4 members—who
(doctor) (teacher) (teacher)
can only be R, W, V and U. So the two
members left for committee C are Z and Y.
Son
264. (b) No other option works under the given
Ajay
conditions.
244. (a) 245. (c) 246. (b) 247. (d)
265. (d) In this order, two secretaries—G and D—
For Qs. 248 to 251, make a diagram: speak one after the other, whereas each
Numerical General Awareness secretary must speak after his/her principal.
Highest Q T 266. (c) A is the third of the secretaries to speak, and
T P L speaks after A, so A cannot be the fourth
P S speaker after M.
R R For Qs. 267 to 271, possible routes are as follows,
Lowest S Q given that the paths are of equal length. P can be
248. (b) 249. (d) 250. (a) 251. (c) at several positions.

3 . 84
Logical Reasoning and Analytical Ability

M
P 275. (b) Option (a) does not conform to the condition
that the green ball is under cup 5. In (c) purple
F G P comes above orange. In (d) red and magenta
are not adjacent.
D G
276. (d) If magenta is under 6, red cannot be adjacent
N
(as green is under 5); and if magenta is in
E G 6, red cannot be adjacent to it. So options
P (a) and (c) are out. Option (b) does not work,
O
as purple has to be under a lower-numbered
267. (a) The route could be NOMNP, PNMON or cup than orange, and 6 is the highest-
PNOMN; in all cases N has to be touched numbered cup.
twice. By any other route, the person would
have to walk a path more than once—which 277. (c) As green is under cup 5, only if purple comes
goes against the given condition. under a lower-numbered cup than 5 can it
conform to the condition of coming lower than
268. (a) See the diagram. If E and G connecting O orange. The other options may or may not
and P are not in a straight line, OP will be be true.
shorter than the route covered by E and G.
278. (c) Unless the three 18th century paintings come
269. (d) in the first three places, the required empty
270. (a) The only route under the conditions is PNM spaces between groups will not be possible
or along GF. if both spaces 4 and 5 are to be empty. You
271. (d) Option (a) is not a ‘result’ of the new paths need not necessarily have 10 as empty
between M and P and between O and P. space, as the arrangement could have the two
Options (b) and (c) are obviously wrong; see 19th century paintings in 6 and 7, and the
the following diagram. four 20th century paintings in 9, 10, 11 and
@UPSC_THOUGHTS

M
12 positions. So option (a) is out. Since you
could have the four 20th century paintings in
slots 6, 7, 8 and 9, to be followed by the
19th century paintings in 11 and 12, option
N
P (b) need not be true. Option (d) does not work:
as already explained, only the 18th century
paintings can occupy the first three slots if
O
space 4 is to be empty.
For Qs 272 and 273, sketch the routes. 279. (c) Reverse chronological order means starting
B with the 20th century paintings which are 4
3k in number. As 5 is to be empty as per given
m
condition, options (a) and (b) may be rejected.
D Route 1 : ABDF
2 km After the four 20th century paintings come the
Route 2 : ACDF
two 19th century paintings in slots 6 and 7—
m

A F Route 3 : ACF
4k

after which at least one space has to be left


5k m at 8. Now if the three 18th century paintings
m 5k are placed in slots 9, 10 and 11, space 12
C will be empty. This shows option (c) is correct.
272. (d) Route 1 is longest and is 13 km. So, if Option (d) is not correct as it does not allow
DF = 2km and BD = 3 km, AB will be 13 the paintings to be shown without breaking the
– (2 + 3) = 8 km. groups.
273. (a) 280. (d) J, K and L belong to the same group, so they
274. (d) As the accountant looks after the accounts will occupy at least three spaces together; if
of Mukherjee and Chatterji, neither of them J is in 11, K and L will have to be in 10
is the accountant. Chatterji has a nursing and 12. In that case, space 9 will have to
home but is not a doctor, so he is the dentist. be empty. If J, K and L are 20th century
We are told Chatterji does not know Bannerji, paintings, space 9 will also be occupied. Now
so the latter cannot be the accountant; since option (a) does not work as F and M are 18th
Bannerji works in an office, he must be the century paintings and there has to be space
lawyer. Now, clearly, Mukherji is the doctor between different groups. G in 9 does not work
and Pestonji is the accountant. either, as G does not belong to the same

3 . 85
Logical Reasoning and Analytical Ability

group as J, and if it is from another group, 287. (d) The information provided in both the statements
there has to be space between groups. So is necessary to answer the question.
options (b) and (c) are out. Option (d) is A — Thursday — 23rd of the month
possible if H is the fourth 20th century D — Friday — 24th of the month
painting. B — Saturday — 25th of the month
281. (d) In option (a), there are three software manuals C — Sunday — 26th of the month
(R, S, T) whereas only two can be selected. Thus, seminar C was organised on Sunday,
Besides, with S on display, T cannot be there. 26th of the month.
In (b), there are four hardware items against
three allowed; besides, with M on display 288. (d) From the first statement alone we cannot get
there should also have been L besides R. the answer. From the second statement alone
Option (c) cannot be correct as M cannot be it is not possible to get the answer. Both
displayed without L and R. So none of (a), together provide the answer: Navin’s mother
(b) and (c) can be displayed. is Kamal’s mother as Navin and Kamal are
brothers. So the father of Kamal will be
282. (a) With K on display, U will have to be on
Ashok’s brother, the husband of Navin’s
display. So the two software manuals have
mother.
to be T and U. The other two hardware items
cannot include M (with which L and R need 289. (d) Statement I alone does not help us reach the
to be displayed as well). The choice can only answer:
be N and O (both of which have to be D < B < C
displayed together). So there is only one II alone does not suffice to get the answer:
acceptable group for display. C < E < A
283. (c) Option (a) goes against the condition that R From I and II we get the following
must be in the same class as Y. Option (b) A > E > C > B > D
goes against the condition that X must be A scored the highest.
@UPSC_THOUGHTS

assigned a class that meets earlier than the 290. (c) From I, we can arrange A B C D E as A
class to which W is assigned. Option (d) is C B E D. We can say who stands in the
not correct as S and X are not be assigned middle.
to the same class. From II, too, we can arrange the order, and
284. (d) With S and Q in the 6 p.m. class, that class get the same answer.
is complete (as Q can be with one student 291. (a) We know that there are only books in English,
only). So T can now only attend the 10 a.m. Hindi and Telugu. If the total number of books
class. Options (a) and (b) are out. Option (c) is 1000, and 50 per cent is in English and
is not correct as, if W gets this class, the Hindi as per statement I, we can easily
condition for X attending an earlier class conclude that 50 per cent is in Telugu, and
cannot be met. Option (d) is correct.
the answer is found. So statement I is
285. (b) T has to be in that class, so option (a) is sufficient to find an answer to the question.
out. Option (c) brings S and X in the same
Statement II is not enough to get an answer
class, so it cannot be correct. If W is given
to the question.
the 10 a.m. slot, where will X go, as X has
to go to a class before W? So option (d) is 292. (c) We already know that Mohan is taller than
not correct. Pradeep, and that Ram is taller than Deepak.
Statement I says Pradeep is taller than Ram.
286. (b) According to statement I,
70 km So Pradeep is taller than Deepak. So we have
Akola Raipur
Mohan > Pradeep > Ram > Deepak
But the information that Nagpur is equidistant Statement I is sufficient to answer the question.
from Akola and Raipur does not help us to Statement II tells us Mohan is taller than
know the exact distance between Akola and
Ram. We already know Ram is taller than
Nagpur.
Deepak and that Mohan is taller than Pradeep.
According to statement II, So we can establish from statement II who
35 km 35 km is the tallest.
Nagpur Bilaspur Akola
So the distance between Nagpur and Akola 293. (c) From I we get the following arrangement
is (35 + 35) km = 70 km. F G ] Right end
So only statement II is enough to answer the Now wherever the others stand in the row,
question. G will remain in the middle.

3 . 86
Logical Reasoning and Analytical Ability

From II we get the following standing


arrangement 303. (b) Nitrogen Atmosphere Hydrogen
F E G H I or E F G H I
We may not know the position of E and F.
But we know for sure that G is in the middle.
So either statement is sufficient to get the
answer.
Machine
294. (d) Statement I by itself is not enough, the
304. (a) Mathematics
positions as given being: Lathe
S P T R
From which we can’t say where Q stands—
at the extreme right end or the extreme left
end.
305. (d) Novelists Poets Books
Statement II by itself leads us little forward,
but in combination with statement I, helps in
getting the answer: Some novelists may be poets, and some poets
S P T R Q may be novelists. Both may write books, but
295. (b) Statement I is of no help. The last day of are separate from the class of books.
the month is a Wednesday. But the last day
306. (d) Some teachers may be graduates and some
of a month could be 30th, 31st, 29th or 28th.
teachers may be athletes. All classes may
But given that the third Saturday of the month have some members in common:
was 17th (statement II), it is obvious that 14th
of the month fell on Wednesday.
Teachers Graduates
So statement II alone is sufficient to answer
@UPSC_THOUGHTS

the question.
imals
An Athletes
Amphibians
296. (c)
Frogs 307. (d) Sad as it is, all three classes may have
common members. So:

297. (b) Police


Red Flower Grass

ro
Eu pe Lawyers Criminals

298. (a) Russia Angola


308. (d) All classes have common members, though
none is contained in another.
299. (d) Doctors Dogs Vessels 309. (c) Working wives are in the class of women who
are married:
n
P la e
t

Married
300. (a) Earth Sun women
Married (wives)
persons
Wives Women
who
301. (b) Protons Atoms Electrons
work

Brinjal Vegetable Cauliflower


302. (c) Physics Science Chemistry 310. (b)

3 . 87
Logical Reasoning and Analytical Ability

311. (d) None of the diagrams fits the group which is the circle, so III is true. There is a region
best represented as: where square, triangle and circle overlap; so
IV is true.
Artist
324. (b) As the rectangle is entirely outside the circle,
option (a) cannot be true. There are some
Painter Brush
Hindi-speaking people who know German, and
some others who know English and French;
these groups do not overlap. Option (c) cannot
be true as a large part of the square (English-
312. (d) Some animals are carnivores. Some carnivores speaking people) is outside the rectangle,
are animals. (Remember, there are some circle and triangle. Option (d) is obviously
plants, too, that are carnivores.) The cow is wrong.
an animal but not a carnivore. 325. (d) Option (a) is represented by the region of the
triangle that is outside the rectangle and the
Animals
ellipses. Option (b) is represented by the part
Cow of the bigger ellipse that is outside the
Carnivore
triangle, rectangle and the smaller ellipse.
Option (c) is represented by the overlap region
of (i) the smaller ellipse, bigger ellipse and
313. (c) There is furniture made of wood and there is triangle or (ii) the rectangle, bigger ellipse and
furniture made of steel but wood and steel triangle.
are separate. 326. (d) It may be seen that the small ellipse
314. (b) 315. (b) (shopkeepers) is entirely outside the rectangle
(policemen), so the two classes have no
@UPSC_THOUGHTS

316. (d) The required region is the one which lies


members in common. Moreover, as the
inside the triangle (army officers) and the rectangle is entirely contained within the
square (tall men) but is outside the circle triangle, all policemen are educated persons.
(strong men).
327. (d) Number of students who took both chemistry
317. (d) The required region is the one which lies and zoology: 12 + 13 + 20 + 18 = 63. As
inside the triangle (inspectors of police), the the question has not asked us to exclude
circle (post-graduates) and the parallelogram these students who have taken more than two
(women). subjects, we include students who, apart from
chemistry and zoology, have taken botany or
318. (a) The required region lies inside the triangle
physics as well.
(honest) and the square (learned) but outside
the circle (rich people). 328. (b) 20 is the only number common to all the
circles.
319. (a) Required region is shared by circles x and
z (physics and mathematics) but is outside 329. (d) All three statements are correct.
y (chemistry); it is T. 330. (c) You can estimate it by just working at the
numbers that fall within the circle for physics:
320. (a) The region marked 5 is the region occupied
all numbers are not only in double digits but
by the Hindus and the Muslims.
also mostly of higher value than those outside
321. (a) A large portion of the square (representing the circle. You may, of course, check by
politicians) lies outside the triangle (representing calculation.
the educated). Hence a number of politicians Number of students who study chemistry—
are not educated. So, ‘All politicians are 16 + 14 + 13 + 12 + 18 + 20 + 18 = 111
educated’ is not true. Number of students who took zoology—
13 + 12 + 20 + 18 + 9 + 13 + 16 = 101
322. (b) Parts of the triangle, the rectangle and the
Number of students who took botany—
circle overlap with a part of the square 9 + 14 + 13 + 20 + 18 + 15 + 13 = 102
showing that (b) can be true. Number of students who took physics—
323. (c) The circle (French-speaking) is contained in 19 + 16 + 13 + 15 + 18 + 20 + 18 = 119
the triangle (Hindi-speaking), so I is true but Therefore, physics was taken by the largest
II is wrong. The rectangle is entirely outside number of students.

3 . 88
4
DECISION-MAKING
AND
PROBLEM-SOLVING
We make decisions and solve problems all through life. However, a
person who is in a responsible position in an organisation or holds an
administrative post is required to make decisions that could have an effect
on many people, and not just on oneself and one’s close relatives/friends.
@UPSC_THOUGHTS

Such decision-making involves taking into account several pros and cons of
possible consequences and difficulties in implementation. Administration no
longer can take recourse to secrecy, as more and more information and the
right to information become available in the increasingly democratic
ambience of governance. In the circumstances, there is a need for
transparency and openness on the part of administrative officials. Decisions
that affect a large number of people—the public at large, in fact—have even
more than ever before come to be governed by ethical considerations as
well as legal requirements. Civil servants have to be fair in their dealings,
credible, and capable of getting the trust of the public. They have also to
develop an ability to deal with emergencies and crises; this requires an
ability to retain coolness in the face of disturbance, and the ability to think
clearly and reach quick effective decisions to solve the problem at hand.
And these problems are not mathematical ones with ‘correct’ and ‘incorrect’
solutions. The UPSC syllabus has included decision-making in the syllabus for
the objective type test, which is a first of sorts. The questions are more
likely to test your ability to respond to given situations than your knowledge
of theories; it is from your response to situations that it would become
possible to gauge your attitude, your way of thinking, and your
responsiveness to the people you are expected to serve. In this section, we
have, however, given a brief outline of the theories of decision-making which
might help you in developing your approach to situational questions too.
Decision-Making and Problem-Solving

@UPSC_THOUGHTS

4.2
Decision-Making and Problem-Solving

Decision-Making and
Problem-Solving
ORIENTATION
● It is a rational and purposeful activity designed to
BASIC IDEAS
attain well-defined objectives. It is a means to an
What is Decision-Making? end. Decision-making is always purposeful; there
The word ‘decision’ is derived from the Latin words may even be a decision not to decide.
de ciso which mean cutting off or cutting away or to ● The aim of decision-making is to find out the best
come to a conclusion. An almost universal definition possible course of action. In order to identify the best
of decision-making is choosing between alternatives. alternative, it is necessary to evaluate all available
According to Gangadhar Rao and Surya P. Rao, alternatives.
“Decision-making is the process by which individuals ● It is an intellectual and rational process. It is the end
select a course of action from among alternatives to process preceded by reasoning and judgement.
produce a desired result.” T.N. Chhabra and R.K. Suri ● Decision-making involves a time dimension and a
@UPSC_THOUGHTS

define decision-making as “a process of selection from time lag.


a set of alternative courses of action which is thought ● It is always related to a situation. One may take one
to fulfil the objectives of the decision problem more decision in a particular situation and quite the
satisfactorily than others. It is a course of action which opposite decision in a different situation.
is consciously chosen for achieving a desired result... ● Decision-making in an organisation is a continuous
Managerial decision-making process involves process.
establishing of goals, defining tasks, searching for ● It is a pervasive function of management. Herbert A.
alternatives and developing plans in order to find the
Simon equates decision-making with management.
best answer to the decision problem.”
Thus, decision-making involves selection from Kinds of Decisions
among alternative courses of action. The possible courses Organisational decision may be classified on different
of action may be given to you (or be available as bases into the following categories:
procedural alternatives) or you may have to think them (i) Organisational and personal decisions Decisions
up in specific situations. The essence of decision-
taken by executives in their official capacity or on behalf
making is choice.
of the organisation are known as organisational
Characteristics of Decision-Making decisions. These decisions are aimed at furthering the
Decision-making involves the following characteristics: interests of the organisation and have a direct bearing
● The need for decision-making arises only when on the functioning of the organisation. For example,
alternatives exist for doing the work. When there is promotion of employees made by a senior officer is an
no choice of action, no decision is possible. organisational decision. Change in procedures to be
● The need for making any decision occurs when some followed is an organisational decision.
uncertainty as to outcome exists. Personal decisions are taken in one’s personal
● Decision-making is a human and social process. It capacity concerning one’s personal affairs. However,
not only involves intellectual abilities but also some personal decisions can have an effect on an
intuition, knowledge, experience, subjective values organisation. The personal decision to leave an
and judgement. organisation would also affect the organisation.

4.3
Decision-Making and Problem-Solving

(ii) Programmed and unprogrammed decisions (ii) Risk taking Decision-making often involves
Programmed decisions are routine and repetitive. They risks. Organisations (and individuals) generally want
are taken under the rules and policies framed by the to make least risky choices. Decisions are often avoided
organisation. For example, if an employee applies for because of the risks involved.
leave, the supervisor can decide the case according to (iii) Time pressure With the increase of time
the rules and regulations. pressure, the ability to examine and compare choice
Unprogrammed decisions are non-repetitive and alternatives is challenged, and the decision-making
novel. There are no pre-established policies or procedures process is modified.
to rely on and every decision is a unique case. Such (iv) Personal biases We cannot rule out personal
decisions involve judgement, intuition and creativity. biases in decision-making. There can be no neutral
An example is requisitioning additional supplies if the principle to determine the rationality of competing
existing stock in the store drops below the prescribed reasons. Each principle has its own reasons or kinds
level. of logic. So decisions are influenced by habits, reflexes,
(iii) Active and passive decisions To decide to do prejudices, emotions and appetites.
something is active decision-making whereas to decide (v) Experience Experience is knowledge which
‘not to decide’ is passive decision-making. Passive helps in decision-making. The greater the number of
decision, in fact, means to let the present situation successful decisions, greater the confidence one has to
continue without actually making a decision to that take more complex decisions. However, experience
effect. should not be followed blindly.
(iv) Strategic and routine decisions Strategic or (vi) Intuition Decision-making is also influenced by
basic decisions are concerned with policy matters which intuition and hunches. Intuition may provide the
@UPSC_THOUGHTS

may require huge investments. Such decisions influence essential direction for solving a problem in a certain
the objectives, facilities and structure of the organisation. way.
They are generally taken by top management or (vii) Seeking advice Taking advice can greatly
administrators. enlarge the potential of making a good decision.
The routine decisions are repetitive in nature and (viii) Age The cognitive resources decrease with
are concerned with short term commitments. They are age. On the other hand, experience and wisdom increase
generally taken by the lower level managers. They with the age.
translate policies into action. (ix) Limitation of resources Limited physical,
(v) Individual and group decisions Individual financial and human resources restrict the choice of
decisions are decisions taken by an individual or a selection.
single person in an organisation. Individual decisions
Decision-Making Processes
are usually taken in small organisations or organisations
with autocratic leadership. The decision-making process describes the elements of
Group decisions are collective decisions taken by decision-making. The process consists of a sequential
a group of members of the organisation. They are set of steps which vary between five and eight. These
usually practised in committee or board types of steps, in fact, represent a simplification of the process
organisations. They tend to be more balanced, acceptable and in actual decision situations, the orderly step-by-
and practical but involve greater expenditure of time, step process is rare. Several steps may go on
money and effort. simultaneously and some steps may be skipped or
repeated. In general, the various stages in the process
Factors Influencing Decision-Making of rational decision-making are as follows:
Decision-making is influenced by the fallowing factors: (i) Defining the problem A problem is a situation
(i) Fear of failure Fear of failure may block the or condition of people or the organisation that could
decision-making process. The worry about the unknown exist in the future, and that is considered undesirable
result of a decision sometimes paralyses the decision- by the person or organisation. The first step in decision-
making. making is to recognise, identify, determine and define

4.4
Decision-Making and Problem-Solving

the problem clearly. Accurate diagrosis or perception of contribution to the goal is selected. This is the point of
the problem is necessary to find the right solution. The culmination at which point a plan of action is adopted.
problem must be understood in relation to higher level The criteria for choosing the best alternative may involve
goals of the organisation. The definition of the problem past experience, experimentation, and research and
helps in collecting relevant data and in finding the analysis.
correct solution. (vi) Implementing the decision Taking a decision
(ii) Analysing the problem After being defined, the is of no use unless it is put into practice. The manager
problem must be analysed in terms of the nature, must ensure that systematic steps are taken to implement
impact, periodicity, future course or impact, etc. of the the decision. Implementation of the decision involves
decision. Analysis also involves enumeration of the development of detailed plan, communication of the
limiting or strategic factors relevant to the decision. By decision, gaining acceptance of the decision, getting
analysis of the problem we determine who should take support and cooperation of those concerned with
the decision, what information is required and how it implementation, and developing control to ensure that
can be gathered. A decision is as good as the information the decision is being carried out properly.
on which it is based. (vii) Evaluation of decision process The final step
(iii) Developing alternative solutions Despite time in decision-making is evaluation or follow-up of both
and cost constraints, an attempt should be made to the decision and the process of decision-making. If the
identify as many alternative courses of action as follow-up shows unsatisfactory results, the process
possible. The ability to develop a reasonable number of should be reviewed and the decision may be modified.
alternatives is often as important as making the right Herbert A. Simon, the well-known Nobel Prize
@UPSC_THOUGHTS

choice among alternatives. It requires considerable winning theorist on organisation and decision,
imagination experience and judgement. This is the conceptualised three major phases in the decision-
stage of investigation into problem. Brainstorming, making process:
synectics and nominal grouping are the techniques of (i) Intelligence activity Borrowed from the military
generating alternative courses of action. The alternative meaning of intelligence, it involves finding occasions
solutions also include the negative aspects of decision- for making a decision. The executives spend a large
making. portion of their time surveying the economic, technical,
(iv) Evaluating alternatives The alternative solutions political and social environment to identify new
are compared and evaluated in terms of their costs, conditions that call for new actions.
time, feasibility and contribution to objectives. Evaluation (ii) Design activity The design activity consists of
involves deliberation or measurement of the merits and inventing, developing and analysing possible course of
demerits of various alternatives. The primary objective action, that is, finding alternative courses of action. The
of evaluation is not to find one magic solution. The executives spend an even larger fraction of their time,
attempt is made chiefly to limit the alternatives to a individually or with their associates, seeking to invent,
manageable and economically feasible number. The design and develop possible courses of action for
general criteria for evaluating the probable consequences handling situation where a decision is needed.
of different alternatives include the risk involved in (iii) Choice activity The choice activity involves
each course of action; economy of time, money and selecting a particular course of action from the given
efforts; timing; and limitation of physical and human alternatives. The executives spend a small fraction of
resources. their time in choosing among alternative actions already
(v) Selecting the best alternative The selection of developed and analysed for their consequences, to meet
the best alternative is done when all the alternatives an identified problem.
have been enumerated and evaluated against the In general, intelligence activity precedes design,
decision criteria. Depending upon the priorities given and design activity precedes choice. Each phase in
to the various selections criteria, the best alternative is making a particular decision is, in itself, a complex
selected. The alternative which can make a net maximum decision-making process.

4.5
Decision-Making and Problem-Solving

Mintzberg and his colleagues, on a more empirical rationality as “the selection of preferred behaviour
basis, described three phases of decision-making as alternatives in terms of values whereby the consequences
follows: of behaviour can be evaluated”. According to Simon,
(i) The identification phase During identification the decision-making behaviour could best be described
phase recognition of a problem or opportunity arises, as follows.
and a diagnosis is made. It was found that severe, ● The decision-makers, in choosing between
immediate problems did not have a very systematic, alternatives, attempt to satisfice, or look for the one
extensive diagnosis but that mild problems did. which is satisfactory or ‘good enough’. Satisficing
(ii) The development phase During the development criteria would be adequate profit or share of the
phase there may be a search for existing standard market and fair price.
procedures or solutions already in place or the design ● The decision-makers recognise that the world they
of a new, tailormade solution. It was found that the perceive is a drastically simplified model of the real
design process was a grouping, trial-and-error process world. They are content with this simplification
in which the decision-makers had only a vague idea because they believe the real world is mostly empty
of the ideal solution. anyway.
(iii) The solution phase During the solution phase ● As they satisfice rather than maximise, they can
the choice of a solution is made. There are three ways make their choice without first determining all
of reaching this solution: (a) by the judgement of the possible behaviour alternatives and without
decision-maker, on the basis of experience or intuition ascertaining that these are, in fact, all the alternatives.
rather than logical analysis; (b) by analysis of the ● As they treat the world as rather empty, they are
@UPSC_THOUGHTS

alternatives on a logical, systematic basis; and (c) by able to make decisions with relatively simple rules
bargaining when the selection involves a group of of thumb or tricks of the trade or from force of habit.
decision-makers and all the political manoeuvring that These techniques do not make impossible demands
this entails. Once the decision is formally accepted, an upon their capacity for thought.
authorisation is made. In contrast to the rational economic model, Simon’s
model is rational and maximising but it is bounded.
Models of Decision-Making The decision-makers end up satisficing because they do
There are various models of or approaches to decision- not have the ability to maximise. The factors responsible
making in organisations. Decision-making in an for bounded rationality leading to satisficing decisions
organisation is always logical, unemotional and rational. are inadequate information as well as limited capacity
Different models of decision-making are as follows. to process the available information; time constraints;
(i) The rational economic model or economic man resources constraints; dynamic nature of organisational
model The economic man model, also called normative objectives; environ-mental forces or external factors;
decision theory, and rational comprehensive model personal factors of the decision-maker like habit, attitude,
assumes that the decision- maker is an economic being etc.; organisational factors like rules, procedures,
and makes decisions that would maximise his (or her) communication, etc.; lack of awareness of the decision-
advantage by searching and evaluating all possible makers of all the possible alternatives available and
alternatives. That alternative is selected which could their consequences; and impossibility of the alternatives
maximise the satisfaction of the decision-maker. being quantified in an ordered preference.
However, this model ignores the influence of powerful (iii) Judgemental heuristics and biases model This
individuals and groups like labour unions, consumer model takes the bounded rationality model one step
councils, government agencies, etc. farther. It identifies specific systematic biases that
(ii) Administrative man model or behavioural influence judgement. This model is mostly drawn from
theory Administrative man model is also called bounded cognitive decision theories which suggest that decision-
rationality model or descriptive decision theory. The makers rely on heuristics (simplifying strategies or rules
exponent of this theory, Herbert A. Simon, viewed of thumb). Such judgemental heuristics reduce the

4.6
Decision-Making and Problem-Solving

information demands on the decision-maker and diagnosed, classified and interpreted in arriving at a
realistically help in the following ways: decision.
● Summarise past experiences and provide an easy (ii) Experience Experience means practical
method to evaluate the present. knowledge. It is quite a common practice to fall back
● Save considerable mental activity and cognitive on experience while taking decisions because experience
processing. provides a familiar situation where a decision has been
Even though these cognitive heuristics simplify and taken in the past in similar circumstances. Thus,
help the decision-makers, their use can, under certain experience provides guidance in decision-making.
conditions, lead to errors and systematically biased (iii) Intuition Intuition means use of hunches, inner
outcomes. feelings or the gut feeling of the person taking decision.
(iv) Incremental model The incremental model Suggestions and influences and the psychological make-
recognises the practical problems in the rational up of the decision-maker play an important role in
comprehensive approach. It highlights various intuition-based decision-making. Persons who take up
limitations like money, time, information, politics, etc., decisions based on intuition generally lean heavily on
which govern the actual decision-making process in the instincts, on the personal feel for a situation, but they
administration. It argues that the decision-makers always never overlook realism in making decisions.
continue the existing programmes and policies with (iv) Considered opinion The use of considered
some additions. It has been argued that what actually opinion means use of logic in reaching the decision.
occurs in administrative decisions is ‘incrementalism’, The logic is derived from careful analysis of the
that is, a virtual continuation of the previous activities situation. Statistics are collected and related to the
@UPSC_THOUGHTS

with few modifications. The model assumes that the decision, which may be described as a quantification
past activities and experiences are used by the of tentative decision.
administrators to make future decisions. (v) Operating research Operating research involves
(v) Optimal model The optimal model was bringing together available data on a specific problem,
presented by Yelezkel Dror, who claimed that his processing these data, and from them resolving
model was superior to all the existing normative models quantitative reports on the relative merits of various
of decision-making and was a combination of potential courses of action. In operating research, the
economically rational model and extra-rational model. concepts of optimisation, input-output, and
The optimal model is a rationalist model of policy- mathematical models are used.
making and has five major characteristics: (vi) Linear programming Linear programming helps
● The model is economically rational. to determine the optimal combination of limited
● The model is concerned with meta policy-making. resources to solve problems and achieve organisational
● The model is qualitative and not quantitative. objectives. The problems suitable for linear programming
● The model contains both rational and extra-rational are maximising production output, minimising
elements. distribution cost, fixing optimal inventory levels, etc.
● The model contains a built-in feedback. (vii) Simulation Simulation is the imitation of some
real thing, state of affairs, or process. In simulation, it
Bases for Decision-Making
is possible to trace the activities as relationships and
The bases for decision-making vary from guess, at one variable change. It is basically a systematic trial-and-
extreme, to complex mathematical analysis on the other. error approach to complex problems and is not used
They depend upon the background, experience, for optimisation.
knowledge and resources of the decision-makers. (viii) Queuing Sometimes, there is loss to an
Different bases for decision-making are as follows. organisation because the employees, machines or
(i) Facts Facts are regarded as the most important materials are made to wait as the facilities to handle
basis for decision-making. A decision should be based them immediately are insufficient. The objective of
on adequate facts. However, facts must be carefully queuing is to minimise the loss. Queuing is related to

4.7
Decision-Making and Problem-Solving

the flow of machines, human resources, work processes, ● There should be separate groups to monitor the
communication and materials. environment, develop new technologies, and generate
(ix) Gaming Gaming is a kind of simulation. It is new ideas.
applied when the problem is concerned basically with ● To reduce the tendency to neglect gathering negative
the actions of competitors. The technique is used for long-term information, managers should solicit worst-
optimising the standards of action under competitive case scenarios as well as forecasts that include long-
conditions including the minimising of losses on the term costs.
actions to be taken. ● Checkpoints and limits should be worked in to every
(x) Monte Carlo Monte Carlo is a narrow form of plan.
simulation but it also includes probability factors. It is ● When limits are reached, it may be necessary to have
a trial-and-error means to see what would take place an outside, independent, or separate review of the
when certain events occur. The approach predicts what current plan.
will probably happen in actual events without analysing ● Judge people on the way they make decisions and
comparable existing events. not only on outcomes, especially when the outcomes
may not be under their control.
Group Decision-Making
● Shifting emphasis to the quality of the decision
The understanding of group dynamics has become process should reduce the need of the decision-
relevant to decision-making since individual decision- maker to appear consistent or successful when
making has largely given way to group decision- things are not going well.
making in today’s organisations. The schemes or rules ● Organisations may establish goals, incentives, and
which can predict the final outcome of group decision-
@UPSC_THOUGHTS

support systems that encourage experi-mentation


making are as follows. and taking risks.
(i) The majority-wins scheme In this commonly In addition to the above simple guidelines, group
used scheme, the group arrives at the decision that was decision techniques given below can also be used to
initially supported by the majority. This scheme guides help eliminate the dysfunctions of groups and help
decision-making most often when there is no objectively them make more effective decisions—
correct decision. (i) The Delphi technique is named after the oracle at
(ii) The truth-wins scheme Under this scheme the Delphi in ancient Greece. It has many variations, but
group comes to recognise that one approach is objectively generally it works as follows.
correct because more information is provided and (a) A group, usually of experts, is formed. The
opinions are discussed. members of the group are not in face-to-face interaction
(iii) The two-thirds majority scheme This scheme with one another. In this way, the expenses of bringing
is frequently adopted by juries, who tend to convict a group together are eliminated.
defendants when two-thirds of the jury initially favours (b) Each member of the group is asked to make
conviction. anonymous predictions or inputs into the problem
(vi) The first-shift rule Under the first-shift rule the decision the panel is charged with.
group tends to adopt the decision that reflects the first (c) Each member then receives composite feedback
shift in opinion expressed by any group member. from the inputs of the others. In some variations the
Avoiding Tendencies that Hinder Effective reasons are listed (anonymously), but mostly just a
Decision-Making Tendencies such as accepting the composite figure is used.
status quo affect group decision-making. The following (d) Another round of anonymous inputs are made
suggestions may be used to help reduce and combat on the basis of the feedback. These repetitions take place
such tendencies and to make more effective group for a pre-determined number of times or until the
decisions: composite feedback remains the same, which means
● Even if things are going well, decision-makers should everyone is sticking with his or her position.
be vigilant in examining alternatives. A major key to the success of the technique lies in

4.8
Decision-Making and Problem-Solving

its anonymity. In the traditional decision-making hat’. By mentally wearing and switching ‘hats’ one can
technique of interacting group, the experts may be more easily focus or redirect thoughts and conversation. Each
concerned with defending their vested positions than thinking hat is a different thinking style. The six
with making a good decision. thinking hats are as follows.
The Delphi technique can be applied to a wide (a) The white hat is cold, neutral and objective. It
variety of programme planning and decision problems focuses on known information. It implies looking at the
in any type of organisation. However, a major criticism information presented to you and seeing what you can
of this technique centres on its time consumption, cost learn from it.
and Ouija-board effect (much like the parlour game of (b) The yellow hat symbolises brightness and
that name, Delphi can claim no scientific basis or optimism. The optimistic viewpoint helps to see all the
support). benefits and values of a decision. In a gloomy situation,
(ii) The Nominal Group technique (NGT), closely the yellow hat helps one to keep going and explore the
related to the Delphi technique, has been used by social positives.
psychologists. A nominal group is simply a ‘paper (c) The black hat is probably the most powerful and
group’, i.e., group in name only as no verbal exchange useful of the hats. It is gloomy and negative. This
is allowed between members because a general viewpoint looks at things pessimistically, cautiously
conclusion is that interacting groups inhibit creativity. and defensively. It tries to see why ideas and approaches
The NGT consists of the following steps: might not work. It highlights the weak points in a plan
(a) Generation of ideas in writing silently. or course of action. It allows one to eliminate the
(b) Round-robin feedback from group members, obstacles, alter the approach, and prepare contingency
@UPSC_THOUGHTS

who record each idea in a terse phrase on a flip chart plans in case of failure of the original plan. The black
or blackboard. hat thinking makes plans tougher and more resilient
(c) Discussion of each recorded idea for clarification and helps the decision-maker to fully prepare to counter
and evaluation. future difficulties in advance.
(d) Individual voting on priority ideas, with a (d) The red hat signifies feelings, hunches and
group decision being mathematically derived through intuition in making decisions. The viewpoint also tries
rank ordering or rating. to take into account how other people will react
Evidence shows that NGT-led group come up with emotionally and tries to understand the intuitive
many more ideas than traditional interacting groups responses of the people.
and even slightly more than groups using the Delphi (e) The green hat viewpoint focuses an creativity,
technique. possibilities, alternatives and new ideas. It gives an
opportunity for the expression of new concepts and
Techniques of Decision-Making and new perceptions. It helps develop creative solutions to
Problem Solving a problem.
The different techniques of decision-making and problem (f) The blue hat is used to manage the thinking
solving help structure the factors involved in the process process. The chairman of a meeting generally wears this
of decision-making and problem solving. Some important hat for process control.
techniques are given below. The thinking hats may also be used in pairs; such
(i) Six thinking hats The six thinking hats is a as white and red, black and yellow, and green and blue.
parallel thinking process which helps people to be more In fact, most of the times one takes decisions, all the
productive, focused, and mindfully involved. The process hats come into use, one by one, or together.
helps one to understand the complexity of a decision, (ii) Pareto Analysis The Pareto analysis is also
and spot issues and opportunities which might known as 80/20 rule. It is a statistical technique in
otherwise pass unnoticed. This technique separates decision-making that is used for the selection of a
thinking into six clear functions and roles. Each thinking limited number of tasks that produce a significant
role is identified with a coloured symbolic ‘thinking overall effect. The Pareto principle is that by doing 20

4.9
Decision-Making and Problem-Solving

per cent of the work one can generate 80 per cent of (vi) Brain storming Brain storming is a technique
the benefit of doing the whole work. Also, in terms of for generating useful ideas through open, freewheeling
quality improvement, a majority of problems (80 per discussion among team members. It is aimed at
cent) are produced by a few key causes (20 per cent). expanding available alternatives, looking beyond
This analysis is also known as ‘the vital few and the obvious solutions, encouraging innovation, shifting
trivial many’. points of view, challenging tradition, reducing
The 80/20 rule can be applied to almost everything, inhibitions and tapping the team’s creative resources.
viz., 80 per cent of customer complaints arise from 20 After a brain storming session all ideas are recorded
per cent of one’s products or services; 80 per cent of and then the most fruitful alternatives are selected,
delays in schedule arise from 20 per cent of the possible either by having participants vote for the best ideas or
causes of the delays; 20 per cent of a system’s defects by reaching consensus through discussion. The top
cause 80 per cent of its problems; 20 per cent of one’s choices are discussed in detail.
workforce produces 80 per cent of the unit’s work or (vii) Starbursting Starbursting is a form of
revenue; and 20 per cent of one’s products or services brainstorming used to generate questions in a systematic
account for 80 per cent of the profit. comprehensive way. It helps one to understand all
There are many applications of the Pareto principle aspects and options more fully by generating questions
in quality control. The Pareto diagram, one of the key rather than answers. It can be used repeatedly, with
tools used in total quality control, is based on the Pareto further layers of questioning about the answers to the
principle. The value of the Pareto principle for a project initial set of questions.
manager is that it focuses on the 20 per cent of things (viii) Cost-Benefit Analysis Cost benefit analysis
@UPSC_THOUGHTS

that matter. is done to determine how well/poorly a planned action


(iii) Paired Comparison Analysis In paired will turn out. In this analysis positive factors are added
comparison analysis or ‘paired choice analysis’, a and negative factors are subtracted to determine a net
number of options are compared and the results are result. The net result indicates whether the planned
tallied to find an overall winner. Many plausible options action is advisable or not. In its simplest form, the
are listed. Each option is compared against each of the analysis is carried out using only financial costs and
other options to determine the preferred option in each financial benefits. However, in its more sophisticated
case. The results are tallied and the option with the form financial values are put on intangible costs and
highest score is the preferred option. The analysis is a benefits. But the analysis can be carried out only in
good way of weighing up the relative importance of terms of finance.
different courses of action, particularly where priorities (ix) Decision Tree Analysis A decision-making tree
are not clear, or are competing in importance. is a diagram that represents, in a specially organised
(iv) Cause-and-effect diagram The cause-and-effect way, the decisions, the main external or other events
diagram helps to identify all the possible factors causing that introduce uncertainty, as well as possible outcomes
a specific problem. The cause and effect relationships of all those decisions and events. The analysis provides
govern everything that happens and as such are paths a highly effective structure within which one can
of effective problem solving. It is also known as explore options, and investigate the possible outcomes
‘ishikawa’ or ‘fishbone’ diagram. of choosing those options. It also helps to form a
(v) Plus Minus Interesting (PMI) The PMI is also balanced picture of the risks and rewards associated
known as ‘the pros and cons of a decision’. It involves with each possible course of action. Decision trees are
considering the positive, negative and interesting or particularly helpful in situations of complex multistage
thought-provoking aspects of an idea or alternative decision problems.
using a balance sheet grid where plus and minus refer (x) Cash flow forecasting Cash flow forecasting is
to criteria identified in the step of analysing the problem. carried out on a spreadsheet package by which the
The PMI is a good technique to use to check that a impact of changing factors within the forecast is
selected course of action is worth taking. investigated. The forecasting helps to build a model of

4 . 10
Decision-Making and Problem-Solving

the way in which cash moves within a project or employees, clientele groups, and the public in general
organisation. It helps to predict whether the sales or will behave much as they have in the past.
income one forecasts will cover the costs of operation. (iii) Oversimplification The decision-maker tends
It also helps to analyse whether a project will be to simplify the problem and prefers a simple solution.
sufficiently profitable to justify the efforts invested For example, for many officials, the effective solution for
in it. riots is to suppress them and insist on law and order
(xii) Flow Chart A flow chart is an analytical tool rather than go into the causes and other factors and
used to identity problems. It illustrates the flow of an try to remove them.
activity, a process or a set of inter-related decisions or (iv) Over-reliance on one’s own experience In
communications from the beginning to the end. It may general decision-makers place great weight on their
be applied to a range of things from processing of a own previous experience and personal judgement.
tax return to the flow of materials in a manufacturing However, a person’s own experience may not be the
process. best guide. Shared decision-making produces wiser
Others Other tools of decision-making and problem- decisions.
solving worth mention are: (a) SWOT analysis for (v) Preconceived notions In many cases, decisions,
generating strategic alternatives from a situation analysis allegedly based on facts, in truth reflect the preconceived
applied to either the corporate level or the business unit ideas of the decision-maker. Decision-making is
level; it frequently appears in marketing plans; (b) dishonest when facts are doctored or distorted to justify
Histogram, a bar chart, mainly used to show the the decision. However, in some cases, officials are
capable of seeing only the facts that support their own
frequency of certain activities; (c) PEST analysis, a
biases. Anything else is ignored or considered not
@UPSC_THOUGHTS

scanner used to describe a framework for the analysis


credible and therefore does not qualify as a fact.
of political, economic, social and technological macro
(vi) Unwillingness to experiment The right way to
environmental factors; (d) Check sheet, used to compile,
determine the workability of proposals is to test them
summarise and track observations, interview results or
in practice on a limited basis. But for various reasons,
other data to translate opinions into facts; (e) Scatter
officials do not encourage such experimentation.
diagram, a tool for determing cause-and-effect
Pressure for immediate, large scale action often
relationship; and (f) APC (standing for alternatives,
convinces policy-makers that there is no time to proceed
possibilities and choice) tool, to select seemingly the
cautiously with pilot projects. At the same time, some
best solution by thinking alternatives or different
administrators are unimaginative and neither innovate
approaches.
nor encourage those who want to do so.
(vii) Reluctance to decide Even when adequate
Errors in Decision-Making
facts are available, some people try to avoid making a
Certain common errors in decision-making are as
decision. The personal experience of these people shows
follows. that decision-making is a burdensome task. In
(i) Cognitive nearsightedness There is a human government organisations red-tapism is common. Red
tendency to make decisions that satisfy immediate tapism refers to excessive formalities and routine
needs and to brush aside doubts of long-range wisdom. involved before official action is taken. However, passing
However, a tempting immediate solution may create the buck or the refusal to accept responsibility, is a
infinitely greater difficulties for the future. Related to practice that exists in private companies as well.
cognitive nearsightedness is the narrow view that
considers one aspect of a problem and neglects all Ethical Decision-Making
others. A set of standards that guides our behaviour, both as
(ii) Assumption that future will repeat the past In an individual and as a member of an organisation, is
making decisions, officials must forecast future called ethics. It refers to well-founded standards of right
conditions and events. However, in relatively stable and wrong that prescribe what humans ought to do,
periods of history, an assumption is made that usually in terms of rights, obligations, benefits to society,

4 . 11
Decision-Making and Problem-Solving

fairness or specific virtues. Ethical standards also enjoin ● Strive for fairness rather than polarised ‘winner takes
virtues of honesty, compassion and loyalty, and all’ outcome.
encompass standards relating to human rights, viz., the ● Learn from history and previous situations.
right to life, the right to freedom from injury, and the ● Get the facts from all possible perspectives.
right to privacy. It is also important to note that ethics ● Understand the long-term consequences.
is not the same as feelings; it is not religion; it is not ● Check the law.
necessarily following the law; it is not following ● Consult widely.
culturally accepted norms; and it is not science. ● Consider cause and effect in the deepest possible
Ethical decision-making in organisations has both sense.
negative and positive connotations. Negatively—the ● Resist the delusion and arrogance that power and
‘don’ts’, in other words—it means not to exceed one’s authority tend to foster.
authority and not to use one’s position to seek personal ● Beware of justifying decisions according to religious
gains in the form of soliciting gifts, making official faith or community customs.
decisions that benefits one financially, using inside ● Aim for solutions and harmony; facilitate rather than
information of the organisation to benefit oneself or influence decision.
one’s family and friends, using organisation’s time and ● Show transparency.
property for personal reasons, and using official position
to endorse a person or product. The positive meanings Ethics and Decision-Making in
of ethics include placing the law and ethical principles Public Administration
above private gain; acting impartially; protecting and In a position of responsibility, one may be confronted
@UPSC_THOUGHTS

conserving organisation’s property; and putting forth with complex situations in which it is difficult to decide
an honest effort in everything connected to one’s official what to do and how to act. There are contrasted values
position. or decisional premises that could apply in the situation.
Ethical decision-making in an organisation contains One has to face ethical dilemmas or what are called
three elements. ‘hard choices’. A dilemma is something wider and more
(i) Ethical commitment involves showing a strong demanding than a problem, however difficult or
desire to act ethically and the right thing, particularly complex the latter may be. Dilemmas, unlike problems,
when ethics imposes financial, social or psychological cannot be solved in the terms in which they are initially
costs. presented to the decision-maker. The decision-maker is
(ii) Ethical consciousness or awareness includes faced with opposed and perhaps equally unwelcome
seeing and understanding the ethical implications of alternatives; moreover, their incompatible juxtaposition
our behaviour and applying ethical values in our daily also implies that they are mutually exclusive in the
lives. sense that the satisfaction of the one can only be made
(iii) Ethical competency involves the skills in ethical if the other is sacrificed. This can hardly be effective.
decision-making, which include: (a) evaluation or an A dilemma may, however, be dealt with in a more
ability to collect and evaluate relevant facts and knowing effective and appropriate way if full account is taken
when to stop collecting facts and to make prudent and due respect paid to the warring value options,
decisions based on incomplete and ambiguous facts; (b) which are then ordered and linked among themselves
creativity or the capacity to develop resourceful means in a more systematic and coherent manner. Naturally,
of accomplishing goals in ways that minimise ethical
dilemmas abound in complex organisations, which fail
problems; and (c) prediction or the ability to foresee the
to tackle them effectively. As a result, state officials and
potential consequences of conduct.
civil servants exposed to acute dilemmas can hardly
Some principles for ethical decision-making are as help succumbing to a state of confusion and
follows: embarrassment in which they are often quite unwillingly
● Step back from every decision before making it and thrust. In circumstances like these, public administration,
look at it objectively. instead of functioning as a well-ordered state of

4 . 12
Decision-Making and Problem-Solving

legitimate purposes, degenerates into a state of confusion taken and due respect is paid to the four functionally
in which there is a lack of determination. It is then that associated imperatives: (i) the principle of democratic
ethical vagueness and lack of clarity about overall legitimacy and accountability of public bureaucracy
values to guide action and choices in ‘hard cases’ may and administration; (ii) the rule of law and the principle
come close to releasing a spirit of unbound relativism of legality, whereby law and only law should govern
if not cynicism. In such a situation, nothing can be the administration; (iii) the principle of meritocracy,
taken seriously – not the values, not the rights and professional integrity, autonomy and capacity of the
duties of public servants and citizens alike. administrative apparatus of the state; and (iv) the
In an effort to make some sense out of the multitude principle of responsiveness and responsibility of
of criteria that one way or another enter and frequent administration to civil society.
the organisational landscape of public administration, Modern political science and public administration
a set of ground rules have been distinguished which, draws a distinction between politics and administration.
first, classify in an orderly way basic administrative This distinction refers not only to their division of
dilemmas; and second, ought to be taken into account function and their structural separation but also to the
whenever one is engaged in the business of dealing subordination of the administration to the political
with them. dispensation. Thus, the primacy of politics in the
The fundamental principles or criteria that could politico-administrative nexus explains the ultimate
be said to govern the process of dealing with ethical political or rather governmental control of the
dilemmas in public administration are: administrative machinery of the state in a democracy.
(i) democratic accountability of administration; The loyalty of the bureaucracy to its political masters
(ii) the rule of law and the principle of legality; is grounded on the obligation of ministers in
@UPSC_THOUGHTS

(iii) professional integrity; and parliamentary democracies to be answerable and


(iv) responsiveness to civil society. responsible to the legislature (ministerial responsibility
This may perhaps be described as the ALIR model of to parliament). It is only by that means that the
imperatives of ethical reasoning in public administration representatives of the nation may hold the bureaucracy
(from the initials of its key notions). accountable to the will of the people and the general
The major determinants of administrative conduct interest.
in the public sector include: (i) the wider political As it is the ministers, and not the civil servants,
context within which administrative organisations who are accountable to parliament, it follows that the
operate, (ii) the system of law and the kind of legal order latter are obliged to execute the orders of the former even
that applies to it, (iii) the public employees, who if they disagree with their content, provided that they
constitute the staff of the public services and help originate from a legitimate source of authority in the
deliver their products to the public at large, and (iv) the institutional hierarchy, and that authority insists on the
variety of citizens and users of public services who in order being executed despite the objections put forward
the multitude of their associations form the particular by officials.
type of civil society. The subordination of civil servants to elected
It will be obvious that the ALIR imperatives of representatives who act as law-makers and policy-
ethical reasoning involves issues of good governance setters forms a precondition of democratic politics.
and the moral standards obtaining in it. Such a type Unless subordinated to political control, the bureaucracy
of governance and administration would have to be and administration usurps power, which does not
democratic, bound to the rule of law, capable of carrying belong to it; if that happens the bureaucracy (civil or
out its missions and tasks efficiently and effectively, military) enters the political arena, undermines
and responsive to civil society. The interdependence representative democracy and controls politics and
and interconnectedness among these major premises of government to its own interests and commands.
ethical reasoning in public administration suggests that Thus, a fundamental ethical duty of civil servants
dilemmas can only be dealt with in an effective and in pluralist parliamentary democracies is to subordinate
morally accepted way to the extent that full account is themselves to political authority. They owe a duty of

4 . 13
Decision-Making and Problem-Solving

loyalty to the duly elected or appointed political masters, has to be guided and determined by a system of rules
however transient they may be. In the same vein, they and laws. Their application in concrete cases by
would have to show a spirit of neutrality and discretion administrative or judicial authorities then forms the
in their official capacity as members of the administrative essence of the principle of legality and the rule of law.
infrastructure of the state with regard to partisan The control of legality of administrative action,
politics, besides controlling their own personal initially exercised by the administration itself and
preferences in the performance of their duties and ultimately by independent courts of justice, aims to
responsibilities. It should, however, also be stressed ensure, in particular, that proper procedures have been
that the supremacy of politics over the administration followed and observed, as well as that equity,
in the constitutional division of powers does not amount reasonableness and impartiality have been respected.
to and cannot be taken to mean the politicisation of The application of the principle of legality in
public services and the state bureaucracy, because that administrative performance would serve and promote
would then undermine the instrumental value of the the rule of law and the avoidance of abuse of power.
executive branch of government and administration. The idea that people should be governed by law
Administrators are not the clients or servants of rather than by fiat is basically relevant both to democracy
politicians in the partisan sense. Modern governance and modernity. Law could also be said to establish
and administration is founded on the idea that the what may be described as the minimum standard of
differentiation of roles and functions between politics morality. Usually, unethical conduct—be it bribery,
and administration increases the quality and potential theft, favouritism, or abuse of power—is a violation of
of both of them. In the circumstances, ‘speaking truth law, that is, putting someone above or beyond the law.
to power’ can be considered as a vital ingredient of Thus, consistent and fair enforcement of law can be a
@UPSC_THOUGHTS

professional ethics and moral integrity of civil servants first priority of an ethics reform strategy. However, it
and the administrative machinery of the state in general. is also a necessity that the people accept the sovereignty
The rule of law has a relevance of a very specific of law.
and significant kind to administrative conduct. Respect In order to carry out their functions, public agencies
for and adherence to the principle of legality are the employ personnel having the status of public officers
manifestations of a spirit of constitutionalism and who are governed by special provisions for recruitment,
forms an essential prerequisite for the legitimacy of state career, discipline and control. Professional integrity
action and the exercise of authority. and autonomy—the ‘professional virtue’, as one may
Respect for and application of the principle of call it—qualifying absolute hierarchical subordination
legality entails a particular type of control on entails that public administration may be brought
administrative action that aims to see that public under political guidance and control but its staff is
administration operates within the context of the law recruited and serves under the authority of law and in
established by the legislature (parliament). Since the the public interest, and not on the basis of partisan
source of all power is ultimately the people, according favouritism.
to the fundamental constitutional principle of popular Civil servants are supposed to be fully competent
sovereignty that is enshrined in most democracies on the basis of their ascertained knowledge, experience
nowadays, it follows that all power must be exercised and expertise, and independent enough to offer official
in the name of and in the general interest of the people. advice to ministers as well as to implement public
Consequently, government and administration alike as policies and decisions in an efficient and effective
the executive branch of the state and indirectly of the manner in the public interest.
nation itself have to respect the law, which is the It is necessary that recruitment should take place
expression of the will of the nation. The state is, on the basis of merit, and not be subject to spoils system
therefore, no more than the mechanism which the practices (favouritism, patronage). This is an important
nation sets up for its own governance and precondition of professional integrity and autonomy.
administration. If governance is to be effective rather There should also be a special set of rights and
than arbitrary, the running of the business of the state obligations promoting loyalty and integrity, including

4 . 14
Decision-Making and Problem-Solving

neutrality of practice, namely, a readiness to serve an enabling environment for sustainable human and
different legitimate governments regardless of one’s social development. Responsiveness is not restricted to
personal preferences or choices. market forces but also and even primarily concerns
Professionalism in public service could then be citizens’ participation in all levels and layers of
seen to accrue from a combination of knowledge, government, as well as empowering people in human
expertise, judgement and conduct in accordance to communities. Responsiveness also entails consultation
standards, as well as commitment to the field. in governance. In this context, civil society not only
Competence on the basis of knowledge, experience and furnishes the state with needs and demands of an
expertise is in all established professions (doctors, individual nature, but also with valuable sources of
lawyers, engineers, accountants, journalists, academics, information, feedback criticism and evaluation of
etc.). There is also a set of shared values held by peers performance.
on how to use and apply this knowledge. In other The imperative for responsiveness to civil society
words, knowledge and expertise have to be used with calls for an increased awareness and readiness to adapt
certain standards defining professional ethics such as, to changing values and conditions in society at large,
for instance, avoiding corruption in the delivery of and stresses the need for an overall ‘external’ or societal
services. accountability of state authorities and administration.
The imperative for integrity constitutes a source of Indeed, the notions of responsiveness and responsibility
internal self-control in administrative conduct based on are closely related to each other, and both to the notion
ethical standards and criteria shared and respected by of accountability. The bottom line is an obligation and
the corps of professional administrators. Avoiding, for a readiness on the part of the civil service to sincerely
instance, corruption and exhibiting integrity would be explain and justify its action in the public interest.
@UPSC_THOUGHTS

for them a matter of personal and professional honour Furthermore, it should consistently act in such a way
and prestige in a culture of ethics, and not simply an that the interests are well served and looked after by
externally imposed obligation. the administrators. Thus, new public management
Hegel considered public functionaries to be the emerges as an alternative to old bureaucratic
servants of the state only, and not of civil society, since administration.
it was the former that expressed the general will of the In our highly organised societies, often enough
people. Civil society, to Hegel, was a conglomerate of moral and ethical standards enter more as constraints
particular and inconsistent wishes and interests. to be taken into consideration than as legitimate
Whereas the state served the general interest, civil objectives. Thus they tend to be overlooked or even
society was the state of partial interests. Hegel’s theory ignored in the actual decision-making process. In other
needs to be understood in the context of the fact that words, ends often matter much more than means. As
he lived in a country where most citizens were simply a result, administration and management become
‘subjects’, without participation in the work of divorced from ethics and morals.
government, and where, therefore, a political life and Of course, conflicting demands may make
tradition was almost wholly lacking. Now, almost 200 administration sound like a paradox; indeed, each one
years later, we are aware that, unless there is an of the ALIR imperatives of ethical reasoning taken to
articulate and independent civil society, no political life its extreme would, far from being a virtue, turn into a
and even less democratic polity is likely to grow and vice. The basic aim would therefore be not the triumph
flourish. That is why civil society has been declared as of one principle or ethical imperative over the other, but
one of the most essential preconditions of liberty and rather the reduction of incompatibility among them and
democracy. The view that now prevails is that the state the provision of conditions for their harmonious
should neither command civil society nor be subservient coexistence, mutual support and complementary
to it. The ‘civic virtue’ of ethical reasoning in state fulfilment. Turning mutually exclusive dilemmas into
action entails that public institutions be responsive to solvable problems would then require a holistic and
society and pay attention to the needs and demands reflexive approach to ethical reasoning. Here ethics has
of the people, facilitating access to services and creating been conceived as a body of norms and values that

4 . 15
Decision-Making and Problem-Solving

guide official conduct in the public services. Their consider the risks and results of various alternatives
consistent application is more than a matter of sanction, before making a final decision.
an issue of acculturation of ethics and the respective
Tips for Effective Decision-Making
socialisation of civil servants. It signifies a shift in focus
Some tips to enhance the effectiveness of one’s decision
from external to internalised control and standards, as
are as follows.
well as an enrichment of the normative content of
● Do not let the decisions accumulate. Make decisions
administrative behaviour. One will then choose not to
as you go along. A backlog of many little decisions
be unethical in action because one is by character
sometimes become harder to deal with.
ethical, and not only because there are laws or rules
● Do not make decisions that are not yours to make.
forbidding certain actions.
● Keep in mind those affected by your decision and

Characteristics of Effective Decision-Makers get them involved to increase their commitment, if


possible.
Effective decision-makers need to have the following
● It is not possible to know with hundred per cent
characteristics:
certainty that your decision is correct as it would be
(i) Advice-seeking Effective decision-makers know
implemented only in the future.
that they need help from others. They identify people
● A decision-maker must have the luxury of having
who can make specific contributions to the decision-
the right to be wrong.
making process and ask them for their advice and
● Trust yourself to make a decision.
counsel.
● A decision should always be made at the lowest
(ii) Selectivity Effective decision-makers seek
possible level and as close to the scene of action as
pertinent data and avoid getting bogged down by
@UPSC_THOUGHTS

possible.
extraneous facts and figures. ● A decision should always be made at a level ensuring
(iii) Currency Effective decision-makers consider that all activities and objectives affected are fully
current conditions and take advantage of opportunities considered.
that exist at the time. ● Not making a decision is a decision not to take
(iv) Knowledge Effective decision-makers have deep action.
understanding of all relevant facts to make a decision. ● Making decisions that do not have to be made is
The effectiveness of the decision depends on how wastage of time.
informed a decision-maker is. ● Before gathering data to make a decision, determine
(v) Initiative Effective decision-makers assume alternative courses of action.
responsibility for starting the decision-making process ● Assess the risk involved in the best choice before
and seeing it through. They actively participate in implementing it.
making things better. ● Consider always how the decision is to be
(vi) Good judgement Effective decision-makers must implemented as part of your decision-making process.
exercise their best judgement in considering factors ● Do not continue prolonged deliberation about your
particular to the situation. decision. Make it and carry it through.
(vii) Comprehensiveness Effective decision-makers ● Remember, making decisions is simply choosing
need to look at all available options and consider every from among alternatives and is not always making
possible alternative so as to make the best choice. a choice between right and wrong.
(viii) Flexibility Effective decision-makers remain ● Do not make snap decisions.
open-minded about new concepts and ideas. They are ● Make a decision while you still have time. Choosing
willing to change course or try a different approach if the right alternative at the wrong time is not any
better results seem likely. better than choosing the wrong alternative at the
(ix) Self-knowledge Effective decision-makers know right time.
their own abilities, biases and limitations. ● Make your decision on paper. Make notes to keep
(x) Calculated risk-taking Effective decision-makers your ideas visible in order to consider all the relevant
take calculated and moderate risks. They carefully information in making a decision.

4 . 16
Decision-Making and Problem-Solving

● Make decision based on what is right, not who is (i) Rational problem solving Rational thinking
right. includes the perception of sensation, learning, reasoning
● Pen down the pros and cons of a line of action to and memory.
clarify your thinking. In rational problem solving, problems are identified
● Once a decision is made and its implementation by (a) comparing actual performance with an expected
started, removing all doubts, do it with commitment. standard performance; (b) presenting a precise and
complete description of the problem to identify a
Problem-Solving and Decision-Making
solution; (c) identifying the cause of the problem
Compared
by comparing problem and non-problem situations;
All decisions are about problems, and problems may (d) noting that recent problems may be a result of some
be considered at three levels. A problem of macro context change in the situation that has caused an unwanted
would involve global issues (exchange rates, for deviation from expectations.
example), national concerns (the cultural orientations The steps followed in rational problem-solving
toward decision processes of different countries), and are—(a) define the problem; (b) identify the decision
provincial and state laws and cultures within nations. criteria; (c) weigh the criteria to determine rank of
In the meso context, the problem relates to organizational
importance; (d) generate possible alternative solutions;
cultures and structure. In the micro context, problems
(e) rate each alternative on each criteria; and (f) compute
relate to the immediate decision environment—the
the optimal decision.
organization’s employees, board, or office.
(ii) Emotional problem solving Emotional thinking
Problem-solving is a mental process. It is a complex
takes place in the limbic system which is associated
intellectual function. Good problem-solving skills
with basic needs and emotions, i.e., hunger, pain,
@UPSC_THOUGHTS

include development of creative and innovative


pleasure, satisfaction, sex, etc. Emotional thinking is
solutions, development of practical solutions, application
mostly clear-cut, black and white, all or nothing.
of a range of strategies to problem-solving, independence
(iii) Creative problem solving Creative problem-
and initiative in identifying problems and solving them,
solving is a subconscious process based on past distilled
and application of problem-solving strategies across a
experience. It is based more on the gut feeling of the
range of areas.
Problem-solving, sometimes referred to as problem decision-maker than on an objective process of weighing
management, may be divided into two parts—process alternatives. Intuitive method is preferred when a high
and decision. The process of problem-solving is level of uncertainty exists, variables are not reliably
predicated on the existence of a system designed to predictable and there are several plausible solutions.
address issues as they crop up. The second part of the Problem-Solving Process
problem management equation is the decision, or choice.
There are many problem-solving processes, each one
Thus, the relationship between decision-making and
consisting of steps. A common seven-step problem
problem-solving is strong. Both attempt to make
solving process is as follows.
organisations work more effectively and utilise similar
(i) Identifying the problem Sizing up the situation
models. Problem-solving also has the characteristic of
to identify the problem is the first step in the problem
making a choice (decision-making). An additional
solving process. It is a simple process, but in a complex
similarity between the two is that any decision made
or problem solved must enhance the organisation’s situation one may ask one’s friends or a professional
ability to meet its objectives. In fact, problem-solving expert and may use techniques like comparison with
requires a series of decisions as people choose from and others, monitoring for weak signals, brain-storming,
evaluate alternatives to overcome the obstacles that checklists, comparison of current performance with
stand between them and a satisfactory resolution. objectives or past performance, role playing and listing
complaints.
Models of Problem Solving (ii) Exploring the problem The problem is analysed
In general three approaches to problem-solving are to see the root cause of it without allowing oneself to
used in an organisation. be affected by emotional issues.

4 . 17
Decision-Making and Problem-Solving

(iii) Set goals One sets goals in a written statement ‘groupthink’ in which the discussion leads to a
which focuses on what is the successful end of the significant shift towards greater risk. The decision of
process. The goal statement helps one to clarify the the group reflects the dominant decision-making norm.
direction to take in solving the problem and gives one (iv) Conformation bias Conformation bias is the
something definite to focus on. tendency of problem-solvers to look for only that
(iv) Look for alternatives One begins to develop information which supports their perceived notions
possible solutions. In this creative as well as practical and not to search for information objectively.
step every possible solution is identified. To identify (v) Fixation Fixation means inability of problem
various possible solutions different techniques may be solvers to see a problem from a fresh perspective.
used including analysis of past solutions, reading, (vi) Insufficiency of hypotheses Insufficiency of
researching, thinking, asking questions, discussing, hypothesis reflects insufficient thought. The problem-
viewing problem with fresh eyes, brain storming, solver seizes upon the first explanation that comes to
sleeping on it, and mind mapping. mind and stops thinking about the problem.
(v) Select the best solution One prioritises the (vii) Other causes Limited organisational capacity;
solutions by their effectiveness, and narrows down the the costliness of analysis; interdependence between
choices to one best possible solution expected to give facts and value; decision avoidance psychosis; and
the optimum outcomes. decision randomness.
(vi) Implementation Implementation is a crucial
step in the problem-solving process. One prepares an
QUESTION TYPES
action plan in order to implement the best solution
chosen. The action plan is communicated to those who Few competitive examinations, if any, have this subject
@UPSC_THOUGHTS

are directly or indirectly affected. for the objective type test. Some management institutions
(vii) Evolution In this final step in the problem do include questions on decision-making, but for the
solving process, one reviews the effectiveness of the civil services, you may be tested for a different kind of
solution against desired outcomes. This stage requires decision-making ability. In the corporate set-up, many
careful analysis that improves upon the best solution. decisions have just the economic or manufacturing
Evaluation may lead one to revise the steps or add new dimensions to be considered. A civil servant is likely
steps. There may be a need to consider a different to face at least some problems that would be very
solution, if the current one is not producing the desired different from those faced by a person in a corporate
result. organisation, though some basic elements of decision-
making would remain the same. The civil servant works
Causes of Poor Problem-Solving with a larger public and a more diverse one; he or she
There are various factors that make problem-solving has to work within certain parameters which are
ineffective. Some of them are as follows. somewhat different from those that govern corporate
(i) Bounded rationality People are only partly managers. While credibility and trust have to be built
rational, and are in fact emotional or irrational in the up in all sectors by those who have to take and
remaining part of their actions. Two major causes of implement decisions, the civil servant has to build these
bounded rationality are limitations of the human mind, up on a larger canvas—with people whom he or she
and the structure within which the mind operates. might not know personally and whose welfare is very
People identify and implement solutions that are much his or her concern. It is very important for anyone
satisficing (combination of words, ‘satisfaction’ and aspiring to the civil services in a country like India to
‘sufficing’) and which do not necessarily lead to develop a broad tolerance for diversity, an unbiased
optimum solution. attitude as far as community, gender, and caste are
(ii) Groupthink Group think is a phenomenon in concerned, a readiness to accept variety of customs and
which the norm for consensus overrides the realistic practices and suspend judgement unless something
appraisal of alternative courses of action. violates the rule of law or the legal framework in which
(ii) Groupshift Groupshift is a special case of government functions. However, even when such

4 . 18
Decision-Making and Problem-Solving

violations take place, the civil servant cannot be in the form of a decision, followed by two other
insensitive; the situation has to be assessed and the statements which are termed arguments. You will be
fundamental reasons for restlessness or resentment asked to assess which of the arguments is strong or
among people probed even as the law is allowed to act, weak for the particular decision. A strong argument
and the findings conveyed to those who have the power is one that leaves you in no doubt that it can be the
and mandate to improve things. basis upon which a decision can be taken. It is
There are situations in which an official has no possible that an argument may have a negative
discretion to change or circumvent procedures, which connotation and yet be a strong argument.
may have to be followed, but the official can, indeed ● A situation may be presented to you, in which you
should, take some initiative to help people out if they have to choose a course of action; in other words,
are genuinely facing problems. While the civil servant decide what you must do to tackle the given situation.
is expected to act within certain well-defined parameters, If the courses of action involve unfairness or injustice
if he or she finds that people in authority are subverting even though they may be efficacious in solving the
the law or doing something inimical to people’s welfare, problem, you should reject that course of action as
he or she should take appropriate steps to counter such a responsible and sensitive person with values.
acts by bringing them to the notice of regulatory Humanitarian considerations, as well as social and
institutions or other authorities who are equipped to national goals should guide you in choosing the
take action. Acting without fear or favour is part of a option. Thus, the issue may be how to control the
civil servant’s duty. population growth in India. The courses of action
The questions in this section will be designed to may be (i) see to it that a one-child norm is
test your ability to analyse problems and take decisions promulgated and strictly implement it, and (ii) offer
@UPSC_THOUGHTS

that mark you out as possessing the potential required incentives for adopting family planning methods. If
for the civil services. you have sensitivity to people’s feelings and the
● Questions may require you to make decisions based socio-economic and cultural ethos of the country,
on a set of rules or restrictions. This is elementary besides an ingrained sense of democracy, you would
problem solving. Such questions would give a set of adopt the latter course of action, though the first one
basic conditions that are required to be met by a may get quicker results.
candidate, for instance, to be recruited to a certain ● Then there are ‘situation reaction’ questions. In these
position or be eligible to be promoted. There would questions, you are given a situation followed by
be a set of sub-conditions for those candidates who alternative answer options which offer you a choice
lack one or more of the conditions stipulated before. of actions that can be adopted. In many such
You need to consider the conditions carefully and questions, it is rather easy to spot the ‘politically
decide whether a candidate passes the first hurdles, correct’ response. And, after all, that is what you are
or can be adjusted with the help of the second set required to do. If you know what is the right thing
of conditions, or has to be rejected. Or the question to do, you may end up doing it in real life. However,
may pose a number of constraints—budget, number many of these questions have two responses that can
of persons, deadline—and ask you to choose the be easily rejected, but another two which make
response that provides optimum results. It may help choice difficult. Neither of these options is, sometimes,
if you make a chart/table and fill in the data to get wholly ‘wrong’. It is in these cases that your true
the answer fast and accurately. In these questions, aptitude and attitude tend to get exposed. You may
your ability to apply rules and follow procedures is as well accept that these questions cannot be
tested. answered by ‘rote’ or by mere ‘practice’: you must
● Decision-making often requires you to check whether actually develop the traits that will make you choose
the reasons or arguments put forward for particular the correct option in the given situation. Once again,
decisions are good enough or not acceptable. The your integrity, honesty, quickness to think rightly in
arguments have to be strong if the decision has to emergencies, your humanitarian concerns, your sense
work. So, there may be questions with a statement of social responsibilities, your sense of fair play and

4 . 19
Decision-Making and Problem-Solving

justice come into the picture. In many cases it may ● Somewhat related to the situation reaction questions,
not do to be dishonest to yourself, either. The are the situational judgement questions. The former
practicality of an action/response too has to be have a greater emphasis on immediate response—
taken into account. Sometimes, it is unavoidable to rather instinctive and quick; the latter test the
compromise with principles, and a good decision- judgement required for solving problems in work-
taker/problem solver is aware of such situations. If related situations. These questions present you with
you are asked what you would do on seeing an hypothetical and challenging situations that one
unarmed person being attacked by two others or a might encounter at work, and that involve working
woman being assaulted by some youths, you may with others as part of a team, interacting with others,
honestly reply that you would intervene to help the and dealing with workplace dilemmas. In response
victims, even at some cost to your own safety, if you to each situation, you are presented with a set of
are to live up to high principles of fighting against possible actions that one might take in dealing with
crime and injustice. You might even have to be ready the problem described. Often enough these questions
to break the law in the process of self-defence. test the ‘relative’ effectiveness of your responses
However, if you are asked what you will do when rather than whether it is simply ‘right’ or ‘wrong’.
confronted by ten armed ‘goondas’ demanding your These tests are considered to be valid predictors of
valuables, would it reflect your bravery (which is an job performance and are less susceptible to adverse
admirable principle) or foolhardiness if you choose bias such as cultural differences than other traditional
the option that you will refuse and offer them a fight methods of assessment. They do not test academic
to death? Even if you are well-versed in martial arts, knowledge.
you would have to be an old-time Hindi film hero
@UPSC_THOUGHTS

to succeed in the circumstances. It might be more Some of the qualities probed by these types of
practical and wiser in the circumstances to hand questions would include:
● Ability to evaluate information and make judgment
over the valuables and report to the police or to run
away swiftly if you get the chance to do so and then decisions
● Adaptability and ability to work with unfamiliar
report to the police.
There may be a question on what you would do if and changing situations
● Ability to prioritise, plan and organise
you see an injured person lying in a pool of blood
● Professional integrity and moral judgement: : this
at an accident site. Would you rush him/her to
hospital, call the police and ambulance, or just go involves the courage to uphold truth and justice as
your way? Clearly, the last option is least acceptable well as acting according to rules and within the law.
● Ability to withstand undue influence as you conduct
because it shows a lack of a sense of duty and
compassion as a human being. The other two options your duties.
● Readiness to uphold transparency in public life.
show that you would do something to help another
● Ability to relate effectively with others—which
person. But which is the better choice? It is difficult
to say. One might opt to take the person to hospital includes having an empathy for the common people
(thus showing that one is an exemplary citizen with whom you ought to be both sincere and
ready to go out of one’s way and spend the time to responsible.
do one’s duty by fellow human beings). However, You will notice that the last has elements of
if the person is seriously wounded, medical advice interpersonal skills as well.
always says one must allow experts to move the A frequent question in many of these tests is what
patient. So it really depends on the severity of the you will do if you see a colleague stealing office
injuries and your own expertise. If there is an option material—stationery, etc. Would you immediately report
that you would check the person’s state and then him to the boss? Would you gang up on him by telling
decide what to do, that would be the most sensible everyone in the department and putting a social ban
course of action – showing your sense of duty as on him? Would you confront him and ask for an
well as responsibility. explanation? Would you just keep quiet and mind your

4 . 20
Decision-Making and Problem-Solving

own business? Only two options appear acceptable: action can be taken. But your scope of action is limited
either tell the boss or confront the colleague, and of by the fact that there are procedures laid down and
those two, the more sensible one would be to confront your senior officer expects you to adhere to procedures.
the colleague and ask for an explanation. This might You need to act in such a way that procedural
well be enough to deter him from committing theft formalities are not ignored even as the old lady does
again. Why this option first? Because he is your not get her pension claim rejected. Your options are
colleague—give him a chance to mend his ways, even restricted to the answer responses given.
as you do not allow the illegal activity of pilferage of If you simply adhere to procedure as response (a)
office property go unchecked. The second option makes suggests, the poor old lady is not likely to get her much
you out to be aggressive and the last makes you out needed pension, though you satisfy your superior
to be ineffective and unethical besides showing disloyalty officer’s procedural demands and your own duty to
to your organisation. Or you may be asked to rank the uphold rules. Response (b) gives you the chance to do
responses in order of effectiveness: both – help an old lady in a way that will serve her
(a) Gather more evidence and catch him ‘red not just for the present occasion but in the future too
handed’ again when such documents are needed, even as you are
(b) Confront your colleague and ask him about faithful to procedures. It is the best response. Response
what you have noticed. (c) would help the lady financially, but would hurt her
(c) Don’t do anything; if guilty he will be caught. dignity and self-worth; she is not a beggar, she is
(d) Privately ask some of your colleagues if they entitled to her claim of pension. It shows that you do
have noticed anything. not respect human dignity; besides, you are taking the
easy way out so that you do not have to go out of your
@UPSC_THOUGHTS

The best order would be: (b), (a), (d) and (c).
way to help or do extra work. Response (d) would be
Some Questions Analysed acceptable only if you have the discretion as an official
(Some of these questions are based on the samples given to overlook procedures, or your senior officer allows it
by the UPSC and the questions that were in the 2011 (again if he/she has the discretion to waive rules). You
Civil Services Preliminary Examination test.) must follow procedural rules.
Q.1. An old sick lady is unable to submit her life Q.2. You have a house in City A which you had left
certificate and claim pension to meet her treatment vacant for some time as you were working in
expenses. Your superior has a strict procedural another city. You have now been transferred back
approach in handling issues. In the circumstances, to City A and, on reaching your house, you are
you… shocked to see it occupied by some people who
(a) would adhere to the procedures as your turn out to be local goondas. On confronting them,
superior officer demands it you are given the ultimatum of either selling the
(b) take initiative to help the lady to arrange for house at a very low price to them or face dangerous
alternative documents that would enable you consequences. What would you do?
to process her claim (a) Sell the house as asked
(c) assist the lady with some money on your own (b) Place a complaint with the police and wait
but would not compromise on procedures. for their action
(d) overlook some procedural steps since the old (c) Negotiate with the goondas to get a higher
lady is in dire need of her pension claim being price
settled. (d) Locate a rival gang of goondas and take their
Analysis As this is a multiple choice question, the help in evicting the illegal occupants of your
question head has stated the problem. The problem is house.
that an old sick lady needs her pension claim to be Analysis The problem has been stated in the
settled, but she has either lost or does not have the question head. The decision options have been given
relevant documents which she must submit before to you. The situation is one in which an illegal action

4 . 21
Decision-Making and Problem-Solving

and injustice have to be faced and countered. Your Analysis The problem is: a woman is being sexually
action needs to be within legal parameters as well as harassed and the culprit is an officer in charge of a
designed to solve the problem of getting the house project that is at a crucial stage of completion. Of course,
vacated. If you are a law abiding person and want to if action is to be taken against the culprit, that person’s
fight injustice in the ethical way, the option is to report attention is likely to be diverted, and the project may
to the police—option (b)—even though the process may suffer. If the project is very important, you may be
cause some delay and trouble (number of visits to the tempted to accept response (a) as an easy way out. But
police station, threats from the goondas, etc.) to you. the option violates principles of justice. It lets the
Option (a) shows that you are not ready to put up a offender go free and makes the victim suffer. As for
fight against injustice and illegal actions and would averting scandal, the woman may proceed legally against
rather take the easiest way out - that of ‘no resistance’. the entire department and there would be even greater
Option (c) is somewhat better than option (a) as you scandal. So the decision would hardly solve the problem
are at least trying to get something out of a bad but may in fact lead to a greater problem. Option (b)
situation, but it still indicates that you are more ready is an evasion of duty and of escapist nature. If the man
to fly at the sign of opposition and do not bother about continues to behave as he did in the past, the woman
the larger issues of illegality and injustice. Option (d) suffers for no fault of hers. This is the kind of expediency
may work, i.e., achieve your end of getting the goondas that revolts against fair play and justice. You need to
to leave your house and get a tit-for-tat kind of brute remember that sexual harassment of a co-worker is a
justice, but it is as illegal as what you are trying to
crime punishable in law. Response (c) is little better
fight. The means of achieving an end should also be
than (b); in fact it compounds your lapse by adding
good.
emotional blackmail to it. The only option is (d): you
@UPSC_THOUGHTS

Q.3. You are a senior administrative officer in charge must let the law take its course, and not allow the
of a department. An officer heading the team for importance of the project cloud your vision of justice.
implementation of a project, which is at an
Q.4. A member of your team working on an important
important phase, has been accused by a co-worker
project comes to you as head of the team, asking
of sexual harassment. The victim is now at the
for a week’s leave to attend the marriage of his
end of her patience and tells you that either
only sister in a fortnight’s time in their native
suitable action should be taken against the culprit
town. The date of the wedding has been fixed
or she would resign. She also has video phone
rather suddenly. However, you are aware that his
recordings to prove what she has charged, and
other colleagues are ready to corroborate her absence will cause a setback to the project which
complaint. What would you do? needs to be completed in another two months.
(a) Accept the resignation as the project is of Which of the following options would you choose
great public interest, and this also would to settle the problem?
ensure that an unsavoury scandal will be (a) The application for leave should be rejected
averted as the work to which he owes allegiance
(b) Tell the affected woman that you will comes first, and not a sister’s marriage
reprimand the culprit but put it off till the (b) The person should be told to find someone
project work is completed from among the team members who will
(c) Explain the importance of the project to the shoulder his work if he wants leave
affected woman and appeal to her loyalty to (c) Leave should be granted unconditionally or
the department and request her to postpone the applicant should be referred to a senior
her official complaint till after the completion officer to ask leave
of the project (d) Leave should be granted on the condition that
(d) Ask the affected woman to file a written the person puts in some extra hours at work
complaint so that you can call the accused before he goes so that a large portion of his
official to explain his conduct work is completed

4 . 22
Decision-Making and Problem-Solving

Analysis The problem involves a conflict between (c) allow the demonstrations but beef up the
personal and organisational needs. As team head, you security and ask the security personnel to be
must keep both needs in mind while coming to a extra alert and ready to prevent any breach
decision. Option (a) takes only the need of the of law and order
organisation into consideration, and leaves out the (d) try to talk the leaders of the groups into
employee’s needs; after all it is his sister’s wedding he presenting to the visiting minister written
wants to attend and he cannot help it of the date has petitions on their demands and unhappiness
been suddenly fixed. Option (b) is not a bad choice, as with the government policies instead of
it solves the problem – keeps both organisational and holding demonstrations
employee’s needs in mind. Option (c) shows that you
Analysis You are, of course, a government servant
do not like to take hard decisions or that you evade
and thus obliged to make sure that law and order is
responsibility and are unable to make decisions: as
not disturbed. However, in a democratic country,
team head, you should be able to decide what to do
protests and demonstrations against official policies are
in the present instance, though granting leave
not only allowed as the rights of people but they often
unconditionally when the project may be delayed
provide important feedback to government. The views
because of a team member’s absence is hardly a good
of local party people must not influence your decision.
decision. Option (d) is also acceptable; indeed, if
As such, you must try to avert trouble without banning
compared to (b), the other good option, (d) is better. Here
you give the chance to the employee concerned a chance peaceful protests or making preventive arrests. So option
to show his dedication to the project and to the (a) is not really an option at all in the circumstances.
organisation; it saves him and you the trouble of Option (b) means that you are allowing personal views
@UPSC_THOUGHTS

explaining all the ins and outs of what is left to be done and biases colour your official decision—something
to another team member (who may not be too happy you need to avoid. You might think you know the
to shoulder extra work, anyway); and it shows fair leaders, but what about the followers? What if they
judgement leaving everybody satisfied to the best extent become uncontrolled, lose patience and become violent?
under the circumstances. You need to be balanced in your judgement and not
allow personal views of any kind to colour your
Q.5. A senior central minister is to visit your city. You, decision or take risks where public order and safety is
as an administrative officer associated with law concerned. Option (c) is the best: you are following
and order in the state, have become aware that democratic tenets but you are showing the good sense
certain groups of people intend to demonstrate at to be alert enough to meet any consequence. It is not
the venue of his rally to show their unhappiness
certain that option (d) would work, even though it is
with some of the policies of the government. In
not a bad idea. The would-be demonstrators may not
your talks with the leaders of the groups, you
agree and may have reached the idea of demonstration
have been assured that the demonstrations are
as a last resort. In any case, the right to demonstrate
going to be absolutely peaceful. The local leaders
peacefully is not wrong, and you must take such
of the political party to which the minister belongs,
demonstrations in your stride.
however, want the demonstration banned and
prohibitory orders imposed. You would Q.6. You are part of a team working on a programme
(a) use the preventive arrest facility and arrest the of relief for an earthquake-hit district. It involves
leaders of the groups intending to protest and great responsibility and heavy work and little rest.
impose prohibitory orders in the areas to be Suddenly, one of your team mates falls seriously
visited by the minister ill and has to be allowed leave. There is a shortage
(b) believe in the integrity of the leaders whom of workforce and a substitute is not likely to be
you personally know and admire and allow found soon. Headquarters asks you to shoulder
the demonstrations as they are going to be your team mate’s share of work. This is certainly
peaceful and this is a democratic country going to increase your workload greatly. You
allowing freedom of speech would

4 . 23
Decision-Making and Problem-Solving

(a) explain to headquarters how you are the motorcade stops and a local MLA and his
overburdened and ask for additional help if companions get out, catch hold of one of the
you have to do extra work protesting youths and beat and kick him despite
(b) accept the responsibility as the circumstances your efforts to stop them. They cause serious
are special injuries to the youth. You would
(c) pretend to hurt yourself so that you can also (a) pacify the MLA and his companions and tell
go on leave them youth will be youth and they should not
(d) tell headquarters that the workload is too take things so seriously
much and suggest that it be assigned to two (b) arrange to hospitalise the youth after charging
people and not just to you him of breach of security and place charges
of assault against the MLA
Analysis You must read the problem carefully; if
(c) smoothen things down, promise the MLA that
you do so, you would reject option (a) as you have
you would arrest the youth and ask the MLA
already been told that there is a shortage of workforce to go on his way
and as such you are not likely to get an additional hand (d) contact your senior officer for instructions
to help you. Option (b) is what a dedicated civil servant
would be expected to do. Additional work and Analysis The problem here is that an elected
representative of the people is the one who has been
responsibility are things you have to learn to cope with
violent. The youths may have breached security though
and be ready to accept, especially in an emergency such
they have not assaulted or hurt anyone. Clearly, your
as the one under consideration. Option (c) is
duty is to charge both the offenders, so (b) is the correct
unscrupulous and shows a total lack of responsibility
@UPSC_THOUGHTS

thing to do though you may face some trouble from the


and commitment to work and lack of empathy for the
political leaders for your action. But as a public official,
people who are in need of your help. Option (d) is
your duty is to be fair and act with integrity in tackling
second best; it is not wrong to ask for help when
violence, from whichever quarter it comes. You cannot
overburdened, and if another team mate shares the
allow anyone to take the law into their own hands. And
work, the shortage of workforce should not be a limiting
you have to act without fear or favour. Options (a) and
factor. However, the suggestion should follow
(c) are not acceptable. Option (d) is not acceptable either
acceptance of the work.
as it shows you incapable of taking a decision (unless,
7. You are an officer on duty controlling crowds of course, you have been specifically told not to act
lining the sides of the road along which a visiting without asking your senior).
minister is to travel. Suddenly a group of youths
waving black flags and shouting slogans breaks In the following pages, we have given a variety
the cordon of policemen and moves towards the of questions ranging over all the types, including some
motorcade of the minister and his companions. based on theory. Most of the answers are accompanied
You are able to take control and push back the by explanatory notes which give you an idea why a
group behind the security line. One of the cars in certain answer response is better than another.

4 . 24
Decision-Making and Problem-Solving

PRACTICE EXERCISES
7. Ethical decision-making in organisations includes
1. Consider the following statements.
I. not to exceed one’s authority.
I. Decision-making is a process of selection or
II. not to use inside information of the
choice from among a set of alternatives.
organisation for personal benefit.
II. Decision-making is always related to a
situation. III. acting impartially.
Which of the above statements is/are correct? IV. protecting the organisation’s property.
(a) I only Select the correct code from those given below.
(b) II only (a) I, II and III
(c) Both I and II (b) II, III and IV
(d) Neither I nor II (c) I, III and IV
(d) All the above
2. Which of the following is not a factor that hinders
decision-making? 8. Which of the following is not an element of ethical
(a) Fear of failure decision-making in an organisation?
(b) Intuition (a) Ethical commitment
(c) Time pressure (b) Ethical order
(d) Personal bias (c) Ethical consciousness
3. The first stage in rational decision-making is (d) Ethical competency
@UPSC_THOUGHTS

(a) analysing the problem 9. Which of the following is/are principles involved
(b) developing alternative solutions in decision-making?
(c) defining the problem I. Get the facts from all possible perspectives.
(d) evaluating alternatives
II. Check the law.
4. The most important basis for decision-making is III. Consider cause and effect in the deepest
(a) experience possible sense.
(b) intuition IV. Justify decisions according to religious faith.
(c) facts Select the correct code from those given below.
(d) operating research (a) I, II and III
5. ‘The use of logic behind a decision’, as a basis (b) II, III and IV
of decision-making, comes under (c) I, III and IV
(a) intuition (d) All the above
(b) considered opinion
10. Which of the following is not a characteristic of
(c) experience
making a good decision?
(d) simulation
(a) Advice-seeking
6. Ethics is (b) Knowledge
I. religion (c) Collecting all the facts
II. law (d) Initiative
III. following culturally accepted norms
IV. science 11. An effective decision-maker who remains open-
Select the correct code from those given below. minded about new concepts and ideas shows
(a) I, II and III (a) flexibility
(b) II, III and IV (b) self-knowledge
(c) I, III and IV (c) calculated risk-taking
(d) None of the above (d) comprehensiveness

4 . 25
Decision-Making and Problem-Solving

Information and instructions (for Qs. 12 to 16): Note: No case shall go to both the minister and
Some entrepreneurs have applied to the government for the secretary.
permission to establish cement factories. The government Mark your answer as under:
will grant the permission after the following conditions (a) Some required information is lacking
are satisfied. (b) Permission can be granted on the
1. The applications on prescribed forms must recommendation of the minister
have reached the Head Office on or before (c) Permission can be granted on the
14th November, 2001 with the name of the recommendation of the secretary
factory proposed. (d) Permission denied
2. The site proposed should be less than 10
12. Vimal Cements applied on 3rd December, 2000.
kilometres but more than 1 kilometre away
They have a deposit of ` 30 million in their
from the main road.
account and have received an NOC. There is not
3. The plants to be installed should have the much demand of cement in that area as one
approval of the government. cement factory is 35 km and another one is 17 km
4. There should be no other cement factory from the proposed site which is 16 km from the
within a range of 50 kilometres. main road. The proposal is to install a plant
5. The owners of the proposed factories must which is approved by the government.
have obtained NOC (no objection certificate)
from the Board of Environmental Preservation. 13. Kismat Cement India has proposed a site five km
6. The proposed owners must have a deposit of from the main road. Cement is in great demand
@UPSC_THOUGHTS

at least ` 10 million in their accounts. in the area as there is no factory within a range
7. Cement should be in demand in the state. of 75 km. The entrepreneurs applied on 16th
December, 2000. They have an NOC and want to
The minister concerned can give special sanction install a plant which is being successfully used
to the proposals if in France for a decade. Their deposits are ` 5
(i) the application reaches the department on or million and people are prepared to buy shares
before 30th November, 2001, thus giving a worth ` 12 million.
relaxation of about a fortnight.
(ii) the plant is not approved by the Indian 14. Kali Cements applied on 26th November, 2001.
government but is being used in some foreign The proposed site is nine km from the main road
country. and there is no cement factory around for 62 km.
(iii) the owner has a deposit of ` 5 million only They have not obtained an NOC but propose to
install an anti-pollution plant as well as a
but people are prepared to buy shares for the
government-approved plant. There is great demand
remaining amount.
of cement in the area and the party has a deposit
The secretary of the department concerned can of ` 15 million.
give special sanction to the parties if
15. Narasimha Rao Cements with ` 40 million in
(i) the proposed site is less than 50 kilometres
account applied on 29th November, 2001. The
from other factories but cement is in much
proposed site is seven km from the main road.
demand.
Cement is much in demand, the other factory
(ii) the proposed site is more than 10 kilometres
being 52 km away. They want to install a plant
from the main road.
which is very popular in Spain and have obtained
(iii) the Board of Environmental Preservation had
an NOC.
put up an objection because there were green
fields nearby but the owners are prepared to 16. Rita Cements has chosen a site where cement is
install anti-pollution plants in their factories. so much in demand that a factory 34 km away

4 . 26
Decision-Making and Problem-Solving

is unable to meet the demand. The firm has an 18. Raja has obtained highest marks among all the
account of ` 14 million, has obtained an NOC and employees in the PT aggregate as well as in each
proposes to install a government approved plant. Part. He is a graduate with 80 per cent marks. He
It applied on 3rd June, 2001 and the site is six was 47 years old as on 1.9.2001 and there is no
km from the main road. adverse remark in his CR. He has completed 10
Information and instructions (for Qs. 17 and 18): years of service in Grade III.
The criteria for promotion from Grade III to II in one Information (for Qs. 19 to 21): There are some
institute is as follows. The employee must conditions for allotment of flats built by the Town
A. be a graduate with minimum 50 per cent Council in the area of Gurgaon. The applicant must
marks.
1. produce domicile certificate of the state.
B. not be more than 45 years of age as on
2. be employed or self-employed in Gurgaon for
10.11.2001.
a minimum of five years.
C. obtain the minimum prescribed marks in
3. be ready to pay the entire amount in five
Promotion Test (PT). Minimum marks are Part
years.
A-35, Part B-25 and in aggregate 70 (total of
4. not be owner or co-owner of residential
Part A and Part B).
accommodation within the city limits of
D. have at least 10 years of service in the institute
out of which at least four years should be in Gurgaon.
Grade III. 5. not be less than 35 years of age as on December
E. not have any adverse remark in his/her 31, 2006.
@UPSC_THOUGHTS

Confidential Report (CR). In case of an applicant who satisfies all other


However, criteria except
(i) if a candidate fulfils all other criteria but is I. 1 above, he/she be referred to the president
more than 45 years and less than 50 years,
of Town Council.
the case is to be referred to the Governing
II. 2 above, but is ready to produce ration card
Board.
for the last five years, he/she should be
(ii) if a candidate fulfils all other criteria but does
referred to the vice-chairman of the House
not have four years of service in Grade III, the
Allotment Committee.
case is referred to the Director, provided the
III. 3 above, but is a freedom-fighter or ex-
employee has obtained 120 or more marks in
aggregate in the promotion test. serviceman or first relation, i.e., son/daughter/
husband/wife of freedom fighters/ex-
Keeping the above criteria in mind, consider each servicemen, he/she should be referred to the
of the cases in Qs. 17 and 18 and give answer chairman of the House Allotment Committee
(a) if the employee is to be promoted
who can give concession for payment upto 15
(b) if the case is to be referred to the Governing
years in such cases.
Board
Conditions, set out in terms of age or duration of
(c) if the case is to be referred to the Director
stay, are to be fulfilled as on December 31, 2006.
(d) if the data is not sufficient to make the
Consider them and choose the most appropriate action
decision
in each case.
17. 37-year-old (as on 14.3.1999) Dipti Gulhati is a
commerce graduate with 59 per cent marks. She 19. Maganlal is a 38-year-old senior clerk in the local
has obtained 125 marks in aggregate in PT. She builder’s office in Gurgaon. He has put in service
has no adverse remark in her CR and has of 13 years but still does not own a house. He has
completed 12 years in the institute out of which produced domicile certificate and is ready to pay
four years are in Grade III. the entire amount in eight years. He is the nephew

4 . 27
Decision-Making and Problem-Solving

of freedom fighter Jaganlal who stays in the (iii) He has led his cricket team at college level at
nearby village. least thrice and has taken 10 or more wickets
(a) Do not allot a flat either by bowling or while wicket-keeping, or
(b) Data inadequate has made aggregate 1000 runs in college level
(c) Refer to the vice-chairman matches.
(d) Refer to the chairman (iv) He has represented his state in national level
20. Rima Mohanty is daughter of a renowned freedom matches at least thrice with a remarkable
fighter from another state. She is domiciled in the bowling or batting or wicket-keeping record.
state and employed in the town council of Gurgaon (v) He has six centuries to his credit in college
for the last six years. She can pay the entire level matches and is a spin or medium fast
amount in five years. She has completed 34 years bowler having taken at least one wicket per
as on December 10, 2004. She does not own a match in college level matches.
house in Gurgaon. Based on the above conditions and the data given
(a) Refer to the chairman in each of the following cases, you have to choose your
(b) Allot a flat answer. You have received all the facts as on February
(c) Do not allot a flat 1, 1999.
(d) Refer to the president
22. Ameya started his cricket career exactly five years
21. Shamim Hasan is an ex-serviceman who is a earlier by celebrating his 18th birthday by scoring
native of Gurgaon. He stays in a rented house and a century. He is ready to pay ` 40,000 at entry
is working as a security officer in a factory for the level. He scored three fifties representing his state
@UPSC_THOUGHTS

last two years after his retirement from the army as captain. He is an excellent leg-spinner.
at the age of 35 years. He has a ration card issued (a) Membership be given as he satisfies (ii) only.
to him recently. He is ready to pay the entire (b) Data inadequate
amount in five years.
(c) Membership be given as he satisfies (ii)
(a) Allot a flat
and (iv)
(b) Refer to the chairman
(d) Membership not to be given
(c) Do not allot a flat
(d) Data inadequate 23. Satyen is a good wicket-keeper having 11 stump-
outs and 16 behind the wicket catches in his six-
Information (for Qs. 22 to 24): ‘Our Glory is
year tenure as a state level player. He has also
Cricket’ club intends to give membership to a selected
led his team in inter-university tournaments and
few players based on the following criteria.
has scored two centuries and one fifty. He is
The player must be above 16 years and not more willing to pay entrance fee as well as monthly
than 24 years of age as on 1.2.1999. He must pay charges. He started representing his state in March
` 15,000 as entrance fee and ` 1,000 as monthly fee 1991 at the age of 16 years.
throughout his membership period. In case he pays (a) Membership be given as he satisfies only (ii)
` 25,000 as additional entrance fee, the monthly payment (b) Membership be given as he satisfies only (iv)
condition is waived. In addition to this he should (c) Data inadequate
satisfy at least one of the following conditions: (d) Membership not to be given
(i) He has won any one inter-college cricket
tournament by leading his college team and 24. Anil has been playing for his college, university
has scored at least one century in college level and state during his seven years’ cricket tenure.
tournaments. He started playing for his college in January 1992
(ii) He has scored at least one century and two when he was 17 years old. He has seven centuries
fifties in inter-university or inter-state and five fifties to his credit aggregating 1600 runs.
tournaments. He led his university and state for two years and

4 . 28
Decision-Making and Problem-Solving

three years respectively. He has taken 11 wickets II. No, students will visit crowded places like
as medium fast bowler while playing for his state malls, markets, playgrounds etc., in more
in national level matches. He is willing to pay the numbers and spread the disease, as they will
requisite entrance fee and monthly fee. have a lot of spare time at their disposal.
(a) Membership to be given as he satisfies only III. Yes, young persons are more prone to get
(ii) and (iv) affected by the viral infection and hence they
(b) Membership to be given as he satisfies only should remain indoors.
(iv) and (v) (a) Only arguments I and II are strong
(c) Membership to be given as he satisfies only (b) Only arguments II and III are strong
(ii), (iv) and (v) (c) Only arguments I and III are strong
(d) Membership not to be given (d) All the arguments, I, II and III are strong
Directions (For Qs. 25 to 50): In making decisions, 27. Should the government ban export of all types of
it is desirable to be able to distinguish between ‘strong’ foodgrains for the next one year to tide over the
and ‘weak’ arguments. Strong arguments must be both unpredicted drought situation in the country?
important and directly related to the question. Weak
arguments may not be directly related to the question Arguments:
and may be of minor importance or may be related to I. Yes, there is no other way to provide food to
the trivial aspects of the question. its citizens during the year.
II. No, the government does not have jurisdiction
Each question below is followed by two or three
over private exporters for banning exports.
arguments. You have to decide which of the arguments
@UPSC_THOUGHTS

III. Yes, the government should not allow the


is/are ‘strong’ and select your answer accordingly.
exporters to export foodgrains and it should
25. Should the government restrict use of electricity procure all the foodgrains held by such
for each household? exporters and make it available for home
consumption.
Arguments:
(a) Only arguments I and III are strong
I. Yes, this will help the government tide over
(b) Only arguments II and III are strong
the problem of inadequate generation of
(c) All the arguments, I, II and III, are strong
electricity.
(d) None is strong
II. No, citizens have the right to consume
electricity as per their requirement as they pay 28. Should there be a common syllabus for all subjects
for using electricity. in graduate courses in all the universities across
III. No, the government does not have the the country?
machinery to put such a restriction on use of
Arguments:
electricity.
I. Yes, this is the only way to bring in uniformity
(a) Only argument I is strong
in the education system in the country.
(b) Only argument II is strong
II. Yes, it will help standardise the quality of
(c) Only arguments I and II are strong
graduation certificates being given by different
(d) All the arguments, I, II and III, are strong
universities in the country.
26. Should the government order closure of all III. No, each university should have the autonomy
educational institutions for a month to avoid fast to decide its syllabus based on the specific
spread of the contagious viral infection? requirement of the university.
Arguments: (a) Only arguments I and II are strong
I. No, closure of educational institutions alone (b) Only arguments II and III are strong
is not the solution for curbing spread of viral (c) All the arguments, I, II and III, are strong
infection. (d) None is strong

4 . 29
Decision-Making and Problem-Solving

29. Should all those students who failed in one or two 32. Should mutual funds be brought under strict
subjects for High School Certificate (HSC) be government control?
allowed to take admission in degree courses and
Arguments:
continue their study subject to their successfully
I. Yes, that is one of the ways to protect the
passing in the supplementary examination?
interest of the investors.
Arguments: II. No, strict government control is likely to be
I. Yes, this will help the students to complete counter-productive.
their education without a break of one year. (a) Only argument I is strong
II. Yes, this is a forward looking strategy to help (b) Only argument II is strong
the students and motivate them for higher (c) Either argument I or II is strong
studies. (d) Neither argument I nor II is strong
III. No, such students do not choose to continue
33. Should all the profit-making public sector units
their studies without having passed in all the
be sold to private companies?
subjects in HSC.
(a) Only argument I is strong Arguments:
(b) Only arguments I and II are strong I. Yes, this will help the government to augment
(c) Only arguments I and III are strong its resources for implementing the development
(d) Only arguments II and III are strong programmes.
II. No, the private companies will not be able to
30. Should there be only one rate of interest for term
run these units effectively.
deposits of varying durations in banks?
@UPSC_THOUGHTS

(a) Only argument I is strong


Arguments: (b) Only argument II is strong
I. No, people will refrain from depositing money (c) Both the arguments, I and II, are strong
for longer duration resulting in reduction of (d) Neither argument I nor II is strong
liquidity level of banks.
34. Should the public sector undertakings be allowed
II. Yes, this will be much simpler for the common
to adopt hire-and-fire policy?
people and they may be encouraged to keep
more money in banks. Arguments:
(a) Only argument I is strong I. Yes, this will help the public sector
(b) Only argument II is strong undertakings to get rid of non-performing
(c) Both the arguments, I and II, are strong employees and will also help to reward the
(d) Neither argument I nor II is strong performing employees.
II. No, the management may not be able to
31. Should all those who have come in contact with
implement the policy in an unbiased manner
the patients’ infectious respiratory disease be
and the employees will suffer due to the high-
quarantined in their houses?
handedness of the management.
Arguments: (a) Only argument I is strong
I. No, nobody should be quarantined unless (b) Only argument II is strong
they are tested and found to be infected by the (c) Either argument I or II is strong
virus causing the disease. (d) Neither argument I nor II is strong
II. Yes, this is the only way to control the spread
35. Should the major part of school examinations be
of the dreaded disease.
made objective-type?
(a) Only argument I is strong
(b) Only argument II is strong. Argument:
(c) Either argument I or II is strong I. No, objective-type examination does not test
(d) Neither argument I nor II is strong the students’ ability to express.

4 . 30
Decision-Making and Problem-Solving

II. Yes, this is the best method of assessing one’s (a) Only argument I is strong
ability and knowledge. (b) Only argument II is strong
(a) Only argument I is strong (c) Neither argument I nor argument II is strong
(b) Only argument II is strong (d) Both the arguments, I and II, are strong
(c) Neither argument I nor argument II is strong
39. Should there be a total ban on use of plastic bags?
(d) Both the arguments, I and II, are strong
Arguments:
36. Should government service in rural areas at least I. No, instead the thickness of plastic bags,
for two years after completion of graduation be which can be used without much damage to
made compulsory for the students of medicine? the environment, should be specified.
Arguments: II. Yes, use of plastic bags causes various
I. Yes, it is everyone’s duty to serve the people problems like water pollution and water-
in rural areas and contribute to their logging and hence it is necessary to ban it.
(a) Only argument I is strong
upliftment.
(b) Only argument II is strong
II. No, it cannot be applied only to the students
(c) Neither argument I nor argument II is strong
of medicine since anyway they are contributing
(d) Both the arguments, I and II, are strong
during their studies and particularly in the
period of internship. 40. Should the parents in India in future be forced to
(a) Only argument I is strong opt for only one child as against two or many at
(b) Only argument II is strong present?
@UPSC_THOUGHTS

(c) Neither argument I nor argument II is strong


Arguments:
(d) Both the arguments, I and II, are strong.
I. Yes, this is the only way to check the ever-
37. Should all the factories in the cities be shifted to increasing population of India.
the outskirts, far away from the main city? II. No, this type of pressure tactic is not adopted
Arguments: by any other country in the world.
I. Yes, this is an essential step for controlling (a) Only argument I is strong.
pollution in the city. (b) Only argument II is strong
II. No, such a step will lead to a lot of inconven- (c) Neither argument I nor II is strong
ience to the employees of the factories and (d) Both the arguments, I and II, are strong
their families as well. 41. Should the sex determination test during
(a) Only argument I is strong pregnancy be completely banned?
(b) Only argument II is strong
(c) Neither argument I nor argument II is strong Arguments:
(d) Both the arguments, I and II, are strong. I. Yes, this leads to indiscriminate female
foeticide and eventually will lead to social
38. Should the practice of rewarding high scores be imbalance.
stopped to handle frustration among the moderate II. No, people have a right to know about the sex
scorers? of their unborn child.
Arguments: (a) Only argument I is strong
I. No, it is necessary to motivate the high scorers (b) Only argument II is strong
and reward is one of the best ways of (c) Either argument I or II is strong
motivating. (d) Both the arguments, I and II, are strong
II. Yes, too much appreciation for high scores 42. Should all the slums in big cities be demolished
affects the moderate students adversely at and the people living in such slums be relocated
times leading to extreme situations. outside the city limits?

4 . 31
Decision-Making and Problem-Solving

Arguments: (a) Only argument I is strong


I. No, all these people will lose their home and (b) Only argument II is strong
livelihood and hence they should not be (c) Neither argument I nor II is strong
relocated. (d) Both the arguments are strong
II. Yes, the big cities need more and more space
to carry out developmental activities and hence 46. Should there be a ban on recruiting women
these slums should be removed. employees in companies required to work in night
(a) Only argument I is strong shifts?
(b) Only argument II is strong Arguments:
(c) Neither argument I or II is strong I. No, companies should instead arrange for
(d) Both the arguments, I and II, are strong transport upto the doorsteps of the employees.
43. Should there be a complete ban on mining coal II. Yes, it is necessary in view of the increasing
in India? number of cases of rape/molestation and
cheating.
Arguments:
(a) Only argument I is strong
I. Yes, the present stock of coal will not last long
(b) Only argument II is strong
if we continue mining at the present rate.
II. No, we do not have an alternate energy source (c) Neither argument I nor II is strong
of sufficient quantity. (d) Both the arguments are strong
(a) Only argument I is strong 47. Should the retirement age for employees of central/
(b) Only argument II is strong
@UPSC_THOUGHTS

state government be increased by two years?


(c) Neither argument I nor argument II is strong
Arguments:
(d) Both the arguments, I and II, are strong
I. Yes, this is required in view of the increasing
44. Should there be uniforms for students in the longevity.
colleges in India as in the schools? II. Yes, experienced employees are more useful
Arguments: than the new recruits.
I. Yes, this will improve the ambience of the (a) Only argument I is strong
colleges as all the students will be decently (b) Only argument II is strong
dressed. (c) Neither argument I nor II is strong
II. No, college students should not be regimented (d) Both the arguments are strong
and they should be left to choose their clothes
for coming to college. 48. Should the government make it compulsory for all
(a) Only argument I is strong the private medical institutes to join the Common
(b) Only argument II is strong Admission Test conducted by the government?
(c) Neither argument I nor II is strong Arguments:
(d) Both the arguments are strong I. No, the private institutes should be allowed
45. Should all the universities switch over to online to decide their own admission strategy to
admissions at all levels all over the country with make them more suitable for jobs.
immediate effect? II. Yes, all the medical institutes, whether private
Arguments: or government, should follow common criteria
I. No, all the students may not have easy access for admission.
to internet. (a) Only argument I is strong
II. Yes, this will save the students and parents (b) Only argument II is strong
from all the hassles of visisting various colleges (c) Neither argument I nor II is strong
and standing in queue. (d) Both the arguments are strong

4 . 32
Decision-Making and Problem-Solving

49. Should there be a complete ban on consumption II. Yes, quality education is not possible without
of tobacco by youngsters till the age of 25 years? teachers and classrooms.
Arguments: 53. Should only nuclear power be used to generate
I. Yes, this will be a good step as youngsters are electricity?
fully matured by the age of 25 years.
Arguments:
II. No, it is not necessary as all the adults
I. Yes, this will help reduce air pollution to a
above 18 years of age can understand their
great extent.
responsibility.
II. No, radioactive material used in nuclear plants
(a) Only argument I is strong
is unsafe for large scale use.
(b) Only argument II is strong
(c) Neither argument I nor II is strong 54. Should the government remove all the slums in
(d) Both the arguments, I and II, are strong major cities?
Arguments:
50. Should the government have the authority to
I. Yes, slums are a nuisance to the people living
censor all electronic media programmes?
in big cities.
Arguments: II. No, inhabitants of slums are also citizens of
I. No, the people can judge what to watch and the country and they contribute towards the
should have the freedom of information. growth of the nation.
II. Yes, because people with impressionable
55. Should cricket replace hockey as the national
minds watch and may get wrong ideas.
sport of India.
@UPSC_THOUGHTS

(a) Only argument I is strong


(b) Only argument II is strong Arguments:
(c) Both arguments are strong I. Yes, the performance of the Indian hockey
(d) Neither argument is strong team has been dismal in the last few years.
II. No, cricket is the national sport of Australia
Directions: For Questions 51 to 55 answer as follows: and no two countries must have the same
(a) Only argument I is strong national sport.
(b) Only argument II is strong 56. Should there be a cap on drawing groundwater
(c) Neither argument I nor II is strong for irrigation purposes in India?
(d) Both the arguments, I and II, are strong I. No, irrigation is of prime importance for food
51. Should the number of holidays given to production in India and it is heavily
government employees be reduced to only five in dependent on groundwater in many parts of
a year? the country.
II. Yes, water tables have gone down to alarmingly
Arguments:
low levels in some parts of the country where
I. Yes, such holidays subsequently reduce
irrigation is primarily dependent on
working hours, thus adversely affecting the
groundwater, which may lead to serious
economy of the nation.
environmental consequences.
II. No, employees require intermittent rest from
III. Yes, India just cannot afford to draw
hectic work schedule.
groundwater any further as the international
52. Should all correspondence courses at the graduate agencies have cautioned India against it.
level be stopped? Arguments:
Arguments: (a) Only I is strong
I. No, correspondence courses help needy (b) Both II and III are strong
students to pursue studies and earn at the (c) Both I and II are strong
same time. (d) I, II and III are strong

4 . 33
Decision-Making and Problem-Solving

57. Should road repair work in big cities be carried (a) Only I and III are strong
out only late at night? (b) Only II is strong
Arguments: (c) Only III is strong
I. No, this way the work will never get (d) Either II or III is strong
completed. 60. Should there be a complete ban on setting up of
II. No, there will be unnecessary use of electricity. thermal power plants in India?
III. Yes, the commuters will face a lot of problems
Arguments:
due to repair work during the day.
I. Yes, this is the only way to arrest further
(a) Only I is strong
environmental pollution.
(b) Only II is strong II. No, there is a huge shortage of electricity in
(c) Only III is strong most parts of the country and hence generation
(d) Either II or III is strong of electricity needs to be augmented.
58. Should there be a restriction on the construction III. No, many developed countries continue to set
of high rise buildings in big cities in India? up thermal power plants in their countries.
I. No, big cities in India do not have adequate (a) Only I is strong
open land plots to accommodate the growing (b) Either I or II is strong
population. (c) Both I and II are strong
II. Yes, only the builders and developers benefit (d) Only II is strong
from the construction of high rise buildings. 61. Should the government supply free textbooks up
@UPSC_THOUGHTS

III. Yes, the government should first provide to Std. X to all the students in the government-
adequate infrastructural facilities to existing run schools in India?
buildings before allowing the construction of
Arguments:
new high rise buildings.
I. No, many students of these schools come from
Arguments: rich families and they should not be given free
(a) Only I is strong books.
(b) Only III is strong II. Yes, this will considerably reduce the
(c) Both I and III are strong percentage of school dropouts as many parents
(d) Both II and III are strong cannot afford the burden of buying books.
III. No, the prices of books prescribed up to
59. Should all the deemed universities be derecognised
Std. X are otherwise very low and there is no
and attached to any of the central or state
need to distribute books free of cost to the
universities in India?
students.
Arguments:
(a) Only I and II are strong
I. Yes, many of these deemed universities do not
(b) Only II and III are strong
conform to the required standards of a full- (c) Only I and III are strong
fledged university and hence the level of (d) I, II and III are strong
education is compromised.
II. No, these deemed universities have been able 62. Should all the engineering graduates passing out
to introduce innovative courses suitable to the of government colleges be compulsorily made to
requirement of various industries as they are work in government organisations/public sector
free from strict government controls. undertakings?
III. Yes, many such universities are basically Arguments:
money spinning activities and education takes I. Yes, these students have used huge resources
a backseat in these institutions. of the government and they should pay back

4 . 34
Decision-Making and Problem-Solving

by serving in government/public sector IV. No, nuclear power installations are sensitive
undertakings. in nature and are integrated to the safety and
II. Yes, otherwise these students will join the security of our country and therefore we
multinational companies and earn huge sums should not allow foreigners to have access to
of money leading to loss of government such installations.
resources. (a) All the arguments are strong
III. No, each individual has a right to choose his/ (b) Only I and II are strong
her job and serving in government/public (c) Only III is strong
sector undertaking should not be made (d) Only II is strong
mandatory.
65. Should lottery schemes be banned in India?
(a) None is strong
(b) Only III is strong Arguments:
(c) Only I and III are strong I. No, many states derive substantial funds for
(d) I, II and III are strong development from their lottery schemes.
II. Many persons who now sell lottery tickets for
63. Should the government privatise all the coal livelihood will be rendered unemployed.
mines in the country? III. Yes, it causes depravity and feeds greed.
Arguments: (a) Only I and II are strong
I. Yes, this will result in optimum mining as the (b) Only II is strong
private companies are in a position to bring (c) I, II and III are strong
@UPSC_THOUGHTS

in experts from other countries to maximise (d) None of them is strong.


the output.
II. No, the coal mines are our country’s wealth Directions (Q. 66 to 105): In each question below
and we should not allow them to go into is given a situation followed by two possible decisions
private hands. or courses of action numbered I and II. On the basis
III. Yes, the coal mines in India are in a very bad of the information given in the situation, assuming
shape and the government is unable to manage everything in the situation to be true, which of the
them efficiently. decisions or courses of action is/are correct?
(a) Only I and III are strong Give answer (a) if only I is right.
(b) Only I and II are strong Give answer (b) if only II is right
(c) Only II and III are strong Give answer (c) if neither I nor II is right
(d) I, II and III are strong Give answer (d) if both I and II are right.
64. Should the government allow foreign construction 66. Situation: Many pilgrims died in a stampede
companies to set up nuclear power plants? while boarding a private ferry to the holy
Arguments: place on the first day of the ten-day-long
I. Yes, the foreign construction companies are festival.
technically superior to the construction Decisions: I. The government should immedi-
companies in our country and therefore they ately cancel the licences of all
will be able to build better installations. the private ferry operators with
II. Yes, we need to construct such facilities with immediate effect.
the help of foreign construction companies II. The government should deploy an
which have experience of building such adequate number of its personnel to
installations in a time-bound manner to avoid guide and manage the pilgrims on
unnecessary delay. their journey to the holy place.

4 . 35
Decision-Making and Problem-Solving

67. Situation: Some students of the local college were 71. Situation: There have been sporadic cases of
caught travelling in the train without stone throwing and damaging vehicles in
purchasing valid tickets. the locality during the day following an
Decisions: I. The parents of these students altercation between two local youth clubs
should be informed about the last night.
incident and requested to counsel Decisions: I. The local police administration
their wards. should arrest all those who are
II. The students should be put behind caught committing these acts.
bars for travelling without bona II. The local police administration
fide credentials. should call a meeting of office
bearers of both the clubs of the
68. Situation: A large part of a locality was flooded
locality to bring the situation under
as the main pipe supplying drinking water
control.
burst while the workers of a utility company
were laying cables in the area. 72. Situation: A huge truck overturned on the middle
Decisions: I. The civil authority should of the main road and blocked most part of
immediately arrange to repair the the road, causing a huge traffic jam.
damage and stop loss of water. Decisions: I. The traffic department should
II. The civil authority should seek an immediately deploy its personnel to
explanation and compensation from build another road to divert the
@UPSC_THOUGHTS

the utility company for the damage traffic.


caused by it. II. The traffic department should
immediately send men and
69. Situation: Millions of pilgrims are expected to
equipment to move the truck and
take a dip in the Ganges at the holy place
clear the road.
during the next fortnight.
Decisions: I. The government should restrict the 73. Situation: Some workers of the company making
number of pilgrims who can take diamond jewellery were caught while they
the dip each day during the were leaving the premises as they were
fortnight. trying to smuggle small pieces of diamonds
II. The government should deploy an hidden in their purses.
adequate number of security Decisions: I. The management of the company
personnel to maintain law and should immediately put on hold all
order during the fortnight at the activities in the premises till a fool-
holy place. proof security system is in place.
II. The belongings of all the workers
70. Situation: The rate of inflation has reached its
should thoroughly be searched
peak in the last twenty years and there is
before they leave the premises of the
no sign of it softening in the coming months.
company.
Decisions: I. The government should initiate
steps like reducing government taxes 74. Situation: A huge tidal wave swept away many
on essential commodities with fishing boats and huts of the fishermen
immediate effect. living along the coastline.
II. Farmers should be asked by the Decisions: I. The fishermen should henceforth
government to sell their produce at be restrained from constructing their
lower price. huts along the coast line.

4 . 36
Decision-Making and Problem-Solving

II. The local administration should share in global agricultural trade is less
send a team of officials to assess the than its share of agricultural export out of
extent of damage and suggest its total exports.
remedial measures. Decisions: I. Efforts should be made to increase
India’s agricultural production.
75. Situation: A large number of invitees who
II. The exports of non-agricultural
attended a marriage function fell ill due to
commodities should be reduced.
food poisoning and were rushed to various
hospitals located in the area. 80. Situation: Huge amount of resources are required
Decisions: I. The government should ban such to develop a tourist place in a country like
marriage functions till further India which is endowed with vast coastal
notice. lines, rivers, forests, temples, etc.
II. The local hospitals should be Decisions: I. More tourist-resorts should be
advised by the government to started only along the coastal line.
provide best services to the affected II. The tourist-potential of India should
people. be exploited.
76. Situation: Air export volumes have increased 81. Situation: The government has decided not to
substantially over the past decade causing provide financial support to voluntary
backlogs and difficulties for air cargo agents organisations from the next Five Year Plan
because of increased demand for space and and has communicated that all such
@UPSC_THOUGHTS

service. organisations should raise funds to meet


Decisions: I. Airlines and air cargo agents their financial needs.
should jointly work out a solution Decisions: I. Voluntary organisations should
to combat the problem. collaborate with foreign agencies.
II. The reasons for the increase in the II. Voluntary organisations should
volume of air export should be explore other sources of financial
found out.
support.
77. Situation: The world conference on ‘Education of
82. Situation: Exporters in the capital are alleging
All’ took place in Thailand in 1990. The
that commercial banks are violating a
widely attended conference endorsed the
Reserve Bank of India directive to operate
Framework for Action for Meeting the Basic
a post-shipment export credit denominated
Learning Needs of all Children.
in foreign currency at international interest
Decisions: I. India should suitably implement
rates from January this year.
the action points of this conference.
Decisions: I. The officers concerned in the
II. India should immediately organise
commercial banks are to be
a conference of this type.
suspended.
78. Situation: About 30 to 40 per cent of children II. The RBI should be asked to stop
who are enrolled do not attend school on giving such directives to commercial
any given day. banks.
Decisions: I. More schools should be started.
83. Situation: The chairman stressed the need for
II. Reasons for their absenteeism
making the education system more flexible
should be found out.
and regretted that the curriculum had not
79. Situation: Although the Indian economy is still been revised in keeping with the pace of
heavily dependent on agriculture, India’s changes taking place.

4 . 37
Decision-Making and Problem-Solving

Decisions: I. Curriculum should be reviewed and 88. Situation: Many school children have died in
revised periodically. accidents caused due to poor maintenance
II. System of education should be of school buses during the last few months.
made more flexible. Decisions: I. The government should set up an
84. Situation: The daytime temperatures this summer expert group to inspect the
have been four to five degrees Celsius above condition of school buses to avoid
the normal temperature across the country. such accidents.
Decisions: I. The government machinery should II. The government should suspend
be put on high alert and provided the licence of all the school buses
with necessary equipment to prevent till these buses are properly checked.
any untoward incident.
89. Situation: A very large number of recently
II. The government should make
recruited officers left the company after
necessary arrangements to provide
attending the programme on branch
water in all the areas affected due
to the extreme heat wave. operations conducted by the company.
Decisions: I. The company should take a bond
85. Situation: If the faculty members also join the of at least two years from each
strike, then there is going to be a serious
newly recruited officer in future
problem.
before putting him into the training
Decisions: I. The faculty members should be
programme.
persuaded not to go on strike.
@UPSC_THOUGHTS

II. The company should conduct the


II. Those faculty members who join
training programme at least one
the strike should be suspended.
year after the newly recruited officers
86. Situation: Many customers complained to the have joined the company.
manager of the local branch of the bank
against the high-handedness of the bank 90. Situation: A huge oil tanker overturned on the
employees while dealing with the highway leading to the state capital and
customers. blocked most part of the highway from
Decisions: I. The bank management should vehicular traffic.
immediately suspend all the Decisions: I. The government should immedi-
employees of the branch and ately constitute a high level enquiry
employ a different set of employees. committee to look into the matter.
II. The bank management should ask II. The oil tanker should be removed
for a report from the manager of the from the road immediately.
branch for taking necessary steps.
91. Situation: A very large number of people gathered
87. Situation: Two persons, while on their daily
outside the local police station to submit a
walks in the jogger’s park, were killed by
memorandum on behalf of the residents
unidentified miscreants early in the
highlighting police inaction in curbing
morning.
incidents of theft and burglary in the
Decisions: I. The police authority should deploy
neighbourhood for the past few months.
police constables near the jogger’s
park to prevent such criminal acts Decisions: I. The police authority should form a
in future. team of officers to talk to the
II. The citizens of the locality should representatives of the residents and
go for early morning walks in assure them that proper steps will
groups to avoid such attacks. be taken to stop the menace.

4 . 38
Decision-Making and Problem-Solving

II. The police authority should advise guidelines for all establishments
the people gathered outside the regarding installation and
police station to disburse them and maintenance of electrical fittings.
promise them quick action. II. The government should relocate all
the markets to the outskirts of the
92. Situation: At least five students were killed in a
city.
stampede in one city school as the students
tried to leave the school building fearing fire 96. Situation: According to latest statistics, the
due to a short circuit. number of rhinos killed by tigers was much
Decisions: I. The principal of the school should more than the number of rhinos killed by
immediately be arrested. poachers in a wildlife sanctuary.
II. The government should immedi- Decisions: I. Stricter regulations should be framed
ately order closure of the school in order to punish the poachers.
permanently. II. As done in some sanctuaries, the
93. Situation: Two local passenger trains collided areas where tigers hunt and the
while running in opposite directions on the areas with dominant rhino
same track as the signalling system failed population should be artificially
for a brief period. separated.
Decisions: I. The services of the motormen of the 97. Situation: A mid-air collision was narrowly
trains should immediately be avoided when the pilot of one of the aircrafts
@UPSC_THOUGHTS

terminated. neglected the air traffic controller’s


II. The government should immedi- instructions.
ately constitute a task force to Decisions: I. Pilots of both of the aircrafts should
review the functioning of the be immediately reprimanded by a
signalling system. revoking of their licenses.
94. Situation: Almost ninety per cent of the flights of II. The training of air traffic controllers
one of the private airline companies were should be improved and made more
cancelled for the fourth consecutive day as comprehensive in order to avoid
the pilots refused to join their duties in such incidents in future.
protest against the sacking of two of their 98. Situation: Oil spill from the oil carrier of one of
colleagues by the airline management. the biggest oil companies has severely
Decisions: I. The management of the airline affected marine life in a large area near the
company should be ordered by the Gulf region.
government to immediately reinstate Decisions: I. The oil company should be
the sacked pilots to end the crisis. penalised for the negligence and
II. The government should immedi- the harm caused to the environment.
ately take steps to end the impasse II. Efforts should be made to shift as
between the management and many marine animals in the area as
the pilots to help the hapless possible to safer habitats.
passengers.
99. Situation: Many private schools have been
95. Situation: A major part of the local market in the reportedly denying admission to students
city was gutted due to a short circuit causing from poor families while preferring those
extensive damage to goods and property. with a sound financial background
Decisions: I. The government should issue strict regardless of their merit.

4 . 39
Decision-Making and Problem-Solving

Decisions: I. Poor students should be encouraged 103. Situation: Many motorists driving on the highway
to take admissions only in govern- within the city are found to be driving much
ment-run schools where such beyond the permissible speed limit.
discrimination does not exist. Decisions: I. The traffic police officials should
II. The government should enforce a personally monitor the movement
certain percentage of seats for of vehicles on the highway within
students from poor families in every the city.
school. II. The government should
immediately put in place a
100. Situation: The number of malaria cases has been
mechanism to identify and punish
rising significantly in the city for the last erring drivers.
few months.
Decisions: I. The municipal authorities should 104. Situation: Majority of the city employees of a
take immediate steps to destroy the renowned BPO company have left their jobs
breeding places of mosquitoes and in protest against inhuman treatment meted
out to them by the company.
improve hygiene in the area.
Decisions: I. The government should
II. A campaign to educate people about
immediately order the BPO company
the ways to prevent the disease
to close down its operation.
should be started using local media
II. The BPO company should shift its
and social workers.
@UPSC_THOUGHTS

operations to some other place in


101. Situation: The local college principal has ordered order to continue its operations.
that all the students must strictly adhere to
105. Situation: The management of the organisation
the dress code stipulated by the college
has issued a circular to all its employees
authority in the mission brochure.
stating that each employee must report for
Decisions: I. Those students who are found to
duty at 10.00 A.M. sharp and should remain
violate the dress code should be
in his/her workplace till 5.30 P.M. everyday.
expelled from the college.
Decisions: I. The management should evolve a
II. Those students who are found to
mechanism to identify such
violate the dress code for the first
employees who may not adhere to
time should be reprimanded and be
the time schedule.
warned against further violation. II. All such employees who are found
102. Situation: The railways have decided to repair to be failing to maintain time
the main tracks within the city on the schedule should be summarily
following Sunday and has decided to suspended.
suspend operations for the whole day. 106. You are walking along a railway track when you
Decisions: I. The railway authority should issue notice some sleepers are missing. You see a train
public notification well in advance approaching in the distance. You would
to ease inconvenience to the (a) rush back to the nearby station to give an
passengers. alarm
II. All the long distance trains entering (b) shout to passersby to do something
the city during the repair hours (c) do something to stop the oncoming train
should be terminated outside the (d) give a call immediately to the police control
city limit. room

4 . 40
Decision-Making and Problem-Solving

107. You are travelling in a train that has been stopping (c) tell the child to sit quietly in a restaurant
every now and then and is running inordinately while you go and find out what happened
late. Everyone is restive and some passengers are (d) ask other people what happened
showing signs of aggressiveness towards the
111. You are driving along a road when a car coming
railway caterers who have come to collect their at a speed from the opposite direction swerves
bills. What would you do? into your path. In trying to avoid a collision, you
(a) Make your way to the toilet swerve too and hit a street vendor’s cart, injuring
(b) Join the passengers as you are also angry at him and damaging his goods. What would you
the delay and suggest a deputation to the do?
railway board (a) Speed away, as it was in reality the other
(c) Point out to the railway staff that the driver’s fault and he has sped away
passengers’ anger is justified (b) Stop and get out of your car and explain to
(d) Pacify the passengers and point out to them the vendor that it was not really your fault
that the railway caterer is not responsible for (c) Step down and take the injured man to hospital
the delay (d) Offer a token amount to pay for the damage,
108. On your way home from office, you see a boy pointing out that the vendor had no right to
being beaten by a group of people shouting, be on the road anyway
‘Thief, Thief’. You would 112. While travelling by public transport, you see some
(a) intervene and stop the people beating the boy boys misbehaving with a girl. You would
(b) join the others in beating the boy who is, after
@UPSC_THOUGHTS

(a) intervene and ask the boys to behave


all, a thief themselves
(c) call upon other passersbys to help you stop (b) keep out of it, as you feel the girl’s modern
the people beating the boy way of dressing has invited unwanted
(d) make your way home; it is none of your attention
business (c) not consider it your business
(d) tell the conductor and driver of the bus to deal
109. You are walking along a road when you see a
with the situation
man snatch a purse from a woman some distance
ahead of you, stab her and run away. The woman 113. You are walking down the road when a car
lies there bleeding. You would driven rashly knocks down a cyclist. You would
(a) use your mobile and ring up the police control (a) run after the car shouting at the driver to stop
room (b) first attend to the cyclist who may be injured
(b) run after the bag snatcher with the intention and then ring up the police
of catching him (c) phone the police control room and describe
(c) find out the condition of the woman and the car to the best of your ability and then
make arrangements to get her medical attention attend to the cyclist
(d) ask the woman to go to the nearby clinic (d) place a written complaint with the police on
while you run after the bag snatcher the increase of rash driving in your locality

110. You have taken your friend’s child for a treat at 114. You are walking home from office when you see
a shopping mall when there is a loud bang and a badly injured man lying by the roadside. He is
you see billows of smoke. You would bleeding from the head. You would
(a) immediately go and investigate what the (a) go on your way as you do not have medical
source of the noise and smoke is knowledge
(b) take the child to a place of safety away from (b) try to take him to hospital after bandaging
the scene him

4 . 41
Decision-Making and Problem-Solving

(c) phone the police, try to stop the bleeding and 118. There is a train accident and a number of people
stay by the man till the patrol van comes are injured. Though the site is near your
(d) try to find out who he is and how he came administrative centre, you do not have jurisdiction
by his injuries over the area where the accident occurred. You
would
115. You are standing on the verandah of your house
(a) begin a rescue operation even as you inform
when you see someone climb up the drainpipe of
the authorities under whose jurisdiction the
the neighbouring house which you know is empty
site falls
as the inhabitant is out of station. The first thing
(b) report to the headquarters and await
you would do is to
instructions
(a) raise a hue and cry
(c) inform the authorities under whose jurisdiction
(b) ring the police
the area falls
(c) go up and tackle the person
(d) wait and watch (d) call a meeting of your colleagues and take
their inputs on what to do
116. You are sitting in an examination hall writing
your test. You see another examinee, who you 119. You are on your way to an important meeting that
know is the son of a member of parliament, get is to result in the signing of an agreement with
out a slip from his shoe and copy something onto a major concern, after which there is to be an
his answer paper. What would you do? official lunch. You get a call from the sister of one
(a) Go up to the examinee and snatch the slip of your friends that he has met with an accident
@UPSC_THOUGHTS

from him and threaten to report him unless and is in the ICU of a city hospital some fifteen
he shares the information with you kilometres from where you are. What would you
(b) Shout out the situation and call upon the do?
other examinees to boycott the exam (a) Make a detour immediately for the hospital,
(c) Report the matter straight to the invigilator phoning your colleague who is immediately
and express your conviction that proper action junior to you to officiate in your place at the
would be taken meeting
(d) Do nothing, as everyone knows how power (b) Pray for the well-being of your friend
is abused by our legislators and their favoured (c) Ask the sister to ring up if there is any further
ones news
(d) Attend the meeting and sign the agreement,
117. You are coming out of a stall at an exhibition
but make excuses from the lunch that is to
when you see a girl just ahead of you drop a
follow and go to the hospital
mobile phone and walk on. You shout out to her
telling her that she has dropped her mobile 120. You have been instrumental in implementing a
phone. Instead of turning back as you expect her project of providing potable water in a village
to, she takes off speedily. You would successfully. The villagers are grateful and the
(a) laugh at her foolishness and pick up the community leaders express their appreciation by
phone meeting you at the office with a box of sweets and
(b) suspect something amiss and pick up the an envelope of cash. How would you respond?
phone and throw it far away (a) You will accept it as refusing it will disappoint
(c) suspect something amiss and ring up the the simple villagers, but put the cash in the
police welfare fund of the office workers
(d) suspect something amiss and warn people off (b) You will accept the sweets and share them
the area even as you ring up the police with the department, but politely refuse the

4 . 42
Decision-Making and Problem-Solving

cash saying you could not accept cash in the (d) Explain to the senior authorities that your
office team is caught up in the final stage of the
(c) You will accept the sweets and share them earlier project and request them to keep the
with your department colleagues, but politely next project on hold for a short while
refuse the cash saying you were merely doing
123. You have noticed in your in-box fax messages
your work and needed no further payment for
unrelated to your organisation’s work but
that
involving some official matter that relates to a
(d) You would firmly send the man about his colleague’s side business. This is not the first time
business after scolding him for trying to offer it has happened. Once earlier, you had talked to
you a bribe your colleague about it and told her that you
121. It is the time of year when a lot of pending work would have to tell the senior officer about it if she
has to be cleared up within a deadline – a very continued to use office resources for business
busy time for everyone in the department of which purposes. What would you do?
you are head. A subordinate comes up with an (a) Ignore it as your warning has had no effect
application for leave for a week as a family on her and you do not want to jeopardise
holiday had been planned earlier and he could your relations with a colleague
not get out of it. You would (b) Talk to her again and repeat to her you would
(a) grant leave on compassionate grounds have to go to the senior officer if she did not
(b) just refuse leave as the situation is self- stop this behaviour
@UPSC_THOUGHTS

explanatory and granting him leave would (c) Take the matter to the senior officer
make others in the department resentful (d) Put the fax in the senior officer’s mailbox
(c) refresh his memory about the work load, tell without saying anything to anyone
him that you understand his problem, but he 124. You are on duty overseeing security at a state fair.
must do his best to postpone the holiday, as You notice that the giant wheel has come to a halt
it would not be possible to grant leave in the with people still on it, and the operator is telling
circumstances them that it will start soon. Two unarmed teenagers
(d) sit over the application long enough for the are in a heated argument and a crowd is forming
applicant to give up hope for the leave around them. A man, obviously drunk, is lying
permission to come through on the ground near the ticket booth. A garbage
122. Your team is working on a sensitive project that container placed next to the tent housing the main
is nearing completion, when another very pavilion has caught fire. Which of these incidents
important project comes in calling for your urgent will get your attention first?
attention. How would you set your priorities? (a) The giant wheel
(a) Be systematic: complete the project in hand (b) The teenagers who are causing a disturbance
and then take up the other one and give it all and may come to violence and draw the
the attention and complete it fast crowds into it
(b) Assign the project that is nearing completion (c) The drunk who may cause obstruction at the
to the senior members of the team after a ticket booth and put off customers
briefing, and take up the second project and (d) The garbage container on fire
start working on it simultaneously 125. You have promoted a woman in your department
(c) Take up the second project immediately as it because she is good at her work, has the right
is very important, keeping aside the earlier qualifications, and has shown potential for
one decisive action, which is a requirement in the new

4 . 43
Decision-Making and Problem-Solving

post. The post is one coveted by many others in asking him to forward the mail to the correct
the department. The woman promoted happens to person whose ID you give
be your boss’s daughter, and three of those not (c) Send the mail again, this time to the right
promoted are complaining about favouritism, and person, before you leave office
are threatening to report the matter to higher (d) Deal with the problem the next day
authorities. What would you do? 128. Your senior officer has asked you to identify two
(a) Call the complaining group and explain how workers who have specific skills required for a
your move was based on principles and that particular project. You know that Rohit and
the best person satisfying all criteria required Sukanya fit the bill; they have the requisite skills
for the post was selected, and tell them that and are highly competent. The difficulty is that
you would not be changing your decision because of some personal grudge they don’t want
(b) Call each of the complaining group separately to work together. However, there is no other
and tell them not to spread rumours or it worker who can be substituted for either of them.
would go badly against them in their What would you do?
confidential reports, further jeopardising their (a) Get back to your senior officer and explain the
chances of promotion situation and ask him or her to seek elsewhere
(c) Ignore the rumours and the complaints, as for at least one other worker
you know you have been guided by principles, (b) Meet Rohit and Sukanya separately and talk
and some people are bound to be disgruntled to them, explaining the importance of the
at every promotion that they miss, and the project and the need for each of them to work
@UPSC_THOUGHTS

rumours will soon subside on it together


(d) File a complaint with the authorities concerned (c) Call the two workers over and threaten them
against the group before they get to complain with disciplinary action if they do not put
against you their childish personal problems aside and
work on the project
126. You work in an oil refinery and have put in (d) Assign specific jobs to each one so that they
overtime all week. You are tired out and have at can work individually without having
last signed off and are on your way home. You anything to do with each other
suddenly notice oil leaking from a transit pipe. It
129. In a village over which you have jurisdiction, two
is a small leak, and you want badly to rest. What
leading political groups have entered into a fight
would you do?
with each other during the election campaign,
(a) Write a note for the senior officer and leave
and there is a danger of the fight escalating into
it on his desk
greater violence. What would you do immediately?
(b) Go home as your working hours are over and
(a) Send a message to the state election
the next man in will take care of any problem
commissioner asking him to tackle the matter
(c) Place a sign near the leak warning people
by disqualifying the candidates belonging to
about the leaking oil
the two parties
(d) Fix the leak or see to it that someone fixes it
(b) Reach the place with speed and use a loud-
immediately
speaker to ask the crowds to disperse, warning
127. At the end of a busy day at work, you are sending them that you will order firing if they do not
an urgent departmental message by e-mail, but take heed
send it to the wrong person. What would you do? (c) Reach the place and order a lathi charge
(a) Call up your senior officer and explain the followed by firing in the air
situation and ask for instructions (d) Call the leaders of both parties and counsel
(b) Leave office and phone the person from home them on the necessity of keeping the peace

4 . 44
Decision-Making and Problem-Solving

130. The meteorological department has forecast severe 133. As administrative officer in charge of a district,
cyclone over coastal Andhra Pradesh in the next you receive a call from an unidentified person
24 hours. In spite of a standing order that people who says that some anti-social elements threw
should not inhabit the coastal areas beyond certain stones at members of a certain community
boundary, people have made kucchha houses and celebrating their religious festival, as a result of
live there. You are the administrative officer with which tension has built up. What would you do?
jurisdiction over the region. What would you do? (a) You will ignore the call as it is an anonymous
(a) Order mandatory evacuation of people in the call, and anonymous calls are usually just
region under threat made to create mischief
(b) Call for a meeting and ask officials concerned (b) You would take a small group of security and
why they have allowed blatant contravention police personnel, after alerting a larger force
of the rules to be ready on call, and reach the place of the
(c) Order the evacuation of those willing to leave alleged incident
their houses (c) You would tell the police commissioner or his
(d) Make provision for rescue operations in case equivalent to take strong action
the cyclone washes away the houses (d) You would ask the telecom department to
trace the call and then the informer
131. You are the administrative officer in charge in an
134. You notice one afternoon that an officer from your
area where a relatively new six-storey building
vigilance department has eaten a meal at a roadside
housing several families has collapsed in the
@UPSC_THOUGHTS

dhaba and not paid for it. You learn from one of
heavy rains. Your immediate action would be to
the other customers that this man never pays for
(a) order rescue operations to begin immediately
his meal. You would
and alert nearby hospitals to be ready with
(a) enquire from the dhaba incharge details about
assistance for the injured
the matter
(b) order the police to file an FIR against the
(b) call the officer to your room and reprimand
builder
him and suspend him
(c) announce compensation for the victims
(c) shout at the officer to go back and pay for
(d) inform the chief minister of the state
what he has eaten
132. Your senior officer has passed an order for (d) turn a blind eye as such things always happen
allowing a block of houses to be built in an area and it is a minor event
recently declared as an environmentally protected
135. Your organisation has invited tenders for a project
area. You have been asked to process the order.
and you are in charge of scrutinising the
What will you do?
applications. One applicant comes to meet you at
(a) Do what you have been told to do, as it is your home and offers you gift of a car if you pass his
job to carry out orders tender. You would
(b) Complain against your senior officer to (a) tell the man to get out
somebody higher than him or her, suspecting (b) tell the man that you would oblige but reject
some nexus between the builders and the the tender out of hand
officer or just blatant incompetence (c) pretend that your hands are tied and senior
(c) Seek out the officer and discuss your view officers would decide the matter so that you
with him or her, politely pointing out that the can get rid of the man
order could contravene the law (d) tell him that if he tries such means again, you
(d) Ask your colleagues what to do in the would complain to the vigilance department
circumstances and blacklist him

4 . 45
Decision-Making and Problem-Solving

136. You have been called to a house to deal with a 139. Your friend asks you to hand in an important
complaint of domestic violence. When you reach, document in his department as he has to go to
you find a man and woman who are both injured. his child’s school in the morning. When you
You would reach his department you are told that the
(a) arrest the man on the charge of hitting his document has to be signed by the official
wife concerned, and this is a policy matter. You call
(b) ask the woman if she was alright and offer
your friend and he tells you to sign his name. You
first aid and then arrest the man
would
(c) ask both of them to explain what occurred
(a) do as he tells you as it is just a formality
before deciding on further action
(d) tell both to sit quietly while you ask neighbours (b) try to convince the official receiving the
about what happened document that he should overlook the rule as
it was an emergency
137. You are assigning duties to certain junior officers
(c) urge your friend to come and sign the
that include fieldwork and desk jobs. A physically
document as required
handicapped officer, Karan, approaches you and
(d) take the document with you and tell your
explains his difficulty in completing the field job
assigned to him. You would friend later what had happened
(a) tell him you cannot discriminate, and he has 140. A resident of a block of apartments close to a
to do his work heavily wooded park calls you—an officer with
(b) call a few other officers who have desk jobs, the local administration—and tells you that he
@UPSC_THOUGHTS

discuss the situation and work out an


has just seen a couple of suspicious looking
exchange of jobs with Karan
characters carrying what looked like rifles going
(c) complain to the other junior officers that such
into the park. What would you do?
people should not be recruited and assign one
(a) Calm down the resident and tell him he is
of them Karan’s share of fieldwork
(d) tell Karan that you will think over what to do safe in his house and should not bother
and tell him himself about such things
(b) Go straight to the park and search out the
138. A local shopkeeper contacts you and shows you
culprits
a number of fake ` 500 notes that he has been
(c) Tell the resident that you will see to it and
given by a customer in payment of some goods.
ask the police authorities to check the situation
He accepted them in good faith but now his bank
has rejected them. He asks you to look into the (d) Get the persons arrested on suspicion of
matter as a senior administrative officer, as he intending mischief
does not want to get involved with the police. You 141. A disastrous accident at an unmanned level
would
crossing has claimed the lives of several of the
(a) consult your senior officers before advising
passengers of the bus that was hit by the train.
the shopkeeper on what to do
An important step you would recommend for
(b) convince the shopkeeper that it is necessary
the prevention of such accidents in the future
to file a complaint with the police before any
action can be taken would be
(c) tell the shopkeeper to pass off the notes one (a) suspension of the train driver
by one to unwary customers if he did not (b) arrest of the bus driver and the proprietor of
want to involve the police the bus for negligence
(d) ask for details of the customer and interrogate (c) construction of overbridges wherever roads
that person to get information on the source cross railway tracks
of the fake notes (d) that all level crossings be manned

4 . 46
Decision-Making and Problem-Solving

142. There have been several incidents of robbery (c) get water pipelines supplying water to the
along an expressway in your district. As an officer locality checked by the engineers
in charge of administration in the district you (d) do all the above
would
146. Your department has launched a socio-economic
(a) restrict use of the expressway at night
programme. It has been received well but some
(b) issue instructions that vehicles using the
expressway should take armed security guards shortcomings gradually come to light. What would
with them you do as an officer in charge of implementing
(c) get police pickets set up along the expressway the programme?
to step up security (a) Suspend it till the shortcomings are removed
(d) get together local youth and train them in (b) Take various stakeholders’ views on how to
using arms to tackle the robbers deal with the shortcomings and introduce the
rectifications
143. Thick fog envelops the region and more of it is
(c) Withdraw the programme and reintroduce it
forecast by the meteorological office. As an officer
under another name
in charge which of the following actions would
(d) Accept the fact that no programme can be
you take immediately?
I. Temporarily suspend air and rail services. perfect and continue with it as it is
II. Issue advisories in the media that people 147. You have been involved in the investigation of a
defer travel plans in view of the weather. scam in which some local bigwigs are apparently
III. Get the authorities to install modern machines involved. You have almost completed the
@UPSC_THOUGHTS

that can guide air and rail services even in investigation and some important people are likely
thick fog. to be arrested. Media persons press you to let them
(a) I only
know the names in advance. You would
(b) II only
(a) simply refuse to comment
(c) I and II
(b) say you do not know the names
(d) I, II and III
(c) say that disclosing the names at the time
144. Your city is the site of a famous annual festival would harm the investigation
at which huge crowds are expected. As in charge (d) say you have no authority to disclose the
of the security arrangements, you would names before charges are filed against the
(a) restrict the number of people entering the city people concerned
(b) ask the central government to arrange for
extra security personnel 148. You are posted to a district in which the custom
(c) put your police forces on high alert to maintain of child marriage continues in spite of being
law and order and ask hospitals to be ready illegal. One of your colleagues invites you to the
for emergencies wedding of a relative’s daughter who is underage.
(d) round up all suspected criminal elements and You would
take them in preventive custody (a) not go, and make it clear to your colleague
145. You receive complaints from residents of a locality that such marriages are illegal and he should
in the town where you are stationed that for some stop it
time dirty water has been coming in the taps (b) point out to him the illegality of the matter
meant for drinking water. You would and take steps to stop the marriage
(a) get samples of the water checked by (c) take it up with local NGOs to spread
laboratories awareness against the custom
(b) arrange for supply of drinking water to the (d) keep quiet as you cannot interfere in local
locality in tankers customs

4 . 47
Decision-Making and Problem-Solving

149. An old man has misplaced the relevant documents colleges. Amongst the following options, what
that entitle him to subsidised foodgrains. He would you choose to do, if your company has
comes to you for help and asks you to permit him vacancies?
to draw his ration. Procedural policy insists that (XAT 2008)
documents be produced. You would (a) I would hire him for a production or finance
(a) overlook procedure for once job but not for a marketing job, which requires
(b) change the procedures which are cumbersome good communication skills.
(c) help the man to get alternative documents (b) I would ask him to improve his communication
made quickly skills and come back again.
(d) tell him to approach someone else as you are (c) I would not hire him as he might be a burden
helpless on the organisation because of his poor
150. You have been a finance manager in an MNC and communication skills.
felt that gender discrimination at the workplace (d) I would hire him for the job he is good at, and
hampered one’s career growth. Frustrated, you provide training in other areas.
quit the job and started a company. While starting 152. A database software manufacturing company
your company, you decided that you would have found out that a product it has launched recently
an equal proportion of males and females. Over had a few bugs. The product has already been
the last six years, you emerged as a very successful bought by more than a million customers. The
entrepreneur and expanded your business to eight company realised that bugs could cost its
@UPSC_THOUGHTS

locations in the country. However, you recently customers significantly. However, if it informed
started facing an ethical dilemma because you the customers about the bug, it feared losing
realized that female employees were not willing credibility. What would be the most ethical option
to travel across cities and work late hours, when for the company?
the work required them to do so. Male employees (XAT 2008)
did not hesitate undertaking such work. You (a) Apologise and fix up the bug for all customers
started to feel the pressure of reducing the even if it has to incur losses.
proportion of female employees. On the other
(b) Do not tell customers about bugs and remove
hand, you are aware that equal representation
them only when customers face problems,
was one of the strongest reasons for you to have
even if it meant losses for the customers.
founded the company. What should you do as a
(c) Keep silent but introduce an improved product
conscientious entrepreneur?
that is bug-free at the earliest.
(a) See if the unwilling female employees could
(d) Take the product off the market and apologise
be given assignments which do not require
to customers.
travel and involve less overtime
(b) Reduce the number of female employees, as For Q. 153: Read the following caselet and choose the
it is a business requirement and you should best alternative.
not let anything affect your business Om Chowdhury was one of the supervisors in the
(c) Let the status quo continue Fire and Safety (F&S) department of Maqsood Textile
(d) Henceforth hire only male employees Mills. He was a distant cousin to Bhiwani, General
151. You, a recruitment manager, are interviewing Manager (Personnel and Administration). The personnel
Mayank, a hard-working young man, who has and administration department was given the
problems in speaking fluent English. He has responsibility of all personnel-related decisions. It was
studied in vernacular medium schools and often rumoured that Om had obtained the job due to

4 . 48
Decision-Making and Problem-Solving

his cousin’s influence. However, Om was meticulous 153. The options below give combinations of the
in the performance of his duties and didn’t give anyone possible root cause of the problem and the
reason for complaint. It was known that Om was not justifications thereof. Given the details in the case,
much given to talking and kept to himself and to his which one can be inferred to be the best option?
duties. (a) Hiring of Om. Reason: That ensured Om was
All F&S supervisors reported to Rabindra, the shop- perpetually casual towards his duties.
floor manager. The mill operated on a three-shift basis (b) Om favouring to work during night shift.
and Rabindra allocated the supervisors to different Reason: Absence of Rabindra ensured that
shifts. They were required to be present at all times Om could relax.
during the shift operation and carry out scheduled (c) Rabindra’s bias against Om. Reason: Rabindra
checks on machinery and fire fighting equipments. For had been assigning too many night shifts to
some reasons, Om was allocated the night shift more Om while for other supervisors he was lenient.
often than other supervisors. Om accepted these (d) Rabindra jumping to conclusions. Reason: He
allocations without any objection, while it was known should have investigated whether Om had
that other supervisors would often plead and bargain carried out his duties.
with Rabindra to be allocated the day shifts. During the
night shift, keeping awake and remaining mentally Questions 154 to 156 are based upon the following
alert were some of the major challenges faced by the passage.
supervisors. Bhola, an avid nature lover, wanted to be an
@UPSC_THOUGHTS

Of late, Rabindra observed signs of indifference entrepreneur. He dreamt of establishing a chain of huts
from Om. On two occasions he found Om absent from in Chatpur region to cater to tourists, who came attracted
his cabin. Rabindra heard from others that Om was by the beauty and splendour of the Himalayas. However,
often found in a different part of the shop. Rabindra he was appalled by the degradation of the Himalayan
called him to his office and reminded Om of his office environment. He remembered the earlier times when
responsibilities. Om did not counter Rabindra. He everything was so green, clean and peaceful. Now,
promised that he would not be lax in his duties again. greenery was replaced by buildings, peace was shattered
Rabindra also broached the subject with Bhiwani. by honking of vehicles and flocking of tourists, and
Bhiwani called Om to his office and talked on a very cleanliness was replaced by heaps of plastics. Bhola
personal basis. He reminded Om that their family had a strong sense of right and wrong. On speaking
relations made it uncomfortable to all concerned. Om to a few locals about the issue, he realised that the
nodded and agreed to do better. Soon his performance locals were aware of these issues. However, they pointed
became that of a model supervisor. It was often found out the benefits of development: pucca houses for locals,
he went beyond his official duties to sort out the higher disposable income and with that, ability to send
problems of the employees. their children to better schools and colleges, better road
About three months later, Rabindra happened to connectivity, and access to latest technology in
visit the plant during night. As he looked into the F&S agriculture. Most locals wanted the development to
office, he found Om playing solitaire on the office continue. Saddened by the lack of support from the
computer. Rabindra immediately fired Om. locals, Bhola took up the issue with the government.
The next morning Bhiwani called Rabindra and He met the chief minister of the state to find out if the
asked him how he could fire an employee. He suggested government could regulate the developmental activities
that Rabindra reconsider Om’s dismissal. “This decision to prevent environmental degradation. However, the
has already been made. There will be no turning back,” chief minister told Bhola that such an action would
replied Rabindra. slow down the economic progress. That also meant loss
(XAT 2009) of substantial tax revenues for the government. Bhola

4 . 49
Decision-Making and Problem-Solving

needed to resolve the dilemma. Bhola had always 156. Bhola had listed five concrete measures he would
wanted to be an entrepreneur, who could contribute to take to protect the environment if he were to head
the society and earn money as well. However, his the government. Choose the best alternative.
business would also be responsible for destroying (a) Charge environmental cess from all businesses
environment. If he did not set up in business, he would operating out of Himalayas.
not be able to earn money and contribute to the society. (b) Charge cess from anyone who pollutes the
After mulling over the issues, he went to his mentor environment, be it citizens or industries and
‘Guruji’. Guruji realised that it was really a difficult
reward those who have contributed to
puzzle: if one saves the environment, there seems to be
afforestation the most.
no development and if the people and the government
(c) All profit making organisations have to take
sought development, the environment and hence the
the responsibility of afforestation pro-
future of this planet and human beings was at stake.
portionate to their profitability.
After careful thought, he felt that the dilemma could be
resolved. He fixed up a meeting with Bhola to answer (d) Think about maximising the revenues and
Bhola’s queries. forget about the environment.
(XAT 2010) 157. Some environmentalists tired of waiting for ‘green
154. Should Bhola still think of doing business? economics’ to catch up with the society at large
(a) Yes, where there is a will, there is a way. have adopted their own strategies for tipping the
(b) No, saving the earth for our children is more financial calculation in favour of the land. In the
@UPSC_THOUGHTS

important than earning money. forest surrounding Vancouver, where trees are
(c) Yes, Bhola should do business while ensuring being felled for paper to print philosophy books,
no environmental damage is done. groups have used metal spikes hidden in trees to
(d) Yes, but only if the government puts strict prevent the chainsaws from operating safely,
environment regulations in place. pushing up the price of harvesting the trees. In
155. Bhola wanted to advise the government about the Phoenix, Arizona, where mountain nature
new tourism policy. Bhola had developed a few reserves have been encroached on by new houses,
alternatives as given below. Choose the best hooded vigilantes have burnt down the new
alternative. residences. The arsonists, according to the local
(a) Stop environmental degradation by stopping paper, pray before they burn down a house that
the developmental activities. no one will get hurt, thinking primarily of the fire-
(b) Forget about the environment: think about the fighters—the new houses are burnt while still
people as they are the vote banks for the empty. “We don’t pray for ourselves not to get
politicians to come back to power. caught—that’s God’s will,” one is quoted as
(c) Suggest that the government should try to
saying. As per the activists, all aforementioned
promote eco-tourism, which would be
activities seem clearly very principled. But are
controlled and regulated by the government,
they ethical?
as the government could think about the
(XAT 2010)
welfare of a majority of stakeholders.
(a) Yes, arsonists are right.
(d) Suggest that the government should promote
eco-tourism with public private partnership (b) No, they have no justification for damaging
with the involvement of NGOs, so that there other people’s property.
are checks and balances for inefficiencies and (c) No, as it is not taken up in a peaceful manner.
promotion for synergetic efforts between (d) No, as the activities are not carried out in a
government and private entrepreneurs. legal manner.

4 . 50
Decision-Making and Problem-Solving

158. Local villagers have reported an increase in recent principals and trustees of the schools found
months in the instances of illegal felling of trees to be violating the directives
in the forest areas. They accuse a certain (c) issue notices to the schools concerned asking
industrialist of being behind this. You as a district them to explain violation of directives and
officer would threaten them with punitive action if they
(a) tell the villagers to form groups and guard the failed to comply with government directives
trees by turns while you report the matter to immediately
forest officials (d) sue the schools on behalf of the government
(b) arrest the industrialist accused by the
160. You are running a relief camp for people affected
villagers
by floods on the outskirts of a village. After a night
(c) call officials concerned with this subject for
of heavy rains you find much of the provisions
an urgent meeting and devise ways of
damaged and what is left is inadequate to make
stopping the illegal activity and bringing the
a meal for the camp inmates. The local village
culprits to book
shop cannot meet the requirements. You would
(d) increase security presence in the forested area
(a) address the camp inmates and share the
159. Many private schools in your jurisdiction have problem with them and arrive at a solution
been found to be violating government directives (b) reprimand the officials in charge of the
and charging higher fees than the specified limits. provisions for allowing them to be damaged
You would (c) send the officials to the headquarters which
@UPSC_THOUGHTS

(a) recommend to the government to keep away is some 50 km away to get fresh provisions
from the private education sector and (d) talk to the sarpanch of the nearby village and
withdraw its directive officially request him to get villagers to donate
(b) get arrest warrants issued against the or sell some essential provisions to you

4 . 51
Decision-Making and Problem-Solving

ANSWERS WITH EXPLANATORY NOTES

1. (c) The aim of decision-making is to find out the the recommendation of the secretary
best possible course of action. It is always permission is granted.
related to a situation. One may take one
17. (d) The marks scored by Dipti in the promotion
decision in a particular situation and an
test, Part A and Part B, have not been given.
opposite decision in a different situation.
18. (b) Raja fulfils all other criteria except the one
2. (b) Intuition may provide the essential direction
regarding age. But as he is more than 45
for solving a problem in a certain way.
years but less than 50, he is to be referred
3. (c) to the Governing Board.
4. (c) A decision should be based on adequate 19. (a) Maganlal satisfies neither condition 3 nor III.
facts. However, facts must be carefully 20. (b) Rima satisfies all the conditions for being
diagnosed, classified and interpreted in arriving allotted the flat.
at a decision.
21. (d) It is not mentioned whether Shamin owns a
5. (b) In considered opinion, the logic is derived house in Gurgaon. It is also not mentioned
from a careful analysis of the situation. whether he is employed in Gurgaon or
6. (d) 7. (d) 8. (b) somewhere else.
9. (a) One of the principles is to beware of justifying 22. (b) It is not mentioned at which level he scored
@UPSC_THOUGHTS

decisions according to religious faith. the century.


10. (c) Collecting all the facts is difficult. 23. (d) Does not mention any of the conditions under
11. (a) (i) to (v).
24. (d) Anil will be more than 24 years of age on
12. (d) Vimal Cements does not fulfil conditions 4
the stipulated date.
and 7.
25. (c) Arguments I and II are strong. The decision
13. (b) The party does not fulfil conditions 3 and 6,
of the government to restrict the use of
but on the recommendation of the minister
electricity will certainly help the government
permission can be granted.
to solve the problem of inadequate generation
14. (d) Kali Cements does not fulfil conditions 1 and
of electricity. But, at the same time, the
5. Had only condition 1 not been fulfilled, the
decision would violate the right of citizens to
case could have been referred to the minister consume electricity according to their
concerned; had only condition 5 not been requirement. Argument III is weak as the
fulfilled the case could have been referred to machinery to enforce such a restriction may
the secretary of the department. Since no be made available by the government.
case can go to both the minister and the
26. (c) Argument I is strong because closure of
secretary, permission cannot be granted in
educational institutions alone cannot curb
this case.
spread of viral infection. More medical and
15. (b) The party does not fulfil condition 1, but on hygiene measures are needed for this.
the recommendation of the minister, permission Argument II is weak. It is not necessary that
can be granted. Also, though nothing is if educational institutions are closed students
mentioned about condition 3, the minister will visit crowded places as they also are
could grant sanction under (ii). aware of the break-out of viral infection. Also,
16. (d) The party does not fulfil condition 4, but on the argument seems to suggest that students

4 . 52
Decision-Making and Problem-Solving

are the source of the disease. Argument III comprehended by common people; and there
is strong as young persons are more prone is no link between a simpler regime of single
to get affected by the viral infection because rate of interest and ‘larger’ deposits. Long
their immune system is in the developing term deposits are made attractive because
phase. of higher interest rates.

27. (d) None of the arguments is strong. The issue 31. (a) Argument I is strong as people just coming
is export of all types of foodgrains. It is not in contact with infected patients need not to
clear which kind of foodgrains would be most be quarantined unless they are found infected.
needed for the people. Argument I is not Instead of mass quarantining, it is better to
strong as one cannot say there is ‘no other go about selectively on the basis of tests.
Argument II is weak as it is neither the only
way to provide food to its citizens’. Foodgrains
way to control the spread of disease nor a
may even be imported at a lower cost.
sound manner of dealing with the situation.
Argument II is factually incorrect; the govern-
ment may take such a step. Argument III 32. (c) Argument I is strong because mutual funds
seems a draconian step which may be are intermediary between the investors and
acceptable in wartime emergencies or the like the capital market and the investors bear the
but not in the present situation, and is weak risk of changes in capital market. Thus, strict
for that reason. The correct argument would governmental control would protect the interest
of the investors which is highly desirable.
be to ban export of specified foodgrains and
Argument II is also strong because the
import others if required.
@UPSC_THOUGHTS

mutual funds may lose the flexibility needed


28. (a) Arguments I and II are strong. For bringing to work as market players.
uniformity in education system in the country
33. (a) Argument I is strong because if government
and for standardising the quality of graduation
sells profit-making public sector units, it
certificates, a common syllabus for all subjects
would get a higher price in comparison to sale
in graduate courses in all the universities
of loss-making enterprises. The government
could help. Argument III is weak as the
would thus have more money to allocate in
specific requirement of a university has developmental programmes. Argument II is
nothing much to do with the syllabus of the weak as private companies are capable
courses. enough to run big enterprises.
29. (b) Arguments I and II are strong because 34. (c) Argument I is strong because, in this age of
allowing students to take admission in degree intense competition, to remain in the market
courses even if students fail in one or two and to earn more profit the public sector
subjects in HSC will help the students to undertakings should be allowed to adopt hire-
complete their education without a break of and-fire policy. The performance of the
one year and it will also motivate the students employees is the basis of performance of the
for higher studies. Argument III is weak undertakings. Argument II is also strong as
because it is a traditional myopic view. the employee’s interest should not be
compromised in this way.
30. (a) Only argument I is strong because only one
rate of interest for term deposits of varying 35. (a) Argument I is strong because it is true that
duration would discourage people wanting to the objective-type examination does not test
opt for long term deposits. This would a student’s ability to express. Argument II is
adversely effect the liquidity level of banks. weak because no reason is given for calling
Argument II is weak. The interest rates are the objective test as the best method of
not so complicated that they cannot be assessing one’s ability and knowledge.

4 . 53
Decision-Making and Problem-Solving

36. (c) Both the arguments are weak. Argument I is involved in this condemnable act. Argument
weak because it makes a sweeping argument II is weak because the right to know about
about the duty of everyone to serve the one’s unborn child is neither a legal right nor
people in rural areas when only medical a natural right. The right may be sacrificed
students are mentioned in the issue. Argument in the light of the misuse it is put to.
II is also weak because though the argument 42. (d) Argument I is strong as relocation outside city
that it cannot be applied only to the students limits will certainly cause loss of home and
of medicine seems strong, the reason put livelihood of poor people. Argument II is also
forward for it is not convincing—it does not strong as more space is needed to carry out
explain why the condition should not be developmental activities which may be
applicable only to medical students. acquired by relocation of slums.
37. (a) Argument I is strong because pollution control 43. (d) Argument I is strong: as coal is an exhaustible
in the city is an issue to be dealt with and resource, the present stock of coal will not
for this it is a right step to shift factories last long if mining is continued at the present
from cities to the outskirts. Argument II is rate. Argument II is also strong as the
weak because the inconvenience caused to alternative sources of energy resources are
the employees of the factories and their not sufficient to meet the present need.
families can be overcome.
44. (d) Argument I is strong as same kind of clothes
38. (a) Argument I is strong as rewarding a high would help to improve the environment of the
@UPSC_THOUGHTS

score is one way of motivating people. college. Argument II is also strong as college
Motivation is the cause of action and will help students should be considered mature enough
students to work harder. Argument II is to choose their own clothes and should have
merely a restatement of the question and the responsibility to do so.
gives no argument as such.
45. (d) Both the arguments are strong. Switching
39. (d) Argument I is strong as doing away with over to online admissions at all levels all over
plastics now is impracticable. The suggestion the country with immediate effect is not a
given is feasible. Argument II is strong good decision as all the students do not have
because it is true that plastic bags cause easy access to internet. Because of the
water pollution and waterlogging as they are digital divide, students from rural areas would
made up of non-biodegradable materials. find it difficult to use internet for getting their
40. (c) Both the arguments are weak. This is not the admissions. Argument II is also strong. Use
only way to check the ever-increasing of information technology for a hassle-free
population of India. For one, it goes against admission process is a wise decision.
the democratic principle; secondly, positive 46. (a) Argument I is strong. Instead of banning
motivation may be a better method. Argument recruitment of women employees the
II is also weak because China has already companies should arrange for transport upto
enforced the one-child norm. Furthermore, the doorsteps of the women employees. A
even if no country has so far adopted this ban on recruitment of women is a violation
pressure technique, that is no reason why of the fundamental right to employment.
some country cannot take the initiative. Argument II is weak as increasing number
41. (a) Sex determination test during pregnancy has of cases of rape, molestation and cheating
caused female foeticide leading to social is basically a law and order problem which
imbalance. It should be completely banned should be dealt with by the government
as even the rich and educated people are machinery in a more efficient way.

4 . 54
Decision-Making and Problem-Solving

47. (d) Both the arguments are strong. Argument I valid as excessive holidays do affect the
is strong as, with increased longevity, older economy adversely with work being
people are at a loss as to how to occupy postponed.
their time on retirement when their faculties 52. (d) Argument I is strong because correspondence
are good enough. Argument II is also strong courses provide opportunities to pursue one’s
because experienced employees are more studies while employed somewhere. Poor
useful in comparison to new recruits as students avail opportunity to continue their
experience means knowledge. studies. Argument II is also strong as
48. (b) Argument I is weak because conducting of classroom studies provide opportunity of
separate admission tests by private medical interactive learning which enhances the quality
institutes does not ensure suitability for the of education.
job. Argument II is strong because uniform 53. (a) Argument I is strong because nuclear power
criteria for admission ensure equality of plants will certainly help reduce air pollution
opportunity. No medical institute whether to a great extent. The thermal power plants
private or government funded should fix burn coal and cause air pollution. Argument
criteria at its discretion. II is weak: radioactive material used in
49. (c) Argument I is weak because it is not clear nuclear plants cannot be called unsafe for
what the ban on tobacco consumption till the large scale use as the technology of safe
age of 25 will achieve. Argument II is also disposal of nuclear waste is available.
@UPSC_THOUGHTS

weak as the ban does not have much to do 54. (b) Argument I is weak as just being a nuisance
with the understanding of responsibility. Also, (and what kind of nuisance?) to the people
it is doubtful if crossing a certain age means living in big cities is not a strong reason to
an instant gain of maturity or responsibility, remove slums in major cities. Argument II is
or how that connects with consumption of strong as inhabitants are citizens of the
tobacco. country and more importantly, they contribute
50. (a) Censorship by government is not acceptable towards the growth of the nation. Slum
in a democracy; people should be seen as dwellers are mostly manual workers who are
mature enough to make a choice on what an indispensable part of the developmental
they want to see. It is the parents and other process of cities. (Removing the slums
mentors who ought to see that impressionable without providing rehabilitation is no solution
minds are guided properly. to the problem—which is why slums come
51. (b) Only argument II is strong as intermittent rest into being at all.)
is essential to maintain enthusiasm and 55. (d) Both the arguments are weak. Argument I is
motivation in the employees. Holidays help weak because the dismal performance for the
employees to restore their energy. Argument last few years is no cause for changing the
I is weak as different research studies have national sport. Argument II is also weak as
proved that intermittent rest leads to an two or more than two countries may declare
increase in overall production of an a particular sport as a national sport, for
organisation. It reduces absenteeism and example, cricket is the national sport of not
apathy to work. Furthermore, it is not clear only Australia but also of England, and the
from the question whether the five holidays Bermudas.
are to be the only holidays or exclusive of 56. (c) Argument I is strong in the context of the
the weekly offs and other leave due to importance of agriculture and food security in
employees. If the latter, argument I may be the Indian economy. II is also strong, as

4 . 55
Decision-Making and Problem-Solving

uncontrolled depletion of groundwater is an arguments in favour of these students being


environmental concern that cannot be ignored. made to work for the government—as it
Argument III is weak as India does not have applies to those students passing out of
to make decisions merely on the ‘caution’ of ‘government colleges’ in which the education
international agencies; it has to make its is heavily subsidised by government or public
policies on the basis of what is good for its money.
people. 63. (c) There is no certainty about optimum mining
57. (c) I does not make sense; many roads are just because private enterprises can bring in
constructed in phases and there is no reason foreign experts. So I is weak. II is strong
why the work should not get completed. II against privatising the coal mines in the
is also weak as the use of electricity in these country while III is strong in favour of the
cases cannot be termed ‘unnecessary’. III is move.
the only strong argument, as work on roads 64. (a) Each argument is strong in its own way. In
in daytime causes a lot of inconvenience to these days of liberalisation and joint ventures,
commuters. it may not be ‘fashionable’ to give argument
58. (c) I is strong as a rising urban population needs III, but it remains valid, nevertheless.
housing but land is a fixed commodity and 65. (a) Both I and II give valid reasons for not
space is limited. III is equally strong: without banning lottery schemes. As for III, it seems
adequate infrastructure, habitations can a self-righteous conclusion—it is not clear
@UPSC_THOUGHTS

degenerate into slums; so the existing buildings how lottery schemes ‘cause depravity’.
should first be attended to before allowing
66. (b) Decision II will help to prevent such accidents
more buildings to come up without requisite
and manage the crowd in a better way.
infrastructure facilities. II is rather weak, as
Decision I is not a sensible way to tackle
not just builders and developers benefit from
such a problem. Every stakeholder may lose
high rise buildings; buyers too benefit.
something. Private ferry owners may lose
59. (b) I and III make generalisations that cannot be livelihood, government may lose revenue or
accepted: if ‘many’ make mistakes, ‘all’ credibility, and pilgrims may abandon their
cannot, or should not, be penalised. journey. Moreover, it is not established that
60. (d) Environmental pollution is not caused only by private ferry operators were in any way
thermal power plants, so I is quite weak. The responsible for the stampede. So, why punish
justification offered by III is dubious. Just them, and all of them at that?
because others—even if they are developed 67. (a) Students are generally prone to reform. The
countries—do something, there is no need for ethical decision in this case would be decision
India to follow. Other considerations are of I. There is no serious criminal intent, so
importance. II is strong as the power needs putting students behind bars is not a sensible
of the country cannot be ignored. option; it is too drastic. If the act is repeated,
61. (b) I is a dubious argument, and most probably decision II may be considered.
not true in any case. II is a valid argument 68. (d) The immediate objective of the civil authority
in favour of the free supply of books, and should be to restore water supply which can
III is also valid for not supplying free books. be fulfilled by taking decision I. The long term
62. (d) Each argument is strong in its own way. One objective to get compensation can be achieved
may agree more with III if one is of a liberal by taking decision II.
democratic view, but I and II are also strong 69. (b) Decision I is not feasible because pilgrimage

4 . 56
Decision-Making and Problem-Solving

is a sensitive issue and religious freedom has reasons for the increase in volume of air
also been guaranteed by the Constitution of export will not help to reduce backlogs and
India. [However, if the crowds are too much difficulties.
to handle, for safety considerations, some 77. (a) Only decision I is required. The action points
kind of queuing system could be devised.] for meeting the basic learning needs of all
Decision II is the right decision because children adopted in the conference should be
maintaining law and order is the duty of the suitably implemented in India. However, there
government. Big crowds can be managed by is no point in immediately organising this type
the government by deploying more security of conference in the country.
personnel and planning well.
78. (b) Only II is needed. In order to increase
70. (c) Decision I will lead to reduction of government’s attendance, reasons for the absenteeism of
earning. Decision II does not ensure reduction children should be found out. Lack of schools
in inflation because farmers are only one, cannot be a cause of absenteeism of enrolled
often a weak, agent in the chain of supply children.
of produce.
79. (a) Decision I is acceptable. If agricultural
71. (d) The police should first arrest all those who production is increased, exports of agricultural
are caught committing these acts to produce and products could rise. Though the
immediately end the violence, after which a second decision would give the result of
meeting of office bearers may be called. increasing the share of agricultural export on
@UPSC_THOUGHTS

72. (b) Building another road may be a long term paper, the step is retrograde as far as the
remedy for preventing the traffic jam. The economy is concerned. Decisions cannot be
immediate action needed is to send men and based on such narrow considerations.
equipment to clean the road. 80. (c) Neither decision leads to the creation of the
73. (b) Putting on hold all the activities of the huge resources required to develop tourist
company till the introduction of a foolproof places in India.
security system will cause loss to the 81. (b) If government has decided not to provide
company and unemployment to the workers. financial support to voluntary organisations
Thus the workers should be thoroughly the obvious decision would be the exploration
searched before they leave the premises of of other sources of financial support.
the company till a foolproof security system Collaboration with foreign agencies would not
is put in place. necessarily solve the financial problems of
74. (b) It is not practical to restrain fishermen from voluntary organisations.
constructing their huts along the coast line 82. (c) Before taking any action, the allegation should
until the alternative arrangements are not be inquired into. Both decisions I and II are
made. However, the second decision should incorrect.
be immediately followed. 83. (d) Both decisions I and II could follow the
75. (b) It is not practical to ban marriage functions statement. System of education should be
till further notice. The illness was caused by flexible in order to review and revise the
food poisoning so action may be taken to curriculum periodically.
provide the best medical facilities to the 84. (d) Both decisions I and II follow the statement.
affected people. If temperatures are four to five degree Celsius
76. (a) Both airlines and air cargo working together above normal temperature, it is an emergency
may overcome the difficulties. But finding situation justifying I. Provision of water during

4 . 57
Decision-Making and Problem-Solving

extreme heat waves is a dire necessity that big crowd may become restive at what
a government with social commitment should members perceive to be an insensitive
take in the circumstances as per II. response. Moreover, making a promise of
85. (a) To ward off any serious problem the faculty ‘quick action’ seems to be promising something
members should be persuaded not to go on without really being able to accomplish it. The
strike. Decision II is incorrect because crowd has collected precisely because of
suspension of faculty members is not a ‘police inaction’.
solution before a problem arises. (Note the 92. (c) The short circuit did not take place. It was
‘if’ clause.) Again, the question of suspension merely speculation which led to a stampede;
comes only when persuasion and other neither the principal nor the school authorities
methods fail. can be blamed for the incident. Neither
86. (b) The complaints of customers should be decision is suitable.
inquired into by the management to take 93. (b) The collision of running trains took place
necessary steps against the employees. because of the failure of the signalling
Decision I is not correct because this kind system, so the government should
of extreme action is unfair to the employees immediately constitute a task force to review
and may not solve the problem. the functioning of the signalling system.
87. (d) Both decisions may be taken to prevent such Decision I is not correct because the accident
type of incidents in future. was not the result of any mistake committed
@UPSC_THOUGHTS

by the motormen.
88. (a) The given situation calls for decision I, not
decision II. Decision II is impractical and the 94. (b) The government should not resolve the issue
action should only be taken against those of a private airline by passing an order in
found guilty after the expert group inspection. favour of the employees; thus decision I is
not correct. Instead, the government should
89. (a) As the recently recruited officers left after
take steps to end the impasse in order to
attending the programme on branch operations,
help the passenger; thus decision II is
the company should take a bond of at least
correct.
two years. Conducting training programme
after one year of recruitment is not a solution: 95. (c) Both the decisions are incorrect. Issuing strict
training is for making an employee more guidelines after the accident is too late a
efficient; and the employee may still leave decision and relocation of all the markets to
after the training. So the purpose of getting the outskirts of the city will not solve the
benefit from trained employees is lost anyway. problem, besides causing inconvenience to
consumers.
90. (b) High level of inquiry is not needed in incidents
such as a road accident of this intensity. 96. (b) The situation indicates that killing of rhinos
by tigers is the bigger problem. Thus artificial
91. (a) If a very large number of people gather
separation of the habitats of both the animal
outside the police station, it means the people
would be the right decision. Decision I may
are highly disturbed with regard to some
be correct in itself but not in the present
situation. The police authority should form a
context.
team to talk to the representatives of the
people. Advising a big dissatisfied crowd to 97. (c) Neither of the decisions is right. A mistake
disperse and promising them quick action is has been committed by one of the pilots, not
not the right decision: a small group may be by both of them. So revoking the licenses
persuaded to behave in a certain way but a of both the pilots is arbitrary action and does

4 . 58
Decision-Making and Problem-Solving

not make any sense. Secondly, the mistake 104. (c) Both the decisions are not desirable in this
has been committed by a pilot and not by situation. The government’s order to close
the air traffic controller so training of traffic down the operation of the BPO companies
controllers here has no relevance. is drastic without finding out all the facts
98. (d) Both the decisions should be taken related to the matter. Shifting of the BPO
simultaneously. The oil company should pay company to some other place would not solve
compensation for its negligence and efforts the problem of the complaint of inhuman
should be made to shift, as far as possible, treatment.
the marine animals to safer habitats. 105. (a) Decision I is correct. The management must
99. (b) If schools are denying admission to students have a mechanism to identify employees who
from poor families, it is the right decision on violate the circular. The circular may be
the part of the government to enforce a considered to be a kind of warning, but
certain percentage of seats to be reserved summary suspension for failing to maintain
for them as a welfare measure. Encouraging time schedule is unwarranted. Erring
the poor to take admissions only in employees should be given a chance to
government-run schools is a violation of their explain. So decision II is not acceptable.
right to education in a place of their choice. 106. (c) By the time you reach the station or call the
100. (d) Both the decisions should be taken PCR, the train would have met with disaster.
simultaneously and immediately so that the You just have to try something to stop the
@UPSC_THOUGHTS

situation can be improved. train. Shouting to passersby will hardly do


much to help. You may, of course, shout to
101. (b) Decision I is not right because expulsion from
passersby simultaneously for help as you try
college is an extreme action to be taken only
to stop the train.
after appropriate warning and that too in some
serious case of infringement of rules. Decision 107. (d) Option (a) shows you up as a person who
II is right because the violation of the order tries to escape unpleasantness. Option (b)
of the principle for the first time should will not solve the problem, and the suggestion
obviously lead to a warning. [Incidentally, the of a deputation is hardly appropriate to the
issue of freedom to dress as one likes does situation. It is obvious that the railway
not arise here as the dress code is stipulated caterers are in no way responsible for the
in the brochure and students have joined the delay, so option (c) is not going to be of any
college, fully aware of the restriction.] use, as the caterers will not be able to do
anything to solve your problem.
102. (d) Both the decisions are right. Prior notification
will ease inconvenience to the passenger. To 108. (a) You should first stop people beating up
ward off any untoward happening during repair anyone. Even if the boy is a thief, he should
work long distance trains entering the city be handed over to the police; no one has
should be terminated outside the city limit. the right to take the law into their own hands.
103. (a) Only decision I is right because traffic police And you do not know the boy is a thief,
system is already a mechanism to deal with anyway. So (b) is not acceptable. Option (c)
erring drivers. Thus there is no need to can also be tried but only after you try to
establish yet another mechanism for the intervene and are not successful. Option (d)
same purpose. As many motorists are erring, is what most people would do, unfortunately,
the traffic police officials should personally but you should not.
monitor the movement of vehicles on the 109. (c) Your first action in the circumstances should
highway. be to see to the injured person. If you could

4 . 59
Decision-Making and Problem-Solving

leave her in other responsible hands, you 116. (c) The first two options may just result in your
could pursue the snatcher. However, even eviction from the examination hall without a
then, it would be better to ring the police and hearing. Moreover, the first option makes you
give them a good description of the man and also a cheat. However, if the invigilator does
the situation. not do his duty, you may be forced to follow
110. (b) Your priority would be to see to the safety the second option. The last option expresses
of your friend’s child. It would be irresponsible apathy and cynicism that must be avoided
to leave the child in a restaurant in the if one wants to do anything worthwhile in this
circumstances while you go and find out what world.
it is all about. 117. (d) Obviously something is amiss if a person
111. (c) On humanitarian grounds you should see that runs away in spite of being told she has
the injured man gets medical aid, even if it dropped a phone. So (a) is just stupid. But
was not your fault, and the vendor has no (b) is not very clever either, as the phone
right to be where he was. Offering money for may be an explosive device and may explode
the damage is secondary. It would, of course, causing much damage even if you are quick
also be necessary to point out to the vendor enough to throw it. Remember it is an
that he was adopting unsafe practices by exhibition you are at and the place may be
standing on the road, but that would have to crowded. Don’t be in a hurry to mark (c),
be done later. If you were able to note the however, as it is only part of the answer. It
@UPSC_THOUGHTS

number of the car that swerved at speed, that is necessary to warn off other people.
would be something to report to the police. 118. (a) Rescuing the injured must be the priority.
112. (a) 113. (b) Responses (b) and (c) would lead to delay
in the rescue of people. Response (d) is not
114. (c) If the man is badly injured and bleeding from
relevant in this context.
the head, it might not be a good idea to move
him without medical expertise. The police, 119. (d) You know that your friend is already in the
once informed, would bring an ambulance. ICU, so under proper medical care; you could
hardly do anything for him till further news
115. (a) This would probably be the best course of
comes. But if the sister has rung you up,
action. It might frighten away the intruder, but
some expectation of what is called ‘moral
at least it would prevent robbery or other
support’ is implied. Rushing off immediately
damage intended by the intruder. On the other
to the hospital, dropping your official duties,
hand, you may be able to gather some
is not behaviour expected of a responsible
helpers with whose cooperation the intruder
officer; moreover, there is a lot of protocol
could be nabbed. Ringing up the police is fine
involved in these meetings, and one cannot
but what happens before they turn up? It is
simply delegate one’s function at short notice
better as a simultaneous or a second step.
to someone else, especially as the situation
Going and tackling the intruder on your own
does not quite call for it. The best option is
may not be exactly the most sensible thing
(d). But suppose you had received a call that
to do as he may be armed, and you may
your friend was lying on the road after the
even be killed. The situation does not quite accident? You could use your contacts to
demand martyrdom. This is one of those ensure that he/she is attended by medical
questions which do not really have a ‘right’ personnel immediately, and request some
and ‘wrong’ answer – except for option (d) close friend or relative to see to things,
here which is truly wrong. keeping in touch by phone to make sure that

4 . 60
Decision-Making and Problem-Solving

things are being attended to. It sounds a bit the tent next to it. The other events can be
cold, but as a responsible officer, you have tackled after you have seen to the fire.
to work out the best possible plan without
125. (a) It is best to nip rumours in the bud and tackle
leaving off your duty.
the complaining group without changing your
120. (c) This is the best option. Option (a) may not decision if that decision has been based on
be really ‘wrong’ but it does not seem entirely principles. And it is best to talk to all the
‘right’ either to accept gifts – cash or members of the disgruntled group together.
otherwise – for doing one’s official duty either. It is necessary to convey the message that
Option (b) seems to suggest that you would you are not likely to be cowed by their
be quite ready to accept the cash outside behaviour, even as they get an opportunity
the office. Option (d) is unnecessarily self- to get an explanation for your action. Option
righteous: the act of the villager does not (b) is underhand behaviour and puts you in
quite accord with the act of offering of a bribe. a dubious position, besides being likely to
121. (c) It is clear that leave cannot be granted in prove harmful to the organisation in the long
the circumstances, but (c) is the best way run. Option (d) does not show a mature
to refuse leave. The question of approach to the problem at hand. If your
‘compassionate grounds’ do not apply in this decision is based on right principles, a
case, even if the family disappointment could complaint against you is not going to work,
be great. so why anticipate the others’ action? Option
@UPSC_THOUGHTS

(c) is not advisable as the rumours cast


122. (b) This is the best option, as work on both aspersion on your work ethics, and letting
projects needs to be done. As the earlier them continue till they die a natural death
project is about to be completed, the senior may, instead, boomerang on you.
members of the team would be well conversant
with it to handle it with some supervision from 126. (d) If you are a responsible worker, (d) would
you. You cannot put any of the projects on have to be your option, however tired you are
hold. or may be desirous of rest. It is not just a
matter of duty, as leaking oil pipes could be
123. (c) You don’t have much choice, anyway. Having dangerous to the life and property. Leaving
threatened her earlier to take the matter to a note on your senior officer’s table may not
the senior officer if it happens again, you ensure timely action. A sign of warning is
have to do so. In any case, what she is doing hardly sufficient in the circumstances. Just
is unethical and by keeping silent, when you going home without any further concern would
find that you have not been successful in be unethical and irresponsible behaviour.
dissuading her, is unethical on your part as
well. Option (b) is not entirely wrong – it 127. (c) For any normal message, this would be the
means you believe in giving a person more correct thing to do. There is no need for
than one chance, but it would be more instructions from senior officers in such a
appropriate if it was your own equipment that matter. Options (b) and (d) show that you
was involved and not the office equipment. take your work rather casually. If, however,
Option (d) is sneaky and shows that you the matter was confidential or only for the
would not be ready to face up to person to whom it was to be sent, you are
unpleasantness even when it is required. in a slightly different and more difficult
position because your carelessness may
124. (d) The garbage container fire could easily be a prove harmful to the organisation. In that
danger to many people if the fire spreads to case, option (a) would have to be taken.

4 . 61
Decision-Making and Problem-Solving

128. (b) The best decision would be to persuade both cannot blindly obey orders when you know
to work together; such personal grudges them to be against the law. Option (d) is not
cannot be allowed to fester to the extent of going to resolve the problem, though it might
adversely affecting the organisation’s work help you to understand your colleagues
and goals. Threatening disciplinary action is better.
inappropriate in this case. Option (a) is not 133. (b) You have to take such calls seriously even
to be resorted to before you have tried other if they are anonymous and eventually turn out
options to get the two to work together. to be hoaxes. You cannot afford tension of
Option (d) is not feasible in a project where a communal sort to build up.
team work is almost always required. 134. (a) This is the first thing you need to do. You
129. (b) This should get the groups to break up, as do not know the other customer is correct;
they are just political supporters and not after all, the officer may have a weekly/
mobsters (though some may not accept the monthly account with the dhaba owner.
Response (c) would make you look silly if
difference). Option (c) can come if (b) fails
there is some justification for the officer’s
to get the result, though better than the lathi
action. Response (d) would not be correct,
which leads to avoidable injuries would be a
especially when another customer has made
water cannon – if we had the facility. Option
his comment.
(a) has no relevance, as the election
commissioner is not in the picture at this 135. (d)
@UPSC_THOUGHTS

stage to disqualify parties because their 136. (c) You cannot presume anything; before arresting
supporters get into a fight. Option (d) is not a person you need to clarify what happened.
an immediate action to take up; it can come You cannot be sure that the woman was
later. entirely innocent, as the man is also injured.
130. (a) This is the only option for you. Option (b) 137. (b)
can come later, and you too cannot escape 138. (b) It is absolutely necessary that circulation of
responsibility in this matter, as it would have fake notes must be brought to the notice of
been your job to see that officers under you the police. You are yourself a senior officer
did their work properly. and should decide on your own and not run
131. (a) This is the first thing you have to do. Options to your senior officers as the first step
(b) and (c) can be done later. Option (d) need [response (a)]. Response (c) is unscrupulous
not be relevant at all. and absolutely unacceptable in an officer.
Response (d) is taking upon yourself the work
132. (c) At the very first stage, option (b) should not
that the police is trained to do.
be adopted. Maybe it is a genuine mistake
– it could happen to anyone. And one mistake 139. (c) This is the best thing to do. Response (a)
does not mean incompetence. Maybe the is fraudulent, even if the procedure is a
senior officer knows something you don’t – ‘formality’; further it renders you liable to
some exceptions in the law that you are not charges of forgery or impersonation. Response
aware of. You should clarify first before (b) is not likely to work if the procedure is
assuming corruption or incompetence. Of a policy matter. Response (d) does not meet
the condition that the document is important
course, if you are not given a satisfactory
and had to be deposited that morning—as
explanation, or if the officer evades the
implied in your friend’s request to do so.
question or puts you down, you might adopt
option (b); you would be ethically obliged to 140. (c) Response (a) implies that you do not take
do so. As a responsible officer yourself, you the person seriously, and after all he may

4 . 62
Decision-Making and Problem-Solving

have seen something worth investigating. 151. (d) The best decision would be to select Mayank
Response (b) is hasty action and, if the men for the job he is good at, and provide training
are armed, may prove avoidably dangerous; in other areas. Communication skills is a
on the other hand, you do not know if they learnt skill and may be improved by giving
are ‘culprits’. You cannot get people arrested training. He cannot be hired for a production
without making sure of what they are up to. or finance job as it not clear whether he is
141. (d) Response (c) is impractical. Responses (a) good in these areas or not. Asking him to
and (b) do not relate to ‘prevention in the improve his communication skills and come
future’. back again is useless as the company has
a vacancy now. Not hiring him, thinking he
142. (c) Responses (a) and (b) are impractical.
would be a burden on the organisation
Response (d) is somewhat outside legal
because of his poor communication skills,
provisions; it is the state’s duty to provide
may not be a good decision as it is not
its citizens with security and order.
specified that the company has vacancies
143. (c) Action III is not something that is possible for only those having good communication
immediately. skill.
144. (c) 145. (d) 146. (b) 152. (a) The most ethical decision for the company
147. (c) Response (a) is too rude, and you need the would be to apologise and fix up the bug for
media to remain friendly. Response (b) would all customers even if it has to incur losses
not be believed. Response (d) is also because the mistake has been committed by
@UPSC_THOUGHTS

acceptable, though (c) is better. the manufacturing company. This is the best
way to maintain credibility in the given
148. (b) Response (c) is a long term procedure. An
situation. Not telling customers about bugs
illegal activity ought to be prevented, especially
and removing them only when customers face
when minor children are concerned. It is
problems, even if it meant losses for the
doubtful if (a) would achieve the purpose of
customer, is not a good decision because it
preventing the marriage. Response (d) is a
would cause annoyance and loss to the
refusal to take hard decisions.
customer, hence loss of credibility of the
149. (c) Procedures cannot be overlooked if company. To keep silent but introduce an
alternatives exist. Nor can you easily change improved product is also not a good decision;
procedures that are laid down. So options (a) besides it being unethical, the customers may
and (b) are not acceptable. Option (d) shows be doubtful of the new product as well. Taking
you as shirking responsibility and not taking the product off the market and apologising
a decision. to customers is only a partially ethical
150. (a) You should try this before making the hard decision as there is no remedy to the
choice of reducing the number of female customers who have already suffered.
employees, as female employees in this
153. (d) Option (a) is not correct as Om was casual
country specially have problems with long
towards his duty only for a short period. As
travel and overtime. The status quo cannot
soon as he was reminded of his duty he
be continued as the situation demands a
change or the business will suffer. Henceforth started working as a responsible employee.
hiring only male employees is not a right His casual attitude was temporary. Option (b)
decision as it goes against your basic is wrong because Om never showed any
principle in starting your own business. This dislike or any particular liking towards the
can be your decision only if all other night shift. Option (c) is also not true as
alternatives fail. Ravindra assigned more night shifts to Om

4 . 63
Decision-Making and Problem-Solving

as a matter of chance. Unlike other 157. (c) This is one of those questions to which there
employees, Om never bargained or pleaded can be more than one correct answer. Option
with Rabindra to get the day shift. There is (a) may even be considered right if a person
no indication here that Rabindra had a bias is a ‘fundamentalist’ type of environmentalist.
against Om. Option (d) is correct as Rabindra However, on the whole, (c) is most acceptable
jumped to the conclusion that Om was casual if we consider ethics to be beyond mere law
towards his duty when he saw him playing
and the legal. Damaging other people’s property
solitaire on the computer. Before coming to
is illegal as well as unethical. But that is not
the conclusion he should have conducted on
the only thing the activists do. They also use
investigation into the matter.
metal spikes hidden in trees to prevent
154. (c) Earning money is also important if it can be chainsaws from operating safely; this is a
done without damaging the environment or
kind of violence as is the burning of property.
causing other harm.
It is not ethical to adopt violent ways to get
155. (d) Involving NGOs will ensure civil society even with what you consider wrong.
participation which is necessary for such
158. (c)
projects to be successful and follow required
parameters. 159. (c) This ought to be the first step.
156. (b) This is the most fair suggestion. 160. (d)
@UPSC_THOUGHTS

4 . 64
5
GENERAL
MENTAL
ABILITY
@UPSC_THOUGHTS

General mental ability is a comprehensive term: it involves


the ability to remember facts and principles, apply acquired
knowledge to emergent or new situations or problems, and
analyse, evaluate and come to decisions. Clearly then, mental
ability is required of the candidate in solving problems relating
to every subject, every field of activity. However, since this
particular examination prescribes General Mental Ability as a
distinct discipline, we may conclude that it refers to certain
specific skills. Prior to 2011, the UPSC syllabus included in this
section question on numerical ability, verbal ability or the
ability of comprehending the meaning of words and symbols,
spatial discrimination or skill at recognising shapes and
manipulating figures, and reasoning ability which includes
analysis and deduction—whether from statements or graphical/
diagrammatic representations. But in the syllabus for the 2011
examination, there is finer differentiation. With logical reasoning,
analytical ability, decision-making and basic numeracy getting
their own slots, what remains must necessarily be considered in
this section, General Mental Ability.
General Mental Ability

@UPSC_THOUGHTS

5.2
General Mental Ability

General Mental Ability


ORIENTATION

SCOPE OF THIS SECTION A Chart to Help You


In many questions based on series, analogy,
General Mental Ability may be considered in two broad
classification or coding based on alphabets and their
sections—(i) verbal and non-verbal. But often enough numerical equivalents, it helps to remember the order
the two may mix, as in locating the answer to a verbal of letters with their respective numbers, i.e., A—1 to
problem, you may need to sketch a diagram of the given Z—26 and vice versa Z—1 to A—26 as given below
data so that it becomes easier to solve the problem. in the chart:
In the verbal portion, there are problems on series,
1 2 3 4 5 6 7 8 9 10 11 12 13
classification, analogy; questions involving rule
A B C D E F G H I J K L M
application, coding-decoding, interpreting symbolic 26 25 24 23 22 21 20 19 18 17 16 15 14
relationships, and working out directions. There may 14 15 16 17 18 19 20 21 22 23 24 25 26
also be problems relating to the calendar and clocks. N O P Q R S T U V W X Y Z
Some language skills and basic arithmetic will be 13 12 11 10 9 8 7 6 5 4 3 2 1
required for solving some of these problems.
These numbers may shift—say A may be 3. In that case,
Visual-spatial or what is more commonly known
@UPSC_THOUGHTS

Z will be 2. Imagine the alphabets and numbers in a


as non-verbal reasoning deals in shapes and graphical circle.
relationships. You are required to understand diagrams.
Abstract reasoning test questions are in the form of and cut as indicated. There are many variations, but
symbols arranged in rows or squares. You have to work in every case you are required to imagine what would
through the three distinct steps of (i) identifying the happen.
different sorts of symbols and deciding the common We give here examples of most of the types of
feature/s among them; (ii) identifying the pattern questions along with instructions on how to solve them.
governing the arrangement or ordering of the symbols; Short-cut methods are included.
and (iii) pointing out the next item or part of the
sequence on the basis of the logic of the pattern you
have discovered. Perceptual reasoning involves the
SERIES
understanding and application of information presented In such problems the items given—whether they are
in a diagrammatic form. Questions on non-verbal letters of the alphabet, numbers or figures, or a mixture
analogies require you to work out the answer by of all these—follow a certain pattern or sequence. The
determining the visual logic of the information provided. pattern is to be determined before the answer can be
Visual logic and a visual problem-solving strategy chosen.
are required in non-verbal problem-solving. Movement
of figures, similarity/dissimilarity in shape, trans- Alphabet Series
formation of shapes—these are some examples. Also In an alphabet series, letters are given in a particular
included is purely spatial reasoning which measures sequence or order with gaps that follow a certain
your ability to manipulate shapes in two dimensions, pattern. The pattern may be uniform—a gap of one letter
or to visualise solid objects presented in two-dimensional (or two, three) each time—or it may involve a
patterns. You may be asked to choose from the progressively increasing or decreasing skipping of letters,
alternatives the correct component to fit a missing or a mixed pattern of skipping one letter, then two, then
portion in the given pattern. You may have to visualise one again between successive terms. There may be two
how an object looks when rotated to various degrees: (even three) interlinked series, each one along its own
or reflected—in a mirror or in a water body; or if folded pattern.

5.3
General Mental Ability

● Take the series As the term we require comes in the series A-F-M,
A D G J we may assume that 6 + 2 = 8 letters now need
To find the next term, you need to reason out that to be skipped from M. Skipping N O P Q R S T
taking the first two terms given here, A and D, we U, we get V.
see that B and C have been skipped. Check if that However, note that the link between the two series
pattern is repeated to get the next term G. Yes: E is also visible.]
and F are skipped after D to get G. Between G and ● The series may be of grouped letters, e.g.,
J again, two letters, H and I, are skipped. So to get QPO SRQ UTS WVU
the next term to continue the series, we skip two
letters after J: How do we get the next term?
The series is made up of groups of three letters each.
To continue, the next term must be according to the
A (BC) D (EF) G (HI) J (KL) M
pattern of each of these groups.
Gaps in→ 2 2 2 2
Each letter group shows consecutive letters in
letters
reverse alphabetical order. The first letter of one
The next term in the series is M. term becomes the last letter of the next term. By that
The series progresses by skipping two letters token, the next term’s third letter will be W.
alphabetically after every term. Consecutive three letters in reverse alphabetical
order beginning with W will be YXW.
● Now, take the series
A D F J M R The pattern of skipping letters from one term’s last
At first sight the string of letters seems baffling, letter to the first letter of the successive one is 3
@UPSC_THOUGHTS

with no apparent pattern, except that no reverse letters in alphabetical order. Thus
order is visible. Let’s work in the gaps, and assign QPO (PQR) SRQ (RST) UTS (TUV) WVU
numbers, and work out the difference in the gaps If we add YXW we see the pattern is maintained:
of letters: WVU (VWX) YXW
This is a shortcut for the technically accurate
Letters description of the pattern which is given below.
skipped
Difference The next term to continue the series is YXW.
between Each group is of three consecutive letters in reverse
gaps
alphabetical order.
A pattern emerges: the gaps in the letters between
Each letter skips one letter in alphabetical order to
the terms show an alternating pattern of decreasing
move into its respective position in the next group
by 1 and increasing by 2. The next turn is of
(or term).
decreasing by 1. So if the last gap between terms
was of 4 letters, this time the gap will be 4-1, i.e., The gap between the last letter of each term and
3 letters. Count off 3 letters after R (STU) to get V. the first letter of the next term is of 3 letters in
alphabetical order.
The next term in the series is V.
This pattern governs the series.
[The same result can be arrived at by considering
the given series as comprising two intermingling ● The series may be made up of a string of letters
series: of alternate terms, each progressing by (generally, small letters) repeated according to a
skipping letters in an increasing order of 2. Thus pattern. These questions are usually of the multiple
5 7 choice type with answer responses given from
(E F G H I) (K L M N O P Q) which you have to choose the correct letters in
order.
A D F J M R
The easiest method to solve these questions is try
(B C D E) (G H I J K L) out the responses to see which one forms a pattern.
4 6 Usually, a cluster of letters gives you a clue.

5.4
General Mental Ability

Take the series limits to the number of such patterns, but the tools you
c – b b a – c a b – a c – a b – a c need to work with are the basic arithmetical functions
There are two clusters of letters bba and cab. If we of addition, subtraction, multiplication and division.
put a in the first blank, we get c a b b a. Assuming And, since you may be required to do without calculators
that b a follows c a b, let’s put b in the third blank; in these tests, you must learn to do these calculations
we get both accurately and fast. The pattern could be one of
c a b b a – c a b b a c – a b – a c simple arithmetical progression.
If we take into account the longest cluster of letters, It could skip one/two/three number/s in sequence:
a pattern emerges: c a b followed by b a c. Fill in 1 3 5 7 9 11 ... (one number skipped)
c in the second and third blanks, and b in the fifth 2 6 10 14 18 22 ... (three numbers skipped)
blank: 5 8 11 14 17 ... (two numbers skipped)
c a b b a c c a b b a c c a b b a c It could be in regular decreasing order:
The letters required in order are a c b c b. 12 10 8 6 4 ...
It could be in an irregular decreasing order:
Format may differ Letter series may assume different
12 11 9 6 2 ...
shapes, but the principle remains the same.
The terms could be multiples of a number, or each
● Which letter will replace the question mark (?)
term could be divided successively by a particular
in the last figure?
number; terms could be squares or roots of consecutive
E N T ? numbers, or there could be a combination of all these
operations.
@UPSC_THOUGHTS

A H L E
● Take the series
The gap between the top and bottom letters is 24 20 16 12 8
of 3, 5, and 7 letters, respectively, in reverse What will be the next term?
alphabetical order. We note that the gap
The terms decrease in some order. The difference
increases by 2 letters at successive steps in
between the first two is 4 (24 – 4 = 20); the same
reverse order from top to bottom. So the letters
difference is found between 20 and its successive
skipped between E and (?) will be 7 + 2, i.e.,
term, 16.
9, in alphabetical order [as E is the bottom term,
and (?) is at top.] The required letter is [E The term to replace the question mark is 8 – 4 = 4.
(FGHIJKLMN)] O. ● Now take the series
● Which letter will come in place of the question 64 32 36 18 22 ?
mark (?) in the following circle? Looking at the first two terms, you note that dividing
the first by 2 will get the second. The same is not
true for the second and third, however; there, an
A C
addition of 4 seems to be the rule. Could these be
? E
the alternate steps?
M G Check.
K I 64 32 36 18 22 ?=11
÷2 +4 ÷2 +4 ÷2
The required letter will be O. The letters The pattern is clear: the next step is of dividing
represented are alternate letters in alphabetical by 2. The term to replace the question mark is 11.
order.
Format may differ As in alphabetical series, number
Number Series series may also be in different formats. The principle,
A number series involves a sequence of numbers however, remains the same. Thus there may be numbers
following a specific identifiable pattern. There are no in squares, triangles or circles.

5.5
General Mental Ability

● Find the missing number in It is easier tackling this type of series problem in
multiple choice mode, as the answer responses are
1 3 7 15 ? given from which you can choose the correct one.
240 48 12 4 ?
Figure Series
When figures are involved, the question format is
31 generally a set of problem figures which establishes the
The answer is:
pattern and a set of answer figures from which the next
2
figure to complete the series is to be chosen.
Pattern Top row : add 2, 4, 8, and 16. Some of the things you need to note are whether
Bottom row: divide by 5, 4, 3 and 2 (i) there is a change in the shape of symbols and,
if so, according to what pattern.
● Find the number to replace the question mark
(ii) there is a steady increase or decrease (or
(?) in
fluctuating increase/decrease) in the number
of symbols at each step.
1 2 (iii) The symbols within each figure move in a
? 2 particular direction—clockwise/anti-clock-
8 4 wise—and by what degree.
All these changes may occur at once, or one by one
The number required is 32. at every step, or in alternate steps.
Pattern Multiply the preceding number by the ● Which figure will come next?
following number to get the next number.
@UPSC_THOUGHTS

Problem Figures
1 × 2 = 2; 2 × 2 = 4; 4 × 2 = 8; 8 × 4 = 32

Letter-Number Mixed Series


In a letter-number mixed series, the letters and numbers
are presented in combination. The letters and numbers Answer Figures
may form a series on their own; i.e., the letters progress
according to a pattern, and the numbers progress
according to another pattern. Or, the letters and numbers
could be related to one another by virtue of the alphabet- (a) (b) (c) (d)
number grid. The principles of reasoning, however, are The answer is (d). There are parallel lines—two
the same as those applied in the letter and number in the first, three in the second, then two in the
series separately. third box. So we choose three parallel lines for
● Take the series the next figure. But not (b) because the circle is
missing in that response.
C3 D4 F6 I9 ?
● Which figure will replace the question mark?
Taking the letters first, the gaps are as follows:
C D (E) F (GH) I Problem Figures

So now three letters (JKL) will be skipped after I to


get M.
As for the numbers, they clearly correspond to the
positions of the letters in the alphabet. As M is the Answer Figures
13th letter of the alphabet, so the number in the
required term is 13.
The term required to replace the question mark is
M 13.

5.6
General Mental Ability

Observe the pattern. The blackened quarter of Now take the question item
the circle moves clockwise and will next be in
Writer : Book as — : —
the lower left quarter—which is shown by all (a) Book : Publisher
the answer responses. So you need to find out (b) Printer : Printer’s ink
how the small circles move. They move in an (c) Building : Builder
anti-clockwise direction. So the two small circles (d) Tailor : Suit
will be in the left half of the bigger circle. It is
Here there are two gaps you have to fill. Find a pair
to be noted that one small circle is not seen
that shows the same relationship as the two given
when it comes into the darkened portion.
words.
The correct answer response is (a).
A writer writes—or produces—books. The pair that
is similar is tailor—suit: the tailor stitches—or
ANALOGY produces—suits. Correct response is (d). The
relationship is one of worker and article produced.
The relationship between groups of letters, words,
numbers or figures is the concern of analogy questions. You may wonder why not (c). The order of words
Find the relationship between the given items, and you is reversed in (c). The product comes before the
can correctly locate the answer. maker. You need to keep the pair of words in the
correct order.
Verbal Analogy ● Now take a set of letters in analogy.
Relationships may be based on antonyms, synonyms, LOM : NMK : : PKI : ?
cause and effect, worker and work/article/tool, tool (a) RIG
@UPSC_THOUGHTS

and object, whole and part, part and part, difference or (b) RIJ
intensity, function, gender, age, and many others. In (c) RHJ
verbal analogy items, you have to be careful of the part (d) SHG
of speech a given word belongs to, and the exact The answer is (a). See how the groups of letters are
relationship that bears between the words. Observe the related: L-N, O-M, and M-K. The same relationship
sequence of items and the grammatical relationship, if exists between P-R, K-I, I-G.
any, and choose the answer response which gives the
same type of relationship. ● Take the item
What is related to ‘fist’ in the same way as ‘grit’
● Take the question item
is related to ‘teeth’?
Chennai : Tamil Nadu :: Bengaluru : ? (a) blow
(a) Trivandrum (b) clench
(b) Mysore (c) boxing
(c) Karnataka (d) open
(d) Chandigarh
Here the relationship between ‘grit’ and ‘teeth’ is
The question head may also be posed as:
one of action and body part. However, note the tone
Chennai is to Tamil Nadu as Bengaluru is to?
of action: it is one of holding hard together. With
Or as:
fist, a similar action would be ‘clench’. So (b) is the
Chennai is related to Tamil Nadu in a certain
correct choice.
way. To which of the following is Bengaluru
related in the same way? Number Analogy
The answer will be the same—(c). You have to spot In questions on number analogy, numbers are given in
the relationship between Chennai and Tamil Nadu. pairs, or in groups or series.
Chennai is the capital of Tamil Nadu. So, with One has to carefully examine the relationship
Bengaluru given, you have to choose the state of between the given pair or group of numbers and
which it is the capital. identify the numbers with the same relationship or
The relationship is that of capital and state. same properties from the given options.

5.7
General Mental Ability

● Sometimes two numbers are given which are ● Now another example:
related to each other in a certain way.
Problem Figures
9 : 81 : : 8 : ?
(a) 46 (b) 64
(c) 70 (d) 80
In the given pair of numbers, the second number
is the square of the first number. Since (b) is the Answer Figures
only option that shows the same relationship
with 8, that is the correct answer.

● Sometimes, a group of numbers is given, all of


which show a common property. You are to
select a number from the given options which
has the same property as the numbers in the
given group. Study the first two figures to see how they are
related, or rather in what way the second figure
(18, 36, 63, ?)
differs from the first. You note the symbols at the
(a) 19 (b) 35 side move to the top and bottom of the main figure,
(c) 54 (d) 22 and the oval symbols move outside, each on its own
Here the given numbers 18, 36 and 63 are side. Now observe the third given figure and see
which answer response shows a change similar to
multiples of 3 and 9. Similarly, in alternative (c),
@UPSC_THOUGHTS

what is observed in the first pair of figures.


54 is also a multiple of 3 and 9.
The symbols at the side move to top and bottom.
Response (a) seems to fit. Yes, and the small
Figure Analogy
triangles move out on their own sides above and
In the non-verbal form of analogy questions, the below the symbols that have moved from the sides.
relationship may be of size, shape, number of items, and
so on. The same points that you have to keep in mind Option (b) is not correct as the lines have been
broken.
for figure series need to be kept in mind here—but in
terms of ‘relationship’ between the figures.
● Let’s take an easy example: CLASSIFICATION/ODD-MAN-OUT
What is the figure to fit (?) Questions on classification test your ability to observe
differences and similarities among various items—
Problem Figures
words, group of letters or figures. Out of a set of items,
you are to find the term that is different from the others.
? Common sense is the basic requirement in getting the
right answer.
Answer Figures Verbal Classification
● Take the group
(a) MOQ
(b) GEC
(a) (b) (c) (d)
(c) SUW
The relationship is one of size and number. The (d) TVX
circles increase in number and differ in size. So the Establish the relationship between the letters of the
rectangles should increase in number and be of groups. If at least two are alike, you have to locate
different size. The answer is (d). the group that shows a different order.

5.8
General Mental Ability

MOQ → M (N) O (P) Q ● Take the following set


Alternate letters have been selected. (a) 1754
GEC → G (F) E (D) C (b) 2690
Alternate letters here too but in backward order. (c) 7324
Take the next group (d) 9611
SUW → S (T) U (V) W
The same order as MOQ. So (b) is likely to be the The numbers do not seem to fit any order. So go
odd-man-out. on to see if they add up (or multiply) to something
Check (d) same.
TVX → T (U) V (W) X Let’s take addition:
Order is the same as MOQ and SUW. So (b) is the 1 + 7 + 5 + 4 = 17
confirmed odd-man-out. 2 + 6 + 9 + 0 = 17
That seems likely.
● Now take
7 + 3 + 2 + 4 = 16
(a) HSRI
(b) MVUN That looks like the odd-man-out.
(c) OLKP Check (d):
(d) PJQX 9 + 6 + 1 + 1 = 17
It helps to keep the alphabet—the order of letters— So, (c) is the odd-man-out.
in mind. Observe that the first three groups show
an easy relationship between the first and last Figure Classification
letters. In non-verbal items, locate the relationship—observe
@UPSC_THOUGHTS

H . . I
repetition pattern, size, shape, direction or inclination
M . . N
of figures. A combination of these factors may also
O . . P
occur.
So they must form a group.
But P . . X do not show that relationship. ● Take the figures—
So (d) is likely to be the odd-man-out.
Now check for the middle letters.
SR → RS in reverse
VU → UV in reverse (a) (b) (c) (d)
LK → KL in reverse The answer is (c). The other figures have three lines
But JQ? No such relationship. intersected by a fourth line, while in (c) two lines
It is confirmed that (d) is the odd-man-out. intersect one another.
Classification questions may involve words as well.
● Take the figures—
● Choose the odd man out.
(a) Mars (b) Mercury
(c) Moon (d) Earth
The answer is (c); the others are all planets.

Numerical Classification (a) (b) (c) (d)


You may have to see whether the order of arrangement
is the same. Look out for consecutive numbers, odd/ Figure (d) is the odd-man-out. While the movement
even numbers, prime numbers, sum/product of the end of all the circles in the figures shows clockwise
numbers, and so on. There are infinite variations. direction 90° at a time, the arrows are wrong in (d).
For one, they are in the same position as in (c); for
● Which item does not fit?
another, their heads are in opposite directions,
(a) 134 (b) 235
(c) 346 (d) 135 which is not so in the other figures.
The answer is (c); others have 3 as the middle digit. So (d) is the odd-man-out.

5.9
General Mental Ability

of circle, triangle, star, etc. Study the example given


MATRIX AND OTHER PATTERNS
below.
In this type of questions, there is a figure in the form 7
of a matrix or any other form divided into various parts 63
in a set pattern. Each part bears a number except one.
2 14 ? 9
You have to understand the pattern of distribution of
the numbers in the figure and find the missing number. 30
15
Matrix
The numbers are arranged in rows and columns. (a) 33 (b) 145
Vertical arrangement of numbers in a matrix forms the (c) 135 (d) 18
columns, and the horizontal arrangement the rows. The Here, the numbers in the circle outside the square
relation between the numbers may be between those in have a relation with the numbers inside the square.
a column or between those in a row or both. You need You need to work out the relationship by trying
to decipher the relationship from the given numbers out various operations—do they add up? Is
and find the number that will come in the blank space multiplication involved? Is it a combination of
from the given options. operations? Here the relation is: the product of two
● Take the example: adjacent numbers outside the square is equal to a
number inside the square.
Row
Thus, 15 × 2 = 30
1 7 9 2 × 7 = 14
Column
@UPSC_THOUGHTS

2 14 ? 7 × 9 = 63
3 105 117 ∴ 9 × 15 = 135
∴ The missing number is 135.
(a) 12 (b) 26 The correct answer response is (c).
(c) 16 (d) 20
● Analogy-based Patterns
Here, after observing the first two columns, we see In questions where two or three figures are given,
that, in each column, the product of the first two the relations are based on analogy. The candidate
numbers plus the first number, gives the third has to study the first figure and the relation between
number. the numbers in it carefully. The same will be the
Thus, 2 × 1 + 1 = 3 relation between numbers in the next figure. Study
14 × 7 + 7 = 105 the example carefully.
x × 9 + 9 = 117
If x is the missing number, 6 2 15 17 23 28
9x = 117 – 9 = 108
28 40 ?
108 3 12 9 8 13 4
x = = 12
9
∴ The missing number is 12.
(a) 11 (b) 5
The correct answer response is (a). [Once you have (c) 1 (d) 14
established the rule governing the pattern, you
could also plug in the options one by one and get Here, studying the first two figures, we establish the
the correct answer by elimination.] relation between their members. We find that the
number in the centre is the difference of the numbers
● Other Patterns obtained by the product of the numbers in the lower
The figures where the numbers are arranged may part and sum of the numbers in the upper part.
be of different shapes. They may be in the shape (Sometimes, these problems can be complicated.)

5 . 10
General Mental Ability

Thus, (12 × 3 ) – (6 + 2) = 28 BLEACH is coded as LBAEHC. In the first step the


(9 × 8) – (15 + 17) = 40 letters in the word are paired BL EA CH. Now
∴ (13 × 4) – (23 + 28) = ? letters in each pair are reversed: LB AE HC. Thus,
or 52 – 51 = 1 the code is obtained.
∴ The missing number is 1. ● Sometimes the word is halved and then each half
The correct answer response is (c). is reversed.
INTIMATE is coded as ITNIETAM.
CODING / DECODING
In the first step, the word is halved into INTI and
Coding is a method of transmitting a signal or a MATE, then they are reversed to form ITNI and
message through a language which is known only to ETAM. Thus, the code is obtained.
the sender and the receiver. A third person cannot Sequence Coding Here the code of letters follows
decipher it. Its application finds place where certain a perfect sequence of letter skipping.
things or information are to be kept secret.
● If in a code language, A is written as B, B is written
Decoding is to decipher the code language.
as C, C is written as D and so on, then how will
The coding and decoding test is therefore set to
SMART be written in that code language?
judge the candidate’s ability to decipher the rules
according to which a particular message has been Here, each letter in the word is moved one step
coded, so that it can be decoded to reveal the message. forward in the alphabet to form its code. So SMART
will be TNBSU.
Letter Coding
● If GODAVARI is coded as KSHEZEVM then
@UPSC_THOUGHTS

In this kind of questions, code values are assigned to


NARMADA is coded as ?
a word in terms of the alphabets. Using the code values
the words are to be encoded or decoded. Here each letter in the word is moved four steps
forward to form the corresponding letters of its
● If POT is coded as LUV, then TOP will be VUL code. So REVQEHE is the required code.
(  P is coded as L, O as U and T as V).
● If GOLD is coded as HQOH, then IRON will be
● Now take coded as
SUSTAIN → XYXZWBC and (a) JTRR (b) JSQR
TRANSPIRE → ZDWCXJBDL (c) HJLP (d) JKPI
So, PRINT → ?
Here each letter in the word is moved forward in
From the given codes we get that
the sequence of +1, +2, +3, +4...in the alphabet to
P → J
obtain the corresponding letter of its code.
R → D
I → B Thus, G + 1 = H, O + 2 = Q, L + 3 = O, D + 4 = H
N→ C Similarly, I + 1 = J, R + 2 = T, O + 3 = R, N + 4 = R
T → Z
Option (a) is the answer.
∴ PRINT → JDBCZ
Coding with rearrangement of positions in the Coding with Numbers
word In this kind of questions the letters in the word In this kind of coding, a number represents a particular
and those in its code are the same, but they only change letter. You are to find out in what manner the number
their positions. represents the letter and find the code of other letters
in a similar way.
● Sometimes, the letters in the word are put in a
reverse order to form its code. ● If EXAMINATION is coded 125, what number
DIUGNAL is reversed to form LANGUID. should be the code for HARDWORK?

● Sometimes the word is formed into pairs of letters (a) 521 (b) 250
and the letters in each pair are reversed. (c) 68 (d) 98

5 . 11
General Mental Ability

Recall the chart for the alphabet with the (a) 246173 (b) 214673
corresponding numbers (c) 214763 (d) 216473
E is the 5th letter in the alphabet.
From the given codes we get that
X is the 24th letter in the alphabet.
S → 2, E → 1, A → 4, R → 6, C → 7, H → 3.
A is the 1st letter in the alphabet.
M is the 13th letter in the alphabet. ∴ SEARCH → 214673.
I is the 9th letter in the alphabet. ∴ Option (b) is the answer.
N is the 14th letter in the alphabet.
Substituted Words
A is the 1st letter in the alphabet.
T is the 20th letter in the alphabet. In this type of questions, particular words are substituted
I is the 9th letter in the alphabet. by other words. The substituted code language is to be
O is the 15th letter in the alphabet. used in answering the questions.
N is the 14th letter in the alphabet. ● If ‘sky’ is called ‘sea’, ‘sea’ is called ‘water’, ‘water’
Now after adding these numbers (5 + 24 + 1 + 13 is called ‘air’, ‘air’ is called ‘cloud’ and ‘cloud’ is
+ 9 + 14 + 1 + 20 + 9 + 15 + 14) we get 125 which called ‘river’, then what do we drink when thirsty?
is the code number of the word EXAMINATION.
Here, using common sense will get you the answer
Similarly, faster than if you go step by step. When we are
HARDWORK is coded as thirsty, we drink water. Now ‘water’ has been
8 + 1 + 18 + 4 + 23 + 15 + 18 + 11 = 98. coded as ‘air’. The answer is, therefore, ‘air’.
Option (d) is the correct answer. Sentence Coding
@UPSC_THOUGHTS

● If Z is coded 52 and ACT is coded 48, then BAT Sometimes a sentence may be coded with a set of words,
is coded as letters or numbers. The code words may not have any
(a) 47 (b) 46 meaning at all.
(c) 51 (d) 53 Thus there are two types of sentence coding.

Given that Z is coded 52 we may infer that, since Sentence Coding with Words Here a definite word
Z is the 26th letter in the alphabet, its code is double in a sentence is coded with another word.
the number. i.e., Z → 2 × 26 = 52 Take the following example:
Similarly, (A) ‘pit dar na’ means ‘you are good’
A is the 1st letter in the alphabet. (B) ‘dar tok pa’ means ‘good and bad’
C is the 3rd letter in the alphabet. (C) ‘tim na tok’ means ‘they are bad’
T is the 20th letter in the alphabet.
In the same language which word stands for ‘they’?
∴ Their codes are
A = 2 × 1 = 2 (a) na (b) tok
C = 2 × 3 = 6 (c) tim (d) pit
T = 2 × 20 = 40 Solution
∴ ACT = A + C + T = 2 + 6 + 40 = 48. ‘pit dar na’ → ‘you are good’ .............(i)
‘dar tok pa’ → ‘good and bad’ ............(ii)
Similarly,
‘tim na tok’ → ‘they are bad’ ...........(iii)
B, A and T are the 2nd, 1st and 20th letters of the
alphabet. In sentences (i) and (ii) common code is ‘dar’ and
∴ Their values are 2 × 2 = 4; 2 × 1 = 2; and the common word is ‘good’. Thus we know that
2 × 20 = 40 respectively. ‘good’ is coded as ‘dar’. In (ii) and (iii) the common
∴ BAT = B + A + T = 4 + 2 + 40 = 46 code is ‘tok’ and the common word is ‘bad’. Thus
∴ Option (b) is the answer. we know that ‘bad’ is coded as ‘tok’. In sentences
● If ROSE is coded 6821, CHAIR is coded as 73456 (i) and (iii) the common code is ‘na’ and the
and PREACH is coded as 961473, then what will common word is ‘are’. Thus it can be inferred that
be the code for SEARCH? ‘are’ is coded as ‘na’.

5 . 12
General Mental Ability

Now in sentence (iii), we know that ‘are’ has been Using the given codes, we get
coded as ‘na’, ‘bad’ has been coded as ‘tok’, and 18 × (12 ÷ 4) + 5 – 6
so ‘they’ should be coded as ‘tim’. = 18 × 3 + 5 – 6
∴ Option (c) is the correct answer. = 54 + 5 – 6 = 53
● If stands for ‘+’, stands for –, stands for
Sentence Coding with Numbers or Letters Take
the following example. ×, stands for ÷, stands for =, and stands
for >, state which of the following equations is
● In a certain code, 253 means ‘books are old’, 546 correct.
means ‘man is old’ and 378 means ‘buy good (a) 3 4 8 2 8 6
books’. What number stands for ‘are’ in that code? (b) 5 2 3 10 10 2
(a) 2 (b) 4 (c) 6 2 5 4 6 2
(c) 5 (d) 6 (d) 3 3 8 4 2 5 1
Solution
Using the symbols we get
‘253’ means ‘books are old’ .............(i)
(1) 3 × 4 – 8 ÷ 2 = 8 – 6 or 12 – 4 = 2 ... not correct
‘546’ means ‘man is old’ ............(ii)
(2) 5 + 2 × 3 – 10 = 10 ÷ 2 or 5 + 6 – 10 = 5 or
‘378’ means ‘buy good books’ ..........(iii)
1 = 5 ... not correct
In sentences (i) and (iii) we find that the word
(3) 6 ÷ 2 + 5 – 4 > 6 + 2 or 3 + 5 – 4 > 8 or 4
‘books’ is common and corresponding to it the
number ‘3’ is common. Therefore ‘3’ stands for > 8 ... not correct
‘books’. In sentences (i) and (ii) the word ‘old’ is (4) 9 + 2 = 10 + 1 or 11 = 11 ... correct
@UPSC_THOUGHTS

common and so also is the number ‘5’. Thus, ‘5’


stands for ‘old’. Now, in sentence (i) we know that Passwords
‘book’ is coded as ‘3’ This type of question requires some decoding as well
‘old’ is coded as ‘5’ as rule application as in analytical reasoning.
∴ ‘are’ should be coded as ‘2’. An example will help to explain this type and the
∴ Option (a) is the correct answer. method for solving it.
Using Mathematical Symbols ● In an office there are eight sections named I, II, III,
In this type of questions mathematical symbols like ‘+’, IV, V, VI, VII and VIII. No visitor is allowed to visit
‘–’, ‘×’, ‘÷’, ‘<’, ‘>’, ‘=’, ‘≠’ etc. are represented by other one section for more than an hour. If a visitor has
symbols or letters or numbers. The candidate has to to visit more than one section he has to come back
substitute the real signs and solve the problems to the security officer to get a fresh visitor's pass
accordingly. The BODMAS formula of simplification by surrendering the earlier section’s pass.
may be kept in mind.
Every hour the security officer generates seven
● If ‘+’ means ‘÷’, ‘×’ means ‘–’, ‘÷’ means ‘+’, and passwords for all sections by choosing a set of
‘–’ means ‘×’, then 16 ÷ 8 × 6 – 2 + 12 = ? seven words, which is used for Section I which is
(a) 22 (b) 24 the reception-cum-visitors hall. Given below is an
(c) 23 (d) 120 illustration of the generation of seven passwords
Using the given codes, we get for each section.
16 + 8 – 6 × 2 ÷ 12
Section I: sri am cue way meat fro le
= 16 + 8 – 12 ÷ 12
II: le cue am way meat fro sri
= 16 + 8 – 1
= 23 III: cue le way am meat sri fro
IV: fro way le am meat sri cue
● If P denotes ÷, Q denotes ×, R denotes + and S V: way fro am le meat cue sri
denotes –, then 18Q12P4R5S6 = ? and so on till section VIII
(a) 53 (b) 36
(c) 59 (d) 65 Questions 1 and 2 are based on this rule.

5 . 13
General Mental Ability

1. A visitor was given a password ‘pit sy me ole The pattern matches that of Section VIII in the
try je no’. If at that time the password for table above.
Section I was ‘me no pit je try ole sy’, which So, the answer is (d)
of the following sections did the visitor want 2. Password of Section VII:
to visit? rye fu che it can si pecha
(a) III (b) IV Corresponding numbers:
(c) VI (d) VIII 2 1 7 6 5 4 3
2. A visitor for Section VII had the password ‘rye (refer to the table above)
fu che it can si pecha’. What was the password ∴ Password of Section I:
for Section I at that time? fu rye pecha si can it che
(a) che pecha rye si can it fu So the answer is (c).
(b) pecha che fu it can si rye
Coded Statements and Conclusions
(c) fu rye pecha si can it che
In this type of questions, there is combination of coding
(d) rye fu si pecha che can it
and deductive reasoning. A little bit of mathematical
Solution: knowledge will also be required. Two worked examples
The rule followed to generate the password in order will explain the type of question and how to solve them.
to visit each section is as follows: In the following questions, the symbols α, β, π, ★,
Let us first number each word in Section I and find γ are used with the meanings given below.
out the pattern in which the words have been A α B means A is greater than B.
interchanged for each section. A β B means A is either greater than or equal
to B.
@UPSC_THOUGHTS

sri am cue way meat fro le A π B means A is equal to B.


I. 1 2 3 4 5 6 7 A ★ B means A is smaller than B.
II. 7 3 2 4 5 6 1 A γ B means A is either smaller than or equal
III. 3 7 4 2 5 1 6 to B.
IV. 6 4 7 2 5 1 3 Now in each of the following questions, assuming
V. 4 6 2 7 5 3 1 the given statements to be true, find which of the two
We note that for Section II, the first and last words conclusions I and II given below them is/are definitely
interchange, and so also the second and third true. Give answer
words. For Section III, first and second words are (a) if only conclusion I is true
interchanged, and so also the third and fourth, and (b) if only conclusion II is true
(c) if neither I nor II is true
sixth and seventh. The two patterns are repeated
(d) if both I and II are true
for Sections IV and V alternately.
1. Statements
Passwords for the other sections can be obtained P α Q; Q γ R; R π S
by repeating the above steps alternately. Conclusions
VI. 1 2 6 7 5 3 4 I. S α Q
VII. 2 1 7 6 5 4 3 II. S π Q
VIII. 3 7 1 6 5 4 2 2. Statements
X α Y; Y γ Z; W α Z
1. Password for Section I:
Conclusions
me no pit je try ole sy
I. W π X
Numbers for the words:
II. W α X
1 2 3 4 5 6 7
The given password in the question: Solution: Break down the coded statements and
pit sy me ole try je no conclusions.
Numbers for the word: 1. Statements
3 7 1 6 5 4 2 P α Q means P > Q

5 . 14
General Mental Ability

Q γ R means Q R He takes a turn towards the north-west and goes


R π S means R = S 5 km. Where is he now with respect to the point
Thus, Q S from where he started?
Conclusions Sketch out the path.
I. S α Q means S > Q
II. S π Q means S = Q 5 km
Both I and II are true
So (d) is correct.

m
5k

5k
2. Statements
X α Y means X > Y
Y γ Z means Y Z
W α Z means W > Z You find the man has reached the starting point.
Thus, W > Z Y < X ● One evening before sunset two friends Sudhir and
Conclusions Montu were talking to each other face to face. If
I. W π X means W = X Montu’s shadow was exactly to his right side,
II. W α X means W > X which direction was Sudhir facing?
The relation between W and X cannot be
determined. Again draw a sketch:
So (c) is correct.

DIRECTION PROBLEMS
@UPSC_THOUGHTS

In questions based on direction, a series of movement


by an object or person is given in various directions.
The candidate is required to find out the final direction
or distance moved, i.e., the distance and direction of the
final point from the initial point.
The problems on direction are of two types:
1. Linear direction
2. Angular direction Given that it is evening time, the sun should be in
the west. Also, Sudhir and Montu are facing each
Problems on Linear Direction other and Montu’s shadow is exactly to his right
In these problems it is necessary to keep in mind the side. This means Montu is facing north and Sudhir
north-south, east-west alignment and draw out the is facing south.
directions as you read the
N Problems on Angular Direction
problem. There is very little
W

chance of going wrong if one In this kind of questions one’s sense of direction based
N

follows the steps without on degree of movements clockwise or anticlockwise is


missing any and also pays W E tested.
attention to the distance
● A boy is facing south. He turns 135o in the
covered each time, if anticlockwise direction and then 225o in the
SW

SE

mentioned. The figure shows


S clockwise direction. Which direction is he facing
the main directions—East (E),
now?
West (W), North (N), South (S) and the cardinal directions
North-East (NE), South-East (SE), South-West (SW) and Draw a sketch and work in the movements, keeping
North-West (NW). the direction (clockwise/anticlockwise; north/
south/east/west) and angles in mind.
● A man starts from a point A, goes east 5 km, then
he takes a turn to the south-west and goes 5 km. The boy first moves 135° anticlockwise from facing

5 . 15
General Mental Ability

south, so he now faces north-east. From that position Problems of Distance and Directions
he turns 225° anticlockwise. Sketch the movements. In this kind of problems, after a successive follow-up
of distances travelled in various directions, the distance
NE
travelled after reaching the final point is to be found.
Boy The distance travelled can be calculated by simple
O addition and subtraction or by the Pythagoras theorem.
West East
225° The Pythagoras theorem is generally used in places
135°
where the distance travelled is in the form of a
South
hypotenuse of a right triangle.

It is clear that the boy now faces west. ● A walks 10 metres towards the front and 10 metres
towards the right. Then every time turning to his
Problems on clocks are based on angular direction. left, he walks 5, 15 and 15 miles respectively. How
● A watch reads 4:30. If the minute hand points east, far is he from his starting point?
what direction will the hour point? Sketch out the movements, marking the distance
(a) North (b) North-west traveled each time.
(c) South-east (d) North-east 15 m
E D
When the clock shows the time 4:30, its hands are
10 m

5m
in the position as shown below.

5m
12
11 1 B C
10 m

15 m
@UPSC_THOUGHTS

10 2

10 m
9 3

End Start
8 4 5m
F A
7 5 From the diagram it is clear that A has travelled
6
from A to F. But before reaching F, A has travelled
Now, according to the question, the minute hand
through the points B, C, D and E.
points east. The candidate has to rotate the watch
in an anticlockwise direction so as to reach a point Now,
where the minute hand points east. Accordingly, AF = ED – BC
the hour hand and the seconds hand change = 15 – 10
directions. The figure shows the position of the AF = 5 m
watch after rotation.
● A man travels 8 km east, then 3 km south, then 4
North km west. How far is he from the starting point?
3

(a) 2 km (b) 4 km
2

(c) 3 km (d) 5 km
1

Sketch the movements and distances carefully:


12

West East
6
11

7
10

8
9

South
Thus, from the diagram we see that the hour hand
points towards the north-east.

5 . 16
General Mental Ability

From the diagram it is clear that the man has Number of girls in the row = 16
travelled from A to D. Let us join D and A such Pinky’s present position from left = 7th
that a right triangle AA’D is formed. Pinky’s earlier position from left = 9th
Number of girls to the right of Pinky = 16 – 9 = 7
Now A’D is 3 km (  A’D || BC)
and AA’ is 4 km ( A'B is 4 km as A’B || DC) So Pinky’s earlier position from the right end of the
Thus, using Pythagoras theorem, row was 8th.
AD2 = (AA2) + (A’D)2
AD2 = 42 + 32 BLOOD RELATIONS
AD = 16  9 In these problems, a round-about description of
relationship between persons is given. You will be
AD = 25 asked how a certain person is related to another or
AD = 5 km others. In these problems, brush up your knowledge of
So the man is 5 km from his starting point. Answer relationships—who are uncle/aunt, cousin, niece and
response (d) is correct. nephew. Also remember what the terms ‘maternal’ and
‘paternal’ mean.
In these problems, a sketch often helps.
RANKING Read the following solved examples.
In this type of questions, the rank of a person is given ● Introducing a man to her husband a woman said,
either from the top or the bottom or both. Or his/her “His brother’s father is the only son of my grand-
@UPSC_THOUGHTS

place in a row is given, either from the left or right or father.” How is this woman related to this man?
both. One has to find out the total number of persons
Let us analyse the problem. Firstly, the woman
or his/her rank from top or bottom, left or right or in
says, the man’s brother’s father; it means the man’s
relation to some other person.
These problems may also be considered a part of father himself. Secondly, she says that the man’s
analytical reasoning. father is the only son of her grandfather. This
means that the man’s father and the woman’s
● Seema ranks 8th from the top and 37th from the father are the same.
bottom in a class. How many students are there in
the class?
FIGURE PARTITION
There are 7 students before Seema and 36 students
below Seema. In such problems, you have to count the number of
∴ Including Seema there are 7 + 1 + 36 = 44 triangles, squares, etc., of small dimensions within a
students. given figure. It requires systematic procedure. Number
There are 44 students in Seema’s class. the vertices or points of intersection and count from
● In a row of 16 girls when Pinky was shifted by two each possible vertex.
places towards the left she became 7th from the left Take the following figure.
end. What was her earlier position from the right
end of the row?
A sketch helps:

7th 9th

Pinky’s Pinky’s
present earlier ● How many triangles are there in the figure?
position position ● How many quadrilaterals are there in the figure?
8th ● How many pentagons are there in the figure?

5 . 17
General Mental Ability

The first step is to name the vertices and and bottom layers will have at least one side
intersections. painted. In the middle layer the outer eight smaller
cubes encircle the centre cube, which is protected
E F on top and bottom by layers of cubes. The centre
cube alone will thus have no painted side.
Q
A B So only one of the small cubes will have no painted
face.
P

C D
DICES
Now set about counting systematically, starting In these problems, figures show a dice in various
from each vertex and taking care not to duplicate. positions, with different faces visible. You will be
Two triangles on AB on top – AEB and AFB; two required to find the number/number of dots on a
below on AB – ABD and ABC; two on EF, i.e., BEF particular face. The procedure is to note down the
and AEF; two on CD – ACD and CBD. Each of these numbers adjacent to the given number. The number left
can be split into two triangles, e.g., ABE has AEQ is the number opposite to the given number.
and AQB; EFB has EQF and BQF. So, the total Study the following solved examples.
number of triangles comes to 16.
● Four different positions of the dice are depicted
The quadrilaterals are ABFE, ACDB, APBQ, APBF, below. Find the number of dots on the face opposite
APBE, ACBQ, ADBQ, ACBE and ADBF. So, there to the face with one dot.
@UPSC_THOUGHTS

are nine quadrilaterals.


The pentagons are ADBFE, ACDBF, CBFEA, BEACD,
APBFE and AQBDC. So, there are six pentagons.

BUILDING DESIGNS
These problems generally involve cubes or blocks packed
You will be able to note that the number of dots
together. The examinee is asked to find the number of
in the face adjacent to the face with 1 dot are 3,
blocks which have maximum sides touching another
2 and 5 (from figures I and II). From figures III and
block or maximum/minimum sides exposed, painted,
IV, it is clear that 6 is also adjacent to 1. Therefore,
and so on. These items, in fact, can be solved by the
number of dots on the face opposite to the face with
use of common sense rather than any formula.
1 dot is 4.
Take an example:
● A dice having numbers 1 to 6 on its 6 sides is
● The figure represents a shown at various angles in the diagram. Which is
wooden block 3 cm on an the number opposite to 3?
edge, all of whose faces
are painted red. If the
1 1 4
block is cut up along the
dotted lines, 27 blocks 3 5 2 3 2 3
result, each 1 cubic cm in
volume. How many of
these will have no painted From the diagrams it is clear that the numbers
faces? adjacent to 3 are 1, 5, 2, 4

Consider the cube. All the small cubes on the top ∴ The number opposite to 3 is 6.

5 . 18
General Mental Ability

● Two positions of a block are given below. When 1 (a) 3.30


is at the top, which number will be at the bottom? (b) 2.30
(c) 9.30
(d) 6.30
Don’t be in a hurry to mark (a) 3.30. Visualise
the hands at 9.30

The numbers adjacent to 1 are 2, 5, 3 and 6. The


number opposite to 1 should be 4.
● Two positions of a block are given. When 2 is at
the bottom, what number is at the top?

The mirror image is laterally inverted. In reality


the clock will show

From the two positions we get that 2 and 3 are


@UPSC_THOUGHTS

adjacent to each other, and so are 3 and 4 also. It


means that 2 and 4 should be opposite each other.
Hence, when 2 is at the bottom, 4 would be at the
top. —which is 2.30.
Answer response (b) is correct.

IMAGES Water Images


In an image formed on a water surface, the object is
Images or reflections are formed in mirrors or any shiny
inverted vertically, but not laterally.
surface. They are also formed in still water. But the two
So the water image of will be
types of images differ in certain features.
letter
Mirror Images
In an image/reflection in a plane mirror, there is lateral water image
inversion of the object reflected in the image. Vertically
A figure reflected in water:
there is no inversion.
So the letter will appear as in a mirror. The
figure in (i) will appear as in (ii) in a mirror. figure

water image
Object (i) (ii) Mirror image
Questions may be asked on how a clock’s hour and
Just remember:
minute hands will look at a certain time as reflected
in a mirror. ● In mirror images, the letters (capital) that do not
● If a clock’s reflection shows 9.30—as the time,
show any change are:
the actual time is A H I M O T U V W X Y

5 . 19
General Mental Ability

● In water images, letters that do not show change


are:
C D E H I K O X

EMBEDDED FIGURES ?
In these problems, your ability to spot a given shape
or figure within a more complicated figure is tested. You
have to catch the lines and shape of the given figure
and locate it in any of the ‘answer figures’. You need
to concentrate and refuse to be distracted by the other
shapes and lines.
See the following example. The figure at the left is This is quite easy, really. The blank space must
embedded in one of the four labelled (a), (b), (c) and have a triangle whose vertex touches the top inverted
(d). Pick out the correct option. one. There is only one triangle and it is attached to the
base. Only (b) fits.
Now try this

?
@UPSC_THOUGHTS

You should note the given left side figure in a


triangle with a stem on its base.
Can you see why (d) is the correct option?
Option (a) has no such shape: the triangle in it is
inverted. Option (b) too does not have it. Option (c) is
your answer. Right in the middle you have the figure FIGURE MATRIX
embedded. Option (d) has a triangle, but its size is
bigger and it has no stem attached to its base. In these problems, a number of figures are given within
a matrix, and these figures follow a particular rule—
something similar to what you saw in series completion.
COMPLETING PATTERNS You have to unravel the pattern and choose the correct
answer response to fit the empty space.
In this type of problem, a matrix with a set of figures
is given, but with one part of it missing—left blank. Take the matrix below:
From the given answer responses, you have to select
the one that fits the blank space to complete the pattern
of the matrix.
Some people have an eye for such visual patterns.
If you don’t, you need to practice. You must note exact
details of the adjacent portions and see how the lines
may continue.
See the problem figure at left and choose the option ?
from (a) (b) (c) and (d) to fit the blank space.

5 . 20
General Mental Ability

Which one of the following completes the matrix?

(a) (b) (c) (d)


The paper has been folded twice. The cut will
(a) (b) (c) (d) appear in each quarter. The shape and size of the
cut should indicate to you that options (c) and (d)
Study the rows that are complete. In the first row,
do not fit. To choose between (a) and (b) you must
the outer line is shed at every step. The same pattern
note that the gap between the cuts in the top and
is repeated in the second row. The last figure in each
bottom quarters will be a little broader than the
row is the innermost part.
margin from the edge. Only answer response (b) fits
The last row will similarly have at the end the
the picture.
innermost symbol which is a circle.
Answer response (d) is correct.
PAPER FOLDING
PAPER CUTTING In these problems, you are required to select a figure
that would most closely resemble the pattern formed
We have all done it idly: fold a piece of paper twice
when a transparent sheet with design on either side of
and tear off or cut a shape into the folded paper. On
a dotted line is folded along this line. Again, you have
opening it, the cut out shape appears on the four
@UPSC_THOUGHTS

to visualise the pattern.


quarters of the piece of paper. The shapes are, however,
precise. In problems on paper cutting, you have to ● The top figure is that of the transparent sheet with
imagine the result with the help of clues given in the the design to be folded along the dotted line. Which
form of figures. Again, your visual ability is on test. of the given figures below shows how the pattern
In these problems, a set of three figures showing would appear when the sheet is so folded?
the manner in which a piece of paper has been folded,
are given. In each of the first two figures, a dotted line
together with an arrow on it is given. The dotted line
is the reference line along which the paper has to be
folded and the arrow indicates the direction of the fold.
In the third figure, there are marks showing the position
and the nature of the cut made in the folded sheet. You
have to select one of the figures from the set of four
answer figures (a), (b), (c) and (d) which would most
closely resemble the pattern when the paper is unfolded.
(a) (b) (c) (d)
● Figures X, Y and Z show a sequence of folding of
When the sheet is folded, the symbols will overlap.
a piece of paper, with Z showing the manner in
The symbol on the right side will retain its position,
which the folded paper has been cut. From the while the one on the left will get laterally inverted.
given answer figures, choose the one that most
So (d) is the best option.
closely resembles the unfolded form of figure Z.
● Another take on this type of problem is given below.
If the given figure is folded along the lines to form
a cube, how many dots would be there opposite the
X Y Z face having six dots?

5 . 21
General Mental Ability

Here is a combination of dice and paper folding.


You see two flaps—one of which will cover the top
and the other, the bottom. So they will be opposite
to one another.
Clearly, the answer is 2—i.e., response (d).

We have covered in these pages most of the question


types that go under General Mental Ability. In the
(a) 5 (b) 3
following pages there is a wide variety of questions
(c) 1 (d) 2
that will give you adequate practice.

@UPSC_THOUGHTS

5 . 22
General Mental Ability

PRACTICE EXERCISES
Directions: In the questions (1 to 25) below, one or 9. a — a b — c b c — d c —
more terms is/are missing in the series. Choose the answer (a) bbcd
response that gives the missing term/s so that the series (b) cbcd
is complete. (c) abcd
1. A, B, D, G, ? (d) bcbc
(a) M
10. a b c – b b c – b c – a b c a a – c a b
(b) L
(c) K (a) a a c b
(d) H (b) a b c b
2. A, C, F, H, ?, M (c) a a b c
(a) L (d) a b a c
(b) K 11. a c – a c b b a c – a c – b – c b a
(c) J (a) b b a c
(d) I
(b) b b c a
3. AZ, CX, FU, ? (c) b b a a
(a) IR
(d) b a a a
(b) IV
(c) JQ 12. a b a – b b a – b a a – a a b – a b b a
@UPSC_THOUGHTS

(d) KP (a) a a a b
4. GH, JL, NQ, SW, YD, ? (b) a a b b
(a) EJ (c) a b b b
(b) FJ (d) b b a a
(c) EL
13. a b – b c b c a c – – a b a b – b c
(d) FL
(a) c a b c
5. PMT, OOS, NQR, MSQ, ?
(b) c a c c
(a) LVP
(b) LUP (c) c a c b
(c) LWP (d) c a a c
(d) LVR 14. a b a – b a b – b a c a – a c b a –
6. ajs, gpy, ?, sbk, yhq (a) c c c a
(a) mve (b) c c c b
(b) oua (c) c c a b
(c) qzi (d) b c a c
(d) dmv
15. a b – a a – b b a a b b –
7. BMX, DNW, FOU, ?
(a) GHO (a) b b b a
(b) GPS (b) b b a a
(c) HPS (c) b a a a
(d) HPT (d) a a a b
8. ADVENTURE, DVENTURE, DVENTUR, ?, 16. 8, 4, 12, ?, 18
VENTU (a) 6
(a) DVENT (b) 8
(b) VENTURE
(c) 9
(c) VENTUR
(d) 15
(d) DVENTU

5 . 23
General Mental Ability

17. 64, 49, 36, ?, 16 Directions: In each of the following questions (26 to
(a) 20 28), a sequence of groups of letters and numbers is given
(b) 24 with one term missing marked by (?). Choose the missing
(c) 25 term from the given answer responses.
(d) 32 26. D-4, F-6, H-8, J-10, ?
(a) L-11
18. 28, 15, 6, ?, 0 (b) L-12
(a) 3 (c) M-12
(b) 2 (d) M-13
(c) 1 27. C 4 X, F 9 U, I 16 R, ?
(d) 0 (a) K 27 P
19. 96, 48, ?, 12, 6 (b) L 25 P
(a) 18 (c) K 25 P
(d) L 25 O
(b) 24
(c) 30 28. Z 1 A, X 2 D, V 6 G, T 21 J, R 88 M, P 445 P, ?
(a) N 267 T
(d) 36
(b) T 2670 N
20. –3, –2, ?, 3, 7 (c) N 2676 S
(a) –1 (d) T 2676 N
(b) 0 29. Which one of the following terms does not fit
(c) 1 in the series:
(d) 2 1 CV, 5 FU, 9 IT, 15 LS, 17 OR
@UPSC_THOUGHTS

(a) 15 LS
21. 27, ?, 18, 15, 13
(b) 5 FU
(a) 20
(c) 17 OR
(b) 21 (d) All of them fit in the series
(c) 22
30. Find the wrong term in the following sequence:
(d) 23 G 4 T, J 10 R, M 20 P, P 43 N, 5 90 L
22. 6, 7, 5, ?, 4, 9 (a) G 4 T
(a) 3 (b) J 10 R
(b) 7 (c) P 43 N
(c) 8 (d) S 90 N
(d) 9 Directions: In each of the following questions (31 to
40), a set of problem figures are given at the top and
23. 1, 8, 27, ?, 125
four answer figures below out of which only one will
(a) 36
be correct in continuing the sequence shown by the
(b) 49
problem figures. Choose the correct option.
(c) 64
(d) 81 31. Problem Figures

24. 2, 3, 6, ?, 18
(a) 9
(b) 11
(c) 13
(d) 15 Answer Figures
25. 0, 1, 3, 6, 10, ?
(a) 13
(b) 14
(c) 15
(d) 16

5 . 24
General Mental Ability

32. Problem Figures 36. Problem Figures

Answer Figures Answer Figures

33. Problem Figures 37. Problem Figures

Answer Figures Answer Figures


@UPSC_THOUGHTS

34. Problem Figures 38. Problem Figures


= +
+ =
+ = + =
Answer Figures Answer Figures

+ = + = +
+ =
=
35. Problem Figures 39. Problem Figures

Answer Figures Answer Figures

5 . 25
General Mental Ability

40. Problem Figures 48. (a) Microscope


(b) Telescope

+
+
(c) Periscope
(d) Stethoscope

+
+
49. (a) Square
(b) Triangle
Answer Figures
(c) Cube
(d) Rectangle
+

50. (a) Door


+

(b) Gate

+
+
(c) Table
(d) Window

Directions: In each of the following questions (41 to Directions: Out of the four figures given in each
50), there are four words, three of which are alike in question (51 to 55), three are similar in a certain way.
some way and one is different. Find the one that is Choose the figure that is different from the rest.
different.
41. (a) Copper
+ ÷
(b) Gold 51.
(c) Silver ÷ +
(d) Steel
@UPSC_THOUGHTS

42. (a) Apple


(b) Mango
(c) Potato 52.
(d) Brinjal
43. (a) Cockroach
(b) Roundworm
(c) Amoeba
(d) Frog
53.
44. (a) Advise
(b) Suggestion
(c) Counsel
(d) Direction
45. (a) Intimacy 54.
(b) Attachment
(c) Enmity
(d) Friendship
46. (a) Cotton
(b) Terene
(c) Silk 55.
(d) Wool
47. (a) Hexagon
(b) Rhombus Directions: In each of the following questions (56 to
(c) Triangle 60), there is a certain relationship between two given
(d) Circle words on one side of : : and one word is given on another

5 . 26
General Mental Ability

side of : : while another word is to be found from the 63. ARTISTS : TROUPE as
given alternatives, having the same relation with this (a) Captain : Team
word as the words of the given pair bear. Choose the (b) Fish : Pond
correct alternative? (c) Market : Crowd
(d) Cattle : Herd
56. Flower : Bud : : Plant : ?
(a) Seed 64. SCALP : HAIR as
(b) Taste (a) Nail : Nailpolish
(b) Horn : Cattle
(c) Flower
(c) Earth : Grass
(d) Twig
(d) Fur : Cap
57. Venerate : Worship : : Extol : ?
65. FILTER : WATER as
(a) Glorify
(a) Expunge : Book
(b) Homage (b) Edit : Text
(c) Compliment (c) Censor : Play
(d) Recommend (d) Curtail : Activity
58. Grain : Stock : : Stick : ? 66. ‘Principle’ is related to ‘rule’ in the same way as
(a) Heap ‘Principal’ is related to
(b) Bundle (a) college
(c) Collection (b) student
(d) String (c) chief
@UPSC_THOUGHTS

(d) scholar
59. Aeroplane : Cockpit : : Train : ?
(a) Wagon 67. What is associated with ‘Premchand’, as ‘poetry’
(b) Coach is associated with ‘Tagore’?
(c) Compartment (a) novel
(b) drama
(d) Engine
(c) biography
60. Wax : Wane : : Zenith : ? (d) criticism
(a) Nadir
68. ‘Wrist’ is related to ‘elbow’ in the same way as
(b) Bottom
‘ankle’ is related to ?
(c) Fall
(a) foot
(d) Depth (b) shin
Directions: In the following questions (61 to 65), (c) leg
choose the option that expresses the same relationship (d) knee
as the words at the top. 69. What is related to ‘traffic lights’ in the same way
61. MOSQUITO : MALARIA as as ‘airport’ is related to ‘control tower’?
(a) Housefly : Food (a) police
(b) Tobacco : Cancer (b) cars
(c) accident
(c) Soil : Erosion
(d) road
(d) Road : Accident
70. Writer is related to Reader as Producer is related
62. BIRD : CAGE as
to
(a) Crime : Punishment
(a) Seller
(b) Antique : Museum
(b) Consumer
(c) Thief : Prison (c) Contractor
(d) Animals : Zoo (d) Film

5 . 27
General Mental Ability

Directions: In each of the following questions (71 to


75), there is some relationship between the figures A
and B. The same relationship exists between the figure 75.
C and one of the four alternatives (a), (b), (c) and (d).
?
Choose that figure alternative.

71. ?

Directions: In each of the following questions (76 to


80), the numbers are written in the cells of a matrix
according to a certain system. From amongst the given
alternatives, find the number which can replace the mark
of ‘?’ as given in a cell of a matrix.

72.
? 76. 7 4 5
8 7 6
@UPSC_THOUGHTS

3 3 ?
29 19 31

(a) 4
(b) 3
(c) 6
(d) 5
73. ? 77. ? 13 49
9 17 69
13 11 59

(a) 9
(b) 5
(c) 10
(d) 21

78. 28 60 48
74. ? 5 6 7
14 39 27
7 ? 16

(a) 23
(b) 24
(c) 27
(d) 18

5 . 28
General Mental Ability

79. 1 4 9 ? 3 8
5 9
1 2 3 4
7 105 24 7
2 4 6 ?
83.
? 21 90 5
(a) 16 and 8
9 9
(b) 25 and 5 8 2
(c) 36 and 4 (a) 7
(d) 49 and 7 (b) 9
(c) 5
80. 3 6 8 (d) 4

5 8 4
2 2
4 7 ?
5 49 25 3
(a) 9 84. 16
(b) 8 6 81 121 ?
(c) 7
(d) 6 3 4

Directions: In each of the following questions (81 to (a) 5


(b) 7
85), numbers have been arranged in a certain pattern.
(c) 11
Find out the missing number.
@UPSC_THOUGHTS

(d) 10

? 8 3

2 27 4
216 27 8 ?
85.
81. 125
343
7 5
125 64 216
6
(a) 329
(b) 343 (a) 16
(c) 305 (b) 64
(d) 4 (c) 40
(d) 75
Directions: Each of the following questions (86 to 91)
3 8 is based on an analogy. There are two or three figures
in each question. In the first figure, the numbers are
1 24 related to each other in a certain way. Similar is the
82. relation between the numbers in other figures. Find out
6 4
the relation and choose the number from the given
2 ? alternatives to replace the question mark (?).
4 6 3 5
86. 24 576 15 ?
(a) 5
(b) 8 (a) 45
(c) 12 (b) 75
(d) 14 (c) 105
(d) 225

5 . 29
General Mental Ability

(a) 52
59 78 (b) 51
72 14 141 53 15 ? (c) 50
87. (d) 49
86 96
93. Find the missing letter:
(a) 140
(b) 250
F I O
(c) 151
(d) 121 A J K

2 4 3 9 1 5 E M ?
88.
(a) S
20 90 ? (b) V
(a) 20 (c) P
(d) R
(b) 25
(c) 26 94. What is the missing number?
(d) 75
2 6 3 5 2 3
84 81 88 168 120 ?
89.
14 12 18 9 ? 11 3 2 2 1 4 5
(a) 81
@UPSC_THOUGHTS

(b) 61 (a) 155


(c) 21 (b) 195
(d) 16 (c) 240
(d) 84
5 6 6 7 4 8 95. Find the missing item in the following:
12 21 ?
90.
4 5 5 4C 2B 3A
(a) 22
(b) 320 28A ? 45B
(c) 14
(d) 32 7C 5A 15B

(a) 10 C
12 14 5 3 11 15 (b) 13 C
23 20 ? (c) 12 C
91. (d) 7 C
6 8 4 7 3 9

96. If QUIZ is coded as RVJA, what is CLASS?


(a) 33 (a) DMBTT
(b) 1
(b) CMBTU
(c) 8
(c) DNBTT
(d) 21
(d) CNBTU
92. Find the missing number in the following:
97. If LODES is coded as 46321, how is DOES coded?
1 2 3
(a) 3261
4 5 6
(b) 3621
7 8 9 (c) 6321
27 38 ? (d) 4261

5 . 30
General Mental Ability

98. HOTEL is coded as 300. How will BORE be 104. FOOD


coded? (a) LUUW
(a) 40 (b) ULLW
(b) 60 (c) UMMW
(c) 200 (d) ULLV
(d) 160 105. WIFE
99. If CBE is coded as ‘bad’, what will GMBH be (a) DURV
(b) DRUV
coded as?
(c) DRVW
(a) Good
(d) DURW
(b) Flug
(c) Glad 106. PASS
(d) Flag (a) KZHH
(b) KHZZ
100. If in a code language, RUSTUM is written as (c) KMHH
INWANZ and RASTOGI is written as IXWAVJK, (d) WZHH
how would RUSSIA be written in that code?
(a) INNWKJ 107. In a certain code DEMOCRACY is written as
(b) INNWKT YEDOMRCCA. How can COMMUNISM be writ-
ten in that code?
(c) INWWKX
(a) MSINUMMOC
(d) INNWNX
(b) MOCMMNUSI
@UPSC_THOUGHTS

101. If in a code PREMIER is written as XOILSIO, (c) MOCNUMMSI


ANTAGONISE is written as MQNMZBQSXI, then (d) MOCMUNMIS
how can REPORT be written in the same code?
108. In a certain code MECHANICS is coded as
(a) OIXBMN HCEMASCIN. How is POSTER written in that
(b) OIXBON code?
(c) OIQBON (a) OPTSRE
(d) OIXBOZ (b) SOPRET
102. If TRACE is written as GIZXV, then how is (c) RETSOP
ROAST written? (d) TERPOS
(a) ILZHG 109. If CONSTITUTION is written as COSNTIUTTINO,
(b) IMZHG how is DISTRIBUTION written in that code?
(c) ILZGH (a) DISTRIUBTINO
(d) IZLMG (b) DITSRIUBTINO
(c) DITSIRUBTINO
103. If in a code language ALTERNATE is written as
(d) DTTSRIUBITON
ZOGVIMZGV and PERICARDIUM is written as
KVIRXZIWRFN, then how would CREDENTIAL 110. In a certain code, SYSTEM is written as SYSMET,
be written in the same code? and NEARER is written as AENRER. How is
(a) XIVMWVGRZO FRACTION written in the same code?
(b) XIUVUWMTRZO (a) CARFTION
(c) XIVWUNGROZ (b) FRACNOIT
(c) CARFTINO
(d) XIVWVMGRZO
(d) ARFCNOIT
Directions (Questions 104 to 106): In the following,
assume that the codes for ABCDEF are ZYXWVU, and 111. If in a code language SOBER is written as RNADQ,
find the codes for the words given from amongst the how will LOTUS be written in the same code
four choices? language?

5 . 31
General Mental Ability

(a) MDUWT (a) 26


(b) KMSTR (b) 30
(c) KMRST (c) 25
(d) KNSTR (d) 42

112. In a certain code language LAKE is written as 119. If N = 28 and PACT = 80, then BATCH will be
PEOI. How will MEAT be written in the same equal to
language? (a) 39
(a) PILO (b) 41
(b) REXO (c) 64
(c) QIEX (d) 68
(d) QEIX 120. If ‘nso ptr kli chn’ stands for ‘Sharma gets mar-
riage gift’; ‘ptr lnm wop chn ’ stands for ‘wife
113. If DELHI can be coded as CCIDD, how would you
gives marriage gift’; ‘tti wop nhi’ stands for ‘he
code BOMBAY?
gives nothing’; what would ‘gives’ stand for?
(a) AMJXVS
(a) wop
(b) AJMTVT
(b) ptr
(c) MJXVSU
(c) nhi
(d) WXYZAX (d) chn
114. If CIGARETTE is coded as GICERAETT, then 121. In a certain code language, ‘col tip mot’ means
DIRECTION will be coded as ‘singing is appreciable’, ‘mot baj min’ means
@UPSC_THOUGHTS

(a) IRDCTIONE ‘dancing is good’ and ‘tip nop baj’ means ‘sing-
(b) NOIETCRID ing and dancing’. Which of the following means
(c) RIDTCENOI ‘good’ in that code language?
(d) NORTECDII (a) mot
115. If in a certain code, CROWN is coded as BSNXM, (b) min
(c) baj
then how will the word BOARD be coded?
(d) Cannot be determined
(a) ANZQC
(b) APZSC 122. In a certain code language (A) ‘tom na rod’ means
(c) CPBSE ‘give me sweet’; (B) ‘jo ta rod’ means ‘you and me’;
(d) CNBQE (C) ‘pot ta noc’ means ‘you are good’; (D) ‘jo mit
noc’ means ‘good and bad’. Which of the follow-
116. If R = 18 and RAT = 39, then how will you code
ing represents ‘bad’ in that language?
RAY? (a) mit
(a) 28 (b) noc
(b) 29 (c) jo
(c) 41 (d) rod
(d) 44
123. In a certain language, “479” means “fruit is
117. If ROSE is coded as 6821, CHAIR is coded as sweet”, “248” means “very sweet voice” and
73456 and PREACH is coded as 961473, then “637” means “eat fruit daily”. Which digit stands
what will be the code for SEARCH? for ‘is’ in that code?
(a) 246173 (a) 7
(b) 214673 (b) 9
(c) 214763 (c) 4
(d) 216473 (d) Cannot be determined

118. If ARUL could be given the code number 52, what 124. In a certain code ‘256’ means “you are good”,
code number can be given to BINA? ‘637’ means “we are bad” and 358 means “good

5 . 32
General Mental Ability

and bad”. Which of the following does ‘and’ 130. If ‘+’ stands for division, ‘×’ stands for addition,
represent in that code? ‘–’ stands for multiplication, ‘÷’ stands for sub-
(a) 2 traction, then which of the following equations is
(b) 5 correct?
(c) 8 (a) 36 + 6 – 3 × 5 ÷ 3 = 24
(d) 3 (b) 36 × 6 + 7 ÷ 2 – 6 = 20
125. If animals which roam on land are called ‘swim- (c) 36 ÷ 6 + 3 × 5 – 3 = 45
mers’, those which crawl on land are called (d) 36 – 6 + 3 × 5 ÷ 3 = 74
‘aviators’, those which live in water are called
131. If ‘×’ stands for ‘addition’, ‘<’ for ‘subtraction’, ‘+’
‘reptiles’ and those which fly in air are called
for ‘division’, ‘>’ for ‘multiplication’, ‘–’ stands
‘prowlers’, what should a house lizard be called?
for ‘equal to’, ‘÷’ for ‘greater than’ and ‘=’ stands
(a) Swimmer
for ‘less than’, state which of the following is true?
(b) Reptile
(a) 3 × 2 < 4 ÷ 16 > 2 + 4
(c) Prowler
(d) Aviators (b) 5 > 2 + 2 = 10 < 4 × 8
(c) 3 × 4 > 2 – 9 + 3 < 3
126. If ‘rain’ is called ‘water’, ‘water’ is called ‘air’, (d) 5 × 3 < 7 ÷ 8 + 4 × 1
‘air’ is called ‘cloud’, ‘cloud’ is called ‘sky’, ‘sky’
is called ‘sea’ and ‘sea’ is called ‘street’, where 132. If ‘×’ stands for addition, < stands for subtraction,
does aeroplane fly? > stands for multiplication, ‘+’ stands for divi-
(a) Water sion, ‘–’ stands for equal to, ‘÷’ stands for greater
(b) Street than, = stands for less than, then which of the
@UPSC_THOUGHTS

(c) Sea given alternatives is correct?


(d) Cloud (a) 8 < 4 × 3 – 3 × 2 × 1
(b) 8 > 4 < 3 – 3 > 2 < 1
127. If ‘green’ means ‘red’, ‘red’ means ‘yellow’, ‘yel-
low’ means ‘blue’, ‘blue’ means ‘orange’ and (c) 8 + 4 < 3 ÷ 3 < 2 < 1
‘orange’ means ‘green’, what is the colour of egg (d) 8 + 4 × 3 = 3 > 2 × 1
yolk? Directions: In the following questions (133 to 135)
(a) Blue $, @, *, ** and # symbols are used.
(b) Red A $ B means A is bigger than B.
(c) Yellow A @ B means either A is bigger than or equal to B.
(d) Green A * B means A is equal to B.
128. If ‘+’ stands for subtraction, ‘÷’ stands for addi- A ** B means A is smaller than B.
tion, ‘–’ stands for multiplication and ‘×’ stands A # B means A is either smaller than or equal to B.
for division, then which of the equations is cor- Now assuming all the given statements as true
rect? find out which of the inferences I or II is definitely
(a) 265 + 11 – 2 × 14 = 22
true. Answer
(b) 2 – 14 × 4 ÷ 11 = 16
(a) if only inference I is true
(c) 46 – 10 + 10 × 5 = 92
(b) if only inference II is true
(d) 66 × 3 – 11 + 12 = 230
(c) if neither inference I nor II is true
129. If ‘+’ means ‘–’, ‘–’ means ‘×’, ‘×’ means ‘÷’ and (d) if both inferences I and II are true
‘÷’ means ‘+’, then
15 × 3 ÷ 15 + 5 – 2 = ? 133. Statements
(a) 0 P @ Q, M # N, N ** Q
(b) 10 Inferences:
(c) 20 I. P $ M
(d) 6 II. N # P

5 . 33
General Mental Ability

134. Statements 140. A direction pole was situated on the crossing. Due
D ** X, F @ Y, D $ F to an accident the pole turned in such a manner
that the pointer which was showing east, started
Inferences
showing south. One traveller went in the wrong
I. X @ Y
direction thinking it to be west. In which direction
II. Y # D
was he actually travelling?
135. Statements (a) South
U * V, X $ W, U ** W (b) East
(c) West
Inferences
I. W $ V (d) North
II. U ** X 141. I travelled eastward 15 km and another 10 km
Directions: In each of the questions (136 to 141) a southward. I turned westward and moved for 10
person is making a series of movements. Identify the km distance and again another 10 km southward.
place/direction of the person at the end of these A travel of another 5 km westward and then 20
movements. km northward ended my journey. Identify the
sketch depicting my journey.
136. A policeman goes straight 7 km eastwards, then
turns right and goes straight 3 km and turns right
again and goes straight 10 km. In which direction
is he from the starting point?
(a) South-West
@UPSC_THOUGHTS

(b) North-West
(a) (b) (c) (d
(c) North-East
(d) South-East Directions: Each of the following questions (142 to
145) on direction sense is based on angular movements.
137. Suman is 40 m south-west of Ashok. Prakash is Identify the direction of the person at the end of these
40 m south-east of Ashok. Prakash is in which movements.
direction of Suman?
142. A man is facing east. He turns 140o in the
(a) South
clockwise direction and then 50o in the anti-
(b) West
clockwise direction. Which direction is he facing
(c) East now?
(d) North-East (a) East
138. Raj starts from his office facing west and walks (b) West
100 metres straight, then takes a right turn and (c) North
(d) South
walks 100 metres. Further he takes a left turn and
walks 50 metres. In which direction is Raj now 143. A man is facing south. He turns 135o in the anti-
from the starting point? clockwise direction and then 225o in the clockwise
(a) North-East direction. Which direction is he facing now?
(b) South-West (a) East
(b) West
(c) North
(c) North
(d) North-West
(d) South
139. In a map, south-east has been shown as north; 144. A clock is so placed that at 12 noon its minute
north-east has been shown as west and so on. In hand points towards North-East. In which direc-
this map, what will west represent? tion does its hour hand point at 1.30 pm?
(a) North-East (a) East
(b) South-East (b) West
(c) South (c) North
(d) North-West (d) South

5 . 34
General Mental Ability

145. If the above clock is turned through an angle of 151. Pointing towards a person in a photograph, Anjali
135o in an anti-clockwise direction, in which said, “He is the only son of the father of my
direction will its minute hand point at 8.45 pm? sister’s brother.” How is that person related to
(a) East Anjali?
(b) West (a) Mother
(c) North (b) Father
(d) South (c) Maternal uncle
146. Rohan ranked eleventh from the top and twenty- (d) Brother
seventh from the bottom among the students who 152. Pointing to a man, a woman said, “His mother
passed the annual examination in a class. If the is the only daughter of my mother.” How is the
number of students who failed in the examination woman related to the man?
was 12, how many students appeared for the (a) Mother
examination? (b) Daughter
(a) 48 (c) Sister
(b) 49 (d) Grandmother
(c) 50
(d) Cannot be determined 153. Pointing to a photograph, a woman says, “This
man’s son’s sister is my mother-in-law.” How is
147. If Rahul finds that he is 12th from the right in the woman’s husband related to the man in the
a line of boys and 4th from the left, how many photograph?
boys should be added to the line such that there (a) Grandson
are 28 boys in the line?
(b) Son
@UPSC_THOUGHTS

(a) 12
(c) Son-in-law
(b) 14
(d) Nephew
(c) 20
(d) 13 154. Pointing to a man in a photograph, a woman said,
“His brother’s father is the only son of my grand-
148. In a row of trees one tree is fifth from either end
father.” How is the woman related to the man in
of the row. How many trees are there in the row?
the photograph?
(a) 11
(a) Mother
(b) 8
(c) 10 (b) Aunt
(d) 9 (c) Sister
(d) Daughter
149. In a class, Vidya ranks 7th from the top. Divya
is 7 ranks ahead of Medha and 3 ranks behind 155. Daya has a brother Anil. Daya is the son of
Vidya. Sushma who is 4th from the bottom is 32 Chandra. Bimal is Chandra’s father. In terms of
ranks behind Medha. How many students are relationship, what is Anil of Bimal?
there in the class? (a) Son
(a) 52 (b) Grandson
(b) 49 (c) Brother
(c) 50 (d) Grandfather
(d) 51 156. Given that : 1. A is brother of B.
150. If every second Saturday and all Sundays are 2. C is father of A.
holidays in a 30-day month beginning on a 3. D is brother of E.
Saturday, how many working days are there in 4. E is daughter of B.
that months? Then, uncle of D is
(a) 21 (a) A
(b) 23 (b) B
(c) 22 (c) C
(d) 24 (d) E

5 . 35
General Mental Ability

157. Q’s mother is sister of P and daughter of M. S is 160. If the dices I, II and III have odd number of dots
daughter of P and sister of T. How is M related on their upper faces, what would be the total
to T? number of dots?
(a) Grandmother (a) 9
(b) Father (b) 11
(c) Grandfather (c) 13
(d) Grandfather or Grandmother (d) 15

158. C is A’s father’s nephew. D is A’s cousin but not 161. If dices I, II and III have even numbers of dots on
the brother of C. Which of the following is a their upper faces and the dices IV, V and VI have
possibility of D’s relationship to C? odd numbers of dots on their bottom faces, what
(a) D is C’s mother would be the difference in the total number of dots
on top face in these two sets?
(b) D is C’s sister
(a) 0
(c) D is C’s aunt
(b) 2
(d) D is C’s uncle
(c) 4
159. Six faces of a cuboid block are coloured green, (d) 6
blue, red, yellow, orange and white in the follow-
162. If the even-numbered dices have odd numbers of
ing manner. dots on their top faces, what would be the total
number of dots?
(a) 7
White Orange
(b) 9
@UPSC_THOUGHTS

(c) 11
Yellow
Blue

(d) 15
Green Red 163. A cylinder is painted in 6 colours—red, green,
blue, yellow, violet and orange. Three positions
are shown below.
When blue is on the top, which colour will be at
the bottom? Orange
(a) Orange Green
Green
(b) Red Yellow Orange
Red

Blue Violet
?

(c) White
(d) Yellow
Directions: Questions 160 to 162 are based on the
What is the colour in the empty space?
following figures of six dices.
(a) Blue
The upper part of each dice is erased. The sum of (b) Green
the dots on the front-back and top-bottom faces is 7. (c) Violet
(d) Yellow
164. What number is opposite 3?

5 2 3 4
4 6 1 4 6 5 1 5
I II III

(a) 1
(b) 2
(c) 4
IV V VI (d) 6

5 . 36
General Mental Ability

165. What should be the number opposite 3? 169. How many faces will have no face coloured?
(a) 0
1 3 4
(b) 4
2 3 1 5 2 3 (c) 8
(d) 16
170. How many faces will have only one face coloured?
(a) 1 (a) 3
(b) 6 (b) 24
(c) 5 (c) 16
(d) 4 (d) 8
166. If the following cube is turned twice to the right 171. How many cubes have two red faces opposite one
( ) then what will the hidden numbers be? another?
(a) 4
(b) 8
(c) 10
(d) None
172. How many cubes have three faces coloured?
(a) 4
(a) 1, 2, 6 (b) 8
(b) 2, 3, 5 (c) 6
(c) 3, 4, 6 (d) 24
(d) Cannot say Directions: Questions 173 and 174 are based on the
@UPSC_THOUGHTS

following information:
167. Two positions of a dice are shown below. If 1 is
at the bottom, which number will be on the top? A cube is painted blue on all faces, and divided
into 125 smaller cubes of equal size.
173. How many cubes are not painted on any face?
(a) 1
(b) 15
(c) 9
(d) 27
(a) 4
(b) 2 174. How many cubes are painted on one face only?
(c) 3 (a) 54
(d) 5 (b) 36
(c) 16
168. Twenty-seven cubes are arranged in a block as (d) 8
shown below. How many cubes will be sur- Directions: Questions 175 and 176 is based on the
rounded by other cubes on all sides? following information:
The six faces of a cube are coloured black, brown,
green, red, white and blue, such that
(i) Red is opposite black
(a) 1 (ii) Green is between red and black
(b) 3 (iii) Blue is adjacent to white
(c) 6 (iv) Brown is adjacent to blue
(v) Red is at the bottom.
(d) 9
175. Which colour is opposite brown?
Directions: Questions 169 to 172 are based on the (a) White
following information:
(b) Red
A cube is coloured red on all faces. It is cut into (c) Green
64 smaller cubes of equal size. (d) Blue

5 . 37
General Mental Ability

176. Which colour is opposite to blue? (a) 44 triangles, 10 squares


(a) Black (b) 14 triangles, 16 squares
(b) Red (c) 24 triangles, 6 squares
(c) Green (d) 24 triangles, 9 squares
(d) Cannot be determined Directions: In each of the questions 181 and 182,
177. How many triangles are in the figure given be- choose the correct mirror image of the figure labelled (x)
low? from the answer responses given below it.

181.
(X)
(a) 27
(b) 25
(c) 23
(d) 21
178. How many triangles are there in the figure given
below?
@UPSC_THOUGHTS

182.
A B

(a) 9 (X)
(b) 10
(c) 11
(d) 12
179. How many triangles and parallelograms are there
in the following figure?

Directions: In each of the following questions (183 to


187), complete the missing portion of the given pattern
by selecting from the given alternatives (a), (b), (c)
and (d).
183.
(a) 21, 17
(b) 19, 13
(c) 21, 15 ?
(d) 19, 17
180. How many triangles and squares are there in the
given figure?
(a) (b) (c) (d)
(a) A
(b) B
(c) C
(d) D

5 . 38
General Mental Ability

184. Directions: In each question (188 to 190), you are


? given a figure (X) followed by four figures (a), (b), (c)
and (d) such that (X) is embedded in one of them. Trace
out the correct alternative.

188.

(a) A (X)
(b) B
(c) C
(d) D
185.

?
189.
@UPSC_THOUGHTS

(X)
(a) (b) (c) (d)
186.

?
190.

(a) (b) (c) (d)


(X)
187.

Directions: In each of the following questions (191 to


192), complete the missing portion of the given pattern
by selecting from the given alternatives (a), (b), (c)
(a) (b) (c) (d) and (d).

5 . 39
General Mental Ability

195.
191.

(X)

?
192.

(X)

Directions: In each of the following questions (196 to


198), there is a diagram marked (X), with one or more
dots placed in it. This diagram is followed by four other
Directions: In each of the following questions (193 to figures, marked (a), (b), (c) and (d) only one of which
195), choose the answer response that can replace the is such as to make possible the placement of the dot(s)
question mark (?) in the figure matrix. satisfying the same conditions as in the original diagram.
@UPSC_THOUGHTS

193. Find the correct alternative in each case.


196.

(X)

194.
197.

?
(X)

5 . 40
General Mental Ability

198.

(X)

200.

199.

If the above piece of paper is folded at the


cutmarks, then how it will look like. Choose your
answer from the four alternatives given below.
@UPSC_THOUGHTS

Which of the following alternatives (a), (b), (c) and


(d) is a part of the complete figure given above?

5 . 41
General Mental Ability

ANSWERS WITH EXPLANATORY NOTES


1. (c) The series progresses with the letters moving The second letter of the required term will
one space more at every step. So be U.
A B (C) D (E F) G (H I J) K Look at the options: only (b) will be suitable.
0 1 2 3 However, check for the third letter. It shows
The next term will be K after skipping 3 reverse order of TSRQ; so the third letter of
letters. the required term will be P.
2. (b) The letters move forward alternately skipping The three letters are LUP. Answer response
two and three steps to get the next term. (b) is correct.
A (B) C (D E) F (G) H 6. (a) The missing term is in the middle. So you
have to trace the pattern in the first two and
A (B) C (D E) F (G) H ? M
the last two terms letters.
1 2 1
Take the first letters of the first and fourth
The next letter after H will come after skipping terms a and s—and see how they progress
2 letters. It will be to their succeeding terms—g and y:
H (I J) K a (b c d e f) g
2 s (t u v w x) y
If one letter is skipped after K, you get M.
So the first letter of the missing term is most
@UPSC_THOUGHTS

So answer response (b) is correct.


likely to be m:
3. (c) Two terms are required. If you observe the
series, you will note that the first and second g (h i j k l) m
letters of the terms have their own patterns. Option (a) has m at the beginning, so it is
probably correct. See if m moves on to the
skip Y skip WV skip TRS
first letter of the next term S in the same
A Z C X F U J Q way: m (n o p q r) s. It does.
skip B skip DE skip GHI But check at least for one more letter. How
does j progress to p, and b to h.
So (c) is the correct answer response.
j (k l m n o) p
4. (d) The first and second letters of the sets each
b (c d e f g) h
has its own pattern of progression:
So, in the third term we should have a similar
skip HI KLM OPQR TUVWX skip 6 skipping of letters from the second letter p
G H J L NQ S W Y D F L of the second term:

skip 3 skip 4 skip 5 skip 6 skip 7 p (q r s t u) v


(XYZABC)
Answer response (a) is correct. If you feel
(After Z, you go back to A, as if the alphabet like confirming for the last letter in each term,
is in a circle.) the progression is along the same pattern—
So (d) is the correct answer response. skip 5 letters at every step:
5. (b) Looking at the first letters of the sets, we note s (t u v w x) y (z a b c d) e (f g h i j) k (l m n o p) q
a reverse order P O N M, so the first letter (G)
of the next set will be L. 7. (d) First letters : B (C) D (E) F H
The second letters show a progression by So either (c) or (d) is likely to be the required
skipping a letter at a time. answer response.
M (N) O (P) Q (R) S Now take the second letters: M N O. They

5 . 42
General Mental Ability

are in alphabetical order; so the second letter 21. (c) The sequence in the given series is
of the required term will be P. Still no luck –5, –4, –3, –2
at elimination. We have to consider the third 22. (c) The given series consists of two given series:
letters: X W U. Try reverse order: X W (V)
U. The pattern is: skip 0, skip 1. So next, Series I : 6, 5, 4 ... (odd places)
we skip 2 letters backward and get U (T S) Series II : 7, 8, 9 ... (even places)
R. No such option. 23. (c) The terms of the given series are
If we accept the S of option (c) we have 13, 23, 33, 33, 54

XW (V) U (T) S 24. (b) The sequence in the given series is


+1, +3, +5, +7
The pattern is uneven. So try with the T of
option (d): 25. (c) The sequence in the given series is
+1, +2, +3, +4, +5
X W (V) U T
26. (b) The letters skip one at every step:
There appears a pattern: no skip, 1 skip, no
skip. So option (d) is best. D (E) F (G) H (I) J

8. (c) If you look at the first three terms, you will So the next letter will be L after skipping K.
note that the first and last letters are left out The number indicates the position of the letter
at alternate steps. The second term lost the in the alphabet. So D-4, F-6, etc.
last letter to become the third term. So the The number for L is 12. So (b) is the correct
next term—the required one—will be the third option.
term minus the first letter, D. Option (b) will
27. (d) Take each letter and number of the term to
@UPSC_THOUGHTS

not fit, as it has ‘E’ at the end; E has been


trace the pattern.
left out much earlier. Option (c) is correct. You
may see if it fits with the next given term; The first letters:
it does, as the last letter ‘R’ is left out in C (D E) F (G H) I
the next term.
So the next term will begin with L (skipping
9. (a) Response (a) fits the bill as its letters JK after I).
complete a pattern of 4 groupings:
Only options (b) and (d) have to be checked.
abab / bcbc / cdcd Since the middle number 25, is the same in
In such questions, you have to see how many both, take the last letter in each given term.
letters form repetitive units. There is a pattern of reverse order:
10. (a) a b c a b/b c a b c/c a b c a/a b c a b X (W V) U (T S) R
11. (b) a c b a c b/b a c b a c/c b a c b a The next will be two letters back from R, i.e,
12. (c) a b a a b/b a b b a/a b a a b/b a b b a O, missing Q and P.

13. (a) a b/c b c b/c a c a/b a b a/b c b c So the correct answer response is (d).

14. (b) a b/a c b c b c/b a c a c a/c b a b The number, you will find, are squares of
numbers in series: 22, 32, 42. So the next
15. (b) a b b a/a b b a/a b b a/a b b a
would be 52 = 25.
16. (a) The sequence in the given series is
28. (c) Take the letters first and establish the pattern.
÷ 2, × 3, ÷ 2, × 3
The first letters of the terms skip one letter
17. (c) The term in the given series are
in reverse order alphabetically
82, 72, 62, 52, 42
18. (c) The sequence in the given series is Z (Y) X (W) V (U) T (S) R (Q) P
–13, –9, –5, –1 The next one will miss one letter—O—to
19. (b) The sequence in the given series is ÷ 2 be N.

20. (b) The sequence in the given series is So you may now consider only options (a)
+1, +2, +3, +4 and (c). The last letters of the terms follow

5 . 43
General Mental Ability

a pattern of skipping 2 letters every time in 32. (a) There is an addition of one limb at every step
order. So after P you skip Q and R to get upwards and horizontally at alternate steps.
S. Only option (c) meets the requirement. So the required figure will have another limb
The numerals follow the pattern: attached horizontally. Only (a) fits the picture.

X1 + 1, X2 + 2, X3 + 3, X4 + 4, X5 + 5 33. (b) At step 1, a small circle is added within the


bigger circle. At step 2, the small circle moves
So the next numeral will be to the opposite side within the bigger circle,
445 × 6 + 6 = 2676 but no other change takes place. In step 3,
Option (c) is correct. there is one more circle inside the small circle
29. (a) Establish the pattern. which has again moved to the opposite side.
Numerals in the first three terms show an Step 4 will be similar to step 2: the small
increasing pattern of +4 circles will move to the opposite side, with
no other change. Option (b) is correct. Option
1 (+ 4) 5 (+ 4) 9
(d) is not correct because the inner circles
But the next one, 15, breaks the pattern. remain in the same place.
It should be (9 + 4 =) 13; then the next 34. (c) In these figures with multiple symbols, take
numeral, 17, will follow with the addition of one at a time and trace the pattern of change
4 to 13. and movement.
Option (a) is thus correct. You could see if a new symbol occupies each
(Incidentally, LS is alright in the sequence.) slot at every step.

30. (b) A quick look at the letters shows that the From the left top first: =, , , .
@UPSC_THOUGHTS

pattern is followed by all the terms.


All have taken as turn except +.
The first letters skip 2 letters progressively.
Among the answer figures, options (b) and (c)
The last letters skip one letter at every step
show + in this position. So take up another
in reverse order. So the flow is in the
symbol—the one in the middle. After +, =,
numerals.
and , it should be the turn of . Option
If you start with 4, you get 10 by adding 6
to it or by X2 + 2. But with neither process (c) seems to be correct. But check for the
can you get the 20 of the next term. So the other places as well. It works for every other
second term is most likely wrong. However, position—right top, right bottom and left
you have to check further. Take the last three bottom—also.
terms. You find Of course, you could trace the movement of
20 × 2 + 3 = 43 each symbol. Take the central + first: the
43 × 2 + 4 = 90 movement from first to fourth is
Pattern : multiply by 2 but add one more at
every step.
Try the pattern backward.
You will get the numeral before 20 by
subtracting 2 first to get 18 and then dividing The movement of = is :
by 2 to get 9. Will 9 fit with the first numeral
of 4? Try the pattern (adding 1 now):
4 × 2 + 1 = 9
So J 10 R is wrong.
The correction option is (b). Deduce from the above that after moving to
31. (d) The figure laterally inverts itself, and one arm centre and out to a corner, the symbol moves
is added at every step. straight to the next corner on the same side.

5 . 44
General Mental Ability

So the next movement for + will be movement and reach their respective positions
shown in both options (c) and (d). So look
at the elements in the central column.
The topmost symbol goes to the opposite
edge. So now it is the turn of the triangle,
and for = will be but when the triangle goes to the opposite
edge, it also gets inverted. Both conditions
are satisfied by option (d). You may note that
the other symbols move step by step to the
top of the column.
All the symbols show this pattern of movement. 41. (d) Steel is an alloy, whereas the others are
And option (c) remains correct. metals.
35. (c) The figure is inverted, then rotates 90° anti- 42. (c) Potato is the only one of the four which grows
clockwise and then inverts itself laterally. Now under the ground.
the next step is an anti-clockwise rotation of
43. (c) Amoeba is unicellular.
90°. Answer figure (c) fits the requirement.
You may wonder why not a clockwise rotation 44. (d) The other words are synonymous; direction
of 90° this time—when (b) would be correct. has a different connotation.
However, a series shows progression for 45. (c) The other words have a positive aspect which
some steps at least before returning to its ‘enmity’ does not.
original position (if at all it does). The 46. (b) Terene is an artificial fibre; the others are
@UPSC_THOUGHTS

movement as per (b) would make the figure natural.


just move repetitiously in a very restricted
47. (d) The other figures are made of straight lines
manner. So (c) is a better response.
and have corners.
36. (d) One line is removed at a time from one set
48. (d) The other instruments are used for ‘seeing’
of lines alternately.
something.
37. (c) The figure gets laterally inverted at every step.
49. (c) The others are two-dimensional.
So the next option must be (c) or (d). The
figure loses a leaf from the bottom end from 50. (c) The others form some means of communicating
alternate sides. The next leaf to go will be from one side to another. Table is a piece
the one on the right side. So (d) is out; (c) of furniture.
is the correct option. It carries the rest of the 51. (c) In each of the other figures, the similar
pattern as well: the dot moves downwards; symbols occur in diagonally opposite corners.
the leaf removed from the lower parts is In (c) they are on the same side.
attached at every step to the top. 52. (d) The circles are attached on the same side
38. (a) In each step, the outermost symbol of the of the line in (d).
previous figure becomes the innermost symbol. 53. (d) In the other figures, the inner and outer circles
So now, it is the turn of the circle to occupy are the same. In (d) we have a circle and
the innermost position. Only option (a) fits the a square.
pattern.
54. (d) In the other figures, both inner figures are the
39. (d) The inner symbol becomes the outer one at same kind. In (d) there are two similar outer
every step. So it is a choice between options figures (circles) and only one triangle inside.
(c) and (d). But at every step no symbol is 55. (c) Note carefully, and you will see that in figures
repeated for the inner figure—some new (a) (b) and (d) the slanting line is in the same
element is added. So (c) may be rejected as direction. In (c) it is in the opposite direction.
it repeats the triangle symbol. Option (d) is In other words, all the figures except (c) can
correct. be rotated into each other making no difference
40. (d) The cross and arrow each moves in a zigzag to the shape or direction of the slanting line.

5 . 45
General Mental Ability

56. (a) A flower grows from a bud; a plant grows from 1st column: (29 – 8) = (7 × 3) = 21
the seed. 2nd column: (19 – 7) = (4 × 3) = 12
57. (a) ‘Extol’ means to ‘praise lavishly’—which is If x is the missing number,
also conveyed by ‘glorify’. The relationship
3rd column: (31 – 6) = (5 × x) = 25
between the words is one of synonyms.
25
58. (b) Stock of grains; bundle of sticks. x = = 5
5
59. (d) Thus, 31 – 6 = 5 × 5 = 25
60. (a) The words are antonyms. ‘Bottom’ and ‘depth’ 77. (b) In each row, the last number = (2 × first
also could be antonyms but in astronomical number) + (3 × second number) of that row
terms, ‘zenith’ and ‘nadir’ are correct as
antonyms. (2 × 13) + (3 × 11) = 59
26 + 33 = 59
61. (b) The relationship is one of causative agent and (2 × 9) + (3 × 17) = 69
disease (or the consequence). None of the
18 + 51 = 69
other options shows this relationship.
(2 × x) + (3 × 13) = 49
62. (c) The sense conveyed by bird in cage is one 2x + 39 = 49
of captivity, not habitation or of things and 2x = 49 – 39
the place where they are found. So (c) is best.
10
63. (d) Artists group into a troupe, as cattle do in x = = 5
2
a herd. A captain ‘leads’ his/her team.
78. (c) The second number in each row = 3 × (third
64. (c) Hair grows on the scalp; grass grows on the number – first number of that row)
earth.
@UPSC_THOUGHTS

60 = 3(48 – 28)
65. (c) One filters water to remove objectionable
6 = 3(7 – 5)
matter; so one censors plays to remove
39 = 3(27 – 14)
objectionable matter. Editing involves correction;
x = 3(16 – 7) = 27
expunge is to delete—something, not
necessarily objectionable, and filtering does 79. (a) The first row contains 12, 22, 32, 42
not quite do that to water. (d) does not The second row contains 1, 2, 3, 4
express the same kind of relation as the The third row contains 2, 4, 6, 8
words given at the top. 80. (d) In each column, the last number =
66. (c) Don’t be in a hurry to mark (a). first number + second number
67. (a) 68. (d) 69. (d) 2
81. (b) The sequence of numbers move in a clockwise
70. (b) Writer writes for a reader; a producer produces
order starting from 8. The sequence is cubes
for a consumer.
of the consecutive numbers starting from 2.
71. (c) The second figure has two more sides, and
i.e., 23 = 8, 33 = 27, 43 = 64, 53 = 125,
it is shaded. So a four-sided figure will
63 = 216, 73 = 343
become six-sided; only (c) fits.
72. (d) 82. (c) The opposite figures have a relation of being
4 times the other.
73. (c) There is an anti-clockwise rotation of 90° in
i.e., 1 × 4 = 4
the second figure.
3 × 4 = 12
74. (b) The entire figure rotates 180°. Don’t just go 2 × 4 = 8
by the arrows; also see that no other change 6 × 4 = 24
takes place in the figure when it rotates.
83. (d) Opposite quadrants behave in a similar way
75. (a) Only the outermost outline remains in the 7 × 5 × 3 = 105
second item. 2 × 9 × 5 = 90
76. (d) In each column, the difference between the 8 + 9 + 7 = 24
last number and the second number is equal 9 + 8 + x = 21
to the product of first and third numbers. x = 21 – 17 = 4

5 . 46
General Mental Ability

84. (b) Let’s simplify each quadrant. x


(5 + 2)2 = 49 × 11 = 88
2
(2 + 3)2 = 25
11x = 176
(4 + x)2 = 121
4 + x = 121 176
x = = 16
x = 11 – 4 = 7 11
So the last quadrant = (6 + 3)2 = 81 90. (d)
In the inner circles,
(25 + 121) – (49 + 81) = 16
5 6 6 7 4 8
12 21 ?
85. (b) 33 = 27
3 4 5 10
43 = 64
2 33 4 5×6×4 6×7×5 4 × 8 × 10
3
2 43 53 = 125 = 120 ÷ 10 = 12 = 210 ÷ 10 = 21 = 320 ÷ 10 = 32
3 3
7 5
7 63 5 63 = 216 91. (d) Sum of the digits in outer circle is 1 less than
6 3
7 = 343 in the inner circle.

12 14 5 3 11 15
86. (d) The product of the number in the upper row
23 20 ?
is equal to the first number in the lower row
6 8 4 7 3 9
which is further squared to the second number
in the lower row.
@UPSC_THOUGHTS

4 × 6 = 24 Sum of the digits outside


(24)2 = 576 1+2+1+4+6+8=22 5+3+4+7=19 1+1+1+5 +3 +9
Similarly, 23 – 22 = 1 20 – 19 = 1 = x – 20 = 1
3 × 5 = 15 x = 1+ 20 = 21
(15)2 = 225
92. (b) Obviously it is the columns that you have to
87. (c) 5 + 9 = 14 study.
8 + 6 = 14
If you multiply the middle two numbers and
7 × 2 = 14
subtract the top one from the product you get
14 × 1 = 14
the bottom-most number.
Similarly,
7 + 8 = 15 (4 × 7) – 1 = 27
9 + 6 = 15 (5 × 8) – 2 = 38
5 × 3 = 15 ∴ (6 × 9) – 3 = 51
15 × 1 = 15 93. (d) Give the letters their position numbers and
Hence, missing number is 15. see if something works out. You will find the
88. (c) 22 + 42 = 20 rows adding up.
32 + 92 = 90 F + I = 6 + 9 = 15 (which is O)
12 + 52 = 26 A + J = 1 + 10 = 11 (which is K)
89. (d) The number in the middle is halved and then ∴ E + M = 5 + 13 = 18 (which is R).
multiplied by the number at the right hand side
94. (b) When large numbers are there, try to see if
to obtain the number above.
they are squares or cubes of some numbers.
Let the unknown number be x. If you add the outer numbers of the first
14 matrix, you get
× 12 = 84
2 2 + 6 + 2 + 3 = 13
18 132 = 169
× 9 = 81
2 Do we see a pattern? 169 – 1 ?

5 . 47
General Mental Ability

Let’s check the next matrix contain the letters in the same positions from
3 + 5 + 1 + 2 = 11 the beginning in the alphabet. They are:
112 = 121, and 121 – 1 = 120 I—9th, Z—26th, X—24th, V—22nd.
So the pattern works It can also be shown diagramatically.
2 + 3 + 5 + 4 = 14 The word T R A C E
142 = 196 7th 9th 26th 24th 22nd
196 – 1 = 195 letters from the end in the alphabet.
95. (a) As every response has a C, that problem is The code G I Z X V
solved for you. But you have to trace a pattern 7th 9th 26th 24th 22nd
in the numbers. The rows do not seem to have letters from the beginning in the alphabet.
a pattern. Check the columns. The middle Similarly,
number is a product of the top and bottom The word R O A S T
numbers. 9th 12th 26th 8th 7th
4 × 7 = 28 letters from the end in the alphabet.
3 × 15 = 45 The code I L Z H G
∴ 2 × 5 = 10 9th 12th 26th 8th 7th
So (a) 10C is the answer. letters from the beginning in the alphabet.
∴ Alternative (a) is the correct answer.
96. (a) Q is R, U is V, I is J, Z is A, i.e., each
letter is coded by the next letter in the 103. (d) A L T E R N A T E — Word
alphabet. 1st 12th 20th 5th 18th 14th 1st 20th 5th

97. (b) Note that the letters of DOES are already letters from beginning in alphabet.
@UPSC_THOUGHTS

there in LODES with specific numbers. Z O G V I M Z G V — Code


1st 12th 20th 5th 18th 14th 1st 20th 5th
98. (d) Here the position numbers of the letters in letters from end in alphabet.
the alphabet are added and then multiplied by C R E D E N T I A L —Word
the number of letters in that word. 3rd 18th 5th 4th 5th 14th 20th 9th 1st 12th
H O T E L letters from beginning in alphabet.
8 + 15 + 20 + 5 + 12= 60 X I V W V M G R Z O—Code
60 × 5 = 300 3rd 18th 5th 4th 5th 14th 20th 9th 1st 12th
B O R E letters from end in alphabet.
2 + 15 + 18 + 5 = 40 104. (b) A B C D E F — Letters
40 × 4 = 160 1st 2nd 3rd 4th 5th 6th
99. (d) Each letter is coded one letter back in the letters in the alphabet from the beginning.
alphabet. Z Y X W V U — Code
So C = b, B = a, E = d, G = f … 1st 2nd 3rd 4th 5th 6th
letters in the alphabet from the end.
100. (c) From the given codes, we have
Similarly,
R → I, U → N, S → W, I → K, A → X
F O O D — Word
Thus RUSSIA → INWWKX.
6th 15th 15th 4th
101. (b) From the given codes, we get letters of the alphabet from the beginning.
P → X, R → O, E → I, O → B, T → N U L L W — Code
∴ REPORT → OIXBON 6th 15th 15th 4th
102. (a) Here, the 1st letter of the word ‘T’ is the 7th letters of the alphabet from the end.
from end in the alphabet and its corresponding 105. (b) W I F E — Word
code letter, G is 7th from beginning. Similarly, 23rd 9th 6th 5th
the 2nd, 3rd, 4th and 5th letters of the word, letters of the alphabet from the beginning.
which are ‘R’, ‘A’, ‘C’, ‘E’, respectively, are D R U V — Code
9th, 26th, 24th and 22nd from the end in the 23rd 9th 6th 5th
alphabet. Their corresponding codes should letters of the alphabet from the end.

5 . 48
General Mental Ability

106. (a) P A S S — Word 116. (d) R + A + T = 18 + 1 + 20 = 39


16th 1st 19th 19th (Given R = 18 and RAT = 39)
letters of the alphabet from the beginning.
The number against each letter denotes the
K Z H H — Code
position in the alphabet.
16th 1st 19th 19th
letters of the alphabet from the end. ∴ R + A + Y = 18 + 1 + 25 = 44

107. (b) DE MO CR AC Y — Word 117. (b) S 2, E 1, A 4, R 6, C 7,


Y ED OM RC CA — Code H 3
Similarly, 118. (a) The method is the same as followed in
CO MM UN IS M — Word Question 7.
M OC MM NU SI — Code B is the 2nd letter of the alphabet.
108. (b) MECH A NICS — Word I is the 9th letter of the alphabet.
HCEM A SCIN — Code N is the 14th letter of the alphabet.
Similarly, A is the 1st letter of the alphabet.
POS TER — Word After adding these numbers, we get
SOP RET — Code 2 + 9 + 14 + 1 = 26
109. (b) CO NS TI TU TI ON — Word 119. (d) Given N = 28
CO SN TI UT TI NO — Code N is the 14th letter of the alphabet.
Similarly, According to the given value,
DI ST RI BU TI ON — Word N = 2(14) = 28
DI TS RI UB TI NO — Code
@UPSC_THOUGHTS

P is the 16th letter of the alphabet


110. (b) This is a little tricky, but no other code works. In the same way
SYS TEM — Word A is the 1st letter of the alphabet.
SYS MET — Code C is the 3rd letter of the alphabet.
NEA RER — Word T is the 20th letter of the alphabet.
AEN RER — Code ∴ Their values are
111. (d) The sequence is –1 in terms of the alphabet. P = 16(2) = 32
A = 2(1) = 2
112. (c) The sequence is +4 in terms of the alphabet.
C = 2(3) = 6
113. (a) The 1st, 2nd, 3rd, 4th and 5th letters of the T = 2(20) = 40
word are moved one, two, three, four and five ACT = P + A + C + T = 32 + 2 + 6 + 40 = 80
steps backward in the alphabet respectively. Similarly,
Thus, BOMBAY AMJXVS B is the 2nd letter of the alphabet.
114. (c) The word is first divided into triplets—CIG, A is the 1st letter of the alphabet.
ARE, TTE. Then letters in each of these T is the 20th letter of the alphabet.
triplets are reversed GIC, ERA, ETT. C, as already stated, is the 3rd letter H is
∴ CIGARETTE GICERAETT the 8th letter.
Similarly, in DIRECTION, ∴ Their values are
DIR RID, ECT TCE and ION NOI B = 2(2) = 4, A = 2(1) = 2, T = 2(20) = 40,
∴ DIRECTION RIDTCENOI C = 3(2) = 6, H = 8(2) = 16
115. (b) The letters in the odd places in the word are BATCH = B + A + T + C + H
moved one step backward in the alphabet, and = 4 + 2 + 40 + 6 + 16 = 68
those in the even places are moved one step 120. (a) Wife gives marriage gift ptr lnm wop
forward in the alphabet to obtain the chn..........(ii)
corresponding letters in the code. He gives nothing tti wop nhi..........(iii)
Thus BOARD APZSC In sentences (ii) and (iii), the common word

5 . 49
General Mental Ability

is ‘gives’ and corresponding to it the common (c) 46 × 10 – 10 ÷ 5 = 46 × 10 – 2 = 460 – 2 = 458


code is ‘wop’. 92
121. (b) Singing is appreciable col tip mot .....(i) (d) 66 ÷ 3 × 11 – 12 = 22 × 11 – 12 = 242 – 12 = 230
Dancing is good mot baj min ......(ii)
129. (b) 15 × 3 ÷ 15 + 5 – 2
∴ is mot
= 15 ÷ 3 + 15 – 5 × 2
Singing and dancing tip nop baj .....(iii)
= 5 + 15 – 10 = 10
From (ii) and (iii) dancing baj
Now in sentence (ii) the word left is ‘good’ 130. (d)
and the code left is ‘min’. (a) Using the codes we get
∴ good min 36 ÷ 6 × 3 + 5 – 3
122. (a) You are good pot ta noc .........(C) = 6 × 3 + 5 – 3 = 18 + 5 – 3 = 20 (wrong)
Good and bad jo mit noc .........(D) (b) Using the codes we get
You and me jo ta rod ..........(B) 36 + 6 ÷ 7 – 2 × 6
From (D) bad mit 6 6
= 36 + – 12 = 24 (wrong)
123. (b) 4 7 9 means ‘fruit is sweet’..........(i) 7 7
(c) Using the codes we get
2 4 8 means ‘very sweet voice’..........(ii)
36 – 6 ÷ 3 + 5 × 3
6 3 7 means ‘eat fruit daily’..........(iii)
= 36 – 2 + 15 = 49 (wrong)
In first and second sentences ‘sweet’ is the
common word and corresponding to it 4 is (d) Using the codes we get
the common number in the code. 36 × 6 ÷ 3 + 5 – 3
@UPSC_THOUGHTS

∴ 4 means sweet. = 36 × 2 + 5 – 3 = 72 + 5 – 3 = 74
In first and third sentences ‘fruit’ is the 131. (b)
common word and number code common (a) 3 + 2 – 4 > 16 × 2 ÷ 4 = 1 > 8 (false)
is 7. (b) 5 × 2 ÷ 2 < 10 – 4 + 8 = 5 < 14 (true)
∴ 7 means fruit. The remaining word is ‘is’ (c) 3 + 4 × 2 = 9 ÷ 3 – 3 = 11 – 3 = 0 (false)
and the number code corresponding to it (d) 5 + 3 – 8 > 8 ÷ 4 + 1 = 1 > 3 (false)
is 9.
132. (d)
∴ ‘is’ means 9 in (i).
Using the given symbols:
124. (c) 2 5 6 means ‘you are good’ ’..........(i)
(a) 8 – 4 + 3 = 3 + 2 + 1 or 5 6
6 3 7 means ‘we are bad ’..........(ii)
(b) 8 × 4 – 3 = 3 × 2 – 1 or 9 5
3 5 8 means ‘good and bad ’..........(iii)
(c) 8 ÷ 4 – 3 > 3 – 2 – 1 or –1 0
∴ From (i) and (ii), we get ‘are’ 6
(d) 8 ÷ 4 + 3 < 3 × 2 + 1 or 5 < 7 (which
From (ii) and (iii), we get ‘bad’ 3
is correct)
From (i) and (iii), we get ‘good’ 5
∴ From (iii), we get ‘and’ 8 133. (a) Statements
P Q; M N; N < Q
125. (d) House lizards are animals which crawl on the
P Q > N M
ground. Here they are called aviators.
Inferences
126. (c) Aeroplane flies in the sky and sky is called P > M is true
sea. N P is not true
127. (a) Colour of clear egg yolk is yellow. Here yellow 134. (c) Statements
means blue. D < X; F Y, D > F
128. (d) X > D > F Y
1 Inferences
(a) 265 – 11 × 2 ÷ 14 = 265 – 11 × 22
7 X Y is not true
14
(b) 2 × 14 ÷ 4 + 11 = 2 × + 11 = 7 + 11 16 Y D is not true
4

5 . 50
General Mental Ability

135. (d) Statements 140. (d) North (East)


U = V; X > W, U < W North-West North-East
X > W > U = V (North-East) (South-East)
Inferences (North) West East (South)
W > V is true
U < X is true South-West South-East
(North-West) (South-West)
141. (b) South (West)
136. (a)
142. (d) E
140°
50°
SW

143. (b)

137. (c)

144. (a) When the clock is placed erect, its minute


@UPSC_THOUGHTS

hand faces north at 12 noon. It is then so


turned that the minute hand faces north-east
at 12 noon. This is possible if the clock is
rotated 45° clockwise. See figure.

138. (d)

Clock when erect Clock when rotated 45°


at 12 noon clockwise at 12 noon

139. (b) North (South-West)


(South) North-West North-East (West)

(South-East) West East (North-West) Clock when rotated


45° at 1.30 pm
(East)South-West South-East (North)
It is clear from the figure that the hour hand
South (North-East)
faces east.

5 . 51
General Mental Ability

145. (d) 150. (b) As the month begins on a Saturday, Sundays


will be on the 2nd, 9th, 16th, 23rd and 30th
of the month, totaling 5.
Every second Saturday will be the 9th and
the 22nd totalling 2.
So with 5 + 2 = 7 holidays working days will
be 30 – 7 = 23 days.
151. (d) Father of Anjali’s sister’s brother will be
The above clock After rotating 135°
Anjali’s father. Her sister’s brother being the
at 12 noon anti-clockwise, the
clock at 8.45 pm only son of Anjali’s father will be her brother.
Thus, it is clear from the figure that the minute 152. (a)
hand faces south at 8.45 pm. 153. (a) The man’s son’s sister is that man’s daughter.
146. (b) Rohan’s rank from the top = 11th If she is the speaker’s mother-in-law, the
Thus, number of students above him = 10 speaker has married that woman’s son—who
Also, Rohan’s rank from the bottom = 27th is thus the grandson of the man in the
Thus, number of students below him = 26 photograph.
Number of students who failed = 12
∴ The total number of students (including grandfather
Rohan) in the class father
= 1 + 10 + 26 + 12 = 49 (only son)
@UPSC_THOUGHTS

147. (d) Rahul’s place from the right = 12th 154. (c) man – brother speaker
Thus number of boys to his right = 11
Rahul’s place from the left = 4th brother – sister
Thus, number of boys to his left = 3
∴ The present number of boys (including 155. (b) 156. (a)
Rahul) in the line 157. (a) If Q’s mother is P’s sister, Q’s mother will
= 11 + 1 + 3 = 15 also be P’s mother. So M is P’s mother. P’s
Now, number of boys to be added so that daughter, S, and T are siblings so T’s mother
the required number of boys in the line is 28 is P, and M is T’s grandmother.
= 28 – 15 = 13
158. (b) If C is A’s father’s nephew, C is A’s cousin.
148. (d) The position of the tree from either end = 5th
D being A’s cousin can also be C’s cousin,
Thus the number of trees from the but not C’s mother, aunt or uncle. However,
right end = 4
the options do not have ‘cousin’. So if D is
and the number of trees from the not a cousin and also not a brother, D must
left end = 4 be C’s sister.
∴ The total number of trees in the row 159. (d) Here two such positions of a cuboid are given
= 4 + 1 + 4 = 9
which do not have a face in common in both
149. (a) Position of Vidya from top = 7th the positions. But, since it is a cuboid, it can
Position of Divya from top = (7 + 3) = 10th be easily determined that the two longer faces
Position of Medha from top = (7 + 10) = 17th
will be opposite to each other and the two
Position of Sushma from top
broader sides will be opposite to each other.
= (17 + 32) = 49
Position of Sushma from bottom = 4th In the example given, green and red, white
∴ Number of students behind Sushma = 3 and orange, yellow and blue are the pairs of
∴ Number of students in the class faces opposite to each other. Thus, when blue
= 49 + 3 = 52 is on the top, yellow should be at the bottom.

5 . 52
General Mental Ability

160. (c) According to the given information, sum of the For Qs. 169 and 170, the given figure will be as follows:
dots in the opposite faces is 7. Therefore, the
possible pairs of opposite faces are 6, 1; 3,
4; 5, 2. In their upper faces, the dots are 5,
5 and 3 because dices I, II and III have odd
number of dots on their upper faces. Hence,
the total number of dots would be 5 + 5 +
3 = 13.
161. (d) In dices IV, V and VI, the pairs of opposite
faces are 5, 2; 6, 1; 4, 3. Since dices IV,
V and VI have odd numbers of dots on their
bottom faces, numbers of dots in the top
faces are 4, 4 and 6.
In dices I, II and III, the opposite pairs are
4, 3; 6, 1; and 5, 2. Since I, II and III have
even numbers of dots on their upper faces,
the dots on the top faces are 2, 2 and 4.
169. (c) There are two middle layers. The middle four
Now the difference in the total number of dots cubes in each layer will have no coloured
on the top face between these two sets is faces. So 8 cubes will have no coloured
(4 + 4 + 6) – (2 + 2 + 4) = 14 – 8 = 6 faces.
@UPSC_THOUGHTS

162. (b) Dices II, IV and VI should have 5, 3, 1 dots 170. (b) There are six faces to the large cube. You
on their top faces. Therefore, the total number can observe that the central four smaller
of dots are 5 + 3 + 1 = 9 central cubes on each side will have only that
163. (c) From the given figures, we get that the colours face coloured. So there are 6 × 4 = 24 such
in the curved side of the cylinder is in the cubes.
order: blue, green, violet and red. Orange and 171. (d) No cube will have its opposite face red.
yellow appear in the sides. Now, putting figure
172. (b) The 4 corner cubes of the top layer and the
II in the position of figure III, we get violet
4 corner cubes of the bottom layer will have
in the empty space.
their three faces coloured.
164. (c) From figures A, C and D, we can deduce that
For Qs. 173 and 174, the cube will look as follows:
numbers on the faces adjacent to face 5 are
4, 6, 3 and 1. Hence face 2 is opposite face
5. From figures A and C, we get that numbers
in the faces adjacent to 6 are 4, 5 and 3.
We also know that face 2 is opposite face
5. Hence face 1 should be opposite face 6
and consequently face 4 should be opposite
face 3.
165. (b) Adjacent to 3 are faces bearing numbers 1,
2, 4 and 5. So only 6 can be on the face
opposite to 3.
166. (b) 167. (c)
168. (a) Only one cube—the central one in the middle
layer—will have cubes on all sides.

5 . 53
General Mental Ability

173. (d) You will observe that each layer has 9 central DEF is the only triangle having eight
cubes. But the top and bottom layers will have components.
colour on one side at least. The central cubes ABC is the only triangle having twelve
in the middle three layers will have no colour components.
on any face. So a total of 9 × 3 = 27 smaller
Thus, there are 8 + 10 + 5 + 2 + 1 + 1 = 27
cubes will have no face coloured.
triangles in the figure.
174. (a) There are six faces to the large cube. You
178. (d) Label the figure as shown.
can observe that the 9 smaller cubes at the
centre of each face of the larger cube have A
one face coloured. So 6 × 9 = 54 such cubes
are painted on one face only.
175. (a) If blue is adjacent to brown, and white is E
adjacent to blue, you can visualise that, in F
a cube, brown and white will be opposite
B C
faces. D
top (black)
Simplest triangles are AFE, EFC, CFD, BFD
and ABF i.e. 5.
brown

white

Triangles having two components are AFC,


blue CFB, ABD and BAE i.e. 4.
Triangles having three components are ADC
@UPSC_THOUGHTS

and EBC i.e. 2.


bottom (red) Triangles having five components are ABC
176. (c) i.e. 1.
177. (c) Label the figure as shown: Total number of triangles in the figure
= 5 + 4 + 2 + 1 = 12

B D C 179. (d) The figure may be labelled as shown.


L M H
G A B C
K N
L M
O Q
I J K
F E H D
P I R

N O
A
G F E
The simplest triangles are GKL, MHN, DLJ,
Number of Triangles:
DMJ, QRE, OPF, PIA and IRA i.e. 8.
Simplest triangles are ILJ, IJN, MJK, OJK,
The triangles having two components each, ABL, BCM, GNF and FOE i.e. 8.
are BDO, CDQ, DLM, PRA, KFI, NEI, HJI,
Triangles composed of two components are
GJI, DKI and DNI i.e. 10.
AHJ, CJD, LJN, MOK, GHJ and EJD i.e. 6.
The triangles having four components each, Triangles composed of three components are
are DIE, DFI, DOA, DQA and DHI i.e. 5. BIK and FIK i.e. 2.
The triangles having six components each, Triangles composed of four components are
are DCA and DBA i.e. 2. AGJ and CDE i.e. 2.

5 . 54
General Mental Ability

The only triangle composed of six components Number of squares:


is BFK. The squares containing two triangles each, are
Thus, there are 8 + 6 +2 + 2 + 1 = 19 triangles GJOK, JOIF, IOLE and LQKH i.e. 4.
in the figure. The squares containing four triangles each,
Number of Parallelograms: are BFOG, AFOE, EOHD and GOHC i.e. 4.
Simplest || gms are BLJM and FNJO i.e. 2. EFGH is the only square containing eight
The || gms composed of two components are triangles.
ABIH, HIFG, CBKD and DEFK i.e. 4. ABCD is the only square containing sixteen
The || gms composed of three components triangles.
are ABKJ, GFKJ, BCJI and IJEF i.e. 4. ∴ The total number of squares in the figure
The only || gm composed of four components = 4 + 4 + 1+1 = 10.
is ABFG.
181. (d) 182. (d)
The || gms composed of five components are
ACDJ, GEDJ, ACJH and HJEG i.e. 4. 183. (b) Ignore the difference in size. Concentrate only
The only || gm composed of six components on the shapes.
is BCEF. 184. (d) 185. (a) 186. (d) 187. (c) 188. (b)
The only || gm composed of ten components 189. (c) 190. (d) 191. (a) 192. (c)
is ACEG.
193. (d) In each row, the symbol rotates 135° anti-
Thus, there are 2 + 4 + 4 + 1 +4 + 1 +1 = 17
clockwise at every step from left to right.
parallelograms in the figure.
194. (d) Observe the shaded and the darkened portions
180. (a) Label the figure as shown below.
@UPSC_THOUGHTS

of each symbol in a row. In the bottom row,


there is an unshaded figure, and there is a
A F B
figure with the top half shaded. So the one
to replace the question mark will have its right
I J
half shaded. So (d) is the only option. You
E G may, however, check the pattern for other
O
K features too.
L
195. (b) Observe the pattern in each row, the shapes
D H C of heads and bodies differ from unit to unit
in each row. In the bottom row, a circle and
Number of triangles: triangle have been used for the faces. So it
is the turn of the square. Option (c) is unlikely.
Simplest triangles are AIF, IFO, IEO, AIE,
As the bodies in the options are all the same,
FBJ, BJG, JGO, FJO, GKC, HKC, HOK,
take up the arms. There is already a figure
GOK, OLH, LDH, ELD and ELO i.e. 16.
with upward slanting arms and another with
The triangles having two simple triangles straight arms. So it is the turn of the third
each, are AFE, EDH, HCG, FBG, EOH, HOG, type—downward slanting. Only option (b) is
GOF, EOF, AEO, BOG, BOF, AOF, DOE, correct. The figure in this option also has the
DOH, GOC and HOC i.e. 16. right kind of feet—small circles. [The arms
The triangles having four simple triangles in option (d) are wrong in shape.]
each, are AOD, DOC, COB, BOA, FEH, EGH, For Qs. 196 to 198 You have to locate the region
GFH and EFG i.e. 8. in which the dot/s fall in (x) whether it is only
The triangles having eight simple triangles in the triangle or circle, etc., or in a combination
each, are ADC, DBC, ABC and BAD i.e. 4. of shapes, and in which combination, if so.

∴ The number of triangles in the figure 196. (a) There are three dots in Figure (X)—one in the
= 16 + 16 + 8 + 4 = 44. region common to square and rectangle,

5 . 55
General Mental Ability

second in the region common to all the four square only; in figure (b), there is no region
figures and third in the region common to common to the circle and rectangle only and
rectangle and triangle. Figure (b) contains no in figure (c), there is no region common to
region common to rectangle and triangle only; the circle and square only. Only figure (d)
figure (c) contains no region common to contains all the three types of regions.
rectangle and square only and figure (d) 198. (a) In figure (X), one dot lies in the region
contains no region common to all the four common to the square, rectangle and triangle;
figures. Figure (a) contains all the three types another dot lies in the region common to the
of regions. square and the rectangle and the third dot lies
197. (d) In figure (X), there are three dots—one in the in the region common to the circle and square.
region common to the square and the circle, Figures (b), (c) and (d) do not contain any
second in the region common to all the four region common to the square, rectangle and
figures and the third in the region common triangle. Only figure (a) contains all the three
to the circle and the rectangle. In figure (a) types of regions.
there is no region common to the circle and 199. (b) 200. (b)

@UPSC_THOUGHTS

5 . 56
6
BASIC NUMERACY
AND
DATA INTERPRETATION
The UPSC syllabus for the civil services preliminary examination has rather
@UPSC_THOUGHTS

limited information on the topics to be tested in the section of Basic Numeracy:


mention is made of “numbers and their relations, orders of magnitude, etc.” The
‘etc.’, has to be interpreted by us. However, it is specified that the level of the
subject in this section is to be of the X class standard. Numeracy is not confined
to arithmetic but extends to other fields of mathematics as well: it means ‘basic
skills in mathematics’. So we may say the subject would cover basic arithmetic,
some algebra, geometry, and may be some trigonometry of an elementary kind,
such as heights and distances.
Data interpretation implies using various statistical tools to draw meaningful
conclusions from data, presented in whatever form. Quantitative techniques
will have to be put to use in data interpretation as well as in questions
on data sufficiency.
In the following pages, a review of basic mathematics is given along with solved
examples. This will familiarise you with the concepts and formulae and an
understanding of how to apply them. The practice exercises cover a wide range
of question types which call for not merely a proficiency in calculations but also
an ability to apply formulae correctly to given situations.
Basic Numeracy and Data Interpretation

@UPSC_THOUGHTS

6.2
Basic Numeracy and Data Interpretation

Basic Numeracy and


Data Interpretation
ORIENTATION

NUMBERS AND FUNDAMENTAL Thus, 680957 represents 7 units, 5 tens, 9 hundreds,


0 thousands, 8 ten thousands and 6 lakhs, or six lakh
OPERATIONS
eighty thousand nine hundred and fifty-seven.
The following table will be useful for translating
Numbers, Numerals and Digits
any number in numerical form to word form.
Numbers can be expressed in words and in numerals.
Place value
The number ‘three hundred fifty-seven’ is the word
form and ‘357’ is the numerical form. Expressing a

an nds)
number in words is called numeration and representing

)
(m n L s) akh
Te illion00 L )
s

ho ousa
re

n m s ( ion

ds
a number in numerals is called notation.

Te ns) Cro

(te ore mill

Te ed th

ds
Cr red e

us
Just as all words are made up of alphabets, all

nd
h
1
illio red

nd ror

illio ak
L n s)

re
sa
(hu n C
(B und

nd
(huakh

its
numerals are built up of digits. In our modern number

nd

ns
nt

ou

Hu

Un
Te
Th
H

system, we use ten digits—0, 1, 2, 3, 4, 5, 6, 7, 8 and


@UPSC_THOUGHTS

9—either individually or in combination with each


other to build a numeral. 9 8 7 6 5 4
10 10 10 10 10 10 10 10 10 10
3 2 1 0

Numbers are named in terms of the place value


system with base 10, in which the value of a digit
Digits from the right
depends on the place in which it is located in the
numeral. In integers (i.e., whole numbers and negative Incidentally, in British usage, billion means a million
numbers) the rightmost digit represents the units or ones million (i.e., 1012), while the American equivalent is a
place. Its value is calculated by multiplying its face thousand million (i.e., 109).
value (i.e., the digit itself) with 100 or 1. The second digit [The power to which 10 is raised indicates the
from the right represents the tens place. Its place value number of zeros in the numeral. So 100, i.e., no zero,
is calculated by multiplying its face value with 10. The so 1; 102, i.e., two zeros or 100.]
third digit from the right represents the hundreds place,
Number Classification
and so on.
The number 680957 can be expressed as follows: Natural Numbers
All counting numbers are natural numbers. Thus, a set
of natural numbers = (1, 2, 3, 4, 5 . . . )
Since each natural number is followed by another
natural number, the set of natural numbers is infinite.
Whole Numbers
● Whole numbers include natural numbers and zero.
Thus, the set of whole numbers = (0, 1, 2, 3, 4, 5 . . .)
● So, while all natural numbers are whole numbers,
zero is a whole number which is not a natural
number.
Integers
● Integers include whole numbers and negative
numbers. Positive integers are 1, 2, 3, 4, 5, and so

6.3
Basic Numeracy and Data Interpretation

on. Negative integers are –1, –2, –3, –4, –5, and so relationship between all these numbers is shown by the
on. However, 0 is neither positive nor negative. following Venn diagram.
● So, we may say, 0, 1, 2 … are non-negative integers.
● A number with decimal places is not an integer.
Real Numbers
Decimal Numbers
Decimal numbers are also composed of digits, each of
which has a value depending on the place in which
Natural Irrational
it is located. The place value system for the digits before
the decimal is the same as for the digits in integers.
Thus, in the decimal number 5436.2178,
6 is the units digit (value = 6 × 100 = 6),
3 is the tens digit (value 3 × 101 = 30),
4 is the hundreds digit (value 4 × 102 = 400), and
5 is the thousands digit (value = 5 × 103 = 5000). Numbers for which there exists no solution in the
After the decimal, the first digit is called the tenths digit, set of real numbers are imaginary numbers; 1 , for
the second one is called the hundredths digit, the third, instance, is an imaginary number. The solution of 1
the thousandths digit, and so on. Thus, in the number is i (or iota).
5436.2178, Thus, 4 4 1 = 2i
2 is the tenths digit,
25 25 1 = 5i
1 is the hundredths digit, 2
Also, i = –1
7 is the thousandths digit, and
@UPSC_THOUGHTS

8 is the ten thousandths digit. Odd and Even Numbers


Integers are either even or odd. Those which can be
Rational Numbers
divided fully by 2 are even numbers: –6, –4, –2, 0, 2,
Any number that can be expressed in the form of a
a 4, 6 are examples of even numbers. Any integer, no
fraction, i.e., in the form of , where a and b are
b matter how large, is even if its last digit is fully
integers, is a rational number. All integers can be
divisible by 2.
expressed as rational numbers.
Integers that cannot be divided fully by 2 are odd
4 –6 0 numbers –7, –5, –3, –1, 1, 3, 5, 7 are examples of odd
4= , –6= , 0
1 1 1 numbers. Any integer, no matter how large, is odd if
All decimal numbers whose decimals either its last digit is not fully divisible by 2.
terminate or repeat infinitely can be expressed as
rational numbers. For instance, Prime Numbers
Prime numbers are counting numbers that are divisible
214 107
2.14 by either 1 or themselves. Thus 2, 3, 5, 7, 11 are prime
100 50 numbers. The following are a few facts about prime
1.3 (= 1.3333 . . . = 4/3) are rational numbers. numbers:
Irrational Numbers ● All prime numbers are positive.
a ● Zero is not a prime number. Zero can be divided
A number that cannot be expressed in the form of ,
b by all counting numbers, even though the answer
where a and b are integers, is an irrational number.
Numbers whose decimals do not terminate and do not is always zero.
0 0 0
repeat cannot be expressed as rational numbers. 0, 0, 0
1 10 421
For example, the value of 2 is 1.414213562 . . . The
● 1 is not a prime number. 1 has only one factor—
decimals neither terminate nor repeat. 2 , therefore, is
itself.
an irrational number.
● 2 is the only even prime. All other prime numbers
Real Numbers are odd. That is because all even numbers except
Real numbers include natural numbers, whole numbers, 2 are divisible at least by 1, 2 and themselves. Of
integers, rational numbers and irrational numbers. The course, all odd numbers are not prime.

6.4
Basic Numeracy and Data Interpretation

Mathematical Signs and Symbols ● All counting numbers, except 0, 1, and prime
numbers, can be expressed in the form of multiples
+ Plus, the sign of addition, e.g., 5 + 3. It
of prime numbers. For instance, 92 can be expressed
also denotes a positive quantity, e.g., +3.
as 23 × 2 × 2, and 105 can be expressed as
– Minus, the sign of subtraction. It also
2 × 3 × 17. The product of prime numbers is called
denotes a negative quantity.
complete factorisation of the given counting
× Sign of multiplication.
number. There is only one complete factorisation of
÷ Sign of division.
any counting number.
. Dot at the centre of the two numbers is
● Pairs of prime numbers which have a difference of
the sign of multiplication.
two are known as twin primes. For instance, 3, 5,
. Dot at the base of the two numbers is the
7; 11, 13; 71,73 are twin primes.
sign of decimal.
● The set of three consecutive twin primes is called
= The sign of equality, read as equal to.
a prime triplet.
≠ The sign of not equal to.
There is only one such set—3, 5, 7.
> Sign of greater than.
≥ Sign of greater than or equal to. ● Pairs of numbers which have no common factor are
< Sign of less than. co-prime to each other. For instance, 7, 15; 8, 21;
≤ Sign of less than or equal to. 6,19; 9,16 are co-primes. The numbers in a pair
Sign of square root or under root. may or may not be prime in themselves.
3 Sign of cube root.
Composite Numbers
n Sign of ‘n’th root.
Composite numbers are counting numbers that are
a2 The square or the second power of a;
@UPSC_THOUGHTS

divisible by more than just 1 and themselves. For


a2 = a × a.
instance, 4, 6, 8, 9, 10 are composite numbers. Note the
a3 The cube or the third power of a;
following facts about composite numbers:
a3 = a × a × a.
● All composite numbers are positive.
an The ‘n’th power of a.
● Composite numbers can be even or odd.
|| Two vertical bars denote the absolute value
● Zero is not a composite number. (Zero is neither
of a number or mode of a number, e.g.,
prime nor composite.)
| – 4 | = 4.
● One is not a composite number. (One is neither
∝ Sign of infinity.
prime nor composite.)
( ) Sign of parenthesis.
● Two is not a composite number.
[ ] Sign of bracket.
{ } Figured bracket. Absolute Value
~ Sign of similarity. Absolute value is the number of units a number is away
≅ Sign of congruency. from 0 on a number line.
 Sign of since or because. Each integer has a location on a real number line,
∴ Sign of therefore or hence. where the sign of the number determines to which side
∪ Sign of union. of 0 the number is located.
∩ Sign of intersection.
⊆ Sign of subset. –5 –4 –3 –2 –1 0 +1 +2 +3 +4 +5
∈ Sign of ‘belongs to’ or ‘is a member of’.
∑ Sign of summation. The number 0 is neither positive nor negative.
∫ Sign of integration. Since absolute value is a measure of distance, it is
d always a positive value. The symbol for absolute value
Sign of differentiation.
dn is two bars on either side of a numerical value or
≡ Sign of identical to.
expression: the absolute value of 5 is written as |5|;
! Sign of factorial.
the absolute value of –5 is written as |–5|, and
i Sign of 1.
since –5 is 5 units away from 0 on a number line,
? Question mark.
|–5| = 5.

6.5
Basic Numeracy and Data Interpretation

Fundamental Operations: Addition, switching the sign of the number being


Subtraction, Multiplication and Division subtracted. Then simply follow the rules for
Fundamental operations include addition, subtraction, addition:
multiplication, and division. To find the value of –25 – 5, change the
subtraction sign to addition sign and change
Addition and Subtraction the +5 to a –5. The problem becomes –25 +
The number obtained after addition is called the sum (–5). Since the signs are both negative, add the
and the numbers to be added are called addends. The absolute values to get 25 + 5 = 30, but keep
operation of finding the remaining number when a the solution negative:
number is taken from another number is called –25 – 5 = –30
subtraction. The number which is taken from another ● Addition and Subtraction Suppose you have
number is called subtrahend and the number from to find the value of
which it is subtracted is called minuend. 7568 – 2248 + 732 – 945
In such questions where there are numbers to
Fast-track Ways to Add and Subtract be added as well as subtracted the following
● Speed in adding numbers can come only with steps are useful.
practice. Practise till you are comfortable with Step 1: First add the numbers with +ve
adding numbers mentally. Unless the sign
calculations are too long, you should not Step 2: Add the numbers with –ve sign
require a paper to put down numbers vertically Step 3: Subtract the result of step 2 from
and then add. that of step 1.
● To avoid wastage of time while adding In the given question,
@UPSC_THOUGHTS

numbers, leave out all superfluous words. For Step 1: 7568 + 732 = 8300
instance, while solving 2 + 9 + 6 + 4 + 5, do Step 2: –(2248 + 945) = –3193
not say 2 plus 9 is 11, 11 plus 6 is 17, 17 plus Step 3: Difference = 5107
4 is 21, etc. Just say 2, 11, 17, 21, 26.
● Addition of numbers in a single row Start Multiplication
adding the numbers from the units digit and The operation of finding the sum of a given number
strike off the digits which have been dealt repeated as many times as there are units in the other
with. given number is called multiplication. The number to
Let’s suppose we have to find the value of be repeated is called multiplicand and the other number
72901 + 82405 + 903 + 1142 + 31 is called multiplier.
As the first step, add up the digits at the The result of multiplying given numbers is called
units’ place of all the numbers and strike the product.
them off. Study the following methods:
= 2 (i) a b c
(1 is to be carried over to the tens’ place.) × d e f
Striking off the digits already added will
a×d a×e+b×d a×f+b×e+c×d b×f+c×e c×f
help in avoiding duplication.
= no = pm = lk = ji = hg
After this, take up the digits at the tens
Answer is nomkig
place and add carried over number, if any:
(h, j, l and p are the numerals to be carried over and
= 8 2
In the hundreds place— added to the next stage.)
=382 For example, 214 × 635
—and so on. (carry over 2 to thousands’ 2 1 4
place) × 6 3 5
The sum will be 157382. 2×6 2×3+1×6 2×5+1×3+4×6 1×5+4×3 4×5
● When subtracting integers, rewrite the = 13 = 15 = 38 = 19 = 20
problem as an addition problem by changing
the subtraction sign to an addition sign and Answer is 135890

6.6
Basic Numeracy and Data Interpretation

(ii) a b c d (i) 3856 × 99 = 3856 (100 – 1)


× e f g = 385600 – 3856 = 381744
(ii) 43516 × 999 = 43516 (1000 – 1)
a×e a×f+ a×g+b×f b×g+c×f c×g+d×f d×g = 43516000 – 43516 = 43472484
= rs b×e +c×e +d×e = kj = ih
= qp = on = ml ● Multiplication by 11: Put down the last digit
of the multiplicand as the units digit. Each
Answer is rspnljh successive digit of the multiplicand is added
to its neighbour at the right.
For example, 1435 × 620
1 4 3 5 (i) 432 × 11 = ?
× 6 2 0 Step I: Put down the last figure of 432 as
the units digit of the answer:
1×6 1×2+ 1×0+4×2 4×0+3×2 3×0+5×2 5×0
432 11
=8 4×6 +3×6 +5×6 = 10 =0
= 28 = 29 = 37 2
Step II: Each successive figure of 432 is
Answer is 889700 added to its right-hand neighbour.
432 11
(3 + 2 = 5, put 5 below the line)
Fast-track Ways to Multiply with Certain 52
Numbers 432 11
● Multiplication by 5, 50, 500, 5000, .... : To (4 + 3 = 7, put 7 below the line)
752
multiply a number by 5, 50 and so on, multiply 432 11
(first figure is 4, put it below the
@UPSC_THOUGHTS

10 100 10 100 4752


by , , and so on. 5, 50 ... line)
2 2 2 2
10 Answer is 4752.
8790
(i) 879 × 5 = 879 × = = 4395
2 2 (ii) 6547 × 11 = ?
1000 879000 6547 11
(ii) 879 × 500 = 879 × (Put 7 below the line)
2 2 7
6547 11
= 439500 (4 + 7 = 11, put 1 below the line
2 17
2
10 and carry over 1)
Also note: 879 × 25 = 879 × 5 = 879 ×
2
6547 11
879 100 87900 (5 + 4 + 1 = 10, put 0 below the
= 21975 017
4 4
line and carry over 1)
You can see that if any big number has to be
multiplied or divided by any multiple or power 6547 11
(6 + 5 + 1 = 12, put 2 below the
of 5, it can be done by expressing 5 as equal 2017
10 line and carry over 1)
to .
2
6547 11
Rather than full scale multiplication, this short (6 + 1 = 7, put 7 below the line)
72017
cut is useful for multiplying numbers by the
powers of 5. Answer is 72017.
For example, 56328 × 125 = 56328 × 53 ● Multiplication by 12: Double each digit in
3
10 56328000 turn and add to its neighbour. Same procedure
= 56328 × as done in multiplication by 11 except that
2 8
= 7041000 before addition, double the number.
846 × 12 = ?
● Multiplication by 9, 99, . . . : Place same number
of zeros to the right of the multiplicand as is 846 12
Step I (6 × 2 = 12, put 2 below the
the number of nines and from this number 2
subtract the real number to get the result. line and carry over 1)

6.7
Basic Numeracy and Data Interpretation

846 12 Take (95)2


Step II (4 × 2 + 6 + 1 = 15, put 5 (95)2 = 9 (9 + 1) / 25
52
= 90 / 25
below the line and carry over 1)
= 9025
846 12 When a number has every digit as one, the
Step III (8 × 2 + 4 + 1 = 21, put
152 method is to count the digits in the given
1 below the line and carry over 2) number and then write numbers in ascending
846 12 order from one to the number of digits and
Step IV (8 + 2 = 10, put 10 below the then in descending order up to 1.
10152
line) Take (111)2
Answer is 10152. There are three digits, so
ascending order
● Multiplication by 13: Triple each digit in turn
and add to its right neighbour. 1 2 3 2 1
348 × 13 = ? descending order
and 12321 is (111)2.
348 13 Now if it is (4444)2
Step I (8 × 3 = 24, put 4 below the
4 This is 42 (1111)
line and carry over 2) = 16 (1234321)
= 19749136
348 13 When a number is nearer to 10 or any of its
Step II (4 × 3 + 8 + 2 = 22, put 2 powers, such as 100, 1000, and so on, such
24
@UPSC_THOUGHTS

below the line and carry over 2) as 96 (which is just 4 short of 100) or 994
(which is just 6 short of 1000), use the formula:
348 13 x2 = (x + y) (x – y) + y2
Step III (3 × 3 + 4 + 2 = 15, put (where y is the difference between the number
524
whose square is to be found and the nearest
5 below the line and carry over 1)
power of 10)
Take an example: you have to find the square
348 13
Step IV (3 + 1 = 4, put 4 below the of 996. 996 is nearest to 1000. So,
4524
(996)2 = (996 + 4) (996 – 4) + 42
line) = (1000) (992) + 42
Answer is 4524. = 992000 + 16
● Multiplication by 14 and 15: The same = 992016
procedure as for multiplication by 12 and 13 Suppose you have to square 103. Now 103 is
nearest 100. So,
is used, except that instead of doubling or
(103)2 = (103 – 3) (103 + 3) + 32
tripling, the digit is multiplied by 4/5, and so = (100) (106) + 32
on before adding to the right neighbour. = 10600 + 9
● Multiplication by 25: Multiply the number by = 10609
100 and then divide by 4.
Division
46 × 25 = ?
The operation of finding how many times one number
46100 can be taken from another number is called division.
46 × 25 = = 1150
4 The number to be taken is called the dividend, and that
● Multiplying by same number or squaring in which tells us how many times it can be subtracted is
some cases: called the quotient. The number left behind from which
When a number ends in 5, say X5. the divisor cannot be taken any more is called the
Use the method: remainder.
(X5)2 = X (x + 1) / 25 Dividend = Divisor × Quotient + Remainder.

6.8
Basic Numeracy and Data Interpretation

Some rules to remember are: The least three-digit number is 100.


When we divide the dividend by divisor and no So divide 100 by 17
remainder is left, then the division is called exact
17 100 5
division.
In case of exact division, 85
Dividend = Divisor × Quotient 15
If there is a remainder, Now, if we simply minus 15 (remainder) from
Dividend = (Divisor × Quotient) + Remainder 100, we get 85. This may be divisible by 17,
or but it is not the least 3-digit number to be
Dividend – Remainder divisible by 17. So we use method (ii) and add
Divisor =
Quotient (divisor – remainder) to the dividend. We have
These formulae are useful to keep in mind. 100 + (17 – 15) = 102
If a number when divided by another leaves a So the required 3-digit number is 102.
remainder, and we need to find the closest number
Test for Divisibility
exactly divisible by the given divisor, we can use two
Divisibility rules for whole numbers are helpful in
methods:
finding factors. Some of these rules are given here.
(i) Subtract the remainder from the number
(dividend) ● Divisibility by 2 A number is divisible by 2 if the
So, here the remainder is the least number to be number is an even number, or if the unit’s digit
subtracted from any number to make that number is 0.
exactly divisible. ● Divisibility by 3 A number is divisible by 3 if the
@UPSC_THOUGHTS

(ii) Add to the dividend (divisor – remainder). sum of the individual digits in the number is
So, here the (divisor minus remainder) is the least divisible by 3. The number 18,036 is divisible by 3
number to be added to a number to make it exactly because 1 + 8 + 0 + 3 + 6 = 18, which is divisible
divisible. by 3.
● Suppose you have to find a number that when
● Divisibility by 4 A number is divisible by 4 if the
divided by 17 leaves a remainder of 14. If the last two digits taken together is divisible by 4 or if
quotient is 18, what is the number? the last two digits are 0s. Thus 2,11,736 and 4,01,700
Number or Dividend = Quotient × Divisor + are both divisible by 4, but 2,11,722 is not.
Remainder.
Now, Number = 18 × 17 + 14 = 320 ● Divisibility by 5 A number is divisible by 5 if the
● Suppose you have to find the greatest 3-digit
unit’s digit is either 5 or 0.
number divisible by 17. ● Divisibility by 7 A number is divisible by 7 if on
What is the greatest three digit number? It is 999 doubling the unit’s digit and deducting the sum
(as, after that we have 1000, a four-digit number). from the remaining number the result is divisible by
Divide 999 by 17: 7. Take 6,412. Double the unit’s digit 2 to get 4.
Deduct 4 from 641 to get 637, which is divisible by
17 999 58 7. Now, if 637 appears too big a number, repeat the
85 process. Double 7 to get 14, deduct 14 from 63 to
149 get 49, which is divisible by 7. So 6412 is divisible
136 by 7.
13 ● Divisibility by 8 A number is divisible by 8 if its
So 13 is the remainder. last three digits are 0, or a multiple of 8. Thus the
If we subtract 13 from 999 we get 986, which number 1,79,128 is divisible by 8 because the last
is the greatest 3-digit number exactly divisible three digits taken together, 128, is divisible by 8.
by 17. Again, the number 1,79,000 is divisible by 8.
● Suppose we have to find the least 3-digit number ● Divisibility by 9 A number is divisible by 9 if the
exactly divisible by 17. sum of the individual digits in the number is

6.9
Basic Numeracy and Data Interpretation

divisible by 9. Take the number 38,322. If we add Check the divisibility of x by all the prime numbers
the digits 3 + 8 + 3 + 2 + 2, we get 18, which is less than k. If x is not divisible, then x is a prime
divisible by 9. So the given number is divisible number.
by 9. ● Suppose you have to find out if (i) 287 and (ii) 191
● Divisibility by 10 Any number ending in 0 is are prime numbers.
divisible by 10.
(i) 17 is nearly the root of 287
● Divisibility by 11 If the difference between the sum
172 = 289 > 287
of digits in the even places and the sum of digits
in the odd places is either 0 or is a multiple of 11, The prime numbers less than 17 are 2, 3, 5, 7,
the number is divisible by 11. Take 4059. The 11, 13.
difference between (9+0) and (4+5) is 0. The number Sum of the digits of the given number, 287 =
is divisible by 11. Take 291918. The difference 2 + 8 + 7 = 17, not divisible by 3. (Refer to
between (2+1+1) and (9+9+8) is 22, which is divisible shortcut rules of divisibility.) Obviously 287 is
by 11. not divisible by 2 or 5. But 287 is divisible
● Divisibility by 25 A number is divisible by 25 if the by 7.
last two digits of the number are 0s, or taken together ∴ It is not a prime number.
is divisible by 25. The numbers divisible by 25 end in
(ii) The nearest value of 191 is 14.
00, 25, 50, 70. 2
14 = 196 > 191
● Divisibility by 99 Starting from the unit’s place
The prime numbers less than 14 are 2, 3, 5, 7,
make pairs of the digits. Add the pairs and any odd
number left. If the sum is divisible by 99, the given 11 and 13
@UPSC_THOUGHTS

number is also divisible by 99. Take 95337. Making 191 is not divisible by 2 and 5.
pairs as told, we get 37 + 53 + 9 = 99, which is Sum of the digits = 1 + 9 + 1 = 11, not divisible
divisible by 99. So 95337 is divisible by 99. by 3 either. It is not divisible by 7, 11 or 13.
The methods given above can be used to find ∴ 191 is a prime number.
whether a number is divisible by a divisor which
can be written as a product of two or more co-prime Some Facts to Remember
numbers.
● Addition or multiplication of any type of
● To test for divisibility by 6, test if the number is numbers (natural, whole, integers, or rational)
divisible by 2 as well as 3 (2 and 3 being co-primes)
gives the same type of number as a result.
● For divisibility by 12, test for divisibility by co-
For example,
primes 3 and 4.
2 + 3 = 5 (all natural numbers)
● For divisibility by 14, test for divisibility by co- 2 × 3 = 6 (all natural numbers)
primes 2 and 7.
2 + (–3) = –1 (all integers)
● For divisibility by 15, test for divisibility by 3 2 × (–3) = –6 (all integers)
and 5.
● Addition or multiplication of numbers does not
● For divisibility by 18, test for divisibility by 2
and 9. depend on the order of the addends or
multiplicands:
Test of a Prime Number a + b = b + a
Prime numbers less than 100 are: a × b = b × a
2, 3, 5, 7, 11, 13, 17, 19, 23, 29, 31, 37, 41, 43, a c
For rational numbers, and ,
47, 53, 59, 61, 67, 71, 73, 79, 83, 89, 97 b d
a c c a
Finding out a prime number > 100
Let x be the given number. Find a number bigger than b d d b
the approximate square root of the number x. Let it a c c a
be k such that k2 > x. b d d b

6 . 10
Basic Numeracy and Data Interpretation

● Addition or multiplication of numbers does not number, then the sum will definitely be positive
depend on how the numbers are grouped. For (+ve).
numbers a, b and c: For example (+) 10 + (+) 12 = (+) 22
a + (b + c) = (a + b) + c (+) 7 + (+) 8 = (+) 15
● If a positive number is added to a negative
a × (b × c) = (a × b) × c
number, then the difference between the numbers
a c e
For rational numbers, , and f is found out and then the sign of the bigger
b d number among them is given. For example:
a c e a c e +10 – 12 = –2
b d f b d f 7 – 18 = –11
a c e a c e 100 – 125 = –25
b d f b d f 125 – 100 = +25
18 – 7 = 11
Also note the following: ● If both the numbers are negative and then the
● The property of identity is applicable in case of sum is found out, the negative (–ve) sign is
whole numbers, integers and rational numbers. given.
In addition of numbers, the additive identity For example: –10 – 12 = –22
is 0. –7 – 8 = –15
a + 0 = a –100 – 125 = –225
a 0 a ● Now in the case of multiplication, when a

b c b positive number is multiplied by a positive


number, then the product will be a +ve number.
@UPSC_THOUGHTS

In multiplication of numbers, the multiplication


When a negative number is multiplied by a
identity is 1.
negative number, then the product will be a
a × 1 = a positive number.
a 1 a
When a positive number is multiplied by a
  negative number, the product is always
b 1 b
● Additive inverse is applicable for integers and negative.
rational numbers. This property is based on the For example:
fact that every number has an inverse. If the 2 × 4 = 8, –2 × –4 = (+)8
operation is addition, the number ‘–a’ is said to 7 × 3 = 21 –7 × –3 = (+)21
be an additive inverse of the number ‘a’. Thus (Note: +8 and 8 are the same. If a sign is not
a + (–a) = 0 given specifically before the number, it’s taken
as a positive number.)
a a
0 Now, –2 × +4 = –8, 2 × –4 = –8
b b 7 × –3 = –21, –7 × 3 = –21
b Some Rules on Counting Numbers
● The number is said to be the multiplicative
a (i) Sum of all the first n natural numbers =
a
inverse of . n (n + 1)
b
2
a b
1 (ii) Sum of first n odd numbers = n2
b a
(iii) Sum of first n even numbers = n (n +1)
● a × (b + c) = (a × b) + (a × c) (iv) Sum of squares of first n natural numbers
However, n (n + 1) (2n + 1)
a + (b × c)  (a + b) × (a + c) = 6
● A number is considered positive if no sign is (iv) Sum of cubes of first n natural numbers
given before the number. n (n + 1)
2
=
● If a positive number is added to another positive 2

6 . 11
Basic Numeracy and Data Interpretation

FRACTIONS AND DECIMALS 6 3


(ii)
7 4
6 × 4 = 24; 7 × 3 = 21
Vulgar Fractions
a 6 3
A fraction is a rational number written in the form .
b 7 4
The variable a is called the numerator, and the variable
b is called the denominator. The denominator cannot Operations on Fractions
have the value of 0. ● To add or subtract fractions, the denominators must

This type of fraction is known as common fraction be converted to become the same: the LCM must be
or vulgar fraction. found to do so. Then add or subtract the numerators
The reciprocal of a fraction is that fraction inverted. and keep the denominator. If necessary, simplify the
Since every whole number has the denominator 1 result and fraction. (The method is discussed in the
understood, the reciprocal of a whole number is a next part—HCF and LCM.)
fraction having 1 as numerator, and the number itself ● When performing operations using mixed numbers,
1
as the denominator; e.g., the reciprocal of 6 is . it is helpful to first change the mixed number into
6
Comparison of Fractions an improper fraction and then use the correct
● To find an equivalent fraction, multiply or divide the procedure for fractions.
a
fraction by the number one, in the form of
a
. For Change A x to an improper fraction by multiplying
3 1 3 3 1 y
example, is equivalent to because . To y and A, then adding the product to x. Put this
9 3 9 3 3
1
number over y . ( A y) x
@UPSC_THOUGHTS

find an equivalent fraction to , multiply by some .


3 y
1 4 4
fractional form of one: . ● To multiply fractions (a) change mixed numbers, if
3 4 12
any, to improper fractions. (b) Multiply all numerators
● If two fractions have the same denominator, the one and place this product over the product of
3
having the larger numerator is the greater fraction: denominators.
2 7
is greater than . 2 4 2
7 Take  2  = ?
● If two fractions have the same numerator, the one 3 7 3
having the larger denominator is the smaller fraction: Change mixed numbers into improper fractions.
5 5 4 18
is smaller than . 2 =
12 11
7 7
● To compare two fractions having different numerators
2 18 5 180 20
and different denominators, change the fractions to Now,    
3 7 9 189 21
equivalent fractions having common denominators, To divide fractions, change mixed numbers to
and compare. (See the next part on LCM.) improper fractions. Invert the second fraction (or
● When fractions are to be compared, the fraction with divisor) and multiply.
lower numerator and higher denominator is lower 2 1
Suppose we have to divide by 2
215 217 3 4
in value than the other fraction. Of and , 1 9
341 334 2 =
the first is smaller in value. 4 4
● To compare two fractions, cross multiply denominator 2 9 2 4 8
   
and numerator of the fractions as shown below: 3 4 3 9 27
5 3
(i) Decimals
7 4
5 × 4 = 20; 7 × 3 = 21 Decimal numbers are also composed of digits, each of
5 3 which has a value depending on the place in which
7 4 it is located. The place value system for the digits before

6 . 12
Basic Numeracy and Data Interpretation

the decimal is the same as for the digits in integers (as decimal. So mark off decimal point in the above
explained earlier). Thus, in the decimal number product taking 4 places from the right.
5436.2178,
Thus 21.255 × 13.4 = 284.8170 = 284.817
6 is the units digit (value = 6 × 100 = 6),
3 is the tens digit (value 3 × 101 = 30), ● In division, both dividend and divisor are multiplied
4 is the hundreds digit (value 4 × 102 = 400), and by 10 or a power of 10 so that the divisor becomes
5 is the thousands digit (value = 5 × 103 = 5000). a whole number, i.e., the decimal points in both the
After the decimal, the first digit is called the tenths dividend and divisor are shifted by as many places
digit, the second one is called the hundredths digit, the to the right as will make the divisor a whole
third, the thousandths digit and so on. Thus, in the number. For example:
number 5436.2178,
2 12.6061 12.6061 102 1260.61
2 is the tenths digit ,
10 11.02 11.02 10 2 1102
1
1 is the hundredths digit , Now you may go ahead with the division proper.
100

7 Just place the decimal in the quotient when bringing


7 is the thousandths digit , and down the digit after the decimal in the dividend.
1000
8 1102 1260.61 1.14
8 is the ten thousandths digit .
10000 1102
A decimal is actually a fraction, the denominator 1586
of which is understood to be 10 or a power of 10. The 1102
@UPSC_THOUGHTS

number of digits, or places after a decimal point, 4841


determines which power of 10 the denominator is: 4408
3 56 641
441
7121
.3 ; .56 ; .641 ; .7121
10 100 1000 10000 You may now add zeros to the remainders and go
The addition of zeros after a decimal point does not on dividing till you get no remainder or you get just
change the value of the decimal; 7 = .70 = .700 . . . a repetition of the number in the quotient, in which case
you have a recurring decimal, and you may stop.
Operations on Decimals Example of recurring decimal are:
● In adding and subtracting decimals, (i) the numbers 1.3333 …
are so arranged that the decimal points come in the 3.142857142857
same vertical line, units come under units, tens These are pure recurring decimals, i.e., a set of
under tens, etc., (ii) numbers so arranged can now figures is repeated.
be added and subtracted in the usual manner, (iii) If it is a single figure repeated, we put a dot over
.
the decimal point in the answer is put directly the figure: 1.3 ; if it is a set of figures repeated, we put
under the column of decimals. a line (or bar) over the figures: 3.142857 .
● To multiply a decimal by another, multiply the two Mixed recurring decimals occur when there is no
numbers ignoring the decimal points. In the product, order in the repeated figures, e.g., 0.18444; this can be
mark off as many decimal places from the right as expressed as 0.184 .
the sum of the decimal places in the multiplicand Convert to Decimal to Compare Fractions
and the multiplier, perplexing zeros if necessary. ● The decimal form of numbers makes comparisons

For example, when we are to multiply 21.255 by easy. To order or compare vulgar fractions, convert
13.4, we ignore the decimal point and multiply the fractions to decimals by dividing, and then order
21255 by 134, and get the product 2848170. Now the decimal equivalents.
11 16
count: the multiplicand contains three places of If you have to compare and , divide the
3 5
decimal and the multiplier contains one place of numerators by the respective denominators.

6 . 13
Basic Numeracy and Data Interpretation

11 16 (i) Let the maximum number of digits along all


3.7 ; 3.2
3 5 the numerators be n, and the maximum number
11 16 of digits among all the denominators be d.
Now, 3.7 > 3.2, so
3 5 Find d – n.
Converting Decimals into Vulgar Fractions (ii) If d – n = 0 or 1, multiply each fraction by 10.
If you have to convert a decimal into a vulgar fraction,
If d – n = 2, 3, 4, … multiply each fraction by
put 1 in the denominator under the decimal and annex
10 to an equivalent power: 102, 103, 104, ...
to it as many zeros as there are digits after the decimal
respectively.
point. So,
(iii) After multiplication, find only the integer value
5 1 of the resultant fraction at first stage.
(i) 0.5 =
10 2
(iv) If, in the above step, any two fractions have
225 9 1 the same integer value, find the decimal values
(ii) 2.25 = 2
100 4 4 one after another till you get differences in
If a pure recurring decimal is to be converted to a value for all fractions.
vulgar fraction, write the repeated digits only once in (v) Now you can compare the fractions or arrange
the numerator and place as many 9s in the denominator them as required.
as the number of repeating digits. So,
[Ascending order: from the smallest to the biggest.
. 4
(i) 0.4 = Descending order: from the biggest to the smallest.]
9
143 Suppose you have to arrange these fractions in
@UPSC_THOUGHTS


(ii) 1.43 =
99 ascending order:
71
(iii) 0.071 = 8 494 87 123
999 , , ,
If a recurring decimal is to be converted into a 13 971 165 235
vulgar fraction, follow the steps: You note that the maximum number of digits
1. for the numerator, take the difference between among the numerators, n = 3 [494, 123]
the number formed by all the digits after the
decimal point (taking repeated digits only once) The maximum number of digits among the
and the number formed by the non-repeated denominators, d = 3 [971, 235]
digits. d – n = 3 – 3 = 0, so multiply each fraction by 10.
2. for the denominator, put as many 9s as the
8 80
number of repeating digits followed by as many 10 =
13 13
zeros as the number of non-repeating digits.
So, 494 4940
10 =
. 49 4 45 1 971 971
(i) 0.49 =
90 90 2 87 870
10 =
2271 22 2249 165 165
(ii) 0.2271 =
9900 9900 123 1230
10 =
(iii) 2.071 = 2 0.071 235 235
Find the integer value; you have 6, 5, 5, 5
71 0 71 71 respectively.
= 2 2 2
990 990 990 8
So you have the greatest fraction,
Another method can be used if we have to compare 13
a number of fractions with large denominators and Now you need to compare the other three as the
numerators, and arrange them in order. integers are same.

6 . 14
Basic Numeracy and Data Interpretation

Divide further to first decimal digit. Now let’s find the HCF of 1560 and 1755
4940 1560 = 2 × 2 × 2 × 3 × 5 × 13
= 5.0
971
1755 = 3 × 3 × 3 × 5 × 13
870
= 5.2 HCF = 3 × 5 × 13 = 195
165
1230 We can use the division method to find the HCF
= 5.2 too. Let’s take the case of two numbers.
235
Step 1 Divide the bigger number by the smaller
Now you have the smallest fraction, 494 . number and find the remainder.
971 Step 2 Divide the first divisor by this remainder
We need to go further to second decimal digit for and find the second remainder.
the other two fractions. Step 3 The second divisor is to be divided by the
870 second remainder to find the third remainder
= 5.27 and so on. . ., till no remainder is left. The
165
last divisor will be the HCF of the given
1230 numbers.
= 5.23
235
Now you can arrange the fractions in ascending ● If the HCF is to be found for 15 and 475, we follow
order: the steps:
494 123 87 8 15) 475 (31 ... Step I
, , ,
@UPSC_THOUGHTS

971 235 165 13 45


25
HCF AND LCM 15
10 ) 15 (1 ... Step II
A factor of a number a is a whole number that divides 10
into a evenly without remainder. 5) 10 (2 ... Step III
All the factors of a number can be reduced down 10
into only prime numbers. Every number has a unique × HCF is 5.
set of prime factors.
A common factor of two or more numbers is a ● If there are more than two numbers, (say, three
number that divides each of them exactly. Thus, 2 is numbers), find the HCF of any two numbers, then
a common factor of 4, 12, 18, 22. find the HCF of the third number and the HCF of
the first two numbers. That will be the final HCF.
Highest Common Factor (HCF)
The greatest common factor (GCF), also called highest Suppose we have to find the HCF of 48, 72 and 96.
common factor (HCF), is the largest common factor of
two or more numbers.
∴ HCF is 24.
Prime factorisation is a convenient way to find the
GCF of two or more numbers. After finding the prime
factors of the numbers given to you, find the common
factors in pairs. Multiply these common factors to get Now find the HCF of 24 and 96
the GCF.
To find GCF of 24 and 96, break up each into prime 24 is the HCF.
factors:
24 = 2 × 2 × 2 × 3
HCF for Decimals First, make the same number of
96 = 2 × 2 × 2 × 2 × 2 × 3 places of decimals in the given numbers. Then consider
Common factors are 2 × 2 × 2 × 3 = 24 these numbers without decimals. Now calculate their

6 . 15
Basic Numeracy and Data Interpretation

HCF. Lastly, there must be as many decimal places in product of the divisors and the undivided number is
the result as there are in each of the numbers. the required LCM.
● To find HCF of 16.50, 0.45 and 15 you do as ● Suppose we have to find the LCM of
follows: 26, 48, 88, 110
Step I 16.50, 0.45 and 15.00 We divide as shown:
Step II Consider these numbers without 2 26, 48, 88, 110
decimals: 1650, 45, 1500 11 13, 24, 44, 55
Step III Now calculate their HCF, which is 15. 4 13, 24, 4, 5
Step IV Now place decimal in the result to 13, 6, 1, 5
get 0.15
LCM = 2 × 11 × 4 × 13 × 6 × 5 = 34320
Least Common Multiple (LCM) LCM of Decimals The same procedure is applied
A common multiple of two or more numbers is a as in ‘HCF of decimals’. Make the same number of
number which is exactly divisible by each of them. Thus decimal places in the given numbers, then find their
20 is a common multiple of 2, 4, 5 and 10. LCM as if they are integers; in the end, mark decimal
The least common multiple (LCM) is the smallest places in the result.
number that two or more numbers will divide into If you have to find the LCM of 0.6, 9.6 and 0.36,
evenly. Again, the prime factorisation of numbers is the you do as follows.
way to find the LCM of given numbers. Step I 0.60, 9.60, 0.36
Write the factors of two numbers. Strike out the Step II Consider the numbers without decimals:
factors in the second number, which are already there 60, 960, 36
@UPSC_THOUGHTS

in the first number. Multiply the remaining factors of Step III Calculate their LCM, which is 2880.
the second number with the factors of the first number Step IV Put decimal places in the result to get 28.80
to get the LCM of the two numbers. If there is another
number, write down its factors. Strike out the factors Fast-Track Formulae and Facts
which are already included in the LCM of the first two
● If HCF of two numbers is 1, then they are
numbers. Multiply the remaining factors to get LCM of
co-primes.
the three numbers.
The process may be repeated for the other numbers HCF of Numerator
● HCF of fraction =
if there are more. LCM of Denominator
For example, to find the LCM of 12, 15 and 18,
LCM of Numerator
factorise 12 and 15. ● LCM of fraction =
HCF of Denominator
(i) 12 = 2 × 2 × 3
(ii) 15 = 3 × 5 ● Product of two numbers = Product of their HCF
Ignore 3 in (ii) as it is included in (i). and LCM.
LCM of 12 and 15 is 2 × 2 × 3 × 5 ● HCF (being a factor or a divisor) will be smaller
Now, entity than LCM (being a multiple or dividend.)
(iii) 18 = 2 × 3 × 3 ● To find the greatest number that will exactly
As 2 × 3 is already there in the LCM of 12 divide x, y and z, the required number = HCF (the
and 15, ignore it, retaining only the 3 left. greatest divisor) of x, y and z.
LCM of 12, 15 and 18 is 2 × 2 × 3 × 5 × 3 = 180 ● To find the greatest number that will divide x, y
and z leaving remainder a, b and c respectively,
We can also find the LCM by division method.
the required number = HCF of (x – a), (y – b) and
Arrange the given numbers in a row. Divide by a
(z – c).
number which divides exactly at least two of the given
numbers. Write the quotients and undivided numbers ● To find the least number which is exactly divisible
in the next line. Repeat the process until you get a line by x, y and z, the required number = LCM of x,
of numbers which are prime to one another. The y and z.

6 . 16
Basic Numeracy and Data Interpretation

● To find the least number (smallest number) which


INDICES, ROOTS AND SURDS
when divided by x, y and z leaves the remainders
a, b and c respectively, the required number = Indices
(LCM of x, y and z) – k
If a is a real number > 0, and m is a rational number,
where k = x – a = y – b = z – c.
then
● To find the least number which when divided by a a a ... a
x, y and z leaves the same remainder ‘k’ in all am =
m times
three cases, then the required number = LCM of
x, y and z + k. am is ‘a raised to m’ or ‘a raised to the power of m’.
a is the base; m is the exponent or index.
● (i) To find the greatest n-digit number which is
exactly divisible by x, y and z (that means, Rules for Indices
remainder in each case is zero). Here, first The expression n
● am is read as ‘a raised to the
find the LCM of x, y and z, then the greatest
power m, the whole raised to n’.
n-digit number ÷ LCM gives a remainder,
n
say, k; (the quotient is immaterial here) then am = am n

the required greatest n-digit number exactly 3


22 = 22 3
2 6 = 64
divisible by x, y and z = the greatest n-digit
number–k. ● am an = am n

(ii) If in the above case, it left a remainder ‘r’ in


22 23 = 22 3
2 5 = 32
each case, then the required number = the n
[It will be seen that am is not the same as am × an;
above answer in (i) + r. The greatest 2, 3, 4,
@UPSC_THOUGHTS

n
nor is it to be confused with a m .
5. . . digit numbers are 99, 999, 9999, 99999. . . n
● a m is read as a raised to the power m power n.
respectively.
3
22 = 2
8
● (i) To find the smallest n-digit number, which
is exactly divisible by x, y and z, find the am
LCM of x, y and z; then the smallest n-digit ● = am n
an
number ÷ LCM gives a remainder say k. 23
∴ The required smallest n-digit number = 23 2
21 = 2
22
exactly divisible by x, y and z = smallest
n-digit number – k + LCM. When the bases are different, we have
n
(ii) If in the above case, it leaves behind a ● an bn = a b
2
remainder ‘r’ in each case then the required 22 32 = 2 3 62 = 36
smallest n-digit number = the above answer m
am a
in 7(i) + r. ●
The smallest 2, 3, 4, 5, . . . digit numbers are bm b
10, 100, 1000, 10000, . . . respectively. 2
22 2
(i)
● (i) To find a relation between product of 3 32 3
numbers p, q, r and their LCM and HCF. 2 2
22 . 82 2 8 16
LCM (p, q, r) (ii) 42
42 4 4
p. q. r. HCF ( p, q, r )
=
HCF ( p, q) . HCF (q, r ) . HCF ( p, r ) Remember also…
(ii) HCF (p, q, r) a0 = 1 (always where a is a rational number)
n 0
p. q. r. LCM ( p, q, r ) a 1
=
LCM ( p, q) . LCM (q, r ) . LCM ( p, r ) 0
an 1

6 . 17
Basic Numeracy and Data Interpretation

If am = bm, then a = b. There are two possibilities for the square root of
If am = an, then m = n. positive numbers; the positive one is called the square
If an = 1, then n is 0. root. Thus we say 9 = 3 although (–3) × (–3) is also
1 9. Since the square of any non-zero number is positive,
a n
an the square root of a negative number is not defined as
1
an n
a , i.e., the nth root of a. a real number. Thus 2 is not a real number.
m There are cube roots of negative numbers.
an n
a m , i.e., the nth root of a to the power m. 3
8 = –2, because (–2) × (–2) × (–2) = –8.
You can also write root as exponents: for example
If you raise a number a to the nth power and the n
a a1/n ; so a a1/2 , 3 a a1/3
result is b, then a is called the nth root of b, which is a1/n b1/n ( a . b )1/n or n a n
b
usually written n b = a. This formula is the basic formula for simplifying
n
The second root is called the square root and is b roots, square roots, cube roots and so on.
written as , the third root is called the cube root, 3 .
Finding Square Root
Squares and cubes of natural numbers The square root of a given number can be found by
(1 to 25) means of
(i) prime factorisation method,
Square Cube (ii) long division method
Prime factorisation method In this method, the
1 1 1
given number is expressed in the form of its prime
@UPSC_THOUGHTS

2 4 8
factors. Then we make pairs of the similar factors and
3 9 27
choose one out of every pair. Now the product of these
4 16 64
5 25 125 chosen numbers is the square root of the given number.
6 36 216 For example, suppose you have to find the square root
7 49 343
of 676. Find the prime factors.
8 64 512
9 81 729 2 6 7 6
10 100 1000
2 3 3 8
11 121 1331
12 144 1728 13 1 6 9
13 169 2197 13 1 3
14 196 2744 1
15 225 3375
676 = 2  2  13  13
16 256 4096
= 2×13
17 289 4913
18 324 5832 = 26
19 361 6859 Long division method Make the pairs of the digits
20 400 8000
of the given number from right to left and then divide
21 441 9261
the number. The quotient is the answer.
22 484 10648
23 529 12167 It can be better understood with the help of the
24 576 13824 following example.
25 625 15625
Suppose we want to find the square root of 119716.
Note: (i) The square of a number, other than 1, is
either a multiple of 4 or greater than a Step I We make pairs of the digits of the number
multiple of 4 by 1. from right to left.
(ii) No perfect square ends in an odd number,
zero, or in any of these—2, 3, 7, 8. 11 97 16

6 . 18
Basic Numeracy and Data Interpretation

The last digit may be either single or a Square root of decimals It can be understood better
pair. with the help of the following example.
Step II Now we start dividing the number. Firstly, Square root of 27.04 is
we have to find a number whose square
5 27.04 5.2
may be equal to or less than the first pair
25
(here, 11).
102 204
In our case the number is 3. 204
×
∴ Square root of 27.04 is 5.2.

Finding Cube Root


Step III We get the remainder (here, 2) and bring
The cube root of a given number can be found by means
down the next pair. Double the quotient
of prime factorisation method. In cube root, grouping
(here, 3) and write to the left.
of three similar factors is done.
3 11 97 16 3
9 For example, the cube root of 551368 is
6 297
Step IV At this stage, the divisor will be equal to 2 551368
double the above quotient (here, 6) with 2 275684
the new quotient (here, 4). 2 137842
41 68921
41 1681
@UPSC_THOUGHTS

41 41
1

3
Step V We get the remainder (here, 41) and bring 551368
down the next pair. Now we take 34 × 2 = 2  2  2  41  41  41
= 68 for new divisor with the quotient of = 2×41
this stage (here, 6)
= 82
[A variety of problems are asked on square roots
and cube roots. We have given many types in the
practice exercises with detailed explanatory notes on
how to solve them. You have to remember the formulae
given in this chapter and learn to apply them.]

Surds
We repeat these steps till all the pairs get exhausted. If a is a positive rational number, and n is a positive
Here, square root of 119716 is 346. 1
Let’s take another example: find the square root of integer such that a or a n is irrational, then
n n
a is
15625. called a surd of order n.
1 15625 125 n
a is called the radical sign.
1
22 056 In n a , a is the radicand, and n is the order of the
44 surd, also known as radical index.
245 1225
1225 Rules to remember
1
× (i) n
a an
n n n
∴ 15625 = 125 (ii) ab a b

6 . 19
Basic Numeracy and Data Interpretation

n 1 1
a a
(iii) n
n These terms are 6 3 and 4 5
b b
(iv) (n a )n a We need to find the LCM of 3 and 5.

mn
LCM of 3 and 5 is 15.
mn
(v) a a
Now,
n m n m
(vi) ( a) a 1 5 1 1
3 15 15 15
6 6 65 7776
(vii) If x = n(n + 1), then
1 3 1 1
and 4 5 15 15 15
n  x  x  x  ....... 4 4 3
64
3 5
We may conclude 6 4
n+1 = x  x  x  .......
[We need not even calculate 65, as obviously
Rationalising Surds 65 > 43.]
Rationalisation is the process of removing the radicals, ● You may be asked to arrange a set of surds in
i.e., roots ( ) from an expression or a part of it without ascending/descending order.
changing the value of the whole expression.
Take 4 6 , 2 and 3 4
In a given fraction in the form of roots, we have
Find the LCM of the order of the surds.
to try and change the root form of the denominator by
LCM of 4, 2 and 3 is 12.
multiplying the denominator with a suitable quantity
@UPSC_THOUGHTS

1 3 1 1
such that it becomes an integer. 4 4 12 3 12 12
6 6 6 6 216
If n a n b is a rational number, then each of the
two surds is known as a rationalising factor of the 1 6 1 1
2 12 6 12 12
other. 2 2 2 2 64
If the surd is of the type, a + b , its rationalising factor 4 1
1 1
is a – b . 3
4 4
3
4
12
44
12
256
12
Rationalising factor of
In descending order, we have
1 1 1 1
(i) is a
a 12 12 12
256 , 216 , 64
1 Or 3
4 4
6 2
(ii) is a  b
a b
4 3
(The ascending order will be 2 6 4)
1
(iii) is a b
a b
SIMPLIFICATION
Comparing Surds
Surds can be compared only if they are of the same Using VBODMAS
order. The radicals can then be compared.
To simplify a given expression or problem fast and
3 3
● Comparing 4 and 8 , we may say that since accurately, the magic word is VBODMAS. In a lengthy
8 > 4, and complicated algebraic expression, start from the left
3 3 and apply VBODMAS in the same order as it appears
8 4
in the term, where
● If the surds are of different order and different
V stands for vinculum or bar : ——
base, we need to reduce them to the same order.
B stands for brackets : [{( )}]
3 5
Take 6 and 4 O stands for of : of

6 . 20
Basic Numeracy and Data Interpretation

D stands for division :  Fast-track formulae to Remember and


M stands for multiplication : × their Application
A stands for addition : + Sometimes, simplification needs some specific formulae
S stands for subtraction : – for solving problems fast and accurately. Some of these
In simplification of a problem, we first remove the important formulae are as follows:
bar and brackets in the order parenthesis ( ), then curly (i) (a + b)2 = a2 + b2 + 2ab
brackets { }, then the square bracket [ ]. Then we proceed (ii) (a – b)2 = a2 + b2 – 2ab
with the arithmetic operations of division, multiplication, (iii) (a + b)2 = (a – b)2 + 4ab
addition and subtraction in that order. or
(a + b)2 – (a – b)2 = 4ab
● Let’s simplify (iv) (a + b)2 = 2(a2 + b2) – (a – b)2
 1  1  1 1 or
1 1  
7 of 1  2      (a + b)2 + (a – b)2 = 2(a2 + b2)
3  4  2  2 2 4  

 (v) a2 + b2 = (a + b)2 – 2ab
or
1 5 5 1 1
Solution 7 of + + a2 + b2 = (a – b)2 + 2ab
3 4 2 2 4
(vi) a2 – b2 = (a + b) (a – b)
(vii) a3 + b3 = (a + b) (a2 – ab + b2)
= 7  1 of  5   5  3  or
3  4 2 4 
a3 + b3 = (a + b)3 – 3ab (a + b)
(viii) a3 – b3 = (a – b) (a2 + ab + b2)
= 7  1 of  5  7 
4 4 or
@UPSC_THOUGHTS

3  
a3 – b3 = (a – b)3 + 3ab (a – b)
= 7  1 of 3 (ix) (a + b)3 = a3 + 3a2b + 3ab2+b3
3
or
= 7 1  3 (a + b)3 = a3 + b3 + 3ab (a + b)
3 (x) (a – b)3 = a3 – 3a2b + 3ab2– b3
=7 – 1 or
=6 (a – b)3 = a3 – b3 – 3ab (a – b)
(xi) a4 – b4 = (a2 + b2) (a + b) (a – b)
● Now let’s simplify (xii) (a+b+c)2 = a2 + b2 + c2 + 2ab + 2bc + 2ca
(xiii) (a + b + c)3 = a3 + b3 + c3 + 3(b + c) (c + a) (a + b)
1  1  1 1 1  (xiv) a3 + b3 + c3 – 3abc
3   of  3  2  
2  3  2 2 2  = (a + b + c) (a2 + b2 + c2–ab–bc–ca)

or
1  1  1 5 1   a3 + b3 + c3 – 3abc
Solution 3   of  3     1
2 3  2 2 2  = (a + b + c) [(a – b)2 +(b–c)2 +(c–a)2]
   2
(xv) If a + b + c = 0, then a3 + b3 + c3 = 3abc
7 1  7 
=   of   2  a3  b3  c3  3abc
2  3  2  (xvi) a + b + c =
a2  b2  c2  ab  bc  ca
7 1 3 (xvii) (a  b) (b  c) (c  a)  ab(a  b)  bc(c  b) 
=   of 
2 3 2 ca (c  a)  2abc
7 1 3 (xviii) (a  b) (b  c) (c  a)   [a2(b  c)  b2(c  a) 
=   
2 3 2 c2(a  b)]
(xix) (a  b  c) (ab  bc  ca)  a2(b  c)  b2 (c  a) 
7 1 6
=   c2 (a  b)  3abc
2 2 2
=3 (xx) (x  a) ( x  b)  x2  x (a  b)  ab

6 . 21
Basic Numeracy and Data Interpretation

You can apply these formulae to solve the following: Percentage is also important in interpreting the
data given in graphs and tables. It is necessary to
● Find the value of 0.87 × 0.87 + 0.13 × 0.13 understand clearly how to work it out.
+ 2 × 0.87 × 0.13
The given expression is in the form a2 + b2 + Facts and Fast-Track Formulae to Remember
2ab, where a = 0.87 and b = 0.13
(i) To change a per cent to a decimal, remove the per
= (a + b)2
cent symbol (%) and divide by 100.
= (0.87 + 0.13)2
= (1.00)2 (ii) To change a decimal to a per cent, multiply by
= 1 100.

(1.3)3  (1.2)3 a
● ? (iii) To change a fraction to per cent, multiply it
b
(1.3)2  1.3  1.2  (1.2)2
with 100.
The given expression is in the form 3
For example, = ? per cent
3 3 12
a b , where a = 1.3 and b = 1.2 3 100 300
a  ab  b2
2
It is = = 25%
12 1 12
( a b) ( a 2 ab b 2 ) (iv) To convert a per cent into a fraction, put the given
( a 2 ab b 2 ) per cent over 100, removing the symbol %, and
= a + b simplify if required.
1.3 + 1.2 = 2.5 10 1
@UPSC_THOUGHTS

10% = 
100 10
● Find the value of
14 7
0.538  0.538  0.462  0.462 14% = 
100 50
1  0.924
(v) Most percentage problems involve three quantities:
The given expression is in the form
(a) the rate (R) which is followed by a % sign;
a2  b2 (b) the base number (N) which follows the word
, where a = 0.538 and b = 0.462
ab ‘of’;
( a b) ( a b) (c) the amount of percentage (P) which usually
( a b) follows the word ‘is’.
= a + b P P
P=R×N ; N= ; R=
R N
0.538 + 0.462 = 1.00
(vi) Per cent change, per cent increase, or per cent
decrease are special types of percentage problems
PERCENTAGE in which the difficulty is in making use of the
right numbers to calculate the percentage. The
Basics full formula is:
The term per cent means ‘for every hundred’ or ‘out of Per cent change =
hundred’. A fraction whose denominator is 100 is
called a percentage, and the numerator of the fraction (New Amount) (Original Amount)
100
is called the rate per cent. The term per cent is denoted Original Amount
by the symbol ‘%’.
Where a new amount is less than the original
24 amount, the number on top will be a negative
24 per cent, written as 24% =
100 number and the result will be a per cent decrease.
31 However, the negative sign is omitted and per
31 per cent, written as 31% = .
100 cent decrease is given as such. If the new amount

6 . 22
Basic Numeracy and Data Interpretation

(xii) If the present population of a town be P and the


Remember these fractions and their
population changes at r% per annum, then
equivalent per cents . . .
(a) Population in n years
1 1 n
1 = 100% = 50% = 33.33% Annual increase
2 3 = Original population 1
1 1 1 100
= 25% = 20% = 16.67% n
4 5 6  r 
= P 1  
1 1 1  100 
= 12.5% = 11.11% = 10%
8 9 10 Population in n years
1 1 1 n
= 9.09% = 8.33% = 6.25% Annual decrease
11 12 16 = Original population 1
1 1 1 100
= 5% = 4% = 2% n
20 25 50  r 
2 2 3 = P 1  
= 66.67% = 40% = 60%  100 
3 5 5 (xiii) Population n years ago
is greater, the per cent change is positive and is
Original Population
called per cent increase. = n
Annual Increase
(vii) If two values are respectively x% and y% 1+
100
more than a third value, then the first is
 100  x  P
  =
 100  y  100 %
of the second n
 r 
  1  
 100 
and the second is
@UPSC_THOUGHTS

Population n years ago


 100  y 
  100 % of the first. Original Population
 100  x  = n
Annual Decrease
1
(viii) If two values are respectively x% and 100
y% less than a third value, then the first is
 100  x  P
  =
 100  y  100 % of the second  r 
n
  1  
 100 
and the second is
(xiv) If the population of a town is P which increases
 100  y 
  100 % of the first. by x% during the first year, by y% during the
 100  x  second year and again increases by z% during
(ix) If A is x% of C and B is y% of C, then A is
the third year then the population after 3 years
x 
  100 % of B. will be
y 
 
P  (100  x ) (100  y ) (100  z )
(x) x% of a quantity is taken by the first, y% of the 100  100  100
remaining is taken by the second and z% of the
(xv) If the population decreases by y% during the
remaining is taken by the third person. Now, if second year and increases in first and third year
A is left in the fund, then there was
by x% and z%, then the population after 3 years
A  100  100  100 will be
in the beginning
(100  x ) (100  y ) (100  z ) P (100  x ) (100  y ) (100  z )
(xi) x% of a quantity is added by the first, y% of the 100  100  100
increased quantity is added again, z% of the (xvi) If the price of a commodity increases by r%, then
again increased quantity is added. Now, it the reduction in consumption so as not to increase
becomes A, then the initial amount is given by the expenditure is
A  100  100  100  r 
(100  x ) (100  y ) (100  z )   100 %
 100  r 

6 . 23
Basic Numeracy and Data Interpretation

(xvii) If the price of a commodity decreases by r%, then (xxvi) If the sides of a triangle, rectangle, square, circle,
increase in consumption, so as not to decrease rhombus or any other two dimensional figure, are
expenditure on this item, is increased by x%, its area is increased by
 r   x  200   x 2 
  100 % x  % or  2x  %
  100   100 
 100 r  

(xviii) If A is x% more than B, then B is less than A by (xxvii)Error Percentage: For any given quantity, if a
particular value is given false by chance then
 x 
  100 % Error percentage
 100  x 
(False value – Actual value)
= × 100
(xix) If A is x% less than B then B is more than A by Actual value
 x  (xxviii) % Excess or % Shortfall: If a number p exceeds
  100 %
 100  x  another number q by a particular per cent, then

(xx) If the value of a number is first increased by x%, % Excess of p


% Shortfall of q = 100 + % Excess of p × 100
and later decreased by x%, the net change is
always a decrease which is equal to x% of x or
If p is shorter than q by a certain %, then
x2
. % Shortfall of p
100
% Excess of q = 100 – % Shortfall of p × 100
(xxi) If the value is first increased by x% and then
@UPSC_THOUGHTS

 xy 
decreased by y% then there is  x  y  % Worked Examples
 100 
increase or decrease, according to the positive ● Convert 9 per cent and 351 per cent into decimal
and negative sign respectively. fractions.

(xxii) If the value is increased successively by x% Solution Divide by 100. However, what you have
and y% then the final increase is given by to do is to place a decimal point after two places
(for the two zeros in 100) from the extreme right of
 xy 
x  y  % the integer. If the percentage is a single digit, add
 100  one zero to the left of it so that it becomes a 2-digit
(xxiii) If the pass marks in an examination is x% and number and then put the decimal point.
if a candidate who scores y marks fails by 9 per cent = 0.09
z marks, then the maximum marks, 351 per cent = 3.51
100 ( y  z ) ● What percentage is 14 of 24?
M 
x
Solution Base number = 24
(xxiv) A candidate scoring x% in an examination fails
Percentage = 14
by a marks while another candidate who scores
y% marks gets b marks more than the minimum Percentage
Rate = Base Number
required pass marks. Then the maximum marks
for that examination are 1
= 14 ÷ 24 = 58 %
100 (a b) 3
M
y x ● A piece of wood weighing 10 kg is found to weigh
(xxv) In an examination, x% failed in one subject and 8 kg after drying. The moisture content was what
y% failed in other. If z% students failed in both per cent?
the subjects, the percentage of students who Solution: Moisture content in wood
passed in both the subjects is 2
= 10 – 8 = 2 kg or × 100 = 20%
100 – (x + y – z) 10

6 . 24
Basic Numeracy and Data Interpretation

● A man divided his 35-hour work schedule as ● The population of a town increases 10% annually.
1 1 If its present population is 1986000, what will it
follows: of the time—sorting mail; of the
5 2 be in 2 years’ time?
1
time—filing letters; of the time—answering Solution: Here, P = 1986000, r = 10 and n = 2
7
enquiries. The rest of the time he spent in running n
r
errands. What is the percentage of time spent on ∴ Population after 2 years = P 1
100
running errands?
2
10
Solution: The total time spent on sorting mail, filing = 1986000 1
100
1 1 1 1
letters, and answering enquiries is 29
110 110
5 2 7 2 = 1986000
hours. The time left for running errands is 100 100
1 1 = 2403060
35 29 5 hours
2 2 ● The population of a town increases at the rate of
1
5 5% annually. If the present population is 8820,
Per cent of time spent on running errands = 2 what was it 2 years ago?
35
11 1 11 5 Solution: Here, P = 8820, r = 5 and n = 2
= 15 % or app. 16%
2 35 70 7
P
● Two numbers are 30% and 50% more than a third. ∴ Population 2 years ago = n
What percentage is the first of the second. r
1

100  x  100
 100  30 
Solution:  100  y  100 % =   100 % 8820
@UPSC_THOUGHTS

   100  50  = 2
5
 130  1
=   100 % 100
 150 
= 86.66% 8820
=
105 105
= 87% (approx.)
100 100
● A number, instead of being multiplied by 5, is
8820
divided by 5. What % is the result of the required = 100 100
correct value? 105 105

Solution: Suppose the number is 1, then the correct = 8000


answer is 5. ● The population of a town is 100000. It increases by
1 2% during the first year, 5% during the second year
The incorrect answer is
5 and 5% during the third year. What is the
∴ Required % =  1  100 % = 4%
population after 3 years?
 5 5 
Solution: Here, P = 100000, x = 2, y = 5 and z = 5
● Two years ago, a man had Rs 5000 in his locker. Population after 3 years
In the first year, he deposited 10% of the amount
in his locker. In the second year, he deposited 30% P (100 x ) (100 y ) (100 z )
=
of the increased amount in his locker. Find the 100 100 100
amount at present in his locker.
100000 (100 2) (100 5) (100 5)
100  10 100  30 =
Solution: and 100 100 100
100 100
5000  110  130 100000 102 105 105
∴ Required amount = =
100  100 100 100 100
= Rs 7150 = 112455

6 . 25
Basic Numeracy and Data Interpretation

● The population of a town is 72000. It increases by 12


Thus, Dheeraj’s salary is × 100
5% during the first year, decreases by 10% during 112
= Rs 10.71
the second year and increases by 15% during the
less than that of Shyam’s.
third year. What is the population after 3 years?
● If Geeta’s salary is 40% less than that of Vini’s, then
Solution: Here, P = 72000, x = 5,
how much per cent is Vini’s salary more than that
y = –10 and z = 15
of Geeta’s?
∴ Population after 3 years Solution: Let Vini’s salary be Rs 100/month,
 x 
P (100  x ) (100  y ) (100  z ) then using the formula   100 %
=  100  x 
100  100  100
 40 
72000 (100  5) (100  10) (100  15) =   100 %
=  100  40 
100  100  100
72000  105  90  115 = 66.66% = 66.67%
=
100  100  100
● If the side of a square is increased by 30%, then
= 78246 find the percentage increase in area.
Solution: Area of square = side × side
● If the price of honey increases by 30%, find by how
much per cent its consumption should be reduced ∴ Net % change in area
so as not to increase the expenditure.
 xy 
= x  y  %
@UPSC_THOUGHTS

Solution: Here, r = 30  100 


Reduction in consumption
 30  30 
 r  =  30  30  %
=   100 %  100 
 100  r 
= 69%
 30 
=   100 % ● Tax on a commodity is increased by 15% and its
 100  30 
consumption decreases by 10%. Find the effect on
= 23.07% revenue.
Solution: Since, Tax × Consumption = Revenue
● If the price of almonds decreases by 15%, find by
how much per cent its consumption can be increased ∴ Net % change in revenue
so as not to decrease the expenditure.  xy 
= x  y  %
 100 
Solution: Here, r = 15
Increase in consumption  15  10 
=  15  10  %
 100 
 r 
=   100 % = 3.5%
 100  r 
∴ The revenue increases by 3.5%
 15 
=   100 % ● In an examination, the pass marks is 50%. If a
 100  15  student scores 250 marks, fails by 150 marks, then
find the maximum marks.
= 17.64%
Solution: Here, x = 50, y = 250 and z = 150
● Shyam’s salary is 12% more than Dheeraj’s. How 100 ( y  z )
∴ M =
much per cent of Dheeraj’s salary is less than that x
of Shyam’s? 100 (250  150)
=
Solution: Suppose Dheeraj’s salary is Rs 100/ 50
100 400
month then Shyam’s salary becomes Rs 112/month. =
50
Dheeraj’s salary is 12% less than Shyam’s salary. = 800

6 . 26
Basic Numeracy and Data Interpretation

● In an examination, a candidate scores 60% and weighted (i) multiply the value of each quantity by its
fails by 30 marks, while another candidate who respective weight (ii) add the products (iii) add up the
scores 80% marks gets 25 marks more than the weights (iv) divide the sum of products by the sum of
minimum required pass marks. Find the maximum the weights.
pass marks.
Fast-Track Formulae to Remember
Solution: Here, x = 60, y = 80, a = 30 and b = 25
(i) If every observation is increased/decreased by a
100 (30  25) constant, the mean of the observations so obtained
∴ M =
80  60 also increases/decreases by the same constant.
100 55 New mean = Previous mean + Number
=
20 added to each term
= 275 New mean = Previous mean – Number
● In an examination, 30% students failed in Sanskrit subtracted from each term
and 45% failed in Mathematics. If 20% failed in (ii) If each observation is multiplied/divided by a
both the subjects, find the percentage of those who constant, the mean of the observations can be
passed in both the subjects. obtained by multiplying/dividing the mean by
Solution: Here, let x = 30, y = 45 and z = 20 the same constant.
New mean = Previous mean × Constant
∴ Percentage of students who passed in
multiplied to each term.
both the subjects is 100 – (x + y – z)
New mean
= 100 – (30 + 45 – 20)
@UPSC_THOUGHTS

= 45% Previous mean


= Number by which each mean is divided

AVERAGE (iii) (a) If a person travels a distance at a speed of


x kmph and the same distance at a speed of
Basics y kmph, then the average speed during the
An average is the sum of ‘n’ different data divided by 2xy
whole journey is given by kmph.
n. For example, if three students score 98, 74 and 80 xy
or
marks in English respectively, then their average marks
(b) If half of the journey is travelled at a speed
in English is equal to:
of x kmph and the next half at a speed of
98 74 80 252 y kmph, then the average speed during the
= = 84 marks
3 3 2xy
whole journey is kmph.
In statistics, average is also known as arithmetic mean. xy
or
There are other ‘averages’, mainly used in statistics,
such as ‘median’ and ‘mode’, which we will discuss (c) If a man goes to a certain place at a speed
in a later unit. of x kmph and returns to the original place
Here, we may say at a speed of y kmph, then the average speed
2xy
Total of data during up and down journey is kmph.
Average = xy
Number of data
(iv) If a person travels three equal distances at a
Total of data = Average × Number of data speed of x kmph, y kmph and z kmph
respectively, then the average speed during the
Total of data
Number of data = 3xyz
Average whole journey is kmph.
xy  yz  xz
When some numbers among terms to be averaged
occur more than once, they must be given the appropriate (v) If a person covers A km at a speed of x kmph,
weight. To obtain the average of quantities that are B km at a speed of y kmph and C km at a speed

6 . 27
Basic Numeracy and Data Interpretation

of z kmph, the average speed during the entire Worked Examples


A BC ● A secretary wrote 73 letters on Monday, 85 on
journey is kmph. Tuesday, 54 on Wednesday, 92 on Thursday and
A B C
 
x y z 66 on Friday. What was the average number of
letters written per day?
(vi) If a person covers Ath part of the distance at x
kmph, Bth part of the distance at y kmph and Solution:
the remaining Cth part at z kmph, then the Average 73 85 54 92 66 370
74
average speed during the entire journey is 5 5
1 ● A census shows that in a certain block of flats the
kmph.
A B C
  number of children in each family is 3, 4, 4, 0, 1,
x y z 2, 0, 2 and 2 respectively.
(vii) The average of n quantities is equal to x. When
Find the average number of children.
a quantity is added or removed, the average
becomes y. The value of the new quantity is Solution:
n(y – x) + y or n(x – y) + y. 3 4 4 0 1 2 0 2 2 18
Average 2
(viii) (a) The average of first n natural numbers is 9 9
n 1 ● Mr. Sharma drove for 6 hours at an average rate
.
2 of 50 km per hour and for 2 hours at an average
rate of 60 km per hour. What was his average rate
(b) The average of even numbers from 1 to n is
for the entire trip?
last even number + 2
@UPSC_THOUGHTS

.
2 Solution: His average rate for the entire trip
(c) The average of odd numbers from 1 to n is 6(50) 2(60)
last odd number + 1 8
.
2 300 120 420 1
= 52
(d) The average of squares of natural numbers 8 8 2
(n  1)(2n  1) (Do not make the mistake of simply adding the two
till n is .
6 rates and dividing by 2).
(e) The average of cubes of natural numbers till ● What are the average marks of a student who
n(n  1)2 received 90 in English, 84 in Maths, 75 in Hindi,
n is .
4 and 76 in Economics, if the subjects have the
following weights: English 4, Maths 3, Hindi 3 and
(f) The average of first n consecutive even
Economics 1?
numbers is (n + 1).
Solution:
(g) The average of first n consecutive odd number
is n. English 90 × 4 = 360
Maths 84 × 3 = 252
(h) The average of squares of first n consecutive Hindi 75 × 3 = 225
2(n  1)(2n  1) Economics 76 × 1 = 76
even numbers is .
3 913
Weight = 4 + 3 + 3 + 1 = 11
(i) The average of squares of consecutive even
( n  1)(n  2) Average = 913 ÷ 11 = 83
numbers till n is .
3 ● The average of 5 numbers is 12. What will be the
(j) The average of squares of consecutive odd new average if 4 is added to each number?
n(n  2)
numbers till n is . Solution: New average = 12 + 4 = 16
3

6 . 28
Basic Numeracy and Data Interpretation

● The average of 5 numbers is 12. What will be the 1 th


● A train covers of the journey at 40 kmph, the
new average if each of the numbers is multiplied 4
3 th
by 4? next of the journey at 30 kmph and the
5
remaining distance at 20 kmph. Find the average
Solution: New average = 12 × 4 = 48
speed of the train during the entire journey.
● If half of the journey is travelled at a speed of 10 Solution: The average speed
kmph and the next half at a speed of 8 kmph, find 1
the average speed during the entire journey. =
A B C
 
2xy x y z
Solution: The average speed = 1
xy = 1 3 3
2  10  8 8 4  5  20
= = 8 kmph
10  8 9 40 30 20
● If a person travels a distance at a speed of
5 kmph and the same distance at a speed of 7 1
= 1 1 3
kmph, then find the average speed during the
whole journey. 160 50 400
1
2xy =
Solution: The average speed = 5  16  6
xy
800
2  5 7 800 17
= = = 29 kmph
57 27 27
@UPSC_THOUGHTS

5 ● The average age of 20 students in a class is 5 years.


= 5 kmph
6
If the teacher’s age is included, the average age
● If a person travels three equal distances at a speed becomes 8 years. Find the teacher’s age?
of 10 kmph, 12 kmph and 8 kmph respectively, then
find the average speed during his whole journey. Solution: The teacher’s age = n(y – x) + y
= 20(8 – 5)+8 = 68 years
3xyz
Solution: The average speed = Find the average of the first 50 natural numbers.
xy  yz  xz ●

3  10  12  8 n1
= Solution: Average =
10  12  12  8  10  8 2
2880 27 50  1
= = 9 kmph =
296 37 2
51
● A person covers 5 km at a speed of 2 kmph, 6 km = = 25.5
2
at a speed of 4 kmph and 7 km at a speed of 6
kmph, then find the average speed during the entire ● Find the average of even numbers from 1 to 50.
journey. last even number + 2
Solution: Average =
Solution: The average speed 2
50  2
=
A BC 567 2
= = 52
A B C 5 6 7 = = 26
    2
x y z 2 4 6
● Find the average of odd numbers from 1 to 50.
18 12
= = 18 
30  18  14 62 last odd number + 1
Solution: Average =
12 2
15 49+1 50
= 3 kmph = 25
31 2 2

6 . 29
Basic Numeracy and Data Interpretation

● Calculate the average of squares of the natural RATIO AND PROPORTION


numbers from 1 to 50.
Basics of Ratio
(n  1)(2n  1)
Solution: Average = A ratio expresses a comparison of two (or more)
6
(50  1)(2  50  1) quantities in terms of numbers. The mark used to
= indicate ratio is (:) and is read ‘is to’.
6
51  101 Any ratio of two terms may be written as a fraction
= = 858.5
6 and any fraction, as a ratio.
● Calculate the average of cubes of natural numbers The ratio of two numbers, say x and y, is written
from 1 to 50. x
as and is denoted by x : y. Here x is called antecedent
y
n(n 1)2
Solution: Average = and y is called consequent.
4
50(50 1)2 Ratio have the following properties:
=
4
(i) A ratio is not affected on each of its terms being
50 2601
= = 32512.5 multiplied or divided by the same non-zero
4
number. But the addition or subtraction of the
● Find the average of first 50 consecutive even
same number can change the ratio.
numbers.
a ma
Solution: Average = (n + 1) = such that a : b = ma : mb
b mb
= 50 + 1
a
@UPSC_THOUGHTS

= 51 a a b
= m such that a : b = :
● Find the average of first 50 consecutive odd numbers. b b m m
m
Solution: Average = n = 50
(ii) A ratio a : b is said to be in the simplest form
● Calculate the average of squares of first 50
if the HCF of a and b is 1 (i.e., a and b are prime
consecutive even numbers. to each other).
2 (n 1) (2n 1) If A is the given quantity to be divided in the
Solution: Average =
3 ratio a : b, then
2 (50 1) (2 50 1)
= a
3 First part = A
2 51 101 a b
=
3 b
= 3434 Second part = A
a b
If the ratio is given in the form of fraction, say
● Find the average of squares of consecutive even
1 1
numbers from 1 to 50. : , then the LCM of the denominators of
m n
(n 1) (n 2) the fractions has to be found and then the
Solution: Average =
3 fractions should be multiplied by the LCM so
(50 1) (50 2) that the ratio can be expressed in the form of
=
3 integers.
51 52
= = 884
3 Basics of Proportion
● Find the average of squares of consecutive odd Proportion is the equality of two ratios. If a, b, c, d are
numbers from 1 to 50. in proportion then we can write
n (n 2) a : b :: c : d
Solution: Average =
3 where a is the first term, b the second term, c the third
49 51 term and d the fourth term. The first and the fourth
= = 833
3 terms are called extremes (end terms), and the second

6 . 30
Basic Numeracy and Data Interpretation

and the third terms are called means (middle terms). Many problems in which three terms are given and
(The symbol : stands for ‘is to’ and the symbol : : stands one term is unknown can be solved by using
for ‘as’.) proportions. However, the proportion should be
Extremes formulated carefully according to the facts given—if
any term is misplaced, the solution will be incorrect.
a : b :: c : d
Fast-Track Methods and Formulae
means (i) To simplify a complicated ratio of two terms
There are different types of proportion: containing fractions, decimals or percentages,
Continued Proportion If a : b :: b : c, then a, b and divide the first term by the second, write as a
c are said to be in continued proportion. Here c is called fraction in lowest terms, and write the fraction as
the third proportional to a and b. 5 7
a ratio. Suppose you have to simplify : ,
Direct proportion Two quantities ‘a’ and ‘b’ are 6 8
said to be directly proportional to each other when one 5 7 5 8 40 20
= 20 : 21
increases or decreases, and the other also increases or 6 8 6 7 42 21
decreases accordingly to the same extent. It is written (ii) To solve problems in which the ratio is given, add
as a b. the terms in the ratio, divide the total to be put
a
i.e., a = k × b or = k, where k is a constant. into a ratio by this sum, and multiply each term
b
in the ratio by this quotient.
Inverse proportion Two quantities ‘a’ and ‘b’ are
said to be inversely proportional to each other when The sum of Rs 360 is to be divided among three
one increases and the other decreases and vice-versa people in the ratio 3:4:5. How much does each
@UPSC_THOUGHTS

1 one receive?
to the same extent. It is written as a .
b Solution: 3 + 4 + 5 = 12
1
i.e., a = k Rs 360 ÷ 12 = Rs 30
b
Rs 30 × 3 = Rs 90
a.b = k
Rs 30 × 4 = Rs 120
where k is a constant.
Rs 30 × 5 = Rs 150
The following points are to be kept in mind. The money is thus divided as Rs 90, Rs 120,
● If a : b : : c : d are in proportion then, Rs 150.
product of extremes = product of means
(iii) (a) A number which, when subtracted from the
ad = bc
terms of the ratio a : b, makes it equal to the
● If a, b and c are in continued proportion
(a : b :: b : c) then b 2 = ac ratio c : d is bc  ad .
c d
● Mean proportional between a and b is ab . (b) A number which, when added to the terms
● If a : b :: c : d then, of the ratio a : b, makes it equal to the ratio
a b c d ad  bc
(i) c : d is .
b d c d
a b c d
(ii) (iv) If the sum of two numbers is S and their difference
b d
a b c d
is d, then the ratio of the numbers is given by
(iii) S + d : S – d.
a b c d
a b (v) If two quantities A and B are in the ratio
(iv)
c d a : b, then
b d
(v) A+B : A–B = a+b : a–b
a c
(vi) If two numbers are in the ratio of a : b and the
a c
(vi) sum of these numbers is x, then these numbers
a b c d
a c ax bx
(vii) Each ratio = will be and respectively.
b d ab ab

6 . 31
Basic Numeracy and Data Interpretation

or (b) If a : b = n1 : d1, b : c = n2 : d2 and


If in a mixture of x litres, two liquids A and B c : d = n3 : d3, then
are in the ratio a : b, then the quantities of liquids a : b : c : d = (n1 × n2 × n3) : (d1 × n2 × n3) :
ax
A and B in the mixture will be litres and (d1 × d2 × n3) : (d1 × d2 × d3)
ab
bx
litres, respectively. (x) (a) In any two dimensional figures, if the
ab
or corresponding sides are in the ratio
a : b, then their areas would be in the ratio
If three numbers are in the ratio of a : b : c and
a2 : b2.
the sum of these numbers is x, then these numbers
ax bx cx (b) In any two 3-dimensional figures, if the
will be , and ,
abc abc abc corresponding sides or other measuring
respectively. lengths are in the ratio a : b, then their
volumes would be in the ratio a3 : b3.
(vii) If two numbers are in the ratio of a : b and
difference between these numbers is x, then these
Worked Examples
numbers will be
● There are 16 boys and 12 girls in a class. What is
ax bx the ratio of the number of girls to the number of
(a) and , where a > b
ab ab children in the class?
ax bx Solution Total number of children in the class = 16
(b) and , where a < b
ba ba + 12 = 28. The ratio of number of girls to the number
(viii) (a) The ratio between two numbers is of children is 12 : 28 which is 3 is to 7.
@UPSC_THOUGHTS

a : b. If x is added to each of these numbers,


● A certain pole casts a shadow 24 metres long. At
the ratio becomes c : d. The two numbers are
the same time another pole 3 metres high casts a
given as :
shadow 4 m long. How high is the first pole given
ax (c  d ) bx(c  d ) that the heights and shadows are in proportion?
and
ad  bc ad  bc Solution: If m is the height of the first pole, the
(b) The ratio between two numbers is proportion is
a : b. If x is subtracted from each of these m 3 24 × 3
= or m =
numbers, the ratio becomes c : d. The two 24 4 4
numbers are given as: = 18 metres
ax (d  c) bx(d  c )
and ● Aluminium Bronze consists of copper and
ad  bc ad  bc
aluminium usually in the ratio of 10 : 1 by weight.
or If an object made by this alloy weights 77 kg. how
The incomes of two persons are in the ratio many kg. of aluminium does it contain?
a : b and their expenditure are in the ratio Solution: Since only two parts of proportion are
c : d. If each of them saves Rs R, then their known (77 is total weight), the problem must be
incomes are given by solved by the ratio method. The ratio 10:1 means
10 parts of copper and 1 part of aluminium would
Ra(d  c) Rb(d  c) form the total of 10 + 1 = 11 of the alloy.
Rs and Rs
ad  bc ad  bc 77 ÷ 11 = 7 kg per part.
and their expenditures are given by The alloy has 1 part aluminium, so it comes to
7 × 1 or 7 kg of aluminium.
Rc(b  a) Rd(b  a)
Rs and Rs ● A, B and C invested Rs 9000, Rs 7000 and Rs 6000
ad  bc ad  bc
respectively. Their profits were to be divided
(ix) (a) If a : b = n1 : d1 and b : c = n2 : d2, then according to the ratio of their investments. If B uses
a : b : c = (n1 × n2) : (d1 × n2) : (d1 × d2) his share of the firm’s profit of Rs 825 to pay a

6 . 32
Basic Numeracy and Data Interpretation

personal debt of Rs 230, how much will he have ax bx


left? ∴ The required numbers are and
ab ab
Solution: The ratio of investment is 9 : 7 : 6 ax 9  28
9 + 7 + 6 = 22 ∴ =
ab 95
Rs 825 + 22 = Rs 37.50 each share of profit. 9  28
= = 18
7 × Rs 37.50 = Rs 262.50 which is B’s share of 14
profit. bx 5  28
and =
Rs 262.50 – Rs 230 = Rs 32.50 which is the amount ab 95
B has left. 5  28
= = 10
14
● Find the number that must be subtracted from the
terms 4 : 7 to make it equal to 1 : 3. ● Three numbers are in the ratio 1 : 2 : 3 and the sum
of these numbers is 18. Find the numbers.
Solution: We have a : b = 4 : 7 and c : d = 1 : 3
Solution: We have a = 1, b = 2, c = 3 and x = 18
bc  ad ax bx
∴ Required number = ∴ The required numbers are ,
c d abc abc
71  4 3 cx
= and
1 3 abc
7 12 5
= = ax 1  18 18
2 2 ∴ = = = 3
abc 12 3 6
● Find the number that must be added to the terms
2  18 2  18
@UPSC_THOUGHTS

1 : 5 to make it equal to 1 : 3. bx
= = = 6
abc 12 3 6
Solution: We have a : b = 1 : 5 and c : d = 1 : 3
ad  bc cx 3  18 3  18
∴ Required number = and = = = 9
c d abc 12 3 6

1 3  5  1 ● Two numbers are in the ratio of 2 : 5. If the


=
13 difference between these numbers is 9, then find the
35 numbers.
= = 1
2
Solution: We have a = 2, b = 5 and x = 9.
● If the sum of two numbers is 9 and their difference Here a < b
is 5, find the ratio of these numbers.
ax bx
∴ The required numbers are and
Solution: We have S = 9 and d = 5 ba ba
∴ Required ratio = S + d : S – d ax 2 9 18
∴ = = = 6
= 9+5 : 9–5 ba 5  2 3
= 14 : 4 = 7 : 2 bx 5 9 45
and = = = 15
ba 52 3
● If two quantities A and B are in the ratio 3 : 2, then
what is A + B : A – B is equal to? ● The ratio of two numbers is 1 : 3. If 7 is added to
each of them, their ratio is changed to 11 : 19. Find
Solution: We have a = 3 and b = 2
the numbers.
∴ A+B : A–B = a+b : a–b
= 3 + 2 : 3–2 Solution: We have a = 1, b = 3, c = 11, d = 19 and
= 5 : 1 x = 7.
● Two numbers are in the ratio 9 : 5 and the sum ax (c  d )
∴ The required numbers are and
of these numbers is 28. Find the two numbers. ad  bc
bx(c  d )
Solution: We have a = 9, b = 5 and x = 28. ad  bc

6 . 33
Basic Numeracy and Data Interpretation

ax (c  d ) 1 7(11 19) 7(8) Rc(b  a)


∴ = = = 4 ∴ Arav’s expenditure is Rs
ad  bc 1 19 3 11 19  33 ad  bc
5000  4(4  5)
bx(c  d ) 3  7(11  19) 21(8) =
and = = = 12 5  3  4 4
ad  bc 1  19  3  11 19  33
20000  (1)
=
● The ratio of two numbers is 1 : 3. If 2 is subtracted 15  16
from each number, their ratio is changed to 1 : 5.
Find the numbers. = Rs 20,000

Solution: We have a = 1, b = 3, c = 1, d = 5 and Rd(b  a)


and Shreya’s expenditure is Rs
x = 2. ad  bc
5000  3(4  5) 15000  (1)
ax (d  c) = =
∴ The required numbers are 5 3  4  4 15  16
ad  bc
bx(d  c ) = Rs 15,000
and
ad  bc ● If A : B = 4 : 5 and B : C = 6 : 7, find A : B : C.
ax (d  c) 1  2(5  1) 2 4
∴ = = = 4 Solution: Here n1 = 4, n2 = 6, d1 = 5 and d2 = 7
ad  bc 1 5  3  1 53
∴ A : B : C = (n1 × n2) : (d1 × n2) : (d1 × d2)
bx(d  c ) 3  2 (5  1) 6 4
and = = = 12 = (4 × 6) : (5 × 6) : (5 × 7)
ad  bc 1 5  3  1 53
= 24 : 30 : 35
● Annual income of Arav and Shreya is in the ratio
5 : 4 and their annual expenses bear a ratio of ● If A : B = 1 : 3 and B : C = 5 : 6, and
@UPSC_THOUGHTS

4 : 3. If each of them saves Rs 5000 at the end of C : D = 8 : 9. Find A : D.


the year, then find their annual income and annual Solution: Here n1 = 1, n2 = 5, n3 = 8, d1 = 3,
expenditures. d2 = 6 and d3 = 9
Solution: We have a = 5, b = 4, c = 4, d = 3 and ∴ A : B : C : D
R = Rs 5000
= (n1 × n2 × n3) : (d1 × n2 × n3)
Ra(d  c) Rb(d  c ) : (d1 × d2 × n3) : (d1 × d2 × d3)
∴ Annual income = Rs and Rs
ad  bc ad  bc
= (1 × 5 × 8) : (3 × 5 × 8) : (3 × 6 × 8) : (3×6×9)
Ra(d  c ) = 40 : 120 : 144 : 162
∴ Arav’s income is
ad  bc
= 20 : 60 : 72 : 81
5000  5(3  4) Thus, A : D = 20 : 81
=
5 3  4  4
25000  (1)
● In two squares, the corresponding sides are in the
= ratio 1 : 4. Find the ratio of the areas of these two
15  16
squares.
= Rs 25,000
Solution: We have a = 1 and b = 4
Rb(d  c)
and Shreya’s income is ∴ The ratio of their areas = a2 : b2
ad  bc
= 12 : 42
5000  4(3  4)
= = 1 : 16
5 3  4  4
20000  (1) ● In two cubes, the corresponding sides are in the
=
15  16 ratio 2 : 3. Find the ratio of their volumes.
= Rs 20,000
Solution: We have a = 2 and b = 3
Rc(b  a)
∴ Annual expenditure = Rs ∴ The ratio of their volumes = a3 : b3
ad  bc
= 23 : 33
Rd(b  a)
and Rs = 8 : 27
ad  bc

6 . 34
Basic Numeracy and Data Interpretation

UNITARY METHOD Money spent on purchasing is directly proportional


to the number of articles purchased.
Basics Inverse/Indirect proportion Two quantities are said
to be indirectly proportional if one quantity increases,
The method in which the value of a unit is first found
the other quantity decreases and vice versa, i.e.,
is called unitary method. Unitary method is applied for
1
solving work and time problems, time and distance C C D r , say, where r is a constant.
D
problems, percentages problems, ratio and proportion Speed of a vehicle is indirectly (inversely)
problems, profit and loss problems, simple interest and proportional to the time taken to cover a distance.
compound interest problems.
The faster the pace of work, lesser is the time
Unitary method is also called method of reduction
required to complete the work.
to the unit. This we can explain more easily with the
Most problems on time and work, time and distance,
help of an example.
and pipes and cisterns are based on these principles.
● If I earn Rs 480 in 2 weeks, how long will it
take to earn Rs 1440? Points to Remember
Solution: ● The fewer men working on a project, longer is the
I earn Rs 480 in 2 weeks time to complete it.
2 ● The more men working on a project, the shorter is
I earn Re 1 in weeks
480 the time to complete it.
2 ● The more efficient is the worker, the less time he
I earn Rs 1440 in  1440 weeks
480 takes to complete the work.
= 6 weeks ● The faster the speed of a car, the less time to cover
@UPSC_THOUGHTS

In this example, earning in 2 weeks is given and the given distance.


we have found earning of Re 1 by compound ● The greater the volume of a gas, the less is the
division. From this time period we have found pressure at the given temperature.
the days required to earn Rs 1440.
For Direct Proportion
Variation and Proportion ● Number of persons Amount of work done
There are two types of variations: ● Time (no. of days) Amount of work
a ● Efficiency of worker Amount of work
Direct Variation If = constant, then a is said to
b ● Ratio of work Amount of work
be in direct variation with b. ● Duration (no. of hours) Amount of work
Inverse Variation If a . b= constant, then a is said ● Man × Time × Efficiency × Work rate Amount
to be in inverse variation with b. In other words, if two of work
quantities are so related that an increase in one causes
corresponding decrease in the other (or vice versa), then For indirect Proportion
they are said to be in inverse variation. 1
● Number of men
Chain rule defines the repeated use of ‘rule of three’ No. of days
in which the fourth proportional is found out when the Worked Examples
other three are given. ● If 12 men do a piece of work in 45 days, in how
Direct Proportion Two quantities are said to be many days will 27 men do it.
directly proportional when one quantity increases or Solution: More men, less days (inverse)
decreases, the other quantity also increases or decreases Men Days
correspondingly, i.e., 12 45
A
A B k , say 27 x
B
or A = Bk where k is a constant. ∴ 12 : 27 : : x : 45
12  45
Distance covered is directly proportional to the time ∴ x = = 20 days
27
taken to cover the distance, i.e., the more distance to be
covered, more is the time required to cover the distance. It will take 20 days for 27 men to complete the work.

6 . 35
Basic Numeracy and Data Interpretation

● If 8 men or 12 women can do a piece of work in ● If 2 horses are worth 3 oxen, and 5 oxen are worth
25 days, in how many days can the work be done 12 sheep, and 2 sheep are worth Rs 500, find the
by 6 men and 11 women working together? value of one horse.
Solution: Given that, 8 men = 12 women Solution:
Þ 2 men = 3 women Given 2 horses = 3 oxen
or 6 men = 9 women 5 oxen = 12 sheep
∴ 6 men + 11 women = 9 women + 11 women 2 sheep = Rs 500
= 20 women Let the cost of one horse be Rs x, then
Now we can put the question in the form, that if x × 2 × 5 × 2 = 3 × 12 × 500
12 women can do a job in 25 days, then 20 women
3  12  500
can do the same job in how many days?’’ x =
2 5 2
Women Days
12 25 x = Rs 900
20 x
∴ 12 : 20 :: x : 25 The value of one horse is Rs 900.
● If Rs 160 maintains a family of 8 persons for 80
12  25
x = = 15 days days, for how long will Rs 210 maintain a family
20
20 women or 6 men and 11 women can do the work of 12 persons?
in 15 days. Solution:
Rupees Persons Days
● If 10 books cost Rs 35. Calculate the cost of 16
@UPSC_THOUGHTS

books. 160 8 80
Solution: 210 12 x
Books Rupees
10 35 Rupees 160 : 210 
∴  : : 80 : x
16 x Persons 12 : 8 
∴ 10 : 16 :: 35 : x 210  8  80
∴ x =
16  35 160  12
x = = Rs 56
10
x = 70 days
The cost of 16 books is Rs 56.
● Six men earn as much as 8 women, 2 women as Rs 210 will maintain 12 persons for 70 days.
much as 3 boys, and 4 boys as much as 5 girls.
● If the wages of 5 men for 12 days be Rs 60, what
If a girl earns 50 paise per day, what does a man
would be the wages of 6 men for 20 days?
earn a day?
Solution:Let earning of one man be Rs x. Solution:
Given, 6 men = 8 women Men Days Wage
2 women = 3 boys
4 boys = 5 girls 5 12 60
1 6 20 x
1 girl = rupee
2 Men 5 : 6 
1 ∴  : : 60 : x
∴ x×6×2×4×1 = 1×8×3×5× Days 12 : 20
2
6  20  60
1 8  3 5 1 ∴ x =
x =  5  12
6  2 4  1 2

x = Rs 1.25 x = Rs 120

A men earns Rs 1.25. The wages of 6 men will be Rs 120 for 20 day.

6 . 36
Basic Numeracy and Data Interpretation

TIME AND WORK, WORK AND WAGES (ii) If A can do a piece of work in x days and B can
do it in y days then A and B working together
xy
Basics will do the same work in days.
xy
Something that one does as part of one’s regular work
may consist of many small tasks or a large one. But the (iii) If A and B together can do a piece of work in x
work is always to be completed in a given period of days and A alone can do it in y days then B alone
xy
time. To complete the given task earlier one has to can do the work in days.
y x
increase the number of persons engaged in doing that
(iv) If A, B and C can do a work in x, y and z days
task or vice versa. While dealing with problems on
respectively then all of them working
wages, it may be kept in mind that the money obtained xyz
is always divided in the ratio of the work done by each together can finish the work in days.
xy  yz  xz
person. (v) If A number of persons doing a piece of work is
In work problems there are three items involved: the increased (or decreased) in a certain ratio, the time
number of people working, the time, and the amount needed to do the same work will be decreased (or
of work done. increased) in the same ratio.
● The number of people working is directly (vi) If p men and q women can do a piece of work
proportional to the amount of work done, i.e., in N days then x men and y women can do the
the more people on the job, the more the work same work in
that will be done, and vice versa.
1 pqN
● The number of people working is inversely days or days.
x y qx py
proportional to the time, i.e., the more people on p N q N
@UPSC_THOUGHTS

the job, the less time it will take to finish the


(vii) If A and B, working together can finish a piece
job, and vice versa.
of work in x days, B and C in y days, C and A
● The time spent on the work is directly
in z days, then
proportional to the amount of work done; i.e.,
(a) A, B, C working together can finish the work
more time spent, more work done, and vice 2xyz
versa. in days.
xy  yz  zx
2xyz
Points to Remember (b) A alone will finish the work in
xy  yz  zx
(i) If A can do a piece of work in n days, then A’s days.
1 2xyz
1 day’s work = . (c) B alone will finish the work in
n yz  zx  xy
1 days.
(ii) If A’s 1 day’s work = , then A can finish the work
n 2xyz
in n days. (d) C alone will finish the work in
zx  xy  yz
(iii) If A is twice as good as a workman as B, then the days.
ratio of work done by A and B is 2 : 1 and the ratio
of time taken by A and B to do the same piece of Worked Examples
work is 1 : 2. ● It takes 4 people working at equal rates to finish
a job in 48 days. How long will 3 workers take for
Fast-Track Methods the same job?
(i) (a) If N1 men can do W1 work in D1 days in T1
Solution: One worker can do the job in 48 × 4 or
time and N2 men can do W2 work in D2 days
192 days.
in T2 time, then the relationship between them
3 workers can do the job in 192 ÷ 3 = 64 days
is
N1W2D1T1 = N2W1D2T2 ● Three workers can do a job in 12 days. Two of them
(b) In the above problem, if time is not given, then work twice as fast as the third. How long would
the formula becomes it take for one of the faster workers to do the job
N1W2D1 = N2W1D2 himself?

6 . 37
Basic Numeracy and Data Interpretation

Solution: If there are 2 fast workers and one slow 26 days, how many workers will be needed to
worker, by the terms given, there are actually 5 perform the same job in 12 days?
slow workers working at equal rates.
Solution: The job would be performed by 1 worker
1 slow worker will take 12 × 5 or 60 days.
1 fast worker = 2 slow workers : therefore, he will in 18 × 26 = 468 days.
60 To perform the job in 12 days would require
take or 30 days to finish the job.
2
468 ÷ 12 = 39 workers
● If A takes 3 days to dig a ditch whereas B takes
● If 40 men working 8 hours a day, can write 1920
6 days and C, 12 days for the same work, how long
pages in 4 days then 60 men working 6 hours a
would it take for all three to do the job?
day, can write how many pages in 2 days?
Solution: A can do the work in 3 days or he can
Solution: We have N1 = 40, W1 = 1920, D1 = 4,
1
do of the work in 1 day. T1= 8, N2 = 60, W2 = x, D2 = 2 and T2 = 6
3
1 1 Using formula N1W2D1T1 = N2W1D2T2
Similarly, in one day B can do and C, of
6 12
the work. Together in one day A, B and C can do 40 × x × 4 × 8 = 60 × 1920 × 2 × 6
1 1 1 7 60  1920  2  6
of the work x =
3 6 12 12 40  4  8
12 5
Therefore, it will take them or 1 days to = 1080
7 7
1
complete the job working together. ● If 10 men can do rd of the work in 3 days, then
3
@UPSC_THOUGHTS

● Four men working together can dig a ditch in 42 1


how many men can do work in 5 days?
days. They begin but one man works only half- 2
days. How long will it take to complete the same 1
Solution: We have N1 = 10, W1 = , D1 = 3,
job? 3
1
N2 = x, W2 = , D2 = 5
Solution: It would take for 1 man 42 × 4 = 168 days 2
to complete the work alone. Using formula N1W2D1 = N2W1D2
As one man works half-days and 3 work full days,
1 1
1 1 10   3 = x  5
we may say 3 men are working; for 3 men the 2 3
2 2
1 1
job would take 168 ÷ 3 = 48 days. 10   3
2 x = 2
1
● A stenographer has been asked to fill 500 forms. 5
3
She fills 25 forms by the end of half-an-hour, when
10  3  3
she is joined by another steno who fills forms at =
2 5
the rate of 45 an hour. The entire assignment will = 9
be carried out in how many hours? ● If A can do a piece of work in 10 days and B can
do it in 15 days, then A and B working together
Solution: At the end of the first half hour there are
can do it in how many days?
500 – 25 = 475 forms remaining.
The first steno’s rate per hour is 25 × 2 = 50 forms. Solution: We have x = 10 and y = 15
xy
The combined rate is 50 + 45 = 95 forms per hour. Using formula
xy
475 A and B can together finish the work in
The remaining forms can be completed in =
95 10  15
5 hours. Adding the first half-hour the entire job days
10  15
1
requires 5 hours. 150
2 = = 6 days
● If a certain job can be performed by 18 workers in 25

6 . 38
Basic Numeracy and Data Interpretation

● If A and B together can do a piece of work in 12 1


days and A alone can do it in 24 days, then in how =
1 8
many days B alone can do this work? 
21 135
Solution: We have x = 12 and y = 24 1
=
xy 135  168
Using formula
y x 2835
B alone can finish the work in 2835
12  24 =
days 303
24  12
36
= 24 days = 9 days
101
● If A, B and C can do a work in 5, 8 and 7 days
respectively, then in how many days can all of ● If A and B, working together can finish a piece of
them working together finish the work? work in 5 days, B and C in 3 days, C and A in
6 days, how much time will they take to complete
Solution: We have x = 5, y = 8 and z = 7 the work if A, B and C work together? Also how
xyz long would each take separately to do the same
Using formula
xy  yz  xz work?
A, B and C can finish the work in Solution: We have x = 5, y = 3 and z = 6
5 87 ∴ A, B and C working together can finish
days
5 8  87  57 2xyz
the work in days.
18 xy  yz  zx
= 2 days.
@UPSC_THOUGHTS

131 2 5 3  6
● If 5 persons can complete a piece of work in 4 days, =
5 3  3 6  5 6
then 7 persons will be able to do the same piece 2  5 3  6 20
of work in how many days? = =
63 7
Solution: 6
= 2 days
Persons Days 7
5 4 2xyz
A alone can do the work in days
7 x xy yz zx
∴ 5 : 7 :: x : 4 180
=
5 4 6 15  18  30
x = = 2 days
7 7 180
= = 60 days
● If 7 men and 9 women can do a piece of work in 3
15 days then 5 men and 8 women can do the same 2xyz
B alone can do the work in days
work in how many days? yz zx xy
180
Solution: We have p = 7, q = 9, N = 15, =
18  30  15
x = 5 and y = 8
180 5
1 = = 5 days
Using formula 33 11
x y
2xyz
p N q N C alone can do the work in days
zx xy yz
5 men and 8 women can do the work in
180
1 =
30 15 18
5 8 days
 2
7  15 9  15 180
= = 6 days
1 27 3
=
5 8 ● A working alone takes 4 days more than A and B

105 135 working together, B working alone takes 9 days

6 . 39
Basic Numeracy and Data Interpretation

more than A and B together. How many days did (b) If a pipe fills a tank in x hours and another
A and B together require to finish the work? pipe fills the same tank in y hours, but a third
pipe empties the full tank in z hours, and all
Solution: We have a = 4 and b = 9
of them are opened together, the net part
Using formula ab 1 1 1
A and B together complete the work in filled in 1 hour is x y z.
4  9 days.
∴ Time taken to fill the tank is
= 6 days
xyz
hours.
yz xz xy
PIPES AND CISTERNS
(iv) A cistern has a leak from which the cistern could
get empty in x hours. A pipe which admits y litres
Basics
of water per hour into the cistern is turned on and
Tank Problems are similar to work problems. Completely now the cistern gets emptied in z hours. The
filling (or emptying) a tank may be thought of as
xyz
completing a job. capacity of the cistern is litres.
z x
If a pipe can fill a tank in × hours, then the part
1 Worked Examples
filled in 1 hour is
x
If a pipe can empty a tank in y hours, then the part ● Pipe A can fill a tank in 3 minutes and pipe B can
1 fill it in 4 minutes. How long would it take both
of the full tank emptied in 1 hour is y .
pipes, working together, to fill it?
@UPSC_THOUGHTS

1
Fast-Track Methods Solution: Pipe A can fill of the tank in 1 minute.
3
(i) If a pipe can fill a tank in x hours and another 1
pipe can fill the same tank in y hours, then the Pipe B can fill of the tank in 1 minute.
4
net part filled in 1 hour, when both the pipes are Pipe A and Pipe B can together fill in one minute
1 1
opened is  . 1 1 7
x y of the tank.
3 4 12 12 5
xy
∴ Time taken to fill the tank is . Therefore they can fill the tank in
7
or 1 and
7
xy minutes.
(ii) (a) If a pipe can fill a tank in x hours and
● A tank is three-fourths full. Pipe A can fill the tank
another pipe can empty the full tank in y
in 12 minutes. Pipe B can empty it in 8 minutes.
hours, then the net part filled in 1 hour, when
If both pipes are open, how long will it take to
1 1 empty the tank?
both the pipes are opened is  .
x y
1
xy Solution: Pipe A can fill of the tank in 1 minute
∴ Time taken to fill the tank is 12
y x
1
Pipe B can empty of the tank in 1 minute.
(b) A pipe can fill a tank in x hours. Due to a 8
leak in the bottom it is filled in y hours. If 1 1 1
the tank is full, the time taken to empty the In 1 minute or 24
of the tank is emptied.
8 12
xy
tank is hours. It would take 24 minutes to empty the whole tank.
y x
3
(iii) (a) Three pipes A, B and C, individually, can fill However, the tank is only full.
4
a cistern in x, y and z hours respectively. If 3
So it will take 24 = 18 minutes to empty the
all the three pipes are opened together, the 4
time taken to fill the cistern is given by tank.
xyz ● A certain tank can be filled by Pipe A in 12
. minutes. Pipe B can empty the tank in 18 minutes.
xy  yz  zx

6 . 40
Basic Numeracy and Data Interpretation

If both pipes are open, how long will it take to fill ● If pipe A fills a tank in 2 hours and pipe B fills
or empty the tank? the same tank in 4 hours, but pipe empties the full
1 tank in 6 hours. If all of them are opened together,
Solution: Pipe A fills of the tank in 1 minute.
12 how much time will it take to fill the tank?
1
Pipe B empties of the tank in 1 minute. Solution: We have x = 2, y = 4 and z = 6
18
1 1
Since
12
is greater than
18
, the tank will ultimately ∴ Time taken to fill the tank
be filled. 2 4  6 48
= =
In one minute
1 1
or
3 2 1
of the tank is 4  6  2  6  2 4 28
12 18 36 36 36
actually filled. 12 5
= = 1 hours
Therefore, the tank will be completely filled in 36 7 7
minutes. ● A leak in the bottom of a tank can empty the full
● If a pipe can fill a tank in 14 hours and another tank in 4 hours. An inlet pipe fills water at the rate
pipe fills the same tank in 12 hours, then how long of 2 litres per minute. When the tank is full, the
will it take if both the pipes are opened together? inlet is opened and due to the leak, the tank gets
Solution: We have x = 14 and y = 12 emptied in 6 hours. Find the capacity of the tank.
14  12 Solution: We have x = 4, y = 2×60 = 120
∴ Time taken to fill the tank =
14  12 and z = 6
84 6 4  120  6
= = 6 hours ∴ Capacity of the tank =
13 13 6 4
@UPSC_THOUGHTS

● If a pipe can fill a tank in 7 hours and another


= 1440 litres
pipe can empty the full tank in 13 hours, then how
long will it take to fill the tank?
Solution: We have x = 7 and y = 13 TIME AND DISTANCE
7  13
∴ Time taken to fill the tank =
13  7 Basics
91 1 In time and distance problems there are usually three
= = 15 hours
6 6 quantities involved: the distance, the time, and the rate
● A pipe can fill a tank in 12 hours. Due to a leak
or speed (distance per unit time).
in the bottom, it is filled in 16 hours. If the tank
is full, how long will it take to empty from the leak. Distance = Speed × Time
Solution: We have x = 12 and y = 16 Distance
Speed =
∴ Time taken by the leak to empty the full tank Time

12  16 Distance
= Time = Speed
16  12
= 48 hours
When two people or objects are travelling towards
● Three pipes A, B and C, individually can fill a
each other, the rate at which they approach each other
cistern in 5, 6 and 7 hours respectively. If all the
is the sum of their respective rates (or speeds).
three pipes are opened together, then how much
When two people or objects are travelling in directly
time is required to fill the cistern.
opposite directions, the rate at which they are separating
Solution: We have x = 5, y = 6 and z = 7
is the sum of their respective speeds.
∴ Time taken to fill the tank Speed of an object is measured in kilometres per
5 67 210 hour (kmph) or metres per second (m/s). To convert the
= =
5 6  6  7  5  7 107 unit, we use following formulae
103 1000 m 5
= 1 hours 1 kmph = = m/s
107 60 60 s 18

6 . 41
Basic Numeracy and Data Interpretation

5 ∴ Speed of moving body


∴ x kmph = x m/s
18
18 Length of moving body + Length of bridge
and x m/s = x  kmph =
5 Time taken
∴ It follows that 5 m/s = 18 kmph, (vi) (a) If a train of length ‘x’ km has a speed of ‘a’
10 m/s = 36 kmph, 15 m/s = 54 kmph kmph and another train of length ‘y’ km has
20 m/s = 72 kmph, 25 m/s = 90 kmph a speed of ‘b’ kmph and a > b, then the time
taken by the faster train to cross the slower
If the speed of a body is changed in the ratio train, while moving in the same direction is given
a : b, then the ratio of the time taken changes in the xy
by = hours
ratio b : a. ab
(b) If the above trains are moving in the opposite
Fast-Track Methods xy
direction then the time taken is given by =
(i) If a body covers distance D1 and D2 at a speed of ab
hours
S1 and S2, then the total time taken is
(vii) If a train of length x kms crosses another train of
D1 D2
T =  length y kms in ‘t1’ seconds if they are moving in
S1 S2
opposite direction and t2 seconds if they are moving
(ii) (a) If a distance D from A to B is covered at a kmph in the same direction, then
in time t and from B to A at b kmph, then the xy  1 1
2ab The speed of the faster train =   
average speed is given by kmph. 2  t1 t2 
ab
and,
@UPSC_THOUGHTS

(b) If the distance from A to B is covered at a kmph


xy 1 1
and from B to A is covered at b kmph and total The speed of the slower train =   
2  t1 t2 
T hours is taken in the whole journey, then the

ab 
distance between A and B is T   km. Worked Examples
ab
a
(iii) If the new speed is of the original speed, then ● A and B are walking towards each other over a
b
the change in time taken to cover the same distance road 120 km long, A at the rate of 6 kmph and B
b at the rate of 4 kmph. How soon will they meet?
is given by, change in time = 1 original time
a
Solution: The factor to be found is the time.
(iv) If A and B start at the same time from two points Distance = 120 km
P and Q towards each other and after crossing Rate = 6 + 4 = 10 kmph
each other they take T1 and T2 hours in reaching
Distance 120
Q and P respectively, then, Time = = 12 hours
T Rate 10
A's speed
 2 They will meet in 12 hours.
B 's speed T1
● Two cars are travelling along the same road. The
(v) (a) If a moving body such as a train, overtakes a
first one, which travels at the rate of 30 kmph,
man or a pole or a milestone, then the distance
starts 6 hours ahead of the second one, which
covered in overtaking = length of the moving
travels at the rate of 50 kmph. How long will it
body
take the second car to catch up with the first one?
∴ Speed of moving body
Solution: The first car starts out 6 hours ahead of
Length of moving body the second at the rate of 30 kmph.
=
Time taken So it has travelled 6 × 30 or 180 km by the time
(b) If a moving body overtakes a bridge or a tunnel the second one starts.
or a platform or another moving body, then The second car travels at the rate of 50 kmph.
Distance covered = Sum of two lengths. Therefore it gains 50 – 30 or 20 kmph.

6 . 42
Basic Numeracy and Data Interpretation

The second car has to cover 180 km.


30 4 4
It will take 180/20 or 9 hours to catch up with the  =
Gunjan's speed 7 25
first car. 2
9 9
● A boy walks from home to school at the speed of 2 6
= =
5 5
2 kmph and from school to home at the speed of
3 3
2 kmph. Find the average speed of the boy. 30  5
8 Gunjan’s speed = = 25 kmph
6
2ab ● A train 450 m long crosses a man in 5 seconds.
Solution : Average speed =
ab What is the speed of the train in kmph?
3
2 2  2
= 8 Solution: Speed of the train
3
22 Length of train
8 =
76 Time taken to cross the man
= = 2.17 kmph
35 450
= m/s = 90 m/s
● A boy walks from home to school at the speed of 5
3 kmph and from school to home at the speed of 18
1 = 90× kmph
3 kmph and total 4 hours is taken in the whole 5
4
journey. Calculate the distance between home and = 324 kmph
school.
● A train 300 m long passes a bridge in 15s moving
Distance = T ab at a speed of 100 kmph. Find the length of the
@UPSC_THOUGHTS

Solution:
a b bridge.
1 Solution: Speed of the train
3 3
= 4 4 = 6.24 km.
1 Length of train + Length of bridge
3 3 =
4
Time taken in crossing the bridge
3
● By walking at of his usual speed, Raman is 10
7 5 300 + x
minutes late to his office. Find his usual time to 100× =
18 15
cover the distance.
5
Change in time 300 + x = 100× 15
Solution: Original time = 18
b 
  1 300 + x = 416.67
a 
x = 116.67m
10 30
= =
 7  4 ● A train of length 600m is running at a speed of
  1
 3  50 kmph and a man walking in the same direction
1 of the train walks at a speed of 5 kmph. In what
= 7 hours
2 time will they pass each other.
● Aashish and Gunjan start their journey from Goa 600  0 600  18
Solution: Time taken = =
and Bombay. After crossing each other they finish 5 45  5
(50  5) 
7 18
their remaining journey in 2 hours and 4 hours,
9 = 48 seconds
respectively. Find Gunjan’s speed, if Aashish’s
speed is 30 kmph. ● A train 140m long is running at a speed of 70
kmph. In what time will it pass a train of length
Aashish's speed T2 120m running at a speed of 80 kmph in the
Solution: 
Gunjan's speed T1 opposite direction.

6 . 43
Basic Numeracy and Data Interpretation

140  120 ● A monkey climbing up a greased pole ascends 12m


Solution: Time taken =
5 and slips down 3 m in alternate minutes. If the pole
(70  80) 
18 is 63 m high, how long will it take him to reach
260  18
= the top?
150  5
Solution Climb in 1 minute = 12 metres (m)
156 6
= = 6 seconds. Fall in the next minute = 3m
25 25
● A train 37.5 km long crosses another train of length Effective rise in 2 minutes
40 km in 40s when they are moving in opposite = 12 – 3 = 9
directions and 52s when they are moving in the Time taken to cover 6 × 9 = 54
same direction. What is the speed of both the = 6 × 2 = 12 minute
trains? Remaining distance = 9 m
12 m in 1 minute
Solution: Speed of the faster train
5 5 9 3
37.5   40  9 m in = minutes
= 18 18  1  1  12 4
 
2  40 52  3
∴ Total time taken = 12 minutes
10.41 11.11 52 40 4
=
2 2080

10.76  92 BOATS AND STREAMS


= = 0.47 m/s
2080
Speed of the slower train Basics
@UPSC_THOUGHTS

5 5 The two terms that we frequently come across while


37.5   40 
= 18 18  1  1  solving questions on boats and stream are ‘upstream’
 
2  40 52  and ‘downstream’. If the boat or a swimmer moves
12 against the current of the stream then it is called
= 10.76  = 0.06 m/s
2080 upstream and if the boat or the swimmer moves with
● Two stations A and B are 110 km apart on a the current of the stream, it is called downstream.
straight road. A motorcyclist starts from A at 7 AM Upstream reduces the speed while downstream
and travels towards B at 20 kmph. Another enhances speed. For example, let the speed of the boat
motorcyclist starts from B at 8 AM and travels be x kmph and speed of stream be y kmph then,
towards A at 25 kmph. At what time will they
meet? Speed of the boat, upstream = x – y kmph
Solution: From 7 AM-8 AM, the distance covered Speed of the boat, downstream = x + y kmph
by the first man = 20 km.
Remaining distance = 110 – 20 = 90 km ● A man’s rate in still water is half the sum of his
∴ Required speed = 20 + 25 = 45 km/hr rates with and against the current.
90
Time taken = = 2 hr. ● The rate of current is half the difference between
45
∴ They will meet at 10 AM. the rates of the man with and against the current.

● To cover the same distance, the time taken by three


Fast-Track Methods
cars are in the ratio 2 : 3 : 5. Find the ratio of their
(i) A man can row x kmph in still water. If in a
speed.
stream which is flowing at y kmph, it takes him
Solution: Ratio of time taken = 2 : 3 : 5
z hours to row to a place and back then the
1 1 1
∴ Ratio of speed = : : z(x 2  y 2 )
2 3 5 distance between the two places is .
[ LCM of 2, 3, 5 = 30] 2x
1 1 1 (ii) A man rows a certain distance downstream in
= 30 : 30 : 30
2 3 5 x hours and returns the same distance in y hours.
= 15 : 10 : 6
If the stream flows at the rate of z kmph then the

6 . 44
Basic Numeracy and Data Interpretation

z( x  y )
speed of the man in still water is given by z( x 2  y 2 )
y x Solution: Distance travelled =
kmph. 2y

(iii) A man can row a boat in still water at x kmph. 8 (10)2  (2)2 
=
In a stream flowing at y kmph; if it takes z hours 2 2
more to go upstream than to go downstream for 8(100  4)
=
the same distance, then the distance is given by 4
z( x 2  y 2 )
km. = 192 km
2y
● A man can row 7 kmph in still water. If the stream
(iv) If a man capable of rowing at the speed of x kmph
is flowing at 3 kmph, find the average speed of the
in still water, rows the same distance upstream
total journey if he rows from a point A to B and
and downstream and if the stream flows at a rate
from B to A.
of y kmph, then man’s average speed throughout
(x  y )(x  y )
the journey is given by: (x  y )( x  y ) kmph. Solution: Average speed =
x
x
(v) If a man rows a boat downstream at x kmph and (7  3)(7  3)
=
upstream at y kmph, then the speed of the boat 7

1 4  10
in still water is given by( x  y ) kmph and speed =
2 7
1 = 5.71 kmph
of the stream is given by ( x  y ) kmph.
2
● A man rows a boat at the rate of 14 kmph
@UPSC_THOUGHTS

downstream and 12 kmph upstream. Find the


Worked Examples
speed of the stream and the speed of the boat in
● A man can row 14 kmph in still water. In the still water.
stream flowing with the speed of 10 kmph he takes
Solution: Speed of the boat in still water
4 hours to move with the stream and come back.
Find the distance he rowed the boat. 1
= (x y)
2
z( x  y )
2 2
Solution: Distance travelled = 1
2x = (14  12)
2
4 (14)2 (10)2 4 (196 100)
= = = 13 kmph
2 14 28
1
4 96 Speed of the stream = ( x  y )
= = 13.71 km 2
28
1
● A man rows downstream in 12 hours and returns = (14  12) =1 kmph
2
back in 16 hours. If the stream is flowing with the
speed of 10 kmph, then what is the speed of man
in still water? RACES AND GAMES
z( x  y )
Solution: Speed of man =
y x Basics
10(12  16) A race is any contest of speed in running, driving,
=
16  12 riding or rowing.
= 70 kmph The starting point is the point from where the race
begins.
● A man rows a boat at 10 kmph in still water. In The finishing point (winning post) is the point
a stream flowing at 2 kmph, he takes 8 hours more where the race finishes.
to get upstream than to go downstream for the The winner is the contestant who first reaches the
same distance. Find the distance travelled by him? finishing point (destination point).

6 . 45
Basic Numeracy and Data Interpretation

If all the contestants reach the finishing point Solution: Let the time taken by A be ‘x’ seconds
exactly at the same time, then the race is said to end
in a deadlock or that the race is a dead heat race. The path The time taken by B = (x + 30) seconds.
or the ground where the race is conducted is called the Total distance = winner’s (A’s) distance = 1 km
race course or the track.
Initial start (x) = 100m, beat distance = 25m
If A and B are two contestants, and A is the winner,
and B the loser, A could have beaten B either by d length
Winner’s time Loser’s time
units or t seconds (time). In other words, when A ∴
Loser’s distance Winner’s distance
reaches the finishing point, B was d metres behind A.
We can also say that A reached the destination ‘t’ x x  30
i.e., =
seconds ahead of B. 1000 (100 25) 1000

Points to remember x x  30
=
875 1000
● A beats B by x seconds means, A finishes the race
x seconds before B finishes. 1000x = 875x + 875 × 30
● A gives B a start of x seconds means, A starts x ∴ Time taken by A = 3 min 30 seconds
seconds after B starts from the same point. and time taken by B = x + 30 = 4 minutes
● If in a game of 100, ‘A can give B 20 points’, it
● Usha and Shiny ran 220 metres in 10 seconds and
implies that while A scores 100 points, B scores 80
11 seconds respectively. If they started together,
points (100 – 20).
find how far was Shiny from the finishing line
● Game of 100 means, a game in which the participant
when Usha completed her 220 metres?
@UPSC_THOUGHTS

scoring 100 points first wins the game.


● A gives B a start of x metres means, if A starts at Solution: Winner’s distance (K) = 220 m
the starting point, B starts x metres ahead from the Winner’s time Loser’s time
starting point at the same time. =
Loser’s distance Winner’s distance

Fast-Track Formulae 10 11
i.e., =
Loser’s distance 220
Winner’s time Loser’s time
(i) 
Loser’s distance Winner’s distance 220 10
Loser’s distance= = 200 m
Start time Beat time 11
Start distance Beat distance Shiny was 20 metres away from the finishing
line when Usha completed 220 metres.
(ii) If A is x times as fast as B and A gives B a start
of a m, then the length of the race course, so that 1
● Ankit is 2 times as fast as Anuj. If Ankit gives
3
both A and B reach the winning post at the same Anuj a start of 40m, how long is the race course
x when both of them reach at the same time?
time, is given by a m.
x 1
1
(iii) If A can run a m race in x seconds and B in y Solution: We have a = 40 and x = 2
3
seconds, where x < y, then A beats B by a distance
a  x 
× (y – x) m. ∴ Length of race course = a  
y  x 1
 7  7
   
Worked Examples = 40  3  = 40  3 
 7  1 4
● A can run a km in half minute less time than B.    
3  3
In a km race, B gets a start of 100 metres and loses
by 25 metres. Find the time A and B take to run 7
= 40  = 70 m
a km. 4

6 . 46
Basic Numeracy and Data Interpretation

● Sachin can run 200 m in 40 sec and Kush in 54


PROFIT AND LOSS
seconds. By what distance Sachin beats Kush?

Solution: We have a = 200, x = 40 and y = 54 Basics


Distance between Sachin and Kush Profit is an amount of money that is gained in business
200 or trade. When the cost of making something is less
= (54  40)
54 than the amount it is sold for, it is considered a profit.
And, if a business makes a loss, it earns less money
200
=  14 than it spends and is therefore in debt. In other words,
54
when a business or shop sells something at a loss, it
= 51.85 m
sells it for a price which is less than what it cost to
● In a game of billiards, A can give B 10 points in produce or less than what it cost to buy it.
60 and A gives C 15 in 60. How many can B give The terms profit and loss are largely used in trade
C in a game of 90? problems. Some other terms used in the context are cost
price, selling price, marked price, and discount which are
Solution: A scores 60, B scores 50 and C scores 45
If B scores 90, then C’s score normally used in problems relating to profit and loss.
Cost price (C.P.) is the price at which an article is
90
=  45 = 81 points purchased. Selling price (S.P.) is the price at which an
50
article is sold. When selling price is greater than cost
B gives C 9 points in a game of 90. price there is always some profit (S.P. > C.P. = Profit).
When the selling price is less than the cost price, there
● A can run 440 metres in 51 seconds and B in 55
@UPSC_THOUGHTS

is always a loss (S.P. < C.P. = Loss). Marked price is


seconds. By how many seconds will B win if he
the list price of the goods. Discount is a certain amount
has 40 metres start?
of rebate given to attract customers.
Solution: Time taken by A to cover 440 m = 51 sec ● Profit or loss is always calculated as a percentage

Time taken by B to cover 440 m = 55 sec of cost price.


Exception : In some cases it may be specifically
B gets a start of 40 m
mentioned that profit or loss has to be calculated
Distance to cover = 440 – 40 = 400 on the selling price. In such cases alone, S.P. will
Time taken by B to cover 400 m be taken into consideration.
● If the article is sold at its cost price itself, then
400
=  55 = 50 sec. there is neither profit nor loss. This is because
440
in such cases S.P. becomes C.P. and thus there
i.e., B will win by 51 – 50 = 1 sec. is no loss or no profit.
● Profit or loss are expressed in terms of rupees
● In a 500 metres race, the ratio of speeds of two
contestants A and B is 3 : 4. If A has a start of 140 or in terms of per cent.
metres, then by how much distance will A win? ● There are certain expenses like transportation
cost, rent, salaries, packing costs, etc. Such costs
Solution: A gets start of 140 m are added to the cost price before ascertaining
profit/loss.
 distance covered by
Thus actual cost price = cost price + other costs.
A = 500 –140 = 360 m
The basic formulae to remember are:
∴ We get the relation, 3 : 4 :: 360 : x
● Profit = S.P. – C.P.
360  4 ● Loss = C.P. – S. P.
Distance covered by B = = 480 m
3
● Percentage of profit or loss
 20 metres behind the final point
Actual profit or loss
 A wins by 20 metres. = 100
Cost Price

6 . 47
Basic Numeracy and Data Interpretation

● When an article is sold at a profit, xy


% gain or loss =  100
100 Profit percentage y
S.P. = C.P. × [gain in case of x > y and loss in case of x < y]
100
(ii) If cost price of two items is Rs x, and if one is
100
C.P. = S.P. × 100 Profit percentage sold at a loss of a% and the other at a gain of
b% then,
x  (100  b)
● When an article is sold at a loss, C.P. of the item sold at loss =
(100  a)  (100  b)
100 Loss percentage x  (100  a)
S.P. = C.P. × C.P. of the item sold at gain =
100 (100  a)  (100  b)
100
C.P. = S.P. × 100 Loss percentage (iii) If two articles have same S.P. and loss % on
one is equal to the gain % on the other then, on
● If an article is sold at a gain of 10%, then S.P. the whole there is always loss and loss % is
= 110% of C.P. 2
Gain % or Loss %
● If an article is sold at a loss of 10%, then S.P. given by %.
10
= 90% of C.P.
(iv) A man purchases a certain number of articles
Error
● Gain % =  100 at Rs x and the same number at Rs y. He mixes
True Value – Error
them together and sells them at Rs z. Then his
True weight – False Weight  2xy 
 100  1  100 according as
@UPSC_THOUGHTS

Gain % = gain or loss % = 


False Weight  z( x  y ) 
the sign is ‘+ve’ or ‘–ve’.
Discount: The term ‘discount’ is common in
(v) When a shopkeeper uses false scale or false
business transactions, essentially meaning the selling
weights and announces that he sells his goods
of a product below its marked price.
at loss/gain of x%, then his overall gain % (g)
● If x % discount is allowed on the marked price is given as
of an item, the selling price of the item is 100  g True weight
(100 – x) % of the marked price. =
100  x False weight
● Amount payable after a series of two discounts
True weight
100 + g =  (100  x )
100 First discount False weight
= Marked price ×
100
If the shopkeeper sells his goods at cost price,
100 Second discount then x = 0
×
100
True weight
This rule can be used for more discount series. 100 + g =  100
False weight
● Single discount rate equivalent to series of two
(vi) If ‘A’ sells an article to ‘B’ at a gain/loss of x%
Discounts = First discount + Second discount
and ‘B’ sells it to ‘C’ at a gain/loss of y%, then
– First discount × Second discount if x% and y% are profits
100  xy 
Resultant profit % =  x  y  %
● Two successive discounts of a% and b% on the  100 
listed price do not equal (m + n)% of listed price. If x% is profit and y% loss then Resultant loss
The discount should be calculated step by step.
 xy 
% = x  y   % according to the –ve and +ve
Fast-Track Methods  100
(i) If the cost price of x articles is equal to the selling signs respectively.
price of y articles, then (vii) If a man buys ‘x’ items for Rs ‘p’ and sells ‘y’

6 . 48
Basic Numeracy and Data Interpretation

items for Rs ‘q’ then the gain or loss per cent 10%. After three months it was sold after being
x q discounted 15%. How much was the bicycle sold
made by him is equal to 1 × 100%
y p for?
(In case of loss per cent the result obtained bears Solution: After 10% discount the selling price of the
–ve sign) bicycle was Rs 90. A further discount of 15% is
made. On Rs 90 the discount will be Rs
(viii) If ‘A’ sells an article to ‘B’ at a gain/loss of x%
and ‘B’ sells to ‘C’ at a gain/loss of y%. If ‘C’ 15
90 = Rs 13.50
100
pays Rs ‘p’ for it to ‘B’ then the cost price for
‘A’ is equal to: Selling price will be Rs 90 – 13.50 = Rs 76.50
1002 p ● A machine cost a dealer Rs 516. He wishes to mark
(100 x ) (100 y ) it so that he may deduct 20% from the marked
(–ve value of x or y indicates loss) price, and still make a profit of 25% of the cost.
What is the list price?
(ix) If two different articles are sold at the same
selling price, getting gain or loss of x% on the Solution: Cost of machine = Rs 516
first and gain or loss of y% on the second, then 25
100% – 20% = 516 + (516 × )
100
the overall % gain or % loss in the transaction
or 80% = Rs 516 + Rs 129 = Rs 645
is equal to:
Rs 645 = 80% of list price
100 ( x y ) 2xy List price = Rs 645 ÷ 80%
%
(100 x ) (100 y ) 100
= 645 × = Rs 806.25
@UPSC_THOUGHTS

80
(–ve sign would indicate overall loss %)
● A microwave oven priced at Rs 15,000 is available
at two successive discounts of 10% and 20%. What
Worked Examples
is the single equivalent discount?
● A stationer buys diaries at Rs 75 per dozen and
sells them at Rs 25 per piece. What is the profit. Solution: Single equivalent discount rate
10 × 20
Solution: Each dozen diaries cost Rs 75 and are = 10 + 20 – = 30 – 2 = 28%
100
sold for 12 × 25 = Rs 300. [There is no need to use the marked price in thus
calculating the discount.]
The profit is Rs 300 – 75 = Rs 225.
● The marked price of a microwave oven is
The profit % based on cost = 225 × 100 = 300%. Rs 10,000. What would I need to pay if two
75 successive discounts of 20% and 20% are available
● An article was sold for Rs 2.50 which is 25% of on the oven?
the cost of the price. What is the cost price? 100 20 100 20
Solution: I would pay 10,000 ×
100 100
Solution: Since the selling price represents the
= 80 × 80 = Rs 6,400
whole cost price plus 25% of the cost price,
● What would be the equivalent discount of a series
Rs 2.50 = 125% of the cost price
of discounts of 10%, 20% and 25%?
2.50 = 1.25 of the cost price
C.P. = 2.50 ÷ 1.25 = Rs 2.00 Solution: Discount equivalent to 10% and 20%
● Find the selling price of an article listed at Rs 10.00 10 20
= 10 + 20 – = 10 + 20 – 2 = 28%
on which there are discounts of 20% and 10%. 100

Solution: Rs 10 × 20% = Rs 2.00 Discount equivalent to 28% and 25%


Rs 10 – Rs 2 = Rs 8 (Price after 1st discount) 25 28
= 25 + 28 – = 25 + 28 – 7 = 46%
Rs 8 × 10% = 80p 100
Rs 8 – 80p = Rs 7.20 So the equivalent discount of the series of three
● A bicycle originally cost Rs 100 and was discounted discounts is 46%.

6 . 49
Basic Numeracy and Data Interpretation

● Find the profit % if C.P. = Rs 240 and S.P. 782  115


= Rs 320. =
85  115
Profit 782  115
Solution: Profit %=  100 =
C.P. 200
320  240
=  100
240 = Rs. 449. 65
80 1 C.P. of the cycle at gain
=  100 = 33 %
240 3
x  (100  a)
= (100  a)  (100  b)
● Find the S.P. if loss = Rs 24.50 and loss % = 5%.
782  (100  15)
Loss % × C.P. = (100  15)  (100  15)
Solution: Loss =
100
782  85
Loss × 100 =
∴ C.P. = 85  115
Loss %
782  85
24.50  100 =
= 200
5
2450 = Rs 332.35
= = Rs 490
5
● If the S.P. of two tables is Rs 500 each and 5% loss
∴ S.P. = 490 – 24.50
on one table is equal to the 5% gain on the other.
= Rs 465.50
Then find the total loss %.
● If profit % = 12% and profit = Rs 96, then find
@UPSC_THOUGHTS

2
Loss % =  
the selling price. 5
Solution:
 10 
Profit % × C.P.
Solution: Profit= 25
100 = = 25%
100
Profit × 100 ● A shopkeeper buys two types of rice at Rs 1200
C.P. =
Profit % and Rs 700 respectively. He mixes them together
96×100 and sells them at Rs. 900. Find his gain % or
= = Rs 800
12 loss %.
∴ S.P. = 800 + 96 Solution : We have x = 1200, y = 700 and z = 900
= Rs 896  2xy 
● The S.P. of 24 articles is equal to the C.P. of 20 Gain % or loss % =   1  100
 z( x  y ) 
articles. Find the loss percentage.
 2  1200  700 
Solution: We have x = 20 and y = 24. =   1  100
 900(1200  700) 
xy
Loss % =  100
y
 1680000 
=   1  100
24  20 2  1710000 
=  100 = 16 %
24 3
3
● A man bought two cycles for Rs 782 each. On one =  100
171
he gains 15% and on the other he loses 15%. What
1
is the C.P. of both the cycles? =  100 = –1.75%
57
Solution: We have x = 782, a = 15 and b = 15 ∴ The shopkeeper’s loss is 1.75%.
x  (100  b) ● A dealer professing to sell at cost price, uses a 900
C.P. of the cycle at loss =
(100  a)  (100  b) gm weight for a kilogram. Find his gain (%).
782  (100  15) 1000
= Solution: 100 + g =  100
(100  15)  (100  15) 900

6 . 50
Basic Numeracy and Data Interpretation

1000 100 ( x y ) 2xy


g% =  100 = %
9 (100 x ) (100 y )
100 1
= = 11 % 100 (20 25) 2 20 ( 25)
9 9 = %
(100 20) (100 25)
● Dinesh sells a bed to Rohan at a profit of 25% and
Rohan sells it to Gagan at a loss of 5%. Find the 500 1000
= %
resultant profit or loss. 195
1500
Solution: We have x = 25 and y = 5 = = –7.69%
195
  25  5 Overall loss = 7.69%
Resultant profit % =  25  5  %
 100 
125 ALLIGATION AND MIXTURES
= 20 %
100 Basics
 1875  Alligation literally means ‘linking’.
=   % = 18.75%
 100  Also called the Rule of Mixture, the Alligation Rule
● A man bought 8 pens for Rs 5 and sold 4 of them is useful for finding (i) the combined average of a
at Rs 3. Find gain %. mixture of items which are divided into two groups,
each group with its own average; (ii) the ratio in which
xq
Solution: Gain % = 1 100% items from two or more groups should be taken to get
y p
a combined average for all the items.
8 3
@UPSC_THOUGHTS

= 1 100% First average (a) Second average ( b)


5 4

24
= 1 × 100% Combined average (m)
20

4
= 100 = 20% (b – m )
20 (m – a )
|(m – b)| |(m – a)|
● A sells an article to B at a gain of 10% and B again
The required ratio is (b–m) : (m–a)
sells it to C at a gain of 15%. If C pays Rs 253 to
[or |(m–b)| : |m–a| ]
B, when is the cost price of the article for A?
[|(m–b)| and |(m–a)| being absolute values, they
Solution: Here x = 10, y = 15, p = 253 do not carry a negative sign.]
Using the formula, If two ingredients are mixed, then
1002 p Quantity of cheaper C.P. of superior Mean price
C.P. =
(100 x ) (100 y ) Quantity of superior Mean price C.P. of cheaper

1002 253 Here, the cost price of a unit quantity of the mixture
= is called the mean price.
110 115
We can also write the above formula as
100 100 253
= C.P. of a unit of C.P. of a unit of
110 115 cheaper quantity (a) superior quantity (b)
= Rs 200
● A man sold two articles, each for Rs 1500. If he Mean price
makes 20% profit on the first and 25% loss on the (m)
second, what is his profit or loss per cent in the
transaction? (b – m) (m – a)

Solution: Here x = 20, and y = –25 Quantity of cheaper b m


∴ Quantity of superior m a
Over all gain/loss %

6 . 51
Basic Numeracy and Data Interpretation

Fast-Track Methods Solution:

(i) If a container contains x units of liquid from Boys Girls


which y units are taken out and replaced by 11.2 10.9
water, then after n operations, the quantity of
  y 
n 11
pure liquid would be  x  1    units.
x   
11 – 10.9 = 0.1 11.2 – 11 = 0.2
(ii) Suppose we have n number of vessels of equal Ratio of boys to girls = 0.1 : 0.2 or 1 : 2
size filled with mixtures of liquids A and B in If there are 50 boys in the bus,
the ratio a1 : b1, a2 : b2, .... an : bn respectively. If There are 50 × 2 = 100 girls in the bus
the contents of all the vessels are poured into a Total number of children is 150.
single large vessel, then
● In an examination, correct answer gets the candidate
a1 a2 an 3 marks, but there is negative marking of 1 for every
...
Quantity of liquid A a1 b1 a2 b2 an bn wrong answer. Marup gets 0 in the paper of 100
questions. How many answers did he get correct?
Quantity of liquid B b1 b2 bn
...
a1 b1 a2 b2 an bn Solution:

Correct Wrong
(iii) Suppose we have n number of vessels of size c1, 3 –1
c2,...,cn filled with mixtures of liquids A and B in
@UPSC_THOUGHTS

the ratio a1 : b1, a2 : b2,...,an : bn, respectively. If the 0


contents of all the vessels are poured into a single
large vessel, then 1 3

a1c1 a2c2 ancn The ratio of correct to wrong answers is 1 : 3


... 1
Quantity of liquid A a1 b1 a2 b2 an bn
∴ Correct answer = × 100 = 25
4
Quantity of liquid B b1c1 b2c2 bncn ● A mixture of chemical and water contains 10%
...
a1 b1 a2 b2 an bn water. How many litres of water must be added
to make it 37% of the new mixture?
Worked Examples Solution:
● Two mixtures of milk and water contain 10% and 10
10% of 70 l = 100 × 70 = 7 l of water
20% of water. In what ratio should the two mixtures
be mixed so that the new mixture has 16% water? So chemical is 70 – 7 = 63 l
In the new mixture, water is 37%, and chemical
Solution:
will be 100 – 37 = 63%
Mixture 1 Mixture 2 Now, if chemical is 63 l, water is 37 l.
10% 20% ∴ Water to be added is 37 – 7 = 30 l

16% ● How much water must be added to 100 cc of 80%


solution of boric acid to reduce it to a 50% solution?
|(16 – 20)| = 4 |(16 – 10)| = 6
Solution: In 100 cc solution, quantity of boric acid
The ratio required is 4 : 6 or 2 : 3 is 80 cc and quantity of water is 20 cc. Suppose
x cc of water is added to make the solution 50%.
● The average age of children going in a bus is 11
years. If the average age of boys is 11.2 years and ∴ 50% (100 + x) = 80 x = 60 cc
that of girls is 10.9 years, how many children are ● How many kg of apple costing Rs 8 per kg must
in the bus given that the boys total 50? be mixed with 20 kg of oranges costing

6 . 52
Basic Numeracy and Data Interpretation

Rs 4 per kg so that 20% gain may be obtained by n


y
selling the mixture at Rs 7.20 per kg? = x 1
x
Solution: S.P. of 1kg mixture =Rs 7.20 per kg,
3
gain 20% 50
 100  150 1
∴ C.P. of 1 kg mixture = Rs   7.20  = 150
 120 
= Rs 6 100
3

By alligation rule, we have = 150


150

C.P. of 1 kg apple C.P. of 1 kg of


100  100  100
800 paise orange 400 paise = 150 
150  150  150
Mean price 400 4
600 paise = = 44 litres
9 9
(d – m ) (m – c) ● We have four vessels of equal size filled with
= 200 = 200 mixtures of milk and water in the ratio 2 : 1, 3 :
Quantity of apple 200 1 2, 4 : 3 and 5 : 4 respectively. If all the contents
∴ Quantity of orange 200 1 are poured into a single large vessel, then what is
Let x kg of apple be mixed with 20 kg of orange the new proportion of milk and water in it?
then, 1 : 1 = x : 20 Solution: We have a1 = 2, a2 = 3, a3 = 4, a4 = 5
x = 20 kg and b1 = 1, b2 = 2, b3 = 3, b4 = 4.
@UPSC_THOUGHTS

● A person has a chemical of Rs 40 per litre. In what Quantity of milk


∴ Quantity of water
ratio should water be mixed in that chemical so
that after selling the mixture at Rs 30/litre he may
2 3 4 5
get the profit of 30%?   
2 1 32 4  3 5 4
= 1 2 3 4
Solution: S.P. of mixture = Rs 30 per litre,   
profit 30% 2 1 32 4  3 5 4
100
∴ Average price =  30 2 3 4 5
130   
3 5 7 9
= Rs 23.07 per litre = 1 2 3 4
  
3 5 7 9
Chemical Water
40 0
2  105  3  63  4  45  5  35
Mean price = 315
1  105  2  63  3  45  4  35
23.07
315
( d – m) (m – c )
23.07 16.93
210  189  180  175
=
∴ Chemical : Water = 23.07 : 16.93 105  126  135  140

● A container contains 150 litres of vinegar. 50 litres 754 377


= =
of vinegar was taken out of the vessel and replaced 506 253
by water. Then 50 litres of mixture was again
● We have three vessels of capacities 2 litres, 4 litres
withdrawn and replaced by water. The operation
and 6 litres filled with mixtures of liquids, milk
was again repeated for the third time. How much
and water in the ratio 1 : 2, 2 : 3 and 3 : 4,
vinegar is now left in the vessel?
respectively. If the contents of the vessels are
Solution: We have x = 150, y = 50 and n = 3 poured into a single large vessel, then find the ratio
∴ amount of vinegar left in the vessel of milk and water in the new mixture?

6 . 53
Basic Numeracy and Data Interpretation

Solution: We have a1 = 1, a2 = 2, a3 = 3 and Formulae for Simple Interest


b1 = 2, b2 = 3, b3 = 4 and c1 = 2, c2 = 4, c3 = 6. If Simple interest = S.I., Principal = P, Rate = R% per
1 2 2 4 3  6 annum, Amount = A and Time = T years
 

Quantity of milk
= 12 2 3 3 4 P  R T
Quantity of water 2 2 3 4 4  6 ● S.I. =
  100
12 2 3 3 4
100  S.I .
2 8 18 ● P =
  R T
= 3 5 7
 12 24 100  S.I .
  ● R =
3 5 7 P T
2  35  8  21  18  15 100  S.I .
● T =
105 PR
= 4  35  12  21  24  15  RT 
● A = P + S.I. = P  1  
105  100 
70  168  270
= Formulae for Compound Interest
140  252  360
If compound interest is C.I., Amount is A,
508 127 Principal is P, Rate is R% per annum, and Time is
= =
752 188 T years.
 T 
C.I. = A – P = P  1 
R 
●   1
INTEREST  100  
● When interest is compound annually :
@UPSC_THOUGHTS

Basics
T
Interest is the sum which is paid for the use of other’s  R 
A  P 1  
money. In other words, interest is the payment made  100 
for the money borrowed from banks or moneylenders. 1
A T
The money borrowed is called principal and the sum R 1 % per annum
P
of the principal and interest is called amount. The
interest is usually paid yearly, half-yearly or quarterly ● When interest is compounded half-yearly:
2T
as agreed upon. If it is payable yearly, it is called rate R
per cent per annum. A = P 1 2
There are two types of interest. 100
When the interest on a certain sum borrowed for
a certain period is reckoned uniformly, then it is called 1
A 2T
simple interest. Throughout the loan period, interest is R = 2 100 1 % per annum
P
charged on the original sum (principal) borrowed.
The interest charged every year on the amount of 2T
R
the previous year is called compound interest. Money C.I. = P 1 1
100 2
is said to be lent at compound interest when at the end
of a fixed period, the interest that has become due and ● When interest is compounded quarterly :
is not paid to the lender, but is added to the sum lent, 4T
R
and the amount thus obtained becomes the principal
A = P 1 4
for the next period. The process is repeated until the 100
amount for the last period has been found. The difference
1
between the original principal and the final amount is A 4T

called compound interest. R = 4 100


P
1 % per annum
When counting the number of days, the day of
4T
deposit is not counted, but the day of withdrawal is R
included. C.I. = P 1
100 4
1

6 . 54
Basic Numeracy and Data Interpretation

● When the interest is compounded n times a (vii) If P1 changes to P2 and R, T are fixed then
year, then RT
Change in S.I. = (P1 P2 )
n T 100
R
A = P 1 (viii) If P is fixed, but R1 changes to R2 and T1 changes
100 n
1 to T2, then
A nT

R = n 100 1 P
P Change in S.I. = (R1 T1 R2 T2 )
100
n T
R (ix) (a) If a sum of money P lent out at S.I. amounts
C.I. = P 1
100 n
1
to A1 in T1 years, and to A2 in T2 years,
A1 T2 A2 T1
Fast-Track Methods P =
T2 T1
For Simple Interest Problems A1 A2
(i) If a certain sum amounts to A in T years at R % and R = × 100%
A1 T2 A2 T1
per annum then the sum will be
(b) If P is lent out for a certain time T at S.I. and
100 A amounts to A1 at R1% per annum and to A2
P =
100 R T at R2% per annum,
(ii) The time T in which a sum of money becomes x
A2 R1 A1 R2
times at S.I. rate of R % is P =
R2 R1
100 (x 1)
T = years.
@UPSC_THOUGHTS

R A1 A2
and T = × 100 years
(iii) If a sum of money doubles itself in n years at A2 R1 A1 R2
simple interest, then the rate of interest is given
(x) If a debt of Rs M is paid in ‘n’ number of
by
instalments and if the value of each instalment is
100 (Multiple number of principal 1) Rs a, then the borrowed (debt) amount is given
R =
n by
(iv) The annual payment that will discharge a debt ra n (n  1)
M = na + 
of Rs A due in T years at the rate of interest R% 100  y 2
per annum is Where,
r = rate of interest per annum
100 A
= n = number of instalments per annum
RT (T 1)
100 T y = 1, when instalments are paid yearly
2
y = 2, when instalments are paid half-yearly
(v) Two equal amounts of money are deposited at
y = 4, when instalments are paid quarterly
R1% and R2% for T1 and T2 years respectively.
y = 12, when instalments are paid monthly
If the difference between their interest is Id
then sum (xi) (a) If a certain sum of money becomes n times
I d 100 itself at R% per annum simple interest in T
=
R1 T1 R2 T2 years, then
1  n1
(vi) Out of a certain sum P, if part is invested at T =   × 100 years
a  R 
1 1
R1%, part at R2% and remaining part at R3%, (b) If a certain sum of money becomes n times
b c itself in T years at a simple interest, then the
and the annual income from all these investments
in A, then the original sum is given by time T’ in which it will become m times itself
A 100 is given by
 m  1
P = R1 R2 R3 T’ =    T years
a b c  n1 

6 . 55
Basic Numeracy and Data Interpretation

For Compound Interest Case II : When T = 3 years and n = 1 per year


(i) When the rate of interests are different for different   R  
3 2
 R 
years, then C.I. – S.I. = P    3  
 100   100  
A = P  1  R1 
 1 
R2 
 1 
R3 
 .... 2
 100  100 100   S.I . R R
= 3
3 100 100
(ii) When the rate of interest is compounded annually
1
and the time is in fraction, say 4 years then,
2 Worked Examples
 1  2
R
R   2 
4
 ● Find the amount if P = Rs 2460, R = 6 % and
A = P 1   1   3
 100   100  1
  T = 3
2
years.
(iii) If a sum of money becomes x times in T years at 2 20
Solution: P = 2460, R = 6 % = %
compound interest, then the same amount of 3 3
money becomes xy times in Ty years. 7
T = years
2
(iv) If a certain sum becomes x times in t years, the 20 7
rate of compound interest is given by RT
A = P 1 = 2460 × 1 3 2
1
100 100
R = 100 (x )t 1
20 7
= 2460 1
(v) Present worth of Rs x due t years hence at R% 3 2 100
@UPSC_THOUGHTS

per annum compounded yearly is given by 7


A = 2460 1
30
t
R
1 37
100 = 2460 ×
(vi) When a certain sum of money at C.I. amounts to 30
Rs x in p years and Rs y in q years, then the rate = Rs 3,034
of interest per annum is ● In how many years will a sum of Rs 10,000 become
1 Rs 20,000 if the rate of interest is 14%?
y q p
R = 1 × 100%
x Solution: Amount = 20,000
Principal = 10,000
Difference between C.I. and S.I. S.I. = 10,000, R = 14%

PRT  R 
Tn
 PRT
S.I. = , C.I. = P  1  100  n   1 S.I. =
100
100  
100  S.I .
∴ T =
C.I. > S.I., n × T > 1 PR
100 10,000
=
R
Tn
RT 10,000 14
C.I. – S.I. = P 1 1
100 n 100 2 1
= 7  7 years
14 7

Case I : When T = 2 years and n = 1 per year 3


● At what rate will the interest on Rs 3800 be th
5
2 of itself in 8 years? How much will it become in
 R 
C.I. – S.I. = P   14 years?
 100 
Solution: P = 3800; T = 8 years;
R  S.I . 3
= S.I. =  3800 = 2280
2  100
5

6 . 56
Basic Numeracy and Data Interpretation

S.I .  100 Solution: We have P = 1000, change in S.I. = 150,


∴ R = R1 – R2 = 8
PT
2280 100 PT
= Change in S.I. = (R1 – R2)
3800  8 100
15 1000 T
= = 7.5% 150 =  (8)
2 100

RT 150 = 80T
In 14 years, A = P 1
100
150 7
 7.5  14  ∴ T = 1 years
= 3800  1   80 8
 100 
● If simple interest on Rs 2,000 be more than the
105
= 3800 1 interest on Rs 1,500 by Rs 100 in 5 years, find the
100
rate per cent per annum.
= 3800  205 Solution: We have change in S.I. = 100,
100
= Rs 7,790 P1–P2 = 500, T = 5
RT
● A sum of money doubles itself in 12 years at simple Change in S.I. = (P1  P2 )
100
interest. What is the rate of interest?
5R
Solution: We have multiple number of 100 =  500
100
principal = 2, n = 12
@UPSC_THOUGHTS

100 (Multiple number of principal 1) 100 = 25R


Rate =
n R = 4%
100 (2  1) ● If the simple interest on a certain sum at 2% per
=
12 annum for 5 years is Rs 50 more than the interest
= 8.33% on the same sum for 3 years at 3% per annum, find
the sum.
● What annual payment will discharge a debt of
Rs 848 in 4 years at 4% per annum? Solution: We have change in S.I. = 50, R1 = 2,
R2 = 3, T1 = 5, and T2 = 3
100 A
Solution: Annual payment = P
100T
RT (T 1) Change in S.I. = (R1 T1  R2 T2 )
2 100
84800
= P
4  100 
4(3) (4) 50 = (2  5  3  3)
2 100

= Rs 200 P
50 =
100
● The difference between the interest received from
P = Rs 5,000
two different banks on Rs 500 for 2 years is Rs 2.5.
● If a sum of money at simple interest amounts to
Find the difference between their rates.
Rs 6,000 in 2 years and to Rs 6,800 in 3 years, what
I d 100 is the sum and the rate of interest?
Solution: (R1 R2 )
sum t
Solution: We have A1 = 6000, A2 = 6800,
2.5 100 T1 = 2, T2 = 3
= 0.25%
500 2
A1T2 A2T1
Principal =
● Simple interest on Rs 1000 increases by Rs 150, T2 T1
when the rate % increases by 8% per annum, find 6000  3  6800  2
=
the time. 32

6 . 57
Basic Numeracy and Data Interpretation

9r
= 18000 – 13600 140 = 120 +
10
= Rs 4400 2
r = 22 %
( A2 A1 ) 100 9
Rate = T1 A2 T2 A1 ● In what time a sum of money will double itself at
a rate of simple interest of 9% per annum?
(6800 6000) 100 Solution: We have n = 2, R = 9
=
2 6800 3 6000
(n  1)  100
T = years
800 100 R
=
4400 (2  1)  100
= years
800 2 9
=  18 %
44 11 1
● A sum of money is invested for certain time. It = 11 years
9
amounts to Rs 700 at 4% per annum. But when
invested at 3% per annum, it amounts to Rs 610. ● A sum of money put out on simple interest triples
Find the sum and time. itself in 15 years. In how many years would it
become five times itself?
Solution: We have A1 = 700, A2 = 610, R1 = 4
R2 = 3 Solution: We have T = 15, n = 3, m = 5

A2R1  A1R2  m 1


Principal = T’ =    T years
R1  R2  n1 
@UPSC_THOUGHTS

610  4  700  3 (5  1)
=  15
43 = (3  1)
= 2440 – 2100
4
=  15 years
= Rs 340 2
 A1  A2 
Time =    100

= 30 years
 2R1  A1R2 
A
● What principal will amount to Rs 700 at 9% per
 700  610  annum in 2 years?
=    100
 610  4  700  3  Solution: We have, A = Rs 700, R = 9% per annum,
90
T = 2 years.
=  100
340 100 A 100 700
∴ P = 100 R T 100 9 2
8
= 26 years
17 100 700
=
● A sum of Rs 140 is lent to be paid back in 4 equal 118
half yearly instalments of Rs 30 each. Find the rate = Rs 600
per cent. 1 1
● Out of a certain sum, rd is invested at 6%, th
Solution: We have M = 100, a = 30, n = 4, y = 2 3 4
ra n (n  1) at 8% and the rest at 3%. If the annual income is
M = na +  Rs 250, then the original sum is:
100  y 2
r  30 4(4  1) 1 1 1 1 1 1 1 1
140 = 4  30   Solution: Here , , 1
100  2 2 a 3 b 4 c 3 4 3
3r
140 = 120 +  (2  3) R1 = 6%, R2 = 8%, R3 = 3%, A = Rs 250
20

6 . 58
Basic Numeracy and Data Interpretation

= 8000 × (1.16 – 1)
A 100
∴ The original sum = (correct to two decimal places)
R1 R2 R3
= 8000 × 0.16 = 1280
a b c
5
250 100 S.I. = 8000  3
100
=
6 8 3
= Rs 1200
3 4 3
∴ The difference = 1280 – 1200 = Rs 80
250 100 25000
= ● Compute the compound interest on Rs 12000 for
2 2 1 5 1
1 years at 10% p.a. and compounded half yearly.
= Rs 5,000 2
Tn
● Find the amount on Rs 2000 for 2 years at 5% p.a. R
Solution: A = P 1
compound interest. n 100
3
T 2
 R   10  2
Solution: A = P 1  = 12000   1  
 100   2 100 
3
 5 
2
 21 
= 2000 1  = 12000   
  20 
 100 
2 12000  9261
 105  =
= 2000   8000
 100 
= 13891.50
@UPSC_THOUGHTS

= 2000 × 1.052 = Rs 2,205


● Find the compound interest on Rs 4000 for 3 years ∴ C.I. = 13891.50 – 12000
at 10% p.a. = Rs 1891.50
 R 
T
 ● A sum becomes Rs 4630.50 in 3 years at 5% C.I.
Solution: C.I. = P  1    1
 100   p.a. Find the sum.
T
 10 
3
 R
4000  1  Solution: A = P 1
=   1 100
 100  
3 3
 5   105 
 110 3  4630.50 = P 1   = P  
= 4000    1  100   100 
 100   3
 21 
= 4000 × [(1.1) – 1] 3 = P  
 20 
= 4000 × (1.331 – 1) 3
20
= 4000 × 0.331 = Rs 1,324 ∴ P = 4630.50
21
● Find the difference between C.I. and S.I. on a sum 4630.50  8000
= = Rs 4,000
of Rs 8,000 for a period of 3 years at 5% p.a. 9261
 R 
T
 ● In what time will Rs 6,000 amount to Rs 6,615 at
Solution: C.I. = P  1    1
 100   5% p.a. compounded annually.
T
 3
  R 
5  Solution: A = P 1  
= 8000   1    1  100 
 100  
T
 5 
 105  3
 6615 = 6000  1  
= 8000     1  100 
  100   T
 105 
= 8000 × [(1.05)3 – 1] = 6000  
 100 

6 . 59
Basic Numeracy and Data Interpretation

6615 ● Sarika invests Rs 10,000 in a bond which gives


= (1.05)T interest at 5% per annum during the first year, 6%
6000
during the second year and 10% during the third
1.1025 = (1.05)T
year. How much does she get at the end of the third
(1.05)2 = (1.05)T
year.
T = 2 years
Solution: We have P = 10,000, R1 = 5,
● What is the rate of interest (compounded annually)
R2 = 6, R3 = 10
if Rs 800 amounts to Rs 882 in 2 years?
 R 
T  R  R  R 
Solution: A = P 1
Amount = P  1  1  1  2  1  3  . . . .
  100  100  100 
 100 
882
2 5 6 10
R = 10000 1 1 1
= 1 100 100 100
800 100
2 105 106 110
 R  = 10000
1.1025 = 1  100 100 100
 100 
R 21 53 11
∴ 1 = = 10000
1.1025 20 50 10
100
= 1.05 = Rs 12,243
R ● What will be the compound interest on Rs 5,000
= 1.05 – 1 = 0.05
100 3
for 2 years at 8% per annum?
R = 0.05 × 100 = 5% 4
@UPSC_THOUGHTS

3
● What will be the compound interest on Rs 800 for Solution: We have P = 5000, R = 8% and T = 2
4
2 years at 10% p.a., interest being charged (i) 3
2 R
annually, (ii) half yearly? R
A = P 1 1 4
100 100
Solution: (i) Here n = 1
n T
 3 
R 8
8  
2
A = P 1  4
n 100 = 5000  1   1  
2
 100   100 
 10   
= 800  1  
 100  108 108  6 
2 = 5000    1  
 110  100 100  100 
= 800   
 100 
= 800 × (1.1)2 108 108 106
= 5000   
100 100 100
= 800 × 1.21 = 968
∴ C.I., reckoned annually = 968 – 800 5000 
27 27 53
 
=
= Rs 168 25 25 50
(ii) Here n = 2 = Rs 6,181.92
10
2 2 ● What is the difference between compound interest
A = 800 1 and simple interest on a sum of Rs 12,500 for 2
2 100
years at 4% per annum?
21 21 21 21 Solution: We have P = 12,500, R = 4%
= 800    
20 20 20 20 2
 R 
= 972.405 C.I. – S.I. = P 
 100 
∴ C.I.= 972.405 – 800 2
= Rs 172.405 4
= 12500
= Rs 172.41 100
(correct to two decimal places) = Rs 20

6 . 60
Basic Numeracy and Data Interpretation

● The difference between compound interest and


A
simple interest on a sum of money for 3 years at Present worth = t
R
4% per annum is Rs 50, find the sum. 1
100
Solution: We have C.I. – S.I. = 50, R = 4 9261
= 3
 R   R  
3 2
 5 
C.I. – S.I. = P    3   1 
 100 
 100   100  
9261  20  20  20
 4 3  4 2  =
50 = P    3   21  21  21
 100   100  
= Rs 8,000
 1 3  1 2 
50 = P    3    ● A sum of money at C.I. amounts to Rs 1,500 in 2
 25   25   years and to Rs 2,535 in 4 years. Find the rate of
interest per annum.
 1 3 
50 = P 
 15625 625  Solution: Here x = 1500, y = 2535,
 1  75  p = 2, q = 4
50 = P
 15625  2535
1
4 2
∴ R = 1 × 100%
 76  1500
50 = P
 15625  1
= 1.69 2
1 100%
50  15625
P =
@UPSC_THOUGHTS

76 = [1.3 – 1] × 100%
= Rs 10,279.60 = 0.3 × 100%
● A sum of money becomes 3 times in 15 years at = 30%
compound interest. In how many years will the
same amount of money become 9 times itself.
DISCOUNT
Solution: We have x = 3, T = 15, y = 2
The given sum will become 9 times itself in Ty Trade discount has already been discussed in the unit
years, i.e., 15 × 2 = 30 years on Profit and Loss. Just to recapitulate:
● At what rate per cent compound interest does a ● If the discount allowed is d% then,
sum of money become 16 times in 4 years? 100  d
Solution: We have x = 16, T = 4 SP =  MP
100
1 ● For successive discounts
R = 100 (x ) n 1 100  d1 100  d2 100  d3
SP =    ....  LP / AP / PP / MP
100 100 100
1
where d1, d2, d3 are successive discounts.
R = 100 (16)T 1
Discount ×100
● Rate of discount =
Marked Price
 1
 MP  (100  d)
R = 100 (24 ) 4  1 ● CP =
  (100  P%)
R = 100 [2 – 1] Some terms:
R = 100% Net price = Marked price – Discount
There other terms related to the purchase of an
● Find the present worth of Rs 9261 due in 3 years
article include:
hence at 5% per annum compounded yearly.
MP = Marked Price
Solution: We have A = 9261, t = 3 and R = 5 AP = Advertised Price

6 . 61
Basic Numeracy and Data Interpretation

PP = Printed Price Amount or sum due = Rs 1500, R% = 10%


LP = Listed Price and T = 5 years.
NP = Net Price True discount = Amount – Present Worth
All these prices are the prices at which the article = 1500 – 1000
is listed or marked to be sold. In other words, they are = Rs 500
the price that are labelled on the article at which the
article shall be sold. Formulae to Remember for True Discount
Let Amount = A, Present Worth = PW,
True Discount True discount = TD, Rate = R, Time = T,
True discount is the difference between the amount or Simple interest = SI
sum due at the end of a given time and its present (i) A = PW + TD
worth.
A  R T A  R T A
The money which amount to the sum due or (ii) TD = = =
100 100  RT 1
100
amount in the given time at the already agreed upon
RT
rate of interest is the present worth (PW) of the amount. A
(iii) PW =
RT
True discount = Amount – Present Worth 1
100
In effect, true discount is the simple interest
ART
calculated on the present worth of the amount, though (iv) SI =
100
discount is seen as a discount on amount, i.e.,
SI  TD
Present worth = Amount – True Discount (v) P =
SI  TD
TD  R  T
Assume that, I have borrowed some money say (vi) SI on TD = SI – TD =
100
@UPSC_THOUGHTS

Rs 1,000 from X and I have to give this money back A  (RT )2


after 5 years. The given time has been agreed upon by =
100(100  RT )
X and myself. The rate of interest agreed upon is 10%
A
per annum on S.I. basis. So after 5 years, at a given rate (vii) PW = T
 R 
of 10% p.a., the amount to be paid back to X (i.e. the 1  
sum due after 5 years) amounts to Rs 1,500. But I would  100 
like to clear off my debt now, after paying some money. PW1 100  RT2
(viii) PW = 100  RT
Clearly, I do not want to pay the entire amount in 2 1

advance either. The present worth or value of the sum


TD1 PW1  R  T1
which amounts to Rs 1,500 in 5 years at 10% p.a. is (ix) TD = PW  R  T
2 2 2
calculated.
(x) Where, A is the amount due after T years hence
Amount (A) = Principal (P)+Simple Interest (SI)
P is the money of each instalment
= P  PRT R is the rate per cent per annum on
100 simple interest.
P (100  RT ) n is the number of equal instalments to
A =
100 be paid to clear the debt.
A  100 k is the number of instalments in a year.
∴P =
100  RT
P  R n(n  1)
A = n P  
Here, we have A = 1500, R = 10 and T = 5 100 2k
(xi) Where, n is the number of equal instalments
1500  100
∴P = = Rs 1000 T1 is the time period for the first
100  10  5
instalment.
The present worth or present value is Rs 1,000. If T2 is the time period for the second
I pay Rs 1,000 now, I would be able to clear off my debt. instalments.
So, in this example, 100  A  1 1 1 
PW =    ....  
Present Worth = Rs 1000,
n  (100  RT1 ) (100  RT2) 100  RTn 

6 . 62
Basic Numeracy and Data Interpretation

(xii) Where the sum is put at compound interest, (ii) TD = Bill Amount (A) – Present Worth (PW)
A PW  R  T BG  100
PW = T
= S.I. on PW= =
R 100 RT
1
100 ART A BD
= =
100  RT A BD
Banker’s Discount
Let’s suppose a person A buys goods worth a sum of
= PW  BG
Rs 10,000 from a merchant B on credit for a period of
= (A  TD )(BD  TD )
4 months. B prepares a bill called the bill of exchange.
This bill is an agreement duly signed by the debtor (iii) Banker’s Gain (BG) = Banker’s Discount (BD)
stating that he has accepted the bill and the money can –True discount (TD)
be withdrawn from his bank by the creditor on the due
T .D.  R  T
date. = SI on TD =
100
The sum due or the amount of the bill is called the 2
= Present Worth  
face value of the bill. RT

The due date—the date on which the face value is  100 
due from the debtor—is known as the nominally due (TD)2 A( RT )2
= =
date. Usually a grace period of 3 days or so is given; PW 100(100  RT )
the date so arrived at is called the legally due date. BD  TD BD  TD
The debtor (the borrower) is bound by law, through (iv) Amount = =
BD  TD BG
the Bill of Exchange, to pay the money at the mutually
@UPSC_THOUGHTS

A  100 (TD )2
agreed upon terms, to the creditor at the latest by the (v) Present Worth (PW)= =
100  RT BG
legally due date.
If the creditor, B, wants his money before the legally Worked Examples
due date, he can encash the bill of exchange from a
● A fan is listed at Rs 1,500 and a discount of 20%
bank or a broker. The bank (or broker) will charge
is offered on the list price. What additional discount
simple interest on the face value for the unexired time,
must be offered to the customer now to bring the
i.e., the period from the date the bill was ‘discounted’
net price to Rs 1,104?
(paid by the banker/broker) and the legally due date.
(If the date of the bill is not given, the days of grace Solution: We have MP = 1500, d1 = 20%, d2 = x
are not added.) This simple interest amount charged by and SP = 1104
the banker is known as the banker’s discount. 100  d1 100  d2
Banker’s discount is slightly more than the True ∴ SP =   MP
100 100
Discount. The difference between the banker’s discount 100  20 100  x
(B.D.) and the true discount (T.D.) is called the banker’s 1104 =   1500
100 100
gain (B.G.)
80 100  x
1104 =   1500
Formulae to Remember for Banker’s Discount 100 100

(i) Banker’s discount = SI on bill for its 1104 = 4 × 100 – x × 3


unexpired time
1104
Bill amount (A) Rate(R ) Unexpired time (T ) = 100 – x
= 12
100 92 = 100 – x
TD(100  RT ) x = 100 – 92
=
100 x = 8
∴ Additional discount of 8% is offered to the
BD(100  RT ) A  BG A  TD
= = = customer to bring the net price to Rs 1,104.
RT TD A  TD

6 . 63
Basic Numeracy and Data Interpretation

● The marked price of an article is Rs 200. A discount


A 100
1 Solution: PW=
of 12 % is allowed on the marked price and a 100 RT
2
profit of 25% is made. Find the cost price of the
article. 3756 100
= 13 8
1 100
Solution: We have MP = 200, d = 12 , P = 25 2 12
2
MP  (100  d)
∴ C.P. = 3756 100 3756 100 3
(100  P%) = =
13 313
 1 100
200   100  12  3
 2
= = Rs 3,600
(100  25)
TD = 3756 – 3600 = Rs 156
25
200 100 ● Find the present worth of a bill of Rs 5,512.50 due
2
= in 2 years hence at 5% per annum compound
125
interest. Calculate the true discount also.
100  175
= Amount
125 Solution: PW= T
= Rs 140  R 
1  
● Successive discounts of 10% and 20% are equivalent  100 
to a single discount of how much? 5512.50 5512.50
= =
Solution: Total discount = MP – SP  5 
2
1.05  1.05
Here, let MP be Rs 100 1 
 100 
@UPSC_THOUGHTS

100  10 100  20
and S.P. =   100 5512.50
100 100 = = Rs 5000.00
1.1025
90 80
=   100 TD = A – PW
100 100
5512.50 – 5000 = Rs 512.50
= Rs 72
∴ Total discount = 100 – 72 ● The true discount on bill due 6 years hence at 9%
= Rs 28 on Rs 100 per annum simple interest is Rs 810. Find the
= 28% amount of the bill and its present worth.
● Find the present worth of Rs 9280 due 2 years PW RT
Solution: TD =
hence at 8% p.a. on simple interest. Find the true 100
discount also. TD  100 810  100
PW = = = 1500
RT 9 6
A  100
Solution: PW = ∴ Amount = 1500 + 810 = Rs 2,310
100  RT
9280 100 ● The difference between the S.I. and the true discount
=
100 2 8 on a certain sum of money for 6 years at 9% per
9280  100 annum is Rs 437.40. Find the sum.
=
116 TD  RT
Solution: SI – TD =
∴ PW = Rs 8,000 100
Let TD be x.
TD = 9280 – 8000 = Rs 1,280
x  9 6
∴ 437.40 =
● Find the present worth of Rs 3,756 due in 8 months 100
1 437.40 43740
at 6 % per annum on simple interest basis. What x =  100 =
2 96 9 6
is the true discount? = 810

6 . 64
Basic Numeracy and Data Interpretation

∴ TD = 810 ● If the true discount on a certain sum due in 5


months at 5% is Rs 9, what is the banker’s discount
A  RT
TD = for the same sum at the same rate for the same time
100  RT
period?
TD  (100  RT )
∴ A = Solution: TD = 9
RT
∴ BD = SI on TD
810  154 9  5 5 3
= = =
54 13  100 16
A = Rs 2310
∴ BD = TD + BG
● The true discount on Rs 925 due after a certain time 3 3
= 9 = 9
1 16 16
at 3 % p.a. is Rs 175. Find the time after which
2 ● The present worth of a bill due some time hence
it is due.
is Rs 2460 and the true discount is Rs 410. Find
Solution: PW = A – TD = 925 – 175 = 750 the banker’s discount and the bankers gain.
PW  R  T TD2 410  410
If TD = Solution: BG = =
100 PW 2460
TD 100 2 1
Then T = = 68 = 68
PW R 6 3
175  100  2 1
= ∴ BD = BG + TD = 410  68
750  7 3
@UPSC_THOUGHTS

1
100 = 478
= 3
15 ● The banker’s discount and true discount on a
20 2 certain sum of money due 5 months hence is Rs
= = 6 years
3 3 78.75 and Rs 75 respectively. Find the sum due and
● A shopkeeper purchased a mobile phone handset the rate per cent.
for Rs 3000, and sold it for Rs 3960, on credit for
BD  TD 78.75  75
some time, thereby gaining 10%. If the rate of Solution: Amount = =
BD  TD 78.75  75
interest is 5%, find for how long was the credit
7875  75  100
allowed? = = Rs 1575
100  375
Solution: CP = 3000; A = (SP) = 3960 ART
R = 5%; T = ? BD =
100
CP + Gain = Present Worth of the amount
BD  100 78.75  100  12
∴ R = =
Gain % C .P. AT 1575  5
i.e. PW= 3000 
100 7875  12
= = 12%
10  3000 1575  5
= 3000 
100 ● If the true discount on Rs 1750 due in a certain
= 3300 time is Rs 350, find the banker’s discount and the
A  100 banker’s gain.
PW =
100  RT BD TD
Solution: A =
BD TD
3960  100
i.e. 3300 = BD 350
100  5T i.e., 1750 =
BD 350
3960  100
100 + 5T = 1750BD – 350BD = 1750 × 350
3300
= 120 1750  350
5T = 20 BD =
1400
T = 4 years = Rs 437.50

6 . 65
Basic Numeracy and Data Interpretation

BG = Rs 437.50 – 350
PARTNERSHIP
= Rs 87.50
Basics
● A banker paid Rs 7938.75 for a bill of Rs 8700
drawn on July 4, 2000 for 2 years. When did the An association of two or more persons who put their
bill get discounted, if the rate of interest is 7%? money together in order to carry on a certain business
is called partnership and these persons are called
Solution: BD = 8700 – 7938.75 = 761.25 partners.
i.e., Rs 761.25 is the simple interest on
Partners may be active or sleeping. An active
Rs 8700 at 7% for the unexpired time T.
partner is a partner who devotes his time for the
BD  100
∴ T (unexpired time) = business in addition for his investment. A sleeping
A R
partner is a partner who only invests money in the
761.25  100 76125
= = business.
8700  7 8700 7
In simple partnership the capitals of all the partners
= 1.25 years = 1.25 × 12 = 15 months are invested for the same time period, and the profit/
Date on which the bill was loss is distributed proportionally according to the
drawn = July 4, 2000
investment of the partners.
Nominal due date = July 4, 2002
In compound partnership capitals are invested for
Legally due date = July 7, 2002
different time periods, and the capital of each partner
∴ The date of encashment of the bill
is multiplied by the number of months the money was
= 15 months prior to July 2, 2002
invested in the business, and the profit/loss is
@UPSC_THOUGHTS

= April 7, 2001
proportionately distributed.
● The banker’s gain on a certain sum of money for
4 years at 10% is Rs 192. Find the present worth Formulae to Remember
and the sum due.
(i) If the capitals of two partners be x and y for the
TD  RT same period and the total profit be P, then the
Solution: BG =
100 shares of the partners in the profits are
BG  100 192  100 P P
∴ TD = = = 480 x y
RT 10  4 and
x y x y
TD2 480  480
∴ PW= = = Rs 1200
BG 192 (ii) If the capitals of three partners be x, y, and z
∴ Sum due = 1200 + 480 = Rs 1680 for the same period and the total profit be P, then
● Calculate the BD on a bill due 8 months hence at the shares of the partners in the profits are
9% per annum if the present worth of the bill is P P P
x, y and z
Rs 3200. x y z x y z x y z
A  100
Solution: PW =
100  RT (iii) If the capitals of two partners be x and y for the
PW (100  RT ) periods t1 and t2, respectively and the total profit
∴ A = be p, the shares of the partners in the profits are:
100
 8  x t1 P y t2 P
3200  100   9 
=  12  x t1 y t2 and x t1 y t2
100
= 32×106 = Rs 3392 (iv) If the capitals of three partners be x, y and z for
3392  9  8 the periods t1, t2 and t3, respectively and the total
BD = = 203.52
12  100 profit be P, then the shares of the partners in
BD = 203.52 the profits are

6 . 66
Basic Numeracy and Data Interpretation

x t1 P y t2 P 12
Atul’s share = of Rs 50,00,000
x t1 y t2 z t3 , x t1 y t2 z t3 45
= Rs 13,33,333.30
z t3 P
and x t y t z t 15
1 2 3 Amit’s share = of Rs 50,00,000
45
(v) If two partners A and B invest x and y amounts
= Rs 16,66,666.60
in a business for t1 and t2 respectively,
18
x t1 Profit of A Loss of A Sohail’s share = of Rs 50,00,000
or 45
y t2 Profit of B Loss of B
= Rs 20,00,000
(vi) If more than two persons invest money in a
business, say A, B and C invest x, y and z for ● A, B and C opened a business by contributing Rs
t1, t2 and t3 time respectively, 10,000, Rs 20,000 and Rs 15,000. At the year end,
Amount of investment of A × No. of months the profit was Rs 4500. How should they distribute
invested by A : the profit?
Amount of investment of B × No. of months
Solution: Ratio of investments
invested by B :
= 10,000 : 20,000 : 15,000
Amount of investment of C × No. of months
= 2 : 4 : 3
invested by C
Ratio of profit = 2 : 4 : 3
x t1 : y t2 : z t3
2
A’s share =  4500
= Profit of A : Profit of B : Profit of C 2 4 3
@UPSC_THOUGHTS

(Or Loss of A : Loss of B : Loss of C) 2


=  4500 = Rs 1000
9
(vii) If the times of the investments of three partners
4
are in the ratio t1 : t2 : t3, and their profits are B’s share =  4500 = Rs 2000
9
in the ratio P1 : P2 : P3, then their capitals x, y,
3
z are in the ratio C’s share =  4500 = Rs 1500
9
P1 P P ● A, B and C are three partners. A invested Rs 2,000,
: 2 : 3
t1 t2 t3 B invested Rs 10,000 and C invested Rs. 6,000. If
(viii) If the investments of three partners are in the C got a profit of Rs 450 at the end of the year, how
ratio x : y : z, and their profits are in the ratio much did A and B get?
P1 : P2 : P3, the time periods of their investment
Solution: Ratio of investments
are in the ratio
= 2,000 : 10,000 : 6,000
P1 P P = 1 : 5 : 3
: 2 : 3
x y z 3
C’s share of profit =  x = 450
9
where x is the total profit.
Worked Examples
450  9
● Atul, Amit and Sohail started a business in Total profit is x = = Rs 1350
3
partnership. Atul invested Rs 1,20,000, Amit
invested Rs 1,50,000 and Sohail invested Rs 1,80,000 1
∴ A’s share of profit =  1350 = Rs 150
respectively in business. How should they divide 9
5
a profit of Rs 50,00,000? B’s share of profit =  1350 = Rs 750
9
Solution: Profit should be divided in the ratios of
the capitals, i.e., in the ratio ● Prasad and Prakash start a business with Rs 1500
1
120000 : 150000 : 180000 = 12 : 15 : 18. and Rs 1200 respectively. Prakash gets 12 % of
2
Now, 12 + 15 + 18 = 45 the profit for managing the business. If the total

6 . 67
Basic Numeracy and Data Interpretation

profit gained in a year is Rs 2,160, find the share ● X, Y and Z enter into a partnership and their
of Prakash. 1 1 1
investments are in the ratio : : . X withdraws
Solution: Ratio of investments = 1500 : 1200 3 2 5
= 15 : 12 half the capital after 4 months. At the end of the
= 30 : 24 = 5 : 4 year, a profit of Rs 20,003 is divided among them.
∴ Prakash’s share for managing the business Find the share of each.

25 Solution: Ratio of investment


=  2,160 = Rs 270
200
1 1 1
X : Y : Z = : :
∴ Prakash’s ratio of profit 3 2 5
= 10 : 15 : 6
= 2,160  270  4 Ratio of profit of X : Y : Z.
9
4 = 5 × 12 + 5 × 4 : 15 × 12 : 6 × 12
= 1890  = Rs 840
9 = 80 : 180 : 72
∴ Total share of Prakash = 840 + 270
= 20 : 45 : 18
= Rs 1,110
Sum of ratios = 83
● Reena and Meena are partners in a business in
20
which Reena has contributed Rs 1,470 and Meena ∴ X’s share =  20,003 = Rs 4,820
83
Rs 1,050. After 4 months, Venu joined them with
a capital of Rs 1,890. Find the ratio in which the 45
Y’s share =  20,003 = Rs 10,845
profit should be divided at the end of the year. 83
@UPSC_THOUGHTS

18
Solution: Reena’s investment = 1,470 × 12 months Z’s share =  20,003 = Rs 4,338
83
= Rs 17,640
● A, B and C enter into a partnership investing
Meena’s investment = 1,050 × 12
Rs 2000, Rs 4000 and Rs 6000, respectively. The
= Rs 12,600
net profit for the year was Rs 72,000 which was
Venu’s investment = 1,890 × 8 divided in proportion to investments. Find the
= Rs 15,120 amount of profit each partner earned.
Ratio of their investment
= 17,640 : 12,600 : 15,120 Solution: We have x = 2000, y = 4000, z = 6000
and P = 72000
(HCF = 2,520 and then dividing each
P
amount by HCF) = 7 : 5 : 6 Therefore, profit share of A = x
x y z
● The ratio of investments of two partners is 5 : 7 72000
= 2000
and the ratio of their profit is 20 : 21. If A invested 2000 4000 6000
the money for 8 months, find for how long B
72000
invested the money? = 2000
12000
Solution: Let us assume that B invested the money
for x months. = 6 × 2000 = Rs 12,000
Then, 5 × 8 : 7 × x= 20 : 21 P
Profit share of B = y
40 20 x y z
=
7x 21
72000
40  21 = 4000
7x = 2000 4000 6000
20
2  21 72000
x = = 4000
7 12000
= 6 months
= Rs 24,000

6 . 68
Basic Numeracy and Data Interpretation

P STOCKS, SHARES AND DEBENTURES


Profit share of C = z
x y z
Basics
72000
= 6000 When a group of individuals come together to start a
2000 4000 6000 business venture, involving large amount of money, a
company or a corporation is said to be formed.
72000 The capital of the company is called the stock. Stock
= 6000
12000 is a unit of ownership of a company. It consists of a
group of shares. The total investments required by the
= Rs 36,000
company or the corporation is known as capital stock.
Shares can be explained as “a fixed and indivisible
● A and B enter into a partnership. A contributes Rs
section of the capital of a company”. Shares are generally
5000 for 6 months and B Rs 4000 for 5 months.
worth Rs 10 and Rs 100 each. Money derived from the
If the total profit is Rs 40,000, find their respective
sale of shares in a business and used for carrying it
shares. on is termed as share capital. Company issues a share
Solution: We have x = 5000, y = 4000, t1 = 6, certificate to the person (shareholder) who owns a
t2 = 5 and P = 40000 share or shares to certify the ownership of its shares.
The government, at times raises money from the
x t1 P
∴ Profit share of A = public at a certain fixed rate of interest, to meet the plan
x t1 y t2 expenditure. Bonds or promissory notes each having
a fixed value are then issued to public for sale.
@UPSC_THOUGHTS

5000 6 40000
= If a bond is purchased by a person for Rs 100 at
5000 6 4000 5
which, say, 7% interest is fixed by the government, we
= Rs 24,000 say that the person (holder of the bond) has a Rs 100
stock at 7%. Here Rs 100 is the face value of the stock.
y t2 P
Profit share of B = The holder of the bond gets an annual interest of 7%
x t1 y t2
of the face value, i.e., if the face value of the stock is
4000 5 40000 Rs 100, then the annual interest is Rs 7.
= 5000 6 4000 5 Though a fixed period is set for the bonds to
mature, stocks are bought and sold in the open market
= Rs 16,000
through brokers at stock exchanges. The broker’s charges
are known as brokerage.
● A and B together invested Rs 1,800 in a business. When stock is purchased, brokerage is added to the
At the end of the year, out of a total profit of cost price. When stock is sold, brokerage is subtracted
Rs 1,500, A’s share was Rs 500. What was the from the selling price.
investment of A?
1 1
Brokerage means Rs for Rs 100 stock.
5 5
A’s investment Profit share of A The selling price of a Rs 100 stock is said to be
Solution:
B’s investment Profit share of B below par, at par or above par, depending upon
whether the selling price of the stock is less than
500 500 1
∴ Rs 100 or exactly Rs 100 or more than Rs 100.
1500 500 1000 2 The original value of the share is called its nominal
value or face value or par value. Face value or the
1 1 par value of a share is the value stated in the stock
∴ Investment of A = 1800 1800
1 2 3 certificate. Fave value always remains the same.
The market value (or the cash value) of a share is
= Rs 600 the price of the share in the market.

6 . 69
Basic Numeracy and Data Interpretation

If the market value (cash value) is equal to its Since total investment = total purchase cost,
nominal value, then the share is said to be at par. Annual income =
If the market value is greater than the nominal Investment
value, the share is at a premium and if the market value % rate of int. on stock ×
MV+brokerage
is less than the nominal value, then the share is at a
discount. (v) % rate of interest is the rate % of interest received
Dividend is the profit of the company which is per annum per Rs 100 stock. But if the stock is
divided among shareholders in proportion to their at a premium or at a discount on the total
holdings. Dividend is always paid on the face value
investment, actual rate % obtained will change
of a share. A company may declare the dividend as a
accordingly.
certain percentage of par value of a specified number
of rupees or paise per share. For example, if the stock is at a premium of 10,
A 12% dividend means that, on a share of Rs 100, then MV of the stock = 100 + 10 = 110
a shareholder gets Rs 12. If 5% is the rate of interest, then actual rate %
Preference shares: A fixed rate of dividend is paid 5
obtained = 100 = 4.55%
on these shares to the shareholder. 110
Ordinary or Equity shares: After paying the If the stock is at a discount of 10, then for 5% rate
preference shareholders, the dividends are paid to the %, actual rate % obtained
holders of equity shares. These dividends vary 5
= 100 = 5.56%
depending upon the profits of the company. 90
Debentures is the written acknowledgement of a In a discount stock, actual rate % obtained
debt. Debenture is a security issued by a company for increases while in a premium stock, the actual
@UPSC_THOUGHTS

money borrowed on the company’s property, having a rate % obtained actually decreases.
fixed rate of interest to the shareholders or public for
a fixed period. A debenture-holder gets interest Total investment
(vi) Number of shares =
calculated on the face value of the debenture, at a fixed Purchase value of 1 share
rate of interest of the company and hence the interest Total investment
does not vary. = MV (1 + % brokerage)

Formulae for Problems on Stocks Total amount realised on sale


(vii) Number of shares =
Sale value of one share
(i) Changes from time to time in the market value of
a share Total Sale realisation
= MV (1 % brokerage)
(a) If the share is at a premium, then
Market value (MV) = Face value (FV) + Premium
(b) If the share is at a discount, then Formulae for Problems on Shares
Market value (MV) = Face value (FV) – Discount All the formulae for shares are also applicable for
(face value may be Rs 10 or Rs 100) solving questions on debentures.
(ii) If the stock is purchased, then the purchase cost
(i) If the stock is at premium,
MV + brokerage Market value = 100 + premium
= × Amount of stock
100 (ii) If the stock is at a discount,
then MV = 100 – discount
(iii) If the stock is sold, then the sale realisation
(iii) Brokerage is calculated on market value–as a per
MV – brokerage
= × Amount of stock cent of market value.
100 % Brokerage is added to market value, in a
(iv) If the rate % of stock is given, annual income on purchase.
the total stock can be found. Purchase Value = MV+% brokerage on MV
Annual income (iv) (a) For one share,
Amount of stock
= % rate of stock × Purchase Cost = MV + (1 + % bokerage)
100

6 . 70
Basic Numeracy and Data Interpretation

(b) For the stock, Worked Examples


Purchase cost ● Find the cost of Rs 2,000, 7% stock at 10 premium.
MV(1+% brokerage)×Amount of Stock Solution: Market Value = 100 + 10 = Rs 110
=
100 Total Investment = Total purchase cost
Number of shares 2000  110
= = Rs 2,200
Total purchase cost 100
=
MV (1 + % brokerage) ● Find the cost of Rs 3,000, 6% stock at 8% discount,
During sale of stock, brokerage is to be deducted 1
brokerage %.
from M.V. 2
1 1 1
(v) (a) For one share, Solution: MV = 100  8  = 92  = 92 %
2 2 2
Sale Value = MV (1 – %brokerage) 185
∴ Purchase cost =  3000
(b) For the stock, sale realisation 2  100
185
MV (1 % brokerage) × Amount of stock =  3000
= 200
100
Also, number of shares 5550
= = Rs 2,775
Sale realisation 2
= ● How much stock should be sold to realise Rs 1,850
MV(1 % brokerage) 1
(brokerage % ) from 6 % stock at 7% discount?
Purchase Cost 2
Amount of stock
(vi) = Solution: Sale Value of Rs 100 stock
100 MV(1 + % bokerage)
@UPSC_THOUGHTS

= MV – brokerage
Investment
= 1
MV(1 + % brokerage) = (100  7) 
2
Sale realisation
= 1 1
MV(1  % brokerage) = 100  7 = 92
● Investment is the same as total purchase cost. 2 2
(vii) Annual income 185
=
% rate of stock × amount of stock 2
= Amount of stock
100
% rate of stock × Investment Total sale realisation×100
= =
MV(1 + % brokerage) Sale value of 1 stock
(viii) Actual rate % obtained 1,850 × 2
= 100 = 2000
% rate of stock × 100 185
=
MV(1 + % brokerage) ● Find the annual income received by investing Rs
(ix) Dividend (expressed usually as a % rate of 2,800 in 4% stock at 112.
dividend) is calculated on face value, not on
Solution: Total investment = 2,800
market value.
MV = 112
Income earned from dividend
rate % = 4%
% rate of dividend × FV × No. of shares
= rate % × Investment
100 ∴ Annual Income = (MV + brokerage)
Actual rate % on investment
Dividend % × FV 4×2,800
= = Rs 100
= MV (1 + % brokerage) 112

Annual Income ● Find the cash realised by selling Rs 3,200, 6% stock


Investment × Div. % × FV 1
at 5% premium, while brokerage is %.
= 100 MV (1 + % brokerage) 4

6 . 71
Basic Numeracy and Data Interpretation

18000 – 2x = 13,200
1
Solution: Sale price of Rs 100 stock = 100  5 
4 18000 – 13200 = 2x
3 419 4,800 = 2x
= 104 =
4 4
x = 2400
419
∴ Total cash realised = 3200
4 100 ∴ Rs 2,400 invested in 3% stock at 99
= Rs 3,352 and Rs 3,600 invested in 12% stock at 132.
● Find the total investment made to purchase
1 ● Mr Harshad invested Rs 6,000 in the shares of face
Rs 3,600, 5% stock at 5% premium (brokerage % ).
4 value of Rs 100 and 12% dividend. His income
Solution: Cost price of Rs 100 stock from dividend was Rs 900. Find the market value
1 421
of the share if the brokerage is nil%.
= 100  5  =
4 4
Solution: Let the market value of the share be x.
421
∴ Total investment = 3600 Income earned from dividend
4 100
% rate of dividend × FV × No. of shares
= Rs 3,789 =
100
● A man sells Rs 5,000, 5% stock at 120 and invests
the proceeds partly in 3% stock at 99 and partly % dividend × FV × Investment
=
in 12% stock at 132; thereby increasing his income 100  MV(1 + % brokerage)
@UPSC_THOUGHTS

by Rs 150. How much of the sale proceeds were


invested in each stock? % dividend × FV × Investment
MV =
Income from dividend × 100
Solution: Sale realisation
12 × 100 × 6000
(MV  brokerage)×Amount of stock = = Rs 80
= 900 100
100
120 5000 ● A man buys 500 debentures of face value of Rs 100
(Brokerage-nil) = each at Rs 92 and sells the same when the price
100
rises to Rs 98. If the brokerage is 2%, find his gain
= Rs 6,000 or loss.
5
Annual income of 5% = 5000
100 Solution: Investment while buying 500 debentures
= MV(1 + %brokerage) × No. of debentures
= Rs 250
Income after re-investment = 250 + 150
 2 
= Rs 400 = 92  1    500
Let Rs x from the proceeds of sale be re-invested  100 
in 3% stock at 99 and Rs (6,000 – x) be invested 92  102  500
in 12% stock at Rs 132. Total annual income is = = 46,920
100
given
 2 
3 x 12 (6000 x )
= 400 (given) Total sale amount = 98  1    500
99 132  100 

x 3(6000 x ) 98  98  500
= 400 =
33 100
x + 18000 – 3x = 400 × 33 = 48,020
= 13,200 ∴ Gain = Rs 1,100

6 . 72
Basic Numeracy and Data Interpretation

January 31 31
CALENDARS
February +28 59 (60 in case of leap year)
Basics March +31 90
A day is the chief measure of time. A day consists of April 30 120
24 hours, which is the average time in which earth May 31 151
turns round on its axis. June 30 181
An ordinary year has 365 days, i.e., 52 weeks and July 31 212
one day. ( 365 = 52 × 7 + 1) August 31 243
September 30 273
These extra number of days left in a year/month
October 31 304
after the complete number of weeks are called odd days.
November 30 334
Every year which is divisible by 4 is a leap year;
December 31 365 days
but any century year (year ending in 00) which is not
divisible by 400 is not a leap year.
In case of a leap year, from February onwards, the
For example: 1964, 1988, 1992, 2000, are all leap cumulative sum increases by one day.
years; where as 100, 200, 1500, 2100, 1989, 1990... are
Division of a year (Quarterwise)
all examples of ordinary years.
Period No. of days
A leap year has 366 days.
The following facts should be remembered: February 28 (29)
(i) 1 week = 7 days 1st quarter (Jan 1 to March 31) 90 (91)
2nd quarter (April 1 to June 30) 91
@UPSC_THOUGHTS

(ii) In an ordinary year there are 52 weeks + 1


3rd quarter (July 1 to Sept. 30) 92
day = 365 days. Therefore, it contains 1 odd
4th quarter (Oct 1 to Dec 31) 92
day.
Jan 1 to Dec 31 365 (366)
(iii) A leap year contains 2 odd days. (A leap year
has 366 days which is 52 × 7 + 2 days.) Number of days in a leap year is given in brackets.
(iv) 100 years = 76 ordinary years + 24 leap years
(xi) A day repeats itself after 7, 14, 21, ... 364 days
∴ Number of odd days in 100 years (including the present day).
= 76 odd days + 24 × 2 odd days
(xii) An ordinary year thus contains 1 odd day,
= 124 odd days = 17 weeks + 5 odd days therefore in a non-leap year, the weekday of a
100 years contain 5 odd days specific date will increase by one in the next year.
(v) As 100 years contain 5 odd days, 200 years But a leap year contains 2 odd days. Therefore,
contain 5 + 5 = 10 odd days and therefore 3 for each leap year, the day of the week advances
odd days. by two for a date.
(vi) 300 years contain 5 + 5 + 5 = 15 odd days To find the day of the week on a particular date
and therefore 1 odd day. when no reference day is given:
(vii) 400 years contain 5 + 5 + 5 + 6 (400th year
(a) Count the net number of odd days on the given
is a leap year) = 21 days and therefore 0 odd
date.
days.
(b) For 0 odd day write Sunday
(viii) Last day of a century cannot be aTuesday, a For 1 odd day write Monday
Thursday or a Saturday. For 2 odd days write Tuesday
(ix) The first day of a century must be a Monday, For 3 odd days write Wednesday
a Tuesday, aThursday or a Saturday. For 4 odd days write Thursday
(x) Cumulative number of days in the months of For 5 odd days write Friday
an year. For 6 odd days write Saturday

6 . 73
Basic Numeracy and Data Interpretation

Worked Examples (b) the hands are straight (point in


● If April 4, 1988 was a Monday, what day was opposite directions) once;
September 2, 1988? (c) the hands are twice at right angles.
(iii) The minute hand moves through 6° in
Solution: From April 5, 1988, the number of days
each minute; an hour hand moves through
till September 2, 1988
= Apr May June July Aug Sept. 1/2° in each minute.
26 + 31 + 30 + 31 + 31 + 2 (iv) Starting from 12 a.m., the hands coincide
= 151 days = 21 weeks + 4 odd days 11 times in every 12 hours or 22 times in
i.e., September 2, 1988 was Friday. 24 hours; between 11 and 1 o’clock there
is a common position 12 o’clock when the
● Arshu was born on August 19, 1992. What day of
hands coincide;
the week was he born?
(v) The hands point towards each other 11
Solution: 1992  1991 years + 7 months + 19 days times in 12 hours or 22 times in 24 hours
We begin with 1600 as reference year as it is (between 6 and 7 there is a common
known that it had no odd days. position at 6 o’clock when the hands are
1600 years contain 0 odd days straight);
300 years contain 1 odd day (vi) The hands of a clock are at right angles
91 years have 69 twice every hour. In 12 hours, they are at
ordinary years 69 odd days right angles 22 times and thus 44 times in
22 leap year 44 odd days a day. Two positions common in every 12
69 + 44 = 113  1 odd day hours, are at 3 o’clock and 9 o’clock.
@UPSC_THOUGHTS

From 1st January 1992, the number of days till


August 19, 1992 Worked Examples
= 232 days ● Find at what time between 8 and 9 o’clock will the
= 33 weeks + 1 odd days
hands of a clock be in the same straight line but
 1 odd day not together.
Total number of odd days
= (1+1) = 2 odd days Solution: At 8 o’clock, when the hour hand is at
8 and the minute hand is at 12, they are 20 minute
 August 19, 1992 was Tuesday
divisions apart. For them to be in the straight line
(but not together) they will have to be 30 minute
CLOCKS divisions apart. The minute hand will have to gain
10 minutes over the hour hand.
Basics 55 minutes gained in 60 minutes
The dial of a clock is divided into 60 equal parts, called 120
60
minute spaces. The minute hand passes over 60 minute = 10 minutes gained in × 10 =
55 11
spaces whilst the hour hand goes over 5 minute spaces.
10
One day contains 24 hours or 1440 minutes or 86400 = 10 minutes
11
seconds.
∴ The hour hand and the minute hand of a clock
Points to Remember will be in a straight line between 8 o’clock and
10
(i) The minute hand (MH) takes 60 minutes 9 o’clock after at 10 minutes past 8.
11
to gain 55 minute space (MS) over the hour
hand (HH). Therefore to gain 1 M.S. over ● At what time between 3 and 4 are the hands of
60 12 the clock 5 minutes apart?
the H.H., the M.H takes min. or
55 11
minutes. Solution: At 3 o’clock, the minute hand is 15
(ii) In every hour, minutes apart. The hands of the clock can be 5
(a) the hands coincide once. minutes apart in two cases—whether minute hand

6 . 74
Basic Numeracy and Data Interpretation

is 5 minutes before the hour hand and when it is equations work helps in solving arithmetic problems
5 minutes after the hour hand. faster. Some algebraic formulae have already been given
in an earlier unit.
When the minute hand is 5 minutes behind the
Algebraic fractions are solved on the same principles
hour hand, the minute hand has to gain
as arithmetical ones, except that we are dealing here
15 – 5 = 10 minutes.
with symbols as well as numbers.
55 minutes gained in 60 minutes One must be careful not to cancel terms where such
60 10 cancelling is not warranted. Take the term
10 minutes gained in 10 10 minutes
55 11 6  t
10
i.e., the hands will be 5 minutes apart at 10 8  t
11
minutes past 3 There is no scope of cancelling here as there is no
factor that divides the entire numerator as well as the
Now when the minute hand is ahead of (after)
entire denominator.
the hour hand, then the minute hand has to gain
For example, if we have to reduce to its lowest factor
15 + 5 = 20 minutes
the term
60 240
∴ 20 minutes gained in 20 3x 2 6 x
55 11 ,
4x3 8x2
9
= 21 = minutes we should factorise the numerator and denominator.
11
9 We get
i.e., the hands will be 5 minutes apart at 21 3x (x + 2)
11
minutes past 3’oclock. 4x2 (x + 2)
@UPSC_THOUGHTS

The factors common to both numerator and


● The minute hand of a clock overtakes the hour denominator are x and (x + 2). Dividing these out, we
hand at intervals of 65 minutes of correct time. 3
How much does the clock gain or lose in a day? get x.
4

Solution: In one hour, the minute hand gains 55 1 1


Suppose we have to find the sum of and .
minutes over the hour hand. a b
In order to coincide, it must gain 60 minutes Cross multiply and put the sum over the
60 720 denominator product and add the cross product.
i.e., 60 minutes gained in 60
55 11 1 1 a+b
+ =
5 a b ab
= 65
11
= minutes a3 b3
Now, what is the product of and 2 .
But here, they are together after 65 minutes itself. b2 a
2
5 Divide a into the first numerator and second
∴ Gain in 65 minutes = minutes denominator; then divide b2 into the first denominator
11
a b
24 60 5 1440 and second numerator; the expression is now 
Gain in 24 hours = 1 1
65 11 143 or ab.
10 Basic to using algebra in problems is conceptualising
= 10 = minutes.
143 an equation. The first step is to determine what quantity
or letter you wish to isolate. Solving an equation for x
means getting x on one side of the ‘=’ sign and
ALGEBRAIC FORMULAE AND everything else on the other.
EQUATIONS
Linear Equations
Basics A linear equation in one variable is an equation of the
While higher algebra need not be read in depth for this type
examination, an elementary knowledge of the way ax + b = 0 or ax = c,

6 . 75
Basic Numeracy and Data Interpretation

where a, b and c are constants, a 0, and x is an Then we combine the two equations; one of the
unknown variable. variables will be eliminated. Then follow the steps of
The equation substituting the value of the variable as in the other
ax = c method.
is solved for x, as Suppose we have to solve
3x + y = –2, 7x – 3y = –26
x = c.
a Take the variable y. In the first equation, it has a
A linear equation in two variables is an equation coefficient of 1, in the second it has a coefficient of
of the type –3. Multiply the first equation by 3 (positive) we have
ax + by + c = 0 9x + 3y = –6
or ax + by = d, 7x – 3y = –26
where a, b, c and d are constants, and a 0, b 0; Now combine the like terms of the two equations.
the unknown variables are x and y. The ‘y’ terms get eliminated and we have
16x = –32
To solve such equations, there are two methods:
x = –2
substitution and elimination.
Substitute the value of x in any equation as in the
In the substitution method, one of the equations for earlier example to get the value of y.
one of the variables, say y, is solved, and then this value Algebraic formulae are useful in solving even
for y is substituted in the second equation for y. arithmetical problems. A problem can be translated into
The second equation then becomes an equation algebra.
with one variable, in this case x, which can be solved. Let’s take an example:
@UPSC_THOUGHTS

Now, substituting this value of x, thus obtained, in the Q. Three consecutive odd integers have a sum of
first equation, the value of y can be found. 33. What is the average of these integers?
If we have to solve 3x + y = –2, 7x – 3y = –26, we Solution: Represent the integers as x, x + 2 and
solve the first equation for y; x + 4 (Consecutive odd or even integers will be 2 apart).
y = –3x – 2 Write an equation whose sum is 33.
Substituting this value of y in the second equation, 3x + 6 = 33
we get 3x = 27, or x = 9
7x – 3 (–3x – 2) = –26 The integers are 9, 11, 13.
Now the equation contains only one variable, so Age problems—which usually involve a comparison
applying the distributive property, we have of ages at the present time, or several years later—can
7x + 9x + 6 = –26 be solved easily with algebra.
16x = –26 – 6
16x = –32 Quadratic Equations
32
If the terms of an equation contain squares of the
x = = –2 unknown as well as linear terms, the equation is called
16
Taking any of the given equations, we can substitute quadratic. Some examples of quadratic equation are
the value of x : x2 + 4x = 3,
3x + y = –2 2x2 = 2x,
3 × (–2) + y = –2 and
a + b = a2 + 6
–6 + y = –2
To solve a quadratic equation:
y = –2 + 6 = 4
● Group all the terms on one side of the equation
So, x = –2, y = 4
so that the other side is zero.
In the elimination method, we multiply both ● Combine the terms on the non-zero side.
equations by such numbers so as to make the coefficients ● Factor the expression into linear expressions.
of one of the unknown variables numerically the same. ● Set the linear factors equal to zero and solve.

6 . 76
Basic Numeracy and Data Interpretation

The method depends on the fact that if a product ● Algebraic expressions containing terms created by
of expressions is zero then at least one of the expressions the product of constants and variables are called
must be zero. polynomials. A single term is called a monomial.
Take an example: solve x2 + 5x = –4 Examples of monomials: 2, 3x, –4bc, and 33x2y.
x2 + 5x + 4 = –0 Examples of polynomials: a–b, 4x–3y, a2+2a–4, etc.
x2 + 5x + 4 = (x + 4) (x + 1) = 0 ● The term like terms are polynomial terms containing
So x = –4 and x = –1 exactly the same variable(s) and exponent(s); e.g.,
3x and 4x; 7xy and –6xy; 8x2y and –10x2y.
● An equation is a mathematical sentence that states
Points to Note that two expressions are equal.
● A quadratic equation will usually have 2
● When solving an equation, the goal must be to
different solutions, but a quadratic may also
isolate the variable.
have only one solution or elude a solution
altogether. ● When solving an equation, these steps need to be
● Certain principles of inequalities should also
followed:
be kept in mind. It is an axiom that if a and (i) Simplify each side of the equation separately.
b are two rational numbers then either (ii) Move the variable to one side of the equation.
a = b
(iii) Perform the inverse operations of either
or a > b addition or subtraction.
or a < b
(iv) Perform the inverse operations of multi-
Also the following are true:
plication or division.
@UPSC_THOUGHTS

(i) If a < b and b < c, then a < c


(ii) If a < b then a + c < b + c (v) Answer may be checked by substituting the
(iii) If a < b and c > 0, then ac < bc value of the variable into the original equation.
(iv) If a < b and c < 0 then ac > bc ● If factoring is not possible in a quadratic equation
(v) The product of xa and yb is maximum such as in the form of ax2 + bx + c = 0, use the
(provided x + y = constant) when formula
x a
.
b b 2 . 4ac
x =
y b 2a
However, if there is any property of inequalities
that you are unable to remember, try out some
specific numbers in the given expressions. PROBLEMS ON AGES

Algebraic expressions and equations come in handy in


Some Facts about Algebraic Expressions solving problems on ages. The following solved
and Equations examples will help you understand the method. Just be
● An algebraic expression contains numbers, variables, careful to get the equations right.
and operations to state a relationship. An equation ● Tipu’s grandfather was 8 times older to her 16
is two algebraic expressions set equal to each other.
years ago. He would be 3 times of her age 8 years
An equation thus contains an equal (=) sign, but
from now. Eight years ago, what was the ratio of
an expression does not.
Tipu’s age to that of her grandfather?
● Numbers, in algebra, are called constants.
Solution: Let Tipu’s (T’s) age 16 years ago be x
● Letters used to represent unknown quantities are
years.
variables.
Her grandfather’s (G’s) age was 8x years.
● A number that comes in front of a variable (that is
connected by multiplication) is known as a 8 years from now, T will be (x + 16 + 8) years
coefficient. = (x + 24) years

6 . 77
Basic Numeracy and Data Interpretation

and G will be (8x + 16 + 8) years ● One year ago, the ratio of Gaurav’s and Sachin’s
= (8x + 24) years age was 6 : 7 respectively. Four years hence, this
We are given that G will be 3 times ratio would become 7 : 8. How old is Sachin?
T’s age 8 years from now. Solution: Ratio merely means that if Gaurav’s age
∴ (8x + 24) = 3 (x + 24) was 6x, Sachin’s age was 7x one year ago.
Or 8x – 3x = 72 – 24 Gaurav’s age 4 years from now = (6x + 1) + 4
48 = (6x + 5) years
Or 5x = 48, or x =
5 (Remember, it was 6x one year ago)
We have to find the ratio of their ages 8 years ago. Sachin’s age 4 years from now
48 = (7x + 1) + 4 = 7x + 5 years
8
T x 8 5 After 4 years the ratio of the ages will become
G 8x 8 48 7 : 8.
8 8
5
Gaurav’s age 4 years hence 7
88 11 ∴ :
= Sachin’s age 4 years hence 8
424 53
The ratio of Tipu’s age to her grandfather’s age 8 6x 5 7
Or
years ago was 11 : 53 7x 5 8
Or 8 (6x + 5) = 7 (7x + 5)
● The present ages of three persons are in proportions
Or 48x + 40 = 49x + 35
@UPSC_THOUGHTS

4 : 7 : 9. Eight years ago, the sum of their ages was


Or x = 5
56. Find their present ages (in years).
Sachin’s present age is (7x × 5 + 1) = 36 years.
Solution: Let their present ages be 4x, 7x and 9x.
(Don’t forget to add the ‘1’, as 7x refers to Sachin’s
By the given conditions,
age one year ago.)
(4x – 8) + (7x – 8) + (9x – 8) = 56
Or 20x = 56 + 24 ● The ratio between the present ages of P and Q is
6 : 7. If Q is 4 years older than P, what will be
80
Or x = = 4 the ratio of the ages of P and Q after 4 years?
20
∴ The present ages of the three persons are Solution: Let P’s age and Q’s age be 6x years and
(4 × 4), (7 × 4) and (9 × 4) 7x years respectively.
Or 16 years, 28 years and 36 years Then, 7x – 6x = 4
Or x = 4
● The ages of two persons differ by 16 years. If 6 ∴ Required ratio = (6x + 4) : (7x + 4)
years ago, the elder one be 3 times as old as the = 28 : 32
younger one, find their present ages. = 7 : 8

Solution: Let the younger person’s age be x years. ● A is two years older than B who is twice as old
The older person will then be (x + 16) years. as C. If the total of the ages of A, B and C be 27,
By the given conditions, then how old is B?

3 (x – 6) = (x + 16 – 6) Solution: Let C’s age be x years


Then, B’s age = 2x years
Or 3x – 18 = x + 10
A’s age = (2x + 2) years
Or 2x = 28
∴ (2x + 2) + 2x + x = 27
Or x = 14
Or 5x = 25
The younger person is at present 14 years old,
while the older person is (14 + 16 =) 30 years old. Or x = 5

6 . 78
Basic Numeracy and Data Interpretation

different choices for the first place, 5 for the second


PERMUTATION AND COMBINATION
place, and 4 for the third. So the number of arrangements
Basics are 6 × 5 × 4 = 120.]
Questions are often asked on the lines, ‘in how many A combination is the total number of groupings of
ways can seven children be seated at a round table’ or a set of objects. Order is not important with combinations.
‘in how many ways can 3 balls be picked up of a group The grouping of objects changes the number of
of 10 and kept in a row’. One can get the answer to combinations that exist.
such questions without actually writing down all the The formula for the number of combinations (C) of
different possibilities. Some basic principles and n objects taken r at a time is
formulae will have to be understood and remembered.
A permutation of objects is the number of possible n! n(n 1) (n 2)...(n r 1)
Cr =
arrangements for that set of objects. If 5 different bottles n
r ! (n r )! r(r 1) (r 2)...3.2.1
are to be arranged on a shelf, there are 5 choices for
the first spot on the shelf, 4 choices for the second spot, Note that nCn will always be 1, as there is only one
3 choices for the third spot, 2 choices for the fourth spot, way to select all the numbers of a group.
and only 1 left for the fifth spot. The total number of
Formulae to Remember
arrangements for those 5 bottles can be represented by
an operation: 5 ! = 5 × 4 × 3 × 2 × 1 = 120. This is Multiplication principle of counting If an operation can
known as 5 factorial. be performed in m different ways, following which a
The factorial of a whole number n is the product second operation can be performed in n different ways,
of that number and each of the natural numbers less the two operations in succession can be performed in
@UPSC_THOUGHTS

than the number. m × n different ways.


It is written as n or n ! ● How many possible outcomes are there if a coin
= n × (n–1) × (n–2) × … × 1 is tossed four times?
In other words, the number of permutations (P) of
Solution: 2 × 2 × 2 × 2 = 16
n objects taken n at a time is
● There are 4 entrances and 3 exists to a hall. In
n
Pn = n!
how many ways can one enter into and exit
(n Pn or nPn mean the same.)
from the hall?
At times, all the objects are not considered for each
Solution: 4 × 3 = 12
different arrangement. In such a case, n represents the
total number of objects from which one has to choose, Addition principle of counting If an operation can
and r is the number actually chosen to be arranged in be performed in m different ways and another operation,
a particular way. In this case, independent of the first one, can be performed in n
n! different ways, then either of the two operations can be
n
Pr =
(n r )! performed in (m + n) ways.
Take an examples ● There are 8 doors in a room two on one side
Q. There are 6 runners in a race. How many and 6 on another. If a man has to go out of the
different orders are there for the first, second, and third room, he can do so in 8 ways. He can go out
places? of 2 doors on one side and out of 6 doors on
n! another, the total being (6+2) = 8 ways.
Solution: Use the formula nPr =
(n r )!
6! 6! ● C0 = nCn = 1
n
P =
6 3
(6 3)! 3! ● C1 = n
n

6 5 4 3 2 1 ● If nCx = nCy, either x = y or y = n – x or


= 120
3 2 1 x + y = n
[There is a faster way. There are 3 different places ● Cr + nCr–1 = Cr
n n+1
(first, second and third) to be considered. There are 6

6 . 79
Basic Numeracy and Data Interpretation

● Number of combinations of n different things Solution: Here n = (5 + 3 + 2) = 10,


taken r at a time p = 5, q = 3; r = 2
(i) When p particular things are always Different arrangements possible
included
10 !
= n–pCr–p =
5 ! 3 ! 2!
(ii) When p particular things are not included 10 9 8 7 6 5 4 3 2 1
= n–pCr = = 2520
5 4 3 2 3 2 2
(iii) When p particular things are not together ● In how many ways can 5 toy cars be distributed
in any selection among 4 boys, there being no restriction on the
= nCr – n-pCr-p number of toy cars a boy gets?
● Number of selections of r consecutive things out Solution The required number of ways = 45 = 1024
of n things in a row
= n–r + 1 ● A question paper has two parts I and II, each
● Number of selections of r consecutive things out containing 10 questions. If a candidate has to
of n things in a circle choose 8 from Part I and 5 from Part II, in how
= n, when r < n; many ways can the questions be selected?
= 1, when r = n n!
Solution: Using the formula nCr = , from
● Number of ways of dividing m + n different r ! (n r )!
Part I, the choice will be
things in two groups containing m and n things,
respectively, when m n 10! 10 9 8 7 6 5 4 3 2 1 10 9
@UPSC_THOUGHTS

8! (10 8)! 8 7 6 5 4 3 2 1 (2 1) 2
m+n
Cm = (m n)!
m!n! From Part II, the choice will be
● The number of permutations of n things, taken
all at a time, out of which p are alike and of 10! 10 9 8 7 6 5 4 3 2 1
one type and q are alike and of one type, and 5! (10 5)! 5 4 3 2 1 (5 4 3 2 1)
the rest are different 10 9 8 7 6
n! 5 4 3 2 1
=
p!q !
Total number of choices will be
● The number of permutations of n different things
taken r at a time when each thing may be 10 9 10 9 8 7 6
repeated any number of times is nr. 2 5 4 3 2 1

● Number of arrangements of n different objects = 5 × 9 × 3 × 2 × 7 × 6 = 11,340


in a circle
● In an examination paper, there are two parts, I and
= (n – 1) ! II, each containing 4 questions. A candidate has
● Number of arrangements of n different things in to attempt 5 questions in all but not more than 3
a circle when clockwise or anticlockwise questions from any one part. In how many ways
arrangements are not different (i.e., when can the questions be selected?
observations can be made from both sides) Solution: In this case, the sum of choices from each
1
= (n–1) ! part is 3 Qs from I and 2 Qs from II or 2 Qs from
2 I and 3 Qs from II.
Worked Examples One set of choices are 4C3 × 4C2
● There are 5 green, 3 black and 2 white balls in a 4! 4! 4! 4!
bag. They are all drawn out one by one at random 3! (4 3)! 2!(4 2)! 3! 1! 2! 2!
and arranged in a row. How many possible 4 3 2 1 4 3 2 1
arrangements are there? 4 6
3 2 1 1 2 1 2 1

6 . 80
Basic Numeracy and Data Interpretation

Another set of choices are 4C2 × 4C3 take them together, we may consider them as one
4! 4! entity. So there are 4 letters to arrange:
6 4
2!(4 2)! 3!(4 3)! JDG (UE), which can be done in
Total number of choices = 4C3 × 4C2 + 4C2 × 4C3 4! = 24 ways.
= 4 × 6 + 6 × 4 However, the vowels, UE, can be arranged among
= 24 + 24 = 48 themselves in 2 ways. So the required number of
ways is
● How many different arrangements of 5 students
24 × 2 = 48
can be made in a row of 3 desks?
● From a group of 7 men and 6 women, five persons
Solution: Since the order is important here, it is a
are to be selected to form a committee so that at
permutation problem. Arranging 5 students in 3
least 3 men are there on the committee. In how
desks will be as
many ways can it be done?
P = 5 × 4 × 3 = 60
5 3
Solution: There are three ways in which there can
● How many 3-digit numbers can be formed from the
be at least 3 men on the committee of 5 : 3 men
digits 2, 3, 5, 6, 7 and 9, which are divisible by + 2 women; 4 men + 1 woman; or 5 men.
5 and none of the digits is repeated? There are 7 men and 6 women.
Solution: We need to consider units, tens and So the number of ways will be
hundreds place for a 3-digit number. If the number 7
C3 6
C2 7
C4 6
C1 7
C5
is to be divisible by 5, the unit place of the number
@UPSC_THOUGHTS

must be 5. So there is only 1 way of getting the 7 6 5 6 5 7 6 7


= C3 C1 C2
unit place right. 3 2 1 2 1
In the tens place, any of the other five numbers can
7 6 5 7 6
be placed: so there are 5 different ways of filling = 525 6
3 2 1 2 1
the tens place.
The hundreds place can now be filled with any of = (525 + 210 + 21) = 756
four digits: so there are 4 different ways of filling
the place.
PROBABILITY
∴ So the number of 3-digit numbers from the given
digits will be (1 × 5 × 4) = 20 Basics
Ordinarily speaking the probability of an event denotes
● In how many different ways can the letters of the
the likelihood of its happening. If an event is certain
word ‘RUMOUR’ be arranged?
to happen its probability would be 1 and if it is certain
Solution: The given word has 6 letters: that the event would not take place, then the probability
R-2, U-2, M-1 and U-1 of its happening is 0. Therefore, probability is always
So one can arrange the letters in measured between 0 to 1.

6! 760 Certainty of Event Percentage Probability


(2 !) (2 !) (1 !) (1 !) 4 = 180 ways Will occur 100% 1
Will not occur 0% 0
● In how many different ways can the letters of the
Chance to occur 25% ¼
word ‘JUDGE’ be arranged in such a way that the
Chance to occur 50% ½
vowels always come together?
Solution: While the word has 5 letters, the given Probability is used to predict the outcomes of
condition is that vowels always come together. random experiments. Random experiments are those
There are two vowels, U and E, and if we always experiments in which results are not, however,

6 . 81
Basic Numeracy and Data Interpretation

essentially same even though the conditions may be (iii) the odds in favour of event E are p : q or
nearly identical. A few examples of random experiments p : (1–p)
are: drawing a card from a pack of playing cards; (iv) the odds against the event E are q : p or
tossing a coin; selecting an odd number from a given (1–p) : p
set of numbers; throwing a pair of dice; and drawing Thus, according to this theory, the probability of
a ball of a particular colour from a bag having balls getting the number 6 in a single throw of a dice. (or
of different colours. 1
p) is and the probability that this event will not take
To perform a random experiment is called a trial 6
5
and the outcome of the trial is called an event. place (or q = 1–p) is .
6
Probability of an event denotes the likelihood of its
happening. The general rule of the happening of an Theoretic Notations of Events
event is that if an event can happen in m ways and Let A and B be two events of a random experiment
fail to happen in n ways the probability of the happening whose sample space is 5. We have observed that the
of the event, events and the relations among them can be best
m described by using concepts of ‘set theory’. The following
p
m n table gives the equivalent forms of probability statements
and the corresponding statements in the set theory.
or number of cases favourable to the event divided by
the total number of ways in which the event can
Probability theory Set theory
happen or divided by the number of exhaustive cases.
How the number of cases favourable to the event or how Sample space S
the number of exhaustive cases should be computed is Outcome or a sample point
@UPSC_THOUGHTS

a matter on which opinions differ. Event A A S


Event A has occurred A
Measuring Probability Event A has not occurred A
There are different ways of measuring probability. Event A implies event B A B
Classical Method When the event (E) occurs in m Event ‘not A’ AC
different ways out of a total number of n possible ways, Event ‘A or B’ A B
Event ‘A and B’ A B
all of which are equally likely, then the probability of
m Event ‘A but not B’ A – B = A BC
the event P(E) is . Events A and B are A B =
n
Frequency Method If an event (E) is repeated n mutually exclusive
number of times (n being very large) and it occurs only
m Types of Events
m times, the probability of the event, P(E) is . Let us get clear about the types of events.
n
For example, when a dice is rolled once, we have 1. Mutually exclusive events Two events A and B
6 equally likely sample points, 1, 2, 3, 4, 5 and 6 and are called mutually exclusive if they do not
1 occur simultaneously, i.e., if A B = . For
hence the probability of each of these 6 outcomes is ,
6 example, the events A = {1, 2, 4} and B = {3, 5,
i.e.,
1 6} associated with the experiment of rolling a
P(1) = P(2) = P(3) = P(4) = P(5) = P(6) =
6 dice are mutually exclusive.
Similarly, if E is the event, ‘a number multiple of 2. Complementary events Two events where it is
2 1 certain that one or the other will occur, but not
3 shows up’, then E = {3, 6} and hence P(E) =
6 3 both the events together are complementary
If the probability of an event E associated with a events. For example, if a single coin is tossed,
random experiment is ‘p’ (0 p 1), i.e., P(E) = p, the event ‘heads’ and the event ‘tails’ are
then complementary.
(i) the chance that event E happens is p 3. Compatible events Two events when they
(ii) the chance that event E fails to happen is q happen simultaneously are called compatible
or 1–p events.

6 . 82
Basic Numeracy and Data Interpretation

Probability Theorems Otherwise also if 13 cards of Spade and 4 Kings


Important theorems of probability are as follows: were added, we would have subtracted 1 from the
total (as the thirteen cards of Spade already contain
(i) Addition theorems (a) If A and B are two
the Spade King) and thus the total number of
mutually exclusive events associated with a random
favourable cases would have been 13 + 4 – 1 or
experiment, then
16. Since the total number of ways in which a card
P(A or B) = P(A) + P(B)
can be drawn is 52, the required probability of
i.e., P(A B) = P(A + B) = P(A) + P(B) 16 4
drawing a Spade or a King would be or .
(b) If A and B are any two events associated with 52 18
a random experiment, then (ii) Multiplication theorems (a) If A and B are two
P(A or B) = P(A) + P(B) – P(A and B) events associated with a random experiment such that
i.e., P(A B) = P(A) + P(B) – P(A B) P(A) 0 and P(B) 0, then
(c) Thus if the probabilities of N mutually exclusive B
events are P1, P2, P3, Pn then the probability that one of P(A B) = P(A) P
A
these events will happen would be
A
P1 + P2 + P3 + ... Pn P(A B) = P(B) P
B
The theorem of addition discussed above will not
(b) If A and B are two independent events, then
be applicable when the events are not mutually exclusive
P(A B) = P(A) . P(B)
(where two or more events can take place together). The
addition formula in such cases then takes the following P( A B) P( A) . P( B)
@UPSC_THOUGHTS

shape. P( A B) P( A) . P( B)

P (A or B) = P (A) + P (B) – P (AB) P( A B) P( A) . P( B)

If there are three such events which overlap, the [Note: P( A) 1 P( A) and P(B) 1 P( B)]
probability ● A box of fuse contains 20 fuses, of which 5 are
P (A or B or C) = P (A) + P (B) + P (C) – P (AB) defective. If 3 of the fuses are selected at random
– P (AC) – P (BC) + P (ABC) and removed from the box without replacement,
what is the probability that all three fuses would
● If from a pack of cards a single card is drawn what
be defective?
is the probability that it is either a Spade or a King?
Solution: Suppose A, B and C are the events that
13 all the three fuses are defective.
Solution: The probability of a Spade card =
52 5 B 4 C 3
4 P( A) ,P ,P
The probability of a King = 20 A 19 A B 18
52
But in the Spade card also there will be a King. 5 4 3 1
.
P( A B C)
As such if we add the above two probabilities we 20 19 18 114
will be counting the Spade King twice. Therefore
Worked Examples
the answer would be
Probability of a Spade or a King ● If two coins are tossed 4 times, then what is the
probability that we get heads on both the coins?
13 4 13 4
52 52 52 52 Solution: Result can be indicated as
17 1 16 4 (Head–H, Tail–T)
52 52 52 13 (TT, HT, TH, HH)
So, the probability of having heads on both the
Note the probability of a card being both (A) and coins is only 25%.
(B) is P (A) × P (B). In the above case the value is
1 ● If E1 is the event “drawing an ace” from a deck
.
52 of cards and E2 is the event “drawing a king”, then

6 . 83
Basic Numeracy and Data Interpretation

4 1 4 1 (a) without replacement


P (E1) = = and P (E2) = = . What (b) with replacement
52 13 52 13
is the probability of drawing either an ace or a king
Solution: (a) If the first card is not replaced before
in a single draw?
the second card is drawn, then probability of
Solution: P (E1+E2) getting aces in succession is
4 3 1
= P (E1) + P (E2) = 0.0045
52 51 221
1 1
= + (b) If the first card is replaced before the second is
13 13
2 drawn, the probability of getting aces is
= 4 4 1
13
= 0.0059
52 52 169
● If the probability that A will be alive in 20 years
is 0.7 and the probability that B will be alive in ● Three coins are tossed 8 times (HHH, HHT, HTH,
20 years is 0.5, then what is the probability that THH, HTT, THT, TTH and TTT). If A is the event
they both will be alive?
that a head occurs on each of the first two coins,
Solution: P(A) = 0.7 B is the event that a tail occurs on the third coin,
P(B) = 0.5 and C is the event that exactly two tails occur in
The probability that they will both be alive in 20
the three coins, show that
years is (0.7) (0.5) = 0.35
(a) events A and B are independent
A consumer research organisation has studied the
@UPSC_THOUGHTS


(b) events B and C are dependent
services under warranty provided by 50 new car
dealers in a certain city, and its findings are as Solution: A = 2 times {HHH, HHT}
given below: B = 4 times {HHT, HTT, THT, TTT}
Time Good service Poor Service C = 3 times {HTT, THT, TTH}
under warranty under warranty [As given in result of tosses: HHH, HHT, HTH,
THH, HTT, THT, TTH and TTT]
In business 10 16 4
years and more A  B = {HHT}
B  C = {HTT, THT}
In business less 10 20
According to the above information we get:
than 10 years
1 1 3
P(A) = , P(B) = , P(C) =
If a person randomly selects one of these new car 4 2 8
dealers, what is the probability that he will get one 1 1
P ( A  B) = and P ( B  C ) =
who provides good service under warranty? 8 4

n(G) 16 10 (a) Probability that events A and B are independent


Solution: P(G) = = = 0.52 is
n(S) 50 1 1 3
P A B P A .P B
Here P(G) = Probability of good dealer 4 2 8
giving warranty (b) Probability that events B and C are dependent
n(G) = Number of elements in G is
n(S) = Number of elements in whole 1 3 3
P(B C) P ( B ) . P (C )
sample space 2 8 16
3
● Find the probability of randomly drawing two aces But here P(B C)
16
in succession from an ordinary deck of 52 playing
∴ Events B and C are not independent.
cards, if we sample

6 . 84
Basic Numeracy and Data Interpretation

● A speaks truth in 75 per cent cases and B in 80 If the terms of a sequence follow a certain pattern,
per cent of the cases. In what percentage of cases the sequence is known as a progression. Each of the
are they likely to contradict each other, narrating following sequences is a progression.
the same incident? (i) 2, 4, 6, 8, 10, … , 24.
(ii) 2, 4, 8, 16, 32, … , 1024.
Solution:
(iii) 2, 4, 16, 256, 65536, …
Let A = Event that A speaks the truth
and B = Event that B speaks the truth In (i) each term after the first is formed by adding
2 to the preceding term; each term in (ii) is multiplied
75 3 80 4
Then, P (A) = , P (B) = by 2 to form the next term; each term in (iii) is a square
100 4 100 5
of the preceding term. Further, while (i) and (ii) are
3 1 4 1 finite—the last term is given, (iii) is infinite as it can
∴ P ( A) 1 and P (B ) 1
4 4 5 5 go on indefinitely.
P (A and B contradict each other)
= P [(A speaks the truth and B tells a lie) or Arithmetic Progression
(A tells a lie and B speaks the truth)] Arithmetic Progression (A.P.) is a sequence whose terms
increase or decrease by a fixed number, and the fixed
= P [(A and B ) or ( A and B)]
number is called the common difference of the A.P.
= P (A and B ) + P ( A and B) Usually in an A.P., the first term is denoted by a,
= P (A) . P ( B ) + P ( A ) . P (B) the common difference by d, and the nth term by tn.

3 1 1 4 3 1 (i) d = tn – tn–1
=
(ii) The general term of an A.P. is given by
@UPSC_THOUGHTS

4 5 4 5 20 5
tn = a + (n–1) d
7 7
= 100 % = 35% (iii) If an A.P. has n terms, the nth term is called
20 20
the last term (l) of the A.P.:
∴ A and B contradict each other in 35% of the
l = a + (n–1) d
cases.
(iv) If a is the first term and d the common
difference of an A.P. of m terms, the nth term
PROGRESSIONS from the end is the (m–n+1)th term from the
beginning. Therefore,
Basics nth term from the end = a + (m–n) d
The concept of sequence and series comes into use in (v) If Sn is the sum of n terms of an A.P. whose
matters such as calculating premiums on life insurance first term is a, and the last term is l,
and instalments to be paid on loans. n
(a) Sn = (a + l)
A sequence is a function whose domain is the set 2
(N) of natural numbers and range, a subset of real n
(b) Sn = [2a + (n–1) d] … d being the
numbers or complex numbers. Here we are dealing with 2
real sequences only. common difference.
● The different terms of a sequence are usually Geometric Progression
denoted by a1, a2, a3, … or by t1, t2 … (the
Geometric Progression (G.P.) is a sequence of non-
subscript, 1, 2, etc., denoting the position of the
zero numbers in which every term, except the first
term in the sequence).
one, bears a constant ratio with its preceding term.
● The number at the nth place in a sequence, i.e., This ratio is called common ratio of the G.P. The
tn is known as the general term of the sequence. sequences below are examples of G.P.
● A sequence is finite if it has a finite number (i) 3, –6, 12, –24, 48, …
of terms, and infinite if the number of terms is
(ii) b1, b2, b3, b4, … (b being a fixed real number)
infinite.

6 . 85
Basic Numeracy and Data Interpretation

In (i) the ratio of every term to its preceding term A line is understood to be extending without end
is –2. in both directions. Between any two points only one
[3 (×–2=) –6, (×–2=) 12, … ] line segment is possible.
In (ii) it is b. [b×b = b2; b2×b = b3; b3×b = b4, … ] ● Two lines are parallel  if they do not intersect
The first term and common ratio of a G.P. are no matter how far they are extended.
usually denoted by a and r respectively.
● If two lines are parallel to a third line, then they
(i) The general term tn = arn–1
are parallel to each other.
(ii) If the G.P. is a finite one with m terms, the nth
term from the end is given by arm–n. ● Two different lines cannot have more than one
(iii) The nth term from the end of a G.P. with last point of intersection.
l An angle, ∠ is the figure formed by two lines
term l is
rn 1
meeting at a point.
(iv) The sum of the first n terms of a G.P.
a (r n 1)
Sn =
r 1
Harmonic Progression
Harmonic Progression (H.P.) is a sequence of non-zero
1 1 1 x
numbers a1, a2, a3, … provided the sequence , , , ...
a1 a2 a3
is in A.P.
(i) The nth term of an H.P. The vertex of the angle is B, the point where
1
the two sides meet. The above angle will be
@UPSC_THOUGHTS

= denoted ∠ABC or ∠x.


nth term of the corresponding H.P.
When two straight lines intersect, four angles
(ii) No term of an H.P. can be zero. are formed. If these angles are equal, each one
Worked Examples is a right angle, i.e., equal to 90°. A right angle
● Find the nth term and 17th term of the sequence is represented by the symbol L. An angle less
5, 2, –1, –4, … than a right angle is an acute angle. An angle
greater than a right angle is an obtuse angle.
Solution: In this A.P., first term a = 5, and the If two sides of an angle extend in opposite
common difference d = –3 directions forming a straight line, the angle is
tn = a + (n–1) d = 5 + (n–1) (–3) = –3n + 8 a straight angle which contains 180°.
For the 17th term, n = 17. So, ● Two adjacent angles are complementary if their
t17 = –3(17) + 8 = –51 + 8 = 43 sum is 90°; they are supplementary if their sum
is 180°.
● Find the sum of 16 terms of an A.P. whose first If two lines are parallel and a third line intersects
term is 3 and the last term is 50. them, eight angles are formed.
Solution: a = 3, l = 57, n = 16
n
Sn = (a + l)
2
16
Sn = (3 + 57) = 8(60) = 480
2

GEOMETRY

Lines and Angles


A point has no size. All geometric figures consist of In the given figure
points. A line consists of points. However, lines can be (i) ∠4 = ∠6 and ∠3 = ∠5
measured. Alternate interior angles are equal.

6 . 86
Basic Numeracy and Data Interpretation

(ii) ∠1 = ∠5, ∠2 = ∠6, ∠3 = ∠7, ∠4 = ∠8 ● If a polygon has n sides, it can be divided into
Corresponding angles are equal n – 2 triangles.
(iii) ∠3 + ∠6 = 180°; ∠4 + ∠5 = 180° ● The sum of all the angles in a polygon is (no.
of sides – 2) × 180°.
● Interior angles on the same side of the
transversal are supplementary.
Triangle
The reverse is also true. A triangle is a closed, three-sided figure, the sum of
● If a pair of alternate interior angles are equal; its three angles being 180°.
if a pair of corresponding angles are equal; if
● If a triangle has sides of different lengths, it is
a pair of interior angles on the same side of the
transversal are supplementary; then the two a scalene triangle.
lines cut by a transversal are parallel. ● If a triangle has two equal sides it is an
● If two lines intersect to form right angles, the isosceles triangle. In such a triangle the angles
two lines are perpendicular to each other. opposite the equal sides are also equal.
● If two lines in a plane are perpendicular to the ● If a triangle has all three sides equal, it is an
same line, then the two lines are parallel. equilateral triangle. Each angle in such a triangle
is of 60°.
● A triangle which has a right angle is a right
triangle. In such a triangle, the two acute angles
are complementary. The side opposite the right
@UPSC_THOUGHTS

angle is the hypotenuse and is the longest side.

AD ⊥ CD, BP ⊥ CD, so AO  BP ● Within any triangle, the largest side is opposite


the largest angle; the smallest side is opposite
Polygon the smallest angle; the sum of the lengths of any
A polygon is a closed figure in a plane that is made two sides must be longer than the remaining
of line segments meeting at end points only. A polygon side.
may have three or more sides. We are here taking up
● The measure of an exterior angle to a triangle
only ‘convex’ polygons, i.e., a polygon in which the
is equal to the sum of the two remote interior
measure of each interior angle is less than 180°.
angles of the triangle.
A triangle is a 3-sided polygon; a quadrilateral is
a 4-sided polygon, and so on. A
● A polygon has as many sides as it has angles.

F B B D
C

In the above triangle, ∠ACD = ∠ABC + ∠BAC


E C

D The Pythagorean Theorem states that in a right


● A, B, C, D, E and F in the above polygon are triangle, the square of the hypotenuse is equal to
its vertices. the sum of the squares of the other two sides. In
a ABC where AC is the hypotenuse,
● A polygon is known as regular if all its sides are
AB2 + BC2 = AC2.
equal and all its angles are equal.

6 . 87
Basic Numeracy and Data Interpretation

● Two triangles are congruent if The corresponding sides of the two triangles
(i) two pairs of corresponding sides and the are proportional:
corresponding included angles are equal: DE AE AD
= =
BC AC AB

Also, if a line-segment joins the midpoints of


two sides of a triangle, it is parallel to the third
side and measures half of it.
● In a right triangle, the two acute angles are
complementary, that is, they add up to 90º.
(ii) two pairs of corresponding angles and the ● In a right-angled triangle ABC right angled at
corresponding included sides are equal: 1
A, if ∠B = 60°, then AB = BC, the hypotenuse.
2

(iii) all three pairs of corresponding sides of


two triangles are equal: ● The sum of the lengths of any two sides of a
@UPSC_THOUGHTS

triangle must be larger than the length of the


third side.

Quadrilaterals
A quadrilateral is a closed figure of four sides or a
polygon of four sides. The sum of its four angles is 360°.
Quadrilaterals may be of several types:
(iv) any two corresponding sides of a right
triangle are equal, the third sides are equal ● Parallelogram is a quadrilateral in which both
pairs of opposite sides are parallel and equal.
and the triangles congruent.
Opposite angles of a parallelogram are also
equal.
A diagonal divides the parallelogram into two
equal triangles.
The diagonals bisect each other, i.e., intersect at
each other’s midpoint.
● Two triangles, are similar if all three pairs of If all the sides of a parallelogram are equal, the
corresponding angles are equal. figure is a rhombus. The diagonals of a rhombus
In the figure below, if DE  BC, then Δ ADE is are perpendicular to each other.
similar to Δ ABC. ● A rectangle has all the properties of a
A parallelogram besides having its four angles
equal to right angles. (If all angles of a
quadrilateral are equal, then the figure is a
D E rectangle.)
The diagonals of a rectangle are equal and
B C bisect each other.

6 . 88
Basic Numeracy and Data Interpretation

● A square is a rectangle with all its four sides Secant: a line which intersects a circle in two
of equal size. (If a rhombus has all its sides distinct points. PQ is a secant line.
equal, it is a square.) Arc: any part of the circumference.
● A trapezoid is a quadrilateral with two parallel
sides and two sides which are not parallel. The Points to Remember
parallel sides are the bases; the non-parallel ● A diameter divides the circle into two equal halves
sides are called legs. called semicircles.
An isosceles trapezoid is a trapezoid with non- ● A line from the centre which is perpendicular to
parallel sides of equal lengths. Also, the two a chord bisects the chord and vice versa.
pairs of angles are congruent; and the diagonals ● The radius from the centre to the point where a
are congruent. tangent touches the circle is perpendicular to the
tangent.
B C
A B

A D O
Y X
BC  AD AB = CD
@UPSC_THOUGHTS

∠ABC = ∠BCD; ∠BAD = ∠CDA Z


AC = BD
● In the above figure, ∠AOB is a central angle and
Circles is equal to the number of degrees contained in its
intercepted arc. ∠XYZ is an inscribed angle and
A circle is a closed plane curve, all points of which are
it has half the number of degrees of arc XZ.
equidistant from a fixed point within called the centre.
A complete circle contains 360°. ● Two chords equidistant from the centre are equal
in length. Conversely, two equal chords are
equidistant from the centre.
B C
● Angles in the same segment of a circle are equal.
E
d

radius ● The angle in a semicircle is a right angle.


or
ch

A O Conversely, a circle drawn with the hypotenuse


centre
ter
me of a right triangle as diameter passes through its
d ia
D secant Q opposite vertex.
P tangent
F
X Y

Chord: a line segment connecting any two points


on a circle. AB is a chord.
Radius: a line segment connecting the centre with
any point on the circumference. OC is the radius.
Diameter: a chord passing through the centre of the ● In equal circles or the same circle, equal chords cut
circle. DE is the diameter. It is always twice the radius. off equal arcs. Conversely, if two arcs subtend
Tangent: a line touching a circle at one and only equal angles at the centre (or at the circumference),
one point. XY is a tangent to the circle at F. the arcs are equal.

6 . 89
Basic Numeracy and Data Interpretation

B A B
O P A

O
E C
A D D
C D
B C ● If two chords AB and CD intersect either inside
or outside the circle (when produced at a point
If ∠AOB = ∠CPD, If arc AB = arc CD,
E) then AE × BE = EC × ED.
arc AB = arc CD and ∠AOB = ∠COD and
● A tangent at any point of a circle is perpendicular
vice versa vice versa
● Equal chords of a circle subtend equal angles at to the radius through the point of contact.
the centre. Conversely, if the angles subtended by Conversely, a line drawn through the end of a
two chords at the centre of a circle are equal, the radius and perpendicular to it, is a tangent to the
chords are equal. circle.
● The angle subtended by an arc of a circle at the A

centre is double the angle subtended by it at any B


A D
point on the remaining part of the circle.
E
D C E
B D
C
C

O ● If two tangents are drawn from an external


@UPSC_THOUGHTS

point to a circle, they subtend equal angles at


the centre. They are also equally inclined to the
line segment joining the centre to that point. The
A B
lengths of two such tangents are equal.
Arc AB subtends ∠AOB at the centre and ∠ADB
and ∠ACB on the circle A
∠AOB = 2∠ADB
∠AOB = 2∠ACB
● If two circles touch, their point of contact lies on O B AB = BC
the line that joins their centres.

P P
C

A
A ● A cyclic trapezium is isosceles and its diagonals
O1 O2 O2 O1
are equal.
Q Q

S R
A, O1, O2 are collinear.
● The sum of a pair of opposite angles of a cyclic
quadrilateral (a quadrilateral within a circle whose
vertices lie on the circumfernce of the circle) is 180°.
P Q
Also, the sum of the opposite pair of sides are
equal. If PQ  SR,
∠ABC + ∠ADC = 180° PS = QR and PR = QS
∠ADE = ∠ABC Also, an isosceles trapezium is always cyclic.
AB + CD = AD + BC Also if two opposite sides of a cyclic quadrilateral

6 . 90
Basic Numeracy and Data Interpretation

are equal, the other two sides are parallel. We are aware that x + w = 180°, and
We know that y + z + w = 180°
Worked Examples So, x + w = y + z + w, and x = 4 + z
Geometry problems occur frequently. If you are not
● In the triangle ABC; if AC= 6, AD = 5 and
provided with a diagram, draw one for yourself.
BC = 10, then AB is
Think of any condition which will help you answer
the question. It may help to draw some diagonals,
altitudes, or other auxiliary lines in your diagram.
● In the figure below, AB is the diameter and
x
OC = BC. What is the value of ?
2

Solution: Using AB2 + AC2 = 2(AD2 + BD2)

Solution: As OC = BC and OC and OB are radii, AB2 + 36 = 2 (25 + 25)


BOC is an equilateral triangle AB2 = 100 – 36 = 64
AB = 8
Angle BOC = 60°
x ● In the figure, AB=BC and angles BAD and BCD are
∴ x = 120° and = 60° right angles.
@UPSC_THOUGHTS

2
● In the triangle below DC is paralled to FE,
AD = DF, DC = 4 and DF = 3. What is FE?

Which one of the following conclusions may be


drawn?
Solution: If angles BAD and BCD are right angles,
Solution: Since CD is parallel to EF, the triangles they are equal. Angle BAC = angle BCA as they
are base angles of an isosceles triangle. Subtracting
ACD and AEF are similar. Therefore,
equals from equals, angle DAC = angle DCA.
corresponding sides are proportional. So CD is
AD 1 Therefore ACD is an isosceles triangle, and
to EF as AD is to AF. Since AD = DF, is . AD = CD.
AF 2
Therefore EF is twice CD or 8. ● Consider the figure below:
● In the figure below which statement can be true?
(AB is parallel to CD) A B
A y B AC = 6
x O C
z D
C
Solution: We extend the lines to give the figure: What is the area of the circle O?
Solution: The figure ABCO is a rectangle. If AC,
one of its diagonals is 6, OB, its other diagonal is
also 6. OB is also the radius of circle O. So area
of the circle is 6 × 6 × π or 36π

6 . 91
Basic Numeracy and Data Interpretation

● Suppose you have to find the distance between


COORDINATE GEOMETRY
two points Q (5, 2) and R (–2, –4).
Basics and Some Formulae The given distance formulae is
In coordinate geometry, there are two perpendicular d = ( x a )2 ( y b )2
lines in the plane intersecting at a point called the plane If Q’s coordinates is (x, y) and R’s is (a, b)
origin. The horizontal line is the x-axis and the vertical We have
one, the y-axis. The origin corresponds to zero.
QR = [5 ( 2)]2 [2 ( 4)]2
y
= (5 2)2 (2 4)2

Quadrant II Quadrant I = 49 36
x negative x positive
y positive y positive QR = 85
● The distance of a point P (x, y) from the origin
O 2 2
x O (0, 0) is given by OP = x y
Quadrant III Quadrant IV The section formula says that the coordinates
x negative x positive of point P (x, y) which divides a line segment
y negative y negative
joining two points (x1, y1) and (x2, y2), internally,
in the ratio m1 : m2 are
m1 x2 m2 x1 m1 y2 m2 y1
,
@UPSC_THOUGHTS

To the right of origin x-axis is positive; to the left m1 m2 m1 m2


of origin it is negative. Positive numbers on the x-axis Also, if the ratio in which P divides the line
are above the origin; negative numbers on the y-axis are segment is k : 1, the coordinates of P will be
below the origin.
An ordered pair of coordinates has the general form kx2 x1 ky2 y1
,
(a, b). The first element always refers to the x-axis, while k 1 k 1
the second element gives the value on y-axis. In the 1
Area of a triangle is base × altitude. The
following figure, 3, 2; –3, 2; –3, –2; and 3, –2 are ordered 2
pairs of coordinates. area (A) of a triangle whose vertices are (x1, y1),
(x2, y2) and (x3, y3) is given by
y
1
A = x1 y 2 y 3 x2 y 3 y1 x3 y1 y2
2
● If the area of the triangle is 0 square units,
(–3, 2) (3, 2) then its vertices will be collinear.
Knowing how to find the area of a triangle,
x one can find the area of other polygons.
O

(–3, –2) (3, –2)


MENSURATION: AREA AND VOLUME

Perimeter, Area, Volume


The perimeter of a two-dimensional (plane) figure
is the distance around the figure, or the total length
The formula for finding the distance (d) of two of the sides enclosing the figure.
points, say coordinates (x, y) and (a, b) is The area of any figure is the amount of surface
within its bounding lines. Area is expressed in square
d = ( x a)2 ( y b)2 units.

6 . 92
Basic Numeracy and Data Interpretation

For solids or three- yotta, (Y), meaning 1024 deci, (d), meaning 10-1
dimensional figures, it is the zetta, (Z), meaning 1021 centi, (c), meaning 10-2
surface area that is taken into exa, (E), meaning 1018 milli, (m), meaning 10-3
account. peta, (P), meaning 1015 micro, (u), meaning 10-6
A solid is a figure bounded tera, (T), meaning 1012 nano, (n), meaning 10-9
by one or more surfaces; it has giga, (G), meaning 109 pico, (p), meaning 10-12
three dimensions, namely, length, mega, (M), meaning 106 femto, (f), meaning 10-15
breadth (or width), and height kilo, (k), meaning 103 atto, (a), meaning 10-18
(or thickness). The plane surfaces hecto, (h), meaning 102 zepto, (z), meaning 10-21
that bind the object are its faces. deka, (da), meaning 101 yocto, (y), meaning 10-24
The area of the plane surfaces or
faces of the solid is called its Units of Length
surface area. The surface area of 10 millimetres (mm) = 1 centimetre (cm)
any three-dimensional figure is 10 centimetres = 1 decimetre (dm) = 100 millimetres
the sum of the areas of its faces. 10 decimetres = 1 metre (m) = 1000 millimetres
The volume of a three- 10 metres = 1 dekametre (dam)
dimensional figure is the amount 10 dekametres = 1 hectometre (hm) = 100 metres
of space enclosed within its 10 hectometres = 1 kilometre (km) = 1000 metres
bounding faces. It is measured in
cubic units, e.g., cubic Units of Area
100 square millimetres (mm2 ) = 1 square centimetre (cm2)
centimetres (cc) or cm3; m3, etc.
100 square centimetres = 1 square decimetre (dm2)
@UPSC_THOUGHTS

Tables of Metric Units of 100 square decimetres = 1 square metre (m2)


Measurement 100 square metres = 1 square dekametre (dam2) = 1 are
100 square dekametres = 1 square hectometre (hm2) = 1 hectare (ha)
In the metric system of
100 square hectometres = 1 square kilometre (km2)
measurement, designations of
multiples and sub-divisions of
Units of Liquid Volume
any unit may be arrived at by
10 millilitres (mL) = 1 centilitre (cL)
combining with the name of the
10 centilitres = 1 decilitre (dL) = 100 millilitres
unit the prefixes deca, hecto, and 10 decilitres = 1 litre1 = 1000 millilitres
kilo meaning, respectively, 10, 10 litres = 1 dekalitre (daL)
100, and 1000, and deci, centi, 10 dekalitres = 1 hectolitre (hL) = 100 litres
and milli, meaning, respectively, 10 hectolitres = 1 kilolitre (kL) = 1000 litres
one-tenth, one-hundredth, and
one-thousandth. In some of the Units of Volume
following metric tables, some 1000 cubic millimetres (mm3) = 1 cubic centimetre (cm3)
such multiples and sub-divisions 1000 cubic centimetres = 1 cubic decimetre (dm3)
have not been included for the = 1 000 000 cubic millimetres
reason that these have little, if 1000 cubic decimetres = 1 cubic metre (m3)
any currency in actual usage. = 1 000 000 cubic centimetres
In certain cases, particularly = 1 000 000 000 cubic millimetres
in scientific usage, it becomes Units of Mass
convenient to provide for 10 milligrams (mg) = 1 centigram (cg)
multiples larger than 1000 and 10 centigrams = 1 decigram (dg) = 100 milligrams
for sub-divisions smaller than 10 decigrams = 1 gram (g) = 1000 milligrams
one-thousandth. Accordingly, the 10 grams = 1 dekagram (dag)
following prefixes have been 10 dekagrams = 1 hectogram (hg) = 100 grams
introduced and these are now 10 hectograms = 1 kilogram (kg) = 1000 grams
generally recognized: 1000 kilograms = 1 megagram (Mg) or 1 metric ton(t)

6 . 93
Basic Numeracy and Data Interpretation

Two-Dimensional Figures For an isosceles right-angled triangle,


As all the figures considered here are also in the Perimeter = c 2 1
Geometry section, it would be useful to read that section c
where c is the hypotenuse.
and recall the special features of each figure. a
c = a 2
1. Triangle
a2
A triangle is bounded by three sides. Area = a
2

b 2. Quadrilateral
c
h A quadrilateral is a figure bounded by four sides.
D C
a p1
(i) For any triangle, perimeter (P) = a + b + c p2

1
or P = 2s where s = (a + b + c)
2
A B
1 1
Area = base × height = ah ● Perimeter = Sum of the 4 sides
2 2 1
● Area = d (p1 + p2)
or s ( s - a) ( s b ) ( s c ) 2
(where d is a diagonal and p1 and p2 are
(ii) For equilateral triangle, perimeter (P) = 3a
perpendiculars to that diagonal from opposite
@UPSC_THOUGHTS

(All angles and all sides are equal angles = 60°


vertices)
each.)
● If the lengths of the four sides and one diagonal
3 3 2
Area = side2 a are known,
4 4
3 3 Area = Area of Δ ABC + Area of Δ ADC
Altitude (h) = side a For cyclic quadrilateral (i.e., quadrilateral whose
2 2 a a
h2 h vertices can be on the circumference of a circle)
Area =
3
● Area = s (s a) (s b) (s c) (s d)
a
(iii) For isosceles triangle,
a b c d
Perimeter (P) = (2a + b) where s
2
(Two sides are equal.)
● The sum of the angles = 2π
1
Height (h) = 4 a2 b 2
2 3. Parallelogram
b In the given figure, AB = DC = base b
Area = 4 a2 b 2
4 AD = BC = a
(iv) For right-angled triangle
D
c2 = a2 + b2 C
Or c = a2 b2
a h d
(c being the hypotenuse or side opposite the
right angle and also the largest side)
Area = base into perpendicular
A B
c base (b)
a
● Perimeter = 2(a + b)
where a and b are adjacent sides of the
b parallelogram.

6 . 94
Basic Numeracy and Data Interpretation

7. Trapezium
● Area = 2 s (s a) (s b) (s d)
where a and b are adjacent sides, d is the a
a b d
diagonal and s
2 d
c h
● The sum of the squares of the diagonals
d12 d22 = 2(a2 + b2)
b
4. Rectangle
l 1
D C ● Area = h (a + b)
2
where a and b are the parallel sides.
b d b
a b
Or Ax
x
A B (if all four sides are known but h is not known)
l
● Perimeter = 2(l + b) where Ax = s (s c) (s d) (s x)
● If one side and diagonal are given, (x = b – a if b > a
perimeter = 2(l d2 l2 ) and a – b if a > b
● Area = l × b c d x
and s = )
If one side and diagonal are given, area 2
@UPSC_THOUGHTS

= (l d2 l2 ) or b d2 b2 8. Regular Polygon
A polygon is a figure bounded by four or more
● If perimeter (P) and diagonal are given, area straight lines. It is called ‘regular’, when all its
P2 d2 sides and angles are equal.
=
8 2
a
● Diagonal = l2 b2
a
5. Rhombus
a
Perimeter = 4a a
or 2 d12 d22 d1
1 a a
Side = d12 d22 h
2 a
d2 ext. a
Area = a × h angle
1
or d × d2
2 1 a ● Sum of exterior angles = 2π
6. Square
a ● Sum of interior angles = (n–2)π
Perimeter (P) = 4 a
or 2d 2 (n being the number of sides)
Area = a2 n (n 3)
a a ● Number of diagonals =
2 2 d d 2
d P
or or ● Perimeter = n × a
2 16
n 2
Length of d = 2 a ● Each interior angle θ =
P a n
or 2 area or 2
2 2 ● Each exterior angle =
[Height of a square is the same as its side.] n

6 . 95
Basic Numeracy and Data Interpretation

1 1 AxB is the minor segment.


● Area = r or n a r
2 2 AYB is the major segment.
(where r is the radius of the inscribed circle Area of circle = AxB + AYB
or circle drawn inside the polygon touching its Area of minor segment
sides) = Area of sector – Area of triangle AOB
● Area of a regular hexagon (six-sided regular
3 3
Fast-Track Formulae
polygon) = (side)2 ● If the length of a rectangle is increased by x% and
2
● Area of a regular octagon (eight-sided regular the same area were to be maintained, the breadth
polygon) = 2( 2 1) (side)2 100x
would have to be decreased by %.
100 x
9. Circle ● If the length and breadth of a rectangle are
A circle is a plane figure bounded by one line increased by x% and y% respectively, the area of
such that the distance of this line from a fixed xy
the rectangle will increase by x y %.
point within it remains constant throughout. 100
● Perimeter is known as the circumference (c )
● If any defining dimension or side of any plane
in the case of a circle, and c = 2πr or πd (r figure (triangle, quadrilateral, circle, etc.) is
is radius and d is diameter), O is the centre x
changed by x%, its area changes by x 2 %.
of the circle and r is the constant distance 100
from O to the circumference. ● If the sides of any plane figure are changed by
r x%, its perimeter also changes in the same way
d2
Area = πr2 or
@UPSC_THOUGHTS

● d (i.e., increases or decreases) by x%.


4 O ● If all sides of a quadrilateral are increased or
c2 decreased by x%, its diagonals also increase or
or
4 decrease by x%.
area circumference ● If the diagonal of a square increases by x times,
● Radius r = or
2 the area of the square becomes x2 times.
● A sector is the figure enclosed by two radii and ● If the ratio of the area of two squares be a : b,
an arc between those radii. the ratio of their other dimensions (side, diagonals,
A etc.) each will be in the ratio a : b .
r ● If the length and breadth of a rectangular area
l O are l and b, and a carpet of width w is used to
O
cover the area, the carpet length required will
B
1 l b
Area of the sector AOB = lr be .
2 w
(where l is the length of the arc) ● If the length and breadth of a rectangular area
r 2  are l and b, the least number of square tiles
or
360  required to cover the area will be
l b
● A segment of a circle is bounded by a chord
(H.C.F. of l and b )2 and the size of the largest tile
and the corresponding arc.
if the tiles are to fit exactly will be H.C.F. (l, b).
Y ● If a path of w units surrounds a rectangular area
of length l units and breadth of b units, the area
O of the path = 2w (l + b + 2w) sq. units
r r (measurements being in the same unit).
r
A B ● Ratio of the area of a circle
x circumscribing a square to the area of
2
6 . 96 a circle inscribed in the square is .
1
Basic Numeracy and Data Interpretation

● Ratio of the area of a square


circumscribing a circle to the area of G
2
a square inscribed in the circle is .
1
B F
● If the area of a square is a sq. cm, the area of
a circle of same perimeter as that of the square a
C
4a
is sq. cm.
A E
● The area of the largest circle that can be inscribed
a a
a2
in a square of side a is .
4 D
● The area of a square inscribed in a circle of radius Diagonals are BE, AF and GD.
r will be 2r2. Length, breadth and height are all same = a
● A circular wheel of radius r travelling a distance ● Diagonal (d) = a 3
d
d will make revolutions. ● Total surface area = 6 (edge)2 = 6a2 or 2d2
2 r
3 3
d surface area
3
● Volume = a or or
Three-Dimensional Figures 3 6
While area is in square units, volume is in cubic units. For two cubes:
(i) Ratio of volumes = (ratio of sides)3
1. Rectangular Solid
@UPSC_THOUGHTS

(ii) Ratio of surface areas = (ratio of sides)2


A rectangular solid has six rectangular faces and
equal opposite rectangles. 3. Prism
A prism is a three-dimensional solid with two
G F congruent top and bottom faces, and any number
of other faces that are all rectangles.
h
B C
E
H
b
A
l D

Diagonals are BE, AF and GD,


l, b and h are the length, breadth and height of
the figure.
● Diagonal (d) = l2 b2 h2
● Total surface area = 2(lb + bh + lh)
● In a prism of n sides, number of vertices = 2n
● Volume = l × b × h = A1 A2 A3 where and number of faces = n +2
● Lateral surface area
A1 = area of base or top
= perimeter of base (or top) × height
A2 = area of one side face
● Total surface area
A3 = area of other side face = 2 × base area + lateral surface area
● Volume = area of base × height
2. Cube
A cube is a three-dimensional figure in which 4. Cylinder
each of the six faces is a square. So all faces are A cylinder (a right circular one) is bound by two
equal. The length, breadth and height of a cube congruent circular ends with one rectangular
are equal and are called the edges of the cube. face wrapped around these ends.
6 . 97
Basic Numeracy and Data Interpretation

● Area of curved surface = O vertex


circumference of base ×
height = 2πr × h edge slant
● Area of total surface = area height ( l)
of curved surface + area of
side
circular ends face
= 2πrh + 2πr2
= 2πr (h + r) sq. units
● Volume = area of base × X
height base ( b)
= πr2h cubic units
b (base side)

5. Cone ● Slant surface area


A right circular cone is obtained by rotating a 1
right-angled triangle around its height. = base perimeter × slant height
2
1
Given r is radius of base, h is = l × b × n (n = number of sides)
2
height and l is slant height,
● l = h2 r2 Ellipse
An ellipse is a closed symmetrical curve like an
● Area of curved surface = πrl
elongated circle—the higher the eccentricity, the
sq. units greater the elongation. Any chord through the centre
@UPSC_THOUGHTS

● Total surface area = area of is a diameter. The ellipse has two diameters that
base + area of curved surface are axes of symmetry: the longest diameter is the
major axis and the shortest the minor axis. A line
= πr2 + πrl = πr (r + l) sq. units
segment from the centre to the ellipse along the
1 2 major axis is a semi-major axis; one along the minor
● Volume = πr h cubic units
3 axis is a semi-minor axis. Each of the two points
A cone with a portion of its top cut off is called at which the major axis meets the ellipse is a vertex
the frustum of the original cone. of the ellipse.
The area of an ellipse is πab, where a is the
Given that R is radius of the length of the semi-major axis and b the length of
base of the frustum, r is the the semi-minor axis.
radius of the top of the frustum,
h is height and l is slant height, Ellipsoid
Ellipsoid is a closed surface such that its plane
● l = h2 (R r )2 units sections are ellipses or circles. The centre of the
● Area of curved surface of ellipsoid is its centre
c
of symmetry, and any b
frustum = π (R + r) l sq. units
chord through the
● Total surface area of frustum
centre is a diameter
= [( R2 r 2 ) l( R r)] sq. units of the ellipsoid. Three a
● Volume of frustum of these chords are
h 2 axes of symmetry—
= (R r2 Rr ) cubic units
3 as with the ellipse,
the largest is the
6. Pyramid major axis and the smallest the minor axis. The
A pyramid is a three-dimensional solid with a third axis, perpendicular to the other two, is the
regular polygon (of 3 or more sides) as its base mean axis. The semi-major (a), semi-minor (b), and
and triangles for its other faces meeting at a semi-mean (c) axes are defined as for the ellipse.
vertex. In a right pyramid, the foot of the The volume of an ellipsoid is 43 πabc.
perpendicular from the vertex is to the midpoint
of the base.
6 . 98
Basic Numeracy and Data Interpretation

● Whole surface area = base area + total slant 1


Area of the triangle = × b × h
surface area (base area depends on shape of 2
1
base polygon) Area of the triangle = × 3 × 4 = 6
1 2
● Volume = base area × h 1 1
3 Area of semicircle = area of circle = π r2
(OX is the altitude = h) 2 2
1
Radius = × 4 = 2
7. Sphere 2
A sphere is a solid contained 1
Area = π × 2 × 2 = 2π
by one curved surface, such 2
that all points on it are Total area = 16 + 6 + 2 π = 22 + 2 π
equidistant from a fixed point r
● Find the area of the shaded region if every circle
within it, namely the centre.
is of unit radius.
It is formed by revolving a
semicircle on its diameter. The
midpoint of the diameter is called the centre of
the sphere, and the radius of the semicircle is the
radius of the sphere.
● Surface area of sphere = 4πr2
4 3
● Volume = r Solution: It may be deduced that, as the circles
3
touch, the lines passing through the points of
● A zone of a sphere
contact will pass through the centre of the circle.
@UPSC_THOUGHTS

is the part be-


If we join the centres, we have an equilateral
tween two paral- h
triangle of sides 2 units each (as we are given
lel planes. If the
r radius of circle is unit).
sphere has radius
r, and the distance
between the
2 2
planes is h, the
area of the curved surface of the zone is 2πrh.
● A hemisphere is a half-sphere: part of a 2
sphere cut off by a plane through its centre.
It is thus a zone of one base with an altitude As the angles subtended at each centre by the
equal to the sphere’s radius. equilateral triangle will be 60°, the sector formed
60º 1 2
in each case will be πr πr . The area of the
2

Worked Examples 360 6


shaded portion is the area of the triangle – the area
● Find the area of the following figure:
of the three sectors.
3 2
Area of triangle = × 2 (area of equilateral
4
triangle being 3 side2)
4
3 1
Area of shaded portion = × 4 – 3. (since r = 1)
4 6
3 1 1
Solution: You have to find the area of three figures— = 4 3 units
4 2 2
a square of side 4; a semicircle of diameter 4 (lower
● Poornima is standing 180 metres due north of
side of square); a right angled triangle with legs
point P. Fatima is standing 240 metres due west
3 and 4 (right side of the square).
of point P. What is the shortest distance between
Area of the square = a2 = 42 = 16 Fatima and Poornima?

6 . 99
Basic Numeracy and Data Interpretation

Solution: A quick sketch of the information Its side = 36 = 6 cm


provided shows that we need to employ the
∴ Its perimeter = 6 × 4 = 24 cm
Pythagorean Theorem.
● A man walked diagonally across a square lot.
Poornima N
Approximately, what was the per cent saved by
not walking along the edges?
h
180 W E Solution:

Fatima C
240 B
P S

The shortest distance from Poornima to Fatima


is the hypotenuse of this right triangle.
2 2 2 D A
180 + 240 = h
As 180 and 240 are multiples of 60 (3 × 60 = Let the side of the square be x metres
180; 4 × 60 = 240), it is clear that there is a right Then, AB + BC = 2x metres
triangle of sides 3, 4, 5. AC = 2x = (1.41x) m
Hypotenuse is 5 × 60 = 300 metres Saving on 2x metres = (0.59x) m
0.59 x
● The diameter of a wheel is 20 cm. If it rolls Saving % = 100 % = 30% (approx.)
forward making 10 revolutions, what distance 2x
does it travel?
@UPSC_THOUGHTS

● Consider the volumes of the following:


Solution: Radius of the wheel = 10 cm 1. A parallelopiped of length 5 cm, breadth 3
The distance covered by the wheel in 10 cm and height 4 cm
revolutions 2. A cube of each side 4 cm
= 10 × 2πr 3. A cylinder of radius 3 cm and length 3 cm
= 10 × 2 × 3.14 × 10 = 628 (taking π = 3.14) 4. A sphere of radius 3 cm
State the volumes of these in the decreasing
● The length and breadth of a rectangle are order.
increased by 20% and 10% respectively. By what
per cent does its area increase? Solution:
Solution: Percentage increase in area Volume of parallelopiped = (5 × 3 × 4) cm3 = 60 cm3
xy Volume of cube = (4)3 cm3 = 64 cm3
= x y %
100
22
20 10 Volume of cylinder = 3 3 3 cm3
= 20 10 % 7
100
= 84.86 cm3
= (20 + 10 + 2)% = 32%
4 22
● The perimeters of two squares are 40 cm and 32 Volume of sphere = 3 3 3
3 7
cm. Find the perimeter of a third square whose
area is equal to the difference of the areas of the = 113.14 cm3
two squares. So in descending order : 4, 3, 2, 1
Solution: ● A cube of edge 15 cm is immersed completely
40 in a rectangular vessel containing water. If the
Side of Ist square = = 10 cm
4 dimensions of the base of vessel are 20 cm × 15
cm, find the rise in water level.
32
Side of 2nd square = = 8 cm Solution: The water level will increase in the
8
Area of 3rd square = 102 – 82 = 100 – 64 same volume as the volume of the cube, which
= 36 cm2 is
6 . 100 (15 × 15 × 15) cm3
Basic Numeracy and Data Interpretation

Volume position of one ray into the position of the other ray
Rise in water level = is called an angle.
Area
15 15 15 45
=
20 15 10

de

e
id
si

ls
al

a
in
= 11.5 cm

it i
rm

In
Te
e
● A hemispherical bowl is filled to the brim with iv
e iv
it at
os eg
a beverage. The contents of the bowl are P N
O (a) Initial side O
transferred into a cylindrical vessel whose radius (b) Terminal side

is 50% more than its height. If the diameter is


same for both the bowl and the cylinder, the Rotation of ray Angle measure
volume of the beverage in the cylindrical vessel Anticlock wise Positive
is: Clockwise Negative
Solution:
x Measurement of Angles by Sexagesimal System
If x is the height of the vessel, will be the
2 (degrees, minutes and seconds)
radius (r) of the vessel (which is also the radius
1
of the bowl. 1 degree = 1o = right angles
90
3
2 x 1
@UPSC_THOUGHTS

Volume V1 of the bowl = x3 1


3 2 12 1 minute = 1’ = degree
60

x
2
1 1
Volume V2 of the vessel = x x3 1 second = 1’’ = minute
2 4 60
or other way
V2 is greater than V1; so the vessel can contain
90o degree = 90o = 1 right angle
all or 100% of the beverage in the bowl.
60 minutes = 60’ = 1o
60 seconds = 60” = 1’
HEIGHTS AND DISTANCES
Measurement of Angles by Circular System
(Basic Trigonometry) (Radians)
Radian is the angle subtended by an arc at the centre
Trigonometry is a Greek word, having three words
of a circle whose length is equal to the radius of the
in itself. Trio means thrice, Gonia means angle and
circle. One circular measure is denoted by 1c. The S.I.
Metron means measure. So the word meaning of
symbol for a radian is rad. 
trigonometry simply put is “measurement of triangles”
In the given figure AO = OC = AC = r.
(sides and angles).
So, AOC  1c
Trigonometry and Heights and Distances AOC = one radian

Trigonometry helps to calculate the distance between


the points and the heights of the objects, without
measurements. For solving such problems, generally
A r B
angle of elevation and angle of depression are used.
O
Measuring Angles r
r
When the two rays (initial and terminal) unite together
at a point with the rotation in a plane to bring the C

6 . 101
Basic Numeracy and Data Interpretation

Thus, the length of the arc = radius × angle subtended Sign of Trigonometrical Ratios
by the arc at the centre in radians
Given in the figure, y
arc ACB r
AOB = = =  radians
r r
IInd Quadrant Ist Quadrant
 radians = 180o = 2 right angles.
180 x' x
1 radian = degree = 57o17’44.8’’(approx)

IIIrd Quadrant IVth Quadrant
Sectorial Area Let ABC be a sector having central angle
 and radius r. y'

B
Ist Quadrant (0    90o ) = All ratios are positive.
IInd Quadrant (90o    180o ) = sin θ and cosec θ are
positive.
IIIrd Quadrant (180o    270o ) = tan θ and cot  are
A C positive.
IVth Quadrant ( 270o    360o ) = cos  and sec  are
1 2
The area of sector ABC = r  positive.
2
Trigonometric Ratios of Complementary Angles
Trigonometrical Ratios
Two angles are said to be complementary if their sum
is 90 degree and each one of them is complement of
@UPSC_THOUGHTS

P the other, e.g., 47 and 43 (47+43) = 90


(x, y)
C
(9
0–
r y )


B A
O x M o o
For 0    90
AB
In a right-angled triangle OPM, sin (90 ) = = cos 
AC
Perpendicular MP y BC
(i) = = = sine θ cos (90 ) = = sin 
Hypotenuse OP r AC
AB
Adjacent side OM x tan (90 ) = = cot 
(ii) = = = cosine θ BC
Hypotenuse OP r
BC
Perpendicular MP y cot (90 ) tan
(iii) Adjacent side = = = tangent θ AB
OM x
AC
Hypotenuse OP r sec (90 ) = = cosec θ
BC
(iv) = = = cosecant θ
Perpendicular MP y AC
cosec ( 90 ) = = sec 
Hypotenuse OP r AB
(v) Adjacent side = = = secant θ
OM x Also sin 0º cos 90º , sin 90º cos 0º
Adjacent side OM x tan 0º cot 90 º , sec 0º cosec90º , etc.
(vi) = = = cotanget θ
Perpendicular MP y
Trigonometrical Identities
[Abbreviations normally used are sin, cos, tan, An equation involving trigonometric ratios of an
cosec, sec, cot.] angle  , which is true for all values of  for which

6 . 102
Basic Numeracy and Data Interpretation

the given trigonometric ratios are defined, is called observer’s eye and the line joining the object and the
a trigonometric identity. eye is called the angle of depression.
P
O B
(eye) Angle of depression
r
Y
(object)
A A
x M
For any acute angle 
2 2
2 2  y x r2 Working Rule
(i) sin   cos         2 = 1 Step I : Understand the problem first.
r r r
Step II : Make a clear figure of the problem and mark
2 2
(ii) 1  tan 2   1        sec2 
y r all its dimensions.
 x  x Step III : Choose right angle triangle to solve the
2 2 problem.
x r
(iii) 1 cot 2 1 cosec 2 Step IV : If the problem comprises of more than one
y y
triangle, then solve one by one.
Step V : Use correct trigonometric ratio
Angles and Trigonometrical Ratios
unknown side
i.e.,
Angle of Elevation known side
When an object is higher than the observer’s eye, the = known angle of the triangle.
angle between the horizontal line passing through
@UPSC_THOUGHTS

The trigonometrical ratios for right angled triangles


the observer’s eye and
A discussed earlier are useful in determining the height
line joining the object (object) of a distant object or the distance between two far-off
and the eye is called
objects.
the angle of elevation. Angle of elevation
(eye) B For triangles (other than right angled)
Angle of Depression O
When an object is lower than the observer’s eye, the a b c b2  c2  a 2
We can use   or cos A 
angle between the horizontal line passing through the sin A sin B sin C 2bc

Values of Trigonometric Ratios


    3
Angle  0º 300 or 6 450 or 600 or 900 or 1800 or  2700 or  3600 or 2 
4 3 2 2

1 1 3
sin 0 1 0 –1 0
2 2 2
3 1 1
cos 1 0 –1 0 1
2 2 2
1
tan 0 1 3 0 0
3
2
cosec 2 2 1 -1
3
2
sec 1 2 2 –1 1
3
1
cot 3 1 0 0
3

6 . 103
Basic Numeracy and Data Interpretation

Worked Examples Solution:


● A kite is flying with a thread 250m long. If the B
thread is assumed to be stretched straight and D
makes angle of 55° with the horizontal then find
the height of the kite above the ground. h
h

Solution: 60º 30º


A C
B 100 m P x

Let AB and CD be the two towers


0m In ABP
25 h
AB h
tan 60   3
55°
AP 100
C A
h = 100 3 m
AB
In BAC we have  sin 55
CB In CPD
h CD 1 100 3
= sin55° = 0.8192 tan 30 =  =
250 CP 3 x
h = 250  0.8192 x = 100 3  3
= 204.8m
= 300m
● If a 30m ladder is placed against a 15m wall such  AC = Distance between the towers
that it just reaches the top of the wall, what is the = 100+300
@UPSC_THOUGHTS

elevation of the wall?


= 400m
B
Solution: ● Two ships leave a port at the same time. One
In BCA sails at 30km/h in the direction N 32°E while
AB 15 other sails at 20km/h in the direction S 58°E.
sin   
m

BC 30
30

15m After 2 hours how far are the ships from each
1
 sin  other?
2
  30 Solution:
C A
Let A and B be the two ships. After 2 hours
● If the height of a pole is 2 3 m and the length of AB = x
its shadow is 2m, what is the angle of elevation AOB = 180° – (58°+32°)
of the sun? = 90°
Q N A
Solution: OA = 2 × 30 = 60km
In QRP OB = 2 × 20 = 40km
PQ 2 3 In OAB 32°
tan    2 3
RP 2 2
AB = OA + OB2 2 58°
W E
tan   3  tan 60 2 2
O 32°
x = (60)  ( 40)
  60 R 2m P 58°
= 3600  1600
● An observer measures angle of elevation of two
towers of equal heights from a point between = 5200
the towers. If the angle of elevation are 60° and S B
30° and distance of the nearer tower is 100m = 4  100  13
what is the height of each tower and distance
= 2  10 13
between the towers, respectively?
= 20 13 km

6 . 104
Basic Numeracy and Data Interpretation

There are two bases of logarithms: one is base ‘e’


LOGARITHMS
(e = 2.71828 approx.) and the other is base ‘10’. The
logarithms to base e are called natural logarithms and
Order of Magnitude and Logarithm
the logarithms to base 10 are called common logarithms.
When we make rough estimates, comparisons or
calculations, we sometimes round off a number to Properties of Logarithms
zero—the nearest power of 10.
A number rounded to the nearest power of 10 is (i) loga(pq) = logap + logaq
called an order of magnitude. If, for example, we take  p
the average height of a human to be about 1.7 metres, (ii) loga   = log p – log q
a a
q
we may round off 1.7 metres to the nearest power of
(iii) logaa = 1
10, which is 100 m (or 1 m). This does not mean that
the average height of a person is a mere 1 metre, but (iv) loga1 = 0
rather the average height is closer to 1 metre (or 100 (v) loga(pq) = qlogap
metres) than it is to 10 metres (or 101 metres).
(vi) logap × logpa = 1
Similarly, rounding the height of an ant, which is about
8 × 10–4 metres, to the nearest power of ten results in 1
10–3 metres. log p a
We can also say that the order of magnitude of the logb p log p
(vii) logap = =
height of an ant is 10-3 metres. Now, if we compare the logb a log a
height of a human being (100 metres) with the height (Note that the old base a is converted to the new
of an ant (10–3 metres), we come up with the ratio of base b.)
@UPSC_THOUGHTS

human height to ant height = 100/10–3 = 100 – (–3) = 103


(viii) alogap = p
= 1000. So we may conclude that a human being is
about 1000 times (or 103 times) taller than an ant. In (ix) When the base is not given, it is taken as 10.
other words, a human being is 3 orders of magnitude (x) Negative numbers and zero have no logarithms.
(3 powers of 10) taller than an ant.
To be an order of magnitude greater is to be 10 times Characteristic and Mantissa of Logarithm
as large. The logarithmic scale is used to measure such The characteristic of a number is the integer which is
differences in order of magnitude. raised as power of 10 when it is written in standard
An order of magnitude difference between two form. For example,
values is a factor of 10. For example, the mass of the
32.4 = 3.24 × 101 is in standard form. Any positive
planet Saturn is 95 times that of Earth, so Saturn is two
decimal number n can be written in the form n = m × 10x,
orders of magnitude more massive than Earth. Order of
magnitude differences are called decades when where x is an integer and 1 ≤ m < 10.
measured on a logarithmic scale. This is called the standard form of the decimal n.

Logarithm The characteristic of a logarithm of a number greater


than one is one less than the number of digits in the
The word ‘logarithm’ was coined from two Greek
integral part of the number. It is positive.
words, logos which means a ‘ratio’ and arithmos,
meaning ‘number’. Number Digits in integral Characteristic
If a is a positive real number such that ax = b, then part
the exponent x is called the logarithm of b to the
base a. Symbolically, 243.72 243(3) 3–1 = 2
ax = b logab = x 78.345 78(2) 2–1 = 1
4932.751 4932 (4) 4–1 = 3
and it is read as ‘logarithm of b to the base a is x3.’
8.04 8(1) 1–1 = 0
For example, 20 = 1 log21 = 0
1 The characteristic of a logarithm of a number less
641/3 = 4 log644 =
than one is one more than the number of zeros
3

6 . 105
Basic Numeracy and Data Interpretation

immediately after the decimal. It is negative, but instead ● If log 3 = 0.4771, what is the number of digits in
of –1, –2, ..., we write 1 (one bar), 2 (two bar), and 356 ?
so on.
Solution:
356 = log 356
Number Number of zeros Characteristic
after decimal = 56 log 3
= 56 × 0.4771
0.00379 2 2+1 = 3
= 26.7176
0.043 1 1+1 = 2
0.128 0 0+1 = 1 ● What is the value of log10 4 100 ?
0.00064 3 3+1 = 4
Solution: log10 4 100
1
Characteristic is always an integer positive, negative 4
= log 10 100
or zero.
The mantissa is the decimal part of the logarithm 1
of a number. It is never negative and it always lies = log 10 100
4
between 0 and 1. For mantissa, we look through the log
table. 1
= log 10 10 2
Thus, to obtain log n we find characteristic and 4
mantissa of log n, and then add them. 2
= log 10 10 (as log10 10 = 1)
Applications of Logarithm 4
@UPSC_THOUGHTS

The invention of logarithms shortens calculations. It is 1


=
used in numerical calculations; calculating compound 2
interest; calculating population growth; and calculating log 8
depreciation of value. ● What is the value of ?
log 8
Worked Examples
1
● What is the value of log10 0.00001? log 8 log 8 2
Solution: =
Solution: log 8 log 8
1
Log10 0.00001 = log 10 1
100000 log 8
2 1
= log10 10–5 =
log 8 2
= –5 log10 10
= –5 × 1 a b
● If log log log ( a b) , which of the follow-
= –5 b a
ing is true?
12 log 10 10
● Find the value of (a) a + b = 1
2 log10 100
(b) a – b = 1
Solution:
(c) a = b
12 log 10 10
(d) a2 – b2 = 1
2 log 10 100
12 1 a b
= Solution: log log log ( a b)
2 log10 10 2 b a
a b
12 log ( a b) log log 1
= b a
2 2 log 10 10
∴ a + b = 1
12
= 3 The correct answer is (a).
4

6 . 106
Basic Numeracy and Data Interpretation

whether by itself it gives sufficient information to


DATA SUFFICIENCY
solve the problem. Be careful—you must disregard
the information in the other statements when you
Type and Strategies
consider one statement. If one statement gives
Data sufficiency questions are not found as such in
sufficient information, put a tick mark against it
school level examinations; they are, however, set in
and go on to consider the next statement. You may
competitive examinations which have mathematics (or find both (or all in case of more than two) statements
numeracy) as a subject. are by themselves sufficient to solve the problem.
Problems of this type require you to assess whether You need to then choose your answer correspond-
the given statements or data are sufficient to answer a ing to your conclusion.
certain question. You are not required to fully solve the
● If you find neither (or no) statement by itself gives
question to get the correct answer; you may stop as soon
you enough information to solve the problem, you
as you can tell that the data provided will enable you
may need to consider both or all statements together.
to solve the given question, or not, as the case may be.
They might, in combination, be enough to solve the
In data sufficiency items, there is a question followed given problem.
by two or more statements providing some data. It is
In case three or more statements are given, it is
possible that each statement by itself can help to solve
possible that only two in combination help to solve
the problem; or only one of the statements helps, the
the problem. You must pick your answer response
others is/are not of use; or both or all of them are
carefully to reflect the correct combination of
required to get the answer; or, may be, none can on its
statements.
own or in combination with the others help to get the
● You need to seek the answer to the given problem;
@UPSC_THOUGHTS

answer. In other words, the ‘sufficiency’ or the


the answer statement has to be sufficient to get a
‘insufficiency’ of the data is to be judged. And, generally,
single answer. If, for example, you are asked,
this can be done only if you are well-grounded in basic
‘what is the value of x’, and one statement says,
mathematical principles and formulae.
‘x = y + 3’, you do not have sufficient information
Data sufficiency questions are not longer confined
to get a specific value for x. (‘y + 3’ could give a
to mathematical knowledge alone; they often involve
wide range of values for x.)
analytical reasoning from given facts/statements of a
non-mathematical kind. These questions may involve ● If there are figures, remember they are not necessarily
drawn to scale; so if two sides of a quadrilateral
problems of relationships, comparisons or critical
look equal, do not assume they are equal. Just go
reasoning—i.e., assumptions behind statements/
by the information provided.
arguments or meanings, explicit or implicit.
We have already featured non-mathematical data Worked Examples
sufficiency items in the section, ‘Logical Reasoning and
In most questions relating to Data Sufficiency, the
Analytical Ability’. Here, we are concerned with items instructions include a set of answer responses from
mostly based on mathematical principles. Of course, which you have to choose the correct one for each
some knowledge beyond ‘maths’ is to be assumed— question. The options usually read:
such as number of days in the different months, meaning Give answer
of clockwise/anticlockwise, etc. (a) if the data in Statement I alone are sufficient
to answer the question, while the data in
Points to Remember Statement II alone are not sufficient to answer
● You do not have to solve the problem; so do not the question;
waste time trying to do so. You only need to (b) if the data in Statement II alone are sufficient
determine if the data provided is sufficient to solve to answer the question, while the data in
the given problem—for which you need to be well- Statement I alone are not sufficient to answer
versed in methods of solving various problems. the question;
● Whether you are given two statements or three, you (c) if the data either in Statement I or in Statement
first need to consider each statement and decide II alone art sufficient to answer the question;

6 . 107
Basic Numeracy and Data Interpretation

(d) if the data even in both Statements I and II Solution: I by itself does not lead us to the answer.
together are not sufficient to answer the question; In II an indefinite term such as ‘more than’ (which
(e) if the data in both Statements I and II together does not tell us exactly by how much) shows us that
are necessary to answer the question. it by itself does not give the answer. The two
If an examination paper format allows only four together also lead nowhere. [However, if II had said
answer options, one of the options mentioned above “equal to”, the data in both statements together
will be omitted. would have helped us reach the answer.] As it is,
I and II, taken single or together, do not provide
In other Data Sufficiency questions, each question
sufficient data to answer the question.
may carry its own set of options.
In either case, you need to read the options and be Question Type I
careful both while deciding upon the correct one and
when you mark it on the answer sheet. Directions: Each of the questions given below consists of
a statement and/ or a question and two statements numbered
● How much profit was earned by selling a product I and II given below it. You have to decide whether the data
for Rs 100? provided in the statements are sufficient to answer the
I. If the product had been sold for Rs 90, the profit question. Read both the statements and
would have been 20%. Give answer (a) if the data in Statement I alone are
II. The profit was one-third the cost price. sufficient to answer the question, while the data in Statement
Solution: II alone are not sufficient to answer the question;
100 Give answer (b) if the data in Statement II alone are
I. C.P. = 90 × = Rs 75
120 sufficient to answer the question, while the data in Statement
@UPSC_THOUGHTS

Now you can find the profit. So I is sufficient. I alone are not sufficient to answer the question;
x Give answer (c) if the data either in Statement I or in
II. x + = 100,
3 Statement II alone are sufficient to answer the question;
100 3 Give answer (d) if the data even in both Statements I
or x = = 75
4 and II together are not sufficient to answer the question.
Now profit can be found. So II is sufficient.
● What per cent of a group of people are women
Thus either statement by itself will provide the with blue eyes?
answer. I. Of the women in the group, 5 per cent have
● Is ABCD a rectangle? blue eyes.
II. Of the men in the group, 10 per cent have
blue eyes.
Solution: Unless the total number of people in
the group and the total number of women with
blue eyes are known, the problem cannot be
answered.
I. AB = CD
Now take I. It does not give the required
II. AC = BD
information. So it is not sufficient on its own.
Solution: Clearly either statement by itself is not Take II. The information given is irrelevant
enough to get the answer. Taken together, all we unless totals are given. So II alone is also
know is that the figure could be a square or a insufficient.
rectangle. So both statements even if taken together Now, we have to see if both taken together can
are not sufficient to answer the question. supply the answer. Neither statement gives
● What is the age of Zoya? information on the total numbers of people on
I. Aizo is 12 years younger than Zoya. women, so the question cannot be answered
II. Five years later Aizo will be more than half of with the data provided by statements I and II.
Zoya’s age. The correct answer response is (d).

6 . 108
Basic Numeracy and Data Interpretation

● If n is an integer, is n + 1 odd? compound interest without more information.


I. n + 2 is an even integer. So II is not sufficient.
II. n – 2 is an odd integer. Your answer is (a).
Solution: Take I first. If n + 2 is an even integer,
Question Type I: Variance in Answer Responses
n must be even. So n + 1 will be an odd integer.
The set of answer responses given in the directions may
I is sufficient. But you have to take up II as well. vary: the response (c) in Question Type I may read:
If n – 1 is an odd integer, n must be even. So
n + 1 would be an odd integer. Give answer (c) if the data in both Statements I and II
together are necessary to answer the question.
So I is sufficient on its own, and II is sufficient
on its own to answer the question. The other responses will remain as they are.

The correct answer response is (c). If this response is among the options, and you find
● What is the value of the two-digit integer x? that neither I nor II is sufficient by itself to solve the
I. The sum of the two digits is 3. problem, you should not be in a hurry to mark the
II. X is divisible by 3. response that neither is sufficient. You have to check
if both statements together can help you get the answer.
Solution: The digits will be from among the
integers from, 0 to 9, inclusive. Study the example given below.

Let’s take I. The integer may be 12, 21 or 30. ● What is the area of the rectangle?
But no single value for x can be determined. So I. The ratio of the length and breadth is 3 : 2.
I is not sufficient. II. The area of the rectangle is 3.6 times its
perimeter.
Let’s take II. There are several two-digit integers
@UPSC_THOUGHTS

divisible by 3: 12, 15, 18, 24, just to name a few. Solution: Take I. Let’s take length l as 3x and
So II does not help to get a single value for x, breadth b = 2x metres.
and hence is not sufficient. Since you are not Area = 3x × 2x = 6x2 metres2
given the option of I and II together providing
However, a specific value of x, and therefore the
the answer, you may mark (d).
area, cannot be derived. So I is not sufficient.
● What is the rate of compound interest earned Now take II. Without a specific number for at
by Hiren at the end of 2 years? least one dimension, l or b, or for the perimeter,
I. Simple interest at the same rate for one year we cannot get a clear answer with the help of
is Rs 1020 and the rate of interest is 12% II alone. So II also is not sufficient by itself.
per annum.
But with option (c) asking if both statements
II. The amount invested is Rs 8500.
together can get the answer, we need to check
Solution: Take I. You have the rate per cent and further. Now, you can combine what I and II
simple interest for a year. The question gives say to arrive at an answer.
you the time. So you can easily find the principal From I, we have l and b = 3x and 2x.
Interpolating in II, we have
100 S.I.
P = Perimeter = 2 (3x + 2x) m = (10x) m.
R T
From I, we have area = 6x2
Once you know the principal, you can find Now 6x2 = 3.6 times perimeter (from II)
the C.I.
or 6x2 = 3.6 × 10x
T
R
C.I. = P 1 1 x2 (3.6 10)
100 or
x 6
as all elements are available. or x = 6 metres
So I is sufficient. Now as from I we know 3x × 2x to be length
Take II now. Just with the principal and the and breadth, and we know value for x, the area
time (given by the question) you cannot find the can be found.

6 . 109
Basic Numeracy and Data Interpretation

So I and II together are sufficient to answer the three’. So you need to check III—which you
question. should, anyway.
So you must read the answer responses III tells us l : b : : 6 : 5
carefully. If an option asks you to consider the
sufficiency of both statements together, you l 6
Or
cannot stop with just checking I and II b 5
separately. You must also check whether both 5l + 6b = 0
together can supply the answer. Now III and I can also solve the problem
Question Type II and III and II can also solve the problem
In this format, there are three statements below each So the correct answer response is (b).
question, and you have to check the sufficiency of each
statement to answer the question. The answer options ● What is the price of one dozen oranges?
in this kind of question usually follows each question I. Total cost of 2 dozen oranges and 3 dozen
item as various combinations of the statements are bananas is Rs 110.
provided in the options. II. Total cost of 3 dozen apples and 1 dozen
bananas is Rs 170.
Directions: Each of the questions below consists of a question III. Total cost of 1 dozen each of oranges, apples
followed by three statements. You have to study the question and bananas is Rs 95.
and the statements and decide which of the statement(s) is/ (a) I and II only or I and III only
are necessary to answer the question. (b) I and III only or II and III only
● What is the area of the rectangular field? (c) II and III only
I. The perimeter of the field is 110 metres.
@UPSC_THOUGHTS

(d) I, II and III are all required to answer the


II. The length is 5 metres more than the width. question
III. The ratio between the length and the width
is 6 : 5 respectively. Solution: In such questions, take up the options.
(a) I and II only In (a) you have I and II as part of the option.
(b) Any two of the three Statements I and II do not give you an answer:
(c) All I, II and III there are three variables and 2 equations. So (a)
(d) I, and either II or III only cannot be correct; eliminate it.
In (b) you have I and III only as part of the
Solution: The answer options are not simple.
option. Once again you have three unknown
You have not only to check each statement for
variables and two equations. No answer;
its necessity in answering the question, but you
eliminate.
have also to see in which combinations the
statements work. Take (c) II and III—again two equations but three
unknown variables. No answer, eliminate.
From I you can get the combined length and
breadth of the field With three unknown variables, and three
equations, a solution could be possible.
Perimeter 110 m = 2 (l + b)
From I 2x + 3y = 110 … (i)
l + b = 55 m
From II 3z + y = 170 … (ii)
But since it is a rectangle, you need to know the
length and breadth separately. From III z + x + y = 95 … (iii)
From II you get Multiplying (iii) by 3 we get
l = b + 5 3z + 3x + 3y = 285
From I and II now you can get l and b and hence Subtracting (ii)
the area of the field. 3z + 3x + 3y = 285
But don’t be in a hurry to mark (a). 3z + y = 170
– – –
There is option (b) which says, ‘Any two of the We get 3x + 2y = 115 … (iv)

6 . 110
Basic Numeracy and Data Interpretation

We have equations (i) and (iv) for x and y. Now 2. Statistics does not study individuals. Statistics
we can work out the equation for two variables deals with aggregates. This is a limitation, too. If 100
and find the price of one dozen oranges. persons die due to starvation in a country of a huge
But we have used all three statements for arriving population, the percentage would work out to a
at the solution. practically negligible figure: statistically we would be
So answer response (d) is correct. justified in ignoring it; but on an individual level it
would not lessen the horror of death and its
circumstances.
DATA INTERPRETATION 3. Statistical laws are true only on an average. Laws
of Statistics are not universally true like the laws of
Data may be in the form of tables, charts, diagrams or
Physics or Astronomy. If we say the per capita income
just statements. Interpreting data calls for careful reading,
of a certain country is 500 dollars, we cannot conclude
understanding, and organising of facts and figures,
that each person in that country gets an income of 500
doing the required calculations and then reaching
dollars, for it is merely an average figure. Since a
meaningful conclusions.
multiplicity of factors affect a phenomenon studied
Most questions requiring interpretation of data call
statistically and since it is not possible to study the
for a basic knowledge of statistical tools. Principles of
effect of each factor separately, (as is done under
calculating percentages, ratios, rates and averages also
experimental methods), the conclusions arrived at are
need to be kept in mind.
not accurate. It is possible to arrive at slightly different
Statistical Methods conclusions with the same data at different times. Here
we talk in terms of probability and not certainly.
Statistics is a body of methods for collecting, organising,
4. Statistics is only one method of studying a
@UPSC_THOUGHTS

summarising, presenting and analysing data, as well


problem. It does not tell the complete story. Factors like
as drawing valid conclusions and making reasonable
religion, culture and mental attitudes which affect
deductions. On the basis of such analysis, statistics is
human actions do not come in the field of statistics.
a method of handling data. Statistical methods range
from a very simple set of devices to highly complicated Misuses: It is important to be able to discriminate
and complex mathematical procedures which can be between a valid and an invalid use of statistics.
used only by those who have received adequate training A set of figures by itself cannot disclose how
in these areas. For our purpose, the simple set of devices accurate or dependable it is. It is of utmost importance
is adequate. that these figures are rightly collected or arrived at, and
According to the definition of statistics as a method used without bias or prejudice.
of inquiry, there are distinct stages in a statistical (i) If the data is accurate, the conclusion drawn
inquiry, namely, (i) collection of data; (ii) presentation has a better chance of being valid. Conclusions
of data; (iii) analysis of data; and (iv) interpretation of drawn from defective or inaccurate data are
data. bound to be misleading.
Limitations: Statistics is not a precise science and (ii) If the selected sample is not the true
its limitations must be kept in mind while assessing representative of a population from which it
the validity of conclusions drawn from a study or comes, then the conclusions drawn from such
investigation. a sample about the characteristics of a parent
1. Statistics does not study qualitative phenomena population would be wrong. If, for instance,
directly. Honesty, for instance, cannot be measured in we arrive at a conclusion on the educational
figures and, therefore, in a study of honesty statistical level of India’s population after a study of a
methods cannot be of much use. However, it is possible sample drawn solely from a metropolitan town,
to express in figures certain factors related to this the conclusion is bound to give a misleading
phenomenon of honesty and thus study it in a statistical picture. If, for instance, the income level of
light. A study of the number of thefts or cases of India’s population is arrived at after a
cheating or swindling can indirectly tell us something telephone questionnaire, clearly, a wrong
of the problem. picture would emerge.

6 . 111
Basic Numeracy and Data Interpretation

(iii) The data must be sufficient to study the salient 1. Collection of Data
features of the variable under investigation. To Though for this examination, you are not likely to be
arrive at the conclusion that the crime rate has required to answer questions on collection of data, it
increased merely because the number of is necessary to know the basics so that you will be able
convicts in jails has increased is obviously to identify manipulations or shortcomings in the given
reaching a conclusion on inadequate data. data and thus avoid wrong conclusions.
(iv) If the size of the sample is small, the Some terms need to be understood.
conclusions drawn about the characteristics of Primary Data are those which are collected for the
the population would be inaccurate. The first time and are thus original in character. The source
accuracy of a random sample increases with from which such data is collected is called the primary
the square of the number of items included in source.
the sample. The effectiveness of a drug cannot Secondary Data are those which are collected by
be claimed after testing it on merely five some other agency and are available in publications,
patients. A random sample of 400 items will such as reports, books, magazines. The main sources
of secondary data are the government and its
be twice as reliable as a sample of 20 items.
departments, trade associations, research/statistical
It would not be advisable to draw conclusions
service organisations, specialised journals. Secondary
about the nature of a population consisting of
data may also be available in the records of departments
5 million items on the basis of a sample
and research papers which are not published.
constituting 50 items. Every survey involves the collection of the desired
(v) An alteration in the definition of a key term information from a population set or the universe—i.e.
could give a wrong conclusion. If two different the items under study. A population can be finite or
@UPSC_THOUGHTS

definitions of ‘wages’ are used by different infinite. The population of all the tools produced in a
parties, the same figures can be used to arrive factory on a given day is finite; the population consisting
at opposite conclusions to suit each party’s of all possible outcomes (heads/tails) in succeeding
ends. tosses of a coin is infinite. There are two ways in which
(vi) Statistics is certainly concerned with making data can be collected—census and sample.
comparisons. But some comparisons are The census method collects information about each
inaccurate or inappropriate. The statement and every individual item in the population. It is used
that the “incidence of death among sick persons when the field of investigation is limited, when more
is higher in hospitals than at home” is likely accuracy is desired and when finances and ‘time are
to lead to the conclusion that hospitals are not freely available.
the right place for sick persons—obviously Under the sampling method, only a part of the
erroneous if we remember that it is generally universe is studied and the conclusions and inferences
about the population are drawn on that basis. The
only seriously ill persons who are hospitalised.
sample is considered to be representative of the universe.
(vii) Sometimes a certain degree of correlation may
Samples may be selected in two ways; deliberate
be apparent from a set of figures when actually
selection (or purposive sampling) and random sampling.
no such relationship exists. For example, if it
In deliberate selection, the investigator himself
was found that several high income persons chooses from the universe few such units which,
took size ten in shoes, it would be wrong to according to his estimates, are best representatives of
conclude a correlation between shoe size and the population. The technique clearly suffers from the
income level. possible bias of the investigator affecting the selection.
(viii) In many cases the totals mislead us and do In random sampling the selection of the units is
not give a complete picture. If a newspaper done in such a manner that the chance of selection of
reports that 2 million people watched the each unit of the universe is the same.
Asian Games at Delhi, it is likely that many The size of the sample has a relation to the accuracy
people might have attended on more than one expected to be achieved. Ordinarily the bigger the size
day so the figure is quite inaccurate. of a sample the greater would the accuracy be.

6 . 112
Basic Numeracy and Data Interpretation

The technique of random sampling is based on the specific values. Instead, the variable is measured on a
theory of probability—a mathematical concept continuous scale. Examples are the heights, weights or
indicating the likelihood or the chance of the happening ages of objects or people. When dealing with continuous
or not happening of a particular event. If from a pack variables we have problems of precision of measurement
of 52 cards one card is drawn at random the chance and as a result we are usually involved with some
of its being an Ace is clearly 4/52 and the chance of degree of approximation.
its being any card of spade is 13/52. This indicates that Frequency may be defined as the number of times
if the chances of selection of all the units in a universe a value appears in a series.
are equal, and if from it selections are made at random Raw data can be organised into a logical form
then there is a possibility of the so selected sample mainly by (i) array and (ii) frequency distribution.
being representative of the characteristics of the The simplest form of organising raw data
population. This is the law of statistical regularity. systematically is to form an array, i.e. an arrangement
The reliability of a sample depends on size, method of items according to the magnitude.
and control of sampling errors. However, the array would be too unwieldy if the
number of items is in hundreds or thousands. In such
2. Classification and Presentation of Data cases, data has to be condensed, and a frequency
Raw data has to be reduced and simplified into a form distribution may be constructed.
that brings out its salient features. Classification is a Take a sample of 100 students and their percentage
process of arranging data into different classes according of marks, for example: to count the numbers of students
to their resemblances and affinities. with each mark would not really be practical, unless,
The following characteristics mark a good of course, you need to know that specific fact. To
classification: condense the marks it is better to allocate them to
@UPSC_THOUGHTS

(i) It should be unambiguous. Various classes classes and to count the numbers of students in each
should be defined properly so that there is class.
no overlapping. The starting-point and finishing-point of each class
(ii) It should be stable so as to facilitate are quite clear when you are working with a discrete
comparison. variable. The smallest and largest values of a variable
(iii) It should be flexible and have the capacity of that can actually occur in a class are called the class
adjustment to new situations and limits. However, when a frequency distribution is drawn
circumstances. up from a set of observations of a continuous variable,
Every time we make an observation we are interested there may be problems in identifying the starting-and
in the behaviour of a variable, that is, a characteristic finishing-points of each class.
that we know will have different values within a Here is an example on the weights in kilograms of
population. 75 dogs.
There are three types of variables.
36.42 24.23 50.36 50.72 27.20 53.36 30.63
Attribute or categorical variables are descriptive:
for example, colour of hair, colour of eyes or 40.27 103.62 84.72 35.20 42.63 47.83 28.60
defectiveness of a product. For this reason we often 22.27 19.67 105.36 60.37 109.46 63.72 64.83
have difficulty in identifying an object that possesses 52.36 50.72 42.78 73.70 37.63 42.74 50.63
a particular attribute. 64.90 65.60 28.30 70.23 67.20 63.83 57.60
Discrete variables are numerical variables but the 58.73 97.23 63.40 65.23 58.60 51.46 55.60
numerical values taken can only be particular numbers. 58.63 51.30 55.50 48.33 70.58 80.93 38.60
One example is the number of rooms in a house: rooms 29.36 27.32 37.40 33.20 33.40 74.60 87.20
can only be counted in single units. Another example 83.23 63.76 77.60 89.40 43.60 47.60 52.30
is the number of cars passing a certain spot in a given 43.57 74.60 72.20 93.60 95.43 91.30 54.87
period of time. A discrete variable does not have to take
48.98 57.76 45.67 52.89 59.89
only whole number values, however.
Continuous variables are also numerical variables Weight is a continuous variable since there is no
but, unlike discrete variables, they are not restricted to restriction on the value a weight can take within a given

6 . 113
Basic Numeracy and Data Interpretation

range. Our measuring equipment, however good, can the 20 and under 30 kg class whereas weights outside
never record the weight exactly. The recorded weight this range, for example, 19.95 kg and 30.02 kg would
will always be subject to some error. In this case we go in other classes. You must always look very carefully
shall assume that the weights are correct to the nearest at any frequency distribution to determine the true class
0.01 kg since they are given correct to 2 decimal places. boundaries and limits.
Suppose we formed a frequency distribution from
The difference between the class boundaries is
these figures using the following classes:
called the class interval or the length of the class. In
11-20 kg
our example it is 29.995 minus 19.995 kg which is 10
21-30 kg
31-40 kg, etc. kg. You will sometimes find the midpoint of a class
The problem here is that there is a gap between referred to as the class mark. For the 20 and under 30
10
each class. There is one whole kilogram between 20 kg kg group the midpoint is at 19.995 + , which equals
2
and 21 kg but this does not appear to have been 24.995 precisely, although in practice this might be
allowed for in the classes. Into which class would we rounded to 25 kg.
put the weight of 50.36 kg for example? Since it must You will notice that the frequency distribution of
go somewhere, we must make a more careful definition weights used in the above example has two open-ended
of our classes so that we cover all possible values of classes—under 20 kg and over 100 kg. Often at the
the variable. In this example it would be far better to ends; of distributions we have classes, such as those
draw-up the frequency distribution as shown in the above, where the class interval is indeterminate. It is
table below. It is quite clear from this into which class
usually convenient to treat open-ended classes as having
any weight falls.
the same class interval as the following or preceding
@UPSC_THOUGHTS

classes. In the weight example, the open-ended classes


Weights of 75 Dogs
should be treated as though they had an interval of 10
Weight (kg) Number of kg, the same as the other classes. The ‘under 20’ class
(variable) dogs (frequency) is treated as ‘10 and under 20’, therefore, and the ‘over
Under 20 1 100’ is treated as ‘100 and under 110’.
20—30 7 When data are classified in the exclusive method,
30—40 8 the upper limit of a class interval is the lower limit of
40—50 11 the succeeding class interval. See the following example:
50—60 19
60—70 10 Marks No. of Students
70—80 7 30—40 13
80—90 5 40—50 27
90—100 4 50—60 30
Over 100 3 60—70 25
Total 75 70—80 5

As in the discrete case, the class limits are the


Total 100
smallest and largest values that can occur in a class;
so in the 20 and under 30 kg class, the lower class limit In the above example 13 students are securing
is 20.00 kg and the upper class limit is 29.99 kg. The marks ranging from 30 to less than 40 and 27 students,
class limits should be distinguished from the class marks range from 40 to less than 50. A student getting
boundaries. The class boundaries are the dividing lines exactly 40 marks would be classified in the 40-50 group
between the classes; so the class boundaries of the 20 and not in the 30-40 group.
and under 30 kg group are at 19.995 kg and 29.995 kg. Under the inclusive method the upper limit of a
Such values cannot actually be measured but any class interval is included in the interval itself, and does
weight in the range 19.995 to 29.995 kg will be put in not go to the next higher class.

6 . 114
Basic Numeracy and Data Interpretation

Marks No. of Students Although classified data is a step towards


simplification (and summarising) of data, it is not able
30—39 13 to explain the data fully. Neither does it analyse the
40—49 27
data. Data has to be presented in a suitable form before
50—59 30
60—69 25 it can be studied and its salient features and significance
70—79 5 understood. Data can be presented textually, but it is
not an effective form, as matter has to be read over and
Total 100 over again to grasp the entire range of figures. The main
Here the confusion which was observed in the methods of presenting data are (i) tabulation (ii) diagrams
exclusive method is avoided as the upper limit of a and graphs.
For the purpose of this examination, we need not
class here is not the lower limit of the next class.
go into how to present data. We need to understand and
Statistical series can be either simple or cumulative.
analyse what is presented in the tables, diagrams and
In a simple series frequency against each class interval
graphs so as to interpret the data. We will discuss the
or value is shown separately or individually. In a various forms of tables and diagrams as we analyse
cumulative series the frequencies are progressively and interpret them later in this unit. It is more important
totalled and aggregates shown. to get acquainted first with the statistical tools required
There are two types of cumulative, series: (i) ‘less for analysis of data.
than’ type and (ii) ‘more than’ type. A less than type
of classification shows the number of items which is 3. Analysis: Statistical Tools
less than the limit of each class in a frequency Besides brushing up the ways of multiplying and
distribution for example, the heights of 50 students are dividing numbers quickly, you require to master the use
shown below in a ‘less than’ classification. of tools such as ratios, percentages, averages, dispersion
@UPSC_THOUGHTS

and so on. Figures have to be related to one another


Simple series Cumulative series before a meaningful conclusion can be derived.
Height No. of Height No. of
Ratio
(in cm) persons (in cm) persons
In the simplest possible form, a ratio is a quotient or
155—160 5 Less than 160 5 the numerical quantity obtained by dividing one figure
160—165 8 Less than 165 13 by another.
165—170 16 Less than 170 29 For example, production of wheat in country A is
170—175 7 Less than 175 36 4,50,600 tons and in B, it is 2,00,000 tons. We can say
175—180 5 Less than 180 41 that the ratio of production in these countries is
180—185 8 Less than 185 49 4,50,600 : 2,00,000 or 4.5 : 2
185—190 1 Less than 190 50 Ratios make comparisons of one magnitude with
A more than classification shows the number of another as multiples or as fractions and thus aid in the
items which is more than the lower limit of various interpretation, of statistical data. They are one of the
classes in a frequency distribution. The same data as simplest and most commonly used basic techniques in
above in the ‘more than’ classification will look as statistics.
We frequently use the ratio, per capita income. This
follows:
expresses the relationship of national income to the
Simple series Cumulative series population.
Height No. of Height No. of Per capita income in a year
(in cm) persons (in cm) persons National income at current prices in the year
=
155—160 5 More than 155 50 Population in that year
160—165 8 More than 160 45
For example Per capita income in 1981-82
165—170 16 More than 165 37 National income at current prices in 1981-92
=
170—175 7 More than 170 21 Population in 1981-82
175—180 5 More than 175 14 Rs 12,1989 crores
=
180—185 8 More than 180 9 693.99 millions
185—190 1 More than 185 1 = Rs 1757.8

6 . 115
Basic Numeracy and Data Interpretation

Another most commonly used ratio is sex ratio. The is a special kind of ratio which measures the rate of
sex ratio of India according to 1991 census is 929 change over a period of time.
females for 1000 males i.e., 929:1000. This form of For example: If one is driving at a speed of 40 km
expressing a ratio is called ‘proportion’. per hour and then increases the speed to 50 km per
hour, then clearly he has increased his speed by,
Percentages
50 40
Ratios or proportions with the base 100 are more easily 100 25%
40
understood and compared. For example, the ratio of Now if he reduces the speed once more to 40 km
scheduled castes to the total population in Kerala and per hour he has reduced his speed by,
Bihar in 1991 can be expressed as:
40 50
25,49,000 100 20%
SC ratio in Kerala = 29,011,237 100 50
Generally we may express this percentage rate of
= 8.78% change as
1,01,42,000 x1 x0
SC ratio in Bihar = 100 100
8,63,38,853 Percentage rate of change =
x0
= 11.75%
Where x1 = The changed or new value
This reveals that the SC ratio is favourable in Bihar.
x0 = The original value.
Rates
A ratio between two magnitudes usually shown over If the result is positive, the rate of change is positive
a period of time is called a rate or rate of change. This or the new value is greater than the original value; if
@UPSC_THOUGHTS

Demographic Ratios
Crude death and birth-rate Crude death-rate for a Infant Mortality Rate (IMR) Infant mortality rate is an
locality is found by dividing the total number of deaths important specific death rate. It is defined as the number
in that area, during the year, by the number of people of infants under one year of age dying in a year per 1000
living in that locality at the mid-point of that year. It is live births of the same year. It is calculated as follows:
usually expressed per thousand. Infant Mortality rate
Crude death rate (C.D.R.)
No. of deaths of infants the age of
No. of deaths in a locality in a year 1 year in a given year
= × 1000
= × 1000 No. of live births during the year
No. of people living during the mid
point of that year Standard Death Rate (STDR) It is evident from the
Similarly,
definition of C.D.R. that its usefulness is restricted be-
Crude birth rate (C.B.R.) cause it is based on the entire population. It does not
No. of births in a locality during a year reflect the composition of the population with respect to
= × 1000 any particular characteristic like age, region, disease, etc.
No. of people living during the
Similarly we see that by means of specific death rates
mid point of that year
(S.D.R.), we can study various segments of the population
Specific Death Rate (SDR) It is defined as the individually. But if we want to compare the death rates
number of deaths per 1000 population in a specified of two regions or communities, heterogeneity factors have
class in a given year. For e.g., if we are considering a to be removed and the populations standardised. There
certain age group of the population, than the number of are various ways in which standardised sizes of the
deaths per 1000 population of this age group gives us population are obtained.
the age specific death rate for that group. It is calculated Standardised death rate (STDR) is defined as
as follows: Sx Dx
Age specific death rate x
STDR =
No. of deaths in the age Sx
group in the given year Where, x
= × 1000
Mid - year population in that age Sx = Standardised population for group x.
group in the given year Dx = Specific death rate for group x.

6 . 116
Basic Numeracy and Data Interpretation

negative, the rate of change is negative or the new value consumption of cotton in bales but we cannot calculate
is less than the original value. ratios between production of wheat in tons and
We could have expressed the new car speed production of cotton in bales. However, in many cases
50 comparison has to be done between items which are
× 100 = 125% of the original speed and after expressed in different units. The common denominator
40 40
slowing down, the new speed as × 100 = 80% of in such cases is a number or quantity. Thus in
50
the old speed. comparing total income with total population we really
divide the number of rupees representing total income
Co-Efficients: Rate per unit is called a co-efficient. by the number of persons representing the total
If the death rate in a country is 17 per thousand, the population. In comparing the number of kilometres
co-efficient of deaths is .017. If this co-efficient of .017 done with a litre of petrol, though the units are different,
is multiplied by the total population of the country, we it is the number of kilometres which is compared with
shall get the total number of deaths. the number of litres.
Size and Types of Bases: The difference between Measures of Central Tendency
ratios, rates, percentages and co-efficients is only in the We might want to compare two or more frequency
base on which they are calculated; otherwise they all distributions obtained from different population to see
give a relative picture of two interrelated phenomena. if there are differences between the populations—for
Various types of bases can be used for the example, whether the cost of living is higher in Delhi
computation of ratios. than in Kolkata. You might want to look at a frequency
Total to total If one group of figures as a whole distribution to see how you compare with other members
is compared with another group, the base of the ratio of your profession, and whether, for example, your
would be the total of one of the groups. Per capita salary is comparable with other people of your age.
@UPSC_THOUGHTS

income, for example, is calculated by dividing total When making these comparisons it is obviously useful
income by total population. to have a single measure that is representative or typical
Part to total We may compare a part to its whole, of the distribution. Such measures are referred to as
males or females as a proportion of the total population measures of central tendency, measures of location or simply
or work force, for example. In such cases the base of as averages.
the ratio is usually the value of the universe. If these (i) Arithmetic Mean The arithmetic mean (A.M.) is
ratios are expressed as percentages, the total of these the average of a set of numbers. (You may refer to the
ratios would add up to 100 per cent. unit on ‘Average.) Suppose the numbers of visits made
Part to part We may compare part to part within by ten mothers to a clinic were
a whole. We may compare the export earnings from jute, 8 6 5 5 7 4 5 9 7 4
coffee, handicrafts etc. We may express the value of If you were asked to calculate the average number
coffee exports as a ratio of the value of jute exports. of visits, what would you do? Probably you would add
However, a better comparison would be if export earnings together the numbers of visits, making a total of 60, and
in both these items are expressed as a percentage of divide by the number of mothers, giving 6 as the
total exports. average number of visits. This particular type of average
Standard area and units Sometimes the base of the is known as the arithmetic mean. It is calculated by
ratio is a standard area—population per square totalling the values of the observations and dividing
kilometre; standard distance—cost of railway line per that total by the number of observations.
kilometre; a standard unit—students per school, rooms For such ‘ungrouped’ data, the formula is
per house. x
Arbitrary ratios In many enquiries it is possible to x
n
use arbitrary units; examples of such arbitrary units are
horse power, light-year, ton-kilometre. The most common The symbol x (pronounced x-bar) stands for the
arbitrary units are 1, 10, 100, 1000; among these 100 arithmetic mean.
(or per cent) is the most popular. The symbol ∑ stands for summation: whenever you
● It is necessary that the two figures compared see this symbol in a formula it means ‘the total of’. ( ∑
should have the same characteristics and should be is a Greek capital letter which is pronounced sigma.)
expressed either in the same unit or in comparable The letter x refers to the value of an observation, and
units. Production of cotton in bales can be compared to n is the number of observations.

6 . 117
Basic Numeracy and Data Interpretation

Suppose we have to find the A.M. of a frequency In the case of continuous variables, covering a large
distribution. Consider the following table: range of values or a grouped frequency distribution,
there is a different way of calculating the A.M.
Numbers of visits made to clinic Let’s take the data in the following table.
by 100 mothers
Weights of 75 pigs
Number of visits Number of mothers
(variable) (frequency) Weight (kg) Number of pigs
4 8 (variable) (frequency)
5 12
6 15 Under 20 1
7 25 20 and under 30 7
8 17 30 and under 40 8
9 13 40 and under 50 11
10 10
50 and under 80 19
Total 100 60 and under 70 10
70 and under 80 7
Each value of the variable occurs more than once. 80 and under 90 5
To find the sum of the numbers of visits made by all 90 and under 100 4
the mothers we multiply each value of the variable by Over 100 3
the frequency with which it occurs. It is best to set out
Total 75
the calculation in the form of a table.
With such a frequency distribution we have a range
Frequency distribution and sum of visits of values of the variable comprising each group. As our
made by 100 mothers to the clinic values for x in the formula for the arithmetic mean we
@UPSC_THOUGHTS

use the midpoints of the classes. (We again make the


Number of visits Number of mothers Sum of visits
assumption that the open-ended class intervals are of
(variable) (frequency)
the same length as the adjoining classes.) This assumes
x f fx
that within each class the arithmetic mean value of the
observations is at the class midpoint, an assumption
4 8 32 (4×8)
that may not be completely sound but it is the best that
5 12 60 (5×12)
we can do. Having made that assumption, we can
6 15 90
calculate the arithmetic mean.
7 25 175
We have the table below in which midpoints of the
8 17 136
9 13 117 classes are indicated.
10 10 100 Grouped frequency distribution of
weights of 75 pigs
Total 100 710
Weight (kg) Midpoint of Number of pigs
The arithmetic mean is found by dividing the total (variable) class (frequency)
number of visits made by all the mothers (710) by the x f fx
number of mothers (100): this gives 7.1.
Under 20 15 1 15
The formula for the arithmetic mean calculated
20 and under 30 25 7 175
from a frequency distribution has to be amended to
30 and under 40 35 8 280
include the frequency. It becomes:
40 and under 50 45 11 495
fx 50 and under 60 55 19 1045
x 60 and under 70 65 10 650
f
70 and under 80 75 7 525
80 and under 90 85 5 425
In this case, 90 and under 100 95 4 380
fx 710
x 7.1 Over 100 105 3 315
f 100
Total 75 4305

6 . 118
Basic Numeracy and Data Interpretation

fx In the final column of the table each deviation has


Substituting in the formula x we find that been multiplied by its frequency to give the total
f
deviation for each class; for instance, there were 7
the arithmetic mean of the weights is 4305 kg ÷ 75, that readings in the 20 and under 30 kg class. So these 7
is 57.4 kg. readings give a contribution of 7 × (–3) to the total
The calculation of the arithmetic mean can be deviation. The sum of the deviations is divided by the
simplified using a procedure known as the assumed total number of observations to find the mean of the
mean method. In this method we assume that one of deviations but we must-remember to multiply by the
the values of x is the arithmetic mean and then find common factor of 10 that we took out earlier. Adding
the deviations of our observations from that assumed this mean of the deviations to the assumed mean gives
mean. If we happen to choose as our assumed mean us the true arithmetic mean of the weights:
the true value of the arithmetic mean, then the sum of 18
these deviations is zero. 55 + × 10 = 55 + 2.4 = 57.4 kg
75
Let us recalculate the arithmetic mean of the pig The formula for performing this calculation is:
weights using this method, taking an assumed mean
of 55 kg. To ensure simplification of the arithmetic we fd
x = Assumed mean + i
must-choose one of the class midpoints as the assumed f
mean. It is also helpful to choose a midpoint in the
centre of the distribution and one with a high frequency. where i is the common factor that was taken out of the
The calculation is set out as in the table below: deviation to simplify the arithmetic and d is the
difference between each value of x and the assumed
mean in units of i. If the class intervals are of equal
@UPSC_THOUGHTS

Assumed mean method of calculating the


arithmetic mean of the weights of 75 pigs length, then the length of the class interval will always
Weight (kg) Mid-point Number Deviation Deviation
be a common factor of the deviations.
(variable) of pigs of x from in units Arithmetic Mean should be used when (a) the
(frequency) assumed of 10 kg
mean (kg) distribution is not very skew, (b) the distribution does
x (kg) f d fd not have open-end classes, (c) rates, ratios and
percentages are not being studied, (d) when the
Under 20 15 1 –40 –4 –4
distribution does not have very large and very small
20 and under 30 25 7 –30 –3 –21
items, as in such cases extreme items would adversely
30 and under 40 35 8 –20 –2 –16
affect the average.
40 and under 50 45 11 –10 –1 –11
50 and under 60 55 19 0 0 0 (ii) Mode: The mode, as its name might suggest, is
60 and under 70 65 10 10 1 10 the value of the variable which is most ‘fashionable’,
70 and under 80 75 7 20 2 14 the one with the highest frequency. It is at the highest
80 and under 90 85 5 30 3 15 peak of the frequency curve.
90 and under 100 95 4 40 4 16
( fz fl )
7 Over 100 105 3 50 5 15 Mode = L i
( fz fl ) ( fz fh )
Total 75 18 where
L = lower class boundary of the modal class
The deviations in column (4) are found by i = class interval
subtracting the assumed mean from each value of x. To fz = frequency of the modal class
simplify the calculation further, in column (5) each fl = frequency in the adjacent lower class
deviation has been divided by its highest common fh = frequency in the adjacent higher class
factor, in this instance 10. Although taking out a
common factor of 10 does not simplify this particular If we take the data in the table of ‘Weight of 75
calculation a great deal, the procedure can often make Pigs’, we find
a calculation much easier. So always look for a common (19 11)
Mode = 50 10
factor. (19 11) (19 10)

6 . 119
Basic Numeracy and Data Interpretation

8 For a groped frequency distribution,


= 50 10
8 9
1 
8  n  Fm 1 
= 50 10
Median = Lm   2 i
17
 fm 
 
80  
= 50
17
where Lm = lower class boundary of median class
= 54.7 kg. to 1 decimal place n = total number of observations
Graphical methods of estimating mode are possible Fm – 1 = cumulative frequency below the median class
from histograms and frequency curves. fm = median class frequency
i = class interval
Mode should be used when we are dealing with
qualitative data, and where we have to find the Take the table on weights of 75 pigs and create a
preferences of people, say for a particular brand of tea ‘Less than’ cumulative frequency.
or any other article. Consumer preferences are best
studied with the help of mode which gives the most Weights of 75 pigs
common or fashionable item of the series. In some cases Weight (kg) Number of pigs ‘Less than’
of discrete series like the average number of rooms in (variable) (frequency) cumulative
frequency
households, or average size of shirt collar, or shoe or
average number of children per couple, mode is the Under 20 1 1
only appropriate average. 20 and under 30 7 8
@UPSC_THOUGHTS

30 and under 40 8 16
40 and under 50 11 27
(iii) Median: We would expect a measure of central 50 and under 60 19 46
tendency to be near the middle of the distribution to 60 and under 70 10 56
which it refers. The value of the variable which divides 70 and under 80 7 63
the distribution so that exactly half of the distribution 80 and under 90 5 68
has the same or larger values and exactly half has the 90 and under 100 4 72
Over 100 3 75
same or lower values is called the median.
Total 75
To find the median we need to arrange our
observations in ascending sequence and then locate the
We want to find the weight of the pig half-way
middle value.
through the distribution, that is, the weight of the 38th
If the total number n is odd, pig. This is the weight of the (n + l)/2nd pig (where
n is the total number of pigs). The 38th pig is somewhere
n 1 in the ‘50 and under 60 kg’ class since 27 pigs weighed
Median = value of th item
2 less than 50 kg and 46 pigs weighed less than 60 kg.
If the total number n is even, We assume that the weights of the pigs in the ‘50 and
under 60 kg’ class are evenly spread across the class.
Median We have accounted for 27 of the 38 pigs before we
1 n n reached this class. So we need to consider 11 out of the
= value of th item value of 1 th item
2 2 2 19 pigs in the class to arrive at the 38th pig. The median
weight will thus be 11/19 of the way across the ‘50
If it is a discrete series, and under 60 kg’ class interval:
n 1 11
Median = th item 50 + × 10 = 50 + 5.79 = 55.79
2 19
where n is the number of observations The median weight is 55.8 kg to 1 decimal place.

6 . 120
Basic Numeracy and Data Interpretation

We conclude that half the pigs weighed less than 55.8 Let us find the geometric mean of the numbers 3, 25
kg and half weighed 55.8 kg or more. and 45. There are three observations, so n = 3.
It is common practice when dealing with grouped Geometric mean
data to calculate the median as the value of the n/2 item = 33 25 45
1
in the distribution, in this case the weight of the 37 th = 3 3375
2
pig, although strictly speaking it should be the value
= 15 (by inspection, since 153 = 3375)
of the (n + l)/2nd item.
Using the formula for the median of grouped The geometric mean cannot be calculated if we have
frequency distribution, we have negative or zero observations. The geometric mean of
75 a set of readings is always less than the arithmetic
27 mean (unless all the readings are identical) and is less
Median = 50 2
10
19 influenced by very large items. Take, for example, a
small company where the following salaries (in
10.5
= 50 10 thousands of rupees per month) are paid to the staff:
19
2 3 4 4 4 5 7
= 55.5 kg to 1 decimal place. The arithmetic mean monthly salary is Rs 4143 to
1 the nearest rupee. The geometric mean of the salaries
This value corresponds to the weight of the 37 th is:
2
pig whereas our previous value was the weight of the
= 72 3 4 4 4 5 7
38th pig.
Median can also be graphically determined. = 7 13440
@UPSC_THOUGHTS

Finding roots such as this is not an easy arithmetical


Median should be used when we have open end
operation.
distribution and more particularly when plotted as a
curve, the distribution is found to be of J-shape or An alternative method is to use logarithm tables
reverse J-shape. log 13440 = 4.1284
Relationship between Mean, Mode and Median: Divide by 7 = 0.5898
The following formulae may be kept in mind:
Antilog (0.5898) = 3.889
(a) In symmetrical distribution
Mean = Median = Mode To the nearest rupee, the geometric mean monthly
(b) In asymmetrical distribution salary is Rs 3889.
Mode = Median – 2 Mean Suppose that the owner of the company, having
(c) In positive skewed curve paid himself Rs. 7000 per month, now decides that the
Mode < Median < Mean success of the company warrants him trebling his
(d) In negative skewed curve salary to Rs 21000 so that the salaries (in thousands
Mean < Median < Mode of rupees per month) now are:
(e) In moderately skewed distribution 2 3 4 4 4 5 21
Mode = 3 Median – 2 Mean The arithmetic mean salary is Rs 6143 per month
Or A.M. – Mode = 3 (A.M. – Median) to the nearest rupee. The geometric mean is:
= 72 3 4 4 4 5 21
(iv) Geometric Mean: A less commonly used
measure of central tendency is the geometric mean. It is = 7 40320
calculated using the formula:
= 4.549 to 3 decimal places
Geometric mean = n x1 x2 x3 ... xn
The geometric mean salary is Rs 4549 per month
where n is the number of observations made of the to the nearest rupee. The trebling of the owner’s salary
variable x and x1, x2, ..., xn are the values of these has had less of an effect on the geometric mean because
observations. A simple example will explain the formula. the geometric mean is only affected by a factor of 7 3.

6 . 121
Basic Numeracy and Data Interpretation

The geometric mean is useful when only a few rate of 50 documents per hour. This does not take into
items in a distribution are changing: it is then much account the fact that invoices take longer to produce
more stable than the arithmetic mean. The geometric than statements and statements take longer to produce
mean is useful for making estimates from data which than reminders. The true average rate is found by taking
grow or decay in geometric progression. Population the harmonic mean:
grows in this way; that is, an increase in population
3 3 3 240
is proportional to the number in the population at any
1 1 1 17 17
one time, not proportional to the number in the 30 40 80 240
population at the start of the time span as it would be
= 42 (to the nearest decimal)
if it grew arithmetically.
Geometric Mean should be used when we have to Harmonic Mean is useful when we have to compare
average rates, ratios and percentages and we are values of a variable with a constant quantity of another
interested in measuring relative changes rather than variable, e.g., time, distance covered within certain time,
absolute ones. It is an appropriate average for the speed, and quantities purchased or sold at a particular
construction of index numbers. price. This average is very useful in such cases where
small items have to be given greater importance.
(v) Harmonic Mean: The harmonic mean is used for
averaging rates. Worked Examples on Measures of Central Tendency
If a car travelled on an outward journey at 60
● The arithmetic mean of 10 observations is 50 and
kilometres per hour and on the return journey at 40
that of 6 observations is 35. The combined mean
kmph, what is its average speed? It is not 50 kilometres
@UPSC_THOUGHTS

of all the 15 observations will be


per hour as it might appear. That would be the average
(a) 44.3 (b) 43.3
speed if the car had travelled for one hour at 60 kmph
(c) 42.3 (d) 41.3
and then a further hour at 40 kilometres per hour. To
find the average speed when different speeds have been 10 50 6 35
Solution: x
attained over the same distance, the harmonic mean is 10 6
used. The formula for this is: 500 210
44.3
16
n
Harmonic mean =
1 ● Mean weight of a group of 25 persons was found
x to be 78.4 kg. Later on, it was discovered that the
weight of one person was misread as 69 kg instead
where n is the number of observations.
of the correct weight of 96 kg. Then the correct
In our example, the speeds of 60 and 40 kmph are mean would be
the observations. The average speed, therefore, is: (a) 79.5 (b) 79.2
(c) 79.3 (d) 79.4
2 2 2 120
1 1 5 5 Solution: Mean weight of 25 persons = 78.4 kg
60 40 120 Total sum of weight = 78.4 × 25
= 48 kmph = 1960.0
Correction = 96 – 69 = 27
The principle can be applied to a business situation.
Suppose that a typist in an office can type invoices at  Corrected sum of weight = 1960 + 27
a rate of 30 per hour, statements at a rate of 40 per hour = 1987
and reminders at a rate of 80 per hour and that she 1987
 Correct mean = = 79.48
has to type equal numbers of these documents. The 25
office manager is asked to report on the average output = 79.5
per hour of the typist: he adds together the rates, 39 ● What would be the modal weight of the following
+ 40 + 80 = 150, and divides by three giving an average group of the college football players?

6 . 122
Basic Numeracy and Data Interpretation

Weight (lbs) No. of Players or 31.2 + 2.6y = 28 + 3y


60-70 2 or 0.4y = 3.2
70-80 8 4y = 32
80-90 20 y = 8
90-100 17
● A train covered a distance of 500 km four times;
100-110 3
the first time at a speed of 50 km/h, the second
(a) 79.5 (b) 70.4 time at 20 km/hr, the third time at 40km/hr and
(c) 86.8 (d) 45.6 the fourth time at 25 km/hr. The average speed
of the train is
Solution: Here modal class is 80 – 90
(a) 67.5 km/h (b) 76.2 km/h
c. f2 (c) 20 km/h (d) 29.6 km/h
Mode = l
f1 f2
Solution: Total distance covered by the train
10  17 = 500 × 4 = 2000 km
= 80  = 86.8
8  17 Time taken by the train, first time
● The median of 13, 9, 3, 7, 12, 15, 10 is distance 500
(a) 9 (b) 10 = =
speed 20
(c) 12.5 (d) 7
= 10 hours
Solution: Arrange in order : 3, 7, 9, 10, 12, 13, 15 Second time the train took
Hence, median is 10. 500
= = 25 hours
40
@UPSC_THOUGHTS

● What would be the mode of the following data of Third time the train took
the weights in kg of a group of people? 500
= = 12.5 hours
52, 59, 50, 43, 50, 59, 59, 61, 64, 65, 61, 66, 65 50
Fourth time the train took
(a) 65 (b) 43 500
(c) 59 (d) 61 = = 20 hours
25
Total time taken = 67.5 hours
Solution: By arranging in series we have
Thus the average speed of the train is
x 43 50 52 59 61 64 65 66 2000
= = 29.6 km/h
67.5
y 1 2 1 3 2 1 2 1
● A man goes by car from town X to town Y and
back. The outward journey is uphill and he gets
Since the highest frequency is 3 which corresponds only 20 km to the gallon of petrol. On the return
to 59 journey, he gets 30 km to the gallon. Harmonic
 Mode = 59 mean of his petrol consumption in km per gallon
● If the mean of the distribution is 2.6, then the value is
of y is (a) 24 km/g (b) 25 km/g
(c) 26 km/g (d) 27 km/g
Variate X 1 2 3 4 5
n
Solution: Harmonic Mean =
Frequency F of X 4 5 y 1 2 1
 
x

(a) 13 (b) 3
(c) 8 (d) 24 2 2
= 1 1 =
5
Solution: Mean = 1 4 2 5 3 y 4 1 5 2 20 60 30
4 5 y 1 2 120
4  10  3 y  4  10 =
5
2.6=
12  y = 24 km per gallon

6 . 123
Basic Numeracy and Data Interpretation

● The average marks of 600 students in an ● A student’s final marks in Mathematics, Physics,
examination is 42. The average marks of top 150 English and Hygiene are respectively 82, 86, 90
students is 76, while that of the last 250 students and 70. If the respective credits received for these
is 28. Average marks of the remaining 200 students courses are 3, 5, 3 and 1. Then the approximate
is average marks is
(a) 34 (b) 30 (a) 80 (b) 85
(c) 27 (d) 25 (c) 90 (d) 95
Solution: Total marks of 600 students is wX
Solution: X
600 × 42 = 25,200 w
Total marks of top 150 students is
(3) (82) ( 5)(86) (3)(90) (1)(70)
150 × 76 = 11,400 =
3 5 3 1
Total marks of the last 250 students is
= 85
250 × 28 = 7,000
● Twenty per cent of the workers in a firm employing
 25200 – 18400 = 6800
a total of 2,000 earn less than Rs 2 per hour, 440
i.e., the average marks for the remaining 200
earn from Rs 2 to Rs 2.24 per hour, 24% earn from
students is
Rs 2.25 to Rs 2.49 per hour, 370 earn from Rs 2.50
6800 ÷ 200 = 34
to Rs 2.74 per hour, 12% earn from Rs 2.75 to Rs
● The number of overtime hours worked by 1000 2.99 per hour and the rest earn Rs 3 or more per
employees of a company during the last six months hour. Modal wage of the workers is
is as follows: (a) 2.31 (b) 5.21
@UPSC_THOUGHTS

Overtime (in hours) No. of employees (c) 4.8 (d) 3.24


0-20 401 Solution:
20-40 226
Earning per hour No. of workers
40-60 182
60-80 74 Less than Rs 2.00 400
80-100 43 Rs 2.00 to Rs 2.24 440
100-120 38 Rs 2.25 to Rs 2.49 480
Rs 2.50 to Rs 2.74 370
120-140 20 Rs 2.75 to Rs 2.99 240
140-160 9 Rs 3.00 and more 70
160-180 5
Total 2,000
180-200 2
The percentage of the employees having worked f1  f 0
Mode = l1  i
less than 40 overtime hours during these months 2 f1  f 0  f 2
is 480  440
(a) 50% (b) 62.7% = 2.245   0.25
960  440  370
(c) 75.2% (d) 80%
Solution: No. of employees having overtime less = 2.245  40
 0.25
than 40 hours is in class 40  110
0 – 20 and 20 – 40 = 2.245 + 0.07
 401 + 226 = 627
= Rs 2.31
Total number of employees = 1000
Percentage of employees who worked less than 40 Measures of Variation
627 There are ways of measuring and comparing the spread
overtime hours is =  100 = 62.7% of data in a population. These measures are called
1000
measures of dispersion or measures of variation.

6 . 124
Basic Numeracy and Data Interpretation

(i) Range: The range of a set of results is the the lowest interval with values in that interval is 1-5.
difference between the smallest and the largest figure Choosing the bottom limit we have 1. The highest
in the data. It is literally the ‘spread’ or ‘width’ of our interval with values in that interval is 41-45. The top
data. If we have the figures: limit of this interval is 45, so the range is:
7, 1, 6, 3, 8, 8, 15, 4, 7 Range = 45 – 1 = 44
then the range is the largest figure minus the smallest L S
figure, in other words: Coefficient of range =
L S
Range = 15 – 1 = 14
where L = value of the largest item
Similarly, if our measurement is continuous, for the
S = value of the smallest item
figures:
7.4, 12.2, 8.6, 7.7, 9.3, 5.8, 11.6, 8.8, 9.0 (ii) Quartile Deviation: A problem with using the
the range is still the largest figure minus the smallest range is that extreme values in a distribution can cause
figure. the range to be misleading as a measure of dispersion.
Range = 12.2 – 5.8 = 6.4 The quartile deviation helps with this problem because
If we have a frequency table for a discrete variable it just looks at the middle 50% of the distribution,
therefore leaving out the extreme values.
where the table has frequencies for individual values,
Quartiles are the values of the variable that belong
then we simply find the lowest value which has a
to the members of the population that are 25%, 50% or
frequency of 1 or more, find the highest value which
75% of the way through the distribution. There are
has a frequency of 1 or more and take the difference
therefore three quartiles, the first known as Q1, the
between these values.
second as Q2 and the third as Q3. The second quartile—
Number of cars purchased per day we have already come across it—is the median, the
@UPSC_THOUGHTS

value of the variable that belongs to the item half-way


Number of cars Frequency through the distribution. The values of the other
0 3 quartiles can be calculated in exactly the same way as
1 8 the median was calculated.
2 20 Q Q1
3 14 Quartile Deviation = 3
2
4 6
5 3 Q3 Q 1
Coeff. of Q.D. =
6 0 Q3 Q 1
then the lowest value with any observations is zero, the (iii) Deciles and Percentiles: It is convenient,
largest with any observations is 5, so the range is: particularly when dealing with statistics of income and
Range = 5 – 0 = 5 employment, to consider values similar to the quartiles
but which subdivide the distribution more finely. Such
If the grouped table uses class intervals, we take
partition values are deciles and percentiles. The deciles
the bottom limit of the lowest interval with a frequency
are the values which divide the total frequency into
of 1 or more. This is then subtracted from the top limit tenths and the percentile are the values which divide
of the highest interval with a frequency of 1 or more. the total frequency into hundredths. It is only meaningful
For the table with number of transactions: to derive such statistics when we are dealing with a
1-5 1 very large number of observations, preferably thousands.
6-10 9 The deciles are denoted by D1, D2 , . . . , D9. The third
11-15 12 decile D3, for example, is the value below which 30%
16-20 13 of the data lies. The method of calculation follows the
21-25 16 same pattern as the calculation of the median and
26-30 11 quartiles.
31-35 6 The percentiles are denoted by P1, P2 , . . ., P99 and
36-40 2 for example P6 is the value below which 6% of the data
41-45 2 lies and P66 is the value below which 66% of the data
46-50 0 lies.

6 . 125
Basic Numeracy and Data Interpretation

(iv) Mean Deviation: The mean deviation measures


fx 4305
how far, on an average, the readings are from the x =
f 75
arithmetic mean. (The median is very occasionally used
instead of the arithmetic mean.) If the data have a small
spread about the mean, the mean deviation has a lower f |x x| 1260.8
MD =
value than for data which show large variations about f 75
the mean.
= 16.8 kg to 1 decimal place
To calculate the mean deviation
(i) find the arithmetic mean, x , of the data; (v) Standard Deviation: The Standard deviation is
(ii) find the deviation of each reading from x i.e. the most important measure of dispersion. It takes the
work out the difference between each reading value of every observation into account but does not
and x ; suffer from the same arithmetical deficiencies as the
(iii) find the arithmetic mean of the deviations, mean deviation.
ignoring their signs. When we calculated the mean deviation for
For ungrouped data the formula for the mean ungrouped data, we used the formula:
deviation is: |x x|
|x x| MD
Mean deviation (MD) = n
n
where |x x| meant the total of the absolute
where | x x | stands for the absolute deviation, that is,
deviations from the mean. We had to take the absolute
the deviation ignoring the sign.
deviations because the sum of actual deviations was
When calculating the mean deviation of a grouped
zero. There is another way of getting round this problem:
@UPSC_THOUGHTS

frequency distribution the formula is modified to include


instead of ignoring the signs we can remove them
the class frequencies: simply by squaring the deviations. From your elementary
f |x x| mathematics you will know that if you multiply like
MD =
f signs the product always has a positive sign. It does not
x in this formula is the class midpoint. Using the matter therefore whether | x x | has a positive or a
example of the weights of 75 pigs, we set out the negative sign: squaring it will always give a positive
calculation as in the table. We again assume that the result.
open-ended classes are of the length as adjacent classes. If we find the mean of the sum of the squared
deviations, we obtain the variance:
Calculation of the mean deviation
(x x )2
Weight (kg) Midpoint Number Deviation
Variance =
(variable) of pigs n
(frequency)
Standard deviation (SD) = Variance
x f fx | x x | F | x x |
( x x )2
Under 20 15 1 15 42.4 42.4 =
n
20 and under 30 25 7 175 32.4 226.8
30 and under 40 35 8 280 22.4 179.2 To calculate the standard deviation for ungrouped
40 and under 50 45 11 495 12.4 136.4 data using this formula:
50 and under 60 55 19 1045 2.4 45.6 (i) find the arithmetic mean, x , of the data;
60 and under 70 65 10 650 7.6 76.0 (ii) find the deviation of each reading from x ;
70 and under 80 75 7 525 17.6 123.2 (iii) square each of these deviations;
80 and under 90 85 5 425 27.6 138.0 (iv) total the squared deviations;
90 and under 100 95 4 380 37.6 150.4 (v) divide this sum by the total number of readings
Over 100 105 3 315 47.6 142.8 to obtain the variance:
(vi) find the square root of the variance to obtain
Total 75 4305 1260.8
the standard deviation.

6 . 126
Basic Numeracy and Data Interpretation

Probability 19×2
44.70
100 = 114.0
Probability has already been discussed in an earlier 39.20
unit. The rules and principles need to be kept in mind. 51.40
19×3 100 = 131.1
39.20
Index Numbers
An index number is a statistic used to reduce a series 52.50
19×4 100 = 133.9
of data to some common level so that we can make 39.20
comparisons of variations between items or groups of The figures obtained are often called price relatives.
items. They are used to describe changes in prices, All the figures are calculated as percentages of the
output, income, etc. and can be divided into three types: figure for 19×1 and so they show percentage changes
(a) price index numbers, which measure changes from 19×1 (e.g., the price for 19×3 is 31.1% higher than
in price; that in 19×1).
(b) quantity index numbers, which measure However, you cannot say the percentage rise from
changes in quantity; and 19×3 to 19×4 is:
(c) value index numbers, which measure changes 133.9 – 131.1 = 2.8%
in the value of service or activities or goods. It is in fact:
The base period is the time at which all comparisons 52.50
are made. It may be a single date, a month or a year. 100 102.1
51.40
The length of the base period usually depends on the so a 2.1% increase. Using the index number we work
interval at which the index number is to be calculated. out the ratio of the appropriate index numbers as:
The index number for the base period is given the value
133.9
100 and this might be allocated to: 100 102.1 (as before)
@UPSC_THOUGHTS

131.1
(a) a year - 19×5 = 100;
(b) a month - August 19×5 = 100; Adjusting income and prices by price index or
(c) a single date - 16 August 19×5 = 100. deflating income and prices The consumer is concerned
If we have the price of an item recorded at different with real wage rather than with money wage. Real
times, then an index number can be constructed to wage implies the quantity of commodities or services
show changes in price. that can be bought by money wage. When commodity
Year Change of car service prices are high, purchasing power is low. Thus it is the
real income and not money income that determines the
19×0 Rs 36.50
standard of living of the people in a country. Price
19×1 Rs 39.20
changes have to be eliminated for any change in the
19×2 Rs 44.70
standard of living to be examined. Deflating means
19×3 Rs 51.40
neutralising the effect of an increase in prices.
19×4 Rs 52.50
Money income
If we use 19×1 as base year it has index number Real income =
Price index
= 100. The index number for 19×0 is calculated as:
Cost of living index numbers A cost of living index
36.50
100 = 93.1 is a statistical tool devised to show the average change
39.20
in prices of goods and services purchased by a group
Similarly, for 19×2 we calculate: of persons living at the same place. The cost of living
44.70
index numbers are based upon retail prices which are
100 = 114.0 more sensitive to change than are wholesale prices.
39.20
Before constructing the cost of living index numbers, a
and if the other years’ index numbers are calculated, decision must be taken about which class of people is
we have: to be studied, as people of different classes consume
36.50 different commodities. It is also important to define the
19×0 100 = 93.1
39.20 area covered by an index because people of the same
19×1 class in the same area consume more or less the same

6 . 127
Basic Numeracy and Data Interpretation

type of commodities. Next, a family budget enquiry (i) There is a cause and effect relationship between two
should be conducted to estimate how much an average variables. For example heat and temperature
family spends on different commodities of consumption. may be correlated because heat affects
The last step in the construction of the index is to assign temperature. There may be a cause and effect
weights to selected commodities. Quantities of relationship between them. However, in social
commodities consumed in the base year may be used sciences, such relationships are rarely found.
as weights, or the aggregate expenditure incurred upon (ii) Both the correlated variables are being affected by
the purchase of different commodities by an average a third variable or by more than one variable. For
family may be used as weights. example we may find a high degree of
In the aggregate expenditure method, the cost of correlation between the prices of rice and
living index is computed by the formula. prices of jute. In reality it may be found that
the prices of both these commodities have been
P1q0
100 affected by their production which in turn is
P0 q0 affected by rainfall.
(iii) Related variables might be mutually affecting each
where P1q0 is the total expenditure incurred on the
other so that neither of them could be designated
purchase of commodities in the current year and
as a cause or effect. This situation particularly
P0q0 is the total expenditure on the same basket of
holds good in the field of economics and
goods and services in the base year.
business. For example the demand of a
In the family budget method (also called weighted
commodity may go down as a result of rise in
relatives method) the cost of living index is
prices. One would normally presume that price
RW is the cause and demand is the effect. However,
@UPSC_THOUGHTS

W it may be that the demand of the commodity


has gone up due to anticipated shortage in
Current price (P1 )
R stands for price relatives; R future and has resulted in the price rise. Now
Base year price (P0 )
demand would be the cause and price would
W stands for weights, i.e. expenditure incurred on be the effect.
different commodities in the base year. (iv) The correlation may be due to random or chance
factors. Many times correlation is noticed
Correlation between two variables without any real
The term correlation (or co-variation) indicates the relationship between them. It may happen due
relationship between two such variables in which with to chance. This generally happens when a very
changes in the values of one variable, the values of the small sample is chosen from a large universe.
other variable also change. (v) There might be a situation of nonsense or spurious
The utility of the study of correlation is immense correlation between the two variables under study.
both in physical as well as social sciences. The study One may find a high degree of correlation
of correlation reduces the range of uncertainty associated between the number of divorces per year and
with decision-making. In social sciences, particularly in the export of televisions sets. Obviously there
the business world, forecasting is an important cannot be any relationship between divorces
phenomenon, and correlation studies help us to make and television exports and that is why it
relatively more dependable forecasts. should be understood clearly that correlation is
Though the word correlation is used in the sense a relationship between related variables only.
of mutual dependence of two or more variables yet it
is not at all necessary that it should always be so. Even Regression
a very high degree of correlation between two variables Regression is the measure of the average relationship
does not necessarily indicate a cause and effect between two or more variables in terms of the original
relationship between them. There can be correlation units of the data. Regression analysis attempts to
between two variables due to any one or more of the establish the nature of the relationship between variables—
following reasons: that is, to study the functional relationship between the

6 . 128
Basic Numeracy and Data Interpretation

variables and thereby provide a mechanism for The cyclical variations in a series are the recurrent
predicting, or forecasting. variations whose duration is more than one year.
Regression analysis is done for estimating or Though cyclical movements, by and large, are regular,
predicting the unknown value of one variable from the they are not necessarily uniformly periodic.
known value of the other variable. Irregular variations are the effect of random factors.
There may be no cause and effect relationship These are generally mixed up with seasonal and cyclical
between the variables under study and yet they may variations and are caused by irregular and accidental
be correlated. Regression on the other hand presumes one factors like floods, famines, wars, strikes, lockouts, etc.
variable as a cause and the other as its effect. The There is no regular period or time of their occurrence
independent variable is supposed to be affecting the and that is why they are called random or chance
dependent variable and as such we can estimate the fluctuations.
values of the dependent variable by projecting the The analysis of time series consists of two major
relationship between them. steps which are (i) identifying the various forces or
influences whose interaction produces the variations in
Analysis of Time Series the time series, and (ii) isolating, analysing and
Time series refers to such a series in which one variable measuring the effect of these forces separately and
is time. In other words if we have chronologically independently, by holding other things constant.
arranged values of a variable over successive time
periods it would be a time series. Interpolation and Extrapolation
Time series analysis is done primarily for the Statistical data are not always available in continuous
purpose of making forecasts and also for the purpose series in as much detail as necessary. In such situations,
of evaluating past performance.
@UPSC_THOUGHTS

it becomes absolutely essential to estimate the missing


Characteristic movements of time series may be values with utmost care and accuracy in order to
classified into four main types, often called components examine the behaviour of the variable over a period of
of a time series—secular trend, seasonal variations, time.
cyclical variations and irregular variations. Interpolation may be defined as the technique of
The general tendency of the time series data to
estimating the most likely value of a dependent variable
increase or to decrease or to remain segregated during
corresponding to the given value of an independent
a long period of time is called secular trend. Hence
variable for any intermediate period.
secular trend is said to refer to long-term tendency of
Extrapolation may be defined as a process of
a phenomenon.
projecting the most likely future value of a dependent
If the values of the phenomenon under study, when
variable corresponding to the given value of an
plotted on a graph paper cluster more or less around
independent variable. Extrapolation is always related
a straight line, the trend is said to be linear or a straight
to the future whereas interpolation is always concerned
line trend. In a straight line trend the values increase
with the past.
or decrease by a constant absolute amount. In such
The technique of interpolation is needed to fill up
cases the rate of growth {positive or negative) is more
the gaps in statistical data which may be due to the
or less constant. If the plotted points of the values of
following causes:
a phenomenon do not fall in the pattern of a straight
(a) statistical data may have been lost or destroyed
line the trend is said to be non-linear or curvi-linear.
in fire, earthquake, floods or war.
Here the rate of growth is uneven.
(b) statistical data may have been collected with
Seasonal variations refer to such movements in a
insufficient details.
time series due to forces which are rhythmic in nature
and which repeat themselves periodically every season. (c) data may not have been collected for a particular
These variations repeat themselves in less than one- week or month due to certain special causes
year’s time. Seasons could be weekly, monthly, quarterly such as war or natural calamity.
or half-yearly depending on the nature of the (d) it may not be possible to collect data due to
phenomenon. These are the results of such factors which huge amount of expenditure and organisational
uniformly and regularly rise and fall in magnitude. problems.

6 . 129
Basic Numeracy and Data Interpretation

The technique of interpolation is also employed to minimum percentage increase/decrease, or trend of


compute the values of median and mode when the data movement of a variable. Simple arithmetical computation
are expressed in the form of continuous series. should help you arrive at the correct answer. It is a good
The following are the main assumptions behind idea to estimate the numbers that you have to use from
interpolation and extrapolation: a table rather than use them in their printed form.
(i) There are no sudden jumps in the values of Usually, the first and second digit are all that is needed.
the variable from one period to another. In
case of violent changes in the intermediate Worked Examples
period, the technique of interpolation would
result in wrong estimates. ● A student obtained the following marks in a test.
(ii) There is regularity in fluctuation which implies
Subject Marks obtained Maximum marks
that the rise and fall in data are uniform. For
example while forecasting the sales of a Physics 60 100
particular product for the coming year, it is Chemistry 80 100
assumed that the trend of sales will remain the Mathematics 80 100
same and there will be no sudden changes in Hindi 40 50
sales. English 40 50
The graphical method is the simplest method of
What is the aggregate percentage marks obtained
interpolation. It may be employed for estimating the
value of one series on the basis of given values of by the student?
another series particularly in time series, correlated (a) 50 per cent (b) 60 per cent
series or series showing periodicity. (c) 75 per cent (d) 80 per cent
@UPSC_THOUGHTS

According to this method, the independent variable Solution: First total (i) the marks obtained and (ii)
is represented along the horizontal axis and the the maximum marks. You have
dependent variable upon the vertical axis. The values Marks obtained = 300
of the two variables are plotted on the graph paper and Maximum marks = 400
then joined. The resulting line or curve is employed for
Now if from 400, marks obtained are 300, from
estimating the value of the dependent variable
100, they will be:
corresponding to the given value of the independent
300
variable. Aggregate percentage = 100 = 75
400
Interpretation
● The following table shows data on the weight of
Once data is tabularly or graphically presented and various organs of different animals on the basis
analysed, interpretation can be attempted. In the context
of (1) absolute weight of the organ and (2) its
of the present examination, a general awareness of
percentage of total body weight.
statistical principles and ground rules as well as
common sense are enough. It would do well to recall Brain Heart Liver Spleen
the fallacies and misuses that statistics is susceptible
(1) (2) (1) (2) (1) (2) (1) (2)
to in the hands of an ignorant person or a clever
manipulator. You should be able to avoid these fallacies A. 650 g 1.0 3.4 kg 0.7 5.0 kg 1.5 1.2 kg 0.16
yourself and be alert enough to locate any in the data
B. 120 g 0.9 45 g 0.4 2.0 kg 1.7 424 g 0.12
presented for your interpretation.
C. 130 g 1.1 2.4 kg 0.4 700 g 1.5 120 g 0.17
Interpreting From Tables
D. 450 g 1.2 2.5 kg 0.5 0.5 kg 1.2 1.1 kg 0.17
The key to understanding tables is to read the title and
the stubs and footnotes, if any; these will tell you what
the numbers in the table mean. Be alert to the units 1. Which animal has the heaviest brain and
which are used in a table—which may be different from heaviest heart?
the units asked for in the problem. The question (a) D (b) C
generally relate to annual rate of increase, maximum/ (c) B (d) A

6 . 130
Basic Numeracy and Data Interpretation

2. The heaviest liver and lightest heart as a Now you have to find the percentage, which
percentage of body weight is found in is
(a) B (b) A 34
(c) D (d) C 100 = 52.3%
65
Solution: The data can be easily read to arrive at (b) Number of employees earning less than
the answers. Just be careful of the units, and do Rs 100.00 but at least Rs 60.00 per month
not let the plethora of numbers confuse you. = 10 + 14 + 16 + 10 = 50.
1. Look for the animal with the heaviest brain: Percentage of employees earning less than
it is A. Does it also have the heaviest heart?
Yes, its heart weighs 3.4 kg. (Don’t go by Rs 100.00 but at least Rs 60.00 per month
numbers alone or you may think it is B—but 50
= = 76.9%.
the 45 is grams, not kg.) 65
You can choose (d).
2. Column 2 for each animal gives the percentage
● Table shows a frequency distribution of the lifetimes
of organ to body weight. You have to look in
of 400 radio tubes tested at the L&M Tube
the columns for liver first—the heaviest liver
to body weight per cent is that of B. Now look Company.
for the lightest heart to body weight per cent Lifetime (hours) Number of Tubes
in the heart column: both B and C have the
same percentage. But in combination with the 300- 399 14
heaviest liver to body weight percentage, it 400- 499 46
is B. 500- 599 58
@UPSC_THOUGHTS

So (a) is your answer. 600- 699 76


● Table shows a frequency distribution of the 700- 799 68
monthly wages in Rs of 65 employees in the P and 800- 899 62
R company. 900- 999 48
Table 1000- 1099 22
Wages (Rs) Number of Employees 1100- 1199 6
Total 400
50.00 — 59.99 8
60.00 — 69.99 10 Questions (i), (ii) and (iii) are based on the above table.
70.00 — 79.99 16 (i) What is the percentage of tubes whose lifetime
80.00 — 89.99 14 do not exceed 600 hours.
90.00 — 99.99 10 (a) 116 (b) 118
100.00 — 109.99 5 (c) 58 (d) None of these
110.00 — 119.99 2 (ii) Determine the percentage of tubes with lifetime
Total 65 greater than or equal to 900 hours.
(a) 17% (b) 18%
What is
(c) 19% (d) 20%
(a) The percentage of employees earning less than
(iii) What is the percentage of tubes whose lifetime
Rs 80.00 per month.
are at least 500 but less than 1000 hours.
(b) The percentage of employees earning less than
(a) 50% (b) 31.2%
Rs 100.00 but at least Rs 60.00 per month.
(c) 75% (d) 78%
Solution: First find the total number of employees
in the required range of wages. Solution: In each question, read carefully what is
(a) Those earning less than Rs 80 per month will required and choose your data accordingly.
be in the wage classes Rs 50 to Rs 79.99. This (i) Tubes which do not exceed 600 hours have
adds up to 16 + 10 + 8 = 34. to be taken as tubes which have lifetime less

6 . 131
Basic Numeracy and Data Interpretation

than 600 hours: these will be all those with an increase with the increase in number of men per
lifetime from 300 – 599 hours; i.e., sq. km. from 1 to 4. With 5 men/sq. km, whereas
14 + 46 + 58 = 118 the total output shows increase, the average
As you are asked the percentage, you have to product/man becomes stationary indicating that
work it out: peak production has been achieved at the level.
118 Subsequently, the total output/sq. km. shows a
100 = 29.5% slight increase upto 8 men/sq. km. but average
400
production per man decreases. Even total output
As none of the answer choices (a), (b) or (c)
per sq. km. decreases at 10 men/sq. km. From the
gives this, you have to choose (d).
above, it is clear that it is most economical to
(ii) Total tubes with lifetime greater than or equal employ 5 men/sq. km. when total output/ sq. km.
to 900 hours are those that have lifetime shows a considerable increase and the average
between 900-1199 hours; viz., product/man reaches the peak.
48 + 22 + 6 = 76 So answer response (b) is correct.
76
Percentage = 100 = 19% ● Study the following table and answer the question
400
So answer response (c) is correct. below it.
Admission Figures for a University in Arts
(iii) The required number of tubes that have lifetime and Science Faculties (1986 to 1988)
of between 500-999 hours;
1986 1987 1988
viz., 58 + 76 + 68 + 62 + 48 = 312
312 Arts Sc. Total Arts Sc. Total Arts Sc. Total
@UPSC_THOUGHTS

Percentage = 100 = 78%


400 Males 4,805 695 5,500 5,030 870 5,900 5,362 1,128 6,490
The correct answer choice is (d). Females 675 325 1,000 724 396 1,120 759 473 1,232

● The following table gives product of labour (in Total 5,480 1,020 6,500 5,754 1,266 7,020 6,121 1,601 7,722
units of wheat) per square km of land:
Number of men Average product Total output (i) What is the percentage increase of male
per sq. km per sq. km per man students in 1988 as compared to 1986?
1 50 units 50 units (a) 99 (b) 18
2 164 units 82 units (c) 50 (d) 10
3 285 units 95 units Solution: Note the years—1986 and 1988, so skip
4 400 units 100 units the data for 1987.
5 500 units 100 units
6 576 units 96 units First find the increase in male students: the figures
7 630 units 90 units you need are total male students in 1986 and total
8 656 units 82 units male students in 1988; these are 5500 and 6490
9 656 units 73 units respectively. Now find the difference between the
10 640 units 64 units two figures:
Examine the above table. Assuming that land is of 6490 – 5500 = 990
the same type, labour is equal in efficiency and the To find the percentage increase, we will use as base
inputs (e.g., type of implements, fertilisers, seeds, the total male students in 1986. So,
etc.) are the same, the most economical number of
men to be used is 990
Percentage increase = 100 = 18%
(a) 4 (b) 5 5500
(c) 7 (d) 8 Answer response (b) is correct.
Solution: It is clear from the table that total output/ [You have to be careful in the choice of the base
sq. km. as well as average product per man shows while finding the percentage.]

6 . 132
Basic Numeracy and Data Interpretation

● Table showing the number of workers of a A line graph shows trends, generally over a period
factory in the years 2000 and 2005. of time. There may be more than one line, each one
Workers Year 2000 Year 2005
representing a different item. A line that goes up shows
increasing trend, one that goes downwards indicates
Union Non- Total Union Non- Total
Members union Members union
decline and one that is horizontal indicates ‘no change’.
Members Members Two or more variables can be plotted on a graph
provided the unit of measurement is the same.
Men 1175 375 1550 1290 180 1470
Histogram is a very popular method of presenting
Women 25 175 200 220 28 248 frequency distributions. The classes are marked along
Total 1200 550 1750 1510 208 1718 the horizontal, i.e., x-axis. Taking the class interval as
the base, rectangles are erected with heights proportional
(i) What is the increase in the percentage of the to the frequencies of the respective classes. These
female workers belonging to the union in the frequencies are measured along the vertical line, i.e. the
five year period? y-axis. With equal class intervals, all rectangles will
(a) 78% (b) 195% have equal bases. Hence, the area of each rectangle will
(c) 780% (d) 700% be proportional to the frequency in that class.
Solution: The five-year period is 2000-2005. In 2000, If the class intervals are unequal, the width of the
the women workers who belonged to the union rectangles will vary and the heights of the rectangles
were 25. In 2005, the figure went up to 220. The will be proportional to the frequency densities or the
increase is 220 – 25 = 195. adjusted frequencies.

195
@UPSC_THOUGHTS

Percentage increase is 100 = 780%


25
(The base is the figure of women union members
in the earlier year.)
Answer response (c) is correct.
(ii) In 2000, the ratio of men workers to women
workers in the factory was
(a) 21 : 9 (b) 0.88 : 0.11
(c) 31 : 7 (d) 9 : 1
Solution: The figures involved are the total men
workers and total women workers in 2000, which
is 1550 and 200 respectively. Their ratio is
1550 : 200 = 31 : 7
Answer response (c) is correct,.
A frequency polygon is a curve representing a
Interpreting Graphs and Diagrams frequency distribution. A frequency polygon is
constructed from a histogram by joining the midpoints
Graphs of the tops of the different rectangles with straight lines.
Graphs may be geometrical or statistical. Statistical The total area of the frequency polygon is equal to
graphs illustrate comparisons and trends in given data. the total area of all the rectangles put together in a
The fundamental idea about graphs is that they all use histogram.
some distance to represent value. A frequency curve emerges when the points of a
One should read the labels, margins and notes frequency polygon are joined by free hand ‘smoothed’
given carefully. A basic knowledge of economics and curves and not by straight lines. Obviously the area of
Indian economic facts may be necessary to interpret the frequency polygon and of the frequency curve is the
some of the tables and graphs in such examinations. same as that of the histogram.

6 . 133
Basic Numeracy and Data Interpretation

Frequency curves generally take characteristic the curve is pulled towards low-value items which are
shapes as shown in the figures. The symmetrical bell not compensated for by high-value items in the
shaped curve is characterised by the fact that distribution.
observations equidistant from the central maximum A J-shaped curve shows a movement from low
have the same (or near same) frequency. The normal frequencies in low classes to its highest frequency in
curve is a special type of bell-shaped curve. the highest class in the distribution. If we disregard
In the moderately asymmetrical or skewed frequency infant mortality and plot a frequency distribution relating
curves, the tail of the curve to one side of the central to death rates by age, we shall get a J-shaped curve.
maximum is longer than the curve to the other. If the A reverse J-curve has the highest frequencies at the
longer tail occurs to the right the curve is said to be lowest values, gradually diminishing as we move
skewed to the right or have positive skewness. This is towards higher values, i.e., to the right.
because the curve is being pulled towards the high- A U-shaped curve has maxima at both ends. If we
valued items which are not compensated for by the plot unemployment among males we find that
presence of low-value items in the distribution. The employment is at the highest in the earliest working
curve is said to be skewed to the left or to have negative years and at the latest working years.
skewness if the longer tail occurs to the left. In this case A bimodal curve has two maxima. Here it may be
noted that while a U-shaped curve is bimodal, all
bimodal curves need not be U-shaped. A multimodal
curve has more than two maxima.
The graphic representation of cumulative frequency
distribution is the Ogive or Cumulative frequency
@UPSC_THOUGHTS

curve.
Bell Shaped J-Shaped An ogive shows a distribution on a ‘less than’ or
an ‘or more’ basis. The values of the variable are
represented along the horizontal axis while the
cumulated frequencies are plotted upon the vertical
axis.
Ogives help us to compare frequency distributions.
They help us to determine the values of median,
Skewed to the right Reverse J quartiles, deciles etc., graphically. The frequencies or
(Positively skewed) number of values above or below a certain value can
be determined with the help of ogives.

Diagrams
Bar diagrams are one-dimensional diagrams in the
shape of vertical or horizontal bars.
Skewed to the left The thickness of the bars is not relevant to the data.
U shaped
(Negatively skewed) The simple bar diagram is used to represent only one
variable. The values of the variables are shown in the
shape of bars which are of equal width but of varying
heights. The height of the bar represents the value of
a variable, therefore it becomes easy to compare the
values of a variable. The bars are arranged in time
sequence or according to the size of the variable.
Bimodal Multimodal The base may be on the vertical line also, in which
Characteristic Frequency Curves case the bars will be horizontally placed.

6 . 134
Basic Numeracy and Data Interpretation

A bar diagram can be adapted to compare changes


Percentage component bar diagram showing
in more than one variable in which case it is termed families above poverty line in 1983-87
a multiple or compound bar diagram. 100 Key
Sub-divided Bar Diagram are also called 6 or more
90 children
component bar diagrams (or charts); these are used to
5 children
show the breakdown of a total into its component parts. 80
4 children
70 3 children

Percentage of families
2 children
80 60 1 child

70 50
Men 21+
Average daily earnings (Rs.)

40
60 Women 18+ (full time)
30
50
20

40 10

30 0
1983 1984 1985 1986 1987
Year
20

10 Two dimensional diagrams are in the shape of


figures with two dimensions, that is, figures like
0
1980 1981 1982 1983 1984 rectangles, squares or circles. The areas of rectangles
square or circles are in proportion to the size of the
@UPSC_THOUGHTS

Year
items they represent.
In many cases, it is meaningless to compare absolute Though a rectangle would appear similar to a bar
values, and it is better to employ relative values. For diagram, in the latter case only the height is taken into
this purpose, percentage bar diagrams are employed consideration, while in the former it is the area—length
to depict the relative change in the values of a × width—which is taken into account.
variable.
The pie diagram is the most common of two-
Component bar diagram showing dimensional diagrams.
Key
families above poverty line The term pie diagram has been derived from the
6 or more
(1983-87) children word ‘pie’—the diagram looks like a pie and its parts,
5 children
4 children like the slices cut from a pie. A circle is sub-divided
Number of Families (thousands)

70 3 children into various sectors and the diagram shows the


2 children
60 1 child relationship of the parts to the whole. A pie diagram
50
Pie Diagram Showing Expenditure on Various
40
Heads by Family A and Family B
30

20

10

0
1983 1984 1985 1986 1987
Year

The percentage bar diagram tells us nothing about


the total number of families each year but shows the
breakdown into components as a proportion of the
whole.

6 . 135
Basic Numeracy and Data Interpretation

may be defined as a circle divided into sectors by radii, (b) exports have always been less than imports
each sector corresponding in area, arc and angle formed (c) exports have increased steadily
by the radii to the number of frequencies represented. (d) imports were less than exports till 1976-
The angle at the centre of the circle is 360° which 77 but rose steeply after that
represents the total. This angle is sub-divided depending
Solution: Note that uniform increase would be
upon the nature of data to be represented. 1 per cent
is equal to 3.6°. The angle representing each item or shown by a straight line; fluctuation would be
sector may be calculated with the aid of the following indicated by at least one decrease; exports and
formula: increase are same in 1976-77, so (b) cannot be
Value of Sector A × 360 true. Also be careful to follow the correct lines
Angle of Sector A =
Total Value representing the items.
The correct answer is (c).
Worked Examples
While solving any question related to graphs or diagrams, 2. In which year did imports register maximum
look at the entire picture, what units are being used and growth over the preceding period?
what units the question involves. Answer should be in (a) 1974-75 (b) 1977-78
the correct units. Also make sure you get your (c) 1979-80 (d) 1980-81
percentages and decimals clear.
Solution: It is visually established that (d) is the
Do not go beyond what is actually available in the
answer, as the slope for that period is the
data given when drawing inferences, unless you are
specifically asked to apply additional knowledge. Of maximum.
course, you must make use of mathematical formulae
@UPSC_THOUGHTS

and calculations if and when necessary. ● Figure 2 gives details of cereals production in India
over a period of years. Questions 3 and 4 are based
● Study Graph I and answer Questions 1 and 2. on it.
I. Exports and Imports of India from 1974-75
to 1980-81 (Rs Crores)
S
RT
PO

S
RT
IM

O
E XP

Figure 2

3. Which of the following statements can be


concluded from the above diagram?
I. The only cereal to show steady increase
is rice.
Figure 1 II. The trend in cereal crops is one of growth
in production.
1. During the period under review III. Wheat and other cereals show fluctuations
(a) exports and imports have shown uniform in production.
increase

6 . 136
Basic Numeracy and Data Interpretation

(a) I and III (b) I only 5. What was the approximate state expenditure
(c) I, II and III (d) II only per capital in cities having populations of
Solution: A careful observation of the given 2,00,000 to 2,99,000?
multiple bar diagram will help you reach the (a) Rs 100 (b) Rs 150
(c) Rs 180 (d) Rs 200
conclusion. You must be careful to identify the
Solution: The diagram is a subdivided bar
components correctly to compare accurately.
diagram. The middle bar of the seven shown
The trend is the average tendency over a long
represents the cities with population of 2,00,000
period of time and it is seen to be an increasing
and 2,99,000. It reaches about halfway between
one.
100 and 200, so approximate estimate is Rs 150
Take each statement one by one. per capita. Answer response (b) is correct.
I is true; the bars for ‘wheat’ and ‘other cereals’
6. Of the three categories of expenditures which
show a decline in at least one of the years.
was least dependent on city size?
II is also true. The general movement of the line (a) Health (b) Utilities
of growth is positive on the whole. The decline (c) Administration (d) None
is never too large.
Solution: Note that the dark part of each bar
III is true; in fact, we saw it when consider- (the part that represents utilities) varies least
ing I. as city size increases. It shows that even as city
So (c) is the correct answer. size increases, utilities have not shown much
change in expenditure per capita. So, (b) is the
4. In which year did all three crops register a rise
@UPSC_THOUGHTS

correct answer.
in production over the preceding years?
(a) 1972-73 (b) 1973-74 ● Base your answers to question 7 on the following
(c) 1974-75 (d) 1975-76 piecharts in Figure 4.
Solution: Be careful that the increase is to be A B
in all three items—which is there only in 1974-
75 over 1973-74. So (c) is the correct answer. F C C F

● Questions 5 and 6 are based on the diagram in E M M E


Figure 3. R R

State Expenditure Per Capita


Figure 4

7. If F = Food, C = Clothes, R = Rent, E =


Entertainment, M = Misc., which of the following
cannot be said of the figures?
(a) Family A spends more on food than on
entertainment
(b) Family B spends most of its income on
clothes and rent
(c) Both families A and B spend least on
miscellaneous
(d) Family A spends more than Family B on
Figure 3 food

6 . 137
Basic Numeracy and Data Interpretation

Solution: This is a simple question just requiring Family A Family B


step by step elimination based on observation. Total Expenditure Total Expenditure
Rs 20,000 p.a. Rs 1,00,000 p.a.
The question asks what is not correct. (a) (b)
and (c) are conclusions that can be drawn from
the diagrams. Only (d) is not correct: both
families appear to spend the same amount on
food.

● The pie charts in Figure 5 give the characteristics


of foreign tourists in one year.

American Below 20 years


60% 60%
Figure 6
si an ee n
Rus 0% British Be tw 4 0 Above
1 20 -
40 years
9. From these diagrams we can conclude that
Others 15% a r s
ye (a) Family A spent more money on Food than
15% 20% 20%
Family B
Tourists Countrywise American Tourists in (b) Family B spent more money on Food than
100,000 = 100% Age Groups Family A
@UPSC_THOUGHTS

Figure 5 (c) Family A and Family B spent same amount


of money on Food
8. From this chart, the number of American
(d) The expenditure on Food by Family A and
tourists in the age group 20-40 who visited
Family B cannot be compared
during the year, is
Solution: These are percentage bar diagrams.
(a) 12,000 (b) 20,000
(c) 40,000 (d) 60,000 For Family A the break-up of total expenditure
is as follows:
Solution: Both pie diagrams must be considered Food 50% of 20,000 = Rs 10,000
together. 20% American tourists are between Other Items 30% of 20,000 = Rs 6,000
20-40 years and Total American tourists are Education 20% of 20,000 = Rs 4,000
60% of 100,000 = 60,000. Therefore, American
For Family B, the break up is as follows:
tourists between 20-40 years are 20% of 60,000
Food 10% of 1,00,000 = Rs 10,000
= 12,000. Answer response (a) is correct.
Other items 60% of 1,00,000 = Rs 60,000
● The expenditure on the various items by two Education 30% of 1,00,000 = Rs 30,000
families A and B are represented in Figure 6. The answer is now clear: (c) is correct.

6 . 138
Basic Numeracy and Data Interpretation

PRACTICE EXERCISES
1. The sum of prime numbers lying between 60 and 75, 10. A number when divided by the sum of 555 and 445
is gives two times their difference as quotient and 30
(a) 199 (b) 201 as the remainder. The number is
(c) 211 (d) 272 (a) 1220 (b) 1250
2. There are four prime numbers written in ascending (c) 22030 (d) 220030
order. The product of the first three is 385 and that of 11. 998 × 1002 = ?
the last three is 1001. The last number is (a) 99996 (b) 999996
(a) 11 (b) 13 (c) 999994 (d) 2000
(c) 17 (d) 19 12. 212 × 188 is equal to
3. If we write all the whole numbers from 200 to 400, (a) 38956 (b) 35986
then how many of these contain the digit 7 once and (c) 39856 (d) 39866
only once? 13. The difference between the squares of two consecu-
(a) 32 (b) 34 tive numbers is 35. The numbers are
(c) 35 (d) 36 (a) 14, 15 (b) 15, 16
4. If n is a natural number, then n is (c) 17, 18 (d) 18, 19
(a) always a rational number 14. The sum of two numbers is twice their difference. If
(b) always a natural number one of the numbers is 10, the other number is
(c) always an irrational number 1
@UPSC_THOUGHTS

(d) sometimes a natural number and sometimes (a) 3 (b) 30


3
an irrational number 1 1
(c) 30 or  3 (d) 30 or 3
5. Which one is a true statement among the following 3 3
options? 15. The largest number of five digits exactly divisible by
(a) 1 is prime 77 is
(b) 1 is prime and composite (a) 99946 (b) 99956
(c) 1 is composite (c) 99964 (d) 99965
(d) 1 is neither prime nor composite 16. The number 23*7 is divisible by 3. The missing digit
6. The sum of all prime numbers between 61 and 89 is (*) is
(a) 610 (b) 523 (a) 1 (b) 2
(c) 460 (d) 373 (c) 3 (d) 4

7. Which of the following numbers is exactly divisible 17. The difference between the largest 4 digit number
by 24? and the smallest 3 digit number is
(a) 35718 (b) 63810 (a) 9899 (b) 8999
(c) 537804 (d) 3125736 (c) 9989 (d) 9889

8. In a sum involving division, the divisor is 12 times 18. On dividing 59761 by a certain number, the quotient
the quotient and 5 times the remainder. If the remain- is 189 and the remainder is 37. The divisor is
der is 48, then what would be the dividend? (a) 517 (b) 316
(a) 2147 (b) 3100 (c) 711 (d) 418
(c) 3694 (d) 4848 19. The sum of three consecutive odd numbers is al-
9. A student was asked to divide a number by 6 and ways divisible by
add 12 to the quotient. He, however, first added 12 to (a) 2 (b) 3
the number and then divided it by 6, getting 112 as (c) 5 (d) 7
the answer. The correct answer should have been 20. The sum of first 45 natural numbers is
(a) 143 (b) 138 (a) 1035 (b) 1280
(c) 122 (d) 136 (c) 2070 (d) 2140

6 . 139
Basic Numeracy and Data Interpretation

21. A number when divided by 114 leaves the remain- 29. The smallest five digit number exactly divisible by
der 21. If the same number is divided by 19, then the 476 is
remainder will be (a) 10000 (b) 10472
(a) 1 (b) 2 (c) 10476 (d) 47600
(c) 7 (d) 21 30. 997 × 997 is equal to
22. The difference between two numbers is 1365. When (a) 9994009 (b) 994000
the larger number is divided by the smaller one, the (c) 994009 (d) 9949
quotient is 6 and the remainder is 15. The smaller 31. The least number of five digits exactly divisible by
number is 567 is
(a) 240 (b) 270 (a) 10206 (b) 10702
(c) 295 (d) 360 (c) 10361 (d) 10567
23. While solving a question involving division with 32. What least number must be subtracted from 1000 to
zero remainder, a candidate took 12 as divisor in- get a number exactly divisible by 17?
stead of 21. The quotient obtained by him was 35. (a) 21 (b) 17
The correct quotient should have been (c) 14 (d) 9
(a) 0 (b) 12 33. The number which is nearest to 3006 and exactly
(c) 13 (d) 20 divisible by 29 is
24. The sum of three consecutive odd numbers is al- (a) 3042 (b) 3031
ways divisible by (c) 3024 (d) 3016
I. 2 II. 3 34. The largest number of four digits exactly divisible
@UPSC_THOUGHTS

III. 5 IV. 6 by 88 is
(a) Only I (b) Only II (a) 9768 (b) 8888
(c) I and III (d) II and IV (c) 9988 (d) 9944

25. If the number 357*25* is divisible by both 3 and 5, 35. 128 players start in the men’s singles at a tennis
then the missing digits in the unit’s place and the tournament, where this number reduces to half on
thousandth place respectively are every succeeding round. How many matches are
played totally in the event?
(a) 0, 6 (b) 5, 6
(a) 60 (b) 83
(c) 5, 4 (d) None of these
(c) 127 (d) 153
26. The smallest number that must be added to 803642
36. A boy was asked to multiply a certain number by 53.
in order to obtain a multiple of 11 is
He multiplied it by 35 and got his answer less than
(a) 1 (b) 4
the correct one by 1206. Find the number to be multi-
(c) 7 (d) 9
plied.
27. The number which is nearest to 457 and is exactly (a) 68 (b) 67
divisible by 11 is (c) 77 (d) 88
(a) 450 (b) 451 37. A number is divided successively in order by 4, 5
(c) 460 (d) 462 and 6. The remainders are respectively 2, 3 and 4.
28. The owner of a local jewellery store hired 3 watch- The number is
men to guard his diamonds, but a thief got in and (a) 214 (b) 476
stole some diamonds. On the way out, the thief met (c) 954 (d) 1908
each watchmen, one at a time. To each he gave half 38. A number when divided successively by 4 and 5
of the diamonds he had taken, besides 2 more. He leaves remainders 1and 4 respectively. When it is
escaped only with one diamond. How many did the successively divided by 5 and 4, then the respective
thief originally steal ? remainders will be
(a) 16 (b) 26 (a) 1, 2 (b) 2, 3
(c) 36 (d) 46 (c) 3, 2 (d) 4, 1

6 . 140
Basic Numeracy and Data Interpretation

39. The HCF and LCM of two numbers are 18 and 3780 48. Three men start together to travel the same way
respectively. If one of them is 540, then the other num- around a circular track of 11 km. Their speeds are 4,
ber is 5.5 and 8 km per hour respectively. They will meet
(a) 142 (b) 126 after
(c) 118 (d) 112 (a) 10 hours (b) 20 hours
(c) 22 hours (d) 28 hours
40. Three measuring rods are 64 cm, 80 cm, and 96 cm
49. There are three consecutive road crossings at which
in length. The least length of cloth that can be mea-
traffic lights change after every 48 seconds, 72 sec-
sured exact number of times using anyone of the
onds and 108 seconds respectively. If the lights
above rods is change simultaneously at 8 : 20 : 00 hours, then at
(a) 0.96m (b) 19.20m what time will they again change simultaneously?
(c) 9.60m (d) 96.00m (a) 8 : 27 : 12 hours (b) 8 : 27 : 24 hours
41. Greatest number which when subtracted from 10,000 (c) 8 : 27 : 36 hours (d) 8 : 27 : 48 hours
so that the remainder may be divisible by 32, 36, 48 50. A, B and C start at the same time in the same direc-
and 54 is tion to run around a circular stadium. A completes a
(a) 9,136 (b) 9,250 round in 252 seconds, B in 308 seconds and C in 198
(c) 8,478 (d) 8,784 seconds, all starting at the same point. After what
time will they meet again at the starting point?
42. In a school, 391 boys and 323 girls have been di- (a) 46 minutes 5 seconds
vided into the largest possible classes, so that each (b) 46 minutes 9 seconds
class of boys numbers the same as each class of girls. (c) 46 mintues 12 seconds
The number of classes is
@UPSC_THOUGHTS

(d) 46 minutes 16 seconds


(a) 5 (b) 17 51. A rectangular courtyard 3.78 metres long and 5.25
(c) 37 (d) 53 metres wide is to be paved exactly with square tiles,
43. The HCF and LCM of two numbers are 27 and 2079 all of the same size. What is the largest size of the tile
respectively. If the first number is 189, the other num- which could be used for the purpose?
ber is (a) 24cm (b) 23cm
(c) 22cm (d) 21 cm
(a) 287 (b) 297
(c) 278 (d) 279 52. Find the maximum number of students among
whom 456 bottles of soft drinks and 874 pieces of
44. If the LCM and HCF of two numbers are 2400 and 16
chocolates can be equally distributed?
and if one number is 80, the other number is (a) 38 (b) 51
(a) 460 (b) 470 (c) 57 (d) 62
(c) 480 (d) 490
53. The greatest and the smallest four-digit numbers
4 10 20 which are exactly divisible by 3, 4 and 5 are
45. The HCF of , and is
9 21 63 (a) 9999 and 1000 (b) 9980 and 1010
2 2 (c) 9970 and 1010 (d) 9960 and 1020
(a) (b)
63 9 54. Five bells beginning together toll at intervals 4, 5, 7,
10 5 8 and 10 seconds respectively. After what interval of
(c) (d)
63 63 time will they toll together?
(a) 4 minutes 40 seconds
46. If the HCF of 456 and 874 is 38, then the LCM is
(b) 4 minutes 30 seconds
(a) 665 (b) 10844
(c) 4 minutes 20 seconds
(c) 10488 (d) 10456 (d) 4 minutes 10 seconds
47. The smallest sum of money which contains 55. The least number of square tiles required for a ter-
Rs 2.50, Rs 20, Rs 1.20 and Rs 7.50 is race 23m 80cm long and 8m 16cm wide is
(a) Rs 100 (b) Rs 80 (a) 420 (b) 410
(c) Rs 60 (d) Rs 40 (c) 340 (d) 310

6 . 141
Basic Numeracy and Data Interpretation

56. Three different containers contain milk of different 63. Which of the following is correct?
quality like Mother Dairy, Milma and Parag respec- 5 3 2 8 5 2 3 8
tively whose measurements are 280 litres, 315 litres (a)    (b)   
7 9 3 11 7 3 9 11
and 385 litres respectively. What biggest measure 8 5 2 3 8 2 3 5
should be used to measure all different qualities an (c)    (d)   
11 7 3 9 11 3 9 7
exact number of times?
3 7 8
(a) 15 (b) 20 64. The value of   is
(c) 30 (d) 35 1 10 100
(a) 3.078 (b) 3.78
57. Which one of the following sets of fractions is in the (c) 3.87 (d) 3.087
correct sequence of ascending order of values? 65. Value of 2.5 ÷ 0.0005 is
1 5 4 3 5 3 (a) 5 (b) 50
(a)  , ,  (b)  ,  ,
2 6 9 7 6 5 (c) 500 (d) 5000
1 4 5 4 5 1
(c)  ,  , (d)  , , 66. The value of (2 + 0.2 + 0.02 + 0.002) is
2 9 6 9 6 6
(a) 2.002 (b) 2.022
58. Among the following fractions the largest fraction is (c) 2.006 (d) 2.222
10
(a) (b) 0.91  0.1 0.1 0.1  0.02  0.02 0.02 
11 67. The value of   is
7.3 8.3  0.2  0.2 0.2  0.04  0.04  0.04 
(c) (d) (a) 1.25 (b) 0.125
8 9
(c) 0.0125 (d) 0.00125
59. A decimal number has 16 decimal places. The num-
ber of decimal places in the square root of this num- 68. Given 168 × 32 = 5376, then 5.376 ÷ 16.8 is equal to
@UPSC_THOUGHTS

ber will be (a) 0.32 (b) 320


(a) 2 (b) 4 (c) 32 (d) 3.2
(c) 8 (d) 16 69. The sum of 2.75 and 3.78 is
60. Which of the following is the smallest fraction? (a) 1.03 (b) 1.53
1 1 9 500 (c) 4.53 (d) 5.53
, , ,
10 100 1000 10,000 70. Write the ascending order of the following rational
1 1 numbers.
(a) (b)
10 100 7 5 2
, ,
9 500 10 8 3
(c) (d)
1000 10000 2 5 7 5 7 2
(a)   (b)  
61. What is the difference between the biggest and the 3 8 10 8 10 3
2 3 4 5 7 2 5 7 5 2
smallest fraction among , , and ? (c)   (d)  
3 4 5 6 10 3 8 10 8 3
1 1
(a) (b) 71. The fraction for the recurring decimal 0.535353 is
30 20
1 26 28
1 (a) (b)
(c) (d) 53 53
12 6
53 27
62. Which of the following is correct? (c) (d)
99 53
5 9
I.  72. What is the least fraction which when added to or
7 13
29 15
11 14 subtracted from  will make the result a whole
II.  12 16
13 17 number?
2 5 17 31
III. (a) (b)
9 6 48 48
(a) Only I (b) Only II 31 21
(c) (d)
(c) I and III (d) II and III 38 38

6 . 142
Basic Numeracy and Data Interpretation

10 12 ? (25 7)  19 8
73.    16 82. = ........
3 5 4 243 (10 9  9)
(a) 6 (b) 2 (a) 109 (b) 103
(c) 8 (d) 4 (c) 107 (d) 2.5
? 54
1 7  3 2  2  83. 
74.        ? 169 39
7 9  9 9  9  (a) 108 (b) 324
1 3 (c) 2916 (d) 4800
(a) (b) 1.21 0.9
9 7
84. The value of is
2 1 1.1 0.11
(c) (d) (a) 2 (b) 3
9 7
1 (c) 9 (d) 11
 4  20 1
75. 2 ? 85. The value of 8 3 is
1
 4  20 (a) 2 (b) 4
2
81 3 (c) 2 (d) 8
(a) (b) 2
88 11 x 21
86. If  , find the value of x.
161 144 36
(c) (d) 1
176 (a) 1296 (b) 441
76. 0.07 × 0.008 × 0.2 =? (c) 196 (d) 49
(a) 0.000112 (b) 0.00056 ?
87. = 0.02
@UPSC_THOUGHTS

(c) 0.00112 (d) 0.0112 200


(a) 0.4 (b) 4
77. (0.6  0.7 0.8  0.3) gives
(c) 16 (d) 1.6
3 33
(a) 2 (b) 2 88. 0.0009  0.01 = ?
10 100
2
(a) 3 (b) 0.3
(c) 2 (d) 2.35 1
3 (c) (d) None of these
3
78. 80.40 ÷ 20 – (–4.2) = ? 89. If 12276 ÷155 = 79.2 the value of 122.76 ÷ 15.5 = ?
(a) 497.8 (b) 5.786
(a) 7.092 (b) 7.92
(c) 947.00 (d) 8.22
(c) 79.02 (d) 79.2
79. It being given that 15  3.88 , the best approxima- 5.7  5.7  5.7  2.3  2.3  2.3
90. ?
5 5.7  5.7  2.3  2.3  5.7  2.3
tion to is
3 (a) 2.3 (b) 3.4
(a) 0.43 (b) 1.89
(c) 5.7 (d) 8.0
(c) 1.29 (d) 1.63
(0.87)3  (0.13)3
1 91. The simplification of
80. What fraction must be subtracted from the sum of (0.87)2  (0.13)2  (0.87)  (0.13)
4 yields the result
1 1
and to have an average of of all 3 fractions? (a) 0.13 (b) 0.74
6 12
1 1 (c) 0.87 (d) 1
(a) (b)
2 3 1.04 1.04  1.04 0.04  0.04 0.04
92. is
1 1 1.04 1.04 1.04  0.04 0.04 0.04
(c) (d)
4 6 (a) 0.001 (b) 0.1
1 1 1 (c) 1 (d) 0.01
81. If  added to of a certain sum is 7000, then the
93. 0.778 0.778  0.222 0.222 = ?
6 9 2
sum is
0.556
(a) 7000 (b) 9000 (a) 3 (b) 1
(c) 10,000 (d) 11,000 (c) 2 (d) 4

6 . 143
Basic Numeracy and Data Interpretation

(0.82)2 (0.284)2 (0.795)2


94. =? 2 1
(8.2)2 (2.84)2 (7.95)2 104. =?
(a) 100 (b) 0.01 2 1
(c) 10 (d) 0.1 (a) 0.732 (b) 1.3142
95. Find the value of (c) 2.4142 (d) 0.3652
0.05  0.05  0.05  0.04  0.04  0.04 105. The greatest number in the following is
0.05  0.05  0.05  0.04  0.04  0.04 (a) 6 3 5 (b) 832
(a) 0.0002 (b) 0.09
(c) 2 3 130 900 (d) 3
(c) 0.009 (d) 0.41 x y ab
106. If a = and b = then is equal to
xy xy ab
1 1 1
96. 2  3  4 = ? x  y 
2
2 3 4  
(a) x  y (b) xy 
1 5  
(a) 11 (b) 5
12 6
xy x2  y2
1 1 (c) 2 2 (d)
(c) 9 (d) 10 x y xy
3 12
107. If the numerator and the denominator of a proper
97. If 15625 = 125, then 15625  156.25 fraction are increased by the same quantity, then the
 1.5625 = ? resulting fraction is
(a) 1.3875 (b) 13.875 (a) always greater than the original fraction
(c) 138.75 (d) 156.25 (b) always less than the original fraction
(c) always equal to the original fraction
@UPSC_THOUGHTS

98. A number of men went to a hotel and each spent as


(d) none of the above
many rupees as there were men. If the money spent
 1   1   1 
was Rs 15,625, find the number of men. 108. The value of 1   1   1  
 x  1  x 2  x 3
(a) 115 (b) 125  1 
1   is
(c) 135 (d) 130  x  4
1 1
99. The value of 0.000064 is (a) 1 
3
(b)
x 5 x 5
(a) 0.1 (b) 0.2
1 x 5
(c) 0.4 (d) 0.6 (c) x  (d)
3
x 5 x 1
 8 4
100. The value of  3  16  is equal to 109. The difference between the squares of the two con-
 625 9 
 4 secutive odd integers is always divisible by
4   6
(a) (b) (a) 3 (b) 5
35 75
(c) 8 (d) 11
8 9
(c) (d)
125 145 110. More than half of the members of a club are women.
101. The largest 3-digit number which is a perfect square 4 7
If of the women and of the men in the club
is 7 11
attended the meeting, then what is the smallest num-
(a) 999 (b) 998
ber of members that the club could have?
(c) 990 (d) 961 (a) 18 (b) 22
102. The least number to be subtracted from 1298 to make (c) 25 (d) 39
it a perfect square is 111. The cost of telephone calls in an industrial town is
(a) 2 (b) 8 30 paise per call for the first 100 calls, 25 paise per
(c) 14 (d) 20 call for the next 100 calls, and 20 paise per call for
103. The largest number in the sequence 1, 2 1/2, calls exceeding 200. How many calls can one make
31/3, 41/4 is for Rs 50?
(a) 1 (b) 21/2 (a) 175 (b) 180
(c) 31/3 (d) 41/4 (c) 200 (d) 225

6 . 144
Basic Numeracy and Data Interpretation

25 x 123. A candidate who gets 30% of the marks fails by 50


112. If 1   1  , then x = ?
marks. Another candidate who gets 320 marks fails
144 12
(a) 1 (b) 2 by 30 marks. The maximum marks is
(c) 5 (d) 7 (a) 1000 (b) 900
(c) 800 (d) 700
113. If 2n = 64, then the value of n = ?
124. Anand donated 5% of his income to a charitable
(a) 2 (b) 4
organisation and deposited 20% of the remainder in
(c) 6 (d) 12
a bank. Now, if he has Rs 1919 left, his total income
5 1 5 1 was
114. If a = and b = , then the value of
5 1 5 1 (a) Rs 2500 (b) Rs 2525
(c) Rs 2558.60 (d) Rs 2570
a2  ab  b2
a2  ab  b2 125. In an examination, 85% students passed in Math-
1 ematics; 15 students failed in the paper and 15 stu-
(a) 1 (b) dents were given a chance for re-test. Find the total
4
1 4 number of students.
(c) (d)
2 3 (a) 115 (b) 200
2
 1 1 (c) 170 (d) 225
115. If  a   = 3, then the value of a3  is
 a a3 126. A owns a house worth Rs 10,000. He sells it to B at a
(a) 0 (b) 3 3 profit of 10% based on the worth of the house. B sells
the house back to A at a loss of 10%. In this transac-
(c) 6 3 (d)
10 3 tion, A gets
3 (a) a profit of Rs 2000
@UPSC_THOUGHTS

(b) a profit of Rs 1100


116. The value of 2 2 2 2 is ?
(c) a profit of Rs 1000
(a) 0 (b) 2 (d) no profit no loss
(c) 215/16 (d) 231/32 127. The price of an article is cut by 10%. To restore it to
117. If 42% of a given number is 12.6, then the number is the former value, the new price must be increased by
(a) 30 (b) 40 1
(a) 10% (b) 9 %
(c) 35 (d) 45 11
1
118. Two numbers are less than a third number by 30% (c) 11 % (d) 11%
9
and 37% respectively. How much per cent is the sec- 128. The salary of a salesperson for two months is in the
ond number less than the first? ratio 2 : 3. By what % was the salary (i) of the second
(a) 3% (b) 4% month more than the first month? (ii) of the first month
(c) 7% (d) 10% less than the second month respectively?
119. The number which exceeds 20% of it by 38 is 1 1
(a) 50% and 33 % (b) 33 % and 50%
(a) 47.5 (b) 46 3 3
(c) 7.6 (d) 5 (c) 50% and 25% (d) None of these

120. An ice-cream seller sells 60% of ice-cream cups and 129. Two positive numbers x and y are related in such a
still has 300 ice-cream cups left with him. The total manner that a% of x plus b% of y is equal to c% of x
number of cups he initially had was plus d% of y. If b  d, then the percentage of y to that
(a) 750 (b) 600 of x will be:
(c) 540 (d) 450 100 (a  c) 100 (c  a)
(a) (b)
121. If 150% of a certain number is 80, what will be 60% d b d b
of that number? 100 (a  c ) 100 (a  c)
(c) (d)
(a) 80 (b) 66 d b b d
(c) 49 (d) 32 130. Passing marks in an examination is 36%. If a stu-
122. Gayatri’s salary is increased from Rs 5000 to Rs dent gets 72 marks and fails by 18 marks, then the
6300. Find the percentage of increase in her salary. maximum mark in the examination would be
(a) 11% (b) 20% (a) 100 (b) 200
(c) 26% (d) 30% (c) 250 (d) 400

6 . 145
Basic Numeracy and Data Interpretation

131. The population of a village is 7986. During last three 140. If the diameter of a sphere is doubled, then the sur-
years, the population increased at the rate of 10% face area is increased by
per year. The population before three years was (a) 100% (b) 200%
(a) 5000 (b) 5500 (c) 300% (d) 400%
(c) 6000 (d) 6600 141. The price of an article was increased by p%. Later
132. In a college election, a candidate secured 62% of the the new price was decreased by p%. If the latest price
votes and is elected by a majority of 144 votes. The is Re 1, the original price was
total number of votes polled is 1  p2
(a) 600 (b) 800 (a) Re 1 (b)
100
(c) 925 (d) 1200 10000
1  p2
133. In a city, 40% of the people are illiterate (c) (d) 10000  p2
100
and 60% are poor. Among the rich 10% are illiterate.
142. In an examination, 70% candidates passed in En-
What percentage of the poor population is illiterate?
glish and 65% in Mathematics. If 27% of the candi-
(a) 36% (b) 40%
dates failed in both the subjects, and 248 passed the
(c) 60% (d) 75%
examination, the total number of candidates who
134. The population of a town increases 4% annually appeared in the examination were
but is also decreased annually by emigration to the (a) 400 (b) 348
extent of 0.5%. What will be percentage of increase (c) 420 (d) 484
in population in 3 years?
(a) 9.8 (b) 10 143. In an election, a candidate who got 40% of total votes
(c) 10.5 (d) 10.8 polled was defeated by the only other candidate by
200 votes. How many votes were polled for success-
@UPSC_THOUGHTS

135. On decreasing the price of a car by 30%, its sale is ful candidate?
increased by 20%. What is the effect on revenue re- (a) 12,000 (b) 10,000
ceived by shopkeeper?
(c) 9,950 (d) 600
(a) 10% increase (b) 10% decrease
(c) 16% increase (d) 16% decrease 144. A woman had a certain sum of money. If she gives
10% to her husband, 20% of the rest to her daughter
136. Water tax is increased by 20% but its consumption
and 20% of the rest to her son, she has Rs 1,32,480
is decreased by 10%. Then the increase or decrease
left with her. Find the total amount of money she
in the expenditure of the money is
had.
(a) 10% decrease (b) 10% increase
(a) 2,20,000 (b) 2,30,000
(c) 8% decrease (d) 8% increase
(c) 2,40,000 (d) 2,50,000
137. In an examination, 52% candidates failed in English,
145. A man used to save 25% of his earnings. Then his
42% failed in Mathematics and 17% failed in both.
The number of students who passed in both the sub- income was increased by 20% and he increased his
jects were expenditure by 10%. Find the % hike in his savings.
(a) 23% (b) 35% (a) 45% (b) 49%
(c) 25% (d) 40% (c) 50% (d) 55%

138. In an examination there were 1500 candidates, out 146. A trader dealing in pressure cookers reduced the
of which 920 were girls and the rest were boys. If price by 20% as a result of which his sale went up by
50% of the girls and 35% of the boys passed the exam, 80%. What was the net effect on his sales income?
then the total percentage of candidates who failed is (a) 33% increase (b) 33% decrease
(a) 58.5% (b) 15% (c) 44% increase (d) 44% decrease
(c) 10% (d) 55.8% 147. In a certain examination, 40% of the students failed
139. If the side of a square is increased by 25%, then by in mathematics, 30% failed in English and 10% failed
how much per cent would the area of it get increased in both. The percentage of students who passed in
by? both the subjects is
(a) 56.25 (b) 50 (a) 20 (b) 40
(c) 125 (d) 156.25 (c) 30 (d) 10

6 . 146
Basic Numeracy and Data Interpretation

148. Sohan’s wages were decreased by 40%. Again the 157. The average temperature of first 3 days is 27° and of
reduced wages were increased by 40%. He has a the next 3 days is 29°. If the average of the whole
(a) loss of 16% (b) gain of 16% week is 28.5 °C, the temperature on the last day was
(c) loss of 10% (d) gain of 10% (a) 31.5° (b) 10.5°
149. The price of cooking oil has increased by 25%. The (c) 21° (d) 42°
percentage of reduction that a family should effect 158. In a primary school, there are 60 boys of age 12 each,
in the use of cooking oil so as not to increase the 40 of age 13 each, 50 of age 14 each and another 50 of
expenditure on this account is age 15 each. The average age of the boys of the school
(a) 15% (b) 20% is
(c) 25% (d) 30% (a) 13.5 (b) 13
150. Two candidates contested an election. One secured (c) 13.45 (d) 14
63% of the votes and won the election by a margin of 159. The average age of a class was 16 years. But 4 boys
32500 votes. Find the number of total votes polled. whose average age was 13 years 6 months were ad-
(a) 12750 (b) 125000 mitted. The class average became 15 years 7 months.
(c) 128200 (d) 127950 How many boys were there after the new admis-
151. The average age of 30 students in a class is 15 years. sion?
If 6 students of this class have the average age of 16 (a) 24 (b) 16
years, then the average age of the remaining 24 stu-
(c) 21 (d) 29
dents would be
(a) 14 years 160. A batsman has a certain average runs for 10 innings.
(b) 14 years and 6 months In the 11th innings, he scored 50 runs thereby in-
@UPSC_THOUGHTS

(c) 14 years and 9 months creasing his average by 4 runs. What was his aver-
(d) 15 years and 3 months age after the 11th innings?
152. The average weight of a group of 150 students in a (a) 11 (b) 8
class is 60 kg. If the mean of the weights of all the 50 (c) 10 (d) 9
boys in the class is 70 kg, then the average weight of
the 100 girls in the class is 161. If a car travels from A to B at 60 km/hr and returns to
(a) 65kg (b) 50kg A from B at a speed of 40 km/hr, the average speed of
(c) 60kg (d) 55 kg the car is
153. The average of first 35 numbers is (a) 45 km/hr (b) 50 km/hr
(a) 24.2 (b) 21 (c) 48 km/hr (d) 55 km/hr
(c) 18 (d) 17.5 162. A man had 5 children. When their average age was
154. The average of 9 results is 40. If the average of first 5 10 years, the child who was 5 years of age died.
results is 39 and that of last 5 results is 42, then the What will be the average age of the surviving chil-
5th result is dren four years after the death of the child?
(a) 32 (b) 38 1 3
(c) 40 (d) 45 (a) 15 years (b) 14 years
4 4
155. A student bought 10 pens of Rs 15 each, 12 note- (c) 15 years (d) 14 years
books of Rs 22 each and 4 books of Rs 250 each. The
average value of the stationary is 163. What is the average of n, n + 1, n + 2, n + 3, n + 4 and
(a) Rs 50 (b) Rs 54.38 n + 5?
(c) Rs 57.46 (d) Rs 59.53 (a) 6n + 15 (b) n + 2
156. Seven men agree with the eighth man to subscribe a 1 1
(c) n + 3 (d) n+ 2
sum of money for a chit fund. The first seven are to 2 2
subscribe Rs 50 each and eighth man Rs 70 more 164. Three years ago, the average age of A and B was 18
than the average of all the eight. Find how much did years. With C joining them, the average age becomes
the eighth man subscribe? 22 years. How old is C now?
(a) 130 (b) 122 (a) 20 years (b) 24 years
(c) 119 (d) 114 (c) 10 years (d) 15 years

6 . 147
Basic Numeracy and Data Interpretation

165. The average height of 40 students is 163cm. On a 173. The average age of husband and wife was 23 years
particular day, three students A, B and C were ab- when they were married 5 years ago. The average
sent and the average of the remaining 37 students age of the husband, the wife and a child, who was
was found to be 162 cm. If A and B have equal height born during the interval, is 20 years now. The present
and the height of C is 2cm less than that of A, then
age of the child is
find the height of A, B and C.
(a) 1 year (b) 2 years
(a) 142 cm, 142 cm, 140 cm
(b) 176 cm, 176 cm, 174 cm (c) 3 years (d) 4 years
(c) 157 cm, 157 cm, 155 cm 174. The average age of 10 men is increased by 2 years
(d) 180 cm, 180 cm, 178 cm when two of them whose ages are 20 years and 22
166. A motorist completes the journey between A and B at years are replaced by two new men. The average age
a constant speed of 20 kmph and covers the return of the two new men is
journey from B to A at a constant speed of 30 kmph. (a) 31 years (b) 33 years
What was his average speed? (c) 35 years (d) 40 years
(a) 24 kmph (b) 21 kmph
175. The average temperature of Monday to Wednesday
(c) 25 kmph (d) 20 kmph
was 37 °C and that of Tuesday to Thursday was
167. The average weight of A and B is 42 kg while the 5
average weights of A, B and C is 44 kg. If the average 34 °C. If the temperature on Monday was th of
4
weight of B and C is 43 kg, find the weight of B. that on Thursday find the temperature of Monday.
(a) 36 kg (b) 38 kg (a) 30° (b) 35°
(c) 39 kg (d) 42 kg (c) 40° (d) 45°
168. The average monthly expenditure for Vijayan is Rs
@UPSC_THOUGHTS

176. In what ratio should Kavita and Kumud divide a


2300 for the first three months, Rs 2100 for the next 5 profit of Rs 312 if their investments are Rs 500 and
months and Rs 2000 for the last 4 months in a year. Rs 300 respectively?
If he managed to save Rs 9400 during the year what
(a) 3 : 5 (b) 5 : 3
is his average monthly income?
(c) 4 : 4 (d) 1 : 3
(a) Rs 2900 (b) Rs 2800
1
(c) Rs 2700 (d) Rs 2600 177. A, B and C enter into a partnership. A contributes
3
169. Out of three numbers, the first is twice the second of the capital while B contributes as much as A and
and is half of the third. If the average of 3 numbers is C together contribute. If the profit at the end of the
56, then the three numbers in order are year amounts to Rs 840, what would A receive?
(a) 48, 96, 24 (b) 48, 24, 96 (a) 280 (b) 420
(c) 96, 24, 48 (d) 96, 48, 24 (c) 120 (d) 600
170. The average age of 3 boys is 15 years. If their ages are 178. The ratio between the ages of Gayatri and Savitri is 6
in ratio 3 : 5 : 7, the age of the youngest boy is
: 5, and the sum of their ages is 44 years. The ratio of
(a) 21 years (b) 18 years
their ages after 8 years will be
(c) 15 years (d) 9 years
(a) 5 : 6 (b) 7 : 8
171. For ten hours, a train travels at a constant speed of (c) 8 : 7 (d) 14 : 13
20 miles per hour and during the next 15 hours, it
travels 240 miles. What is the average speed of the 179. The ratio between the speed of walking of A and B is
train for the whole journey? 3 : 2. If the time taken by B to cover a certain distance
(a) 8.4 miles (b) 11.12 miles is 24 minutes, how much time will be taken by A to
(c) 14.8 miles (d) 17.6 miles cover the same?
172. What is the average speed of a train running at the (a) 12 minutes (b) 16 minutes
rate of 30 kmph during the first 100 km; at 40 kmph (c) 15 minutes (d) 18 minutes
during the second 100 km; and at 50 kmph during
180. Which of the following ratios is the largest?
the third 100 km?
(a) 40 kmph (b) 42.5 kmph (a) 7 : 15 (b) 15 : 23
(c) 38.3 kmph (d) 36.8 kmph (c) 17 : 25 (d) 21 : 29

6 . 148
Basic Numeracy and Data Interpretation

181. Arshad and Prahlad start a business with If at the end of the year, both save Rs 800 each, find
1 the income of Mahavir.
Rs 1500 and Rs 1200 respectively. Prahlad gets 12 %
2 (a) Rs 1600 (b) Rs 1400
of the profit for managing the business. If the total (c) Rs 2000 (d) Rs 2400
profit gained in a year is Rs 2160. Find the share of
190. From 9 AM to 2 PM, the temperature rose at a con-
Prahlad. stant rate from 21°C to 36°C. What was the tempera-
(a) Rs 1050 (b) Rs 1080 ture at noon?
(c) Rs 1110 (d) Rs 1200 (a) 28.5 °C (b) 27 °C
182. Rs 5625 is to be divided among A, B and C so that A (c) 30 °C (d) 32 °C
1
may receive much as B and C together receive, 191. A sum of Rs 2000 is divided into two parts such that
21
and B receives of what A and C together receive. one part is invested at 10% per annum simple inter-
4 est and the second part at 15% per annum simple
The share of A is more than that of B by
(a) Rs 750 (b) Rs 775 interest. After 2 years, if Rs 460 is received as inter-
(c) Rs 1500 (d) Rs 1600 est, then the ratio of investment of first and second
part is
183. If a : b = 2 : 3, then the value of 2a + 3b : 4a + b is (a) 7 : 4 (b) 4 : 3
(a) 8 : 7 (b) 10 : 9 (c) 7 : 3 (d) 6 : 4
(c) 13 : 11 (d) 16 : 5
192. The difference between the simple interest received
184. A man divides his property so that his wife’s share from two different sources on Rs 1500 for 3 years is
to his daughter and his daughter’s share to his son Rs 13.50. The difference between their rates of inter-
are both in the ratio 3 : 2. If the son gets Rs 8000 less est is
than the daughter, find the worth of the total
@UPSC_THOUGHTS

(a) 0.1% (b) 0.2%


property? (c) 0.3% (d) 0.4%
(a) Rs 36000 (b) Rs 24000
(c) Rs 76000 (d) Rs 16000 193. Three years back, a sum of money was remitted in a
bank at 12% per annum S.I. The accounts are now
185. A bag contains fifty paise, twenty five paise, ten paise cleared, the bank paying a sum of Rs 6800. The sum
and five paise coins in the ratio 2 : 3 : 5: 7. If the total originally invested is
amount is Rs 52, the number of 50 paise coins is (a) Rs 20,000 (b) Rs 12,000
(a) 50 (b) 40 (c) Rs 7,500 (d) Rs 5,000
(c) 80 (d) 25
194. A sum of money put at simple interest trebles itself
186. A man has few hens and cows. If the number of heads in 15 years. The rate per cent per annum is
equals 48 and the number of feet equals 140, the (a) 13.33% (b) 10%
number of hens will be (c) 8.5% (d) 5%
(a) 26 (b) 30
(c) 33 (d) 40 195. An amount of Rs 1,00,000 is invested in two types of
shares. The first yields an interest of 9% per annum
187. The monthly salaries of two persons are in the ratio and the second 11% per annum. If the total interest
3 : 5. If each receives an increment of 3
Rs 20 in the salary, the ratio is altered to at the end of one year is 9 %, then the amount in-
4
13 : 21. Their initial respectively salaries are vested in each share was
(a) Rs 120 and 200 (b) Rs 120 and 225 (a) Rs 72,500, Rs 27,500
(c) Rs 240 and 400 (d) Rs 240 and 450 (b) Rs 62,500, Rs 37,500
188. 11 chairs cost as much as 3 tables. If a chair and a (c) Rs 52,500, Rs 47,500
table cost Rs 140, how much does a chair cost alone? (d) Rs 82,500, Rs 17,500
(a) Rs 110 (b) Rs 30 196. What is the rate of annual simple interest at which a
(c) Rs 70 (d) nothing sum of money would double itself in 25 years?
189. Ratio between annual incomes of Mahavir and (a) 4% (b) 5%
Sarosh is 5 : 4 and between their expenditure is 3 : 2. (c) 6% (d) 8%

6 . 149
Basic Numeracy and Data Interpretation

1 1 205. The compound interest on a certain sum of money at


197. A man invested of his capital at 6%; at 8% and
2 4 a certain rate for 2 years is Rs 40.80 and the simple
the remainder at 10%. If his annual income is Rs interest on the same sum for the same time is Rs 40 at
600, the capital is the same rate. The rate of interest is
(a) Rs 5,793.42 (b) Rs 6,545.45 (a) 2% per annum (b) 3% per annum
(c) Rs 7,993.20 (d) Rs 8,000 (c) 4% per annum (d) 5% per annum
198. A certain sum of money at S.I. amounts to Rs 1,012 206. The difference between compound interest and
1 simple interest for 2 years on a sum of money is Rs
in 2 years and to Rs 1,067.20 in 4 years. The rate of 160. If the simple interest for 2 years be Rs 2,880, the
2
interest per annum is rate per cent is
(a) 2.5% (b) 3% 5 1
(a) 5 % (b) 12 %
(c) 4% (d) 5% 9 2
1
199. A certain sum of money at S.I. amounts to (c) 11 % (d) 9%
9
Rs 1,260 in 2 years and to Rs 1,350 in 5 years. The
207. The compound interest on a certain amount for 2
rate per cent per annum is
years at the rate of 3% per annum is Rs 1218. The
(a) 2.5% (b) 3.75%
simple interest on the same amount for the same
(c) 5% (d) 7.5%
period is
200. The rates of simple interest in two banks A and B are (a) Rs 1,200 (b) Rs 1,205
in the ratio 5 : 4. A person wants to deposit his total (c) Rs 1,210 (d) Rs 1,215
savings in two banks in such a way that he receives 208. If the difference between the simple interest and com-
equal half yearly interests from both. He should de- pound interest on same principal amounts to 20%
@UPSC_THOUGHTS

posit the saving in banks A and B in the ratio for 3 years is Rs 48, then the principal amount is
(a) 5 : 2 (b) 2 : 5 (a) Rs 648 (b) Rs 600
(c) 4 : 5 (d) 5 : 4 (c) Rs 375 (d) Rs 350
201. A man borrows Rs 200 at 4% compound interest. If 209. The compound interest on a certain sum is Rs 104
he pays back Rs 58 at the end of each year, then how for 2 years and simple interest is Rs 100. The rate per
much does he owe at the end of 2 years? cent is
(a) Rs 156 (b) Rs 164 (a) 2% (b) 5%
(c) Rs 176 (d) Rs 184 (c) 7% (d) 8%
210. A sum borrowed under compound interest doubled
202. Rakesh invested Rs 15,000 at simple interest rate of
itself in 10 years. When will it become four-fold of
9 per cent per annum for two years and invested Rs
itself at the same rate of interest?
12000 at compound interest rate of 8 per cent per
(a) 15 years (b) 20 years
annum for two years. What is the total amount of (c) 24 years (d) 40 years
interest accrued by Rakesh in two years?
211. Find a single discount equivalent to a series of dis-
(a) Rs 4096.60 (b) Rs 4696.80
counts 20%, 10% and 25%
(c) Rs 4896.60 (d) Rs 4698.80
(a) 46% (b) 52%
203. The compound interest on a certain sum for 2 years (c) 57% (d) 60%
at 10% per annum is Rs 525. The simple interest on 212. Prakash bought a TV with 20% discount on the la-
the same sum for double the time at half the rate per belled price. Had he bought it with 25% discount, he
cent per annum is would have saved Rs 500. At what price did he buy
(a) Rs 400 (b) Rs 500 the TV?
(c) Rs 600 (d) Rs 800 (a) Rs 5000 (b) Rs 10,000
204. The cost of a property depreciates every year at the (c) Rs 12,000 (d) None of these
rate of 10% on its value at the beginning of that year. 213. A trader allows two successive discounts of 20%
If the present value of the property is Rs 65,61,000, and 10%. If he gets Rs 108 for an article. the marked
its worth 3 years ago was price is
(a) Rs 1 crore (b) Rs 90 lakh (a) Rs 250 (b) Rs 200
(c) Rs 82 lakh (d) Rs 75 lakh (c) Rs 150 (d) Rs 175

6 . 150
Basic Numeracy and Data Interpretation

214. If the true discount available on an amount due in 223. A merchant has 1000 kg of sugar, part of which he
1 3 sells at 4% profit and the rest, at 9% profit. He gains
2 years hence at the rate of 3 % is the same as the
3 4 7% on the whole. The quantity sold at 9% profit is
simple interest on a sum of Rs 2400 for the same (a) 600 kg (b) 500 kg
period and rate of interest, the amount due is Rs. (c) 400 kg (d) 300 kg
(a) 2610 (b) 6210
(c) 2160 (d) 3230 224. Pure milk costs Rs 3.60 per litre. A milkman adds
water to 25 litres of pure milk and sells the mixture
215. The true discount on Rs 4180 due after a certain time at Rs 3 per litre. How many litres of water does he
at 6% per annum is Rs 180. Find the time after which add?
it is due. (a) 4 litres (b) 5 litres
(a) 3 months (b) 4 months (c) 6 litres (d) 10 litres
(c) 8 months (d) 9 months
225. Two brass alloys are composed as follows: the first
216. The simple interest and true discount on a certain contains 3 parts tin, 20 parts copper, and 3 parts
amount for a given period at a given rate of interest zinc. The second alloy contains 7 parts tin, 56 parts
are Rs 672 and Rs 480 respectively. The sum due is copper, and 2 parts zinc. These alloys are fused to-
(a) Rs 1200 (b) Rs 1680 gether in equal quantities (by weight). Find the ratio
(c) Rs 1380 (d) Rs 1500 of three ingredients in the resulting alloy.
217. A bill is discounted at 4% per annum. If banker’s (a) 15 : 117 : 19 (b) 29 : 212 : 19
discount be allowed, what should be the rate per (c) 25 : 27 : 28 (d) 23 : 118 : 27
cent for the proceeds to be invested so that nothing 226. A group of 10 students, working one hour per day
may be lost? complete a piece of work in 12 days. If there are 12
1 2
@UPSC_THOUGHTS

(a) 4 % (b) 4 % students in the group and all of them work one hour
6 5 per day, then they will be able to complete the work
1
(c) 6 % (d) None of these in
4
1 4 (a) 8 days (b) 9 days
218. The banker’s discount on a sum due at 8 % is
3 3 (c) 10 days (d) 11 days
times the true discount. Then the unexpired time is 227. A can do a piece of work in 8 days and B can do the
(a) 4 years (b) 3 years same piece of work in 12 days. A and B together
(c) 2 years (d) 1 year complete the same piece of work and get Rs 200 as
219. The banker’s gain of a certain sum of money is Rs 40 the combined wage. B’s share of the wage was.
and the true discount on the same sum of the same (a) Rs 90 (b) Rs 85
time and at the same rate is Rs 30. Calculate the sum. (c) Rs 80 (d) Rs 75
(a) Rs 120 (b) Rs 140 228. A can do a work in 6 days, while B and C can do the
(c) Rs 160 (d) Rs 180 same work in 12 and 18 days respectively. If they
220. The present worth and the bankers gain on a bill is start together to do the work on the same day and C
Rs 10,000 and Rs 25 respectively. Find the true stops working after 3 days, then the time required by
discount of the bill. A and B together to finish the rest of the work will be
(a) Rs 700 (b) Rs 600 (a) 1 day and 12 hours
(c) Rs 500 (d) Rs 400 (b) 16 hours
221. Milk and water are in the ratio of 3 : 2 in a mixture (c) 8 hours
of 80 litres. How much water should be added so (d) 1 day and 6 hours
that the ratio of milk and water becomes 2 : 3? 229. A and B did a piece of work together and received Rs
(a) 40 litres (b) 37 litres
300. If A alone can do piece of work in 2 weeks and B
(c) 32 litres (d) 30 litres
alone in 3 weeks, how should the money be divided
222. In what ratio must a grocer mix two varieties of between them?
pulses costing Rs 15 and Rs 20 per kg respectively (a) A = Rs 180 and B = Rs 120
so as to get a mixture worth Rs 16.50 per kg? (b) A = Rs 120 and B = Rs 180
(a) 7 : 3 (b) 9 : 1 (c) A = Rs 150 and B = Rs 150
(c) 12 : 5 (d) 15 : 4 (d) A = Rs 200 and B = Rs 100

6 . 151
Basic Numeracy and Data Interpretation

230. A piece of work can be done by 6 men and 5 women 238. A does half as much work as B in three-fourth of the
in 6 days or 3 men and 4 women in 10 days. How time. If together they take 18 days to complete the
many days will it take by 9 men and 15 women to do work, how much time shall B take to do it?
the same work? (a) 21 days (b) 30 days
(a) 1 day (b) 2 days (c) 34 days (d) 38 days
(c) 3 days (d) 4 days 239. 6 men and 8 women can complete a work in 9 days,
231. A can do a work in 7 days and B in 8 days. A and B while 5 men and 9 women can complete it in 10
undertake to do this work for Rs 56. If with the assis- days. In how many days will 15 women complete it?
tance of a boy they finish the work in 3 days then the (a) 40 days (b) 21 days
boy will get (c) 38 days (d) 15 days
(a) Rs 5 (b) Rs 11 240. A can do a certain job in 12 days. B is 60% more
(c) Rs 19.50 (d) Rs 25 efficient than A. How many days does B alone take
232. A takes as much time as B and C together to finish a to do the same job?
job. A and B working together finish the job in 10 1 1
(a) 4 days (b) 5 days
days. C alone can do the same job in 15 days. In how 2 2
many days can B alone do the same work? 1 1
(c) 7 days (d) 9 days
(a) 30 days (b) 60 days 2 2
(c) 90 days (d) 120 days 241. A can do a piece of work in 10 days, B in 15 days.
They work for 5 days. The rest of the work was fin-
233. 3 men or 5 women or 8 boys can do a work in 30 ished by C in 2 days. If they get Rs 1500 for the whole
days. Then in how many days will 1 man, 1 woman work, the daily wages of B and C are
and 1 boy do the work?
@UPSC_THOUGHTS

(a) Rs 225 (b) Rs 300


(a) 45.5 days (b) 40 days (c) Rs 354 (d) Rs 397
(c) 38 days (d) 35.5 days 242. A, B and C together can finish a work in 10 days. All
234. Rajat can do a piece of work in 8 days and Raman the three started working at it together and after 4
can do the same work in 12 days. How long will days A left. Then B and C together completed the
they take, if they work together? work in 10 more days. A alone could complete the
4 work in
(a) 4 days (b) 3 days
5 (a) 23 days (b) 24 days
7 5 (c) 25 days (d) 26 days
(c) 2 days (d) 2 days
8 7 243. A certain industrial loom weaves 0.128 metres of
235. 45 women get Rs 15525 for 48 days of work. How cloth every second. Approximately, how many sec-
many men must work 16 days to receive Rs 5750, the onds will it take for the loom to weave 25 metres of
daily wages of a man being double those of a woman? cloth?
(a) 20 men (b) 25 men (a) 200 seconds (b) 195 seconds
(c) 190 seconds (d) 185 seconds
(c) 30 men (d) 35 men
244. 30 men can produce 1500 units in 24 days working
236. A, B and C can do a work in 10 days. The three to-
6 hours a day. In how many days, can 18 men pro-
gether work for 4 days. Then A left the work. B and C
duce 1800 units working 8 hours a day?
finished the remaining work in 10 days. In how many (a) 30 days (b) 34 days
days will A alone do the work? (c) 36 days (d) 38 days
(a) 10 days (b) 14 days
245. Two candles of the same height are lighted at the
(c) 18 days (d) 25 days
same time. The first is consumed in 4 hours and the
1 second in 3 hours. Assuming that each candle burns
237. A and B can do a job in 10 days. A is 1 times as
2 at its own constant rate, find the number of hours,
efficient as B. The same job can be done by A alone in after being lighted, after which the first candle will
(a) 10 days (b) 14 days be twice the height of the second candle.
1 2 (a) 1.5 hours (b) 1.9 hours
(c) 15 days (d) 16 days (c) 2.4 hours (d) 2.7 hours
4 3

6 . 152
Basic Numeracy and Data Interpretation

246. A certain number of artisans can complete a shoe 253. A man riding a cycle at 12 kmph can reach a village
fabrication consignment in 16 days. 8 additional 1 1
in 4 hours. If he is delayed by 1 hours at the start
artisans had to be deployed for the same consign- 2 2
then in order to reach his destination in time, he
ment and together they completed it in 4 days less
should ride with a speed of
than the earlier estimate. The number of artisans ini- (a) 15 kmph (b) 16 kmph
tially employed was (c) 18 kmph (d) 20 kmph
(a) 24 (b) 20
254. Two cyclists A and B start from the same place at the
(c) 18 (d) None of these
same time, one going towards north at 18 kmph and
247. 5 men can do a piece of work in 12 days and 10 the other towards south at 20 kmph. What time will
women can do it in 10 days. In how many days can they take to be 95 km apart?
3 men and 5 women do the same piece of work? 1 3
(a) 4 hours (b) 4 hours
(a) 12 (b) 6 2 4
(c) 10 (d) 8 4 1
(c) 5 hours (d) 2 hours
15 2
248. A and B can weave a carpet in 10 days and 15 days
255. A train leaves Meerut at 6 a.m. and reaches Delhi at
respectively. They begin to work together but B leaves
10 a.m. Another train leaves Delhi at 8 a.m. and
after 2 days. In what time will A complete the re- reaches Meerut at 11.30 a.m. At what time do the two
maining work? trains cross one another?
1 2 9 12
(a) 6 days (b) 6 days
3 3 (a) 5 hours (b) 4 hours
17 13
@UPSC_THOUGHTS

(c) 7 days (d) 8 days 19 14


(c) 3 hours (d) 2 hours
249. A garrison of 2000 men had enough food to last for 20 15
30 days. After 10 days 500 more men joined. How 256. A thief steals a car at 2.30 pm and drives it at 60
long did the food last then? kmph. The theft is discovered at 3 pm and the owner
(a) 20 days (b) 18 days sets off in another car at 75 kmph. When will the
(c) 17 days (d) 16 days owner overtake the thief?
250. To finish a certain job X takes twice as long as Y and (a) 7 pm (b) 6 pm
Z together and Z three times as long as X and Y (c) 5 pm (d) 4 pm
together. If X, Y and Z working together complete the 257. Two men start together to walk to a certain destina-
job in 6 days, how long would X take to complete the tion, one at 3 kmph and another at 3.75 kmph. The
work alone? latter arrives half an hour before the former. The dis-
(a) 16 days (b) 18 days tance is
(c) 22 days (d) 26 days (a) 7.5 km (b) 8 km
251. In a 500 metres race, Q starts 45 metres ahead of P. (c) 9.5 km (d) 10 km
But P wins the race while Q is still 35 metres behind. 258. The ratio between the speeds of two trains is 7 : 8. If
The ratio of their speeds, assuming that both start at the second train runs 400 kms in 4 hours, then the
the same time, is speed of the first train is
(a) 5 : 7 (b) 5 : 3 (a) 87.5 kmph (b) 84 kmph
(c) 5 : 6 (d) 25 : 21 (c) 75 kmph (d) 70 kmph
252. A truck completes a journey in 20 hours, the first 259. A scooterist completes a journey in 10 hours, the
half at 80 kmph, and the rest at 84 kmph. The dis- first half at the rate of 21 kmph and the second half
tance covered by the truck is at the rate of 24 kmph. The total distance travelled is
(a) 1639 km (b) 1639.02 km (a) 192 km (b) 204 km
(c) 1639.12 km (d) 1639.32 km (c) 224 km (d) 256 km

6 . 153
Basic Numeracy and Data Interpretation

260. A monkey climbs a slippery pole 10 m high in such 267. A train 100 m long meets a man going in opposite
a way that in every one minute it rises 50 cm and in direction at the rate of 5 km per hour and passes him
every next minute it slips 25 cm. How soon will he in 7.2 seconds. At what rate is the train going?
reach the top? (a) 50 kmph (b) 45 kmph
(a) 43 minutes (b) 51 minutes (c) 42 kmph (d) 40 kmph
(c) 68 minutes (d) 77 minutes 268. A passenger train running at the speed of 80 km/hr
261. A man standing on a platform notes that a train go- leaves the railway station 6 hours after a goods train
ing in one direction takes 3 seconds to pass him; a leaves, and overtakes it in 4 hours. What is the speed
train of the same length going in opposite direction of the goods train?
takes 4 seconds. How long did they take to pass (a) 32 kmph (b) 40 kmph
each other? (c) 45 kmph (d) 50 kmph
3 4 269. A person standing on a railway bridge which is 50
(a) 3 seconds (b) 3 seconds
7 7 1
m long finds that a train crosses the bridge in 4
2
1 1
(c) 3 seconds (d) 3 seconds seconds, but himself in 2 seconds. The length of the
3 8
train is
262. Two trains of equal length are running on parallel (a) 50 m (b) 100 m
lines in the same direction at 46 and 36 kmph. The (c) 200 m (d) 40 m
faster train passes the slower train in 36 sec. The
length of each train is 270. Kerala Express train consists of 15 bogies, each 15
(a) 47 m (b) 50 m metres long. The train crosses a telegraph post in 18
(c) 53 m (d) 56 m seconds. Due to some problems, 3 boggies were de-
@UPSC_THOUGHTS

263. Two guns were fired from the same place at an tached. Find the time taken by the train to cross the
interval of 10 minutes and 30 seconds, but a person telegraph post now.
in a train approaching the place hears the second (a) 14.4 seconds (b) 13 seconds
shot 10 minutes after the first. The speed of the train, (c) 12.8 seconds (d) 11 seconds
supposing that sound travels at 330 metres per
271. The length of the bridge which a train, 130 metres
second is
long and travelling at 45 km per hour, can cross in
(a) 59.4 kmph (b) 57 kmph
(c) 55.03 kmph (d) 51.2 kmph 30 seconds, is
(a) 200 m (b) 225 m
264. A motorist can complete a journey in 6 hours, travel-
(c) 245 m (d) 250 m
ling at a speed of 24 km/hr. If he is delayed by two
hours at the start of the journey, then to make up for 272. A train overtakes two persons who are walking in
the lost time, he should drive at a speed of the same direction at the rate of 2 and 4 kmph and
(a) 36 km/hr. (b) 20 km/hr. passes them in 9 and 10 seconds respectively. The
(c) 16 km/hr. (d) 12 km/hr. length of the train is
265. Two cars start at the same time from A and B which (a) 35 m (b) 40 m
are 120 km apart. If the two cars travel in opposite (c) 45 m (d) 50 m
direction they meet after one hour and if they travel
273. A is twice as fast as B and B is thrice as fast as C.
in the same direction (from A towards B) they meet
after 6 hours. What is the speed of the car starting What time will A take to cover the same distance
from A? that C covered in 42 minutes?
(a) 70 km/hr (b) 60 km/hr (a) 5 minutes (b) 6 minutes
(c) Data inadequate (d) None of these (c) 7 minutes (d) 8 minutes
266. The speeds of three cars are in the ratio of 274. A man walking at the rate of 5 km/hr. crosses a square
2 : 3 : 4. The ratio between the time taken by these field diagonally in 5 minutes. Find the area of the
cars to travel the same distance is field?
(a) 2 : 3 : 4 (b) 3 : 2 : 4 (a) 831.28 acres (b) 854.43 acres
(c) 6 : 4 : 3 (d) 4 : 3 : 2 (c) 860.9 acres (d) 868.05 acres

6 . 154
Basic Numeracy and Data Interpretation

275. Neera leaves home at 11 a.m. and rides to Asha’s 283. A swimmer swims from a point A against a current
house to return her bicycle. She travels at 12 km per for 5 minutes and then swims in favour of the cur-
hour and arrives at 11.30 a.m. She turns right around rent for the next 5 minutes and comes to the point B.
and walks home. How fast does she walk if she re- If AB = 100m, find the speed of the current.
turns home at 1 p.m.? (a) 0.6 km/hr (b) 1 km/hr
(a) 5 kmph (b) 4 kmph (c) 2 km/hr (d) None of these
(c) 7 kmph (d) 6 kmph 284. A man’s speed of rowing in still water is 5 km/hr. If
276. Two ships are 1550 km apart sailing towards each the speed of the stream is 1 km/hr, it takes him 75
other. One sails at the rate of 85 km per day and the minutes to row to a place and back. How far is the
other at 65 km per day. How far apart will they be at place?
the end of 9 days? (a) 3 km (b) 4 km
(c) 3.5 km (d) 4.5 km
(a) 150 km (b) 200 km
(c) 220 km (d) 785 km 285. A can run a kilometre in 4 minute 50 seconds and B
in 5 minutes. By what distance can A beat B?
277. A man can row 20 kmph in still water. It takes him
thrice as long to row up as to row down the river. 1 2
(a) 33 m (b) 966 m
Find the rate of stream. 3 3
2
(a) 10 kmph (b) 12 kmph (c) 66 m (d) 100 m
3
(c) 30 kmph (d) 100 kmph 1
286. A runs 2 times as fast as B. If A gives B a start of 60
278. If a boat goes 10 km upstream in 50 minutes and 3
metres, how far must be the destination point if the
speed of the stream is 2 kmph then the speed of boat race ends in a dead lock?
in still water is (a) 100 m (b) 200 m
@UPSC_THOUGHTS

(a) 5 kmph (b) 14 kmph (c) 105 m (d) 150 m


(c) 26 kmph (d) 30 kmph
287. A can run 100 metres in 22.5 seconds and B can run
279. A man rows 6 kmph in still water. When the river is in 25 seconds. A will beat B by......
running at 1.2 kmph, it takes him 1 hour to row to a (a) 9 metres (b) 8 metres
place and back. How far is the place? (c) 10 metres (d) 5 metres
(a) 1 km (b) 1.5 km 288. In a 100 metres race A can beat B by 25 metres and B
(c) 2 km (d) 2.88 km can beat C by 4 metres. In the same race, A can beat C
280. A man swims downstream 20 km and upstream 8 by
km, taking 4 hours each time. Then the velocity of (a) 21 m (b) 26 m
stream is (c) 28 m (d) 29 m
(a) 1.05 kmph (b) 1.25 kmph 289. A and B ran a race which lasted a minute and a half.
(c) 1.5 kmph (d) 1.75 kmph A gave B a start of 9 metres and beat him by 1 metre.
281. A boat, while going downstream in a river covered a A ran 40 metres while B ran 39 metres. Find the length
distance of 50 miles at an average speed of 60 miles of the race.
per hour. While returning, because of the water re- (a) 200 m (b) 100 m
(c) 400 m (d) 800 m
sistance, it took 1 hour 15 minutes to cover the same
3
distance. What was the average speed during the 290. S is 1 times as fast as J. If S gives J a start of 150
8
whole journey? metres, how far must the winning post be placed so
(a) 40 mph (b) 48 mph that the race ends in a dead heat?
(c) 50 mph (d) 55 mph (a) 100 m (b) 200 m
(c) 440 m (d) 550 m
282. A boat goes 20 km upstream in 2 hours and down-
stream in 1 hour. Find how much time this boat will 291. A can give B and C a start of 20 metres and 25 metres
take to travel 30 km in still water. respectively in a 100 metres race; B can give C one
1 second over the course. How long does A take to run
(a) 1 hr (b) 1 hr 100 metres?
2
1 (a) 10 sec (b) 12 sec
(c) 2 hrs (d) 2 hrs (c) 15 sec (d) 16 sec
2

6 . 155
Basic Numeracy and Data Interpretation

292. A racecourse is 400 m long. A and B run a race and A 300. Three pipes A, B and C can fill a tank in 6 hours.
wins by 5 m. B and C run over the same course and After working at it together for 2 hours, C is closed
B wins by 4 m. C and D run over it and D wins by 16 and A and B fill the remaining part in 7 hours. The
m. If A and D run over it, then who would win and number of hours taken by C alone to fill the tank is
by how much? (a) 14 hours (b) 13 hours
(a) A, by 8.4 m (b) D, by 8.4 m
(c) 12 hours (d) 11 hours
(c) A, by 7.2 m (d) D, by 7.2 m
301. Pipes A and B running together can fill a cistern in 6
293. At a game of billiards, A can give B 5 points in 30 minutes. If B takes 5 minutes more than A to fill the
and he can give C 7 points in 30. How many can B
cistern, then the time in which A and B will fill the
give C in a game of 40?
cistern separately will be respectively.
(a) 4 (b) 3
(c) 2 (d) 1 (a) 15 minutes, 20 minutes
(b) 25 minutes, 20 minutes
294. Raj, Aryan and Rohit walks around a circle 1760
(c) 15 minutes, 10 minutes
metres in circumference at the rate of
160 m, 120 m and 105 m per minute. If they all start (d) 10 minutes, 15 minutes
together and walk in the same direction, when will 302. Two pipes A and B fill a cistern in 5 minutes and 6
they be first together again? minutes, respectively. If these pipes are turned on
(a) 3 hour 38 min (b) 4 hour 30 min alternately for 1 minute each, how long will it take
(c) 5 hour 52 min (d) 6 hour 21 min
for the cistern to fill?
295. Raman and Aditya run a 10 km race on a course 300
1 5
metres round. If their rates by 5 : 4, how often does (a) 5 minutes (b) 5 minutes
3 6
the winner pass the other?
@UPSC_THOUGHTS

(a) 11 times (b) 9 times 1 5


(c) 6 minutes (d) 6 minutes
(c) 8 times (d) 5 times 3 6
296. Pipes A and B can fill a tank in 3 hours and 6 hours 303. A cistern is normally filled in 10 hours but takes one
respectively. Both the pipes together can fill the tank hour longer to fill because of a leak in its bottom. If
in the cistern is full, the leak will empty it in
(a) 9 hours (b) 3 hours (a) 80 hours (b) 90 hours
(c) 2 hours (d) 1 hour (c) 100 hours (d) 110 hours
297. A tank is filled by pipe A in 12 hours and pipe B in 304. A tap can fill a tank in 16 minutes and another can
15 hours. When full, it can be emptied by pipe C in 9 1
hours. If all the three pipes are opened simulta- empty it in 8 minutes. If the tank is already full
2
neously, half of the tank will get filled in and both the taps are opened together, will the tank
5 be filled or emptied? How long will it take before the
(a) 18 hours (b) 25 hours
7 tank is either filled or emptied completely as the case
2
(c) 30 hours (d) 32 hours may be?
5
(a) Filled, 8 minutes
298. Two pipes A and B fill a tank in 24 minutes and 32
(b) Filled, 12 minutes
minutes respectively. If both the pipes are opened
(c) Emptied, 8 minutes
simultaneously, after how much time should pipe B
(d) Emptied, 12 minutes
be closed so that the tank is full in 18 minutes?
(a) 8 minutes (b) 20 minutes 305. Two pipes A and B can fill a tank in 7 hours and 4
(c) 35 minutes (d) 40 minutes hours respectively. If they are opened in alternate
hours and if pipe A is opened first, in how many
299. One pipe can fill a tank three times as fast as another
hours, will the tank be full?
pipe. If together the two pipes can fill the tank in 36
minutes, then the slower pipe alone will be able to 2
(a) 5 hours (b) 5 hours
fill the tank in 7
(a) 81 minutes (b) 108 minutes 1
(c) 4 hours (d) 4 hours
(c) 144 minutes (d) 192 minutes 2

6 . 156
Basic Numeracy and Data Interpretation

306. Taps A, B and C are connected to a water tank and the business for 2 months only. At the end of the
the rate of flow of water is 42 litres/hr, 56 litres/hr year, the business earns a profit of Rs 9,100. The
and 48 litres/hr respectively. Taps A and B fill the share of C in the profit is
tank while tap C empties the tank. If all the three (a) Rs 1,400 (b) Rs 2,000
taps are opened simultaneously, the tank gets com- (c) Rs 2,200 (d) Rs 2,500
pletely filled up in 16 hours. What is the capacity of
the tank? 313. A and B enter into a partnership with their capitals
(a) 960 litres (b) 2346 litres in the ratio 7 : 9. At the end of 8 months, A with-
(c) 1600 litres (d) 800 litres draws his capital. If they receive the profits in the
ratio 8 : 9, find for how long B’s capital was used.
307. An electric pump can fill a tank in 3 hours. Because (a) 4 months (b) 7 months
1 (c) 9 months (d) 1 year
of a leakage in the tank it took 3 hours to fill the
2
tank. The leakage can drain out all the water of the 314. A and B entered into a partnership. A invested Rs
tank in 8000. If the shares of profits of A and B be Rs 360 and
1 Rs 450, the investment of B is
(a) 10 hours (b) 12 hours
2 (a) Rs 4,000 (b) Rs 5,000
(c) 21 hours (d) 24 hours (c) Rs 8,000 (d) Rs 10,000
308. A leak in the bottom of the tank can empty the full 315. A started a business with Rs 76,000. After few
tank in 12 hours. An inlet pipe fills water at the rate months, B joined him with Rs 57,000. At the end of
of 6 litres a minute and the tank is full when the inlet the year, the total profit was divided between them
is opened for 8 hours. How many litres does the
@UPSC_THOUGHTS

in the ratio 2 : 1. After how many months did B join?


cistern hold? (a) 4 months (b) 6 months
(a) 7580 (b) 7960 (c) 8 months (d) 11 months
(c) 8210 (d) 8640
316. Rs 56,250 is to be divided among A, B and C so that
309. A pipe can fill a bath in 20 minutes, and another can
A may receive half as much as B and C together and
fill it in 30 minutes. A person opens both the pipes
B receives one-fourth of what A and C together re-
simultaneously; when the bath should have been
ceive. The share of A is more than that of B by
full, he finds that the waste pipe was open. He then
(a) Rs 7,500 (b) Rs 7,750
closes the waste pipe and in 3 minutes the bath is
(c) Rs 15,000 (d) Rs 16,000
full. In what time will the waste pipe empty it?
(a) 20 minutes (b) 48 minutes 317. Two merchants A and B are in partnership A puts in
(c) 30 minutes (d) 45 minutes Rs 20,000 and then after eight months he adds an-
other Rs 10,000 to the capital. B withdraws Rs 4,000
310. A cistern when full is emptied by a waste pipe in 30
at the end of 4 months. At the end of the year, A and
minutes; but if a tap is opened it takes 40 minutes to
empty the cistern. How long will the cistern take to B’s ratio of profit is equal. Find B’s capital in the
become full if both the pipes are working? beginning.
(a) 60 minutes (b) 120 minutes (a) Rs 26,000 (b) Rs 30,000
(c) 140 minutes (d) None of these (c) Rs 35,000 (d) Rs 40,000

311. Three partners A, B and C invest Rs 5,000, 318. Three partners A, B and C invested Rs 8,000, Rs 5,000
Rs 7,000 and Rs 10,000 respectively in a business. If and Rs 7,000 respectively. They decide to distribute
at the end of the year they earned Rs 17,600 as profit, 60% of the profit equally and the remaining 40% as
the share of partner C in this profit amounts to the interest on their respective capitals. If A received
(a) Rs 5,000 (b) Rs 6,000 Rs 840 less than the sum of the other two find the
(c) Rs 7,000 (d) Rs 8,000 total profit.
312. A starts a business with Rs 5,000. B joins him after 3 (a) Rs 2,500 (b) Rs 3,000
months with Rs 8,000. C puts a sum of Rs 12,000 in (c) Rs 4,800 (d) Rs 9,000

6 . 157
Basic Numeracy and Data Interpretation

319. A sum of money is to be divided among A, B and C in 327. Vineet invests Rs 5,400 partly in 7% stock at 147 and
the ratio 2 : 3 : 7. If the total share of A and B together partly in 6% stock at 144. If his income is the same,
is Rs 1,500 less than C. What is A’s share in it? find his two investments.
(a) Rs 1,500 (b) Rs 1,700 (a) Rs 2,500 and 2,900
(c) Rs 2,000 (d) Rs 2,200 (b) Rs 2,600 and Rs 2,800
320. A, B and C enter into a partnership and their shares (c) Rs 2,520 and Rs 2,880
1 1 1 (d) Rs 2,580 and Rs 2,820
are in the ratio : : . After 2 months, A with-
2 3 4 328. A man invests Rs 12,100 partly in 7% stock at 80 and
draws half of his capital and after 10 months, a profit
partly in 9% stock at 117. If his total income is Rs
of Rs 378 is divided among them. What is B’s share? 990, how much did he invest in each kind of stock?
(a) Rs 129 (b) Rs 144 (a) Rs 5,500 and Rs 16,600
(c) Rs 156 (d) Rs 168 (b) Rs 5,000 and Rs 7,100
321. Find the cost of Rs 1,000, 10% stock at par. (c) Rs 5,600 and Rs 6,500
(a) Rs 1,000 (b) Rs 1,100 (d) Rs 6,000 and Rs 6100
(c) Rs 990 (d) Rs 1,010 329. Arvind has 1,000 Rs 8 shares in a company, out of
322. Find the cost of Rs 2,500, 9.5% stock at 5 discount, which he sells 300 at Rs 10 each. He reinvests the
1 proceeds in a stock at 75. Find Arvind’s total income
brokerage % .
2 in one year from the shares and the stock if a divi-
(a) Rs 2,387.50 (b) Rs 2,388 dend of 8% per annum is paid on the shares and
(c) Rs 2,375 (d) Rs 2,625.00 11% is paid on the stock.
@UPSC_THOUGHTS

(a) Rs 888 (b) Rs 448


1
323. Which is a better investment: 5 % stock at 102 or (c) Rs 440 (d) None of these
3 2
4 % stock at 98 330. Jessy invested Rs 3,000 in 13% stock at 75 and sold it
4
a certain market value. She invested the proceeds in
(a) 4 3 % stock at 98 15% ‘‘Mavelikara rice’’ at 90 and increased her in-
4
come by Rs 56.00. What was the market value when
1
(b) 5 % stock at 102 she sold the first stock?
2
(a) Rs 86 (b) Rs 84
(c) Both are equally good
(c) Rs 86.60 (d) Rs 86.40
(d) Neither of them is good
331. The market value of Rs 10 share of a company is Rs
324. A man invested Rs 8,800 when he bought Rs 100
16. Mahavir buys 250 shares of the company. What
shares at Rs 110. When a 12% dividend is declared,
will be his annual income from these shares if the
his annual income is 1
company declares 6 % dividend.
(a) Rs 900 (b) Rs 960 4
(a) Rs 200 (b) Rs 225
(c) Rs 920 (d) None of these (c) Rs 250 (d) Rs 275
325. A man sold 300 shares with face value Rs 100 and 332. What is the sum due whose present worth at 6% per
the market value Rs 680. If he had purchased the annum in 3 years hence is Rs 1,000?
shares at 65% premium, what was his gain? (a) Rs 1,150 (b) Rs 1,160
(a) Rs 1,50,000 (b) Rs 1,54,000 (c) Rs 1,170 (d) Rs 1,180
(c) Rs 1,54,500 (d) Rs 1,60,000 333. How much 4% stock at 95 can be purchased by in-
1
326. A man bought 250 shares with face value Rs 100 vesting Rs 2667 (brokerage % )?
4
and the market value Rs 150. If he sold the shares at (a) Rs 3,400 (b) Rs 3,200
150% premium, what was his gain? (c) Rs 3,000 (d) Rs 2,800
(a) Rs 15,000 (b) Rs 20,000 334. An amount of Rs 1,00,000 is invested in two types of
(c) Rs 25,800 (d) Rs 25,000 shares. The first yields an interest of 9% per annum

6 . 158
Basic Numeracy and Data Interpretation

and the second 11% per annum. If the total interest 342. At what time between 5.30 and 6 in the morning will
3
at the end of one year is 9 % , then the amount in- both the hands of a clock be at right angles?
4
vested in each share was 7
(a) 43 minutes past 5
(a) Rs 72,500, Rs 27,500 11
(b) Rs 62,500, Rs 37,500 6
(b) 40 minutes past 5
(c) Rs 52,500, Rs 47,500 11
(d) Rs 82,500, Rs 17,500 5
(c) 39 minutes past 5
2 13
335. Rahul purchased stock for Rs 1500 and sold of it 3
3 (d) 34 minutes past 5
after its value doubled. He sold the remaining stock 16
at 5 times its purchase price. What was his total Directions for questions 343 to 345: Refer to the data be-
profit on the stock? low and answer the questions that follow:
(a) Rs 1,500 (b) Rs 2,000
Manoj uses a toothpaste tube of 400g in a month.
(c) Rs 2,500 (d) Rs 3,000
He brushes twice a day except on Sundays when he
336. A clock strikes once at 1 o’clock, twice at brushes only once a day.
2 o’clock thrice at 3 o’clock and so on. The total num-
343. If the first of January is Monday, then what is his
ber of strikes in a day is
average usage in gms/brushing in January?
(a) 24 (b) 98
(a) 6.72 (b) 6.89
(c) 124 (d) 156 (c) 7.01 (d) 7.19
337. How many times are the hand of a clock at right 344. If the first of January is a Sunday, then what is his
angle in a day? average usage in gms/brushing in January?
@UPSC_THOUGHTS

(a) 22 (b) 24 (a) 6.72 (b) 6.89


(c) 44 (d) 48 (c) 7.01 (d) 7.19
338. How many times do the hands of a clock coincide in 345. What should be the first of January, if the average
a day? usage in gms/brushing should be minimum?
(a) 20 (b) 22 (a) Monday (b) Tuesday
(c) 21 (d) 24 (c) Wednesday (d) Any of these
339. What day of the week will be 95 days from today, if 346. If the fourth Saturday of a month is the 22nd day.
today is Monday? What day is the 13th of that month?
(a) Monday (b) Wednesday (a) Tuesday (b) Wednesday
(c) Thursday (d) Friday (c) Thursday (d) Friday
340. How many days are there from April 18 to June 6, 347. At what time between 11 and 12 will the hands of a
2002? clock coincide?
(a) 45 days (b) 50 days 5
(a) 5 min. past 11
(c) 49 days (d) 51 days 11
(b) At 12 o’clock
341. A man goes out between 5 p.m. and 6 p.m. When he (c) Never happens
comes back between 6 p.m. and 7 p.m., he observes (d) None of these
that the two hands of a clock have interchanged their
348. In between 9 a.m. and 10 am, when will both hands
positions. Find when the man went out.
be in opposite direction?
1
(a) 31 minutes past 4 4
11 (a) 16 minutes past 9
11
4 4
(b) 32 minutes past 5 (b) 22 minutes past 9
13 11
4
(c) 43 minutes past 5 (c) 26 minutes past 9
11
3
(d) 24 minutes past 6 (d) None of these
16

6 . 159
Basic Numeracy and Data Interpretation

349. If the two hands of a watch coincide every 66 min- 356. The ratio of the present age of two brothers is 1 : 2
utes, then the watch and the ratio 5 years back was 1 : 3. What will be the
(a) gains time ratio of their ages after 5 years?
(b) loses time (a) 1 : 4 (b) 2 : 3
(c) shows correct time (c) 3 : 5 (d) 5 : 6
(d) alternatively loses and gains time 357. Shaurya’s mother was four times as old as Shaurya
350. Two clocks are set correct at 10.00 a.m. on Friday. ten years ago. After 10 years, she will be twice as old
The first clock gains 2 minutes every hour and gains as Shaurya. How old is the mother today?
(a) 32 years (b) 50 years
twice as much as the second clock. What time will
(c) 43 years (d) 71 years
the second clock register when the correct time is
2.00 p.m. on the following Monday? 358. The sum of the present ages of a father and his son is
(a) 3.15 pm (b) 3.16 pm 60 years. Six years ago, the father’s age was five
(c) 4.32 pm (d) 3.18 pm times the age of the son. After 6 years, son’s age will
be
351. The ratio between the present ages of P and Q is 6 : 7. (a) 20 years (b) 18 years
If Q is 4 years older than P, what will be the ratio of (c) 14 years (d) 12 years
their ages after 4 years?
359. Present ages of X and Y are in the ratio 5 : 6 respec-
(a) 7 : 8 (b) 6 : 7
tively. Seven years hence this ratio will be 6 : 7 re-
(c) 5 : 6 (d) 4 : 5
spectively. What is X’s present age in years?
352. The sum of the ages of 5 children each born at 3 (a) 22 (b) 28
years interval is 50 years. What is the age of the (c) 35 (d) 43
@UPSC_THOUGHTS

youngest child? 360. Z’s age is twice the average age of X, Y and Z. X’s
(a) 10 years (b) 8 years age is one half of the average of X, Y and Z. If Y is 5
(c) 4 years (d) None of these years old, what is the average age of X, Y and Z?
353. A is two years older than B who is twice as old as C. (a) 12 years (b) 11 years
If the total of the ages of A, B and C is 27, then how (c) 10 years (d) 9 years
old is B? 361. There are 5 letters and 5 directed envelopes. The
(a) 7 (b) 8 number of ways in which all the letters can be put in
(c) 9 (d) 10 wrong envelopes is
(a) 119 (b) 44
354. A person was asked to state his age in years. His
(c) 59 (d) 40
reply was, “Take my age three years hence, multiply
it by 3 and then subtract three times my age three 362. If the letters of the word WOMAN are written in all
years ago and you will know how old I am.” What possible orders and these words are written out as
in a dictionary, then the rank of the word ‘WOMAN’
was the age of the person?
is
(a) 12 years (b) 18 years
(a) 117 (b) 120
(c) 16 years (d) 14 years
(c) 119 (d) 118
355. A demographic survey of 100 families in which two
363. If a polygon has 44 diagonals, then the number of its
parents were present revealed that the average age
1 sides are
A, of the oldest child, is 20 years less than the sum (a) 11 (b) 7
2
of the ages of one parent F and the age of the other (c) 8 (d) None of these
parent, M. Which of the following is equivalent to 364. Out of 18 points on a plane, no three are in the same
A? straight line except five points which are collinear.
FM FM The number of straight lines that can be formed join-
(a)  20 (b)  20
2 2 ing them is
(a) 143 (b) 144
F  M  20 F  M  20
(c) (d) (c) 153 (d) None of these
2 2

6 . 160
Basic Numeracy and Data Interpretation

365. Everybody in a room shakes hands with everybody 373. Three boys and three girls are to be seated around a
else. The total number of handshakes is 66. The total table in a circle. Among them the boy X does not
number of persons in the room is want any girl neighbour and the girl Y does not want
(a) 11 (b) 12 any boy neighbour. How many such arrangements
(c) 8 (d) 14 are possible?
(a) 5 (b) 6
366. The number of ways in which 5 prizes can be
(c) 4 (d) 2
distributed among 4 boys, while each boy is capable
of having any number of prizes is 374. A box contain 2 white balls, 3 black balls and 4 red
balls. In how many ways can 3 balls be drawn from
(a) 64 (b) 45
4 the box, if at least one black ball is to be included in
(c) 4! 2 (d) 6 . (4!)
the draw?
367. The number of diagonals for n-sided polygon is (a) 64 (b) 69
n (n 1) n (n 1) (n 2) (c) 75 (d) 80
(a) (b)
2 6 375. In a group of 6 boys and 4 girls, four children are to
n (n 3) be selected. In how many different ways can they be
(c) n (n 1) (d)
2 selected such that at least one boy should be there?
368. How many words can be made out of the letters of (a) 109 (b) 190
the word ASSASSINATION taken all together? (c) 209 (d) 290
(a) 1396421 (b) 37289300 376. In how many different ways can the letters of the
(c) 9112143 (d) 10810800 word ‘SOFTWARE’ be arranged in such a way that
the vowels always come together?
@UPSC_THOUGHTS

369. In how many ways can 5 persons be seated at a


(a) 4320 (b) 5430
round table, so that all shall not have the same
(c) 6540 (d) 7650
neighbours in any two arrangements?
(a) 10 (b) 12 377. There are 12 buses running between Jaipur and
Delhi. In how many ways can a man go to Delhi and
(c) 17 (d) 5
return by a different bus?
370. In an examination paper there are two groups, each (a) 132 (b) 134
containing 4 questions. A candidate is required to (c) 136 (d) 138
attempt 5 questions but not more than 3 questions 378. A department had 8 male and female employees
from any group. In how many ways can 5 questions each. A project team involving 3 male and 3 female
be selected? member needs to be chosen from the department
(a) 24 (b) 48 employees. How many different project teams can
(c) 96 (d) None of these be chosen?
(a) 3112 (b) 3136
371. A box contains 10 balls out of which 3 are red and
(c) 3143 (d) 3179
the rest are blue. In how many ways can a random
sample of 6 balls be drawn from the bag so that at 379. Five persons A, B, C, D and E occupy seats in a row
such that A and B sit next to each other. In how many
the most 2 red balls are included in the sample and
possible ways can these five people sit?
no sample has all the 6 balls of the same colour?
(a) 48 (b) 50
(a) 105 (b) 168
(c) 52 (d) 54
(c) 189 (d) 120
380. In an examination paper there are two groups, each
372. A cricket team of 11 players is to be formed from 20 containing 4 questions. A candidate is required to
players including 6 bowlers and 3 wicketkeepers. attempt 5 questions but not more than 3 questions
The number of ways in which a team can be formed from any group. In how many ways can 5 questions
having exactly 4 bowlers and 2 wicketkeepers is be selected?
(a) 20790 (b) 6930 (a) 48 (b) 54
(c) 10790 (d) 360 (c) 58 (d) 64

6 . 161
Basic Numeracy and Data Interpretation

381. If we toss a coin 1000 times and find that it comes up 389. An urn contains 5 red and 7 yellow balls. Two balls
tails 703 times, then the probability of heads is are drawn at random from the urn. The number of
(a) 0.703 (b) 0.382 elements in the space when one ball is red and other
(c) 0.297 (d) 4.83 is yellow is
382. If a man has 24 shirts and 9 ties then in how many (a) 12 (b) 24
ways can he choose a shirt and a tie? (c) 35 (d) 2
(a) 216 (b) 2.66 390. A coin is tossed twice. “Number of heads” is odd
(c) 33 (d) 15 and denoted by A whereas “number of tails is at
383. The number of ways in which 4 cards can be chosen least one” denoted by B. Cases favourable for
from 9 cards is A  B are
(a) 190 (b) 112 (a) HH, TT (b) HT, TT
(c) 110 (d) 126 (c) HT, HH (d) HT, TH
384. In a large metropolitan area, the probabilities that a 391. Four dice are thrown together. The probability of
family owns a colour television set, a black and white getting a sum less than 8 is
set, or both kind of sets are 0.86, 0.35 and 0.29. What
13 13
is the probability that a family owns either or both (a) (b)
kinds of sets? 1296 216
(a) 0.92 (b) 0.83 8 8
(c) 0.19 (d) 0.27 (c) (d)
216 435
385. If we randomly pick two television tubes in succes- 392. 5 boys and 5 girls sit in a row at random. The prob-
@UPSC_THOUGHTS

sion from a shipment of 240 television tubes of which ability that all the girls sit together is
15 are defective, what is the probability that they
both will be defective? 1 1
(a) (b)
7 7 32 42
(a) (b)
1932 1793 1 1
7 7 (c) (d)
(c) (d) 52 62
1912 1780
386. A box of fuse contains 20 fuses, of which 5 are 393. Letters of the word MATHEMATICS are placed at
defective. If 3 of the fuses are selected at random and random in a row. Probability of getting both the M
removed from the box without replacement, what is together is
the probability that all three fuses would be 2 11
defective? (a) (b)
11 2
1 1
(a) (b) 10 7
110 114 (c) (d)
1 1 7 10
(c) (d)
176 125 394. For two vacancies for the same post, a husband and
387. In how many ways can 10 people be seated on a a wife appear in an interview. If probability of
bench if only 4 seats are available? 4 2
husband’s selection is and wife’s selection is ,
5 5
(a) 5040 (b) 918
then the probability that both are selected is
(c) 1075 (d) 7850
8 7
388. A bag contains 3 black balls. What is the sample (a) (b)
25 12
space if the random experiment consists of choos-
ing 2 balls? 6 10
(c) (d)
(a) R1R2, R2R3, R1R3 5 6
(b) R1R1, R2R2, R3R3 395. A bag contains 9 white and 3 black balls. Another
(c) R2R3, R2R1, R2R2 bag contains 6 white and 10 black balls. If a ball is
(d) R1R2, R2R1, R2R3, R3R2, R1R3, R3R1 transferred from the first bag to the second bag and a

6 . 162
Basic Numeracy and Data Interpretation

ball is drawn from the second, find the probability 401. The probability that a man now aged 55 years will
that the ball drawn is white. 5
be alive in 1993 is , while the probability that his
8 5
19 41 wife now aged 53 will be alive in 1993 is . Prob-
(a) (b) 6
25 60 ability that in 1993 at least one of them will be alive
3 27 is
(c) (d)
28 68 15 16
(a) (b)
396. 5 persons enter the lift cabin on the ground floor of 16 15
an 8-floor building. Each of them independently and 13 14
(c) (d)
with equal probability, can leave the cabin at any 14 13
floor beginning with the first. Probability of all five 402. A proposal, to put more emphasis on improved su-
persons leaving at different floors is perannuation rather than wage increase, was put
5
P7 7
P5 by the union executive to the members. The follow-
(a) 7 (b) ing results are received:
5 75

57 Opinion
75
(c) 5 (d) 7P
P7 5 Members In favour Opposed Undecided Total
397. A card is taken out of a pack of 52 cards numbered 2 Skilled 800 200 300 1300
to 53. The probability that the number on the card is Unskilled 100 600 200 900
a prime number less than 20 is
@UPSC_THOUGHTS

2 2 Total 900 800 500 2200


(a) (b)
13 15
Probability that the member selected at random will
3 4
(c) (d) be skilled and in favour of the proposal is
13 15
(a) 0.289 (b) 0.364
398. There are 9 oranges and 12 guavas in a box. If the (c) 0.500 (d) 0.227
two fruits are chosen at random, the probability that
403. The probability of a company obtaining contract A
one is a orange and other is a guava is
is 0.3, contract B is 0.4 and contract C is 0.6. If the
(a) 0.62 (b) 0.91
company obtains none or only one of these contracts,
(c) 0.37 (d) 0.51
it will bid for D with a 0.8 probability of obtaining it.
399. A positive integer is chosen at random. The prob-
What is the probability that the company will ob-
ability that sum of the digits of its square is 33 is
tain C and D?
1 2 (a) 0.1344 (b) 0.125
(a) (b)
33 33 (c) 0.227 (d) 0.2016
1 404. Out of 5 mathematicians and 7 physicists, a com-
(c) (d) None of these
11 mittee consisting of 2 mathematicians and 3 physi-
400. A basket contains 10 oranges out of which 4 are rot- cists is to be less formed. In how many ways can it be
ten. If two oranges are taken out together in which done so that one particular physicists must be in the
one is good then the probability that the other is also committee?
good is (a) 350 (b) 150
17 5 (c) 105 (d) 495
(a) (b)
30 18 405. A man has two purses. One purse contains 5 copper
5 4 coins and 3 silver coins where as the second purse
(c) (d) contains 8 copper coins and 10 silver coins. The prob-
9 10

6 . 163
Basic Numeracy and Data Interpretation

ability of taking out copper coin from any of the 412. There are 6 positive and 8 negative numbers. Four
purses is numbers are chosen at random and multiplied. The
73 77 probability that the product is a positive number is
(a) (b) 500 503
65 144 (a) (b)
1001 1001
10 18
(c) (d) 505 101
51 93 (c) (d)
1001 1001
406. A non-leap year is selected randomly. Probability
413. A bag contains 6 red and 9 black balls. Two draws of
that the year has 53 Sundays is
4 balls are made. Find the probability that the first
1 1
(a) (b) draw will give 4 red and the second draw 4 black
2 5 balls, if the balls are returned to the bag after the first
1 1
(c) (d) draw.
4 7
(a) 0.1 (b) 0.01
407. A secretary wrote 5 letters to 5 different clients. Dif- (c) 0.001 (d) 0.0001
ferent addresses on 5 envelopes were also written.
414. Amar can hit a target with a pistol 3 times in 5 shots;
Without looking at the addresses what is the prob-
ability that the letters go into the right envelopes? Akbar can hit 2 times in 5 shots; and Anthony can
hit 3 times in 4 shots. If Amar, Akbar and Anthony
1 1 fire a volley of shots, what is the probability that two
(a) (b)
25 80 shots are hit?
1 1 (a) 0.15 (b) 0.32
(c) (d)
100 120 (c) 0.39 (d) 0.45
@UPSC_THOUGHTS

408. A single letter is selected from the word 415. From a bag containing 5 red, 4 white and 3 blue
‘DEVELOPMENT’. The probability that it is a vowel marbles, three marbles are drawn, with replacement.
is What is the probability that one is white and two are
7 3 red?
(a) (b)
11 5 2 2
4 1 (a) (b)
(c) (d) 19 17
11 8 2 2
409. The probability that an event happens in one trial of (c) (d)
13 11
an experiment is 0.4. Three independent trials of the n2
416. What is the fifth term of the sequence an ?
experiment are performed. The probability that the 3n 2
event A happens at least once is 25 5
(a) (b)
(a) 0.936 (b) 0.784 17 17
(c) 0.904 (d) 0.888 9 8
(c) (d)
410. Suppose six coins are flipped. Then the probability 11 7
of getting at least one tail is 417. The first term of an A.P. is 5 and its common differ-
71 53 ence is –3. Find the 11th term of the A.P.
(a) (b)
72 54 (a) 20 (b) –25
63 1 (c) 30 (d) None of these
(c) (d)
64 12
411. A bag contains 3 white balls and 2 black balls. An-
418. Find the sum of the series 1 + 4 + 7 + ... to 25 terms.
other bag contains 2 white balls and 4 black balls. A
(a) 1025 (b) 1850
bag and a ball are picked at random. The probabil-
(c) 925 (d) 1295
ity that the ball will be white is
7 7 419. The sum of all natural numbers between 2 and 101
(a) (b) which are divisible by 5 is
11 30
5 7 (a) 1295 (b) 1050
(c) (d) (c) 1690 (d) 3285
11 15

6 . 164
Basic Numeracy and Data Interpretation

420. The sum of two-digit natural numbers divisible by 3 (a) 156 (b) 120
is (c) 100 (d) Cannot be determined
(a) 1665 (b) 1329
430. The population of a country doubles every two
(c) 1831 (d) 5496
decades. How many decades will it take for the
1
421. The nth term of the series 3, 3 , 1, ... is , then n is population to grow from 1000 to 512000?
243
(a) 12 (b) 13 (a) 15 (b) 512
(c) 14 (d) 15 (c) 18 (d) 21

422. A dramatic society consists of members each one 431. What is the area of the shaded portion in the given
22
progressively differing in age by 3 months. The figure?
youngest member is 7 years old and the sum of the 7
ages of all the members is 250 years. How many
members does the dramatic society have?
(a) 20 (b) 25 m 7c
7c m
(c) 30 (d) Cannot be found out
423. The 99th term of the series
2 + 7 + 14 + 23 + 34 + ... is
(a) 9999 (b) 9998
(c) 10000 (d) None of these (a) 42 sq cm (b) 48 sq cm
1 1 1
424. If ax b y
cz and a, b, c are in G.P. then x, y, z are in (c) 76 sq cm (d) 152 sq cm
@UPSC_THOUGHTS

(a) A.P. (b) G.P. 432. The area of the shaded portion is
(c) H.P. (d) None of these

3 5 9 17
425. The next term of the series ... is 1 cm
2 4 8 16 2 cm
2cm
25 29
(a) (b)
32 32
37 33
4 cm
(c) (d)
32 32 (a) 8 sq cm (b) 6 sq cm
1 1 (c) 16 sq cm (d) 4 sq cm
426. The value of 3 1 ... is equal to
3 9 433. An isosceles right triangle has an area of 200 sq cm.
20 9
(a) (b) The area of a square drawn on its hypotenuse is
9 20
9 4 (a) 400 sq cm (b) 400 2 sq cm
(c) (d)
4 9 (c) 800 sq cm (d) 800 2 sq cm
427. The sum of numbers between 100 and 500 which 434. The perimeter of a rectangle is 48 m and its area is
are divisible by 6 is
135 m2. The sides of the rectangle are
(a) 67 (b) 498
(a) 15m, 9m (b) 19m, 5m
(c) 2010 (d) 20100
(c) 45m, 3m (d) 27m, 5m
428. If the average of 148, 146, 144, 142, 140,… in A.P. is
125, then the total numbers in the series will be 435. The areas of a rectangle and a square are equal. The
(a) 18 (b) 24 side of the square is 5 cm and the smaller side of the
(c) 30 (d) 48 rectangle is half that of the square. The length of the
429. A clock rings once at 1 o’clock, twice at 2 o’clock and other side of the rectangle is
thrice at 3 o’clock and so on. How many times in (a) 5 cm (b) 8 cm
total will the clock ring in a day? (c) 10 cm (d) 12.5 cm

6 . 165
Basic Numeracy and Data Interpretation

436. A rectangular plate of size 9 cm by 4 cm is to be 444. A towel, when bleached, was found to have lost 20%
melted and cast into 4 squares of equal area but of of its length and 10% of its breadth. The percentage
same thickness as the rectangular plate. The side of of decrease in area is:
each such square is (a) 10% (b) 10.08%
(a) 2 cm (b) 3 cm (c) 20% (d) 28%
(c) 4 cm (d) 4.5 cm 445. What is the least number of square tiles required to
437. The sides of a rectangular field are in the ratio 3 : 1 pave the floor of a room15 m 17 cm long and 9 m 2
and its area 7500 sq m. The cost of fencing the field cm broad?
at 25 paise per metre is (a) 814 (b) 820
(a) Rs 100 (b) Rs 94.50 (c) 840 (d) 844
(c) Rs 80 (d) Rs 57.50
446. An error of 2% in excess is made while measuring
438. In a circular plot of radius 105 m, a path of uniform the side of a square. The percentage of error in the
width all around its inside border with an area of calculated area of the square is:
4466 sq m is constructed. The width of the path is (a) 2% (b) 2.02%
(a) 8 m (b) 7 m (c) 4% (d) 4.04%
(c) 6.5 m (d) 6 m
447. The sides of a triangle are 3 cm, 4 cm and 5 cm. The
439. The area of a circle is 13.86 hectares. The cost of area (in cm2 ) of the triangle formed by joining the
fencing it at the rate of 60 paise per metre is mid-points of the sides of this triangle is:
(a) Rs 784.00 (b) Rs 788.00
3 3
(c) Rs 792.00 (d) Rs 796.00 (a) (b)
@UPSC_THOUGHTS

4 2
440. The number of revolutions made by a bicycle wheel (c) 3 (d) 6
56 cm in diameter in covering a distance of 1.1 km is 448. A triangle and a parallelogram are constructed on
(a) 31.25 (b) 625 the same base such that their areas are equal. If the
(c) 62.5 (d) 312.5 altitude of the parallelogram is 100 m, then the alti-
441. The area of the smallest circle containing a square of tude of the triangle is:
side ‘a’ is (a) 10 2 m (b) 100 m
a
(c) 100 2 m (d) 200 m
a a 449. The area of a field in the shape of a trapezium mea-
sures 1440 m2. The perpendicular distance between
a its parallel sides is 24 m. If the ratio of the parallel
sides is 5 : 3, the length of the longer parallel side is:
a a2
(a) (b) (a) 45 m (b) 60 m
2 2
(c) 75 m (d) 120 m
(c) 2 a2 (d) a
2 450. If a wire is bent into the shape of a square, then the
442. A square and an equilateral triangle have equal
area of the square is 81 sq. cm. When the wire is bent
perimetres. If the area of the equilateral triangle is
into a semi-circular shape, then the area of the semi-
16 3 cm2, then the side of the square is
circle will be
(a) 4 cm (b) 4 2 cm
(a) 22 cm2 (b) 44 cm2
(c) 6 2 cm (d) 6 cm (c) 77 cm2 (d) 154 cm2
443. The diagonal of a rectangle is 41 cm and its area is 451. The edges of a cuboid are in the ratio 1 : 2 : 3 and its
20 sq. cm. The perimeter of the rectangle must be: surface area is 88 cm2. The volume of the cuboid is:
(a) 9 cm (b) 18 cm (a) 24 cm3 (b) 48 cm3
(c) 20 cm (d) 41 cm (c) 64 cm3 (d) 120 cm3

6 . 166
Basic Numeracy and Data Interpretation

452. A hall is 15 m long and 12 m broad. If the sum of the 2 4


areas of the floor and the ceiling is equal to the sum (a) cm (b) cm
9 9
of areas of the four walls, the volume of the hall is: 9 9
(a) 720 (b) 900 (c) cm (d) cm
4 2
(c) 1200 (d) 1800
461. What is the angle of elevation of the sun when the
453. How many cubes of 3 cm edge can be cut out of a length of the shadow of a tree is 3 times the height
cube of 18 cm edge?
of the tree?
(a) 36 (b) 216
(a) 30° (b) 45°
(c) 218 (d) 432
(c) 60° (d) 90°
454. Three cubes with sides in the ratio 3 : 4 : 5 are melted
462. From a point P on a level ground, the angle of eleva-
to form a single cube whose diagonal is 12 3 cm. tion of the top of a tower is 30°. If the tower is 100 m
The sides of the cubes are: high, the distance of point P from the food of the
(a) 3 cm, 4 cm, 5 cm tower is:
(b) 6 cm, 8 cm, 10 cm (a) 149 m (b) 156 m
(c) 9 cm, 12 cm, 15 cm (c) 173 m (d) 200 m
(d) None of these
463. The angle of elevation of the top of a tower at any
455. Two right circular cylinders of equal volumes have point on the ground is 30° and moving 20m towards
their heights in the ratio 1 : 2. The ratio of their radii the tower it becomes 60°. The height of tower is
is:
(a) 10 m (b) 10 3m
(a) 1 : 2 (b) 1 : 4
@UPSC_THOUGHTS

10 3
(c) 2 : 1 (d) 2:1 (c) m (d) none of these
3
464. A kite is flying with a string 150m long. It makes an
456. 66 cubic centimetres of silver is drawn into a wire 1
angle of 60° with the horizontal. Find the height of
mm in diameter. The length of the wire in metres will
the kite from the ground.
be:
(a) 132 m (b) 139.5 m
(a) 84 (b) 90
(c) 129.9 m (d) 175.2 m
(c) 168 (d) 336
465. From the top of a cliff 100m high, the angle of depres-
457. A right triangle with sides 3 cm, 4 cm and 5 cm is sion of the top and bottom of a tower are observed to
rotated about the side of 3 cm to form a cone. The be 32.6º and 45°. Find the height of the tower.
volume of the cone so formed is: (a) 47 m (b) 55.7 m
(a) 12π cm3 (b) 15π cm3 (c) 36.05 m (d) 26.78 m
(c) 16π cm3 (d) 20π cm3
466. If logx 4 = 0.4, then the value of x is:
458. The volume of the largest right circular cone that (a) 1 (b) 4
can be cut out of a cube of edge 7 cm is: (c) 16 (d) 32
(a) 13.6 cm3 (b) 89.8 cm3
(c) 121 cm 3
(d) 147.68 cm3 log 8
467. is equal to:
log 8
459. If the radius of a sphere is increased by 2 cm, then its
surface area increases by 352 cm2. The radius of the 1 1
(a) (b)
sphere before the increase was: 8 4
(a) 3 cm (b) 4 cm 1 1
(c) (d)
(c) 5 cm (d) 6 cm 2 8
460. A cylindrical vessel of radius 4 cm contains water. A 1
468. 2 log10 5 + log10 8 – log10 4 = ?
solid sphere of radius 3 cm is lowered into the water 2
until it is completely immersed. The water level in (a) 2 (b) 4
the vessel will rise by: (c) 2 + 2 log10 2 (d) 4 – 4 log10 2

6 . 167
Basic Numeracy and Data Interpretation

Directions: In the following questions, a question is 476. A shop sells only toy cars and toy boats. It sells
given followed by two statements. You have to consider each in only two colours—red and blue. Last year
the statements and decide whether they provide sufficient the shop sold 900 toys, half of which were red. How
data to answer the given question. Give answer many toy boats did the shop sell?
(a) if Statement I alone is sufficient but statement I. The shop sold three times as many blue toy
II alone is not sufficient boats as red toy cars last year.
(b) if each Statement alone is sufficient II. The shop sold half as many blue toy cars as
(c) if both Statements together are sufficient but blue toy boats last year.
neither Statement alone is sufficient
(d) if Statements I and II are not sufficient and 477. If x is a positive integer, is x divisible y 48?
more data is required to answer the question. I. x is divisible by 8.
II. x is divisible by 6.
469. What is the value of integer x?
I. x is a multiple of 7. 478. A box contains a number of toys, including 5 dolls.
II. 575 < x < 590 If two toys are selected from the box at random,
470. Shyam can chop down 4 trees in an hour. How what is the probability that both will be dolls?
long does it take Mohan to chop down 4 trees? I. The probability that the first toy picked will
I. Mohan takes twice as long as Shyam to chop 1
be a doll is .
down trees. 2
II. The probability that the second toy picked
II. Mohan spends 6 hours per day chopping
4
down trees. will be a doll is .
9
@UPSC_THOUGHTS

471. 479. Rahul, Anurag and Vivek started a business


A together. In what proportion would the annual
B
profit be distributed among them?
r I. Rahul got one-fourth of the profit.
C D
p q II. Rahul and Vivek contributed 75 per cent of
the total investment.
E F
480. Ravi, Gagan and Nitin are running a business firm
If both ABDC and CDFE are parallelograms, what in partnership. What is Gagan’s share in the profit
is q + r?
earned by them?
I. r = 70
I. Ravi, Gagan and Nitin invested the amounts
II. p = 110
in the ratio of 2 : 4 : 7.
472. Is the average (arithmetic mean) of x, y, and z equal
II. Nitin’s share in the profit is Rs 8750.
to 8?
I. Three times the sum of x, y, and z is equal 481. How long will Machine Y, working alone, take to
to 72. produce x candles?
II. The sum of 2x, 2y, and 2z is equal to 48. I. Machine X produces x candles in 5 minutes.
473. What is the value of x? II. Machine X and Machine Y working at the
I. x2 + 5x + 4 = 0 same time produce x candles in 2 minutes.
II. x is not prime. 482. A and B together can complete a task in 7 days. B
474. What is the sum of a, b, and c? alone can do it in 20 days. What part of the work
I. 2a + b + 3c = 45 was carried out by A?
II. a + 2b = 30 I. A completed the job alone after A and B
475. What is the value of integer x? worked together for 5 days.
I. x
64 = 4 II. Part of the work done by A could have been
II. x2 = 2x + 8 done by B and C together in 6 days.

6 . 168
Basic Numeracy and Data Interpretation

483. How much time did X take to reach the destination? 488. What is the area of the circle?
I. The ratio between the speeds of X and Y is I. An arc of length 4 cm subtends an angle of
3 : 4. 60° at the centre.
II. Y takes 36 minutes to reach the same II. A chord of length 5 cm subtends an angle
destination. of 90° at the centre.
484. Two towns are connected by railway. Can you find 489. What is the capacity of a cylindrical tank?
the distance between them? I. Radius of the base is half of its height which
I. The speed of mail train is 12 km / hr more is 28 metres.
than that of an express train. II. Area of the base is 616 sq. metres and its
II. A mail train takes 40 minutes less than an height is 28 metres.
express train to cover the distance. 490. Is a given rectangular block, a cube?
485. The towns A, B and C are on a straight line. Town I. At least 2 faces of the rectangular block are
C is between A and B. The distance from A to B is squares.
100 km. How far is A from C? II. The volume of the block is 64.
I. The distance from A to B is 25% more than
Direction: Each of the questions consists of a question
the distance from C to B.
1 followed by three statements. You have to study the questions
II. The distance from A to C is of the distance and the statements and decide which of the statement/s is/
4
from C to B. are necessary to answer the questions.

Directions: Each of the questions given below consists of 491. What is the area of the hall?
a statement and/ or a question and two statements numbered I. Material cost of flooring per sq. metre is 250.
@UPSC_THOUGHTS

I and II given below it. You have to decide whether the data II. Labour cost of flooring the hall is 3,500.
provided in the statement(s) is/are sufficient to answer the III. Total cost of flooring the hall is 14,500.
question. Read both the statements and (a) I and II only
(b) II and III only
Give answer (a) if the data in Statement I alone
(c) All I, II and III
are sufficient to answer the question, while
(d) Any two of the three
the data in Statement II alone are not sufficient
to answer the question; 492. What was the percentage of discount offered?
Give answer (b) if the data in Statement II alone I. Profit earned by selling the article for Rs 252
are sufficient to answer the question, while after giving discount was Rs 52.
the data in Statement I alone are not sufficient II. Had there been no discount the profit earned
would have been Rs 80.
to answer the question;
III. Had there been no discount the profit earned
Give answer (c) if the data either in Statement
would have been 40%.
I or in Statement II alone are sufficient to
(a) I and II only
answer the question;
(b) II and either I or III only
Give answer (d) if the data even in both Statements
(c) I and either II or III only
I and II together are not sufficient to answer
(d) None of these
the question.
493. What is the speed of the train?
486. The area of a playground is 1600 m2. What is its I. The train crosses a signal pole in 13 seconds.
perimeter? II. The train crosses a platform of length 250 m
I. It is a perfect square playground. in 27 seconds.
II. It costs Rs 3200 to put a fence around the III. The train crosses another train running in
playground at the rate of Rs 20 per metre. the same direction in 32 seconds.
487. Area of a square is equal to the area of a circle. (a) I and II only
What is the circumference of the circle? (b) I and III only
I. The diagonal of the square is x inches. (c) II and III only
II. The side of the square is y inches. (d) Any two of the three

6 . 169
Basic Numeracy and Data Interpretation

494. What is the population of state A? 499. Who among M, N, P, Q and R earns the maximum?
I. After increasing the population of state A by I. M earns less than P but not less than R.
15% it becomes 1.61 lakh. II. Q earns more than M but not as much as N.
II. Ratio of population of state A to that of state III. N earns more than M and R.
B is 7 : 8 respectively. (a) Question cannot be answered even with the
III. Population of state B is 1.6 lakh. data in all three statements
(a) I only (b) I and II only
(b) II and III only (c) I and II only or I and III only
(c) I and II only (d) I and III only
(d) Either only I or II and III
500. What is the price of one dozen oranges.
495. How many workers are required for completing the
I. Total cost of 2 dozen oranges and 3 dozen
construction work in 10 days?
bananas is Rs 110.
I. 20% of the work can be completed by 8
II. Total cost of 3 dozen apples and 1 dozen
workers in 8 days.
bananas is Rs 170.
II. 20 workers can complete the work in 16
days. III. Total cost of 1 dozen each of oranges, apples
III. one eighth of the work can be completed and bananas is Rs 95.
by 8 workers in 5 days. (a) I and III only or II and III only
(a) I and III only (b) I and II only or II and III only
(b) II and III only (c) II and III only
(c) I only (d) All I, II and III are required to answer the
@UPSC_THOUGHTS

(d) Any one of the three question.


496. What is the area of the isosceles triangle? 501. What is R’s share of profit in a joint venture?
I. Perimeter of the isosceles triangle is 14 metres I. Q started business investing Rs 80,000.
II. Base of the triangle is 8 metres. II. R joined him after 3 months.
III. Height of the triangle is 5 metres. III. P joined after 4 months with a capital of
(a) I and II only Rs 1,20,000 and got Rs 6,000 as his share of
(b) II and III only profit.
(c) I and II only or II and III only (a) All I, II and III
(d) I and III only (b) I and III only
497. What is the speed of the train A? (c) II and III only
I. Train A crosses 200 metres long train B (d) Even with all I, II, and III, the answer cannot
running in opposite direction in 20 seconds. be arrived at
II. Speed of train B is 60 kmph.
III. Length of train A is twice that of train B. 502. Three friends P, Q and R started a partnership
(a) I and II only business investing money in the ratio of 5 : 4 : 2
(b) II and III only respectively for a period of 3 years. What is the
(c) I and III only amount received by P as his share in the total profit?
(d) All I, II and III I. Total amount invested in the business in Rs
498. What is the cost of flooring a rectanaular hall? 22,000.
I. Perimeter of the rectangle is 60 metres. II. Profit earned at the end of 3 years is 3/8 of
II. Angle between diagonal and breadth is 60°. the total investment.
III. Cost of flooring per square metre is Rs 125. III. The average amount of profit earned per year
(a) I and III only is Rs 2750.
(b) II and III only (a) I or II or III
(c) I and III only or II and III only (b) Either III only, or I and II together
(d) Question cannot be answered even with data (c) Any two of the three
in all three statements (d) All I, II and III are required

6 . 170
Basic Numeracy and Data Interpretation

503. In how many days can 10 women finish a work? 508. I. p2 + 16 = 8p


I. 10 men can complete the work in 6 days. II. 4q2 + 64 = 32q
II. 10 men and 10 women together can complete 509. I. 2p2 + 12p + 16 = 0
3
the work in 3 days. II. 2q2 + 14q + 24 = 0
7
III. If 10 men work for 3 days and thereafter 10 Directions (Qs. 510-513): Study the following graph
women replace them, the remaining work is carefully and answer the questions given below it.
completed in 4 days.
Percentage profit earned by two companies
(a) Any two of the three
A and B over the given years.
(b) I and II only
Percentage profit/loss =
(c) II and III only
(d) I and III only Income – Expenditure
100
Expenditure
504. 8 men and 14 women are working together in a
field. After working for 3 days, 5 men and 8 women B
leave the work. How many more days will be A
70
required to complete the work? 65

Percentage of Profit
60
I. 19 men and 12 women together can complete 55
the work in 18 days. 50
45
II. 16 men can complete two-third of the work 40
in 16 days. 35
30
III. In a day, the work done by three men is equal 25
@UPSC_THOUGHTS

to the work done by four women. 20


15
(a) I only 10
(b) II only 5
0
(c) III only 1995 1996 1997 1998 1999 2000
(d) I or II or III
Years
505. If both the pipes are opened, how many hours will
be taken to fill the tank? 510. Expenditures of company B in 1996 and 1997 are
I. The capacity of the tank is 400 litres. Rs 12 lakh and Rs 14.5 lakh respectively. What was
II. The pipe A fills the tank in 4 hours. the total income of company B in 1996 and 1997
III. The pipe B fills the tank in 6 hours. together (in lakh rupees)?
(a) Only I and II (a) 39.75 (b) 37.95
(b) Only II and III (c) 38.75 (d) 38.5
(c) All I, II and III 511. Ratio of expenditures of companies A and B in 1999
(d) Any two of the three was 3 : 4 respectively. What was the respective ratio
Directions: In each question below one or two equation(s) of their incomes in 1999?
is/are provided. On the basis of these you have to find (a) 21 : 26 (b) 13 : 14
out relation between p and q. (c) 14 : 13 (d) None of these
Give answer (a) if p = q 512. If the expenditures of company A and B in 2000
Give answer (b) if p > q were equal and the total income of the two
Give answer (c) if q > p companies was Rs 5.7 lakh, what was the total
Give answer (d) if p q and expenditure of the two companies in 2000?
506. I. p2 + 24 = 10p (a) 4 lakh
II. 2q2 + 18 = 12q (b) 2 lakh
507. I. p2 + 2p – 3 = 0; (c) 4.2 lakh
II. 2q2 –7q + 6 = 0 (d) Can’t be determined

6 . 171
Basic Numeracy and Data Interpretation

513. If the incomes of company B in 1997 and 1998 were Percentage Population Below Poverty Line
in the ratio of 2 : 3 respectively. What was the in Six Different States and Proportion
respective ratio of expenditures, of that company of Males and Females
in these two years? Percentage Proportion of Males
(a) 20 : 29 (b) 9 : 10 and Females
(c) 29 : 45 (d) 10 : 29 State Below Below Above
Powerty Line Poverty Line Poverty Line
Directions (Qs. 514-517): Study the following table M F M F
carefully and answer the questions given below:
A 12 3 : 2 4 : 3
Number of students admitted to the school and left B 15 5 : 7 3 : 4
C 25 4 : 5 2 : 3
the school at five different schools in six academic
D 26 1 : 2 5 : 6
years since inception in 1990. E 10 6 : 5 3 : 2
F 32 2 : 3 4 : 5
School A B C D E

Year Ad. Left Ad. Left Ad. Left Ad. Left Ad. Left
518. If the total population of state A is 3000, what is the
1990 1025 — 950 — 1100 — 1500 — 1450 —
1991 230 120 350 150 320 130 340 150 250 125 approximate number of females above poverty line
1992 190 110 225 115 300 150 300 160 280 130 in the state?
1993 245 100 185 110 260 125 295 120 310 120 (a) 1700 (b) 2112
1994 280 150 200 90 240 140 320 125 340 110 (c) 1200 (d) 1950
1995 250 130 240 120 310 180 360 140 325 115
519. If the population of states C and D together is 18,000,
what is the total number of female members below
@UPSC_THOUGHTS

514. What is the difference between the total number of poverty line in the two states?
students admitted at school D and the total number (a) 5000 (b) 5500
of students admitted at school B during the years (c) 4800 (d) Data inadequate
1991 to 1995? 520. If the number of male members below poverty line
(a) 514 (b) 1065 for state B is 500, what is the total population of
(c) 965 (d) 415 that state?
(a) 14400 (b) 5400
515. What is the pooled average of the number of (c) 8000 (d) 9800
students studying in five schools in the year 1992?
521. If the number of females above poverty line for state
(a) 1494 (b) 1294 E is 19,800, what will be the number of male
(c) 8810 (d) 7470 members below poverty line?
516. The total number of students who left the school C (a) 5500 (b) 3000
(c) 2870 (d) Data inadequate
is approximately what percent of the total number
of students admitted at school C since 1990 till Directions (Qs. 522-526): Study the following table
1995? to answer the given questions.
(a) 25% (b) 48%
Centre and Post-wise Number of Candidates
(c) 36% (d) 29%
Post Officer Clerk Field Supervisor Specialist
517. What was the number of students studying in Centre Officer Officer
Bangalore 2000 5000 50 2050 750
school B in the year 1994?
Delhi 15000 17000 160 11000 750
(a) 465 (b) 1910 Mumbai 17000 19500 70 7000 900
(c) 1449 (d) None of these Hyderabad 3500 20000 300 9000 1150
Kolkata 14900 17650 70 1300 1200
Directions (Qs. 518-521): Study the following table Lucknow 11360 15300 30 1500 650
carefully and answer the questions given below: Chennai 9000 11000 95 1650 500

6 . 172
Basic Numeracy and Data Interpretation

522. In Kolkata, number of specialist officers is (a) 18 (b) 22


approximately what per cent of that of officers? (c) 20 (d) 21
(a) 8.7 (b) 9 525. Which centre has 300% more number of clerks as
(c) 6.5 (d) 8 compared to Bangalore?
523. What is the difference between total number of (a) Lucknow (b) Hyderabad
officers and clerks? (c) Delhi (d) Chennai
(a) 31,690 (b) 34,180 526. Which centre has the highest number of
(c) 32,690 (d) 28,680 candidates?
524. In Chennai, the number of clerks is approximately (a) Chennai (b) Kolkata
how much per cent more than that of officers? (c) Hyderabad (d) Mumbai

Directions (Qs. 527-531): Study the given table carefully and answer the question given below it.
The given table represents the no. of candidates appeared and
qualified during 1991 to 1996 from 6 states

Year 1991 1992 1993 1994 1995 1996


State app. qual. app. qual. app. qual. app. qual. app. qual app. qual.

A 5600 840 7250 925 8250 876 7856 824 8349 932 7964 853
B 7200 864 8100 840 7865 792 8425 896 7658 878 8107 940
C 4850 588 6450 650 7120 685 7763 735 6984 792 7058 827
@UPSC_THOUGHTS

D 6325 745 7185 795 8545 842 6987 898 5896 685 6754 746
E 5200 640 6225 685 7962 934 7645 888 7389 843 7766 812
F 6500 820 7380 860 6895 788 7844 762 8105 798 8934 911

527. In which of the given years the No. of canditates (a) 1.25 (b) 2.25
appeared from state D has maximum percentage of (c) 2.50 (d) 1.00
qualified candidates? 530. What is the difference between total qualified
(a) 1992 (b) 1995
candidates in six states in year 1993 and 1996?
(c) 1993 (d) 1994
(a) 217 (b) 172
528. What is difference between total candidates (c) 168 (d) 182
appeared in six states in year 1991 and 1993?
(a) 6915 (b) 10962 531. What is per cent of the total qualified candidates to
(c) 10692 (d) 9615 the total number of appeared candidates among all
the six states in 1996?
529. The percentage increase/decrease in the number
of the appreared to the qualified candidates in the (a) 15 (b) 13
state B is—in 1991. (c) 19 (d) 11

Directions (Qs. 532-534): Study the following table carefull and answer the question given below.
Marks obtained by seven students in different subject
in their annual school examination
Student/Stubject A B C D E F G
Physics out of 100 65 58 73 75 68 56 51
Chemistry out of 100 59 63 69 52 71 49 72
Maths out of 150 132 105 141 128 119 120 135
Social Studies out of 200 138 126 162 144 150 118 132
English out of 50 32 27 33 41 29 35 37
Biology out of 150 105 98 112 106 119 107 96
Total out Of 750 531 477 590 546 556 485 523

6 . 173
Basic Numeracy and Data Interpretation

532. Approximately what is the maximum difference 534. What is the difference between the percentage of
between the percentage of total marks obtained by marks obtained by student E in Chemistry and
any two students? Maths together and the percentage of marks
(a) 15 (b) 10 obtained by student A in Social Studies and
(c) 13 (d) 17 English together?
533. Approximately what is the average of marks (a) 6 (b) 8
obtained by the seven students in Biology? (c) 12 (d) 14
(a) 126 (b) 106
(c) 116 (d) 96

Directions (Qs. 535-539): Study the following table carefully and answer the questions given below it.
A factory was opened in 1994 with certain initial strengths in different units as shown in the table. In the
beginning of the subsequent years some of the workers left and some new workers were deployed. No worker left or
joined in between. Details are given in the table given below. Study it carefully and answer the questions that follow:

UNIT
Year A B C D E
1994 156 132 98 76 125
(Initial Left Joined Left Joined Left Joined Left Joined Left Joined
Strength)
1995 12 15 23 32 12 36 06 26 11 13
1996 17 18 16 14 08 19 17 28 11 15
@UPSC_THOUGHTS

1997 09 20 12 12 17 14 09 16 19 16
1998 32 40 14 17 23 35 12 23 23 14
1999 22 35 11 15 18 25 14 24 32 38
2000 26 32 17 21 13 18 11 19 21 36

535. What was the strength of Unit B in 1998? 538. What was the total strength of workers in all the
(a) 142 (b) 125 five units in 1996?
(c) 135 (d) 124 (a) 570 (b) 607
536. In 1999 the strength of workers was maximum in (c) 670 (d) 690
which unit?
539. What was the approximate increase/decrease in
(a) C (b) D
(c) A (d) B the strength of the workers in unit D in 1998 with
respect to its initial strength?
537. The strength of workers in unit C in 1996 is
(a) 57.37% increase
approximately what percent of the strength in unit
E in 1997? (b) 64.47% increase
(a) 98 (b) 109 (c) 64.47% decrease
(c) 104 (d) 110 (d) 57.37% decrease

Directions (Qs. 540-544): Study the following table carefully and answer the questions given below:
Percent Marks obtained by 6 students in different subjects
Students/ Physics Chemistry Maths History Geography English
Subject (out of 150) (out of 75) (out of 200) (out of 100) (out of 50) (out of 75)
A 77 63 89 55 64 72
B 69 72 71 78 69 66
C 82 78 69 65 75 57
D 73 81 76 67 58 63
E 58 69 54 74 66 75
F 66 57 61 62 71 59

6 . 174
Basic Numeracy and Data Interpretation

540. What is the total marks obtained by B in all the 547. For company A, in which year is the percentage
subjects? increase decrease in the production from the
(a) 425 (b) 542 previous year the highest?
(c) 469.5 (d) 461.5 (a) 2001 (b) 1995
541. What is the average marks obtained by 6 students (c) 1998 (d) 1996
in Chemistry out of 75 marks? 548. What is the difference in the total production of the
(a) 55.5 (b) 70.5 two companies for the given years?
(c) 52.5 (d) 72.5 (a) 2700000 (b) 3100000
542. What is the difference in the total marks obtained (c) 270000 (d) 310000
by C in Physics and Chemistry and that obtained 549. Which of the following is the closest average
by E in the same subjects? production in lakh units of company B for the given
(a) 43 (b) 38 years?
(c) 38.75 (d) 52.75 (a) 4.1 (b) 3.9
543. What is the per cent marks obtained by A in both (c) 4.0 (d) 3.75
Maths and History? Find upto two decimal places. Directions (Qs. 550-553): Study the following graph
(a) 75.88 (b) 77.67 carefully and answer the questions given below it:
(c) 48.67 (d) 78.88
Per cent profit earned by two companies A and B
544. What is the average marks obtained by 6 students over the years 1991 to 1997.
in Geography out of 50 marks? Find upto two
decimal places. 100 A
B
@UPSC_THOUGHTS

(a) 38.18 (b) 31.47 90


(c) 33.58 (d) 36.26 80
Per cent Profit

70
Directions (Qs. 545-549): Study the following graph 60
50
to answer the given questions.
40
Production of two Companies A and B over 30
the years (Production in lakh units) 20
10
0

1991 1992 1993 1994 1995 1996 1997

Year
550. Investment of company B in 1997 is more by 40%
than that in the previous year. Income in 1997 was
what per cent of the investment in 1996?
(a) 252% (b) 280%
(c) 242% (d) 52%

1994 1995 1996 1997 1998 1999 2000 2001


551. Income of company A in 1995 was Rs 21.7 lakh.
What was the investment?
545. For company A, what is the per cent decrease in (a) Rs 14 lakh (b) Rs 15.4 lakh
production from 1994 to 1995? (c) Rs 15.8 lakh (d) Rs 14.6 lakh
(a) 25 (b) 50 552. Income of company A in 1995 is equal to the
(c) 75 (d) 10 investment of company B in 1996. What is the ratio
546. In 2001, the production of company B is of the investment of company A in 1995 to the
approximately what per cent of that of in 2000? investment of company B in 1996?
(a) 60 (b) 157 (a) 37 : 36 (b) 31 : 20
(c) 158 (d) 50 (c) 20 : 31 (d) Data inadequate

6 . 175
Basic Numeracy and Data Interpretation

553. Investment of company B in 1993 was Rs 15,40,000, (c) Income of company B was more than the
what was its income in that year? income of company C by 10% of the
(a) Rs 22.33 lakh (b) Rs 22 lakh expenditure
(c) Rs 22.23 lakh (d) Rs 23.33 lakh (d) None of these
558. Expenditure of company B in 1996 is equal to the
Directions (Qs. 554-559): Study the following graph
income of company A in 1998. What is the respective
carefully and answer the questions given below it.
ratio of the expenditures of companies B and A in
Per cent profit earned by three 1996 and 1998 respectively?
companies A, B and C over the (a) 15 : 8 (b) 3 : 2
years 1994 to 2000 (c) 33 : 20 (d) 7 : 5
(Profit = Income – Expenditure)
559. Expenditures of company B in 1998 was Rs 6.5 lakh
A B C which was equal to the income of company C
100
90
in 1996. What was the ratio of income of company
80 B in 1998 to the expenditure of company C in
Percent Profit

70 1996?
60 (a) 1 : 42 (b) 81 : 25
50 (c) 25 : 81 (d) 18 : 25
40
30 Directions (Qs. 560-564): Study the following graph
20 and answer the questions given below.

7020
10
1994 1995 1996 1997 1998 1999 2000
@UPSC_THOUGHTS

8000
Years

6250
7000

5730
554. Income of company C in 1999 was Rs 8.5 lakhs. 6000

4970
4940
4800
What was its income in the year 2000?
Price (Rs)

5000
3980

(a) Rs 9.35 lakh


3210

4000
(b) Rs 9.5 lakh
(c) Rs 9.15 lakh 3000
(d) Cannot be determined 2000

555. Income of company C in 1999 was Rs 1,68,000. 1000


What was the expenditure in that year?
Feb. Mar. April May June July Aug. Sep.
(a) Rs 2,68,000 (b) Rs 1,50,000 1999 1999
(c) Rs 1,05,000 (d) Rs 1,00,800 Month
556. Expenditure of the three companies in the year 2000 560. In how many months the increase in the price of
were equal. What was the ratio of incomes of crude oil was more than 10% with respect to the
Companies A, B and C respectively in that year? corresponding previous years?
(a) 25 : 27 : 24 (b) 35 : 37: 34 (a) 2 (b) 3
(c) 15 : 17 : 14 (d) 32 : 35 : 34 (c) 4 (d) 6
557. Expenditures of companies B and C in 1995 were 561. If in April the crude oil price would have been lesser
equal. Then which of the following statements is than the given by Rs 223 per metric tonne then how
definitely true about the incomes of two companies much would have been the percentage increase in
in 1995? price over the earlier month?
(a) Incomes of companies B and C in 1995 were (a) 10 (b) 12
equal. (c) 14 (d) 15
(b) Income of company C was more than the 562. Which month/s experienced more than 10 per cent
income of company B by 10% of the but less than 20 per cent increase in the price of
expenditure crude oil over the earlier month?

6 . 176
Basic Numeracy and Data Interpretation

(a) July and September 568. The total production in all states in 1999 was
(b) June and September approximately what per cent of that in 1998?
(c) April and July (a) 90% (b) 110%
(d) March and July (c) 120% (d) 75%
563. What is the approximate percentage increase in the
Directions (Qs. 569-573): Study the following graph
price of crude oil from February to September?
carefully and answer the questions given below it:
(a) 100 (b) 140
(c) 130 (d) 120
564. Which month/s had less than one per cent increase
in crude oil price over the earlier month? 600
(a) August and April only 500
(b) April only 400
(c) June only 300
(d) May only 200
100
Directions (Qs. 565-568): Study the following graph
carefully and answer the questions given below:
Production of wheat in 6 states
in 1997, 1998 and 1999 Districts
Production of Wheat (in lakh tonnes)

70 1997 569. How many districts in Himachal Pradesh were


@UPSC_THOUGHTS

1998
60
visited by more than 10% of the total Indian tourists?
1999
(a) 5 (b) 3
50
(c) 4 (d) 2
40
570. By what percentage the Indian tourists visiting
30 Chamba were less than those visiting Shimia?
20
(a) 50 (b) 55
(c) 60 (d) 75
10
571. Approximately what percentage was shared by
A B C D E F total foreign tourists among all the tourists visiting
States Himachal Pradesh?
565. The total production in three years in state A is (a) 7 (b) 2
what per cent of the total production in state F in (c) 4 (d) 5
three years? 572. What was the ratio between the Indian tourists to
(a) 120% (b) 125% foreign tourists visiting Kullu?
(c) 90% (d) 80% (a) 105 : 3 (b) 170 : 3
566. What is the average difference between the average (c) 107 : 3 (d) 35 : 13
production for all states in 1998 and that in 1999? 573. Which of the following districts in Himachal
(a) 4.17 (b) 2.17 Pradesh were visited by less than 10% of the total
(c) 6.14 (d) 4.14 foreign tourists who visited Himachal Pradesh?
567. From 1998 to 1999 the per cent increase/decrease (a) Chamba, Kullu
is minimum for which state? (b) Solan, Kangra
(a) B (b) D (c) Solan, Chamba, Shimla
(c) A (d) C (d) Solan, Chamba

6 . 177
Basic Numeracy and Data Interpretation

Directions (Qs. 574-578): Study the following pie Total Girls: 800
chart and answer the questions given below. Percentage of Girls in courses
Sales of car in Country X according
to their colour F
14% A
Blue Yellow E 30%
10%
Gre 12% 14%
e
8% n Red
20%
B
Golden D 10%
10% 30%

n 5%
ow a ck White
2 5 l v er

B r 3% Bl

2 %C
25%
%
Si

579. For course D what is the respective ratio of boys


and girls ?
574. 3% increase in the production of cars of which
(a) 3 : 4 (b) 4 : 5
colour along with red cars will make them 35% of
(c) 3 : 5 (d) 5 : 6
the total cars?
(a) Silver (b) Blue 580. For which pair of courses is the number of boys the
(c) Black (d) Yellow same?
(a) E and F (b) A and D
575. The cars of which colours together make them 50%?
(a) Blue, Black, Red (c) C and F (d) B and D
(b) White, Black, Red 581. For course E, the number of girls is how much per
@UPSC_THOUGHTS

(c) White, Silver, Blue cent more than the boys for course E?
(d) White, Blue, Green (a) 150 (b) 350
576. White car is 18% more popular than which cars? (c) 250 (d) 180
(a) Black (b) Brown 582. For which course is the number of boys the
(c) Silver (d) Blue minimum?
577. If the total production of cars during a certain period (a) E (b) F
was 4,20,000, how many more blue cars were sold (c) C (d) A
is comparison to golden cars? 583. How many girls are in course C ?
(a) 12580 (b) 8400 (a) 18 (b) 16
(c) 12850 (d) 840
(c) 80 (d) 160
578. Which colour is 17% less popular than white
colour? Directions (Qs. 584-587): Study the following graph
(a) Blue (b) Golden and table carefully and answer the questions given below
(c) Green (d) Yellow it:
Directions (Qs. 579-583): Study the following Distribution of candidates appeared
information to answer the given questions. in a competitive examination
Percentage of students in various courses from seven states.
(A, B, C, D, E, F) and Percentage of girls
G
out of these. 7% A
15%
F F
13% A 19%
Total students: 1200 20% B
E 18%
(800 girls + 400 boys) 12%
E
Percentage in various B 12%
15% C
courses D D 6%
35% 23%
5%C

Total candidates appeared = 3 lakh

6 . 178
Basic Numeracy and Data Interpretation

Cost of the total production (both items together) by


State-wise percentage and ratio of male
seven companies = 25 crores
and female qualified candidates
Ratio of production between items I and II and the per
State % Qualified Ratio of
over Qualified
cent profit earned for the two items
appeared Candidates Ratio Per cent
from a state Male : Female of profit
A 49 4 : 5 Company Production earned
B 61 6 : 4 Item Item Item Item
C 54 7 : 8 I II I II
D 45 3 : 2
A 2 3 25 20
E 65 7 : 6
B 3 2 32 35
F 57 11 : 8
C 4 1 20 22
G 48 9 : 11
D 3 5 15 25
584. What is the number of male candidates qualified E 5 3 28 30
from state G? F 1 4 35 25
G 1 2 30 24
(a) 4568 (b) 5454
(c) 5544 (d) 4536
588. What is the amount of profit earned by company D
585. What is the total number of female candidates on item II?
qualified from states A and B together?
(a) Rs 3.125 crore (b) Rs 31.25 crore
(a) 26526 (b) 26426
(c) Rs 3.125 lakh (d) Rs 31.25 lakh
(c) 24526 (d) 25426
589. Cost of production of item I by company F is what
@UPSC_THOUGHTS

586. What is the percentage of candidates qualified from


state A and C together, rounded-off to two decimal per cent of the cost of production of item II by
points? company D?
(a) 49.93 (b) 50.93 (a) 20% (b) 33.33%
(c) 50.43 (d) 51.26
(c) 66.67% (d) 12.5%
587. What is the total number of candidates qualified
590. What is total profit earned by company G for items
from states E and D together?
(a) 54460 (b) 45540 I and II together?
(c) 54540 (d) 54450 (a) Rs 1.62 crore (b) Rs 78 lakh
(c) Rs 7.8 crore (d) Rs 16.2 lakh
Directions (Qs. 588-590): Seven companies A, B, C,
D, E, F and G are engaged in production of two items Directions (Qs. 591-595): Study the following pie-
I and II.The comparative data about production of these
charts and table to answer the questions.
items by the seven companies is given in the following
graph and the table. Study them carefully and answer Statewise details of the adult population of a country
the questions given below.
Graduate and above Upto XII STD Pass
Percentage of the total production Total No. = 24 Lakh Total No. = 32 Lakh
produced by the seven companies

G 12% A 15% F A F A
F5 14% 16% 15%
% 20%
B 11% E B
B
20% 18% E 16%
E 27%
19%
C 22% D C C
D
17% 15% 12% 18%
D 8%

6 . 179
Basic Numeracy and Data Interpretation

Male-Female (M : F) Radio participated in baseball. One-fourth of the hockey players


are females. 20 per cent of badminton players are males.
Graduate and Avobe Upto XII Std Pass
Half the players who have participated in lawn tennis
State M : F M : F are males. There are 45 female cricket players. No
A 7 5 7 9 female player has participated in baseball.
B 5 3 3 5 596. The number of female players participating in
C 5 4 4 5 badminton is approximately what percentage of
D 9 8 5 7 the number of players participating in baseball?
E 9 7 9 10 (a) 75 (b) 80
(c) 95 (d) 86
F 4 3 3 2
597. What is the difference between the number of male
players participating in hockey and the number of
591. What is the difference between the graduate male
female players participating in lawn tennis?
population and XII Std male population from
(a) 92 (b) 98
state A?
(c) 102 (d) 108
(a) 24,000 (b) 14,000
(c) 28,000 (d) 36,000 598. If due to certain reason cricket game was dropped
and all the cricket players left the tournament, then
592. What is the ratio of the graduate female population what would be the total number of male players in
of state E to Std XII female population of state D? the tournament?
(a) 7 : 5 (b) 5 : 7 (a) 200 (b) 210
(c) 16 : 15 (d) 15 : 16
@UPSC_THOUGHTS

(c) 190 (d) 220


593. The Graduate female population of state C is what 599. What is the ratio of the number of male players
per cent of the Std XII female population of that participating in badminton to the number of female
state? players participating in hockey?
(a) 40 (b) 62.5 (a) 3 : 11 (b) 3 : 10
(c) 50 (d) 52.5 (c) 6 : 11 (d) 11 : 6
594. The total graduate population of state F is what per 600. What is the total number of female players
cent of the total Std XII population of state A? participating in the tournament?
(a) 56 (b) 72 (a) 130 (b) 120
(c) 68 (d) 70 (c) 145 (d) 155
595. What is the ratio of the total graduate population
Directions (Qs. 601-605): Study the following
of state D to the total Std XII population of that
information carefully and answer the questions that
state? follow—
(a) 17 : 16 (b) 16 : 17
An office consists of 520 employees working in
(c) 64 : 51 (d) 51 : 64
different departments, viz., HR, IT, Production and
Directions (Qs. 596-600): Study the following Marketing. The ratio of men to women in the
information carefully to answer the questions that follow: organisation is 5 : 3 respectively. 20 per cent of the men
In a tournament, a total number of 400 players have work in the IT department. 40 per cent of the women
participated in five different sports, viz., badminton, work in the HR department. The total number of
hockey, lawn tennis, cricket and baseball. 15 per cent employees in the Production department is 135. Two-
of the total players have participated in badminton. fifth of the women work in the IT department and the
Two-fifths of the total players have participated in remaining work in the marketing department. 40 per
hockey. 6 per cent of the total players have participated cent of the men work in the Production department.
in lawn tennis. 25 per cent of the total players have Four per cent of the men work in the HR department
participated in cricket. Remaining players have and the remaining work in the Marketing department.

6 . 180
Basic Numeracy and Data Interpretation

601. Number of men working in the Marketing is 600. 20 per cent of the total number of products is
department forms what per cent of the total number televisions. 40 per cent of the remaining products are
of employees in the organisation? refrigerators. 50 per cent of the number of computers
(a) 22.5 (b) 34.5 as well as the number: Mobile phones is of category-
(c) 19.5 (d) 38.5 A. 25 per cent of the number of televisions is of category-
602. What is the ratio of the number of men working in B. One-fourth of the number of refrigerators is of
the HR department to the women working in the category-A.
same? 606. What is the ratio of mobile phones of category-A to
(a) 1 : 5 (b) 2 : 3 the number of computers of category-A?
(c) 4 : 7 (d) 1 : 6 (a) 7 : 6 (b) 5 : 9
603. What is the number of women working in the (c) 7 : 3 (d) 7 : 5
Marketing-department? 607. What is the approximate average number of
(a) 41 (b) 34 products of category-A together?
(c) 46 (d) 39 (a) 81 (b) 86
604. Total number of employees working in the (c) 90 (d) 95
Production department forms approximately what 608. What is the total number of televisions of category-
per cent of the total number of employees working A, refrigerators of category-B and mobile phones of
in the organisation? category-A together?
(a) 12 (b) 17 (a) 318 (b) 328
(c) 21 (d) 26 (c) 296 (d) 266
@UPSC_THOUGHTS

605. What is the total number of employees working in 609. The combination of which of the two products of
the IT department? category-B is exactly equal in number?
(a) 130 (b) 124 (a) Mobile Phone and Computer
(c) 143 (d) 101 (b) Mobile Phone and Refrigerator
Directions (Qs. 606-610): Study the following (c) Television and Refrigerator
information carefully and answer the questions that (d) Television and Computer
follows: 610. What is the difference between the number of
A company produces four different products, viz. computers of category-B and the number of
mobile phone, televisions, refrigerators and computers. refrigerators of category-A?
Each of the products has two categories, viz. category- (a) 42 (b) 18
A and category-B. The total number of all the products (c) 32 (d) 36

6 . 181
Basic Numeracy and Data Interpretation

ANSWERS WITH EXPLANATORY NOTES

1. (d) Prime numbers lying between 60 and 75 are 6. (d) Prime numbers between 61 and 89 are 67,
61, 67, 71 and 73. 71, 73, 79 and 83.
Their sum is 61+67+71+73 = 272 ∴ The sum of prime numbers between 61
2. (b) Let the four prime numbers be p, q, r and and 89 is 67 + 71 + 73 + 79 + 83 = 373
s. Given pqr = 385 and qrs = 1001 7. (d) Any number divisible by 24 should also be
qrs 1001 divisible by 3 and 8.
∴ =
pqr 385
s 35718 is not divisible by 8.
13
⇒ = 63810 is not divisible by 8.
p 5
Thus, the last number is 13. 537804 is not divisible by 8.
[Alternative method : You are given that p, 3125736 is divisible by both 3 and 8.
q, r, s are in ascending order. Now factorise
8. (d) Divisor = 12 × Quotient
385; 385 = 5×7×11. Clearly the last number
Divisor = 5 × Remainder
must be greater than 11. Factorise 1001. You
may safely assume 7 and 11 to be two Remainder = 48
factors in 1001. The factors in 1001 are ∴ Divisor = 5 × 48 = 240
7×11×13. So the last number is 13.] Divisor 240
∴ Quotient = = = 20
3. (d) The whole numbers between 200 and 300, 12 12
Dividend = Divisor × Quotient + Remainder
@UPSC_THOUGHTS

which contain the digit 7 once and only once


= 240 × 20 + 48
are 207, 217, 227, 237, 247, 257, 267, 270,
= 4848
271, 272, 273, 274, 275, 276, 278, 279, 287,
and 297. 9. (c) Let the number be x
∴ (x + 12) ÷ 6 = 112
Similarly, the total number of whole numbers
x  12
between 300 and 400 which contain the digit = 112
6
7 once and only once are also 18.
x = 660
∴ Total of such numbers from 200 to 400 is
x 660
18 + 18 = 36. ∴ Correct answer is  12 =  12
6 6
[Take the first 100 numbers. Whole numbers = 110+12
with 7 as last digit occur once in every 10 = 122
numbers. However, once it occurs double (in
10. (d) Dividend = Divisor × Quotient + Remainder
77) which goes against the given condition.
∴ Number = (555+445)×2×110+30
So we have only 7 occuring as last digit for
9 times. Now, as first digit it also occurs 10 = 220000+30
times, (70 to 79) but rejecting 77 for the same = 220030
reason as given above, we have it 9 times.
11. (b) 998 × 1002
So, a total of 18 times. This would be true
= (1000 – 2) (1000 + 2)
for every hundred. From 200 to 400, we have
two sets of 100s so the whole numbers = 10002 – 22 [ (a–b) (a+b) = a2–b2]
containing 7 once and only are 18 + 18 = 36.] = 10,00,000 – 4
4. (d) Take n = 2; n = 2 which is an irrational = 999996
number, so n is not always rational.
12. (c)  212×188 = (200 + 12)(200 – 12)
If n = 4, 4 = 2 which is a natural number.
Thus, n may be a natural number or an = (200)2 – (12)2
irrational number. = 40000–144
5. (d) = 39856

6 . 182
Basic Numeracy and Data Interpretation

13. (c) Let the consecutive numbers be x and x + 1 n(n  1)


20. (a) We know that: 1 + 2 + 3 + ... n =
(x + 1)2 – x2 = 35 2
x + 2x + 1 – x2
2
= 35 45(45  1) 45  46
∴ =
2x + 1 = 35 2 2
2x = 35 – 1 = 34 = 1035

x =
34
= 17 21. (b) 114 x ∴ x = 114×1+21 = 135
2 1 → 21 (Taking quotient = 1)
x + 1 = 17 + 1 When 135 is divided by 19, then the remainder
= 18 will be 2.
∴ The next number is 18. 19 135
14. (b) Let the required number be x. 7 → 2
Then, x + 10 = 2(x – 10) 22. (b) Let the smaller number be x. Then larger
= 2x – 20 number is 1365 + x
10 + 20 = 2x – x = x
∴ 1365+x = 6x + 15
∴ x = 30
5x = 1350
15. (a) Largest number of 5 digits = 99999 x = 270
∴ 77)99999(1298 ∴ 99999
77 23. (d) Dividend = Divisor × Quotient + Remainder
 53
229 = 12×35+0
154 99946
= 420
759
693 Now, dividend = 420 and divisor = 21
@UPSC_THOUGHTS

669 420
616 ∴ Correct quotient = = 20
21
53
24. (b) Let the three consecutive odd numbers be
16. (c) Divisiblity by 3 : If the sum of the digits of (2x + 1), (2x + 3) and (2x + 5)
a number is divisible by 3, then the number
Their sum = (6x + 9) = 3(2x + 3), which is
is divisible by 3.
always divisible by 3.
23 * 7 = 2+3+*+7
25. (b) Given that 357*25* is divisible by both 3 and
= 12 + *
5. For divisibility by 5, we must have 0 or
Substitute the options in place marked *and 5 at the unit place of the given number.
add digits. When we put * = 3. We get 15 For divisibility by 3, sum of the digits of the
which is divisible by 3. given number should be divisible by 3.
17. (a) Largest four digit number = 9999 Let the unit place of the given number have
5, then 3 + 5 + 7 +* + 2 + 5 + 5 = 27 + *. When
Smallest three digit number = 100
we put * = 6, the number is divisible by 3.
∴ 9999 – 100 = 9899
∴ The missing digits in the unit’s place and the
Dividend Remainder thousandth place respectively are 5 and 6.
18. (b) Divisor =
Quotient 26. (c) On dividing 803642 by 11, we get remainder 4
59761  37 ∴ required number to be added is 11 – 4 = 7
= = 316
189
27. (d) On dividing 457 by 11, remainder is 6.
19. (b) Three consecutive numbers are
∴ required number is either 451 or 462.
(2x + 1), (2x + 3) and (2x + 5)
So, 462 is the number which is nearest to
Their sum = 2x + 1 + 2x + 3 + 2x + 5 457 and exactly divisible by 11.
= 6x + 9
28. (c) Thief escaped with one diamond.
= 3(2x+3)
Before he met the 3rd watchman, the thief
which is always divisible by 3. had (1 + 2) × 2 = 6 diamonds.

6 . 183
Basic Numeracy and Data Interpretation

Before he met the 2nd watchman, the thief Total number of matches played
had (6 + 2) × 2 = 16 diamonds. = 64 + 32 + 16 + 8 + 4 + 2 + 1
Before he met the 1st watchman, the thief = 127
had (16 + 2) × 2 = 36 diamonds.
36. (b) By the given condition 53x – 35x = 1206
29. (b) Smallest number of 5 digits is 10000. ⇒ 18x = 1206
On dividing 10000 by 476, we get remainder 4. 1206
∴ required number is 10000+(476 – 4) ⇒ x = = 67
18
= 10472
37. (a) 4 x ∴ z = 6×1+4 = 10
30. (c)  997×997 = 9972 5 y → 2 y = 5z+3 = 5×10+3 = 53
= (1000–3)2 6 z → 3 x = 4y+2 = 4×53+2 = 214
= 1000000 – 6000+9 1 4
[ (a – b)2 = a2 – 2ab + b2]
= 994009 38. (b) A number when divided successively by 4
and 5 leaves remainders 1 and 4 respectively.
31. (a) The least number of five digits is 10000. 4 x
∴ 567 )10000(17 5 y → 1
567
1→ 4
4330
3969 ∴ y = 5×1+4 = 9
361 ∴ x = 4y + 1 = 4 × 9 + 1 = 37
Now, 37 when divided by 5 and 4, leaves
∴ The least 5 digit number divisible by 567
remainders 2 and 3 respectively.
= 10000 + (567–361)
= 10000 + 206 39. (b) Product of two numbers = their HCF × their
@UPSC_THOUGHTS

= 10206 LCM
32. (c) On dividing 1000 by 17, the remainder 540 × second number = 18×3780
obtained is 14. 18  3780
58 second number = = 126
17 1000 540
40. (c) Find the LCM of the given lengths
986
LCM of 64, 80 and 96 is 960 cm = 9.60m
14
∴ The required number is 14. 41. (a) LCM of 32, 36, 48, 54 is 864.
∴ The required number = 10000 – 864
33. (d) On dividing 3006 by 29, the remainder is 19 = 9136
103
42. (b) Number of classes = HCF of 391 and 323
29 3006
= 17
2987
19 LCM × HCF
43. (b) Second number =
So, we have to add 29 – 19 = 10 to the First number
number. 2079  27
= = 297
∴ Required number = 3006 + 10 = 3016 189
34. (d) 88)9999(113 44. (c) Product of 2 numbers
88 = their HCF × their LCM
119 2400  16
∴ Second number =
88 80
319 = 480
264
55 HCF of Numerators
45. (a) HCF of fractions =
∴ Required number =9999–55 LCM of Denominators
= 9944
HCF of 4, 10 and 20
35. (c) At first matches played = 64 =
(i.e., 2 players at a time) LCM of 9, 21 and 63

6 . 184
Basic Numeracy and Data Interpretation

HCF of 4, 10, 20 = HCF of [22; 2 × 5; 22 × 5] 53. (d) The LCM of 3, 4 and 5 = 60


= 2 The greatest 4-digit number
3 9, 21, 63
LCM of 9, 21 and 63 = 63 3 3, 7, 21 = 9999
2 7 1, 7, 7 ∴ The required number
∴ HCF of fraction = 1, 1, 1 = 9999 – 39 = 9960
63

Product of the Numbers


46. (c) LCM =
HCF The smallest 4-digit number
= 1000
456  874
=
38
= 10488
47. (c) LCM of 2.5, 20, 1.2, 7.5
= (LCM of 25, 200, 12 and 75) × 0.1 ⇒ The required number
= 600 × 0.1 = 1000 – 40 + 60 = 1020
54. (a) LCM of 4, 5, 7, 8
= Rs 60
and 10
48. (c) Time taken by them to complete one = 2×2×5×7×2
revolutoion = 280 seconds
11 11 11 = 4 minutes 40 seconds.
= , and hours
4 5.5 8 55. (a) Number of tiles will be the least when the
11 2 11 size of one tile is maximum. Hence HCF of
@UPSC_THOUGHTS

= , and 2380 and 816 (the length has to be converted


4 1 8
into cms) will be the maximum size of the
11 2 11 LCM of 11, 2, 11
LCM of , and = square tile.
4 1 8 HCF of 4, 1, 8
22 ∴ HCF of 2380 and 816 = 68cm
= = 22 hours
1 816×2380
∴ They will meet after 22 hours. ∴ The number of tiles = = 420
68×68
49. (a) LCM of 48, 72, 108 is 432. 56. (d) The required answer
∴ Light will change after every 432 seconds, = HCF of 280, 315
i.e., after every 7 minutes and 12 seconds. and 385
= 35
50. (c) LCM of 252, 308 and 198 is 2772.
So, A, B and C will again meet at the starting
point in 2772 seconds, i.e., 46 minutes 12 (Note: First find the HCF of any two numbers, say
seconds. 280 and 315. That is 35. Then find the HCF of 35
and 385. Thus, we get the HCF of 3 numbers.)
51. (d) Largest size of the tile = HCF of 378 and 525
57. (b)
= 21cm
10
52. (a) The given items have to be divided among 58. (d) = 0.9090
11
the students equally. 0.91 =0.91
∴ HCF of 456 and 874 is 38. 7.3
= 0.9125
8
8.3
= 0.922
9
59. (c) For every pair of decimal places, there is one
decimal place in the square root; for example,
0.04 = .2
0.0001 = .01

6 . 185
Basic Numeracy and Data Interpretation

1.21 = 1.1 2.5


65. (d) 2.5 ÷ 0.0005 =
So for 16 decimal places, there will be 8 0.0005
decimal places in the square root. 25000
= = 5000
5
1 1 9 500
60. (c) , , , 66. (d) 2 + 0.2 + 0.02 + 0.002 = 2.222
10 100 10000 10000
Make the denominators the same:  0.1  0.1 0.1  0.02  0.02  0.02 
67. (b)  
 0.2 0.2  0.2  0.04  0.04  0.04 
1000 100 90 500
, , ,
10000 10000 10000 10000  0.001  0.000008 
=  
 0.008  0.000064 
90 9
Clearly, or is the smallest fraction.  0.001008  1008
10000 1000 =   =
 0.008064  8064
2 3
61. (d)  0.666... 0.75 1
3 4 = = 0.125
4 5 8
 0.8  0.8333...
5 6 68. (a) 168×32 = 5376
∴ 0.833 > 0.8 > 0.75 > 0.66 ⇒ 5376 ÷ 168 = 32
5 4 3 2 5.376 5.376
so Now, 5.376 ÷ 16.8 = =
6 5 4 3 16.8 16.800
5376 32
5 2 1 = =
∴ required difference is   168  100 100
6 3 6 = 0.32
@UPSC_THOUGHTS

5 9
62. (d) and = 5 × 13 > 9 × 7 69. (c) 2.75  3.78 = (2 0.75) (3 0.78)
7 13
= 5 + 1.53
11 14 = 5 + 1 + 0.53
and = 11 × 17 > 14 × 13
13 17 = 4 + 0.53
2 5 = 4.53
and = 2×6<5×9
9 6 7
70. (c) = –0.7
5 3 2 8 10
63. (c) The given fractions are , , and 5
7 9 3 11 = –0.625
LCM of 7, 9, 3 and 11 is 693. 8
2
5 5  99 495 and = –0.666
∴ = = 3
7 7  99 693
∴ –0.7 < –0.666 < –0.625
3 3  77 231 7 2 5
= =  
9 9  77 693 ⇒
10 3 8
2 2 231 462 53
= = 71. (c) 0.5353 .... = 0.53 =
3 3 231 693 99

8 8  63 504 29 15 29  4  15  3 116  45
= = 72. (a)  = =
11 11  63 693 12 16 48 48
161 17
8 5 2 3 = = 3
∴    48 48
11 7 3 9 17
So will have to subtracted
 When denominators of fractions are same, 48
the fraction with the largest numerator is
considered the largest. 10 12 x
73. (c)    16
3 5 4
3 7 8
64. (b)   = 3 + 0.7 + 0.08 = 3.78 16
1 10 100  x  8
2

6 . 186
Basic Numeracy and Data Interpretation

1 7  3 2  2   1  7  5  2  x2 54
     
7  9 9 9  
74. (d) 83. (b)
7 9  9 9  9 169 39
1 1 x 54
 0 
7 7 13 39
54
1 1 1 x 13 18
  20  20 39
75. (c) Given expression = 2 4  8 x2 = (18)2
1 22
 4  20 = 324
2
1  160 161 1.21  0.9 121 9
  84. (b) =
8 22 176 1.1  0.11 11  11
= 3 [ (11)2  121]
76. (a) 0.07 × 0.008 × 0.2 = 0.000112
1 1
6 7 8 3 85. (c) ( 8 ) 3 = ( 23 ) 3
77. (c) Given expression =   
9 9 9 9 = 22  3
3 1

24 2
= 2 = 2
9 3
80.40 x 21
78. (d) Given expression =  4.2 86. (d) =
20 144 36
x 21
= 8.22 = [ (12)2  144]
12 36
@UPSC_THOUGHTS

5 5 3 15 3.88 21 12
79. (c)    x = = 7
3 3 3 9 3 36
x = (7)2  x = 49
= 1.29
x
1 1 1 87. (c) = 0.02
80. (d)   x = 3 200
4 6 12
x = 200 × 0.02
1 1 1 =4
∴ x =  
4 6 4 x = 16
1 0.0009 9  100
= 88. (b) =
6 0.01 10000 1
1 1 1 329 9
81. (b)   = =
6 9 2 18 100
3
14 7
 =  0.3
= 10
18 9
122.76 12276 10
7
of x = 7000 89. (b) 
9 15.5 155  100
9 79.2
x = 7000 = 9000   7.92
7 10

25 7  19  8 175  152 90. (d) Given expression


82. (a) =
243  (10  9  9) 243  (90  9)
a3  b3
= where a = 5.7 and b = 2.3
175  152 a  b2  ab
2
a + b = 8.0
=
243  81
=a + b
327
=  109 = 8.0
3

6 . 187
Basic Numeracy and Data Interpretation

a3  b3 13
91. (d) Given expression = = 9
a2  b2  ab 12
1
= 91
( a b)( a 2 ab b 2 ) 12
=
a2 b 2 ab 1
= 10
=a + b 12
= 0.87 + 0.13 97. (c) Given expression = 125 + 12.5 + 1.25
=1 = 138.75

98. (b) Let there be x number of men and each spent


a2  ab  b2
92. (c) Given expression = Rs x, then
a3  b3
x × x = 15625
a2 ab b 2 x 2 = 15625
=
( a b)( a2 ab b 2 ) ∴ x = 15625
1
= = 125
ab
where a = 1.04 and b = 0.04
99. (b) 3
0.000064 = 3
0.00064
1
= 1
1.04 0.04 = 3
(0.2)3 = 0.2
a2  b2
93. (b) Given expression = ab 3 3 3
ab  38  16  4  8 4  4  8 4  4
@UPSC_THOUGHTS

100. (c)   =  3 2    3 2 
4   54  (32 )4   54  38 
where a = 0.778 and b = 0.222  625 9     
∴ a + b = 1.000
= 1 4 3 3
2 4 23 2 8
=    
94. (b) Given expression 4 3 53
 5 125
5 4
a2  b2  c2
= 101. (d) The largest three-digit number is 999
(10a)2  (10b)2  (10c)2
3 999 31
where a = 0.82, b = 0.284, c = 0.795 9
61 x99
a2 b2 c2
= 61
100a2 100 b2 100c2 38
(a2  b2  c2) 1 ∴ 999 – 38 = 961 is the largest three-digit
= 
2 2
100 (a  b  c ) 2 100 perfect square, the root of which is 31.

= 0.01 102. (a) 3 1298 36


3 3 9
a b
95. (b) Given expression = 66 x398
a2  ab  b2 296
( a b)( a 2 ab b 2 ) 2
=
a 2 b 2 bb ∴ 1298 – 2 = 1296 is a perfect square of 36.
=a + b
= 0.05 + 0.04 1
103. (c) 112 1, (22 )12 26 64
= 0.09
1 1
1 1 1 1 1 1 (33 )12  34  81, (44 )12  43  64
96. (d) 2  3  4 = 2  3  4    1
2 3 4 2 3 4 ∴ 33 is the largest number in the given
643 options.
= 9
12

6 . 188
Basic Numeracy and Data Interpretation

109. (c) Let the two consecutive odd numbers be


2 1 ( 2  1) ( 2  1)
104. (c) = 2x + 3 and 2x + 1
2 1 ( 2  1) ( 2  1)
According to the given condition
[Multiplying numerator and denominator by
( 2  1) ] (2x  3)2  (2x  1)2
= 4x2  12x  9 (4x2  4x  1)
2
( 2  1) = 8x + 8
=
1
= 8(x + 1)
= 2 1 110. (c) 14 women and 11 men. The number of
= 1.4142 + 1 women should be a multiple of 7 and the
number of men should be a multiple of 11.
= 2.4142
Also, more than half the member are women,
therefore the smallest number of members is
105. (a) (6 3 5)3 = 216 × 5 = 1080 14 + 11 = 25

(83 2)3 = 512 × 2 = 1024 111. (b) For first 100 calls, 0.30 × 100 = 30
Remaining money 20
(23 130)3 = 8 × 130 = 1040
Money per call
=
0 .25
20  100
( 3 900)3 = 900 =  80
25
∴Total calls = 100 + 80 = 180
x y
106. (c) a  and b 
@UPSC_THOUGHTS

xy x y
112. (a) 1
25
= 1
x
144 12
xy x2  y 2 144  25 169 13 x
∴ ab  , ab  = 1
x y
2 2
x y
2 2 144 144 12 12
1 x
ab xy = 1  1
∴ ab  2
12 12
x  y2 ∴x = 1
113. (d) 2n = 64 = 26 (∴ 64 = 26)
1 1 1 1 2
107. (a)    1
2 2 1 2 3 n = 26
2n = 2 2
2 1 1
2
 
2

3 ∴ n =6
3 31 3 4 2
n = 12
2 2 1 2 1
   5 1
5 51 5 2 114. (d) Given that a 
5 1
You may try it out with any other fraction.
5 1 5 1
a =
 1  1  1  1  5 1 5 1
108. (d)  1   1   1   1  
 x  1  x 2  x  3  x 4 5 1 2 5 6 2 5
=
4 4
 x 11  x  2 1  x  31  x  4 1
=      5 1
 x 1   x 2   x 3   x  4  and b =
5 1
x 2 x 3 x  4 x 5
=    5 1 5 1
x 1 x 2 x 3 x  4  b =
5 1 5 1
x 5 5 1 2 5 6 2 5
= =
x 1 4 4

6 . 189
Basic Numeracy and Data Interpretation

63x
a2  ab  b2 (a  b)2  ab and second number = 63% of x =
 = 100
a2  ab  b2 (a  b)2  3ab
7x
70x 63x
91 8 4 Difference =  =
=   100 100 100
93 6 3  7x 100 
∴ Required percentage =    100 %
 1
2
 100 70x 
115. (a)  a    3 = 10%
 a
1 Direct approach
 a  3
a 100 y
Second number is 100 % of First
3
 1 3  1 1 100 x
 a   = a  3  3 a   100 37
 a  a  a  100 %
100 30
3 1
 3 3 = a  3 3 3
a 63
= 100 % 90%
3 1 70
 a  0
a3 Second number is less than the first by
1
(100 – 90)% = 10%
116. (c) 2 2 2 2 = 2 2 2 22 119. (a) Let the number be x. Then,
3 x – 20% of x = 38
= 2 2 22 [ am  an  am n ] 20
 x x = 38
100
3 1 1
2 2  22 [ a  a 2 ] 1
x  x = 38
@UPSC_THOUGHTS

= 2

5
3 4
= 2 2  24  x = 38
5
5
1 3
 x = 38 = 47.5
4
= 2 2 4 [ am  an  am n ]
120. (a) Ice-cream seller has (100 – 60) = 40% of
7
ice-cream cups.
= 2 24
7 1 1
40% of x = 300
= 2(2 4 ) 2 [ a  a 2 ] 40
x = 300
7 100
= 2 28 300  100
x =  750
40
1 7
= 2 8
121. (d) 150% of x = 80
15
= 150 160
28 x  80  x 
15  1 100 3
= 28 2
60 160
15 ∴ 60% of x =   32
= 100 3
2 16
117. (a) 42% of a number = 12.6 122. (c) Increase in salary = Rs (6300 – 5000)

Let the number be x, then = Rs 1300


42 1300
× x = 12.6 Percentage increase =  100
5000
100
100 = 26%
 x = 12.6 ×
42
123. (a) Suppose x is the maximum mark
 x = 30
Then, the pass marks for the first candidate
118. (d) Let the third number be = x = 30% of x + 50
70x and the pass marks for the second candidate
Then, first number = 70% of x =
100 = 320 + 30

6 . 190
Basic Numeracy and Data Interpretation

Therefore, 30% of x + 50 = 320 + 30 130. (c) Total marks needed to pass the exam
3x = 72 + 18 = 90
or  300
10 Passing marks in exam = 36%
100
300  10 Maximum marks in the examination = 90 
∴ x =  1000 36
3 = 250
124. (b) Let Anand’s income be Rs x
P
Then, x – 5% of x – 20% of 95% of x = 1919 131. (c) Population 3 years ago = 3
x 20  95  x  R 
 x  = 1919 1  100
20 100  100  
x 19x 7986
 x  = 1919 =
20 100 3
 10 
 100x – 5x – 19x = 191900 1  100
 
191900
∴ x = 7986
76 =
11 11 11
= 2525  
10 10 10
Thus, Anand’s total income is Rs 2525.
125. (b) Let x be the total number of students 7986000
=
15 11  11  11
 x = 30
100
30 100 = 6000
 x = = 200
15
132. (a) (62% of x – 38% of x) = 144
110
126. (b) Cost paid by B = 10,000 × = 11,000 24% of x = 144
@UPSC_THOUGHTS

100
90 144 100
Cost paid by A = 11,000 × = 9,900 ∴ x =
100 24
∴ A’s profit in both the transaction = 600
133. (c) Let the total population be x, then
= 1000 + 100 = 1,100
60 3
127. (c) Let the original price be Rs 100 Poor population = x  x
100 5
Decreased price = Rs 90 40 2
Illiterate population = x  x
 10  1 100 5
Difference % =   100 %  11 % Rich illiterates = 10% of (100 – 60)% of x
 90  9
128. (a) Let the salaries be 2x and 3x  10 40  x
=   x 
% excess of second month’s salary  100 100  25
 (3x  2x )  2 x  9x
=   100 % Poor illiterates =  x   
 2x  5 25  25
x  9x 5 
=  100  50% ∴ Required percentage =    100 %
2x  25 3x 
% shortness of first month’s salary = 60%
(3x  2x )
=  100 134. (d) Let original population = 100
3x
x 1 3
 100  33 %  1 
=
3x 3  3 
Population after 3 years = 100   1  2 
 100 
129. (c) According to the question  
 
ax by cx dy
   207 207 207
100 100 100 100 = 100   
200 200 200
∴ percentage of y to that of x = 100 (a  c ) = 110.87
db

6 . 191
Basic Numeracy and Data Interpretation

∴ Increase = [110.87 – 100]% ∴ % of candidates who failed


= 10.87% (1500  663)
= 10.8% =  100
1500
135. (d) Let, price = Rs 100, and sale = 100 837
=  100
Then, sale value = Rs (100 × 100) 1500
= 55.8%
= Rs 10000
New sale value = Rs (70 × 120) 139. (a) A = a2
2
=  a 
= Rs 8400 25a 
New Area 
 1600   100 
Decrease % =   100 % 2 2
 10000   a  5a 
= a     
= 16%  4  4
25 2
Direct approach = a
16
Inc.%  Dec.% 25a2 2
Inc. % – Dec.% – Difference in area = a
100 16
20  30 2
 20 – 30 – 9a
100 =
= –10 – 6 16
= –16%  9a2 
% difference =   100 %
 16a2 
Thus, there is a total loss of 16%  
Inc. %  Dec. % 225
= %  56.25%
@UPSC_THOUGHTS

136. (d) Inc. % – Dec.% – 4


100
20  10 Short Method
= 20 – 10 –
100 x2
= 10 – 2 Increase in area = 2x
100
= 8%
Thus, 8% increase  (25)2 
= 2  25  
137. (a) % of students who failed in both subjects  100 

= 50  625 
= 17%
% of students who failed only in English  100 
= 52–17=35%  5625
% of students who failed only in Maths =    56.25
 100 
= 42–17=25%
140. (c) Let diameter be 2r  radius = r
Total % of students who failed = 17 + 35 + 25
Surface area of a sphere = 4πr2
= 77%
If the diameter doubles,
∴% of students who passed = 100 – 77
then d = 4r  new radius = 2r
= 23%
Direct approach New surface area = 4π × (2r)2
100 – (52 + 42 – 17) = 4π × 4r2
= 23 = 16π r2
138. (d) No. of boys = 1500 – 920 Increase % in surface area
= 580  (16r 2  4r 2) 
=   100 %
Total students who passed 4r 2
 
50 35
=  920   580  12r 2 
100 100 =  2
 100 %
= 460 + 203  4r 
= 663 = 300%

6 . 192
Basic Numeracy and Data Interpretation

141. (d) Let the original price be x 145. (c) Let his earnings be Rs 100
px p px Savings = Rs 25 ⇒ Expenditure = Rs 75
x x =1
100 100 100 After the increase, his income= Rs 120
110
px px p2 x Expenditure after the increase = 75
x =1 100
100 100 10000 = 82.5
 p2  ∴ Present saving = 120 – 82.5 = 37.5
x 1 
 10000  =1  12.5 
 ∴ % hike in savings =   100 %
 25 
1
∴ x = = 50%
p2
1 146. (c) Here, x = –20 and y = 80
10000
xy
10000 Using the shortcut method x + y +
= 100
10000  p2 (20)  (80)
∴  20  80 
142. (a) Let x be the total number of candidates 100
% of candidates who failed in both subject = 60 – 16 = 44% increase
= 27 147. (b) The required percentage
% who failed only in English = 30 – 27 = 3 = 100 – (40 + 30 – 10)%
% who failed only in Maths = 35 – 27 = 8 = 40%
∴ % of those who failed in all = 27 + 3 + 8
148. (a) Let Sohan’s wage be Rs 100
= 38%
62 Reduced wage = Rs 40
 % who passed =  x  248
@UPSC_THOUGHTS

100 Increased wage = 140% of Rs 40


248  100
 x =  400 140
62 = × 40 = Rs 56
Direct approach 100
% of those who failed = 100 – (30 + 35 – 27) ∴ Change in wage = Rs 40 – Rs 56
= 62 = –16%
62 x = Loss of 16%
% who passed  248,
100 Direct approach
24800
x =
62
400 Inc. % – Dec. % – In.%value  Dec.%value
4040 100
143. (d) % of votes for the defeated candidate = 40% = 40 40 = –16%
∴ % of votes for the winning candidate = 60% 100
= Loss of 16%
Difference % = 20%
20 149. (b) % change in consumption
 x = 200
100 25
200 =  100
x =  100 = 1000 100 25
20 25
∴ Successful candidates’s votes = 600 =  100
125
144. (b) Let x be the total sum = 20%
 10   20   20  150. (b) % of votes secured by the winning candidate
Then  1   1   1  
 100   100   100  = 63%
× total amount = 132,480 ∴ % of the votes secured by the defeated
candidate = 100 – 63 = 37%
90 80 80
  × x = 132,480 Difference in percentage = (63–37)% = 26%
100 100 100 Let x be the number of votes polled
9 4 4 26
   x = 132,480 ∴  x = 32,500
10 5 5 100
x =
132480 250 32,500 100
144 ∴ x =
26
Total sum = 230,000
i.e., total votes polled = 1,25,000

6 . 193
Basic Numeracy and Data Interpretation

151. (c) According to the condition (i) n 1


153. (c) Average of first n numbers =
Total of data 2
= 15
30 35 1
Average of first 35 numbers =
∴ Total of data = 15 × 30 = 450 ...(i) 2
According to the condition (ii) 36
18
Total of data of 6 students 2
= 16
6 154. (d) The total of 9 results = 40 × 9 = 360
∴ Total of data of 6 students = 96 ...(ii) The total of first 5 results = 5 × 39 = 195
Thus, using equation (i) and (ii), we get The total of last 5 results = 5 × 42 = 210
Average of remaining 24 students The fifth result is common to both;
∴ Fifth result = 195+210–360
450  96
= = 45
24
354 1015  1222  4250
= = 14.75 155. (b) Average =
24 10  12  4
= 14 years and 9 months 150  264  1000
Shortcut =
26
30 students × average = 6 students × average  414
+ 24 students × average =
26
30 × 15 = 6 × 16 + 24 × x = Rs 54.38
30 15 6 16 354 59 156. (a) Let the average subscription of all 8 men
@UPSC_THOUGHTS

x =
24 24 4 be x .
= 14 years and 9 months 350  x  70
Average subscription is = x
8
420 + x = 8x
152. (d) According to the condition (i)
⇒ x = 60
Total weight
= 60 ∴ 8th man’s subscription = 60 + 70 = 130
150
∴ Total weight = 150 × 60 157. (a) Sum of the temperature for the full week
= 9000 kg ...(i) = 7 × 28.5
According to the condition (ii) = 199.5
∴ The temperature on the last day
Total weight of 50 male students
= 70 = 199.5 – [3 × 27 + 3 × 29]
50
= 199.5 – [81 + 87]
∴ Total weight of 50 boys
= 70 × 50 = 31.5
= 3500 kg ...(ii) 158. (c) Required answer
Thus using equations (i) and (ii), we get 60  12  40  13  50  14  50  15
Average of remaining 100 girls =
60  40  50  50
9000  3500
= 720  520  700  750
100 = = 13.45
5500 200
= = 55 kg
100 159. (a) Let the number of students before the new
Shortcut
admissions be x.
150 students × average = 50 male × average
∴ Total age of the class = 16x
+ 100 females × average
Total age of the four new boys = 4 × 13.5
150 × 60 = 50 × 70 + 100 × x
= 54 years
150 60 50 70
x = 16 x  54 7
100 ⇒ = 15
= 55 kg x4 12

6 . 194
Basic Numeracy and Data Interpretation

16x  54 187 given by x + x + x – 2 = 40 × 163 – 37 × 162


⇒ =
x4 12 3x – 2 = 6520 – 5994
3x – 2 = 526
12(16x + 54)= 187(x + 4)
3 x = 528
⇒ 192x – 187x = 748 – 648
528
5 x = 100 x =
x = 20 3
∴ x + 4 = 24 = 176
160. (c) Let the average runs for 10 innings be x ∴ A’s height = 176 cm
⇒ Total runs for 10 innings = 10x B’s height = 176 cm
Average for the 11th innings is given by
C’s height = 176 – 2 = 174 cm
10x  50
= x+4 2xy
11 166. (a) Average speed =
xy
⇒ 10x + 50 = 11x + 44
50 – 44 = 11x – 10x 2 20  30
=
x =6 20  30
∴ Average after 11 innings 1200
=
= 6 + 4 = 10 50
= 24 kmph
2xy
161. (c) Average Speed = km/hr AB
xy 167. (b) Average weight of A and B = = 42
2  60  40 2
@UPSC_THOUGHTS

= ∴ A + B = 2×42 = 84 kg ...(i)
60  40
2 60  40 Similarly, A + B + C = 3×44 = 132 kg ...(ii)
= = 48 km/hr.
100 B + C = 2×43 = 86 kg ...(iii)
162. (a) Total age of the surviving children Subtracting (ii) from (iii), we get A = 46 kg
= 5 × 10 – 5 = 45 Now putting this value of A in (i) we get
After 4 years, average age of those children B = 84 – 46 = 38 kg
45 168. (a) Total yearly income
= 4
4
= Total expenditure + Saving
1 1
= 11  4 = 15 years = 3 × 2300 + 5 × 2100 + 4 × 2000 + 9400
4 4
= 6900 + 10500 + 8000 + 9400
n  n  1 n 2 n  3 n  4  n  5
163. (d) Average = = 34800
6
34800
6n  15 3(2n  5) ∴ Average monthly income = = Rs 2900
= = 12
6 6
(2n  5) 5 1 169. (b) Let the numbers be 2x, x and 4x
= = n  = n2
2 2 2 2x + x + 4x = 56×3
164. (b) (A – 3) + (B – 3) = 36 (given average is 18) 7 x = 168
168
A + B = 42 x =
7
Also, A + B + C = 66 = 24
42 + C = 66 ∴ The numbers are 48, 24, 96.

⇒ C = 24 years 170. (d) 3x + 5x + 7 x = 15×3 = 45


x =3
165. (b) Let the height of A, B and C be x, x and
∴ The ages are 9, 15, 21
x – 2 respectively.
∴ The total heights of A, B and C are ∴ Required answer = 9 years

6 . 195
Basic Numeracy and Data Interpretation

20  10  240 200  240 1 1   1 1 


171. (d) Average speed = = : : 1   
3 2   2 3  
25 25 A : B : C=
440 88
= = 17.6 miles 1 1 5
25 5 = : : 1
3 2 6
300
172. (c) Average speed = 1 1 1
100 100 100 = : :
  3 2 6
30 40 50
300 1 1 1
= = 6 : 6 : 6
10 5 2 3 2 6
  =2 : 3 : 1
3 2 1
300  6 2
= = 38.3 kmph A’s share of profit =  840
47 6
173. (d) Age of the child = [(20 × 3) – (23 × 2 + 5 × 2)] = 280
= [60 – (46 + 10)] 178. (c) Let Gayatri’s and Savitri’s age be 6x and 5x.
= [60 – 56] = 4 years 6x + 5x = 44
174. (a) Total age increased = 10 × 2 44
x = = 4
11
= 20 years
∴ Gayatri’s age = 24
Sum of ages of two new men Savitri’s age = 20
= (20 + 22 + 20) After 8 years, G : S = 32 : 28 = 8 : 7
= 62 years
179. (b) Ratio of the speeds of A and B = 3 : 2
@UPSC_THOUGHTS

62
∴ Average age of two new men= Ratio of the time taken by A and B
2
= 31 years 1 1
= : = 2 : 3
3 2
175. (d) Temperature on Monday + Tuesday ∴ 2 : 3 = x : 24
+ Wednesday = 3×37° = 111°
2 x
Temperature on Tuesday + Wednesday + Thur =
3 24
day = 3 × 34 = 102° ...(i) 24  2
5 ∴ x = = 16 minutes
Temperature on Monday = Thursday 3
4
5 7 15
∴ Thursday + Tuesday + Wednesday = 111 180. (d)  0.47  0.65
4 15 23
...(ii)
17 21
Subtracting (i) from (ii), we get  0.68  0.72
25 29
5
Thursday  Thursday = 111 – 102 = 9°
4 181. (c) Ratio of investment = 1500 : 1200
1 = 30 : 24
∴ of temperature on Thursday = 9°
4
Prahlad’s share for managing the business
∴ Temperature on Thursday = 36°
5 25
∴ Temperature on Monday =  36 =  2160
4 2  100
= 45° = Rs 270
176. (b) Ratio of investment = 500 : 300 Remaining profit = 2160 – 270 = 1890
= 5 : 3 24
Prahlad’s share of profit =  1890
( Ratio of share in profit = Ratio of investment) 54
177. (a) Let B’s contribution = (A + C)’s contribution = Rs 840
∴ Total share of Prahlad = 840+270
1
= of the investment = Rs 1110
2

6 . 196
Basic Numeracy and Data Interpretation

182. (a) A + B + C’s share = Rs 5625 184. (c) W : D = 3 : 2 and D : S = 3 : 2


1 W : D : S = 9 : 6 : 4
Given A = (B  C )
2 6x – 4x = 8000
1 2x = 8000
i.e., ( B  C )  ( B  C ) = 5625
2 x = 4000
3 ∴ Total property = (9 + 6 + 4 ) × x
(B  C ) = 5625
2 = 19 × 4000
2 = Rs 76000
B + C = 5625  = 3750 ...(i)
3
1 185. (b) Let the no. of coins be 2x, 3x, 5x and 7x
∴ A’s share = (B  C ) = 1875
2 respectively.
1
Also given B = (A  C ) ...(ii) Total value of the coins = No. of coins ×
4
Value of Coins
4B = (A + C)
2x × 50 + 3x × 25 + 5x × 10 + 7x × 5 = 5200
∴ A + B + C = 5625 ...(iii)
100x + 75x + 50x + 35x = 5200
Putting (ii) in (iii)
260x = 5200
⇒ B + 4B = 5625
5200
5625 x =
B= = 1125 260
5
∴ C = 3750–1125 = 20

= 2625 from (i) ∴ The numbers of 50 p coins = 2 x


= 2×20
@UPSC_THOUGHTS

A’s share = Rs 1875, B’s share = Rs 1125


and C’s share = Rs 2625 = 40
A – B = 1875 – 1125 = Rs 750 186. (a) Let the number of hens be = x
and number of cows be = y
Shortcut Method
Then x + y = 48
A’s share : B and C’s share = 1 : 2
and 2x + 4y = 140
sum of ratio = 1 + 2 = 3
By solving these equations, we get
5625  1
A’s share = = Rs 1875 2y = 44 or y = 22
3
B’s share : A and C’s share = 1 : 4 so, x = (48 – 22) = 26
sum of ratio = 1+4 = 5 ∴ Number of hens = 26
5625  1 187. (c) Let the initial salaries be 3x and 5x
B’s share = = Rs 1125
5 3x  20 13
A – B = 1875 – 1125 = Rs 750 =
5x  20 21
a 2 21(3x + 20) = 13(5x + 20)
183. (c) 
b 3 63x + 420 = 65x + 260
∴ 2b = 3a 420 – 260 = 65x – 63x
3
160 = 2 x
∴ b = a 160
2 x = = 80
3 2
2a  3b 2a  3. a
2 ∴The initial salaries are Rs 240 and 400.
∴ =
4a  b 3
4a  a Short cut
2
4a  9a The initial respective salaries are
=
8a  3a ax (c d) bx (c d )
,
ad bc ad bc
13a
= = 13 : 11 (here, a = 3, b = 5, c = 13, d = 21, x = 20)
11a

6 . 197
Basic Numeracy and Data Interpretation

3 20 (13 21) 5 20 (13 21) Case II:


,
3 21 5 13 3 21 5 13 (2,000  x )  15  2
S.I.2 =
Rs 240, Rs 400 100
188. (b) Cost of 11 chairs = Cost of 3 tables Total S.I. = S.I.1 + S.I.2 = 460
11 C = 3 T x  10 2 (2,000 x)  15 2
11
∴ 460 = 
i.e., T = C 100 100
3
11C ∴ x = 1,400
+C = 140
3 Other part = 2000 – x
11C+3C = 140×3
= 2000 – 1400
14C = 140×3
140×3 = 600
C = = Rs 30 1,400
14
∴ Ratio of investment = = 7 : 3
189. (c) If 5x and 4x are their annual incomes, 600

5x  800 3  1,500 R1  3   1,500 R2  3 


then = 192. (c)     = 13.50
4x  800 2  100   100 
10x – 1600 = 12x – 2400 ⇒ 4,500 (R1 – R2) = 1,350
2400 – 1600 = 12x – 10x
1,350
2x = 800 ⇒ R1 – R2 =
4,500
x = 400
= 0.3%
∴ Mahavir’s income = 5 × 400 = Rs 2000
@UPSC_THOUGHTS

Shortcut Method 193. (d) Interest for 3 years = 3 × 12


= 36% of the principal
Ra (d c )
Mahavir’s income =
ad bc The amount in 3 years = 100 + 36
800 5(2 3) = 136% of the principal = Rs 6,800
5 2 4 3
6,800 100
= Rs2000 ∴ Original sum invested =
136
190. (c) Rate of increase of temperature = Rs 5,000

36  21 194. (a) Let the sum be Rs 100


=
14  9 After 15 years it becomes Rs 300
15 ∴ Interest = 300 – 100
=
5 = Rs 200
= 3 °C per hour. S.I . 100
[ 2 p.m. = 1400 hrs] Then, rate =
P T
∴ Temperature at 12’0 clock
200  100
= 3 × (12 – 9) + 21°C =
100  15
= 3 × 3 + 21
= 13.33%
= 9 + 21
= 30 °C 195. (b) Suppose amount invested in the first type of
share = Rs x
191. (c) Let one part be Rs x
∴ amount invested in the second type of
∴ Other part will be Rs (2000 – x)
share
Case I: = Rs (100000 – x)
x  10 2 9x 11 (100000 x) 39 100000
S.I.1 = ∴
100 100 100 4 100

6 . 198
Basic Numeracy and Data Interpretation

x 1260 × (100 + 5R) = (100 + 2R) × 1350


⇒ = 9750 – 11000
50 1260 × 5R–1350 × 2R = 135000–126000
⇒ x = 50 × 1250 = Rs 62500 6300R – 2700R = 9000
The amount invested in the second type of
9000
share R =
3600
= (100000 – 62500 =) Rs 37,500
= 2.5%
196. (a) Let sum = x then Shortcut Method
A = 2P = 2x 100 [ A2  A1 ]
∴ SI = 2x – x = x Use formula
A1t2  A2t1
100 S.I .
Rate = 100 [1350  1260]
PT ⇒ = 2.5%
1260 5  1350 2
100  x
=
x  25 200. (c) Let P1 and P2 be the principals,
= 4% The rates are 5R and 4R respectively
197. (d) Let the capital be Rs x P1  5R  1 P2  4R  1
=
x 6 x 8 x 10 100  2 100  2
∴ 600
2 100 4 100 4 100 P1 4R  100 2
=
6 x 4x 5x P2 100 2  5R
600
200 200 200 P1 : P2 = 4 : 5
@UPSC_THOUGHTS

15x = 120000
200  4  1
x = Rs 8000 201. (a) S.I. for Ist year = = 8
100
198. (c) Use formula A = 208
100 [A2  A1] He paid Rs 58, remaining amount = Rs 150
Rate % =
A1 t2  A2 t1
150 41
∴ S.I. for IInd year = = 6
100 [1,067.20  1,012] 100
=
1,012 4  1,067.20  2.5 A = 150 + 6
5,520 = Rs 156
=
1,380
15000  9  2  8 
2

= 4% 202. (b) A =  12000  1    1
100  100  
 RT  270000 11664
199. (a) A = P  1   = 12000 1
 100  100 10000
 2R  270000 1664
1260 = P  1   ... (i) = 12000
 100  100 10000
 5R 
1350 = P  1   ... (ii) =
270000 199680
 100  100 100
Divide equation (i) by (ii), 469680
= = Rs 4696.80
100  2R 100
1260 100
⇒ = 203. (b) Let the sum be Rs P, then
1350 100  5R
100   10 
2

P 1   P  = 525
  100  
1260  100  2R  100
=    11  2

1350  100  (100  5R) P    1 = 525
  10  

6 . 199
Basic Numeracy and Data Interpretation

525  100
P=
= Rs 100     1
21 12 12 12
∴ Sum = Rs 2500  10 10 10 

= Rs 100 
2500  5  4  728 
So, S.I. = Rs   
 1000 
 100 
= Rs 500 = Rs 72.8
3 Difference between C.I. and S.I.
 10 
204. (b) P  1   = 65,61,000 = Rs 72.8 – Rs 60
 100  = Rs 12.80
 656100010 1010  If the difference is Rs 12.80, the sum is
∴ P = Rs  
 9  9 9  Rs 100.
But difference is Rs 48, so sum is
= Rs 90,00,000 1
Rs 100 48
12.80
205. (c) P [(1 + R)2 – 1] = 40.80
= Rs 375
⇒ PR (R + 2) = 40.80
Shortcut Method
But PR × 2 = 40 ( S.I. Rs 40) 3 2
R R
∴ PR = 20 C.I. – S.I. = P 3
100 100
40.80
R+2 = = 2.04 3 2
20 20 20
48 = P 3
4 100 100
R = 0.04 = = 4% p.a.
100
@UPSC_THOUGHTS

1 3
206. (c) Simple interest for 1 year = Rs 1440 = P
125 25
∴ Simple interest on Rs 1440 for 1 year
16
= Rs 160 48 = P
125
100  160
Hence, rate per unit =
1440  1 P=
125 48
= Rs 375
1 16
= 11 %
9 209. (d) Difference between C.I. and S.I.
207. (a) Let the sum be Rs 100 = 104 – 100 = Rs 4
Simple interest for 2 years on Rs 100 at 3% 2 4 100
Therefore, Rate = = 8%
per annum = Rs 6 100
Compound interest for 2 years on Rs 100 at 10
 R 
3% per annum = Rs 6.09 210. (b) 2P = P  1  
 100 
If compound interest is Rs 6.09
10
2 =  1 
R 
Simple interest is Rs 6 ∴ 
 100 
But compound interest is Rs 1218
1
R
Simple interest =
6
 1218 ∴ 1 = 210
6.09 100
  R  
n
= Rs 1200 A  P  1  
  100  
208. (c) Let the sum be Rs 100 n
4 P = P  1 
R 

Simple interest for 3 years at 20% = Rs 60  100 
n 1
4 =  1 
 20 
3
 ⇒
R  =
Compound interest = Rs 100  1   (210 )n
  1  100 
 100  

6 . 200
Basic Numeracy and Data Interpretation

n 215. (d) PW =4180 – 180 = 4000


∴ 22 = 210
TD  100 180 100
n T= =

10
=2 PW  R 4000  6
⇒ n = 20 years 3
= years = 9 months
4
211. (a) Let MP be Rs 100
 100  20   100  10   100  25  SI  TD 672 480
⇒ SP = 100      216. (b) Amount = =
 100   100   100  SI  TD 672  480
80 90 75 672 480
= 100    = 54 = = Rs 1680
100 100 100 192
⇒ Single discount = 100 – 54 = 46% 217. (a) Let the amount of the bill be Rs 100. It is
discounted at 4%.
212. (d) Let the LP be x ⇒ Proceeds of the bill = Rs 96
Nothing has to be lost
(100  20) (100  25)
x x = 500 ⇒ SI Rs 4 to get on 96 for 1 year.
100 100
100  4 25
5 x = 500 × 100 ∴ Rate % = =
96  1 6
500  100
x = 1
5 = 4 %
6
@UPSC_THOUGHTS

= 10,000
218. (a) Let TD be x.
10000  80 4
∴ Price after discount = ∴ BD = x
100 3
= Rs 8,000 4 x
⇒ BG = xx =
80 90 3 3
213. (c) 108 = MP   BG 100
100 100 T=
TD R
108  100  100 x  100  3
⇒ MP = = = 4 years
80  90 3  x  25
BD  TD  40  30 
219. (a) Sum = = Rs  
= Rs 150 BD  TD  40  30 
= Rs 120
PRT ART
214. (a) =
100 (100  RT ) 220. (c) TD = PW  BG
= 10000 25
2400157 A 15 7
i.e. , =
43100 4 3 100
15 7 = 250000
4 3 = Rs 500
35A 221. (a) Quantity of milk in the mixture = 48 litres
210 =
 35  Quantity of water in the mixture = 32 litres
4  100  
 4  Let x litres of water be added in the mixture
35  A 35A so that
= =
435 435 48 2
4 =
4 32  x 3

210 435 ⇒ 2x + 64 = 144


∴ A= = Rs 2,610 ⇒ x = 40 litres
35

6 . 201
Basic Numeracy and Data Interpretation

222. (a) 228. (c) Work done by A, B and C in 3 days


1 1 1 
=    3
 6 12 18 
11
= of the work
12
11 1
∴ remaining work = 1 =
12 12
3.50 35 7 Work done by A and B in one day
∴ Required ratio = = =
1.50 15 3 1 1 1
= 7 : 3 =
6 12 4
1 1
223. (a) ∴ A and B complete the work in 4  =
12 3
1
day i.e.,  24 = 8 hours
3
229. (a) Ratio of working capacity of A and B is 3 : 2
3
∴ A’s share =  300
5
∴ ratio of 1st and 2nd parts = 2 : 3 = Rs 180
3  2
∴ quantity of 2nd kind =   1000  kg B’s share =  300
5  5
@UPSC_THOUGHTS

= 600 kg = Rs 120
224. (b) Cost of water = 0 230. (c) 6 men + 5 women = 6 days
Pure milk 3.00 0 3 5 or 3 men + 4 women = 10 days
So,
Water 3.6 3.0 0.6 1 Work done by each group in one day
∴ For every 5 litres of milk, the milkman adds 1
⇒ 6 men + 5 women = ...(i)
1 litre of water, and for every 25 litres of milk, 6
he adds 5 litres of water. 1
or 3 men + 4 women = ...(ii)
10
225. (b) Tin : Copper : Zinc
Now, by solving equations (i) and (ii) we get
3 7 20 56 3 2 1 1 1
= : : 3 women =  =
26 65 26 65 26 65 5 6 30
= 29 : 212 : 19 1 1
1 women can do the work in =
3 30 90
226. (c) Using formula : N1W2D1 = N2W1D2
1
Here work is the same, W2 = W1 and 1 man can do work in .
Now we can derive the formula
54
9 15
⇒ N1D1 = N2D2 ∴ 9 men + 15 women = 
⇒ 10 × 12 = 12 × D2 54 90
1 1
10  12 = 
D2 = 6 6
12 2 1
= 10 days = =
6 3
227. (c) Ratio of A : B = 8 : 12 = 2 : 3 Therefore 9 men and 15 women can finish the
2
∴ B’s share = 200  work in 3 days.
2 3
231. (b) A’s 3 day’s work + B’s 3 days work
2
= 200  = Rs 80 + Boy’s 3 days work = 1
5

6 . 202
Basic Numeracy and Data Interpretation

Number of men
3 3
∴ Boy’s 3 days work = 1    
Total wages
7 8 =
11 Number of days × 1 man's 1day's wage
=
56 5750  8
= = 25 men
3 3 11 16  115
Ratio of shares = : :
7 8 56 236. (d) Work done by A, B and C in 4 days
3  56 3 56 11 56 1 2
= : : = 4 =
7 8 56 10 5
= 24 : 21 : 11 2 3
Remaining work = 1  =
56 5 5
∴ Boy’s share = 11 = Rs 11 3
24 21 11 Now, work is done by B and C in 10 days
5
15  10 Whole work will be done by B and C in
232. (b) (A + B) + C can do work in = 6 days 5 50
15  10 = 10  = days
Since A’s days = (B + C)’s days 3 3
1
B + C can do in 6 × 2 = 12 days (A + B + C )’s one day work =
10
15  12 3
∴ B [B = (B + C) – C] can do in (B + C )’s one day work =
15  12 50
1 3 2 1
= 60 days A’s one day work =  = =
10 50 50 25
1 1 ∴ A alone could complete the work in 25 days.
233. (a) =
1 1 1 1  1 1 1
 
@UPSC_THOUGHTS

30  3 30  5 30  8     237. (d) If B is 1, then A is 1


1
, So
30  3 5 8  2
1 1 1
= 1 + 1 time = 2 officient A and B
1  40  24  15  2 2
 
30  120  complete the work in 10 days.
1 1
= ∴ B will do the work in 10  2days = 25 days
1  79  2
  2
30  120  ∴ A can finish the work in 25 days
2 3
30  120 = 16
3
days
=
79 Alternate Method
Let A finish the work in x days
= 45.5 days
3
Then B finishes the work in x days
2
8  12 96 4
234. (a) = = 4 days
8  12 20 5 x ×3x 2
A and B can finish the work in
3x
15525 x×
235. (b) Wage of a woman for a day = 3x 2 2
45  48 ∴ = 10 ⇒ x = 16 2 3
3x
115 238. (b) Suppose B takes x days to do the work.
= Rs
16  3  3x
∴ A takes  2  x   days to do it
115  4  2
Thus, wage of a man for a day = 2  1
16 (A+B)’s 1 day work =
18
1 2 1
115 ∴  =
= Rs x 3x 18
8
⇒ x = 30 days

6 . 203
Basic Numeracy and Data Interpretation

239. (b) Let 1 man’s 1 day’s work = x and 1 woman’s 242. (c) Work done by A, B and C in 4 days
1 day’s work = y 1 2
= 4 =
1 10 5
Then, 6x + 8y = 2 3
9 Remaining work = 1  =
5 5
1 Whole work will be done by B and C in
and 5x + 9y =
10 5 50
By solving the above two equations we get 10  = days
3 3
1 1
y = (A+B+C)’s 1 day work =
315 10
1 3
∴ 1 woman’s 1 day work = (B+C)’s 1 day work =
315 50
1 1 3 2 1
⇒ 15 women’s 1 day’s work =  15 A’s 1 day work =  = =
315 10 50 50 25
1
= ∴ A alone could complete the work in 25 days.
21
25
Hence, 15 women will complete the work in 21 243. (b) = 195 seconds
0.128
days.
244. (c) 30 men can produce 1500 units in 24 days
240. (c) Ratio of times taken by A and B working 6 hours a day, i.e., 24 × 6 = 144
= 160 : 100 hours.
=8 : 5 30 men can produce 1800 units in
Suppose B alone takes x days to do the job. 144 864
Then, 8 : 5 :: 12 : x  1800 = hours
1500 5
@UPSC_THOUGHTS

⇒ 8 x = 5 × 12 18 men can produce 1800 units in


15 1
⇒ x = = 7 days 864 30
2 2  hours
1 1 5 18
241. (a) Part of work done by A = 5 = = 288 hours
10 2
1 1 = 36 days of 8 hours each.
Part of work done by B = 5 =
15 3 Alternative method
 1 1 1 Using N, W2 D1 T1 = N2 W1 D2 T2
Part of work done by C = 1     =
 2 3 6 = 30 × 1800 × 24 × 6 = 18 × 1500 × D2 × 8
So, A’s share : B’s share : C’s share
30 1800 24 6
1 1 1 = D2
= : : = 3 : 2 : 1 18 1500 8
2 3 6
3  36 days = D2
∴ A’s share = Rs   1500 = Rs 750
6  245. (c) Quantity of the Ist candle burnt in one hour
2  1
B’s share = Rs   1500 = Rs 500 =
6  4
1
1  Quantity of IInd candle burnt in one hour =
C’s share = Rs   1500 = Rs 250 3
6  Let t be the required number of hours.
750 At t hours, remaining of the Ist candle
A’s daily wage = Rs = Rs 150
5 t 4t
= 1 =
500 4 4
B’s daily wage = Rs = Rs 100
5 At t hours, remaining of IInd candle
250 t 3t
C’s daily wage = Rs = Rs 125 = 1 =
2 3 3
∴ daily wages of B and C = Rs (100 + 125) 4t 3t 
Given = 2  
= Rs 225 4  3 

6 . 204
Basic Numeracy and Data Interpretation

1
i.e. , 3(4 – t) = 8(3 – t) = (X + Y + Z)’s daily work =
6
1 1 1
i.e. , 12 – 3t = 24 – 8t ∴ X’s daily work =  =
6 3 18
i.e. , 8t – 3t = 24 – 12 ∴ X alone can finish the work in 18 days.
⇒ 5t = 12
251. (d) Suppose the speeds of P and Q are
12
⇒ t = respectively
5
x m/min and y m/min.
= 2.4 hours Distance covered by P = 500 m
Distance covered by Q in the same time
246. (a) M1 × 16 = (M1 + 8) × 12
= 500 – (45 + 35)
= 12M1 + 96 = 500 – 80
16M1 – 12M1= 96 = 420 m
96 500 420
M1 = = 24 men Thus, =
4 x y
247. (c) Work done by 5 × 12 men
or x : y = 500 : 420 = 25 : 21
= work done by 10 × 10 women
i.e. , 60 men = 100 women 2  Total time  Product of speeds
252. (b) Distance =
3 men = 5 women Sum of speeds
∴ work done by 2 20 80 84
=
3 men + 5 women = 5 women + 5 women 80  84
= 10 women 268800
=
@UPSC_THOUGHTS

i.e., 3 men + 5 women will take 10 days to do 164


the work. = 1639.02 km
248. (b) (A and B)’s 2 day’s work 9
253. (c) Distance covered by the man in hours
2
1 1
= 2  2 =
9
 12 = 54 km
10 15 2
32 5 1 1
= = = To cover 54 km in 1 hours less time,
15 15 3 2
2 i.e., in 3 hours, he should ride with the speed
Remaining work =
3
Time taken by A to complete the remaining 54
= = 18 kmph
work 3
2
2 20 2 254. (d) They are 38 km apart in 1 hour
= 3 =  10 = = 6 days
1 3 3 3 1
10 ∴ They will be 95 km apart in 95 hours
38
249. (d) At the end of 10 days, the food for 2000 men
= 2.5 hours
is left for 30 – 10 = 20 days. At this stage
number of men are 2500 1
= 2 hours
Men Days 2
2500 20
255. (d) Let the distance between Meerut and Delhi
2000 x
be y km
⇒ 2500 : 2000 :: 20 : x
Average speed of the train leaving Meerut
2000  20
⇒ x = = 16 days y
2500 = kmph
4
250. (b) 2(X’s) daily work = (Y + Z)’s daily work
Average speed of the train leaving Delhi
Adding X’s daily work on both sides, we get
2y
3 times X’s daily work = kmph
7

6 . 205
Basic Numeracy and Data Interpretation

Suppose they meet x hours after 6 am 8x  7x


= 10
xy 2y (x 2) 336
Then, y
4 7 15x = 3360
x 2x 4 ⇒ x = 224 km
1
4 7 260. (d) Rise in two minutes = 50 – 25 = 25 cm
∴ 15x = 44
Pole length = 10 m = 1000 cm
44 14 Part of pole climbed in two minutes
x = 2 hours
15 15 1000  50 950
256. (c) Suppose the thief is overtaken x hours after = = = 38
25 25
2.30 pm. ∴ total time = 38 × 2 + 1
Then, distance covered by the thief in x hours= = 77 minutes
 1
distance covered by the owner in  x   hours 261. (a) Time taken by trains to pass each other
 2
 1
2xy 2  3 4
∴ 60x = 75 x   = =
 2 xy 3 4
75 24 3
⇒ 15x = = = 3 seconds
2 7 7
5 262. (b) Let the length of both the trains be x metres
⇒ x = hours
2 Difference in their speeds is 46 – 36
So, the thief is overtaken at 5 p.m. = 10 kmph
@UPSC_THOUGHTS

257. (a) Let the total journey be x km 5 25


= 10  = m/s
x x 1 18 9
Then,  = Distance covered in 36 seconds at this speed
3 3.75 2
2.5x – 2x = 3.75 25
= 36 
3.75 9
⇒ x = = 100 m
0.50
∴ Length of each train is 2x = 100
15
⇒ x = ⇒ x = 50 m
2
⇒ x = 7.5 km 263. (a) Let the speed of the train be x m/sec. Then,
distance travelled by the train in 10 minutes
258. (a) Let the speed of two trains be = distance travelled by sound in 30 seconds
7x and 8x kmph ⇒ x × 10 × 60 = 330 × 30
400
Then, 8x = (given) ⇒ x = 16.5
4
18
x = 12.5 ∴ speed of the train = 16.5 = 59.4 kmph
∴ speed of the first train = 7 × 12.5
5
264. (a) Distance covered = 24 × 6 = 144 km
= 87.5 kmph
Required Speed to make up for lost time
259. (c) Let the distance be = x km
x 144
Time taken in travelling km at the rate of =
2 4
1 x x = 36 km/hr
21 kmph =  = hour
21 2 42
x 265. (a) When they travel in opposite directions
Time taken in travelling km at the rate of 24
2 Relative Speed = Sum of speeds and time
1 x x
kmph =  = hour = 1 hr.
24 2 48
x x 120
But,  = 10 i.e., = 120 = Sum of speeds ...(i)
42 48 1

6 . 206
Basic Numeracy and Data Interpretation

When they travel in the same direction 15  12 15  12  2


= =
Speed = difference of speed; and time = 6 hrs. 25 25
120 2
i.e. , = 20 = difference of speed ...(ii) 72
6 = = 14.4 seconds
5
from (i) and (ii) it’s clear that the speeds are
70 and 50 kmph.
271. (c) Let the bridge be ‘x’ metres long.
∴ The required answer = 70 km/hr
266. (c) Ratio of speeds of the cars = 2 : 3 : 4 130  x
i.e., = 30
5
⇒ ratio of time =
1 1 1
: : 45 
2 3 4 18
1 1 1
= 12  : 12  : 12  (130 + x) × 18 = 30 × 45 × 5
2 3 4
=6 : 4 : 3 30  45  5
x =  130
267. (b) Let x m/s be the speed of the train 18
5 100 x = 375 – 130
x  5 = = 245 m
18 7.2
18x  25 100  10 272. (d) Speeds of two men are 2kmph
=
18 72 5 5
= 2 = m/s
100  10  18 18 9
18x =  25 5
72 and 4 kmph = 4  = 10 m/s
225 18 9
x = Let the speed of the train be x m/s.
18
@UPSC_THOUGHTS

Then the relative speeds are


25 18
= km/hr  5  10 
2 5  x   m/s and  x   m/s
= 45 km/hr  9  9
Now, length of train = Relative speed × time
268. (a) Distance covered by the passenger train in
taken to pass a man
4 hrs.

 5
length of train =  x    9
= 80 × 4 = 320 kms
= distance covered by goods trains in 10 hrs.  9
320  10 
∴ Speed of the goods train = = x    10
10  9 
= 32 km/hr. 100 45 55
∴ x =  = m/s
9 9 9
269. (d) Let the length of the train be x metres. 55 18
∴ Speed of the train (while passing the man) ∴ Speed of train =  = 22 kmph
9 5
x  5
= m/s and length of train = x  9
2  9
The speed of train while passing the bridge is  55 5 
the same =     9 = 50 m
 9 9
x x  50
i.e. , = Shortcut Method
2 9
  Length of the train
2
Diff. in speed × T1 T2 of two men
x 2x  100 =
i.e. , = T2 T1
2 9
 10 5 
9 x = 4x + 200     9  10
= 
9 9
5 x = 200 = 50 m
x = 40 m 10  9
273. (c) Let C’s speed = x km/hr
15 15 25
270. (a) Speed of the train = = m/s. B’s speed = 3x km/hr
18 2
Time taken after the detachment of 3 boggies and A’s speed = 6x km/hr

6 . 207
Basic Numeracy and Data Interpretation

∴ Ratio of speeds of A, B and C 1


= 6x : 3x : x Hence, rate of the stream = (30  10)
2
=6 : 3 : 1 = 10 kmph
1 1
Ratio of time taken = : : 1
6 3  10 
278. (b) Rate upstream =   60  kmph
=1 : 2 : 6  50 
If C takes 6 minutes, A takes 1 minute = 12 kmph
If C takes 42 minutes, A takes   42  min
1 Speed of the stream = 2 kmph
6  Let the speed in still water be x kmph.
= 7 minutes Then, the speed upstream = (x – 2) kmph
∴ x – 2 = 12
 5 ⇒ x = 14 kmph
274. (d) 5 km/hr =  5  m/sec.
 18 
z( x 2  y 2 )
25 279. (d) Using formula =
= m/sec. 2x
18 Here z = 1, x = 6 and y = 1.2
Distance travelled in 5 minutes or 300 seconds 1(62  1.22 ) 36  1.44
⇒ = =
25 2 6 12
=  300 34.56
18 = = 2.88 km
1250 12
= metres
3 Distance
280. (c) Speed =
1250 Time
∴ Diagonal of the field = metres
@UPSC_THOUGHTS

3 20
Man’s speed downstream = = 5 kmph
1 1250 1250 2 4
∴ Area of the field =   m 8
2 3 3 Man’s speed upstream = = 2 kmph
= 86805.55 m2 4
(5 2)
= 868.05 acres ∴ Velocity of the stream = = 1.5 kmph
2
275. (b) It may be solved algebraically 281. (b) Time taken during downstream journey
Rate × Time = Distance
50 5
1 = = hours
Going 12 × = 6 60 6
2
Time taken during upstream journey
1 3
Coming x × 1 = x = 1 hour 15 minutes
2 2
15 5
The distances are equal. = 1 = hours
60 4
3 Total time taken during the whole journey
6 = x or 12 = 3x or x = 4 kmph
2 5 5 25
276. (b) 85 km × 9 days = 765 km =  = hours
6 4 12
65 km × 9 days = 585 km Total distance covered = 50 + 50 = 100 miles
765 + 585 = 1350 km total distance covered
∴ average speed =
1550 – 1350 = 200 km apart at the end of total time taken
9 days. 100
= = 48 mph
25
277. (a) Let the man’s rate upstream be x kmph
12
Then, rate downstream is 3x kmph 20
282. (c) x + y = downstream rate =
1
1
∴ Rate in still water = (3x  x ) = 2x kmph = 20 km/hr ...(i)
2
So, 2x = 20 ⇒ x = 10 20
x – y = upstream rate =
∴ Rate upstream is 10 kmph, rate downstream 2
is 30 kmph. = 10 km/hr ...(ii)

6 . 208
Basic Numeracy and Data Interpretation

where x is speed of boat in still water and 286. (c) A : B = 7 : 3


y is speed of water. In a race of 7 metres, A gains 7 – 3 = 4 m
from (i) and (ii), we get over B
2x = 30 ∴ The distance covered by A if A gains 60
⇒ x = 15 km/hr metres
and y = 5 km/hr 60
=  7 = 105 metres
30 4
Time = = 2 hours
15 22.5 25
283. (a) Let the speed of the swimmer be x km/hr 287. (c) 
x 100
and the speed of the stream be y km/hr. where x is the distance covered by B
∴ Swimmer’s downstream rate = (x + y) km/hr
22.5  100
Swimmer’s upstream rate = (x – y) km/hr  x =
25
Distance travelled in 5 minutes is given as = 90
5(x y ) 5(x y ) 100  A can beat B by 10 metres
= km
60 60 1000 288. (c) A : B = 100 : 75
(x y ) (x y ) 1 B : C = 100 : 96
=
12 12 10 A B 100 100
A : C =  = 
12 B C 75 96
( x + y) – ( x – y) =
10 = 100 : 72
1.2 A beats C by (100 – 72) = 28 metres.
y = = 0.6 km/hr
@UPSC_THOUGHTS

2
284. (a) Let ‘d ’ be the distance one way 289. (c) Let k be the length of the race
d d 75 ⇒ Distance covered by A = k metres
 = Distance covered by B = k – (9+1)
51 51 60
= k – 10 metres
d d 75
 = The rate of speed of A and B is given
6 4 60
40 : 39 :: k : k – 10
2d  3d 75 i.e., 39k = 40 (k – 10)
=
12 60 400 = 40k – 39k = k
75 12 i.e., The length of the race course = 400 m
d = = 3 kms.
60  5 290. (d) The race ends in a dead heat
285. (a) Let loser’s distance be x m ⇒ time taken by S and J are the same.
Winner’s time Loser’s time Distance
 Time =
Loser’s distance Winner’s distance Speed
J’s distance S’ s distance
290 sec 300 sec
i.e., 
i.e., = J’ s speed S’ s speed
x 1000
If k is the distance, and ‘d’ is the speed then
290 1000
∴ x =
300 k  150 k 8k
 i.e., k – 150 =
d 11 11
2900 d
= 8
3
11k – 150 × 11 = 8k
2
= 966 m 11k – 8k = 150 × 11
3
2 1
i.e., A beat B by 1000 – 966 = 33 150 11
3 3 k =
3
metres. = 550 metres

6 . 209
Basic Numeracy and Data Interpretation

291. (b) When A runs 100 metres, B runs 80 and C Thus Raj and Aryan will be together after 44
runs 75 metres mins, while Raj and Rohit are together again
When B runs 100 metres, the distance covered after 32 mins.
Now taking LCM of 44 and 32 we get 352
75 375
by C =  100  = 93.75 m So, Raj, Aryan and Rohit are first together
80 4 again after 352 minutes or 5 hours 52 minutes
⇒ B beats C by 100 – 93.75 = 6.25 m or 1 sec.
295. (c) When Raman makes 5 rounds, then Aditya
i.e., Distance covered by C in 1 sec = 6.25 m
makes 4 rounds
∴ Time taken by C to cover 75 m Distance covered by Raman in 5 rounds is
75  5  300  3
= = 12 sec.   km = km
6.25  1000  2
⇒ Time taken by A to run 100 m Distance covered by Aditya in 4 rounds is
= 12 seconds  4  300  6
  km = km
 1000  5
292. (d) When A covers 400 m, B covers 395 m
So in covering 10 km Raman passes Aditya
When B covers 400 m, C covers 396 m 6 2
  10 = 8 times
396 5 3
When B covers 395 m, C covers × 395
400 3 6
296. (c) A + B together fill the tank in hours
 391.05 m 3 6
∴ When A covers 400 m, C covers 391.05 m = 2 hours

When D covers 400 m, C covers 384 m 297. (b) A + B + C fill the tank in
@UPSC_THOUGHTS

When C covers 391.05 m, D covers 407.3 m 12 15 9 12  15  9


=
∴ When A covers 400 m, D covers 407.3 m 15 9  12 9  15 12 63
When D covers 400 m, A covers 392.8 m 180 5
= = 25 hours
∴ D wins by 7.2 m 7 7

293. (b) A scores 30 points, then B scores 25 points 298. (a) Let B be closed after x minutes.
if A scores 30 points, then C scores 23 points Then part filled by (A + B) in x minutes + Part
filled by A in (18 – x) minutes = 1
Now, when B scores 25 points, C scores 23
1 1 1
points ∴ x     (18  x)  =1
∴ When B scores 40 points  24 32  24
7x 18  x
⇒  =1
 23  96 24
C scores  40 = 36.8
 25  ⇒ 7x + 4(18 – x) = 96
= 37 points ⇒ x =8
Hence, B can give C, 3 points in a game of Hence, B must be closed after 8 minutes.
40. 299. (c) Let the slower pipe alone fill the tank in x
x
294. (c) Raj takes 1 min to gain 40 m over Aryan. minutes. Then, faster pipe will fill it in
3
To gain 1760 m over Aryan, Raj takes minutes.
1 3 1
1 ∴
= × 1760 x x 36
40
4 1
= 44 mins x 36
Raj takes 1 min to gain 55 m over Rohit ∴ x = 36 × 4
∴ To gain 1760 m over Rohit, = 144 minutes
1 Thus, the slower pipe fills the tank in 144
Raj takes = × 1760 = 32 mins
55 minutes.

6 . 210
Basic Numeracy and Data Interpretation

2 1 303. (d) We have x = 10 and y = 11


300. (a) Part filled in 2 hours = 10  11
6 3 ∴ the leak will empty the cistern in
11  10
2 1 = 110 hours
Remaining part = 1
3 3 304. (c) Half the tank is filled by one tap in 8 minutes
2 Half the tank is emptied by one tap in 4
∴ A + B’s 7 hours’ work = minutes
3
1 1 1
2 1 2 In 1 minute = – is filled when both
A + B’s 1 hour work = 8 4 8
3 7 21
1
∴ C’s 1 hour work = (A + B + C)’s 1 hour work taps are open i.e., of the tank is emptied
– (A + B)’s 1 hour work 8

1 2 1 ∴ It takes 8 minutes to empty the tank.


=
6 21 14
∴ C alone can fill the tank in 14 hours. 1
305. (a) A’s 1 hour work =
301. (d) Let pipe A fill the cistern in x minutes 7
∴ Pipe B will fill the cistern in (x + 5) minutes 1
B’s 1 hour work =
∴ In one minute, pipes A and B together can fill 4
1 1 (A + B )’s 4 hour work when opened
 of the cistern. 1 1 11
x x 5 alternatively =  =
7 4 28
1 1 1
⇒  =
x x 5 6 (A + B)’s 4 hour work when opened alternately
@UPSC_THOUGHTS

⇒ x = 10 22
=
∴ pipes A and B can fill the cistern in 10 minutes 28
and 15 minutes respectively. 22 6
Remaining part = 1  =
28 28
302. (b) As the pipes are operating alternately, thus 3
1 1 11 =
their 2 minutes job is  = 14
5 6 30 1
Now, it is A’s turn and part is filled by A
In next two minutes, the pipes can fill another 7
in 1 hour.
11 3 1 1
part of cistern. ∴ Remaining part to be filled by B =
30
14 7 14
∴ In 4 minutes, the two pipes which are
1 4 2
B fills in = hours
11 11 22 14 14 7
operating alternately will fill  =
30 30 30 ∴ Total time taken to fill the tank is
11 2
= , part = 4 + 1 +
15 7
11 2
The part of the cistern left unfilled = 1  = 5 hours
15 7
4
= 306. (d) Quantity of water flowing into the water tank
15
1 in 1 hour
Pipe A can fill of the cistern in one minute. = 42 + 56 – 48
5
1 1 = 98 – 48
Pipe B can fill of the cistern in 5  = 50 litres
6 6
5 ∴ Capacity of the tank = 16 × 50
= minutes
6 = 800 litres
5
Total time taken to fill the cistern 5 
6 307. (c) Part filled 1 in hour (without the leak)
5 1
= 5 minutes =
6 3

6 . 211
Basic Numeracy and Data Interpretation

2 311. (d) The profit should be divided in the ratios of


Part filled in 1 hour, because of the leak =
7 the capitals, i.e., in the ratio 5 : 7 : 10.
Part emptied by the leak in one hour
Sum of ratio = 5 + 7 + 10 = 22
1 2 76 1
=  = = 10
3 7 21 21 C’s share =  17,600 = Rs 8000
⇒ Time taken by the leak to empty the tank 22
312. (a) Ratio of capitals
= 21 hours.
= 5,000 × 12 : 8,000 × 9 : 12,000 × 2
308. (d) Work done by the inlet pipe and the leak
= 5 × 12 : 8 × 9 : 12 × 2
1 1 32 1
=  = = =5 : 6 : 2
8 12 24 24
Sum of ratio = 5 + 6 + 2 = 13
⇒ Tank can be full in 24 hours. 2
C’s share =  9,100 = Rs 1,400
⇒ Tank can be full in 24 × 60 = 1440 minutes 13
∴ Volume of tank = 1440 × 6 litres/minute 313. (b) Let B’s capital be used for x months.
The ratio of A’s and B’s capital = 7×8 : 9× x
= 8640 litres
7 8 8
So, we have =
309. (b) Let x be the time in which the waste pipe 9 x 9
can empty the bath. Part filled by both pipes 789
⇒ x =
in 1 minute 89
⇒ x =7
1 1 32 5 1 ∴ B’s capital is used for 7 months.
=  = = =
20 30 60 60 12
314. (d) Let B’s investment be Rs x.
⇒ Bath can be filled in 12 minutes.
Then, 8,000 : x : : 360 : 450
@UPSC_THOUGHTS

But the bath was full in 12 + 3 = 15 minutes, 8,000 360


⇒ =
due to the waste pipe. For the first 12 x 450
minutes, the waste pipe was open. 8000  450
⇒ x =
360
∴ the whole work done (bath filled) is given as ⇒ = Rs 10,000
 1 1 1  1 1 
12     3   = 1 315. (a) Suppose B joined after x months.
 20 30 x   20 30 
Then, B’s money was invested for (12 – x)
1 1 1
i.e., 12   12   3  =1 months.
12 x 12
76,000  12 2
12 1 ∴ 57,000  (12  x )
=
1  = 1 1
x 4
⇒ 9,12,000 = 1,14,000 (12 – x)
12 1
i.e., = 114(12 – x) = 912
x 4
⇒ 12 – x = 8
⇒ x = 48 minutes
⇒ x =4
310. (b) Part emptied in one minute (without open tap)
Hence, B joined after 4 months.
1
= 1
30 316. (a) A = (B  C ) ⇒ 2A = B + C
2
Part emptied in one minute, when the tap is 1
B = (A  C ) ⇒ 4B = A + C
1 4
opened =
40 A + B + C = 56,250
1 1
Work done by filling tap =  ⇒ A + 2A = 56,250
30 40
43 1 ⇒ A = 18,750
= =
120 120 Similarly, B = 11250 and C = 26250.
⇒ cistern will be full in 120 minutes. ∴ Share of A is more than that of B by Rs 7,500.

6 . 212
Basic Numeracy and Data Interpretation

317. (a) Let ‘x’ be B’s initial capital 840  25


Ratio of profit of A and B ⇒ x = = 3,000
7
= 20,000×12 + 10,000×4 : x × 4 +(x – 4000)×8
∴ total profit is 3,000.
= 2,40,000 + 40,000 : 4x + 8x – 32,000
= 2,80,000 : 12x – 32,000 are equal (given) 319. (a) A : B : C = 2 : 3 : 7
i.e., 12x – 32,000 = 2,80,000 i.e.,these shares are 2 x , 3x and 7x
12x = 2,80,000 + 32,000 respectively.
i.e., 2x + 3 x = 7x–1,500
= 3,12,000
i.e., 1,500 = 7x – 5x = 2x
3,12,000 1,500
x = = 26,000 x = = 750
12 2
318. (b) Ratio of investment = 8,000 : 5,000 : 7,000 ∴ A’s share = 2 × 750 = 1,500
Sum of the ratios = 8 : 5 : 7
1 1 1
Let the total profit be x. 320. (b) Ratio of initial investments = : :
2 3 4
60
Then 60% of the profit = x =6 : 4 : 3
100
3x Let their investments be 6x, 2x and 3x
=
5 respectively.
Let A, B and C be the partners. A : B : C
3x 3x x = (6x × 2 + 3x × 10) : (4x × 12) : (3x × 12)
Then share of each = = =
5 3 5 5 = 42 : 48 : 36
@UPSC_THOUGHTS

3
∴ A’s share of the remaining profit = 7 : 8 : 6
 8
=
8 40
 x =
4x ∴ B’s share = Rs  378   = Rs 144
20 100 25  21 

5 40 MV×Amount of Stock
x 321. (a) Purchase Cost =
B’s share of profit =  x = 100
20 100 10
100 1,000
7 40x 7x = = Rs 1,000
C’s share =  = 100
20 100 50 322. (a) Purchase Cost
x 4x 5x  4x 9x
A’s total share =  = = (MV+brokerage)×Amount of stock
5 25 25 25 =
100
x x 3x
B’s total share =  = 1
5 10 10 100 5 2,500
2
=
x 7x 17 x 100
C’s total share =  =
5 50 50
= 95.5 × 25 = 2387.50
9x 3x 17x
 840 =  323. (b) Let the amount of investment in each case
25 10 50
be 102 × 98.
15x  17 x 32x 1
= =
50 50 ∴ Income received from 5 % stock at 102
2
9x 32x 11 102 98
⇒  840 = = = 539
25 50 2 102
1
32x 9x Income received from 4 % stock at 98
⇒ 840 =  2
50 25 9 102 98
= = 459
32x  18x 2 98
=
50 By investing the same amount, the income
14x 7x 1
840 = = received from 5 % stock at 102 is better
50 25 2
than the other one.

6 . 213
Basic Numeracy and Data Interpretation

8,800 12 100 Dividend from these 700 shares


324. (b) Annual income =
110 100 5,600 8
= = Rs 448 ...(i)
= Rs 960 100
Cash realised on sale of 300 shares
325. (c) Sale value of 1 share = 680
= 300 × 10 = 3,000
Purchase value of 1 share = 165
Income from this stock
⇒ gain = 300 × (680 – 165)
% rate of stock × Investment
= 300 × 515 =
MV
= 1,54,500 11 3,000
= = Rs 440 ...(ii)
326. (d) Sale value of 1 share = 100 + 150 = 250 75
Purchase value = 150 ∴ Arvind’s total income = 448 + 440 = Rs 888
Gain = 250(250 – 150) 3,000  100
330. (d) Amount of stock = = Rs 4,000
= Rs 25,000 75
13  3,000
327. (c) Annual Income Income from this stock = = Rs 520
75
% rate of stock × Investment
= Income from ‘‘Mavelikara rice” = 520 + 56
MV + brokerage = Rs 576
Let the investments be x and 5400 – x. Cash realised from the sale of first stock
7 x 6 (5,400 x ) = Investment for ‘‘Mavelikara rice”
∴ = Income × Market Value
147 144 =
% rate of stock
24x = 21(5400 – x)
576  90
= = 3,456
24x + 21x = 21 × 5400 15
@UPSC_THOUGHTS

21 5,400 So, in the case of first stock


x =
45 Cash realised on sale = Rs 3,456
i.e., one investment @ 7% stock = Rs 2520 Amount of stock = Rs 4,000
∴ The second investment = 5,400 – 2,520 ∴ Market value at which the stock is sold
= Rs 2,880 Sale realised × 100
=
328. (c) If x be the amount invested in 7% stock and Amount of stock
12,100 – x be the amount in 9% stock. 3,456  100 3,456
= = = Rs 86.40
7 x 9 (12,100 x ) 4,000 40
i.e., = 990
80 117
331. (c) Cash value of 250 shares = 250 ×16
7x 12,100 x 1 25 1
i.e., = 990 ∴ annual income at 6 % = 4,000  
80 13 4 4 100
= Rs 250
13 × 7x + 80(12,100 – x) = 990 × 80 × 13
332. (d) Simple interest on Rs 1,000 at 6% for 3 years
91x – 80x = 990 × 80 × 13 – 80 × 12,100
1000 6 3
11x = 80 × 110(9 × 13 – 110) = = Rs 180
100
80 110 7 ∴ Sum due = 1,000 + 180 = Rs 1,180
x =
11
 1
investment at 7% stock = Rs 5,600 333. (d) ∴By investing Rs  95   ,
 4
⇒ the other investment at 9% stock
stock purchased = Rs 100
= 12,100 – 5,600 By investing Rs 2667, the stock purchased
= Rs 6,500 100 4 2667
= Rs
329. (a) Face value of 700 shares = 700 × 8 381
= Rs 5,600 = Rs 2,800

6 . 214
Basic Numeracy and Data Interpretation

334. (b) Suppose amount invested in the first type of 340. (b) April 18 May June 6
shares = Rs x. 13 + 31 + 6 = 50 days
∴ The amount invested in the second type of 341. (b) Let the man go out x minutes past 5
shares
= Rs (1,00,000 – x) Then x = 720  780  5 minutes past 5
143
9 x 11(1,00,000  x ) 39  1,00,000 720  3900 4620
∴  = = 
100 100 4  100 143 143
x 44 4
⇒ = 9,750 – 1,100 = 32 32 minutes past 5
50 143 13
⇒ x = 50 × 1250 342. (a) The minute hand is ahead of 5.30. At 5
o’clock, the hands are 25 minute spaces
⇒ x = Rs 62,500
100000 – x = 100000 – 62500 apart. Then for the hands to be at right angles
= Rs 37,500 after 5.30, 25 + 15 = 40 minute spaces have
to be gained over the hour hand, i.e., 40
2 40 60 480
335. (d) CP of of the stock = Rs 11,000 minutes gained in
3 55 11
2 7
SP of of the stock = Rs 2,000 = 43 minutes.
3 11
1 7
CP of of the stock = Rs 500 At 43 minutes past 5, the hands will be
3 11
1 at right angles between 5.30 and 6 o’clock.
SP of of the stock = Rs 2,500
3 343. (b)  1st January is Monday
@UPSC_THOUGHTS

∴ Total profit = SP – CP ∴ Sunday falls on 7th, 14th, 21st, 28th


= 4,500 – 1,500 ∴ 4 Sundays + 27 other days
= Rs 3,000 ∴ he brushes 4×1+27×2 = 58 times in January
336. (d) After every 12 hours, the system is repeated. 400
∴ Average usage = = 6.89
In 12 hours, the total number of strikes 58
= 1 + 2 + 3 + ........ + 11 + 12 344. (c) 1st January is Sunday
12 (1  12)  n(n  1)  ∴
 Sn 
8th, 15th, 22nd and 29th January are also
2 
=
2  Sundays
12  13 5 Sundays
= ∴ he brushes 5 × 1 + 26 × 2 = 57
2
400
= 78 ∴ average usage = = 7.01
57
∴ Total number of strikes = 2 × 78 = 156 345. (d) If first of January is Monday or Tuesday or
337. (c) In one hour, the hands are at right angles Wednesday, Sunday comes four times.
twice, so in 12 hours, the hands are at right ∴ If 1st January is Sunday or Saturday then
angles 22 times,—the exception because 2 Sunday comes 5 times.
positions 3 o’clock and 9 o’clock are common. Average usage will be minimum if Sundays
In 24 hours, they are at right angles 44 times. are less.
338. (b) In every hour, both hands coincide once. 346. (c) Ist Saturday = 22 – 21 = 1
⇒ In 12 hours, the hands coincide 11 times, So the day on the 13th of the month will be
— because between 11 and 1 o’clock, the 1 + 7 = 8 = Saturday
hands coincide only at 12 o’clock and, 8 + 5 = 13 = Thursday
⇒ In 24 hours, the hands coincide 22 times 347. (b) At 11 o’clock, the minute hand is 55 minute
339. (d) Same day of the week is repeated after 7 spaces apart. To be coincident, it must gain
days 55 minute spaces.
∴ After 91 days, it would be Monday 55 minutes gained in 60 minutes or
∴ 95th day will be Friday. ⇒ the hands coincide at 12 o’clock only

6 . 215
Basic Numeracy and Data Interpretation

348. (a) At 9 a.m., both the minute hand and hour 352. (c) Let the age of the children be x, x + 3, x + 6,
hand are 15 minute spaces apart. To be in x + 9 and x + 12
straight line, i.e., opposite directions, it has By the given condition, we get
to gain 30 minute spaces. x + x + 3 + x + 6 + x + 9 + x + 12 = 50
⇒ 5x + 30 = 50
∴ It has to gain 30 – 15 = 15 minute spaces ⇒ 5x = 20
over the hour hand. ⇒ x =4
i.e., 15 minutes will be gained in ∴ The age of youngest child is 4 years.
60  15 180 4 353. (d) Let C’s age be x years, B’s age be 2x years
 = 16 minutes
55 11 11 and A’s age be 2x + 2 years
i.e., the hands will be in opposite direction at
4 A + B + C = 27
16 minutes past 9 a.m.
11 2x + 2 + 2 x + x = 27
349. (b) 55 minutes are gained in 60 minutes
60
⇒ 5x = 25
60 minutes are gained in 60
55 ⇒ x =5
5
= 65 minutes ∴ B’s age is 2 × 5 = 10 years
11
5
Loss in 66 minute = 66 – 65 354. (b) Let the present age of the person be x years.
11
6 Then,
= minutes
11 3(x + 3) – 3(x – 3) = x
350. (b) Time from 10.00 a.m. on Friday to 2.00 p.m.
@UPSC_THOUGHTS

⇒ (3x + 9) – (3x – 9) = x
on Monday = 24 × 3 + 4 = 76 hours
IInd clock gains 1 minute every hour ⇒ x = 18 years

∴ In 24 hours, second clock gains 24 minutes FM


355. (b) A =  20
∴ 24 hours of correct clock 2

= 24 hours + 24 minutes of second clock 356. (c) Let the present age of the two brothers be
x and 2x years, respectively. Then,
122
= hours x 5 1
5 =
122 76 1159 2x  5 3
∴ 76 hours of correct clock =  
5 24 15 ⇒ 3(x – 5) = (2x – 5)
= 77 hours 16 minutes
∴ required time on second clock ⇒ x = 10
= 77 hours 16 minutes
After 5 years,
after 10 a.m. = 3.16 p.m. on Monday.
x 5 15 3
351. (a) Let P’s age be 6x, then Q’s age = 7x. ∴ Required ratio = = = or 3 : 5
2x  5 25 5
Given that7 x = 6x + 4 357. (b) Let Shaurya’s age be x years and her
⇒ x =4 mother’s age be y years
∴ P’s present age = 6×4 = 24 and (y – 10) = 4(x – 10) ...(i)
Q’s present age = 7×4 = 28 and (y + 10) = 2(x + 10) ...(ii)
After 4 years P’s age would be 24 + 4 = 28 By solving (i) and (ii), we get
years
x = 20 years and y = 50 years
and Q’s age would be 28 + 4 = 32 years
358. (a) Let the son’s age be x and the father’s age
P 28 7 be y
∴  
Q 32 8 From the given conditions, we get
p : q = 7 : 8 x + y = 60 ...(i)

6 . 216
Basic Numeracy and Data Interpretation

andy – 6 = 5(x – 6) 5  5  20
∴ average age of X, Y And Z =
⇒ y – 6 = 5x – 30 3
⇒ 5x – y = 24 ...(ii) 30
= = 10 years
Adding equations (i) and (ii), we get 3
x + y + 5x – y = 60 + 24 361. (b) Number of ways in which all the five letters
6 x = 84 are posted in wrong envelopes
x = 14 1 1 1 1 1
∴ After 6 years, the son’s age will be 14 + 6 |5 1
|1 |2 |3 |4 |5
= 20 years
1 1 1 1
Alternative Method 120
S t (n 1) 2 6 24 120
Son’s present age = years
n 1 = 60 – 20 + 5 – 1
[Here, S = 60, t = 6, n = 5] = 44
60 6 (5 1)
= = 14 years 362. (a) Words starting with A are |4
5 1
Words starting with M are |4
After 6 years, son’s age = 14 + 6 = 20 years
Words starting with N are |4
359. (c) Let the present ages of x and y be 5x and
Words starting with O are |4
6x years respectively. Then,
Words starting with WA are |3
5x  7 6
= Words starting with WM are |3
6x  7 7
7(5x + 7) = 6(6x + 7) Words starting with WN are |3
35x + 49 = 36x +42 Words starting with WOA are |2
⇒ x =7 Next word will be WOMAN
@UPSC_THOUGHTS

∴ Position of WOMAN
∴ present age of x is 5 × 7 = 35 years
= |4 +|4 +|4 +|4 +|3 +|3 +|3 +|2 +1
Shortcut method = 117
a = 5, b = 6, t = 7, c = 6, d = 7
363. (a) If n is the number of sides, then the number
at (c d ) of diagonals = nC2 – n = 44
x’s present age =
ad bc
1
5 7 (6 7) ⇒ n (n – 1) – n = 44
= 2
5 7 6 6
= 35 years ⇒ (n – 11) (n + 8) = 0
⇒ n = 11
X Y Z
360. (c) Z = 2 ...(i) 364. (b) The number of straight lines
3
= 18C2 – (5C2 – 1) = 144
⇒ 3Z = 2X + 2Y + 2Z ...(ii)
365. (b) If n is number of persons in a room,
Z = 2X + 2Y
total number of handshekes = nC2 = 66
1
1 X Y Z ⇒ n (n – 1) = 66
and X = 2
2 3
⇒ n2 – n – 132 = 0
⇒ (n + 11) (n – 12) = 0
⇒ 6X = X + Y + Z
⇒ n = 12
(Putting value of z)
366. (b) Every prize can be given by any of the four
6 X = X + Y + 2X + 2 Y boys, so number of ways
⇒ 6 X = 3X + 3Y (Given Y = 5) = 4 × 4 × 4 × 4 × 4 = 45
⇒ 3X = 3 × 5 367. (d) Total number of lines obtained by joining n
⇒ X=5 vertices of polygon is nC2. Out of these, n
∴ Z = 2X + 2Y lines are sides and remaining are diagonals
= 2×5+2×5 n (n 3)
∴ Number of diagonals = nC2 – n =
= 20 2

6 . 217
Basic Numeracy and Data Interpretation

368. (d) There are 13 letters in the given word. 3 are 374. (a) We may have 1 black and 2 non-black balls
a’s, 4 are s’s, 2 are n’s, 2 are i’s and rest or 2 black and 1 non-black balls or 3 black
2 are different balls.
13 ! ∴ Required number of ways
∴ Required number of words =
3! 4 ! 2 ! 2 ! = (3C1 × 6C2) + (3C2 × 6C1) + (3C3)
= 10810800
369. (b) Five persons can be seated at a round table 6 5 3 2
= 3 6 1
in 2 1 2 1
(5–1)! 4! = 24 ways = 45 + 18 + 1
Required number of arrangements
= 64
1
24 = 12 375. (c) We may have 1 boy and 3 girls or 2 boys
2
4 and 2 girls or 3 boys and 1 girl or 4 boys
370. (b) C3 × 4C2 + 4C2 × 4C3
6
C1 × 4C3 + 6C2 × 4C2 + 6C3 × 4C1 + 6C4
4! 4! 4! 4!
=
(4 3)! 3! (4 2)! 2! (4 2)! 2! (4 3)! 3! 6! 4! 6! 4! 6! 4! 6!
=
= 4 × 6 + 6 × 4 5! 1! 1! 3! 4! 2! 2! 2! 3! 3! 3! 1! 2! 4!

= 24 + 24 = 48 = 6 × 4 + 15 × 6 + 20 × 4 + 15
371. (b) The possible ways are as follows: = 24 + 90 + 80 + 15 = 209
(i) 1 red ball out of three and 5 blue balls out 376. (a) The word SOFTWARE’ has 3 vowels and
of seven. 5 consonants. We can take 3 vowels as one
@UPSC_THOUGHTS

(ii) 2 red balls out of three and 4 blue balls out letter.
of seven.
∴ Word can be written in 6! ways = 720 and
∴ Total number of ways in which a random
3 vowels can be arranged in 3! ways = 6
sample of six balls can be drawn
= 3C1 × 7C5 + 3C2 × 7C4 ∴ Required number of words = 720 × 6 = 4320
3! 7! 3! 7!
= (3 1)! 1! (7 5)! 5! (3 2)! 2! 377. (a) Required number of ways
(7 4)! 4!
=3 × 21 + 3 × 35 = (ways of going to Delhi)
=63 + 105 = 168 × (ways of returning to Jaipur)
372. (a) There are 6 bowlers, 3 wicketkeepers and 11 = 12 × 11 = 132
batsmen in all.
8! 8!
The number of ways in which a team of 4 378. (b) 8
C3 × 8C3 =
bowlers, 2 wicketkeepers and 5 batsmen can 5! 3! 5! 3!
be chosen 8 7 6 8 7 6
=
6 3
= C4 × C2 × 11
C5 3 2 1 3 2 1
= 56 × 56
6! 3! 11!
= (6 4)! 4! (3 2)! 2! (11 5)! 5! = 3136
379. (a) 4! × 2 ways, i.e., 24 × 2 = 48
= 15 × 3 × 462
4
= 20790 380. (a) C3 × 4C2 + 4C2 × 4C3

373. (c) 1, 2, X are boys; 3, 4, Y are girls 4! 4! 4! 4!


=
1, 2 can be interchanged in x 1! 3! 2! 2! 2! 2! 1! 3!
2 1
P2 = 2 ways
2 = 4 × 6 + 6 × 4
3 and 4 can also be
3 = 24 + 24
interchanged in 2 ways
4
∴ Required number of ways = 2 × 2 =y 4 = 48

6 . 218
Basic Numeracy and Data Interpretation

381. (c) No. of tosses = 1000 3


388. (a) No. of elements in S = C2
No. of tails = 703
3!
No. of heads = 1000 – 703 = 297 = 2! 1! = 3
297 S = [R1R2, R2R3, R1R3]
Probability of heads is = = 0.297
1000
389. (c) Let S be the event of getting one red and
382. (a) Total shirts = 24 one yellow ball.
Total ties = 9  No. of elements in S is
No. of ways of choosing a shirt and a tie is
= 5C 1 7
C1
= 24 × 9 = 216
= 5 × 7 = 35
n!
383. (d) nCr = (n = 9), (r = 4) 390. (d) S = {HH, HT, TH, TT}
r !(n r )!
A = {HT, TH}
9! 9 8 7 6 B = {HT, TH, TT}
= =
4! 5! 4!
(A B) = {HT, TH}
9 8 7 6
= = 126 391. (a) Here S = {(1, 1, 1,1), (1, 1, 1, 2)...
4 3 2 1
...(6,6,6,6)}
384. (a) Let A be the event having a colour set and
B be having a black and white set  No. of possible outcomes = 6×6×6×6
= 1296
P(A) = 0.86, P(B) = 0.35, P( A  B) = 0.29
@UPSC_THOUGHTS

Let A be the event of getting a sum less than 8


P( A  B) =0.86 + 0.35 – 0.29 = 0.92
A = {(1,1,1,1), (1,1,1,2), (1,1,1,3), (1,1,1,4)
385. (c) Probability of the first tube being defective
(1,1,2,1), (1,1,3,1) (1,1,4,1), (1,2,1,1)
is
(1,3,1,1), (1,4,1,1) (2,1,1,1), (3,1,1,1)
15
= (4,1,1,1)}
240
P  (sum > 8)
Probability that the second tube will be
n(A) 13
defective is P( A) = =
14 n(S) 1296
= 392. (b) Total number of arrangements = (5+5)!
239
= 10 !
Thus, the probability that both the tubes might Considering 6 girls as a group of one
15 14 7 ∴ No. of favourable cases = (5+1)! 5!
be defective = =
240 239 1912 = 6! 5!
386. (b) Suppose A, B, C are the events that all the 6! 5!
three fuses are defective ∴ P(all girls together) =
10!
5 B 4 C 3 5!
P (A) , P , P =
20 A 19 A B 18 10 9 8 7
5 4 3 2 1
5 4 3 1 =
P(A B C) = = 10 9 8 7
20 19 18 114 1
=
387. (a) The first seat can be filled in 10 ways, 42
second seat in 9 ways, the third in 8 ways,
393. (a) The word MATHEMATICS contains
and the fourth seat in 7 ways.
2 – M, 2 – A, 2 – T, 1 – A, 1 – E, 1 – T,
Number of arrangements of 10 people taken
4 at a time = 10 9 8 7 1 –C and 1–S.
 Total number of arrangements
= 5040

6 . 219
Basic Numeracy and Data Interpretation

398. (d) Let A be the event of choosing an orange


11!
= = 4989600 and a guava
2!2!2! 1!1!1!1!1!
Then n(S) = 21C2 = 210
Now consider 2 – M as one block n(A) = 12 × 9 = 108
 No. of favourable cases 108
 P(A) = = 0.51
10! 210
= = 907200 399. (d) Suppose x be an integer, whose sum of the
1!2!2!1!1!1!1!1!
digits square is 33.
907200 2
 Required probability = =  3 divides x2 since 3 is prime.
4989600 11  3 divides x
394. (a) P (Husband) =
4  9 divides x2
5 Any number is divisible by 9 if its sum is
2 divisible by 9. But 33 is not divisible by 9.
P (Wife) =
5 Hence, probability cannot be found.
P (both are selected) = P(H) P(W) 400. (b) No. of ways in which two good oranges can
4 2 8 be taken out is 6 C2 = 15
= =
5 5 25 No. of ways that at least one orange selected
is good = 6 × 9 = 54
9 white 6 white
395. (d) 3 black 10 black 15 5
Required probability = =
54 18
Let W1 and B1 are the events of transferring
401. (a) P(at least one is alive)
a white ball and a black ball from bag I to
@UPSC_THOUGHTS

bag II respectively. = 1 – P(both be dead)


Let W2 be the event of drawing a white ball 3 1 15
from bag II after the transfer of one ball. = 1– =
8 6 16
 Required probability = P(W2) 402. (b) No. of skilled and in favour = 800
P(W2) = P(W1W2) or P(B1W2) 800
= P(W1W2) + P(B1W2) so probability is equal to = 0.364
2200
W2 W2 403. (d) P(C and D) = P( A and B and C and D)
= P(W1)P W P(B1)P
B1
1
= 0.7 × 0.6 × 0.6 × 0.8
9 6 1 3 6 = 0.2016
=
9 3 (6 1) 10 9 3 6 (10 1)
404. (b) 2 mathematicians out of 5 can be selected
9 7 3 6 27
= = = 5C2
12 17 12 17 68
2 physicist out of 6 can be selected
396. (b) No. of floors where persons can leave the
cabin = 8 – 1 = 7 = 6 C2
Total number of ways for 5 persons to leave Total number of possible ways
the cabin= 7 × 7 × 7 × 7 × 7 = 75 = 5 C 2 6 C 2 = 10×15 = 150
Total number of ways for 5 persons to leave
the cabin at different floors 1 5 1 8
405. (b) Required probability =
= 7×6×5×4×3 = 7 P5 2 8 2 18
7
P5 5 2 77
 Required probability =
75 =
16 9
=
144
397. (a) Prime numbers less than 20 are
2, 3, 5, 7, 11, 13, 17, 19 406. (d) A non-leap year contains 365 days which has
52 weeks and one day more. This one day
 n(A) = 8, n(S) = 52
8 2 may be one of the seven days of a week
 P(A) = 52 = 13 in which there is only one sunday.

6 . 220
Basic Numeracy and Data Interpretation

Thus n(S) = 7 412. (c) Ways of event occurring Probability of


Number of favourable cases = 1 occurrence
Required probability 6 5 4 3 15
(i)
14 13 12 11 1001
No. of favourable cases
= 8 7 6 5 70
total no. of cases
1 (ii)
14 13 12 11 1001
=
7
6 5 8 7 70
407. (d) Total number of ways of putting the letters (iii)
14 13 12 11 1001
in the envelope is
5! = 120 ways 6 8 5 7 70
(iv)
The favourable case = 1 14 13 12 11 1001
1
The required probability = 6 8 7 5 70
120 (v)
408. (c) Sample space S = {D,E,V,E,L,O,P,M,E,N,T} 14 13 12 11 1001

 n(S) = 11 8 6 5 7 70
(vi)
Event A = {E,E,O,E} 14 13 12 11 1001
 n(E) = 4 8 6 7 5 70
(vii)
n(E ) 4 14 13 12 11 1001
The required probability is =
n(S) 11
8 7 6 5 70
409. (b) Probability of happening (p) = 0.4 (viii)
14 13 12 11 1001
@UPSC_THOUGHTS

∴ Probability of not happening (q) = 1 – 0.4 = 0.6


Probability of the event that the product of
Probability (event does not happen)
four numbers is a positive number is
= q × q × q = (0.6)3 = 0.216
Probability (event A happens at least once) 15 70 505
7
= 1 – 0.216 = 0.784 1001 1001 1001
413. (c) Total number of balls = 15
410. (c) Total number of outcomes that would occur
The probability of drawing 4 red balls in the
by flipping six coins = 26 = 64
first draw is given as P(E1)
1
Probability that no tail occurs =
64 No. of cases favourable to E1
P( E1) =
∴ Probability of at least one tail occurring Total number of cases
1 63 6
= 1 – C
64 64 = 15 4
1 C4
411. (d) Probability that bag A is drawn =
2 Probability of drawing 4 black balls in the
Probability that white ball is drawn from bag second draw is given as P(E2)
1 3 3 9
C
A = P(E2) = 15 4
2 5 10
C4
1 The required probability = P(E1 E2)
Probability that bag B is drawn =
2
= P(E1) . P(E2)
Probability that white ball is drawn from bag
6 9
1 2 1 C4 C4
B = =
15 15
2 6 6 C4 C4
Probability that white ball is drawn either from 6! 9!
bag A or from bag B 2! 4! 5! 4!
=
15! 15!
3 1 7
= 11! 4! 11! 4!
10 6 15

6 . 221
Basic Numeracy and Data Interpretation

418. (c) Here a = 1, d = 4 – 1 = 3


6! 11! 4! 9! 11! 4!
=
2! 4! 15! 5! 4! 15! n = 25
6 25
= S25 = [2 × 1 + (25 – 1) 3]
13 5 7 13 2
25
6 = (2 + 72)
= = 0.001 2
5915
3 25
414. (d) The prob. of Amar hitting a target = = × 74
5 2
2
The prob. of Akbar hitting a target = = 925
5
3 419. (b) First number between 2 and 101 which is
The prob. of Anthony hitting a target =
4 divisible by 5 is 5
Two shots can be hit in the following ways:
Similarly last number is 100
Let E1 be the event where Amar and Akbar
hit and Anthony fails ∴ A.P. is 5, 10, 15 ... 100
E2 be the event where Amar and Anthony hit a= 5, d = 5, tn = 100
and Akbar fails and tn = 100 = a + (n – 1) d
E3 be the event where Akbar and Anthony hit = 5 + (n–1) 5 = 100
and Amar fails 5 + 5n–5 = 100

6 5 n = 100
3 2 3 3 2 1
∴ P(E1) = 1 =
5 5 4 5 5 4 100 100
@UPSC_THOUGHTS

n= = 20
5
3 3 2 3 3 3 27
P(E2) = 1 = n
5 4 5 5 4 5 100 Sn (a l )
2
2 3 3 2 3 2 12
P(E3) = 1 = 20
5 4 5 5 4 5 100 S20 (5 100)
2
6 27 12
Required probability = = 10 × 105
100 100 100
45 = 1050
= 0.45
100
415. (d) 3 marbles are taken out of 12 in 12C3 ways. 420. (a) Smallest two-digit number divisible by 3 is
1 white marble out of 4 can be taken out 12 and largest is 99
in 4C1 ways. A.P. is 12, 15, 18, ... , 99
2 red marbles tan to taken out of 5 in 5C2 a = 12, d = 15 – 12 = 3
ways.
tn = 99
∴ Prob. of drawing 1 white and 2 red marbles
4C 5C tn = 99 = a + (n–1) d
1 2 2
= 12 + (n–1) 3 = 99
12C 11
3 12 + 3n – 3 = 99
n2 3n = 99 – 9
416. (a) an
3n 2
3n = 90
(5)2 25
3 5 2 17 n = 30
417. (b) Here a = 5, d = –3 n
Sn (a l )
tn = a + (n – 1) d 2
t11 = a + (11 – 1) d 30
= (12 + 99)
= a + 10 d 2
= 5 + 10 (–3) = 15 × 111
= 5 – 30 = –25 = 1665

6 . 222
Basic Numeracy and Data Interpretation

n 1 1 1 1 1
1 1 1 1 1 1 ...
421. (b) tn 3 425. (d)
3 243 2 4 8 16
1 33
33 n /2 3 5 So next term = 1 +
32
=
32
1
2
(n – 3) = 5 426. (c) 3 1 1 1 ... is in G.P.
3 9 1
n = 13 Where a = 3 and r =
3
1
422. (b) Three months is of a year. So the ages 3 3 9
4 ∴ S
1 1 1 1 4
are 7 years, 7 years, 7 years and so 1 1
4 2 3 3
on. 427. (d) Sn = 102 + 108 + 114 + ... + 498
The ages are in arithmetic progression with the If there are n terms in Sn then
1 498 = 102 + (n – 1) 6
first term a = 7, difference d = and the total
4 n = 67
of ages, Sn = 250. We have to find n, the
1
number of members. ∴ Sn = (67) (102 + 498) = 20100
2
n 428. (b) Let the total numbers in the series be n
Sn =
2
[2a + (n - 1) d ] a = 148, d = 146 – 148 = –2
n é 1ù Sn
125
Þ ê2 ´ 7 + (n - 1) 4 ú n
2 ë û
⇒ Sn = 125n
@UPSC_THOUGHTS

é (n - 1) ù n
Þ n ê14 + 4 ú = 250 × 2 = 500 Sn =
2
[2a +(n –1) d] = 125n
ë û
n
Þ n [56 + (n – 1)] = 500 × 4 = 2000 ⇒ [2 × 148 + (n –1) (–2)] = 125n
2
Þ n (n + 55) = 2000 n
2
Þ n + 55n – 2000 = 0 ⇒ [296 – 2n + 2] = 125n
2
2
Þ n + 80n – 25n – 2000 = 0 n
⇒ [298 – 2n] = 125n
(breaking up 55 into factors of 2000) 2
Þ (n + 80) (n – 25) = 0 ⇒ 149n – n2 = 125n
Þ n = 25 ( n cannot be – 80) ⇒ n2 = 24n ⇒ n (n–24) = 0

The number of members in the dramatic ⇒ n = 24 ( n 0)


society is 25. 429. (a) This is an arithmetic progression with the first
423. (b) S = 2 + 7 + 14 + 23 + ... + t99 ... (i) term a = 1, difference d = 1 and n = 12
S = 0 + 2 + 7 + 14 + 23 + ... +t98 + t99 ... (ii) (as after 12 o’clock the rings will be repeated
Subtracting (i) from (ii), we get 1, 2, 3 …) and last term l = 12
0 = [2 + 5 + 7 + 9 + ... + 99 terms] –t99 Total number of rings = 2 (1 + 2 + 3 + … + 12)
n
t99 = 2 + [5 + 7 + 9 + ... + 98 terms] Now (1 + 2 + 3 + … + 12) = (a + l )
2
1 12
= 2 + (98) (2 × 5 + 97 × 2) = (1 + 12) = 6 × 13 = 78
2 2
= 9998 Total rings = 2 × 78 = 156
1 1 1
424. (a) a x 430. (c) This is a case of geometric progression.
by cz k
Let the growth be 1000, 2000, 4000, … 512000
a = k , b = ky, c = kz
x
Here a = 1000, r = 2, and tn = 512000
Now a, b, c are in G.P.
b2 = ac tn = ar n–1 Þ 1000 × 2n–1 = 512000
k 2 y = kx+z 2n–1 = 512 = 29
n–1 = 9
2y = x + z
Time taken is 2 × 9 = 18 decades
x, y, z are in A.P.

6 . 223
Basic Numeracy and Data Interpretation

434. (a) Perimeter of the rectangle = 48m


431. (a) D C
2 (a + b) = 48
m 7c a + b = 24 m … (i)
7c m
O1 O2 Area of rectangle = 135 m2
E
ab = 135 … (ii)
A B From (i) and (ii) we get
135
a + = 24
Here AB = O1O2 = O1E + O2E = 7 + 7 a
2
a – 24a + 135 = 0
= 14 cm
a2 – 15a – 9a + 135 = 0
∴ Area of square ABCD = (14)2 = 196 sq cm
a(a – 15) –9 (a – 15) = 0
Also, sum of areas of semi-circles AED and
(a – 9) (a – 15) = 0
BEC = Area of a circle of radius 7 cm
22 a = 9, 15
= π (7)2 = × 7 × 7 = 154 sq cm 135
7 Hence b = = 15, 9
Hence, area of shaded portion = 196 – 154 a
∴ The sides of the rectangle are 15 m and 9 m.
= 42 sq cm
435. (c) Area of rectangle = Area of square (Given)
432. (b) D C ab = a2
E H
5
1 cm 2 cm × b = 25
2cm 2
G 25 2
F
@UPSC_THOUGHTS

B b =
A 4 cm 5
b = 10 cm
Area of rectangle ABCD = AB × BC
Hence, the other side of the rectangle is 10 cm
= 4 × 2 = 8 sq cm
436. (b) Area of rectangular plate = 4 (Area of one
Area of rectangle EFGH = EF × FG square)
= 1 × 2 = 2 sq cm 9 × 4 = 4 (a)2
∴ Area of shaded portion = (8 – 2) sq cm 9 = a2
= 6 sq cm ∴ a = 3
433. (c) A Hence, the side of each such square is 3 cm.
437. (a) Let the length and breadth of a rectangular
x field be 3x and x.
Area of field = ab
B C 7500 = 3x × x
x
7500
= x2
Let x cm be the side of a right-angled 3
isosceles triangle ABC. ∴ x = 50
1 1 Perimeter of rectangular field = 2 (a + b)
Given that, AB × BC = x2 = 200
2 2
= 2 (3 × 50 + 50)
x2 = 400 sq cm
= 2 (150 + 50)
From ΔABC, AC2 = x2 + x2 = 2x2
= 400m
Hence, area of a square drawn by hypotenuse
AC = 2x2 Cost of fencing is 25 paise per metre
= 2 × 400 Cost of fencing 400 m is 400 × 0.25
= 800 sq cm = Rs 100

6 . 224
Basic Numeracy and Data Interpretation

442. (d) Let the side of square be x and that of


438. (b) triangle be y
Path
∴ 4x = 3y
105m 3
x = y
4
3 2
Area of triangle = y
Area of circle = π r 2 4
22 3 2
= × 105 × 105 16 3 = y
7 4
= 34650 sq m y =8
Area of path = 4466 sq m 3
and x = 8 = 6 cm
∴ Area of inner circle = 34650 – 4466 4
π r 2 = 30184 sq m
443. (b) l 2 b2 = 41
2 30184
r = sq m
or l 2 + b2 = 41. Also, lb = 20
r2 = 9604 (l + b)2 = (l 2 + b 2) + 2lb
r = 98 = 41 + 40 = 81
⇒ (l + b) = 9
Radius of inner circle is 98 m
∴ Perimeter = 2 (l + b) = 18 cm.
Hence, width of path is 105 – 98 = 7 m
444. (d) Let original length = x and original breadth
439. (c) Area of circle = πr2
@UPSC_THOUGHTS

= y.
13.86 = π r2
80 90
Decrease in area = xy – x y
4.41 = r2 100 100
r = 2.1 18 7
= xy
xy = xy
22 25 25
Perimeter of circle = 2πr = 2 × × 2.1
7 7 1
= 13.2 ∴ Decrease% = xy 100 % 28%
25 xy
= 1320 m 445. (a) Find H.C.F. for the tile. Note the units are
The cost of fencing is = 1320 × 0.60 in metre and cm.
Length of the largest tile
= Rs 792
= H.C.F. of 1517 cm and 902 cm
440. (b) Distance covered by wheel in one revolution = 41 cm
= 2πr Area of each tile = (41 × 41) cm2.
56
= 2 1517 902
2 ∴ Require number of tiles = = 814
= 176 cm = 0.00176 km 41 41
The distance of 1.1 km is covered by wheel 446. (d) 100 cm is read as 102 cm.
in ∴ A1 = (100 × 100) cm2
and A2 = (102 × 102) cm2
1.1
= 625 revolutions (A2 – A1) = [(102)2 – (100)2]
0.00176
= (102 + 100) × (102 – 100) = 404 cm2
1 a
441. (b) Radius of circle = 2a [Remember algebraic formulae of a2 – b2
2 2
= (a + b) (a – b), etc.]
a a
∴ Area of circle = 404
2 2 ∴ Percentage error = 100 %
2 100 100
a
= = 4.04%
2

6 . 225
Basic Numeracy and Data Interpretation

447. (b) a = 3 cm, b = r cm and c = 5 cm 452. (c) 2(15 + 12) × h = 2(15 × 12) or
It is a right-angled triangle with base = 3 cm 180 20
h = m = m
and height = 4 cm. 27 3
1
∴ Its area = 3 4 cm2 = 6 cm2 20
2 ∴ Volume = 15 12 m3 = 1200 m3
1 3
Area of required triangle = 6 cm2
4 18 18 18
453. (b) Number of cubes = = 216
3 8 3 3
= cm2
2
454. (b) Let the sides of the three cubes be 3x, 4x
448. (d) Let the altitude of the triangle be h1 and the
and 5x
common base be b.
Then, Volume of the new cube
1
Then, × b × h1 = b × h2, where h2 = 100 m = [(3x)3 + (4x)3 + (5x)3]
2 = 216x3
⇒ h1 = 2h2 = (2 × 100) m = 200 m Edge of the new cube = (216x3)1/3 = 6x

1 Diagonal of the new cube = 6 3x


449. (c) Area of field = (5x 3x ) 24 m2
2 ∴ 6 3x = 12 3
= (96x) m 2 ⇒ x = 2
So, the sides of the cubes are 6 cm, 8 cm
1440
⇒ 96x = 1440 ⇔ x = ⇔ x = 15 and 10 cm.
96
Hence, the length of longer parallel side 455. (d) Let their heights be h and 2h and radii be
@UPSC_THOUGHTS

r and R respectively. Then we have


= (5x) = 75 m
r2 2h 2
450. (c) Length of each side of the square π r2h = π R2 (2h) ⇒
R2 h 1
= 81 cm = 9 cm r 2

Length of wire = (9 × 4) cm = 36 cm R 1
Perimeter of semicircle will be diameter i.e. 2 : 1
(= 2R ) + perimeter of half (2πR) circle.
456. (a) Let the length of the wire be h.
or πR + 2R = 36 ⇔ (π + 2)R = 36
1 1
36 Radius = mm = cm
⇒ R = = 7 cm 2 20
22 (Be careful of the units of measurements)
2
7 Now we have
1 2 22 1 1
Area of Semi-circle = r h = 66
2 7 20 20
1 22 66 20 20 7
= 7 7 ⇒ h = = 8400 cm = 84 m
2 7 22
457. (a) We have r = 3 cm and h = 4 cm
= 77 cm2
1 2 1
∴ Volume = r h = 32 4 cm3
451. (b) Let the dimensions of the cuboid be x, 2x 3 3
and 3x. = 12π cm2
Then, 2(x × 2x + 2x × 3x + x × 3x) = 88 458. (b) The volume of the largest cone
⇒ 2x2 + 6x2 + 3x2 = 44 = Volume of the cone with diameter of base
⇒ 11x2 = 44 7 cm and height 7 cm
⇒ x2 = 4 1 22
⇒ x = 2 = 3.5 3.5 7 cm3
3 7
∴ Volume of the cuboid = (2 × 4 × 6) cm3 269.5
= 48 cm3. = cm3 = 89.8 cm3
3

6 . 226
Basic Numeracy and Data Interpretation

465. (c)
459. (d) 4π (r + 2)2 – 4πr2 = 352
X C (Top)
45°
7 1 32.6°
or (r + 2)2 – r2 = 352 = 28
22 4
(Top) Cliff 100m
or (r + 2 + r) (r + 2 – r) = 28 T 32.6°
D
or 2r + 2 = 14 Tower a
14 45°
⇒ r = 1 = 6 cm B A
2 Draw TDCA
460. (c) Let the rise in the water level be h cm
Then CTD = XCT = 32.6°
4 CBA = XCB = 45°
Then, π × 4 × 4 × h = π × 3 × 3 × 3
3 AB
3 3 9 In CAB , = cot 45° (cot 45  1)
⇒ h = = cm AC
4 4 AB = AC × 1 = 100
DT = 100
461. (a) Let AB be the tree and AC be its shadow.
In CDT
B
Let ∠ ACB = θ. DT
= cot(32.6°)
AC DC
Then, = 3
AB DT = DC cot(32.6°)
⇒ cot θ = 3 100 = (AC – AD) cot(32.6°)
⇒ θ = 30° C A 100 = (100 – BT) cot32°36’
@UPSC_THOUGHTS

100 a
462. (c) Let AB be the tower. Then, ∠ APB = 30° 100 =
and AB = 100 m. tan 32 36’
B
AB 1 100 tan32°36 ’ = 100 – a
= tan 30° = a = 100 (1 – tan32°36’)
AP 3
100 m = 100 (1 – 0.6395)
⇒ AP = (AB × 3) = 100 × 0.3605
30°
= 100 3 m. C A = 36.05 m

= (100 × 1.73) m = 173 466. (d) logx 4 = 0.4


x x  20 D
463. (b) cot 60  , cot 30  logx 4 =
4
=
2
h h 10 5
20 h
 cot 30  cot 60  or x2/5 = 4
h
20 A
30° 60° or x = x5/2 = (22)5/2
h = 20 B x C 5
cot 30  cot 60
2
2
or x = 2 = 25
20
 3 or x = 32
1
3 = 20  10 3
3 2 1
log(8)1/2 log 8
log 8 2 1
PQ P 467. (c) = =
464. (c) In PQO = sin60° log8 log 8 log 8 2
OP
h 3
 1
150 2 150 h 468. (a) 2 log10 5 + log10 8 – log10 4
3 2
 h = 150 ×
2 = log10 (52) + log10 8 – log10 (41/2)
h = 75 3 O 60° Q
25 8
= 75 × 1.732 = log10 25 + log10 8 – log10 2 = log10
2
= 129.9m = log10 100 = 2.

6 . 227
Basic Numeracy and Data Interpretation

469. (d) There are an infinite number of multiples of If the sum is known, we can find the average:
7, so I is not sufficient to answer the 24
= 8
question; (a) is eliminated and so is (b). 3
Consider statement II: it narrows the range This is the average mentioned by the question,
of possibilities, and with the help of I could so I alone is sufficient. But we have to check
give the answer. If x is a multiple of 7 and II before option (b) can be eliminated.
lies between 590 and 575, we have to see
Statement II tells us that
how many numbers between the two are
multiples of 7. Divide 590 by 7; you get 84 2 (x + y + z) = 48
and remainder 2. So (590 – 2 =) 588 is a 48
Or x + y + z = = 24
multiple of 7. If you subtract 7 from 588, you 2
get another multiple of 7 : 83, which is also 24
Again, = 8
between 575 and 590. So there are two 3
multiples of 7 that can be x, and no definite So II alone is also sufficient
value of x is possible even with both Answer response (b) is correct.
statements together. The answer is (d).
473. (d) In statement I there is a quadratic equation,
470. (a) Statement I tells you that Mohan take twice
so we need to factorise and get our options
as long as Shyam to chop down trees. As
for x.
the question tells you the time taken by
x2 – 5x + 4 = 0
Shyam to cut down 4 trees, you can calculate
(x – 4) (x – 1) = 0
the time it would take Mohan to do the same.
∴ x could be either 4 or 1
So I is sufficient. However, you have to
@UPSC_THOUGHTS

check II before you can eliminate option (b). Statement I alone is not sufficient, as we
Statement II tells you how long. Mohan works have no single value for x. Options (a) and
every day at chopping down trees, but this (b) are eliminated. But we have to check II
gives you no data on time taken to cut down to see if (c) can be the answer.
4 trees. So II is not sufficient. Answer From statement II we can have many values
response (a) is correct. for x, as the majority of numbers are not
471. (b) As the given figures are parallelograms, there prime. And, as both 1 and 4 (what we
are only two angle measures—one acute and deduced from I) are not prime, we still have
the other obtuse. We know that the two no specific value for x. the statements
angles together add up to 180°. From together are not sufficient to answer the
statement I, we have r = 70. Angle q will question. Answer response (d) is correct.
be the same. So I alone is sufficient. 474. (c) Statement I is insufficient on its own as it
However, we have to check II before we can provides a three-variable equation which cannot
eliminate option (b). Statement II gives us the be solved. So (a) is eliminated; so is (b). We
measure of the obtuse angle which we can have now to check for (c). Let’s take both
subtract from 180° to get the value of the statements together. Add the two equations:
smaller angle, and we can calculate the value
I. 2a + b + 3c = 45
of q. So II is also sufficient on its own. The
II. + a + 2b = 30
correct answer is (b).
3a + 3b + 3c = 75
472. (b) We are asked only about the average and
not the individual values of x, y and z. We 3 (a + b + c) = 75
can find the average if we know the sum of 75
a + b + c = = 25
the three numbers. Statement I tells us that 3
3 (x + y + z) = 72.
So the two statements together can get you
72 the answer to the question. The correct
So x + y + z = = 24 answer response is (c).
3

6 . 228
Basic Numeracy and Data Interpretation

3 So the shop sold (2 × 150 =) 300 blue toy


475. (a) From I it is clear that x = 3 as 64 = 4
boats.
We have to check II if (b) is to be eliminated.
Now, we can add the information to what we
x2 – 2x + 8 have in I and get an answer to the question.
x2 – 2x – 8 = 0 As the shop sold 3 times as many blue toy
Factorising we get boats as red toy cars (from I), the number
(x – 4) (x + 2) = 0 300
of red toy cars sold is = 100.
We x can be either 4 or –2. As we get no 3
single value for x, statement II is not Subtract 100 from 450 to get total of red toy
sufficient. boats sold. So 350 red toy boats were sold.
476. (c) Draw up a table to solve the question. The A total of 350 + 300 (blue toy boats) add
data of the question can be filled in. up to 650 toy boats.
Answer response (c) is correct.
Red Blue Total
477. (d) Check statement I. Plug in some numbers
Toy boats for x divisible by 8 and see if all of them
Toy cars are divisible by 48. Actually, there are many
numbers divisible by 8 that are not divisible
Total 450 450 900 by 48. Take numbers less than 48, say 32,
24, 16 … all of which are divisible by 8
As half the toys were red, half will be blue. but not by 48. So I is not sufficient. By the
Now, let’s enter the data of statement I in same token, statement II is also insufficient.
@UPSC_THOUGHTS

the table as: And taken together, the statements solve


nothing. Don’t be misled by 8 × 6 being 48:
Red Blue Total
there are several numbers divisible by both
Toy boats 3x 8 and 6, but not by 48, such as 24, 72, 120.
So (d) is the correct answer response.
Toy cars x
478. (b) If there are 5 dolls, and the probability of
Total 450 450 900 picking one first is ½, it means
1 5
There are still two unknowns—red toy boats (taking x as total toys)
2 x
and blue toy cars which cannot be related.
So I is not sufficient. x = 10. So we can go on to the probability
of the second toy being a doll. After picking
We now fill in data from II in the table to one, we have 4 dolls and 9 toys left. So the
see if (c) can be an option. If 2x toy boats probability of selecting a doll a second time
were sold, half that or x toy cars were sold: is
Red Blue Total 1 4 2
or
2 9 9
Toy boats 2x Statement I is sufficient.
Toy cars x But we have to check II to see if (b) could
be an option.
Total 450 450 900
Statement II tells us the probability of picking
Though two unknowns remain here, too, we a doll for the second selection is 4/9. It
can get some useful information. implies that the probability of the first toy
If 2x + x = 450, picked being a doll is 5/10. So statement II
also can answer the question on its own.
450
x = = 150 Answer response (b) is correct.
3

6 . 229
Basic Numeracy and Data Interpretation

479. (c) Let the total investment be Rs x 5 2


Remaining work = 1
7 7
x
Then, R = 2
4 ∴ work was carried by A.
7
75 3x II. is irrelevant.
R + V = x
100 4
483. (c) I. If Y takes 4 min., then X takes 3 min.
3x x x 3
V = II. If Y takes 36 min., then X takes 36
4 4 2 4
min = 27 min.
x x x
∴ A = x – Thus, I and II together give the answer.
4 2 4
x x x 484. (d) Let the distance between the two stations be
R : A : V = : : = 1 : 1 : 2
4 4 2 x km.
Thus, both I and II are needed to get the I. Let the speed of the express train be y
answer. km/hr.
∴ Correct answer is (c). Then, speed of the mail train
480. (c) Let Ravi, Gagan and Nitin be R, G and N = (y + 12) km/hr.
respectively
x x 40
I. R : G : N = 2 : 4 : 7 II.
y (y 12) 60
II. N = 8750
From I and II, we get: Thus, even I and II together do not give x.
@UPSC_THOUGHTS

When N = 7, then G = 4. When N = 8750, 485. (b) Let AC = x km. Then, CB = (100 – x) km.
4
then G = 8750 = 5000. (100 – x)
7 A x C B
Thus, both I and II are needed to get the
I. AB = 125% of CB
answer. Correct answer is (c).
x 125
481. (c) I. Gives, Machine X produces candles in 100 = × (100 – x)
5 100
1 min.
x 100 100
II. Gives, Machines X and Y produce 100 – x = = 80
2 125
candles in 1 min.
x x 3x x = 20 km
From I and II, Y produces
2 5 10 ∴ AC = 20 km.
candles in 1 min.
3x Thus, I alone gives the answer.
candles are produced by Y in 1 min. 1 1
10
II. AC = CB x = (100 – x)
10 4 4
x candles will be produced by Y in x
3x 5x = 100 x = 20
10
min = min. ∴ AC = 20 km.
3
Thus, I and II both are necessary to get the Thus, II alone gives the answer.
answer. 486. (c) Area = 1600 m2
1
482. (a) B’s 1 day’s work = (A + B)’s 1 day’s work I. Side = 1600 m = 40 m. So, perimeter
20
1 = (40 × 4) m = 160 m.
= .
7 ∴ I alone gives the answer.
5 Total cost 3200
I. (A + B)’s 5 day’s work = .
6 II. Perimeter = = 160 m
Cost per metre 20

6 . 230
Basic Numeracy and Data Interpretation

∴ II alone gives the answer. 489. (c) I Gives, h = 28 m and r = 14 cm.


∴ Correct answer is (c) ∴ Capacity = πr 2h, which can be obtained.
Thus, I alone gives the answer.
487. (c) I. Area of the circle = Area of the square
1 II Gives, πr 2 = 616 m2 and h = 28 m
= x 2 sq. inches.
2 ∴ Capacity = (πr 2 × h) = (616 × 28) m3
1 2 Thus, II alone gives the answer.
r2 x
2
Correct answer is (c).
x2 x 490. (d) I Gives, any two of l, b, h are equal
r
2 2
II Gives, Ibh = 64
∴ Circumference of the circle = 2πr, which
can be obtained. From I and II, the values of l, b, h may
be (1, 1, 64), (2, 2, 16), (4, 4, 4)
∴ I alone gives the answer.
Thus, the block may be a cube or cuboid.
II. Area of the circle = Area of the square ∴ Correct answer is (d).
= y 2 sq. inches.
491. (c) Let xm2 be the area of the hall
2 2
r y Material cost of flooring per sq mt = 250
y ∴ For total area it would be 250 × x
r
Labour cost of flooring the hall = 3,500
∴ Circumference of the circle = 2 πr, which Total cost of flooring the hall = 14,500
can be obtained. Total cost = Material cost + Labour cost
@UPSC_THOUGHTS

Thus, II alone gives the answer 14500 = 250x + 3500


∴ Correct answer is (c). 250x = 11000
11000
2 R x = = 44 sq m
488. (c) I. Length of arc = 250
360
22 ∴ All the three statements are necessary
2 R 60 to calculate the answer
4 7
360 492. (d) Given statement (I)
Here selling price (S.P.) = Rs 252
Profit earned P1 = Rs 52
Let cost price (after discount) = C.P.1
O We know that
R o R S.P. = C.P. + Profit
90
252 = C.P.1 + 52
A B ∴ C.P.1 = 252 – 52 = Rs 200
By statement II,
When there is no discount. profit earned
This gives R and therefore, area of the
would be Rs 80
circle = πR2.
Let cost price without discount be C.P.2
Thus, I only gives the answer.
S.P. = C.P. + Profit
II. R2 + R2 = 52
252 = C.P2 + 80
2R2 = 25
C.P.2 = 252 – 80 = 172
25
or R2 = This cannot be possible since cost price
2
before discount should be more that cost
∴ Area of the circle = πR2
price after discount, i.e.,
22 25 C.P.2 should be > C.P.1
= sq. cm.
7 2 but here C.P.2 < C.P.1
Thus, II only gives the answer. (172 < 200)
Correct answer is (c). ∴ Statement (II) is not relevant.

6 . 231
Basic Numeracy and Data Interpretation

Now consider statement (III) train running in the same direction, distance
Let cost price without discount be C.P.3 covered = difference in lengths of both
Here without discount profit earned = 40% trains
S.P. = C.P. + Profit Here length of the other train is not given.
252 = C.P.3 + C.P.3 × 40% So statement III is not complete.
[ Profit percentage is always calculated on ∴ I and II only are necessary to answer the
cost price] questions.
100 C.P.3 + 40 C.P.3 Alternative Method
252 =
100
We know that distance covered by a train
25200 = 140 C.P.3
crossing a platform
25200
C.P.3 = = 180 = Length of train + Length of platform
140
Here again this is not possible since From statement II we get the length of
platform and time taken to cross it.
C.P.3 < C.P.1
From statement I we get
So statement III is also not relevant
Length of train
493. (a) Consider statement I = 13 sec
Speed of train
Distance(D)
= Time (T) Distance
Speed (S) [ Time = and distance covered by
Speed
Length of Train Lt a train crossing the pole = length of train]
= 13 sec ... (1)
@UPSC_THOUGHTS

Speed S
So it is clear that statements I and II only
[ When a train crosses a pole, Distance are necessary to solve the question
covered by train = Length of train]
Moreover statement III is not complete since
Consider statement II it does not give the length of the other train
Distance covered by a train crossing a 494. (d) Consider statement I
platform = Lt + Length of platform Let x be the population of state A
After increasing the population by 15%, we
D = Lt + 250 m get
Time taken = 27 sec x + 15% x = 1.61 lakh
D 100 x 15x
S = = 1.61 lakh
T 100

Lt 250 115x = 1.61 × 100 lakh


∴ S =
27
161
Lt 250 x = = 1.4 lakh
27 = 115
S
∴ Population of state A = 1.4 lakh
Lt 250
27 = Now from statement II,
S S
250 Ratio of population of state A to that of
27 = 13 + [ From (1)]
S B = 7 : 8
250 From statement III,
= 14
S
Population of state B = 1.6 lakh
250
S = = 17.86 m/s (approx) From II and III,
14
Now consider statement III, Population of state A(P) 7
Population of state B =
We know that when a train crosses another 8

6 . 232
Basic Numeracy and Data Interpretation

4 8 8 5 4
P 7 In 2 days, work is completed by
= 5 2 5
1.6 lakh 8
= 128 workers
7×1.6 lakh
P = ∴ Additional workers required to complete the
8
work in 10 days
P = 1.4 lakh
= (128 – 8) = 120 workers
∴ Statement I by itself or II and III
∴ Statement I can answer the question.
together are necessary for answering
the question. Now consider statement II

Alternative Method 20 workers can complete the work in 16 days


In 16 days, 1 work is completed by 20
Consider statement I.,
workers
Let x be population of State A then 1
x + 15%x = 1.61 lakh In 1 days, work is completed by 20
16
and x can be found workers
So statement I is necessary to find answer. 1 20
In 10 days, work is completed by
Now consider statement II 16 10
workers
Ratio of population of state A: state B = 7 : 8
In 10 days, 1 work is completed by
and population of state B (1.6 lakh) is given
in statement III 20
× 16 = 32 workers
From both statements II and III 10
∴ Statement II can answer the question
We get an equation
@UPSC_THOUGHTS

Now consider statement III,


7 Population of state A 1
= of work can be completed by 8 workers
8 1.6 lakh 8
in 5 days
and the population of state A can be easily
∴ Remaining days = 10 – 5 = 5 days
found. 1 7
Remaining work = 1 – =
∴ Either statement I or both II and III are 8 8
necessary to answer the question. 1
In 5 days, work is completed by 8 workers
8
495 (d) No. of days in which construction should be 1
completed = 10 days In 1 day, work is completed by 8 × 5
8
workers
Consider statement I,
In 1 day, 1 work is completed by 8 × 5 × 8
20% of work can be completed by 8 workers
workers
in 8 days.
8 5 8
In 5 days, 1 work is completed by
∴ Remaining days = 10 – 8 = 2 days 5
20 workers
Remaining work = 1 – 20% = 1 –
100 7
1 In 5 days, work is completed by
= 1 – 8
5
4 8 5 8 7
= work = 56 workers
5 5 8
1
In 8 days, work is completed by 8 workers ∴ Additional workers needed to complete the
5
1 work in 10 days
In 1 day, work is completed by 8 × 8
5
workers = 56 – 8 = 48 workers
In 1 days, 1 work is completed by 8 × 8 × 5 ∴ Statement III caan answer the problem.
workers
8 8 5 ∴ Any one of the three statement can answer
2 days, 1 work is completed by
2 the question.
workers

6 . 233
Basic Numeracy and Data Interpretation

496. (c) Let speed of train A be x mps.


We also know that when two trains cross
each other.
a h a Distance covered = Length of train A
+ Length of train B
Speed of train A + Speed of train B

b Length of train A + Length of train B


=
Time taken
Area of isoceles triangle
base Statement I gives us length of train B i.e.
= 4(side)2 (base)2
4 200 metres and time taken i.e. 20 seconds
b Statement II gives us Speed of train B i.e.
= 4(a )2 (b )2
4 60 kmphs
Statement I: Statement III says that length of train A
Perimenter = 14 meters
= 2 length of train B
2a + b = 14 (1)
Length of train A = 2 × 200 metres
Statement II:
= 400 metres
Base ‘b’ = 8 metres (2)
Putting these values in the formula
Putting b = 8 in (1), We get
We get
2a + 8 = 14
60 1000
2a = 14 – 8 Speed of train A + mps
@UPSC_THOUGHTS

3600
6 400 200
a = = 3 (3) =
3 20
∴ From statements I and II, area can be
50 600
determined x + =
3 20
Again consider statement II
50
base ‘b’ = 8 metres (Given) x + = 30
3
Consider statement III 3x + 50 = 90
height h = 5 metres (Given) 3x = 90 – 50
3x = 40
We know that
Area of a triangle 40
x =
3
1 40
= × base × height ∴ Speed of Train A = metre per second.
2 3
∴ We can find area from statement II and III.
All statements I, II and III are necessary to
Either statement I and II
answer the question
or statement II and III are sufficient to answer
the question. 498. (d) Cost of flooring a hall
497. (d) We know that when two trains are running = Area of the floor × Cost per square metre
in the opposite directions
Consider statment I
Their Relative Speed
Given: Perimeter of the rectangle = 60 metres
= Speed of first train
We know that
+ Speed of second train
Perimeter of a rectangle = 2(l + b)
Here relative speed
Where l is length and b is the breadth of the
= Speed of train A + Speed of train B rectangle
Also Relative speed of approach 60 = 2(l + b)
Distance covered 60
= l + b =
Time taken 2

6 . 234
Basic Numeracy and Data Interpretation

l + b = 30 Statement II
l = 30 – b Q earns more than M but not as much as N
Consider statement II M < Q < N
Given: Angle between diagonal and breadth Statement III
is 60° N earns more than M and R
This information does not provide sufficient N > M
data to calculate the area of the floor. N > R
Consider Statement III From the three statements we can say that
N and P earn more than any one else. But
Cost of flooring per square metre = Rs 125
among N and P who earns more cannot be
We know that determined.
Area of a rectangle = l × b (1) ∴ The answer cannot be determined even
Putting l = 30 – b in (1) we get with data in all three statements
Area = (30 – b)b 500. (d) To find: Price of one dozen oranges
= 30b – b 2 Let price of one dozen oranges be Rs C, price
But value of b is not given of one dozen apples be Rs A and price of
So cost of flooring cannot be found even with one dozen bananas be Rs B
data in all three statements Consider statement I
Alternative Method 2C + 3B = 110 ... (1)
Consider statement II
D C
3A + B = 170 ... (2)
@UPSC_THOUGHTS

60° Consider statment III


C + A + B = 95 ... (3)
Solving (2) and (3) we get,
3A + B = 170 ... (2)
A B C + A + B = 95 ... (3)
Multiplying (3) by 3 we get
We know that 3C + 3A + 3B = 285 ... (3)
Cost of flooring rectangular hall 3A + B = 170 ... (2)
= Area of hall × Cost per sq. metre Subtracting (2) from (3) we get
Area = length × breadth 3C + 2B = 115 ... (4)
Now Consider
= l × b
3C + 2B = 115 ... (4)
Statement I says perimeter = 60 metres
2C + 3B = 110 ... (3)
We know that Multiplying (4) by 3 and (3) by 2
Perimeter of a rectangle = 2(l + b) We get
60 = 2 (l + b) (4) × 3 9C + 6B = 345 ... (4)
Statement II gives us the angle between (3) × 2 4C + 6B = 220 ... (3)
diagonal and breadth which is not sufficient
Subtracting (3) from (4) we get
to find area
9C + 6B = 345
Statement III gives cost of flooring per sq.
4C + 6B = 220
metre – – –
But none of the statements gives us value 5C = 125
of l or b
125
So question cannot be answered even with C =
data in all three statements. 5
C = 25
499. (a) Statement I
∴ Cost of one dozen oranges = Rs 25
M earns less than P but not less than R
All I, II and III are required to answer the
R M < P question

6 . 235
Basic Numeracy and Data Interpretation

Alternative Method
240 1 240
Let price of one dozen oranges be Rs C women’s 1
7 60 7
Let price of one dozen apples be Rs A day’s work = 1.
Let price of one dozen bananas be Rs B
240
From statement I, we get women’s 1 day’s work
7
2C + 3B = 110 ... (1)
4 3
From statement II, we get = 1
7 7
3A + B = 170 ... (2)
10 women’s 1 day’s work
From statement III, we get
3 7 1
C + A + B = 95 ... (3) = 10
7 240 8
We can see that there are three unknown
variables A, B, C, and three equations (1), So, 10 women can finish the work in 8
(2), (3) days.
∴ Value of A, B, C, can be determined. III. (10 men’s work for 3 days) + (10 women’s
work for 4 days) = 1
All I, II and III are required to answer the
question. (10 × 3) men’s 1 day’s work + (10 × 4)
women’s 1 day’s work = 1
501. (d) From I, II and III, we get P : Q : R
30 men’s 1 day’s work + 40 women’s
= (120000 × 8) : (80000 × 12) : (x × 9) 1 day’s work = 1.
Since R’s investment is not given, the above Thus, I and III will give us the answer.
@UPSC_THOUGHTS

ratio cannot be given


And, II and III will give us the answer.
Given data is inadequate. Correct answer response is (a).

502. (b) I and II give, profit after 3 years 504. (d) Clearly, I only gives the answer
Similarly, II only gives the answer
3
= Rs 22000 = Rs 8250 And, III only gives the answer
8
∴ Correct answer is (d).
From III also, profit after 3 years
= Rs (2750 × 3) = Rs 8250 505. (b) II. Part of the tank filled by A in 1 hour
1
=
5 4
∴ P’s share = Rs 8250 = Rs. 3750
11 III. Part of the tank filled by B in 1 hour
1
Thus, (either III is redundant) or (I and II are =
6
redundant).
1 1 5
(A + B)’s 1 hour’s work =
503. (d) I. (10 × 6) men can complete the work in 4 6 12
1 day.
∴ When both A and B are opened together,
1 5
1 man’s 1 day’s work = they will fill the tank in hrs
60 12
24 24 = 2 hrs 24 min.
II. 10 men + 10 women can
7 7 So, II and III are needed
complete the work in 1 day.
∴ Correct answer is (b).
240 240 506. (b) I. p2 + 24 = 10p (Given)
men’s 1 day work +
7 7 ⇒ 2
p – 10p + 24 = 0
women’s 1 day work = 1
⇒ p2 – 4p – 6p + 24 = 0

6 . 236
Basic Numeracy and Data Interpretation

Splitting the middle term ⇒ (p – 1) (p + 3) = 0


⇒ p(p – 4) –6 (p – 4) = 0 ⇒ (p – 1) = 0; (p + 3) = 0
⇒ (p – 4) (p – 6) = 0 ⇒ p = 1 ; p = –3 ... (1)
⇒ (p – 4) = 0: (p – 6) = 0 II. 2q – 7q + 6 = 02
(Given)
⇒ p = 4; p = 6 ... (1) ⇒ 2q2 – 4q – 3q + 6 = 0
II. 2q2 + 18 = 12q (Given) [∴ splitting the middle term]
⇒ 2
2q – 12q + 18 = 0 ⇒ 2q(q – 2) –3 (q – 2) = 0
Simplifying we get ⇒ (q – 2) (2q – 3) = 0
q2 – 6q + 9 = 0 ⇒ (q – 2) = 0; (2q – 3) = 0
⇒ 2
q – 3q – 3q + 9 = 0 3
⇒ q = 2; q = ... (2)
Splitting the middle term 2
⇒ q(q – 3) –3 (q – 3) = 0 From (1) and (2) we can see
⇒ (q – 3) (q – 3) = 0
3
⇒ (q – 3)2 = 0 When p = 1 or –3, q = 2 or
2
⇒ q =3 ... (2)
From (1) and (2) we can see that ⇒ q>p
when p = 4 or 6, q = 3 Alternative Method
⇒ p>q I. p2 + 2p – 3 = 0
Alternative Method 2 4 4 ( 3) (1)
We know that p
2
Roots of a quadratic equation
2 4 12
@UPSC_THOUGHTS

p
b b2 4ac 2
=
2a 2 4
p=
I. p2 + 24 – 10p = 0 2
2 4 2 4
p2 – 10p + 24 =0 p= ;
2 2
10 (10)2 4(1)(24) p = 1 ; –3 ... (1)
p=
2 1 II. 2q2 – 7q + 6 = 0
10 100 96 10 2
p= = 7 (7)2 4(2)(6)
2 2 q=
2 2
10 2 10 2
p= ; 7 49 48 7 1
2 2 q= =
4 4
p = 6; 4 ... (1) 7 1 7 1
q= ;
4 4
2
II. 2q + 12q – 18 = 0 3
q = 2 ; ... (2)
q2 – 6q + 9 = 0 2
From (1) and (2), q > p
6 (6)2 4(1)(q )
q= 508. (a) I. p2 + 16 = 8p (Given)
2 1
⇒ 2
p – 8p + 16 = 0
6 36 36
q= ⇒p2 – 4p – 4p + 16 =0
2 (∴ splitting the middle term)
6
q= =3 ... (2) ⇒ p(p – 4) – 4 (p – 4) = 0
2
⇒ (p – 4) (p – 4) = 0
From (1) and (2), p > q
⇒ (p – 4)2 = 0
2
507. (c) I. p + 2p – 3= 0 (Given) ⇒ p – 4= 0
⇒ 2
p – p + 3p – 3 =0 ⇒ p= 4 (1)
[∴ splitting the middle term] II. 4q2 + 64 = 32q (Given)
⇒ p(p – 1) + 3 (p – 1) =0 ⇒ 2
4q – 32q + 64 = 0

6 . 237
Basic Numeracy and Data Interpretation

Simplifying, we get Alternative Method


⇒ q2 – 8q + 16 = 0 I. 2p2 + 12p + 16 = 0
⇒ q – 4q – 4q + 16 = 0
2
p2 + 6p + 8 = 0
Splitting the middle term
6 36 4 1 8
⇒ q(q – 4) – 4(q – 4) = 0 p=
⇒ (q – 4) (q – 4) = 0 2 1
⇒ (q – 4)2 = 0 6 36 32
p=
⇒ (q – 4) = 0 2
⇒ q= 4 (2) p=
6 2
∴ From (1) and (2) we can see that when 2
6 2 6 2
p = 4, q = 4 p= ;
⇒ p= q 2 2
p = –2 ; – 4 ... (1)
Alternative Method
2
II. 2q + 14q + 24 = 0
I. p2 – 8p + 16 = 0
⇒ q2 + 7q + 12 = 0
8 (64) 4 (16)
p= 7 49 4 12
2 1 q=
8 0 2
p=
2 7 1
q=
8 2
p= =4 ... (1)
2 7 1 7 1
q= ;
II. 2
4q – 32q + 64 = 0 2 2
⇒ q2 – 8q + 16 = 0 q=–3 ; –4 ... (2)
@UPSC_THOUGHTS

This is the same equation as above From (1) and (2), p ≥ q


∴ q= 4 ... (2)
510. (b) Given expenditure of company B
⇒ p= q
in 1996 = Rs 12 lakh
509. (d) I. 2p2 + 12p + 16 = 0 (Given) in 1997 = Rs 14.5 lakh
Simplifying, we get Profit earned by compnay B
p2 + 6p + 8 =0 in 1996 = 35%
⇒ 2
p + 2p + 4p + 8 =0 in 1997 = 50%
[ splitting the middle term] We know that
⇒ p(p + 2) + 4(p + 2) =0 Percentage profit
⇒ (p + 2) (p + 4) =0
⇒ (p + 2) = 0 ; (p + 4) =0 Income Expenditure
= ×100 ... (1)
⇒ p = –2 ; p = – 4 ... (1) Expenditure

II. 2
2q + 14q + 24 = 0 (Given) For on 1996
Simplifying, we get Putting the amount of expenditure and per
⇒ q2 + 7q + 12 = 0 centage of profit, we get
⇒ 2
q + 3q + 4q + 12 = 0 Income 12
35 = ×100
[ Splitting the middle terms] 12
35 ×12
⇒ q(q + 3) + 4 (q + 3) = 0 Income – 12 =
100
⇒ (q + 3) (q + 4) = 0
420
⇒ (q + 3) = 0 ; (q + 4) = 0 Income = 12
100
⇒ q=–3 ; q=–4 ...(2) = 4.2 + 12
∴ From (1) and (2) we can see that = Rs 16.2 lakh
When p = – 4, q = –4 For 1997
⇒ p = q Putting the amount of expenditure and percent-
also when p = – 2, q =–3 age profit in (1) we get
⇒ p>q Income – 14.5
∴ p≥q 50 = 100
14.5

6 . 238
Basic Numeracy and Data Interpretation

50 14.5 589. (a) 160 x


Income – 14.5 = ⇒ Income = 4x
100 100
725 160 x 400 x
Income = + 14.5 ⇒ Income of B =
100 100
= 7.25 + 14.5 560 x
=
100
= Rs 21.75 lakh
∴ Ratio of incomes of A and B in 1999
∴ Total Income = (Rs 16.2 + Rs 21.75) lakh 390x
= Rs 37.95 lakh 100
=
560x
Alternative Method
100
We know that
390 x
⇒ ⇒ 39 : 56
Expenditure (100 + Percent profit) 560x
Income =
100 Alternative Method
Required Income Expenditure (100+ Percent Profit)
= Income of 1996 + Income of 1997 Income =
100
12 (100 35) 14.5(100 50) Given ratio of expenditure of A and B = 3 : 4
=
100 100 Let x be the common variable
1620 2175 ∴ Expenditure of A = Rs 3x
=
100 Expenditure of B = Rs 4x
= Rs 37.95 lakh ∴ Required ratio is
3x (100 30)
@UPSC_THOUGHTS

511. (d) Ratio of expenditures of company Income of A 100


=
A and B in 1999 = 3 : 4 Income of B 4 x (100 40)
Let x be the common variable 100
∴ Expenditure of A in 1999 = 3x 3 130
=
Expenditure of B in 1999 = 4x 4 140
Profit in 1999 (from the graph) 39
Company A = 30% =
56
Company B = 40% ∴ Required ratio = 39 : 56
We know that
512. (a) Let Rs x lakh be the expenditures of both
Percentage profit =
the companies in 2000.
Income Expenditure
= ×100 ... (1) Total income of both the companies
Expenditure
= Rs 5.7 lakh
Putting the values for company A in ... (1) Let income of company A be I
Income – 3x ∴ Income of company B would be
30 = 100
3x Rs 5.7 lakh – I
90 x We know that
⇒ = Income – 3x
100 Percentage profit/loss
90 x Income – Expenditure
⇒ Income = 3x = ×100 ... (1)
100 Expenditure
90 x 300 x Profit for company A in 2000
⇒ Income =
100 = 40%
390 x Putting the values in (1) we get
⇒ Income of A =
100 I –x
40 = ×100
Putting the values for company B in (1) x
40 I–x
Income – 4x =
⇒ 40 = ×100 100 x
4x
160 x 40 I
⇒ = Income – 4x = –1
100 100 x

6 . 239
Basic Numeracy and Data Interpretation

I 40 From equation (1)


= +1
x 100 1.4x + 1.45x = 5.7
I 5.7
40+100 x= =2
= 2.85
x 100
I 140 ∴ Total expenditure of A and B
=
x 100 = Rs (2 + 2) lakh
I = 1.4x ... (2) = Rs 4 lakh
Now profit for company B in 2000 = 45%
Putting the values in (1) we get 513. (c) The ratio of incomes of company B in 1997
and 1998 = 2 : 3
(5.7 I) x
45 = × 100 Let expenditure for 1997 be E1 and for 1998 be
x
E2
45 5.7 I x
= – Let x be the common variable
100 x x
∴ Income of B in 1997 = Rs 2x
45 5.7 I
= –1 and Income of B in 1998 = Rs 3x
100 x
Percentage profit of B:
5.7 I 45 in 1997 = 50
= 1
x 100 in 1998 = 45
5.7 I 45 100 We know that
= Percentage profit/loss
x 100
Income – Expenditure
= ×100 ... (1)
@UPSC_THOUGHTS

5.7 I 145 Expenditure


=
x 100
For 1997
5.7 – I = 1.45x
putting the respective values in (1), we get
I = 5.7 – 1.45x ... (3)
From (2) and (3) 2x Expenditure (E1 )
50 = 100
Expenditure (E1 )
1.4x = 5.7 – 1.45x
2x E1
1.4x + 1.45x = 5.7 50 = 100
E1
2.85x = 5.7 50 2x
= –1
x =2 100 E1
2x 50 100
⇒ Expenditure of company A = Expenditure of =
E1 100
company B = Rs 2 lakh
2x 150
∴ Total expenditure of two companies in 2000 = = 1.5
E1 100
= Rs 2 lakh + Rs 2 lakh = Rs 4 lakh
2x = 1.5E 1 ... (2)
Alternative Method For 1998
Let expenditure of A = B = Rs x putting the respective values of in (1), we get
Income of A = IA 3x E2
45 = 100
Income of B = IB E2
45 3x E2
IA + IB = Rs 5.7 lakh ... (1) =
100 E2
45 3x
Expenditure (100 + Profit percent = –1
Income = 100 E2
100
3x 45 100
=
x (140) E2 100
IA = = 1.4x
100 3x 145
x (145) =
E2 100
IB = = 1.45x
100 3x = 1.45E 2 ... (3)

6 . 240
Basic Numeracy and Data Interpretation

Dividing (2) by (3) we get Total no. of students


studying in 5 schools
2x 1.5 E1 =
= No.of schools
3x 1.45 E2
1.5 E1 Total no. of students studying in 5 schools in
2
⇒ = 1.45 E 1992
3 2
= Total no. of students admitted upto 1992
E1 2(1.45) – Total no. of students left upto 1992
⇒ =
E2 3(1.5) Total no. of students admitted in 5 schools upto
E1 2.9 29 1992
⇒ = = = 1025 + 950 + 1100 + 1500 + 1450
E2 4.5 45
+ 190 + 225 + 300 + 300 + 280 + 230
or E1 : E2 = 29 : 45
+ 350 + 320 + 340 + 250
Alternative Method = 8810
Total no. of students left upto 1992
Ratio of Incomes of B in 1997 and 1998
= 120 + 150 + 130 + 150 + 125
=2:3
+ 110 + 115 + 150 + 160 + 130
Expenditure (100 + per cent profit = 1340
Income =
100 ∴ Total no. of students studying in 5 schools in 1992
Let Expenditure in 1997 be E1 and = 8810 – 1340
Expenditure in 1997 be E2 = 7470
7470
@UPSC_THOUGHTS

E1(100 50) ∴ Required average =


∴ Income in ‘97 = 5
100
= 1.5 E1 = 1494

E2(100 45) 516. (d) Total no. of students who left the school C
Income in ‘98 =
100 from 1990 to 1995
= 1.45 E2 = 130 + 150 + 125 + 140 + 180
= 725
1.5 E1 2
⇒ = = Total no. of students admitted to school C
1.45 E2 3
from1990 to 1995
E1 2 1.45 2.9 29
∴ = = = = 1100 + 320 + 300 + 260 + 240 + 310
E2 3 1.5 4.5 45
= 2530
⇒ E1 : E2 = 29 : 45
725
Required percentage = × 100
514. (d) Total no. of students admitted at school B 2530
during 1991 to 1995 = 28.66%
= 350 + 225 + 185 + 200 + 240 or
29% (approx)
= 1200
Total no. of students admitted at school D dur- 517. (d) No. of students studying in school B in the
ing 1991 to 1995 year 1994,
= 340 + 300 + 295 + 320 + 360 = No. of students admitted till 1994
= 1615 – No. of students left till 1994
∴ Required difference = 1615 – 1200 = (950 + 350 + 225 + 185 + 200)
= 415 – (150 + 115 + 110 + 90)

515. (a) Average of no. of students studying in five = 1910 – 465


schools in the year 1992 = 1445

6 . 241
Basic Numeracy and Data Interpretation

518. (c) Total population of state A = 3000 521. (b) No. of females above poverty line for state
Percentage population below poverty line E = 19,800 ... (1)
= 12% Let x be the total population of state E
Population below poverty line No. of people below poverty line
= 12% × 3000 = 10%x
∴ = 360 10
= x = 0.1x
∴ Population above poverty line 100
= 3000 – 360 ∴No. of people above poverty line
= 2640 = x – 0.1x
No. of females above poverty line = 0.9x
3 No. of females above poverty line
= × 2640
4 3 2
= 0.9 x
3 5
= × 2640 2
7 But 0.9 x = 19800
5
= 1131.43
[ from (1)]
or
1.8 x
1200 (approx) ⇒ = 19800
5
519. (d) Total population of C and D together 19800 5
= 18,000 ⇒ x=
1.8
Let population of state C and D be a and b ⇒ x = 55,000
@UPSC_THOUGHTS

respectively ∴ Total population of state E = 55,000


⇒ a + b = 18000 ∴ No. of people below poverty line
⇒ a = 18000 – b = 0.1 × 55000
∴People below poverty line in state = 5500
C = 25% (18000 – b) ∴ No. of males below poverty line
But value of neither a nor b is given so data 6
is inadequate = 5500
11
520. (c) Let x be the population of state B = 3,000
No. of people below poverty line
= 15% of x 522. (d) Percentage of no. of specialist officers in
Kolkata to that of no. of officers
15
= x No. of specialist officers
100 = 100
No. of male members below poverty line No. of officers
5 15 = 1200
= × x 100 = 8.05%
12 100 14900
or
Given that no. of male members below poverty 8% (approx)
line for state B
523. (c) Total of all officers and all clerks.
= 500
Officers Clerks
5 15
⇒ 500 = × x 2,000 5,000
12 100
Cancel out as many common terms as possible 15,000 17,000
17,000 19,500
1 1 3,500 20,000
⇒ 500 = × x
4 4 14,900 17,650
⇒ x = 500 × 4 × 4 11,360 15,300
⇒ x = 8000 9,000 11,000
∴ Total population of state B = 8000. Total 72,760 1,05,450

6 . 242
Basic Numeracy and Data Interpretation

∴ Difference between clerk and officers Total candidates appeared in 1993


= Total no. of clerks – Total no. of officers = 8250 + 7865 + 7120 + 8545 + 7962 + 6895
= 105450 – 72760 = 46637
∴ Required difference
= 32690
= 46,637 – 35,675
524. (b) The percentage of no. of clerks more than = 10,962
that of no. of officers in Chennai
529. (c) Percentage increase/decrease in the no. of
No. of clerks – No. of officers appeared to qualified candidates in state B
= 100
No. of officers in 1991
11000 – 9000 Increase/Decrease in those qualified
= × 100 = Increase/Decrease in those appeared × 100
9000

2000 840 864


= × 100 = 100
9000 8100 7200

= 22.22% 24 (decrease)
= 900(increase) 100
or 22% (approx)
525. (b) No. of clerks in Bangalore = 5000 = 2.67%
or
300% more number of clerks 2.5% (approx.)
= 5000 + 300% of 5000 530. (b) Total qualified candidates in 6 states:
= 5000 + 15000
(a) In year 1993
@UPSC_THOUGHTS

= 20000 = 876 + 792 + 685 + 842 + 934 + 788


From the table, we can see that Hyderabad is = 4917
the centre which has 20000 clerks, i.e., 300% (b) In year 1996
more than that of Bangalore.
= 853 + 940 + 827 + 746 + 812 + 911
526. (b) In fact, looking at the table, you can conclude = 5089
that only Delhi or Mumbai or Kolkata or
∴ Required difference
Hyderabad could be the answer, as these
= 5089 – 4917
centres have larger numbers than the others.
Delhi is not mentioned in the option. So you = 172
can get the answer just by counting up for
531. (d) Per cent of total qualified candidates to those
Mumbai and Delhi.
appeared in all 6 states in 1996
Mumbai : 17000 + 19500 + 70 + 7000 + 900
Total qualified candidates
= × 100
= 44470 Total candidates appeared
Hyderabad : 3500 + 20000 + 300 + 9000 + 1150 Total qualified candidates in 1996
= 33950
= 853 + 940 + 827 + 746 + 812 + 911
Kolkata : 14900 + 17650 + 70 + 1300 + 1200 = 5089
= 35120 Total appeared candidates in 1996
527. (d) We can see from the table that in the year = 7964 + 8107 + 7058 + 6754 + 7766 + 8934
1994, the no. of candidates from state D had = 46583
maximum percentage of qualified candidates
5089
i.e. 898 candidate = 100
46583
528. (b) Total candidates appeared in six states in
= 10.92%
1991
= 5600 + 7200 + 4850 + 6325 + 5200 + 6500 or
= 35675 11% (approx)

6 . 243
Basic Numeracy and Data Interpretation

532. (a) Highest total marks obtained by a student = 138 + 32


= 590 marks = 170
Lowest total marks obtained by a student Percentage of marks obtained in Social Stud-
= 477 marks ies and English together
Percentage of highest total marks obtained by Total marks obtained in
a studnet
= S. Studies and English
590
= × 100 Total max. marks in
750
S. Studies and English
= 78.66%
170
= 100
Percentage of lowest total marks obtained by 200 50
a student
170
477 = 100
= 100 250
750
= 63.6% = 68%
∴Required difference = (76 – 68)%
∴ Maximum difference between the per cent
= 8%
age of total marks obtained by any two
students 535. (a) Strength of Unit B in 1998
= 78.66 – 63.6 = Initial strength + Total no. of workers who
= 15.06% joined till 1998 – Total no. of workers who
or 15% (approx) left till 1998
Initial Strength = 132
533. (b) Average marks obtained by seven students
@UPSC_THOUGHTS

in Biology Total no. of workers who joined till 1998


Total marks obtained in Biology = 32 + 14 + 12 + 17 = 75
=
No. of students Total no. of workers who left till 1998
= 23 +16 + 12 + 14 = 65
105 98 112 106 119 107 96
= ∴ Strength of unit B in 1998
7
743 = 132 + 75 – 65
= = 106.14
7
or 106 = 142

534. (b) Student E 536. (c) Year 1999


Total marks obtained in Chemistry and Maths Unit Initial + Joined Left Total
together = 71 + 119 Strength till 1999 till 1999 Strength
= 190
(15+18+20 (12+17+9
Percentage of marks obtained +40+35) +32+22)
A 156 + 128 – 92 = 192
(Total marks obtained in
Chemistry and Maths) B 132 + 90 – 76 = 146
= 100 (32+14+12 (23+16+
Total Max. marks in Chemistry
and Maths +17+15) 12+14+11)

190 C 98 + 129 – 78 = 149


= 100 (36+19+ (12+8+17
100 150
14+35+25) +23+18)
190
= 100 D 76 + 117 – 58 = 135
250
(26+28+16 (6+17+9
= 76%
+23+24) +12+14)
Student A E 125 + 96 – 96 = 125
Total marks obtained in Social Studies and (13+15+16 (11+11+
English together +14+38) 19+23+32)

6 . 244
Basic Numeracy and Data Interpretation

∴ From above table, we can see that in 1999 = 93 – 44


strength of workers is maximum in Unit A = 49 (i.e. increase)
537. (c) Strength of workers in Unit C in 1996 Percentage of net increase in workers in Unit
= Initial strength + No. of workers joined till 1996 D to that of initial strength
– No. of workers left till 1996 = 98 49
=
76
+ (36 + 19) – (12 + 8)
= 64.47% increase.
= 98 + 55 – 20 = 133
540. (d) Marks obtained by B in a subject
Similarly, strength of workers in Unit E in 1997
= 125 + (13 + 15 + 16) – (11 + 11 + 19) Percent marks
= × Max. marks
100
= 125 + 44 – 41
This can be calculated in the following way.
= 128
∴ The required percentage Subject Marks obtained
Strength of workers in C in 1996 69
= × 100 Physics 150 = 103.50
Strength of workers in E in 1997 100
133
= 100 72
128 Chemistry 75 = 54
100
= 103.9%
or 104%(approx) 71
Maths 200 = 142
100
538. (c) Strength of Unit A in 1996
78
= 156 + (15 + 18) – (12 + 17) History 100 = 78
@UPSC_THOUGHTS

100
= 156 + 33 – 29
69
= 160 Geography 50 = 34.50
100
Strength of Unit B in 1996 66
English 75 = 49.50
= 132 + (32 + 14) – (23 + 16) 100
= 132 + 46 – 39 Total marks obtained = 461.5
= 139
∴ Total marks obtained by B in all subjects = 461.5
Strength of Unit C in 1996
541. (c) Marks obtained by students in Chemistry
= 98 + (36 + 19) – (12 + 8)
= 98 + 55 – 20 Per cent marks
= × 75
= 133 100

Strength of Unit D in 1996 Subject Marks obtained


= 76 + (26 + 28) – (6+ 17)
63
= 76 + 54 – 23 A 75 = 47.25
100
= 107 72
B 75 = 54
Strength of Unit E in 1996 100
= 125 + (13 + 15) – (11 + 11) 78
C 75 = 58.50
= 125 + 28 – 22 100
= 131 81
D 75 = 60.75
∴ Total strength of workers in all five unit in 1996 100
= 160 + 139 + 133 + 107 + 131 69
E 75 = 51.75
100
= 670
539. (b) Increase/Decrease strength of workers in Unit 57
F 75 = 42.75
D in 1998 100
(26 + 28 + 16 + 23) – (6 + 17 + 9 + 12) Total marks obtained = 315

6 . 245
Basic Numeracy and Data Interpretation

Average marks obtained by 6 students in 544. (c) Total marks obtained by 6 students in Ge-
Chemistry ography
Total marks obtained Sum of percent marks obtained
= = × 50
6 100
315
= = 52.5
6
= 64 69 75 58 66 71 50
542. (d) Marks obtained in Physics 100
82 403
by C = × 150 = 123 = 50
100 100
58 = 201.5
by E = × 150 = 87
100 Average marks obtained by 6 students in
Marks obtained in Chemistry Geography

78 Total marks obtained


by C = 75 = 58.5 Number of students
100
69 201.5
by E = 75 = 51.75 =
100 6
= 33.58
Total marks obtained in Physics and
Chemistry 545. (a) Percentage decrease in production from 1994
to 1995 of company A.
by C = 123 + 58.5 = 181.5
by E = 87 + 51.75 = 138.75 Production in 1995 - Production in 1994
× 100
Production in 1994
@UPSC_THOUGHTS

Difference in total marks obtained by C in Phys-


ics and Chemistry and that obtained by E in 4 3
= 100
same subjects 4
1
= 181.5 – 138.75 = 100
4
= 42.75 = 25%
543. (b) Marks obtained by A in Maths 546. (b) Percentage of production of company B in
89 2001 to that of in 2000
= × 200 = 178
100
Production in 2001
= 100
Marks obtained by A in History Production in 2000

55 11 lakh units
= × 100 = 55 = 100
100 7 lakh units
Total marks obtained by A in both
= 157.14%
Maths and History = 178 + 55 = 233
or
Total max. marks in both Maths and History 157% (approx)
= 200 + 100 = 300 547. (d) We can see from the given graph that in 1996
the percentage increase in production from the
∴ Required percent marks
previous year is the highest. This inference
Total marks obtained by A can be drawn from the fact that the line joining
in both Maths and History the production in 1995 with the production in
= 100
Total max. marks in both 1996 is the longest among the lines in the
Math and History graph.
233 ∴ The correct answer is 1996.
= × 100
300 548. (a) We must take the readings from the graph,
= 77.67% though exact figures will not be available.

6 . 246
Basic Numeracy and Data Interpretation

Difference in total production of the two Percentage of income in 1997 of the investment
companies for the given years. in 1996
= Total production of company A for the given
Income in 1997
years – Total production of company B for = × 100
the given years
Investment in 1996
= (4 + 3 + 8 + 8 + 8 + 7 + 8 + 12) lakh units 2.52x
= × 100
– (1 + 1 + 1 + 2 + 3 + 5 + 7 + 11) lakh units x
= 58 lakh units – 31 lakh units
= 252%
= 27 lakh units
or 2700000 units 551. (a) Given that income of company A in 1995

549. (b) Average production of company B for given = Rs 21.7 lakh


years Let investment be Rs x lakh
Total production for given years Profit per cent in 1995 = 55%
= No. of years
We know that
1 1 1 2 3 5 7 11 Income = Investment + Profit on Investment
= lakh units
8
21.7 = x + 55%x
31 lakh units
= = 3.875 lakh units 55
8 ⇒ 21.7 = x + x
or 100
3.9 lakh units 100 x 55x
⇒ 21.7 =
550. (a) We know that 100
@UPSC_THOUGHTS

Income = Investment + Profit on Investment 155x


⇒ 21.7 =
Company B 100
The per cent profit earned in 1996 21.7 100
= per cent profit earned in 1997 ⇒ x=
155
= 80%
Let Investment in 1996 be Rs x ⇒ x = 14
⇒Income in 1996 = x + 80% x ∴ Investment of company A in 1995 = Rs 14 lakh
80 552. (c) Let income of company ‘A’ in 1995
=x+ x
100
= Investment of company ‘B’ in 1996
(100 80)x
= = Rs x
100
180 x Let Investment of company ‘A’ in 1995 be Rs I
=
100 Profit of ‘A’ in 1995 = 55%
∴Income in 1996 = 1.8 x We know that
Given that investment in 1997 is more than that Income = Investment + Profit on Investment
in 1996 by 40%
⇒ x = I + 55% I
⇒Investment in 1997 = x + 40%x
55
= x+
40
x ⇒ x = I+ I
100 100

140 100 55
= x ⇒ x= I
100 100
= 1.4x 155
∴ Income in 1997 = 1.4x + (80%) 1.4x ⇒ x=
100
= 1.4x + 1.12x
100 x
= 2.52x ⇒ I=
155

6 . 247
Basic Numeracy and Data Interpretation

Ratio of Investment of company A in 1995 to


Alternative Method
that of company B in 1996
We know that
Investment of Company A in 1995 Income×100
= Expenditure =
Investment of Company B in 1996 100 Profit percent
100 100 20 1,68,000 100
= x= = =
155 155 31 100 60
x
or 20 : 31 1,68,00,000
=
160
553. (a) Given that investment of company B in 1993
= Rs 1,05,000
= Rs 15,40,000
556. (b) We know that
Per cent profit in 1993 = 45%
We know that Income×100
Expenditure =
Income =Investment + Profit on Investment 100 Profit percent
= 15,40,000 + 45% × 15,40,000
Let Income of the three companies in 2000 be
45 IA, IB, IC respectively
= 15,40,000 + × 15,40,000
100
= 15,40,000 + 6,93,000 Let Expenditure = E for the three companies

= Rs 22,33,000 I A × 100 I × 100 I × 100


⇒ = B = C
100 75 100 85 100 70
or Rs 22.33 lakh
Cancelling the common numerator 100 we get
@UPSC_THOUGHTS

554. (d) The question gives information regarding


income of 1999. But there is no information IA IB IC
= 175 : 185 : 170
regarding the relationship between income of 175 185 170
1999 and that of 2000. So income of 2000 = 35 : 37 : 34
cannot be determined. Morover there is no 557. (d) Let Income of company B and C for 1995
information about expenditure of 2000 also. be IB and IC respectively
So it is not possible to determine income of We know that
2000 from the given information. Income × 100
Expenditure =
555. (c) Given, Income of company C in 1999 100 Profit per cent
= Rs 1,68,000 IB ×100 I ×100
⇒ = C
Let Expenditure in 1999 be E 100 50 100 60
We know that IB I
⇒ = C
Income – Expenditure 150 160
Profit per cent = × 100
Expenditure IB 15
⇒ =
168000 E IC 16
⇒ 60 = 100
E ∴ IB : IC = 15 : 16
168000 E 6 So we can see that the statement given in
⇒ – =
E E 10 options (a), (b) and (c) are not ‘definitely true’
168000 6 about the incomes of two companies in 1995
⇒ = 1
E 10 So the answer is Option (d)
168000 16
⇒ = 558. (c) Let Expenditure of company B in 1996
E 10
= Income of company A in 1998
⇒ 16E = 168000 × 10
Let income of company B in 1996 = IB
1680000 and expenditure of company A in 1998 = EA
⇒ E= = 1,05,000
16
To find: Ratio of x : EA in 1996

6 . 248
Basic Numeracy and Data Interpretation

For 1998 Months Percentage increase in price


We know that 3980 3210
March 100 = 23.99%
Expenditure (100 + Percent profit) 3210
Income =
100 4800 3980
April 100 = 20.60%
E (100 65) 3980
x= A
100
33 4940 4800
x 165 May 100 = 2.91%
= = 4800
EA 100 20

x : EA = 33 : 20 4970 4940
June 100 = 0.61%
4940
559. (b) Consider year 1998
We know that 5730 4970
July 100 = 15.29%
4970
Expenditure (100 + Percent Profit)
Income = 6250 5730
100 August 100 = 9.08%
5730
∴ Income of company B
6.5(100 80) 7020 6250
IB = September 100 = 12.32%
100 6250
1170 117 ∴ The increase in price of crude oil was more than
= =
100 10 10% in 4 months i.e. March, April, July and
Consider year 1996 September
@UPSC_THOUGHTS

We know that
561. (d) Crude oil price in April = Rs 4800
Income ×100 Had it been lesser by Rs 223, then the new
Expenditure = 100 Percent profit
crude oil price in
∴ Expenditure of company C April = Rs 4800 – Rs 223

6.5 100 = Rs 4577


EC =
100 80 % increase in crude oil price over earlier month
650 65 Price in April – Price in March
= = = × 100
180 18 Price in March
∴ Required ratio 4577 3980
= 100
3980
117 597
IB = 100
= = 10 3980
EC 65
18 = 15%

562. (a) From the table in solution to Question No.


117 18 81
= = 560, we can see that July and September
10 65 25
are the months which experienced more than
(  Cancelling all common terms) 10% but less than 20%increase in the price
∴ Required ratio = 81 : 25 of crude oil over the earlier month.
560. (c) Percentage increase in price of crude oil 563. (d) % increase in price of crude oil from February
to September
Price in current month – Price in
previous month Price in September – Price in February
=100 × = 100
Price in previous month Price in February

The % increase in price of crude oil for various 7020 3210


= 100
months can be calculated as follows: 3210

6 . 249
Basic Numeracy and Data Interpretation

3810 567. (d) Per cent increase/decrease from 1998 to 1999


= 100
3210 Production in 1999 – Production in 1998
= 118.69% = ×100
Production in 1998
or For state A
120% (approx) 45 40
= 100 = 12.5% increase
564. (c) From the table in solution to Question 560, 40
we can see that the month which had less For state B
than one percent increase in crude oil price 45 35
= 100 = 28.57% increase
over earlier month was June. 35
For state C
565. (d) Total production in 3 years in State A
40 45
= (35 + 40 + 45) = 100 = 11.11 decrease
45
= 120 lakh tonnes
For state D
Total production in 3 years in State F
55 45
= (60 + 50 + 40) lakh tonnes = 100 = 22.22% increase
45
= 150 lakh tonnes For state E
∴ Required percentage =
55 40
100 = 37.5% increase
40
Total production of State A
= Total production of State F × 100 For state F
40 50
@UPSC_THOUGHTS

= 100 = 20% decrease


120 50
= 100
150 ∴ The percentage is minimum for State C.
= 80% 568. (b) Total production in all states in 1999
= (45 + 45 + 40 + 55 + 55 + 40) lakh tonnes
566. (a) Average production for all states
= 280 lakh tonnes
Total production for all states Total production in all states in 1998
=
No. of states =(40 + 35 + 45 + 45 + 40 + 50) lakh tonnes
= 255 lakh tonnes
Average production for all states in 1998

40 35 45 45 40 50 ∴ Required percentage
= lakh tonnes
6 Total production in all states in 1999
= Total production in all states in 1998 ×100
255
= lakh tonnes
6 280
= 100
= 42.5 lakh tonnes 255
= 109.8%
Average production for all states in 1999
or
(45 45 40 55 55 40)
= lakh tonnes 110%
6
569. (b) Total Indian Tourists
280 = (540 + 220 + 130 + 535 + 140)
= lakh tonnes
6
= 1565 thousand
= 46.67 lakh tonnes
10% of Total. Indian Tourists
∴ The average difference
10
= 1565 ×
= (46.67 – 42.5) lakh tonnes 100
= 4.166 = 156.5 thousand
or 4.17 (approx) = 156500

6 . 250
Basic Numeracy and Data Interpretation

∴The no. of districts in Himachal Pradesh which ∴ Required Ratio


were visited by more than 10% of total Indian
Indian tourists visiting Kullu
tourists = 3 (i.e. Shimla, Solan, Kullu). =
Foreign tourists visiting Kullu
570. (d) No. of Indian tourists visiting Chamba
535000
=
= 130 thousand 15000
= 1,30,000 107
= or 107 : 3
No. of Indian tourists visiting Shimla 3

= 5,40,000 573. (d) Total no. of foreign tourists who visited


Himachal Pradesh = 34,500
∴ Required percentage
(From solution to Ques. 571)
(Tourists in Shimla – Tourists in Chamba) 10% of total foreign tourists who visited
= × 100
Tourists in Shimla Himachal Pradesh
540000 130000 = 10% × 34,500
= 100
540000
10
410000 = × 34,500
= 100 100
540000
= 3450
= 75.92
∴ The districts which were visited by less than
or
10% of total foreign tourists are Solan and
75% (approx.)
@UPSC_THOUGHTS

Chamba.
571. (b) Total foreign tourists visiting Himachal Pradesh
574. (b) We can see from the pie chart, percentage
= (7 + 2 + 0.5 + 15 + 10) thousand of red car = 20%
= 34.5 thousand
Given that 3% increase in production of a car
= 34,500
of a particular colour along with red cars will
Total. tourists visiting Himachal Pradesh make them 35% of total cars
= Total Indian tourists + Total Foreign
Let x % be the present per cent production of
tourists
that particular car
= 1565000 + 34500
⇒Per cent of red cars + x% + 3% = 35%
= 1599500
⇒ 20% + x% + 3% = 35%
[∴ Total Indian Tourists from solution to
⇒ x % = 35% – (20 + 3)%
Question No. 569]
∴ Required percentage ⇒ x = 35 – 23

Total foreign tourists ⇒ x = 12%


= 100
Total no. of tourists ∴ 12% is the per cent of blue cars.
34500 So, 3% increase in production of blue cars along
= × 100 with red cars will make them 35% of total cars.
1599500

= 2.16% 575. (b) We can see from the pie chart that:
white + black + red cars together make 50%
or
2% (approx.) White + Black + Red
= 25% + 5% + 20%
572. (c) Foreign tourists who visited Kullu = 15000
Indian tourists visiting Kullu = 535000 = 50%

6 . 251
Basic Numeracy and Data Interpretation

576. (c) Percent of white cars = 25% No. of boys in course D


Let x be the cars that which white cars are 18% = Total no. of students in Course D
more popular – No. of girls in course D
⇒ White car (%) = x cars (%) + 18% = 420 – 240 = 180 boys
⇒ 25% = x cars (%) + 18% ∴ Ratio of boys and girls in Course D
= 180 : 240
⇒ x cars (%) = (25 – 18)
= 18 : 24
= 7%
= 3: 4
We can see that percent of silver cars = 7%
Alternative Method
∴ White cars are 18% more popular than silver
cars Required ratio of Boys : Girls

577. (b) Total production of cars = 420000 35 30 30


= 1200 800: 800
100 100 100
Production of blue cars
= 12% × 420000 = 50,400 420 – 240 : 240
Production of golden cars 180 : 240
= 10% × 420000 = 42,000 3: 4
∴ No. of blue cars sold more than that of
580. (c) No. of boys in various courses
golden cars
= 50400 – 42000 = Total no. of students in the particular course
– No. of girls in the particular course
@UPSC_THOUGHTS

= 8,400
This can be calculated as follows:
578. (c) Percentage of white colour car = 25%
Percentage of car which is 17% less popular Course Total no. No. of Difference
than white colour = (25 – 17)% of students girls

= 8% A 20% × 1200 = 240 30% × 800 = 240 0

We can see that it is the green colour car which B 15% × 1200 = 180 10% × 800 = 80 100
has 8% sales. C 5% × 1200 = 60 2% × 800 = 16 44
∴ Green colour is 17% less popular than white D 35% × 1200 = 420 30% × 800 = 240 180
colour.
E 12% × 1200 = 144 14% × 800 = 112 32
For Qs. 579 to 590, you need to consider both pie
F 13% × 1200 = 156 14% × 800 = 112 44
diagrams. Be careful to read the diagrams and the
questions before answering.
∴ We can see that for the courses C and F the no.
579. (a) No. of girls in course D of boys is the same i.e. 44.
= 30% × Total no. of girls
= 30% × 800 581. (c) No. of girls for course E

30 = 14% × 800 = 112


= × 800
100 No. of boys for course E
= 240 girls = Total no. of students for course E – No. of

Total no. of students in course D girls for course E


= 12% × 1200 – 112
= 35% × Total no. of students
= 144 – 112
= 35% × 1200
= 32 boys
35
= × 1200 = 420 Students Per cent of no. of girls more than that of boys
100
for course E

6 . 252
Basic Numeracy and Data Interpretation

No. of candidates who qualified from


No of girls – No. of boys
× 100 State B = 61% × 54000
No. of boys
= 32,940
112 32
= 100 No. of female candidates who qualified from
32
80 4
= 100 State B = × 32940
32 10
= 250% = 13,176
Total no. of female candidates who qualified
582. (d) We can see from the table given in solution from both state A and B
to question 580 that for course A, the no.
= 12250 + 13176 = 25,426
of boys is minimum, i.e., 0.
586. (c) Total no. of candidates who qualified from
583. (b) No. of girls in course C
State A = (15% of 300000) × 49%
= 2% × 800 = 16 girls 49
= 45000 ×
584. (d) Total no. of candidates who appeared from 100
= 22050
State G = 7% × 300000 Total no. of candidates who qualified from
7
State C = (6% of 300000) × 54%
= × 300 000 54
100 = 18000 ×
100
= 21,000 = 9720
Total no. of candidates who qualified from
@UPSC_THOUGHTS

No. of candidates who qualified from


States A and C together
State G = 48% × 21000 = 22050 + 9720
48 = 31770
= × 21000
100 ∴ Percentage of candidates who qualified from
= 10,080 States A and C together
No. of male candidates who qualified from Total candidates qualified
9 from A and C
State G = × 10080 = × 100
20 Total candidates who appeared
= 4,536 from state A and C together

585. (d)Total. no. of candidates who appeared from 31770 31770


= × 100 = ´ 100
State A = 15% × 300000 (45000 + 18000) 63000

= 45,000 = 50.428%
No. of candidates who qualified from or
State A = 49% × 45000 50.43%

= 22,050 587. (d) Total no. of candidates who qualified from


No. of female candidates who qualified from State E = 65% × 36,000
= 23,400
5
State A = × 22050 Total no.of candidates who qualified from
9
= 12,250 State D= 45% × 69,000
= 31,050
State B
∴ Total no. of candidates who qualified from
Total. no. of candidates who appeared States E and D together
= 18% × 300000 = 23,400 + 31,050
= 54,000 = 54,450

6 . 253
Basic Numeracy and Data Interpretation

588. (d) Cost of production of both item I and II for Cost of production of item II
company D 2
= × Rs 3 crore = Rs 2 crore
= 8% of Rs 25 crore 3
Profit earned
= Rs 2 crore
= Percent profit × Cost of production
Cost of production of item II
∴ Profit earned on item I
5 30
= × Rs 2 crore
8 = × Rs 1 crore = Rs 0.3 crore
100
= Rs 1.25 crore Profit earned on item II
24
Amount of profit earned by company D on = × Rs 2 crore = Rs 0.48
item II 100
∴ Total profit earned by G for item I and II
= Percent profit earned on item II × Cost of
= Rs 0.3 crore + Rs 0.48 crore
production of item II = Rs 0.78 crore
= 25% × Rs 1.25 crore = Rs 78 lakh
25 [1 crore = 100 lakh)
= × Rs 1.25 crore
100
= Rs 0.3125 crore 591.(b) Total graduate male population = 24 lakh
or 24 × 105
We know that
Total XII std. pass = 32 × 105
I crore = 100 lakh
Graduate male population of state A
∴ Rs 0.3125 crore
7 16
= Rs 0.3125 × 100 lakh = × × 24 × 105 = 2,24,000
@UPSC_THOUGHTS

12 100
= Rs 31.25 lakh
XII std male population of state A
589. (a) Cost of production of Item I by company F
7 15
Cost of production of both Item I and II by F = × × 32 × 105 = 2,10,000
16 100
5
= × Rs 25 crore The difference is 2,24,000 – 2,10,000 = 14,000
100
= Rs 1.25 crore
592. (d) Graduate female population of state E:
Cost of production of Item I
1 Std. XII female population of state D
= × Rs 1.25 crore
5 7 20 7 12
⇒ × × 24: × × 32
= Rs 0.25 crore 16 100 12 100
Cost of production of Item II by Comapny D
= Rs 1.25 crore [We can leave out 105 as it is common to
(From Q. 589) both sides.]
∴ Required percentage
210000 15
(Cost of production of item I by F)
100
⇒ 210000 : 224000 = =
= 224000 16
(Cost of production of item II by D)
Rs 0.25 crores
= 100 ⇒ Ratio is 15 : 16
Rs 1.25 crore
= 20%
Graduate female population of state C
590. (b) Cost of production of both items I and II for 593. (c) × 100
XII std. female population of state C
company G
12 4
15 24
= × 25 crore 9
100 ⇒ 100
5
= Rs 3 crore 18 32
9
Cost of production of item I
1
1 ⇒ 100 = 50%
= × Rs 3 crores = Rs 1 crore 2
3

6 . 254
Basic Numeracy and Data Interpretation

594. (d) Total graduate population of state F


48
596. (d) Required no. = 100 = 85.71
14 56
= × 24 lakh
100
⇒ 86% approx
Total XII std. population of state A
597. (d) Difference = 120 – 12 = 108
15
= × 32 lakh 598. (a) The required number is the difference between
100 total male players and players of cricket :
14 255 – 55 = 200.
24
Required percentage
100 100 599. (b) The required ratio = 12 : 40 ⇒ 3 : 10
15
32 600. (c)
100
336 Chart for Qs. 601 to 605
⇒ 100 = 70%
480 Total employees = 520
17
24lakh Ratio of men to women = 5 : 3
17
595. (a) Required ratio = 100
12 16 5
lakh ∴ Men = 520 = 325
100 8
⇒ 17 : 16 Women = 520 – 325 = 195
Department Men Women Total
Note: Questions based on passages are similar to
analytical reasoning problems with arithmetical HR 4% of 325 = 13 40% of 195 = 78 91
calculations included. It is best to represent the data
@UPSC_THOUGHTS

in the form of a chart and plug in the data you can 2


IT 20% of 325 = 65 of 195 = 78 143
calculate. Then the question become easy to answer. 5
Questions 596 to 610 are of this type.
Production 40% 325 = 130 135–130 = 5 135

Chart for Qs. 596 to 600: Marketing 325–(13 + 65 + 130) 195–(78+78+5)


= 117 = 34 151
Total Male Female

Badminton 15% of 400 20% of 60 60 – 12 601. (a) Required percentage


No. of men in marketing
= 60 = 12 = 48 = × 100
Total employees
2 1 117
Hockey of 400 160 – 40 of 160 = × 100 = 22.5%
5 4 520
= 160 = 120 = 40
602. (d) Ratio of men to women in HR
Lawn Tennis 6% of 400 12 12
⇒ 13 : 78
= 24
⇒ 1 : 6
Cricket 25% of 400 100 – 45 45
= 100 = 55 603. (b)
Baseball 400 – 56 Nil
604. (d) Total employees = 520
(60 + 160 +
Employees in Production = 135
24 + 100 =)
135
= 344 Required percentage × 100 = 25.96%
520
= 56
= 26% approx
Total 400 255 145 605. (c)

6 . 255
Basic Numeracy and Data Interpretation

Chart for Qs. 606 to 610


84 7
606. (d) The required ratio is 84 : 60 or
Product Category A Category B Total 60 5
⇒ 7 : 5
Mobile 50% of 168 = 84 84 600–432
= 168 322
607. (a) Average = = 80.5 ⇒ 81 approx.
4
1
T.V. 200–150 = 150 25% of 200 of 600
3 608. (a) Required total = 150 + 84 + 84 = 318
= 50 = 200
609. (b) Both equal 84
1
Refrigerators of 112 = 28 112 – 28 = 84 40% of 610. (c) Required difference =
4
280 = 112
No. of computers in category B – No. of
Computers 20% of 120 = 60 60 20% of 600 refrigerators in category A
= 120
⇒ 60 – 28 = 32
Total 322 278 600

@UPSC_THOUGHTS

6 . 256
7
PRACTICE
SESSION
It is a well-worn cliché that practice makes perfect. This section, which
@UPSC_THOUGHTS

aims to provide that practice, is divided in two parts. The first comprises
Practice Sets: these are sets of 220 questions each for practice. A wide
range of questions have been carefully assembled along with their an-
swers, and explanations where required.
These questions cover all the areas of the syllabus
except comprehension passages.

The second part consists of Three Practice Test Papers. The UPSC
Civil Services Preliminary Examination General Studies Paper II in 2011
contained 80 questions, all carrying equal marks. (Incidentally, the
questions on Interpersonal Skills and Decision-Making were not to be
subject to negative marking.) We have conformed, broadly, to the
pattern set by the examination papers since 2011. Attempting the
papers in ‘test mode’—i.e., each one in two hours—will help to build up
confidence as well as locate weak areas that require further attention.
Practice Session

@UPSC_THOUGHTS

7.2
Practice Session

Practice Session
I. PRACTICE EXERCISE SETS

PRACTICE SET 1
Directions: Find the missing number in each of the Directions: Read the following:
following series in Questions 1-6. P, Q, R, S, T and U are members of a family. There are
two married couples in the family. Q is a trader and
1. 1, 1, 2, 4, 3, 9, 4, . . .
is father of T. U is grandfather of R and is a teacher.
(a) 5 (b) 16
S is grandmother of T and is a housewife. R is daughter
(c) 6 (d) 15
of P. There is one trader, one teacher, one housewife,
2. 1, 1, 4, 8, 9, 27, . . . one engineer and two students in the family. Now
(a) 16 (b) 64 answer Questions 12 to 16.
(c) 30 (d) 18
12. Which of the following two pairs are married
3. CFL, EIK, GLJ, IOI, . . . couples in the family?
(a) KRH (b) KRJ I. US II. QP
(c) JRH (d) KQH III. RP IV. QT
(a) I and III (b) I and II
4. QIF, S2E, U6D, W21C, . . . (c) II and III (d) II and IV
@UPSC_THOUGHTS

(a) Y66B (b) Z88B


(c) Y88B (d) Z66B 13. Which of the following is P’s profession?
(a) Housewife or engineer
5. 1, 2, 3, 6, 9, . . ., 54 (b) Engineer
(a) 18 (b) 36 (c) Teacher
(c) 81 (d) 27 (d) Cannot say
6. ccbab — caa — bccc — a — 14. Which of the following is definitely a group of
(a) b a b b (b) b b b a male members of the family?
(c) b a a b (d) b a b c (a) UQT (b) TUR
7. In a certain code, HUMIDITY is written as (c) QUR (d) QU
UHMIIDTY. How is POLITICS written in that 15. Who is the husband of P?
code? (a) R (b) Q
(a) OPILITCS (b) OPLIITCS (c) T (d) Cannot say
(c) OPLITISC (d) None of these
16. How are T and R related?
8. If CHAIR is written as EGCHT, how is AUDIT (a) T is R’s brother
written in that code? (b) T and R are sisters
(a) CSFHV (b) CTFHV (c) R is T’s sister
(c) BTFHV (d) BTSHV (d) Cannot be determined
9. If GTQQX means HURRY, then RSNO means 17. ‘Table’ is related to ‘Wood’ in the same way as
(a) STOP (b) SHOP ‘Shirt’ is related to
(c) SHIP (d) STEP (a) Cotton (b) Textile
(c) Cloth (d) Yarn
10. CBE means ‘bad’, GMBH means
(a) Good (b) Flug 18. ‘Flower’ is related to ‘Bud’ as ‘Fruit’ is related to
(c) Glad (d) Flag (a) Seed (b) Flower
(c) Petal (d) Tree
11. If GIGANTIC is written as GIGTANCI, how is
MIRACLES coded? 19. Although there are no physical differences be-
(a) MIRLACSE (b) MIRLCAES tween the visual organs of the two groups, inhab-
(c) RIMLCAES (d) RIMCALSE itants of Island A, when shown a card displaying

7.3
Practice Session

a spectrum of colours, perceive fewer colours 27. Some hens are cows. All cows are dogs. Therefore
than do the people of Island B. Which of the we may conclude
following conclusions can most reliably be drawn (a) no dog is a hen
from the information above? (b) some hens are dogs
(a) Inhabitants of Island A are taught to (c) all hens are dogs
(d) none of the above
recognise fewer colours than persons on
Island B 28. Some dogs are bats. Some bats are cats. So we
(b) Differences in physical environment influ- may conclude
ence colour perception (a) some dogs are cats
(c) It is possible that inhabitants of Island A (b) some cats are dogs
subsist on a poorer diet than those of (c) some bats are dogs
(d) none of the above
Island B
(d) Physical structure of visual organs alone 29. Some cats are dogs. No dog is blue. Which
do not determine colour perception conclusion follows from the above?
1. No cat is blue.
20. Five persons sitting around a table were playing 2. Some dogs are cats.
a card game. Makku was to the left of Rukku, 3. Some cats are blue.
Vikku was to right of Akku and between Akku 4. Some cats are not blue.
and Nakku. Who was to the right of Nakku? (a) 1 only (b) 2 and 3
(a) Vikku (b) Rukku (c) 2 and 4 (d) None of them
(c) Makku (d) Cannot say
30. All cats are bats. All bats are rats. Which con-
@UPSC_THOUGHTS

clusion definitely follows from the above?


In the following questions 21 to 25, three classes (a) All cats are rats
of items are given. Choose the diagram that best (b) All bats are cats
illustrates the relationship among the given classes (c) No rat is a cat
(d) All of them follow
in each case.
31. If 2x2 + x – 2 = 1, and x > 0, then x = ?
(a) 1 (b) 3/2
(a) (b) (c) 3 (d) 6
1
32. The ratio of x to y is . If the
2
2
ratio of x + 2 to y + 1 is , then what is the value
(c) (d) 3
of x?
21. Diseases, Leprosy, Scurvy (a) 1 (b) 2
(c) 2 (d) 4
22. Venus, Mars, Planets
2
23. Human beings, Postgraduates, Teachers x y x y
33. If 9 , then ?
x2 y2 x y
24. Oranges, Fruits, Vegetables
1 1
25. Sheep, Dog, Wolf (a) (b)
3 9
26. All sides of a cube are of different colours. The (c) 1 (d) 9
red side is opposite to the black. The green side 34. In a certain population, x of every y persons are
is between red and black. The blue side is adja- found to have characteristic k. If 200 persons were
cent to white. The brown side is adjacent to blue. found to have characteristic k, what is the total
The red side is face down. Which side is opposite number of persons in the population?
to brown? (a) 200 x (b) 200 y
(a) White (b) Black
200 y 200 x
(c) Red (d) Green (c) (d)
x y

7.4
Practice Session

35. In a group of 15, 7 can speak German and 8 can cm. If a cube is shown on the drawing with a
speak Italian, while 3 can speak neither. What side of 10 cm, what is the actual volume (in cubic
part of the group can speak both German and metres)?
Italian? (a) 4.5 (b) 9
1 4 (c) 27 (d) 30
(a) (b)
5 15
3 1 42. In a list of numbers, each number after the first
(c) (d) 1
5 3
is exactly the number immediately preceding
3
36. A society of 356 voters has to choose a president. it. If the fifth number in the list is 3, what is the
5 candidates are seeking office. If all voters second number?
exercise their votes, what is the least number of 8 1
votes a successful candidate could receive and (a) (b)
3 9
yet have more votes than any other candidate? (c) 27 (d) 81
(a) 71 (b) 72 2
(c) 81 (d) 82 43. Avinash spent of his income for a month on
5
3
37. Two hours after a goods train leaves Nagpur an rent and of the remainder on other expenses.
4
express train leaves the same station travelling in The remaining Rs 180 he put in his savings
the same direction at an average speed of 60 account. How much was his income for the
kmph. After travelling four hours, the express month?
train overtakes the goods train. The average (a) Rs 1200 (b) Rs 1400
speed of the goods train was . . . kmph. (c) Rs 1600 (d) Rs 1800
(a) 30 (b) 40
@UPSC_THOUGHTS

44. Gunjan went to the stationers and bought things


(c) 58 (d) 60
worth Rs 25, out of which 30 p went on sales
38. Arusha would like to complete all her homework tax on taxable purchases. If the tax rate was 6
before 10 p.m. so that she can watch the special per cent, what was the cost of the tax-free items?
programme on television. She has 40-minute as- (a) Rs 15 (b) Rs 20
signments in each of five subjects. What is the (c) Rs 15.70 (d) Rs 19.70
latest time at which she can start and still
45. Kamal receives marks of 91, 88, 86 and 78 in four
complete her homework in time for the
out of five subjects. What must he receive in his
programme? fifth subject in order to average 85?
(a) 6 : 30 p.m. (b) 6 : 40 p.m. (a) 82 (b) 83
(c) 7 : 10 p.m. (d) 7 : 20 p.m. (c) 85 (d) 86
39. Paintwell Company contracts to paint three 46. In an office with 21 workers, 1/3 are men and
houses. Babu can paint a house in 6 days while the rest women. How many women should be
Paintal would take 8 days and Saigal 12 days. hired to obtain a staff in which 1/4 are men?
After 8 days, Babu goes on leave and Paintal (a) 3 (b) 5
begins to work for a period of 6 days. How many (c) 6 (d) 7
days will it take Saigal to complete the contract? 47. Sunita bought a TV with 20% discount on the
(a) 7 (b) 10 labelled price. Had she bought it with 25%
(c) 11 (d) 12 discount, she would have saved Rs 500. At what
40. In a certain shop, a reduction of 30% from the price did she buy the TV?
usual selling price is offered on some items. If the (a) Rs 5,000 (b) Rs 10,000
price of an item has been reduced by Rs 2.40, (c) Rs 12,000 (d) Rs 15,000
what is the usual selling price of that item? 48. Jaya is as much younger to Anil as she is older
(a) Rs 5.80 (b) Rs 8.00 to Prasad. If the sum of the ages of Anil and
(c) Rs 10.50 (d) Rs 7.20 Prasad is 48 years, what is the age of Jaya?
41. On a certain three-dimensional drawing, an ac- (a) 20 years (b) 25 years
tual dimension of 1.5 metres is represented by 5 (c) 24 years (d) Cannot say

7.5
Practice Session

49. Utkal and Nitpal invested Rs 7,000 and Rs 8,500 57. The length and width of rectangle AEFG are each
respectively in a business. They earned a profit 2/3 of the corresponding parts of ABCD.
of Rs 9,300 in the business. What was the share AEB =12; AGD = 6. What is the area of the
of Nitpal? shaded part?
(a) Rs 4,200 (b) Rs 4,650
(c) Rs 5,100 (d) None of these
50. Neera, Mani and Sona hired a video shooting
unit for a week for Rs 9,800. If they use it for 12
hours, 6 hours and 10 hours respectively, how
much of the rent should Neera pay?
(a) Rs 4,200 (b) Rs 2,400
(c) Rs 5,200 (d) Rs 3,250
(a) 48 (b) 40
51. A cricketer scored 180 runs in the first Test and (c) 36 (d) 24
258 runs in the second. How many runs should
he score in the third Test so that his average score 58.
in three Tests should be 230 runs?
(a) 250 (b) 252
(c) 230 (d) 242
52. The average of runs scored by the eleven players
of a cricket team is 60. If the runs scored by the
captain are ignored, the average of runs scored radius of I = 3 cm
by the remaining players increases by 5. How radius of II = 4 cm
@UPSC_THOUGHTS

many runs were scored by the captain? radius of III = 5 cm


(a) 0 (b) 10
(c) 15 (d) 100 What is the perimeter of triangle ABC formed by
joining the centres of the three circles?
53. Arun borrowed Rs 5,000 from Barun at simple (a) 12 cm (b) 15 cm
interest. After 4 years Barun received Rs 1,000
(c) 24 cm (d) Cannot say
more than the amount given in loan. What was
the rate of interest on the loan? 59. In the figure below, AB is the diameter and OC
(a) 5 (b) 5.5 = BC. What is the value of x/2?
(c) 4 (d) 4.5
54. Each term of a data is divided by a non-zero
number, a. In order to obtain the mean of the new
data, the original mean should be:
(a) diminished by a (b) increased by a
(c) multiplied by a (d) divided by a
55. A boy is running at a speed of p km/h to cover (a) 20° (b) 30° (c) 60°
a distance of 1 km. But due to the difficult terrain (d) Cannot be determined
his speed is reduced by q km/h (p > q). If he takes
60. The area of the circle O is 9π. What is the area
r hours to cover the distance then
of ABCD?
1 1 1 1
(a) (b) p c
r p q r
(c) r = p + q (d) r = p – q
56. A train, 700 metres long, is running at the speed
of 72 km/h. If it crosses a tunnel in 1 minute, then
the length of the tunnel (in metres) is
(a) 700 (b) 600
(a) 24 (b) 30
(c) 550 (d) 500
(c) 36 (d) 81

7.6
Practice Session

61. Square ABCD is inscribed in circle O. OV is 65. What is the area of the shaded region in the
perpendicular to DC. OV = 1. What is the area following figure (in square units)?
of the shaded portion?

A B

D (a) π/2 (b) π/3


V C
(c) π/4 (d) π 2
(a) π–4 (b) 2π – 4
66. What is the area of the shaded region in the
(c) 4π – 8 (d) 2π – 2
following figure (in square units)?
62. In the figure below, the rectangle ABCD has an
area equal to 100. Its base DC equals 20. What
are the areas of triangles DEC, DFC and DBC?

a2
(a) a2 1 (b) 1
2 2 2
@UPSC_THOUGHTS

(c) a2 (π – 1) (d) Cannot say


(a) 25, 25, 50 (b) 50, 50, 25 67. In the figure below, ABCD is a square and
(c) All 50 (d) All 25 semicircles are constructed on each side of the
63. Four equal circles of diameter square. If AB is 2, what is the area of the entire
1 cm touch at four points as shown in the figure. figure?

What is the area of the shaded portion (in square


(a) 4 + 8π (b) 4 – 2π
cm.)?
(c) 2 – 4π (d) 4 + 2π
(a) 1 (b) 1
4 68. What is the perimeter of the shaded portion?
(c) 1 4 (d)
4
64. In the figure below, the radii of the outer circles
are each a. What is the radius of the inner circle?

(a) ( 2 2 )a (b) ( 2 1) a (a) 100π (b) 50π


(c) 1/ 2 a (d) Cannot say (c) 100 (d) 50

7.7
Practice Session

69. Perimeter of square ABCD = 24. What is the area Relative Sweetness of Substances Lactose .16
of the shaded portion? Maltose .32
Glucose .74
Sucrose 1.00
Fructose 1.70
Saccharin 675.00
73. What percentage of increase in sweetness is
obtained by substituting equal amounts of mal-
tose for lactose?
(a) 9π – 36 (b) 36 – 3π (a) 16 (b) 50
(c) 9π – 24 (d) 36 – 9π (c) 100 (d) 200
70. 74. What is the ratio of glucose to lactose in a mixture
as sweet as maltose?
(a) 8 : 21 (b) 21 : 8
(c) 15 : 8 (d) 32 : 5
75. Approximately how many times sweeter than
sucrose is a mixture of glucose, sucrose, and
fructose in the ratio of 1 : 2 : 3 ?
The area of the shaded portion is (a) 1 (b) 1.3
(a) 2r2 (8 – π) (b) 2r2 (4 – π) (c) 2.3 (d) 2.9
@UPSC_THOUGHTS

(c) 2r2 (π – 4) (d) r2 (4 – π)


Directions: Refer to the following table and graph and
71. ABCD is a rectangle. answer the questions.
AD = 12; AB = 16; DE =? Table : Family A’s Expenditure
in 1991 (in Rs)
Housing 13,750
Clothing 1,300
Insurance 1,500
Medical expenses 2,600
Education 4,200
(a) 20 (b) 10 Furniture, gadgets 1,700
(c) 14 (d) 8 Entertainment 2,200
Food 4,000
72. A readymade garments shop marked down all Transport 3,000
merchandise as follows: Miscellaneous 750
Group Original Sale Total 35,000
Price Price
Kurta Rs 60 Rs 50
Salwar Rs 65 Rs 55
Jeans Rs 70 Rs 60
Shirt Rs 75 Rs 65
Which garment group was offered at the greatest
rate of discount from its original price?
(a) Kurta (b) Salwar
(c) Jeans (d) Shirt
Directions: Study the following table and answer ques-
tions 73 to 75.

7.8
Practice Session

76. For a family with an annual income of Rs 30,000 80. Which place has extreme variation of diurnal
how much would be put into savings according temperature?
to the figures for the average Indian? (a) A (b) B
(a) Rs 90 (b) Rs 900 (c) C (d) D
(c) Rs 30 (d) Rs 300
81. Which of the following is the equation of the
77. How does the expenditure for food by Family A graph? (Origin O is centre of circle)
compare with the way the average Indian spends
for food?
(a) Less than the average Indian
(b) More than the average Indian
(c) Equal to the average Indian
(d) Cannot be determined
78. The pie chart below represents the animal agri-
cultural production of a State. If the production
of sugar is 3000 tons, what is the total production
of rice and wheat? (a) x+y=a (b) x2 y2 a2
(c) x2 + y2 = a2 (d) None of these
Sugar 82. Which of the following equations expresses the
Wheat
graph line?
@UPSC_THOUGHTS

Rice
Millet

(a) 8000 tons (b) 6000 tons


(c) 4000 tons (d) 3000 tons

Directions: Study the following diagram and answer (a) y = mx (b) y = c


questions 79 and 80. (c) x = c (d) y = mx + c
Diurnal Temperature Variation 83. The following graph gives the production of crop
A over a certain period. Which period indicates
a breakthrough in the production of the crop?
Day max.
Night min.
B
A D

79. In which place was the average maximum tem-


perature and minimum temperature both the
least?
(a) A (b) B (a) 1959-60 (b) 1979-80
(c) C (d) D (c) 1980-81 (d) At no period

7.9
Practice Session

84. What is the length of PQ? If the figure above is a rectangle, what is the area
of the figure?
(a) xy (b) xy2
(c) y(x – 2) (d) x(y – 2)
89. A certain experiment involves repeatedly subject-
ing a chemical sample to the same procedure,
with the result that the sample loses 1/2 its
weight after each repetition. If the weight after the
sixth repetition is 16 gm, what is the weight after
the third repetition?
(a) 8 gm (b) 32 gm
(a) 4 units (b) 5 units (c) 128 gm (d) 256 gm
(c) 6 units (d) Cannot say 90. If the average of five different integers is 1, which
85. In the figure below what are the coordinates of of the following must be true?
point P? I. 1 is one of the integers
II. At least one of the integers is negative
III. 0 is not one of the integers
(a) I only (b) II only
(c) I and II (d) II and III
91. A gymnast’s score for an item is the average of
the scores awarded by ten judges on a scale
@UPSC_THOUGHTS

ranging from 0 to 10. If the first seven judges have


awarded the gymnast scores of 7, 8, 7.5, 9, 8.2,
(a) (–5, –2) (b) (–2, 5) 8.5 and 7.8, and she does not receive a score
lower than 6 from any of the other judges, then
(c) (–5, 2) (d) (2, –5)
her final score for the item will be
86. A circle whose radius is 7 has its centre at the (a) greater than 8.0
origin. Which of the following points are outside (b) greater than or equal to 7.8
the circle? (c) between 7.0 and 7.8
I. (4,4) II. (5,5) (d) greater than or equal to 7.4
III. (4,5) IV. (4,6) 92. A traveller has booked a holiday plan with agent
(a) I and II only X for a total cost of Rs 1200 and has paid agent
(b) II and III only X a non-refundable deposit equal to 10% of the
(c) II, III and IV only cost of the plan. On learning that she can get the
(d) II and IV only same holiday plan from agent Y for 20% less, she
87. The coordinates of the vertices of a quadrilateral decides to forfeit her deposit with agent X and
PQRS are P (0,0), Q (9,0), R (10,3) and S (1,3) purchase the plan through agent Y. Is it a prof-
respectively. The area of PQRS is itable decision?
(a) No, as it involves an increase in the cost
(a) 9 10 (b) 27/2
of the holiday by Rs 240
(c) 27 (d) Cannot say (b) Yes, as it involves a decrease in the cost of
88. the holiday by Rs 240
(c) Yes, as it means a decrease in the cost of
the holiday by Rs 120
(d) Immaterial, as no change in cost is in-
volved after forfeiting the deposit
93. From March 1 to March 31 the price of a certain
commodity fell by 1/4, and from April 1 to April

7 . 10
Practice Session

30 the price fell by 1/3. By what percentage Directions: Questions 98 and 99 are based on the
would the price of the commodity have to in- following table that shows the number of candidates
crease in the month of May to bring it back to appeared and projected to appear in the civil service
the level of March 1? examination over the years from various disciplines (in
(a) 25% (b) 50% lakhs).
(c) 66.5% (d) 100% Years
94. A certain manufacturer has three machines pro- Discipline 1990 1991 1992 1993 1994 1995
ducing the same item. If machine X produces 1/ Arts 10 6 12 24 30 36
4 as many of the item as Machine Y produces in Commerce 6 8 18 32 46 51
the same time and Machine Y produces twice as Science 55 58 45 60 72 75
many of the item as Machine Z in the same time, Engineering 3 2 2 1 3 1
then during a fixed period Machine Z produces Management 4 6 13 12 11 15
what fraction of the total number of items pro- Total 78 82 90 129 162 178
duced?
(a) 1/14 (b) 2/7 98. In which year was the number of candidates from
(c) 1/3 (d) 4/7 science discipline exactly half of the total number
of candidates in that year?
Directions: Study the following graph carefully and
(a) 1990 (b) 1991
answer questions 95 to 97.
(c) 1992 (d) 1993
Number of Branches of Five
Banks in Urban and Rural Areas 99. In which year was the number of candidates from
the Arts and Management disciplines together
@UPSC_THOUGHTS

equal to the number of candidates who appeared


from commerce discipline?
(a) 1991 (b) 1992
(c) 1993 (d) 1995
100. Three times the first of three consecutive odd
integers is 3 more than twice the third. Find the
third integer.
(a) 9 (b) 11
(c) 13 (d) 15
101. Raju is 15 years older than his brother Suraj.
95. What is the total number of rural branches of all
However, y years ago Raju was twice as old as
the five banks together?
Suraj. If Suraj is now b years old and
(a) 650 (b) 750
b > y, find the value of b – y.
(c) 875 (d) 765
(a) 13 (b) 14
96. What is the ratio of the rural branches to urban (c) 15 (d) 16
branches in Grindlay’s Bank?
(a) 3 : 4 (b) 2 : 1.5 102. Fujia left Ahmednagar to drive to Bazpur at 6:15
(c) 1 : 2 (d) 4 : 3 p.m. and arrived at 11:45 p.m. If she averaged 30
kmph and stopped one hour for dinner, how far
97. In the case of which of the following pairs of is Bazpur from Ahmednagar?
banks is the total number of branches of each of (a) 120 km (b) 135 km
the two banks the same? (c) 150 km (d) 165 km
(a) UCO and Grindlay’s Bank
(b) State Bank and Grindlay’s 103. The vertex of the square MNOP is located at the
(c) State Bank and City Bank centre of circle O. If arc NP is 4π units long, what
(d) City Bank and Hong Kong Bank is the perimeter of the square MNOP?

7 . 11
Practice Session

108. In the figure below which statement can be true?


(AB is parallel to CD)

A y B
x
z
(a) 32 (b) 32 π C D
(c) 64 π (d) Cannot say (a) y = 90° (b) y = x + z
104. In the triangle below DC is paralled to FE, AD (c) y + z = x (d) x + y + z =180°
= DF, DC = 4 and 109. Excessive amounts of mercury in drinking water
DF = 3. What is FE? associated with certain type of industrial pollu-
tion have been shown to cause Hobson’s Disease.
Island R has an economy based entirely on
subsistence-level agriculture; modern industry of
any kind is unknown. The inhabitants of Island
R have an unusually high incidence of Hobson’s
Disease.
Which of the following can be validly inferred
(a) 5 (b) 6 from the above statements?
(c) 7 (d) 8 I. Mercury in drinking water is actually per-
fectly safe.
Directions: Questions 105 and 106 refer to the
@UPSC_THOUGHTS

II. Mercury in drinking water must have


following table. sources other than industrial pollution.
% of PRO- % of CARBO- % of COST PER III. Hobson’s Disease must have causes other
TEIN HYDRATES FAT 100 GRAMS than mercury in drinking water.
(a) III only
FOOD A 10 20 30 Rs 1.80
(b) I or III but not both
FOOD B 20 15 10 Rs 3.00 (c) II or III but not both
FOOD C 20 10 40 Rs 2.75 (d) II or III or both

105. Which of the following diets would supply the Directions: Questions 110 to 113 are based on the
most grams of protein? following passage.
(a) 250 grams of B
(b) 350 grams of C Professor Kittu’s literature class includes students with
(c) 150 grams of A and 200 grams of B varied tastes in poetry. All those in the class who enjoy
(d) 200 grams of B and 200 grams of C the poetry of Browning also enjoy the poetry of Eliot.
Those who enjoy the poetry of Eliot despise the poetry
106. All of the following diets would supply at least of Coleridge. Some of those who enjoy the poetry of Eliot
75 grams of fat. Which of the diets costs the least? also enjoy the poetry of Auden. Some of those who enjoy
(a) 500 grams of B, 100 grams of A the poetry of Auden despise the poetry of Coleridge. All
(b) 200 grams of C those who enjoy the poetry of Donne also enjoy the
(c) 150 grams of A, 100 grams of C poetry of Frost.
(d) 300 grams of A
110. Guddi enjoys the poetry of Donne. Which of the
107. In a school in which 40% of the enrolled students following must be true?
are boys, 80% of the boys are present on a certain (a) She may or may not enjoy the poetry of
day. If 1152 boys are present, the total enrollment Coleridge
in the school is (b) She does not enjoy the poetry of Browning
(a) 1440 (b) 2880 (c) She enjoys the poetry of Auden
(c) 3600 (d) 5760 (d) She does enjoy the poetry of Eliot

7 . 12
Practice Session

111. Hari enjoys the poetry of Browning. He may also 118. Rajni scored more than Shyam. Priti scored as
enjoy any of the following poets except much as Divya. Sujata scored less than Anju.
(a) Auden (b) Coleridge Shyam scored more than Priti. Anju scored less
(c) Donne (d) Eliot than Divya. Who scored the lowest?
112. Ila enjoys the poetry of Coleridge. Which of the (a) Anju (b) Priti
following must be false? (c) Sujata (d) Shyam
(a) She enjoys the poetry of Donne 119. If the word PEARL is written as MBXOI, then
(b) She enjoys the poetry of Frost how would the word DIAMOND be written in
(c) She does not enjoy the poetry of Browning
that code?
(d) She may enjoy the poetry of Eliot
(a) AFXJLKA (b) AFXJKLA
113. Based on the information provided, which of the (c) AFXKLNA (d) BGYKMKB
following statements concerning the members of
the class must be true? 120. In a certain code 130584 is written 485031. How
(a) All those who enjoy the poetry of Eliot also is 27096 written in that code?
enjoy the poetry of Browning (a) 60972 (b) 69072
(b) None of those who despise the poetry of (c) 67092 (d) 60792
Frost enjoy the poetry of Auden 121. In a certain code language, (i) ‘guda buka’ means
(c) Some of those who enjoy the poetry of ‘clear water’, (ii) ‘pin gola’ means ‘overcast sky’
Auden despise the poetry of Coleridge
and (iii) ‘pin saf buka’ means ‘clear blue sky’.
(d) None of those who enjoy the poetry of
Which word in that language means ‘blue’?
Browning despise the poetry of Donne
(a) Guda (b) Buka
@UPSC_THOUGHTS

114. In a row of children, Raj is fifth from the left and (c) Saf (d) Pin
Ravi is sixth from the right. When they exchange
positions, Raj will be thirteenth from the left. 122. Doctor is related to Diagnosis in the same way
What will be Ravi’s position from the right? as Judge is related to . . . ?
(a) Fourth (b) Fifth (a) Law (b) Judgement
(c) Thirteenth (d) Fourteenth (c) Court (d) Punishment
115. Ramesh walks 20 metres North. Then he turns 123. Three of the following four are alike in a certain
right and walks 30 metres. Then he turns right way and so form a group. Which is the one that
and walks 35 metres. Then he turns left and does not belong to that group?
walks 15 metres. Then he turns left and walks (a) Wall (b) House
15 metres. Then he again turns left and walks 15 (c) Beam (d) Roof
metres. In which direction and how many metres
away is he from the original position? 124. Examine the following statements.
(a) 30 metres towards East I. All men are heavier than women
(b) 15 metres towards West II. Some men are heavier than women
(c) 45 metres towards East III. No men are heavier than women
(d) 30 metres towards West IV. Some men are not heavier than women
Two of these statements cannot be true but both
116. If B says that his mother is the only daughter of
can be false. They are
A’s mother, how is A related to B?
(a) I and IV (b) I and II
(a) Mother (b) Sister
(c) Father (d) Son (c) I and III (d) II and IV

117. If (i) six persons A, B, C, D, E and F are standing Directions: In questions 125 and 126, a statement is
in a circle, not necessarily in the same order, (ii) given followed by two assumptions numbered I and II.
B is between F and C, (iii) A is between E and An assumption is something supposed or taken for
D and (iv) F is to the left of D, which of the granted. You have to consider the statement and both
following is between A and F? the assumptions following it together and decide which
(a) B (b) C of the assumptions is implicit in the statement. Give
(c) D (d) E answer

7 . 13
Practice Session

(a) if only I is implicit; Directions: Questions 133 and 134 are based on the
(b) if only II is implicit; diagram given. The triangle represents educated people,
(c) if both I and II are implicit; the rectangle represents scientists, the circle represents
(d) if neither I nor II is implicit. poets, and square represents politicians.
125. Statement:
Educated
Buy Breezy—the fan with ten years’ guarantee.
Assumptions:
I. People are attracted by longer period of guar-
Scientists
antee.
II. People want to buy fans. Poets

126. Statement:
Politicians
Government has permitted unaided colleges to
increase their fees.
133. Which statement is not true?
Assumptions:
(a) All politicians are educated
I. Unaided colleges are in financial difficulties.
(b) Some politicians are poets but not scientists
II. Aided colleges do not need to increase fees.
(c) Some poets are educated but not politicians
127. The ages of A and B are in the ratio of 6 : 5 and (d) All scientists are educated
sum of their ages is 44 years. What will be the
ratio of their ages after 8 years? 134. Which of the statements can be true?
(a) 5 : 6 (b) 7 : 8 (a) All poets are scientists
(c) 8 : 7 (d) 14 : 13 (b) Some educated politicians are scientists
@UPSC_THOUGHTS

and poets
128. At a railway station one train passes every 2 (c) Some scientists are politicians but not edu-
minutes in Down direction and every 3 minutes cated
in Up direction. How many trains pass in 30
(d) None of them is true
minutes?
(a) 10 (b) 18 Directions: Study the following table and answer ques-
(c) 15 (d) 25 tions 135 and 136 given below it.
129. A coin has heads on both sides. On tossing it, ANNUAL INCOME OF 5 SCHOOLS
the probability of heads is Sources of School
(a) 1/2 (b) 1 income
(c) 1/3 (d) None A B C D E

130. In an examination, 1100 were boys and 900 were Tuition fee 12000 6000 21000 9000 1200
girls. 50% of the boys and 40% of the girls passed Term fee 2400 1200 4500 2400 3000
the examination. The percentage of candidates Donations 5400 2100 6000 5100 6000
that failed is Grants 6000 5400 12000 4200 7500
(a) 45% (b) 45.5% Miscellaneous 1200 300 1500 300 1500
(c) 54.5% (d) 59.2%
27000 15000 45000 21000 30000
131. A discount series of 10%, 20% and 40% is equal
to a single discount of
(a) 50% (b) 56.80% 135. In case of which school, tuition fee forms the
(c) 60% (d) 70.28% highest percentage of total income?
(a) A (b) B
132. The price of cooking oil has increased by 25%. (c) C (d) D
The percentage of reduction, that a family should
effect in the use of cooking oil so as not to change 136. In case of how many schools the income by way
its expenditure on cooking oil, is of tuition fee is less than four times of term fee?
(a) 15% (b) 20% (a) Nil (b) One
(c) Two (d) Three
(c) 25% (d) 30%

7 . 14
Practice Session

137. 143. The average age of the father and his 6 children
is 12 years, which is reduced by 5 years if the
father be excluded. How old is the father?
(a) 42 years (b) 40 years
(c) 48 years (d) 50 years
144. Milk and water are in the ratio of 3 : 2 in a
mixture of 80 kg. How much water should be
added so that the ratio of the milk and water is
AB = BD = DE = EA = 2.5 cm; BX = XD; 2 : 3?
CX = 2.16 cm. The area of the figure given above (a) 40 kg (b) 25 kg
(c) 35 kg (d) 20 kg
with the necessary measurement is
(a) 18.41 sq. cm (b) 15.40 sq. cm 145. How many km per hour does a man walk who
(c) 12.70 sq. cm (d) 8.95 sq. cm passes through a street 600 m long in 5 minutes?
(a) 10 (b) 8
138. The hypotenuse of a right-angled triangle is 10
(c) 6 (d) 7.2
cm and one of its sides is 6 cm. The area of the
triangle is 146. A number exceeds its three-fifths by 14. What is
(a) 24 sq. cm (b) 30 sq. cm the number?
(c) 40 sq. cm (d) 80 sq. cm (a) 23 1/2 (b) 70
(c) 35 (d) 40
139. ABCD is a trapezium in which AB||DC and AB
= 2 CD. If its diagonals intersect each other at O, 147. At the end of three years what will approximately
be the compound interest on Rs 10,105 at the rate
@UPSC_THOUGHTS

then the ratio of areas of the triangles AOB and


of 10% per annum?
COD is
(a) Rs 4,500 (b) Rs 3,000
(a) 1:2 (b) 1 : 4
(c) Rs 3,300 (d) Rs 3,600
(c) 4:1 (d) 2 : 1
148. A boat takes four hours for travelling down-
Directions: The items in questions 140 and 141 given
stream from A to B and coming back to A
below have been represented by sets of circles on the upstream. If the velocity of the stream is 2 km per
basis of their relationship. Your task is to match the hour and the speed of the boat in still water is
right set of items with the right set of circles. 4 km per hour, what is the distance between A
and B?
1. 2. (a) 6 km (b) 8 km
(c) 4 km (d) 9 km
149. Six years ago the ratio of the ages of Kamal and
3. 4. Suresh was 6:5. Four years hence the ratio of their
ages will be 11 : 10. What is Suresh’s age at
140. sheep : dog : wool present?
(a) 1 (b) 2 (a) 20 years
(c) 3 (d) 4 (b) 18 years
(c) 16 years
141. yak : zebra : bear (d) Cannot be determined
(a) 1 (b) 2 1
3 2
(c) 3 (d) 4 150. If x , then the value of x is
3 2 x
(a) 10 (b) 11
142. A travelled a distance of 20 km out of a total
(c) 12 (d) 13
distance of 50 km. What percentage of total
distance has he yet to travel? 151. If 523 + 173 + x3 – 51 × 52x = 0, the value of x is
(a) 30 per cent (b) 25 per cent (a) 26 (b) 32
(c) 60 per cent (d) 40 per cent (c) 38 (d) None

7 . 15
Practice Session

152. In the triangle ABC given below, the length of AD 157. Statements:
(in cm) is Some trees are horses.
Some ships are trees.
Conclusions:
I. Some horses are ships.
II. Some trees are not ships.
(a) Only 1 follows
(b) Only 2 follows
(a) 18 (b) 12 (c) Both 1 and 2 follow
(c) 6 (d) 32 (d) Neither 1 nor 2 follows
153. In the triangle ABC; if AC=6, AD = 5 and BC = 158. Statements:
10, then AB is “You take care of yourself. We will take care of
your money; so save with us”—an advertisement
from a Bank
Assumptions:
1. People read advertisements
2. Catchy slogans appeal to people
(a) Only 1 is implicit
(a) 5 (b) 6 (b) Only 2 is implicit
(c) 7 (d) 8 (c) Both 1 and 2 are implicit
(d) Neither 1 nor 2 are implicit
Directions: Which of the following diagrams given
@UPSC_THOUGHTS

below best represents the relationship between different 159. If the word PENCIL is coded as LICNEP, how
groups given in questions 154 and 155. would the word INKPOT be coded?
(a) TOPINIK (b) JOKQPU
(C) HMKOPS (d) TOPKNI
1. 2.
160. In a certain code language ‘lee su jak’ means ‘he
eats apples’, ‘jee tic’ means ‘sweet oranges’ and
‘lee kee jo tic’ means ‘Mohan eats sweet bananas’.
3. 4. In that language which of the following means
‘oranges’?
154. Stars : Planets : Moons (a) tic (b) jee
(a) 1 (b) 2 (c) kee (d) none of these
(c) 3 (d) 4 161. Three of the following four are alike in a certain
155. Fruits : Bananas : Potatoes way and so form a group. Which is the one that
(a) 1 (b) 2 does not belong to the group?
(c) 3 (d) 4 (a) July (b) May
(c) August (d) January
156. Statement:
All papers are pencils. 162. Which of the following is related to ‘Hygiene’ in
All pencils are erasers. the same way as ‘Plant’ is related to ‘Botany’?
Conclusions: (a) Health (b) Disease
I. Some erasers are papers. (c) Sterilisation (d) Physiology
II. Some pencils are not papers. 163. Pointing to a man, a woman said, ‘His mother
(a) Only 1 follows is the only daughter-in-law of my mother-in-law’.
(b) Only 2 follows How was the man related to the woman?
(c) Both 1 and 2 follow (a) Husband (b) Son
(d) Neither 1 nor 2 follows (c) Brother (d) Uncle

7 . 16
Practice Session

164. A race always has 170. In the figure below, the coordinates of points P
(a) referee (b) spectators and Q are (6, 0) and (0, 6) respectively.
(c) rivals (d) prize
Directions: To answer questions 165 and 166 use the
information given below. Six plays A, B, C, D, E and
F are to be staged one on each day from Monday to
Saturday. The schedule of the plays is to be in accor-
dance with the following:
I. A should not be staged on Friday What is the area of the circle?
II. C should not be staged on the first day of (a) 36 (b) 12
the schedule (c) 6 (d) 3
III. D should be staged on Wednesday only but 171. Three of the following four are alike in a certain
should not be followed immediately by C way and so form a group. Which is the one that
IV. There should be a gap of two days between does not belong to that group?
D and F (a) Actor (b) Artist
V. E should be immediately followed by A (c) Dancer (d) Musician
165. Which of the following is the order of staging the 172. In a row of girls, if Meena who is 10th from the
plays starting from Monday? left and Sarita who is 7th from the right inter-
(a) EADBCF (b) EADCBF change their seats, Meena becomes 15th from the
(c) AEDBCF (d) EADFBC left. How many girls are there in the row?
@UPSC_THOUGHTS

166. Which of the statements given in the information (a) 17 (b) 20


is superfluous for getting the order of staging all (c) 21 (d) 22
the plays? 173. In a certain code TELEPHONE is written
(a) I (b) II ETPELENOH. How is STATEMENT written in
(c) IV (d) V that code?
167. How many triangles and squares are there in the (a) TSTAMENET (b) TSETANEMT
given figure? (c) SATMETTNE (d) TSETATNEM
174. The town of Paranda is located on Green lake.
The town of Akram is west of Paranda. Tokhada
is east of Akram but west of Paranda. Kakran is
east of Bapri but west of Tokhada and Akram.
If they are all in the same district, which town
is the farthest west?
(a) 10 squares, 44 triangles (a) Paranda (b) Kakran
(b) 4 triangles, 16 squares (c) Akram (d) Bopra
(c) 24 triangles, 5 squares
Directions: In questions 175 to 177 a letter series is
(d) 9 squares, 24 triangles
given which has some blanks. Which set of letters when
168. Disease is related to Pathology in the same way placed consecutively one after another in the blanks
as Planet is related to shall complete the series?
(a) Astrology (b) Astronomy
(c) Orbit (d) Milky Way 175. ab-babab-ba-aba-abab
(a) b b a a (b) b a b a
169. If water is called food, food is called tree, tree is (c) a b a b (d) a a b b
called sky, sky is called wall, on which of the
176. a-abcbcb-acaca-a-abc
following grows a fruit?
(a) a c a b (b) b a b b
(a) Water (b) Food
(c) b c b b (d) c b c c
(c) Sky (d) Wall

7 . 17
Practice Session

177. abc-cab-acabc-b-bc 182. Teacher, Principal, Income Tax payers


(a) b c a b (b) b b a a (a) 1 (b) 2
(c) b c a a (d) b c b a (c) 3 (d) 4
178. In the following sequence of instructions 1 stands 183. Administrative Service, I.A.S., I.P.S.
for Run, 2 stands for Stop, 3 stands for Go, 4 (a) 1 (b) 2
stands for Sit and 5 stands for Wait. If the (c) 3 (d) 4
sequence were continued, which instruction will
184. Police, Criminals, Lawyers
come next?
(a) 1 (b) 2
44545345314531245453453 (c) 3 (d) 4
(a) Run (b) Stop
185. A husband and wife have 2 sons. Both sons are
(c) Go (d) Sit
married and have one child each. How many
179. Assuming that the statement ‘Every Library has members are there in the whole family?
books’ is true, which of the following statements (a) 4 (b) 6
is definitely true? (c) 7 (d) 8
(a) Books are only in a library
(b) Some libraries do not have readers 186. A rod of length 5 times its radius is melted. How
(c) Libraries are meant for books only many balls can be made of the same radius?
(d) No library is without books (a) 3 (b) 5
(c) 4 (d) 6
180. Statements:
All tigers are ships, some ships are 187. C’s mother was four times as old as C 10 years
cupboards. ago. After 10 years, she will be twice as old as
@UPSC_THOUGHTS

Conclusion: C. How old is C today?


1. Some tigers are cupboards. (a) 5 years (b) 10 years
2. Some cupboards are tigers. (c) 20 years (d) 30 years
(a) Only conclusion 1 follows
188. If the speed of a train is 92.4 km/hr, then how
(b) Only conclusion 2 follows
many metres are covered by it in 20 minutes?
(c) Both 1 and 2 follow
(a) 30,800 (b) 3,080
(d) Neither 1 nor 2 follows
(c) 308 (d) 30.8
181. Statements:
Be humble even after gaining victory. 189. A watch gains 4 seconds per hour. What time
Assumptions: will it show at 8 a.m. on 22nd of a month, if it
1. Many people are humble after gaining victory. is set right at 1 p.m. on the 20th of the same
2. Generally people are not humble. month?
(a) Only assumption 1 is implicit (a) 8 hrs 2 min. 40 sec.
(b) Only assumption 2 is implicit (b) 8 hrs 3 min. 20 sec.
(c) Both 1 and 2 are implicit (c) 8 hrs 2 min. 52 sec.
(d) Neither 1 nor 2 is implicit (d) 8 hrs 3 min. 12 sec.

Directions: In questions 182 to 184, three classes are 190. 9 persons can plough a field in 25 days. In how
given. Out of the four response figures given below, you many days will 15 persons be able to complete
are to indicate which figure will best represent the the job?
relationship amongst the three classes: (a) 11 (b) 20
(c) 18 (d) 15
191. A square of a number cannot end with
1. 2. (a) 3 (b) 4
(c) 5 (d) 2
x 3 y
192. If , the value of 2x 2 y equals
2y 2 x
3. 4. (a) 1/7 (b) 7
(c) 7.1 (d) none of these

7 . 18
Practice Session

Directions: For Questions 193 and 194 refer the follow- 200. If only the length of a rectangular field is in-
ing Table. creased by 50%, by what per cent will its area
be increased?
Size of Distribution of (a) 20 (b) 25
Agricultural Holdings (c) 30 (d) None of the above
Category Size Number Land
Directions: Each of Questions 201 to 203 is followed by
(hectare) (million) (million
hectares) two statements numbered I and II. You have to decide
whether the data provided in the statements are suf-
Marginal Less than 1 35.7 14.5 ficient to answer the question. Give answer
Small 1 to 2 13.4 19.3 (a) If statement I alone is sufficient to answer
Semi-medium 2 to 4 10.6 30.0 the question and statement II alone is not
Medium 4 to 10 7.8 48.2 sufficient to answer the question
Large 10 and above 2.5 50.0 (b) If either statement alone is sufficient to
Total 70.0 162.0 answer the question
(c) If both statements together are required to
answer the question
193. What percentage of the total farmers constitute
(d) If neither statement is sufficient to answer
the small and marginal farmers?
the question
(a) Nearly 90% (b) Nearly 80%
(c) Nearly 70% (d) Nearly 50% 201. If Arun has filled 92 envelopes with letters, what
per cent of the total letters has he put into the
194. One of the categories of farmers is about 15% of
@UPSC_THOUGHTS

envelopes?
all the cultivators. Which is this category?
I. The total number of letters is 250.
(a) Large (b) Medium
II. The number of letters already put into the
(c) Semi-medium (d) Small
envelopes represents two-fifths of the total
195. What will be the approximate value of 151% of number of letters.
631?
202. If only those people who were given invitation
(a) 950 (b) 1050
cards were to attend the peace concert, how many
(c) 850 (d) 750
people attended it?
196. If a + b + c = 0, then a3 + b3 + c3 is I. 80 people received invitation cards to
(a) 0 (b) 1 attend the peace concert.
(c) a3b3c3 (d) 3abc II. 60 per cent of the people given invitation
197. The square of which of the following numbers is cards attended the peace concert.
not a perfect cube? 203. Rainbow paint contains only alcohol and pig-
(a) 4 (b) 12 ment. What is the ratio of alcohol to pigment in
(c) 16 (d) None of the above Rainbow paint?
198. Three years ago Rama’s age was one-eighth of I. Exactly 70 g of pigment are contained in
her father’s age. Seven years hence, the sum of a 120 g can of Rainbow paint.
their united ages will be 56 years. Rama’s age II. Exactly 50 g of alcohol are contained in a
now is 120 g can of Rainbow paint.
(a) 7 years (b) 8 years 204. You and a co-worker are employed in a complex
(c) 9 years (d) 10 years project that demands hard work from both of you.
199. A man can row with the stream at 8 km per hour Your co-worker is often absent from work as a
and against the stream at 2 km per hour. The result of some personal problems which is caus-
speed of the current is ing him a lot of stress. You have not known your
(a) 2 km/hr (b) 3 km/hr co-worker for long, and you do not know much
(c) 4 km/hr (d) 5 km/hr about his problem. But as a result of his frequent

7 . 19
Practice Session

absence, you are over loaded with the work, as point out the wrong of such activities, but to no
you put in a lot of overtime to keep the project effect. You discuss the matter with the senior
deadline. You are now feeling that your health officer who merely nods and shrugs and says he
may suffer. What would the most effective re- is helpless. In the circumstances, you should
sponse be? (a) follow the behaviour of your senior officer
(a) Continue to put in overtime as it is all as, after all your efforts, there is nothing
important that the project moves to comple- you can do
tion (b) ask for a transfer to some other department
(b) Meet your co-worker and request him to do or city
his share of work (c) resign from the job in despair
(c) Offer help to solve your co-worker’s per- (d) write to higher authorities about the pre-
sonal problem vailing situation and ask for action
(d) Raise the issue with your senior or super-
208. While another person is speaking to you,
visor and request additional help so that
(a) you should not speak
the project is not adversely affected
(b) you should nod or make brief verbal re-
205. You notice your co-worker often using the official sponses
transport for personal business. Your response (c) you should constantly seek clarifications
would be: (d) you should maintain eye contact with the
I. Warn your co-worker that if he did not stop speaker
misusing official transport, you would re-
209. Which of the following sentences is clear and
@UPSC_THOUGHTS

port him to the senior officer.


conveys its meaning?
II. Report your co-worker to the superior of-
(a) Anita took her daughter with her to the
ficer.
shopping mall and while she bought some
III. Remind him that official equipment is not
toys she got lost and was found by the
to be used for personal business.
guards later.
IV. Keep quiet as it is not really your business.
(b) Anita took her daughter to the shopping
Let someone else find out and take action.
mall to buy some toys but she got lost only
(a) III, I, and II in that order
to be found later.
(b) I and III in that order
(c) Anita took her daughter with her to the
(c) II
shopping mall, but while Anita was buy-
(d) IV
ing some toys, the child got lost, though she
206. You are an easy-going person and much given was later found by the guards.
to putting your arm about the shoulders of an- (d) All are correct
other person while talking to him. You meet a
210. Rearrange the following sentences in logical order:
man who frowns when you do this, and seems
I. Strict obedience to these rules is called
uncomfortable with your action. You would
discipline.
(a) explain to him that this is what you do
II. In the same way, a society in which rules
with all people, and there is no harm in it
are not followed cannot survive for long.
(b) withdraw your arm and continue with
III. Only then can a society be run in an
whatever you were discussing
orderly fashion.
(c) laugh and tell him not to be so ‘touch-me-
IV. A society can exist properly only when
not’
people living in it agree upon certain rules
(d) ask him why he is making a face
of conduct.
207. You have recently been transferred to a new V. For example, if the people on the roads do
department and you find the workers mired in not obey traffic rules, there will be complete
corruption. You try to reason with the others and disorder and confusion .

7 . 20
Practice Session

VI. Students must obey their teachers, children III. Tell the old man to seek shelter in an old
their parents, citizens the laws and so on age home.
and so forth. IV. Sympathise , but express your unwilling-
(a) DEBFCA (b) DABFCE ness to interfere in family quarrels.
(c) ABECFD (d) DEBAFC (a) I
(b) II, and III in the meanwhile
211. Pick the right words for the blanks in the sen- (c) I, and II if I proves ineffective
tence: She did not like to . . . her decision on her (d) IV
subordinates like a . . . .
(a) emphasise; boss 215. Which of the following is a drawback of oral
(b) force; tyrant communication?
(c) enforce; bulldozer (a) It is a slower process than written commu-
(d) thrust; dictator nication
(b) It gets immediate feedback
212. Should students get extra time to read the ques- (c) Matters discussed cannot be used as proof
tions in an examination, above the time allotted (d) It is uneconomical
to the examination on the whole?
216. Written communication suffers less than oral
I. The examination involves the ability to
communication does from
read and understand the question, so no
(a) possibility of misunderstanding
extra time should be allowed.
(b) geographical distance
II. The questions are long and take up a lot
(c) noise
of time of the examinees, so there should (d) all the above
@UPSC_THOUGHTS

be extra time.
III. The examination paper is set taking into 217. Consider this exchange of views:
account the time required for reading the Trilochan: The Internet is making more informa-
questions and answering the questions, so tion available to more people than ever
no extra time is required. before in history. So, people can simply
learn all they need to know without seek-
Which of these statements provide strong argu-
ing the advice of experts.
ments in the context of the question?
Paayal: In the past, the need for experts actually
(a) I only (b) I and II
increased as the volume of knowledge
(c) II only (d) I and III
increased. Therefore, the Internet will surely
213. You observe a person smoking in a restaurant, increase our dependence on experts.
though smoking is prohibited there. What would The dialogue most strongly supports the claim
be your response? that Trilochan and Paayal disagree with each
(a) Ask one of the restaurant staff to tell the other about whether
person to stop smoking (a) experts will be increase their reliance on
(b) Go up to the person and tell him (or her) the Internet
to stop smoking (b) the Internet will contribute to the spread of
(c) Call for the manager and demand action expertise in society
(d) Go to another restaurant (c) the Internet will increase the likelihood of
people seeking the advice of experts when
214. An old man comes to you with a grievance
searching for knowledge
against his son who has made him sign off his
(d) explaining knowledge to specialists can
property to the son and then sent the old man
only be accomplished by Internet experts
out of the house. What would you do?
I. Attempt to make the son realise the wrong 218. Which of the following cannot be called a func-
of his action. tion of interpersonal communication?
II. Guide the old man on the action he can (a) Clarifying duties
take through the court. (b) Expressing emotions

7 . 21
Practice Session

(c) Eliciting cooperation in achieving goals 220. In the light of the increasing pollution and its
(d) Issuing press statements detrimental effects on human health and the
219. You are in a room with other colleagues when environment, which of the following courses of
you get a phone call from a senior officer who action would you follow?
gives you some confidential instructions meant I. Write to people in authority to use more
only for you. You need to verify the instructions. funds to clean up rivers than to beautify
(a) You would repeat the instructions to your
city buildings
officer so that you can be corrected if you
II. Run a campaign to get large industries
have got the message wrong
banned
(b) You will tell your senior officer that you
will call back in half an hour to confirm III. Suggest to authorities concerned to resort
the instructions to solar powered illumination
(c) You would request the caller to repeat the (a) I and II
instructions so that you can be sure of them (b) II and III
(d) You would request your senior officer to (c) I and III
call you back on your mobile (d) I, II and III
@UPSC_THOUGHTS

7 . 22
Practice Session

ANSWERS WITH EXPLANATORY NOTES

1. (b) It is a mixed series. 16. (c) We cannot mark (a) or (b) as we do not know
(1) 1, 2, 3, 4 .... T is male or female.
(2) 12, 22, 32, 42
17. (c) 18. (b) 19. (d)
1, 12, 2, 22, 3, 32, 4, 42
1, 1, 2, 4, 3, 9, 4, 16 20. (c) See the diagram
2 3 2
2 . (a) Two series of squares and cubes: 1 , 1 , 2 ,
23, 32, 33, 42
1, 1, 4, 8, 9, 27, 16
3. (a) The first letter is moved two steps forward,
the second letter is moved three steps
forward and the third letter is moved one step
backward to get the next term.
21. (a) 22. (a) 23. (d) 24. (c) 25. (b)
4. (c) For the letters, first letter is moved two steps 26. (a)
forward, the last one step backward. For the
27. (b) See diagram (H = hens, C = cows, D = dogs)
numbers, the series runs:
1 × 1 + 1 = 2; 2×2+2=6
6 × 3 + 3 = 21; 21 × 4 + 4 = 88

3 3
5. (d) The sequence : 2 × × 2 × and so on.
@UPSC_THOUGHTS

2 2
There is nothing to indicate that no dog is
6. (a) c c b a/b b c a/a a b c/c c b a/b a hen, and there are many hens which are
7. (b) 8. (b) 9. (a) 10. (d) 11. (a) neither cows nor dogs.

For Questions 12 to 16, see the chart below. 28. (d) Do not make the mistake of marking (a),
grandfather
because some dogs may or may not be cats
grandmother
(housewife)
S m U (teacher) (D = dog B = bat C = cat)

(engineer)?
Q m P
trader
and
father of
29. (c) We do not know anything from the statements
T R daughter of P about cats being or not being blue. The
student ? student ? diagram makes it clear that only (2) follows.
D = dog, C = cat, B = blue
Note: There are two married couples—who can
only be S-U and Q-P under the given conditions.
P must be engineer; otherwise, she has to be C
housewife or student. But there must be only one D
B B
housewife. If she is a student, either T or R must
be the engineer. While the most likely arrange-
ment is P = engineer; R and T = students, we 30. (a) C = cat B = bat R = rat
cannot say for sure. Now answer the questions.
12. (b) 13. (d)
14. (d) We do not know about the gender of T.
15. (b)

7 . 23
Practice Session

31. (a) You may substitute the values in the equation least number of votes required for a candidate
and get the answer. Or you can solve for to win is 72.
the roots of the equation: 37. (b) In four hours the express train covered 240
2x2 + x – 2 = 1 km—the distance covered by the goods train
2x2 + x – 3 = 0 in 6 hours. The average speed of the goods
(2x + 3) (x – 1) = 0 train was 240/6 or 40 kmph.
So either 2x + 3 = 0, 38. (b) 5 × 40 minutes = 200 minutes
3
2x = –3 and x =
2
= 3 1 hours
3
or x – 1 = 0, and x = 1 = 3 hours 20 minutes
As we are given that x > 0, correct choice
is 1, or (a) So it is 6 : 40 PM, which is 3 hours 20
minutes before 10 P.M.
x 1 1
32. (d) As ,y 2x 39. (c) Babu completes a house and of a second
y 2 3 6
3
x +2 2 house in 8 days. Paintal does or of
= 8 4
y +1 3 a house in 8 days. Together they have done
1 3 25 1
Substituting value of y, 1 + or or 2 houses and of the
3 4 12 12
11
x +2 2
third house. Saigal must do of the third
12 1
=
2x + 1 3 house. In one day Saigal does of a
12 11
Or 3(x + 2) = 2(2x + 1) house. He therefore needs 11 days to do
@UPSC_THOUGHTS

12
3x + 6 = 4x + 2 of the house.
x = 4
40. (b) Rs 2.40 is 30% of the usual selling price.
( x y) 2 ( x y) ( x y) 30
Rs 2.40 × = Rs 8.00 = the usual selling
33. (b) 2 100
x y2 (x y) (x y)
price.
x y
= 41. (c) As 5 cm in the drawing represents an actual
x y
x y 1 length of 1.5 m., a line 10 cm. long in the
This is the reciprocal of
x y 9 drawing represents an actual length of 3m.
x 200 So the cube’s edge actually measures 3 m.
34. (c)
y Total Its volume is 33 = 27 cubic metres.
200 y 42. (d) Since each term is 1/3 the number preceding
or Total =
x it in the sequence, the numbers get smaller
as the sequence progresses. However, as we
35. (a) There are 12 in the group who speak either
are going backwards from 5th to 2nd position,
German or Italian or both. See figure:
the numbers get larger:
5th 4th 3rd 2nd
3 9 27 81
2 3
43. (a) Avinash spent x on rent, leaving x , if x is the
5 5
3 3
4 + 3 + 5 = 12 income for the month. of x is spent on other
4 5
3 can speak both languages. 1 3 3 3
expenses, leaving of x or x=
3 1 4 5 20 20
15 5 Rs 180, and x = Rs 1200.
36. (b) To win, a candidate needs at least one vote 44. (d) 30 paise is 6 per cent of x if x is the taxable
more than votes polled by the others. If you amount, or 30 p = .06x or x = Rs 5. The
divide 356 by 5, you get 71.2. Clearly, the cost of the tax-free items is

7 . 24
Practice Session

Rs 25 – Rs 5 – Rs 0.30 (tax) 54. (d) Let the n observations be x1, x2 ...xn and let
= Rs 19.70 (Do not forget to subtract the xm be their mean
tax also).
x1 x2 ..... xn
xm
45. (a) 91 88 86 78 x 85 n
5 If each observation is divided by a, the terms
343 + x = 425
x1 x2
x = 82 become , and so on. Their mean
1 a a
46. (d) As of 21 workers are men, there are 7 men 1 x1 x2 .... xn
3
1 =
in the office. 7 men must be of the new a n
4 1
staff total, x. = m
a
1
x = 7, x = 28 55. (b) Actual speed = (p – q) km/h
4
Subtracting the current staff total, i.e., 21 Hence 1 km was covered in
from 28 we get 7, which is the number of 1
q hr.
women to be hired. p
As r hours is the time taken,
47. (b) Let the price be Rs 100
1
She purchased it for r = q hours
p
Rs 100 – Rs 20 = Rs 80.
5
She could purchase it for 56. (d) 72 kmph = 72 × m/s = 20 m/s
18
Rs 100 – Rs 25 = Rs 75
Distance travelled in 1 minute
Difference = Rs 5
@UPSC_THOUGHTS

= 20 × 60 = 1200 m
Actual difference is Rs 500
∴ Length of tunnel = 1200 – 700
100 500 = 500 m.
So price of TV =
5 57. (b) AEB = 12, ∴ AE = 8
= Rs 10,000 AGD = 6, ∴ AG = 4
48. (c) Let the age of Prasad = x years Let the age Area of AEFG = 32
of Jaya = x + y years Let the age of Anil Area of ABCD = 72
= x + 2y Area of shaded part = 40
2x + 2y = 48 58. (c) AB = 3 cm + 5 cm = 8 cm
x + y = 24, which is Jaya’s age BC = 5 m + 4 cm = 9 cm
49. (c) Utkal and Nitpal invested in the ratio of AC = 4 cm + 3 cm = 7 cm
7000 : 8500 i.e., 14 : 17. Perimeter of ΔABC
∴ Nitpal’s share in the profit = 8 + 9 + 7
= 24 cm
17
= = 5100 59. (c) As OC = BC and OC and OB are radii, BOC
31 9300
12 is an equilateral triangle
50. (a) × 9800 = Rs 4200 Angle BOC = 60°
28 x
51. (b) Let x be the score in third Test then ∴ x = 120° and = 60°
2
180 258 x 60. (c) Area of circle = πr2 = 9π
230
3 or r2 = 9; r = 3. Clearly a side of the square
x = 252 ABCD is 2r or 6. Area is 36.
52. (b) 11 × 60 – 10 × 65 = 660 – 650 – 10 61. (b) The diagonal of the square ABCD is the
53. (a) S.I. = Rs 1000 diameter of the circle.
Sum = Rs 5000 As OV is 1, the side of the square is 2.
Time = 4 years AB2 + BC2 = AC2
100 1000 or 22 + 22 = AC2
∴ Rate = 5
5000 4 or 8 = AC2

7 . 25
Practice Session

65. (b) Area of the sector of angle x


or 8 = AC or the radius of the circle
xº 120
8 r2 (1)2
= . 360º 360 3
2 66. (a) Area of shaded portion = area of semicircle
Shaded portion is area of circle – area of – area of triangle
2
8 8 a2 2a a2
square – 4 or π – 4 or 2π – 4 a a2
2 4 2 2 2
62. (c) = a2 1
2
63. (a) If you join the midpoints of the circles you get
67. (d) The required area is area of square + area
a square as shown below, with each side =
of 4 semicircles or 2 circles. Area of square
1.
= 4 sq. units
1 2
Area of a semicircle = r
2 2
(as radius = 1)
The area of the entire figure
4
= 4 + 4 2
2
68. (a) The perimeter of the shaded figure is formed
by the curve of four circle quadrants
It is seen that the area of the shaded portion Perimeter of 4 circle quadrants
2 50
is area of square — area of one circle (four = 4 100
@UPSC_THOUGHTS

2
quarters of circles of equal radius).
69. (d) Area of square ABCD
Thus the area of shaded portion
= 6 × 6 = 36
2
1 (As perimeter is 24, each side = 6)
1 or 1 – m
2 4 Area of circle = πr2 = 9π (as radius of circle
64. (b) Join the mid-points of the outer circles to get is half of side of the square = 3)
a square. The diagonal of the square AC = Area of shaded portion is area of square –
radii of two outer circles + diameter of inner area of circle = 36 – 9π.
circle.
70. (b) Required area = area of rectangle – area of
the two circles.
Area of rectangle ABCD
= 4r × 2r = 8r2
(AB = 4 × r; BC = 2r)
Area of two circles = 2πr2
Area of shaded portion
= 8r2 – 2πr2
= 2r2 (4 – π)
From the given data we can say
71. (b)
AC2 = AB2 + BC2
72. (a) Note that each group was reduced by Rs 10.
= (2a)2 + (2a)2 = 8a2
The greatest rate of discount would be for
or AC = 8 a2 2a 2 the group which was originally the least
Diameter of small circle expensive.
= AC – 2a 73. (c) The increase is from .16 to .32, i.e., a change
of .16.
= 2a 2 2a 2a( 2 1)
2a ( 2 1) Change .16
Radius = a ( 2 1) 1 100%
2 Original .16

7 . 26
Practice Session

74. (a) Obviously less of the glucose than lactose is x-axis is 5 units left of origin, therefore
to be used to get .32, as glucose is sweeter negative, and 2 units above origin on y-axis,
than lactose, (a) is the only alternative that i.e., positive; so the values are (–5, 2).
fits the requirement.
86. (d) Use the distance formula. Distance of (4, 4)
75. (b) To make the mixture from origin
1 gm. glucose = .74
= 16 16 32 7
2 gm. sucrose = 2.00
3 gm. fructose = 5.10 Distance of (5, 5) from origin
Total 6 gm. = 7.84 = 25 25 50 7
1 gm. sucrose = 1.00 Distance of (4, 5) from origin
6 gm sucrose = 6.00
= 16 25 41 7
If x is the number of times the mixture is
sweeter than sucrose, 7.84 = 6x or x = 1.3 Distance of (4, 6) from origin

76. (b) 77. (a) = 16 36 52 7


Hence only II and IV are outside the circle.
78. (a) 90° (i.e., sugar) represents 3000 tons. Rice
and wheat form 240° [360 – (90 + 30)] 87. (c) If you draw the diagram, you will note that
240° represents 8000 tons PQ and RS are parallel and equal and hence
it is a parallelogram of base = 9 and height
79. (c) 80. (d) 81. (c) 82. (d) 83. (c)
= 3.
84. (b) We can find the length of PQ by constructing Hence area = 9 × 3 = 27
a triangle: 88. (c) As the point (x, y) is located y units above
x-axis, the width of the rectangle is y. The
@UPSC_THOUGHTS

length of the rectangle is x – 2 as it runs


from point 2 to point x, parallel to the x-axis.
Since width is y and length is x – 2, the area
must be y(x – 2).
89. (c) Follow the change in the sample’s weight step
by step, from the sixth repetition back to the
third. After test :
6 5 4 3
16 32 64 128
Now, we see that QR runs from (5,6) to (5,2)
90. (b)
and so it must be 4 units long. We see that
PR runs from (2,2) to (5,2) so it is 3 units 91. (d) The first seven judges have awarded the
long. We then use the Pythagorean Theorem gymnast a total of 56 points. If she receives
to determine that PQ, which is the hypot- nothing less than a 6 from the other three
enuse of our triangle, is 5 units long. Actually, judges, she will get at least another 18 points,
the ‘distance formula’ for any two points PQ for a total of 74 points. Hence her average
with values P = x, y, and Q = x2, y2, is score will at least be 7.4 (or greater but not
less).
PQ ( x2 x1 )2 ( y2 y1 )2
92. (c) By forfeiting the deposit, she loses 10% of
85. (c) The positive x values are to the right of the Rs 1200, or Rs 120. The other plan is taken
origin (intersection of x and y axes); the at 20% less than Rs 1200, i.e., Rs 1200 –
negative values on x axis are to the left of .20 (Rs 1200) = Rs 1200 – Rs 240 =
origin. The positive y values are above origin, Rs 960. Her total cost will be Rs 960 + the
the negative y values are below x axis. While forfeited deposit of Rs 120, i.e., Rs 1080,
reading the coordinates, the first is always the which is Rs 120 less than the original price
of Rs 1200.
value on the x-axis and the second on the
y-axis. In the present case the value on the 93. (d) Assign an arbitrary number say 100, to rep-

7 . 27
Practice Session

resent the price of the commodity on March sides are proportional. So CD is to EF as AD


1. First the price falls by 1/4 from 100 to 75. is to AF. Since AD = DF, AD/AF is 1/2.
It falls another 1/3 to 50. To return to its Therefore EF is twice CD or 8.
original level, the price must increase by 50,
105. (d) Since food A is 10% protein, 500 grams of
which is 100% of 50.
food A will supply 50 grams of protein. Food
94. (b) One way to tackle this is to assume a concrete B is 20% protein, so 250 gm of food B will
value. Say, X produces 100 units in a certain supply 50 gm of protein. 350 gm of food C
time period. Machine Y would produce four
will supply 70 gm of protein. 150 gm of food
times that, or 400 units, and Machine Z would
A and 200 gm of food B will supply 15 +
produce half of that, or 200 units. The three
40 = 55 gm of protein. 200 gm of food B
machines together would produce 700 units,
and 200 gm of food C will supply 40 + 40
200 2
of which 200 are produced by Z : . or 80 gm of protein.
700 7
95. (c) 150 + 200 + 200 + 225 + 100 = 875 106. (d)
200 107. (c) 80% of the boys = 1152
96. (d) is 4 : 3.
150 1152
Number of boys = %
97. (a) In both cases, it is 350. 80
= 1152 ÷ 0.80 = 1440
98. (c) 99. (d)
1440
100. (d) Let x = first integer Total number of students = %
40
x + 2 = second integer = 1440 ÷ 0.40 = 3600
x + 4 = third integer 108. (c) We extend the lines to give the figure:
@UPSC_THOUGHTS

3(x) = 3 + 2 ( x + 4)
3x = 3 + 2x + 8
x = 11
Third integer is 11 + 4 = 15
101. (c) b = Suraj’s age now
b + 15 = Raju’s age now
b – y = Suraj’s age y years ago
b + 15 – y = Raju’s age y years ago We are aware that x + w = 180°, and
b + 15 – y = 2 (b – y) We know that y + z + w = 180°
b + 15 – y = 2b – 2y So, x + w = y + z + w, and x = 4 + z
15 = b – y.
1 109. (d)
102. (b) Total time elapsed = 5 hours; however, one
2
hour was used for dinner. Therefore Fujia For questions 110 to 113, a circle diagram showing
1 the inter-relationships of the groups named will help.
drove at 30 kmph for 4 hours covering 135
2 Use solid lines to represent groups whose relationship
km. are definitely established. Use broken lines to represent
103. (a) Since MNOP is a square, we know that angle groups about which some ambiguity exists. Your
O must be a right angle. So arc NP is one- diagram should look more or less like this one:
fourth circumference of the circle = 4 × 4π
= 16π units. MN and OP are sides of the
square as well as radii of the circle.
Now, 2πr = 16π
2r = 16, r = 8
So each side of the square MNOP is 8, and
its perimeter is 4 × 8 = 32.
104. (d) Since CD is parallel to EF, the triangles ACD
and AEF are similar. Therefore, corresponding

7 . 28
Practice Session

110. (a) If Guddi is within the circle labelled D (Lovers ‘MBXOI’. Similarly, ‘DIAMOND’ is coded as
of Donne), she may or may not be within the ‘AFXJLKA’.
circle labelled C (lovers of Coleridge). Each 120. (b) The order of writing the digits is reversed.
of the other statements present as definite
fact something which may or may not be true. 121. (c) (i) and (iii) imply that ‘buka’ means ‘clear’ (A)
(ii) and (iii) imply that ‘pin’ means ‘sky’ ....B
111. (b) As the diagram shows, the circle labelled B : (A), (B) and (iii) imply that ‘saf means ‘blue’
has no overlap with the circle labelled C.
122. (b) 123. (b)
Therefore there are no Browning-lovers who
are also Coleridge-lovers. Browning-lovers 124. (c) A : B = 6 : 5, A + B = 44
may also enjoy Auden, Donne, or Frost and ∴ A = 24, B = 20
they definitely enjoy Eliot; since the circle After eight years, A : B = 32 : 28 = 8 : 7
labelled B is entirely within the circle labelled 125. (a) 126. (a) 127. (c) 128. (d) 129. (b)
E.
130. (c) Number of boys passed = 550 Number of girls
112. (d) All the choices are probably true, with the passed = 360
exception of (d). Since the circle labelled C
So out of 2000 candidates, 910 candidates
and the circle labelled E do not overlap, Ila
passed and 1090 failed i.e., 54.5% of the
definitely does not enjoy the poetry of Eliot.
candidates failed.
113. (c) Since some Auden-lovers are Eliot-lovers, 131. (b) Suppose price = Rs 100
and since all Eliot lovers are Coleridge- After 1st discount of 10%, the price
despisers, there must be some Auden-lovers = Rs 90.
who are Coleridge-despisers After 2nd discount of 20%, the price
@UPSC_THOUGHTS

114. (d) Total number of children in the row will be = Rs 90 – 20% of 90 = Rs 72


eighteen. Ravi’s position from the right will After 3rd discount of 40%, the price
be fourteenth. = 72 – 40% of 72
115. (a) = 72 – 28.80 = Rs 43.20
∴ Net discount equivalent to three discounts =
35 m Rs 56.80
20 m Final 132. (b) Let the original expenditure be Rs x per kg.
35 m Let the original consumption
Start
15 m = y kg.
∴ Total expenditure = Rs xy.
15 m
Now increased price = x + 25% of x
116. (c)
5x
= Rs per kg
4
Suppose reduced consumption
= c kg
5c
∴ Total expenditure = Rs
4
c 4y
Thus 5 × = xy i.e. c =
4 5
y
∴ Consumption is reduced by kg as
5
against the original consumption of y kg.
133. (a) 134. (b)
117. (c)
135. (c) Suppose Tuition Fee forms K% of total
118. (c) Rajni > Shyam; Priti = Divya; Sujata < Anju;
income
Shyam > Priti ∴ The value of K in case of: School A
119. (a) Each letter in the word ‘PEARL’ is moved 1200
three steps backward so as to code it as = = 44.44
27

7 . 29
Practice Session

600 ∴ M = 48 and W = 32
School B = = 40 In order that M : W be 2 : 3, W = 72. Therefore,
15
2100 40 kg of water should be added.
School C = = 46.66
45 600 60
900 145. (d) 7.2
School D = = 42.86 5 1000
21
146. (c) Suppose the number is k
1200
School E = = 40 3
30 ∴ k = + 14, i.e., k = 35
136. (b) 5k
3
10
137. (d) Required area = Area of square ABCD + Area 147. (c) 10105 1
100
of ACD
1 11 11 11
= (2.5)2 + × 2.5 × 2.16 = 8.95 10105 × 10105
2 10 10 19
138. (a) The other side of the right angled triangle = = 13449.755 – 10105
= 3344.755 = 3300 app.
(10)2 (6)2 = 8
1 148. (b) The boat travels downstream
∴ Area = × 8 × 6 = 24 sq m. @ 4 + 2 = 6 km/hr and upstream
2
139. (c) @ 4 – 2 = 2 km/hr
Thus in 2 hrs, boat covers the distance of
8 km.
Let distance between A and B be x km.
@UPSC_THOUGHTS

Therefore, in covering 2x km, the boat takes


2 x
2x hours
8 2
x
It is given that in the trapezium ABCD, AB = ∴ = 4 i.e. x = 8
2
2 × CD Then K 6x 6
149. (c) Suppose (At present)
Area of AOB S 6x 6
Area of COD
6x 6 4 11
1 ∴ i.e. x = 2
AB × OM 5x 6 4 10
= 2
1 ∴ Suresh’s age at present
CD × ON
2 = 16 years
1
2 × CD × 2 × ON  AB = 2 × CD, 3 2
= 2 OM = 2 × ON 150. (a) Given x =
1 3 2
CD×ON Therefore
2
2 × CD × 2 × ON 4 1 3 2 3 2
= x
CD × ON 1 x 3 2 3 2

= 4 : 1 3 2 6 3 2 6
= 10
140. (a) All sheep have wool; dog is different 3 2
141. (b) All the items are entirely different. 151. (d) If a3 + b3 + c3 – 3abc = 0
142. (c) 30 km = 60 per cent of 50 km. then a + b + c = 0
143. (a) Total age of the father and six children = x = –69
12 × 7 = 84 years
152. (d) AD2 = BD × DC
Total age of six children ∴ AD2 = 6 × 3
= 7 × 6 = 42 years AD = 18 3 2
∴ Father’s age = 42 years 153. (d) Using AB2 + AC2 = 2(AD2 + BD2) AB2
M 3 + 36 = 2 (25 + 25)
144. (a) M : W = 3 : 2, i.e.,
W 2 AB2 = 100 – 36 = 64
3W
i.e., M = , also M + W = 80 AB = 8
2

7 . 30
Practice Session

154. (d) Area of a circle is πr2 = (6)2π = 36π


155. (a) All bananas are fruits. No potato is a fruit. 171. (b) 172. (c)

156. (a) Since both the premises are affirmative, con- 173. (d) ‘TE LEP HONE’ is coded as ‘ET PEL ENOH’
clusion should be affirmative. Therefore, con- Similarly, ‘ST ATE MENT’ is coded as ‘TS
clusion II cannot follow. Since all pencils are ETA TNEM’
papers, and all papers are erasers, it follows 174. (d) Bopri - Kakram - Akram - Tokhada - Paranda
all pencils are erasers’ too. So “some erasers
are papers”. 175. (d) a b a b/a b a b/a b a b/a b a b/a b a b

157. (d) The middle term is not distributed even once 176. (c) a b/a b c b c b/c a c a c a/b a b a b c
in any of the premises, hence no conclusion 177. (c)
can be drawn.
178. (a) 4/45/453/4531/45312/45/453/4531
158. (c) 159. (d)
179. (d)
160. (b) ‘lee su jak’ means ‘he eats apples’
—(1) 180. (d) Since the middle term ‘ships’ is not distrib-
‘Jee tic’ means ‘sweet oranges’—(2) uted even once in the premises, therefore,
‘lee keejo tic’ means ‘Mohan eats sweet no conclusion can be drawn.
bananas’—(3) (2) and (3) imply ‘tic’ means 181. (b) 182. (a) 183. (b) 184. (a) 185. (d)
‘sweet’
186. (a) Suppose radius = r
∴ ‘jee’ means ‘oranges’
∴ Volume of rod = πr2 (5r) = 5πr3
161. (c) 162. (a) Volume of the ball = 4πr3/3
@UPSC_THOUGHTS

163. (b) Son’s mother is the woman who herself is ∴ No. of balls that can be made of the radius
the daughter-in-law of her mother-in-law.
15 3
= 3 3
164. (c) 4 4
165. (a) EADBCF 187. (c) Suppose C’s age = x
Her mother’s age = y
166. (b) (y – 10) = 4 (x – 10)
167. (a) There are 44 triangles in the given figure. (y + 10) = 2 (x + 10)
∴ x = 20, y = 50
188. (a) In 60 minutes, 92,400 m are covered
In 20 minutes, 30,800 m are covered
189. (c) Total gain = 4 sec. × 4.3
= 172 seconds = 2 min. : 5 sec.
190. (d) 191. (a)
2x 1 1
3
192. (b) 2x y 2y 2 2 7
x 2y x 3
ABC, ABD, ABO, ACD, ADO, AGH, AGJ, AGO, 1 1
2y 2
AJH, AHO, BCD, BCO, BFG, BFK, BFO, BFK,
BGO, CDO, CEF, CEL, CEO, CFL, CFO, DEH, 193. (c) Total no. of farmers = 70 million
DEM, DEO, DHM, DHO, EFG, EFH, EFO, Total no. of small and marginal farmers
EGH, EHO, ELO, EMO, FGH, FGO, FKO, = 49.1 million
FLO, GHO, GJO, GKO, HJO, HMO. 49.1
∴ required % = × 100
168. (b) 169. (c) 70
170. (a) By connecting Q and O or P and O it can = 70.14%
be seen that the radius of the circle is 6 units. 194. (c) % of marginal farmers

7 . 31
Practice Session

35.7 100 199. (b) Speed of current


= 51
70 8 2
= = 3 km/hr.
% of small farmers 2

13.4 200. (d) Suppose L and B be the length and breadth


= × 100 = 19.14 of a rectangle respectively
70
∴ area = L × B
% of semi-medium farmers
3LB
New area = (L+ 50% of L) × B =
10.6 2
= × 100 = 15.14
70 Increase per cent in area = 50
% of medium farmers 201. (b) When per cent is asked, you have to think
7.8 in terms of part and whole. Put the question
= × 100 = 11.14
70 in the form:
% of large farmers
92 x
2.5 ? 100
= × 100 = 3.57
70
Statement I gives the whole – 250. So you
195. (a)
can find the answer. Statement II gives you
196. (d) Now (a = b + c) = 0 . . . Given information that helps you to form the equation
∴ a + b = –c
2 x
Cubing both sides we get:
(a + b)3 = –c3 5 100
∴ a3 + 3a2b + 3ab2 + b3 = –c3 Again, you can get the answer.
@UPSC_THOUGHTS

a + b3 + c3 + 3ab (a + b) = 0
3
So either statement alone is sufficient to
a3 + b3 + c3 + 3ab (–c) = 0
answer the question.
as a + b = –c
a3 + b3 + c3 = 3abc 202. (c) Statement tells us how many people got
invitation cards; it is not necessary that all
197. (d) of them attended the concert. Statement II
198. (a) Let Rama’s age now be x tells us only what per cent of those who got
Three years ago she was x – 3 years, invitation cards attended the concert. Without
x – 3 = 1/8y the total, we cannot say how many attended.
If y was her father’s age So either statement by itself does not provide
or x = l/8y + 3 ... (i) sufficient data to answer the question. But
taken together, we can arrive at the answer:
Her father’s present age is y + 3
we know 80 people got the invitation cards
7 years hence Rama’s age = x + 7
– of which 60 % attended. So we can
and her father’s age = y + 3 + 7 calculate how many actually attended.
By the given condition 203. (b) Statement I gives us the quantity of one of
x + 7 + y + 3 + 7 = 56 the two ingredients in the paint, so we can
or x + y = 56 – 17 = 39 calculate the quantity of the other ingredient
x + y = 39 ..... (ii) and hence the ratio. The same applies to
statement II. Any of them is sufficient to
Substituting the value of x from (i)
answer the question.
in (ii) we have 1/8 y + 3 + y = 39
204. (d) Your health is also important; if you also
or 9y/8 = 39 – 3 = 36
break down, the project is not going to be
8 36 helped in any way. So (a) is not appropriate.
or y= = 32
9 As your co-worker is not obviously being
Substituting the value of y in (ii) we have wilfully uncooperative, but is under stress, (b)
x + 32 = 39 or x = 39 – 32 = 7 years is hardly going to be effective; he may not
be in a position to help his behaviour. As

7 . 32
Practice Session

you do not know him well enough, (c) is not 212. (d)
appropriate. In any case, you do not know if 213. (a) You are in a restaurant, so it is best that you
you can solve his problem. (d) is the best ask one of the staff to take action. Calling the
option. manager would be a later step if the person
205. (a) Even though you have observed your co- does not stop smoking. Of course, you are
worker often misusing the official transport, free to leave the restaurant, but that is just
it is still correct to remind him of his avoiding the situation.
obligations (perhaps he has really forgotten 214. (c)
or does not know about it), and give him fair
warning of the action you intend taking. If he 215. (c) As there is no written record, there is no
persists after that, you must report him. (d) evidence that the conversation took place. Of
is the worst response. course, this can be circumvented with the use
of a tape recorder.
206. (b) You have to respect the other’s wish for
personal space, and not invade it, especially 216. (d) 217. (c) 218. (d)
when the other apparently does not like it. 219. (c) You cannot do (a) in the light of the
The other responses show you to be either confidentiality of the instructions. Options (b)
self-centred or insensitive. and (d) are not appropriate when you are
207. (d) 208. (b) 209. (c) 210. (d) dealing with a senior officer under the given
conditions.
211. (d) The first part of option (c) is alright, but the
second is not apt. 220. (c)
@UPSC_THOUGHTS

7 . 33
Practice Questions

PRACTICE SET 2
1. A sum of money is divided among three persons same colour. The three colours used for painting the
in the ratio of 4 : 6 : 9. If the largest share is cube are red, blue and green. The cube is then cut into
Rs 1,000 more than the smallest share, what is 64 smaller identical cubes. Answer questions 5 to 7
the total sum? based on the above information.
(a) Rs 4000 (b) Rs 9500 5. How many cubes are coloured on three sides?
(c) Rs 3600 (d) Rs 3800 (a) 4 (b) 16
2. A 70 cm long wire is to be cut into two pieces (c) 8 (d) 32
such that one piece will be 2/5 as long as the 6. How many cubes have only one coloured face?
other. How many centimetres will the shorter (a) 0 (b) 8
piece be? (c) 16 (d) 24
(a) 28 (b) 20
(c) 14 (d) 10 7. How many cubes have only two painted sides?
(a) 32 (b) 28
Directions: In questions 3 and 4 if the faces shown in (c) 20 (d) 16
the figure are folded to form a cube, in which of the
cubes shown below can it result? 8. From a given piece of cloth 10 cm x 10 cm the
maximum number of 3cm x 3cm pieces that can
3. be cut out would be
@UPSC_THOUGHTS

(a) 10 pieces (b) 9 pieces


(c) 12 pieces (d) 6 pieces
9. In a certain code ‘CONVENTIONAL’ is written
‘NOCENVIOTLAN’. How is ‘ENTHRONEMENT’, writ-
ten in the code?
(a) TNEROHEMNTNE (b) TNEORHMENTNE
(c) NTEROHEMNTE (d) TNEROHEMNNTE
10. In a certain shipment of 120 new cars, 2/3 of the
cars were equipped with radios and 2/5 with
airconditioners. If 20 of the cars are equipped
(a) (b) (c) (d) with neither a radio nor an airconditioner, how
4. many cars in the consignment are equipped with
both radio and airconditioners?
(a) 20 (b) 32
(c) 56 (d) 28
11. All of the senior students of a school are members
of the dramatic club, debating club or both. If 90
are in the debating club and 70 in the dramatic
club, how many are in both clubs?
(a) 30 (b) 40
(c) 20 (d) 25
12. If A is the brother of B, C is the daughter of B,
D is the sister of E, and F is the brother of C, then
(a) (b) (c) (d) who is uncle of F?
Directions: The six faces of a 4-inch cube are painted (a) A (b) B
in such a manner that no two adjacent faces have the (c) E (d) D

7 . 34
Practice Questions

13. Which is the third number to the left of the 19. Statements:
number which is exactly in the middle of the Some cooks are lazy. All boys are lazy.
following sequence of numbers? Conclusions:
1 2 3 4 5 6 7 8 9 2 4 6 8 9 7 5 3 1 9 8 7 6 5 1. Some boys are cooks.
4 3 2 1 2. Some cooks are boys.
(a) 7 (b) 6 (a) Only 1 follows
(c) 5 (d) 4 (b) Only 2 follows
14. Vijay starts walking straight towards east. After (c) Both 1 and 2 follow
walking 75 metres he turns to the left and walks (d) Neither 1 nor 2 follows
25 metres straight. Again he turns to left, walks 20. Statements:
a distance of 40 metres straight, again he turns
Delink degress with jobs. Then boys will think
to left and walks a distance of 25 metres. How
twice before joining colleges.
far is he from the starting point?
(a) 35 metres Assumptions:
(b) 50 metres 1. Boys seek college education for getting jobs.
(c) 25 metres 2. A degree is of no use in getting a job.
(d) None of the above (a) Only 1 is implicit
(b) Only 2 is implicit
15. Profit is related to loss as
(a) Success is related to failure (c) Both 1 and 2 are implicit
(b) Whole is related to part (d) Neither 1 nor 2 is implicit
(c) Adult is related to child Directions: In Questions 21 to 26 each item is repre-
@UPSC_THOUGHTS

(d) Rupee is related to paise sented by a circle. Your task is to match the right parts
16. P, Q, R and S are playing a game of carrom. P, on the basis of the relationship among the words in
R, and S, Q are the partners. S is to the right of each question.
R who is facing west. In which direction Q is
facing? A. B.
(a) East (b) West
(c) North (d) South
17. One-third of the students in a class are Marathi- C. D.
speaking. One-half of the students know Hindi.
The number of Gujarati-speaking students is
twice that of Marathi-speaking students. Two-
thirds of the students are girls. Which of the E. F.
following statements is definitely true?
(a) All Marathi-speaking students are boys
21. Water : Atmosphere : Hydrogen
(b) A half of the Marathi-speaking students
(a) A (b) B
know Hindi
(c) Some Gujarati-speaking students know (c) D (d) E
Hindi 22. Shirt : Collar : Pocket
(d) All Gujarati-speaking students are girls (a) E (b) D
18. Some chairs are rabbits. A table is a chair. So we (c) A (d) B
may conclude: 23. English : Latin : Greek
1. All rabbits are chairs (a) A (b) F
2. Table is not a rabbit. (c) C (d) D
(a) Only 1 follows
(b) Only 2 follows 24. Week : Day : Year
(c) Both 1 and 2 follow (a) F (b) E
(d) Neither 1 nor 2 follows (c) C (d) A

7 . 35
Practice Questions

25. Frame : Model : Painting included the average will be increased by 500
(a) A (b) B grams. Find the weight of the teacher.
(c) E (d) F (a) 75 kg (b) 68 kg
(c) 70 kg (d) 77 kg
26. City : Children : State
(a) F (b) E 36. Marked price of an article is 25% above cost price
(c) D (d) C what will be gain % after allowing a discount of
12% to the customer?
27. Which number should replace question marks in (a) 20 (b) 15
? 96 (c) 10 (d) 8
54 ?
37. Two men undertake to do a piece of work for Rs
(a) 72 (b) 27
200. One alone can do it in 6 days, the other in
(c) 36 (d) 63 8 days. With the help of a boy they finish it in
28. If 7 7 9
1 , which of the following should
3 days. How much is the share of the boy?
8 * 16 (a) Rs 45 (b) Rs 40
be in place of the asterisk?
(c) Rs 30 (d) Rs 25
(a) 2 (b) 5
33 38. The compound interest on a certain sum for 2
(c) 3 (d) 16 years is Rs 40.80 p and the simple interest is Rs
24
40. Find the sum
29. The area of four walls of a room whose length, (a) Rs 500 (b) Rs 400
width and height are l, w and h respectively will (c) Rs 300 (d) Rs 450
be
@UPSC_THOUGHTS

39. A certain sum lent out at S.I. amount to Rs 575


(a) l2 + w2 + h2 (b) 2l + 2w + h
in 3 years and to 625 in 5 years. Find the rate
(c) 2 (l + w) h (d) 2 lwh per cent
25 (a) 4% (b) 4.5%
30. ?
(c) 5% (d) 5.5%
0.25
(a) 0.1 (b) 1 40. In an enclosure of a zoo there are some rabbits
(c) 10 (d) 100 and pigeons. If their heads are counted they are
90 while their legs are 224. Find the number of
31. 25 – [6+ {11 – (11 – 4)}]
pigeons.
(a) 15 (b) 5
(a) 22 (b) 44
(c) 20 (d) 23
(c) 34 (d) 68
32. A man runs around a circle of radius 50 m at
41. 20% of a number when added to 20 becomes the
a speed of 12 km/hour. Find the time taken by number itself. Find the number.
him for going around it ten times, (π = 3.14) (a) 20 (b) 25
(a) 10 min. (b) 12 min. (c) 50 (d) 80
(c) 15 min. (d) 15.7 min.
42. In the given diagram, circle represents strong
33. What % is 13 of 20? men: square represents tall men; triangle repre-
(a) 65 (b) 75 sents Army officers. Which region represents
(c) 95 (d) None of these Army Officers who are tall but not strong?

34. Three cubes whose edges measure 3 cm, 4 cm and


5 cm respectively to form a single cube. Find the
surface area of the new cube.
(a) 216 cm2 (b) 126 cm2
(c) 226 cm2 (d) 316 cm2
35. The average weight of students in a class of 35 (a) 1 (b) 2
(c) 3 (d) 4
is 50 kg. If, however, the weight of the teacher be

7 . 36
Practice Questions

43. Money doubles itself in five years. A man invests fond of G, and a person who likes F also likes
Rs 5,000 in each of the years 1960, 1965 and 1970. E. The paper-setter of a particular year is very
The amount he gets in 1980 is fond of F but does not like to set a question on
(a) Rs 65,000 (b) Rs 75,000 him as he has written a book on him (F). The
(c) Rs 1,40,000 (d) Rs 1,25,000 question set the year before that was on the poet
A. The poet on whom the question is most likely
44. The average height of the students in a class of
to be set this particular year is
strength 10 is 105 cm. 20 students with average
(a) F (b) E
height of 120 cm are admitted into the class. What
(c) G (d) H
would be the new average height?
(a) 115 cm (b) 112 cm 51. How many cubes are there in the diagram?
(c) 110 cm (d) 105 cm
45. A man spends a fixed amount per month for his
petrol consumption, as follows:
Price (Rs): 1.50 2.00 3.00 4.50 6.00
Litres : 60 45 30 20 ?

What is the amount he will buy if price is Rs 6.00 (a) 8 (b) 9


per litre? (c) 10 (d) 13
(a) 10 (b) 12.5
52. A cube is painted on all sides. If it is cut into
(c) 15 (d) 16
several more cubes so as to have the side of the
@UPSC_THOUGHTS

46. A man has to walk 6 km to cover all the bound- new cubes exactly 1/4th of the original cube, how
aries of a rectangular farm of area 2 sq. km. What many of these cubes will have one side only
is the difference between the two sides of the painted?
farm? (a) 16 (b) 24
(a) 0 km (b) 1 km (c) 12 (d) 32
(c) 2 km (d) 0.5 km
53. A dice having numbers 1 to 6 on its 6 sides is
47. Continue the series 4, 196, 16, 144, 36, 100, 64, . . . shown at various angles in the diagram. Which
(a) 48 (b) 64 is the number opposite to 3?
(c) 125 (d) 256
48. The code for certain letters are indicated in the
following words. What is the code for D?
BRAIN 12345 GRADE 72308
DRAIN 02345 STATE 78388 (a) 1 (b) 2
(a) 0 (b) 1 (c) 5 (d) 6
(c) 2 (d) 9
54. How many different triangles can be identified in
49. Point out the odd one. the given diagram?
(a) S O K I (b) R N J F
(c) T P L H (d) N J F B
50. A question must be set on one of the eight poets
ABCDEFGH every year. Of these, the first 4 are
considered to be medieval and the last 4 modern.
The general trend is that if the question is set on
the first group in one year, then the following
year the question is set from the other group. (a) 5 (b) 8
Generally, if one is fond of H, then one is also (c) 10 (d) 14

7 . 37
Practice Questions

55. Which of the following statements can be con- Directions: Figure for Questions 58 and 59.
cluded from the graph given?

B
Distance

A Time D C

(a) The man covers the distance BD in time AC 58. What is the maximum number of bus routes
(b) The man walks a certain distance with a possible from X to Y so that the bus does not come
certain speed and comes back to the starting to one junction more than once in a route?
point at a higher speed (a) 4 (b) 6
(c) The man walks a certain distance at a given (c) 8 (d) 9
speed and returns to the starting point at a 59. Which is the most crowded junction, assuming
lower speed that each arrow denotes equal traffic?
(d) The distance travelled by the man is indi- (a) U (b) X
cated by area of the diagram ABC (c) Y (d) Z
56. ABCD and WXYZ are two square card-boards of 60. If × is + ; < is – ; > is × ; + is +; + is > ; = is
size 2a. Square ABCD is fixed and square WXYZ
@UPSC_THOUGHTS

< and – is = , which of the following gives true


is fixed at W to the centre of the other square and relationship?
can rotate about W in the same plane. Now what
(a) 3 × 2 × 4 = 6 + 3 < 2
is the relation between distance BO and area
(b) 3 + 2 × 4 = 6 < 3 + 2
WOCP?
(c) 3 + 2 < 4 + 6 > 3 × 2
B (d) 3 × 2 < 4 + 6 + 3 < 2
A X
61. Ax = b, by = c, cz = a, then xyz is equal to:
O (a) –1 (b) 0
(c) 1 (d) a + b + c
W Y
D P C 62. Which of the following diagrams correctly repre-
sents elephants, lions, animals?
Z
(a) As BO increases, area decreases (a) (b)
(b) As BO increases, area increases
(c) As BO changes, area remains constant
(d) No definite relation can be established be-
tween the two
(c) (d)
57. The population of a town at different time inter-
vals are shown below. Point out during which
period was the population increase maximum? 63. A gives half of his salary to his wife, half of
remaining salary to his son and 1/3 of what then
Year : 1940 1950 1960 1970 1980
remains to his daughter. He is left with Rs 500.
Population: 40,000 50,000 60,000 72,000 85,000
What is his salary?
(a) 1940-50 (b) 1950-60 (a) Rs 1200 (b) Rs 1500
(c) 1960-70 (d) 1970-80 (c) Rs 3000 (d) Data insufficient

7 . 38
Practice Questions

64. Two pipes A and B can fill a tank in 20 and 30 (a) Skilled workers (b) Unskilled workers
minutes respectively. If both pipes are used to- (c) Agriculturists (d) None
gether, how long will it take to fill the tank?
69. All good athletes want to win. All good athletes
(a) 50 minutes (b) 12 minutes
eat well. Therefore:
(c) 25 minutes (d) 15 minutes
(a) All those who eat well are good athletes
65. A goes from X to Y in half an hour at a speed (b) All those who win eat well
of 40 kmph. He goes from Y to Z in half an hour (c) Some who eat well are good athletes
at the speed of 60 kmph. His average speed from (d) All who want to win should eat well
X to Z is:
(a) 100 kmph (b) 50 kmph 70. The cumulative expenditure on food items and
(c) 45 kmph (d) 75 kmph other items for a family during January-May in
a year is plotted in the following graph.
66. In the following graph which period shows
maximum growth in industrial production?

expenditure
Cumulative
s
I tem
h er m
s
Ot ite
od
Fo
@UPSC_THOUGHTS

From the graph one can conclude that the maxi-


mum total expenditure for both the items oc-
(a) 1960-65 (b) 1965-70 curred in the month of
(c) 1970-75 (d) 1975-80 (a) February (b) March
67. In the figure below, AB is parallel to CD. What (c) April (d) May
is angle x? 71. If n is an integer between 20 and 80, then any
of the following could be n + 7 except
(a) 47 (b) 58
(c) 84 (d) 88
72. The following graph illustrates the paths of two
motorists A and B in driving from P to Q. From
(a) 60° (b) 50°
this graph we can conclude that
(c) 70° (d) Not known
68. In the graph below which category shows a 10% 200 km
increase in income over the period shown?
Distance in Km

400 A
1960
1980 of
th
300 Pa
B
of
h
200 P at

100 Time in hours

(a) Average speed of A is greater than that of B


Labour
Skilled

Labour
Agricultural
Unskilled
Labour

(b) Average speed of B is greater than that of A


(c) Average speed of A is the same as that of B
(d) Speed of A is always greater than that of B

7 . 39
Practice Questions

73. In the following figures the area of triangle ABC 76. A sheep S is tied by two ropes each 15 metres
is 1 sq unit. If BC is equal to a side of the hexagon, long to two points P and Q which are 20 metres
the area of the hexagon DEFGHI is apart.
S

15
15

m
P Q
20 m
(a) 3 (b) 4
(c) 5 (d) 6 Which one of the following shaded regions illus-
trate the regions over which the sheep can move?
74. Below is the time-velocity graph of two vehicles
A and B starting at the same time from rest.
(a) (b)
Which one of the following statements can be
deduced from the graph as correct?

(c) (d)

77. In the following figure, ABCD is a rectangle in


Velocity

@UPSC_THOUGHTS

a horizontal plane. E and F are vertically above


D and C respectively.

Time

(a) Acceleration of A is higher than that of B


(b) Acceleration of B is higher than that of A
(c) Acceleration of A is increasing at a slower
rate than that of B
(d) Velocity of B is higher than that of A
The length of BE is
75. The following diagram shows a box tied with a (a) 200 cm (b) 250 cm
ribbon. Assuming an extra 10 cm for tying the (c) 300 cm (d) 350 cm
knot, the minimum length of the ribbon required
78. A grocer brought 10 kg of apples for Rs 81 out
to tie the box as shown would be
of which one kg was found rotten. If he wishes
to make a profit of 10%, he should sell it @ . . .
20

per kg.
cm

(a) Rs 9.00 (b) Rs 9.90


(c) Rs 10.10 (d) Rs 10.20
2 0 cm

79. A wheel that has 6 cogs is meshed with a larger


wheel of 14 cogs. When the smaller wheel has
made 21 revolutions, the number of revolutions
30 cm made by the larger wheel is
(a) 200 cm (b) 190 cm (a) 12 (b) 49
(c) 180 cm (d) 160 cm (c) 9 (d) 4

7 . 40
Practice Questions

80. The following table shows the marks obtained by 85. In the rectangle below, what is the ratio of the
two students in different subjects. area of the shaded region to the area of the
Student Maximum Student Maximum
unshaded region?
A marks B marks
English 60 100 80 150
Psychology 70 100 70 100
History 50 100 60 100
Sanskrit 30 50 15 25

The difference in the mean aggregate percentage


marks of the students is 1 1
(a) (b) (c) 1
(a) 2.5% (b) 13.75% 4 2
(c) 1.25% (d) Zero (d) Cannot be determined

81. A train 270 metres long is moving at a speed of 86. If the price of an item is increased by 10% and
25 kmph. It will cross a man coming from the then decreased by 10%, the net effect on the price
opposite direction at a speed of 2 kmph in of the item is
(a) 36 seconds (b) 30 seconds (a) an increase of 99%
(c) 26 seconds (d) 20 seconds (b) an increase of 1%
(c) no change
82. A pencil costs 5 paise. If they are sold at 20% (d) a decrease of 1%
profit what is the selling price of a dozen such
pencils? Directions: Use the following table for Questions 87
(a) 60 paise (b) 72 paise to 89.
@UPSC_THOUGHTS

(c) 80 paise (d) 1 Rupee Speed of a train over a 3-hour period


83. ABCD is a square and the area of the triangle BCE Time Period 0 30 45 60 90 120 150 180
is 8. (in minutes)
Speed at Time 40 45 47.5 5 0 55 60 65 70
(in kmph)
1
87. How fast was the train travelling 2 hours after
2
the beginning of the time period?
(a) 50 kmph (b) 55 kmph
What is the area of the square ABCD? (c) 60 kmph (d) 65 kmph
(a) 4 (b) 8 88. During the three hours shown on the table the
(c) 32 (d) 16 speed of the train increased by
84. In the figure, AB=BC and angles BAD and BCD (a) 25% (b) 50%
are right angles. (c) 75% (d) 125%
89. At time t measured in minutes after the beginning
of the time period, which of the following gives
the speed of the train in accordance with the
table?
1
(a) t (b) 10t
6
Which one of the following conclusions may be (c) 40 + t (d) 40 + 1 t
drawn? 6
(a) AC = CD 90. New problems require new solutions. And new
(b) AD = CD problems arise with new populations and new
(c) BC is shorter than CD technologies. The solutions of these problems
(d) None of these require new institutions as well as new political,

7 . 41
Practice Questions

economic and social mechanisms. Yet institu- to do. Office workers grow increasingly produc-
tions and political and economic arrangements tive as the temperature drops, so long as it does
grow slowly and die slowly. Because old insti- not fall below 68° F.
tutions die slowly, new institutions should be
The passage leads most naturally to which of the
given every chance of success.
following conclusions?
The writer of the passage makes the assumption (a) The temperature-efficiency formula is appli-
(a) If there were no growth, old institutions cable to all kinds of work
would die more slowly (b) Employees are most efficient when the tem-
(b) Socio-technological change requires new perature is 68°F
forms of institutional arrangements (c) Office workers will be equally efficient at 67°
(c) New institutions are needed because the old F and 69° F
institutions are inefficient (d) Some efficiency gains will be short-term only
(d) As old institutions are phased out, new ones
94. The following information was given at a bus
would die more slowly
station. “The bus for Surat left 15 minutes before.
Directions: Read the following and answer Questions As per schedule, after every 45 minutes there is
91 and 92. a bus for Surat. The next bus is at 8.30 A.M.” At
which time was this information given?
From amongst six girls A, B, C, D, E and F and
(a) 9.15 A.M. (b) 9.00 A.M.
five boys P, Q, R, S and T, a team of six is to be
(c) 7.45 A.M. (d) 8.00 A.M.
selected under the following conditions :
A and D have to be together. 95. The accused is more powerful than the dead. The
@UPSC_THOUGHTS

B cannot be teamed with E. police is less powerful than the court but more
C and Q have to be together. powerful than the lawyer. The accused bows his
D cannot go with P. head in the presence of the police. Who is the
C cannot go with S. most powerful?
B and R have to be together. (a) Police (b) Lawyer
S and T have to be together. (c) Accused (d) Court
91. If four members including E have to be girls, the 96. Statements:
members other than E are Some doctors are fools. Maya is a doctor.
(a) A B C Q R Conclusions:
(b) A C D F Q I. All fools are doctors.
(c) A D F S T II. Maya is a fool.
(d) B C F Q R (a) Only conclusion I follows
(b) Only conclusion II follows
92. If four members have to be boys, the members of
(c) Both conclusions I and II follow
the team are
(d) Neither conclusion I nor conclusion II fol-
(a) B C P Q R S
lows
(b) B C P Q R T
(c) B F P R S T Directions: In the following diagram, triangle represents
(d) B C Q R S T the Christians, square represents the Hindus and circle
the Muslims. Each part is numbered. Study the figure
93. Efficiency experts will attempt to improve the
and answer Questions 97 and 98.
productivity of an office by analysing production
procedures into discrete work tasks. They then
study the organisation of those tasks and advise
production, such as rescheduling of employee
breaks or relocating various equipment such as
the copying machines. I have found a way to
accomplish increases in efficiency with much less

7 . 42
Practice Questions

97. Choose the number that represents the area in- 103. Two boys came to a doctor.
habited by all three religions? “I have swallowed a marble,” said one boy.
(a) 2 (b) 4 “I see”, said the doctor, “and is this your brother
(c) 5 (d) 6 with you?”
“No”, said the other boy, . . .
98. Which communities occupy area indicated by 5?
I. “I own the marble.”
(a) Muslims and Hindus
II. “I saw him swallowing it.”
(b) Hindus and Christians
(c) Christians and Muslims 104. Study the following table:
(d) All three
Distribution No. In Population
99. A swimmer can swim 10 km/hr in a stationary
Having X 25
river. If the river is flowing at the rate of 2 km/
Having Y
hr, the time taken by him to travel 12 km from
one place to another along the river and come Having X 10
back would be . . . hours. Lacking Y
(a) 4 (b) 3.6
Lacking X 25
(c) 3 (d) 2.5
Having Y
100. In a group of 60 people, 40 take tea but not coffee
Lacking X 40
and 15 take coffee but not tea. How many of them
Lacking Y
take tea and how many coffee respectively?
(a) 45, 15 (b) 55, 20
The table gives the distribution of two genetic
@UPSC_THOUGHTS

(c) 45, 20 (d) 35, 25


characteristics, X and Y, in a population of 100
Directions: In each of the questions 101 to 103 an subjects. What is the ratio of number of subjects.
incident is described but the ‘punch line’ or climax is What is the ratio of number of subjects having
missing, indicated by dotted lines. The ‘punch line’ X to the number of subjects having Y?
would give the incident an unexpected twist making (a) 7 : 5 (b) 1
the incident funny. So study the two alternatives pro- (c) 7 : 10 (d) 5 : 7
vided after each incident and answer
105. Triangle ABC is inscribed in a semicircle:
(a) if you think I fits best
(b) if you think II fits best
(c) if you think both I and II fit equally well
(d) if you think neither I nor II fits
101. The cleaning lady of a bank gave notice saying
to the manager: “You don’t trust me.”
The area of the shaded portion is
The branch manager replied: “How can you say
(a) 2π – 2 (b) 2π – 4
that? I even leave the keys of the safe lying
(c) 4π – 4 (d) π – 4
around.” Said the cleaning lady. . .
I. “That’s true, but there is no cash in it.” 106. A figure that can be folded over along a straight
II. “That’s true, but none of them fits.” line so that the result is two equal halves which
are then lying on top of one another with no
102. One afternoon a customer came into the shop
overlap is said to have line of symmetry. Which
hurriedly, and said, “A mousetrap, please. Quickly
one of the following has only one line of sym-
as I need to catch the next bus.”
metry?
The shopkeeper replied . . .
(a) Circle
I. “But there are no mice in the bus.”
(b) Square
II. “I am sorry, we do not have that big a mouse-
(c) Isosceles triangle
trap.”
(d) Rectangle

7 . 43
Practice Questions

Directions: In Questions 107 and 108 each statement is 110. Which of the following could be the measures of
followed by two numbered facts. You are to determine the sides of a single triangle?
whether the data given in the statements is sufficient I. 3, 4, 5 II. 12, 5, 18
for answering the question. Use the data given, plus III. 3, 3, 3
your knowledge of maths and everyday facts, to choose (a) I only (b) II only
between the four possible answers. (c) I and II (d) I and III
(a) If statement (1) alone is sufficient to answer
the question, but statement (2) alone is not 111. Sixteen cylindrical cans, each with a radius of 1
sufficient unit, are placed inside a cardboard box four in
(b) If statement (2) alone is sufficient to answer a row. If the cans touch adjacent cans and/or the
the question but statement (1) alone is not walls of the box which of the following could be
sufficient the interior area of the bottom of the box in square
(c) If either statement by itself is sufficient to units?
answer the question (a) 16 (b) 32
(d) If not enough facts are given to answer the (c) 64 (d) 128
question
112. Six blocks shown below are to be rearranged so
107. Rita’s class in history is held once each week on that the letters are in alphabetical order, reading
Thursday afternoons. If it was held every Thurs- from left to right.
day in the month of May, how many times was CBEFDA
it held in that month?
What is the minimum number of blocks that must
@UPSC_THOUGHTS

1. There were five Wednesdays in the month.


be moved to arrive at the desired arrangement?
2. The seventeenth of May was a Friday
(a) 2 (b) 3
108. Is quadrilateral PQRS a square? (c) 4 (d) Cannot say
113. An express train travelled at an average speed of
100 kilometres per hour, stopping for 3 minutes
after every 75 kilometres. How long did it take
to reach its destination 600 kilometres from the
starting point?
1. PR = SQ (a) 6 hr. 21 min.
2. All the interior angles are equal (b) 6 hr. 24 min.
(c) 6 hr. 30 min.
109. Some judges are members of the club. No member
(d) 6 hr. 27 min.
of the club is a convicted felon. Therefore, some
judges are not convicted felons. Which of the 114. The graph below shows the distribution of marks
following is logically most similar to the argu- of a certain class.
ment developed above?
(a) The Taj Mahal is beautiful. The Taj Mahal
No. of students

20
is in Agra. Therefore, Agra is beautiful 15
(b) All men are excluded from the women’s 10
hostel, but some men are polite. Therefore
5
some polite men are not allowed in the
women’s hostel. 70 80 90 100
(c) Anyone who jogs in the heat will be sick.
I do not jog in the heat and therefore not What was the average marks of the class?
likely to be sick (a) 75 (b) 78
(d) None of the above (c) 85 (d) 87

7 . 44
Practice Questions

115. If the figure below is a rectangle, what is the area 119. Sameer is shorter than Salma. Paul is taller than
of PQRS? Sameer. Sudin is taller than Salma but shorter
than Hasan. Salma is taller than Paul. Who will
be in the middle if they stand in a row according
to height?
(a) Sameer (b) Salma
(c) Paul (d) Sudin
(a) 4 (b) 5
(c) 6 (d) 10 120. Effort : Success : :
(a) Health : Vitamin
Directions: Questions 116 and 117 are based on this (b) Climb : Mountain
passage. (c) Reading : Knowledge
A behavioural psychologist interested in animal behav- (d) Write : Read
iour noticed that dogs who are never physically disci- Directions: In Questions 121 and 122 two statements
plined (e.g., with a blow from a rolled-up newspaper) are given followed by two conclusions I and II. Taking
never bark at strangers. He concluded that the best way the statements to be true even if they seem to be at
to keep a dog from barking at strange visitors is to not variance from commonly known facts, you are to decide
punish the dog physically.
which of the given conclusions logically follows from
116. The psychologist’s conclusion is based on which the two statements.
of the following assumptions? (a) if only conclusion I follows;
I. The dogs he studied never barked (b) if only conclusion II follows;
II. Dogs should not be physically punished (c) if either conclusion I or II follows;
@UPSC_THOUGHTS

III. There were no instances of an unpunished (d) if neither conclusion I nor II follows.
dog barking at a stranger which he had
121. Statements:
failed to observe
All lemons are mangoes. No mango is cheap.
(a) I only (b) II only
Conclusions:
(c) III only (d) I and III
I. All mangoes are lemons.
117. Suppose the psychologist decides to pursue his II No lemon is cheap.
project further, and he studies 25 dogs which are
known to bark at strangers. Which of the follow- 122. Statements:
ing possible findings would undermine his origi- Some chalks are pens. Some pens are pencils.
nal conclusion? Conclusions:
(a) Some of the owners of the dogs studied did I. Some chalks are pencils.
not physically punish the dog when it barked II. Some pencils are chalks.
at a stranger 123. If ‘water’ is called ‘food’, ‘food’ is called ‘tree’,
(b) Some of the dogs studied were never physi- ‘tree’ is called ‘sky’, ‘sky’ is called ‘well’ on
cally punished which of the following would you expect a fruit
(c) The owners of some of the dogs studied to grow?
believe that a dog which barks at strangers (a) Well (b) Food
is a good watch dog
(c) Sky (d) Tree
(d) None of the above
124. Which one of the following groups of letters will
118. If in the following alphabet, ‘A’ is written for ‘A
appear the same in a mirror?
and B’, ‘B’ is written for ‘C and D(. ‘C for ‘E and
(a) YUMOVMY (b) HVRTRVH
F’ and so on, what will be the third letter from
(c) VOAMAOV (d) YMOVONY
the end in the new structure?
125. The marks obtained by 20 students in an exami-
ABCEDFGHIJKLMNOPQRSTU
nation for 100 marks are given below:
V W X Y Z
(a) L (b) J 40, 55, 63, 60, 35, 26, 42, 86, 72, 54, 77, 49, 29,
(c) K (d) None of these 44, 84, 64, 72, 59, 63, 31

7 . 45
Practice Questions

Вам также может понравиться